Download as pdf or txt
Download as pdf or txt
You are on page 1of 1147

Christian Apologetics

&
Research Ministry

MOAN
Mother Of All Notebooks

i
ii
CARM - Christian Apologetics & Research Ministry

Christian Apologetics Notebook


Volumes 1-5

A collection of information designed to aid the Christian in defending the Christian faith.

• Vol. 1 • Vol. 3
o Doctrine o Oneness Pentecostal
o The Bible o Universalism
o Baptism o Roman Catholicism
o Various Articles o Christadelphianism
o Parables o Christian Science
o Dictionary of Theology o Shepherd's Chapel
o Christian Issues o International Church of Christ
o To the Christian Church • Vol. 4
• Vol. 2 o Cults
o Apologetics o Jehovah’s Witnesses
o Apologetics Dialogues o Mormonism
o 40 Objections with Answers • Vol. 5
o Evidence and Answers o Atheism
o Bible Difficulties o Relativism
o Islam
o Heresies
o New Age Movement

By Rev. Matthew J. Slick, M. Div.


www.carm.org
copyright 2003

Permission is granted to copy the material via photocopying machine for distribution.
Please do not alter the text. Please provide author information if and when possible.

Scriptures are quoted in italic so they may stand out from the rest of the text.

Note to reader: The purpose of this volume, and all the other volumes, is to provide information for
the Christian by which the Lord Jesus may be glorified, the Christian equipped, and the unbeliever
encouraged to come to a saving relationship with Jesus. To this end, please feel free to copy the
material and distribute it where needed. But retain author and copyright notice.

iii
Table of Contents

Christian Doctrine...............................................................1
Introduction..................................................................................................................... 1
Christian Doctrine............................................................................................................... 2
The Bible........................................................................................................................ 2
God............................................................................................................................... 3
Trinity............................................................................................................................ 4
Creation......................................................................................................................... 4
Man............................................................................................................................... 5
Jesus............................................................................................................................. 6
The Holy Spirit................................................................................................................ 7
Salvation........................................................................................................................ 8
Justification and Sanctification.......................................................................................... 8
The Church..................................................................................................................... 8
The Resurrection............................................................................................................. 8
The Millennium................................................................................................................9
The Rapture....................................................................................................................9
The Final Judgment......................................................................................................... 9
The New Heavens and the New Earth.................................................................................9
Basic Christian Doctrine..................................................................................................... 11
Three Essential Doctrines of Christianity.............................................................................. 12
Who is God?..................................................................................................................... 14
The Holy Spirit..................................................................................................................15
The Atonement................................................................................................................. 17
James 2:24, not by faith alone........................................................................................... 20
The law and the gospel...................................................................................................... 21
Salvation: What Does it Mean to Be a Christian?................................................................... 24
Being a Christian Means Fellowship with Jesus...................................................................... 24
Covenant......................................................................................................................... 26
The Two Main Covenants................................................................................................... 30
Divisions under the covenant of grace................................................................................. 31
Jesus' Two Natures........................................................................................................... 32
Jesus............................................................................................................................. 32
Bible verses that show Jesus is God.................................................................................... 33
Jesus is Jehovah (YHWH)................................................................................................... 35
The True Jesus................................................................................................................. 36
100 Truths About Jesus..................................................................................................... 38
Who is Jesus according to John the Apostle?.........................................................................41
Jesus' Resurrection was physical......................................................................................... 44
Objections to Jesus' physical resurrection answered.............................................................. 46
What did Jesus come to do?............................................................................................... 49
Jesus is a man right now....................................................................................................50
Jesus is God..................................................................................................................... 53
If Jesus were not God, then explain.....................................................................................55
The Trinity, what is it?..................................................................................................... 56
Trinity is indeed biblical..................................................................................................... 57
The Trinity Chart...............................................................................................................58
Another Look at the Trinity.................................................................................................59
Early Trinitarian Quotes..................................................................................................... 61
The Bible..........................................................................64
Introduction................................................................................................................... 64
The Bible......................................................................................................................... 65
The Greek and Hebrew Alphabet with numeric equivalents..................................................... 66
iv
Papyri p1 through p76 200 A.D. to 700 A.D..........................................................................68
Minuscules 2 through 399 - 9th to 16th century copies.......................................................... 69
Minuscules 404 through 999 -- 8th to 16th century copies.....................................................70
Old Testament Books.........................................................................................................72
New Testament Books....................................................................................................... 74
When was the Bible written and who wrote it?...................................................................... 76
Other books mentioned in the Bible.....................................................................................78
Chronology of the Old Testament........................................................................................ 79
Bible Chronology of the New Testament............................................................................... 81
Non biblical accounts of New Testament events and/or people................................................ 83
The Documentary Hypothesis of the Pentateuch....................................................................86
Answering the Documentary Hypothesis.............................................................................. 88
Biblical Interpretation........................................................................................................ 91
Scientific Accuracies of the Bible......................................................................................... 95
Prophecy, the Bible and Jesus.............................................................................................96
What is Redaction Criticism?.............................................................................................101
Baptism.......................................................................... 102
Introduction..................................................................................................................102
Is Baptism Necessary for Salvation?.................................................................................. 103
Baptism and Mark 16:16.................................................................................................. 107
Baptism and Roman 6:3-5................................................................................................109
Baptism and Gal. 3:27..................................................................................................... 112
Baptism and 1 Pet. 3:21.................................................................................................. 113
Baptism and John 3:5...................................................................................................... 115
Baptism and Acts 2:38.....................................................................................................117
Various Articles..............................................................122
Introduction..................................................................................................................122
Man...............................................................................................................................123
Let Us make man in our image......................................................................................... 125
God............................................................................................................................... 126
Angels........................................................................................................................... 127
Pharisee.........................................................................................................................130
Moloch........................................................................................................................... 131
The Elder in the Church....................................................................................................132
The Crucifixion of Jesus....................................................................................................133
Plurality Study................................................................................................................ 135
Col. 2:9 and Eph. 3:19.................................................................................................... 138
Parables.........................................................................140
Introduction..................................................................................................................140
The Fig Tree Luke 13:1-9................................................................................................. 144
The Prodigal Son Luke 15:1-2, 11-32................................................................................ 147
The Good Samaritan Luke 10:25-37.................................................................................. 150
The Unjust Steward Luke 16:1-8.......................................................................................153
Dictionary of Theology....................................................158
Introduction..................................................................................................................158
Evangelism..................................................................... 204
Introduction..................................................................................................................204
Why Should We Witness?................................................................................................. 205
Foundations are First....................................................................................................... 206
Three Important Verses in Witnessing................................................................................208
Salvation is God's work.................................................................................................... 210
How to Memorize Scripture...............................................................................................212
The Importance of Prayer in Evangelism............................................................................ 214
The Do's and Don't's of Witnessing.................................................................................... 216
Law and Gospel...............................................................................................................217

v
The Four Spiritual Laws....................................................................................................219
Christian CPR..................................................................................................................220
Leading Someone to the Lord........................................................................................... 222
Trust God and Go Witness................................................................................................ 223
Christian Issues..............................................................224
What is a Christian World View and Why do Christians Need One?......................................... 225
What are Some Elements of a Christian World View?........................................................... 227
Christians and Education.................................................................................................. 228
Christianity and Homosexuality......................................................................................... 230
The Christian Family........................................................................................................ 232
Christianity and Science................................................................................................... 234
The Failure of the Christian Church....................................................................................236
To the Christian Church..................................................238
Introduction..................................................................................................................238
Introduction: Why this topic?........................................................................................... 239
What is the Christian church?........................................................................................... 241
What is the Christian church supposed to be?..................................................................... 243
The need for unity in the church........................................................................................245
There is pride in the Christian Church................................................................................ 247
Apostasy in the Christian church....................................................................................... 249
Examples of Apostasy in the Christian church..................................................................... 252
What are signs that a church is becoming secular?.............................................................. 254
The elder in the church.................................................................................................... 256
Should unbelievers lead Christians in worship in a church service?........................................258
What kind of a Christian are you?......................................................................................260
Do you know the basics of the Christian faith?.................................................................... 262
Answers to the questions on basics................................................................................... 264
Are you comfortable?.......................................................................................................267
Apologetics.....................................................................270
Introduction..................................................................................................................270
An Introduction to Apologetics.......................................................................................... 271
Are you an apologist?...................................................................................................... 273
Logic in Apologetics......................................................................................................... 274
Prayer in Apologetics....................................................................................................... 276
Are there Guidelines for doing Apologetics?........................................................................ 277
Presuppositional Apologetics............................................................................................. 280
Evidential Apologetics...................................................................................................... 281
The Cosmological Argument............................................................................................. 282
The Teleological Argument............................................................................................... 283
Apologetics Bibliography.................................................................................................. 284
Apologetics Dialogues....................................................286
Introduction..................................................................................................................286
Jesus' Resurrected Body, the Atonement, and Islam............................................................ 287
Discussion with a Gnostic on Jesus' Resurrection................................................................. 289
Does God have a Body?................................................................................................... 291
A Christian having doubts because of school....................................................................... 293
Is Baptism Necessary for Salvation?.................................................................................. 294
The Resurrection of Jesus: Literal or Symbolic?................................................................. 298
Pain can make us doubt God............................................................................................ 300
An Agnostic Questions the Trinity...................................................................................... 302
Claims to be a god.......................................................................................................... 303
Is it right to tell people that what they believe in is false?....................................................307
Dialogue with someone who claims to be one of the two witnesses of Revelation....................310
Discussion on the possibility of Jesus' resurrection..............................................................315
Condemning words and pompous attitude.......................................................................... 318
An evolutionist says evolution is a fact............................................................................... 323
vi
Discussion with a Jehovah's Witness about a relationship with Jesus ......................................326
Can the Muslim do enough good works to go to paradise?.................................................... 328
Discussion on how God can be one person as flesh and spirit................................................331
A Satanist judges God..................................................................................................... 334
Dialogues with Atheists.................................................................................................. 335
An Atheist Says There Is No Evidence For God.................................................................... 335
An Atheist Says He Knows There Is No God........................................................................ 336
Sickness as an argument against God's existence................................................................338
Discussion with an obnoxious atheist................................................................................. 341
Discussion on logical absolutes as a proof for God's existence............................................... 343
Dialogues with Catholics.................................................................................................345
Two Catholics say baptism is necessary for salvation........................................................... 345
Discussion with a Catholic on interpreting the Bible............................................................. 348
Dialogues with Mormons................................................................................................ 352
Two Mormons state that 3 Gods is really 1 God...................................................................352
Discussion with a Mormon on God's Nature........................................................................ 357
What is salvation and who is God?.....................................................................................361
Did Joseph Smith see God the Father?............................................................................... 364
Feelings, gods, and Joseph Smith......................................................................................366
Mormon and salvation and works...................................................................................... 369
40 Objections with Answers...........................................372
Introduction..................................................................................................................372
I am not a sinner............................................................................................................ 373
What is sin?....................................................................................................................373
I am too big a sinner....................................................................................................... 373
What is salvation?........................................................................................................... 373
What do I do to get saved?.............................................................................................. 373
Is baptism necessary for salvation?................................................................................... 373
I am already good enough................................................................................................374
I am doing the best I can and I'm sincere.......................................................................... 374
I am skeptical................................................................................................................. 374
I tried Christianity once....................................................................................................374
I knew some Christians once and they wronged me.............................................................374
I'll take my chances.........................................................................................................375
I am not that bad a person...............................................................................................375
I am too old or too young.................................................................................................375
I can't believe in a God who would send people to hell......................................................... 375
I will worry about it in the next life.................................................................................... 375
I don't want to give up what I like doing............................................................................ 376
Christianity is boring........................................................................................................376
I am an atheist. I don't believe in God............................................................................... 376
I am trying to be a Christian.............................................................................................377
I am already religious...................................................................................................... 377
I don't need God............................................................................................................. 377
I have things I need to do before I become a Christian........................................................ 377
I prefer to remain open minded........................................................................................ 377
I already believe in God................................................................................................... 377
I'll choose God later.........................................................................................................378
There are too many hypocrites in the church...................................................................... 378
Why are we here? Or, Why did God make us?.....................................................................378
What about those who have never heard the Gospel?.......................................................... 378
Jesus is only one of many great men of history................................................................... 378
Why is there evil and suffering in the world?.......................................................................379
What makes Jesus so special?...........................................................................................379
Why did Jesus have to die in order for me to go to heaven?.................................................380
What makes you think the Bible is the word of God?............................................................380
The New Testament was written so that it would only look like Jesus fulfilled prophecy. ...........380

vii
The Bible is full of contradictions....................................................................................... 380
How do I know which religion is right?............................................................................... 381
Religion is whatever you feel is right..................................................................................381
All religions are different paths to the same place................................................................381
What about dinosaurs and evolution?.................................................................................381
Evidence and Answers.................................................... 383
Introduction..................................................................................................................383
Inerrancy and inspiration of the Bible................................................................................ 384
The Miracles of Jesus....................................................................................................... 386
Evidence supporting the Bible......................................................................................... 387
Manuscript evidence for superior New Testament reliability..................................................387
Illustration of Bible text manuscript tree............................................................................ 390
Non biblical accounts of New Testament events and/or people.............................................. 391
Archaeological Evidence verifying biblical cities................................................................... 394
The writings of Josephus mention many biblical people and places........................................397
When were the gospels written and by whom?....................................................................399
Evidence of biblical inspiration.......................................................................................... 402
Answers to Questions about the Bible.............................................................................. 405
Can we trust the New Testament as a historical document?.................................................. 405
Wasn't the New Testament written hundreds of years after Christ?........................................408
Hasn't the Bible been rewritten so many times that we can't trust it anymore?.......................411
Since the New Testament writers were biased, can we trust their testimony?......................... 413
What is the gospel of Q and does it prove the Gospels are false?...........................................414
Why isn't there other evidence of the massacre of the babies?..............................................416
Do the lost books of the Bible prove that the Bible has been altered?.....................................419
Is there non-biblical evidence of a day of darkness at Christ's death?....................................420
Evidence supporting Jesus' existence............................................................................... 421
Regarding the quotes from the historian Josephus about Jesus.............................................421
1 Cor. 15:3-4 demonstrates a creed too early for legend to corrupt.......................................423
Answers to objections concerning Jesus' miracles and resurrection......................................425
Extraordinary claims require extraordinary evidence............................................................425
Does the Bible provide extraordinary evidence for Jesus' Resurrection?..................................428
It is improbable that Jesus rose from the dead....................................................................430
The Christians were mistaken about Jesus' resurrection....................................................... 431
The New Testament writers conspired together to gain power and influence...........................432
Jesus was a magician who made people hallucinate about His miracles..................................435
Jesus only appeared to have died on the cross - Swoon Theory.............................................436
The Disciples stole Jesus' body and faked His resurrection....................................................438
There are no non-biblical accounts to the resurrection......................................................... 441
Miracles cannot happen....................................................................................................443
Answers to Questions about Jesus................................................................................... 444
Was Jesus just a myth?....................................................................................................444
Did Jesus really die on the cross?...................................................................................... 447
Did Jesus rise from the dead?........................................................................................... 448
Didn't Jesus simply rise in a non-physical, spirit form?......................................................... 449
If Jesus is God in flesh, why did He not inherit original sin?..................................................450
Can't all Jesus' miracles be explained naturally?..................................................................451
Other Objections answered............................................................................................. 455
Are the New Testament themes found in the Old Testament?...............................................455
Apollonius of Tyana also did miracles and rose. What about him?.......................................... 457
Doesn't the religion of Mithra prove that Christianity is false?...............................................458
Why believe in Christianity over all other religions?............................................................. 460
If God is all powerful and loving, why is there suffering in the world?.....................................463
A loving God would never send anyone to hell.................................................................... 466
It is intolerant to say that Christianity is the only true religion. .............................................468
Why did animals have to die for the sins of Adam and Eve and others?.................................. 469
Why would God have to die to save people from Himself?....................................................470

viii
If God is not the author of confusion, what about the Tower of Babel?...................................471
If babies die when they go to heaven, why is abortion wrong?.............................................. 471
Bible Difficulties.............................................................472
Introduction..................................................................................................................472
Introduction to Bible Difficulties and Bible Contradictions.....................................................473
1 & 2; Don't Gen. 1 and 2 present contradictory creation accounts?......................................474
Genesis........................................................................................................................ 474
1:26, How many Gods are there, one or many?.................................................................. 475
3:9, Doesn't God saying ‘Adam where are you?’ show God didn't know something?.................476
4:17, Where did Cain get his wife?.................................................................................... 476
5:1-31, Did people really live hundreds of years according to Genesis?.................................. 477
6:19-20, How many kinds did Noah bring into the ark, two or seven?.................................... 478
37:28, Who purchased Joseph, the Ishmaelites or the Midianites?.........................................478
38:9, God kills a man for spilling his seed on the ground......................................................479
Exodus.........................................................................................................................479
6:2-3, Has anyone seen God or not?................................................................................. 479
20:5, Should you make graven images or not?................................................................... 480
20:8, Should we keep the Sabbath or not?......................................................................... 481
Deuteronomy................................................................................................................482
5:9, Do the sons bear the sins of the fathers or not?........................................................... 482
1 Samuel......................................................................................................................483
16:19-23, Did or did not Saul know who David was?...........................................................483
17:50, Who killed Goliath, David or Elhanan?.....................................................................484
31:4, Who killed Saul, Saul or the Amalekite?.....................................................................484
2 Samuel......................................................................................................................485
6:23, Did Michal have any children or not?......................................................................... 485
10:18, How many charioteers were killed, 700 or 7000?...................................................... 485
24:1, Who incited David to count the fighting men of Israel? God or Satan?..........................486
24:9, How many fighting men were found in Judah and Israel?.............................................486
24:13, God sent his prophet to threaten David with how many years of famine?.....................487
1 Kings........................................................................................................................ 487
4:26, How many stalls of horses did Solomon have, 4,000 or 40,000?................................... 487
5:16, How many supervisors were there?...........................................................................488
7:26, How many baths, 2000 or 3,000?............................................................................. 489
2 Kings........................................................................................................................ 489
8:26, How old was Ahaziah when he began to rule over Jerusalem?......................................489
24:8, How long did Jehoiachin rule over Jerusalem?............................................................ 490
2 Chronicles..................................................................................................................490
11:20, Who was King Abijah's mother?.............................................................................. 490
36:9, How old was Jehoiachin when he became king?.......................................................... 490
Ezra – Nehemiah........................................................................................................... 491
2, Why are the statistics in Ezra 2 and Nehemiah 7 different?............................................... 491
Job.............................................................................................................................. 492
1, If Job was blameless, why did God allow Satan to afflict him?........................................... 492
Psalm.......................................................................................................................... 492
5:5; 11:5, Does God hate people or love them?.................................................................. 492
Isaiah.......................................................................................................................... 493
7:14, in Hebrew ‘alma’ means maiden, not virgin. Therefore, it is not a prophecy..................493
45:7, Is the Lord good or bad to people?............................................................................494
Jeremiah...................................................................................................................... 495
22:28-30, Did Coniah have children or not?........................................................................495
32:27, Is the Lord omnipotent or not?............................................................................... 495
Jonah...........................................................................................................................496
3:4,10, Did God destroy Nineveh or not?........................................................................... 496
Malachi........................................................................................................................ 496
3:6, Does the Lord change or not?.................................................................................... 496
Matthew....................................................................................................................... 497

ix
1, Why are there different genealogies for Jesus in Matthew 1 and Luke 3?............................497
4:5,8, Where did the devil take Jesus first, the pinnacle or somewhere else?..........................499
4:18, Where did Jesus first meet Simon Peter and Andrew?..................................................501
5:1; 6:9-13; 7:28, Who did Jesus tell the Lord's Prayer to?.................................................. 502
5:16; 6:1-4, Should or should we not let our good works be seen?.......................................503
5:22; 23:17, Can we call someone a fool or not?................................................................ 504
8:5-13, Who brought the Centurion's request to Jesus?.......................................................504
9:9, Was the taxman named Matthew or Levi?....................................................................506
9:18, Was Jairus' daughter alive or dead when he came to Jesus?.........................................506
11:2, When did John find out Jesus was the Messiah?..........................................................507
11:13-14, Was John the Baptist really Elijah?..................................................................... 509
12:40, How long was Jesus in the tomb?............................................................................510
17:1, After how many days did Jesus take the three men up the mountain?...........................512
20:20-21, Who made the request to sit beside Jesus in His kingdom?....................................512
21:12, Did Jesus cleanse the Temple on the first or second day?...........................................513
26:34,74-75, Did the cock crow once or twice before Peter's third denial?.............................. 513
27:3-8, How did Judas die, by hanging or falling down?....................................................... 515
27:34, Was the vinegar given to Jesus on the cross mingled with gall or myrrh?.....................515
Mark............................................................................................................................ 517
1:7-13, What did Jesus do after encountering John the Baptist?............................................517
3:29, Can you be forgiven of all sins or not?.......................................................................518
6:8, Did Jesus tell His disciples to take a staff or not?.......................................................... 518
10:46, How many blind men did Jesus encounter when leaving Jericho?................................519
11:2-7, Were one or two animals brought to Jesus?............................................................ 519
11:14, Did the tree that Jesus cursed wither immediately or overnight?................................ 520
14:43, Who arrested Jesus?............................................................................................. 522
15:20-21, Did Jesus or Simon of Cyrene carry the cross?..................................................... 523
15:25, At what hour was Jesus crucified?........................................................................... 524
15:26, What was written on the sign on the cross?..............................................................524
15:34, What are the last words of Jesus?........................................................................... 525
15:40, Were the women close or far from the cross?........................................................... 525
16:5, How many men or angels appeared at the tomb?.......................................................526
16:6, What did the angels tell Mary?................................................................................. 526
16:8, Did or did not the women tell what happened?........................................................... 529
16:9, Who saw Jesus first?............................................................................................... 529
16:9-20, Is the ending of Mark really scripture?.................................................................. 530
Luke............................................................................................................................ 531
14:26, Are we supposed to hate or not?.............................................................................531
John............................................................................................................................ 531
3:13, Did anyone ascend into heaven................................................................................ 531
8:14, Was Jesus' witness of Himself true or not?................................................................. 532
18:19, Who did Jesus see first upon his arrest, Annas or Caiaphas?.......................................533
Acts............................................................................................................................. 533
5:29, Shall we obey God's Law or human law?....................................................................533
9:3-4, When Paul saw the light, did all fall to the ground or not?...........................................533
9:7, Did the men with Paul hear the voice or not?............................................................... 534
Romans........................................................................................................................535
3:23, Have all people sinned or not?..................................................................................535
5:1, Are we saved by faith or by baptism?..........................................................................535
9:17, God hardened Pharaoh's heart. Is that right?............................................................ 536
15:33, Is the Lord a God of Peace or of war?...................................................................... 536
1 Corinthians................................................................................................................ 537
1:19, Will wisdom stand or not?....................................................................................... 537
15:29, Is baptism for the dead really Christian?.................................................................. 537
Galatians......................................................................................................................538
4:22, How many children did Abraham have, one or two?.................................................... 538
6:2,5, Do we bear one another's burdens or not?................................................................ 539
Ephesians.....................................................................................................................539
x
2:8-9, Are we saved by grace or works?............................................................................ 539
Colossians.................................................................................................................... 540
1:15-17, Is Jesus or God the creator of all things?..............................................................540
Hebrews.......................................................................................................................540
11:21, Did Joseph worship at the head of the bed or leaning on a staff?................................540
2 Peter.........................................................................................................................541
3:10, Does the earth abide forever or not?......................................................................... 541
James.......................................................................................................................... 541
1:13, Does God tempt people or not?................................................................................ 541
Jude............................................................................................................................ 542
14, Jude 14 quotes the book of Enoch. Is it scripture?.........................................................542
Table: Cleansing of the Temple........................................................................................ 543
Table: Crucifixion Chronology.......................................................................................... 545
Table: Total statistics of population from Ezra 2 and Nehemiah 7.........................................548
Table: Resurrection Chronology....................................................................................... 550
Table: Chronology of Jesus' Baptism and Temptation..........................................................553
Bibliography................................................................................................................... 555
Oneness Pentecostal......................................................557
Introduction..................................................................................................................557
What is Oneness Pentecostal theology?..............................................................................558
What does Oneness Pentecostal teach?.............................................................................. 559
Issues and Answers....................................................................................................... 560
Oneness and the word .................................................................................................... 560
Another look at Jesus, the Father, and two wills.................................................................. 562
Is Baptism Necessary for Salvation?.................................................................................. 563
Baptism and Mark 16:16.................................................................................................. 567
Baptism and Roman 6:3-5................................................................................................569
Baptism and Gal. 3:27..................................................................................................... 572
Baptism and 1 Pet. 3:21.................................................................................................. 573
Baptism and John 3:5...................................................................................................... 575
Baptism and Acts 2:38.....................................................................................................577
Must baptism be \........................................................................................................... 581
Is speaking in tongues a necessary sign of salvation?.......................................................... 582
Who did Jesus pray to?.................................................................................................... 583
Jesus' resurrection and ascension......................................................................................584
God was seen in the Old Testament. Who was it?............................................................... 584
Isaiah 9:6, Is Jesus the Everlasting Father?........................................................................586
What is the real gospel message?......................................................................................587
Witnessing to Oneness People......................................................................................... 588
Witnessing to those who are in oneness churches................................................................588
Questions to ask Oneness Pentecostal believers.................................................................. 589
Answers and response to Questions to ask Oneness Pentecostal believers..............................590
Universalism...................................................................593
Introduction..................................................................................................................593
Universalism...................................................................................................................594
Christian Universalism..................................................................................................... 596
Can a Christian be a universalist?......................................................................................598
Universalism and the Cults............................................................................................... 599
Texts examined.............................................................................................................600
Matt. 25:46 and Universalism........................................................................................... 600
Mark 3:28-29 and Universalism........................................................................................ 602
1 Tim. 4:10 and universalism........................................................................................... 605
1 Tim. 2:4 and 2 Pet. 3:9. Is it God's will that all people be saved?......................................607
Objections Answered to the paper, “Is it God's will that all
people be saved?............................................................ 610
xi
More objections answered to “Is it God's will that all people
be saved?.......................................................................614
Words examined........................................................................................................... 615
A look at the word ‘aionion’.............................................................................................. 615
What do Greek dictionaries say about ............................................................................... 617
Forever and Ever.............................................................................................................619
Issues and Answers....................................................................................................... 621
Scriptures that say not all are saved..................................................................................621
The unforgivable sin and the age to come.......................................................................... 622
Does God hate anyone?................................................................................................... 626
Is Hell Eternal?............................................................................................................... 628
The demonic forces will not be saved................................................................................. 631
Fallen angels go to the lake of fire forever.......................................................................... 632
Does eternal punishment deny God's justice?..................................................................... 634
The Danger of Universalism.............................................................................................. 635
The Danger of Universalism Illustrated...............................................................................637
If election is true what is the danger in universalism?.......................................................... 639
Satan and universalism.................................................................................................... 643
A Challenge to Universalists..............................................................................................645
What is Unitarianism?...................................................................................................... 646
Concluding thoughts on Universalism.................................................................................648
Roman Catholicism......................................................... 651
Introduction..................................................................................................................651
Why is it necessary to write about Roman Catholicism?........................................................652
Roman Catholicism, the Bible, and Tradition....................................................................... 653
Issues and Answers....................................................................................................... 656
Is the Bible Alone Sufficient for Spiritual Truth?..................................................................656
Catholic Terminology....................................................................................................... 658
Mary..............................................................................................................................660
Did Mary Have Other Children?......................................................................................... 661
Purgatory....................................................................................................................... 663
Does Purgatory Deny the Sufficiency of Christ’s Sacrifice?.................................................... 664
Purgatory and 1 Cor. 3:15................................................................................................666
Council of Trent: Canons on Justification........................................................................... 668
The Roman Catholic view on justification............................................................................670
Comparison Grid............................................................................................................. 674
Is the Catholic Catechism's view on the Muslim god wrong?.................................................675
Introduction..................................................................................................................677
What do the Christadelphians Teach?.................................................................................678
Christadelphian History.................................................................................................... 679
Is Christadelphianism Christian?....................................................................................... 680
Answering a “refutation....................................................................................................682
Issues and Answers....................................................................................................... 685
The Christadelphians, John 1_1, and ................................................................................ 685
Is God ever seen?........................................................................................................... 687
Interesting Quotes from Christadelphianism....................................................................... 689
Did Jesus have a sin nature?............................................................................................ 690
Was Jesus' sacrifice blemished according to Christadelphian theology?...................................694
Can the Christadelphian Jesus with a fallen nature save anyone?..........................................697
The Christadelphian view of the Holy Spirit.........................................................................699
Questions for Christadelphians.......................................................................................... 700
Has God performed the greatest act of love?...................................................................... 701
Christian Science............................................................ 703
Introduction..................................................................................................................703
What does Christian Science Teach?.................................................................................. 704
Christian Science History..................................................................................................705
xii
Is Christian Science Christian?.......................................................................................... 706
Terms and Definitions of Christian Science......................................................................... 707
Jesus is the Christ........................................................................................................... 709
Interesting Quotes from Mary Baker Eddy.......................................................................... 711
Questions to Ask Christian Scientists................................................................................. 712
Shepherd’s Chapel.......................................................... 713
Introduction..................................................................................................................713
What is the Shepherd's Chapel?........................................................................................ 714
Who is Arnold Murray?..................................................................................................... 715
What does the Shepherd's Chapel teach?........................................................................... 716
Is the Shepherd's Chapel Christian?...................................................................................717
Did we exist as souls prior to Adam's creation?................................................................... 718
The serpent seed and the Kenites......................................................................................719
Shepherd's Chapel and the rapture....................................................................................721
An open letter to Arnold Murray of Shepherd's Chapel.........................................................723
International Church of Christ........................................727
Introduction..................................................................................................................727
What does the International Church of Christ teach?............................................................728
Is the International Church of Christ a cult?........................................................................729
The International Church of Christ..................................................................................... 730
Verses showing justification by faith.................................................................................. 733
Is Baptism Necessary for Salvation?.................................................................................. 735
Baptism and Mark 16:16.................................................................................................. 739
Baptism and Roman 6:3-5................................................................................................741
Baptism and Gal. 3:27..................................................................................................... 744
Baptism and 1 Pet. 3:21.................................................................................................. 745
Baptism and John 3:5...................................................................................................... 747
Baptism and Acts 2:38.....................................................................................................749
Cults............................................................................... 753
Introduction..................................................................................................................753
Cults!............................................................................................................................ 754
What makes a church or group non-Christian?.................................................................... 757
Comparison Chart........................................................................................................... 759
Justification and Sanctification: What is the Difference?.......................................................761
Do we have the right to make these judgments?................................................................. 762
What is the truth?........................................................................................................... 763
What about the testimonies of people in cults?...................................................................766
An easy way to witness to Mormons and Jehovah's Witnesses..............................................768
The True Jesus................................................................................................................772
Jehovah’s Witnesses......................................................775
Introduction..................................................................................................................775
What do the Jehovah's Witnesses Teach?........................................................................... 778
Jehovah's Witness History................................................................................................ 779
Is the Jehovah's Witness religion Christian?........................................................................ 780
Are Jehovah's Witness are Really Watchtowerites?..............................................................781
Has Jehovah performed the greatest act of love?................................................................783
Issues and Answers....................................................................................................... 784
The Watchtower Organization points to itself as the truth..................................................... 784
Does the Watchtower organization control the Jehovah's Witnesses' thinking?.......................785
Does Annihilation and resurrection make sense?................................................................. 788
Does the Watchtower say the Bible teaches the Trinity?....................................................... 789
Questions for Jehovah’s Witnesses.................................................................................... 790
The Lord's Supper and the 144,000 Anointed Class of Jehovah's Witnesses............................791
Are the Jehovah's Witnesses the faithful and discreet slave?.................................................793
Salvation according to the Watchtower Organization............................................................795
The New World Translation and Proskuneo (worship)........................................................... 797
xiii
Bad Translations of the Jehovah's Witness Bible, the New World Translation (NWT).................801
Jehovah's Witnesses and Mental Health..............................................................................802
A Biblical Response to Jehovah's Witnesses........................................................................ 803
Watchtower Quotes....................................................................................................... 807
Interesting Quotes from Watchtower Literature...................................................................807
Regarding the Trinity, Jesus, Adam, and immortality of the soul............................................809
Problems in the Watchtower Organization.........................................................................810
False Prophecies of the Jehovah's Witnesses.......................................................................810
1914 A.D., 607 B.C., 586 B.C. and the Jehovah's Witnesses................................................. 812
Jehovah's Witness doctrine is not from the Bible alone.........................................................814
Contradictions in Watchtower Literature............................................................................. 815
More Contradictions in Watchtower Literature..................................................................... 816
Jehovah’s Witness Attacks on Jesus................................................................................. 818
The Jehovah's Witnesses and the Resurrection of Jesus.......................................................818
Did Jesus die on a stake or a cross?.................................................................................. 820
Bible Verses Examined................................................................................................... 821
1 Chron. 29:20, is Jesus worshipped the same way David was?............................................821
Matt. 3:3, Prepare the way of the LORD............................................................................. 823
John 1:1 and the Jehovah's Witnesses ............................................................................. 824
John 5:30-32, By Myself I can do nothing.......................................................................... 826
John 8:58 and the Jehovah's Witnesses and John 8:58....................................................... 827
John 8:58, & 10:30-33, I AM, and the Jeohovah’s Witnesses................................................831
John 17:3 and the Only True God......................................................................................833
1 Cor. 1:2, Call upon the name of the Lord Jesus................................................................ 835
Col. 1:15 and the Jehovah's Witnesses.............................................................................. 841
Col. 1:16-17 and the Jehovah's Witnesses ........................................................................842
Heb. 1:6 and the Jehovah's Witnesses .............................................................................. 844
Heb. 1:8 and Psalm 45:6 and the Jehovah's Witnesses ...................................................... 846
Mormonism.....................................................................849
Introduction..................................................................................................................849
Are you studying with the Mormons or thinking of joining.....................................................850
What does Mormonism teach?.......................................................................................... 852
Mormonism in a Nutshell.................................................................................................. 854
Mormonism's History....................................................................................................... 855
Is Mormonism Christian?.................................................................................................. 856
Mormon Writings and the Church.................................................................................... 858
A Quick Look at the Book of Mormon................................................................................. 858
Some of the Many Changes in the Book of Mormon............................................................. 860
The Book of Abraham Papyri and Joseph Smith................................................................... 862
The Mormon Church Statistics...........................................................................................866
Mormon Church Structure................................................................................................ 867
Issues and Answers....................................................................................................... 868
Does Mormonism Attack Other Religions?...........................................................................868
Mormon words don't mean the same thing......................................................................... 870
A Response From (and to) S.H.I.E.L.D.S............................................................................ 873
Jehovah is Elohim............................................................................................................883
A Biblical Response to Mormons........................................................................................ 885
A Comparison Between Christian Doctrine and Mormon Doctrine........................................... 889
What is Baptism for the Dead mentioned in 1 Cor. 15:29?....................................................891
The True Jesus................................................................................................................891
Mormon Objections Answered........................................................................................... 894
Was the LDS Jesus born of the Virgin Mary?....................................................................... 897
Miscellaneous................................................................................................................899
Hinckley says Mormons Believe in a Different Jesus............................................................. 899
The Mormon Plan of Eternal Progression.............................................................................900
Mormonism and the Negro............................................................................................... 901
Quotes......................................................................................................................... 903

xiv
Interesting Quotes from Joseph Smith the Founder of Mormonism........................................903
Interesting Quotes from Brigham Young the Second Prophet of the Mormon Church................904
Interesting Quotes from the book Articles of Faith, by James Talmage...................................906
Interesting Quotes from Various Mormon Authorities........................................................... 908
Mormonism Unvailed..................................................................................................... 911
Mormonism Unvailed, eyewitness testimonies against Joseph Smith.....................................911
Mormonism Unveiled Defended......................................................................................... 912
Mormonism Unveiled: Testimonies of Barton Stafford and Henry Harris.................................918
Mormonism Unveiled: Testimonies of Abigail Harris and Lucy Harris (Martin Harris' wife)........920
Mormonism Unveiled, Testimony of Artemas Cunningham................................................... 922
Mormonism Unveiled, Testimonies of Nahum Howard and Oliver Smith.................................. 924
Mormonism Unveiled, Testimony of John N. Miller............................................................... 925
Mormonism Unveiled, Testimonies of Roswell Nichols & Joshua Stafford................................. 926
Atheism.......................................................................... 928
Introduction..................................................................................................................928
Terms and Definitions...................................................................................................... 931
Can Atheists be ethical?................................................................................................... 934
Issues and Answers....................................................................................................... 935
Mistake Christians make when dialoguing with Atheists........................................................935
Mistakes Atheists make when dialoguing with Christians......................................................937
Is Atheism viable?........................................................................................................... 938
Response to criticism of .................................................................................................. 939
Another response to criticism of ....................................................................................... 943
I................................................................................................................................... 956
Response to criticism of - I lack belief in God...................................................................... 958
Another response to criticism of - I lack of belief in a god.....................................................961
Additional response to criticism of - lack of belief, and, Is atheism viable? ..............................964
Lack-of-belief analysis outline........................................................................................... 975
Comments from atheists.................................................................................................. 979
Answering Atheist's Objections........................................................................................981
Concerning atheist attacks on Theism................................................................................ 981
I don't' see any convincing evidence for God......................................................................983
Can God make a rock so big He can't pick it up?................................................................. 984
God cannot exist because His attributes would require limits................................................985
There is no proof that God exists.......................................................................................987
If God is unchanging, why does the world change if it reveals God?......................................988
Only atheism offers a predictable universe......................................................................... 989
All of reality and God's existence.......................................................................................991
If God exists, then...........................................................................................................992
Any entity that is not the source of all power within reality is not God....................................994
If everything needs a creator, then who or what created God?..............................................996
How can something that cannot be described be said to exist?..............................................997
Why do you believe in Jesus but not Santa Claus?............................................................... 998
Why believe in Christianity over all other religions?............................................................. 999
Proofs for the Existence of God......................................................................................1002
Entropy and Causality used as a proof for God's existence.................................................. 1002
Atheism, Evolution, and Purpose..................................................................................... 1003
The Christian Worldview, the Atheist Worldview, and Logic................................................. 1005
An answer to a refutation of the Transcendental Argument................................................. 1007
An answer to another response to the Transcendental Argument......................................... 1009
Relativism....................................................................1017
Introduction................................................................................................................ 1017
What is relativism?........................................................................................................ 1018
Ethical relativism........................................................................................................... 1020
Cognitive relativism....................................................................................................... 1022
Refuting relativism........................................................................................................ 1023
What if relativism were true? An illustration..................................................................... 1025
xv
What is truth?............................................................................................................... 1027
Islam............................................................................ 1029
Introduction to Islam................................................................................................... 1029
The Gospel for Muslims.................................................................................................. 1030
What is Islam?.............................................................................................................. 1032
Regarding Islam.......................................................................................................... 1034
The Qur'an................................................................................................................... 1037
Chronology of early Islam...............................................................................................1038
Divisions within Islam.................................................................................................... 1041
Doctrines of Islam....................................................................................................... 1043
What are the doctrines of Islam?..................................................................................... 1043
The Five Pillars of Islam................................................................................................. 1044
True faith in Islam......................................................................................................... 1045
Islamic Terms............................................................................................................... 1046
Issues and Answers..................................................................................................... 1048
Methods Muslims use to attack Christianity....................................................................... 1048
Comparison grid between Christianity and Islamic doctrine................................................. 1052
Does Islam teach salvation by works?.............................................................................. 1053
Questions for Muslims.................................................................................................... 1057
More questions for Muslims.............................................................................................1059
Differences between the Bible and the Qur'an................................................................... 1060
The Qu'ran says the Bible is not corrupt........................................................................... 1061
Who has performed the greatest act of love? Yahweh or Allah.............................................1062
Jihad: holy struggle or holy war?.................................................................................... 1064
Is the Trinity possible?................................................................................................... 1067
Objections Answered....................................................................................................1068
The Trinity makes no sense. It isn't logical......................................................................1068
Jesus cannot be God's son.............................................................................................. 1070
If Jesus is God, then who did He pray to?......................................................................... 1071
God cannot be tempted. Jesus was tempted. Therefore, Jesus cannot be God.....................1072
God is infinite. Matter is finite. God could not become a man............................................1073
Why is it necessary for God to die for our sins?................................................................. 1075
The Qur'an..................................................................................................................1077
Contradictions in the Qur'an........................................................................................... 1077
Interesting quotes from the Qur'an.................................................................................. 1079
Interesting Quotes about women from the Qur'an.............................................................1081
The Hadith..................................................................................................................1082
Interesting Quotes from the Hadith, Part two.................................................................... 1086
Interesting Quotes from the Hadith about Forgiveness....................................................... 1090
Interesting Quotes from the Hadith on Jesus.....................................................................1092
Interesting Quotes from the Hadith on Satan.................................................................... 1098
Bibliography................................................................................................................. 1101
Heresis.........................................................................1102
Introduction................................................................................................................ 1102
Heresies.......................................................................1103
Gnosticism....................................................................................................................1106
New Age Movement...................................................... 1112
Introduction................................................................................................................ 1112
What is the New Age Movement...................................................................................... 1113
More on the New Age Movement..................................................................................... 1116
A Biblical Responses to the New Age Movement................................................................ 1118
Witnessing to New Agers................................................................................................ 1119
Interesting Quotes from New Age Sources........................................................................ 1120

xvi
Christian Doctrine
Introduction
Christian doctrine is the basis of proper understanding of the Bible and of God. The Lord gave us the
Bible and we have developed doctrinal statements which reflect the theology of God’s word.

When reading through this section, note the basics of the faith. If you were to learn this section well,
you would be able to recognize error.

Knowing the truth is the way to strength and accuracy.

Following are some questions you might want to answer.

1. How many books in the Bible? p. 2


2. Is the Bible reliably transmitted to us from ancient times? p. 2
3. Who is God? p. 3, 13
4. What is the Trinity? p. 4, 53
5. How many persons in the Trinity? p. 4
6. How did the universe come into existence? p. 4
7. What are the effects of sin on mankind? p. 5.
8. What is original sin? p. 5, 184
9. How many natures does Jesus have? pp. 6, 30
10. What is the hypostatic union? p. 6
11. What kind of body did Jesus rise in? p. 6, 41-45, 47
12. What priesthood does Jesus hold? p. 6
13. Is the Holy Spirit a person? p. 7, 14
14. What are some of His works? p. 7, 14
15. What is salvation? p. 7-8, 22
16. What must we do to be saved? p. 7-8
17. What is the difference between Justification and sanctification? p. 8
18. What is the church? p. 8
19. Is the Resurrection physical or spiritual? p. 8
20. What is the millennium? p. 8
21. What is the rapture? p. 9
22. What is the final Judgment? p.9
23. What are some verses that show justification by grace/faith? pp. 17-18
24. What is the difference between the law and the gospel? pp. 19-21
25. What is a covenant? pp. 24-29
Christian Doctrine
The following is an outline of basic Christian Doctrine. It's brief, accurate, and informative. As with
any venture into learning you must first begin with the basics. The basics are the foundation of your
Christian life. If you learn what is here, you will be well informed and knowledgeable. All you need to
do is get grounded in the fundamentals and from there cultivate your Christian walk: "Therefore let us
leave the elementary teachings about Christ and go on to maturity" (Heb. 6:1). A building is only as
strong as its foundation.

1. The Bible
A. The Bible consists of 66 books: 39 in the OT and 27 in the new. (Note: 3 x 9 = 27).
i. The OT has 23,214 verses. The NT has 7,959 verses.
B. The Bible took about 1600 years to write.
i. It was written in three languages (Hebrew, Aramaic, and Greek) by about 40 authors
and is internally consistent throughout.
ii. It was written on three continents: Africa, Asia, and Europe.
iii. It was written by a variety of people: prophets, priest, cupbearer, a king, judges,
fishermen, etc.
C. The first English translation of the Bible was begun by John Wycliffe and completed by John
Purvey in A.D. 1388.
D. The first American edition of the Bible was perhaps published some time before A.D. 1752.
i. The Bible has been translated in part or in whole as of 1964 in over 1,200 different
languages or dialects.
E. The Bible was divided into chapters by Stephen Langton about A.D. 1228.
i. The Old Testament was divided into verses by R. Nathan in A.D. 1448 and the New
Testament by Robert Stephanus in A.D. 1551.
F. Old Testament -- a total of 39 books and has 5 main divisions:
i. Pentateuch (Genesis to Deuteronomy), Historical (Joshua to Esther), Poetic (Job to Song
of Solomon), Major Prophets (Isaiah to Daniel), Minor Prophets (Hosea to Malachi).
G. New Testament -- a total of 27 books and has 4 main divisions
i. Gospels (Matthew to John), History (Acts), Epistles (Romans to Jude), Prophetic
(Revelation).
H. Reliability of the biblical documents.
i. The Bible is 98½ percent textually pure. This means that through all the copying of the
Biblical manuscripts of the entire Bible, only 1½% has any question about it. Nothing in
all of the ancient writings of the entire world even approaches the accuracy of
transmission found in the biblical documents.
ii. The 1½ percent that is in question does not affect doctrine. The areas of interest are
called variants and they consist mainly in variations of wording and spelling.
iii. The OT does not have as many supporting manuscripts as the NT but it is, nevertheless,
remarkably reliable.
a. The Septuagint, a Greek translation of the Hebrew OT done around 250 B.C., attests
to the reliability and consistency of the OT when it is compared to existing Hebrew
manuscripts.
b. The Dead Sea Scrolls discovered in 1947 also verify the reliability of the OT
manuscripts.
c. The Dead Sea Scrolls were ancient documents that were hidden in a cave in Israel
about 2000 years ago. The scrolls contained many OT books, one of them being
Isaiah.
i. Before the Dead Sea scrolls, the earliest existing manuscript of the OT was
dated around 900 A.D. called the Masoretic Text. The Scrolls contained OT
documents 1000 years earlier. A comparison between the manuscripts revealed
an incredible accuracy of transmission through copying, so much so that critics
were silenced.
iv. The NT has over 5000 supporting Greek manuscripts existing today with another 20,000
manuscripts in other languages. Some of the manuscript evidence dates to within 100
years of the original writing. There is less than a 1% textual variation in the NT
manuscripts.

v. Estimated time of writing of the NT documents 1

a. Paul's Letters, 50-66 A.D.


b. Matthew, 70-80 A.D.
c. Mark, 50-65 A.D.
d. Luke, early 60's
e. John, 80-100 A.D.
f. Revelation 96 A.D.
vi. Some of the supporting manuscripts of the NT are:
a. John Rylands MS written around 130 A.D., the oldest existing fragment of the NT
b. Bodmer Papyrus II (150-200 A.D.)
c. Chester Beatty Papyri (200 A.D.), contains major portions of the NT
d. Codex Vaticanus (325-350 A.D.), contains nearly all the Bible.

e. Codex Sinaiticus (350 A.D.), contains almost all the NT and over half of the OT 2

f. No other ancient writing can boast of having copies so close to the original time of
writing. With the Bible, the difference is about 50 years. With Plato and Aristotle, for
example, the difference is measure in hundreds of years.
I. Prophecy and mathematical odds of fulfillment.
i. The odds of Jesus fulfilling 48 of the 61 major prophecies concerning Him are 1 in 10 157;
that is a one with 157 zeros behind it.
ii. By comparison, the estimated number of electrons in the entire known universe is about
1079; that is a one with 79 zeros behind it.
J. Inspiration and Inerrancy - The Bible is inspired by God. Inspiration means that God,
through the Holy Spirit, caused the writers of the Bible to write the accurate and
authoritative revelation of God. It is God breathed (2 Tim. 3:16) through the instrumentation
of the apostles and prophets (2 Pet. 1:21).
i. It is without error in the original manuscripts and absolutely reliable and true in all areas
it addresses.
ii. Every true Christian accepts the inspiration and authority of the Bible.
K. Scientific Accuracies in the Bible.
i. The spherical shape of the earth (Isaiah 40:22).
ii. The earth is suspended in nothing (Job. 26:7).
iii. The stars are innumerable (Gen. 15:5).
iv. The existence of valleys in the seas (2 Sam. 22:16).
v. The existence of springs and fountains in the sea (Gen. 7:11; 8:2; Prov. 8:28).
vi. The existence of water paths (ocean currents) in the seas (Psalm 8:8).
vii. The water cycle (Job. 26:8; 36:27-28; 37:16; 38:25-27; Psalm 135:7; Ecc. 1:6-7).
viii. The fact that all living things reproduce after their own kind (Gen. 1:21; 6:19).
ix. The nature of health, sanitation, and sickness (Gen. 17:9-14; Lev. 12-14).
x. The concept of entropy, that energy is running down (Psalm 102:26).
2. God

1
McDowell, Josh, Evidence that Demands a Verdict, Here’s Life Publishers. San Bernardino, CA, 1979, pp. 39-52.
2
Ibid. pp. 46-47.
A. God is the only Supreme Being. He is Holy (Rev. 4:8), Eternal (Isaiah 57:15), Omnipotent
(Jer. 32:17,27), Omnipresent (Psalm 119:7-12), Omniscient (1 John 3:20); etc.
B. He is Love (1 John 4:8,16); Light (1 John 1:5); Spirit (John 4:24); Truth (Psalm 117:2);
Creator (Isaiah 40:12,22,26), etc.
C. He is to be worshiped (Gen. 24:26; Exodus 4:31; 2 Chron. 29:28; 1 Cor. 14:25; Rev. 7:11).
D. He is to be served (Matt. 4:10; 1 Cor. 6:19; Phil. 3:7; 1 Thess. 1:9; Heb. 9:14).
E. He is to be proclaimed (Matt. 28:19f.; John 14:15f.; Acts 1:8)
i. "To worship God is to serve and proclaim Him; to serve God is to proclaim and worship
Him; to proclaim God is to worship and serve Him."
ii. The name of God is Jehovah, or Yahweh. It is comprised of the four Hebrew consonants
yod-he-vah-he. The precise pronunciation of God's name has been lost. In Exodus 3:14
God proclaims His name to be "I AM." "God said to Moses, ‘I AM WHO I AM. This is what
you are to say to the Israelites: ‘I AM has sent me to you.'" (NIV)
3. Trinity
A. The Trinity is the doctrine that there is one God who exists simultaneously in three persons.
Each is coequal, copowerful, and coeternal with the other. Each person, Father, Son and Holy
Spirit, is not the other. Without either there is no God; all comprise the one God.
i. The Doctrine of the Trinity is opposed to:
a. Modal Monarchianism, also known as Jesus Only - There is one person in the
Godhead who took three consecutive forms or modes. First there was the Father
who then became the Son who then became the Holy Spirit.
i. Present groups are the United Pentecostal and United Apostolic churches. This
doctrine is incorrect. It denies the true doctrine of the Trinity.
b. Dynamic Monarchianism - Only one person in the Godhead, the Father. Jesus and
the Holy Spirit are not God.
i. Present groups are the Jehovah's Witnesses, World Wide Church of God,
Christadelphianism, and The Way International. This doctrine is incorrect. It
denies the Trinity, the deity of Christ, and the deity of the Holy Spirit.
c. Tritheism – the teaching that the godhead is three separate gods: Father, Son, and
Holy Spirit.
B. Christianity is monotheistic - Only one God in existence, anywhere, anytime. See Isaiah
43:10; 44:6,8; 45:5,14,18,21,22; 46:9; 47:8; John 17:3; 1 Cor. 8:5-6; Gal. 4:8-9 for
verses that teach monotheism.
C. Christianity is opposed to:
i. Polytheism - Belief in many Gods.
a. Monolatry - Belief in more than one God but serves and worships only one, i.e.
Mormonism.
b. Henotheism - Belief in one God without denying the existence of others.
ii. Pantheism - God is in the world, God is the world - New Age Movement.
iii. Panentheism - The belief that God is in the universe. It differs with pantheism which
states that God is the universe and all that it comprises.
iv. Deism - God exists, but is not involved in the world.
v. Theism - God exists, and is involved in the world.
4. Creation
A. God created the physical and spiritual universe out of nothing (Gen. 1:1f; Psalm 33:6; John
1:3; Rom. 4:17; 1 Cor. 1:28).
i. He did not make the world out of part of Himself.
ii. He did not make the world out of a substance called "nothing."
B. Specifically it was Jesus, the firstborn (Col. 1:15), second person of the Trinity, who created
all things (Col. 1:16-17; Isaiah 44:24).
C. Because God created all things, He is before all things and beyond all things. Therefore, the
entire universe is under His control.
D. Because God created all things, He is able to provide for His creation through the means of
His creation, i.e. weather, rain, plants, animals, sunshine, etc.
E. Opinions on the duration of creation differ. Some say six days; others say six long
periods.
5. Man
A. Creation of man
i. Man is not only the crown of creation, but also the object of God's special care.
ii. Man was originally made pure, without sin.
iii. "Then God said, ‘Let us make man in our image, in our likeness, and let them rule over
the fish of the sea and the birds of the air, over the livestock, over all the earth, and
over all the creatures that move along the ground.' So God created man in his own
image, in the image of God he created him; male and female he created them" (Gen.
1:26-27; see also, 2:7,21-23).
a. "Let us make man..." is a disclosure of the divine counsel before the creation of
man, "us" being the Trinitarian counsel. See also Gen. 11:7.
b. Man was created different than the animals. He had the breath of life breathed into
him from God (Gen. 2:7). The animals did not. Also Man was given dominion over
the animals. Man can know God, worship Him, and love Him. Animals cannot.
iv. Is Man made of two or three "parts"?
a. Dichotomy is a term which signifies a division into 2 parts: Body and Soul. The
words "spirit" and "soul" are often used interchangeably.
i. "Mary said: ‘My soul glorifies the Lord and my spirit rejoices in God my Savior,'"
(Luke 1:46-47).
ii. "My soul yearns for you in the night; in the morning my spirit longs for you...,"
(Isaiah 26:9).
iii. For the term "Body and Soul" see Matt. 6:25; 10:28.
iv. For the term "Body and Spirit" see 1 Cor. 5:3,51.
b. Trichotomy is a term which signifies a division into 3 parts: Body, Soul, and Spirit.
i. "May God himself, the God of peace, sanctify you through and through. May
your whole spirit, soul and body be kept blameless at the coming of our Lord
Jesus Christ,” (1 Thess. 5:23).
ii. "For the word of God is living and active. Sharper than any double-edged
sword, it penetrates even to dividing soul and spirit, joints and marrow; it
judges the thoughts and attitudes of the heart," (Heb. 4:12).
c. There is no official orthodox position on the number of parts of man.
v. The Origin of the soul
a. Traducianism: "The souls of men are propagated along with the bodies by
generation, and are therefore transmitted to the children by the parents." (Berkhoff,
Systematic Theology. p. 197.)
b. Creationism: "The soul is a creation of God, owing its origin to a direct creative act."
(Berkhoff, p. 199).
c. Except for Adam, the Bible makes no clear remark regarding the origin of the soul.
vi. Man was created in the image of God.
a. This means that Man has moral and intellectual abilities similar to God though not as
perfect and vast.
i. "God said, ‘Let us make man in our image, in our likeness...'" (Gen. 1:26).
ii. "...and have put on the new self, which is being renewed in knowledge in the
image of its Creator" (Col. 3:10).
b. Man is above the animals in "rational ability, moral awareness, pursuit of beauty,
use of language, and spiritual awareness."
B. Man before the Fall.
i. The Law of God was written in their hearts. Adam and Eve were without sin and "endued
with knowledge, righteousness, and true holiness after God's own image, with the ability
to keep the Law of God." (Westminster Confession of Faith, 4:2.)
ii. In this state man had free and unhindered access to God. This is exemplified in the
account in Gen. 3:8 where God was walking in the Garden.
C. Man, the Fall, and its effects
i. Adam and Eve rebelled against God and sinned by eating the forbidden fruit.
a. "Therefore, just as sin entered the world through one man, and death through sin,
and in this way death came to all men, because all sinned" (Rom. 5:12, NIV).
ii. What was their sin?
a. They listened to Satan and ate of the fruit that was forbidden by God ( Gen. 3:1-13).
iii. What was the consequence of their sin?
a. Death (Rom. 6:23) and separation from God's presence (Isaiah 59:2)
b. Transmission of the sin nature to their (and our) children (Psalm 51:5).
c. Creation also fell (Gen. 3:17; Rom. 8:22).
iv. How did their sin affect God?
a. They became unfit for the presence of God (Isaiah 59:2).
b. They became unable to do God's will (Rom. 6:16; 7:14).
c. They became subject to the curse of the Law and death (Deut. 27:26; Rom. 6:23).
v. Original Sin - The doctrine that we inherit our sin natures from Adam ( Rom. 5:12-21).
a. Adam was the Federal Head of all humanity; that is, he represented all people in the
Garden of Eden.
i. "For as in Adam all die..." (1 Cor. 15:22).
ii. The phrase "in Adam" indicates our relation to Adam, that he represented us in
the garden. In the same way, our being "in Christ" indicates our relation to
Jesus, that He represent us on the cross (Rom. 5:18; 6:11; 8:1; 1 Cor. 1:2;
15:22; 2 Cor. 5:19).
b. Our sin with Adam: "Therefore, just as sin entered the world through one man, and
death through sin, and in this way death came to all men, because all sinned" (Rom.
5:12). See also Rom. 5:18; 1 Cor. 15:22.
D. Man after Death and before the resurrection.
i. The intermediate state
a. This is the condition of the soul between the death of the body and the resurrection.
b. There is little spoken of it in the Bible, but it is a state of consciousness ( 2 Cor. 5:5-
8; Luke 16:19-31).
c. We are self aware and, apparently, with the Lord (Phil. 1:21-23).
i. For the righteous this is a time of blessedness and joy (Luke 16:19-31).
ii. For the unrighteous this is a time of suffering (Luke 16:19-31) as is exemplified
in the account of Lazarus and the rich man.
6. Jesus
A. He is the creator (John 1:1-3; Col. 1:15-17).
B. He is uncreated (John 1:1-3; Col. 1:15-17).
C. He is God in flesh (John 1:1,14; 8:58 with Exodus 3:14; Col. 2:9; Phil. 2:5-8; Heb. 1:8).
D. His Incarnation and His deity
i. Hypostatic Union - Jesus has two natures in one person. He was not half God and half
man. He is both Human and Divine. He was completely God and completely man. This is
the correct position concerning His two natures. See Col. 2:9; Phil. 2:5-8; John 8:58 and
Exodus 3:14.
ii. Jesus will remain as both God and man for eternity.
iii. Jesus was born of the Virgin Mary (Matt. 1:18; Luke 1:35).
a. He was born under the Law (Gal. 4:4) and fulfilled all of the Law of God (John 4:34 ;
8:29), even to the point of death (Phil. 2:8). In His death He bore the curse of the
law by becoming a curse for us (Gal. 3:13). Thus in the death of Christ the sins of
His people were judged (Rom. 3:23-26) and forgotten (Heb. 8:12), and the result of
His act of righteousness was eternal life (Rom. 5:18).
iv. Jesus is worshiped - (Matt. 2:2,11; 14:33; John 9:35-38; Heb. 1:6).
v. Jesus is prayed to - (Acts 7:55-60; Psalm 116:4 and Zech. 13:9 with 1 Cor. 1:1-2).
vi. Jesus is called God - (John 20:28; Heb. 1:8).
vii. He is the exact representation of the nature of God (Heb. 1:3).
E. His death and the atonement
i. Jesus bore the sins of the world (1 John 2:2) in His body on the cross (1 Pet. 2:24).
ii. He was a propitiation, a satisfaction to God that appeased God's wrath.
iii. He atoned. He made right that which was wrong between us and God. His shed blood is
what cleanses us from sin (Lev. 17:11; Heb. 9:22; Rom. 5:9; 1 John 1:7-9).
a. He removed the enmity between God and Man (Rom. 5:10).
iv. For whom did He die? - Some say for the sheep (Christians) only (John 10:11,15).
a. The Sheep are the Christians. The Goats are the non-Christians (Matt. 25:32-46).
v. Others say He died for everyone (1 John 2:2). Each side has good arguments.
F. The Resurrection of Christ (John 2:19-21; 1 Cor. 15:1-4).
i. Jesus rose in the same body that He died in (John 2:19-21; Luke 24:36-43).
a. Jesus' body is ‘resurrected.' We do not know exactly what His body is like, but the
nature of the resurrected body is discussed by Paul in 1 Cor. 15:35-58.
G. Right now Jesus is in heaven, still as, and eternally to be both God and man ( 1 Tim. 2:5;
Col. 2:9).
i. This is important because Jesus is the High Priest forever: "where Jesus has entered as
a forerunner for us, having become a high priest forever according to the order of
Melchizedek,” (Heb. 6:20). A spirit cannot be a high priest, only a man can do that.
Furthermore, Jesus always lives to make intercession for us "Hence, also, He is able to
save forever those who draw near to God through Him, since He always lives to make
intercession for them," (Heb. 7:25).
H. The Ascension of Christ (Acts 1:1-11.).
i. After the resurrection Jesus appeared to His disciples during a period of forty days. He
completed His message to them then.
ii. In light of the cloud in the O.T. (Exodus 40:34; 1 Kings 8:10f.; Luke 9:34f.) as a
manifestation of God's glory and presence, we have the necessary expectation of His
glorious ascension.
iii. He ascended in full view of the apostles who wrote of what they saw.
I. The Doctrine of the Deity of Christ is opposed to:
i. Docetism - Jesus was truly spirit and only appeared to be a man.
ii. Gnosticism - Jesus was only a man taken over by the heavenly Christ which never
became incarnate. The heavenly Christ returned to heaven before the crucifixion.
iii. Arianism - Jesus was created slightly lower than God. Then Jesus created all things.
J. The Hypostatic Union (Jesus having two natures in one person) is opposed to:
i. Kenosis - Jesus lessened Himself in the incarnation, i.e., God minus something.
ii. Eutychianism - The two natures of Jesus are completely ‘mixed' and indiscernible.
iii. Nestorianism - The two natures are not in contact with each other and that Jesus was
two persons.
iv. Monophycitism - The two natures combined and became one, a new type of being.
(Then Jesus would be neither God nor man, but a third something.)
7. The Holy Spirit
A. With the ascension of Christ we have the arrival of the Holy Spirit (John 14:26; Acts 2) who
ministers to the Church through the mediation of Christ (1 Tim. 2:5) and the Scriptures.
B. He is fully God; He is not a force. He is the third person of the Trinity.
i. He has a will - 1 Cor. 2:11
ii. He speaks - Acts 13:2
iii. He loves - Rom. 15:30
iv. He can be grieved - Eph. 4:30
v. He convicts of sin - John 16:8
vi. He creates - Gen. 1:2; Job 33:4
vii. He gives gifts - 1 Cor. 12:8
viii. He Intercedes - Rom. 8:26
ix. He teaches - John 14:26
x. He testifies of Jesus - John 15:26
xi. He baptizes - 1 Cor. 12:13
xii.He guides - John 16:13
xiii.
He encourages - Acts 9:31
xiv.He empowers - Micah 3:8
xv. He gives joy - Rom. 14:17
xvi.He comforts - John 14:16-26
C. The Holy Spirit indwells the believer (Rom. 8:11) and continues to work in him to bring
about sanctification (Rom. 15:16).
D. The Holy Spirit illuminates the mind of the believer (1 Cor. 2:12,13) and reveals to Him the
things of God (1 Cor. 2:10,13; 1 John 2:27).
8. Salvation
A. Salvation is the deliverance out of or the saving from the judgment of God upon the sinner.
This judgment is known as damnation and consists of God casting the unsaved into the lake
of eternal fire. The saved go to heaven to be with the Lord forever.
B. God is the sole agent of salvation (Eph. 2:8-9; John 1:12-13; Acts 13:48). Man does not
cooperate with God to earn or keep salvation. If a person needed to do anything towards
his/her salvation, then Jesus died needlessly (Gal. 3:21).
C. Salvation is by faith, not by works (Rom. 3:21; Rom. 4:5; Gal. 3:21). It is a free gift (Rom.
6:23; Eph. 2:8-9).
D. In salvation, the sins of the Christian are borne in Christ on the cross and the merits of
Christ's righteousness are counted to the Christian.

E. The two main views on salvation in respect to man's choosing.


i. Free will - Man is totally able to accept or reject God (John 3:16) based upon some
quality or ability within him.
ii. Predestination - God predestines who He chooses into salvation (Eph. 1:1-11; Acts
13:48). There is nothing within man that will allow him to choose God. God must call.
9. Justification and Sanctification
A. Justification is the instantaneous event where God imputes to the believer, the righteousness
of Christ.
B. Sanctification means to be set apart for holy use. It means to consecrate.
C. Where justification is that position of being declared righteous before God (Rom. 4:5; 5:9),
sanctification is the growth in the life of the Christian in holiness in understanding, intent,
thought, and action (1 Thess. 4:3-7).
D. Sanctification is a transformation of the believer produced by the Holy Spirit ( Eph. 5:22-23)
where godly fruit is the result.
E. The Christian's sanctification is tied to Christ: "I have been crucified with Christ and I no
longer live, but Christ lives in me..." (Gal. 2:20, NIV).
F. Further scriptures dealing with this are Rom. 6:1-23; Eph. 5:10-Gal. 6:10; Eph. 4:17-6:18)
10. The Church
A. The church can be viewed in two ways: The visible church and the invisible church.
i. The visible church is all who profess to be disciples of Christ.
ii. The invisible church is all who truly are saved.
B. The church is called the body of Christ (Eph. 1:22-23) with Christ as the head (Eph. 5:23).
C. The church is to be united (Eph. 4:1-16) under one God (Eph. 4:4).
D. The church is to be holy (1 Cor. 1:1-2; Eph. 5:27; 1 Pet. 2:9).
E. The church is open to all (John 3:16) and to preach the word of God (Matt. 28:19-20).
F. The church is called the bride of Christ (Eph. 5:22-23; Rev. 19:7), the church of the firstborn
(Heb. 12:23), the church of God (1 Cor. 1:2), God's building (1 Cor. 3:9), etc.
11. The Resurrection
A. The resurrection is when the dead in Christ are raised imperishable (1 Cor. 15:42,52-54).
i. In general, God raises the dead (2 Cor. 1:9). Specifically it is said that Jesus raises the
dead (John 5:21,25,28,29; 6:38-40,44,54; 1 Thess. 4:16).
ii. It is also said to be the work of the Holy Spirit (Rom. 8:11).
B. The resurrection occurs at the return of Christ, (1 Thess. 4:16-17; 1 Cor. 15).
C. The resurrection is physical.
i. Jesus is called the first-fruits of the resurrection (1 Cor. 15:20,23) and the firstborn from
the dead (Col. 1:18; Rev. 1:5). He was raised in the same body He died in (John 2:19-
21; Luke 24:39). Therefore, we shall also be raised in physical form as He was.
ii. It is not known exactly what our bodies will be like but it is thought that they will be like
Jesus' resurrection body (Phil. 3:21; 1 Cor. 15:42-54), not in His divinity, but in the
state of His resurrection.
D. There will be a resurrection of the good and of the wicked (Acts 24:15).
i. The good, the Christians, will be raised to everlasting life Matt. 25:31-34).
ii. The bad, the non-Christians, will be raised to everlasting punishment ( Matt. 25:4-46).
12. The Millennium
A. Millennium means 1000 years. There are three main views concerning the Millennium.
i. Amillennialism - that we are in the millennial reign of Christ now.
a. This view asserts that Satan was bound when Jesus first came to earth. It holds that
at the return of Christ the rapture occurs, the judgment of the wicked takes place,
and the new heavens and earth are created.
ii. Premillennialism - that the millennial reign of Christ has not yet happened.
a. This view asserts that Jesus will return (the rapture occurs near or at His return)
and then bind Satan, cast him into the abyss, and rule on earth for 1000 years. At
the end of that period Satan will be let loose to lead a rebellion. Jesus will then
destroy him. Then comes the final judgment, followed by the new heavens and
earth.
iii. Postmillennialism - that the church will usher in the millennium of Christ through the
preaching of the word and the conversion of the world.
B. There is debate on whether or not the millennium is a literal or figurative period. Some say
the period must be a literal 1000 years (Rev. 20:2), others say the period may be
interpreted figuratively (2 Pet. 3:8). There are very good arguments on both sides of the
issue.
C. Historically, the church has held mainly to Amillennialism and Premillennialism with each
gaining prominence at one time or another during the past 2000 years.
13. The Rapture
A. The rapture is the time when, at Christ's coming, the Christians who are alive are changed
into their resurrected bodies (1 Thess. 4:15-17). They are literally caught up to where Jesus
is as He descends from heaven to collect His church.
B. Those who have died beforehand come with Jesus and precede those who are on earth.
C. The main debate on the Rapture is when it will occur in relation to the Tribulation.
i. Pretribulation - the rapture will happen before the tribulation period.
ii. Midtribulation - the rapture will occur half-way through the tribulation period.
iii. Postribulation - the rapture will occur at the end of the tribulation period.
14. The Final Judgment
A. This is the judgment of all people (Matt. 25:31-46) at the end of all things (Matt. 13:40-43).
B. This judgment for the Christian is regarding his works (2 Cor. 5:10). It does not affect
salvation because being in Christ (Rom. 8:1) our works play no part in our salvation (Rom.
4:5).
i. The reward of the Christian is to be with the Lord forever (1 Thess. 4:17) in the new
heavens and new earth.
C. For the wicked the Day of Judgment (2 Pet. 3:7) is a judgment upon all their sinful actions
(Acts 17:31; 1 Cor. 13:11-15).
i. The wicked will be cast into hell (2 Thess. 1:6-10; Matt. 13:40-42).
15. The New Heavens and the New Earth
A. At the consummation of all things, God will destroy the elements with intense heat ( 2 Pet.
3:12).
B. There will be a new Earth which is the home of the righteous (2 Pet. 3:13).
C. This heavenly life will be social since it is spoken of in the context of a perfect city ( Heb.
12:28), as a holy temple (Ezekiel 40-48), and as a wedding feast (Rev. 19:7).
D. This heavenly life will have no more marriage (Matt. 22:30), no death (Luke 20:36), no
sorrow (Rev. 7:17), no pain (Rev. 21:4), etc.
E. This condition of perfection and fellowship with the Lord will be without end ( Matt. 25:46) in
a condition of light without darkness (Rev. 22:5).
Basic Christian Doctrine
1. There is only one God - Isaiah 43:10; 44:6,8; John 17:3; 1 Cor. 8:5-6; Gal. 4:8-9
2. God is a Trinity - 2 Cor. 13:14; 1 Pet. 1:2
3. There are no Gods before or after God - Isaiah 43:10
4. God knows all things - 1 John 3:20
5. God is all powerful - Psalm 115:3
6. God is everywhere - Jer. 23:23,24
7. God is sovereign - Zech. 9:14; 1 Tim. 6:15-16
8. God is spirit - John 4:24
9. God created all that exists - Gen. 1:1; Isaiah 44:24
10. Spirit does not have a body of flesh and bones - Luke 24:39
11. God has always been God - Psalm 90:2
12. Jesus is God - John 1:1,14; 10:30-33; 20:28; Col. 2:9; Phil. 2:5-8; Heb. 1:8
13. Jesus became a man - Phil. 2:5-8
14. Jesus has two natures: divine and human - Col. 2:99; 1 Tim. 2:5
15. Jesus was sinless - 1 Pet. 2:22
16. Jesus is the only way to God the Father - John 14:6; Matt. 11:27; Luke 10:22
17. The Holy Spirit is God - Acts 5:3-4
18. The Holy Spirit is not a force. He is alive - Acts 13:2
19. The Bible is inspired by God - 2 Tim. 3:16
20. All people have sinned - Rom. 3:23, 5:12
21. Man did not evolve, he was created - Gen. 1:26
22. Adam and Eve were real people - Gen. 3:20; 5:1; 1 Tim. 2:13
23. Death entered the world because of Adam's sin - Rom. 5:12-15
24. Sin separates us from God - Isaiah 59:2
25. Jesus died for all our sins - 1 John 2:2; 2 Cor. 5:14; 1 Pet. 2:24
26. Jesus' sacrifice was a substitution, for us - 1 Pet. 2:24
27. Jesus rose from the dead in His physical body - John 2:19-21
28. Those who reject Jesus will go to Hell - Rev. 20:11-15
29. Hell is a place of fiery punishment - Matt. 25:41; Rev. 19:20
30. Hell is eternal - Matt. 25:46
31. The unsaved go to hell forever - Rev. 21:8
32. Salvation is a free gift of God - Rom. 4:5; 6:23; Eph. 2:8-9
33. The Bible is the Word of God - 2 Tim. 3:16
34. Jesus will return visibly to earth - Acts 1:11
35. Christians will be raised from the dead when Jesus returns - 1 Thess. 4:14-17
36. There will be a rapture (being caught up into the clouds with Jesus) - 1 Thess. 4:14-17
37. There will be a final judgment - 2 Pet. 3:7
38. The damned will be thrown into a lake of fire - Rev. 20:15
39. Satan will be cast into the lake of fire - Rev. 20:10
40. There will be a new heavens and a new earth - 2 Pet. 3:13; Rev. 21:1
Three Essential Doctrines of Christianity
The Bible itself reveals those doctrines that are essential to the Christian faith. They are 1) the
Deity of Christ, 2) Salvation by Grace, and 3) the Resurrection of Christ. These are the doctrines the
Bible says are necessary. Though there are many other important doctrines, these three are the only
ones that are declared by Scripture to be essential. The truly regenerate may be ignorant to some
extent of one or more of them at the beginning of his/her new life in Christ, but he will come to a
proper understanding of these three issues as he studies the Word of God. A non-regenerate person,
or a cultist (i.e., Mormon or Jehovah's Witness), will deny one or more of these essentials.

1. The Deity of Christ


A. Jesus is God in flesh (John 8:58 with Exodus 3:14). See also John 1:1,14; 10:30-33; 20:28;
Col. 2:9; Phil. 2:5-8; Heb. 1:8
i. 1 John 4:2-3: "This is how you can recognize the Spirit of God: Every spirit that
acknowledges that Jesus Christ has come in the flesh is from God, but every spirit that
does not acknowledge Jesus is not from God. This is the spirit of the Antichrist, which
you have heard is coming and even now is already in the world."
a. The above verse needs to be cross referenced with John 1:1,14 (also written by
John) where he states that the Word was God and the Word became flesh.
b. 1 John 4:2-3 is saying that if you deny that Jesus is God in flesh then you are of the
spirit of Antichrist.
ii. John 8:24, "I said, therefore, to you, that you will die in your sins. For if you do not
believe that I am, you will die in your sins."
iii. Jesus said here that if you do not believe "that I am" you will die in your sins. In Greek
“I am” is 'ego eimi.' These are the same words used in John 8:58 where Jesus says
"...before Abraham was, I am." He was claiming the divine title by quoting Exodus 3:14
in the Greek Septuagint. (The Septuagint was the Hebrew Old Testament translated into
Greek.)
B. Jesus is the proper object of faith
i. It is not simply enough to have faith. Faith is only as valid as what it is put in. You must
put your faith in the proper object. Cults have false objects of faith; therefore, their faith
is useless--no matter how sincere they are.
ii. If you put your faith in a vacuum cleaner, then you will be in a lot of trouble on the Day
of Judgment. You might have great faith, but so what? It is in something that can't save
you.
C. The Doctrine of the deity of Christ includes:
i. The Trinity - There is one God who exists in three persons: The Father, the Son, and the
Holy Spirit. They are all coeternal, and of the same nature.
ii. Monotheism - There is only one God in all existence (Isaiah 43:10; 44:6,8;
45:5,14,18,21,22; 46:9; 47:8). Mormons believe that many gods exist though they
serve and worship only one. Therefore, they are polytheists which excludes them from
the camp of Christianity.
D. The Hypostatic Union - That Jesus is both God and man.
i. The sufficiency of the sacrifice of Christ - The sacrifice of Christ is completely sufficient
to pay for the sins of the world
ii. As God - Jesus must be God to be able to offer a sacrifice of value greater than that of a
mere man.
a. He had to die for the sins of the world (1 John 2:2). Only God could do that.
iii. As man - Jesus must be man to be able to be a sacrifice for man.
a. As a man He can be the mediator between God and man (1 Tim. 2:5).
2. Salvation by Grace
A. "For it is by grace you have been saved, through faith -- and this not from yourselves, it is
the gift of God -- not by works, so that no one can boast,” (Eph. 2:8-9, NIV).
B. "You who are trying to be justified by law have been alienated from Christ; you have fallen
away from grace,” (Gal. 5:4).
i. This verse and its context plainly teach that if you believe that you are saved by faith
and works then you are not saved at all. This is a common error in the cults. Because
they have a false Jesus, they have a false doctrine of salvation. (Read Rom. 3-5 and Gal.
3-5).
ii. You cannot add to the work of God. Gal. 2:21 says, "I do not set aside the grace of God,
for if righteousness could be gained through the law, Christ died for nothing!" (NIV)
C. "Therefore no one will be declared righteous in his sight by observing the law; rather,
through the law we become conscious of sin,” (Rom. 3:20).
i. "However, to the man who does not work but trusts God who justifies the wicked, his
faith is credited as righteousness,” (Rom. 4:5).
ii. "Is the law, therefore, opposed to the promises of God? Absolutely not! For if a law had
been given that could impart life, then righteousness would certainly have come by the
law,” (Gal. 3:21).
3. The Resurrection of Christ
A. "And if Christ has not been raised, our preaching is useless and so is your faith,” ( 1 Cor.
15:14). "And if Christ has not been raised, your faith is futile; you are still in your sins,” ( 1
Cor. 15:17).
B. To deny the physical resurrection is to deny Jesus' work, sacrifice, and our resurrection.
C. These verses clearly state that if you say that Jesus did not rise from the dead (in the same
body He died in -- John 2:19-21), then your faith is useless.

A comment on Gal. 1:8-9, "But even if we or an angel from heaven should preach a gospel other
than the one we preached to you, let him be eternally condemned! As we have already said, so now I
say again: If anybody is preaching to you a gospel other than what you accepted, let him be eternally
condemned!" (NIV).
These two verses here in Galatians could be considered a fourth self declarative statement of the
essentials. But, Gal. 1:8-9 is simply stating the necessity of believing the gospel message which, in its
entirety, is that Jesus is God in flesh, who died for sins, rose from the dead, and freely gives the gift of
eternal life to those who believe.
1 Cor. 15:1-4 defines what the gospel is: "Now, brothers, I want to remind you of the gospel I
preached to you, which you received and on which you have taken your stand. By this gospel you are
saved, if you hold firmly to the word I preached to you. Otherwise, you have believed in vain. For what
I received I passed on to you as of first importance: that Christ died for our sins according to the
Scriptures, that he was buried, that he was raised on the third day according to the Scriptures,” ( NIV).
Within these verses are the essentials: Christ is God in flesh (John 1:1,14; 10:30-33; 20:28; Col.
2:9); Salvation is received by faith (John 1:12; Rom. 10:9-10), therefore it is by grace; and the
resurrection is mentioned in verse 4. Therefore, this gospel message automatically includes the
essentials.
Who is God?

1. God Is One - Deut. 6:4; 1 Cor. 8:4


2. God Is Truth -Psalm 117:2; Jer. 10:10
3. God is Light - 1 John 1:5
4. God is Love - 1 John 4:8,16
5. God Is Infinite - Jer. 23:24; Psalm 147:5
6. God is All Knowing - 1 John 3:20
7. God is Everywhere - Psalm 139:7-12
8. God is All Powerful - Jer. 32:17,27
9. God is Unequaled - Isaiah 40:13-25
10. God Is Perfect - 1 Kings 8:27; Psalm 139
11. God Is A Most Pure Spirit - John 4:24
12. God Is Invisible - 1 Tim. 1:17
13. God Does Not Have A Human Body - Luke 24:39; Deut. 4:15-16
14. God Does Not Change - Num. 23:19; Mal. 3:6; James 1:17
15. God Is Without Limit - 1 Kings 8:27; Jer. 23:23-24
16. God Is Eternal - Psalm 90:2; 1 Tim. 1:17
17. God Is Incomprehensible - Rom. 11:33; Psalm 145:3
18. God Is The Almighty One - Rev. 1:8, 4:8
19. God Is Most Wise - Rom. 16:27; Jude 25
20. God Is Most Holy - Isaiah 6:3; Rev. 4:8
21. God Is Most Free - Psalm 115:3
22. God Is Most Absolute - Isaiah 44:6; Acts 17:24-25
23. God Works According To His Will - Eph. 1:11; Rom. 8:28
24. God Receives Glory - Rom. 8:36; Rev. 4:11
25. God Is Most Loving - 1 John 4:8-10
26. God Is Gracious - Exodus 33:19; 1 Pet. 2:3
27. God Is Merciful - Exodus 34:6; Psalm 67:1; James 5:11
28. God Is Long-suffering - Psalm 86:15; 2 Pet. 3:15
29. God Abounds In Goodness - Psalm 31:19; 52:1; Rom. 11:22
30. God Is Forgiving - Dan. 9:9; Eph. 1:7; Psalm 86:5
31. God Rewards Those Who Seek Him - Heb. 11:6
32. God Is Just In All His Judgments - Neh. 9:32-33; 2 Thess. 1:6
33. God Hates Sin - Psalm 5:5-6; Hab. 1:13
34. God Is the Creator - Isaiah 40:12,22,26
35. God Is Shepherd - Gen. 49:24
The Holy Spirit
The Holy Spirit is the third person in the Trinity. He is fully God. He is eternal, omniscient,
omnipresent, has a will, and can speak. He is alive. He is a person. He is not particularly visible in the
Bible because His ministry is to bear witness of Jesus (John 5:26).
Some cults like the Jehovah's Witnesses say that the Holy Spirit is nothing more than a force
(Reasoning from the Scriptures, 1985, pp. 406-407). This is false. If the Holy Spirit were merely a
force, then He could not speak (Acts 13:2); He could not be grieved (Eph. 4:30); and He would not
have a will (1 Cor. 12:11).
The truth is that the Holy Spirit is a person the same as the Father and the Son are within the
Trinity.

Power in
His Names His Attributes Symbols of Sins Against
Christ's Life
God Eternal Dove Blasphemy Conceived of
Acts 5:3-4 Heb. 9:14 Matt. 3:15 Matt. 12:31 Matt. 1:18,20
Lord Omnipotent Wind Resist (Unbelief) Baptism
2 Cor. 3:18 Luke 1:35 John 3:5 Acts 7:51 Matt. 3:15
Spirit Omnipresent Fire Insult Led by
1 Cor. 2:10 Psalm 139:710 Acts 2:3 Heb. 10:29 Luke 4:1
Spirit of God Will Lied to Filled with Power
*****
1 Cor. 3:16 1 Cor. 12:11 Acts 5:3 Luke 4:14,18
Spirit of Truth Loves Grieved Witness of Jesus
*****
John 15:26 Rom. 15:30 Eph. 4:30 John 15:26
Eternal Spirit Speaks Quench Raised Jesus
*****
Heb. 9:14 Acts 8:29; 13:2 1 Thess. 5:19 Rom. 8:11

The Works of the Holy Spirit

Access to God - Eph. 2:18 Inspires prayer - Eph. 6:18; Jude 20


Anoints for Service - Luke 4:18 Intercedes -Rom. 8:26
Assures - Rom. 8:15-16; Gal. 4:6 Interprets Scripture - 1 Cor. 2:1,14;
Authors Scripture - 2 Pet. 1:20-21 Leads - Rom. 8:14
Baptizes - John 1:232-34; 1 Cor. 12:13-14 Liberates - Rom. 8:2
Believers Born of - John 3:3-6 Molds Character - Gal. 5:22-23
Calls and Commissions - Acts 13:24; 20:28 Produces fruit - Gal. 5:22-23
Cleanses - 2 Thess. 3:13; 1 Pet. 1:2 Empowers Believers - Luke 24:49
Convicts of sin - John 16:9,14 Raises from the dead - Rom. 8:11
Creates - Gen. 1:2; Job 33:4 Regenerates - Titus 3:5
Empowers - 1 Thess. 1:5 Sanctifies - Rom. 15:16
Fills - Acts 2:4; 4:29-31; 5:18-20 Seals - Eph. 1:1314; 4:30
Strengthens - Eph. 3:16; Acts 1:8; 2:4;
Gives gifts - 1 Cor. 12:8-11
1 Cor. 2:4
Glorifies Christ - John 16:14 Teaches - John 14:26
Guides in truth - John 16:13 Testifies of Jesus - John 15:26
Helps our weakness - Rom. 8:26 Victory over flesh - Rom. 8:2-4; Gal. 4:6
Indwells believers - Rom. 8:9-14; Gal. 4:6 Worship helper - Phil. 3:3
The Atonement

Why is the atonement necessary? Because God is holy and we are not. Follow the outline below
which leads us through scriptures that demonstrate the reason, the problem, and the solution for our
sin problem.

1. God - is the standard of righteousness


A. Is holy (1 Sam. 2:2; Isaiah 43:3,14,15; Rev. 4:8)
B. Just (Deut. 32:4; Psalm 89:14; 97:2; 145:17)
C. Righteous (Psalm 145:17)
D. Judge (Psalm 50:6; 96:10,13; Isaiah 33:3-4)
E. Visits wrath on the ungodly (Rom. 1:18)
F. Too pure to look upon evil (Hab. 1:13)
2. The Law - is a reflection of His character
A. Comes from God (Exodus 20:1-26; Isaiah 33:22; James 4:12)
B. Is holy (Rom. 7:12)
C. Is covenantal (Deut. 4:13,23)
D. Inaugurated with blood (Heb. 9:18-23)
E. Brings the knowledge of sin (Rom. 3:20)
F. Is perfect (Psalm 19:7)
G. Cannot make man perfect (Heb. 7:19; 10:1)
3. Man - is the Sinner or Law Breaker
A. Sin is breaking the Law of God (1 John 3:4)
B. Man is a law breaker (Rom. 3:23)
C. Original Sin - Our inherited sinful nature from Adam (Gen. 3:1-6; Rom. 5:12)
D. Human nature - We are by nature children of wrath because we are sinners (Eph. 2:3)
i. Heart is wicked (Jer. 17:9; Mark 7:21-23)
ii. Spiritually blind (1 Cor. 2:14)
iii. Does not seek for God (Rom. 3:11).
iv. Is lawless, rebellious, unholy, and profane (1 Tim. 1:9).
v. Suppresses the truth of God in unrighteousness (Rom. 1:18)
vi. Futile in heart and mind (Rom. 1:21 )
vii. Man is at enmity with God (Rom. 5:10)
4. Judgment - is God's lawful action upon the sinner
A. God punishes evil (Exodus 20:5; Isaiah 11:4)
B. According to the Law (Deut. 29:21; Joshua 8:34; Rev. 21:8)
C. Eternal punishment (Matt. 3:12; Rev. 14:11)
D. Separation from God (Isaiah 59:2)
5. Reconciliation - Man's Need before God
A. Reconciliation is the means God has ordained to make peace between Him and us.
B. We need our sin removed.
C. We need to regain fellowship with God.
D. We need to find God's favor.
E. We need to escape God's lawful judgment.
6. Atonement - The Means of Reconciliation
A. The Nature of the Atonement is in the shedding of blood (Lev. 17:11)
B. Law requirements of the atonement
C. The sacrifice must be unblemished (Lev. 22:19)
D. By appointed priests (1 Sam. 2:28)
E. The High Priest had to be lawfully clean (Exodus 29:1-9;19-35)
F. Jesus as the Atonement, the Sacrifice
G. Unblemished (1 Pet. 1:19)
H. According to the Law (Heb. 9:22; Lev. 17:11 )
I. As the High Priest (Heb. 4:14; 6:20)
J. Substitutionary (1 Pet. 2:24; Isaiah 53; Eph. 5:2)
K. Our propitiation - He removed God's wrathful judgment (1 John 2:2; 4:10)
L. Jesus as God and Man (Col. 2:9; Phil. 2:5-8)
M. Man - to atone for men (Heb. 2:14; 5:1)
N. God - to offer an infinite and satisfactory sacrifice to God (Eph. 5:2,10)
7. Justification - Result of the Atonement
A. We are lawfully righteous before God (Rom. 3:24-26)
B. We are clothed in righteousness (Isaiah 61:10)
C. We have Imputed righteousness (Rom. 4:6)
i. Active - Christ's obedience to the Law and his fulfillment of it ( Rom. 8:3-4)
ii. Passive - Christ being led to the cross to atone for us (John 19:16-18; 1 Pet. 2:24)
D. We escape the judgment of God (Rom. 8:1)
E. We are restored to fellowship (1 Thess. 5:9-10)
F. We are at peace with God (Rom. 5:1)
G. We are reconciled to God (2 Cor. 5:19)
H. We are righteous before God (2 Cor. 5:21)
I. We have access to God (Eph. 2:18)
J. We have an advocate with the Father (1 John 2:1)
Verses showing justification by faith

Justification is the legal act where God declares the sinner to be innocent of his or her sins. It is
not that the sinner is now sinless, but that he is "declared" sinless. This declaration of righteousness is
being justified before God. This justification is based on the shed blood of Jesus, "...having now been
justified by His blood..." (Rom. 5:9) where Jesus was crucified, died, was buried, and rose again (1
Cor. 15:1-4). God imputes (reckons to our account) the righteousness of Christ; at the same time our
sins were imputed to Christ when he was on the cross. That is why it says in 1 Pet. 2:24 , "and He
Himself bore our sins in His body on the cross, that we might die to sin and live to righteousness; for
by His wounds you were healed." Also, 2 Cor. 5:21 says, "He made Him who knew no sin to be sin on
our behalf, that we might become the righteousness of God in Him." Additionally, we are justified by
faith (Rom. 5:1) apart from works of the Law (Rom. 3:28).
To be saved means that God has delivered us (saved us) from His righteous wrathful judgment
due us because of our sins against Him. It means that we will not be judged for our sins and be
sentenced to eternal damnation. To be saved means that we are justified before God. Only Christians
are saved. Only Christians are justified. The issue at hand is whether or not this salvation, this
justification, is attained by faith or by faith and something else.
Following is a list of verses that show that salvation/justification is by faith. Bold references are
particularly pointed.

1. John 3:16, "For God so loved the world that He gave His only begotten Son, that whoever
believes in Him should not perish, but have eternal life."
2. Rom. 3:22, "even the righteousness of God through faith in Jesus Christ for all those who
believe; for there is no distinction."
3. Rom. 3:24, "being justified as a gift by His grace through the redemption which is in Christ
Jesus;"
4. Rom. 3:26, "for the demonstration, I say, of His righteousness at the present time, that He
might be just and the justifier of the one who has faith in Jesus."
5. Rom. 3:28-30, "For we maintain that a man is justified by faith apart from works of the Law.
29
Or is God the God of Jews only? Is He not the God of Gentiles also? Yes, of Gentiles also,
30
since indeed God who will justify the circumcised by faith and the uncircumcised through
faith is one."
6. Rom. 4:3, “For what does the Scripture say? "And Abraham believed God, and it was reckoned
to him as righteousness."
7. Rom. 4:5, "But to the one who does not work, but believes in Him who justifies the ungodly,
his faith is reckoned as righteousness,"
8. Rom. 4:11, "And he received the sign of circumcision, a seal of the righteousness of the faith
which he had while still uncircumcised, that he might be the father of all those who believe,
though they are uncircumcised, that righteousness might be imputed to them also,"
9. Rom. 4:16, "Therefore it is of faith that it might be according to grace, so that the promise
might be sure to all the seed, not only to those who are of the law, but also to those who are
of the faith of Abraham, who is the father of us all."
10. Rom. 5:1, "therefore having been justified by faith, we have peace with God through our Lord
Jesus Christ,"
11. Rom. 5:9, "Much more then, having now been justified by His blood, we shall be saved from
the wrath of God through Him."
12. Rom. 9:30, "What shall we say then? That Gentiles, who did not pursue righteousness, attained
righteousness, even the righteousness which is by faith."
13. Rom. 10:4, "For Christ is the end of the law for righteousness to everyone who believes."
14. Rom. 10:9-10, "that if you confess with your mouth Jesus as Lord, and believe in your heart
that God raised Him from the dead, you shall be saved; 10 for with the heart man believes,
resulting in righteousness, and with the mouth he confesses, resulting in salvation."
15. Gal. 2:16, "nevertheless knowing that a man is not justified by the works of the Law but
through faith in Christ Jesus, even we have believed in Christ Jesus, that we may be justified
by faith in Christ, and not by the works of the Law; since by the works of the Law shall no
flesh be justified."
16. Gal. 3:5-6, "Does He then, who provides you with the Spirit and works miracles among you, do
it by the works of the Law, or by hearing with faith? 6Even so Abraham believed God, and it
was reckoned to him as righteousness."
17. Gal. 3:8, "And the Scripture, foreseeing that God would justify the Gentiles by faith, preached
the gospel beforehand to Abraham, saying, "All the nations shall be blessed in you."
18. Gal. 3:14, "in order that in Christ Jesus the blessing of Abraham might come to the Gentiles, so
that we might receive the promise of the Spirit through faith."
19. Gal. 3:22, "But the Scripture has shut up all men under sin, that the promise by faith in Jesus
Christ might be given to those who believe."
20. Gal. 3:24, "Therefore the Law has become our tutor to lead us to Christ, that we may be
justified by faith."
21. Eph. 1:13, "In Him, you also, after listening to the message of truth, the gospel of your
salvation—having also believed, you were sealed in Him with the Holy Spirit of promise."
22. Eph. 2:8, "For by grace you have been saved through faith; and that not of yourselves, it is the
gift of God."
23. Phil. 3:9, "and may be found in Him, not having a righteousness of my own derived from the
Law, but that which is through faith in Christ, the righteousness which comes from God on
the basis of faith."
24. 1 Tim. 1:16, "And yet for this reason I found mercy, in order that in me as the foremost, Jesus
Christ might demonstrate His perfect patience, as an example for those who would believe in
Him for eternal life."

James 2:24, not by faith alone

It should be clear that we are saved (justified) by faith in with Christ has done on the cross. This
faith alone saves us. However, James 2:24 says, "You see that a man is justified by works, and not by
faith alone."
James chapter 2 has 26 verses: Verses James 1:1-7 instruct us to not show favoritism. Verses 8-
13 are comments on the Law. Verses 14-26 are about the relationship between faith and works.
James begins this section by using the example of someone who says he has faith, verses 14. He
then immediately gives an example of what true and false faiths are. He begins with the negative and
demonstrates what an empty faith is (verses 15-17). Then he shows that that type of faith isn't much
different from the faith of demons (verse 19). Finally, he gives examples of living faith by showing
Abraham and Rahab as examples of people who demonstrated their faith by their deeds.
James is examining two kinds of faith: one that leads to godly works and one that does not. One is
true, and the other is false. One is dead, the other alive; hence, "Faith without works is dead," ( James
2:20).
Also, notice that James actually quotes the same verse that Paul uses to support the teaching of
justification by faith in Rom. 4:3. James 2:23 says, "and the Scripture was fulfilled which says, ‘and
Abraham believed God, and it was reckoned to him as righteousness.'" If James was trying to teach a
contradictory doctrine of faith and works than the other New Testament writers, then he would not
have used Abraham as an example.

Conclusion

Justification is by faith. True faith results in regeneration of the sinner which, in turn, results in
good works. But it is not these works that earn our place with God. That was accomplished by Jesus
on the cross. All that we need, we have in Jesus. All we need to do to be saved, to be justified, is to
truly believe in want God has done for us in Jesus on the cross. This true belief will result in
justification and regeneration, which results in good works.

The law and the gospel

The Law is the do’s and don’t’s of moral behavior. God gave the Law so that people would have a
guide to live by and a standard by which they might recognize God’s purity and their sinfulness. There
are 613 commandments in the OT. They oversee moral, judicial, and religious behavior.
The Law is a reflection of the character of God because the Law comes forth from the very heart of
God. The Bible says out of the abundance of the heart the mouth speaks (Matt. 12:34). When God
gave the Law, He was speaking out of the abundance of His heart. He was speaking from what was in
Him. Therefore, the Law is good, pure, right, and holy. It is wrong to lie, because it is against God’s
nature to lie. It is wrong to steal because it is against God’s nature to steal.
This Law, then, by its very nature of coming out of the heart of God, and being spoken to men, is a
standard for human conduct, a perfect standard. Because it was perfect, and we are not, it is
impossible for sinful people to keep. It was for this reason that the Law became a stumbling block. It
became an obstacle to Man because it is an unattainable perfect standard. The Law, then, brings about
the opposite of what it requires. The Law says to be perfect, but shows you where you are not. It says
to be holy but condemns you when you are not. Since it is not possible for us to keep the Law and
therefore earn our position with God, we then need the holiness of God given to us -- because there
simply isn’t any way for us to attain to the standard of God. Therefore, "...the law was put in charge to
lead us to Christ that we might be justified by faith," (Gal. 3:24). That is, the Law shows us that we
can’t get to God by what we do. We need the grace of God in Christ Jesus manifested in His sacrifice.

1. The Law reveals our sinfulness.


A. "Therefore no one will be declared righteous in his sight by observing the law; rather,
through the law we become conscious of sin," (Rom. 3:20).
B. "What shall we say, then? Is the law sin? Certainly not! Indeed I would not have known what
sin was except through the law. For I would not have known what coveting really was if the
law had not said, ‘Do not covet,’" (Rom. 7:7).
2. The Law is for those who are not under grace.
A. "Now we know that whatever the law says, it says to those who are under the law, so that
every mouth may be silenced and the whole world held accountable to God," (Rom. 3:19).
B. "For sin shall not be your master, because you are not under law, but under grace," (Rom.
6:14).
3. The Law justifies no one.
A. "Therefore no one will be declared righteous in his sight by observing the law," (Rom. 3:20).
4. The Law makes no concessions; it makes demands.
A. "Cursed is every man who does not abide by everything written in the book of the law to
perform them," (Gal. 3:10).
5. The Law is spiritual: It works on the Spirit, not on the body.
A. "For we know that the Law is spiritual, but I am of the flesh," (Rom. 7:14).
B. "Thou shalt not..." applies to the heart, not the body.
6. We are made righteous in God’s eyes by grace apart from the Law of God.
A. “For we maintain that a man is justified by faith apart from observing the law," (Rom. 3:28).
B. "Therefore, since we have been justified through faith, we have peace with God through our
Lord Jesus Christ," (Rom. 5:1).
C. "knowing that a man is not justified by observing the law, but by faith in Jesus Christ. So
we, too, have put our faith in Christ Jesus that we may be justified by faith in Christ and not
by observing the law, because by observing the law no one will be justified," (Gal. 2:16).
7. The Law brings judgment.
A. "...because law brings wrath," (Rom. 4:15).
8. The Law prepares us for the gospel
A. The Law shows us that the free gift of the gospel is the only way to attain righteousness.
B. "The law was put in charge to lead us to Christ that we might be justified by faith," (Gal.
3:24).
i. Being saved by grace through faith (Eph. 2:8) is only found in the Christian religion.
Only Christianity has the message of free, unearned, grace.
9. The Law is for the ungodly.
A. "But we know that the Law is good, if one uses it lawfully, realizing the fact that law is not
made for a righteous man, but for those who are lawless and rebellious, for the ungodly and
sinners, for the unholy and profane, for those who kill their fathers or mothers, for
murderers and immoral men and homosexuals and kidnappers and liars and perjurers, and
whatever else is contrary to sound teaching, according to the glorious gospel of the blessed
God," (1 Tim. 1:8-11).
10. The Law differs from the gospel in:
A. The manner of revelation.
i. The Law is revealed in the hearts of man.
a. "For when Gentiles who do not have the Law do instinctively the things of the Law,
these, not having the Law, are a law to themselves, in that they show the work of
the Law written in their hearts..." (Rom. 2:14-15).
b. It would be impossible to convert anyone if the Law had not been written on their
hearts because the Law reveals sin, (Rom. 3:20).
ii. The gospel is by direct revelation; it is not written on the heart.
a. "Now, brothers, I want to remind you of the gospel I preached to you, which you
received and on which you have taken your stand," (1 Cor. 15:1).
B. Contents.
i. The Law tells what people are to do (our works). It makes demands (Deut. 27:26).
ii. The Gospel reveals what God is doing (God’s work). Therefore, it makes no demands on
us except faith (Rom. 6:23).
iii. The Law is the list of do’s and don’t’s (Exodus 20)
iv. The Gospel is the death, burial, and resurrection of Jesus for sins (1 Cor. 15:1-4).
a. It contains grace and truth (John 1:17) because the gospel is about Jesus.
C. Promises
i. The Law and the gospel both promise eternal life:
a. The Law by complete obedience to all its commands (Lev. 18:5; Luke 10:26).
b. The gospel by grace unconditionally (Rom. 3:22-24, Eph. 2:8-9). It demands
nothing, makes no threats. It removes from sinners the desire to sin.
11. Effects of preaching the Law.
A. It tells us what to do, but does not enable us to do it. This can frustrate us because we
cannot keep it!
B. Reveals to man his sins. It offers no help to get him out; hurls man into despair.
i. "...I would not have come to know sin except through the Law; for I would not have
known about coveting if the Law had not said, ‘You shall not covet,’" (Rom. 7:7).
C. It brings to our awareness damnation, hell, and hopelessness.
i. "But your iniquities have separated you from your God; your sins have hidden his face
from you, so that he will not hear," (Isaiah 59:2).
ii. "Christ redeemed us from the curse of the law by becoming a curse for us, for it is
written: ‘Cursed is everyone who is hung on a tree,’" (Gal. 3:13).
12. Effects of preaching the gospel
A. It demands faith and gives it to us.
i. "Faith comes by hearing and hearing by the word of Christ," (Rom. 10:17).
B. It does not reprove the sinner.
i. "Therefore, there is now no condemnation for those who are in Christ Jesus," (Rom.
8:1).
C. It does not require anything good for man to do, either in heart, mind or body because it is a
free gift.
i. "For the wages of sin is death, but the gift of God is eternal life in Christ Jesus our Lord,"
(Rom. 6:23).
13. Who the Law and the Gospel are preached to.
A. The Law is preached to sinners, those secure in their sin.
i. "But we know that the Law is good, if one uses it lawfully, realizing the fact that law is
not made for a righteous man, but for those who are lawless and rebellious, for the
ungodly and sinners, for the unholy and profane, for those who kill their fathers or
mothers, for murderers and immoral men and homosexuals and kidnappers and liars
and perjurers, and whatever else is contrary to sound teaching," (1 Tim. 1:8-10).
B. The Gospel is preached to those who are alarmed, frightened, smitten by the law; to those
who are made thirsty for the Gospel message.
i. "...through the law we become conscious of sin," (Acts 4:20).
ii. "So the law was put in charge to lead us to Christ that we might be justified by faith,"
(Gal. 3:24).
Salvation: What does it mean to be a Christian?
Theologically speaking, a Christian is someone who has received the Lord Jesus as Savior ( John
1:12), trusts Him alone for the forgiveness of sins (Acts 4:12), has put not trust in His own efforts
(Isaiah 64:6) to please God, and repented from his/her sins (Mark 1:15).
Experientially speaking, the life of a Christian does not consist only of theological knowledge. It is
theology that defines who Jesus is and what He has done, but it is not the end of all things. We are
Christians who believe the above points, yes, but we have a living and open relationship with the Lord
Jesus. We experience Him through His indwelling Spirit. As Christians, we seek to do the will of the
Lord, to follow in His footsteps, and to honor and glorify God in all he does.
It is not necessary as a Christian to perform good works in order to please God because, first of
all, our good deeds are but filthy rags to God (Isaiah 64:6) and, most important, we are made
righteous in the eyes of God by the finished work of Jesus on the cross (Rom. 5). This is one of the
areas where the cults error. They confuse good works with the forgiveness of sins. They combine the
two and teach that God will not accept us if we are not trying to be good. Because they have a wrong
view of who Jesus is, they have a wrong view of salvation.
A common objection to this doctrine of justification by faith is that if a Christian believes in God
the way I say, then he does not need to do anything good, that he could then go out and sin all he
wanted. First of all, this objection is answered in Romans 6. We are not saved for the purpose of
impurity, but in sanctification (1 Thess. 4:7). We do not use the grace of God to sin. Second, a
Christian is called to be Holy (1 Pet. 1:16). Third, a Christian is called to do good works (Eph. 2:10). It
is just that these works are not combined with our faith to merit the forgiveness of our sins; they are,
instead, a natural result of our saved condition. We do good works because we are Christians, not to
become Christians.
Additionally, being a Christian means that you are serving the true Jesus, not a false one. In order
for a person to follow Jesus, he must first accurately understand who He is. If someone called their pet
iguana Jesus, even though he had great faith in Jesus the iguana, his faith is useless. Faith is only as
good as the object in which it is placed.
The Mormon Jesus is the brother of the devil begotten through sexual intercourse from a god and
goddess who used to be people on another planet (Mormon Doctrine, by Bruce McConkie, p. 321). The
Jehovah's Witness Jesus is Michael the archangel who became a man, died on a torture stake, did not
rise from the dead in the same body he died in, and then went back to being an angel (Aid to Bible
Understanding, p. 1152; New Heavens and a New Earth, p. 30). The New Age Jesus is a man in tune
with the divine consciousness. In opposition to this, the Jesus of Christianity is both God and Man. See
the two natures of Jesus for more information on this.

Being a Christian Means Fellowship with Jesus

Why did God create? Was their some lack in God that moved Him to create the universe and man
in it? Was God lonely? We can't fully answer these questions, but we can look into the Bible for clues
to their answer.
1 John 4:8 says that God is love. John 3:16 says that "...God so loved the world He gave..." The
nature of love is to give. It is "other" centered. It focuses on another. Read 1 Cor. 13 for confirmation
of this. That is why God gave His Son. That is why, I believe, that God created us: to love us, to give
to us Himself which is the very best thing in the universe. But sin entered the picture and God, in His
loving mercy, sent His Son into the world to save the world.
Love is not a doctrine; it is an experience, an action. In the Garden of Eden, Adam and Eve walked
with God. They actually walked with the creator of the universe. They had fellowship with Him.
Fellowship is an intimate communion between two or more persons. Adam and Eve had this intimate
communion with the Lord. But when they sinned, that fellowship was broken. God then shed blood, by
killing an animal to get the skins, and covered Adam and Eve. Incidentally, Jesus said in John 6:46
that no one has ever seen the Father. If Adam and Eve were walking with God in the Garden of Eden,
but it wasn't the Father, then who was it. It must have been Jesus.
So God sought Adam and Eve, remember they hid themselves from Him. In Exodus 25:8, God told
the Israelites to build a sanctuary so that He might dwell among His people. In John 1:14, Jesus, God
in flesh, dwelt again among His people. In 1 Cor. 1:9 we are called by God to be in fellowship with
Jesus. In these statements are profound clues. We are called to have a personal relationship, the way
it was in the Garden of Eden, with Jesus. This can only be done through Jesus.
Additionally, the word for ‘fellowship' in the Greek is the same word used for ‘communion.' When
we partake of communion, we are partaking in fellowship with the Lord. Communion is a covenantal
sign of the promise of God to give us eternal life and it is representative in that sense of God's
promises to be with His people. But the real communion, the real fellowship with the Lord, is through
the indwelling Holy Spirit who always bears witness of Jesus (John 15:26). Therefore, the Christian,
the true Christian, will have an intimate and real personal relationship with the Lord Jesus.
The cultist cannot have this intimate and personal relationship with Jesus, first because their Jesus
is false (Matt. 24:24). Second, because their Jesus is not prayed to the way the Jesus of the Bible is
(Zech. 13:9 with 1 Cor. 1:1-2; Acts 7:55-60); third, because their Jesus is not worshiped equally with
the Father (John 5:22-23; Matt. 2:2,11; 14:33; 28:9; John 9:35-38; Heb. 1:6); and fourth, because
their Jesus is not their Lord and God (John 20:28; Heb. 1:8).
The Jesus of the cultist is not God (he might be one god among many, or he might be a lesser
god). Therefore, he is not to be sought in a personal and intimate way.
The Christian, on the other hand, has a real relationship with the real Lord Jesus. This is
accomplished only through the real Jesus, the Jesus of the Bible.
To be a Christian is to experience the Lord, to have a sweet and real fellowship with Jesus, to be
able to pray to Him, and seek Him.

"God is faithful, by whom you were called into the fellowship of His Son,
Jesus Christ our Lord," (1 Cor. 1:9).
Covenant
A covenant is a contract or agreement between two or more parties. Covenant is how God has
chosen to communicate to us, to redeem us, and to guarantee us eternal life in Jesus. These truths,
revealed in the Bible, are the basis of Christianity. The Bible is a covenant document. The Old and New
Testaments are really Old and New Covenants. The word "testament" is Latin for Covenant.
There is a pattern to the covenants found in the Bible. Basically, it is as follows. The initiating party
describes himself and what He has done, then there is a list of obligations between the two (or more)
parties. What follows is the section dealing with rewards and punishments that govern the keeping and
breaking of the covenant. The Ten Commandments fit this pattern and are a covenant document.
Covenant is how God first decided to deal with Mankind. We know this from studying the Eternal
Covenant mentioned in Heb. 13:20, "May the God of peace, who through the blood of the eternal
covenant brought back from the dead our Lord Jesus, that great Shepherd of the sheep" (NIV). In this
covenant God the Father and the Son made an agreement with regard to the elect. This covenant was
made before the universe was created and it consisted of the Father promising to bring to the Son all
whom the Father had given Him (John 6:39; 17:9,24). The Son would become man (Col. 2:9; 1 Tim.
2:5), become for a while lower than the angels (Heb. 2:7), and be found under the Law (Gal. 4:4-5).
The Son would die for the sins of the world (1 John 2:2; 1 Pet. 2:24) and the Father would raise the
Son from the Dead (Psalm 2).
The Eternal Covenant, then, leads to the Covenant of Grace. Where the Eternal Covenant was
made between the Father and the Son, the Covenant of Grace is made between God and Man. This
latter covenant is where God promises to Man eternal salvation based upon the sacrifice of Jesus on
the cross. The manifestation of that covenant occurs in our world in a sequence of additional covenants
that God made with individuals: Adam (Gen. 2:15-17), Noah (Gen. 9:12-16), Abraham (Gen. 17), the
Israelites at Mount Sinai (Exodus 34:28), believers in the New Covenant (Jer. 31:31-37), etc. I
present the view that there are two main covenants. However, there is disagreement as to the number
of Covenants. Some say there is really only one, the Eternal Covenant, with all others falling under it.
Some say two, some say three, and others four, etc. There really is no absolute answer.
Understanding Covenant is important for several reasons:

1. We learn that God deals with Man covenantally.


2. Since a Covenant is an agreement, it is a promise made by God. Since we
can rely on God's word for eternity, we can take great comfort in His covenant
promising us eternal life in His Son.
3. It helps us to see the Bible as a covenant document. The Old and New
Testaments are Old and New Covenants.
4. With Covenant understood as a framework through which the Bible was
written we can better understand it, God’s dealings with us through it, and our
responsibilities to God as well as His to us.
5. We can better understand the symbols used by God in covenant
ratification: The Lord’s Supper and Baptism.

1. Requirements and Promises in the Eternal Covenant


A. The Father required of the Son, that He should atone for the sins of those whom the Father
had given Him (1 John 2:2; John 6:39; 10:11,15), and should do what Adam failed to do by
keeping the law (Gal. 4:4-5; 1 Pet. 2:22).
B. This requirement included the following particulars:
i. That he should assume human nature (John 1:1,14; Col. 2:9).
ii. That He should place Himself under the law (Gal. 4:4-5)
iii. That He, after accomplishing forgiveness of sins and eternal life, should apply them to
the elect (Rom. 5:18; 1 Cor. 15:22; 2 Cor. 5:14).
2. The Relation of the Eternal Covenant and the Covenant of Grace
A. The Eternal Covenant is the model for the Covenant of Grace. The former is eternal, that is,
from eternity, and the latter temporal in the sense that it is realized in time. The former is a
compact between the Father and the Son as a surety and head of the elect, while the latter
is a compact between the triune God and the elect sinner.
i. If there had been no Eternal Covenant between the Father and the Son, there could
have been no Covenant of Grace between God and sinful man.
ii. The Holy Spirit, which produces faith in the sinner, was promised to Christ by the
Father, and the acceptance of the way of life through faith was guaranteed by Christ.
3. The Covenant with Adam also known as the Covenant of Works
A. This was a covenant made between God and Adam where Adam would have everlasting life
based upon obedience to God. This apparently was possible since Adam did not have a sin
nature.
i. "And the LORD God commanded the man, ‘You are free to eat from any tree in the
garden; but you must not eat from the tree of the knowledge of good and evil, for when
you eat of it you will surely die,’" (Gen. 2:16-17).
B. God entered into a covenant with Adam.
i. The promise connected to that covenant was life. The condition was perfect obedience.
Its penalty was death.
4. The Covenant with Noah
A. This covenant was God’s promise to Noah to never again destroy the world with a flood. God
gave the rainbow as a sign.
i. "I now establish my covenant with you and with your descendants after you and with
every living creature that was with you -- the birds, the livestock and all the wild
animals, all those that came out of the ark with you -- every living creature on earth. I
establish my covenant with you: Never again will all life be cut off by the waters of a
flood; never again will there be a flood to destroy the earth." And God said, "This is the
sign of the covenant I am making between me and you and every living creature with
you, a covenant for all generations to come: I have set my rainbow in the clouds, and it
will be the sign of the covenant between me and the earth. Whenever I bring clouds
over the earth and the rainbow appears in the clouds, I will remember my covenant
between me and you and all living creatures of every kind. Never again will the waters
become a flood to destroy all life. Whenever the rainbow appears in the clouds, I will see
it and remember the everlasting covenant between God and all living creatures of every
kind on the earth." So God said to Noah, ‘This is the sign of the covenant I have
established between me and all life on the earth,’" (Gen. 9:9-17).
5. The Covenant with Abraham
A. God promised a land and descendants to Abraham, who was commanded to "keep" the
covenant (Gen. 17:9f., 14) and was given circumcision as the sign (Gen. 15:8-18; 17:1-14).
i. On that day the LORD made a covenant with Abram and said, "To your descendants I
give this land, from the river of Egypt to the great river, the Euphrates," (Gen. 15:18).
6. The Covenant with Moses
A. In the giving of the Law, the nation of Israel was constituted a holy nation and given
stipulations to follow to ensure fellowship with God. The covenant was ratified by a covenant
sacrifice and the sprinkling of blood (Exodus 24:4-8).
B. "Moses then wrote down everything the LORD had said. He got up early the next morning
and built an altar at the foot of the mountain and set up twelve stone pillars representing
the twelve tribes of Israel. 5Then he sent young Israelite men, and they offered burnt
offerings and sacrificed young bulls as fellowship offerings to the LORD. 6Moses took half of
the blood and put it in bowls, and the other half he sprinkled on the altar. 7Then he took the
Book of the Covenant and read it to the people. They responded, ‘We will do everything the
LORD has said; we will obey.’ 8Moses then took the blood, sprinkled it on the people and
said, ‘This is the blood of the covenant that the LORD has made with you in accordance with
all these words,’" (Exodus 24:4-8, NIV).

7. The Covenant with David


A. God gave a promise to David that his descendants should have an everlasting kingdom and
be known as his sons.
i. "You said, ‘I have made a covenant with my chosen one, I have sworn to David my
servant, I will establish your line forever and make your throne firm through all
generations,’" (Psalm 89:3).
ii. It was through the descendants of David that Jesus was born.
8. The New Covenant
A. This is the new covenant of the Messianic age where the Law of God would be written upon
the hearts of men.
i. "The time is coming," declares the LORD, "when I will make a new covenant with the
house of Israel and with the house of Judah...This is the covenant I will make with the
house of Israel after that time," declares the LORD. "I will put my law in their minds and
write it on their hearts. I will be their God, and they will be my people," (Jer. 31:31,33).
B. It was promised in Eden
i. "And I will put enmity between you and the woman, and between your offspring and
hers; he will crush your head, and you will strike his heel" (Gen. 3:15).
C. It was proclaimed to Abraham
i. "I will bless those who bless you, and whoever curses you I will curse; and all peoples
on earth will be blessed through you," (Gen. 12:3).
D. It was fulfilled in Christ
i. "Praise be to the Lord, the God of Israel, because he has come and has redeemed his
people. He has raised up a horn of salvation for us in the house of his servant David (as
he said through his holy prophets of long ago), salvation from our enemies and from the
hand of all who hate us -- to show mercy to our fathers and to remember his holy
covenant, the oath he swore to our father Abraham: to rescue us from the hand of our
enemies, and to enable us to serve him without fear in holiness and righteousness
before him all our days. And you, my child, will be called a prophet of the Most High; for
you will go on before the Lord to prepare the way for him, to give his people the
knowledge of salvation through the forgiveness of their sins, because of the tender
mercy of our God, by which the rising sun will come to us from heaven to shine on those
living in darkness and in the shadow of death, to guide our feet into the path of peace,"
(Luke 1:68-79).
9. The Covenant of Grace
A. This may be defined as that gracious agreement between the offended God and the
offending sinner, in which God promises salvation through faith in Christ, and the sinner
accepts this by faith, promising a life of faith and obedience ( John 1:12-13; 3:16; Rom.
10:9-10).
10. Comparison of the Covenant of Works (the Adamic Covenant) and the Covenant of
Grace

Covenant of Works Covenant of Grace

God is the Creator and Lord. Established God is the Redeemer and Father. Established
because of His love and benevolence. because of His mercy

Man appears simply as God’s creature, Man appears as a sinner who has perverted his
rightly related to his God ways, and can only appear in union with Christ
and grace.

No Mediator Jesus is Mediator

Righteousness is based upon the Based on the obedience of Christ as Mediator


obedience of a changeable man which is which is absolute and certain.
uncertain.

The way of life is by keeping the Law. The way of life is by faith in Jesus Christ.

The covenant is partly known in nature, The covenant is known exclusively through
since the law of God is written in the special revelation: the Bible.
heart of Man.

Just as in the covenant of works, so in the covenant of grace God is the first of the contracting
parties; He takes the initiative and determines the relation in which the second party will stand to Him.
It is not easily determined who the second party is. But in general, it may be said that God
naturally established the covenant of grace with fallen man.
The idea that the covenant is fully realized only in the elect is a perfectly scriptural idea, as
appears, for instance, from Jer. 31:21-34; Heb. 8:8-12. It is also entirely in line with the relation in
which the Covenant of Grace stands to the Eternal Covenant.
The Two Main Covenants
God works covenantally in the Bible. A covenant is a pact or agreement between two or more
parties. In fact, the Latin word "testamentum" is the word "covenant." Following is a chart that
represents a two-division analysis of covenant. There are different interpretations of how many
covenants there are, but I lean towards two.
Therefore, following is a brief chart demonstrating the breakdown of the two main covenants.

THE TWO MAIN COVENANTS


ETERNAL COVENANT ALSO CALLED
COVENANT
THE COVENANT OF REDEMPTION COVENANT OF GRACE
NAME
(HEB. 13:20)
PARTIES
Father and Son (Inter Trinitarian) God and Man
INVOLVED
The Father would...

• Prepare the Son a body (Luke 1:35; Heb.


10:5).
• Give the Son the Spirit without measure
(Isaiah 43:1,2; 61:1). Eternal life for mankind
• Always support and comfort the Son
(Isaiah 49:1-7; 49:8). The complete restoration of
PROMISE
• Deliver the Son from the power of Death fellowship between God and man.
(Psalm 2; Psalm 16:10).
• Bring to the Son all whom the Father had
given Him (John 6:39; 17:9,24).

• Give the Son a number of redeemed that


no one could number (Psalm 22:27;
72:17).
The Son...

• Was to assume human nature (Gal. 4:4,5;


Heb. 2:10,44,14,15).
CONDITION Faith in Jesus Christ
• Was to be under the Law (Psalm 40:8;
Gal. 4:4,5; Phil. 2:5-8).

• Was to bear our sins (1 Pet. 2:24).

MEDIATOR No Mediator Jesus Christ


Divisions under the covenant of grace
God works covenantally. A covenant is a pact or agreement between two or more parties.
The chart below helps you to see some of the major covenants made between God and man. This
is important because it helps us see that God is a keeper of His word. That is, since a covenant is an
agreement, God cannot break His word, His covenant. Furthermore, God has signs to His covenants.
They help us to remember the promises and faithfulness of God.

Some Divisions Under the Covenant of Grace


COVENANT
ADAMIC NOAHIC ABRAHAMIC SINAITIC DAVIDIC NEW
NAME
God and God and God and God and the God and God and the
Adam; Noah Abraham Israelites at David Elect
therefore all (his Mt. Sinai
PARTIES
Mankind (his descendants)
INVOLVED
descendants
and all living
creatures)
Eternal Life To not Land and Continued His Law written on
based on destroy the descendants Fellowship descendants their hearts. God
keeping world again to Abraham with God would have will be their
THE
God’s word. with a flood (Gen. 15:8- (Exodus an people
PROMISE
(Gen. 9:8- 18; 17:1-14) 34:27-28; everlasting
18) 19:5; 24:4- kingdom (Jer. 31:31-33)
8) Psalm 89:3)
THE Perfect No Condition Circumcision Perfect No Condition Faith in Jesus
CONDITION Obedience Obedience
The Tree A Rainbow Circumcision A sacrifice None The Blood of
and because it Christ (Baptism
SIGN AND sprinkling of was to be and the Lord’s
SEAL blood fulfilled in Supper)
(Exodus Jesus’
24:4-8) sacrifice.

MEDIATOR No Mediator The Messiah Jesus Christ


Jesus' Two Natures
Jesus is God in human flesh. He is not half God and half man. He is fully God and fully man. At the
incarnation He added to His divine nature the nature of man. Thus He has two natures: divine and
human. He is both God and man at the same time. He is not merely a man who "had God within Him"
nor is he a man who "manifested the God principle." He is God, second person of the Trinity. "The Son
is the radiance of God's glory and the exact representation of his being, sustaining all things by his
powerful word," (Heb. 1:3, NIV). Jesus' two natures are not "mixed together," nor are they combined
into a new God-man nature. They are separate yet act as a unit. This is called the Hypostatic Union.
The following chart should help you see the two natures of Jesus "in action":

GOD MAN
He is worshiped (Matt. 2:2,11; 14:33). He worshiped the Father (John 17).
He was called God (John 20:28; Heb. 1:8) He was called man (Mark 15:39; John 19:5).
He was called Son of God (Mark 1:1) He was called Son of Man (John 9:35-37)
He is prayed to (Acts 7:59). He prayed to the Father (John 17).
He is sinless (1 Pet. 2:22; Heb. 4:15). He was tempted (Matt. 4:1).
He knows all things (John 21:17). He grew in wisdom (Luke 2:52).
He gives eternal life (John 10:28). He died (Rom. 5:8).
All the fullness of deity dwells in Him (Col. 2:9). He has a body of flesh and bones (Luke 24:39).

One of the most common errors that non-Christian cults make is not understanding the two
natures of Christ. For example, the Jehovah's Witnesses focus on Jesus' humanity and ignore His
divinity. The repeatedly quote verses dealing with Jesus as a man and try and set them against
scripture showing that Jesus is also divine. On the other hand, the Christian Scientists do the reverse.
They focus on the scriptures showing Jesus' divinity to the extent of denying His true humanity.
For a proper understanding of Jesus and, therefore, all other doctrines that relate to Him, His two
natures must be properly understood and defined. Jesus is one person with two natures. This is why
He would grow in wisdom and stature (Luke 2:52) yet know all things (John 21:17). He is the Divine
Word that became flesh (John 1:1,14).
The Bible is about Jesus (John 5:39). The prophets prophesied about Him (Acts 10:43). The Father
bore witness of Him (John 5:37; 8:18). The Holy Spirit bore witness of Him (John 15:26). The works
Jesus did bore witness of Him (John 5:36; 10:25). The multitudes bore witness of Him (John 12:17).
And, Jesus bore witness of Himself (John 14:6; 18:6).
Other verses to consider when examining His deity are John 10:30-33; 20:28; Col. 2:9; Phil. 2:5-
8; Heb. 1:6-8; and 2 Pet. 1:1.
1 Tim. 2:5 says, "For there is one God, and one mediator also between God and men, the man
Christ Jesus." Right now, there is a man in heaven on the throne of God. He is our advocate with the
Father (1 John 2:1). He is our Savior (Titus 2:13). He is our Lord (Rom. 10:9-10). He is Jesus.
Bible verses that show Jesus is God
Following are verses used to show that Jesus is God in flesh. The scriptures used here are from the
New American Standard Bible. The links to the verses are to the King James Version of the Bible here
on CARM.

1. John 1:1, "In the beginning was the Word, and the Word was with God, and the Word was God."
A. John 1:14, "And the Word became flesh, and dwelt among us, and we beheld His glory,
glory as of the only begotten from the Father, full of grace and truth."
2. John 5:18, "For this cause therefore the Jews were seeking all the more to kill Him, because He
not only was breaking the Sabbath, but also was calling God His own Father, making Himself
equal with God."
3. John 8:24, "I said therefore to you, that you shall die in your sins; for unless you believe that I
am He, you shall die in your sins."
A. Note: In the Greek, "He" is not there.
4. John 8:58, - "Jesus said to them, 'Truly, truly, I say to you, before Abraham was born, I am.'"
5. Exodus 3:14, "And God said to Moses, 'I AM WHO I AM'; and He said, Thus you shall say to the
sons of Israel, ‘I AM has sent me to you.’"
6. John 10:30-33 - "I and the Father are one." 31The Jews took up stones again to stone Him.
32
Jesus answered them, "I showed you many good works from the Father; for which of them are
you stoning Me?" 33The Jews answered Him, "For a good work we do not stone You, but for
blasphemy; and because You, being a man, make Yourself out to be God."
7. John 20:28, "Thomas answered and said to Him, "My Lord and my God!"
8. Col. 2:9, "For in Him all the fullness of Deity dwells in bodily form."
9. Phil. 2:5-9, , “Have this attitude in yourselves which was also in Christ Jesus, 6who, although He
existed in the form of God, did not regard equality with God a thing to be grasped, 7but emptied
Himself, taking the form of a bond-servant, and being made in the likeness of men. 8And being
found in appearance as a man, He humbled Himself by becoming obedient to the point of death,
even death on a cross. 9Therefore also God highly exalted Him, and bestowed on Him the name
which is above every name
10. Heb. 1:8, "But of the Son He says, "Thy throne, O God, is forever and ever, and the righteous
scepter is the scepter of His kingdom."
A. Quoted from Psalm 45:6, "Thy throne, O God, is forever and ever; a scepter of uprightness
is the scepter of Thy kingdom."
11. Matt. 4:10, "Then Jesus *said to him, 'Begone, Satan! For it is written, "You shall worship the
Lord your God, and serve Him only."’"
12. Matt. 2:2, - "Where is He who has been born King of the Jews? For we saw His star in the east,
and have come to worship Him."
13. Matt. 2:11, "And they came into the house and saw the Child with Mary His mother; and they
fell down and worshiped Him; and opening their treasures they presented to Him gifts of gold and
frankincense and myrrh."
14. Matt. 14:33, - "And those who were in the boat worshiped Him, saying, "You are certainly God’s
Son!"
15. Matt. 28:9, "And behold, Jesus met them and greeted them. And they came up and took hold of
His feet and worshiped Him."
16. John 9:35-38, "Jesus heard that they had put him out; and finding him, He said, "Do you
believe in the Son of Man?" 36He answered and said, "And who is He, Lord, that I may believe in
Him?" 37Jesus said to him, "You have both seen Him, and He is the one who is talking with you."
38
And he said, "Lord, I believe." And he worshiped Him."
17. Heb. 1:6, "And when He again brings the first-born into the world, He says, 'And let all the
angels of God worship Him.'"
Jesus is prayed to

1. Acts 7:55-60, "But being full of the Holy Spirit, he gazed intently into heaven and saw the glory
of God, and Jesus standing at the right hand of God; 56and he said, "Behold, I see the heavens
opened up and the Son of Man standing at the right hand of God." 57But they cried out with a loud
voice, and covered their ears, and they rushed upon him with one impulse. 58And when they had
driven him out of the city, they began stoning him, and the witnesses laid aside their robes at the
feet of a young man named Saul. 59And they went on stoning Stephen as he called upon the Lord
and said, "Lord Jesus, receive my spirit!" 60And falling on his knees, he cried out with a loud
voice, "Lord, do not hold this sin against them!" And having said this, he fell asleep."

2. 1 Cor. 1:1-2, "Paul, called as an apostle of Jesus Christ by the will of God, and Sosthenes our
brother, 2to the church of God which is at Corinth, to those who have been sanctified in Christ
Jesus, saints by calling, with all who in every place call upon the name of our Lord Jesus Christ,
their Lord and ours." (The phrase, "to call upon the name of the Lord" is a phrase used to
designate prayer.)
A. 1 Kings 18:24, "Then you call on the name of your god, and I will call on the name of the
Lord, and the God who answers by fire, He is God." And all the people answered and said,
"That is a good idea."
B. Zech. 13:9, “"And I will bring the third part through the fire, refine them as silver is refined,
and test them as gold is tested. They will call on My name, and I will answer them; I will
say, ‘They are My people,’ and they will say, ‘The Lord is my God.’"
C. Rom. 10:13-14, "for 'whoever will call upon the name of the Lord' will be saved." 14How
then shall they call upon Him in whom they have not believed? And how shall they believe in
Him whom they have not heard?" (Paul is speaking of calling upon Jesus. (The phrase "Call
upon the name of the Lord" is a quote from Joel 2:32).
i. Joel 2:32, "And it shall come to pass, that whosoever shall call on the name of the
LORD shall be delivered: for in Mount Zion and in Jerusalem shall be deliverance, as the
LORD hath said, and in the remnant whom the LORD shall call." (LORD here is YHWH,
the name of God as revealed in Exodus 3:14. Therefore, this quote, dealing with God
Himself is attributed to Jesus.)

3. First and Last


A. Isaiah 44:6, "Thus says the Lord, the King of Israel and his Redeemer, the Lord of hosts: ‘I
am the first and I am the last, and there is no God besides Me."
B. Rev. 1:17, "Do not be afraid; I am the first and the last, 18and the living One; and I was
dead, and behold, I am alive forevermore, and I have the keys of death and of Hades."
Jesus is Jehovah (YHWH)
Jehovah Jesus
Psalm 102:25, Heb. 1:10,
"Of old Thou didst found the earth; and the "And, "Thou, Lord, in the beginning didst lay
heavens are the work of Thy hands. the foundation of the earth, and the heavens are
the works of Thy hands";
Isaiah 45:20 Phil. 2:10-11,
"I have sworn by Myself, The word has gone "that at the name of Jesus every knee should
forth from My mouth in righteousness and will bow, of those who are in heaven, and on earth,
not turn back, That to Me every knee will bow, and under the earth, and that every tongue should
every tongue will swear allegiance." confess that Jesus Christ is Lord, to the glory of
God the Father."
Deut. 10:17, Rev. 17:14,
"For the Lord your God is the God of gods "These will wage war against the Lamb, and
and the Lord of lords, the great, the mighty, and the Lamb will overcome them, because He is Lord
the awesome God who does not show partiality, of lords and King of kings, and those who are with
nor take a bribe." Him are the called and chosen and faithful."
Isaiah 44:6 Rev. 22:12-13,
"Thus says the Lord, the King of Israel and "Behold, I am coming quickly, and My reward is
his Redeemer, the Lord of hosts: 'I am the first with Me, to render to every man according to what
and I am the last, And there is no God besides he has done. 13"I am the Alpha and the Omega,
Me." the first and the last, the beginning and the end."
Psalm 130:7-8, Titus 2:14,
"O Israel, hope in the Lord; for with the Lord "who gave Himself for us, that He might
there is lovingkindness, and with Him is redeem us from every lawless deed and purify for
abundant redemption. 8And He will redeem Himself a people for His own possession, zealous
Israel from all his iniquities." for good deeds."
Zech. 12:10, Rev. 1:7,
"And I will pour out on the house of David "Behold, He is coming with the clouds, and
and on the inhabitants of Jerusalem, the Spirit of every eye will see Him, even those who pierced
grace and of supplication, so that they will look Him; and all the tribes of the earth will mourn over
on Me whom they have pierced; and they will Him. Even so. Amen."
mourn for Him, as one mourns for an only son,
and they will weep bitterly over Him, like the
bitter weeping over a first-born.
Joel 2:32, Rom. 10:13 ,
"And it will come about that whoever calls on "for 'Whoever will call upon the name of the
the name of the Lord will be delivered; for on Lord will be saved.'"
Mount Zion and in Jerusalem there will be those
who escape, as the Lord has said, even among
the survivors whom the Lord calls."
The True Jesus
There is a simple way to see if someone has the true Jesus or not. By true Jesus, I mean the one
of the Bible, not the one of Mormonism who is the brother of the devil, nor the Jehovah's Witness
Jesus who is Michael the Archangel, and certainly not the one of the New Age Movement who is simply
a man in tune with the divine consciousness.

• The Jesus of the Bible is prayed to (Acts 7:55-60; Psalm 116:4 and Zech. 13:9 with 1 Cor. 1:1-
2).
• The Jesus of the Bible is worshiped (Matt. 2:2,11; 14:33; 28:9; John 9:35-38; Heb. 1:6)
• The Jesus of the Bible is called God (John 10:28; Heb. 1:8).

In cult theologies, Jesus is a creation in one form or another (this is why the Jehovah's Witnesses
add the word ‘other' four times to Col. 1:16-17). Therefore, He is not to be prayed to, worshiped, or
called God.
If you are a Christian then you will be able to pray to Jesus, not just through. You will be able to
worship Jesus equally with the Father. And you will be able to call Jesus your Lord and God. A cultist
cannot do this. A cultist has a false Jesus, and, therefore, a false hope of salvation.

The following is an expansion of the above points

If you put your faith in a Jesus that is not true, then your faith is useless. The power of faith does
not rest in the act of believing, but in its object; the greatest faith in someone false is the same as no
faith at all. Sincerity and false messiahs do not bridge the chasm of sin between God and man, only
the Jesus of the Bible does that. Who then, is the true Jesus?
Jesus said that He was the only One who reveals the Father (Matt. 11:27 and Luke 10:22): "All
things have been committed to me by my Father. No one knows who the Son is except the Father, and
no one knows who the Father is except the Son and those to whom the Son chooses to reveal him,"
(NIV).
So, to know the true Father you must first know the true Jesus. The question is how do you
recognize the true Jesus? Simple: Look in the Bible.
If you were to say, "Father receive my spirit," who would you be praying to? The Father, right?
If you were to say, "Jesus receive my spirit," who would you be praying to? Jesus.
In Acts 7:59, Stephen, while full of the Holy Spirit (v. 55), prayed to Jesus:

And they went on stoning Stephen as he called upon the Lord and said, "Lord Jesus,
receive my spirit." (See also Acts 9:14; Rom. 10:13.)

Stephen prayed to Jesus, not just through Him. If it is acceptable for him then it should be alright
for you. The Jesus of the Bible is prayed to. I pray to Jesus. Do you? If yes, good. If not, why?
But you might say, "Jesus said to pray to the Father." I do. But I also pray to Jesus as Stephen
did. If the church is only to pray to the Father then why did Stephen, under the inspiration of the Holy
Spirit, address Jesus in His prayer? Was he wrong? See also 1 Cor. 1:1-2 with Psalm 116:4 where
calling upon the name of the Lord is prayer and prayer is addressed to Jesus by the Corinthian church.

Jesus was also worshipped. The verses are:

• “And those who were in the boat worshiped Him, saying, "You are certainly God's
son!” (Matt. 14:33).
• “And behold, Jesus met them and greeted them. And they came up and took hold of
His feet and worshiped Him,” (Matt. 28:9).
See also Matt. 2:2,11; 14:33; 28:9; John 9:35-38; Heb. 1:6.
The Jesus of the Bible is prayed to and worshiped. Do you do what Jesus' disciples did? Do you
pray to and worship the true Jesus?
Since it is against Mormon and Jehovah's Witness theologies to pray to Jesus but only through if
you do worship Jesus, how can you do that without praying to Him? And, do you honor Him equally
with the Father as Jesus said to do in John 5:23? If you do not, then why not?
There is just one more issue to address. Do you call Jesus your Lord and God?
After Jesus' resurrection He showed Himself to many people. One of them was Thomas. John
20:28:

• Thomas answered and said to Him [Jesus], "My Lord and my God!" The literal Greek
says, "The Lord of me and the God of me."

"My God!" is a pagan expression used today. Two points can be made from this. First, do you
agree that Thomas a devout Jew was swearing, like a pagan of today? Second, there is no biblical
account of swear words. Peter did swear in Mark 14:71 by swearing he did not know Jesus. To say
Thomas was swearing, or merely exclaiming profound surprise has no evidence.

God calls Jesus God in Heb. 1:8:

• But of the Son He [the Father] says, "Thy throne, O God, is forever and ever..."

Unfortunately, in the Jehovah's Witness Bible in Heb. 1:8 you'll see that it says, "God is your
throne, forever and ever." This, technically speaking, is a legitimate translation. The reason this is so
lies in the nature of the Greek language and the fact that the form of the word "God" and "Throne"
both end in a noun construction that is interchangeable, therefore making the NWT translation
legitimate. It is unfortunate that the Watchtower has chosen to do this. Nevertheless, if you'd like to
read more about this, then go to The Jehovah's Witnesses and Heb. 1:8 and Psalm 45:6.

Conclusion

The Jesus of the Bible is prayed to (Acts 7:55-60; Psalm 116:4 and Zech. 13:9 with 1 Cor. 1:1-2),
worshiped (Matt. 2:2,11; 14:33; 28:9; John 9:35-38; Heb. 1:6), and called Lord and God (John
20:28; Heb. 1:8). If I have the wrong Jesus, and therefore I serve the wrong God, then why do I pray
to Jesus, worship Him, and call Him my Lord and God as the Scriptures teach? But, if you have the
true Jesus, why is it you don't do those things? Why does JW theology not agree with the scriptures?
I think the answer is simple. The Jesus of the cults is not the true Jesus. Therefore, they are
wrong.
100 Truths about Jesus

1. Jesus claimed to be God - John 8:24; 8:56-59 (see Exodus 3:14); John 10:30-33
2. Jesus created all things - John 1:3; Col. 1:15-17
3. Jesus is before all things - Col. 1:17
4. Jesus is eternal - John 1:1,14 ; 8:58
5. Jesus is honored the same as the Father - John 5:23
6. Jesus is prayed to - Acts 7:55-60
7. Jesus is worshipped - Matt. 2:2,11; 14:33; John 9:35-38; Heb. 1:6
8. Jesus is called God - John 1:1,14; 20:28; Col. 2:9; Titus 2:13
9. Jesus is omnipresent - Matt. 28:20
10. Jesus is with us always - Matt. 28:20
11. Jesus is our only mediator between God and ourselves - 1 Tim. 2:5
12. Jesus is the guarantee of a better covenant - Heb. 7:22; 8:6
13. Jesus said, "I AM the Bread of Life" - John 6:35,41,48,51
14. Jesus said, "I AM the Door" - John 10:7,9
15. Jesus said, "I AM the Good Shepherd" - John 10:11,14
16. Jesus said, "I AM the Way the Truth and The Life" - John 14:6
17. Jesus said, "I AM the Light of the world" - John 8:12; 9:5; 12:46; Luke 2:32
18. Jesus said, "I AM the True Vine" - John 15:1,5
19. Jesus said, "I AM the Resurrection and the Life" - John 11:25
20. Jesus said, "I AM the First and the Last" - Rev. 1:17; 2:8; 22:13
21. Jesus always lives to make intercession for us - Heb. 7:25
22. Jesus cleanses from sin - 1 John 1:9
23. Jesus discloses Himself to us - John 14:21
24. Jesus draws all men to Himself - John 12:32
25. Jesus forgives sins - Matt. 9:1-7; Luke 5:20; 7:48
26. Jesus gives eternal life - John 10:28; 5:40
27. Jesus gives joy - John 15:11
28. Jesus gives peace - John 14:27
29. Jesus has authority - Matt. 28:18; John 5:26-27; 17:2; 3:35
30. Jesus judges - John 5:22,27
31. Jesus knows all men - John 16:30
32. Jesus opens the mind to understand scripture - Luke 24:45
33. Jesus received honor and glory from the Father - 1 Pet. 1:17
34. Jesus resurrects - John 5:39; 6:40,44,54; 11:25-26
35. Jesus reveals grace and truth - John 1:17 see John 6:45
36. Jesus reveals the Father - Matt. 11:27; Luke 10:22
37. Jesus saves forever - Matt. 18:11; John 10:28; Heb. 7:25
38. Jesus bears witness of Himself - John 8:18; 14:6
39. Jesus' works bear witness of Himself - John 5:36; 10:25
40. The Father bears witness of Jesus - John 5:37; 8:18; 1 John 5:9
41. The Holy Spirit bears witness of Jesus - John 15:26
42. The multitudes bear witness of Jesus - John 12:17
43. The Prophets bear witness of Jesus - Acts 10:43
44. The Scriptures bear witness of Jesus - John 5:39
45. The Father will honor us if we serve Jesus - John 12:26 see Col. 3:24
46. The Father wants us to fellowship with Jesus - 1 Cor. 1:9
47. The Father tells us to listen to Jesus - Luke 9:35; Matt. 17:5
48. The Father tells us to come to Jesus - John 6:45
49. The Father draws us to Jesus - John 6:44
50. Everyone who's heard & learned from the Father comes to Jesus - John 6:45

51. The Law leads us to Christ - Gal. 3:24


52. Jesus is the Rock - 1 Cor. 10:4
53. Jesus is the Savior - John 4:42; 1 John 4:14
54. Jesus is King - Matt. 2:1-6; Luke 23:3
55. In Jesus are the treasures of wisdom and knowledge - Col. 2:2-3
56. In Jesus we have been made complete Col. 2:10
57. Jesus indwells us - Col. 1:27
58. Jesus sanctifies - Heb. 2:11
59. Jesus loves - Eph. 5:25
60. We come to Jesus - John 5:50; 6:35,37,45,65; 7:37;
61. We sin against Jesus - 1 Cor. 8:12
62. We receive Jesus - John 1:12; Col. 2:6
63. Jesus makes many righteous - Rom. 5:19
64. Jesus is the image of the invisible God - Heb. 1:3
65. Jesus sends the Holy Spirit - John 15:26
66. Jesus abides forever - Heb. 7:24
67. Jesus offered up Himself - Heb. 7:27; 9:14
68. Jesus offered one sacrifice for sins for all time - Heb. 10:12
69. The Son of God has given us understanding - 1 John 5:20
70. Jesus is the author and perfecter of our faith - Heb. 12:2
71. Jesus is the Apostle and High Priest of our confession - Heb. 1:3
72. Jesus is preparing a place for us in heaven - John 14:1-4
73. Jesus cleanses us from our sins by His blood - Rev. 1:5; Rom. 5:9
74. Jesus is the Light of the world - Rom. 9:5
75. Jesus has explained the Father - John 1:18
76. Jesus was crucified because of weakness - 2 Cor. 13:4
77. Jesus has overcome the world - John 16:33
78. Truth is in Jesus - Eph. 4:21
79. The fruit of righteousness comes through Jesus Christ - Phil. 1:11
80. Jesus delivers us from the wrath to come - 1 Thess. 1:10
81. Disciples bear witness of Jesus Christ - John 15:27
82. Jesus died and rose again - 1 Thess. 4:14
83. The Christian dead have fallen asleep in Jesus - 1 Thess. 4:15
84. Jesus died for us - 1 Thess. 5:10
85. Jesus tasted death for everyone - Heb. 2:9
86. Jesus rendered the devil powerless - Heb. 2:14
87. Jesus is able to save completely - Heb. 7:25
88. Jesus was a ransom for many and to serve - Matt. 20:28
89. Jesus came to be a high priest - Heb. 2:17
90. Jesus came to save - John 3:17; Luke 19:10
91. Jesus came to preach the kingdom of God - Luke 4:43
92. Jesus came to bring division - Luke 12:51
93. Jesus came to do the will of the Father - John 6:38
94. Jesus came to give the Father's words - John 17:8
95. Jesus came to testify to the truth - John 18:37
96. Jesus came to die and destroy Satan's power - Heb. 2:14
97. Jesus came to fulfill the Law and the Prophets - Matt. 5:17
98. Jesus came to give life - John 10:10,28
99. Jesus came to taste death for everyone - Heb. 2:9
100. Jesus came to proclaim freedom for believers - Luke 4:18
Who is Jesus according to John the Apostle?
Who is Jesus according to John the apostle? Is He a mere man, an angel in flesh, or is He God
incarnate? The answer is very important because it determines where you stand in relationship to the
truth. Since faith is only as good as who you place it in, it is crucial that you place your faith in the
true Savior. So, who is the true Savior? Is he God or not? Is he an angel who became a man or not?
Or is he merely a great teacher?
John's gospel is different than the other three. In fact, Matthew, Mark, and Luke are called the
synoptic gospels because they are so similar. However, John presents Jesus in a different light from
the other three.
Additionally, John wrote the epistles of John and the book of Revelation. In each of them, Jesus is
presented in a special way. Let's take a look at how John sees Jesus.
John's concept of Jesus begins with the introductions of his gospel ( John 1:1,14) and his first
epistle (1 John 1:1,10). It is not a mere coincidence that John writes in such parallel to the opening
chapters of Genesis. Undoubtedly, John's opinion of Jesus was sufficient to equate him with God's
creative work of "in the beginning." Let's look.

Gospel of John 1 John Genesis

'In the beginning was the word "What was from the beginning, "In the beginning. . .," (1:1a)
and the word was with God what we have heard what we
and the Word was God 2He was have seen with our eyes, what
in the beginning with God," we behold and our hands handled
(1:12). concerning the word, . .” (1:1a).

"All things came into being by " . . . God created the heavens
Him, and apart from Him and the earth," (1:1b)
nothing came into being that
has come into being," (1:3)

In Him was life, and the life ". . . of life,” (1:1b). "Then God said, 'Let there be
was the light of men. 5And the light'; and there was light. And
light shines in the darkness, God saw that the light was
and the darkness did not good; and God separated the
comprehend it.. . . (1:4-5). light from the darkness," (1:3-
4)

…and the word became flesh " . . .God is light and in Him "And they heard the sound of
and dwelt among us. . ." there is no darkness," (1:5). the LORD God walking in the
(1:14). garden in the cool of the
day. . .," (3:8).

The parallels between the gospel of John, 1 John, and Genesis are immediately evident. The
terminology is very similar. The themes are almost identical. Obviously, John considers Jesus to be of
preeminent importance and uses many figures of speech equated with God.
But John does not abandon the thematic comparison between the Word and God after the opening
chapters. He continues to show the divine qualities of Jesus throughout his writings.
In the Gospel of John

I have already mention John 1:1,14 where the Word is in the beginning with God, and was God,
the Word became flesh. This is obviously referencing Jesus. John also presents Jesus as…

1. giving eternal life (John 10:27);


2. the bread of life (John 6:35,51 - an obvious allusion to the manna given by God out of
heaven in 6:32,35);
3. the way the truth and the life (14:6);
4. the light of the world (8:12);
5. proceeding from the Father (8:24).
6. being the 'I am" (8:58) -- see also Exodus 3:14 where God calls Himself “I AM.”
7. being one with the Father (10:30) for which the Jews wanted to kill him - see Lev. 24:16;
8. sharing the glory of God before creation (17:5; note that God shares His glory with no one,
Isaiah 42:8);
9. calling Jesus His own Father making Himself equal with God ( John 5:18);
10. receiving the same honor that you give to the Father (John 5:23);
11. Lord and God (John 20:28).
12. knowing all things (John 21:17 - something only God can do).
13. And in John 18:5, in the Garden of Gethsemane when Jesus answers those who came to
arrest Him with the statement, "I am", they fall back to ground.

Is it safe to say that John in his gospel merely considers Jesus a man or even a special angel? Is a
mere man or an angel the giver of eternal life or is this something God does? Is a mere man or an
angel the way, the truth, and the life, or the light of the world. Is a creature one with the Father, or
does a creature share in God's glory, or even knowing all things? No. Not at all.

In the Book of Revelation

John continues with OT themes dealing with God and applies them to Jesus in the book of
Revelation.

"Do not be afraid; I am the first and the last, "Thus says the Lord, the King of Israel and his
18
and the living One; and I was dead, and Redeemer, the Lord of hosts: 'I am the first and
behold, I am alive forevermore, and I have the I am the last, and there is no God besides Me,"
keys of death and of Hades," (Rev. 1:1718). (Isaiah 44:6).
"Behold, I am coming quickly, and My reward is "Behold, the Lord God will come with might, with
with Me, to render to every man according to His arm ruling for Him. Behold, His reward is
what he has done. 13"I am the Alpha and the with Him, and His recompense before Him,"
Omega, the first and the last, the beginning and (Isaiah 40:).
the end," (Rev. 22:12-23).
"These will wage war against the Lamb, and the "that you keep the commandment without stain
Lamb will overcome them, because He is Lord or reproach until the appearing of our Lord Jesus
of lords and King of kings, and those who are Christ, 15which He will bring about at the proper
with Him are the called and chosen and time--He who is the blessed and only Sovereign,
faithful," (Rev. 17:14; 19:16). the King of kings and Lord of lords," (1 Tim.
6:15-16).
"for the Lamb in the center of the throne shall "The Lord is my shepherd, I shall not want. 2He
be their shepherd, and shall guide them to makes me lie down in green pastures; He leads
springs of the water of life; and God shall wipe me beside quiet waters," (Psalm 23:1-2)
every tear from their eyes," (Rev. 7:17).

Divine themes run through the book of Revelation. Both Jesus and God are called the first and last.
Both are coming to give out their reward. Both are the Lord of Lords and the King of Kings. Both are
the divine shepherds.
It is no wonder in three significant verses in the gospel; John records Jesus saying about Himself:

• "I said therefore to you, that you shall die in your sins; for unless you believe that I am, you
shall die in your sins," (John 8:24).
• "Jesus therefore said, "When you lift up the Son of Man, then you will know that I am He, and I
do nothing on My own initiative, but I speak these things as the Father taught Me," (John
8:28).
• "Jesus said to them, "Truly, truly, I say to you, before Abraham was born, I am," (John 8:58),
see Exodus 3:14.

It is apparent that John considered Jesus more than a man and more than an angel. He is God in
flesh: "In the beginning was the Word, and the Word was with God, and the Word was God . . . and
the Word became flesh and dwelt among us. . ., " (John 1:1,14).
Jesus' Resurrection was physical

The resurrection of Jesus is a fundamental and essential doctrine of Christianity. The resurrection
of Jesus is so important that without it Christianity is false. Paul said in 1 Cor. 15:14, "and if Christ
has not been raised, then our preaching is vain, your faith also is vain." Three verses later, in verse
17, he again says, "and if Christ has not been raised, your faith is worthless; you are still in your
sins." Though there are many subjects with which Christians may disagree and still be considered
Christian, this is not one of them. To deny the resurrection of Jesus is to deny the heart of Christianity
itself.
However, the problem in the resurrection isn't so much in agreeing that Jesus rose, but in how He
rose. Unfortunately, cults attack the resurrection of Christ and reinterpret it in different ways, thereby
denying His physical resurrection. We must ask if Jesus rose from the dead in the very same body He
died in or did His rise in a spirit body that was not flesh and bones? The answer to this question is
vital. It separates true Christians from false systems. Therefore, here is the correct doctrine of
Christ's resurrection. I consider it so important, that it must be set off by itself as a statement of
truth.

Jesus rose from the dead in the very same physical body that He died in. This
resurrected body was a glorified, spiritual body.

The above statement is the correct doctrine of scripture. As such, it stands against the Jehovah's
Witness and the Shepherd's Chapel groups that state that Jesus did not rise bodily, but spiritually.
Neither group seeks to deny the obvious biblical declaration of Christ's resurrection, but they change
the meaning of the resurrection so that it really didn't happen. Did Jesus rise from the dead in the
same glorified body He died in? Yes!
After the resurrection Jesus was able to eat (Luke 24:4243). He showed people His hands and feet
with the nail prints in them (Luke 24:51; John 20:27), and people even grabbed His feet and
worshipped Him (Matt. 28:9). After the reports of Jesus' resurrection were spreading, Thomas, who
was doubting the resurrection of Christ, said, "Unless I shall see in His hands the imprint of the nails,
and put my finger into the place of the nails, and put my hand into His side, I will not believe," ( John
20:25). Later, Jesus appeared to Thomas and said to him, "Reach here your finger, and see My
hands; and reach here your hand, and put it into My side; and be not unbelieving, but believing,"
(John 20:27).
If Jesus' body had not risen, then He would not have feet and hands with the same holes of the
nails of the crucifixion. Consider the following verses as further proof that His very body was raised:

• "When therefore it was evening, on that day, the first day of the week, and when the doors
were shut where the disciples were, for fear of the Jews, Jesus came and stood in their midst,
and *said to them, "Peace be with you." 20And when He had said this, He showed them both His
hands and His side. The disciples therefore rejoiced when they saw the Lord," (John 20:19-20).
• "And He said to them, "Why are you troubled, and why do doubts arise in your hearts? 39"See
My hands and My feet, that it is I Myself; touch Me and see, for a spirit does not have flesh and
bones as you see that I have," (Luke 24:38-39).

It is obvious that Jesus was raised in the same body He died in, with the same holes in His hands
and feet. We see that Jesus proclaimed He had flesh and bones? Does a "spirit body" consist of flesh
and bones? Not at all.
I have heard it said that Jesus physical body died but His spiritual body was raised. If this is so,
then does the spiritual body consist of flesh and bones as well as the physical one? It makes no
sense. Also, if Jesus did not rise physically, then what happened to His body? Was it dissolved? Was
it moved somewhere? There is no biblical account of what happened to Jesus' body other than that it
was raised from the dead. Therefore, His body was raised from the dead.
John 2:19-21

"Jesus answered and said to them, "Destroy this temple, and in three days I will raise it
up." 20The Jews therefore said, "It took forty-six years to build this temple, and will You
raise it up in three days?" 21But He was speaking of the temple of His body,"
(John 2:19-21).

The phrase "I will raise" is translated from the single Greek word "egeiro." "Egeiro" is the future,
active, indicative, 1st person singular. The active voice in Greek designates who is performing the
action. In this case, since it is first person, singular ("I"), Jesus is saying that He Himself would
perform the action of the resurrection. This is precisely what the Greek says.
However, some still deny that Jesus rose from the dead physically -- even when examining John
2:19-21. We can clearly see that Jesus prophesied that He would raise up the temple of His body as is
clarified in verse 21 by John the apostle who states that Jesus was speaking of "the temple of his
body." Therefore, this should be conclusive proof that Jesus rose from the dead in the same body He
died in. Clearly, John 2:19-21 shows us that Jesus predicted He would raise His very body -- and He
did so. Is this enough to put this issue to rest? You'd think so, but resistance persists.

1 Cor. 15:35, 39, 42-44


35
But someone will say, "How are the dead raised? And with what kind of body do they
come?. . .39All flesh is not the same flesh, but there is one flesh of men, and another
flesh of beasts, and another flesh of birds, and another of fish. . . 42So also is the
resurrection of the dead. It is sown a perishable body, it is raised an imperishable body;
43
it is sown in dishonor, it is raised in glory; it is sown in weakness, it is raised in power;
44
it is sown a natural body, it is raised a spiritual body. If there is a natural body, there
is also a spiritual body.

Verse 44 above is used in an attempt to establish the idea that Jesus did not rise physically, but
spiritually. Of course, I've already established above that Jesus was raised in the same body He died
in, with the same holes in His hands and feet. We also saw that Jesus proclaimed He had flesh and
bones (Luke 24:39). Again, does a "spirit body" consist of flesh and bones? The scripture no where
declares such a thing.
Paul is not stating that there are two separate bodies to each person, the physical and the spiritual
and that after the physical one dies, the second and different spirit body takes over. Rather, when
referencing the same body he states, "it is sown a natural body, it is raised a spiritual body," (v. 44).
The "it" is referring to the same body in both clauses, not separate and different ones. This same
body becomes a resurrected body -- which is the spiritual body He is referring to. In other words, the
spiritual body is the very same body he previously had, though it had been changed into a spiritual
one.

"For this perishable must put on the imperishable, and this mortal must put on
immortality. 54But when this perishable will have put on the imperishable, and this
mortal will have put on immortality, then will come about the saying that is written,
"Death is swallowed up in victory," (1 Cor. 15:53-54).

Our perishable and mortal bodies put on the imperishable and immortal aspects of the spiritual
body which is the physically resurrected and changed body of the believer. Jesus was simply the first
fruits of this resurrection (1 Cor. 15:20). Therefore, we can see that our future resurrected bodies will
be spiritual bodies. But, those spiritual bodies are in fact physical, the same bodies we have now, only
glorified. Otherwise, there is no resurrection.
Objections to Jesus' physical resurrection answered

Even though the bible teaches us that Jesus rose from the dead in the same body that He died in
and that His resurrected body was a glorified body, people still resist accepting this truth -- to their
detriment. Various objections are raised against such biblical support as...

• "Jesus answered and said to them, "Destroy this temple, and in three days I will
raise it up." 20The Jews therefore said, "It took forty-six years to build this temple, and will
You raise it up in three days?" 21But He was speaking of the temple of His body," (John 2:19-
21).
• "When therefore it was evening, on that day, the first day of the week, and when
the doors were shut where the disciples were, for fear of the Jews, Jesus came and stood in
their midst, and *said to them, "Peace be with you." 20And when He had said this, He showed
them both His hands and His side. The disciples therefore rejoiced when they saw the Lord,"
(John 20:19-20).
• "And He said to them, "Why are you troubled, and why do doubts arise in your
hearts? 39"See My hands and My feet, that it is I Myself; touch Me and see, for a spirit does
not have flesh and bones as you see that I have." (Luke 24:38-39).

It seems clear that Jesus' physical resurrection is a reality. Unfortunately, objections to it have
been raised.

Objection 1: Jesus was put to death physically but was raised spiritually according to 1 Pet. 3:18.

1 Pet. 3:18 is often used as a counter John 2:19-21. Instead of harmonizing the Scriptures, some
people use one scripture to "refute" another or to justify their interpretations which seem to favor their
positions. Such is the case with 1 Pet. 3:18-19:

"For Christ also died for sins once for all, the just for the unjust, in order that He might bring us
to God, having been put to death in the flesh, but made alive in the spirit," (1 Pet. 3:18).

The point that they try to make in this verse is that Jesus did not rise in the flesh, but "in the
spirit." Some even say that Jesus ceased to exist and then was made alive" in the spirit. However,
because Jesus is the Word made flesh (John 1:1,14), His spirit is immortal and does not need to be
made alive. Nevertheless, they assert that Jesus was not speaking literally in John 2:19-21, otherwise
it would contradict their doctrine that Jesus did not rise physically. Of course, they are incorrect. Here
is why.
Let's look at the context of 1 Pet. 3:18. Here is 1 Pet. 3:17-20,

"For it is better, if God should will it so, that you suffer for doing what is right rather than for
doing what is wrong. 18For Christ also died for sins once for all, the just for the unjust, in order
that He might bring us to God, having been put to death in the flesh, but made alive in the
spirit; 19in which also He went and made proclamation to the spirits now in prison, 20who once
were disobedient, when the patience of God kept waiting in the days of Noah, during the
construction of the ark, in which a few, that is, eight persons, were brought safely through the
water," (1 Pet. 3:17-20).

We must acknowledge right away that these verses have different interpretations among scholars.
It says Jesus was in the spirit when He went and made proclamation. But what does that mean? Did
Jesus, between the time of his death and resurrection, go and make a proclamation to spirits in prison,
or was it after His resurrection? Also, the Greek word used is "proclaim," (karuso) not "preach,"
(evangelizo), so it was not a message of salvation to those spirits in prison. Also, who are the spirits,
angels or men? In the spirit realm, angels are said to be in prison (Rev. 20:7; 2 Pet. 2:4), but never
people. What was the proclamation? Most probably, it was the proclamation of Christ's victory at the
cross, according to scripture, which was proclaimed to spirits of old who were disobedient in the time
of Noah and who were being held in bonds (See also, 2 Pet. 2:4-5).
In my opinion, between His death and resurrection, Jesus went and made a proclamation of His
victory on the cross to those fallen angels who were being held in prison. But since there is no
definitive answer on this, I am open to further discussion on it.
Verse 18 does not require the interpretation that Jesus did not rise physically. In fact, logically
speaking, if we held to the "spirit only" idea of His resurrection, we would have a contradiction with
other verses in the Bible; namely, John 2:19-21 and Luke 24:39 cited above. Since John 2:19 clearly
teaches that the temple of Christ's body was raised, 1 Pet. 3:18, which has different interpretations
among scholars, cannot be held in a way that would contradict other, clearer scriptures such as John
2:19-21 and Luke 24:39).
Furthermore, different Bibles translate verse 18 differently. Some say Jesus was "made alive by
the Spirit,” (KJV, NKJV, NIV, MLB) while others say "...made alive in the spirit,” (NASB, NEB, RSV, JB,
and the 1901 ASV). It is certainly possible that Jesus was made alive by the Holy Spirit which is
consistent with the Trinitarian aspect of Jesus' resurrection where God raised Jesus (1 Thess. 1:10),
the Father raised Jesus (Gal. 1:1), and Jesus raised Himself (John 2:19-21), and the Holy Spirit was
also involved in His resurrection (Rom. 8:11). It is also accurate to say that Jesus was raised in the
spirit in that His spiritual body, which is His physical glorified body, was quickened, made alive,
became real as the first fruits of all creation (1 Cor. 15:20).
Finally, it is our bodies that are redeemed as well, not just our spirits. "And not only this, but also
we ourselves, having the first fruits of the Spirit, even we ourselves groan within ourselves, waiting
eagerly for our adoption as sons, the redemption of our body," (Rom. 9:23). The body here spoken of
is the physical one, not a "spiritual" non-flesh body.
To summarize about this verse: 1 Pet. 3:18 does not say that Jesus was raised a spirit creature. It
says that He was "made alive in the spirit." What does that mean? Quite simply, it means that Jesus
was raised in an imperishable body. This is what 1 Cor. 15:35-45 says when it refers to the body as
being sown perishable, but raised imperishable; sown in dishonor and raised in glory; sown a natural
body and raised a spiritual body, etc. Jesus was the "Last Adam" a life giving spirit. Paul is typifying
the resurrection body. In this passage Paul is talking about the resurrection of all people. All
Christians will be raised in physical bodies. It is the same with Jesus.

Objection 2: The Bible says that "flesh and blood cannot inherit the kingdom of God" ( 1 Cor.
15:50), therefore, Jesus could not have been raised from the dead in the same body He died in.
The problem with this objection is that it fails to recognize the fact that after the resurrection, Jesus
said, "See My hands and My feet, that it is I Myself; touch Me and see, for a spirit does not have flesh
and bones as you see that I have," (Luke 24:39), not "flesh and blood." This is not simply a play on
words. Every word in the Bible is inspired and Jesus used this phrase on purpose.
The term "flesh and blood" is a phrase used in scripture in different contexts, but denotes the
natural order.

• "And Jesus answered and said to him, "Blessed are you, Simon Barjona, because flesh
and blood did not reveal this to you, but My Father who is in heaven," ( Matt. 16:17).
• "For our struggle is not against flesh and blood, but against the rulers, against the
powers, against the world forces of this darkness, against the spiritual forces of wickedness in
the heavenly places," (Eph. 6:12).
• "Since then the children share in flesh and blood, He Himself likewise also partook of the
same, that through death He might render powerless him who had the power of death, that
is, the devil," (Heb. 2:14).

Jesus had shed His blood on the cross. It quit literally had drained out of His body. We see that
when Jesus rose from the dead, He still had the holes in His hands and feet ( Luke 24:39). Since He
retained the characteristics of His bodily ordeal, it is logical to state that His blood, which was literally
drained from His body, was likewise still shed. Therefore, His body could be raised and the blood
remained shed as the thing that "makes atonement": "For the life of the flesh is in the blood, and I
have given it to you on the altar to make atonement for your souls; for it is the blood by reason of the
life that makes atonement," (Lev. 17:11).
That is why after the resurrection, to prove that He had risen in the same body He died in, Jesus
told people to touch His hands and feet because it was the hands and feet that had the holes in them.
What more proof do you need to but see and touch the very same hands and feet that had the holes in
them from the nails on the cross! Furthermore, in the same statement Jesus said that He possessed
flesh and bones, not flesh and blood. He had risen!
Objection 3: The sacrificial offering was the body of Christ; therefore, it could not rise lest the
sacrifice be made invalid by "being taken back.”
The answer to this objection is similar to the one above. Jesus' resurrection is the proof that His
sacrifice was accepted by the Father who had promised, "For Thou wilt not abandon my soul to Sheol;
Neither wilt Thou allow Thy Holy One to undergo decay," (Psalm 16:10). Because Jesus offered a
perfect sacrifice for sin, He was guaranteed a physical resurrection. You see, physical death is the
result of sin. But, Jesus successfully took care of the sin problem and, in the process, conquered death
which is the result of sin (Rom. 5:12; 1 Cor. 15:56). The proof is found in the fact that He rose from
the dead in the same body He died in.
Furthermore, the truth is that Jesus bore our sins in His body on the cross (1 Pet. 2:24) and took
our place (2 Cor. 5:21). His body was used as the means to shed the blood that cleanses of sin.

• "For the life of the flesh is in the blood, and I have given it to you on the altar to make
atonement for your souls; for it is the blood by reason of the life that makes atonement,"
(Lev. 17:11).
• "And according to the Law, one may almost say, all things are cleansed with blood, and
without shedding of blood there is no forgiveness," (Heb. 9:22).

So, the blood of Christ is what removes our sin and the physical resurrection of Christ is proof that
the sacrifice was accepted by the Father.

Objection 4: Jesus manifested different physical forms in order to convince the disciples that He
had been raised.
This is faulty for several reasons. First, it would mean that Jesus was tricking His disciples into
believing that His body had been raised when it hadn’t. Second, it disregards the clear teaching of
Jesus Himself who said His very body would be raised: "Destroy this temple, and in three days I will
raise it up." 20Then the Jews said, "It has taken forty-six years to build this temple, and will You raise
it up in three days?" 21But He was speaking of the temple of His body," (John 2:19-21). Jesus said
that His body would be raised. Third, 1 Tim. 2:5 says, "For there is one mediator between God and
man, the man Christ Jesus." Jesus is said to be a man. If He was not raised physically, then how could
he be a man without a body of flesh and bones?

Objection 5: The Father raised Jesus; He didn't do it Himself, therefore John 2:19-21 cannot be
literal because Jesus didn't raise Himself.
This objection simply fails to take into account the Trinitarian nature of God and the resurrection.
We see that each of the members of the Godhead was involved in the resurrection of Christ.

• Father - "Paul, an apostle (not sent from men, nor through the agency of man, but through
Jesus Christ, and God the Father, who raised Him from the dead)," (Gal. 1:1).
• Son - "Destroy this temple, and in three days I will raise it up." 20Then the Jews said, "It has
taken forty-six years to build this temple, and will You raise it up in three days?" 21But He
was speaking of the temple of His body," (John 2:19-21).
• The Holy Spirit - "But if the Spirit of Him who raised Jesus from the dead dwells in you, He
who raised Christ Jesus from the dead will also give life to your mortal bodies through His
Spirit who indwells you," (Rom. 8:11).

Likewise, we see that other Trinitarian aspects are observed throughout scripture on different
subjects: Each is called God: Father (Phil. 1:2), the Son (John 1:1,14; Col. 2:9) and the Holy Spirit
(Acts 5:3-4). Each is the Creator: Father (Isaiah 64:8; 44:24), the Son (John 1:3; Col. 1:15-17), and
the Holy Spirit (Job 33:4; 26:13), etc. When looking at the whole of scripture we see no contradiction
dealing with Jesus' resurrection. Instead, we see an affirmation of the truth that Jesus did, in fact,
raise His body just as He said He would in John 2:19-21.
What did Jesus come to do?
1. To reveal the Father, Matt. 11:27, “All things have been committed to me by my Father. No one
knows the Son except the Father, and no one knows the Father except the Son and those to
whom the Son chooses to reveal him."
2. To be a ransom for many, Matt. 20:28, "just as the Son of Man did not come to be served, but to
serve, and to give his life as a ransom for many."
3. To serve, Matt. 20:28, "just as the Son of Man did not come to be served, but to serve, and to
give his life as a ransom for many."
4. To save the world, John 3:17; Luke 19:10, "For God did not send his Son into the world to
condemn the world, but to save the world through him."
5. To preach the good news of the kingdom of God, Luke 4:43, "But he said, "I must preach the
good news of the kingdom of God to the other towns also, because that is why I was sent."
6. To bring division, Luke 12:51, "Do you think I came to bring peace on earth? No, I tell you, but
division."
7. To do the will of the Father, John 6:38, "For I have come down from heaven not to do my will but
to do the will of him who sent me."
8. To give the Father's words, John 17:8, "For I gave them the words you gave me and they
accepted them. They knew with certainty that I came from you, and they believed that you sent
me."
9. To testify to the truth, John 18:37, "You are a king, then!" said Pilate. Jesus answered, "You are
right in saying I am a king. In fact, for this reason I was born, and for this I came into the world,
to testify to the truth. Everyone on the side of truth listens to me."
10. To die and destroy Satan's power, Heb. 2:14, "Since the children have flesh and blood, he too
shared in their humanity so that by his death he might destroy him who holds the power of death
-- that is, the devil."
11. To destroy the devil's works, 1 John 3:8, "He who does what is sinful is of the devil, because the
devil has been sinning from the beginning. The reason the Son of God appeared was to destroy
the devil's work."
12. To fulfill the Law and the Prophets, Matt. 5:17, "Do not think that I have come to abolish the Law
or the Prophets; I have not come to abolish them but to fulfill them."
13. To give life, John 10:10,28, "The thief comes only to steal and kill and destroy; I have come that
they may have life, and have it to the full...I give them eternal life, and they shall never perish;
no one can snatch them out of my hand."
14. To taste death for everyone, Heb. 2:9, "But we see Jesus, who was made a little lower than the
angels, now crowned with glory and honor because he suffered death, so that by the grace of
God he might taste death for everyone."
15. To become a high priest, Heb. 2:17, "For this reason he had to be made like his brothers in every
way, in order that he might become a merciful and faithful high priest in service to God, and that
he might make atonement for the sins of the people."
16. To atone for sin, Heb. 2:17, "For this reason he had to be made like his brothers in every way, in
order that he might become a merciful and faithful high priest in service to God, and that he
might make atonement for the sins of the people."
17. To proclaim freedom for believers, Luke 4:18, "The Spirit of the Lord is on me, because he has
anointed me to preach good news to the poor. He has sent me to proclaim freedom for the
prisoners and recovery of sight for the blind, to release the oppressed."
18. To proclaim the year of the Lord's favor, Luke 4:19, "to proclaim the year of the Lord's favor."
19. To bring judgment, John 9:39, "Jesus said, "For judgment I have come into this world, so that
the blind will see and those who see will become blind."
20. To take away sin, 1 John 3:5, "But you know that he appeared so that he might take away our
sins. And in him is no sin."
21. To preach, Mark 1:38 , "Jesus replied, 'Let us go somewhere else -- to the nearby villages -- so I
can preach there also. That is why I have come.'"
22. To call sinners, Mark 2:17, "On hearing this, Jesus said to them, 'It is not the healthy who need a
doctor, but the sick. I have not come to call the righteous, but sinners.'"

Jesus is a man right now

One of the lesser known biblical doctrines concerns Jesus as a man right now. Many do not know
that right now, in heaven, Jesus is a man, though in a glorified body. Some object to this and cite
various reasons (answered at the end of this paper) for denying His present humanity. They are in
error. Following is a biblical demonstration that Jesus is still both divine and human in nature.
It is biblically correct to say that Jesus is a man right now in heaven -- though a glorified man.
But, it would wrong to say He was only a man. He is both divine and human in nature at the same
time (Col. 2:9); He is both God and man, right now.
Furthermore, Jesus' humanity now is important for two reasons. First, this is what the Bible
teaches. Second, as a man, Jesus is a priest forever after the order of Melchizedek. As a priest He
forever intercedes for us.

• "where Jesus has entered as a forerunner for us, having become a high priest forever according
to the order of Melchizedek, (Heb. 6:20).
• "Hence, also, He is able to save forever those who draw near to God through Him, since He
always lives to make intercession for them," (Heb. 7:25).

In order to be a priest, Jesus has to be a man. A spirit cannot be a priest after the order of
Melchizedek and if Jesus is not a man now, He could not hold His priesthood and He could not be
forever interceding for us. Therefore, to deny Jesus' present humanity is to deny His priesthood and
His intercession on our behalf. Without His intercession, we are lost.

1. Jesus died
There is no dispute that Jesus died on the cross --except for some non-Christian religions and
various atheistic groups who deny the biblical record. Nevertheless, the scriptures teach us that
Jesus died.
• "For if we believe that Jesus died and rose again, even so God will bring with Him those
who have fallen asleep in Jesus," (1 Thess. 4:14).

2. Jesus rose from the dead physically


The bible teaches us that Jesus rose from the dead. Unfortunately, some Christians are not
aware that Jesus actually rose from the dead in the same body He died in, though it was a glorified
body. We see that Jesus prophesied the resurrection of His physical body in John 2:19-21 and
fulfilled this in other verses:
• "Jesus answered and said to them, "Destroy this temple, and in three days I will raise it
up." 20The Jews therefore said, "It took forty-six years to build this temple, and will You
raise it up in three days?" 21But He was speaking of the temple of His body," (John 2:19-
21).

After Jesus' crucifixion and resurrection He appeared to various people to demonstrate that He
had risen physically.

• "See My hands and My feet, that it is I Myself; touch Me and see, for a spirit does not
have flesh and bones as you see that I have," (Luke 24:39).
• "When therefore it was evening, on that day, the first day of the week, and when the
doors were shut where the disciples were, for fear of the Jews, Jesus came and stood in
their midst, and *said to them, "Peace be with you." 20And when He had said this, He
showed them both His hands and His side. The disciples therefore rejoiced when they saw
the Lord," (John 20:19-20).
• "Then He *said to Thomas, "Reach here your finger, and see My hands; and reach here
your hand, and put it into My side; and be not unbelieving, but believing," (John 20:27).

In these verses we see that Jesus said He would raise the temple of His body. This He did and
the body He rose in was the same one He died in since it retained the physical wounds of His
crucifixion -- He still had holes in His hands and side!
I would like to note here that if anyone denies the resurrection of Christ, his faith is in vain and
he is not a true Christian.

1 Cor. 15:14, "and if Christ has not been raised, then our preaching is vain, your faith also is
vain.”

It is not enough to say that Jesus rose. You must acknowledge that He rose physically. A
"spirit" resurrection is not a resurrection of the body and without the resurrection of the body of
Christ, death has not been conquered and our faith would be in vain.

3. Jesus' resurrected body was a glorified body


Jesus rose from the dead physically in the same body He died in. But, what kind of a body was
this physical body He rose in? Was it subject to death again? Would it grow tired or grow old?
The Bible tells us about the resurrected body, of which all Christians will receive in the future.
• "But someone will say, "How are the dead raised? And with what kind of body do they
come?...40There are also heavenly bodies and earthly bodies, but the glory of the heavenly
is one, and the glory of the earthly is another. 41There is one glory of the sun, and another
glory of the moon, and another glory of the stars; for star differs from star in glory. 42So
also is the resurrection of the dead. It is sown a perishable body, it is raised an
imperishable body; 43it is sown in dishonor, it is raised in glory; it is sown in weakness, it is
raised in power; 44it is sown a natural body, it is raised a spiritual body. If there is a natural
body, there is also a spiritual body. 45So also it is written, "The first man, Adam, became a
living soul." The last Adam became a life-giving spirit. 46However, the spiritual is not first,
but the natural; then the spiritual. 47The first man is from the earth, earthy; the second
man is from heaven. 48As is the earthy, so also are those who are earthy; and as is the
heavenly, so also are those who are heavenly. 49And just as we have borne the image of the
earthy, we shall also bear the image of the heavenly," (1 Cor. 15:35,40-49).

These verses tell us that something happens to the body that is raised from the dead. Notice that
verse 44 says that "it is sown a natural body. It is raised a spiritual body." The same body that is
sown (dies) is raised. The natural body is the body we are born with. The natural body dies and is
raised from the dead. But, when it is raised, it is changed into a spiritual body. The resurrected
body is different than the natural body in its abilities and qualities as Jesus demonstrated;
however, and this is vitally important, it is the same body as before -- only "improved," "glorified,"
"spiritualized," etc. We see this in the fact that Jesus retained the wounds of His crucifixion as
evidenced by the holes in His hands and side (John 20:27), yet He was able to simply appear in a
room with the disciples without entering through the door (John 20:19-20). He was raised in the
same body He died in, though it had been glorified.

4. Jesus is a man in a glorified body


We have already seen that Jesus was raised from the dead in the same body He died in, but
that body is a resurrected body. However, some people believe that at Jesus' ascension, He was
somehow changed and His physical body was no longer needed. But, this is not what the Bible
teaches. There is no place where it states that Jesus stopped being a man. If anything, the New
Testament says He is still a man.
• "For in Him all the fullness of Deity dwells in bodily form," (Col. 2:9).
Notice that this verse speaks in the present tense ("dwells"). Colossians was written well after
Jesus' ascension into heaven, yet Paul tells us that Jesus is in bodily form. What body would that
be? Why, it would be the same body He was raised in. To clarify that Jesus is a man, read the
next verse.

• "For there is one God, and one mediator also between God and men, the man Christ
Jesus," (1 Tim. 2:5).
 We see here that Jesus is called a man. Like Col. 2:9 above, this verse uses the
present tense ("is"). It clearly states that Jesus is a man.
• "And when I saw Him, I fell at His feet as a dead man. And He laid His right hand upon
me, saying, “Do not be afraid; I am the first and the last, 18
 and the living One; and I was
dead, and behold, I am alive forevermore, and I have the keys of death and of Hades,"
(Rev. 1:17-18).

In Rev. 1:17-18, Jesus is in heaven and John the Apostle falls at Jesus' feet and Jesus laid His
right hand on him. Clearly, from these verses we can see that Jesus is in bodily form as a man.

1. Objections Answered
A. Flesh and blood cannot inherit the Kingdom of God.
Some argue that the Bible says that flesh and blood cannot go to heaven as is stated in 1
Cor. 15:50, "Now I say this, brethren, that flesh and blood cannot inherit the kingdom of
God; nor does the perishable inherit the imperishable."
The term "flesh and blood" is a phrase used to designate the natural state, even the
carnal state of man.
i. "And Jesus answered and said to him, "Blessed are you, Simon Barjona, because flesh
and blood did not reveal this to you, but My Father who is in heaven," (Matt. 16:17).
ii. "to reveal His Son in me, that I might preach Him among the Gentiles, I did not
immediately consult with flesh and blood," (Gal. 1:16).
iii. "For our struggle is not against flesh and blood, but against the rulers, against the
powers, against the world forces of this darkness, against the spiritual forces of
wickedness in the heavenly places," (Eph. 6:12).
iv. "Since then the children share in flesh and blood, He Himself likewise also partook of the
same, that through death He might render powerless him who had the power of death,
that is, the devil," (Heb. 2:14).

After the resurrection, Jesus said, "See My hands and My feet, that it is I Myself; touch Me
and see, for a spirit does not have flesh and bones as you see that I have," (Luke 24:39).
Jesus specifically stated that He had flesh and bones, not flesh and blood. This may seem like a
word game, but it is not. Every word is inspired in the Bible and Jesus chose His words for a
reason. Remember, Jesus' blood was drained out of His body on the cross. It is His blood that
cleanses us of our sins: "but if we walk in the light as He Himself is in the light, we have
fellowship with one another, and the blood of Jesus His Son cleanses us from all sin," (1 John
1:7). Jesus was the sacrifice and His blood cleanses us. Therefore, flesh and blood cannot
inherit the Kingdom of God, but flesh and bones can.

The last Adam became a life-giving spirit.


1 Cor. 15:45 says, "The first man, Adam, became a living soul. The last Adam became a
life-giving spirit." This verse is not saying that Jesus is without a body, but that He is a life
giving spirit. That is, as the last Adam, He is the one who gives life to people (John 10:27-28).
Furthermore, it is designating that Jesus' resurrected body is equipped to be in both the
physical realm and the spiritual.
Jesus is God
"You are my witnesses," declares the LORD, "and my servant whom I have
chosen, so that you may know and believe me and understand that I am he.
Before me no god was formed, nor will there be one after me," (Isaiah 43:10).

JESUS IS GOD, "YAHWEH"

John 1:3, "Through him all things Job 33:4, "The Spirit of God has made me;
were made; without him nothing was the breath of the Almighty gives me life."
made that has been made." Isaiah 40:28, "Do you not know? Have you
Col. 1:16-17, "For by him all things not heard? The LORD is the everlasting God, the
were created: things in heaven and on Creator of the ends of the earth. He will not
Creator
earth, visible and invisible, whether grow tired or weary, and his understanding no
thrones or powers or rulers or one can fathom."
authorities; all things were created by
him and for him. He is before all things,
and in him all things hold together."

Rev. 1:17, "When I saw him, I fell at Isaiah 41:4, "Who has done this and carried it
his feet as though dead. Then he placed through, calling forth the generations from the
his right hand on me and said: ‘Do not beginning? I, the LORD -- with the first of them
be afraid. I am the First and the Last.’" and with the last -- I am he."
Rev. 2:8, "To the angel of the church Isaiah 44:6, "This is what the LORD says --
First and
in Smyrna write: These are the words Israel's King and Redeemer, the LORD
Last
of him who is the First and the Last, Almighty: I am the first and I am the last; apart
who died and came to life again." from me there is no God."
Rev. 22:13, "I am the Alpha and the Isaiah 48:12, "Listen to me, O Jacob, Israel,
Omega, the First and the Last, the whom I have called: I am he; I am the first and
Beginning and the End." I am the last."

John 8:24, "Therefore I said to you Exodus 3:14, "God said to Moses, "I AM WHO
that you will die in your sins; for if you I AM. This is what you are to say to the
do not believe that I am He, you will Israelites: ‘I AM has sent me to you.’"
die in your sins." (NKJV) I AM Isaiah 43:10, "You are my witnesses,"
John 8:58, "I tell you the truth," declares the LORD, "and my servant whom I
Jesus answered, "before Abraham was "ego have chosen, so that you may know and believe
born, I am!" See Exodus 3:14 eimi" me and understand that I am he. Before me no
John 13:19, "I am telling you now god was formed, nor will there be one after
before it happens, so that when it does me."
happen you will believe that I am He." See also Deut. 32:39

2 Tim. 4:1, "In the presence of God Joel 3:12, "Let the nations be roused; let
and of Christ Jesus, who will judge the them advance into the Valley of Jehoshaphat,
living and the dead, and in view of his for there I will sit to judge all the nations on
appearing and his kingdom, I give you every side."
this charge..." Rom. 14:10, "You, then, why do you judge
Judge
2 Cor. 5:10, "For we must all appear your brother? Or why do you look down on your
before the judgment seat of Christ, that brother? For we will all stand before God's
each one may receive what is due him judgment seat."
for the things done while in the body,
whether good or bad."

Matt. 2:2, "...Where is the one who King Jer. 10:10, "But the LORD is the true God; he
has been born king of the Jews? We is the living God, the eternal King. When he is
saw his star in the east and have come angry, the earth trembles; the nations cannot
to worship him." endure his wrath."
Luke 23:3, "So Pilate asked Jesus, Isaiah 44:6-8, "This is what the LORD says --
"Are you the king of the Jews?" "Yes, it Israel's King and Redeemer, the LORD
is as you say," Jesus replied." Almighty: I am the first and I am the last; apart
See also John 19:21 from me there is no God."
See also Psalm 47

John 8:12, "When Jesus spoke again Psalm 27:1, "The LORD is my light and my
to the people, he said, "I am the light salvation -- whom shall I fear?"
of the world. Whoever follows me will Isaiah 60:20,"our sun will never set again,
never walk in darkness, but will have and your moon will wane no more; the LORD
the light of life." Light will be your everlasting light, and your days of
Luke 2:32, "a light for revelation to sorrow will end."
the Gentiles and for glory to your 1 John 1:5, "God is light; in him there is no
people Israel." darkness at all."
See also John 1:7-9

1 Cor. 10:4, "...for they drank from Deut. 32:4, "He is the Rock, his works are
the spiritual rock that accompanied perfect, and all his ways are just. A faithful God
Rock
them, and that rock was Christ." who does no wrong, upright and just is he."
See also 1 Pet. 2:. See also 2 Sam. 22:32 and Isaiah 17:10.

John 4:24, "They said to the woman, Isaiah 43:3, "For I am the LORD, your God,
‘We no longer believe just because of the Holy One of Israel, your Savior."
what you said; now we have heard for Isaiah 45:21, "...And there is no God apart
ourselves, and we know that this man from me, a righteous God and a Savior; there is
Savior
really is the Savior of the world.’" none but me."
1 John 4:14, "And we have seen and
testify that the Father has sent his Son
to be the Savior of the world."

John 10:11, "I am the good Psalm 23:1, "The LORD is my shepherd, I
shepherd. The good shepherd lays shall not be in want."
down his life for the sheep." Isaiah 40:11, "He tends his flock like a
Heb. 13:20, "May the God of peace, shepherd: He gathers the lambs in his arms and
Shepher
who through the blood of the eternal carries them close to his heart; he gently leads
d
covenant brought back from the dead those that have young."
our Lord Jesus, that great Shepherd of
the sheep,"
See also John 10:14,16; 1 Pet. 2:25

Unless otherwise noted, all quotations are from the NIV.


If Jesus were not God, then explain...
If Jesus is not God, then explain...

1. Why Thomas calls Jesus God in John 20:28? (Note, Thomas addresses Jesus specifically.)
2. Why does God call Jesus God in Heb. 1:8?
3. Why does John the apostle state that Jesus was the Word which was God that became flesh (John
1:1,14)?
4. Why is the phrase "Call upon the name of the LORD" (Hebrew, YHWH, i.e., Psalm 116:4) used
only of God on the OT, and translated into the Greek in the LXX as "Call upon the name of the
LORD (Greek, KURIOS)," applied to Jesus in the NT (1 Cor. 1:2) if Jesus is not God in flesh?
5. Why does the apostle John say that Jesus was, "...calling God His own Father, making Himself
equal to God," (John 5:18)?
6. What did Jesus say that caused the Pharisees to claim that Jesus was making Himself out to be
God?
7. How was it possible for Jesus to know all things, (John 21:17)?
8. How can Jesus know all men, (John 16:30)?
9. How can Jesus be everywhere, (Matt. 28:20)?
10. How can Jesus, the Christ, dwell in you, (Col. 1:27)?
11. How can Jesus be the exact representation of the Nature of God, (Heb. 1:3)?
12. How can Jesus be eternal, (Micah 5:1-2)?
13. How can Jesus be the one who gives eternal life, (John 10:27-28)?
14. How can He be our only Lord and Master, (Jude 4)?
15. How can Jesus be called the Mighty God (Isaiah 9:6) if there is only one God in existence, (Isaiah
44:6-8; 45:5)?
16. How can Jesus be called the Mighty God (Isaiah 9:6) and "God" also be called the Mighty God in
Isaiah 10:21?
17. How was Jesus able to raise Himself from the dead, (John 2:19-21)?
18. How can Jesus create all things (Col. 1:16-17), yet it is God who created all things by Himself,
(Isaiah 44:24)?
19. How can Jesus search the hearts and minds of the people, (Rev. 2:23)?
20. Why was Jesus worshiped (Matt. 2:2,11; 14:33; 28:9; John 9:35-38; Heb. 1:6) when He says to
worship God only (Matt. 4:10)? (Same Greek word for worship is used in each place.)
21. In the OT God was seen (Exodus 6:2-3; 24:9-11; Num. 12:6-9; Acts 7:2), yet no man can see
God, (Exodus 33:20; John 1:18). It was not the Father that was seen in the OT, (John 6:46).
Who, then were they seeing? See John 8:58.
22. Then why did Jesus claim the divine name, "I AM", for Himself in John 8:58? see Exodus 3:14.
23. Then why did Jesus say you must honor Him even as you honor the Father, (John 5:23)?
24. Then why is it that both the Father and the Son give life, (John 5:21)?
25. Then why did Jesus bear witness of Himself, (John 8:18; 14:6)?
The Trinity, what is it?
The word "trinity" is a term used to denote the Christian doctrine that God exists as a unity of three
distinct persons: Father, Son, and Holy Spirit. Each of the persons is distinct from the other, yet
related in essence. Each is divine in nature, but each is not the totality of the Godhead. Each has a
will, loves, and says "I", and "You" when speaking. The Father is not the same person as the Son who
is not the same person as the Holy Spirit who is not the same person as the Father. Each is divine, yet
there are not three gods, but one God. There are three persons individual subsistences, or persons.
The word "subsistence" means something that has a real existence. The word "person" denotes
individuality and self awareness. The Trinity is three of these, though the latter term has become the
dominant one used to describe the individual aspects of God known as the Father, the Son, and the
Holy Spirit.
Included in the doctrine of the Trinity is a strict monotheism which is the teaching that there
exists in all the universe a single being known as God who is self-existent and unchangeable (Isaiah
43:10; 44:6,8). Therefore, it is important to note that the doctrine of the trinity is not polytheistic as
some of its critics proclaim. Trinitarianism is monotheistic by definition and those who claim it is
polytheistic demonstrate a lack of understanding of what it really is.

 The Trinity
 God is three persons
 Each person is divine
 There is only one God

Many theologians admit that the term "person" is not a perfect word to describe the three
individual aspects/foci found in God. When we normally use the word person, we understand it to
mean physical individuals who exist as separate beings from other individuals. But in God there are
not three entities, nor three beings. God, is a trinity of persons consisting of one substance and one
essence. God is numerically one. Yet, within the single divine essence are three individual
subsistences that we call persons.

 Each of the three persons is completely divine in nature though each is not the
totality of the Godhead.
 Each of the three persons is not the other two persons.
 Each of the three persons is related to the other two, but are distinct from them.

The word "trinity" is not found in the Bible. But this does not mean that the concept is not taught
there. The word "bible" is not found in the Bible either, but we use it anyway. Likewise, the words
"omniscience," which means "all knowing," "omnipotence," which means "all powerful," and
"omnipresence," which means "present everywhere," are not found in the Bible either. But we use
these words to describe the attributes of God. So, to say that the Trinity isn't true because the word
isn't in the Bible is an invalid argument.

Is there subordination in the Trinity?

There is, apparently, a subordination within the Trinity in regard to order but not substance or
essence. We can see that the Father is first, the Son is second, and the Holy Spirit is third. The Father
is not begotten, but the Son is (John 3:16). The Holy Spirit proceeds from the Father (John 5:26). The
Father sent the Son (1 John 4:10). The Son and the Father send the Holy Spirit (John 14:26; 15:26).
The Father creates (Isaiah 44:24), the Son redeems (Gal. 3:13), and the Holy Spirit sanctifies (Rom.
15:16).
This subordination of order does not mean that each of the members of the Godhead are not equal
or divine. For example, we see that the Father sent the Son. But this does not mean that the Son is
not equal to the Father in essence and divine nature. A wife is to be subject to her husband but this
does not negate her humanity, essence, or equality. By further analogy, a king and his servant both
share human nature. Yet, the king sends the servant to do his will. Jesus said, "For I have come down
from heaven, not to do My own will, but the will of Him who sent Me,” ( John 6:38). Does this mean
that the one sent must, therefore, be of different nature than the one who sent him? Of course not.
Critics of the Trinity will see this subordination as proof that the Trinity is false. They reason that if
Jesus were truly God, then He would be completely equal to God the Father in all areas and would not,
therefore, be subordinate to the Father in any way. But this objection is not logical. If we look at the
analogy of the king and in the servant we certainly would not say that the servant was not human
because he was sent. Being sent does not negate sameness in essence. Therefore, the fact that the
Son is sent does not mean that He is not divine any more than when my wife sends me to get bread, I
am not human.

Is this confusing?

Another important point about the Trinity is that it can be a difficult concept to grasp. But this
does not necessitate an argument against its validity. On the contrary, the fact that it is difficult is an
argument for its truth. The Bible is the self revelation of an infinite God. Therefore, we are bound to
encounter concepts which are difficult to understand -- especially when dealing with an
incomprehensible God who exists in all places at all times. So, when we view descriptions and
attributes of God manifested in the Father, the Son, and the Holy Spirit, we discover that a completely
comprehensible and understandable explanation of God's essence and nature is not possible. What we
have, however, done is derive from the Scripture the truths that we can grasp and combine them into
the doctrine we call The Trinity. The Trinity is, to a large extent, a mystery. After all, we are dealing
with God Himself.
It is the way of the cults to reduce biblical truth to make God comprehensible and understandable
by their minds. To this end, they subject God's word to their own reasoning and end in error. The
following verses are often used to demonstrate that in the doctrine of the Trinity is indeed biblical.

• Matt. 28:18, “Go therefore and make disciples of all the nations, baptizing them in the name of
the Father and the Son and the Holy Spirit,”
• 1 Cor. 12:4-6, “Now there are varieties of gifts, but the same Spirit. 5And there are varieties of
ministries, and the same Lord. 6And there are varieties of effects, but the same God who works
all things in all persons.”
• 2 Cor. 13:14, “The grace of the Lord Jesus Christ, and the love of God, and the fellowship of
the Holy Spirit, be with you all.”
• Eph. 4:4-7, “There is one body and one Spirit, just as also you were called in one hope of your
calling; 5one Lord, one faith, one baptism, 6one God and Father of all who is over all and
through all and in all. 7But to each one of us grace was given according to the measure of
Christ’s gift.”
• 1 Pet. 1:2, "according to the foreknowledge of God the Father, by the sanctifying work of the
Spirit, that you may obey Jesus Christ and be sprinkled with His blood: May grace and peace be
yours in fullest measure."
• Jude 20-21, "But you, beloved, building yourselves up on your most holy faith; praying in the
Holy Spirit; 21keep yourselves in the love of God, waiting anxiously for the mercy of our Lord
Jesus Christ to eternal life."

Sources:

• Baker's Dictionary of Theology, Everett Harrison, ed. Baker Book House, Grand Rapids,
Michigan, 1960.
• Berkhoff's Systematic Theology, Wm. B. Eerdmans Publishing Company, Grand Rapids,
Michigan, 1988.
• Grudem, Wayne, Systematic Theology: An Introduction to Biblical Doctrine, Zondervan
Publishing House, Grand Rapids, MI, 1994.
The Trinity Chart
God is a trinity of persons: the Father, the Son, and the Holy Spirit. The Father is not the
same person as the Son; the Son is not the same person as the Holy Spirit; and the Holy Spirit is
not the same person as Father. They are distinct persons; yet, they are all the one God. They are
in absolute perfect harmony consisting of one substance. They are coeternal, coequal, and
copowerful. If any one of the three were removed, there would be no God. (See also, "Another
Look at the Trinity")
There is, though, an apparent separation of some functions among the members of the
Godhead. For example, the Father chooses who will be saved (Eph. 1:4); the Son redeems them
(Eph. 1:7); and the Holy Spirit seals them, (Eph. 1:13).
A further point of clarification is that God is not one person, the Father, with Jesus as a
creation and the Holy Spirit is a force (Jehovah's Witnesses). Neither is He one person who took
three consecutive forms, i.e., the Father, became the Son, who became the Holy Spirit. Nor is
God the divine nature of the Son (where Jesus had a human nature perceived as the Son and a
divine nature perceived as the Father (United Pentecostal). Nor is the Trinity an office held by
three separate Gods (Mormonism).
The chart below should help you to see how the doctrine of the Trinity is derived from
Scripture. The list is not exhaustive, only illustrative.
The first step is to establish how many Gods exist: one! Isaiah 43:10; 44:6; 45:14,18, 21,
22; 46:5,9.

"I am the LORD, and there is no other; besides Me there is no God," (Isaiah 45:5).

The Trinity
FATHER SON HOLY SPIRIT
Called God Phil. 1:2 John 1:1,14; Col. 2:9 Acts 5:3-4
Creator Isaiah 64:8 John 1:3; Col. 1:15-17 Job 33:4, 26:13
Resurrects 1 Thess. 1:10 John 2:19, 10:17 Rom. 8:11
Indwells 2 Cor. 6:16 Col. 1:27 John 14:17
Everywhere 1 Kings 8:27 Matt. 28:20 Psalm 139:7-10
All knowing 1 John 3:20 John 16:30; 21:17 1 Cor. 2:10-11
Sanctifies 1 Thess. 5:23 Heb. 2:11 1 Pet. 1:2
Life giver Gen. 2:7: John 5:21 John 1:3; 5:21 2 Cor. 3:6,8
Fellowship 1 John 1:3 1 Cor. 1:9 2 Cor. 13:14; Phil. 2:1
Eternal Psalm 90:2 Micah 5:1-2 Rom. 8:11; Heb. 9:14
A Will Luke 22:42 Luke 22:42 1 Cor. 12:11
Speaks Matt. 3:17; Lk 9:25 Luke 5:20; 7:48 Acts 8:29; 11:12; 13:2
Love John 3:16 Eph. 5:25 Rom. 15:30
Searches the heart Jer. 17:10 Rev. 2:23 1 Cor. 2:10
We belong to John 17:9 John 17:6 ...
1 Tim. 1:1; 2:3; 2 Tim. 1:10; Titus 1:4;
Savior ...
4:10 3:6
We serve Matt. 4:10 Col. 3:24 ...
Believe in John 14:1 John 14:1 ...
Gives joy ... John 15:11 John 14:7
Judges John 8:50 John 5:21,30 ...
Another Look at the Trinity
The Trinity can be a difficult concept to understand. Some think it is a logical contradiction. Others
call it a mystery. Does the Bible teach it? Yes it does, see trinity, but that doesn't automatically make
it easier to comprehend.
The Trinity is defined as one God who exists in three eternal, simultaneous, and distinct persons
known as the Father, the Son, and the Holy Spirit. Such a definition may suffice for some, but for
others this explanation is insufficient.
Therefore, to help understand the Trinity better, I offer the following analogy that, I think, is
hinted at in Rom. 1:20: "For since the creation of the world His invisible attributes, His eternal power
and divine nature, have been clearly seen, being understood through what has been made."
Notice that this verse says God's attributes, power, and nature, can be clearly seen in creation.
What does that mean? Should we be able to learn about God's attributes, power, and nature by
looking at what He has made? Apparently, according to the Bible, this is possible.
When a painter paints a picture, what is in him is reflected in the painting he produces. When a
sculptor creates a work of art, it is from his heart and mind that the source of the sculpture is born.
The work is shaped by his creative ability. The creators of art leave their marks, something that is
their own, something that reflects what they are. Is this the same with God? Has God left His
fingerprints on creation? Of course He has.

Creation

Basically, the universe consists of three elements: Time, Space, and Matter. Each of these is
comprised of three 'components.'

Time Past Present Future


Space Height Width Depth
Matter Solid Liquid Gas

TIME SPACE MATTER

As the Trinitarian doctrine maintains, each of the persons of the Godhead is distinct, yet they are
all each, by nature, God.
With time, for example, the past is distinct from the present, which is distinct from the future. Each
is simultaneous. Yet, they are not three 'times,' but one. That is, they all share the same nature: time
With space, height is distinct from width, which is distinct from depth, which is distinct from
height. Yet, they are not three 'spaces,' but one. That is, they all share the same nature: space.
With matter, solid is not the same as liquid, which is not the same as gas, which is not the same
as solid. Yet, they are not three 'matters,' but one. That is, they all share the same nature: matter.
Note that there are three sets of threes. In other words, there is a trinity of trinities. If we were to
look at the universe and notice these qualities within it, is it fair to say that these are the fingerprints
of God upon His creation? I think so. Not only is this simply an observation, but it is also a good source
for an analogy of the Trinity.
A Criticism of Trinitarianism

Some critiques of the Trinitarian doctrine say that the Trinity is really teaching three gods, not one.
They will say that God the Father, and God the Son, and God the Holy Spirit would make three gods,
since the Father plus the Son plus the Holy Spirit would make three. But this is not a logical necessity.
Instead of adding, why not multiply? One times one times one equals one. Why must addition be the
criteria by which the doctrine is judged? It need not be. Rather, the doctrine should stand or fall based
upon biblical revelation, not human logic. Nevertheless, let me draw an analogy from creation itself to
illustrate the doctrine of the Trinity.

An Analogy of the Trinity

To continue with the observation about the Trinitarian nature of creation, I would like to use 'time'
to illustrate the Trinity. Is the "past" plus the "present" plus the "future" a total of three times? Not at
all. It simply is a representation of three distinct aspects of the nature of time: past, present, and
future. Likewise, the Father and the Son and Holy Spirit are not three separate beings or entities, but
three distinct persons in the one nature of the Godhead.
One more comment about Jesus. All cults deny that Jesus is God, the creator of the universe, in
flesh. Various objections are raised saying that Jesus could not be God, otherwise, He would be
praying to Himself, etc. Let's work with the analogy above, and continue with 'time' as our illustration.
Let's take 'present' and add to it human nature. Present, then, would have two natures: time and
man. If 'present' were truly human then he would be able to communicate with us, tell us much, and
we could see and touch him. But, because he is also 'time' by nature, he would be able to tell us both
the past and the future as he manifested the 'time' nature within him. If 'present' then, communicated
with the past and the future, it would not mean he was communicating with himself, but with the
distinctions known as the Past and the future.
I know that this is only an analogy. But I think it is a good, though basic, illustration of God's
nature as expressed in Trinitarian expression.
Early Trinitarian Quotes
There are cult groups (Jehovah's Witnesses, The Way International, Christadelphians, etc.) who
deny the Trinity and state that the doctrine was not mentioned until the 4th Century until after the
time of the Council of Nicea (325). This council "was called by Emperor Constantine to deal with the
error of Arianism which was threatening the unity of the Christian Church."
The following quotes show that the doctrine of the Trinity was indeed alive-and-well before the
Council of Nicea.

Polycarp (70-155/160). Bishop of Smyrna. Disciple of John the Apostle.


"O Lord God almighty...I bless you and glorify you through the eternal and heavenly high priest
Jesus Christ, your beloved Son, through whom be glory to you, with Him and the Holy Spirit, both now
and forever" (n. 14, ed. Funk; PG 5.1040).

Justin Martyr (100?-165?). He was a Christian apologist and martyr.


"For, in the name of God, the Father and Lord of the universe, and of our Savior Jesus Christ, and
of the Holy Spirit, they then receive the washing with water" (First Apol., LXI).

Ignatius of Antioch (died 98/117). Bishop of Antioch. He wrote much in defense of Christianity.
"In Christ Jesus our Lord, by whom and with whom be glory and power to the Father with the Holy
Spirit for ever" (n. 7; PG 5.988).
"We have also as a Physician the Lord our God Jesus the Christ the only-begotten Son and Word,
before time began, but who afterwards became also man, of Mary the virgin. For ‘the Word was made
flesh.' Being incorporeal, He was in the body; being impassible, He was in a passable body; being
immortal, He was in a mortal body; being life, He became subject to corruption, that He might free our
souls from death and corruption, and heal them, and might restore them to health, when they were
diseased with ungodliness and wicked lusts." (Alexander Roberts and James Donaldson, eds., The
ante-Nicene Fathers, Grand Rapids: Eerdmans, 1975 rpt., Vol. 1, p. 52, Ephesians 7.)

Irenaeus (115-190). As a boy he listened to Polycarp, the disciple of John. He became Bishop of Lyons.
"The Church, though dispersed throughout the whole world, even to the ends of the earth, has
received from the apostles and their disciples this faith: ...one God, the Father Almighty, Maker of
heaven, and earth, and the sea, and all things that are in them; and in one Christ Jesus, the Son of
God, who became incarnate for our salvation; and in the Holy Spirit, who proclaimed through the
prophets the dispensations of God, and the advents, and the birth from a virgin, and the passion, and
the resurrection from the dead, and the ascension into heaven in the flesh of the beloved Christ Jesus,
our Lord, and His manifestation from heaven in the glory of the Father ‘to gather all things in one,' and
to raise up anew all flesh of the whole human race, in order that to Christ Jesus, our Lord, and God,
and Savior, and King, according to the will of the invisible Father, ‘every knee should bow, of things in
heaven, and things in earth, and things under the earth, and that every tongue should confess; to
him, and that He should execute just judgment towards all...'" (Against Heresies X.l)

Tertullian (160-215). African apologist and theologian. He wrote much in defense of Christianity.
"We define that there are two, the Father and the Son, and three with the Holy Spirit, and this
number is made by the pattern of salvation...[which] brings about unity in trinity, interrelating the
three, the Father, the Son, and the Holy Spirit. They are three, not in dignity, but in degree, not in
substance but in form, not in power but in kind. They are of one substance and power, because there
is one God from whom these degrees, forms and kinds devolve in the name of Father, Son and Holy
Spirit." (Adv. Prax. 23; PL 2.156-7).

Origen (185-254). Alexandrian theologian. A disciple of Origen. Defended Christianity. He wrote much
about Christianity.
"If anyone would say that the Word of God or the Wisdom of God had a beginning, let him beware
lest he direct his impiety rather against the unbegotten Father, since he denies that he was always
Father, and that he has always begotten the Word, and that he always had wisdom in all previous
times or ages or whatever can be imagined in priority...There can be no more ancient title of almighty
God than that of Father, and it is through the Son that he is Father" (De Princ. 1.2.; PG 11.132).
"For if [the Holy Spirit were not eternally as He is, and had received knowledge at some time and
then became the Holy Spirit] this were the case, the Holy Spirit would never be reckoned in the unity
of the Trinity, i.e., along with the unchangeable Father and His Son, unless He had always been the
Holy Spirit." (Alexander Roberts and James Donaldson, eds., The Ante-Nicene Fathers, Grand Rapids:
Eerdmans, 1975 rpt., Vol. 4, p. 253, de Principiis, 1.111.4)
"Moreover, nothing in the Trinity can be called greater or less, since the fountain of divinity alone
contains all things by His word and reason, and by the Spirit of His mouth sanctifies all things which
are worthy of sanctification..." (Roberts and Donaldson, Ante-Nicene Fathers, Vol. 4, p. 255, de
Principii., I. iii. 7).

If, as the anti-Trinitarians maintain, the Trinity is not a biblical doctrine and was never taught until
the council of Nicea in 325, then why do these quotes exist? The answer is simple: the Trinity is a
biblical doctrine and it was taught before the council of Nicea in 325 A.D.
Part of the reason that the Trinity doctrine was not "officially" taught until the time of the Council
of Nicea is because Christianity was illegal until shortly before the council. It wasn't really possible for
official Christian groups to meet and discuss doctrine. For the most part, they were fearful of making
public pronouncements concerning their faith.
Additionally, if a group had attacked the person of Adam, the early church would have responded
with an official doctrine of who Adam was. As it was, the person of Christ was attacked. When the
Church defended the deity of Christ, the doctrine of the Trinity was further defined.
The early church believed in the Trinity, as is evidenced by the quotes above, and it wasn't
necessary to really make them official. It wasn't until errors started to creep in, that councils began to
meet to discuss the Trinity as well as other doctrines that came under fire.
The Bible
Introduction

The Bible is the foundation and source of the Christian faith given by God as a record of the creation,
fall, history, and redemption of man. Knowing it, knowing its truths, and knowing the facts concerning
it are important foundational issues necessary to more fully grow in relationship with the Lord Jesus.

1. How many men wrote the Bible over how many years? p. 62
2. What is the Pentateuch? p. 68
3. What is the last book in the Old Testament? p. 69
4. About how many authors wrote Psalms? p. 72
5. Are there other books mentioned in the Bible? Is this important? p. 74
6. Which came first, the birth of Noah or Abraham? p. 75
7. What are some of the non-biblical references to biblical events? pp. 79-81
8. What is the Documentary Hypothesis of the Pentateuch? p. 82-83
9. What did Jesus say about the Pentateuch? p. 86
10. What are some things you should consider in biblical interpretation? p. 87
11. Give two scientific accuracies of the Bible. p. 91
12. Give two O.T. prophecies of Jesus, with verse location. P. 92-95
13. What are the mathematical Odds of Jesus fulfilling 48 prophecies? p. 95
14. What is redaction criticism? p. 96
The Bible
It has been reported for about 50 years that the Bible has been the largest seller of all books
published in the history of the world.
The Bible was written by about 40 men in about 1600 years dating from 1500 B.C. to about 100
A.D. These men wrote as they were moved by the Holy Spirit (2 Pet. 1:21). They wrote not in words of
human wisdom but in words taught by the Holy Spirit (1 Cor. 2:13).

English Bible

• The first translation of the English Bible was initiated by John Wycliffe and completed by John
Purvey in A.D. 1388.
• The first American edition of the Bible was perhaps published some time before A.D. 1752.
• The Bible has been translated in part or in whole as of 1964 in over 1,200 different languages
or dialects.
• Stephen Langton divided the Bible into chapters about A.D. 1228.
• The Old Testament was divided into verses by R. Nathan in A.D. 1448 and the New Testament
by Robert Stephanus in A.D. 1551.
• There are 66 books in the Bible, 39 in the OT and 27 in the new. (Note: 3 x 9 = 27).
• The OT has 929 chapters and 23,214 verses. The NT has 260 chapters and 7,959 verses.
• In the OT, the longest book is Psalms. The shortest book is Obadiah.
• In the NT, the longest book is Acts. The shortest is 3 John.
• The word "God" occurs 4,379 times. The word "Lord" occurs 7,738 times.
• Isaiah is referenced 419 times in 23 NT books; Psalms 414 times in 23 books; Genesis 260
times in 21 books.

Unusual things in the Bible

• Methuselah who lived to be 969 years old (Gen. 5:27).


• Sons of God married the daughters of men (Gen. 6:2).
• Baby had a scarlet thread tied around its hand before it was born (Gen. 38:28-29).
• Battle won because a man stretched out his hand (Exodus 17:11).
• Man was spoken to by a donkey (Num. 22:28-30).
• One who had a bed 13½ feet long and 6 feet wide (Deut. 3:11).
• The women who had to shave their heads before they could marry (Deut. 21:11-13).
• Sun stood still for a whole day (Josh. 10:13).
• An army with 700 left handed men (Judges 20:16).
• Man whose hair weighed about 6 pounds when it was cut annually (2 Sam. 14:26).
• Man who had 12 fingers and 12 toes (2 Sam. 21:20).
• Father who had eighty-eight children (2 Chron. 11:21).
• The sun traveled backward (Isaiah 38:8).
• A harlot was an ancestor of Christ (Matt. 1:5).
The Greek and Hebrew Alphabet with numeric equivalents
In English, we have alpha and numeric characters: a,b,c,d,e, etc., & 0,1,2,3,4, etc. Both Greek
and Hebrew have only one set of characters used for both words and numbers.
When a word in Greek, for example, is written out, it has a mathematical value. This value is
called the gamatria. The Greek word for Jesus (Ιη σ ο υ ς ) has a mathematical equivalent of 888.
There are many very interesting mathematical relationships when the gamatria of words and
sentences are examined.

Greek Greek English Numeric Greek Greek English Numeric


Letter Name Letter Value Letter Name Letter Value
Α ,α alpha father 1 Ν, ν nu new 50
Β,
Beta boy 2 Ξ, ξ xi box 60
β
Γ,γ Gamma god 3 Ο, ο omikron off 70
∆,δ delta dog 4 Π, π pi pet 80
90 is
Ε,ε epsilon end 5
Ρ, ρ rho rat 100
6 is sigma,
ς final Σ, σ
sigma sam (s,V) 6, 200
form ,ς *
Ζ, ζ zeta zebra 7
Η,η eta hey 8 Τ, τ tau test 300
Θ, θ theta thick (q) 9 Υ, υ upsilon up 400
Ι,ι iota it 10 Φ,φ phi phone (f) 500
Κ ,κ kappa kit 20 Χ ,χ chi bach 600
Λ ,λ lamda low 30 Ψ, ψ psi cups (y) 700
800
Μ, µ Ω, ω orange
mu mike 40 omega 900 =
(w)
sampsi

Hebrew Hebrew English Numeric Hebrew Hebrew English Numeric


Letter Name Letter Value Letter Name Letter Value

‫א‬ aleph silent (‫ )א‬1 or 1000 ‫ל‬ lamed low 30

‫ב‬ Bet boy 2 ‫ם‬,‫מ‬ * mem mike 40

‫ג‬ gimel god 3 ‫נ‬,‫ן‬ * nun new 50

deled dog 4 samech sam 60


‫ד‬ ‫ס‬
‫ה‬ heh hi 5 ‫ע‬ ayin silent 70

‫ו‬ vov very, up 6 ‫פ‬,‫ף‬ * pey pet, phone 80

‫ז‬ zayin zebrea 7 ‫צ‬,‫ץ‬ * tzadi nets 90

‫ח‬ ches bach 8 ‫ק‬ kuf q, kit 100

‫ט‬ tes test 9 ‫ר‬ reish race 200

‫י‬ yod you 10 ‫ש‬ shin sam, shem 300

‫כ‬,‫ך‬ * chof kit 20 ‫ת‬ tof test 400

*final form is form of letter found when it occurs at the end of a word.
Letter Hebrew Numeric
final form Name Value

‫ך‬ final chof 500

‫מ‬ final mem 600

‫ן‬ final nun 700

‫ף‬ final pey 800

‫ץ‬ final tzadi 900


Papyri p1 through p76 200 A.D. to 700 A.D.
The following are extant papyrus fragments and copies. These manuscripts, designated by p1, p2,
etc., are part of the overall existing biblical manuscript scope that we now possess. This range of
papyri are very old. They represent the extant copies made in the dates so designated.

Date
Manuscript # Content Location
Copied (Century)
p1 Gospels Philadelphia 3rd
p4 Gospels Paris 3rd
p5 Gospels London 3rd
p6 Gospels Strassburg 4th
p8 Acts Berlin 4th
p10 Epistles of Paul Cambridge, Mass 4th
p13 Epistles of Paul London and Florence 3rd/4th
p15 Epistles of Paul Cairo 3rd
p23 General Epistles Urbana, 111. Early 3rd
p24 Revelation Newton Center, Mass. 4th
p25 Gospels Berlin Late 4th
p27 Epistles of Paul Cambridge 3rd
p30 Epistles of Paul Ghent 3rd
p37 Gospels Ann Arbor, Mich. 3rd/4th
p38 Acts Ann Arbor, Mich. About 300
p39 Gospels Chester, Pa. 3rd
p40 Epistles of Paul Heidelberg 3rd
Dublin: Chester Beatty, and
p45 Gospels, Acts 3rd
Vienna
Dublin: Chester Beatty, and
p46 Epistles of Paul About 200
Ann Arbor, Mich.
p47 Revelation Dublin: Chester Beatty Late 3rd
p48 Acts Florence Late 3rd
p49 Epistles of Paul New Haven, Conn. Late 3rd
p51 Epistles of Paul P. Oxy. 2157 About 400
p64 Gospels Oxford About 200
p65 Epistles of Paul Florence 3rd
p66 Gospels Geneva: P. Bodmer ii About 200
p67 Gospels Barcelona About 200
p70 Gospels P. Oxy. 2384 3rd
p71 Gospels P. Oxy. 2385 4th
p72 General Epistles Geneva: P. Bodmer xvii 3rd/4th

p76 Gospels Geneva: P. Bodmer xiv, xv Early 3rd


Minuscules 2 through 399 - 9th to 16th century copies
Minuscules are lowercase Greek letters. These manuscripts were written in the Greek lower case.
They represent copies made from the 9th to the 16th centuries.

Number Content Date


2 Acts & Catholic Epistles, Epistles of Paul 12th
3 Gospels, Acts & Catholic Epistles, Epistles of Paul 12th
4 Gospels 13th
4 Acts & Catholic Epistles, Epistles of Paul 15th
5 Gospels, Acts & Catholic Epistles, Epistles of Paul 14th
6 Gospels, Acts & Catholic Epistles, Epistles of Paul 13th
7 Epistles of Paul 11th
8 Gospels 11th
10 Gospels 13th
16 Gospels 14th
17 Gospels 15th
Gospels, Acts & Catholic Epistles, Epistles of Paul,
18 1364
Revelation
21 Gospels 12th
22 Gospels 12th
25 Gospels 11th
29 Gospels 10th
31 Gospels 13th
Gospels, Acts & Catholic Epistles, Epistles of Paul,
35 11th
Revelation
36 Acts & Catholic Epistles 12th
37 Gospels 11th
38 Gospels, Acts & Catholic Epistles, Epistles of Paul 13th
39 Gospels 11th
42 Acts & Catholic Epistles, Epistles of Paul, Revelation 11th
43 Gospels, Acts & Catholic Epistles, Epistles of Paul 12th
47 Gospels 15th
51 Gospels, Acts & Catholic Epistles, Epistles of Paul 13th
53 Gospels 14th
55 Gospels, 13th
56 Gospels 15th
57 Gospels, Acts & Catholic Epistles, Epistles of Paul 10th/11th
58 Gospels 15th
Minuscules 404 through 999 -- 8th to 16th century copies
Minuscules are lowercase Greek letters. These manuscripts were written in the Greek lower case.

Number Content Date


404 Acts & Catholic Epistles, Epistles of Paul 14th
407 Gospels 12th
418 Gospels 15th
424 Acts & Catholic Epistles, Epistles of Paul, Revelation 11th
425 Acts & Catholic Epistles, Epistles of Paul 1330
429 Acts & Catholic Epistles, Epistles of Paul; Revelation 14th;15th
431 Gospels, Acts & Catholic Epistles, Epistles of Paul 11th
435 Gospels 10th
437 Acts & Catholic Epistles 11th
440 Gospels, Acts & Catholic Epistles, Epistles of Paul 12th
441 Acts & Catholic Epistles, Epistles of Paul 13th
442 Acts & Catholic Epistles, Epistles of Paul 13th
450 Acts & Catholic Epistles, Epistles of Paul 10th
453 Acts & Catholic Epistles 14th
455 Acts & Catholic Epistles, Epistles of Paul 8th/9th
456 Acts & Catholic Epistles, Epistles of Paul, Revelation 10th
459 Acts & Catholic Epistles, Epistles of Paul, Revelation 1092
460 Acts & Catholic Epistles, Epistles of Paul 13th
462 Acts & Catholic Epistles, Epistles of Paul 13th
463 Acts & Catholic Epistles, Epistles of Paul 12th
464 Acts & Catholic Epistles, Epistles of Paul 11th
465 Acts & Catholic Epistles, Epistles of Paul 11th
466 Acts & Catholic Epistles, Epistles of Paul 11th
467 Acts & Catholic Epistles, Epistles of Paul, Revelation 15th
468 Acts & Catholic Epistles, Epistles of Paul, Revelation 13th
469 Acts & Catholic Epistles, Epistles of Paul, Revelation 13th
471 Gospels 12th
Gospels, Acts & Catholic Epistles, Episltes of Paul,
506 11th
Revelation
Gospels, Acts & Catholic Epistles, Episltes of Paul,
517 11th/12th
Revelation
Gospels, Acts & Catholic Epistles, Episltes of Paul,
522 1515
Revelation
914 Acts & Catholic Epistles, Epistles of Paul 13th
915 Acts & Catholic Epistles, Epistles of Paul 13th
917 Acts & Catholic Epistles, Epistles of Paul 12th
918 Acts & Catholic Epistles, Epistles of Paul 16th
Old Testament Books
Old Testament - 39 books total

Pentateuch - 5 books
Genesis, Exodus, Leviticus, Numbers, Deuteronomy

Historical Books - 12 books


Joshua, Judges, Ruth, First Samuel, Second Samuel, First Kings, Second Kings, First Chronicles,
Second Chronicles, Ezra, Nehemiah, Esther.

Poetic - 5 books
Job, Psalms, Proverbs, Ecclesiastes, Song of Solomon

Prophetic - 17 books
Major Prophets - Isaiah, Jeremiah, Lamentations, Ezekiel, Daniel
Minor Prophets - Hosea, Joel, Amos, Obadiah, Jonah, Micah, Nahum, Habakkuk, Zephaniah,
Haggai, Zechariah, Malachi.

Pentateuch - 5 books

1. Genesis - Creation, the Fall, the Flood, spread of the nations, Abraham, Isaac, Jacob, and
Joseph. Enslavement in Egypt.
2. Exodus - Enslavement, Moses, 10 plagues, Passover, Leave Egypt, Red Sea Crossing, Mt. Sinai
and the 10 commandments
3. Leviticus - Instructions on sacrificial system and the priesthood. Instructions on moral purity.
4. Numbers - Still at Mt. Sinai, people make false idol, punishment, 40 years wandering begins.
5. Deuteronomy - Moses' discourses on God's Acts for Israel the Decalogue, the ceremonial, civil,
and social Laws, and covenant ratification.

Historical Books - 12 books total

1. Joshua - First half of Joshua describes the 7 year conquest of the Land of Promise. The last half
deals with partitioning the Lands to the people.
2. Judges - Time of Judges. Many were quite bad. The Israelites did not drive out all the
inhabitants of Canaan and begin to take part in their idolatry. 7 cycles of foreign oppression,
repentance, and deliverance. In the end, the people failed to learn their lesson.
3. Ruth - Kinsman redeemer in Boaz, redeeming Ruth, a Moabitess. Speaks of righteousness, love,
and faithfulness to the Lord.

The next 6 books trace the time from Samuel to the Captivity

1. First Samuel - Samuel carries them from judges to King Saul


2. Second Samuel - David as King, adultery, and murder.
3. First Kings - Solomon, Israel is powerful. Solomon dies in 931 B.C., then division of tribes: 10
to the north and 2 to the south.
4. Second Kings - The Divided Kingdom. All 19 kings of Israel were bad; therefore, captivity in
Assyria (722 B.C.). In Judah, 8 of 20 rulers were good but went into exile too.
5. First Chronicles - A recounting of the history of Israel to the time of the captivities.
6. Second Chronicles - continued recounting of the life of Solomon, building of temple. History of
Judah only.
The Next 3 books deal with Israel's Restoration.

1. Ezra - Cyrus let the most of the Jews return to their land of Israel. Zerubbabel led the people
(539 B.C.). Ezra returned later with more Jews (458 B.C.) Built the temple.
2. Nehemiah - Building the walls of Jerusalem. Nehemiah got permission from the king of Persia to
rebuild the walls (444 B.C.). Revival in the land.
3. Esther - Took place during chapters 6 and 7 of Ezra. Mordecai. Plot to kill the Jewish people.

Poetical - 5 books

1. Job - a righteous man tested by God. Deals with God's sovereignty.


2. Psalms - Consists of 5 divisions. Worship in song. Large variety of subjects
3. Proverbs - Practical wisdom in everyday affairs.
4. Ecclesiastes - All is vanity. The wisdom of man is futility.
5. Song of Solomon - A song between Solomon and his Shulammite bride displaying the love
between a man and a woman.

Prophetical - 17 books

Major Prophets - 5 books

1. Isaiah - Looks at the sin of Judah and proclaims God's judgment. Hezekiah. Coming restoration
and blessing.
2. Jeremiah - Called by God to proclaim the news of judgment to Judah, which came. God
establishes a New Covenant.
3. Lamentations - 5 lament poems. Description of defeat and fall of Jerusalem.
4. Ezekiel - He ministered to the Jews in Captivity in Babylon. Description of the end of times.
5. Daniel - Many visions of the future for the Gentiles and the Jews.

Minor Prophets

6. Hosea - Story of Hosea and his unfaithful wife, Gomer. Represents God's love and faithfulness
and Israel's spiritual adultery. Israel will be judged and restored.
7. Joel - Proclaims a terrifying future using the imagery of locusts. Judgment will come but
blessing will follow.
8. Amos - He warned Israel of its coming judgment. Israel rejects God's warning.
9. Obadiah - A proclamation against Edom, a neighboring nation of Israel that gloated over
Jerusalem's judgments. Prophecy of their utter destruction.
10. Jonah - Jonah proclaims a coming judgment upon Nineveh which repents and are spared.
11. Micah - Description of the complete moral decay in Israel. God will judge but will forgive.
12. Nahum - Nineveh is now in apostasy (appr.. 125 years after Jonah) and will be destroyed.
13. Habakkuk - Near the end of the kingdom of Judah, Habakkuk asks God why He is not dealing
with Judah's sins. God says He will use the Babylonians. Habakkuk asks how God can use a
nation that is even worse than Judah.
14. Zephaniah - The theme is developed of the Day of the Lord and His judgment with a coming
blessing. Judah will not repent, except for a remnant, who will be restored.
15. Haggai - The people failed to put God first, by building their houses before they finished God's
temple. Therefore, they had no prosperity.
16. Zechariah - Zechariah encourages the Jews to complete the temple. Many messianic
prophecies.
17. Malachi. - God's people are lax in their duty to God. Growing distant from God. Moral
compromise. Proclamation of coming judgment.
New Testament Books

• Historical Books - Matthew, Mark, Luke, John, Acts


• Pauline Epistles - Romans, 1 Corinthians, 2 Corinthians, Galatians, Ephesians, Philippians,
Colossians, 1 Thessalonians, 2 Thessalonians. 1 Timothy, 2 Timothy, Titus, Philemon
• Non-Pauline Epistles - Hebrews, James, 1 Peter, 2 Peter, 1 John, 2 John, 3 John, Jude,
Revelation

Historical Books

1. Matthew - Presents Jesus as the Messiah. Genealogy of Jesus through Joseph. Fulfillment of
O.T. prophecy.
2. Mark - Presents Jesus as the Servant. 1/3 of the gospel deals with the last week of His life.
3. Luke - Presents Jesus as the Son of Man to seek and save the lost. Genealogy of Jesus
through Mary. Largest of the gospels.
4. John - Presents Jesus as God in flesh, the Christ, so that you might believe.
5. Acts - Historical account from Jesus’ ascension to travels of Paul in his missionary journeys.

Pauline Epistles

1. Romans - A systematic examination of justification, sanctification, and glorification. Examines


God’s plan for the Jews and the Gentiles.
2. 1 Corinthians - This letter deals with factions and corrections due to immorality, lawsuits, and
abuse of the Lord’s Supper. Also mentions idols, marriage, and the resurrection.
3. 2 Corinthians - Paul’s defense of his apostolic position.
4. Galatians - Paul refutes the errors of legalism and examines the proper place of grace in the
Christian’s life.
5. Ephesians - The believer’s position in Christ and information on Spiritual warfare.
6. Philippians - Paul speaks of his imprisonment, his love for the Philippians. He exhorts them to
godliness and warns them of legalism.
7. Colossians - Paul focuses on the preeminence of Jesus in creation, redemption, and godliness.
8. 1 Thessalonians - Paul’s ministry to the Thessalonians. Teachings on purity and mention of the
return of Christ.
9. 2 Thessalonians - Corrections on the Day of the Lord.
10. 1 Timothy - Instructions to Timothy on proper leadership and dealings with false teachers, the
role of women, prayer, and requirements of elders and deacons.
11. 2 Timothy - A letter of encouragement to Timothy to be strong.
12. Titus - Paul left Titus in Crete to care for the churches there. Requirements for elders.
13. Philemon - a letter to the owner of a runaway slave. Paul appeals to Philemon to forgive
Onesimus.
Non Pauline Epistles.

1. Hebrews - A letter to the Hebrew Christians in danger of returning to Judaism. It


demonstrates the superiority of Jesus over the O.T. system. Mentions the Melchizedek
priesthood. (Hebrews may be of Pauline origin. There is much debate on its authorship).
2. James - a practical exhortation of to live a Christian life evidencing regeneration. It urges self
examination of the evidence of the changed life.
3. 1 Peter - Peter wrote this letter to encourage its recipients in the light of their suffering and be
humble in it. Mentions baptism.
4. 2 Peter - Deals with the person on an inward level, warnings against false teachers, and
mentions the Day of the Lord.
5. 1 John - John describes true fellowship of the believer with believer and with God. Describes
God as light and love. Encourages a holy Christian walk before the Lord. Much mention of
Christian love.
6. 2 John - Praise for walking in Christ and a reminder to walk in God’s love.
7. 3 John - John thanks Gaius for his kindness to God’s people and rebukes Diotrephes.
8. Jude - Exposing false teachers and uses O.T. allusions to demonstrate the judgment upon
them. Contend for the faith.
9. Revelation - A highly symbolic vision of the future rebellion, judgment, and consummation of all
things.
When was the Bible written and who wrote it?
The following dates are not always exact, but are very good estimates.

Old Testament

Book Author Date Written


Genesis Moses ? - 1445 B.C.
Exodus Moses 1445 - 1405 B.C.
Leviticus Moses 1405 B.C.
Numbers Moses 1444 - 1405 B.C.
Deuteronomy Moses 1405 B.C.
Joshua Joshua 1404-1390 B.C.
Judges Samuel 1374-1129 B.C.
Ruth Samuel 1150? B.C.
1 Samuel Samuel 1043-1011 B.C.
2 Samuel Ezra? 1011-1004 B.C.
1 Kings Jeremiah? 971-852 B.C.
2 Kings Jeremiah? 852-587 B.C.
1 Chronicles Ezra? 450 - 425 B.C.
2 Chronicles Ezra? 450 - 425 B.C.
Ezra Ezra 538-520 B.C.
Nehemiah Nehemiah 445 - 425 B.C.
Esther Mordecai? 465 B.C.
Job Job? ??
David 1000? B.C.

Psalms Son of Korah wrote Psalms 42, 44-49, 84-85, 87; Asaph Psalms 50, 73-83;
Heman Psalm 88; Ethan Psalm 89; Hezekiah Psalms 120-123, 128-130, 132,
134-136; Solomon Psalms 72, 127.
Solomon wrote 1-29
Proverbs Agar wrote 30 950 - 700 B.C.
Lemuel wrote 31
Ecclesiastes Solomon 935 B.C.
Song of Solomon Solomon 965 B.C.
Isaiah Isaiah 740 - 680 B.C.
Jeremiah Jeremiah 627 - 585 B.C.
Lamentations Jeremiah 586 B.C.
Ezekiel Ezekiel 593-560 B.C.
Daniel Daniel 605-536 B.C.
Hosea Hosea 710 B.C.
Joel Joel 835 B.C.
Amos Amos 755 B.C.
Obadiah Obadiah 840 or 586 B.C.
Jonah Jonah 760 B.C.
Micah Micah 700 B.C.
Nahum Nahum 663 - 612 B.C.
Habakkuk Habakkuk 607 B.C.
Zephaniah Zephaniah 625 B.C.
Haggai Haggai 520 B.C.
Zechariah Zechariah 520 - 518 B.C.
Malachi Malachi 450 - 600 B.C.

New Testament

Book Author Date Written


Matthew Matthew 60's
Mark John Mark late 50's, early 60's
Luke Luke 60
John John late 80's, early 90's
Acts Luke 61
Romans Paul 55
1 Corinthians Paul 54
2 Corinthians Paul 55
Galatians Paul 49
Ephesians Paul 60
Philippians Paul 61
Colossians Paul 60
1 Thessalonians Paul 50 - 51
2 Thessalonians Paul 50 - 51
1 Timothy Paul 62
2 Timothy Paul 63
Titus Paul 62
Philemon Paul 60
Hebrews (Paul, Apollos, Barnabas...?) 60's
James James, half brother of Jesus 40's or 50's
1 Peter Peter 63
2 Peter Peter 63 - 64
1 John John late 80's, early 90's
2 John John late 80's, early 90's
3 John John late 80's, early 90's
Jude Jude, half brother of Jesus 60's or 70's
Revelation John late 80's, early 90's

Other books mentioned in the Bible


Sometimes cult groups will attempt to justify the existence of newly inspired books (i.e., Book of
Mormon) by citing reference to other books mentioned in the Bible. They erringly conclude that
because the Bible sometimes mentions another book that the book is either a lost book of the Bible or
it somehow justifies their claim to newly inspired writings.
Just because the Bible references another book does not mean that that book is inspired. The Jews
knew which were and were not inspired books and did not consider the following books to be inspired.

The exception, of course, is when the Bible references other inspired books within itself.
Nevertheless, these books are not lost books of the Bible. They were never considered inspired.

1. The Book of Wars - Num. 21:14


2. The Book of Jasher - Josh. 10:13
3. The Chronicles of David - 1 Chron. 27:24
4. The Book of the Kings of Israel and Judah - 2 Chron. 27:7; 35:27; 36:8
5. The Book of the Kings of Israel - 1 Chron. 9:1; 2 Chron. 20:34.
6. The Words of the Kings of Israel - 2 Chron. 33:18.
7. The Decree of David the King of Israel - 2 Chron. 35:4.
8. The Chronicles of Samuel the Seer - 1 Chron. 29:29
9. The Chronicles of Nathan the Prophet - 1 Chron. 29:29
10. The Book of Gad - 1 Chron. 29:29
11. The Book of the Prophet Iddo - 2 Chron. 13:22
12. The Words of Shemaiah the Prophet - 2 Chron. 12:15
13. The Deeds of Uzziah by Isaiah the Prophet - 2 Chron. 26:22; 32:32
14. The Book of Jehu - 2 Chron. 20:34
15. The Record book of Ahasuerus - Esther 2:23; 6:1
16. The Book of Remembrance - Mal. 3:16
17. The Book of Life - Dan. 12:1; Phil. 4:3; Rev. 20:11; 22:19
18. The Book of Judgment - Dan. 7:10; Rev. 20:12
19. The seven-sealed book - Rev. 5:1, 13.
20. An angel's book - Rev. 10:2
Chronology of the Old Testament
The following chronological list is adapted from The Chronological Bible. Its purpose is to help
you develop an overall understanding of the order of the major people and events of the Bible. The
dates are disputed here and there, but overall, they can give you a reference in time and order.

The Pre-existent Christ John 1


Creation Gen. 1:1
Satan cast out of heaven Isaiah 14:12-17
Six days of creation Gen. 1:3-26
Garden of Eden Gen. 2:8-17 From the Creation
Fall of Adam and Eve Gen. 3:1-7 to the Flood
Expulsion from Eden Gen. 3:21-24
Cain kills Abel Gen. 4
Noah is born Gen. 5:28-29
The Flood Gen. 7:10-24
The Tower of Babel
Gen. 11
Abram (Abraham) is born
Gen. 11:27
Job The Flood
Job 1
Abram becomes Abraham to the Patriarchs
Genesis 17
Birth of Isaac, then Jacob, then
Genesis 21-30
Joseph
Joseph sold into slavery to Egypt Genesis 27- 28
Famine and move of Hebrews to Gen. 41
The Patriarchs
Egypt
to the Exodus
The Hebrew population grows Genesis 47:27
1606 - 1462 B. C.
Their bondage and oppression Exodus 8
Moses is born Exodus 6:20
The 10 plagues against Egypt
Exodus 7 - 11
The Hebrews are freed then pursued
Exodus 12 The Exodus to
Crossing the Red Sea
Exodus 13 - 15 Entering Canaan
Receiving the 10 Commandments
Exodus 20 1462 - 1422 B.C.
Israel wanders in the desert for 40
Numbers 14
years
The conquest and division of Canaan Joshua 6 - 12
Canaan to the Reign
Israel becomes a world power 1200 - 750 B. C.
of King Saul
Samson is born Judges 13
1422 - 1065 B. C.
Saul becomes first King 1 Samuel 9

David and Goliath 1 Samuel 17


David becomes King 2 Samuel 5 The Reign of
David with Bathsheba 2 Samuel 11 King David
Absalom's rebellion 2 Samuel 12 1025 - 985 B. C.
David prepares temple materials 1 Chronicles 22

Solomon becomes King 1 Kings 1


The Reign of
Solomon asks God for wisdom 1 Kings 3
King Solomon
The building of the Temple 1 Kings 6
985 - 945 B. C.
Solomon's downfall 1 Kings 11
The nation of Israel divided into two: Israel to the north
and Judah to the south. This period was full of judges, not The Divided Kingdom
kings, who ruled each realm. Many of the judges were (Israel and Judah)
evil. Other gods were worshiped occasionally. During this From Solomon
time Elijah has his ministry. Hosea preached. Jonah dealt to the Fall of Israel
with Nineveh. Rome was founded. The temple was 945 - 586 B. C.
restored.
Both Israel and Judah fall to foreign powers. Micah
The Fall of Israel
prophesies. Isaiah is martyred. Jeremiah is born. Daniel
to the Fall of Judah
is born. Zephaniah prophesies. Ezekiel is born. Jeremiah
721 - 586 B. C.
preaches.

Ezekiel prophesies as Jerusalem falls Ezekiel 33


Nebuchadnezzar's image Daniel 3 Israel is taken
Daniel's vision of the four beasts Daniel 7 into Captivity
Daniel's vision of the 70 weeks Daniel 9 586 - 516 B.C.
Fall of Babylon to Persians Isaiah 13, Jer. 25

Dedication of the Temple Ezra 6


Deliverance of the Jews Esther 8 The Restoration
Ezra prepares to return to Jerusalem Ezra 7 from Captivity
Building of the wall of Jerusalem Nehemiah 2 516 -400 B. C.
Malachi's Prophecies Malachi

Greece is the world power from


Time Between
Rome is the world power from 333 - 63 B. C.
the Testaments
God does not speak for about 400 63 B. C. - 476 A.D.
400 B. C. - 5. A.D.
years
Bible Chronology of the New Testament
The following chronological list is adapted from The Chronological Bible. Its purpose is to
help you develop an overall understanding of the order of the major people and events of the
Bible. The dates are disputed here and there, but overall, they can give you a reference in
time and order.

Jesus is born 5 B. C. Matt. 1; Luke 2


Flight to Egypt 4 B. C. Matt. 2:13-18
Childhood at Nazareth Luke 2:40
In Jerusalem at 12 Luke 2:41-50
John the Baptist Matt. 3; Mark 1; Luke 3 The life of Jesus
Jesus' Baptism Matt. 3
Temptation in the wilderness Matt. 4 Matthew, Mark,
Turns water into wine John 2 Luke, John
First cleansing of the Temple John 2:13-22
Jesus and Nicodemus John 3:1 5 B.C. - 29 A.D.
Discourse with the woman at the well John 4
Galilean ministry Luke 4:14; John 4:43
Miracles by the sea Matt. 8
Visit to Jerusalem John 7

Jesus anointed by Mary John 12:8-11


His triumphal entry Matt. 21
Second cleansing of the Temple Matt. 21:12
Cursing of the fig tree Matt. 21:18
Jesus'
Judas agrees to betray Christ Matt. 26:14-16
Last
Preparation for Passover Mark 14:12-16
Week
Institution of the Lord's supper Luke 22:14-23
Jesus washes the disciples' feet John 13:1-17
Jesus prays at Gethsemane Mark 14:26-42
Jesus is betrayed and arrested John 18:2-11

Jesus is tried and condemned Matt 26:57-68


Peter denies the Lord Matt 26:58; 69-75
Jesus before Pilate Matt. 27:11-14
Jesus before Herod Luke 23:6-12
Barrabas released Mark 15:6-15
Jesus' walk to Golgotha Matt. 27:32-24
Jesus is crucified Matt. 27:35
Darkness at noon Mark 15:33 The Crucifixion
Jesus dies Matt. 27:50 to
Jesus' body entombed John 19:42 The Ascension
Resurrection Luke 24:1-8
Women visit the tomb Matt. 28:1
Stone rolled away Matt. 28:2-4
Message of angels Matt. 28:5-8
Christ appears to disciples Mark 16:14
The great commission given Matt. 28:19-20
Jesus' ascension John 20:26-29
Pentecost Acts 2
Conversion of Saul (Paul) Acts 9
Peter imprisoned and delivered Acts 12:1
Death of Herod Acts 12:20-23
Paul's first missionary journey Acts 13:1
Paul attends council at Jerusalem Acts 15:1-12
The Ministries
Paul's second missionary journey Acts 15
of Paul and Peter
Paul's third missionary journey Acts 18
29 - 67
Paul goes from Macedonia to Corinth Acts 20
Arrest of Paul Acts 21
Paul's fourth missionary Acts 27
Paul's first imprisonment Acts 28:16
Death of Paul and Peter A.D. 67-68
Temple at Jerusalem destroyed A.D. 70
Non biblical accounts of New Testament events and/or people

1. Flavius Josephus (AD 37?-101?, a Jewish historian) mentions John the Baptist and
Herod - Antiquities, Book 18, ch. 5, par. 2
A. "Now some of the Jews thought that the destruction of Herod's army came from God, and
that very justly, as a punishment of what he did against John, that was called the Baptist:
for Herod slew him, who was a good man, and commanded the Jews to exercise virtue, both
as to righteousness towards one another, and piety towards God, and so to come to
baptism; for that the washing [with water] would be acceptable to him, if they made use of
it, not in order to the putting away [or the remission] of some sins [only], but for the
purification of the body; supposing still that the soul was thoroughly purified beforehand by
righteousness."
2. Flavius Josephus (AD 37?-101?) mentions Jesus - Antiquities, Book 18, ch. 3, par.
A. Now there was about this time Jesus, a wise man, if it be lawful to call him a man; for he
was a doer of wonderful works, a teacher of such men as receive the truth with pleasure. He
drew over to him both many of the Jews and many of the Gentiles. He was [the] Christ. And
when Pilate, at the suggestion of the principal men amongst us, had condemned him to the
cross, (9) those that loved him at the first did not forsake him; for he appeared to them
alive again the third day; (10) as the divine prophets had foretold these and ten thousand
other wonderful things concerning him. And the tribe of Christians, so named from him, are
not extinct at this day.
i. There is debate among scholars as to the authenticity of this quote since it is so
favorable to Jesus.
3. Flavius Josephus (AD 37?-101?) mentions James, the brother of Jesus - Antiquities,
Book 20, ch. 19.
A. "Festus was now dead, and Albinus was but upon the road; so he assembled the Sanhedrim
of judges, and brought before them the brother of Jesus, who was called Christ, whose name
was James, and some others, [or, some of his companions]; and when he had formed an
accusation against them as breakers of the law, he delivered them to be stoned: but as for
those who seemed the most equitable of the citizens, and such as were the most uneasy at
the breach of the laws, they disliked what was done."
4. Flavius Josephus (AD 37?-101?) mentions Ananias the High Priest who was mentioned
in Acts 23:2
A. Now as soon as Albinus was come to the city of Jerusalem, he used all his endeavors and
care that the country might be kept in peace, and this by destroying many of the Sicarii. But
as for the high priest, Ananias (25) he increased in glory every day, and this to a great
degree, and had obtained the favor and esteem of the citizens in a signal manner; for he
was a great hoarder up of money
B. Acts 23:2, "And the high priest Ananias commanded those standing beside him to strike him
[Paul] on the mouth."
5. Tacitus (A.D. c.55-A.D. c.117, Roman historian) mentions "christus" who is Jesus -
Annals
A. "Consequently, to get rid of the report, Nero fastened the guilt and inflicted the most
exquisite tortures on a class hated for their abominations, called Christians by the populace.
Christus, from whom the name had its origin, suffered the extreme penalty during the reign
of Tiberius at the hands of one of our procurators, Pontius Pilatus, and a most mischievous
superstition, thus checked for the moment, again broke out not only in Judaea, the first
source of the evil, but even in Rome, where all things hideous and shameful from every part
of the world find their centre and become popular."
B. Ref. from http://classics.mit.edu/Tacitus/annals.mb.txt
6. Thallus Circa AD 52, eclipse of the sun. Thallus wrote a history of the Eastern Mediterranean
world from the Trojan War to his own time. His writings are only found as citations by others.
Julius Africanus who wrote about AD 221 mentioned Thallus' account of an eclipse of the sun.
A. "On the whole world there pressed a most fearful darkness; and the rocks were rent by an
earthquake, and many places in Judea and other districts were thrown down. This darkness
Thallus, in the third book of his History, calls, as appears to me without reason, an eclipse of
the sun."
B. Is this a reference to the eclipse at the crucifixion? Luke 23:44-45, "And it was now about
the sixth hour, and darkness fell over the whole land until the ninth hour, 45 the sun being
obscured; and the veil of the temple was torn in two."
C. The oddity is that Jesus' crucifixion occurred at the Passover which was a full moon. It is not
possible for a solar eclipse to occur at a full moon. Note that Julius Africanus draws the
conclusion that Thallus' mentioning of the eclipse was describing the one at Jesus'
crucifixion. It may not have been.
D. Julius Africanus, Extant Writings, XVIII in the Ante–Nicene Fathers, ed. by Alexander Roberts
and James Donaldson (Grand Rapids: Eerdmans, 1973), vol. VI, p. 130. as cited in
Habermas, Gary R., The Historical Jesus: Ancient Evidence for the Life of Christ, (Joplin, MO:
College Press Publishing Company) 1996.
7. Pliny the Younger mentioned Christ. Pliny was governor of Bithynia in Asia Minor. Pliny
wrote ten books. The tenth around AD 112.
A. "They (the Christians) were in the habit of meeting on a certain fixed day before it was light,
when they sang in alternate verses a hymn to Christ, as to a god, and bound themselves by
a solemn oath, not to any wicked deeds, but never to commit any fraud, theft or adultery,
never to falsify their word, nor deny a trust when they should be called upon to deliver it up;
after which it was their custom to separate, and then reassemble to partake of food—but
food of an ordinary and innocent kind."
B. Pliny, Letters, transl. by William Melmoth, rev. by W.M.L. Hutchinson (Cambridge: Harvard
Univ. Press, 1935), vol. II, X:96 as cited in Habermas, Gary R., The Historical Jesus: Ancient
Evidence for the Life of Christ, (Joplin, MO: College Press Publishing Company) 1996.
8. The Talmud
A. "On the eve of the Passover Yeshu was hanged. For forty days before the execution took
place, a herald went forth and cried, "He is going forth to be stoned because he has
practiced sorcery and enticed Israel to apostasy. Any one who can say anything in his favor,
let him come forward and plead on his behalf." But since nothing was brought forward in his
favor he was hanged on the eve of the Passover!"
B. Gal. 3:13, "Christ hath redeemed us from the curse of the law, being made a curse for us:
for it is written, Cursed is every one that hangeth on a tree."
C. Luke 22:1, "Now the Feast of Unleavened Bread, which is called the Passover, was
approaching. 2And the chief priests and the scribes were seeking how they might put Him to
death; for they were afraid of the people."
D. This quotation was taken from the reading in The Babylonian Talmud, transl. by I. Epstein
(London: Soncino, 1935), vol. III, Sanhedrin 43a, p. 281 as cited in Habermas, Gary R., The
Historical Jesus: Ancient Evidence for the Life of Christ, (Joplin, MO: College Press Publishing
Company) 1996.
9. Lucian (circa 120-after 180) mentions Jesus. Greek writer and rhetorician.
A. "The Christians, you know, worship a man to this day—the distinguished personage who
introduced their novel rites, and was crucified on that account. . . . You see, these misguided
creatures start with the general conviction that they are immortal for all time, which explains
the contempt of death and voluntary self-devotion which are so common among them; and
then it was impressed on them by their original lawgiver that they are all brothers, from the
moment that they are converted, and deny the gods of Greece, and worship the crucified
sage, and live after his laws. All this they take quite on faith, with the result that they
despise all worldly goods alike, regarding them merely as common property."
B. Lucian, The Death of Peregrine, 11–13, in The Works of Lucian of Samosata, transl. by H.W.
Fowler and F.G. Fowler, 4 vols. (Oxford: Clarendon, 1949), vol. 4, as cited in Habermas,
Gary R., The Historical Jesus: Ancient Evidence for the Life of Christ, (Joplin, MO: College
Press Publishing Company) 1996.
C. Though Lucian opposed Christianity, he acknowledges Jesus, that Jesus was crucified, that
Christians worship him, and that this was done by faith.

___________________
Sources

• McDowell, Josh, Evidence that Demands a Verdict, San Bernardino, CA, Here's Life Publishers,
Inc. 1979.
• Habermas, Gary R., The Historical Jesus: Ancient Evidence for the Life of Christ, (Joplin, MO:
College Press Publishing Company) 1996.
• Encarta on the Web at http://encarta.msn.com .
The Documentary Hypothesis of the Pentateuch
also known as the JEDP Theory
Some of the critics of the Bible have come up with some sophisticated arguments in their attempts
to disprove its authenticity and reliability. One of these attempts is known as the Documentary
Hypothesis, or the JEPD theory. In short, this theory states that the first five books of the Bible, called
the Pentateuch consisting of Genesis, Exodus, Leviticus, Numbers, and Deuteronomy, were not written
completely by Moses, who died in 1451 B.C. according to Bishop Ussher's Chronology, but by different
post-mosaic authors. It is alleged that these authors are detectable through the variations of usage of
different words within those books. These authors are known as the Jehovist, the Elohist, the Priestly,
and the Deuteronomist.
If you are not aware, YHWH (not to be confused with JEDP) are the four letters used to represent
the name of God in the Old Testament. From YHWH we get the word Jehovah, the name of God,
mentioned in Exodus 3:14. The word in Hebrew "elohim" is simply the word 'god.' So, YHWH is the
name of God (elohim).
According to Oswald T. Allis there were four main areas considered by these critics when
supporting the Documentary Hypothesis: 3

1. The Variations in the Divine Names in Genesis;


2. The Secondary Variations in Diction and Style;
3. The parallel or Duplicate Accounts (Doublets);
4. The Continuity of the Various Sources.

One of, if not the earliest, appearances of this type of approach to Scripture was by H.B. Witter in
the early 1700s who asserted that there were two parallel accounts in the creation story that were
distinguishable by the word usage in the text.
This method of analysis really took root in 1753 when a French physician named Austruc analyzed
the book of Genesis and asserted that it had two main sources: a Jehovist and an Elohist. However, he
did not deny Mosaic authorship of the Pentateuch at this time.
What this analysis entails is the assumption that where the word Jehovah appears in large
quantities in a section of writing, it is the result of an author who used the word "Jehovah" or the
tetragramaton YHWH predominantly. Additionally, it is stated that where the term Elohim appears
more frequently, it is the result of an Elohist, or someone who used that word more frequently than
another person.
Another person to use this method was Eichhorn whose analysis of 1787 was similar to Austruc's.
However, neither of these men denied Mosaic authorship and neither carried the analysis past the
book of Exodus.
A few years later, a gentleman named, De Wette (1805), assigned Deuteronomy to the time of
Josiah (post Moses period). This prompted other writers to tackle the issue. In 1823 Eichhorn had
given up on his claim of Mosaic Authorship of the Pentateuch.
The letters associated with this issue are J and E.

Hupfeld

In 1853, Hupfeld proposed that there are two Elohistic source documents in Genesis: chapters 1-19
by one author and chapters 20 - 50 by another. He also put great importance upon the redactor, or
the one who assembled the various documents, who used editor rights during the compilation of the
book of Genesis. Therefore, his arrangement of the documents was thus: First Elohist, Second Elohist,
Jehovist, Deuteronomist: J, E, and D.

3
Oswald T. Allis, The Five Books of Moses, Presbyterian and Reformed Publishing Co., Phillipsburg, New Jersey,
1949, p. 22.
Graf-Wellhausen

Later, Karl H. Graf in the 1860's and Julius Wellhausen in the 1870's said that "according to the
historical and prophetical books of the Old Testament the priestly legislation of the middle books of the
Pentateuch was unknown in pre-exilic time, and that this legislation must therefore be a late
development."4 The letter P became associated with this view.
Basically they arranged the Pentateuch authorship in the following manner:

1. "The earliest part of the Pentateuch came from two originally independent documents, the
Jehovist (850 B.C.) and Elohist (750 B.C.).
2. From these the Jehovist compiled a narrative work (650 B.C.).
3. Deuteronomy came in Josiah's time and its author incorporated this into the Jehovist's work.

4. The priestly legislation in the Elohist document was largely the work of Ezra and is referred to
as the Priestly Document. A later editor(s) revised and edited the conglomeration of documents
by about 200 B.C. to form the extant Pentateuch we have today." 5

There have been slight modifications of this list, but it is basically the same form used by those
holding to the Documentary Hypothesis.

4
Oswald T. Allis, The Five Books of Moses, p. 17.
5
Josh McDowell, More Evidence That Demands a Verdict, Here's Life Publishers, Inc. 1981, p. 45.
Answering the Documentary Hypothesis (JEDP)
It is not my intention to attack the character of those who advocate the Documentary Hypothesis.
But the Bible says in Rom. 1:18-21 that men suppress the truth of God's word in their
unrighteousness. This is what is happening here. They are suppressing the truth. They are devising
elaborate methods to deny the inspiration and authenticity of the Bible, particularly the Pentateuch.
Nevertheless, there are several issues worth examining when answering their claims.

1. Presuppositions:
By far, the majority of those holding to the JEDP theory presuppose that the miraculous cannot
happen. Therefore, they must conclude beforehand that the Pentateuch is not inspired and Moses
could not have written it. They must find another explanation for the mosaic authorship of the first
five books of the Bible.
Such a presupposition does not allow a proper examination of the documents and will result in
inaccurate conclusions.

2. The Critics are claiming a great deal.


The Pentateuch was written centuries ago in a different language, in a different culture, and a
different land. The critics are claiming that "they are able to decide exactly what a writer could or could
not say, and on this basis to determine what part of the document belongs or does not belong to
him."6 In other words, the critics are basing their argument on their own ability to read a document
that is 3000 years old, divide it up into word usage groups, and assert hidden divisions, and separate
authors. And not only this, but they are claiming they can do it on a consistent basis. This is hardly an
exact science and is open to wide range of error depending upon the presuppositions and purposes of
the critic.

3. Writing Styles change within Writers


What writer writes with a consistent style? Yes, there are styles to writers, but the subject matter
affects the content. A technical work is different from a narrative or historical piece. The Pentateuch
has components of all of these. Therefore, different styles are expected.
Additionally, what the writer has in mind can easily cause him to use a different concentration of
words. Should the intention change, so would the word usage. Did Moses sit down at one sitting and
write everything out? Of course not. Upon reflection, reading, prayer, etc., his focus and purpose
within sections of Scripture can change as he moves to a new subject.

4. One writer can produce different analysis results


WordPerfect has a Grammar Analyzer for readability. I ran both the paper explaining the
Documentary Hypothesis and this paper refuting it through the analyzer. The results are interesting.

Analysis Explaining Answering


JEDP JEDP

Flesch-Kincaid grade level 13.64 10.35

Passive voice (% of finite verb phrases) 9% 9%

Sentence Complexity (100 = very complex) 60 43

Vocabulary Complexity (100 = very 38 22


complex)

We could conclude that though there are similarities, because there are definite differences, there
must be two authors. After all, the first paper has more complex sentences and more verb complexity

6
Oswald T. Allis, The Five Books of Moses, p. 70.
than the second as well as being 13th grade level. The funny thing is, I wrote this in two sittings:
One before church and the other after church on the same day.

5. A look at the actual analysis


In the back of Oswald T. Allis' book, pages 291-293, is a breakdown of the JEDP analysis of the
Pentateuch. I chose a small section dealing with Genesis 1 - 7 and supplied the verses (NASB version)
so you can see for yourself if these divisions are warranted.

J P Text of Genesis
1:1-
2:4a
2:4b-
4:26
5:1-28 v. 28, "And Lamech lived one hundred and eighty-two years, and became
the father of a son."
5:29 v. 29, "Now he called his name Noah, saying, 'This one shall give us rest
from our work and from the toil of our hands arising from the ground
which the Lord has cursed.'"
5:30-32 v. 30, "Then Lamech lived five hundred and ninety-five years after he
became the father of Noah, and he had other sons and daughters."
6:1-8
6:9-22
7:1-5 v. 5, "And Noah did according to all that the Lord had commanded him."
7:6 v. 6, "Now Noah was six hundred years old when the flood of water came
upon the earth."
7:7-10 v. 7, "Then Noah and his sons and his wife and his sons' wives with him
entered the ark because of the water of the flood."
7:11 v. 11, "In the six hundredth year of Noah's life, in the second month, on
the seventeenth day of the month, on the same day all the fountains of
the great deep burst open, and the floodgates of the sky were opened."
7:12 v. 12, "And the rain fell upon the earth for forty days and forty nights."
7:13- v. 16a, "And those that entered, male and female of all flesh, entered as
16a God had commanded him. . ."
7:16b v. 16b, ". . . and the Lord closed it behind him."
7:17a v. 17a, "Then the flood came upon the earth for forty days"
7:17b v. 17b, "and the water increased and lifted up the ark, so that it rose
above the earth."
7:18-21 v. 18, "And the water prevailed and increased greatly upon the earth; and
the ark floated on the surface of the water. 19 And the water prevailed
more and more upon the earth, so that all the high mountains everywhere
under the heavens were covered. 20 The water prevailed fifteen cubits
higher, and the mountains were covered. 21And all flesh that moved on
the earth perished, birds and cattle and beasts and every swarming thing
that swarms upon the earth, and all mankind"
7:22-23 v. 22-23, "of all that was on the dry land, all in whose nostrils was the
breath of the spirit of life, died. Thus He blotted out every living thing that
was upon the face of the land, from man to animals to creeping things
and to birds of the sky, and they were blotted out from the earth; and
only Noah was left, together with those that were with him in the ark."
7:24 v. 24, "And the water prevailed upon the earth one hundred and fifty
days."

As you can see, the first seven chapters of Genesis are chopped up into bite-size pieces. In some
places, sentences are cut in half and attributed to different sources. I cannot see any reason to divide
the sections of scripture up the way they have.

6. Jesus attributed the 5 books to Moses


Whether or not a biblical critic wants to take Jesus' word for anything is up to the individual. But no
less than Jesus authenticated the Mosaic authorship of the Pentateuch. Jesus divided the Old
Testament into three sections in Luke 24:44: Moses and the Prophets and the Psalms. Also, he
attributed all the individual JEDP defined sections of the Pentateuch to Moses.
In Mark 10:4-8, Jesus quoted Gen. 2:24, which would be J, as coming from Moses. Mark 7:10,
Jesus quoted the Ten Commandments, which fall into the E category, as coming from Moses. In Mark
10:3, Jesus refers to Deut. 24:1f, which would be D, as being from Moses. In Matt. 8:4, Jesus quoted
Lev. 14, which would be equivalent to P, as coming from Moses.

This is a brief look at the Documentary Hypothesis. In my opinion, it is a fabrication based upon
false presuppositions and inaccurate analysis. It contradicts what Jesus said and it is an unreliable way
to analyze a document that is thousands of years old.
Biblical Interpretation
The Bible is God’s Word. But some of the interpretations derived from it are not. There are many
cults and Christian groups that use the Bible claiming their interpretations are correct. Too often,
however, the interpretations not only differ dramatically but are clearly contradictory. This does not
mean that the Bible is a confusing document. Rather, the problem lies in those who interpret and the
methods they use.
Because we are sinners, we are incapable of interpreting God’s word perfectly all of the time. The
body, mind, will, and emotions are affected by sin and make 100% interpretive accuracy impossible.
This does not mean that accurate understanding of God’s Word is impossible. But it does mean that we
need to approach His word with care, humility, and reason. Additionally, we need, as best as can be
had, the guidance of the Holy Spirit in interpreting God’s Word. After all, the Bible is inspired by God
and is addressed to His people. The Holy Spirit helps us to understand what God’s word means and
how to apply it.
On the human level, to lessen the errors that come in our interpretations, we need to look at some
basic biblical interpretive methods. I’ll list some of the principles in the form of questions and then
apply them one at a time to a passage of scripture.
I offer the following principles as guidelines for examining a passage. They are not exhaustive nor
are they set in concrete.

1. Who wrote/spoke the passage and to whom was it addressed?


2. What does the passage say?
3. Are there any words or phrases in the passage that need to be examined?
4. What is the immediate context?
5. What is the broader context in the chapter and book?
6. What are the related verses to the passage’s subject and how do they affect the understanding
of this passage?
7. What is the historical and cultural background?
8. What do I conclude about the passage?
9. Do my conclusions agree or disagree with related areas of scripture and others who have
studied the passage?
10. What have I learned and what must I apply to my life?

In order to teach you how these questions can affect your interpretation of a passage, I have
chosen one which, when examined closely, may lead you into a very different interpretation than what
is commonly held. I leave it to you to determine if my interpretation is accurate.
The passage that I am going to use is Matt. 24:40, "Two men will be in the field; one will be taken
and the other left"(NIV).

1. Who wrote/spoke the passage and who was it addressed to?


Jesus spoke the words and they were recorded by Matthew. Jesus spoke them to His disciples in
response to a question, which we will get to later.

2. What does the passage say?


The passage simply says that one out of two men in a field will be taken. It doesn’t say where,
why, when, or how. It just says one will be taken. It doesn’t define the field as belonging to someone
or in a particular place.

3. Are there any words in the passage that need to be examined?


No particular word in this verse really stands out as needing to be examined, but to follow this
exercise, I will use the word "taken." By using a Strong Concordance and a dictionary of New
Testament words (Vine’s, for example), I can check the Greek word and learn about it. The word in
Greek is paralambano. It means "1) to take to, to take with one's self, to join to one's self, 2) to
receive something transmitted."
A point worth mentioning about word studies is that a word means what it means in context.
However, by examining how a word is used in multiple contexts, the meaning of the word can take on
a new dimension. For example, the word for "love" in Greek is "agapao." It is generally believed to
mean "divine love." This seems obvious since it is used in John 3:16 in that way. However, the same
word is used in Luke 11:43. Jesus says, "Woe to you Pharisees, because you love the most important
seats in the synagogues and greetings in the marketplaces," (NIV). The word used there is "agapao."
It would seem then that the meaning of the word might mean something more along the lines of "total
commitment to."
However, we must be careful not to insert a meaning of a word from one context into that of
another. For example: 1) That new cadet is green. 2) That tree is green. The first green means "new
and inexperienced." The second one means the color green. Would we want to impose the contextual
meaning of one into the other? It wouldn’t be a good idea.

4. What is the immediate context?


This is where this particular verse will come alive. The immediate context is as follows, Matt.
24:37-42, "As it was in the days of Noah, so it will be at the coming of the Son of Man. 38For in the
days before the flood, people were eating and drinking, marrying and giving in marriage, up to the day
Noah entered the ark; 39and they knew nothing about what would happen until the flood came and
took them all away. That is how it will be at the coming of the Son of Man. 40Two men will be in the
field; one will be taken and the other left. 41Two women will be grinding with a hand mill; one will be
taken and the other left. 42Therefore keep watch, because you do not know on what day your Lord will
come," (NIV).
Immediately we can see that the person taken in verse 40 is paralleled by people being taken in
verse 39. That is, the "being taken" are of the same kind.
A further question needs to be asked. Who was taken in verse 39? Was it Noah and his family or
was it the people who were eating and drinking? The answer to that question might help us
understand the original passage better. Therefore, the next interpretive step will help us greatly.

5. What is the broader context in the chapter and book?


A passage should always be looked at in context, not only in its immediate context of the verses
directly before and after it, but also in the context of the chapter it is in and the book in which it is
written.
Jesus’ discourse from which our verse was taken began with a question. Jesus had just left the
temple and in verse 2 told His disciples that "...not one stone here will be left on another; every one
will be thrown down." Then in verse 3 the disciples asked Jesus, "Tell us," they said, "when will this
happen, and what will be the sign of your coming and of the end of the age?" (NIV). Jesus then goes
on to prophesy about things to come at the end of the age. He speaks of false Christs, of tribulation, of
the sun being darkened, of His return, and of two men in a field where one will be taken and the other
left.
The context then is eschatological. That means that it deals with the last things, or the time
shortly before Jesus’ return. Many people think that this verse in Matt. 24:40 refers to the rapture
spoken of in 1 Thess. 4:16-17. It may. But it is interesting to note that the context of the verse seems
to suggest that the wicked are taken, not the good.
Now, about this time you might be thinking that this method of interpreting passages isn’t that
good. After all, the "one taken, one left" verse is obviously about the rapture. Right? Well, maybe. You
see, we all come to the Bible with preconceived ideas. Sometimes they are right, sometimes wrong.
We should always be ready to have our understanding of the Bible challenged by what it says. If we
are not willing, then we are prideful. And God is distant from the proud (Psalm 138:6).

6. What are the related verses to the passage’s subject and how do they affect the
understanding of this passage?
It just so happens that there are related verses, in fact, a parallel passage found in Luke 17:26-
27. "Just as it was in the days of Noah, so also will it be in the days of the Son of Man. 27People were
eating, drinking, marrying and being given in marriage up to the day Noah entered the ark. Then the
flood came and destroyed them all," (NIV).
Immediately we discover that related verses do indeed affect how we understand our initial verse.
It is clear from this passage in Luke that the ones taken by the flood are those who were eating and
drinking and being given in marriage. In other words, it wasn’t the godly people who were taken, it
was the wicked.
As you can see, this has a profound impact on how we understand our passage in Matt. 24:40.
Does the context suggest that the one in the field who is taken is the one who is wicked? Also, how
does this context affect my preconceived ideas about this verse? Let’s read the verse again in context.
Matt. 24:37-42, As it was in the days of Noah, so it will be at the coming of the Son of Man. 38For in
the days before the flood, people were eating and drinking, marrying and giving in marriage, up to the
day Noah entered the ark; 39and they knew nothing about what would happen until the flood came and
took them all away. That is how it will be at the coming of the Son of Man. 40Two men will be in the
field; one will be taken and the other left. 41Two women will be grinding with a hand mill; one will be
taken and the other left. 42"Therefore keep watch, because you do not know on what day your Lord will
come,” (NIV).
What do you think now? Is the one taken the good or the bad? Also, does this verse refer to the
rapture or not?
Just asking.
Of related interest is a passage in Matt. 13:24-30 where Jesus gives the parable of the sower who
sows good seed in his field and someone sows tares. The servants asked if they should go immediately
and gather up the wheat. But, in verse 30, Jesus says, "Let both grow together until the harvest. At
that time I will tell the harvesters: First collect the weeds and tie them in bundles to be burned; then
gather the wheat and bring it into my barn."
The point worth noting here is that the first ones gathered are the weeds, not the wheat. This is
most interesting since Jesus explains the parable in Matt. 13:36-43 and states that they will be cast
into the furnace.
Additionally, when we turn to Luke 17:1, which is the parallel passage of Matt. 24, we discover
that the disciples ask Jesus a question in response to Jesus’ statement that " two will be in the field and
one will be taken." In verse 37 they ask, "Where, Lord?" they asked. He [Jesus] replied, "Where there
is a dead body, there the vultures will gather."
They are taken to a place of death.

7. What is the historical and cultural background?


This is a more difficult question to answer. It requires a bit more research. A commentary is worth
examining here since they usually provide the historic and cultural backgrounds that help to unravel
the text.
In this context, Israel was under Roman rule. They had been denied the right of capital
punishment, of self-rule, and the ability to wage war. Rome had dominated the small nation. Judaism
was tolerated among the Roman leadership. After all, Israel was a small far-away country with a
people that were fanatical about their religion. So, Rome allowed Israel to be ruled by Jewish political
puppets.
The Temple was the place of worship for the Israelite community. It was there that the blood
sacrifices were made by the high priest for the atonement of the nation. It had taken 46 years to build
(John 2:20). Jesus said the temple would be destroyed which prompted the question which lead to His
discourse which contains the passage we are examining.
Culturally, the Jewish people were dedicated to the Old Testament. Within those pages were
prophecies of the Messiah, of the end of the age, and of the delivery from bondage. The Jewish people
knew that and were in a state of expectation. Along comes Jesus with miracles and words of great
power. Naturally, they would look to him as a possible deliverer.

8. What do I conclude about the passage?


Since the context of the passage suggests that it is the wicked that are taken, I am going to
conclude that the one taken in the field is not the good, but the bad. I also am tempted to conclude
that the wicked are taken to a place of judgment.
9. Do my conclusions agree or disagree with related areas of scripture and others who have
studied the passage?
I’ve already presented other verses which seem to agree with my conclusion. However, it is not in
agreement with all of the commentaries I’ve read on this verse. At this point I would need to present
my conclusion to others to see what they think. Just because I studied the Word and arrived at a
conclusion does not mean that it is correct. But it doesn’t mean it is wrong either.
By consulting with others, by examining the word again, and by seeking God and his illumination,
I can only hope to arrive at the best possible conclusion about a passage.

10. What have I learned and what must I apply to my life?


Interpretation of scripture is for a purpose: To understand God’s word more accurately. With a
better understanding of His word, we can then more accurately apply it to the area that it addresses.
In this case, the passage deals with an area of the future, and area of judgment. It is information that
Jesus has revealed and that He wants us to know about. The application then would be that God will
execute judgment upon the unrighteous at the end of the age.

Concluding remarks:
This article is only an illustration. It is basic and does not cover all the points of biblical
interpretation. But it does give a direction and an example for you to apply. As I said before, pray.
Read His word. Look into the scriptures as best you can with as much understanding and skill as is
possible. Be humble in your approach and test everything by the Bible.

One last thing: did you agree with my conclusion?


Scientific Accuracies of the Bible
Many people doubt the Bible for various reasons. One of them is that the Bible is not accurate
scientifically. But this just isn't so. The Bible is not a book about science, but when it does speak
scientifically, it is accurate. In fact, it was far ahead of any other writing of its time. Please consider
the following.

1. The Spherical Shape of the Earth


A. "He sits enthroned above the circle of the earth, and its people are like grasshoppers.
He stretches out the heavens like a canopy, and spreads them out like a tent to live in,”
(Isaiah 40:22, NIV).
2. The Earth is suspended in nothing
A. "He spreads out the northern [skies] over empty space; he suspends the earth over
nothing" (Job. 26:7, NIV).
3. The Stars are Innumerable
A. "He took him outside and said, "Look up at the heavens and count the stars -- if indeed
you can count them." Then he said to him, "So shall your offspring be,” ( Gen. 15:5,
NIV).
4. The Existence of Valleys in the Seas
A. "The valleys of the sea were exposed and the foundations of the earth laid bare at the
rebuke of the LORD, at the blast of breath from his nostrils,” (2 Sam. 22:16, NIV).
5. The Existence of Springs and Fountains in the Seas
A. "In the six hundredth year of Noah's life, on the seventeenth day of the second month
-- on that day all the springs of the great deep burst forth, and the floodgates of the
heavens were opened" (Genesis 7:11, NIV). See also Gen. 8:2; Prov. 8:28.
6. The Existence of Water Paths (Ocean Currents) in the Seas
A. "O LORD, our Lord, how majestic is your name in all the earth!...When I consider your
heavens, the work of your fingers, the moon and the stars, which you have set in
place,...You made him [man] ruler over the works of your hands; you put everything
under his feet...the birds of the air, and the fish of the sea, all that swim the paths of
the seas,” (Psalm 8:1,3,6,8, NIV).
7. The Hydrologic Cycle
A. "He wraps up the waters in his clouds, yet the clouds do not burst under their weight,”
(Job. 26:8, NIV).
B. "He draws up the drops of water, which distill as rain to the streams; the clouds pour
down their moisture and abundant showers fall on mankind" (Job. 36:27-28, NIV)
C. "The wind blows to the south and turns to the north; round and round it goes, ever
returning on its course. All streams flow into the sea, yet the sea is never full. To the
place the streams come from, there they return again (Ecclesiastes 1:6-7, NIV).
8. The Concept of Entropy
A. "In the beginning you laid the foundations of the earth, and the heavens are the work
of your hands. They will perish, but you remain; they will all wear out like a garment.
Like clothing you will change them and they will be discarded" (Psalm 102:22-26, NIV).
9. The Nature of Health, Sanitation, and Sickness
A. The listing for this section is too large for this page. But the scriptural references are
Leviticus 12-14.
Prophecy, the Bible and Jesus
How do you respond to someone's claim that the Bible is not inspired? Is there a way to
prove inspiration or, at least, intelligently present evidence for its inspiration? The answer is
"Yes!" One of the best ways to prove inspiration is by examining prophecy. There are many
religious books in the world that have many good things to say. But only the Bible has fulfilled
prophecies--with more fulfillments to come. The Bible has never been wrong in the past, and it
won't be wrong in the future. It claims inspiration from God (2 Tim. 3:16). Since God is the
creator of all things (Isaiah 44:24), then He is also the creator of time. It is under His control.
Only God, then, would always be right about what is in the future, our future.
Fulfilled prophecy is strong evidence that God is the author of the Bible because when you
look at the mathematical odds of prophecy being fulfilled, you quickly see a design, a purpose,
and a guiding hand behind the Bible. If just one prophecy failed, then we would know that God is
not the true God, because the creator of all things, which includes time, would not be wrong
about predicting the future. Deut. 18:22 says, "If what a prophet proclaims in the name of the
LORD does not take place or come true, that is a message the LORD has not spoken. That
prophet has spoken presumptuously" (NIV). Isaiah 46:9-10 says, "Remember the former things,
those of long ago; I am God, and there is no other; I am God, and there is none like me. I make
known the end from the beginning, from ancient times, what is still to come. I say: My purpose
will stand, and I will do all that I please."
One approach to use with an unbeliever is to turn to Psalm 22 and read verses 12-18. This is
a detailed description of the crucifixion--1000 years before Jesus was born. After you read the
section ask him what it was about. He'll say, "The crucifixion of Jesus." Then respond with
something like, "You're right. This is about the crucifixion. But it was written 1000 years before
Jesus was born. And on top of that, crucifixion hadn't even been invented yet. How do you think
something like this could happen?" After a brief discussion, you could show him (or her) a few
other prophecies like where Jesus' birthplace was prophesied (Micah 5:2), that He would be born
of a virgin (Isaiah 7:14), that His side would be pierced (Zech. 10:10), etc.

Born of the Seed of the Woman

Gen. 3:15, "And I will put enmity Matt. 1:20, "But after he had considered this, an
between you and the woman, and angel of the Lord appeared to him in a dream and
between your offspring and hers; he will said, "Joseph son of David, do not be afraid to take
crush your head, and you will strike his Mary home as your wife, because what is conceived
heel." in her is from the Holy Spirit."

Born of a Virgin

Isaiah 7:14, "Therefore the Lord himself Matt. 1:18,25, "This is how the birth of Jesus Christ
will give you a sign: The virgin will be came about: His mother Mary...was found to be
with child and will give birth to a son, with child through the Holy Spirit... But he had no
and will call him Immanuel." union with her until she gave birth to a son. And he
gave him the name Jesus."

Son of God

Psalm 2:7, "I will proclaim the decree of


the LORD: He said to me, "You are my Matt. 3:15, "And a voice from heaven said, 'This is
Son today I have become your Father." my Son, whom I love; with him I am well pleased.'"
Seed of Abraham

Gen. 22:18, "and through your offspring Matt. 1:1, "record of the genealogy of Jesus Christ
all nations on earth will be blessed, the son of David, the son of Abraham:"
because you have obeyed me."

Son of Isaac

Gen. 21:12, "But God said to him, 'Do Luke 3:23-34, "Now Jesus himself was about thirty
not be so distressed about the boy and years old when he began his ministry. He was the
your maidservant. Listen to whatever son, so it was thought, of Joseph, the son of
Sarah tells you, because it is through Heli...the son of Jacob, the son of Isaac, the son of
Isaac that your offspring will be Abraham, the son of Terah, the son of Nahor,"
reckoned.'"

House of David

Jer. 23:1, "The days are coming, Luke 3:23-31, "Now Jesus himself was about thirty
declares the LORD, "when I will raise up years old when he began his ministry. He was the
to David a righteous Branch, a King who son, so it was thought, of Joseph, the son of
will reign wisely and do what is just and Heli...the son of Mattatha, the son of Nathan, the
right in the land." son of David,"

Born at Bethlehem

Micah 5:2 "But you, Bethlehem Matt. 2:1, "After Jesus was born in Bethlehem in
Ephrathah, though you are small among Judea, during the time of King Herod, Magi from
the clans of Judah, out of you will come the east came to Jerusalem."
for me one who will be ruler over Israel,
whose origins are from of old, from
ancient times."

He shall be a Prophet

Deut. 18:18, "I will raise up for them a Matt. 21:11,"The crowds answered, "This is Jesus,
prophet like you from among their the prophet from Nazareth in Galilee."
brothers; I will put my words in his
mouth, and he will tell them everything I
command him."

He shall be a Priest

Psalm 110:4, "The LORD has sworn and Heb. 5:5-6, "So Christ also did not take upon
will not change his mind: 'You are a himself the glory of becoming a high priest. But
priest forever, in the order of God said to him, 'You are my Son; today I have
Melchizedek.'" become your Father.' And he says in another place,
'You are a priest forever, in the order of
Melchizedek.'"
He shall be a King

Psalm 2:6, "I have installed my King on Matt. 27:37, "Above his head they placed the
Zion, my holy hill." written charge against him: THIS IS JESUS, THE
KING OF THE JEWS."

He shall judge

Isaiah 33:22, "For the LORD is our John 5:30, "By myself I can do nothing; I judge
judge, the LORD is our lawgiver, the only as I hear, and my judgment is just, for I seek
LORD is our king; it is he who will save not to please myself but him who sent me."
us."

He would be preceded by a Messenger

Isaiah 40:3, "A voice of one calling: 'In Matt. 3:1-2, "In those days John the Baptist came,
the desert prepare the way for the preaching in the Desert of Judea and saying,
LORD; make straight in the wilderness a 'Repent, for the kingdom of heaven is near.'"
highway for our God.'"

Rejected by His own people

Isaiah 53:3, "He was despised and John 7:5, "For even his own brothers did not
rejected by men, a man of sorrows, and believe in him."
familiar with suffering. Like one from John 7:48, "Have any of the rulers or the Pharisees
whom men hide their faces he was believed in Him?"
despised, and we esteemed him not."

His side pierced

Zech. 12:10, "And I will pour out on the John 19:34, "Instead, one of the soldiers pierced
house of David and the inhabitants of Jesus' side with a spear, bringing a sudden flow of
Jerusalem a spirit of grace and blood and water."
supplication. They will look on me, the
one they have pierced, and they will
mourn for him as one mourns for an only
child, and grieve bitterly for him as one
mourns for an only son."
Crucifixion

Psalm 22:1, Psalm 22:11-18, "For the Luke 23:33, "When they came to the place called
director of music. To the tune of "The the Skull, there they crucified him, along with the
Doe of the Morning." A psalm of David. criminals -- one on his right, the other on his left."
My God, my God, why have you forsaken John 19:33, "But when they came to Jesus and saw
me?...Do not be far from me, for trouble that He was already dead, they did not break His
is near and there is no one to help. Many legs."
bulls surround me; strong bulls of John 19:23-24, "When the soldiers crucified Jesus,
Bashan. Dogs have surrounded me; a they took his clothes, dividing them into four
band of evil men has encircled me, they shares, one for each of them, with the
have pierced my hands and my feet. I undergarment remaining. This garment was
can count all my bones; people stare and seamless, woven in one piece from top to bottom.
gloat over me. They divide my garments Let's not tear it, they said to one another. "Let's
among them and cast lots for my decide by lot who will get it." This happened that
clothing." the scripture might be fulfilled which said, "They
divided my garments among them and cast lots for
my clothing." So this is what the soldiers did."

The Mathematical Odds of Jesus Fulfilling Prophecy

The following probabilities are taken from Peter Stoner in Science Speaks (Moody Press, 1963)
to show that coincidence is ruled out by the science of probability. Stoner says that by using the
modern science of probability in reference to eight prophecies, 'we find that the chance that any
man might have lived down to the present time and fulfilled all eight prophecies is 1 in 10 17." That
would be 1 in 100,000,000,000,000,000. In order to help us comprehend this staggering
probability, Stoner illustrates it by supposing that "we take 10 17 silver dollars and lay them on the
face of Texas. They will cover all of the state two feet deep.
"Now mark one of these silver dollars and stir the whole mass thoroughly, all over the state.
Blindfold a man and tell him that he can travel as far as he wishes, but he must pick up one silver
dollar and say that this is the right one. What chance would he have of getting the right one? Just
the same chance that the prophets would have had of writing these eight prophecies and having
them all come true in any one man."
Stoner considers 48 prophecies and says, "we find the chance that any one man fulfilled all 48
prophecies to be 1 in 10157, or 1 in
100,000,000,000,000,000,000,000,000,000,000,000,000,000,000,000,000,000,000,000,000,
000,000,000,000,000,000,000,000,000,000,000,000,000, 000,000,000,000,000,000,000,000,
000,000,000,000,000,000,000,000,000,000.

The estimated number of electrons in the universe is around 10 79. It should be quite evident
that Jesus did not fulfill the prophecies by accident."

__________________
This information was taken from the book Evidence that Demands a Verdict by Josh McDowell.
What is Redaction Criticism?
Redaction Criticism of the Bible is the theory that different copyists and commentators of the early
biblical writings embellished and altered the biblical texts throughout early Jewish and Christian history
to make them appear more miraculous, inspirational, and legitimate. An example of redaction theory
would be the claim that Old Testament prophecies were modified by redactors after the fact to make
them appear as miraculous prophecies. Redaction criticism reduces the quality of the biblical record,
casts strong doubt on its inspiration, and implies that the Bible is not trustworthy as a historical
document.
Originally, redaction criticism was restricted to the synoptic gospels (Matthew, Mark, and Luke), but
it has been applied to other areas of scripture. Norman Perrin in his book "What is Redaction
Criticism?" states, "The prime requisite for redaction criticism is the ability to trace the form and
content of material used by the author concerned or in some way to determine the nature and extent
of his activity in collecting and creating, as well as in arranging, editing, and composing." 7
Redaction Criticism began in Germany in the early 1700's with Hermann Reimarus who was a
professor of Oriental languages in Hamburg. He was a deist who wrote extensively opposing
Christianity. He proposed that Jesus was a failure and that the disciples altered their stories in an
attempt to make Jesus appear messianic and miraculous. Redaction criticism was then taken up by
David Friedrich Strauss (1808-74) who attempted to show that the gospels were altered, were the
expression of myth, and cannot be construed as historical. His main contribution to redaction criticism
was the idea that Mark was used as a source document by Matthew and Luke. 8 Wilhelm Wrede (1859-
1906) was the next major proponent of redaction criticism who attempted to show that the historical
narratives of Mark were not reliable.

Some Evidence and Answers for Redaction

Some evidence offered to support biblical redaction is the ending of Deuteronomy (Chapter 34)
that records Moses' death; the arrangement of the Psalms into five sections is the work of a compiler;
and that the Book of Chronicles state they are based on prior writings (1 Chron. 9:1; 27:24; 29:29; 2
Chron. 9:29; 13:22; 6:11; 20:34; 25:26; 27:7; 28:26; 32:32; 33:19; 35:27; 36:8).9 There are other
alleged evidences but these will suffice.
Though there are accounts of biblical writers arranging or commenting on events, this does not
discount the authenticity or reliability of the biblical documents. It is commonly accepted in
conservative scholarly circles that Joshua probably wrote the ending of Deuteronomy. This does not
invalidate the Mosaic authorship of the Pentateuch. Likewise, simply arranging material, such as the
Psalms, into categories does not affect its inspiration, authenticity, or reliability at all. And, citing
other sources for factual reference likewise, in no way reduces the inspiration of the book of
Chronicles, or the Bible as a whole. The inspired writer simply used other books, which were not
inspired though accurate, in his compilation of the biblical record.
Another twist in redaction criticism is the proposition that there were inspired redactors. But, this
contradicts the doctrine that the original writings were inspired. After all, if the original writings were
inspired, there would be no need for altering the text. It further implies that what is said in scripture
is not trustworthy. The gospels, for example, would not then really contain Jesus' words but only the
words of redactors who wanted to embellish and/or modify "myth stories" into what was apparently
more spiritual and inspirational. With this, deception is implied since the biblical documents claim
authenticity and accuracy. Though it is not within the scope of this paper, redaction criticism is refuted
by the evidence of the reliability of the historic documents (dealt with in Textual Criticism), the fact
that the prophecies were indeed made and fulfilled, and that the Bible is archeologically accurate. Due
to the science of Textual Criticism, the original texts of the Bible can be reconstructed with a great deal
of accuracy, their prophetic nature verified, and their inspiration maintained.

7
Perrin, Norman, What is Redaction Criticism?, Philadelphia, Fortress Press, 1969. p. 2.
8
Perrin, Norman, What is Redaction Criticism?, pp. 4-5.
9
Geisler, Norman, Baker Encyclopedia of Christian Apologetics, Grand Rapids, Michigan, Baker Books, 1999. p.
636.
Baptism
Introduction

Baptism is a Christian ordinance that is taught in the word of God. John the Baptist taught it. Jesus
was baptized. Christians to this day are baptized.

But, is baptism a requirement for salvation? What is baptism? Why is it important? What does it do?
These questions and others are important for the Christian to know.

1. Is baptism necessary in order to be saved? Why or why not? p. 98-101


2. The gospel is what saves us. What is the gospel? p. 99
3. What are some textual issues with Mark 16:16? p. 102
4. What verses demonstrate justification by faith? p. 102
5. Baptism by immersion is a perfect symbol for what? p. 105-106
6. What saved Noah in the context of 1 Pet. 3:21? p. 108
7. What does the water symbolize in 1 Pet. 3:21? p. 109
8. How does Acts 10:44-46 affect the discussion of baptism and salvation? p. 109
9. What are some of the different interpretations of John 3:5? p. 110
10. Is Acts 2:38 teaching that we must be baptized to be saved? Why or why not? p. 112-114
11. What is "The Promise" of Acts 2:39? p. 114
Is Baptism Necessary for Salvation?
One of the most nagging questions in Christianity is whether or not baptism is necessary for
salvation. The answer is a simple, "No." But you might ask, "If the answer is no, then why are there
verses that say things like ‘. . .baptism that now saves you . . . ‘ (1 Pet. 3:21, NIV) and ‘ . . . Repent
and be baptized, every one of you, in the name of Jesus Christ for the forgiveness of your sins . . . "
(Acts 2:38, NIV)? This is an honest question and it needs a competent answer. But, before I tackle this
I need to lay a foundation of proper theology, then I'll address some of those verses that are
commonly used to support the idea that baptism is necessary for salvation.

God Works Covenantally

First, you need to understand that God works covenantally. A covenant is a pact or agreement
between two or more parties. The New Testament and Old Testaments are New and Old Covenants.
The word "testament" comes from the Latin testamentum which means covenant. So, the Bible is a
covenant document. If you don't understand covenant you cannot understand, in totality, the issue of
baptism because baptism is a covenant sign.
If you don't think that God works covenantally then look at Heb 13:20 which says, " May the God
of peace, who through the blood of the eternal covenant brought back from the dead our Lord Jesus,
that great Shepherd of the sheep" (NIV). The Eternal Covenant is the covenant between the Father
and the Son before the creation of the world, whereby the Father would give to the Son those whom
the Father had chosen. That is why Jesus says things like, " All that the Father gives me will come to
me, and whoever comes to me I will never drive away" (John 6:37, NIV). And, "And this is the will of
him who sent me, that I shall lose none of all that he has given me, but raise them up at the last day"
(John 6:39, NIV). And, "I pray for them. I am not praying for the world, but for those you have given
me, for they are yours," (John 17:9, NIV).
If you fail to understand that God works covenantally and that He uses signs as manifestations of
his covenants (rainbow, circumcision, communion, etc.) then you will not be able to understand where
baptism fits in God's covenant system.
Second, you need to know what baptism is. It is an outward representation of an inward reality.
For example, it represents the reality of the inward washing of Christ's blood upon the soul. That is
why it is used in different ways. It is said to represent the death of the person (Rom. 6:3-5), the union
of that person with Christ (Gal. 3:27), the cleansing of that person's sins (Acts 22:16), the
identification with the one "baptized into" as when the Israelites were baptized into Moses (1 Cor.
10:2), and being united in one church (1 Cor. 12:13). Also, baptism is one of the signs and seals of
the Covenant of Grace that was instituted by Jesus. It is in this sense a sacrament. A sacrament is a
visible manifestation of something spoken. It is also said to be a visible sign of an inward grace. For
example, the communion elements of bread and wine are called the sacrament of communion. When
we take communion we are partaking of the sacrament.
The Covenant of Grace is the covenant between God and Man where God promises to Man eternal
life. It is based upon the sacrifice of Jesus on the cross and the condition is faith in Jesus Christ. As the
Communion Supper replaced Passover, baptism, in like manner, replaces circumcision. "They
represent the same spiritual blessings that were symbolized by circumcision and Passover in the old
dispensation," (Berkhoff, Lewis, Systematic Theology, 1988, p. 620.).
Circumcision was the initiatory rite into the Abrahamic covenant; it did not save. A covenant is a
pact or agreement between two or more parties and that is exactly what the Abrahamic covenant was.
God said to Abraham, "I will establish my covenant as an everlasting covenant between me and you
and your descendants after you for the generations to come, to be your God and the God of your
descendants after you" (Genesis 17:7, NIV). God later instructed Abraham to circumcise not only
every adult male, but also 8 day old male infants as a sign of the covenant (Gen. 17:9-13). If the
children were not circumcised, they were not considered to be under the promissory Abrahamic
covenant. This is why Moses' wife circumcised her son and threw the foreskin at Moses' feet. (Ex.
4:24-25). She knew the importance of the covenant between God and her children. But at the same
time we must understand that circumcision did not guarantee salvation to all who received it. It was a
rite meant only for the people of God, who were born into the family of God (who were then the Jews).

An important question here is how is it possible for an infant to be entered into a covenant with
God. There could be a lot of answers given but the point remains: it was done; infants were entered
into a covenant relationship with God -- through their parents.
In the New Testament, circumcision is mentioned many times. But with respect to this topic it is
specifically mentioned in Col. 2:11-12: "In him you were also circumcised, in the putting off of the
sinful nature, not with a circumcision done by the hands of men but with the circumcision done by
Christ, having been buried with him in baptism and raised with him through your faith in the power of
God, who raised him from the dead," (NIV). In these verses, baptism and circumcision are related.
Baptism replaces the Old Testament circumcision because 1) there was a New Covenant in the
communion supper (Luke 22:20), and 2) in circumcision there was the shedding of blood but in
baptism no blood is shed. This is because the blood of Christ has been shed.
If you understand that baptism is a covenant sign, then you can see that it is a representation of
the reality of Christ circumcising our hearts (Rom. 2:29; Col. 2:11-12). It is our outward proclamation
of the inward spiritual blessing of regeneration. It comes after faith which is a gift of God (Rom. 13:3)
and the work of God (John 6:28).
Third, the Bible says that it is the gospel that saves. " By this gospel you are saved..." (1 Cor.
15:2). Also, Rom. 1:16 says, "I am not ashamed of the gospel, because it is the power of God for the
salvation of everyone who believes: first for the Jew, then for the Gentile."

What is the Gospel?

It is clearly the gospel that saves us. But what exactly is the gospel? That too is revealed to us in
the Bible. It is found in 1 Cor. 15:1-4: "Now, brothers, I want to remind you of the gospel I preached
to you, which you received and on which you have taken your stand. By this gospel you are saved, if
you hold firmly to the word I preached to you. Otherwise, you have believed in vain. For what I
received I passed on to you as of first importance: that Christ died for our sins according to the
Scriptures, that he was buried, that he was raised on the third day according to the Scriptures ." The
gospel is defined as the death, burial, and resurrection of Jesus for our sins. Baptism is not mentioned
here.
Paul said that he came to preach the gospel, not to baptize: "I am thankful that I did not baptize
any of you except Crispus and Gaius, so no one can say that you were baptized into my name. (Yes, I
also baptized the household of Stephanas; beyond that, I don't remember if I baptized anyone else.)
For Christ did not send me to baptize, but to preach the gospel..." (1 Cor. 1:14-17). If baptism is
necessary for salvation then why did Paul downplay it and even exclude it from the description of what
is required for salvation? It is because baptism isn't necessary for salvation.
Additionally, in Acts, Peter was preaching the gospel, people got saved, and then they were
baptized. Acts 10:44-46 says, "While Peter was still speaking these words, the Holy Spirit came on all
who heard the message. The circumcised believers who had come with Peter were astonished that the
gift of the Holy Spirit had been poured out even on the Gentiles. For they heard them speaking in
tongues and praising God. Then Peter said, ‘Can anyone keep these people from being baptized with
water? They have received the Holy Spirit just as we have.' So he ordered that they be baptized in the
name of Jesus Christ. Then they asked Peter to stay with them for a few days," (NIV). These people
were saved. The gift of the Holy Spirit was on the Gentiles and they were speaking in tongues. This is
significant because tongues is a gift given to believers, see 1 Cor. 14:1-5. Also, unbelievers don't
praise God. They can't because praise to the true God is a deep spiritual matter that is foreign to the
unsaved (1 Cor. 2:14). Therefore, the ones in Acts 10 who are speaking in tongues and praising God
are definitely saved and they are saved before they are baptized. This simply isn't an exception. It is a
reality.

Let's Suppose...

Another way of making this clear is to use an illustration. Let's suppose that a person, under the
conviction of the Holy Spirit (John 16:8), believed in Jesus as his savior (Rom. 10:9-10; Titus 2:13),
and has received Christ (John 1:12) as Savior. Is that person saved? Of course he is. Let's further
suppose that this person confesses his sinfulness, cries out in repentance to the Lord, and receives
Jesus as Savior and then walks across the street to get baptized at a local church. In the middle of the
road he gets hit by a car and is killed. Does he go to heaven or hell? If he goes to heaven then
baptism isn't necessary for salvation. If He goes to hell, then trusting in Jesus, by faith, isn't enough
for salvation. Doesn't that go against the Scriptures that say that salvation is a free gift (Rom. 6:23)
received by faith (Eph. 2:8-9)?
Saying that baptism is necessary for salvation is dangerous because it is saying that there is
something we must do to complete salvation. That is wrong! See Gal. 2:21; 5:4.
All right, so this sounds reasonable. But still, what about those verses that seem to say that
baptism is part of salvation? I'll address those now. But, because this subject can become quite
lengthy, in fact, sufficient for a book in itself, I'll only address a few verses and then only briefly.

Baptism Verses

John 3:5, "Jesus answered, ‘I tell you the truth, no one can enter the kingdom of God unless he is
born of water and the Spirit.'"
Some say that water here means baptism. But that is unlikely since Christian baptism hadn't yet
been instituted. If this verse did mean baptism, then the only kind that it could have been at that point
was the baptism of repentance administered by John the Baptist (Mark 1:4). If that is so, then baptism
isn't necessary for salvation because the baptism of repentance is no longer practiced.
It is my opinion that the water spoken of here means the water of the womb referring to the
natural birth process. Jesus said in verse three that Nicodemus needed to be born "again." This meant
that he had been born once--through his mother. Nicodemus responds with a statement about how he
can't enter again into his mother's womb to be born. Then Jesus says that he must be born of water
and the Spirit. Then in verse 6 He says that "flesh gives birth to flesh, but the Spirit gives birth to
spirit.." The context seems to be discussing the contrast between the natural and the spiritual birth.
Water, therefore, could easily be interpreted there to mean the natural birth process.
I would like to add that there are scholars who agree with the position and some who do not.
Some believe that the water refers to the Word of God, the Bible, and others claim it means the Holy
Spirit. You decide for yourself.

Acts 2:38, "Peter replied, ‘Repent and be baptized, every one of you, in the name of Jesus Christ for
the forgiveness of your sins. And you will receive the gift of the Holy Spirit.‘"
This verse is a tough one. It seems to say that baptism is part of salvation. But we know, from
other scriptures that it isn't, lest there be a contradiction. What is going on here is simply that
repentance and forgiveness of sins are connected. In the Greek, "repent" is in the plural and so is
"your" of "your sins." They are meant to be understood as being related to each other. It is like saying,
"All of you repent, each of you get baptized, and all of you will receive forgiveness." Repentance is a
mark of salvation because it is granted by God (2 Tim. 2:25) and is given to believers only. In this
context, only the regenerated, repentant person is to be baptized. Baptism is the manifestation of the
repentance, that gift from God that is the sign of the circumcised heart. That is why it says, repent and
get baptized.

1 Pet. 3:21, "and this water symbolizes baptism that now saves you also -- not the removal of dirt
from the body but the pledge of a good conscience toward God. It saves you by the resurrection of
Jesus Christ."
This is the only verse that says that baptism saves. But, the NIV translation of the verse is
unfortunate. A better translation is found in the NASB which says, "and corresponding to that, baptism
now saves you." The key word in this section is the Greek “antitupon.” It means "copy," "type,"
corresponding to," "a thing resembling another," "its counterpart," etc. Baptism is a representation, a
copy, a type of something else. The question is "Of what is it a type?", or "Baptism corresponds to
what?” The answer is found in the previous verse, verse 20: "who once were disobedient, when the
patience of God kept waiting in the days of Noah, during the construction of the ark, in which a few,
that is, eight persons, were brought safely through the water. 21And corresponding to that, baptism
now saves you" (NASB).
What does baptism correspond to? Is it the flood? Or, is it the ark? What was it that saved Noah
and his family? Was it the water or the ark? Obviously, it was the Ark. Noah built and entered the ark
by faith and was saved (Heb. 11:7). The flood waters destroyed the ungodly. Peter, when referring to
the flood waters, refers to them as the means of destruction of the ungodly (2 Pet. 2:5; 3:6). It was
the Ark that saved. Noah entered the ark by faith. Baptism here, in my opinion, refers to the Ark, not
the waters. That is why the rest of the verse says, "not the removal of dirt from the body but the
pledge of a good conscience toward God" which is consistent with what Paul said in Col. 2:11-12 where
He equates baptism with being circumcised of heart.

Acts 22:16, "And now what are you waiting for? Get up, be baptized and wash your sins away, calling
on his name."
Is the washing away of sins done by baptism, the representation of the circumcised heart (Col.
2:11-12) which means you are already saved, or is it by the blood of Christ (Heb. 9:14; Rom. 5:9;
Eph. 1:7)? Obviously it is the blood of Jesus and the washing here refers to the calling on Jesus' name.

Rom. 6:4, "We were therefore buried with him through baptism into death in order that, just as Christ
was raised from the dead through the glory of the Father, we too may live a new life."
Because the believer is so closely united to Christ it is said that the symbol of baptism is our
death, burial, and resurrection. Obviously we did not die--unless, of course, it is a figurative usage.

Titus 3:5, "he saved us, not because of righteous things we had done, but because of his mercy. He
saved us through the washing of rebirth and renewal by the Holy Spirit."
The washing of rebirth can only be that washing of the blood of Christ that cleanses us. It is not
the symbol that saves, but the reality. The reality is the blood of Christ.

Gal. 3:27, "for all of you who were baptized into Christ have clothed yourselves with Christ."
This is speaking of the believer's union with Christ. It is an identification with, a joining to, a
proclamation of loyalty to, etc. In 1 Cor. 10:2 the Israelites were baptized into Moses. That means
they were closely identified with him and his purpose. The same thing is meant here.

Conclusion

Baptism is not necessary for salvation. It is the initiatory sign and seal into the covenant of grace.
As circumcision referred to the cutting away of sin and to a change of heart (Deut. 10:16; 30:6; Jer.
4:4; 9:25,26; Ez. 44:7,9) baptism refers to the washing away of sin (Acts 2:38; 1 Pet. 3:21; Tit. 3:5)
and to spiritual renewal (Rom. 6:4; Col. 2:11-12). The circumcision of the heart is signified by the
circumcision of the flesh, that is, baptism (Col. 2:11-12).
One last thought: If someone maintains that baptism is necessary for salvation, is he adding a
work, his own, to the finished work of Christ? If the answer is yes, then that person would be in
terrible risk of not being saved. If the answer is no, then why is baptism maintained as being
necessary the same way as the Jews maintained that works were necessary?
Baptism and Mark 16:16

"He who believes and is baptized will be saved; but he who does not believe will be
condemned," (Mark 16:16).

This verse is frequently used by baptismal regenerationists to show that baptism is necessary for
salvation. It says he who believes and is baptized will be saved. Therefore, they conclude that
baptism is a necessary part of becoming saved. But, does this verse prove that baptism is necessary
for salvation? Not at all.
Mark 16:16 does not say that baptism is a requirement for salvation. Let me show you why. I
could easily say that he who believes and goes to church will be saved. That is true. But it is belief
that saves, not belief and going to church. Likewise, if you believe and read your Bible, you'll be
saved. But it isn't reading your Bible that saves you. Rather, belief in Christ, in His sacrifice, is what
saves. As I've stated in other papers on this subject, there are numerous verses that clearly
demonstrate that justification is by faith (Rom. 5:1; Eph. 2:8; Phil. 3:9; etc.). Belief in what God has
done, not what man can do, is what results in salvation. Baptism is simply a public demonstration of
the inner work of regeneration. This is why the rest of the verse says, "...but he who does not to
believe will be condemned." Mark 16:16 focuses on the issue of belief, not baptism.

A textual issue with Mark 16:9-20

What I will share here may not be very popular with some readers. Therefore, I need to say
upfront that I believe in the absolute inspiration and authority of the Bible. It is the word of God and
what it says is authoritative. However, the simple fact is that there are textual variations within the
biblical manuscripts. The originals are what are inspired, not the copies. We have copies of inspired
documents. These copies are not perfect, but they are very close to it.
Again, I am not saying the Bible is untrustworthy. It is 98.5% textually pure. The remaining 1.5%
of textual variation are almost entirely of insignificant spelling errors and minor word omissions or
additions that do not change the meaning of the text. However, Mark 16:9-20 is a significant textual
variant. Many scholars, Christian scholars, consider the ending of Mark to lack authenticity. Please
consider the following evidence.

1. Mark 16:9-20 doesn't appear in many of the oldest ancient manuscripts.

A. “The last twelve verses of Mark (16:9-20) are lacking in the two earliest parchment codices,
B and Aleph, in the Old Latin manuscript k,, the Sinaitic Syriac, many manuscripts of the Old
Armenian version, the Adysh and Opiza manuscripts of the Old Georgian version, and a
number of manuscripts of the Ethiopic version. Clement of Alexandria, Origen, and
Ammonius show no knowledge of the existence of these verses; other Church Fathers state
that the section is absent from Greek copies of Mark known to them (e.g. Jerome, Epist.
cxx. 3, ad hedibiam,)...The original form of the Eusebian sections makes no provision for
numbering sections after 16:8. Not a few manuscripts which contain the passage have
scholia stating that older Greek copies lack it (so, for example, MSS. 1, 20,22, &c.), and in
other witnesses the passage is marked with asterisks or obeli, the conventional sigla used by
scribes to indicate a spurious addition to a literary document." 10

10
"The Text of the New Testament," by Bruce Metzger (Professor of New Testament Language and Literature,
Princeton Theological Seminary), 2nd ed., Oxford University Press, New York, 1968, p. 226.
2. There are other endings to Mark.
A. Another ending is found in L, Psi, 099, 0112, and minuscules 274mg 579, k, Syrh and more
is as follows:
i. "But they reported briefly to Peter and those with him all that had been told. And after
this Jesus himself sent out by means of them, from east to west, the sacred and
imperishable proclamation of eternal salvation."
3. Apparent, theological error.
A. Mark 16:12 says, "And after that, He appeared in a different form to two of them, while they
were walking along on their way to the country." This verse may be problematic. Jesus rose
in the same body that he died in (John 2:19), though it was a glorified body. This is
problematic because it suggests "a different form." Jesus did not appear in a different form.
He appeared in the same body he rose in.
4. Evidence against the Mark authorship.
A. There are 17 non-marcan words used in a non-marcan sense in these verses.

This information about the ending of Mark is not intended to cast doubt upon God's word. But
the fact is that the ending is under a large cloud of doubt as to its authenticity. I would not use it
as a defense for baptismal regeneration.
Baptism and Roman 6:3-5

Romans 6:3-5 is often used as a proof text for the claim that baptism is essential for salvation. It
is a strong comparison between our baptism and Christ's death, burial, and resurrection. On the
surface, one could conclude that from these verses, that baptism is part of salvation.

"Or do you not know that all of us who have been baptized into Christ Jesus have been
baptized into His death? 4Therefore we have been buried with Him through baptism into death,
in order that as Christ was raised from the dead through the glory of the Father, so we too
might walk in newness of life. 5For if we have become united with Him in the likeness of His
death, certainly we shall be also in the likeness of His resurrection,"

Is this section of scripture teaching us that baptism is necessary for salvation? No, it is not. First,
we know from the rest of scripture that salvation is by faith, not by faith and something we do Rom.
3:28-30. Second, we can see from other scriptures that baptism follows faith. Take a look at Acts
16:30-33 where the Jailer specifically asks what he must do to be saved and where baptism fits in.

"and after he brought them out, he said, "Sirs, what must I do to be saved?" 31And they said,
"Believe in the Lord Jesus, and you shall be saved, you and your household." 32And they spoke
the word of the Lord to him together with all who were in his house. 33And he took them that
very hour of the night and washed their wounds, and immediately he was baptized, he and all
his household," (Acts 16:30-33, NASB).

If baptism were part of salvation, then Paul should have said, "Believe and be baptized and you will
be saved." But, he did not. Also, consider Acts 10:44-46.

"While Peter was still speaking these words, the Holy Spirit came on all who heard the
message. The circumcised believers who had come with Peter were astonished that the gift of
the Holy Spirit had been poured out even on the Gentiles. For they heard them speaking in
tongues and praising God. Then Peter said, ‘Can anyone keep these people from being baptized
with water? They have received the Holy Spirit just as we have.' So he ordered that they be
baptized in the name of Jesus Christ. Then they asked Peter to stay with them for a few days,"
(NIV).

These people were saved. The gift of the Holy Spirit was on the Gentiles and they were speaking in
tongues. This is significant because tongues is a gift given to believers, see 1 Cor. 14:1-5. Also,
unbelievers don't praise God. They can't because praise to the true God is a deep spiritual matter that
is foreign to the unsaved (1 Cor. 2:14). Therefore, the ones in Acts 10:44-46 who are speaking in
tongues and praising God are definitely saved and they are saved before they are baptized. This isn't
an exception. It is a reality. This proves that baptism is not necessary for salvation.
What is Romans 6:3-5 saying?

"Or do you not know that all of us who have been baptized into Christ Jesus have been
baptized into His death? 4Therefore we have been buried with Him through baptism into death,
in order that as Christ was raised from the dead through the glory of the Father, so we too
might walk in newness of life. 5For if we have become united with Him in the likeness of His
death, certainly we shall be also in the likeness of His resurrection,"

The phrase "baptized into" occurs five times in the NT in four verses as found in the KJV and the
NASB..

1. Rom. 6:3, "Or do you not know that all of us who have been baptized into Christ
Jesus have been baptized into His death?"
2. 1 Cor. 10:2, "and all were baptized into Moses in the cloud and in the sea."
3. 1 Cor. 12:13, "For by one Spirit we were all baptized into one body, whether
Jews or Greeks, whether slaves or free, and we were all made to drink of one Spirit."
4. Gal. 3:27, "For all of you who were baptized into Christ have clothed yourselves
with Christ."

To be baptized "into Christ," "into His death," "into Moses," and "into one body" is to be publicly
identified with the thing you are being baptized into. The focus is not the baptism itself, but on the
thing the baptism represents. In the case of Rom. 6:3-5, being baptized into Christ is a public
identification with Christ's death, burial, and resurrection which is said to be the gospel that saves in 1
Cor. 15:1-4. Baptism then is a public statement proclaiming that the person is trusting in the sacrifice
of Christ.
Baptism by immersion is a perfect symbol for this work of Christ with which the Christian is
identifying himself. As Christ died and was raised to a new life, so to the Christian, in Christ, is said to
have died (Rom. 6:11; Col. 3:3) and has a new life. This new life of regeneration is by faith, the
internal work. Baptism is the external work of identification with Christ. This is why the reference to
baptism in the Bible is dealing more with "our union and identification with Christ than to our water
baptism."11

• Baptism is being identified as a disciple (Matt. 28:18-9).


• Baptism may be compared to a new birth (John 3:5).
• Baptism is compared to Jesus' death and resurrection (Rom. 6:3-5).
• Baptism is compared to Israel's Exodus and passing through the Red Sea (1 Cor. 10:2).
• Baptism is compared to Noah's escaping the flood waters by entering the ark (1 Pet. 3:21).

In each of the references above, baptism is an identification with something. When people were
baptized into John the Baptist's baptism of repentance, it wasn't the baptism that granted them
repentance or made repentance real. Repentance is something that happens internally and is the work
of God (2 Tim. 2:25). To participate in John's baptism was to publicly proclaim that the person being
baptized was accepting John's message or repentance. Hence, it was called a baptism of repentance.
It wasn't the baptism that brought repentance; rather, baptism was the result of repentance. The
person had to first decide to repent, and then become baptized as a proclamation of his decision.
Likewise, the Christian must first decide to repent, to receive Christ (John 1:12), to rely on the
sacrifice of Christ, by faith, and then participate in the public proclamation of identifying with Christ's
work.

11
Enhanced Strong’s Lexicon, (Oak Harbor, WA: Logos Research Systems, Inc.) 1995.
It is an identification with the death, burial, and resurrection of Christ that baptism represents.
Jesus' shed blood is what cleanses us from our sins (Heb. 9:22), not being washed with water. It is
Christ's death that is the payment for sin. Jesus' burial is the proof that He, in fact, died. Jesus'
resurrection is the proof of God the Father's acceptance of the sacrifice of Christ and that death is
conquered. Again, for a Christian to be baptized is to make a public proclamation that he is trusting in
Christ's work, that he is naming himself with Christ and trusting what Christ has done. This is why it
says in Rom. 6:11, "Even so consider yourselves to be dead to sin, but alive to God in Christ Jesus,"
(NASB). Why? Because "I have been crucified with Christ; and it is no longer I who live, but Christ
lives in me; and the life which I now live in the flesh I live by faith in the Son of God, who loved me,
and delivered Himself up for me," (Gal. 2:20). It is on the cross that Jesus paid for our sins, not in His
baptism and not in our baptism. It is our identification with Him, being counted "in Christ" that allows
us to say we have been crucified with Christ so that we can say we are dead to sin. We are not dead
to sin by our baptism. Rather, we are dead to sin, by faith, in what Jesus did in His sacrifice.

Conclusion

Romans 6:3-5 speaks to us of Christ's work and our public identification with it. In that ancient
world of religious plurality in Roman gods, in the strict Laws of the Jewish system, and in the gods of
different cultures, to be baptized was to make a bold statement of commitment to Christ as the risen
Lord. It was not the water that saved, but faith in Christ and His work.
Baptism and Gal. 3:27

Gal. 3:27 is often used by the baptismal regenerationists to support the idea that you must be
baptized to be saved. They maintain that baptism is the place where a person "puts on Christ," where
he is "clothed with Christ" and that it means that baptism saves. They teach that being immersed in
the baptismal water is the place and time of deliverance from sins. This is simply not true.
Gal. 3:27 cannot be understood alone. It must be examined in context.

"Therefore the Law has become our tutor to lead us to Christ, that we may be justified by faith.
25
But now that faith has come, we are no longer under a tutor. 26For you are all sons of God
through faith in Christ Jesus. 27For all of you who were baptized into Christ have clothed
yourselves with Christ. 28There is neither Jew nor Greek, there is neither slave nor free man,
there is neither male nor female; for you are all one in Christ Jesus. 29And if you belong to
Christ, then you are Abraham’s offspring, heirs according to promise," (Gal. 3:24-29).

In Roman society, children were often committed to the care of trusted slaves. This would often
happen when the child was between six or seven, and it would last until puberty. "These slaves were
severe disciplinarians and were charged with guarding the children from the evils of society and giving
them moral training. This was like the Law’s function until Christ came and people could be justified by
faith in Him." 12 The Law was a harsh master to the Jews. It was very difficult to keep. This is why the
Law points to Christ by showing us our inability to keep the Law and by showing us that we must rely
on faith instead. That is why justification is by faith (vv. 24-26), because we cannot attain justification
by Law (Rom. 3:28-30; Phil. 3:9).

"For all of you who were baptized into Christ have clothed yourselves with Christ," (Gal. 3:27).

In Roman society when a child who had been under the care of a tutor and reached a matured
enough age, he was given a special robe, or toga. It was symbolic of his full rights in the family. 13
Therefore, being "clothed with Christ" is a phrase meaning that the Christian moved out from the Law
and into the gospel of grace and can enjoy full acceptance before God the Father. It is not saying that
baptism is what saves us from our sins.

12
Walvoord, John F., and Zuck, Roy B., The Bible Knowledge Commentary, (Wheaton, Illinois: Scripture Press
Publications, Inc.) 1983, 1985, on Gal. 3:24.
13
ibid, on Gal. 3:27.
Baptism and 1 Pet. 3:21
1 Pet. 3:21 says, "and this water symbolizes baptism that now saves you also -- not the removal
of dirt from the body but the pledge of a good conscience toward God. It saves you by the resurrection
of Jesus Christ." This is the only verse that says that baptism saves. Is it teaching that we must be
baptized to be saved? No. But, but to rightly understand it, we need to look at its context.

"For Christ also died for sins once for all, the just for the unjust, in order that He might bring us
to God, having been put to death in the flesh, but made alive in the spirit; 19 in which also He
went and made proclamation to the spirits now in prison, 20 who once were disobedient, when
the patience of God kept waiting in the days of Noah, during the construction of the ark, in
which a few, that is, eight persons, were brought safely through the water. 21 And
corresponding to that, baptism now saves you—not the removal of dirt from the flesh, but an
appeal to God for a good conscience — through the resurrection of Jesus Christ, 22 who is at
the right hand of God, having gone into heaven, after angels and authorities and powers had
been subjected to Him," (1 Pet. 3:18-22, NASB).

The above translation in verse 21 from the NASB is a good translation. "And corresponding to
that, baptism now saves you." The key word in this section is the Greek antitupon. It means "copy,"
"type," "corresponding to," "a thing resembling another," "its counterpart," etc. It is what the NIV
translates as "symbolizes," the NASB as "corresponding to that," and the KJV as "like figure."
Baptism, then, is a representation, a copy, a type of something else. The question is "Of what is it a
type?", or "baptism corresponds to what?"
If we look at the context, an interesting possibility arises, though I will admit, not the favored
interpretation among scholars. What does baptism correspond to? Is it the flood? Or, is it the ark?
What was it that saved Noah and his family, the flood or the ark? Obviously, it was the Ark. Noah built
and entered the ark by faith and he was saved (Heb. 11:7). The flood waters destroyed the ungodly.
Also, Peter consistently refers to the flood waters as the means of destruction of the ungodly ( 2 Pet.
2:5; 3:6), not the salvation of Noah and his family. Rather, it was the Ark that saved, the ark that
Noah entered faith. It may very well be that baptism refers to the Ark, not the waters. That is why
the rest of the verse says, "not the removal of dirt from the body but the pledge of a good conscience
toward God" which is consistent with what Paul said in Col. 2:11-12 where He equates baptism with
being circumcised of heart.
The problem with this interpretation is that it doesn't seem to fit the "water for water typology." It
would seem more natural to equate the water of baptism with the water of the flood. Furthermore, if
we were to look at the flood waters as the thing that removed evil from the land, we could say that
"correspondingly," the waters of baptism remove removes the sin from our hearts. Though this
reading seems a bit more natural, it too has problems.
The water of baptism is not what saves us; the sacrifice of Christ does which we receive by faith.
We read numerous verses about justification by faith (Rom. 5:1), salvation by faith (Eph. 2:8), etc.,
not justification "by faith and baptism," or salvation "by faith and baptism." 14 The fact is that salvation
is received by faith. Peter, not wanting to declare that baptism itself is what saves us, quickly adds,
"not the removal of dirt from the flesh, but an appeal to God for a good conscience." Water baptism,
then, must accompany the work of the Holy Spirit in the person. Peter's explanatory comment shows
us that the act of physical baptism is not what saves, but the "baptism of appeal to God." This appeal
to God is by faith the same as Noah's faith in God led him to build the Ark, enter it, and remain in it.
It was the Ark that saved Noah, not the flood waters.
The flood was for Noah a type of baptism even as the passage through the Red Sea was a type of
baptism for the Israelites.

"I want you to know, brethren, that our fathers were all under the cloud, and all passed
through the sea, 2and all were baptized into Moses in the cloud and in the sea, 3and all ate the

14
Mark 16:16 says, "He who believes and is baptized will be saved; but he who does not believe will be
condemned." Please see the article on Baptism and Mark 16:16 for an examination of this verse.
same supernatural food 4and all drank the same supernatural drink. For they drank from the
supernatural Rock which followed them, and the Rock was Christ," (1 Cor. 10:1-4).

The "baptisms" of both Noah and the Israelites served as types of a transition; that is, they moved
people from the old world to the new, from the old covenant to the new covenant. It is not the water
that saves, but the spiritual thing associated with that water that saves. For Noah it was faith in God.
For Moses it too was faith in God.
But some may say that the work of the Holy Spirit and the act of baptism are simultaneous, that
the Holy Spirit works in and through baptism to bring regeneration. But this cannot be the case since
the Bible tells us that salvation is by faith (Rom. 5:1; Eph. 2:8). Besides, we have a clear instance in
scripture where people are saved before their baptism.

Acts 10:44-48

"While Peter was still speaking these words, the Holy Spirit fell upon all those who were
listening to the message. 45And all the circumcised believers who had come with Peter were
amazed, because the gift of the Holy Spirit had been poured out upon the Gentiles also. 46For
they were hearing them speaking with tongues and exalting God. Then Peter answered,
47
"Surely no one can refuse the water for these to be baptized who have received the Holy
Spirit just as we did, can he?" 48And he ordered them to be baptized in the name of Jesus
Christ. Then they asked him to stay on for a few days," (Acts 10:44-48).

In these verses we see that Peter had been preaching the gospel and the Holy Spirit fell upon the
listeners. In verse 45 we read that "the gift of the Holy Spirit had been poured out upon the Gentiles
also." This gift manifested itself in speaking in tongues. This is significant because tongues is a sign-
gift given to believers, see 1 Cor. 14:1-5. Also, verse 46 says they were "exalting God." Unbelievers
don't praise God. They can't because praise to the true God is a deep spiritual matter that is foreign to
the unsaved (1 Cor. 2:14). Therefore, the ones in Acts 10 who are speaking in tongues and praising
God are definitely saved because they are moving in the Holy Spirit, speaking in tongues, and
glorifying God. It is the Holy Spirit who gives charismatic spiritual gifts to the church ( 1 Cor. 12:27-
28), not to unbelievers. Now, please notice that it was after this movement of the Holy Spirit that the
believers are baptized. If baptism is necessary for salvation, then how is it that the people were
speaking in tongues and exalting God before they were baptized?
If you were to say that the Holy Spirit was simply working upon and through those not yet saved,
then remember that tongues and praise to God are for the church, not the unbelievers. The church
consists of people who are saved, not unsaved. If they were not saved until they were baptized, then
they were not in the body of Christ and would not have moved in the charismatic gifts. Therefore,
they were regenerate before they were baptized. This simply isn't an exception. It is a reality.

Conclusion

1 Pet. 3:21 is not teaching us that baptism is what saves us. Rather, it is showing us that the
water symbolizes a spiritual cleansing through the power of the Holy Spirit gained through Christ’s
victory over death. It is the person's appeal to God that saves the soul, not the washing of water upon
the body.
Baptism and John 3:5

"Truly, truly, I say to you, unless one is born again, he cannot see the kingdom of God."
4
Nicodemus *said to Him, "How can a man be born when he is old? He cannot enter a second
time into his mother's womb and be born, can he?" 5Jesus answered, "Truly, truly, I say to you,
unless one is born of water and the Spirit, he cannot enter into the kingdom of God. 6"That
which is born of the flesh is flesh, and that which is born of the Spirit is spirit. 7"Do not marvel
that I said to you, 'You must be born again.' 8"The wind blows where it wishes and you hear the
sound of it, but do not know where it comes from and where it is going; so is everyone who is
born of the Spirit," (John 3:3-8).

There are five basic interpretations to this section of scripture in reference to water.

1. The water refers to the natural birth.


2. The water refers to the Word of God.
3. The water refers to the Holy Spirit.
4. The water refers to the ministry of John the Baptist.
5. The water refers to the water of baptism as a requirement for salvation.

The first option looks to the context of Jesus' words dealing with being born "again,” ( 3:3).
Nicodemus responds by mentioning the experience of being born from the womb (v. 4). Jesus then
speaks of water and the Spirit and then says, "That which is born of the flesh is flesh, and that which
is born of the Spirit is spirit,” (3:6). However, this view is not the most commonly held view.
The second option holds that the water is referring to the Word of God. Eph. 5:26 says, "that He
might sanctify her, having cleansed her by the washing of water with the word." Some believe that the
washing of water is done by means of the Word of God.
The third view says that the water refers to the Holy Spirit. Perhaps Nicodemus was reminded of
Ezek. 36:25-27, "Then I will sprinkle clean water on you, and you will be clean; I will cleanse you from
all your filthiness and from all your idols. 26"Moreover, I will give you a new heart and put a new spirit
within you; and I will remove the heart of stone from your flesh and give you a heart of flesh. 27"And I
will put My Spirit within you and cause you to walk in My statutes, and you will be careful to observe
My ordinances." Certainly, Jesus' own words are applicable here when He says in John 7:37-39, "Now
on the last day, the great day of the feast, Jesus stood and cried out, saying, "If any man is thirsty, let
him come to Me and drink. 38"He who believes in Me, as the Scripture said, 'From his innermost being
shall flow rivers of living water.'" 39But this He spoke of the Spirit, whom those who believed in Him
were to receive; for the Spirit was not yet given, because Jesus was not yet glorified."
The fourth view holds that the water is in reference to the water baptism of repentance taught by
John the Baptist. Matt. 3:1-6 describes John's ministry in the desert, his teaching about repentance,
and baptizing people into that repentance. Contextually, the first chapter of John mentions John the
Baptist in verses 6-8 and 19-36. Certainly, contextually, John and his ministry is in view here. If this is
the case, then Jesus would have been speaking of the "baptism" (the initiatory ordinance) of
repentance preached by John the Baptist.
The fifth view is the one held by the International Church of Christ; namely, that the water is
referring to baptism and that it is essential to salvation.

Does John 3:5 teach that baptism is essential to salvation?

As you can see, there are different interpretations to John 3:5. But, to simply say that John 3:5
does not teach the necessity of baptism isn't enough. Some sort of proof must be offered. The proof is
found in God's word that has no contradictions. Clearly, salvation is by faith. For example, Rom. 5:1
states that we are justified (declared righteous) by faith. It does not say faith and baptism. If baptism
were part of salvation, then it would say we were justified by faith and baptism. But it does not. If
justification is by faith, then it is by faith. Baptism is not faith. It is a ceremony. Furthermore, please
consider the following verses when declare how we are saved.

1. Rom. 3:22, "even the righteousness of God through faith in Jesus Christ for all those
who believe; for there is no distinction."
2. Rom. 3:26, "for the demonstration, I say, of His righteousness at the present time, that
He might be just and the justifier of the one who has faith in Jesus."
3. Rom. 3:28, "For we maintain that a man is justified by faith apart from works of the
Law."
4. Rom. 4:5, "But to the one who does not work, but believes in Him who justifies the
ungodly, his faith is reckoned as righteousness."
5. Rom. 5:1, "Therefore having been justified by faith, we have peace with God through
our Lord Jesus Christ,"
6. Gal. 3:8, "And the Scripture, foreseeing that God would justify the Gentiles by faith,
preached the gospel beforehand to Abraham."
7. Gal. 3:24, "Therefore the Law has become our tutor to lead us to Christ, that we may be
justified by faith."
8. Eph. 2:8, "For by grace you have been saved through faith; and that not of yourselves,
it is the gift of God."

Additionally, Paul tells us that the gospel is what saves us and that the gospel is the death, burial,
and resurrection of Jesus, (1 Cor. 15:1-4). Baptism is not included in the description of the gospel.
This explains why said he came to preach the gospel, not to baptize: "I am thankful that I did not
baptize any of you except Crispus and Gaius, so no one can say that you were baptized into my name.
(Yes, I also baptized the household of Stephanas; beyond that, I don't remember if I baptized anyone
else.) For Christ did not send me to baptize, but to preach the gospel..." (1 Cor. 1:14-174). If baptism
is necessary for salvation then why did Paul downplay it and even exclude it from the description of
what is required for salvation? It is because baptism isn't necessary for salvation. Therefore, John 3:5
must be interpreted in a manner consistent with the rest of scripture.
Another way of making this clear is to use an illustration. Let's suppose that a person, under the
conviction of the Holy Spirit (John 16:8), believed in Jesus as his savior (Rom. 10:9-10; Titus 2:13),
and has received Christ (John 1:12) as Savior. Is that person saved? Of course he is. Let's further
suppose that this person who confesses his sinfulness, cries out in repentance to the Lord, and
receives Jesus as Savior, then walks across the street to get baptized at a local church. In the middle
of the road he gets hit by a car and is killed. Does he go to heaven or hell? If he goes to heaven then
baptism isn't necessary for salvation. If He goes to hell, then trusting in Jesus, by faith, isn't enough
for salvation. Doesn't that go against the Scriptures that say that salvation is a free gift ( Rom. 6:23)
received by faith (Eph. 2:8-9)? Yes it does. Baptism is not necessary for salvation and John 3:5 cannot
teach that it is.
Baptism and Acts 2:38
Acts 2:38 is one of the more controversy verses in the Bible regarding baptism and whether or not
it is the requirement for salvation. On the surface it seems to support it. But upon closer examination,
we will see that it does not teach baptismal regeneration: that baptism saves.
First of all, rarely is doctrine ever made from a single verse. We need to look at all of what God's
word says about a subject in order to accurately understand what it teaches. I will briefly tackle of this
verse in the following manner.

• Examination of the verse's syntax, grammar and structure.


• Examine other verses dealing with the forgiveness of sins.
• Examine the verse in its covenant context.

Grammar and Structure of Acts 2:38

In Acts 2:38 the main verb is metanoesate (change mind), the aorist direct imperative (a
command) of metanoeo which means to repent (change mind). This refers to that initial repentance of
the sinner unto salvation. The verb translated "be baptized" is in the indirect passive imperative (a
command to receive; hence, passive voice in Greek 15) of baptizo, which does not give it the same
direct command implied in "repent." The preposition "for" in the phrase "for the remission of sins" in
Greek is "eis," unto or into, and it is in the accusative case (direct object). It can mean "for the
purpose of identifying you with the remission of sins." It is the same preposition we find in 1 Cor. 10:2
in the phrase "and were baptized unto Moses." Note that both contexts are dealing with baptism and
identification. These people were baptized or spiritually identifying themselves with the purposes and
vision of Moses. Repentance, therefore, is presented as identifying an individual with the remission of
his sins, even as baptism following repentance provides an external identification visible by others.
Repentance is something that concerns an individual and God while baptism involves others. That is
why baptistheto (let be immersed) is in the passive voice indicating that one does not baptize himself,
but is baptized by another usually in the presence of others. Repentance, however, is an act taking
place within a person's heart as the Holy Spirit moves in the sinner.
But, all this Greek stuff may be confusing. Let me break it down. All people are commanded to
repent for their sins. This is
what believers have already done by
becoming Christians. Baptism,
then, is the outward identification
with being a Christian for those
who have already repented.
Also, as the Israelites were
"baptized into Moses," (1 Cor.
10:2), so too, Christians are
baptized into Jesus. That is, they are
identifying themselves, publicly, with Christ. Likewise, in Rom. 6:1-5 where baptism is related to
death, burial, and resurrection, it is again and identification with Christ in His death, burial, and
resurrection. That is why it is said of Christians that we have died to sin ( Rom. 6:2, 11; Gal. 2:20;
Col. 2:20; Col. 3:3; 1 Pet. 2:24).
This verse is not demonstrating that baptism is essential for salvation, but that baptism is the thing
which we receive, in order to publicly identify ourselves completely and totally with Christ as a
manifestation of the inward work God has done within us.

15
Active voice is "I hit the ball." Passive voice is "The ball hit me." Middle voice is "I was hit by the ball." In
active voice, "I" performed the action. In passive voice, "I" received the action. In middle voice, "I" did
something to myself.
Other verses dealing with salvation

Justification is the work of God where the righteousness of Jesus is reckoned to the sinner so the
sinner is declared, by God, as being righteous under the Law (Rom. 4:3; 5:1,9; Gal. 2:16; 3:11). This
righteousness is not earned or retained by any effort of the saved. Justification is an instantaneous
occurrence with the result being eternal life. It is based completely and solely upon Jesus' sacrifice on
the cross (1 Pet. 2:24) and is received by faith alone (Rom. 4:5; 5:1; Eph. 2:8-9). No works are
necessary whatsoever to obtain justification. Otherwise, it is not a gift ( Rom. 6:23). Therefore, we are
justified by faith (Rom. 5:1).
Nowhere in the Bible does it state that we are justified by grace and baptism or faith and baptism
or faith and anything else. On the contrary, baptism is excluded from the gospel message. Paul said
that he came to preach the gospel, not to baptize: "I am thankful that I did not baptize any of you
except Crispus and Gaius, so no one can say that you were baptized into my name. (Yes, I also
baptized the household of Stephanas; beyond that, I don't remember if I baptized anyone else.) For
Christ did not send me to baptize, but to preach the gospel..." (1 Cor. 1:14-17).
Likewise, Paul told us exactly what the gospel that saves is. He said in 1 Cor. 15:1-4, "Now I make
known to you, brethren, the gospel which I preached to you, which also you received, in which also
you stand, 2by which also you are saved, if you hold fast the word which I preached to you, unless you
believed in vain. 3For I delivered to you as of first importance what I also received, that Christ died for
our sins according to the Scriptures, 4and that He was buried, and that He was raised on the third day
according to the Scriptures." Note that Paul state and that the gospel is what saints and he did not
include baptism in the definition of the gospel.”
So, we must ask if baptism is necessary for salvation, then why did Paul downplay it and even
exclude it from the description of what is required for salvation? It is because baptism isn't necessary
for salvation.
Further proof that baptism is not a requirement of salvation can be found in Acts 10:44-46. Peter
was preaching the gospel, people became saved, and then they were baptized. Acts 10:44-46 says,

"While Peter was still speaking these words, the Holy Spirit came on all who heard the
message. The circumcised believers who had come with Peter were astonished that the gift of
the Holy Spirit had been poured out even on the Gentiles. For they heard them speaking in
tongues and praising God. Then Peter said, ‘Can anyone keep these people from being baptized
with water? They have received the Holy Spirit just as we have.' So he ordered that they be
baptized in the name of Jesus Christ. Then they asked Peter to stay with them for a few days,"
(NIV).

These people were saved. The gift of the Holy Spirit was on the Gentiles and they were speaking in
tongues. This is significant because tongues is a gift given to believers, see 1 Cor. 14:1-5. Also,
unbelievers don't praise God. They can't because praise to the true God is a deep spiritual matter that
is foreign to the unsaved (1 Cor. 2:14). Therefore, the ones in Acts 10:44-46 who are speaking in
tongues and praising God are definitely saved and they are saved before they are baptized. This isn't
an exception. It is a reality. This proves that baptism is not necessary for salvation and that Acts 2:38
is not teaching its necessity either. But, if it isn't saying that, then why is baptism mentioned here?

Biblical Covenant Context

A covenant is a pact or agreement between two or more parties. Very often, covenants have
visible signs to represent them. The elements of bread and wine in the communion support are good
examples of this. Circumcision was both a covenant sign and and the initiatory rite into the Abrahamic
covenant (Gen. 17:10). But this covenant sign did not save anyone.
God said to Abraham, "I will establish my covenant as an everlasting covenant between me and
you and your descendants after you for the generations to come, to be your God and the God of your
descendants after you," (Gen. 17:7, NIV). God later instructed Abraham to circumcise not only every
adult male, but also eight day old male infants as a sign of the covenant ( Gen. 17:9-13). If the
children were not circumcised, they were not considered to be under the promissory Abrahamic
covenant. This is why Moses' wife circumcised her son and threw the foreskin at Moses' feet after
Moses failed to circumcise him, (Exo. 4:24-25). She knew the importance of the covenant between
God and her children. But at the same time we must understand that circumcision did not guarantee
salvation to those who received it. It was a rite meant only for the people of God, who were born into
the family of God (who were then the Jews). It was an outward sign of the covenant promise. To
reject it was to reject the covenant. But, accepting it did not guarantee salvation.

Another theological debate at risk here

There is debate within Christianity on the nature of baptism and to whom it may be administered.
I am not here trying to convince anyone of the proper objects of baptism whether it be infant baptism
or adult only baptism. I only present the following information as a proof that baptism is a covenant
sign, and not essential to salvation.
In the New Testament, circumcision is mentioned many times. But with respect to baptism it is
specifically mentioned in Col. 2:11-12: "In him you were also circumcised, in the putting off of the
sinful nature, not with a circumcision done by the hands of men but with the circumcision done by
Christ, having been buried with him in baptism and raised with him through your faith in the power of
God, who raised him from the dead," (NIV). In these verses, baptism and circumcision are related.
The extent of that relationship is still being debated. Nevertheless, Paul also says in Rom. 2:29, "But
he is a Jew who is one inwardly; and circumcision is that which is of the heart, by the Spirit, not by the
letter; and his praise is not from men, but from God." As you can see, for the Christian, circumcision
is of the heart. And because it is, we Christians are now included the Abrahamic covenant where
before, we, the Gentiles, were not. "Remember that you were at that time separate from Christ,
excluded from the commonwealth of Israel, and strangers to the covenants of promise, having no
hope and without God in the world," (Eph. 2:12, NASB).
In Gal. 3:8, Paul calls the promise of the Abrahamic covenant, the gospel. He says, "And the
Scripture, foreseeing that God would justify the Gentiles by faith, preached the gospel beforehand to
Abraham, saying, 'All the nations shall be blessed in you, 9So then those who are of faith are blessed
with Abraham, the believer.'" (Gal. 3:8-9). So, Paul calls the Abrahamic covenant, the gospel. The
sign of this Abrahamic covenant was circumcision.
Here is the catch. Since the Abrahamic covenant is still valid (we are justified by faith -- Gal. 3:8),
then is there a covenant sign for us today? I think the answer is a resounding, yes. I believe that
baptism replaces the Old Testament covenant sign of circumcision because 1) there was a New
Covenant in the communion supper (Luke 22:20), and 2) in circumcision there was the shedding of
blood, but in baptism no blood is shed. The covenant sign has changed now that the Law has been
fulfilled in Christ.
If you understand that baptism is a covenant sign, then you can see that it is a representation of
the reality of Christ circumcising our hearts (Rom. 2:29; Col. 2:11-12). It is our outward proclamation
of the inward spiritual blessing of regeneration, of "heart-circumcision." It comes after faith which is a
gift of God (Rom. 13:3) and the work of God (John 6:28). Again, baptism is the covenant sign of our
covenant with God.

Acts 2:39 and "The Promise"

This would explain why Peter in verse 39 of Acts 2 says, "For the promise is for you and your
children, and for all who are far off, as many as the Lord our God shall call to Himself." What promise
is Peter speaking of when he says "the promise"? Notice that he does not say "this promise" but "the
promise." If Peter was referring to baptism as the promise he would have said "this promise."
Instead, he used a phrase "the promise." This is significant.
The phrase "the promise" occurs in 26 Bible verses in the New Testament. It is used in reference
to several different topics.

1. The Holy Spirit, (Luke 24:49; Acts 2:33; Gal. 3:14).


2. God's promise to Abraham to multiply his descendents in Egypt, physical as well
as spiritual, (Acts 7:17; Heb. 6:13, 15, 17).
3. The promise of the Messiah, (Acts 13:32; Acts 26:6-7; Rom. 4:13,14,16; Gal.
3:17,19,22; Eph. 3:6; 2 Tim. 1:1).
4. The promise of eternal redemption (Heb. 9:15; 1 John 2:25).
5. The promise that Sarah would have a child (Rom. 4:20; Gal. 4:23).
6. The promise that through Isaac, the world would be blessed, (Rom. 9:8).
7. The promise of Jesus' return (2 Pet. 3:4).
8. The promise to kill Paul by Paul's adversaries (Acts 22:21).

But, we are most interested in its context in Acts 2 which begins with the outpouring of the Holy
Spirit (Acts 2:1-13). Peter then preaches a sermon and quotes many OT scriptures (Acts 2:14-35). In
verse 2:22, Peter specifically says, "Men of Israel, listen to these words..." Peter is speaking to the
Jews. It was to the Jews that "the promise" of the outpouring of the Spirit was given. Peter is
speaking covenant language of God as He quotes the OT. Since Peter quotes Joel 2:28-32 in Acts
2:17-18, we can easily see what Peter is talking about when speaking of "the promise" in Acts 2:39.

"And it shall be in the last days,’ God says, ‘that I will pour forth of My Spirit upon all mankind;
and your sons and your daughters shall prophesy, and your young men shall see visions, and
your old men shall dream dreams, Even upon My bondslaves, both men and women, I will in
those days pour forth of My Spirit," (Acts 2:17-18).
See also, "For I will pour out water on the thirsty land, and streams on the dry ground; I
will pour out My Spirit on your offspring, and My blessing on your descendants," (Isa. 44:3).

Peter states in Acts 2:38, "Repent, and let each of you be baptized in the name of Jesus Christ for
the forgiveness of your sins; and you shall receive the gift of the Holy Spirit." Peter is clearly speaking
of the promise of God to grant the Holy Spirit in a new and better way. But is he saying that people
become saved by baptism in water or that baptism is part of salvation? Not at all. Peter is simply
speaking covenantally about the covenant sign. Baptism! Consider this proof, from Peter, that people
are saved before baptism.

"While Peter was still speaking these words, the Holy Spirit fell upon all those who were
listening to the message. 45And all the circumcised believers who had come with Peter were
amazed, because the gift of the Holy Spirit had been poured out upon the Gentiles also. 46For
they were hearing them speaking with tongues and exalting God. Then Peter answered,
47
"Surely no one can refuse the water for these to be baptized who have received the Holy
Spirit just as we did, can he?" 48And he ordered them to be baptized in the name of Jesus
Christ. Then they asked him to stay on for a few days," (Acts 10:44-48).

Notice that Peter had been preaching the gospel and the Holy Spirit fell upon the people. In verse
45 we see that "the gift of the Holy Spirit had been poured out upon the Gentiles also." These people
were saved. The gift of the Holy Spirit was on the Gentiles and they were speaking in tongues. This is
significant because tongues is a gift given to believers, see 1 Cor. 14:1-5. Also, unbelievers don't
praise God. They can't because praise to the true God is a deep spiritual matter that is foreign to the
unsaved (1 Cor. 2:14). Therefore, the ones in Acts 10:44-48 who are speaking in tongues and praising
God are definitely saved and they are saved before they are baptized. This simply isn't an exception. It
is a reality.

Conclusion

Acts 2:38 so closely ties repentance and baptism because it is contextually covenant language and
covenant concept. It is not stating that you must be baptized in order to be saved. It is saying that
baptism is the complete and total covenantal identification with Christ in His death, burial, and
resurrection. It is not the covenant representation (baptism) of what Christ did that saves us, but the
reality of His sacrifice which we receive by faith (Rom. 5:1; Gal. 3:8). That is why we can see in Acts
10:44-48 a group of people who are saved before they are baptized.
Baptism is not what saves. It is not part of salvation. It is something someone does who is
already saved.
Various Articles
Introduction
Here we have various articles taken from the CARM website.

1. What does being made in God's image mean? p. 118


2. What is dichotomy and trichotomy? p. 118
3. What are some of the attributes of God? p. 121
4. What are some of the different kinds of angles? p. 122
5. What are some of the qualifications of an elder? p. 127
6. What are some of the physical points of suffering in crucifixion? p. 128
7. Was God ever seen in the Old Testament? p. 130
8. What is amillenialism? p. 132
Man
Man is the direct creation of God. In the Garden of Eden, God made man good. He made Man as
both male and female.

"Then God said, ‘Let us make man in our image, in our likeness, and let them rule over the fish
of the sea and the birds of the air, over the livestock, over all the earth, and over all the
creatures that move along the ground.' So God created man in his own image, in the image of
God he created him; male and female he created them" (Gen. 1:26-27; see also, 2:7,21-23).

Because Adam was made in the image of God, and we are his descendents, we are different than
the animals. Adam had the breath of life breathed into him, where the animals did not (Gen. 2:7).
Also, Adam was given dominion over the animals. Additionally, being made in the image of God
means that we have value. All people have value because of this.
Being made in God's image means that we have a soul, that we can reason, know God, worship
Him, and love Him. Animals cannot. We have moral values and abstract conceptual capability. The
human is a wonderful creation of God.
God is concerned with man. Man is the object of God's creative and redemptive work. God loves
us (John 3:16), provides for us (Matt. 5:43-47), and has provided redemption for us through His Son
(John 3:16).
However, originally Adam and Eve were pure and sinless. But, because they rebelled against God,
they became sinners. As a result their offspring inherited their sinful nature. Many Christians today
do not accept the teaching that we have sinful natures. But it is true. We are by nature sinners
worthy of damnation (Eph. 2:3). Of course, we are not as sinful as we can be, but we are all touched
by sin. Consider the following scriptures concerning human nature.

• Our hearts are deceitful and desperately sick (Jer. 17:9).


• We are full of evil (Mark 7:21-23).
• We love darkness rather than light (John 3:19).
• We are unrighteous, do not understand, do not seek for God (Rom. 3:10-12).
• We are helpless and ungodly (Rom. 5:6).
• We are dead in our trespasses and sins (Eph. 2:1).
• We are by nature children of wrath (Eph. 2:3).
• We cannot understand spiritual things (1 Cor. 2:14).

But, thanks to God and His redemption in Christ, we are new creatures (2 Cor. 5:17). Because of
the indwelling Spirit of God we are able to understand spiritual things, seek for God, and are no longer
ungodly. This is the benefit of God's redemptive work in us. So, though we were dead in our sins, we
are alive in Christ. Though we did not want God, now we do.

Dichotomy or Trichotomy

Are we made of two parts (body and soul) or three parts (body, soul, and spirit)? Theologians
have debated the issue for centuries and there has never been a decisive orthodox declaration of
which is true.
Dichotomy is a term which signifies a division into two parts: Body and Soul. The words "spirit"
and "soul" are often used interchangeably and therefore, the dichotomous position holds that man is
comprised of two parts. Note the following verses used to support this position.

• "Mary said: ‘My soul glorifies the Lord and my spirit rejoices in God my Savior,'" (Luke 1:46-
47).
• "My soul yearns for you in the night; in the morning my spirit longs for you...," (Isaiah 26:9).
• For the term "Body and Soul" see Matt. 6:25; 10:28.
• For the term "Body and Spirit" see 1 Cor. 5:3,5.
Trichotomy is a term which signifies a division into 3 parts: Body, Soul, and Spirit. With the
following verses, "spirit" and "soul" seem to be different.

• "May God himself, the God of peace, sanctify you through and through. May your whole spirit,
soul and body be kept blameless at the coming of our Lord Jesus Christ," (1 Thess. 5:23).
• "For the word of God is living and active. Sharper than any double-edged sword, it penetrates
even to dividing soul and spirit, joints and marrow; it judges the thoughts and attitudes of the
heart," (Heb. 4:12).

Does it matter if you believe in dichotomy or trichotomy? No. However, a word of caution. There
are churches that teach it is possible for Christians to be demon possessed. These groups hold the
trichotomous position. They maintain that it is possible for one part of a person to be possessed but
not the other. For example, they might claim that the spirit of a person can be possessed but the soul
cannot. Others reverse it and state that the soul of a person can be possessed but not the spirit. This
is problematic because there are no accounts in Scripture of Christians having demons cast out of
them. Besides, how can a person indwelt by the Holy Spirit also be indwelt by a demon?
Nevertheless, this does not mean that the trichotomous position is wrong. There are many great
scholars on both sides of the issue in spite of the error of Christian demon possession.

In conclusion, it is a wondrous thing that God would create a universe, populate it with people, and
then love us so much that He would die to save us and bring us to Himself. But that is the great God
we serve and love.
Let Us make man in our image

There are several verses in the Old Testament where God speaks as a plurality. Many Trinitarians
quote these verses to help support the Trinity doctrine because they strongly suggest that there is
more than one person in the godhead.

• "Then God said, “Let Us make man in Our image, according to Our likeness; and let them rule
over the fish of the sea and over the birds of the sky and over the cattle and over all the earth,
and over every creeping thing that creeps on the earth,” (Gen. 1:26, NASB).
• "Then the Lord God said, “Behold, the man has become like one of Us, knowing good and evil;
and now, lest he stretch out his hand, and take also from the tree of life, and eat, and live
forever," (Gen. 3:22, NASB).
• “Come, let Us go down and there confuse their language, that they may not understand one
another’s speech,” (Gen. 11:7, NASB).
• "Then I heard the voice of the Lord, saying, “Whom shall I send, and who will go for Us?” Then
I said, “Here am I. Send me!” (Isaiah 6:8, NASB)

Those opposed to the doctrine of the Trinity say that God is speaking of Himself in any "royal" sense,
in a "plural of majesty." They can say this, but biblically there is never any account of a king or a ruler
speaking of himself in a plural sense or in the third person. So, there is no biblical support for God
using it of Himself in this way.
In regards to Gen. 1:26, those who deny the Trinity say that God when God says, "Let Us make..."
He is speaking with the angels in mind. The problem with this is that angels do not create. There is
absolutely no biblical evidence that angels created anything at all. We see in Isaiah 44:24, "Thus says
the Lord, your Redeemer, and the one who formed you from the womb, “I, the Lord, am the maker of
all things, Stretching out the heavens by Myself, and spreading out the earth all alone." God made all
things alone. Therefore, the "us" in "Let Us make man in our image" cannot be the angels.
Furthermore, people are not created in the image of angles, but of God.
The three verses in Genesis do not prove that the Trinity is true. However, they cannot be
dismissed by the assumption that God is speaking of himself in a type of third person way.
Furthermore, notice in the force verse above, Isaiah 6:8, that's God is speaking in the singular and
then switches to the plural. He says, "Whom shall I send, and who will go for Us?" This is on the
unusual construction. The singular speaker refers to himself in the plural.
God
God, according to the Bible, is the only divine and supreme being that exists in the universe. The
Bible says He is Holy (Rev. 4:8), eternal (Isaiah 57:15, omnipotent (Jer. 32:17,27), omnipresent
(Psalm 139:7-12), and omniscient (1 John 3:20). He is described as love (1 John 4:8,16); light
(1 John 1:5); spirit (John 4:24); truth (Psalm 117:2); and as creator (Isaiah 40:12,22,26). He is
worshiped (Gen. 24:26; Ex. 4:31; 2 Chron. 29:28; 1 Cor. 14:25; Rev. 7:11) and served (Matt. 4:10;1
Cor. 6:19; Phil. 3:7; 1 Thess. 1:9; Heb. 9:14).
Though there are many other concepts of God in the world, the Bible alone presents God as the
only God in existence, the Supreme Being who is a Trinity. The Trinity is the doctrine of one God in
three persons, not three gods, not three modes. Pantheism presents God as an impersonal presence
existing in all things, containing all things, and part of all things. Atheism says there is no God. Deism
says God exists but is unknowable and untouchable.
The Bible speaks of God as knowable, personal, real, alive, self-aware, and very much concerned
with the affairs of this world. It does not defend God's existence nor try to prove it. Instead, the Bible
simply assumes it and says that it is the fool who says there is no God (Psalm 14:1).
Some of the biblical names of God are "Jehovah or Yahweh,” (Exodus 3:14, lit. "I AM"), "Elohim"
(the Hebrew word for 'god', i.e. Gen. 1), and "Adonai" which means "master" or "lord." "Jehovah" is
often joined in usage to the word "elohim" as in Gen. 15:2. It is also joined to other words. For
example, Jehovah-jireh which means "the Lord will provide,” (Gen. 22:13-14); Jehovah-rapha which
means "The Lord who heals,” (Exodus 15:26); Jehovah-shalom which means "the Lord our peace,”
(Judges 6:24); and Jehovah-tsidkenu, which is "the Lord our righteousness,” (Jer. 23:6). In the NT,
God is known as the "Father" (John 17) and the "Word" (John 1:1,14), etc. 16
God is Sovereign. He is in control of all things and supreme over all things. No one is more
powerful than Him and He answers to no one. He had no beginning and has no end (Psalm 90:2).
God decrees what will occur; that is, all things occur according to His plan (Acts 2:23. But this is not
to say that God is the author of evil. Instead, in His sovereign plan, He allowed for it to occur.
God is the creator of the Universe (Gen. 1; Isaiah 44:24; John 1:1-3; Col. 1:16-17), not a part of
it. God is the redeemer who became a man in Christ Jesus (John 1:1,14; Col. 2:9; Phil. 2:5-8; Titus
2:13). God is our Savior.
God is forgiving (Eph. 1:7; Psalm 86:5), merciful (Exodus 34:6; Psalm 67:1; James 5:11), holy
(Isaiah 6:3; Rev. 4:8), eternal (Psalm 90:2; 1 Tim. 1:17), and perfect (1 Kings 8:27; Psalm 139).

The Incarnation

In the incarnation, God became man. It says in Col. 2:9, "For in Him all the fullness of deity dwells
in bodily form" (NASB). Jesus is God in flesh, our redeemer. It was necessary that God become a
man to atone for our sins because a sinful man could not fulfill all the Law of God and offer a sacrifice
to God the Father of sufficient quality to remove our sin. God had to become a man so that He could
die for the sins of men. In this, we have the sacrifice of a loving God who humbled Himself to become
one of us and die at our hands for our sins so that He might give eternal life to those who accept His
sacrifice (John 1:12; John 3:16).

16
Chafer, Lewis Sperry, Major Bible Themes, rev. Walvoord, John F., Grand Rapids,
Angels
Angels are very active in the Bible and are used by God as messengers, warriors, and servants.
The word "angel" comes from the Greek word "angelos" which means messenger. Angels are spiritual
beings without bodies of flesh and bones, though they apparently have the ability to appear in human
form (Gen. 19:1-22). Angels had many functions. They praised God (Psalm 103:20), served as
messengers to the world (Luke 1:11-20, 26-38; Luke 2:9-14), watched over God’s people (Psalm
91:11-12), and were sometimes instruments of God’s judgment (Matt. 13:49-50).
The Bible tells us that God created the angels and that at some time in the distant past there was a
rebellion in heaven and many of the angels fell. Apparently, it was the elect angels that did not fall (1
Tim. 5:21). The Bible says that angels were created by Christ (Col. 1:16), that they carry out the will
of God (Psalm 103:20; Matt. 6:10), they worship God and Christ (Phil. 2:9-11; Heb. 1:6), are wise (2
Sam. 14:20), mighty (Psalm 103:20), holy (Matt. 25:31), and innumerable, (Heb. 12:22). However,
angels are not to be worshipped (Col. 2:18; Rev. 19:10; 22:9) since they are creatures.

Are there different kinds of angels?

Apparently, there are different kinds of angels with different characteristics and roles: cherubim,
seraphim, and archangels. It may also be that there are "powers" and "principalities" that further
describe ranks in the angelic realm, but it is debated. Nevertheless, I'll focus on the three main
groups.

1. "Seraphim stood above Him, each having six wings; with two he covered his face, and with two
he covered his feet, and with two he flew,” (Isaiah 6:2).
A. They praise God (Isaiah 6:3).

B. The word "seraphim" (singular is saraph) probably a translation of ‘fiery ones’ and probably
stems from the fiery imagery often associated with the Presence of God (cf. Ezek. 1:27). 17

2. "So he drove out the man; and he placed at the east of the garden of Eden Cherubim, and a
flaming sword which turned every way, to keep the way of the tree of life,” (Gen. 3:24). See also
Exodus 25:18-22; Heb. 9:5.
A. Cherubim are typically represented with wings, feet, and hands, but are described in
different forms as having two faces (Ezek. 41:18) and even four faces (Ezek. 10:21).
B. Cherubim were considered to be angels that guarded sacred things. In Gen. 3:24 they
guarded the tree of life. They were over the Ark of the Covenant on the Mercy Seat (1 Sam.
4:4). See also Psalm 80:1; 99:1
C. Figures of Cherubs were embroidered on the temple veil (Exodus 26:31; 2 Chron. 3:7) and
lavished Solomon's temple (1 Kings 6:26ff).
3. "For the Lord himself shall descend from heaven with a shout, with the voice of the archangel,
and with the trump of God: and the dead in Christ shall rise first,” ( 1 Thess. 4:16).
A. The word "archangel" is not found in the Old Testament. References to Michael archangel
appear only in 1 Thess. 4:16 and Jude 9. However, Gabriel, who is considered an archangel
appears in both the OT and NT. In the OT he is found in Dan. 8:15-26 and 9:21-27. In the
NT he is mentioned in Luke 1:11-20, 26-38. He seems to be a messenger angel.
B. On the other hand, Michael the archangel seems to be a warrior angel (Rev. 12:7) who does
battle (Dan. 10:13, 21; 12:1).
C. An interesting note is that in Rom. 8:38, Eph. 1:21, and Col. 1:16, the word ‘principalities’ is
used. In Greek the word has the prefix of "arche" suggesting archangel. Some think this
means there is a hierarchy of angels as is suggested in 1 Pet. 3:22: "who is at the right
hand of God, having gone into heaven, after angels and authorities and powers had been
subjected to Him" (NASB).

17
Achtemeier, Paul J., Th.D., Harper’s Bible Dictionary, (San Francisco: Harper and Row, Publishers, Inc.) 1985.
What does the Bible say about fallen angels?

Of course, there are fallen angels as well. Lucifer, another archangel, rebelled against God and
became the devil. Following are verses often quoted in reference to the evil one.

"How you have fallen from heaven, O star of the morning, son of the dawn! You have been cut
down to the earth, you who have weakened the nations! 13 "But you said in your heart, ‘I will
ascend to heaven; I will raise my throne above the stars of God, and I will sit on the mount of
assembly In the recesses of the north. 14 ‘I will ascend above the heights of the clouds; I will
make myself like the Most High,’" (Isaiah 14:12-14).

Most scholars agree that one third of the angels fell into sin and became demons.

"And another sign appeared in heaven: and behold, a great red dragon having seven heads and
ten horns, and on his heads were seven diadems. 4 And his tail swept away a third of the stars
of heaven, and threw them to the earth . . . " (Rev. 12:3-4).

In the future, there will be a judgment upon the fallen angels:

• "Then shall he say also unto them on the left hand, Depart from me, ye cursed, into everlasting
fire, prepared for the devil and his angels," (Matt. 25:41).
• "For if God spared not the angels that sinned, but cast them down to hell, and delivered them
into chains of darkness, to be reserved unto judgment," (2 Pet. 2:4).
• "And the angels which kept not their first estate, but left their own habitation, he hath reserved
in everlasting chains under darkness unto the judgment of the great day," (Jude 6).
• "And the great dragon was cast out, that old serpent, called the Devil, and Satan, which
deceiveth the whole world: he was cast out into the earth, and his angels were cast out with
him," (Rev. 12:9).

Whichever view you have of angels, it cannot be escaped that the Bible mentions them a lot and
that they are greatly used by God to accomplish His will.
High Priest
The following pictures were scanned in from a book printed in 1722. The book is about
German-Jewish history and contains many drawings. The following three drawings were
made prior to 1722 and used in the book. I have reproduced them here for your benefit.

Once a year, the High Priest would


enter the Holy of Holies in the
Tabernacle (later the Temple) where he
would offer sacrifices for the nation of
Israel (Exodus 30:10; Lev. 16; Num.
18:2,5,7; Heb. 8:3; 9:7). He would
sprinkle blood on the Mercy Seat which
was actually a lid on top of the Ark of
the Covenant.
His garments were specially made
(Exodus 28). Notice the Breast Plate of
stones on his chest (Exodus 28:15-29).
Each stone was named after a tribe of
Israel.
Pharisee

The Pharisees were influential religious leaders in Palestine around the time of Christ. They
became prominent around 200 BC and were still around through the first century. They were very
pious, held to the Law, and were ritually pure.
The NT accounts of the Pharisees represent the worst of the group. Generally, they were good
people, God-fearing, and sought to honor the Lord. But as with so many things, purity is
sacrificed to power. The Pharisees became powerful in Israel and were challenged by Jesus claims
and miracles.
"According to Josephus: Josephus, a Jewish historian of the first century who wrote for non-
Jews in Greek, calls the Pharisees a ‘choice [of life]’ and a ‘philosophy.’ . . . .According to Josephus,
the Pharisees were the group most influential with the people, were noted for their accurate and
therefore authoritative interpretations of Jewish law, and had their own traditions and way of life to
which they were faithful. They had a simple standard of living and cultivated harmonious relations
with others. . . .Some Pharisees incited opposition to the government, though others worked with
the chief priests to keep order. In the first century Josephus says they numbered six thousand."

--------------
References: Achtemeier, Paul J., Th.D., Harper’s Bible Dictionary, (San Francisco: Harper and Row,
Publishers, Inc.) 1985.
Moloch

Moloch was one of the false gods that Israel would worship during its periods of apostasy.
This false deity is associated with Ammon in 1 Kings 11:7, "Then Solomon built a high place for
Chemosh the detestable idol of Moab, on the mountain which is east of Jerusalem, and for Molech
the detestable idol of the sons of Ammon."
One of the practices of the cult that worshipped Moloch was to sacrifice their children. Of
course, this was forbidden by God's word: Lev. 18:21 says, "Neither shall you give any of your
offspring to offer them to Molech, nor shall you profane the name of your God; I am the Lord."
(See also Lev. 18:21; 20:2-5; 2 Kings. 23:10; Jer. 32:35).
In some passages the reference is clearly to a deity to whom human sacrifice was made,
particularly in the Valley of Hinnom on the SW of the Jerusalem hill (2 Kings 23:10; Jer. 32:35) at
a site known as Topheth (‘fire pit’ in Syriac).
The ancients would heat this idol up with fire until it was glowing, then they would take their
newborn babies, place them on the arms of the idol, and watch them burn to death. I can't help
but compare today's abortion massacre to the sacrifice of children by these ancient pagans. In
both, innocent life is destroyed for the gain of the parent.

References: The New Bible Dictionary, (Wheaton, Illinois: Tyndale House Publishers, Inc.) 1962.
The Elder in the Church
1. The Term "Elder"
A. PRESBUTEROS - elder, an old man, a leader in the church. The term is used
i. of the elder of two persons (Luke 15:25, or more, John 8:9).
ii. of a person advanced in age (Acts 2:17; in Heb. 11:2).
iii. of the forefathers in Israel (Matt. 15:2; Mark 7:3, 5).
iv. of members of the Sanhedrin (Matt. 16:21; 26:47).
v. of those who managed public affairs in the various cities (Luke 7:3).
vi. of those who were the heads or leaders of the tribes and families, as of the seventy who
helped Moses (Num. 11:16; Deut. 27:1). This included:
a. acting as judges in apprehending murderers (Dt. 19:12).
b. conducting inquests (Dt. 21:2).
c. settling matrimonial disputes (Dt. 22:15; 25:7).
d. If theirs was a city of refuge they also heard pleas for asylum (Jos. 20:4).
vii. of those qualified by the Holy Spirit who exercised spiritual care and oversight of the
local congregation.
B. EPISKOPOI - overseers, bishops
i. Titus equates bishop and elder in verses 1:5-9.
2. Regarding the Office:
A. The office of Elder is a divinely appointed office as defined in the Pastoral Epistles.
B. Elders are apparently appointed by the laying on of hands (1 Tim. 4:14; 2 Tim. 1:6).
C. Should receive double honor in the church (1 Tim. 5:17).
D. The pastor is an elder who preaches and/or teaches (1 Tim. 5:17).
i. The pastor (elder) is to equip the body of Christ (Eph. 4:11-13).
E. Must be a man (1 Tim. 2:9-13).
i. All uses of "elder" are in the masculine except for 1 Tim. 5:2 where it means older
women.
3. The Responsibilities of Elders in the NT Church:
A. Must shepherd the flock (1 Pet. 5:2).
B. Must voluntarily exercise oversight upon the flock (1 Pet. 5:2).
C. Must live as examples to the flock (1 Pet. 5:3).
D. Anoint and pray for the sick (James 5:14).
E. They have the tasks of teaching (1 Tim. 5:17; Tit. 1:5, 9).
F. They have the tasks of acting as judges (Acts 15:2, 6, 22-29; 16:4).
4. Qualifications for an elder
A. Must be above reproach (Titus 1:6; 1 Tim. 3:2)
B. Husband of one wife (Titus 1:6; 1 Tim. 3:2).
C. Household must be in order with children who believe (Titus 1:6; 1 Tim. 3:4).
D. Not a new convert (1 Tim. 3:6).
E. Self controlled and temperate (Titus 1:7; 1 Tim. 3:2).
F. Honorable, hospitable, seeking good (Titus 1:8).
G. Have a good reputation (1 Tim. 3:7).
H. Not addicted to wine (1 Tim. 3:3).
I. Not greedy (1 Tim. 3:3).
J. Able to exhort (teach) sound doctrine (Titus 1:9; 1 Tim. 3:2).
K. Able to refute false teaching (Titus 1:9).
L. They must be ready to earn their own living if necessary (Acts 20:17, 33-35).
The Crucifixion of Jesus
The crucifixion of Christ is recorded in all four gospels: Matthew 27:33-44; Mark 15:22-32; Luke
23:33-43; John 19:17-30.
Crucifixion is the process where a person is nailed or bound to a cross or a stake. It was first used
by the Persians and later by the Egyptians, Carthaginians, and Romans as a form of capital
punishment. Alexander the Great introduced it to the Mediterranean area and the Romans perfected it
as a means of capital punishment.
Normally, there was a permanent stake in the ground. The victim carried the crossbar on his back
to the stake which usually weighed between 50 and 75 lbs. Sometimes the person was nailed to the
crossbar, other times he was tied to it. The crossbar and victim were then hoisted into place. One
method was to hoist the crossbar into a notch on top of the stake so the whole thing looked like a T.
Another method was to place the crossbeam a few feet below the top making a cross. Yet another
method was to nail or tie the person to a single stake in the ground. Usually a small sign on a pole
with the crime written on it was carried ahead of the victim in front of the procession to the cross. It
was then nailed to the cross above the head of the victim.
When nails were used, they were driven through the wrists between the radial and ulna bones and
not through the palms since the nail would have ripped through the palm because the palm could not
withstand all the weight of the body.

The Physical aspect of suffering in the crucifixion

Jesus agony began in Gethsemane with the sweating of blood. Hematidrosis is the name given
to the rare occurrence of tiny blood capillaries in the sweat glands that rupture causing an oozing of
blood to occur through the skin.

"And being in agony He was praying very fervently; and His sweat became like drops of blood,
falling down upon the ground," (Luke 22:44).

Next, Jesus was arrested in the Garden of Gethsemane at night. He was brought before the
Sanhedrin and there struck by a soldier when Jesus questioned the High Priest.

"And when He had said this, one of the officers standing by gave Jesus a blow, saying, "Is that
the way You answer the high priest?" 23Jesus answered him, "If I have spoken wrongly, bear
witness of the wrong; but if rightly, why do you strike Me?" (John 18:22-23).

Jesus was then blindfolded and struck in the face repeatedly. Being blindfolded meant he couldn't
"roll with the punches" and the blows would have been that much more destructive. The Bible says
that He was beaten so badly He could hardly be recognized.

"And some began to spit at Him, and to blindfold Him, and to beat Him with their fists, and to
say to Him, "Prophesy!" And the officers received Him with slaps in the face," (Mark 14:65).
". . . So His appearance was marred more than any man, and His form more than the sons
of men," (Isaiah 53:14).
"I gave My back to those who strike Me, and My cheeks to those who pluck out the beard; I
did not cover My face from humiliation and spitting," (Isaiah 50:6).

Next, Jesus was stripped of His clothing and then scourged. In scourging, a soldier used a whip
called a flagrum consisting of leather straps embedded with metal and glass fragments with small
metal balls sewn into the end of each thong. This whip was brought down with full force and when
struck against the back of Jesus, was pulled thus tearing the skin off, exposing muscle, and maybe
even exposing His very bones. Undoubtedly, His back was reduced to an oozing mass of mutilated
flesh. Scourging stops when it is determined that the victim is near death or 39 lashes was reached.
39 was the number of mercy according to Jewish law. By this time, Jesus was in great pain, suffering
severe blood loss, and was becoming very weak and thirsty. Only after this was He taken to be
crucified.
"Then he released Barabbas for them; but after having Jesus scourged, he delivered Him to be
crucified," (Matt. 27:26).

They then stripped Him, put a scarlet robe on Him and placed a crown of thorns on His head. The
robe would stick to the congealing blood on His back and when they ripped it from Him later, it would
have been very painful and would have helped to continue the bleeding even more. They put a crown
of thorns on His head. These thorns were shoved between His scalp and skull as well as ripping and
tearing at the skin. Severe bleeding would ensue along with great pain.

"And they spat on Him, and took the reed and began to beat Him on the head. 31And after they
had mocked Him, they took His robe off and put His garments on Him, and led Him away to
crucify Him," (Matt. 27:30).
"And after weaving a crown of thorns, they put it on His head, and a reed in His right hand;
and they kneeled down before Him and mocked Him, saying, "Hail, King of the Jews!" (Matt.
27:29).

Then Jesus was mocked and beaten another time after the scourging. He would be very weak by
this time and probably could not bear the weight of the cross. So, another person was drafted to carry
the cross for Him.

"And as they were coming out, they found a man of Cyrene named Simon, whom they pressed
into service to bear His cross," (Matt. 27:32).

Jesus was lead away to the cross and finally, He was nailed to a cross-beam. Normally a person
was laid down upon the cross beam and a nail driven into one wrist. Then the other hand was pulled
very tightly and another nail driven into the other wrist. The nails were usually about 6-8 inches long.
Placing the nail in the wrist severs the median nerve resulting in a burning pain as well as paralysis in
the hand.
Once Jesus was lifted to the cross, His feet were nailed to it. But, in order to do this, His knees
were bent and the feet brought up a bit to allow them to lie flat against the stake so they could be
nailed to it. Once suspended, the force of gravity brings the weight of the body down and the
shoulders and elbows dislocate by popping out of joint, ripping ligaments.
Because of the outstretched position of the arms, the chest cavity is in a perpetually expanded
state and it is very difficult to breathe. With the severe loss of blood from the scourging and
crucifixion, Jesus would have become dehydrated and His body would have less blood to carry oxygen.
Therefore, His heart would beat faster as it attempted to compensate and His need for oxygen would
increase greatly. In order to breath, Jesus had to push up on the nails in His feet to allow His chest
enough flexibility to inhale. Pushing up on the nails is not only excruciating, but this meant that He had
to scrap His raw, beaten back against the rough wooden stake. This whole process of breathing and
exhaling by pushing up on the nails only increases in intensity as time passes. Soon, the body gets to
the point of no return and the heart either ruptures or the person dies from asphyxiation. But, before
that happened, the blood loss results in extreme thirst as the body craves water to restore the lost
blood. Jesus said, "I thirst,” (John 19:28), whereupon a soldier offered Him some sour wine (Luke
23:36), but Jesus refused it -- because He would not seek to escape any of the ordeal.
In order to prolong the suffering, sometimes, the Romans would place a very small seat nailed to
the stake so that the victim could partially sit on it. This would allow a small amount of rest and would
greatly increase the time it took to die, sometimes several days. But in Jesus’ case, since He was so
badly beaten before He got to the cross, He died in a short amount of time. It was, therefore, not
necessary to break His legs to prevent Him from pushing up on the nails in His feet so He could breath.
Jesus died a horrible death.

As I read this account, I am stricken by the greatness of His sacrifice and very thankful that He
loves us enough to die for us. He deserves all the glory.
The Plurality Study
The following study is an interesting examination of theophanies. A theophany is an appearance of
God. God appears in the Old Testament in different ways: as an angel of the Lord (Acts 7:30-32; Ex.
3:2; Judges 2:1), apparently in physical form (Gen. 3:8; Ex. 24:9-11), in visions and dreams (Num.
12:6-8), and in flame (Judges 13:20-21). However, there are verses that say that you can't see God:
Exodus 33:20; John 1:18). If this is so, then is there a contradiction in the Bible? No, there isn't.
Study the following verses, read them in context in the Bible, and see if you can figure out what is
going on. If you can't, continue reading and you'll be pleasantly surprised.
These verses are taken from the New American Standard Bible.

(Note: "LORD" is equivalent to YHWH, or Yahweh, or Jehovah, which is the name of God.)

Plurality of God:

1. Gen. 1:26, "Then God said, "Let Us make man in Our image, according to Our likeness . . ."
2. Gen. 19:24, "Then the LORD rained on Sodom and Gomorrah brimstone and fire from the LORD
out of heaven."
3. Amos 4:10-11, "‘I sent a plague among you after the manner of Egypt; I slew your young men
by the sword along with your captured horses, and I made the stench of your camp rise up in
your nostrils; yet you have not returned to Me,' declares the LORD. ‘I overthrew you as God
overthrew Sodom and Gomorrah . . . '"
4. Is. 44:6, "Thus says the LORD, the King of Israel and his Redeemer, the LORD of hosts: ‘I am
the first and I am the last, and there is no God besides me . . . ‘" See also, Isaiah 48:16.

Appearances of God

1. Gen. 17:1, "Now when Abram was ninety-nine years old, the LORD appeared to Abram and
said to him, "I am God Almighty; walk before Me, and be blameless."
2. Gen. 18:1, "Now the LORD appeared to him by the oaks of Mamre, while he was sitting at the
tent door in the heat of the day."
3. Exodus 6:2-3, "God spoke further to Moses and said to him, ‘I am the LORD; and I appeared to
Abraham, Isaac, and Jacob, as God Almighty, but by My name LORD I did not make myself
known to them.'"
4. Exodus 24:9-11, "Then Moses went up with Aaron, Nadab and Abihu, and seventy of the elders
of Israel, and they saw the God of Israel; and under His feet there appeared to be a pavement
of sapphire, as clear as the sky itself. Yet He did not stretch out His hand against the nobles of
the sons of Israel; and they beheld God, and they ate and drank."
5. Exodus 33:11, "Thus the LORD used to speak to Moses face to face, just as a man speaks to
his friend..."
6. Num. 12:6-8, "He [God] said, "Hear now My words: If there is a prophet among you, I, the
LORD, shall make Myself known to him in a vision. I shall speak with him in a dream. Not so,
with My servant Moses, He is faithful in all My household; with him I speak mouth to mouth,
even openly, and not in dark sayings, and he beholds the form of the LORD . . . "
7. Acts 7:2, "And he [Stephen] said, "Hear me, brethren and fathers! The God of glory appeared
to our father Abraham when he was in Mesopotamia, before he lived in Haran. . . "
Can't see God:

1. Ex. 33:20, "But He [God] said, ‘You cannot see My face, for no man can see Me and live!'"
2. John 1:18, "No man has seen God at any time; the only begotten God, who is in the bosom of
the Father; He has explained Him."
3. 1 Tim. 6:16 "[God] who alone possesses immortality and dwells in unapproachable light; whom
no man has seen or can see."
4. John 6:46, "Not that any man has seen the Father except the One who is from God; He has
seen the Father."

The Solution

1. John 8:58, "Jesus said to them, "Truly, truly, I say to you, before Abraham was born, I am."
2. Exodus 3:14, "And God said to Moses, ‘I AM WHO I AM'; and He said, ‘Thus you shall say to
the sons of Israel, ‘I AM has sent me to you.'"
3. Zech. 12:10, "And I [God] will pour out on the house of David and on the inhabitants of
Jerusalem, the Spirit of grace and of supplication, so that they will look on Me whom they
have pierced; and they will mourn for Him, as one mourns for an only son, . . . "

It is evident above that God was seen. But, considering the "Can't-see-God" verses, some would
understandably argue that people have not seen God; otherwise, there would be a contradiction in the
Bible. A possible explanation for this is that people were seeing visions, or dreams, or the Angel of the
LORD (Num. 22:22-26; Judges 13:1-21). But the problem is that the verses cited above do not say
vision, dream, or Angel of the LORD. They say that people saw God (Exodus 24:9-11), that God was
seen, and that He appeared as God Almighty (Exodus 6:2-3).
At first, this is difficult to understand. God Almighty was seen (Exodus 6:2-3) which means it was
not the Angel of the Lord, for an angel is not God Almighty, and at least Moses saw God, not in a
vision or dream, as the LORD Himself attests in Num. 12:6-8. If these verses mean what they say,
then we naturally assume we have a contradiction. Actually, the contradiction exists in our
understanding, not in the Bible--which is always the case with alleged biblical contradictions.
The solution is simple. All you need to do is accept what the Bible says. If the people of the OT
were seeing God, the Almighty God, and Jesus said that no one has ever seen the Father (John 6:46),
then they were seeing God Almighty, but not the Father. It was someone else in the Godhead. I
suggest that they were seeing the Word before He became incarnate. In other words, they were seeing
Jesus; compare John 8:58 with Exodus 3:14 above.
If God is a Trinity, then John 1:18 is not a problem either because in John chapter one, John
writes about the Word (Jesus) and God (the Father). In verse 14 it says the Word became flesh. In
verse 18 it says no one has seen God. Since Jesus is the Word, God then, refers to the Father, and the
apparent contradiction is easily resolved, especially when this is examined in the light of Jesus' words
in John 6:46 where He said that no one has ever seen the Father. Therefore, Almighty God was seen,
but not the Father. It was Jesus before His incarnation. There is more than one person in the Godhead
and the doctrine of the Trinity must be true.
This is an interesting study to present to Jehovah's Witnesses. Since they deny the Trinity, they
have to do a lot of fancy talking to explain away the theophanies. I've never yet met a J.W. who could
adequately explain these verses.
Amillennialism and Premillennialism

The millennium is the period of time that Jesus reigns as King. There is debate as to the nature of
the millennium. Is it a literal 1000 years or is it a figurative length of time? Below is a chart that
simply lays out the two dominant positions: premillennialism and amillennialism.
Premillennialism is the teaching concerning the end times (eschatology). It says that there is a
future millennium (1000 years as mentioned in Revelation 20) where Christ will rule and reign over the
earth. At the beginning of the millennium Satan and his angels will be bound and peace will exist on
the entire earth. At the end of the 1000 years Satan will be released in order to raise an army against
Jesus. Jesus will destroy them and then the final judgment will take place with the new heavens and
the new earth being made.
Amillennialism is the teaching that there is no literal 1000 year reign of Christ as referenced in
Revelation 20. It sees the 1000 year period spoken of in Revelation 20 as figurative. Instead, it
teaches that we are in the millennium now, and that at the return of Christ ( 1 Thess. 4:16 - 5:2) there
will be the final judgment and the heavens and the earth will then be destroyed and remade ( 2 Pet.
3:10).
Col. 2:9 and Eph. 3:19
• "For in Him all the fulness of Deity dwells in bodily form," (Col. 2:9, NASB).
• "and to know the love of Christ which surpasses knowledge, that you may be filled up to
all the fulness of God," (Eph. 3:19, NASB).

Colossains 2:9 is often used by Trinitarians to support the doctrine that Jesus is God in flesh. It
clearly states that in Jesus, deity dwells. What is interesting about this verse is that it contains a word
used only once in the entire Bible: "deity." Deity means, "The essential nature or condition of being a
god; divinity." This verse states that in Jesus dwells all the fullness of God, or all the fullness of the
condition of being divine.
However, critics of the doctrine of Jesus' deity will go to Eph. 3:19 which says something similar
but not identical to Col. 2:9. It says ". . . that you [the Christians] may be filled up to all the fullness
of God." These verses are similar and opponents of the Trinity will attempt to use Ephesians to deny
that Colossians 2:9 states that Jesus is God. They rightly observe that Eph. 3:19 says that Christians
are filled with the fullness of God. They then reason that if we are filled with the fullness of God and
we are not divine, then when Col. 2:9 says that in Jesus dwells all the fullness of deity, then neither is
He divine.
One of the mistakes in the assumption that Eph. 3:19 interprets or clarifies Col. 2:9 is the failure to
read the verses in context. After all, the verses in different books. Without looking at the context of
both, it isn't proper to simply quote the two in juxtaposition and make a pronouncement that Col. 2:9
does not mean Jesus is God because of a slightly similar usage of words in Eph. 3:19. Therefore, let's
look at the context of each.

• Colossians 2
 v. 2 true knowledge of God’s mystery, that is, Christ Himself
 v. 4, that no one deceive you with persuasive arguments
 v. 6, walk in Christ
 v. 7, rooted and built up in Jesus
 v. 8, see to it that no one takes you captive through philosophy.
 v. 9, for in Jesus dwells all the fullness of deity in bodily form.
• Ephesians 3
 v. 1, Paul a prisoner of Christ.
 v. 3, By revelation, Paul received knowledge of 'the mystery.'
 v. 6, that the Gentiles are fellow heirs and fellow members of the body
 v. 10, in order that the manifold wisdom of God might now be made known
 v. 14, Paul prays for the Ephesians
 v. 16, that God would strengthen them with power through the Holy Spirit.
 v. 17, that Christ may dwell in your hearts by faith, being grounded in love
 v. 18, that they may comprehend with all the saints
 v. 19, and to know the love of Christ which surpasses knowledge, that you may be filled
up to all the fullness of God.

As you can see, the context of Col. 2 is a warning against deception and a proclamation that in
Jesus dwells all the fullness of deity. It is in Christ, in whom dwells deity, that we have protection
from deception. He is our safety. In Eph. 3, the context is that the Gentiles are included in the saving
plan of God, that Paul wishes that the Ephesians would be strengthened with power, that Christ would
dwell in their hearts, and that they would be filled up to all the fullness of God.
Obviously, the contexts are different and because they are, the phrases must be interpreted in light
of them. In Col. 2:9, Jesus is the guard against deception. In Eph. 3:19, being filled with the fullness
of God is contextually speaking of the indwelling of the Holy Spirit (v. 16) and the indwelling of Christ
(v. 17). It is in this context of indwelling that the statement is made about Christians being filled with
the fullness of God. It is not saying that they are divine. Rather, it is saying that they are indwelt by
God as is consistent with other scriptures.
• John 14:23, "Jesus answered and said to him, "If anyone loves Me, he will keep My word; and
My Father will love him, and We will come to him, and make Our abode with him."
• Rom. 8:9, "However, you are not in the flesh but in the Spirit, if indeed the Spirit of God dwells
in you. But if anyone does not have the Spirit of Christ, he does not belong to Him. 10And if
Christ is in you, though the body is dead because of sin, yet the spirit is alive because of
righteousness. 11But if the Spirit of Him who raised Jesus from the dead dwells in you, He who
raised Christ Jesus from the dead will also give life to your mortal bodies through His Spirit who
indwells you."

Col. 2 is a different context and is dealing with a different issue than Eph 3. Though Col. 2:9 and
Eph. 3:19 use similar phrases, they are not identical. Furthermore, Col. 2:9 contains the word "deity,"
"theotas," which only occurs in the entire Bible once, in reference to Jesus. In Col. 2:9 it states that in
Jesus dwells the fullness of deity in bodily form. In Eph. 3:19 it says that Christians may be filled up
to all the fullness of God. Likewise, the phrase "fullness of God" only occurs once in the entire Bible,
right here in Eph. 3:19 and it is dealing with believers being indwelt by God through the Holy Spirit
(v.16). So, Eph. 3:19 cannot be used to state that Col. 2:9 does not teach Jesus is God in flesh.
Furthermore, I cannot help but remember the words found in John 1:1 and verse 14 when thinking
of Col. 2:9

• John 1:1, "In the beginning was the Word, and the Word was with God, and the Word was
God."
• John 1:14, "And the Word became flesh, and dwelt among us, and we beheld His glory, glory as
of the only begotten from the Father, full of grace and truth."

A final note: In John 14:23 and Rom. 8:9, we see the indwelling of God as a Trinity. The Father,
the Son, and the Holy Spirit are all said to dwell in the believer. This is the mystery and the wonder of
God as a Trinity.
Col. 2:9 shows us that in Jesus dwells God, bodily. Eph. 3:19 shows us that Christians are indwelt
by God through the Holy Spirit. They are different.

Jesus said to them, "Truly, truly, I say to you, before Abraham was born, I am," (John 8:58).
Parables

Parables
The parables of Jesus are treasure houses of wisdom masterfully woven in story form. They are
deep, theological, practical, sometimes confusing, but always worth the effort needed to unlock their
mysteries.
Basically, a parable is a short story with a moral lesson. Jesus’ parables teach a series of moral
concepts using the culture of the times. Though the parables have much to offer to us in the present
day via a casual reading, they have even more to offer when we understand the culture of the time
and examine them in that light. For instance, in the story of the Prodigal son, when the son asked for
his father’s inheritance, that was equivalent to saying he didn’t care if his father lived or died. He just
wanted his money. Why? Because a son never ever asked for an inheritance until after the death of
his parent. To do so prematurely was to imply he wished his parent’s death!
There are many such cultural gems waiting for us to discover. When laid in the rich framework of
the parables, we can see the majestic beauty and power of Jesus’ living words reflected in the light of
His truth...and we are not left unaffected.
In the presentation of these parables, I have gleaned heavily from the book Poet & Peasant and
Through Peasant Eyes, by Kenneth E. Bailey. This book forced open my eyes when reading the
parables caused me to see things in them I had never thought of before.
It is important to know that the nobleman of ancient Israel did not run, but walked at a dignified
pace. Then what does this mean when the Prodigal’s father runs to his son?
Isolation from impure food and people was especially crucial for the Pharisees when they sat down
to eat. How do we consider this when the Pharisee asked Jesus to eat with him and provided no means
for Jesus to wash?
A person’s ethnic background could be seen through his speech and his clothes. How does this bear
upon the Good Samaritan parable where the man is left unconscious and naked?
A woman could be divorced for letting her hair down in public. What does this mean when the
woman wet Jesus’ feet with her tears and wiped them with her hair?
The parables used familiar symbols so the listener could relate and, if need be, be shocked.
Whatever the outcome in the hearer, the parables required a response. Either the hearer was to
change a behavior, or a thought, or a belief, or something else. But change is the reason for the
parables.
They were not simply stories. They were living words from the mouth of God.

1. What is a parable? p. 135


2. Why is culture so important in understanding parables? p. 135
3. What did you learn from the Great Banquet Parable? pp. 136-138
4. What does the parable of the fig tree teach you about your own life? pp. 139-141
5. The Prodigal Son is about two types of sinner. Who are they? pp. 142-144
6. Why was the parable of the Good Samaritan so important in Jesus' time? pp. 145-147
7. Why was the unjust steward praised for his deceit? pp. 148-151
The Great Banquet Luke 14:15-24
The Old Testament background for this parable is found in Isaiah 25:6-9:

6
And the Lord of hosts will prepare a lavish banquet for all peoples on this mountain; a banquet of
aged wine, choice pieces with marrow, and refined, aged wine.
7
And on this mountain He will swallow up the covering which is over all peoples, even the veil which
is stretched over all nations.
8
He will swallow up death for all time, and the Lord God will wipe tears away from all faces, and He
will remove the reproach of His people from all the earth; for the Lord has spoken.
9
And it will be said in that day, "Behold, this is our God for whom we have waited that He might
save us. This is the Lord for whom we have waited; let us rejoice and be glad in His salvation."

15. And when one of those "To eat bread" is another way of saying, "To eat a meal."
who were reclining at the
table with Him heard this, he
said to Him, "Blessed is
everyone who shall eat bread
in the kingdom of God!"

16. But He said to him, "A It was the custom when giving a dinner, to invite a certain
certain man was giving a big number of people. Those who accepted the invitation were then
dinner, and he invited many; counted. The meal was prepared according to the number who
accepted the invitation. The more people coming, the more food
had to be prepared. For example, a chicken would be for 2-4
guests, a duck for 5-8, a lamb for 10-15, a sheep for 15-35, and a
calf for 35-75. In other words, the amount and type of meat
depends on the number of people who accept the invitation. Once
an animal has been killed it must be eaten soon or else it will spoil.
Therefore, to back out at the last minute would be rude. The
invited guest is duty bound to attend the banquet.
Also, it was considered very rude to attend a banquet if you
were not invited; after all, the meal had not been prepared with
you in mind.

17. and at the dinner hour The second invitation is a notification to the guests that the
he sent his slave to say to meal is ready.
those who had been invited, The Greek word "Come" means literally, continue coming. This
'Come' for everything is is consistent with the custom of a double invitation.
ready now.'

18. but they all alike began The meal has been prepared, the table set, and people notified.
to make excuses. the first To back out now is an insult.
one said to him, 'I have In the middle East, no one buys a field without first examining
bought a piece of land and I it thoroughly. The springs, wells, stone walls, trees, paths, and
need to go out and look at it; anticipated rainfall are all well-known long before a discussion of
please consider me excused.' the purchase is even begun. The excuse is a lie, an obvious one,
and the guest is stating in no uncertain terms that the field is more
important than his relationship with the host. In a community
where interpersonal relationships are very important, this strikes
even harder as an offence.
19. And another one said, 'I Teams of oxen are sold in the Middle East in two ways. They
have bought five yoke of are taken to the market place and a nearby field and there they
oxen, and I am going to try plow the field. Anyone wishing to buy may then drive the oxen
them out; please consider himself and examine the animals thoroughly to see if they work
me excused.' well as a team.
That is like calling your wife at home and saying you'll be late
for the big dinner that's been planned for weeks because you need
to go out an look at five cars you just bought without looking at
them. The other way is to announce that the team is for sale and
say what day the team will be working in the field. Prospective
buyers can then come to the field, watch, examine, and test them
for themselves. Only after the team is examined thoroughly is a
price discussed.
This excuse, like the other one, is also an insult.

20. And another one said, 'I In the tightly knit community of the Middle East a wedding calls
have married a wife, and for for a celebration. At a celebration is food, lots of it. The community
that reason I cannot come.' would have been aware of it and many people would have been
invited. Meals would have been prepared before hand. Therefore,
the banquet would not have been scheduled for the same day as a
wedding.
Also, if the man simply wants to be with his wife then why did
he accept the invitation in the first place?
This one doesn't even say, "Please."

21. And the slave came back Anger would be a natural expectation of the head of the
and reported this to his household. He has been insulted three times.
master. Then the head of the The invited guests refuse to respond to the good news that the
household became angry and feast is ready. What then is the host to do? He cannot have a feast
said to his slave, 'Go out at without guests. He then invites the unworthy, the poor, crippled,
once into the streets and blind, and lame. He brings in the undesirables. So, he gives the
lanes of the city and bring in command to bring in the poor, who aren't normally invited to
here the poor and crippled banquets; the crippled, who cannot test oxen in the field; and the
and blind and lame.' blind and lame who don't normally marry.
They have no way of repaying the host and he knows it.
Therefore, he is being gracious, very gracious in light of the insults
received.

22. And the slave said, Some have already been saved. But there is room for more.
'Master, what you
commanded has been done,
and still there is room.'

23. And the master said to The previous guests were all part of the immediate community.
the slave, 'Go out into the But here, the highways and hedges are sought out for more to join
highways and along the the banquet. Isaiah 25:6-9 is important for a proper understanding
hedges, and compel them to of this parable. In Isaiah the Gentiles are included in the banquet
come in, that my house my with the Lord.
be filled.' Notice also, that the command is not carried out in this
parable. It is given but no account of its fulfillment is mentioned.
This is because those being compelled to enter in have not yet
been all invited. Redemption is still going on.

24. For I tell you, none of


those men who were invited
shall taste of my dinner.
With what would the original audience have identified in the parable?

The Banquet = the messianic banquet that ushers in the new age.
The Original Guests = the leaders of Israel who are rightfully the first to be invited.
The Lame and Poor of the City = the outcasts within the house of Israel.
The Guests from the Highways and the Hedges = the gentiles.

This parable teaches that no one may enter the kingdom of God without an invitation from God. An
invitation by grace.
It also is a warning to heed the invitation when it is heard. Because the invitation does not last
forever.
In between two great banquet parables, each declaring pure grace (the Great Banquet and the
Prodigal Son), is set a collection of sayings that speaks of the high cost of discipleship in clear and
demanding terms (Luke 14:25-35).
The Banquet is free, the invitation by grace, but acceptance carries with it responsibility.
Discipleship is our responsibility.

Luke 14:25-35:

Now great multitudes were going along with Him; and he turned and said to them, "If anyone
comes to Me, and does not hate his own father and mother and wife and children and brothers and
sisters, yes, and even his own life, he cannot be My disciple. Whoever does not carry his own cross
and come after Me cannot be My disciple. For which one of you, when he wants to build a tower, does
not first sit down and calculate the cost, to see if he has enough to complete it? Otherwise, when he
has laid a foundation, and is not able to finish, all who observe it begin to ridicule him, saying, 'This
man began to build and was not able to finish.; Or what king, when he sets out to meet another king
in battle, will not first sit down and take counsel whether he is strong enough with ten thousand men
to encounter the one coming against him with twenty thousand/ Or else, while the other is still far
away, he sends a delegation and asks terms of peace. So therefore, no one of you can be My disciple
who does not give up all his own possessions. Therefore, salt is good; but if even salt has become
tasteless, with what will it be seasoned?"
The Fig Tree Luke 13:1-9

1. Now on the same Josephus, the Jewish historian of the time of Christ and after,
occasion there were records a number of massacres during this period, but does not
some present who mention this one.
reported to Him about Perhaps the people reporting to Jesus were seeking to get Him to
the Galileans, whose comment politically on Pilate and thereby use Jesus as a means of
blood Pilate had rallying support for their cause. Remember, the Jews were under the
mingled [shed along rule of the Roman government and resented it. Ungodly gentiles were
with] with their ruling over the house of Israel. Obviously, the people doing the
sacrifices. reporting are interested in deliverance as well as justice. They want
what is right; at least, right the way they see it.
Another way to look at the situation would be to imagine a church
gathered one Sunday having communion. Then gunmen enter and
shoot everyone present thereby mingling their blood with the wine of
the supper. The natural reaction would be one of horror and hatred.
This is the type of thing that is presented to Jesus.

2. And He answered When Jesus was told about the slaughter in the temple, He
and said to them, "Do responded not with indignant denunciation of Roman brutality, but with
you suppose that these a warning to His own people to "repent".
Galileans were greater This raises an important question: Why were they told to repent
sinners than all other after Jesus heard about the indignity?
Galileans, because The word "fate" is not in the original and is not here intended to
they suffered this fate? support the belief of fatalism.

3. I tell you, no, but, His statement that they repent or perish is a bold confrontation of
unless you repent you sin; something the Jews did not appreciate Jesus pointing out,
will all likewise perish. particularly when they are expecting Jesus to side with them about the
slaughter of the Galileans.
What an unexpected switch! Jesus doesn't comment on the atrocity
of people killed in the act of sacrifice to the God of Israel, instead He
tells the multitudes they need to repent! Jesus will not be used by the
people to foster their political beliefs. Instead, He tells them to repent
or perish.
We know that the judging hand of God fell upon the Jewish nation in
the form of its destruction in 70 A.D. when Israel was scattered and the
temple destroyed. The Jewish nation had not repented of its sins of
legalism, self-righteousness, and ethnic pride, all of which, combined to
bring about the murder of Jesus at their hands. The Jews reaped what
they sowed. The sowed death, they reaped death.
However, there is not intended here a one to one correspondence
on the relationship between sin and its consequences. Elsewhere Jesus
denies such a correspondence. Please consider this: "And as He passed
by, He saw a man blind from birth. And His disciples asked Him, saying,
'Rabbi, who sinned, this man or his parents, that he should be born
blind?' Jesus answered, 'It was neither that this man sinned, nor his
parents; but it was in order that the works of God might be displayed in
him" (John 9:1-3).
4. Or do you suppose Jesus broadens the scope of the discussion by mentioning an
that those eighteen on incident where a tower fell and killed eighteen people. He uses the
whom the tower in word in Greek for "debtor." This word stands in contrast to the word
Siloam fell and killed "sinners" in verse two.
them, were worse We are in debt to God because we have broken His laws; we have
culprits [debtors] than sinned. A debt is what is owed. Matthew renders the Lord's prayer as
all the men who live in "Forgive us our debts as we forgive our debtors.” The Lucan account of
Jerusalem? the same prayer uses the words "forgive us our sins..." Debts are
unfulfilled duties; sins are purposefully commit acts of rebellion.
The people speak of the slaughter and Jesus speaks of 18 who died
long ago. The 18 were no worse than the Galileans. Why then were
they all killed? Perhaps the better question might be, "Why were any
left alive?" Nevertheless, there is no such thing as chance in a universe
governed by God. The deaths under the tower and at the altar of
sacrifice were all permitted by God.

5. I tell you, no but But what is Jesus saying? He mentions the eighteen and says they
unless you repent, you are no worse than those then living in Jerusalem. Jesus' declaration of
will all likewise perish." the need for Israel to repent of their sins, in the light of the slaughter
of the Galileans would almost seem to bring extreme anger, even revolt
against Him by those listening. After all, the Jews felt oppressed and
the incident of the Galileans would only cement their attitudes of
persecution and self righteousness.

6. And He began telling Leviticus 19:23-25 says, "And when you enter the land and plant all
this parable: "A certain kinds of trees for food, then you shall count their fruit as forbidden.
man had a fig tree Three years it shall be forbidden to you; it shall not be eaten. But in
which had been the fourth year all its fruit shall be holy, an offering of praise to the
planted in his Lord. And in the fifth year you are to eat of its fruit, that its yield may
vineyard; and he came increase for you..."
looking for fruit on it, The vineyard owner was ready to eat of the fruit. But there wasn't
and did not find any. any. It was the 7th year of looking: the 5th year fruit would have been
the first year he could have partaken. The 6th year would have been
the second year he could have partaken, and the 7th year would have
been the year spoken of here. Therefore, he says in verse 7...

7. And he said to the The owner has the right to expect fruit from his vineyard.
vineyard-keeper, Symbolically, this parable seems to be teaching that the Jewish
'Behold, for three years leadership has had enough time to repent of their sins. John the Baptist
I have come looking said to the multitudes going out to see him, "Therefore bring forth
for fruit on this fig tree fruits in keeping with repentance..." (Luke 3:8).
without finding any. Years ago I came across a tract of which title alone struck me hard.
Cut it down! Why does It said, "No Fruit? Cut it down."
it even use up the As Christians we are to bear the fruit of the Spirit mentioned in Gal.
ground?' 5:22: love, joy, peace, patience, kindness, goodness, faithfulness,
gentleness, and self-control. If these fruit are not manifested in your
lives, should you be cut down? Apparently, the Jewish leadership were
not manifesting the fruit of the Spirit nor the fruit of repentance.

8. And he answered This section contains simple teachings:


and said to him, 'Let it
alone, sir, for this year 1) The spiritual leaders of the household of faith are planted in
too, until I dig around "God's vineyard" and are expected to produce fruit.
it and put in fertilizer; 2) God will not tolerate fruitlessness indefinitely.
9. and if it bears fruit 3) Mercy and Grace are extended to those who do not bear fruit.
next year, fine; but if
not, cut it down.'"

What is the expected response of the one who hears?


You should examine your own lives and look for fruit. Preferably the fruit of the Holy Spirit
mentioned in Galatians 5:22, for this is how you store up fruit for eternal life (John 4:36). You must
also realize that it is not possible to bear fruit apart from the Branch, Jesus (John 15), for apart from
Him you can do nothing.
There are many types of fruit that could be examined: giving, praying, righteousness, forgiveness,
tithing, discipling others, leading others to Christ, missionary support, etc. Each is different, but from
the same Lord.
Each of us is different with different gifts and fruit, but we are all of the same body.
The Prodigal Son Luke 15:1-2, 11-32
Theme: Great joy in the salvation of the lost.

1A son is lost - "Give me my share"


| 2 Goods wasted in extravagant living
| | 3 Everything lost - "He spent everything-he began to want
| | | 4 The great sin - "feeding pigs for gentiles
| | | | 5 Total rejection - "no one gave him anything
| | | | | 6 A change of mind - "he came to himself-I perish here"
| | | | | 6 An initial repentance - "make me a servant"
| | | | 5 Total acceptance - "his father ran and kissed him."
| | | 4 The great repentance - "I am no more worthy to be called your son.
| | 3 Everything gained - a robe, ring, and shoes
| 2 Goods used in joyful celebration
1A son is found - "My son was dead and is alive, was lost and is found."

In the beginning of this chapter Jesus is with the self righteous. Yet, He eats with sinners. When
the righteous men of Israel complained about his "obvious" error of eating with sinners, they voice
there disapproval.
Jesus' reply was not one of rebuke, but of teaching; hence, several parables. The first two have
three common threads running through each. 1) Something or someone is lost. 2) The lost is sought
for. 3) Great joy is shared at the recovery of the thing (person) found.
The third parable mentioned is slightly different in the second thread only. In it, the one who is lost
returns to where he came from.

1. Now all the tax gatherers and Those in need.


the sinners were coming near him Eating was important in this culture because it implied
to listen to Him. fellowship, a sharing of something in common. To eat with
sinners could be interpreted many ways. Here, Jesus is
2. And both the Pharisees and the identifying, reaching out to the sinners.
scribes began to grumble, saying, Jesus is accused of eating with sinners. He does not
"This man receives sinners and rebuke; He does not revile; He teaches. So should our witness
eats with them. be. We should be loving of all who sin, accepting of all who
repent, willing to humble ourselves before men and God. Trust
Him to do what is right.
11. And He said, "A certain man The prodigal is shown as wishing for his father's death in
had two sons; his request because the estate was never divided among the
children until after the father’s death. The father should
12. and the younger of them said severely rebuke his son. Instead, the father shows incredible
to his father, 'Father, give me the love by granting the request to his son.
share of the estate that falls to
me.' And he divided his wealth
between them.
13. And not many days later, the He could not sell the land in the community during his
younger son gathered everything father's lifetime. No one would buy it. So, he travels to a
together and went on a journey distant land and sells his property, thus losing the right of
into a distant country, and there redemption of the land.
he squandered his estate with
loose living.
14. Now when he had spent Everything is lost.
everything, a severe famine
occurred in that country, and he
began to be in need.
15. And he went and attached Swine is an unclean animal. It would seem this act was one
himself to one of the citizens of of disdain by the pig owner, "Here Jew, feed pigs."
that country, and he sent him
into his fields to feed swine.
16. And he was longing to fill his He was totally rejected by the people around him. (The
stomach with the pods that the Pharisees rejected the tax-gatherers and sinners.)
swine were eating, and no one
was giving anything to him.
17. But when he came to his His motivation was poverty. Servants were an honorable
senses, he said, 'How many of class of people. He could live in the village. He wouldn't need
my father's hired men have more to live under the same roof as the eldest son. He'd have to face
than enough bread, but I am the scorn of the community though. It is possible that he may
dying here with hunger! have wanted to pay something back to his father, but, of
course, it could not possibly be enough.
18. I will get up and go to my Is the son truly repentant at this time?
father, and will say to him,
"Father, I have sinned against It seems his goal is to become a servant, to earn money,
heaven, and in your sight; and maybe to begin to repay what he lost.

19. I am no longer worthy to be


called your son; make me as one
of your hired men.'"
20. And he got up and came to The Father totally accepts his son.
his father. But while he was still a In that culture, older men did not run; it was a sign of
long way off, his father saw him, humiliation. (Phil. 2:5-8) The son should run to the father.
and felt compassion for him, and
ran and embraced him, and
kissed him.
21. And the son said to him, No bargaining is offered. He admits his guilt only. There is
"Father, I have sinned against no mention of servanthood or earning anything.
heaven and in your sight; I am
no longer worthy to be called
your son.
22. But the father said to his Robe: sign of dignity and honor.
slaves, Quickly bring out the best
robe and put it on him, and put a Ring: sign of authority.
ring on his hand and sandals on
his feet; Shoes: sign of not being a servant. Servants did not wear
shoes.
23. and bring the fattened calf, A whole calf is a lot to eat. The whole village would be
kill it, and let us eat and be invited. (Note: blood is shed)
merry.
24. for this son of mine was The lost son is found.
dead, and has come to life again;
he was lost, and has been found.
And they began to be merry.
25. Now his older son was in the Another son is lost. The duties of the eldest son included
field, and when he came and reconciliation between father and son. A host at feasts. The
approached the house, he heard older son is in the field and not in the house where he should
music and dancing. be. This is a public disgrace to the father.
26. And he summoned one of the
servants and began inquiring
what these things might be.

27. And he said to him, "Your


brother has come, and your
father has killed the fattened calf,
because he has received him
back safe and sound.
28. But he became angry, and The father went out to his son to entreat him. He did not
was not willing to go in; and his rebuke as was customary. Again, the father goes to the son.
father came out and began
entreating him.
29. But he answered and said to When addressing the father, it should be as 'Father,' not
his father, "Look! for many years simply 'Look!' This is very disrespectful (unhumble).
I have been serving you, and I
have never neglected a command The eldest son gives two complaints: one about the father,
of yours; and yet you have never and...
given me a kid, that I might be
merry with my friends;
30. but when this son of yours ...the other about his brother, the sinner. (fornication,
came, who has devoured your devoured your life, "ton bion" in the Greek means "the life".
wealth with harlots, you killed the You killed the calf for him and not me.)
fattened calf for him."
31. And he said to him, "My child, Teknon, child, a Greek word/term of endearment.
you have always been with me,
and all that is mine is yours. All that is mine is yours, come join the celebration.
32. But we had to be merry and And you were dead in our trespasses and sins, Eph. 2:1.
rejoice, for this brother of yours
was dead and has begun to live,
and was lost and has been found.

There are mentioned here two types of sinners: the honest manifest one, the younger son, and the
hypocritical sinner, the elder.
There are mentioned two types of repentance: sincere and pharisaical.
The younger son's initial repentance is not sincere, v. 17, because it was motivated from hunger;
but, in v. 21, he openly admits his sin. The older brother is anchored in self righteousness. His
repentance is not sincere.
God's great love extends to all sinners, the honest as well as the hypocritical. It endures
humiliation. It exults joyously when there is true repentance.
God desires sons, not servants.

The lessons in this parable are many; however, the two main ones are:
The unconditional love of God to everyone.
The gentleness of Jesus and His not striking back in word or deed.
May we learn to do as Jesus teaches. See Matthew 5:38-48.
The Good Samaritan Luke 10:25-37
THEME: What must I do to inherit eternal life?

25. And behold, a certain Lawyer: One who is an expert in the Law of Moses. Often this
lawyer stood up and put individual was called upon to settle legal issues. "He stood up." This is
Him to the test, saying, a social courtesy and a greeting of respect. Yet, in his heart he sought
"Teacher, what shall I do to test Jesus. This is a contradiction between his actions and his words.
to inherit eternal life?"
26. And He said to him, Jesus asks the lawyer about what he knows best: the law. He
"What is written in the knows that keeping the law is the appropriate answer. He brings the
Law? How does it read to issue out into the open. This is probably best since the Jewish
you?" leadership were probably concerned about Jesus' teachings on the Law.

27. And he answered and It is interesting that this man of the law would quote something
said, "You shall love the regarding love and not some ritual or set of rules.
LORD your God with all The standard set here is one which no one could keep.
your heart, and with all Perhaps he was testing Jesus by quoting what Jesus had taught
your soul, and with all your before: love.
strength, and with all your
mind; and your neighbor
as yourself."
28. And He said to him,
"You have answered Jesus, the man, instructs the man of the law, "You have answered
correctly; Do this and you correctly."
will live."
The expected reply would be something like, "Your relative and
29. But wishing to justify
your friend." Then the lawyer would be able to say that he has done
himself, he said to Jesus,
this and thereby enjoy honor among the people there listening;
"And who is my neighbor?"
However, Jesus said...
30. Jesus replied and said, There is a road that goes down from Jerusalem to Jericho. It is 17
"A certain man was going miles long. It has long been a hazardous trip due to thieves and
down from Jerusalem to robbers.
Jericho; and he fell among Jesus intentionally leaves the man undescribed. The audience,
robbers, and they stripped being Jewish, would naturally assume that he was a Jew. Being in this
him and beat him, and half dead state he would be unconscious.
went off leaving him half
dead. Since he is stripped, he then is unidentifiable. Historically, a person can
be identified in one of two ways: his dress and his speech, i.e. dialect.
The man is any person: void of ethnic background, void of stature,
void of position
31. "And by chance a The priest was most certainly riding because he was in the upper
certain priest was going classes of society.
down on that road, and The poor walk.
when he saw him, he Since, he moves to the other side, probably the priest did not
passed by on the other actually see it happen. How can he be sure the wounded man is a
side. neighbor since he cannot be identified? If the person lying there is a
non Jew the priest could be risking defilement, especially if the person
were actually dead. If he defiles himself he can not collect, distribute,
and eat tithes. His family and servants will suffer the consequences
with him.
Priests were supposed to be ritually clean, exemplars of the law.
There would be immediate shame and embarrassment suffered by
them at the expense of the people and their peers for such defilement.
Having just completed his mandatory two weeks of service, he would
then need to return and stand at the Eastern Gate along with the rest
of the unclean. Furthermore, in addition to the humiliation involved,
the process of restoring ritual purity was time consuming and costly. It
required finding, buying, and reducing a red heifer to ashes, and the
ritual took a full week. The priest is in a predicament. Moreover, he
cannot approach closer than four cubits to a dead man without being
defiled, and he will have to overstep that boundary just to ascertain
the condition of the wounded man.
32. "And likewise a Levite The road spoken of here is a long one. It is very likely, according to
also, when he came to the those who have walked it, that a person traveling it, could see ahead
place and saw him, passed of him a long way. The Levite, who is of a lower social class, may have
by on the other side. been walking. He most probably saw the priest ahead of him and could
have thought to himself, "If the priest may pass then so should I."
33. "But a certain The Samaritans were a mixed race between the Jews of captivity
Samaritan, who was on a and the Samaritan people of the land they were captive in. The
journey, came upon him; relationship between the Jews and Samaritans was one of hostility
and when he saw him, he because of some bad things that happened in the past. According to
felt compassion, the Mishna, "He that eats the bread of the Samaritans is like to one
that eats the flesh of swine,” (Mishna Shebiith 8:10). The Mishna is the
oral traditions that developed about the law, containing interpretations
and applications to specific questions which the law deals with only in
principle. Specifically, it is the collection of these traditions.
The Samaritan is not a gentile. He is bound by the same law as the
Jews. The Samaritan would not be naturally from that area, so the half
dead man would certainly not qualify as his neighbor.
34. "and came to him, and The Samaritan risks defilement. He approaches this unidentifiable
bandaged up his wounds, man and helps him.
pouring oil and wine on Oil and wine were poured out on the high altar before God. Note
them; and he put him on how the usage is mentioned after the Priest and Levite have failed to
his own beast, and brought do their duty.
him to an inn, and took Blood revenge: "Mosaic legislation established cities of refuge for
care of him. people under the threat of death from blood vengeance retaliation.
This legislation provided an escape valve for a custom it could not
eradicate."
Often when the guilty cannot be reached, vengeance may be
administered to a member of his family. Often the vengeance would
reach even to the most distant relations of the offending party.
"Irrational minds seeking a focus for their retaliation do not make
rational judgments, especially when the person involved is from a
hated minority community."
The Samaritan forfeits anonymity when he stays overnight and then
35. "And on the next day
says he would return. This is an acceptance of the potential threat of
he took out two denarii
blood vengeance.
and gave them to the
innkeeper and said, 'Take
care of him; and whatever The wounded man has no money. When it is time for him to leave, if
more you spend, when I he cannot pay the debt he can be arrested, Matthew 18:23-35. The
return, I will repay you.' Samaritan knows this and volunteers money

and whatever else is needed to see to the needs of this unidentified


man. Additionally, the Samaritan had no way of insuring the return of
his money. Therefore, it is safe to assume he did not expect it to be
returned.

The Robbers The Samaritan

Rob him Pays for him


Leave him dying Leaves him cared for
Abandon him Promises to return

The robbers hurt the man by violence, the Priest and Levite, by neglect. All three are guilty. "To
the one who knows the right thing to do and does not do it, to him it is sin," (James 4:17).
63. "Which of these three Jesus refuses to define who a neighbor is. Instead He asks a
do you think proved to be question proving something greater than the exact answer anticipated.
a neighbor to the man who Being a neighbor to someone is not limited to family relations or
fell into the robbers' proximity. It is showing the love of God to all who are in need, who
hands?" ever they may be, where ever they may be.
73. And he said, "the one The discussion began with a question: what must I do inherit
who showed mercy toward eternal life. The conclusion is answered with what must be done.
him." And Jesus said to If we are to do this, we will quickly find that we are incapable of
him, "Go and do the completing so perfect a love. Since the law requires perfect obedience,
same." the doing of this lesson would be something most difficult for the
lawyer.

This parable teaches the impossibility of earning one's salvation. The standard, which is perfect
love, is too high.
It holds up an ethical level for us to strive for, see Matthew 5:48.
It attacks racial prejudices.
It teaches that love is something you feel and do.
The Unjust Steward Luke 16:1-8

A. Now He was also saying to the disciples, 'There was a Rich Man and Steward
certain rich man who had a steward, and this steward was
reported to him as squandering his possessions.

B. And he called him and said to him, "What is this I hear Problem
about you? Give an account of your stewardship, for you can
no longer be steward."
B. and the steward said to himself, "What shall I do, since my Problem
master is taking the stewardship away from me? I am not
strong enough to dig; I am ashamed to beg.
C. I know what I shall do, so that when I am removed from Idea
the stewardship, they will receive me into their homes."
B. And he summoned each one of his master's debtors, and Solution
he began saying to the first, "How much more do you owe my
master?" And he said, "A hundred measures of oil." And he
said to him, "Take your bill, and sit down quickly and write
fifty."
B. Then he said to another, "And how much do you owe?" And Solution
he said, "A hundred measures of wheat." He said to him, "Take
your bill, and write eighty."
A. And his master praised the unrighteous steward because Rich Man and Steward
he had acted shrewdly; for the sons of this age are more
shrewd in relation to their own kind than the sons of light.

Many commentators agree that this parable is the most difficult of all the parables to interpret.
In fact, it is interesting to note that there are other "unsavory" characters in Jesus’ parables: The
unjust judge, the neighbor who does not want to be bothered in the night, and the man who
pockets someone else’s treasure by buying his field.
The seeming incongruity of a story that praises a scoundrel has been an embarrassment to the
Church at least since Julian the Apostate used the parable to assert the inferiority of the Christian
faith and its founder. We need a more precise understanding of the culture that affects this text.
The disciples are the primary audience, but the Pharisees are also included (v. 14).

Questions to ask:

1. Is the master assumed to be an honorable man, or is he a partner-in-crime


with his steward?
2. Has the steward obliged the renters to sign bills for amounts greater than the
actual debts?
3. Is his reduction of the debts merely a surrender of his dishonest cut?
4. Is the steward an estate manager dealing with land rentals or is he an
authorized agent for a moneylender?

The most probable cultural setting for the parable is that of a large estate consisting of land
divided into portions, where the steward is entrusted with carrying the business of that estate. The
debtors are most likely renters who had agreed to pay a fixed amount of produce for the yearly
rent. The steward was no doubt making extras "under the table," but these amounts were not
reflected in the signed bills. He was a salaried official who, in addition, was paid a specific fee by
the renter for each contract. The master was a man of noble character respected in the community
who cared enough about his own wealth to fire a wasteful manager, and this is the key to
understanding this parable.
Mishnah, a Hebrew term meaning "repetition" or "study," is the name given to the oldest
postbiblical codification of Jewish Oral Law. Together with the Gemara (later commentaries on the
Mishnah itself), it forms the TALMUD. Between 400 BC and the beginning of the Christian Era, the
biblical laws (see TORAH) were intensively studied, applied to new situations, and supplemented
by traditions of popular observance and by precedents established by prominent leaders. This
material, long transmitted by word of mouth and known as the Oral Torah, defined the meaning of
biblical laws. After the fall of Jerusalem and the destruction of the Temple in AD 70, the Jewish
scholars and teachers called tannaim continued to elaborate and systematize the Oral Torah.
About AD 200, Rabbi JUDAH HA-NASI promulgated a collection of the most reliable traditions. This
work, the Mishnah, became the official text out of which further Jewish legal development
occurred.

1. Now He was also saying to the Someone apparently cared enough about the master to
disciples, 'There was a certain rich tell him something was wrong.
man who had a steward, and this Often tenants disliked the landowners and would not
steward was reported to him as voluntarily help him.
squandering his possessions.
2. And he called him and said to him, The servant does not know how much the master knows
"What is this I hear about you? Give and may be frightened into divulging information the
an account of your stewardship, for master does not have. So, he remains silent.
you can no longer be steward."
3. and the steward said to himself, The steward remained silent. He does not defend
"What shall I do, since my master is himself. In the culture of the time, this is almost an
taking the stewardship away from admission of guilt. He thinks of a way to cover himself after
me? I am not strong enough to dig; I he has been let go.
am ashamed to beg. It is not known if the steward is fired now or later? Is he
asked to get the books now or get them ready to be
examined?
The steward acts as though he is not yet fired: he says,
4. I know what I shall do, so that
‘when I am removed...’; yet, earlier, the landowner had
when I am removed from the
said, ‘you can no longer be steward,’ present tense. So, it is
stewardship, they will receive me into
most likely that he was fired on the spot. But, the word
their homes."
apparently isn't out yet.
5. And he summoned each one of his The steward does not say "Hello" or "Friend." He is in a
master's debtors, and he began hurry.
saying to the first, "How much more
do you owe my master?"
6. And he said, "A hundred measures If the renters know that the steward has been fired and
of oil." And he said to him, "Take your they agree to the deal they would risk being thrown off the
bill, and sit down quickly and write land by the landowner, not to mention sinning in their
fifty." dishonesty. The relationship between the owner of the land
and his renters is a significant personal and economic
relationship. "Take your bill, and sit down quickly and write
fifty" is an attempt by the steward to finish before the
master finds out what is going on.
7. Then he said to another, "And how If the renters did not think that the steward was acting
much do you owe?" And he said, "A with the approval of the landowner they would not have
hundred measures of wheat." He said agreed; the risk would be too great.
to him, "Take your bill, and write In this account, the steward receives credit for having
eighty." arranged such a good deal between the landowner and the
renters.
The renters, would be very appreciative and indebted to
the steward.
Generally, reductions of rent were expected if the
conditions warranted it: a dried spring, fruit trees drying,
drought, etc. But the renters would have to ask for the
reduction and then proceed to haggle. Here, though, the
reduction is unsolicited.
8. And his master praised the Why does the landowner praise the unjust steward?
unrighteous steward because he had It would be quite safe to assume that there is some sort
acted shrewdly; for the sons of this of celebrating occurring in the homes of the renters as they
age are more shrewd in relation to rejoice in, what they think is, the generosity of the
their own kind than the sons of light. landowner.

Soon the whole community would be aware of the kind heartedness of the landowner and would
be happy with the renters as well as thinking honorably of the generous landowner.
He has two alternatives: First, he could gather the renters and tell them that the reductions
were unauthorized and thereby showing his stinginess and risking ridicule from them and the
community. Second, he can keep silent, accept the praise that is even now being showered on
him, and allow the clever steward to get away with the scheme.
Obviously, the steward knew the master was a generous person, otherwise he would not have
taken such a risk; after all, he wasn't jailed to begin with.
In verse 9 Jesus is not praising the dishonesty, but the ability of the steward to recognize the
generosity of his master, see what was coming, and use what he had at the time to obtain
something far greater: self preservation.
This is significant. The fear of the Lord is te beginning of Wisdom. God can condemn you to
eternal damnation. It is wise to seek a way out of that. In fact, the judgment of damnation is so
terrible, that praise is offered to the one who, in desperation, seeks a way out of it.
If the unrighteous steward was praised for trusting the master, how much more will you be
rewarded if you trust the true and holy Master, the Lord Himself.
Jesus uses the rabbinic principle of showing "how much more." That is, if the widow got what
she wanted from the judge (18:1-9), how much more you and God? If the man got bread in the
night from his neighbor (11:5-7), how much more you from God?
What, then, does the parable typify?

1. God (the master) is a God of judgment and mercy.


2. Because of his evil, man (the steward) is caught in the crisis of the coming of the kingdom.
3. Excuses will avail the steward nothing.
4. Man's only option is to entrust everything to the unfailing mercy of his generous master
who, he can be confident, will accept to pay the price for man's salvation.

The steward was vindicated because he completely trusted the master to be generous, to be
good. He was right in doing so!!!
This parable is an appeal to people to understand the nature of God.

1. The nature of God: He is merciful


2. The predicament of man: he is sinful
3. The ground for salvation: complete trust in God.
4. How much more dishonest to God are we?
5. How much more deceitful are we?
6. How much more do we owe to God?
7. How much more merciful is God than the rich man?
God (the master) is a God of judgment and mercy. Because of the steward’s evil behavior, he is
found out. This is representative of the coming kingdom of God and its associated judgment.
Excuses will avail the steward nothing. Man’s only option is to entrust everything to the unfailing
mercy of his generous master who, he can be confident, will accept to pay the price for man’s
salvation. This clever scoundrel was wise enough to place his total trust in the quality of mercy
experienced at the beginning of the story. That trust was vindicated. Christians need the same kind
of wisdom.

Who is God to you?


What do you think of Him?
How do you perceive Him?
Is He mean? Loving? Kind? Impatient? Judgmental?

How you perceive God will determine

• how you respond to Him


• how you address Him
• how often you call upon Him
• How you view troubles
• .....and how much you trust Him.

How much more merciful is God than the rich man?


Dictionary of Theology
Introduction

Words are the tools of theology. Do you know what they mean?

Following are a list of words that you should know if you want to understand your theology a little
better and also if you want to be able to defend the faith more clearly.

Active Obedience Hell


Agnosticism Holy Spirit, The
Antinomianism Hypostatic Union
Apologetics Immutability
Armageddon Incarnation
Atheism Inspiration
Atonement Justification
Baptism Martyr
Catholic Mediator
Christ Messiah
Communion Monotheism
Covenant Original Sin
Depravity Pantheism
Divinity Polytheism
Doctrine Postmodernism
Eschatology Predestination
Eternal life Purgatory
Evolution Regeneration
Faith Resurrection
Fellowship Salvation
Firstborn Septuagint, The
Gnosticism Spiritual Gifts
Gospel Tetragrammaton
Grace Tribulation, The
Heaven Trinity
A.D.
From the Latin, "Anno Domini," which means in the year of our Lord. The Western Calendar is
dated from the birth of Christ. Therefore, 2000 A.D. means 2000 years after the birth of Jesus.

A priori
Knowledge, judgments, and principles which are true without verification or testing. It is
universally true.

Abaddon
The Hebrew word for "destroyer" whose Greek equivalent was "Apollyon." Abaddon is the satanic
angel of the Abyss (Rev. 9:11 ).

Absolution
In Catholicism, the act of releasing someone from their sin by God, through the means of a priest.

Abyss
The abode of the dead and the place where evil spirits are imprisoned (Rev. 9:1-11).

Active Obedience
As distinguished from passive obedience in Reformed Theology. Active obedience is Jesus' actively
fulfilling all the law of God. This active obedience is imputed to the believer when he believes; that is,
God reckons to the believer the righteousness of Christ when the believer trusts in Christ and His
work.

Adiaphora
Teachings and practices that are neither commanded nor forbidden in scripture. An example
might be whether or not to use a sound-board in a church, to meet in a tent or a building, to have two
or more services or simply one on the day of worship.

Adoptionism
Adoptionism is an error concerning Jesus that first appeared in the second century. Those who
held it denied the preexistence of Christ and, therefore, His deity. Adoptionists taught that Jesus was
tested by God and after passing this test and upon His baptism, He was granted supernatural powers
by God and adopted as the Son. As a reward for His great accomplishments and perfect character
Jesus was raised from the dead and adopted into the Godhead. Please see Heresies for more
information.

Advent
From the latin, “coming.” The coming of or the arrival of something very important as in the
advent of Christ’s return. Advent is also an Christian time of preparation preceding Christmas.

Agnosticism
The belief that it is not possible to know if there is or is not a God. (Compare Atheism, Deism, and
Theism.)

Albigenses
A heresy during the middle ages that developed in the town Albi in Southern France. This error
taught that there were two gods: the good god of light usually referred to as Jesus in the New
Testament and the god of darkness and evil usually associated with Satan and the "God of the Old
Testament." Anything material was considered evil including the body that was created by Satan. The
soul, created by the good god, was imprisoned in the evil flesh and salvation was possible only through
holy living and doing good works. Please see Heresies for more information.
Amillennialism
The teaching that there is no literal 1000 year reign of Christ as referenced in Revelation 20. It
sees the 1000 year period spoken of in Revelation 20 as figurative. Instead, it teaches that we are in
the millennium now, and that at the return of Christ (1 Thess. 4:16 - 5:2) there will be the final
judgment and the heavens and the earth will then be destroyed and remade (2 Pet. 3:10). The
Amillennial view is as old as the Premillennial view. (Also compare to Postmillennialism).

Anabaptists
Any of a group of sects of the early Reformation period of the 16th century that believed in
rebaptism of people as adults. Infant baptism was not recognized as valid and the Catholic Mass was
rejected. Anabaptist means “one who baptizes again.” They believed in non-violence and opposed
state run churches.

Angel
Angel means messenger. Angels are created (Psalm 148:2,5; Col. 1:16), non-human, spirit beings
(Heb. 1:14). They are immortal (Luke 20:36), innumerable (Heb. 12:22), invisible (Num. 22:22-31),
sexless (Matt. 22:30), and do the will of God (Psalm 103:20). These angels have a ministry to
believers. They guide (Gen. 24:7, 40), protect (Psalm 34:7), and comfort (Acts 27:2, 24).
There are good angels (Gen. 28:12; Psalm 91:11) and bad angels (2 Pet. 2:4; Jude 6). The only
angels mentioned by name are Gabriel (Dan. 8:16; 9:21 ), Michael (Dan. 10:13,21; 112:1), and
Lucifer (Luke 10:18). Michael is always mentioned in the context of battle (Dan. 10:13) and Gabriel as
a messenger (Luke 1:26). Of course, Lucifer, who became Satan, is the one who opposes God.
Angels were originally created for the purpose of serving and carrying out the will of God. The
fallen angels rebelled and became evil angels. Satan is such an angel ( Isaiah 14:12-16; Ezekiel 28:12-
15).

Animism
The belief that everything in the universe contains a living soul. The belief that a spirit indwells
every object.

Annihilationism
The teaching that when a person dies, he is annihilated, most often this doctrine is applied to the
wicked, thereby negating eternal hell fire. This is contradicted by the Bible in Matt. 25:46 which says
“And these will go away into eternal punishment, but the righteous into eternal life.” Also, degrees of
punishment will be given on the Day of Judgment (Rev. 20:11-15). If all, or only the wicked are
annihilated, then degrees of punishment would be pointless.

Anthropic Principle
The idea that the universe exhibits elements of design specifically for the purpose of containing
intelligent beings; namely, humans. Much debate surrounds this issue. Is the universe necessarily
arranged by God so as to make life possible or is it simply that the universe is godless and that life
came into existence due to the chance state that we now find it in?

Anthropomorphic
Manifesting in human form. It is from the Greek "anthropos" meaning "man" and "morphe"
meaning "form." In biblical theology, God is described in anthropomorphic terms; that is, in human
terms with human attributes. For example, God has hands and feet in Exodus 24:9-11 and is loving (1
John 4:8).

Antichrist
A figure who opposes God. The word is used to describe a spirit of rebellion against God, "...the
spirit of the Antichrist..." (1 John 4:3) and of a specific future person identified as the man of
lawlessness (2 Thess. 2:3). He actively opposes Christ (2 Thess. 2:4) and when he arrives, he will be
able to perform miracles (2 Thess. 2:9). Some believe he will be an incarnation of Satan and as such
will be able to deceive many. His number is 666 (Rev. 13:18). A further possible description of him
might be found in Zech. 11:15-17).
Antinomianism
The word comes from the Greek anti, against, and nomos, law. It is the unbiblical practice of living
without regard to the righteousness of God, using God's grace as a license to sin, and trusting grace to
cleanse of sin. In other words, since grace is infinite and we are saved by grace, then we can sin all we
want and still be saved. It is wrong because even though as Christians we are not under the Law
(Rom. 6:14), we still fulfill the Law in the Law of love (Rom. 13:8,10; Gal. 5:14; 6:2). We are to love
God with all our heart, soul, strength, and mind, and our neighbor as ourselves ( Luke 10:27) and,
thereby, avoid the offense of sin which cost God His only begotten Son. Paul speaks against the
concept of antinomianism in Rom. 6:1-2: "Are we to continue in sin that grace may abound? May it
never be! How shall we who died to sin still live in it?" We are not to use the grace of God as a means
of sin. Instead, we are to be controlled by the love of God and in that way bear the fruit of the Holy
Spirit (Gal. 5:22-25).

Apollyon
Greek for "destroyer." He is the angel of the Abyss, "They have as king over them, the angel of
the abyss; his name in Hebrew is Abaddon, and in the Greek he has the name Apollyon," (Rev. 9:11 ).

Apparition
An unexpected, supernatural appearance of a ghost or a person. Folklore of apparitions are
particularly strong among Catholics who claim appearances of the saints, Mary, and Jesus with
messages of repentance and turning to the Catholic sacraments and confession.

Apocalypse
From the Greek word for "revelation," or "unveiling." It is synonymous with Armageddon and
refers to those future things dealing with the return of Christ and the great battle of Armageddon.

Apocrypha
The word apocrypha means hidden. It is used in a general sense to describe a list of books written
by Jews between 300 and 100 B.C. More specifically, it is used of the seven additional books accepted
by the Catholic church as being inspired. The entire list of books of the apocrypha are: 1 Esdras, 2
Esdras, Tobit, Judith, the Rest of Esther, the Wisdom of Solomon, Sirach, (also titled Ecclesiasticus),
Baruch, The Letter of Jeremiah, Song of the Three Young Men, Susanna, Bel and the Dragon, The
Additions to Daniel, The Prayer of Manasseh, and 1 and 2 Maccabees.
The books accepted as inspired and included in the Catholic Bible are Tobit, Judith, 1 and 2
Maccabees Wisdom of Solomon Sirach (also known as Ecclesiasticus), and Baruch
The Jews never recognized these books as being canonical (inspired). There is no record that Jesus
or the apostles ever quoted from the apocryphal books. The Septuagint (LXX) includes the books, not
as scripture, but as part of the translation of the Hebrew manuscripts as a whole.

Apollinarianism
Apollinarianism was the heresy taught by Apollinaris the Younger, bishop of Laodicea in Syria about
361. He taught that the Logos of God, which became the divine nature of Christ, took the place of the
rational human soul of Jesus and that the body of Christ was a glorified form of human nature. In
other words, though Jesus was a man, He did not have a human mind but that the mind of Christ was
solely divine. Please see Heresies for more information.

Apologetics
The word "apologetics" is derived from the Greek word "apologia," which means to make a
defense. It has come to mean defense of the faith. Apologetics covers many areas: who Jesus is, the
reliability of the Bible, refuting cults, biblical evidences in the history and archeology, answering
objections, etc. In short, it deals with giving reasons for Christianity being the true religion. We are
called by God to give an apologia, a defense: "but sanctify Christ as Lord in your hearts, always being
ready to make a defense to everyone who asks you to give an account for the hope that is in you, yet
with gentleness and reverence," (1 Pet. 3:15).
Apostasy
The falling away from the faith. It is a revolt against the truth of God’s word by a believer. It can
also describe a group or church organization that has "fallen away" from the truths of Christianity as
revealed in the Bible.

Apostle
Someone sent with a special message or commission. Jesus is called the apostle and high Priest of
our confession in Hebrews 3:1. The twelve apostles of Jesus were Simon Peter, Andrew, James the son
of Zebedee, John, Philip, Bartholomew, Thomas, Matthew, James the son of Alphaeus, Thaddaeus,
Simon the Zealot, and Judas Iscariot. Paul became an apostle after Jesus' resurrection (2 Cor. 1:1),
along with Barnabas (Acts 14:14), and others.
Apostles established churches (Rom. 15:17-20), exposed error (Gal. 1:6-9), and defended the
truth of the gospel (Phil. 1:7,17). Some were empowered by the Holy Spirit to perform Miracles (Matt.
10:1,8) and they were to preach the gospel (Matt. 28:19,20).

Archangel
In the Bible, a Greek word found only in the New Testament in two places: 1 Thess. 4:16, "For the
Lord Himself will descend from heaven with a shout, with the voice of the archangel, and with the
trumpet of God; and the dead in Christ shall rise first"; and Jude 9, "But Michael the archangel, when
he disputed with the devil and argued about the body of Moses, did not dare pronounce against him a
railing judgment, but said, 'The Lord rebuke you.'" Archangels seem to of a class of angels of great
rank and power. Apparently, there are three archangels named: Michael, Gabriel, and Lucifer.

Argumentum ad hominem
An irrelevant attack upon a person to deflect the argument from the facts and reasons.

Argumentum ad judicium
An argument where appeal is made to common sense and the judgment of people as validating a
point.

Argumentum ad populum
An argument where appeal is made to emotions: loyalties, patriotism, prejudices, etc.

Argumentum ad verecundiam
An argument using respect for great men, customs, institutions, and authority in an attempt to
strengthen one's argument and provide an illusion of proof.

Arianism
An ancient theological error that appeared around the year 320. It taught that God could not
appear on the earth, that Jesus was not eternal and could not be God. Additionally, it taught that there
was only one person in the Godhead: the Father. Jesus, then, was a creation. It was condemned by
the Council of Nicea in 325.
The Jehovah's Witness cult is an equivalent, though not exactly, of this ancient error.

Ark of the Covenant


Also called the "Ark of the Testimony," (Ex. 30:6), "Ark of God," (1 Sam. 3:3), and the "Ark of the
covenant of the Lord," (Deut. 10:8). The Ark of the Covenant was very sacred to the Ancient Jews. It
was a rectangular box made of Acacia wood about 4 x 1.5 x 1.5 feet. It was covered with gold and
was carried by poles that were inserted into rings located on the four corners. On top was a lid called
"The Mercy Seat" which had two Cherubs with outstretched wings pointing towards each other. Inside
of the Ark were the tablets of the Ten Commandments, a jar of manna, and Aaron's Rod that budded
(Heb. 9:4). It served as the symbol of the very presence of God. The Ark of the Covenant was place
in the Holy of Holies in the Tabernacle and later in the Temple. Once a year, the High Priest would
enter the Holy of Holies and sprinkle blood on the Mercy Seat. This was symbolic of the forgiveness of
the sins of the Jewish nation.
Armageddon
The word "armageddon" only occurs in Rev. 16:16. It is the location of the final great battle
between good and evil called the Great Day of God Almighty.

Arminianism
There are five main tenets of Arminianism: 1) God elects or reproves on the basis of foreseen faith
or unbelief, 2) Christ died for all men and for every man, although only believers are saved, 3) Man is
so depraved that divine grace is necessary unto faith or any good deed, 4) This grace may be resisted,
5) Whether all who are truly regenerate will certainly persevere in the faith is a point which needs
further investigation. 18 (Compare with Calvinism)

Ascension
The bodily taking up of Jesus into the heavens to be at the right hand of God the Father. The
account of Jesus' ascension is record by Luke in Acts 1:4-11.

Ascended Master
A New Age term designating a person of great learning and wisdom who lives on the astral plane.
They are not physical beings. An astral plane is another dimension of reality beyond our world and is
outside of time and space. Supposedly, Jesus, Buddha, Confuscious, etc. were Ascended Masters.

Assumption
In Catholicism, the taking of the body and soul of Mary, by God, into glory. Catholic doctrine,
apparently, does not state whether or not Mary died, but tradition holds that she died and was
immediately afterward assumed into heaven both body and soul.

Assurance
Theologically, assurance is the state of being confident in a condition or outcome. Usually it is
applied to one’s assurance of salvation. Texts often used to support assurance of salvation are John
10:28 “and I give eternal life to them, and they shall never perish; and no one shall snatch them out
of My hand,” and 1 John 5:13, “These things I have written to you who believe in the name of the Son
of God, in order that you may know that you have eternal life.” This assurance is given by the Holy
Spirit.

Astral Projection
In Eastern metaphysical and new age philosophies, astral projection is the practice of the soul
leaving the human body and traveling around this world or other planes of existence.

Astrology
A system of divination using the positions of starts, planets, and the moon, in the twelve Zodiac
signs, as a means to predict the future. One's birth date falls into a Zodia sign and this is used in
determining predictive events in relation to the astral body positions. Astrology is very prominent in
the New Age.

Atheism
This word comes from two Greek words, a the negator, and theos, God. Atheism teaches that
there is no God of any kind, anywhere, anytime. Some atheists claim to "Excercise no belief in a god"
the same way they would exercise no belief in pink unicorns. Logically, an atheist would be an
evolutionist.
The Bible teaches that all men know there is a God (Rom. 2:14-15). Therefore, they will be
without excuse (Rom. 1:20 ) on the Day of Judgment. Instead, atheists willingly suppress the
knowledge of God by their unrighteousness (Rom. 1:18-19).

Atonement
To atone means to make amends, to repair a wrong done. Biblically, it means to remove sin. The
18
This information was taken from Baker's Dictionary of Theology, ed. E. Harrison, (Baker Book House, Grand
Rapids, Michigan), 1960. p. 64.
Old Testament atonements offered by the high priest were temporary and a foreshadow of the real
and final atonement made by Jesus. Jesus atoned for the sins of the world ( 1 John 2:2). This
atonement is received by faith (Rom. 5:1; Eph. 2:8-9).
Man is a sinner (Rom. 5:8) and cannot atone for himself. Therefore, it was the love of the Father
that sent Jesus (1 John 4:10) to die in our place (1 Pet. 3:18) for our sins (1 Pet. 2:24). Because of
the atonement, our fellowship with God is restored (Rom. 5:10). (See Reconciliation.)

Aura
A term used frequently used by New Agers to describe a field of energy emitted by every living
thing, particularly people. This aura takes on different colors depending on the person's mood, state
of wellness, etc., and some New Age practitioners claim the ability to interpret the colors and diagnosis
various conditions and states of mind of the person with the aura. See also, Chakra.

Autograph
An original writing of a biblical document. The original manuscript written. The autographs would
be the actual, original written document from which copies are made.

Autonomy
Freedom from all external constraints. Independence consisting of self-determination.

Baal
A Canaanite god. The word means "lord" or "husband." He was a god of weather, associated with
thunder, who appointed the times of the rains, and was considered to be the son of the pagan god
Dagon. The ancient Jews were often tempted to follow Baal because so much of their lives depended
upon the rain that fed the crops.

Babel
The Hebrew name for Babylon, one of the cities founded by Nimrod. In Gen. 11:1-9, it is the
location where the entire world spoke a single language and worked together to build a tower into the
heavens. It was representative of the great pride of man. God confused their languages. The
building of the tower ceased and they dispersed.

Babel, Tower of
The tower built the builders at Babel constructed which became a symbol of their defiance against
God (Gen. 11:1-6). It was probably modeled after a ziggurat that is a mound of sun-dried bricks and
was probably constructed before 4,000 BC.

Baptism
An immersion or sprinkling of water that signifies one's identification with a belief or cause. In
Christianity it is the believer's identification with Christ in His death, burial, and resurrection ( Rom.
6:4-54). It is done in the name and authority (Acts 4:7) of Christ with the baptismal formula of Father,
Son, and Holy Spirit (Matt. 28:19). It does not save us (1 Pet. 3:21). However, it is our obligation, as
believers, to receive it.
Some maintain that baptism is necessary for salvation. It is not. If you want to read more on this
see Is Baptism Necessary for Salvation?

Baptismal Regeneration
The belief that baptism is essential to salvation, that it is the means where forgiveness of sins is
made real to the believer. This is incorrect. Paul said that he came to preach the gospel, not to baptize
(1 Cor. 1:14-17). If baptism were essential to salvation, then Paul would have included it in his
standard practice and preaching of the salvation message of Jesus, but he did not. (See also Col. 2:10-
11.) For more information on this see Is Baptism Necessary for Salvation?

Bible
A book or collection of sacred writings. The term "bible" is best known in reference to the Christian
Scriptures consisting of the both the Old and New Testaments. The word comes from the Greek,
biblios, meaning "book."

Blasphemy
Speaking evil of God or denying Him some good which we should attribute to Him. Blasphemy of
the Holy Spirit is stating that Jesus did his miracles by the power of the devil ( Matt. 12:22-32) and is
an unforgivable sin (Mark 3:28-30). Blasphemy arises out of pride (Psalm 73:9,11), hatred (Psalm
74:18), injustice (Isaiah 52:5), etc. Christ was mistakenly accused of blasphemy (John 10:30-33).

Book of Life
A book kept by God with the list of names of people who will escape God's wrath (Psalm 69:28;
Rev. 21:27). Those who names are not in the book of Life are cast into hell (Rev. 20:15).

Born Again
The new birth enjoyed by a Christian upon his conversion and regeneration. It is a work of the
Holy Spirit within a believer. It is related to faith in Christ and Him crucified (John 3:3-5). It means
that the person is no longer dead in sins (Eph. 2:1), no longer spiritually blind (1 Cor. 2:14), and is
now a new creation in Christ Jesus (2 Cor. 5:17).

Calvinism
A system of Christian interpretation initiated by John Calvin. It emphasizes predestination and
salvation. The five points of Calvinism were developed in response to the Arminian position (See
Arminianism). Calvinism teaches: 1) Total depravity: that man is touched by sin in all parts of his
being: body, soul, mind, and emotions, 2) Unconditional Election: that God’s favor to Man is
completely by God’s free choice and has nothing to do with Man. It is completely undeserved by Man
and is not based on anything God sees in man (Eph. 1:1-11), 3) Limited atonement: that Christ did
not bear the sins of every individual who ever lived, but instead only bore the sins of those who were
elected into salvation (John 10:11,15), 4) Irresistible grace: that God's call to someone for salvation
cannot be resisted, 5) Perseverance of the saints: that it is not possible to lose one's salvation ( John
10:27-28).

Canon
This is another word for scripture. The Canon consists of the 39 books of the Old Testament and
the 27 books of the New. The Canon is closed which means there is no more revelation to become
Scripture.

Capital sins
In Catholicism, the seven causes of all sin: pride, covetousness, lust, anger, gluttony, envy, sloth

Catholic
Universal, the entire Christian Church. Often applied to the Roman Catholic Church. Actually, the
word simply means universal.

Causality
The relationship between cause and effect. The principle that all events have sufficient causes.

Chakra
In Yoga, an alleged focal point of energy in the human body with its own frequency. It is a Hindi
term. Supposedly, there are seven chakras located in the head, throat, chest, abdomen, groin, etc.
Chakras are often utilized in New Age practices. See also, Aura.
Channeling
A new age practice where a person goes into an altered state of consciousness and allows another
spirit or entity to enter him/her in order to communicate to us. According to New Age teaching, this
spirit or entity can be from another dimension, the spirit realm, or from another part of the galaxy or
universe.

Charismatic Gifts
The special spiritual gifts given to the church. They are for edifying and building up the church.
They are mentioned in Rom. 12, 1 Cor. 12, and 1 Cor. 14: Word of wisdom, word of knowledge, faith,
healing, miracles, prophecy, distinguishing of spirits, tongues, interpretation of tongues.

Chiliasm
Also known as millennialism. The belief that there is a future 1000 year reign of Christ where
perfect peace will reign and the Lord Jesus will be King on earth.

Christ
Christ is a title. It is the N.T. equivalent of the O.T. term "messiah" and means "anointed one." It is
applied to Jesus as the anointed one who delivers from sin. Jesus alone is the Christ. As the Christ He
has three offices: Prophet, Priest, and King. As a Prophet He is the mouthpiece of God ( Matt. 5:27-28)
and represents God to man. As Priest He represents man to God and restores fellowship between them
by offering Himself as the sacrifice that removed the sin of those saved. As King He rules over His
kingdom. By virtue of Christ creating all things (John 1:3; Col. 1:16-17), He has the right to rule.
Christ has come to do the will of the Father (John 6:38), to save sinners (Luke 19:10), to fulfill the
O.T. (Matt. 5:17), to destroy the works of Satan (Heb. 2:14; 1 John 3:8), and to give life (John
10:10,28). Christ is holy (Luke 1:35), righteous (Isaiah 53:11), sinless (2 Cor. 5:21), humble (Phil.
2:5-8), and forgiving (Luke 5:20; 7:48; 23:34).

Christian
The word "Christian" comes from the Greek word christianos which is derived from the word
christos, or Christ, which means "anointed one." A Christian, then, is someone who is a follower of
Christ. The first use of the word "Christian" in the Bible is found in Acts 11:26, "And the disciples were
called Christians first in Antioch." It is found only twice more in Acts 26:28 and 1 Pet. 4:16. However,
it is important to note that it is the true Christ that makes someone a Christian, not the Mormon one
(brother of the devil), or the JW one (Michael the Archangel), the New Age Jesus (a man in tune with
the divine Christ Consciousness), etc. The true Christ is God in flesh (John 1:1,14; 20:28; Col. 2:9;
Phil. 2:5-8; Heb. 1:8): Jesus.

Christology
The study of Christ (Jesus) as revealed in the Bible. Some of the issues studied are: 1) His deity,
2) His incarnation, 3) His offices (See Christ), 4) His sacrifice, 5) His resurrection, 6) His teaching, 7)
His relation to God and man, and 8) His return to earth.

Church
The word is used in two senses: the visible and the invisible church. The visible church consists of
all the people that claim to be Christians and go to church. The invisible church is the actual body of
Christians; those who are truly saved.
The true church of God is not an organization on earth consisting of people and buildings, but is
really a supernatural entity comprised of those who are saved by Jesus. It spans the entire time of
man's existence on earth as well as all people who are called into it. We become members of the
church (body of Christ) by faith (Acts 2:41). We are edified by the Word (Eph. 4:15-16), disciplined by
God (Matt. 18:15-17), unified in Christ (Gal. 3:28), and sanctified by the Spirit (Eph. 5:26-27).

Circumcision
An operation (note the shedding of blood) that entered one into the covenant in O.T. times. It was
instituted by God (Gen. 17:10-14) and performed on the eighth day after birth (Luke 1:59). It was a
sign of the covenant God made with Abraham (Gen. 17:12; Rom. 4:11). In the N.T. the physical
operation is not practiced. Instead, a circumcision of the heart of the Christian is taught ( Rom. 2:29;
Col. 2:11-12). This is the true circumcision (Rom. 2:29).

Codex
An early book form made from papyri leaves cut, folded, and sewn together in the middle to make
a book. First used in the 2nd century.

Common Grace
The grace of God given to the creation as a whole. God still allows the sun to shine upon the
unsaved. He feeds them, allows them to work, and have joy. It is common grace that "restrains" the
wrath of God until a later time. It is in special grace that salvation is given to the Christians.

Communion
The Lord's Supper (Matt. 26:26-30; Mark 14:22-26; Luke 22:14-20; 1 Cor. 1:23-26). It is the
breaking of bread (Acts 2:42,46) and a time to give thanks (Luke 22:17,19). It was originally
instituted by Jesus (Matt. 26:26-29) on the night of the Passover meal which was an annual
occurrence celebrating the "passing over" of the angel of death that claimed the firstborn of every
house in Egypt (Exodus 12). The Lord's Supper, or communion, replaces the Passover meal with the
"body and blood," (Mark 14:22-24) of Jesus. It is to be taken only by believers (1 Cor. 11:23-28). (For
further study see John 6:26-58 and 1 Cor. 11:27-34).

Condemnation
Declaring an evildoer to be guilty; the punishment inflicted. Without Jesus we stand condemned
before God not only because of the sin of Adam (Rom. 5:16-18) but also because of our own sin (Matt.
12:37). However, "There is therefore now no condemnation for those who are in Christ Jesus. For the
law of the Spirit of life in Christ Jesus has set you free from the law of sin and of death," (Rom. 8:1-2).
Christians have passed out of condemnation because they are forgiven in Christ.

Conditional immortality
The view that immortality is given only to those Christians who believe in Christ. The rest are
destroyed and do not exist. Some adherents to conditional immortality believe that the wicked will be
punished in hell for a period proportional to their sins and then they are annihilated.

Confession
The act of disclosing one's sins. In Catholicism, it is telling sins to a priest and the Lord forgives
the person through the priest. Biblically, confession of sins is done to the one offended without the
mediatorship of a priest.

Confessional
In Catholicism, a small compartment where the priest hears the confessed sins of a sinner.

Confirmation
In Catholicism, a ceremony performed by a bishop that is supposed to strengthen a person and
enable him to resist sin. It is usually done at the age of 12. The Bishop dips his right thumb in holy oil
and anoints the person on the forehead by making the sign of the cross and says, "Be sealed with the
gift of the Holy Spirit."

Consubstantiation
It means an inclusion of one substance in another where the body and blood of Christ co-exist in
the elements of the Supper. It suggests that a third substance is formed. The body and blood of
Christ are "in, with, and under" the elements. There is no permanent relationship with the elements.
Instead, the association is limited to the sacramental action. The transformation is effected by the
Word of God and not by a priest.
Contrition
In Catholicism, extreme sorrow for having sinned with a deep repentance concerning that sin.

Conversion
Turning from evil to God. God converts (Acts 21:19) the unsaved into the saved, from the
unregenerate to the regenerate. It is produced through the preaching of the gospel (Rom. 10:14; 1
Cor. 15:1-4) and results in repentance (Acts 26:20) and a new creation (2 Cor. 5:17). The fruits of
conversion are listed in Gal. 5:22-23.

Conviction
The work of the Holy Spirit where a person is able to see himself as God sees him: guilty, defiled,
and totally unable to save himself (John 16:8). Conviction of the Holy Spirit of an unbeliever reveals
sinfulness and guilt and brings fear. Conviction of the Holy Spirit of the believer brings an awareness of
sin and results in confession and cleansing. This conviction is produced by the Holy Spirit ( John 16:8),
the Gospel (Acts 2:37), the conscience (Rom. 2:15), and the Law (James 2:9). Conviction of our sins
brings us to the cross. It shows us our need for forgiveness.

Coptic
The Afro-Asiatic language of the Copts, which survives only as a liturgical language of the Coptic
Church.

Cosmological argument
An attempt to prove that God exists by appealing to the principle that all things have causes.
There cannot be an infinite regress of causes, therefore, there must be an uncaused cause: God.

Cosmology
The study of the origin and structure of the universe.

Covenant
An agreement between two parties. The agreement, according to Ancient Near East custom,
consists of five parts: 1) Identification of parties, 2) Historical prologue where the deeds establishing
the worthiness of the dominant party is established, 3) Conditions of the agreement, 4) Rewards and
punishments in regard to keeping the conditions, and 5) Disposition of the documents where each
party receives a copy of the agreement (e.g. the two tablets of stone of the 10 Commandments).
Ultimately, the covenants God has made with man result in our benefit. We receive eternal
blessings from the covenant of grace. (For further study see Gen. 2:16-17; 9:1-17; 15:18; Gen. 26:3-
5; Gal. 3:16-18; Luke 1:68-79; Heb. 13:20).

Covenant Theology
A system of theology that views God's dealings with man in respect of covenants rather than
dispensations (periods of time). It represents the whole of scripture as covenantal in structure and
theme. Some believe there is one Covenant and others believe two and still others believe in more.
The two main covenants are covenant of works in the O.T. made between God and Adam, and the
Covenant of Grace between the Father and the Son where the Father promised to give the Son the
elect and the Son must redeem them. Some consider these to be one and the same. The covenants
have been made since before the world was made (Heb. 13:20).

Creation
Everything that exists except God himself. This includes material as well as immaterial things and
time. God is the creator, (Heb. 11:3) we are the creatures. The creator/creature distinction must be
maintained to properly remain in humble relationship with God. We are not God, cannot create, nor
can we help ourselves do good in order to be saved. Only God is God. Only He can create. And, only
He has the ability to save man.
Cult
A religious group that follows a particular theological system. In the context of Christianity, and in
particular, CARM, it is a group that uses the Bible but distorts the doctrines that affect salvation
sufficiently to cause salvation to be unattainable. A few examples of cults are Mormonism, Jehovah's
Witnesses, Christian Science, Christadelphians, Unity, Religious Science, The Way International, and
the Moonies. (See also Cults)

Damnation
The righteous judgment of God upon a sinner where the sinner is cast out of the presence of God
into Hell for ever.

Death
The word "death" is used in two main ways in the Bible. First, it is used to describe the cessation
of life. Second, death is used in reference to the lost. This refers to their eternal separation from God
as a result of sin (Isaiah 59:2), in a conscious state of damnation without hope (1 Thess. 4:13; Rev.
20:10,14,15).
Death to humans is unnatural. When God created Adam and Eve, death was not part of the
created order. It was not until they sinned that death entered the scene (Rom. 5:12; 6:23). Death will
be destroyed when Christ returns and the believers receive their resurrected bodies.

Decalogue
The Ten Commandments found in Exodus 20. Deca means ten in Latin. Logue comes from "logos"
which means "word."

Decrees, of God
The Decrees of God is His eternal purpose, according to His will, whereby He has foreordained
whatever comes to pass. His Decrees do not negate the responsibility of people for their sins nor does
it mean that God is responsible for sin. But, it necessarily is true that God knows all things actual as
well as potential, and that that which exists, exists due to His creative effort. It also follows that God
has eternally known all events that have occurred, are occurring, and will occur in this creation
including the fall, redemption, glorification, etc. Yet, God is not the one responsible for the sin in the
world but has decreed, by His permission, that it be allowed to exist. Isaiah 46:9-10 says,
"Remember the former things long past, for I am God, and there is no other; I am God, and there is
no one like Me, 10Declaring the end from the beginning and from ancient times things which have not
been done, saying, ‘My purpose will be established, and I will accomplish all My good pleasure."
God's efficacious decrees are those decrees which God has purposed and determined to occur, i.e.,
Acts 2:23, "this Man, delivered up by the predetermined plan and foreknowledge of God, you nailed to
a cross by the hands of godless men and put Him to death." God's permissive decrees are those
decrees where He permits things to occur such as evil.

Deduction
A system of logic, inference and conclusion drawn from examination of facts. Conclusions drawn
from the general down to the specific.

Depravity
Moral corruption, a state of corruption or sinfulness. Total depravity is the teaching that sin has
touched all aspects of the human: body, soul, spirit, emotions, mind, etc.

Deism
The belief that God exists but is not involved in the world. It maintains that God created all things
and set the universe in motion and is no longer involved in its operation. (Compare to Atheism,
Agnosticism, and Theism.)
Demon
A fallen angel that assists Satan in the opposition of God. Demons are evil ( Luke 10:17,18),
powerful (Luke 8:29), and under the power of Satan (Matt. 12:24-30). They recognized Christ (Mark
1:23,24) and can possess non-Christians (Matt. 8:29).

Deontology
The study of moral obligation.

Determinism
The teaching that every event in the universe is caused and controlled by natural law; that there is
no free will in humans and that all events are merely the result of natural and physical laws.

Devil
Greek is "diabolos," which means accuser. The greatest of all the fallen angels. He opposes God
and is completely evil. He is often called Lucifer which is a Latin translation of "light bearer" found in
Isaiah 14:12, and also the accuser of the brethren in (Rev. 12:10), dragon (Rev. 12:9), the devil
(Matt. 4:1), the tempter (Matt. 4:3), the accuser (Rev. 12:10), the prince of demons (Luke 11:15),
the ruler of this world (John 12:31), See Isaiah 14:12-15 for a description of the fall of the devil.
Upon Jesus' return, the Devil will be vanquished -- depending on the eschatological position. His
future is the eternal lake of fire.

Dialectic
The practice of examining ideas and beliefs using reason and logic. It is often accomplished by
question and answer.

Dichotomy
The teaching that a human consists of two parts: body and soul. Sometimes the soul is also
referred to as spirit. (See Trichotomy)

Didactics
The branch of education dealing with teaching.

Diocese
In Catholicism, an area of many parishes presided over by a bishop.

Disciple
A pupil or follower of a religion, a person, or a movement. As Christians we are to be disciples of
Jesus (Luke 14:26,27). We follow in the teaching and example of what He said and did. A disciple is a
convert but not all converts are disciples. As disciples we are to bear our cross daily (Matt. 16:24).
This means to live and die for Him if necessary (Matt. 16:25).

Dispensation, dispensationalism
In the Scofield Reference Bible a dispensation is "a period of time during which man is tested in
respect of obedience to some specific revelation of the will of God" Dispensationalism says that God
uses different means of administering His will and grace to His people. These different means coincide
with different periods of time. Scofield says there are seven dispensations: of innocence, of
conscience, of civil government, of promise, of law, of grace, and of the kingdom. Dispensationalists
interpret the scriptures in light of these (or other perceived) dispensations. Compare to Covenant.

Divination
The practice of predicting the future and/or securing for a person the services of supernatural
powers. It is also used as a means to gain knowledge.

Divinity
The nature or quality of being God. It belongs to God alone. Jesus was divine in nature ( Col. 2:9)
as well as being a man.
Docetism
Docetism was an error with several variations concerning the nature of Christ. Generally, it taught
that Jesus only appeared to have a body, that he was not really incarnate, (Greek, "dokeo" = "to
seem"). This error developed out of the dualistic philosophy that viewed matter as inherently evil, that
God could not be associated with matter, and that God, being perfect and infinite, could not suffer.
Please see Heresies for more information.

Doctrine
A set of accepted beliefs held by a group. In religion, it is the set of true beliefs that define the
parameters of that belief system. Hence, there is true doctrine and false doctrine relative to each belief
set. In Christianity, for example, a true biblical doctrine is that there is only one God in all existence
(Isaiah 43:10; 44:6,8). A false doctrine is that there is more than one God in all existence.

Dogma
A generally held set of formulated beliefs.

Donatism
Donatism was the error taught by Donatus, bishop of Casae Nigrae that the effectiveness of the
sacraments depends on the moral character of the minister. In other words, if a minister who was
involved in a serious enough sin were to baptize a person, that baptism would be considered invalid.
Please see Heresies for more information.

Dowsing
A form of divination using a forked rod or bent wire often used to find objects, people, or things. A
person holds the dowsing instrument and is "lead" around by it until the object is, allegedly, found.

Dualism
In theology, the concept that the world is controlled by two opposing forces, i.e., good and bad,
God and Satan. In Philosophy the idea that the world consists of two main components: thought and
matter.

Ecclesiology
The study of the Christian church, its structure, order, practices, and hierarchy.

Edify
To build up. In the Christian context it means to strengthen someone, or be strengthened, in
relationship to God, the Christian walk, and holiness. As Christians, we are to "let all things be done
for edification," (1 Cor. 14:26). We are edified by the Word of God (Acts 20:32) and by love (1 Cor.
8:1). (See also Rom. 14:19; Eph. 4:29 and 1 Cor. 3:1-4; James 4:1-6).

Efficacy
Producing a result. Christ's atonement was efficacious; it produced the result of forgiveness of sins
for the elect. The atonement is efficacious grace in action.

Eisegesis
Eisegesis is when a person interprets and reads information into the text that is not there. An
example would be in viewing 1 Cor. 8:5 which says, "For though there be that are called gods,
whether in heaven or in earth, (as there be gods many, and lords many," (kjv). With this verse,
Mormons, for example, bring their preconceived idea of the existence of many gods to this text and
assert that it says there are many gods. But that is not what it says. It says that there are many that
are called gods. Being called a god doesn't make it a god. Therefore, the text does not teach what
the Mormons say and they are guilty of eisegesis; that is, reading into the text what it does not say.
See also exegesis.
Elect, Election
The elect are those called by God to salvation. This election occurs before the foundation of the
world (Eph. 1:4) and is according to God's will not man's (Rom. 8:29-30; 9:6-23) because God is
sovereign (Rom. 9:11-16). The view of election is especially held by Calvinists who also hold to the
doctrine of predestination.

Empiricism
The proposition that the only source of true knowledge is experience. It is the search for
knowledge through experiment and observation. Denial that knowledge can be obtained a priori.

Epistemology
The branch of philosophy that deals with the area of knowledge, its source, criteria, kinds, and the
relationship between what is known and the one who is knowing it.

Eschatology
The study of the teachings in the Bible concerning the end times, or of the period of time dealing
with the return of Christ and the events that follow. Eschatological subjects include the Resurrection,
Resurrection, the Rapture, the Tribulation, the Millennium, the Binding of Satan, the Three witnesses,
the Final Judgment, Armageddon, and The New Heavens and the New Earth. In the New Testament,
eschatological chapters include Matt. 24, Mark 13, Luke 17, and 2 Thess. 2. In one form or another
most of the books of the Bible deal with end times subjects. But some that are more prominently
eschatological are Daniel, Ezekiel, Isaiah, Joel, Zechariah, Matthew, Mark, Luke, 2 Thessalonians, and
of course Revelation. (See Amillennialism and Premillennialism for more information on views on the
millennium.)

Eternal life
Life everlasting in the presence of God. "This is eternal life, that they may know Thee the only true
God, and Jesus Christ, whom Thou has sent," (John 17:3).
There are two senses in which this is used. First, as Christians we possess eternal life (1 John
5:13), yet we are not in heaven or in the immediate presence of God. Though we are still in mortal
bodies and we still sin, by faith we are saved (Rom. 4:5; Eph. 2:8-9) and possess eternal life as a free
gift from God (Rom. 6:23). Second, eternal life will reach its final state at the resurrection of the
believers when Christ returns to earth to claim His church. It is then that eternal life will begin in its
complete manifestation. We will no longer sin.

Eternal Security
The doctrine that salvation cannot be lost. Since it is not gained by anything we do, it cannot be
lost by anything we do. This does not mean that we can sin all we want (Rom. 6:1-2) because we have
been freed from sin and are set apart for holy use (1 Thess. 4:7). (See Antinomianism.)

Ethics
The study of right and wrong and wrong, good and bad, moral judgment, etc.

Eucharist
The elements of the communion supper in Christian Churches where the bread and wine are
consumed as a representation the sacrifice of Christ. They correspond, representatively, as the body
and blood of Christ.

Eutychianism
This is similar to Monophycitism. It states that Christ's natures were so thoroughly combined -- in
a sense scrambled together -- that the result was that Christ was not really truly able to relate to us as
humans. The problem is this implies that Jesus was not truly God nor man. Therefore, He would be
unable to act as mediator and unable to truly atone for our sins. (See Hypostatic Union, which is the
correct view of Christ's two natures, and also Nestorianism and Monophycitism which are the incorrect
views of Christ's two natures.)
Evil
Moral rebellion against God. It is contrary to the will of God. There is natural evil (floods, storms,
famines, etc.) and moral evil (adultery, murder, idolatry, etc.). Natural evil is a result of moral evil.
Adam's sin resulted in sin entering the world allowing floods, storms, famines, etc. Evil originated with
Satan (Isaiah 14:12-15) and is carried on by man (Matt. 15:18-19). (See Theodicy.)

Evolution
Though you might not expect to find the subject of evolution in a dictionary of theology, it is
appropriate if you consider that the theory of evolution requires faith. The evidence for evolution is
actually quite weak. There are numerous difficulties facing it and, the theory has undergone many
changes since its inception in the 1800's. It is the theory that over an incredible duration of time, life
developed from random combinations of non-organic materials. This life was improved upon through
mutations and the process of natural selection. The Scriptures do not speak about evolution but
instead negate the theory by stating that God created all things (Gen. 1). See Evolution for more
information.

Excommunication
The act of discipline where the Church breaks fellowship with a member who has refused to repent
of sins. Matt. 18 is generally used as the model of procedures leading up to excommunication. Those
excommunicated are not to partake in the Lord’s supper.
In the Bible, serious offenders of God’s law, who were supposed to be Christian, were "delivered
over to Satan for the destruction of the flesh," (1 Cor. 15:5; 1 Tim. 1:20). However, upon repentance,
the person is welcomed back into fellowship within the body of Christ.Exegesis
Exegesis is when a person interprets a text based solely on what it says. That is, he extracts out of
the text what is there as opposed to reading into it what is not there (eisegesis). There are rules to
proper exegesis: read the immediate context, related themes, word definitions, etc., that all play a
part in properly understand what something does say and not what it does not say.

Existentialism
A philosophical viewpoint that emphasis human freedom and abilities. Therefore, subjectivity and
individual choice are elevated often above conceptual and moral absolutes.

Expiation
The cancellation of sin. Expiation and propitiation are similar but expiation does not carry the
implication of dealing with wrath, of appeasing it through a sacrifice. Generally speaking, propitiation
cancels sin and deals with God's wrath. Expiation is simply the cancellation of sin. Jesus was our
propitiation (1 John 2:2; 4:10 -- "atoning sacrifice" in the NIV).

Faith
"Now faith is the assurance of things hoped for, the conviction of things not seen," (Heb. 11:1). It
is synonymous with trust. It is a divine gift (Rom. 12:3) and comes by hearing the Word of God (Rom.
10:17). It is the means by which the grace of God is accounted to the believer who trusts in the work
of Jesus on the cross (Eph. 2:8). “Without faith it is impossible to please God,” (Heb. 11:6). It is by
faith that we live our lives, "The righteous shall live by faith," (Hab. 2:4; Rom. 1:17).

Fall, The
The fall is that event in the Garden of Eden where Adam and Eve disobeyed the command of God
and ate of The Tree of the Knowledge of Good and Evil (Gen. 2 and 3). Since Adam represented all of
mankind, when He sinned, all of mankind fell with Him (Rom. 5:12).

False Prophet, (The)


The second beast of Revelation (Rev. 13:11-18). He is a person who will manifest himself near the
culmination of this epoch shortly before the physical return of Christ. He will be a miracle worker and
during the Tribulation period will bring fire down from heaven and command that people worship the
image of the Beast (Rev. 11:15). See also (13:16-17).
Jesus warned about false prophets in Matt. 24:24 stating that in the last days many false prophets
would arise and deceive, if possible, even the elect. False prophets teach false doctrine and lead
people away from the true gospel message and teaching of God found in the Bible. Examples of
modern day false prophets are Joseph Smith (Mormonism), Charles Taze Russell (Jehovah's
Witnesses), Mary Baker Eddy (Christian Science), etc. Each of them distorts the truth sufficient to
cause damnation.

Falsifiability
The ability of something to be proven false. A non-falsifiable statement would be, "There is a
green lizard sitting in a rocking chair on the fourth largest moon of Jupiter." This statement is not
falsifiable in that it cannot be proven false because it cannot be verified or denied. Jesus' resurrection
was falsifiable in that all the critics had to do was produce the body, but they did not. Falsifiability,
generally, is a test of the validity of a belief or occurrence. Something that is not falsifiable can be
said to be untrue since it cannot be confirmed or denied.

Fast, Fasting
Depriving oneself of food for a period of time for a specific purpose, often spiritual. It is the
"weakening" of the body in order to "strengthen" the spirit. It is interesting to note that sin entered
the world through the disobedience of eating (Gen. 3:6). We are called to fast in the N.T. (Matt. 6:16).
(See also 1 Kings 21:27; Psalm 35:13; Acts 13:3; 2 Cor. 6:5).

Fatalism
The idea that all things are predetermined to occur and that there is no ability of the person to alter
the predetermined plan of God in any event. This is not the correct biblical view. The Bible teaches us
that we can influence God with our prayers (James 5:16). How this influence is worked out by God
who knows all things from eternity is something apparently unexplainable in Christianity.

Fellowship
There is no specific definition given in the N.T. But we are called into fellowship with one another
(1 John 1:3, with Jesus (1 Cor. 1:9), with the Father (1 John 1:3), and with the Holy Spirit (2 Cor.
13:14). Fellowship implies sharing common interests, desires, and motivations. Fellowship requires
that time be spent with another communicating, caring, etc. It carries with it a hint of intimacy. As
Christians we fellowship with one another because of our position in Christ, because we are all
redeemed and share an intimate personal knowledge of Jesus. We share a common belief (Acts 2:42),
hope (Heb. 11:39-40), and need (2 Cor. 8:1-15).
The Greek word for fellowship is koinonia. This word is also translated communion in 1 Cor. 10:16
in the KJV. This is where we get the term the communion supper.

Fideism
The position that religious doctrines rest not on reason, but only on faith.

Filioque
The doctrine that the Holy Spirit proceeds equally from both the Father and the Son.

Firstborn
The first of the mother's offspring. It stands figuratively for that which is most excellent. The
firstborn male of the family carried certain familial rites and privileges (Gen. 27:1-29; 48:13-14) and
was given a double portion of the inheritance (Deut. 21:17). The term is also applied to Christ as the
pre-eminent one and the first one raised from the dead (Col. 1:15,18). It does not mean first created
as Jehovah's Witnesses believe. In fact, the firstborn rites were transferable. Compare Jer. 31:9 with
Gen. 41:50-52.

Forgiveness
There are seven words in Scripture that denote the idea of forgiveness: three in Hebrew and four
in Greek. No book of religion except Christianity teaches that God completely forgives sins. God
remembers our sins no more (Heb. 10:17). God is the initiator of forgiveness (Col. 2:13).
There is only one sin for which the Father does not promise forgiveness: blasphemy against the
Holy Spirit (Mark 3:28; Matt. 12:32). The contexts suggest this to be the sin of attributing to unclean
spirits the work of the Holy Spirit.
For man to receive forgiveness, repentance is necessary (Luke 17:3-4). For the holy God to
extend forgiveness, the shedding of blood is necessary (Heb. 9:22; Lev. 17:11). Forgiveness is based
upon the sacrifice of Christ on the cross.

Fool
Hater of God. One who is morally weak, who misuses what God has given him for selfish purposes.
He is lustful (Prov. 7:22), lazy (Ecc. 10:15), does not fear God (Prov. 14:1), hates knowledge (Prov.
1:22), and is self-righteous (Prov. 12:15). As Christians, we are to avoid foolishness (Eph. 5:4). (See
Ecc. 7:25; Prov. 3:35, 10:8.)

Foreknow, Foreknowledge
It is God's knowledge about things that will happen. Past, present, and future are all "present" in
the mind of God. He inhabits eternity (Isaiah 57:15). God has infinite knowledge (Isaiah 41:22,23)
and knows all things in advance. In the N.T. it does not always mean "to know beforehand" but also to
cause to be. See 1 Pet. 1:2,20.

Free Knowledge
The free act of God’s will where, after His free act of creation, He knows all things that are going to
happen and that this knowledge is contingent upon His free creative will. Therefore, the free
knowledge of God would be different if He had chosen a different creative fiat. In other words, because
God created one possible existence instead of another, the range of His knowledge regarding actual
existence would have been different had He created something different in the first place. (See also
Natural knowledge and Middle Knowledge.)

Free will
Freedom of self determination and action independent of external causes.
Freethinker
A person who forms his opinions about religion and God without regard to revelation, scripture,
tradition, or experience.
Gehenna
Originally, a location southwest of Jerusalem where children were burned as sacrifices to the god
Molech. It later became a garbage dump with an continuous burning of trash. Therefore, it was used
biblically, to illustrate the abode of the damned in Christian and Jewish theology. Gehenna is
mentioned in Mark 9:43ff and Matt. 10:28 as the place of punishment of unquenchable fire where both
the body and soul of the wicked go after death. It is apparently the future abode of Satan and his
angels (Matt. 25:41).

Gentile
Those who are not Jews. Gentiles were used by God to punish apostate Judea (Deut. 28:49; 1
Kings 8:33) and often included in blessings by God upon the Jewish people. "Gentiles" is often used
biblically in reference to nations.

Gifts, Spiritual Gifts


Spiritual abilities given by God for the purpose of building up the church. Every Christian has at
least one (1 Cor. 7:7). They are listed and discussed in different places in the N.T. (Rom. 12:6-8; 1
Cor. 12:4-11, 28-30; Eph. 4:7-12). Following is a list of the gifts arranged in two groups. The first are
gifts that require supernatural intervention and are possessed only by true Christians. The second are
gifts that do not require supernatural intervention. Even non-Christians can have the second group of
gifts. A further issue is whether or not the gifts are still in use today. Some believe they ceased with
the apostles and the closing of the Canon (the completion of the writings of the Bible) and they are no
longer needed for the building up of the body of Christ (Eph. 4:12). Others believe the gifts are still in
use but not in the pure apostolic sense. In other words, they are still in use but not in the same way
possessed by the apostles. Instead, they are available to the believer if and when God decides it is
beneficial to use them.
Spiritual Gifts
1 Salvation Rom. 6:23
2 Word of Wisdom 1 Cor. 12:8
3 Word of Knowledge 1 Cor. 12:8
4 Faith 1 Cor. 12:9
5 Healing 1 Cor. 12:9
6 Miracles 1 Cor. 12:10
7 Prophecy Rom. 12:6 1 Cor. 12:10
8 Distinguishing of Spirits 1 Cor. 12:10
9 Tongues 1 Cor. 12:10
10 Interpretation of Tongues 1 Cor. 12:10

1 Serving Rom. 12:7


2 Teaching Rom. 12:7
3 Exhortation Rom. 12:8
4 Giving Rom. 12:8
5 Leading Rom. 12:8
6 Showing mercy Rom. 12:8

Gnosticism
A theological error prevalent around the time of Christ. Generally speaking, Gnosticism taught that
salvation is achieved through special knowledge (gnosis). This knowledge usually dealt with the
individual's relationship to the transcendent Being. It denies the incarnation of God as the Son. In so
doing, it denies the true efficacy of the atonement since, if Jesus is not God, He could not atone for all
of mankind and we would still be lost in our sins.

God
The supreme being of the universe. He is the creator of all things (Isaiah 44:24). He alone is God
(Isaiah 45:21,22; 46:9; 47:8). There have never been any Gods before Him nor will there be any after
Him (Isaiah 43:10). God is God from all eternity (Psalm 90:2). In Exodus 3:14, God revealed His name
to His people. The name commonly known in English is Jehovah. This comes from the four Hebrew
consonants that spell the name of God. (See Tetragrammaton.)
God is a Trinity, knows all things (1 John 3:20), can do all things (Jer. 32:17,27 - except those
things against His nature like lie, break His word, cheat, steal, etc.), and is everywhere all the time
(Psalm 119:7-12).

Gods, False
Gods that are not real, but invented by men or inspired by demons the purpose of which is to
deceive people so they do not believe in the true and living God. Some of the false gods listed in the
Bible are Adrammelech and Anammelech (2 Kings 17:31), Asherah (1 Kings 15:13; 18:19), Ashtoreth
(1 Kings 11:5,33), Baal (1 Kings 14:23; 2 Kings 23:7), Baalzebub (2 Kings 1:1-16); Luke 11:19-23),
Dagon (Judges 16:23-30), Molech/Moloch (Lev. 18:21; 20:1-5), Rimmon (2 Kings 5:18, and Tammuz
(Ezekiel 8:14).

Gospel
The Gospel is the good news that we have forgiveness of sins though Jesus. Specifically, the
gospel is defined by Paul in 1 Cor. 15:1-4: "Now I make known to you, brethren, the gospel which I
preached to you, which also you received, in which also you stand, by which also you are saved, if you
hold fast the word which I preached to you, unless you believed in vain. For I delivered to you as of
first importance what I also received, that Christ died for our sins according to the Scriptures, and that
He was buried, and that He was raised on the third day according to the Scriptures."
The gospel comes from God (Gal. 1:10-12), is the power of God for salvation (Rom. 1:16), is a
mystery (Eph. 6:19), and is a source of hope (Col. 1:23), faith (Acts 15:7), life (1 Cor. 4:15), and
peace (Eph. 6:15).
Grace
Grace is unmerited favor. It is God's free action for the benefit of His people. It is different than
Justice and Mercy. Justice is getting what we deserve. Mercy is not getting what we deserve. Grace is
getting what we do not deserve. In grace we get eternal life, something that, quite obviously, we do
not deserve. But because of God's love and kindness manifested in Jesus on the Cross, we receive the
great blessing of redemption.
Grace is God's Riches At Christ's Expense. Grace rules out all human merit. It is the product of
God, that is given by God, because of who He is not because of who we are. It is the means of our
salvation (Eph. 2:8-9). We are no longer under the Law, but under grace (Rom. 6:14). (See 1 Cor.
15:11; Rom. 5:2, 15-20; 2 Cor. 12:9; and 2 Cor. 9:8).

Great White Brotherhood


A New Age term designating the spiritual organization of Ascended Masters (great spiritual leaders
of this world and other worlds) who have moved from the mortal realm to the Astral Plane (another
dimension) and exist in a state of immortality. The members of the Great White Brotherhood often
communicate to mortals here on earth through channeling. "White" refers to the light (aura) around
the great spiritual teachers throughout history.

Guilt
Being responsible for and accountable for an offense. Biblically, it is the state of being under a
present or pending consequence due to a sin against God’s Law. It is also an emotional state as well as
legal condition. Guilt feelings are used by the Holy Spirit to inform the sinner of broken fellowship with
God (Isaiah 59:2; John 16:8). Because of our guilt before God, we need reconciliation (Rom. 5:6-9).

Hades
New Testament term for the Hebrew “sheol,” which is the abode of the conscious dead. It is
apparently a place (Acts 2:31). In Revelation it is referred to as a creature on a horse (Rev. 6:8). In
Rev. 1:18, it says that Christ holds the keys to death and Hades.

Hamartiology
The study of the doctrine of sin.

Heaven
Heaven is the dwelling place of God and for those who go there a place of everlasting bliss.
Scripture implies three heavens, since "the third heaven" is revealed to exist (2 Cor. 12:2). It is
logical that a third heaven cannot exist without a first and second. Scripture does not describe
specifically the first and second heaven. The first, however, apparently refers to the atmospheric
heavens of the fowl (Hosea 2:18) and clouds (Dan. 7:13). The second heaven may be the area of the
stars and planets (Gen. 1:14-18). It is the abode of all supernatural angelic beings. The third heaven
is the abode of the triune God. Its location is unrevealed. (See Matt. 23:34-37; Luke 10:20; and Rev.
22:2, 20-27).

Hedonism
The teaching that pleasure is the principle good and proper goal of all action. Self indulgence.

Hell
Hell is the future place of eternal punishment of the damned including the devil and his fallen
angels. There are several words rendered as Hell: Hades - A Greek word. It is the place of the dead,
the location of the person between death and resurrection. (See Matt. 11:23; 16:18; Acts 11:27; 1
Cor. 15:55; Rev. 1:18; 6:8). Gehenna - A Greek word. It was the place where dead bodies were
dumped and burned (2 Kings 23:13-14). Jesus used the word to designate the place of eternal
torment (Matt. 5:22,29,30; Mark 9:43; Luke 12:5). Sheol - A Hebrew word. It is the place of the dead,
not necessarily the grave, but the place the dead go to. It is used of both the righteous (Psalm 16:10;
30:3; Isaiah 38:10) and the wicked (Num. 16:33; Job. 24:19; Psalm 9:17). Hell is a place of eternal
fire (Matt. 25:41; Rev. 19:20). It was prepared for the devil and his angels (Matt. 25:41) and will be
the abode of the wicked (Rev. 22:8) and the fallen angels (2 Pet. 2:4).
Henotheism
The teaching that there are many gods but that only one of them must be honored and
worshipped.

Heresy
A doctrinal view that deviates from the truth, a false teaching. We are warned against it in Acts
20:29-32 and Phil. 3:2. Heresies include teachings that Jesus is not God and that the Holy Spirit is not
a person (Jehovah's Witnesses, Christadelphians, The Way International), that men may become gods
(Mormonism), that there is more than one God (Mormonism), that Jesus lost His divinity in hell and
finished the atonement there, and that good works are necessary for salvation (all cults say this), to
name a few.

Heterodoxy
A set of beliefs or opinions that are not in agreement with accepted doctrinal beliefs of a church.
See orthodoxy.

Higher Self
A New Age term used to signify the divine part of each person that is capable of attaining the
knowledge of perfection and "ultimate truth."

Homiletics
That branch of theology concerned with preaching and sermons and the proper way in which to
deliver them.

Holy, Holiness
A quality of perfection, sinlessness, and inability to sin that is possessed by God alone. As
Christians we are called to be holy (1 Pet. 1:16). But this does not refer to our nature. Instead, it is a
command of our practice and thought. We are to be holy in obedience (1 Pet. 1:14). God has made us
holy through His Son Jesus (Eph. 1:4; 1 Pet. 2:9).

Holy Orders
In Catholicism, one of the seven sacraments by which men, bishop, deacons, and priests, are given
the power and authority by a bishop to offer sacrifice and forgive sins.

Holy Spirit, The


The third person of the Godhead. He is completely God. He is called God (Acts 5:3-4), has a will (1
Cor. 12:11), speaks (Acts 8:29; 13:2), and knows all things (John 14:17). He is not an "active force"
as the Jehovah's Witnesses mistakenly teach. The Holy Spirit is alive and is fully and completely God.
He is called the Spirit of God (Gen. 1:2), Holy Spirit (Psalm 51:1), the Helper (John 14:16,26), and
Eternal Spirit (Heb. 9:14). He knows all things (1 Cor. 2:10-11), is all powerful (Luke 1:35), and is
everywhere (Psalm 139:7-13). (See Trinity and Holy Spirit.)

Holy Water
In Catholicism, special water that has been blessed by a priest, bishop, etc. or a liturgical
ceremony. It is used to bring a blessing to a person when applied.

Humanism
A philosophical system of thought that focuses on human value, thought, and actions. Humans are
considered basically good and rationale creatures who can improve themselves and others through
natural human abilities of reason and action. Secular Humanism is a late development emphasizing
objectivity, human reason, and human standards, that govern art, economics, ethics, and belief. As
such, no deity is acknowledged.

Humility
The attitude of the Christian that teaches us not to "...think more highly of himself than he ought
to think; but to think so as to have sound judgment..." (Rom. 12:3). It teaches us to prefer others
over ourselves (Rom. 12:10). It is knowing our true position before God. It is not self-abasement or
demeaning one's self. "God is opposed to the proud, but gives grace to the humble," (James 4:6).
Humility is necessary to be a disciple of Jesus (Matt. 18:3-4). The humility of Jesus is described in
Philippians 2:5-8, "Your attitude should be the same as that of Christ Jesus: Who, being in very nature
God, did not consider equality with God something to be grasped, but made himself nothing, taking
the very nature of a servant, being made in human likeness. And being found in appearance as a man,
he humbled himself and became obedient to death - even death on a cross!" (NIV).

Hypostatic Union
This is the union of the two natures (Divine and human) in the person of Jesus. Jesus is God in
flesh (John 1:1,14; 10:30-33; 20:28; Phil. 2:5-8; Heb. 1:8). He is fully God and fully man (Col. 2:9);
thus, He has two natures: God and man. He is not half God and half man. He is 100% God and 100%
man. He never lost his divinity. He continued to exist as God when He became a man and added
human nature to Himself (Phil. 2:5-11). Therefore, there is a "union in one person of a full human
nature and a full divine nature." Right now in heaven there is a man, Jesus, who is our Mediator
between us and God the Father (1 Tim. 2:5). (For related information on Jesus and His two natures,
see Incarnation, and the errors concerning His natures known as Eutychianism, Monophycitism, and
Nestorianism.)

Jesus as God Jesus as Man


He is worshiped (Matt. 2:2,11; 14:33). He worshiped the Father (John 17).
He is prayed to (Acts 7:59). He prayed to the Father (John 17).
He is sinless (1 Pet. 2:22; Heb. 4:15). He was tempted (Matt. 4:1).
He knows all things (John 21:17). He grew in wisdom (Luke 2:52).
He gives eternal life (John 10:28). He died (Rom. 5:8).
All the fullness of deity dwells in Him (Col. He has a body of flesh and bones (Luke 24:39).
2:9).

Idol, Idolatry
An idol is a representation of something in the heavens or on the earth. It is used in worship and
is often worshiped. It is an abomination to God (Exodus 20:4). Idolatry is bowing down before such an
idol in adoration, prayer, or worship. In a loose sense, idolatry does not necessitate a material image
nor a religious system. It can be anything that takes the place of God: a car, a job, money, a person, a
desire, etc. Idolatry is denounced by God at the beginning of the Ten Commandments and is
considered a form of spiritual fornication.

Immaculate Conception
The teaching that Mary was conceived without original sin. Typically believed as true in Roman
Catholicism.

Image of God
Man was made in the image of God (Gen. 1:26). The image of God is generally held to mean that
people contain within their nature elements that reflect God's nature: compassion, reason, love, hate,
patience, kindness, self-awareness, etc. Though we have a physical image, it does not mean that God
has one. Rather, God is spirit (John 4:24), not flesh and bones (Luke 24:39).

Immutability
The divine attribute of unchangeableness. God said in Exodus 3:14, "I AM that I AM," signifying
His eternal sameness and His sovereignty. He cannot change His moral character, His love, His
omniscience, omnipresence, omnipotence, etc. God is "From everlasting to everlasting," (Psalm 90:2).
Immutability does not mean that God does not vary. The incarnation is just such an example of
variation. Also, God's attitude toward a person is changed when the person becomes a Christian. For
example, the enmity between God and man is removed (Rom. 5:10).
Mormonism denies the immutability of God. It says that God was not always God, that He was a
man on another planet who became a God (Mormon Doctrine, by Bruce McConkie, p. 321.).

Immortality
Life without death anytime in the future. God is immortal. The souls of people are immortal
though their bodies are not. All people can die in a physical sense but they continue on after death.
Therefore, it is the soul that is immortal. However, after the return of Christ and the resurrection, the
Christians' bodies will also become glorified and immortal (1 Cor. 15:50-58). The wicked will likewise
be resurrected to immortality but they will be cast into hell for eternal.

Impute, Imputation
To reckon to someone the blessing, curse, debt, etc. of another. Adam's sin is imputed to all
people (Rom. 5:12-21), therefore, we are all guilty before God. Our sins were put upon, imputed, to
Jesus on the cross where He became sin on our behalf (2 Cor. 5:21) and died with them (Isaiah 53:4-
6). Therefore, our sins are forgiven. Understanding imputation is very important. Imputation is the
means of our salvation. Our sins were put upon, imputed, to Jesus on the cross. Our sins were "given"
to Jesus. When He died on the cross, our sins, in a sense, died with Him. The righteousness that was
His through His perfect obedience to the Father in His complete obedience to the Law is imputed,
given, to us. In short, our sins were given to Jesus. His righteousness was given to us. Technically
speaking our sins were imputed to Jesus. His righteousness was imputed to us.

In facto
Something that exists and is complete.

In fieri
Beginning to be, but not yet complete.

Incarnation
The addition of human nature to the nature of God the second person of the Trinity. It is where
God became a man (John 1:1,14; Phil. 2:5-8). It was the voluntary act of Jesus to humble Himself so
that He might die for our sins (1 Pet. 3:18). Thus, Jesus has two natures: Divine and human. This is
known as the Hypostatic Union.
The doctrine is of vital importance to the Christian. By it we understand the true nature of God,
the atonement, forgiveness, grace, etc. It is only God who could pay for sins. Therefore, God became
man (John 1:1,14) to die for our sins (1 Pet. 2:24) that is the atonement. Through Jesus we have
forgiveness of sins. Since we are saved by grace through faith (Eph. 2:8-9) it is essential that our
object of faith be accurate. The doctrine of the incarnation ensures accuracy, the knowledge that God
died on the cross to atone for sin and that the God-man (Jesus) is now in heaven as a mediator (1
Tim. 2:5) between God and us.
Jesus came to reveal the Father (Matt. 11:27; Luke 10:22), to do His will (Heb. 10:5-9), to fulfill
prophecy (Luke 4:17-21), to reconcile the world (2 Cor. 5:18-21), and to become our High Priest
(Heb. 7:24-28). (Contrast with Kenosis.)

Induction
A system of logic where specific facts are used to draw a general conclusion.

Indulgence
In Catholicism, a means by which the Catholic church takes away some of the punishment due the
Christian in this life and/or purgatory because of his sin.

Inerrancy
Without error, non-errant. In Christianity, inerrancy states that the Bible, in its original documents,
is without error regarding facts, names, dates, and any other revealed information. Inerrancy does not
extend to the copies of the biblical manuscripts.

Infant baptism
The practice of baptizing infant children of believing parents. In the Catholic Church infant baptism
washes away original sin and is regenerative. In Reformed circles infant baptism is not regenerative
but covenantal and validated through the believing parent(s). There are no explicit accounts of infant
baptism in the Bible. However, it cannot be completely excluded as a possibility given that entire
households were baptized Acts 16:15,33; 18:8.
Infidel
A person who does not believe in any particular religious system.

Infinity
The state or quality of being infinite, unlimited by space or time, without end, without beginning or
end. God is infinite in that He is not limited by space or time. He is without beginning and without end
(Psalm 90:2).

Infralapsarianism
An issue within Reformed theology dealing with what may have happened in God's mind regarding
the logical order of His considering whom to elect into salvation before the foundation of the world.
The word means "after the fall." The position is that God first decided he would allow sin into the
world and second that he would then save people from it. By contrast, the supralapsarian ("before the
fall") position holds that God first decided that he would save some people and then second that he
would allow sin into the world.

Inspiration
The doctrine that the Bible was written by the influence of God. It is, therefore, without error. It is
accurate and authoritatively represents God's teachings (2 Tim. 3:16). As such it is a revelation from
God which implies direct knowledge about God, creation, man, salvation, the future, etc. It is an
illumination in that it shows us what we could not know apart from it.
One of the ways to prove that the Bible is inspired is to examine the O.T. prophecies fulfilled in the
N.T. concerning Jesus (Luke 24:27-45). Because the Bible is inspired, its words are unbreakable (John
10:34-36), eternal (Matt. 24:35), trustworthy (Psalm 119:160), and able to pierce the heart of man
(Heb. 4:12). Additionally, the inspired Word of God will not go forth without accomplishing what God
wishes it to (Isaiah 55:11).

Intermediate state
The period between death and resurrection. The condition of the person in the intermediate state is
debated. One theory is that the person is without a body, yet is conscious, and that he will receive his
body at the resurrection. Another theory states that the person has a different sort of spiritual body
that will be lost at the resurrection when body and soul are reunited (2 Cor. 5:1-4).

Jehovah
An anglicized pronunciation of the Hebrew tetragrammaton, YHWH, which are the four consonant
letters used to spell God’s name in the Old Testament (Exodus 3:14). The Hebrews considered the
name of God too holy to pronounce and susbstituted the word “Lord” (adonai) when the text was read.
The vowels of the word “adonai” was combined with YHWH to get the word “Jehovah” which was first
used in the 12th century. A more accurate pronunciation of YHWH would be “Yahweh.” However, the
exact and proper pronunciation has been lost.

Jesus
The Bible is about Jesus (Luke 24:27,44; John 5:39; HHeb. 10:7). The prophets prophesied about
Him (Acts 10:43). The Father bore witness of Him (John 5:37, 8:18). The Holy Spirit bore witness of
Him (John 15:26). The works Jesus did bore witness of Him (John 5:36; 10:25). The multitudes bore
witness of Him (John 12:17). And, Jesus bore witness of Himself (John 14:6, 18:6).
Jesus is God in flesh (John 1:1,14). He is fully God and fully man (Col. 2:9) thus, He has two
natures: God and man. He is not half God and half man. He is 100% God and 100% man. He never
lost his divinity. He existed in the form of God and when He became a man, He added human nature 19
to Himself (Phil. 2:5-11). Therefore, there is a "union in one person of a full human nature and a full
19
Jesus' adding to Himself the nature of man by becoming one of us is known as the Hypostatic Union. Errors
dealing with the relationship of Jesus' two natures are: 1) Monophycitism which states that Jesus' two natures
combined into one new one; the problem here is that neither God nor man was represented in Christ. 2)
Nestorianism which states that the two natures of Christ were so separated from each other that they were "not
in contact;" the problem here is that worship of the human Jesus would then not be allowed. 3) Eutychianism is
similar to Monophycitism. It states that Christ's natures were so thoroughly combined -- in a sense scrambled
together -- that a new third thing emerged; the problem is this implies that Jesus was not truly God nor man,
therefore unable to act as mediator.
divine nature."20 Right now in heaven there is a man, Jesus, who is Mediator between us and God the
Father (1 Tim. 2:5). Jesus is our advocate with the Father (1 John 2:1). He is our Savior (Titus 2:13).
He is our Lord (Rom. 10:9-10). He is not, as some cults teach, an angel who became a man
(Jehovah's Witnesses) or the brother of the devil (Mormonism). He is wholly God and wholly man, the
Creator, the Redeemer. He is Jesus. (See also Jesus.)

Jesus Only Movement


This is a movement in some Pentecostal circles. It is an error in the understanding of the nature of
the Trinity. The biblical Trinity consists of three persons simultaneously and eternally existing in one
God. The Jesus Only Movement maintains that there is only one person in the Godhead: Jesus. It
teaches that the person of the Father became the person of the Son who then became the person of
the Holy Spirit and that the persons are consecutive not simultaneous. This movement is incorrect in
its Trinitarian interpretation. Additionally, they mistakenly believe that baptism is necessary for
salvation and that tongues are evidence of true conversion. See the Plurality Study for a study that
refutes their theology.

Jews
Originally, a Jew was a member of the state of Judah during the period of the division of Israel into
two nations: Judah and Israel. It became a common reference from the 8th century B.C. Today it is
used of adherents of the Jewish religion.

Judgment
Condemnation. There are several judgments: the judgment of the believer's sins (John 5:24), the
judgment of the believer's self (1 Cor. 11:31-32), the judgment of the believer's works (2 Cor. 5:10),
the judgment of the nations (Matt. 25:31-46), and the judgment of the wicked (Rev. 20:11-15).
There is no judgment for the Christian in respect to salvation (Rom. 8:1). We were judged in
Christ on the cross 2000 years ago. However, as Christians we will be judged according to our works
(2 Cor. 5:10) with, most probably, varying degrees of rewards. But, remember, the judgment of our
works does not affect our salvation.

Just, Justice
The due reward or punishment for an act. Justice is getting what is deserved. God is merciful but
He is also just (Deut. 32:4 - righteous) and must punish sin. In the grace of God, justice fell upon His
Son so that mercy would fall upon us. (See also Prov. 8:15; Gen. 18:19; Heb. 10:38).

Justify, Justification
To be justified is to be made righteous. It is a divine act where God declares the sinner to be
innocent of his sins. It is not that the sinner is now sinless, but that he is "declared" sinless. This
justification is based on the shed blood of Jesus, "...having now been justified by His blood..." (Rom.
5:9). When God sees the Christian, He sees him through the sacrifice of Jesus and "sees" him without
sin. This declaration of innocence is not without cost for it required the satisfaction of God's Law,
"...without shedding of blood there is no forgiveness," (Heb. 9:22). By the sacrifice of Jesus, in the
"one act of righteousness there resulted justification of life to all men," (Rom. 5:18, NASB). In
justification, the justice of God fell upon Himself--Jesus. We receive mercy--we are not judged
according to our sins. And grace is shed upon us--we receive eternal life. This justification is a gift of
grace (Rom. 3:24), by faith (Rom. 3:28) because Jesus bore our guilt (Isaiah 53:12).

Karma
In Hinduism, the total compilation of all a person's past lives and actions that result in the present
condition of that person. Normally, it is associated with reincarnation.

Kenosis
This is a teaching concerning Jesus' incarnation. The Kenosis attempts to solve some paradoxes
between the nature of God and of man as united in Jesus. For example, how could an all knowing God
become a baby, or how could God be tempted? The Kenosis maintains that God, when becoming a
man, divested Himself of some qualities of being a man. In a sense, the Kenosis is God minus

20
B. Milne, Know the Truth (Downers Grove: InterVarsity Press, 1982), p. 145.
something; God subtracting some qualities of deity to become a man. The Hypostatic Union is God
plus something; God adding human nature to Himself. The Kenosis, then, jeopardizes the true
incarnation because it puts in doubt the full indwelling of God among men in the person of Jesus.
(Compare with Hypostatic Union.)

Kingdom of God
The kingdom of God and the kingdom of heaven seem to be variations of the same idea. A
kingdom implies a king. Our king is Jesus. Jesus said His kingdom was not of this world (John 18:36).
Jesus' authority did not come from man but from God (Luke 22:29).
Entrance into the kingdom of God is by a new birth (John 3:5), repentance (Matt. 3:2), and the
divine call (1 Thess. 2:12). We are told to seek the kingdom of God first (Matt. 6:33) and to pray for
its arrival (Matt. 6:10). "The kingdom of God is not eating and drinking, but righteousness and peace
and joy in the Holy Spirit," (Rom. 14:17). It is also a future kingdom where full rulership in the actual
presence of the king Jesus will occur when He returns to earth.

Laity
The members of the church who are not in the clergy.

Law
The Law is God's instructions concerning the moral, social, and spiritual behavior of His people
found in the first five books of the Bible. The Law is the very reflection of the nature of God because
God speaks out of the abundance of what is in Him. Therefore, since God is pure, the Law is pure.
Since God is holy, the Law is holy. The Law consists of the 10 commandments (Exodus 20), rules for
social life (Exodus 21 - 23), and rules for the worship of God (Exodus 25 - 31). It was a covenant of
works between God and man and was (and is) unable to deliver us into eternal fellowship with the
Lord because of Man's inability to keep it. The Law is a difficult taskmaster because it requires that we
maintain a perfect standard of moral behavior. And then when we fail, the Law condemns us to death.
We deserve death even if we fail to keep just one point of the law: "For whoever keeps the whole law
and yet stumbles in one point, he has become guilty of all," (James 2:10).
The law made nothing perfect (Heb. 7:19). That is why the Law has shown us our need for Jesus
and the free gift we receive through Him (Gal. 3:24).

Law of non-contradiction
The Law of non-contradiction is the law that something cannot be both true and not true at the
same time when dealing with the same context. For example, the chair in my living room, right now,
cannot be made of wood and not made of wood at the same time. In the law of non-contradiction,
where we have a set of statements about a subject, we cannot have any of the statements in that set
negate the truth of any other statement in that same set. For example, we have a set of two
statements about Judas. 1) Judas hung himself. 2) Judas fell down and his bowels spilled out. Neither
statement about Judas contradicts the other. That is, neither statement makes the other impossible
because neither excludes the possibility of the other. The statements can be harmonized by stating:
Judas hung himself and then his body fell down and his bowels spilled out.
In order to make the set of statements contradictory, we would have something like: 1) Judas
hung himself. 2) Judas did not hang himself. Since either statement excludes the possibility of the
other, we would then have a contradiction.

Laying on of hands
Physical contact by touching of the hands. In the OT and NT it was sometimes used in reference to
doing physical harm (Gen. 22:12; Luke 20:19). In the NT it is also used to signify an attempt at
healing (Acts 9:12) and commissioning of Holy Work (1 Tim. 4:14). Usually, during the ordination of
an elder, hands are layed on him as symbolic of a transfer of authority and power.

Liberalism
In Christianity, the movement away from traditional orthodoxy often in an attempt to harmonize
biblical teachings with science, humanism, or other secular fields. The result is often a denial of
essential biblical doctrines such as the Trinity, the deity of Christ, His virgin birth, His resurrection, and
salvation by grace.

Limited atonement
The teaching held in Reformed (Calvinist) circles of Christianity that Jesus bore only the sins of the
elect, and not that of the entire world. It maintains that the sacrifice was sufficient for all, but
intended for the elect.

Logic
From the Greek “logos” meaning “word.” Logic is study of the principles of reasoning. A set of
premises that are examined and arranged so as to bring a conclusion. If A = B and B = C, then A = C.
Deductive logic is the method of validating a claim by means of supportive information where both
the claim and the information are necessarily true. For example, People exist. All people breath.
Therefore, all people breath.
Inductive logic is the method of drawing a conclusion from a set of supportive information, yet the
conclusion has not yet been verified. For example, each night I get tired at 10 PM. Therefore, I
conclude that tonight, I will be tired at 10 PM.

Logos
The Greek word for "word." Mentioned only in the writings of John. John 1:1 says, "In the
beginning was the Word [logos] and the Word [logos] was with God and the Word [logos] was God."
The Logos is sometimes used to refer to the second person of the Trinity as the Son in preincarnate
form. Jesus is the word [logos] made flesh (John 1:1,14).

Lord's Supper
See Communion

Man
Man is the creation of God. It is man alone who reflects God. The first man, Adam, was made in
God's image (Gen. 1:2627), and placed in the Garden of Eden for the purpose of enjoying the
fellowship of the Lord and fulfilling the purpose of God's creation. He was told to "be fruitful and
multiply, and fill the earth, and subdue it; and rule over the fish of the sea and over the birds of the
sky, and over every living thing that moves on the earth," (Gen. 1:28). When Adam and Eve sinned,
all of humanity fell with them (Rom. 5:12-21). Adam represented all humanity: "In Adam all die..." (1
Cor. 15:22). As a result of Adam's disobedience, condemnation resulted to all men (Rom. 5:18).
Therefore we are by nature children of wrath (Eph. 2:3). We do not seek God (Rom. 3:11) nor can we
understand the spiritual things of God (1 Cor. 2:14). Since this is the condition of man in his natural
state, salvation is then impossible for us to achieve (Matt. 19:26). That is why we need the free gift of
salvation (Rom. 6:23) given by God to Christians through faith in Jesus' sacrifice on the cross.

Manuscript
A document or a copy of an original writing. There are thousands of existing manuscripts of the
biblical documents ranging from vellum (animal skins) to papyri (plant material) upon which the
original and copies of the original writings were made.

Martyr
Someone who dies for a belief or cause. A Christian martyr would be a person who dies because of
his or her faith in Christian principles.

Mass
In Catholicism, a reenactment of the sacrifice of Christ cross in a ceremony performed by a priest.
This ceremony is symbolically carried out by the priest and involves Consecration where the bread and
wine are changed into the body and blood of Jesus.

Materialism
The position that only material things exist and that all other things can be explained in terms of
matter and the physical properties of matter.
Means of Grace
This is associated with sacramental theology. A means of grace is a manner in which the Lord
imparts grace to a believer as he partakes in the sacrament. A sacrament is a visible manifestation of
the word. The bread and wine in the Lord's Supper are considered sacraments in that they are visible
manifestations of the covenant promise of our Lord: "In the same way, after the supper he took the
cup, saying, 'This cup is the new covenant in my blood, which is poured out for you,'" (Luke 22:20).
Generally, the means of grace are considered to be the Gospel, baptism, and the Lord’s Supper. The
Catholic Church has seven total: baptism, confirmation, communion, penance, extreme unction, holy
orders, and matrimony.

Mediation, Mediator
A mediator is someone who intervenes, someone who conveys and conciliates. The word
"mediator" is not found in the O.T., but its principle is. God gave the Law to the people through a
mediator, Moses (Gal. 3:19), who was a type of the true mediator, Jesus. The word occurs only a few
times in the N.T.: 1 Tim. 2:5; Heb. 8:6; 9:15; 12:24. It is in the N.T. that the true nature of mediation
is understood in the person of Jesus Christ. He is the mediator of a better covenant (Heb. 8:6). He was
able to become our mediator by becoming man (John 1:1,14) and dying as our substitute (1 Pet.
1:18,19; 2:24). He reconciled us to God (Eph. 2:16).

Mercy
Mercy is the act of not administering justice when that justice is punitive. Because of our sinfulness
we deserve death and eternal separation from God (Rom. 6:23; Isaiah 59:2), but God provided an
atonement for sin and through it shows us mercy. That is, He does not deliver to the Christian the
natural consequence of his sin which is damnation. That is why Jesus became sin on our behalf (2 Cor.
5:21) and bore the punishment due to us (Isaiah 5345). It was to deliver us from damnation.
(Compare with justice and grace.)
God saved us according to His mercy (Titus 3:5) and we can practice mercy as a gift (Rom. 12:8).
"Let us therefore draw near with confidence to the throne of grace, that we may receive mercy and
may find grace to help in time of need," (Heb. 4:16).

Messiah
Messiah is a Hebrew word. It means "anointed one." It is the equivalent of the N.T. word "Christ"
which also means "anointed." Jesus, as the messiah, was anointed by God (Matt. 3:16) to carry out
His three-fold ministry of Prophet, Priest, and King. As the messiah He has delivered the Christian from
the bonds of sin and given to him eternal life. In that sense, messiah means deliverer, for He has
delivered us. The Messiah was promised in the O.T. in the seed of the woman (Gen. 3:15).

Metaphysics
The branch of philosophy involved with examining and discussing the ultimate nature of reality. The
term comes from "meta" which means "after" and "phusika" which means "physics." Around A.D. 70
Andronicus applied to the section of Aristotelian writings that came after the physics section; hence,
metaphysics.
In the New Age, metaphysics deals with spiritual concepts such as reincarnation, auras, chakras,
Ascended Masters, etc. and other such ideas of a spiritual nature not generally associated with
Christianity.

Middle Knowledge
That knowledge of God dealing with what individuals will do in a given set of circumstances. God
has an infinite set of potential circumstances that could exist and knows all actual choices that would
be made by individuals in each set. (See also Free Knowledge and Natural knowledge.)

Millennium
Literally, this word means 1000 years. In the study of end times doctrines (eschatology) the
millennium is the duration of Christ's rule over the earth. The debate has been over when the
millennium will take place and what it actually is. The terms that have arisen out of this debate are
premillennialism, amillennialism, and postmillennialism. Premillennialism teaches that the millennium
is yet future and that upon Christ's return He will set up His earthly kingdom. Amillennialism teaches
that the millennium is a figurative period and that Christ's rule began when He first became man.
Postmillennialism teaches that through the preaching of the Word of God, the world will be converted
and will then usher in Christ and the kingdom of God. There are good arguments for each position.

Minuscule
The Greek characters of lower case: abgde, etc. Different copies of Greek manuscripts appear in
minuscule form. By contrast, uncials are the Greek characters in upper case.

Miracle
A miracle is an out-of-the-ordinary direct and divine intervention in the world. Examples would be
the parting of the Red Sea, Jesus walking on water, the resurrection of Lazarus, etc. Some hold that it
is a violation of the natural order of physical laws. Others maintain that there is no such violation upon
God's part but only a natural manifestation of His work.
They are also known as powers and signs (Mark 9:39; Acts 2:22, 19:11) and mighty works (John
10:25-28). They are a manifestation of the power of God over nature (Joshua 10:121-14), animals
(Num. 22:28), people (Gen. 19:26), and illness (2 Kings 5:1014). They are produced by God's power
(Acts 15:12), Christ's power (Matt. 10:1), and the Holy Spirit's power (Matt. 12:28).

Modalism
The error that there is only one person in the Godhead who manifests himself in three forms or
manners: Father, Son, and Holy Spirit.

Monarchianism
Monarchianism (mono - "one"; arche - "rule") was an error concerning the nature of God that
developed in the second century A.D. It arose as an attempt to maintain Monotheism and refute
tritheism. Unfortunately, it also contradicts the orthodox doctrine of the Trinity. Monarchianism
teaches that there is one God as one person: the Father. Please see Heresies for more information.

Monergism
The teaching that God alone is the one who saves. It is opposed to synergism which teaches that
God and man work together in salvation. Cults are synergistic. Christianity is monergistic.

Monism
The view that there is only one basic and fundamental reality, that all existence is this one reality
even though we perceive different aspects of this reality.

Monophycitism
This is an error regarding the two natures of Jesus (See Hypostatic Union). It states that Jesus' two
natures are combined into one new one; the problem here is that neither God nor man was
represented in Christ but a new third thing. (Other errors regarding the two natures of Christ are
Nestorianism and Eutychianism.)

Monolatry
The belief that there is more than one God, but only one is served and worshiped. Mormonism is an
excellent example of monolatry. Mormonism teaches the existence of many Gods of many worlds, yet
worships only the one of this planet. Therefore, monolatry is a division of polytheism, the belief in
many gods. It is a false teaching contrary to Scripture. See Isaiah 43:10; 44:6,8; 45:5-6.
Monotheism
The belief that there is only one God in all places at all times. There were none before God and
there will be none after Him. Monotheism is the teaching of the Bible (Isaiah 43:10; 44:6,8;
45:5,14,18,21,22; 46:9; 47:8; John 17:3; 1 Cor. 8:5-6; Gal. 4:89).

Moral government theology


A theological error that maintains that God is not immutable but changes His mind, that He does
not exercise sovereign control over earthly matters, that He does not know all future events -
particularly the free-will choices of individuals, etc.. It denies that the atonement pays for our sins,
denies Jesus’ substitutionary death, and denies the imputed righteousness of Christ to the believer. It
asserts that people are capable of keeping the whole Law of God, that there is no depravity of human
nature, and that salvation is up to a person’s free will choice.

Mormonism
A non-Christian cult begun in 1830 by Joseph Smith. The Mormon church, also known as the
Church of Jesus Christ of Latter-day Saints, denies the historic Trinity and efficacious atonement.
Some of its unique doctrines are that God used to be a man on another world who became a God and
came to this world with one of his wives. We all are literally born in heaven as spirit brothers and
sisters and then inhabit human bodies on earth. For more information on this cult, please see
Mormonism on CARM.

Mortal Sin
In Catholicism, a serious and willful transgression of God's Law. It involves full knowledge and
intent of the will to commit the sin. If left unrepentant, according to Catholicism, can damn someone
to eternal hell. Mortal sin is more serious than venial Sin.

Natural knowledge
A term used in describing a type of knowledge possessed by God. Often it is raised in discussions
dealing with individuals’ free will and God’s infinite knowledge. God’s natural knowledge would be His
knowledge of all things of potential existence influenced by individuals though not necessarily in actual
existence. God knows this set of knowledge from all eternity, before the creation of the universe. It is
called natural because it is a natural attribute of God’s existence. See also Free Knowledge and Middle
Knowledge.

Naturalism
The belief that all of human experience can be described through natural law. It asserts that
biological evolution is true and that there are no supernatural realities.

Neo-orthodoxy
A focus on existential and psychological aspects of religious experience and denounces the
literalism of the Bible. Experience with the divine is what makes scripture real, not biblical revelation,
not reason. Neo orthodoxy is subjective and selective in its "orthodox" positions.

Nestorianism
States that the two natures of Christ were so separated from each other that they were "not in
contact"; the problem here is that worship of the human Jesus would then not be allowed. (See also
Hypostatic Union, Eutychianism, and Monophycitism.)

Nun
Especially in the Roman Catholic Church, those women who consecrate their lives to spiritual
service and various religious orders. They do not marry and are normally virgins.

Objectivism
A branch of philosophy that asserts that reality exists apart from the human mind and that the
knowability of this reality based upon observation.

Occam's Razor
The philosophical rule that the simplest explanation is preferred over the more complicated one and
that explanations should be first proposed in relation to concepts that are already known. Another
way of seeing it is to say that the fewer assumptions that need to be made to support an explanation
of something, the better. The principle is attributed to William Occam of the fourteenth century.

Occult
Occult means "hidden". It covers practices that are not approved of by God e.g., astrology (Isaiah
47:13), casting spells (Deut. 18:11), consulting with spirits (Deut. 18:11), magic (Gen. 41:8), sorcery
(Exodus. 22:8), witchcraft (Deut. 18:10), and spiritism (Deut. 18:11).
Occult practices such as Ouija boards, tarot cards, astrology charts, contacting the dead, séances,
etc. are to be avoided by the Christian and Jews alike.

Omnipotence
An attribute of God alone. It is the quality of having all power (Psalm 115:3). He can do all things
that do not conflict with His holy nature. God has the power to do anything He wants to.

Omnipresence
An attribute of God alone. It is the quality of being present in all places at all times (Jer. 23:23.4).
He is not bound by time and space. This does not mean that nature is a part of God and is, therefore,
to be worshiped. Creation is separate from God, but not independent of Him.

Omniscience
An attribute of God alone. It is the quality of having all knowledge (Isaiah 40:14). Omnipotence,
Omnipresence, and Omniscience represent the nature of God concerning His relation to the creation.

Ontological Argument
An attempt to prove God’s existence first postulated by Anselm. In brief, it states that God is a
being of which no greater thing exists or can be thought of. Therefore, since we can conceive of God
as the greatest of all things that exist, then God must exist.

Ontology
The study of the nature of being, reality, and substance.

Oracles
Oracles are the divine revelations given to God's people. God's method of communicating these
oracles varied from dreams and visions (Num. 12:6-8), to wisdom (Prov. 30:1), and even the Urim
and Thummim (Num. 27:21; 1 Sam. 14:337). 21

Ordination
In Christianity it is the ceremony of consecration to ministry. It is usually administered by a
commissioning and a laying on of hands.

Ordo salutis
Latin for “order of salvation.” Theologically it is the order of decrees by God in bringing about the
salvation of individuals. In the Reformed camp, the ordo solutis is 1) election, 2) predestination, 3)
calling, 4) regeneration, 5) faith, 6) repentance, 7) justification, 8) sanctification, and 9) glorification.
In the Arminian camp, the ordo soluits is 1) calling, 2) faith, 3) repentance, 4) regeneration, 5)
justification, 6) perseverance, 7) glorification.

Original Sin
This is a term used to describe the effect of Adam's sin on his descendants (Rom. 5:12-23).
Specifically, it is our inheritance of a sinful nature from Adam. The sinful nature originated with Adam
and is passed down from parent to child. We are by nature children of wrath (Eph. 2:3).

Orthodoxy
Belief in the standards of accepted and true doctrines taught in the Bible. see Heterodoxy.

21
The Urim and Thummim were placed in the breastplate of the high priest (Exodus 28:30) and were used as a
means of communication with God. They mean "light" and "perfection". Unfortunately, they are not described
anywhere in the Bible. Some theories maintain that they were twelve stones that made up part of the High
Priest's garments. The process of the communication with God is not given either.
Panentheism
The belief that God is in the universe. It differs with pantheism which states that God is the
universe and all that it comprises.

Pantheism
This is an identification of the universe with God. With this view there is a blurring of the distinction
between the Creator and the creation as well as an attack upon the personality and nature of God.
Pantheism tends to equate God with the process of the universe and states that the universe is God
and God is the universe. This is not true because God is the creator of the universe (Isaiah 44:24) and
therefore separate from it.

Papyrus
A plant growing along the Nile in Egypt during biblical times. It was used as writing material.
Papyrus scrolls were made by cutting and pressing sections of the papyri plant together at right
angles. They typical maximum length of a scroll was about 35 feet. The scribe, when using papyrus,
would often use the natural horizontal fibers of the papyrus plant as guidelines. He would take a blunt
instrument and score horizontal lines and then score two or more vertical lines as margins for the edge
of the sheet or to define columns on it. We get the word "paper" from this word. Many of the biblical
manuscripts were on papyrus.

Parable
An illustrative discourse or story that uses common events and culture and is meant to convey a
meaning or lesson. Jesus used parables extensively. Some of the OT parables are Trees Making a
King (2 Sam. 12:1-4); The Thistle and the Cedar (2 Kings 14:9); Israel, a Vine Planted by Water
(Ezek. 24:1014), etc. Some NT parables are The Sower (Luke 8:5-8); the Ten Virgins (Matt. 25:1-
13); The Good Samaritan (Luke 10:25-37); The Prodigal Son (Luke 15:11-32), etc. See Parables.

Paradise
Biblically, paradise is the place of uninterrupted bliss. The Garden of Eden was considered a
paradise. Jesus mentioned paradise while on the cross (Luke 23:43) and Paul also mentioned Paradise
(2 Cor. 12:1-4). Some consider paradise to be the abode of people in the intermediate state while
others believe it is the permanent location of the saved.

Parapsychology
The study of things not generally explainable by the scientific method. Examples of subjects
studied by parapsychologists would be telepathy, clairvoyance, ghosts, etc.

Parousia
(par-ooo-see’-a) A Greek term that means “arrival” or “coming.” The term is often referred to as
the time of Christ’s return; hence, the Parousia, i.e., 2 Thess. 2:1.

Pedobaptism
The practice of infant baptism.

Pelagianism
The teaching of a monk named Pelagius in the fifth Century. He taught that man's will was and still
is free to choose good or evil and there is no inherited sin (through Adam). Every infant born into the
world is in the same condition as Adam before the fall and becomes a sinner because he sins. This is
opposed to the Biblical teaching that we are by nature children of wrath (Eph. 2:3) and that we sin
because we are sinners. Pelagius said we are able to keep the commandments of God because God
has given us the ability. Therefore, there is no need of redemption and the crucifixion of Jesus is
merely a supreme example of love, humility, obedience, and sacrifice. This heresy has its relatives in
the form of the cults that deny the total dependence upon God and maintain that salvation is
obtainable through our own efforts. (Compare to Arminianism and Calvinism.)

Penance
In Catholicism, a means by which all sins committed after baptism are removed. The means are
assigned by a priest and usually consist of special prayers or deeds performed by the sinner.

Pentateuch
This word is from the Greek penta, "five" and teuchos, "a tool". It refers to the first five books of
the Bible known as Genesis, Exodus, Leviticus, Numbers, and Deuteronomy. All five were authored by
Moses and are also known as "the Law".

Pentecost
The word comes from the Greek which means fifty. So, Pentecost was a celebration on the fiftieth
day after Passover. It was a culmination of the feast of weeks (Exodus 34:22,23). Pentecost in the NT
is the arrival of the Holy Spirit for the church (Acts 2). At Pentecost the disciples of Jesus were
gathered and upon the filling of the Holy Spirit, they heard a great wind and spoke in tongues as
tongues of fire that settled upon them. The significance of the fire can be found in recognizing it as a
symbol of the dwelling of the Spirit of God (Exodus 19:18; 1 Pet. 4:14).

Permissive decree
In Christian theology, those decrees (ordained events) of God that are different from His direct
decrees. An example of a permissive decree would be the fall of Adam into sin. God does not desire
sin, yet He permitted its occurrence. He decreed that it would occur by permission, not by direct
action of His will. A direct decree of God would be the incarnation of the Son.

Perseverance
To endure to the end. Theologically, the term “perseverance of the saints” is the teaching that
salvation cannot be lost, that the saints will preserver to the end.

Person
A variety of definitions can be offered from a human being to a legal organization with rights. In
biblical reference it is generically a fleshly individual capable of free choice. In reference to the Trinity
as three persons, the word refers to the attributes of personhood: self-awareness, choice, can reason,
love, possessing a will and consciousness, etc. Humans possess these attributes as well.

Pharisee
The Pharisees were a Jewish sect from the second century B.C. to the first century A.D. They
considered the entire Old Testament to be authoritative, unlike the Sadducees who only accepted the
first five books. The Pharisees believed in life after death, the resurrection, the existence of angels and
demons, and that the way to God was through keeping the law. "According to Josephus, the Pharisees
were the group most influential with the people, were noted for their accurate and therefore
authoritative interpretations of Jewish law, and had their own traditions and way of life to which they
were faithful. They had a simple standard of living and cultivated harmonious relations with others. 22

Philosophy
The study of seeking knowledge and wisdom in understanding the nature of the universe, man,
ethics, art, love, purpose, etc.

Pluralism
The idea that reality consists of different kinds of things. The term is used in different fields of
study. Social pluralism deals with the many different types of social structure. Cultural pluralism
deals with the many different types of culture, etc.

Pneumatology

22
"Achtemeier, Paul J., Th.D., Harper's Bible Dictionary, (San Francisco: Harper and Row, Publishers, Inc.) 1985.
The study of the Holy Spirit, His person, works, relation to the Father and Son, relation to man,
ministry in salvation and sanctification, conviction, and indwelling.

Polytheism
The teaching that there are many gods. In the Ancient Near East the nation of Israel was faced
with the problem of the gods of other nations creeping into the theology of Judaism and corrupting the
true revelation of God. Baal was the god of rain and exercised a powerful influence over the religion of
many pagan cultures and even into the Jewish community. This is so because rain was essential to
survival. Rain meant the crops would grow, the animals would have water, and the people would be
able to eat. If there was no rain, death prevailed. Such visible realities as rain, drought, crops, and
death often carried the spiritual character of the nation of Israel into spiritual adultery, that is,
worshiping other gods. The Bible does recognize the existence of other gods, but only as false gods (1
Cor. 8:5-6; Gal. 4:8-9) and clearly teaches that there is only one true God (Isaiah 43:10; 44:6,8;
45:5,14,18,21,22; 46:9; 47:8;). (See Monotheism.)

Pope
In Catholicism, the Pope is supposed to be Christ's representative on earth. He is the alleged,
visible successor of Peter.

Postmillennialism
The belief that through the preaching of the word of God, the entire world will be converted to
Christianity and this will usher in the kingdom of Christ. This is when Christ will return.

Postmodernism
A relativistic system of observation and thought that denies absolutes and objectivity.
Postmodernism has influenced theology, art, culture, architecture, society, film, technology, and
economics. Traditional social, art, social, and cultural, constructs are discarded and reinterpreted in
relativistic terms. An example of postmodern thought would be the validation of homosexuality as an
equally legitimate sexual expression over and against the Judeo-Christian ethic of heterosexual
monogamy. In other words, previously taboo practices and beliefs are given equal validity to
traditional values and norms often to the point of displacing the latter. This equalization and
displacement are not restricted to religious realms, but affect all circles of human interaction.

Pragmatism
A method in philosophy where value is determined by practical results.

Prayer
A privilege and an obligation of the Christian where we communicate with God. It is how we convey
our confession (1 John 1:9), requests (1 Tim. 2:1-3), intercessions (James 5:15), thanksgiving (Phil.
4:6), etc., to our holy God. We are commanded to pray (1 Thess. 5:17).
Some personal requirements of prayer are a pure heart (Psalm 66:18), belief in Christ (John
14:13), and that the prayer be according to God's will (1 John 5:13). We can pray standing (Neh.
9:5), kneeling (Ezra 9:5), sitting (1 Chron. 17:16-27), bowing (Exodus 34:8), and with lifted hands (1
Tim. 2:8).

Pre-Adamites
The teaching that there was a race of people before Adam and Eve lived in the Garden and that the
fall of Satan caused a widespread destruction of the world. The result of this destruction was so vast
that the world needed to be re-made with Adam and Eve being the first of the new order.

Pre-existence
The teaching that before our existence here on earth, we had a prior existence. Biblically, we do
not pre-exist. Our beginning is at our conception. Many aberrant groups teach pre-existence such as
the Mormons and the Shepherd’s Chapel. Also, all groups that teach reincarnation affirm the idea of
pre-existence.Predestine,
Predestination
The doctrine that God has foreordained all things which will come to pass yet He is not the author
of sin. He does, however, use sinful things for His glory and purpose. For example, the crucifixion was
brought about by sinful men who unrighteously put Jesus to death (Acts 4:27); yet, in that death, we
are reconciled to God (Rom. 5:10).
Predestination maintains that God is the one who decides who will be saved (Rom. 9:16) and that
it is not up to the desire of the person (John 1:13). God is the one who ordains the Christian into
forgiveness, "...and as many as had been appointed to eternal life believed," (Acts 13:48). Also, "For
whom He foreknew, He also predestined to become conformed to the image of His Son, that He might
be the first-born among many brethren; and whom He predestined, these He also called; and who He
called, these He also justified; and whom He justified, these He also glorified," (Rom. 8:29-30).
Further verses to examine are Eph. 1:4,11; Rom. 9. (See also Election and Sovereignty.)

Premillennialism
This is a teaching concerning the end times (eschatology). It says that there is a future millennium
(1000 years) where Christ will rule and reign over the earth. At the beginning of the millennium Satan
and his angels will be bound and peace will exist on the entire earth. At the end of the 1000 years
Satan will be released in order to raise an army against Jesus. Jesus will destroy them and then the
final judgment will take place with the new heavens and the new earth being made.

Preterition
The act of passing over something, or neglecting it. In theology, it is the Reformed doctrine that
God passed over people by not electing them into salvation. Instead, only those elected to salvation
will be saved and passed over all others.

Priest
A person having the ability to perform certain religious rites, sacraments. Generally, a priest stands
between God and Man and administers the ceremonial rites on behalf of the individuals as an offering
to God. In many churches (Catholic), the priest is below the Bishop in ecclesiastical order and rank.

Prophet
Someone who is the mouthpiece of God. He stands between God and man to communicate to man
the word of God. When the prophet spoke as the mouthpiece he was inspired and without error. The
prophet, though, is not a puppet or a mindless repeater of what he hears. Instead, he retains his own
will, mind, and thoughts as he speaks for God. God would put His words in their mouths (Deut. 18:18;
Jer. 1:9). A prophet was God's servant (Zech. 1:6) and messenger (2 Chron. 36:15). The prophecies
fell into three categories: concerning the destiny of Israel, the messianic prophecies, and
eschatological prophecies. The term Law and Prophets refers to the writings of the OT divided into two
categories. The Law is the Pentateuch, or Genesis, Exodus, Leviticus, Numbers, and Deuteronomy. The
Prophets are all the rest of the OT books.

Propitiation
This means the turning away of wrath by an offering. It is similar to expiation but expiation does
not carry the nuances involving wrath. For the Christian the propitiation was the shed blood of Jesus
on the cross. It turned away the wrath of God so that He could pass "over the sins previously
committed," (Rom. 3:25). It was the Father who sent the Son to be the propitiation (1 John 4:10) for
all (1 John 2:2).

Purgatory
An incorrect doctrine of the Roman Catholic Church. Purgatory is the belief that there exists a place
after death where some of the sins of people are purged through suffering. After a period of time
corresponding to the suffering necessary for the sins committed, the person is then set free and enters
heaven. "Gifts or services rendered to the church, prayers by the priests, and masses provided by
relatives or friends in behalf of the deceased can shorten, alleviate or eliminate the sojourn of the soul
in purgatory." 23

23
"Achtemeier, Paul J., Th.D., Harper's Bible Dictionary, (San Francisco: Harper and Row, Publishers, Inc.) 1985.
This is an unbiblical doctrine rejected by the Protestant church. It reflects the misunderstanding of
the atonement of Christ as well as adding insult to the finished work of the cross. The error of
purgatory is the teaching that we might perfect ourselves and remove sin through our sufferings. If
that were possible, then why did Christ need to die? Gal. 2:21 says, "I do not set aside the grace of
God, for if righteousness could be gained through the law, Christ died for nothing!" (NIV)
Additionally, on the cross Jesus said, "It is finished," (John 19:30). In the Greek, this was an
accounting term which meant a debt was paid in full. If the payment for our sins was paid in full on the
cross, then how could purgatory be a reality -- especially when the scriptures don't mention it and
even contradict it: "Just as man is destined to die once, and after that to face judgment," (Heb. 9:27).

Rapture
The rapture is an eschatological (end times) event whereupon the return of Christ the true
believers who are "alive and remain shall be caught up together with them [those who already died as
Christians] in the clouds to meet the Lord in the air..." (1 Thess. 4:17). This is the time of the
resurrection where the Christian receives his resurrected body. First to receive their new bodies are
those who have died as Christians, and then "those who are alive and remain."
There is much debate over the time of the rapture. Does it occur at the beginning, in the middle, or
at the end of the tribulation period? (See Tribulation.)

Rationalism
A branch of philosophy where truth is determined by reason.

Reconcile, Reconciliation
Reconciliation is changing for the better a relationship between two or more persons. Theologically
it refers to the change of relationship between God and man. We are naturally children of wrath (Eph.
2:3), and are at enmity with God (Eph. 2:11-15); but, "...we were reconciled to God through the
death of His Son..." (Rom. 5:10). Because of the death of Jesus, the Christian's relationship with God
is changed for the better. We are now able to have fellowship with Him (1 John 1:3) whereas before
we could not. So, we are reconciled to Him (Rom. 5:10-11). The problem of sin that separates us from
God (Isaiah 59:2) has been addressed and removed in the cross. It was accomplished by God in Christ
(2 Cor. 5:18).

Redemption
Redemption means to free someone from bondage. It often involves the paying of a ransom, a
price that makes redemption possible. The Israelites were redeemed from Egypt. We were redeemed
from the power of sin and the curse of the Law (Gal. 3:13) through Jesus (Rom. 3:24; Col. 1:14). We
were bought with a price (1 Cor. 6:20; 7:23).

Regeneration
The act of God whereby He renews the spiritual condition of a sinner. It is a spiritual change
brought about by the work of the Holy Spirit so that the person then possess new life, eternal life.
Regeneration is a change in our moral and spiritual nature where justification is a change in our
relationship with God. Also, sanctification is the work of God in us to make us more like Jesus.
Regeneration is the beginning of that change. It means to be born again.

Reincarnation
The belief in the birth and rebirth of a person's soul over and over again in different human bodies
throughout history. Some forms of reincarnation include incarnations into animals, plants, or
inanimate objects. The purpose of reincarnation is to allow the individual to learn spiritual lessons
through life so that he/she may return to God from whence the soul came. Reincarnation is closely
tied to Karma.

Relativism
The view that truth is relative and not absolute. Truth varies from people to people, time to time
and there are no absolutes. See CARM's Relativism section.

Religion
An organized system of belief that generally seeks to understand purpose, meaning, goals, and
methods of spiritual things. These spiritual things can be God, people in relation to God, salvation,
after life, purpose of life, order of the cosmos, etc.

Repentance
To repent means to turn. In the NT repentance means to turn from sin. We were called by God to
turn from sin. In fact, all men everywhere are commanded by God to repent of their sins (Acts 17:30).
God's longsuffering leads us to repentance (2 Pet. 3:9) as does His kindness (Rom. 2:4).
There is true and false repentance, "For the sorrow that is according to the will of God produces a
repentance without regret, leading to salvation; but the sorrow of the world produces death," (2 Cor.
7:10).

Resurrection, resurrection bodies


Resurrection means to be raised from the dead (John 5:28,29). The word is used in different
contexts in the Bible. Lazarus was raised from the dead (John 11:43). This is a resurrection, but it is
not part of the resurrection that occurs when we receive our new bodies when Christ returns (1 Thess.
4:13-18), on the last day (John 6:39-44) when the last trumpet is blown (1 Cor. 15:51-55). Lazarus
died again. The resurrection of Jesus is promissory in that as we know He was raised, so we will be
raised also. In that context, Jesus is the only one who has received a resurrected body. That is why He
is called the first-fruit from the dead (1 Cor. 15:20-23). We will receive our bodies either at the
rapture or when Jesus returns to earth.
The resurrected body is not subject to death or sin. We know very little about it except what was
manifested by Jesus after His resurrection; namely, that He was able to move about as He desired --
in and out of rooms without the use of doors. Other than that, the rest is conjecture. (See 1 Cor. 15).

Revelation
This means the disclosure of something that was unknown. There are two types of revelation:
natural and special. Natural revelation is that which is revealed about God through what we can see in
creation (Rom. 1:20 ). Through creation we may learn that there is a God, that He is in control, that
He has an order, and that He is concerned for our welfare. However, through natural revelation, we
are not able to discover the plan of salvation. That comes from special revelation.
Special revelation is that which is given to us through Prophets, the Bible, and even visions and
dreams (Num. 12:6-8). The ultimate in revelation is the incarnation of Jesus because He came to
reveal the Father to us (Matt. 11:27; Luke 10:22; Heb. 1:1-3) and to communicate to us the gospel (1
Cor. 15:1-4 by which comes salvation.

Righteousness
Righteousness is an attribute of moral purity belonging to God alone (John 17:25 ). It is He alone
who is truly righteous. No one in the world is righteous in the eyes of the Lord, that is, except the
Christian. We are counted righteous in the eyes of God when we receive Jesus by faith (Phil. 3:9). Our
righteousness is based on what Jesus did on the cross. The righteousness that was Christ's is counted
to us. We, then, are seen as righteous in the eyes of God. Though we are actually worthy of
damnation, we are made righteous (Isaiah 61:10) by Jesus' sacrifice on the cross. As a result, we will
spend eternity in the presence of the holy, pure, loving, kind, gentle, and righteous God who is our
righteousness.

Rosary
In Catholicism, a string of beads containing five sets with ten small beads. Each set of ten is
separated by another bead. It also contains a crucifix. It is used in saying special prayers, usually to
Mary where the rosary is used to count the prayers.

Sacerdotalism
The teaching that ordination imparts special abilities/powers necessary for the operation of the
ministry. Also, the teaching that grace is administered through the one so ordained.

Sacrament
A visible manifestation of the word. The bread and wine in the Lord's Supper are considered
sacraments in that they are visible manifestations of the covenant promise of our Lord: "In the same
way, after the supper he took the cup, saying, 'This cup is the new covenant in my blood, which is
poured out for you,'" (Luke 22:20).
God, in the OT, used visible signs along with His spoken word. These visible signs, then, were
considered to have significance. "Among the OT sacraments the rites of circumcision and the Passover
were stressed as being the OT counterparts of baptism (Col. 1:10-12) and the Lord's Supper (1 Cor.
5:7)."

Sadducee
A group of religious leaders in the Jewish religion from the second century B.C. to the first century
A.D. In Hebrew their names mean "the righteous ones." They were smaller in size and the group of the
Pharisees. The Sadducees were generally on the upper class, often in a priestly line, and the Pharisees
in the middle class, usually merchants and tradesmen. The Sadducees accepted only the Torah, the
first five books of the old Testament, as authoritative. They held rigidly to the old Testament law and a
denying the life after death, reward and punishment after death, the resurrection, and the existence of
angels and demons. They controlled the temple and its services and were unpopular with the majority
of the Jewish population.

Salvation
Salvation is the "saving" of a sinner from the righteous judgment of God. When someone appeals
to God and seeks forgiveness in Jesus, his sins are forgiven. He is cleansed. His relationship with God
is restored, and he is made a new creature (2 Cor. 5:17). All of this is the work of God, not man.
Salvation is a free gift (Rom. 6:3).
We are saved from damnation. When anyone sins, and we all have (Rom. 3:23; 6:23), he deserves
eternal separation from God (Isaiah 59:2). Yet, because of His love and mercy, God became a man
(John 1:1,14 ) and bore the sins of the world in His body on the cross (1 Pet. 2:24; 1 John 2:2). We
are forgiven when we realize that there is nothing we can do to earn the favor of God and we put our
trust in what Jesus did for us on the cross (Eph. 2:8-9; 1 Cor. 15:1-4). Only God saves. The only thing
we bring to the cross is our sin.
Both God the Father (Isaiah 14:21) and Jesus (John 4:42) are called Savior; that is, deliverer from
sin. Remember, it was the Father who sent the Son (1 John 4:10) to be the Savior.

Sanctify, Sanctification
To sanctify means to be set apart for a holy use. God has set us apart for the purpose of
sanctification not impurity (1 Thess. 4:7) and being such we are called to do good works (Eph. 2:10).
Christians are to sanctify Christ as Lord in their hearts (1 Pet. 3:15). God sanctified Israel as His
own special nation (Ezek. 27:28). People can be sanctified (Exodus 19:10,14) and so can a mountain
(Exodus 19:23), as can the Sabbath day (Gen. 2:3), and every created thing is sanctified through the
word of God and prayer (1 Tim. 4:4). 24
Sanctification follows justification. In justification our sins are completely forgiven in Christ.
Sanctification is the process by which the Holy Spirit makes us more like Christ in all that we do, think,
and desire. True sanctification is impossible apart from the atoning work of Christ on the cross because
only after our sins are forgiven can we begin to lead a holy life.

Sanhedrin
The Sanhedrin was a council of 71 individuals, around the time of Christ, that was comprised of
Pharisees and Sadducees who governed the Jewish nation while under the rule of Rome. It often
served as a court to settle legal and religious matters.

Scholasticism
The method of study in the Middle Ages which was used to support the doctrines of the church
through reason and logic.

Scriptures

24
Adapted from Baker's Dictionary of Theology, p. 470.
The scriptures are, quite simply, the Bible which consists of 39 books in the Old Testament and 27
in the New Testament. Each one is inspired, without error, and is completely accurate in all things it
addresses. The entire Bible, though written by many people over thousands of years is harmonious in
all its teachings. This is because each book of the Bible is inspired.

Second Coming, The


The Second Coming is a term applied to the return of Christ. If there is a second coming, it follows
that there must have been a first. The first coming of Christ was His incarnation when He was born. At
the second coming of Christ every eye will see Him (Rev. 1:7) as He descends from heavens in t he
clouds (Matt. 24:30; Mark 14:6).

Septuagint, The
The Septuagint is the Greek translation of the Old Testament. The Old Testament was originally
written in Hebrew. It was during the reign of Ptolemy Philadelphus (285-246 B.C.) that the
Pentateuch, the first five books of the Bible, were translated into Greek. Shortly afterwards the rest of
the Old Testament was also translated. This translation was done by approximately 70 translators.
Hence, the Septuagint is known by the letters LXX, the Roman numerals for seventy.

Sin
Sin is anything that is contrary to the law or will of God. For example: if you lie, you have sinned.
Why? Because God has said not to lie (Exodus 20:16). If you do what God has forbidden, then you
have sinned. In addition, if you do not do what God has commanded, you sin (James 4:17). Either
way, the result is eternal separation from God (Isaiah 59:2). Sin is lawlessness (1 John 1:3) and
unrighteousness (1 John 5:17). Sin leads to blindness (John 9:) and death (Rom. 6:23).
Paul, in the book of Romans, discusses sin. He shows that everyone, both Jew and Greek, is under
sin (Rom. 3:9). He shows that sin is not simply something that is done, but a condition of the heart
(Rom. 3:10-12). In Ephesians Paul says that we are "by nature children of wrath," (Eph. 2:3). Yet,
"while we were still helpless, at the right time Christ died for the ungodly," (Rom. 5:6).

Skepticism
Skepticism is the philosophical approach that denies that the world can be objectively known in any
absolute sense. It further denies the true know ability of God.

Sola Fide
The teaching that faith alone saves a person when he places his faith and trust in the sacrificial
work of Christ.

Sola Gratia
The teaching that God pardons believers without any merit of their own based solely on the
sacrificial work of Christ.

Sola Scriptura
The teaching that the Scriptures contain all that is necessary for salvation and proper living before
God.

Son of God
This is a title of Jesus. It implies His deity (John 5:18) because the title is one of equality with God.
In the OT it was figuratively applied to Israel (Exo 4:22). In the NT it is applied to Christ (Luke 1:35).
It has many facets, for example: It shows that He is to be honored equally with the Father (John 5:22-
23). That He is to be worshiped (Matt. 2:2,11; 14:33; John 9:35-38; Heb. 1:6); called God (John
20:28; Col. 2:9; Heb. 1:8); prayed to (Acts 7:55-60; 1 Cor. 1:1-2).

Soteriology
The study of the doctrine of salvation. It is derived from the Greek word soterious which means
salvation. Some of the subjects of soteriology are the atonement, imputation, and regeneration.

Soul Sleep
The teaching that when a person dies his soul ceases to exist. On the final judgment day he is
brought back to life and judged. This is not a heresy, only an error of interpretation. The Bible is not
specific on the condition of the person between death and resurrection. However, there are scriptures
that strongly suggest man's continued self-awareness and continued existence after death (Luke
16:19-31; 2 Cor. 5:1-10; Phil. 1:21-23).

Sovereignty
The right of God to do as He wishes (Psalm 50:1; Isaiah 40:15; 1 Tim. 6:15) with His creation.
This implies that there is no external influence upon Him and that He also has the ability to exercise
His power and control according to His will.

Spiritual Gifts
Spiritual gifts are gifts given by Jesus to His church. Spiritual gifts are discussed in 1 Cor. 12 - 14
and Rom. 12. They vary in degree and nature. There are some that are obviously supernatural in the
usage: speaking in tongues, discerning of spirits, healing, etc. There are others that are not so
supernatural: administrations, help, admonition, etc.
There is debate over the continuance of the gifts. Some say that the gifts have ceased because we
now have the Bible. They argue that the gifts were used for the building of the body of Christ during
the beginning of the Christian church when the Bible was not complete. Since the Bible is complete
there is no further need for the revelatory gifts like peaking in tongues and the interpretation of
tongues. Others maintain that the gifts are all for today though to a lesser degree. There are good
arguments on both sides.

Subjectivism
The teaching that the individual is the source and judge of all religious knowledge based upon his
own knowledge and experience.

Supralapsarianism
An issue within Reformed theology dealing with what may have happened in God's mind regarding
the logical order of His considering whom to elect into salvation before the foundation of the world.
The word means "before the fall." This position holds that God first decided that he would save some
people and then second that he would allow sin into the world. By contrast, the infralapsarian ("after
the fall") position is the reverse in that it holds that God first decided he would allow sin into the world
and second that he would then save people from it.

Synagogue
A Jewish house of worship. Traditionally the first synagogues were established during the
Babylonian exile. The early synagogues had a place in the center of the room where the sacred scrolls
were kept and from where they were read. It is from the worship order established in synagogues that
our modern church patterns of reading and expounding upon scripture from the pulpit are derived.

Synergism
The teaching that we cooperate with God in our efforts of salvation. This is opposed to monergism
that is the teaching that God is the sole agent involved in salvation. Cults are synergistic in that they
teach that God's grace combined with our efforts is what makes forgiveness of sins possible.

Synoptic Gospels
The first three gospels: Matthew, Mark, and Luke. They are referred to as the synoptics because of
their great similarity.

Tabernacle
The tabernacle was the structure ordered built by God so that He might dwell among His people
(Exodus 25:8). It was to be mobile and constructed to exacting specifications. It is referred to in
Exodus 25-27, 30-31, 35-40; Num. 3:25ff.; 4:4 ff.; 7:1ff. In all of scripture more space is devoted to
the tabernacle than any other topic. Many books have been written on the spiritual significance of the
tabernacle, how it represented Christ, and how it foretold the gospel. The tabernacle consisted of the
outer court and the tabernacle. The outer court was entered from the East. The outer court contained
the altar of burnt offering (Exodus 27:1-8) and the bronze laver (Exodus 30:17-21). The tabernacle
stood within the court (Exodus 26:1ff.). It was divided into two main divisions: the holy place and the
holy of holies which were separated by a veil (Exodus 26:31 ff.), the same veil that was torn from top
to bottom at the crucifixion of Jesus (Matt. 27:51). Where the veil had represented the barrier
separating sinful man from a holy God (Heb. 9:8), its destruction represented the free access sinners
have to God through the blood of Christ (Heb. 10:19ff.).
The tabernacle was a place of sacrifice. The holy place contained three things: first, a table on
which was placed the shewbread, the bread of the presence (Exodus 25:23-30), second, a golden
lampstand (Exodus 25:31-40) and third, an altar of incense (Exodus 30:1-7). In the Holy of Holies was
the ark of the covenant which contained the Ten Commandments (Exodus 25:16). The holy of holies
was entered only once a year by the high priest who offered sacrifice for the nation of Israel.

Teleological argument
An attempted proof of God's existence based upon the premise that the universe is designed and
therefore needs a designer: God.

Teleology
The study of final causes, results. Having a definite purpose, goal, or design.

Temptation
That which moves us to sin. God cannot be tempted (James 1:13). But we can be tempted by our
lusts (James 1:13-15), money (1 Tim. 6:9), lack of self examination (Gal. 6:1), and the boastful pride
of life (1 John 2:16), to name a few. We are commanded to pray to be delivered from temptation
(Matt. 6:13) for the Lord is capable of delivering us from it (2 Pet. 2:9).

Testament
The word testament is a derivation of the Latin word testamentum, which was used in Jerome's
Vulgate to translate the Hebrew word b'rith, covenant. The Greek equivalent is diatheke, which also
means covenant. The word has come to be used in describing the two main divisions of the Bible: The
Old Testament and The New Testament. It should be understood then, that the Bible is generally to be
looked at as a covenant between God and man.

Tetragrammaton (YHWH)
This is a term applied to the four Hebrew letters that make up the name of God as revealed to
Moses in Exodus 3:14. "And God said to Moses, “I AM WHO I AM”; and He said, “Thus you shall say to
the sons of Israel, ‘I AM has sent me to you.’” YHWH make up the base of the verb "to be" from which
God designated His own name as "I AM." In English the letters are basically equivalent to YHWH. It is
from these four letters that the name of God is derived and has been rendered as Yahweh and
Jehovah. The true pronunciation of God's name has been lost through lack of use, because the Jews,
who were first given the name of God, would not pronounce it out of their awe and respect for God.
Theism
The teaching that there is a God and that He is actively involved in the affairs of the world. This
does not necessitate the Christian concept of God, but includes it. (Compare to Deism)

Theodicy
The study of the problem of evil in the world. The issue is raised in light of the sovereignty of God.
How could a holy and loving God who is in control of all things allow evil to exist? The answer has been
debated for as long as the church has existed. We still do not have a definitive answer and the Bible
does not seek to justify God's actions.
It is clear that God is sovereign, and that He has willed the existence of both good and evil, and
that all of this is for His own glory. Prov. 16:4 says, "The LORD works out everything for his own ends
-- even the wicked for a day of disaster"; Isaiah 45:7 says, "I form the light and create darkness, I
bring prosperity and create disaster; I, the LORD, do all these things."

Theology
The study of God, His nature, attributes, character, abilities, revelation, etc. True theology is found
in the Bible which is the self-revelation of God.

Theophany
A theophany is a visible manifestation of God usually restricted to the Old Testament. God has
appeared in dreams (Gen. 20:3-7; Gen. 28:12-17), visions (Gen. 15:1-21; Isaiah 6:1-13), as an angel
(Gen. 16:7-13; 18:1-33), etc.
There is a manifestation known as the Angel of the Lord (Judges 6:20f.) and seems to have
characteristics of God Himself (Gen. 16:7-9; 18:1-2; Exodus 3:2-6; Joshua 5:14; Judges 2:1-5; 6:11).
Such characteristics as having the name of God, being worshiped, and recognized as God has led
many scholars to conclude that the angel of the Lord is really Jesus manifested in the Old Testament.
This does not mean that Jesus is an angel. The word "angel" means messenger.
Other scriptures that describe more vivid manifestations of God are Gen. 17:1; 18:1; Ex. 6:2-3;
24:9-11; 33:20; Num. 12:6-8; Acts 7:2. For further information on theophanies see the Plurality
Study.

Tithe
A portion of one’s earnings, usually one tenth, that are given to those who perform the work of the
Lord since it belongs to the Lord (Lev. 27:30-33). Those who received tithes the OT consisted of
priests (Num. 18:21-32). Further OT references are Gen. 14:20; 28:22; 2 Chron. 31:5f; Mal. 3:7-12).
In the NT there is no command to tithe a tenth (since we are not under law but grace). But the
tithe is mentioned in Luke 18:9-14; 1 Cor. 16:1; 2 Cor. 8).

Total Depravity
The doctrine that fallen man is completely touched by sin and that he is completely a sinner. He is
not as bad as he could be, but in all areas of his being, body, soul, spirit, mind, emotions, etc., he is
touched by sin. In that sense he is totally depraved. Because man is depraved, nothing good can come
out of him (Rom. 3:10-12) and God must account the righteousness of Christ to him. This
righteousness is obtainable only through faith in Christ and what He did on the cross.
Total depravity is generally believed by the Calvinist groups and rejected by the Arminian groups.

Transcendence
A theological term referring to the relation of God to creation. God is "other," "different" from His
creation. He is independent and different from His creatures (Isaiah 55:8-9). He transcends His
creation. He is beyond it and not limited by it or to it.

Transfiguration
This refers to the mysterious change that occurred to Jesus on the mount: "Six days later, Jesus
took with him Peter and James and his brother John and led them up a high mountain, by themselves.
And he was transfigured before them, and his face shone like the sun, and his clothes became dazzling
white," (Matt. 17:1-2). The transfiguration preceded Jesus' time on the cross and may have been the
Father's preparatory provision to strengthen Jesus as He prepared to bear the sins of the world.

Transubstantiation
The theory accepted by Catholicism, that in the Lord's Supper, the elements are transformed into
the actual body and blood of Jesus. However, there is no perceptible or measurable change in the
elements. The transformation occurs during the Mass at the elevation of the elements by the priest.

Tribulation, The
According to premillennialism, this is a severn year period that immediately precedes the return of
Christ and the millennial kingdom of His rule which lasts for 1000 years. It will be a time of great
peace (the first 3 ½ years) and great war (the second 3 ½ years) when the Antichrist rules over many
nations. At the midpoint of the tribulation (at the end of the first 3 ½ years) the Antichrist will proclaim
himself worthy of worship. Many will bow down and worship the Antichrist and many will refuse. Those
who refuse to worship the Antichrist will be killed. The second half of the tribulation is called the Great
Tribulation. It will involve the whole world (Rev. 3:10). There will be catastrophes all over the world.
(See Matt. 24; Mark 13; Luke 17.)
Trichotomy
The teaching that the human consists of three parts: body, soul, and spirit. (Compare with
Dichotomy.)

Trinity
The word "trinity" is not found in the Bible. Nevertheless, it is a word used to describe one fact the
Bible teaches about God: Our God is a Trinity. This means there are three persons in one God, not
three Gods. The persons are known as the Father, the Son, and the Holy Spirit and they have all
always existed as three separate persons. The person of the Father is not the same person as the Son.
The person of the Son is not the same person as the Holy Spirit. The person of the Holy Spirit is not
the same person as the Father. If you take away any one, there is no God. God has always been a
trinity from all eternity: "From everlasting to everlasting, Thou art God," (Psalm 90:2).
God is not one person who took three forms, i.e., the Father who became the Son, who then
became the Holy Spirit. This belief is known today as the "Jesus Only Movement". It is taught by the
United Apostolic and United Pentecostal churches, and is an incorrect teaching.
Nor is God only one person as the Jehovah's Witnesses, the Way International, and the
Christadelphians teach (These groups are classified as non-Christian cults). For proof that there is
more than one person in the Godhead, see the Plurality Study.
The Bible says there is only one God. Yet, it says Jesus is God (John 1:1,14); it says the Father is
God (Phil. 1:2); and it says the Holy Spirit is God (Acts 5:3-4). Since the Son speaks to the Father,
they are separate persons. Since the Holy Spirit speaks also (Acts 13:2), He is a separate person.
There is one God who exists in three persons.
The following chart should help you understand how the Trinity doctrine is derived.

THE TRINITY

Father Son Holy Spirit

Called God Phil. 1:2 John 1:1,14; Col. 2:9 Acts 5:3-4
Creator Isaiah 64:8; 44:24 John 1:3; Col. 1:15-17 Job 33:4, 26:13
Resurrects 1 Thess. 1:10 John 2:19, 10:17 Rom. 8:11
Indwells 2 Cor. 6:16 Col. 1:27 John 14:17
Everywhere 1 Kings 8:27 Matt. 28:20 Psalm 139:7-10
All knowing 1 John 3:20 John 16:30; 21:17 1 Cor. 2:10-11
Sanctifies 1 Thess. 5:23 Heb. 2:11 1 Pet. 1:2
Life giver Gen. 2:7: John 5:21 John 1:3; 5:21 2 Cor. 3:6,8
Fellowship 1 John 1:3 1 Cor. 1:9 2 Cor. 13:14; Phil. 2:1
Eternal Psalm 90:2 Micah 5:1-2 Rom. 8:11; Heb. 9:14
A Will Luke 22:42 Luke 22:42 1 Cor. 12:11
Speaks Matt. 3:17; Luke 9:25 Luke 5:20; 7:48 Acts 8:29; 11:12; 13:2
Love John 3:16 Eph. 5:25 Rom. 15:30
Searches the heart Jer. 17:10 Rev. 2:23 1 Cor. 2:10
We belong to John 17:9 John 17:6
2 Tim. 1:10; Titus 1:4;
Savior 1 Tim. 1:1; 2:3; 4:10 3:6
We serve Matt. 4:10 Col. 3:24
Believe in John 14:1 John 14:1
Gives joy John 15:11 John 14:7
Judges John 8:50 John 5:21,30

Type, Typology
A type is a representation by one thing of another. Adam was a type of Christ (Rom. 5:14) and so
was Isaac (Heb. 11:19). The Passover was a type of Christ (1 Cor. 5:7). There are many types in the
Bible and most of them are too extensive and deep to be listed.
An example of a typology follows: Isaac a type of Jesus.
ISAAC JESUS
Only begotten Son Genesis 22:2 John 3:16
Offered on a mountain, hill 22:2 Matt. 21:10
Took donkey to place of sacrifice 22:3 Matt. 21:2-11
Two men went with him. 22:3 Mark 15:27; Luke 23:33
Three day journey. Jesus: three days in the grave 22:4 Luke 24:13-21
Son carried wood on his back up hill 22:6 John 19:17
God will provide for Himself the lamb 22:8 John 1:29
Son was offered on the wood 22:9 Luke 23:33
Ram in thicket of thorns 22:13 John 19:2
The seed will be multiplied 22:17 John 1:12; Isaiah 53:10
Abraham went down, Son didn't, "not mentioned." 22:19 Luke 23:46
Eph. 5:22-32;
Servant gets bride for son 24:1-4 Rev. 21:2,9; 22:17
The bride was a beautiful virgin 24:16 2 Cor. 11:2
Servant offered ten gifts to bride* 24:10 Rom. 6:23; 12; 1 Cor. 12

Tritheism
Tritheism is the teaching that the Godhead is really three separate beings forming three separate
gods. This erring view is often misplaced for the doctrine of the Trinity that states that there is but one
God in three persons: Father, Son, and Holy Spirit.

Uncial
The Greek characters of upper case: ABGDE, etc. Different copies of Greek manuscripts appear in
Uncial form. Minuscules are the lower case letters of the Greek alphabet
order and rank.

Unitarianism
A theological error that holds to the unity of God by denying the Trinity, the deity of Jesus, and the
deity of the Holy Spirit. Unitarians teach the unity of God and hold to a common system of believing
as you will about God, salvation, sin, etc. They often profess to have no dogma. Unitarians also hold
to the universal redemption of all mankind.

Universalism
The teaching that all people will eventually be saved through the universal redemption of Jesus.
Some universalists teach that even the devil, after a time of punishment, will be redeemed.

Vellum
A material used for writing, like paper. It was made from animal skins, usually from cattle, sheep,
goats, and antelope. The hair was scraped off of the skins, then they were washed, smoothed, and
dressed with chalk. Vellum was used until the late Middle Ages until paper was introduced into Europe
from China via Arab traders. Vellum lasted longer than papyrus and was tougher, but the edges
sometimes became torn and tattered. The two oldest parchment manuscripts are the Codex Vaticanus
(from Egypt) and the Codex Sinaiticus.

Venial Sin
In Catholicism, a sin but not as bad as mortal Sin. It lessens the grace of God within a person's
soul.

Vicarious Atonement
The theory of the atonement that states that Christ’s death was "legal." It satisfied the legal
justice of God. Jesus bore the penalty of sin when he died on the cross. His death was a substitution
for the believers. In other words, he substituted himself for them upon the cross. Jesus hung in our
place as He bore our sin in his body on the cross. See 1 Pet. 2:24.

Word, The
In Greek the word for "word" is logos. It is used in many places, but of special interest is how it is
used of Jesus. In John 1:1 it says, "In the beginning was the Word and the Word was with God and the
Word was God." The Word is divine and the word "became flesh and dwelt among us," (John 1:14). In
other words, Jesus is the Word of God who represents God to us and us to God.
The term is also used to describe the Scriptures (Rom. 9:6; Heb. 4:12), Christ's teaching (Luke 5:1),
and the gospel message (Acts 4:31).

The Word of God:


• is inspired: "All scripture is inspired by God and profitable for teaching, for reproof, for
correction, for training in righteousness," (2 Tim. 3:16).
• is truth: "all thy commandments are truth," (Psalm 119:151).
• makes free: "...If you abide in My word, then you are truly disciples of mine; and you shall
know the truth, and the truth shall make you free," (John 8:32).
• produces faith: "So faith comes from hearing, and hearing by the word of Christ," (Rom. 10:17,
NASB).
• judges: "For the word of God is living and active and sharper than any two-edged sword, and
piercing as far as the division of soul and spirit, of both joints and marrow, and able to judge
the thoughts and intentions of the heart," (Heb. 4:12).

Worship
The obligation of God's creation to give to Him all honor, praise, adoration, and glory due Him
because He is the holy and divine creator. Worship is to be given to God only (Exodus 20:3; Matt.
4:10). Jesus, being God in flesh (John 1:1,14 ; Col. 2:9), was worshipped (Matt. 2:2,11; 14:33; John
9:35-38; Heb. 1:6).

Wrath
Biblically, it is the divine judgment upon sin and sinners. It does not merely mean that it is a casual
response by God to ungodliness, but carries the meaning of hatred, revulsion, and indignation. God is
by nature love (1 John 4:16), however, in His justice He must punish sin. The punishment is called the
wrath of God. It will occur on the final Day of Judgment when those who are unsaved will incur the
wrath of God. It is, though, presently being released upon the ungodly (Rom. 1:18-32) in the
hardening of their hearts.
Wrath is described as God's anger (Num. 32:10-13), as stored up (Rom. 2:5-8), and as great
(Zech. 7:12). The believer's deliverance from God's wrath is through the atonement (Rom. 5:8-10).
"For God has not destined us for wrath, but for obtaining salvation through our Lord Jesus Christ, " (1
Thess. 5:9).

Yin and Yang


A dualistic philosophy of passive and active, good and bad, light and dark, positive and negative,
male and female, etc., and that they are in opposition, each is part of the whole and works together.

Yoga
A philosophical as well as physical way of life emphasizing harmony of body and mind. The
philosophy of yoga is based in Eastern Metaphysical beliefs. The goal of the philosophy is to help a
person become balanced in mind and body and attain self-enlightenment. Yoga, apart from its
metaphysical teachings, is beneficial to the body.
Zodiac
The stars in the heavens divided into twelve main groups. Imaginary patterns are imposed upon
various star patterns as an aid to remember the stars. In Astrology, the Zodiac signs signify
personality types for the people born under them. The Zodiac signs are determined by durations of
time in the calendar. The twelve signs are Aries, Taurus, Gemini, Cancer, Leo, Virgo, Libra, Scorpio,
Sagittarius, Capricorn, Aquarius, and Pisces.
Evangelism
Introduction

Evangelism is the call of every Christian. Whether you want to admit it or not, how you are, what you
do, what you say as a Christian tells others about Christ. Is your representation a good one?

God calls us to tell others about the saving work of Christ. Unfortunately, however, many people are
intimidated with the idea of sharing their faith. So, in order to help alleviate that fear and make you a
better witness, the following papers are here to help.

1. Why should we witness? p. 200


2. Why are foundations important in witnessing? p. 201
3. What are three important verses in witnessing? p. 203-204
4. What are the differences between justice, mercy, and grace? p. 206
5. Are you memorizing scripture? p. 207
6. Why is prayer important in evangelism? p. 209
7. What are some of the do's and don'ts of witnessing? p. 211
8. Should you preach the law before the gospel? Why or why not? pp. 212-213
9. Have you mapped out the Roman Road in your Bible? p. 214
10. What is Christian CPR? p. 215
Why Should We Witness?
Have you ever asked yourself, "Why should I witness?" Several reasons should come to mind.
First, because Jesus commands you to: "Go therefore and make disciples of all the nations, baptizing
them in the name of the Father and the Son and the Holy Spirit," (Matt. 28:19). Also, Ezek. 3:11 says,
"and go to the exiles, to the sons of your people, and speak to them and tell them, whether they listen
or not..."
Second, you must witness because you love the unsaved (if you don't, you should). The most
loving thing you can do is present the gospel in hopes of bringing others to salvation. Galatians 5:22
lists love as one of the fruit of the Spirit. It is love's nature to give. Take for example John 3:16, "For
God so loved the world that He gave His only begotten Son..." Love gives, and if you have only a small
portion of His love, you will want to give to others.
Third, witness because it is a wise thing to do. Prov. 11:30 says, "...he who is wise wins souls."
Now, I know I am not a very wise person. But, since God says I'll be wise to win souls, or try to, then
great, let me at it. I want to be wise in God's sight.
Fourth, witness to keep people out of hell. Hell is a terrifying place of utter anguish and eternal
separation from God. Those who are not saved go there. Witnessing is an attempt to keep them out of
hell.
Fifth, witness because it pleases God and brings glory to His name.
And finally, and most important, witness so they may find the love and fellowship of God ( 1 John
1:3), the greatest of all treasures. I can think of no greater gift than salvation. It frees the sinner from
sin, it delivers the lost from damnation, and it reveals the true and living God to those who don't know
Him.
The angels of heaven rejoice greatly when anyone passes from judgment into salvation (Luke
15:10). Shouldn't we as Christians rejoice too? Shouldn't we weep over the lost? Shouldn't we ask the
Lord of the field to send laborers into His harvest (Luke 10:2)? Certainly! The salvation of others is the
goal of your efforts. The love of God is your motive. Is there anything greater? So, give.
Foundations are First

"Be diligent to present yourself approved to God as a workman who does not need to be
ashamed, handling accurately the word of truth," (2 Tim. 2:15).

Without a proper foundation, buildings don't stand, civilizations don't last, and Christians don't
witness well. The foundation I am talking about is the foundation of basic Christian doctrine. Do you
know what the Trinity is? How many natures does Jesus have, one or two? Are we saved by grace
through faith or by grace and works? For what purpose did Jesus die? Did He rise from the dead? If so,
why?
Perhaps you do not believe that knowing doctrine is important. Maybe you think that we should
just tell people about Jesus and let them choose to accept Him or not. Unfortunately, witnessing isn't
always that simple. Knowing what and why you believe is essential. For example, if someone says he
wants to receive Jesus as Savior but doesn't believe that He is God in flesh, is that important? If
someone says that the Trinity is not biblical, what would you say? Is the Holy Spirit a force or God?
Doctrine is important because it defines who you put your trust in. It is not simply that you have
faith, it's who you put your faith in.

A "Setup" at church.

A few years ago an associate pastor of a local church asked me to give a talk on some biblical
issues to his college and career group. As we talked about the lecture he decided he would like me to
test the group on their doctrinal knowledge. He asked me to pretend I was someone off the street who
happened to walk in to "see what was going on and challenge them." Since no one at the study knew
me, it seemed like a great idea. He said, "Ask them questions that will make them think. Ask them
questions about their faith."
Since he wanted them to answer for themselves, he arranged not to be there. Instead, a Bible
study leader, who knew of the "setup," would be in charge. This way, the group would be forced to
defend their faith because their pastor wouldn't be there to bail them out when things got tough, and
they did.
I asked some questions about the Bible, got some answers, and then asked more questions about
their answers. I asked them how they knew the Bible was true, how they knew they were going to
heaven, why their beliefs were correct and everyone else's was false, and more. All I did was challenge
them. Later I asked them about Jesus. I said, "If Jesus is God then why did He pray to God the
Father?" The sudden silence spoke loudly against them. I continued with, "Some Jehovah's Witnesses I
was talked to a little while ago said that there was no such thing as the Trinity. They had a bunch of
proof texts. Why should I believe you and not them. They have answers and they seem to know their
Bible."
Needless to say, the group was thoroughly upset. One person left to get the pastor. Two girls were
doubting their salvation and a couple of guys told me later they wanted to beat me up (and these were
Christians!).
Just about then it was time to stop. The Bible study leader, who had kept tactfully quiet until then,
interrupted the discussion and, playing along for a few seconds longer said, "All right, all right. Let's
put a stop to this. " He looked confidently at everyone and said, "Don't worry. We are going to have a
speaker here tonight who will be able to answer all these questions and explain why the Bible really is
the word of God, why there is a Trinity, and all that other stuff we talked about."
A couple of people said to me, "Yeah, so why don't you stay and you can hear some answers." I
smiled and said, "Maybe I will." The Bible study leader smiled too and as he pointed to me he said,
"And tonight, he is our speaker." They stared at me for a moment. Then, as if on cue, they all began
to moan and groan. They had been set up. I smiled. After a while, they smiled too.
What are you called to do?

You are called by God to accurately know His word, "Be diligent to present yourself approved to
God as a workman who does not need to be ashamed, handling accurately the word of truth," (2 Tim.
2:15).
You are called by God to grow in your walk with Him, "Therefore, leaving the elementary teaching
about the Christ, let us press on to maturity..." (Heb. 6:1).
You are called by God to search the scriptures daily, "Now these were more noble minded than
those in Thessalonica, for they received the word with great eagerness, examining the Scriptures
daily, to see whether these things were so," (Acts 17:11).

How good is your foundation?

Are you like the people in that church? Are you weak in your basic Christian foundation? Can you
defend the Trinity biblically? Can you show someone in the Bible that Jesus is God in flesh or that
salvation is by faith only and not by our works? Do you know if Jesus rose from the dead in the same
body He died in or was it a different spiritual body? These questions are crucial and you need to have
the correct answers. Do you?
If your foundation is weak, then you need to strengthen it. Without a good foundation you won't be
able to stand against a breeze of opposition. That is why you must first establish your foundation: you
must witness in the strength of truth, not the weakness of error.
Just as a baby must crawl before it walks, a Christian must know the basics before he can mature:
"Therefore, leaving the elementary teaching about the Christ, let us press on to maturity..." (Heb.
6:1).
Let's first learn the elementary teachings before we go on.
Three Important Verses in Witnessing
"So shall My word be which goes forth from My mouth; it shall not return to Me empty, without
accomplishing what I desire, and without succeeding in the matter for which I sent it," (Isaiah
55:11).

If you only study one page on this section on evangelism, this page is it. Why? Because here is
where you will learn three of the most important verses of evangelism: Isaiah 55:11; Rom. 1:16; and
John 12:32.

Isaiah 55:11

Isaiah 55:11 says, "So shall My word be which goes forth from My mouth; it shall not return to Me
empty, without accomplishing what I desire, and without succeeding in the matter for which I sent it."
God's word is unique. It was by His word (speech) that God created, "God said, ‘Let there be light';
and there was light,” (Gen. 1:3). "By faith we understand that the worlds were prepared by the Word
of God..." (Heb. 11:3). Jesus is called the Word, "In the beginning was the Word and the Word was
with God and the Word was God... and the Word became flesh and dwelt among us..." (John 1:1,14).
When Jesus was on the earth, He taught. He taught with words and His words had a very strong
effect on people. They angered some and broke others. But when Jesus spoke, things happened.
Jesus forgave sins by speaking, "And seeing their faith, He said, ‘Friend, your sins are forgiven
you,'" (Luke 5:20). He raised the dead by speaking, "Lazarus, come forth,” (John 11:43). He calmed
the wind and sea with words, "Then He arose, and rebuked the winds and the sea; and it became
perfectly calm" (Matt. 8:26). He cast out demons by speech, "And He said to them, ‘Begone!' And they
came out..." (Matt. 8:32). He healed by speech, "And He stretched out His hand and touched him,
saying, ‘I am willing; be cleansed.' And immediately his leprosy was cleansed," (Matt. 8:3,13). God's
words are powerful.
The Word (the Bible) is described as having many qualities:

It is "All scripture is inspired by God and profitable for teaching, for reproof, for
inspired: correction, for training in righteousness," (2 Tim. 3:16).
It is truth: "The sum of Thy word is truth," (Psalm 119:151).
It makes "...If you abide in My word, then you are truly disciples of mine; and you shall
free: know the truth, and the truth shall make you free," (John 8:32).
It produces "So faith comes from hearing, and hearing by the word of Christ," (Rom.
faith: 10:17)
"For the word of God is living and active and sharper than any two-edged
sword, and piercing as far as the division of soul and spirit, of both joints and
It judges:
marrow, and able to judge the thoughts and intentions of the heart," (Heb.
4:12).

The Word of God is a mighty weapon. That is why you should memorize!
Romans 1:16

Rom. 1:16, "For I am not ashamed of the gospel, for it is the power of God for salvation to
everyone who believes, to the Jew first and also to the Greek." What is the Gospel? 1 Cor. 15:1-4 says
it is the sacrificial death and physical resurrection of Jesus for sins. It is the powerful message of
salvation to sinners. If you know that the Word of God will accomplish what God wants it to and if you
know that the gospel has power to save, then it should ease your mind to know that in witnessing you
are using two very powerful weapons: God's Word and God's Gospel. The Word of God is the Bible.
The Gospel of God is His revelation or redemption.

John 12:32

John 12:32, "And I, if I be lifted up from the earth, will draw all men to Myself." Here Jesus speaks
specifically about His crucifixion. It is He who draws to Himself all who are to be saved. When you
present the gospel (1 Cor. 15:1-4), the sacrificial death and resurrection of Jesus for sins, Jesus draws
the sinner to Himself. He does the work, not you.
As a Christian, you are to witness with truth, honesty, and integrity. As the Lord provides the
opportunity, you should respond in a humble and gentle spirit ( 2 Tim. 2:24). And in that, you should
point people to Jesus. It is He alone who saves.

If you know that God's Word will accomplish what God desires, that the gospel is powerful to save,
and that it is Jesus who draws all men to Himself, then you should realize that the responsibility of
salvation does not rest on you, but on God. You are the teacher, the deliverer of good news. "How
shall they believe in Him who they have not heard?" (Rom. 10:14). You help them hear!
These three verses should help you to gain confidence. You witness; God saves. You plant the
seeds; God waters. He uses you. It is His Word that accomplishes salvation. It is His gospel that is
powerful. It is His Son Jesus who draws. You witness in power when you witness with the Word of God.
Salvation is God's work
"Salvation belongs to the Lord," (Psalm 3:8).

When someone appeals to God and seeks forgiveness in Jesus, his sins are removed, he is
cleansed, his relationship with God is restored, and he is made a new creature (2 Cor. 5:17). All of this
is the work of God, not man.
The Bible has a phrase that describes the non-Christian. It is 'natural man'. In 1 Cor. 2:14 Paul
says, "But a natural man does not accept the things of the Spirit of God; for they are foolishness to
him, and he cannot understand them, because they are spiritually appraised."
Our human condition can be compared to a drop of poison in a glass of water: all the water is
poisoned but it is not as bad as it could be. The water is incapable of being good. We, too, are
incapable of really being good.
When Jesus' disciples asked Him who can be saved, He replied, "With men this is impossible, but
with God all things are possible," (Matt. 19:26). That is why salvation rests in God alone by grace
through faith (Eph. 2:8-9).
About now you are probably wondering what this has to do with witnessing. Why do you need to
know all this? I am glad you asked. It is helpful to know because you must realize it is God who saves
people. Specifically, it is the Holy Spirit who convicts the sinner of sin -- not you. "And He [the Holy
Spirit], when He comes, will convict the world concerning sin, and righteousness, and judgment,"
(John 16:8).
Remember, the gospel is preached after sin is made known. Because the sinner cannot come to
God on his own, he must be convicted of his sin, and thus be made aware of his need for salvation.
The conviction of sin is beyond our control. It is the work of the Holy Spirit ( John 16:8).

Prayer is essential in witnessing

It is, then, vital that you pray and request God to convict as well as save. Prayer is an essential
part of witnessing. When you witness you must pray. Then you are free to spread the gospel as
effectively as you want and to trust God to give the increase (1 Cor. 3:6-7). Ask Him to send the Holy
Spirit; ask Him to convict the world of sin. The work of the Holy Spirit is essential in salvation.

The Holy Spirit

The Holy Spirit works in two types of people during witnessing: the saved and the unsaved. In the
saved, He dwells within (Rom. 8:11), teaches (John 14:26), anoints (1 John 2:27), guides (John
16:13), and sanctifies (1 Pet. 1:2). Without the Holy Spirit we would be like ships without rudders,
unable to live as Christians and certainly unable to witness effectively.
In the unsaved, He convicts of sin (John 14:8). Sinners come to Jesus to have their sins cleansed.
They do this after they discover their guilt before God. This too is the work of the Holy Spirit.
Since the natural man is separated from God because of his sin (Isaiah 59:2), the Holy Spirit uses
the Scriptures (that you quote) to convict him of his sinfulness, convince him of his need for salvation,
and convert him through the Word. When a natural man (or woman) is aware of his sinful condition
then the gospel message of deliverance from sin is preached and becomes effectual.

Sin

Sin does two things: it offends God and it kills man. How? It offends God because it is His law we
break. It kills us because of the nature of Law. Have you ever heard of a law without a punishment? A
law without a punishment is only a slogan. Since God is just and laws have punishments, then God
must punish the lawbreaker. But that is not the end of the story. God is also merciful and gracious. His
justice fell upon Himself -- on the cross. His mercy falls upon us -- by grace through faith.
Justice, Mercy, and Grace

Imbedded in the relationship of Law and Gospel are the concepts of justice, mercy, and grace. One
of the best ways you can show the difference between them is to use illustrations that show their
differences and relationships. For example, Justice is getting what we deserve. Mercy is not getting
what we deserve. Grace is getting what we don't deserve.
Let's suppose you have a bicycle and I want it. So, one night I sneak over to your house and steal
it. You catch me and I go to jail. (Jail would be where I "pay" for my crime of breaking the law.) The
penalty is met and that is justice. I get what I deserve.
Let's change it a little. I sneak over to your house and steal your bike. You catch me. But you
don't send me to jail. Instead, you tell me to forget about it. The penalty, jail, is not met. That is
mercy. I did not get what I deserved.
One more change. I sneak over to your house and steal your bike. You catch me. You don't send
me to jail. In fact, you give me the bike plus a hundred dollars. That is grace. The penalty is met (by
you paying the 'damages') and I was given what I did not deserve (the bike and money).
Justice, which demands payment, does not meet the requirement of mercy, which seeks
forgiveness. Mercy does not meet the requirement of justice. Grace meets both.

The Lamp Analogy25

Let's say I am at your house or apartment with my wife. We are talking about church and in my
zeal I accidentally knock over your lamp. Now, this lamp is special. A dear friend gave it to you and it
has great sentimental value, and besides, you need a light in your room. After a moment or two you
realize that the damage is done and decide to forgive. You say to me, "That is alright, Matt. I forgive
you for breaking the lamp, but give me ten dollars."
Is asking for ten dollars after you've just forgiven me, true forgiveness? Certainly not! When God
forgives our sins, He says He will remember them no more (Jer. 31:34). Forgive and forget are similar
in spelling and similar in meaning. If you forgive me can you demand payment from the one forgiven?
No, because a forgiven debt does not exist.
Let's say that instead of asking me for ten dollars you turn to my wife and say, "Matt broke my
lamp. You give me ten dollars for it." I ask you again. Is that true forgiveness? No. You are simply
transferring the debt to someone who was not involved in the original offense.
But, we have a problem. The lamp needs to be replaced. In true forgiveness, then, who pays for
its replacement? (Think about this a bit before you go on to read the answer.) Who pays? You do!
You're the only one left. Remember, if you've forgiven me the debt, how can you demand payment?
Now, who was my offense against? You. Who forgives? You do. Who pays? You do.
When we sin, who do we sin against? God. Who forgives? God. Who pays? God! Did you get that?
God pays! How does He do that? Simple. 2000 years ago on a hill outside the city of Jerusalem He
bore our sins in His body and died on the cross (1 Pet. 2:24). He took our punishment: "Surely our
griefs He Himself bore, and our sorrows He carried... He was pierced through for our transgressions,
He was crushed for our iniquities; the chastening for our well-being fell upon Him..." (Isaiah 53:4-5).
God is just. God is merciful. God is gracious. In the justice of God, He took our place. In the mercy
of God we don't get punished. In the grace of God, He gives us eternal life. Even though we are
unworthy of salvation, even though we are unworthy of God's love, even though we are unworthy of
mercy, even though we are worthy of wrath, God saved us. He did so not because of who we are, but
because of who He is, not because of what we do, but because of what He did. God is love (1 John
4:16). God is holy (1 Peter 1:16). God is good (Psalm 34:8). We could never fathom the depths of His
purity and kindness (Rom. 11:33). We could never, through our own efforts, attain Him. There is only
one thing left for us. We must worship Him, love Him, and serve Him. He alone is worthy. Blessed be
the name of the Lord.

25
I do not remember where I read the lamp analogy, but I have been using it for many years. Someone contacted
me and mentioned that it was used by Josh McDowell. If that is the case then he receives the credit for the
analogy.
How to Memorize Scripture
Many people don't think they can memorize their way out of a new shopping mall let alone
remember a verse in the Bible. I've spoken with dozens of people about Scripture memorization who
say the same thing: "I have a terrible memory." My answer to them is, "Nonsense!" Everyone
memorizes all the time. You've learned to speak, haven't you? You've memorized thousands of do's
and don't's in life, right? You know your social security number, your phone number, your address,
how many brothers, sisters, or children you have, how to get to work and what to do when you get
there. The problem is not that people can't memorize, it's that they won't memorize.
The Bible is the foundation for witnessing and you must memorize verses to be able to use it --
this applies to your devotional life as well as witnessing. One serious problem I've found is that most
people have memorized fewer than seven verses. The average Jehovah's Witness has committed many
more to memory and, given the opportunity, could make the average Christian feel about as
knowledgeable as banana bread. What I would like to do is help you memorize better.

Four easy steps to memorization

There are four easy steps to Scripture memorization. Let's use 1 Peter. 2:24 as an example: "and
He Himself bore our sins in His body on the cross, that we might die to sin and live to righteousness;
for by his wounds you were healed," (NASB).

Step 1: Location

The first step is to memorize the location, not the verse. The reason for this is if you forget the
verse, but you've memorized the location, you can always go look it up.
First, memorize the location: "1 Peter 2:24." Say "1 Peter 2:24" over and over again. Don't worry
about what it says at first, just memorize the location. Make sure that when you say 1 Peter 2:24 it
flows smoothly off your lips. Say 1 Peter 2:24 emphasizing different syllables. Say 1 Peter 2:24, or 1
Peter 2:24 or 1 Peter 2:24 or even 1 Peter 2:24. But say 1 Peter 2:24 enough times that when you
say 1 Peter 2:24, it is as natural as breathing.

Step 2: Gist

The second step is to learn the gist of what the verse is. In this case it is very simple, "Jesus bore
our sins in His body on the cross." Say, "Jesus bore our sins in His body on the cross" in different
ways. Say, "Jesus bore our sins in His body on the cross," or "Jesus bore our sins in His body on the
cross," and "Jesus bore our sins in His body on the cross," etc. But say "Jesus bore our sins in His
body on the cross" enough times that when you say, "Jesus bore our sins in His body on the cross," it
is as natural as saying 1 Peter. 2:24. (Kind of repetitive, isn't it?)

Step 3: Association

The third part is more fun. This is where you associate the two together. Say, "1 Peter 2:24 is
Jesus bore our sins in His body on the cross. Jesus bore our sins in His body on the cross is 1 Peter
2:24. 1 Peter 2:24 is Jesus bore our sins in His body on the cross..." Say this over and over again,
about ten times. In no time, if you do this, you will memorize.
This association part is important because it helps you to think of one part whenever you think of
the other. For example, if someone asked you "Where does it say that Jesus bore our sin in his body?",
you'd immediately reply with "1 Peter. 2:24." It works.
Step 4: A piece of paper

The fourth and final part is to take a piece of lined paper, 8 1/2" by 11", and draw a vertical line
about one inch from the left hand side.
Write the verse location in the left column on your paper and on the right side simply write the
verse. Do this with each verse you want to remember. Fold it up, put it in your pocket or purse, and
carry it with you everywhere you go. When you forget a verse or its location simply pull out the paper
and refresh your memory. In no time at all, you'll have over one hundred verses committed to
memory.

Memorization is like exercise. The more you do it, the easier it gets; the less you do the harder it
gets. So do it. If you follow this procedure your mind will become like a sponge, you'll end up
memorizing all sorts of stuff with the greatest of ease, like how many socks are in your drawer,
everything that is in your refrigerator, and even where your car keys are.

And one more thing: You will be amazed at how the Lord uses what you've memorized.
The Importance of Prayer in Evangelism
"The harvest is plentiful, but the workers are few. Therefore beseech the Lord of the harvest to
send out workers into His harvest," (Matt. 9:37-38).

Prayer is essential in the Christian's life. Without it your witness will be far less effective and you
will be far more vulnerable to the enemy. When you witness, you need the blessing and support of the
Lord. You need to be in fellowship with Him. Prayer makes this all possible.
When you witness you plant the seeds of the Gospel, but it is God who causes the growth ( 1 Cor.
3:6-7). In prayer you ask God to give that growth. In prayer you ask God to convict the unrepentant
of their sin and by that awaken in them the need for salvation. In prayer you, "...let your requests be
made known to God," (Phil. 4:6). Think back to your own conversion. Were there people praying and
requesting your salvation?
Jesus prayed frequently (Matt. 14:23; 26:36; Mark 6:46; Luke 5:16; John 17). Paul prayed (Rom.
1:9; Eph. 1:16). Stephen prayed (Acts 7:55-60). You must pray. God wants you to pray to Him and
have fellowship with Him (John 1:1-4). Why? One reason is that our battle is not against flesh and
blood but against powers and the spiritual forces of darkness (Eph. 6:12). That is where the real battle
is, in the spiritual realm. You need prayer. Prayer is one of God's ordained means for you to do
spiritual warfare and sharing the Gospel is definitely spiritual warfare.
Another reason to pray is that you can actually influence God with your prayers. If you are
doubtful then look at 2 Kings 20:1-7. King Hezekiah was told by the prophet Amoz that he should set
his house in order because he was surely going to die (v. 1). Hezekiah prayed earnestly (v. 2,3). The
Lord heard his prayers and said, "I have heard your prayer, I have seen your tears; behold, I will heal
you. On the third day you shall go up to the house of the LORD. And I will add fifteen years to your
life," (v. 5).
Hezekiah's prayer made a difference. That is why you, as a Christian, can be an effective witness,
because you have influence with God and because you can ask God to save. Prayer is a vital part of
witnessing.

What should you pray for?

Pray for more people to witness. Jesus specifically asked you to pray to the Father and ask Him to
send workers into the field (Matt. 9:37-38). What is the field? It is the world of sinners. Who are the
workers? They are people like you. Jesus wants people to find salvation and enjoy eternal fellowship
with Him. He wants you to preach the Gospel. He has given the command "Go therefore and make
disciples of all the nations..." (Matt. 28:19). Your witness for God may or may not be verbal. But either
way, you need to pray and ask God to give you strength, love, and insight.
Pray for compassion for the lost. Compassion is a necessary element in witnessing. It motivates
you to speak, to teach, and to pray for others to come into the kingdom of God. Compassion helps you
to cry over the lost and to come to God in humble request for their salvation. Paul said, "Brethren, my
heart's desire and by prayer to God for them is for their salvation," (Rom. 10:1).
Pray for the desire to witness. Pray this regularly and watch the Lord change you and give you a
desire to reach out and tell people about Jesus. God will grant your prayers and joy will fill your heart
as you fulfill the command of God by witnessing.
Pray for boldness. Pray for the courage to step out in faith and speak up when needed. Many
Christians are timid because speaking a word for the sake of the Lord can be risky and frightening.
Boldness gives you the courage to risk ridicule and to endure the scorn. Ask God for it. "For God has
not given us a spirit of timidity, but of power and love and discipline. Therefore do not be ashamed of
the testimony of our Lord," (2 Tim. 1:7-8).
Pray to the Lord to bind Satan and his angels. There is a hierarchy of demons seeking to hinder
your witness and steal the seeds of the Gospel that you plant. You cannot fight spirits with reason or
flesh and blood, but you can ask the Lord to fight. With prayer you can assault the camp of the enemy
and weaken his false kingdom. Prayer is a mighty tool, a powerful tool. You need it if you are going to
witness.
Pray for your needs. Do you have a close walk with God? Do you need a deeper fellowship with
Him? Do you have sins you need to confess and forsake? If so, then pray. Enjoy your privilege of
coming to the Creator of the universe who meets your every need. He loves you. He wants to hear
from you and He wants you to make your needs known to Him. "Be anxious for nothing, but in
everything by prayer and supplication with thanksgiving let your requests be made known to God,"
(Phil. 4:6).

Hindrances to prayer

Prayer is important for many reasons, especially for witnessing. But prayer can be hindered. So
that your prayers and witnessing might be as affective as possible, a discussion of the hindrances of
prayer is necessary. Do any of the following apply to you?
Sin hinders prayer. "If I regard wickedness in my heart, the Lord will not hear," (Psalm 66:18). We
all sin, but do you have unconfessed and unrepented sin in your life? If so, confess your sin, repent
from it as you are commanded in Acts 17:30, and continue in witnessing and prayer.
Selfishness hinders prayer. "You ask and do not receive, because you ask with wrong motives, so
that you may spend it on your pleasures," (James 4:3). Examine yourself. Make sure your prayers are
not motivated by selfish desires. If you find that selfishness is a factor then confess it and repent.
Doubt hinders prayer. "But let him ask in faith without any doubting, for the one who doubts is
like the surf of the sea driven and tossed by the wind," (James 1:6). We all doubt. We all fail. But
when you doubt be reminded of the man who said to Jesus, "Lord I believe, help my unbelief," (Mark
9:24). He believed and yet doubted and Jesus granted his request. Remember that God has given a
measure of faith to every man (Rom. 12:3). Trust God, even when you have doubts. It does not
matter necessarily how much faith you have as much as who your faith is in. Put what faith you have
in Jesus. Trust Him. Watch Him be faithful to you.
Pride hinders prayer. Jesus spoke of the Pharisee and the tax-gatherer who both were praying.
The Pharisee boasted about himself while the tax-gatherer asked for mercy from God. Jesus said in
Luke 18:14 regarding the tax-gatherer, "I tell you, this man went down to his house justified rather
than the other." Jesus shows us that pride is sin and that it hinders prayer (James 4:6). Have the
same attitude that Jesus had in heaven in His full glory as He had on Earth as a man. He was humble.
If you are prideful, confess it as sin, repent, and continue in humility.
A poor husband and wife relationship hinders prayer. This may seem a little out of place here, but
it isn't. A proper relationship with your spouse is very important. If there are problems because of
selfishness, pride, argument, anger, unforgiveness, or any of the other multitudinous obstacles that
can develop in marriage, then your prayers will be hindered. How are you doing with your mate? Are
you witnessing while there is anger between you two? In Matt. 5:23-24 Jesus said, "If therefore you
are presenting your offering at the altar, and there remember that your brother has something against
you, leave your offering there before the altar, and go your way; first be reconciled to your brother,
and then come and present your offering." Are you reconciled to your wife or husband (for that matter,
anyone you know with whom there is strife) before you offer sacrifices of witnessing and prayer to the
Lord? If not, then be reconciled, so your prayers won't be hindered. 1 Pet. 3:7 says, "You husbands
likewise, live with your wives in an understanding way, as with a weaker vessel, since she is a woman;
and grant her honor as a fellow heir of the grace of life, so that you prayers may not be hindered."

Prayer is a privilege

Prayer is a privilege. It is a powerful tool. Without it you will be a foolish worker in the fields of the
dead. Pray and ask the Lord of the harvest to raise the dead to life. Bend your knees in fellowship with
your Lord. Let Him wash you in His presence and fill you with the Holy Spirit. Prayer is where you meet
Him. Prayer is where you are shaped. Pray.
The Do's and Don't's of Witnessing
"Conduct yourselves with wisdom toward outsiders, making the most of the opportunity. Let
your speech always be with grace, seasoned, as it were, with salt, so that you may know how
you should respond to each person," (Col. 4:5-6).

Like most things in life, witnessing has guidelines. Following is a list of 20 Do's and 12 Don't's.
They should aid you while witnessing and help prevent serious errors. If, however, you choose to
ignore them, witnessing will be difficult and awkward.

1. Do's
A. Do Pray.
B. Do speak to please God.
C. Do read your Bible.
D. Do start with a positive witness for Christ.
E. Do keep things simple.
F. Do share your salvation experience with them.
G. Do know what you believe.
H. Do have a genuine love.
I. Do be simple and define your terms.
J. Do memorize appropriate Scriptures if possible.
K. Do be ready to learn from the people you witness to.
L. Do be patient and gentle.
M. Do listen attentively.
N. Do answer their questions.
O. Do ask questions.
P. Do let him save face.
Q. Do bring him, if possible, to a decision about Jesus.
R. Do encourage him to study the Bible by itself.
S. Do use Scripture in context.
T. Do remember that greater is He that is in you than he that is in the world (1 John 4:4).
2. Don'ts
A. Don't attack directly or make fun of someone.
B. Don't jump from one subject to another.
C. Don't expect too much from him.
D. Don't have a spiritual chip on your shoulder.
E. Don't lose patience.
F. Don't come on too strong.
G. Don't debate peripheral issues or doctrines.
H. Don't get sidetracked defending your denomination.
I. Don't be uptight.
J. Don't assume.
K. Don't argue.
L. Don’t speak to fast or unclearly.

I hope these do's and don't's have brought to your attention areas that would improve your
witnessing. If some of them have struck you as being particularly applicable then I would suggest
you think them over and in prayer ask God to work on your heart and teach you the right way to
witness. He will bless you. All you need is to trust Him and go witness.
Law and Gospel
"...for through the Law comes the knowledge of sin," (Rom. 3:20).

In the Old Testament God gave the Law through Moses. It is the commands and precepts that
govern human conduct. In the New Testament God gave the Gospel through Jesus. It is the message
of salvation by grace through the sacrificial death and physical resurrection of Jesus, for our sins.
The Law is the do's and don't's of moral behavior. It consists of the 10 commandments (Exodus
20), rules for social life (Exodus 21:1-23:33), and rules for the worship of God (Exodus 25:1-31:18).
It was a covenant of works between God and man and was (and is) unable to deliver us into eternal
fellowship with the Lord. The Law is a difficult taskmaster because it requires that we maintain a
perfect standard of moral behavior. And then when we fail, the Law condemns us to death. Works do
not earn us salvation or play any part of it. The Bible says that a man is justified by faith apart from
the works of the Law (Rom. 3:28).
The Gospel, on the other hand, is the good news of the death, burial, and resurrection of Jesus for
our sins (1 Cor. 15:1-4). It is the message of what God has done for us, our deliverance from sin and
the punishment of the Law.
"Law and Gospel" are also part of our foundation and a good understanding of their relationship
will greatly help your witnessing. How? If you understand that the Law of God is a standard of
perfection, that it reveals sin, that we are unable to keep from breaking it, and that the Gospel frees
us from the need to keep the law perfectly, then in order to obtain forgiveness of sins you will be
better able to communicate the message of salvation to the unsaved.

The Law is different from the Gospel

Most Christians already have a basic understanding of the difference between Law and Gospel;
they just don't know they do. For example, "You are a sinner (Law). You need Jesus as your Savior
(Gospel)." The Old Testament (Law) came before the New Testament (Gospel). The Law shows us
what we are guilty of and the Gospel delivers us by grace. First we must know we are guilty (Law)
before we recognize our need to ask for forgiveness (Gospel).
The Law kills. The Gospel makes alive. When Moses came down from the mount after receiving the
Law and saw that the Israelites had fallen into idolatry, he threw the tablets of the Law down to them
and 3000 people died (Exodus 32:28). Later, when Peter preached the Gospel, 3000 people were
saved (Acts 2:41).
With a better understanding of the Law, it will be easier for you to explain sin. Without the Law,
sin cannot be known; Romans 3:20 says, "...through the Law comes the knowledge of sin." (See also
Rom. 7:7.) If sin is not known, then the need for Jesus is not felt. This is why you mention the Law to
those with whom you witness. How? By asking them if they have ever sinned. Tell them that lying,
cheating, stealing, lusting, not honoring God, etc. is sin. Everyone is guilty somewhere ( Rom. 3:23),
so everyone needs to be delivered. Everyone needs the Gospel.
The Law is peculiar. It says "be holy," but shows us we are not. It says "do not lie," but shows
where we do. It says "honor the Lord your God," yet shows us where we fail. Since none of us can
keep the whole Law, we are all under condemnation. There is no way out. What can we do? Nothing!
That is why "the Law has become our tutor to lead us to Christ, that we may be justified by faith,"
(Gal. 3:24). The Law guides us to Him. How does it do that? By showing us that the attempt to keep
the Law (our works) is insufficient to give eternal life and that the Gospel of grace is the only way to
God.
In other words, you must help the person realize that they are not good enough to merit God's
favor. People tend to think that because they are sincere or "not that bad," they are going to be with
God when they die. But the Bible reveals that "sincerity" and being "not that bad" are not good
enough. God requires perfection.
Salvation is of God.

That is why salvation belongs to the Lord (Psalm 3:8), by faith and not by works (Rom. 4:5). That
is why it is a free gift of God (Rom. 6:23), through grace (Eph. 2:8-9). That is why God became man
(John 1:1,14) and fulfilled the Law: "For what the Law could not do, weak as it was through the flesh,
God did; sending His own Son in the likeness of sinful flesh and as an offering for sin, He condemned
sin in the flesh," (Rom. 8:3). And also, "For if a law had been given which was able to impart life, then
righteousness would indeed have been based on Law," (Gal. 3:21); "For by grace though faith you
have been saved, not by works..." (Eph. 2:8). And, "...but to the one who does not work, but believes
in Him who justifies the ungodly, his faith is reckoned as righteousness,” ( Rom. 4:5).
In presenting the Gospel, you show how the requirements of keeping the Law perfectly is
removed. Say something like, The Bible says that if you break just one command of God, you are
condemned, (James 2:10-11). I often add, "Sin can be forgiven but the effects continue. The effect of
your sin is death. Your sin is an offense to the Law-giver, God. But Jesus, who is God in flesh, bore our
sins on the cross and died with them. If you want your sins forgiven, then you need to come to Christ
and ask Him to forgive you. He will."
The Four Spiritual Laws
If you've gone through a discussion with someone and you want to present the gospel message in
a simple and systematic way, the well known "Four Spiritual Laws" can be of help. They are simple, to
the point, and use Scripture to convict, convince, and convert. They are:

1. God loves you:


• "For God so loved the world, that He gave His only begotten Son, that whoever believes
in Him should not perish, but have eternal life," (John 3:16).
2. Man is sinful and separated from God.
• "For all have sinned and fall short of the glory of God," (Rom. 3:23); "For the wages of
sin is death," (Rom. 6:23); "But your iniquities have made a separation between you
and your God," (Isaiah 59:2).
3. Jesus Christ is God's only provision for man's sin.
• "I am the way, and the truth, and the life; no one comes to the Father, but through Me,"
(John 14:6); "But God demonstrates His own love toward us, in that while we were yet
sinners, Christ died for us," (Rom. 5:8).
4. We must individually receive Jesus as Savior and Lord.
• "But as many as received Him, to them He gave the right to become children of God,
even to those who believe in His name," (John 1:12); "if you confess with your mouth
Jesus as Lord, and believe in your heart that God raised Him from the dead, you shall be
saved," (Rom. 10:9); "For by grace you have been saved through faith; and that not of
yourselves, it is the gift of God," (Eph. 2:8).

The Roman Road

Another list of verses usable in the same way as the Four Spiritual Laws is the "Roman Road." The
advantage to these seven verses is that they are all in the book of Romans. Sometimes this is an
advantage when you don't want to flip through a lot of pages.

1. Rom. 3:10, "As it is written, 'There is none righteous, not even one...'"
2. Rom. 3:23, "For all have sinned and fall short of the glory of God."
3. Rom. 5:12, "Therefore, just as through one man sin entered into the world, and death through
sin, and so death spread to all men, because all sinned."
4. Rom. 6:23, "For the wages of sin is death, but the free gift of God is eternal life in Christ Jesus
our Lord."
5. Rom. 5:8, "But God demonstrates His own love toward us, in that while we were yet sinners,
Christ died for us."
6. Rom. 10:9-10, "if you confess with your mouth Jesus as Lord, and believe in your heart that
God raised Him from the dead, you shall be saved; for with the heart man believes, resulting in
righteousness, and with the mouth he confesses, resulting in salvation."
7. Rom. 10:13, "For whoever will call upon the name of the Lord will be saved."

I recommend you put the Roman Road in your Bible. Go to Romans 3:10 underline it and write
Romans 3:23 next to it. Then go to Romans 3:28, underline it and write Romans 5:12 next to it, and
so on. That way all you need to do is memorize where you start: Romans 3:10
Christian CPR
"If we confess our sins, He is faithful and righteous to forgive us our sins and to cleanse us
from all unrighteousness,” (1 John 1:9).

Witnessing is a spiritual battle where you not only fight against ignorance and prejudice but also
against Satan. He will attempt to hinder your efforts, attack you spiritually, and do a thousand other
things to stop you. As a result, you can suffer spiritually dry times brought on by his attacks. So what
do you do to save your spiritual life? Simple . . .
Almost everyone has heard of CPR, cardiopulmonary resuscitation. It has saved many lives. I
would like to introduce you to Christian CPR It can save your spiritual life. It's simple: Confess, Pray,
and Read.

Confess

The first of the three letters represents Confession: our human need. Each person who witnesses
should have his own life right with God. That doesn't mean being perfect, but it does mean actively
seeking to walk in accord with God's will. It means regularly confessing your sins to God and forsaking
them. This is done in prayer.
Sin is not something to be taken lightly. It is so bad, so evil, so wicked, that it cost Jesus His life.
The greatness of the sacrifice of Christ only reflects the greatness of the depth of sin. It took
something as incredible as God on the cross to undo sin. Sin can hinder your effectiveness so you need
to make sure you confess any sin to God. He will forgive you and your fellowship with Him is restored.
In that proper relationship, He will guide you and empower you to speak boldly for Him.
The Bible says, "If we confess our sins, He is faithful and righteous to forgive us our sins and to
cleanse us from all unrighteousness,” (1 John 1:9). Confession is good for the soul they say -- and it is
true. It is good to bow before the Lord in humility and seek His forgiveness: "Humble yourselves in the
presence of the Lord and He will exalt you," (James 4:10). Be ready to confess and forsake your sins.
That is what God asks.

Pray

The second letter represents Prayer: your special privilege. Through prayer you are in fellowship
with the Holy Creator of the universe. You can actually speak to Him. You can worship, love, and
spend time with Him. Because of what Jesus has done for you on the cross, God hears your prayers.
When you desire to pray, is it your flesh that seeks Him? No. Since your natural self does not seek
God and since you are still in sinful flesh, when the inclination to pray comes over you, it is God calling
you to spend time with Him. He wants you to fellowship with Him. 1 Cor. 1:9 says, "God is faithful,
through whom you were called into fellowship with His Son, Jesus Christ our Lord." He wants you to be
in His presence and be dependent upon Him. The one who is in the presence of the Lord cannot but
have his heart filled by Him. He cannot but speak of Him: "Out of the abundance of the heart, the
mouth speaks,” (Matt. 12:34). When Moses was in the presence of God, his face shone (2 Cor. 3:7).
When you are in the presence of God, your heart will shine.
Prayer is the practice of the presence of God. To be effective, you need to be in fellowship with
God. To be in fellowship, you need to be praying, constantly.

Read

The third letter represents Reading: your daily bread. It is in reading the Bible that God speaks to
you. The Bible is, of course, the Word of God. It is, "inspired by God and profitable for teaching, for
reproof, for correction, for training in righteousness; that the man of God may be adequate, equipped
for every good work," (2 Tim. 3:16-17).
The Word of God is one of the ways He has made Himself known to you. It is light for your soul,
food for your thoughts, and the guide by which you should shape your life.
By reading the Bible and memorizing Scripture, you are a much greater threat to Satan. When
Jesus was tempted by Satan what did He use to rebuke Him? A miracle? A powerful sweep of His
hand? No. He quoted Scripture (Matt. 4).
Follow Jesus' example. Learn Scripture. Use it. Let it dwell in your heart and mind. Rebuke the
enemy with it. Learn from it. It will nourish you.

Prayer and Fellowship

Fellowship with God is a privilege and a great blessing. There is nothing better or greater to have.
The purpose of Christian CPR is to help remind you of the requirements for fellowship with God.
Fellowship with the Lord should be the number one priority in your life. If it isn't, make it so.
When Solomon was asked by God what he wanted, Solomon desired wisdom (1 Kings 3:5-9).
When David was asked what he wanted, he said, "That I may dwell in the house of the LORD all the
days of my life,” (Psalm 27:4). In the New Testament whenever Solomon is mentioned, it is not in a
favorable light. Jesus said, "Not even Solomon in all his splendor was clothed as beautifully as one of
these lilies,” (Matt. 6:28-29). Yet, David, a murderer and adulterer, was called by God a man after His
own heart (Acts 13:22). Why? I believe it is because David sought fellowship with God. David wanted
only to be in the presence of the Lord. David, the sinner, loved God.
Jesus was at the home of Mary and Martha (Luke 10:38-42). Martha was busy with her
preparations and Mary was at Jesus' feet. Martha mentioned that Mary was doing nothing and had left
all the work to her. Jesus' reply is enlightening. He said, "Martha, Martha, you are worried and
bothered about so many things; but only a few things are necessary, really only one, for Mary has
chosen the good part, which shall not be taken away from her."
You see, the Lord desires that you spend time with Him. It is more important than all your other
responsibilities, even witnessing. If you are in fellowship with the Lord, then your walk will be stronger,
your love bolder, and your sin weaker. You will shine as a light to the world (Matt. 5:14) and the world
will know that you are His disciple (Matt. 5:16).
When you sense the desire to pray, answer it. It is God calling you to spend time in His wonderful
presence and be filled by Him. And there in His presence is where you will gain the greatest benefit
and become the most powerful witness.
As you can see, CPR is an easily remembered tool that should help you understand the importance
of confession, prayer, and reading. In daily prayer time seek God. In daily reading time seek God. Be
like David who wanted to dwell in the house of the Lord forever. Be like Mary who wanted to sit at
Jesus' feet and be with Him. Be like Jesus who so often went to prayer for fellowship with God. Obey
the Father who has commanded you to fellowship with Jesus. In Him are the blessings to keep your
heart alive because, "grace and truth were realized through Jesus Christ,” (John 1:17). It is in true
fellowship with Jesus that witnessing is at its best. Witnessing is not just words -- it is a way of life.
Leading Someone to the Lord
Alright, so you know about the Law, the Gospel, sin, salvation, the Four Spiritual Laws, and the
Roman Road, but how do you lead someone to the Lord? How do you introduce someone to Christ?
First of all, you tell them about God, sin, and Jesus dying on the cross. So, let's say that after all
this, the person you're witnessing to is touched by God's Word and wants to become a Christian. You
say...
"Do you understand what I've been telling you?"
"Yes."
"Do you want to receive Jesus as your Savior?"
"Yes. Yes I do."
"Good. But first, I need to tell you something. Being a Christian means living for Christ. It means
seeking to do His will and not your own. It won't always be easy. Satan will make your life difficult at
times. You may lose friends and things won't necessarily get better overnight. Being a Christian can be
costly. In fact, Jesus said that you must deny yourself, take up your cross daily, and follow Him ( Luke
9:23). That is what Jesus wants you to do, to follow Him always -- even when others don't want you
to.
"Now, If you still want Jesus as your Savior and Lord, I would be glad to pray with you."
"Yes, I do. I understand and I still want Jesus."
"Then let's pray." You tell him to repeat what you say, maybe bow his head and close his eyes, or
not... It doesn't matter.
An example of what to say is "Dear Jesus, I know I have sinned against you. I confess that I am a
sinner. I accept the sacrifice you made on the cross on my behalf. I ask you to come into my heart.
Please cleanse me of my sin. And give me eternal life. I repent from my sins and put my trust in You.
Thank you Jesus. Amen."

The preceding paragraph is only an example of how to lead someone to Christ. As I've said before,
there is no formula, but only a recognition of one's sinfulness before God, confession and repentance
of sin, and trusting in Jesus' sacrifice for sin.
I would like you to take special note to tell the person that being a Christian is not easy. In a way,
when you do this you are trying to talk him out of being saved. But what is really happening is three
things: 1) you are making sure conversion is real, 2) you are warning him of what is to come, and 3)
you are trying to prevent a 'false conversion' and the excuse he might say later like, "Oh, I tried
Christianity once. It didn't help me." Remember, we receive Christ for the forgiveness of sins, not to
"make things better." Also, you want someone to come to Christ with full knowledge of what it means
to be a Christian.
If we aren't willing to follow Jesus through thick and thin, then we aren't worthy to be called His
disciples. We are to love Him more than anything else (Luke 14:26-28).
Also worth noting is the short sentences used in the prayer. Don't worry about exactly what to
say, just speak the truth, confess sin, ask Jesus for forgiveness, speak slowly, and use short
sentences. This way, there will be no confusion.
Trust God and Go Witness
It is one thing to read about how to witness to someone; it is another to put into practice what
you've learned. I tell my students, "If you want to witness, then go witness." That is the best, though
not the easiest, way to learn. I call it "Trust and Go." Trust God and go witness. My very first
evangelistic endeavor was just such a case. This is what happened.

My first witnessing adventure. . . This really happened.

My friend Chuck and I both had eager hearts. We had been studying cults, Mormonism and
Jehovah's Witnesses, for a while and were ready to go out and "teach the world." We prayed, got into
my car, drove for a while, prayed, drove, prayed, drove, and prayed. Well, after a half hour of this we
figured it was time to stop driving and start talking.
The next street we passed was named Omega Street. Jesus is called the Alpha and Omega (Rev.
1:8). Since we were rather unsure of ourselves, we grabbed at this spiritual straw and Omega Street
became our first witnessing attempt--it is amazing how God uses strange things and people to spread
His Word.
Chuck and I prayed again. We got out of the car and approached a home. The door was ajar. We
knocked. From inside we heard a woman say frantically into her phone, "Send the paramedics to . . . "
then she gave an address. Chuck and I exchanged stares.
Five minutes later the paramedics arrived. They entered a nearby house and took away an elderly
man on a stretcher. Now, if you've ever been around when the paramedics screech into a
neighborhood with sirens blaring, you know how quickly a crowd gathers. That is exactly what
happened and after the paramedics took the victim (of a heart attack) away, Chuck and I suddenly
found ourselves, Bibles in hand, staring into the eyes of an excited crowd. A kid on a bike noticed our
Bibles and said, "What are you guys doing here?" Chuck and I aren't geniuses, but it didn't take us
long to see the work of the Lord. We told him we were there to preach the Gospel. I was too afraid to
speak so Chuck spoke to him and others about Jesus, sin, and salvation. People listened. No one
dropped to their knees in sobbing repentance, but they heard the gospel. That was years ago. Since
then, I have witnessed thousands of times.
The point is simple. When you are willing to be used by God and step out in faith, trust God and go,
He can do amazing things.

Are you available?

Are you available to God? Are you willing to pray, take a risk, trust God and go? If you are, if you
have the desire and a willingness to learn, then God will use you. He desires a willing, teachable, and
available person more than the most learned mind. If you make yourself available, He will make you
able. Get ready! He will use you!
Christian Issues
Introduction

Non-Christians don’t see the world the same as Christians do. They have a different perspective, a
different set of presuppositions.

Christians need to not adopt the presuppositions and perspectives of the world. Christians need to be
biblically submitted in mind, heart, and deed. This submission means that all areas of the Christian life
are to be in alignment with God’s word.

1. What is a world view? p. 220


2. Can you name some of the elements of a Christian worldview? p. 222
3. Why are schools so important from a Christian perspective? p. 223
4. Is homosexuality an acceptable life-style alternative for the Christians? p. 225
5. What is a family? p. 227
6. Is Christianity opposed to science? p. 229
7. Does science support the Bible? pp. 229-230
8. What are some of the failures of the Christian church? pp. 231-232
What is a Christian World View and why do Christians Need One?
A world view is a set of presuppositions and beliefs that someone uses to interpret and form
opinions about his humanity, purpose in life, duties in the world, responsibilities to family,
interpretation of truth, social issues, etc. A Christian should view all these things, and more, guided by
the light that is shed upon them by the Bible.
The Bible has much to say about the nature of man, the world, purpose, truth, morality, etc., and
so does the world. More often than not, the secular world view is in conflict with the biblical one. For
example: Where the world asserts that man evolved, the Bible says he was created and ultimately
responsible to God. Where the world says that morals are relative, the Bible says they are absolute.
Where the world says that there is no need of salvation and redemption, the Bible clearly states that
all people are in need of deliverance from their sin. The contrast is obvious and profound. Both cannot
be true.
The secular world exalts man to the apex of evolutionary development, the sovereign over all he
dominates, though only another animal. God is relegated to the belief systems of the uneducated and
superstitious. Such opposing views will clash.

The Condition of Society

The fruit of the secular world view can be seen in around us. As we observe society, it is evident
that not all is well. Television has degenerated into a bordello of violence, soft-pornography, anti-
family sit-coms, commercials that appeal to immediate gratification, and senseless children's cartoons
that are full of violence, occultism, and disobedience to parents. It often portrays pastors as
psychotics, priests as pedophiles, and religious people as insecure, ignorant, and bigoted.
The News is extremely biased and when speaking in areas where religious and secular morals
collide, it uniformly presents information with loaded words. Instead of "pro-life" we hear "anti-
abortion rights." Instead of "conservative" it is "right wing fundamentalist." Other words are used such
as "Bible thumpers," "censorship," "intolerance," "bigoted," etc.
"According to the Center for Media and Public Affairs, the average TV watchers sees 14,000
references to sex and the average child "watches 8,000 murders and 100,000 acts of violence by the
end of elementary school."1
Illegitimacy is on the rise. In 1970 babies born out of wedlock were 10% of all births. In 1991, it
was 30%. Rape is increasing as is violent crime, venereal disease, drug usage, and prison populations.

In many American schools the "Impressions" series is promoting the New Age and the occult.
Some programs have students being taught that they alone are the ones who should decide if drug
use is good or bad. Many school textbooks teach anti-family values, promote homosexuality, teach
moral relativism, encourage sexual conduct, and, of course, instill evolution as a fact. In addition, they
condemn the notion of a Christian God even being mentioned. Consider the following:
"When 10-year old Raymond Raines bowed his head and silently said grace over lunch in a St.
Louis public school cafeteria, he was placed in detention for a week and told that he must eat in a
room by himself if he continued to pray.
"When 30 Texas high school students gathered to pray at the flagpole before school one morning,
the principal politely told them not only to leave, but to pray out of sight.
"In Illinois, a high school principal sent police to break up a similar prayer group. Two students
were arrested."2
Obviously, America (and the world) is in desperate need of the life changing gospel of Jesus.
The Progress of the Gospel

But lest you get discouraged, the gospel is progressing. There are more Christians in the world
now than ever before. In the 1700's less than 20% of the American population went to church where
now it is above 50%. More people have heard the gospel than ever before, and Bibles are produced en
masse and being sent to nations all over the world with unprecedented reception. The Gospel is
preached on Television and Radio. Millions are coming to Christ in third world countries and a new
Christian awakening is working its way through Russia and Africa with China becoming the new
Christian frontier.
Nevertheless, the Christian community has a great deal of work left to accomplish. To fully carry
out the mandate of winning the world for Christ, Christians must adopt a biblical world view in all
aspects of life and present to the world, biblical perspectives on every niche of our existence. This
includes everything: education, medicine, the arts, politics, science, contemporary issues, ethics, and
more.

To God be the glory


What are Some Elements of a Christian World View?
Following is a representative list of statements that can help you establish a Christian World View.
This list is not exhaustive. But it is a good cross section of biblically derived principles that should help
as you seek to understand a biblical world view. As you study them, notice how they would affect your
beliefs and actions about various topics: God, church, evangelism, creation, worship, family, sin,
salvation, homosexuality, abortion, etc.
Principles are like a fountain from which other beliefs and actions flow. The closer you are to
biblical beliefs, the better able you will be to carry out the commission of our Lord and Savior to make
disciples of all nations, to glorify Him, and to understand His creation.

1. There is a God (Isaiah 43:10,11; 44:6,8; 45:5).


2. The Christian Trinity is the only God (Gen. 1:26; Zech. 12:10; 2 Cor. 13:14; John 1:1).
3. God has revealed Himself in three ways: In creation, in the Bible, and in Jesus (Rom. 1:18-20; 1
Cor. 15:3; John 5:39; Acts 10:43; Heb. 1:1-3; John 14:9).
4. God created the universe and all that is in it with order and design -- the universe is not an
accident (Gen. 1; Isaiah 44:24; 45:18; Jer. 27:5; Neh. 9:6).
5. All life on earth was created by God with a design and a purpose -- life did not evolve (Gen.
1:11,12,21,24,25; 1 Cor. 15:38,39).
6. The unseen supernatural world is just as real as the physical world (Eph. 6:12; Job 1:6; Mark
5:2; Matt. 12:22)
7. God made man in His own image distinct from the animals -- man did not evolve (Gen. 1:26-27;
2:7; 1 Cor. 11:7).
8. Man, from conception, is human and possesses dignity due to being made in God's image (Job
31:15; Ps. 22:10; 139:13; Hosea 12:3; Luke 1:41-44).
9. The first humans were Adam and Eve (Gen. 2; Rom. 5:14; 1 Cor. 15:22,45; 1 Tim. 2:13).
10. Adam and Eve were the first family (male and female) according to the purpose of God for
procreation and glorifying Him -- homosexuality, therefore, is unnatural (Gen. 1:28; 2:21-25).
11. Man is morally responsible and answerable to God (Ex. 15:26; 1 Kings 11:38; Rom. 2:16; Ps.
50:6; 82:8; James 1:21).
12. God gave dominion of the earth to Adam and Eve and, thus, to their descendants (Gen. 1:28;
Titus 1:7)
13. Man is steward of God's creation and is to subdue the world in a manner consistent with biblical
revelation (Gen. 1-2; 2 Tim. 3:16-17).
14. Sin entered the world through Adam and Eve (Gen. 3:1-6; Rom. 5:12-14)
15. All people have sinned and are in need of salvation (Rom. 3:23).
16. Only God can save. Man cannot save himself (Matt. 19:25-26).
17. Jesus is the only way to escape the judgment of God (Acts. 4:12; John 14:6).
18. The Christian Gospel is the key to the conversion of all people (1 Cor. 15:1-4; Mark 8:35; 13:10;
Rom. 1:16)
19. The Bible is the inspired revelation from God and correct and authoritative in all it addresses (2
Tim. 3:16-17; Acts 17:11; Amos 3:7).
20. The Bible reflects the true and moral character of God; therefore, truth and morals are knowable
and absolute, not relative (Exodus 20:1-17).
21. Government is ordained by God and is God's provision for order and safety in society (Rom. 13:1-
7; John 19:11).
22. Christians are to follow the laws of the land except where they contradict the Bible (Acts 5:29;
4:19).
23. Christians are to evangelize the world (Matt. 28:18-19; Ps. 22:17).
24. All areas of life are subject to God and should have Christian principles guiding them: personal,
public, and political matters, as well as medicine, science, art, literature, etc.. (Gen. 1:28; Matt.
28:19-20).
25. All areas of life are Christian mission fields: political, medicine, science, art, literature, education,
technology, family, economics, etc. (Gen. 1:28; Matt. 28:19-20).
Christians and Education
Harvard, Princeton, and other renowned colleges in America were originally founded by Christians
who wanted to educate people in biblical principles. Princeton was founded as a seminary, for example.
But history teaches us that educational institutions tend to drift away from what they were originally
designed.
Many colleges in America have abandoned classical biblical education for pragmatism,
utilitarianism, and a curriculum designed to indoctrinate its people with secular humanism. We have a
vast educational void in America that is slowly growing. Home Schooling alternatives and private
schools are increasing. The need is obvious. More and more, Christians do not want their children
indoctrinated by unbelievers who actively teach beliefs contrary to biblical theology.

The Schools are the Battle Ground

What cannot be conditioned into the minds of the mature must be propagandized to the young.
Therefore, today's schools are battlegrounds for culture and political correctness. Reading, writing, and
arithmetic have given way to humanism, anti-Christian indoctrination, and "values clarification." The
Conservative Students for America conducted a survey of 13 colleges in the Southeast and found that
of those surveyed, "56 percent said that right and wrong is a matter of personal opinion." Is this the
kind of graduates the schools are producing? Would you want a lawyer to defend you if he thought
morality was relative? What about politicians and those in power over us? Those who make our laws
don't seem to give a second thought at exempting themselves from many of the laws and taxes they
pass upon us. It is a scary thought. And things are getting worse.
Sure, the three "R's" are still taught in schools, but poorly. The educated in America are
increasingly less knowledgeable and trustworthy. Since the 1960's, Americans have become less
capable and are falling behind the rest of the world in educational ratings. What is being promoted as
the solution? Money! In 1996 the total Federal funding of public education was over 17 billion dollars.
State funding totaled over 113 billion dollars. It hasn't helped. In fact, the educational system seems
to be getting worse.
In a national survey conducted by the International Communications Research group, in
Pennsylvania, 58% of the parents polled strongly supported school choice as an option for parents.
When asked if they thought that children were receiving the education they needed, 16% said yes and
79% said no. Clearly, people want a change.

What is the Christian's Responsibility Regarding Education?

As Christians our responsibilities are to make sure that we and our children are educated in the
godliest manner possible. We should not let unbelievers teach our children ungodly philosophies. God
has entrusted us with children and He will want an accounting of what we did with them.
As people of God who are under His Lordship, we have been given the command to multiply and to
subdue the earth (Gen. 1:28). To do this, we need to learn about God's creation and seek to discover
what God has hidden in His creation. By doing this, we learn more about God's majesty and grow in
our ability to praise Him and glorify Him.
Electricity, radio waves, light, sound, physics, biology, medicine, etc., are all treasures God
created and hid in creation. He gave us the ability to discover them, to become educated about them.
All areas of knowledge, science, math, art, philosophy, medicine, etc, are merely categories of
discovery made by people. Therefore, all education for the Christian is to be God-centered because it is
disclosure of God's creative glory. The queen of the educational sciences should be biblical theology.
I recommend that Christian parents seriously look into home schooling as a preferable option over
letting unbelievers teach in a manner contrary to biblical revelation. Nevertheless, some will say that
it is difficult. I know. My wife home schools our children and we are financially strapped because of
it. It is work and it is sacrifice. But, our children are given to us by God to care for, raise for Him, and
nourish in the ways of the Lord. We parents will answer to the Lord for how we raised our children.
Connect with a local Christian home schooling organization. The support there is great as well as
encouraging. There are many viable options to public school. Do some research.
Another excellent alternative to public school is the Christian Day school. These are Christian
schools that are dedicated to teaching biblical perspectives. The faculties are Christian, God-fearing
people who know the great value of raising children for God. There are many great ones all over this
nation.

What does the Bible Say?

Please consider the following verses (NASB):

2 Tim. 3:16-17, "All Scripture is inspired by God and profitable for teaching, for reproof, for
correction, for training in righteousness; that the man of God may be adequate, equipped for every
good work."
Rom. 1:20, "For since the creation of the world His invisible attributes, His eternal power and
divine nature, have been clearly seen, being understood through what has been made. . ."
Psalm 78:1, "Listen, O my people, to my instruction; Incline your ears to the words of my mouth."

Eph. 6:4, "Fathers, do not provoke your children to anger; but bring them up in the discipline and
instruction of the Lord."
2 Chr. 17:9, "And they taught in Judah, having the book of the law of the Lord with them; and
they went throughout all the cities of Judah and taught among the people."
John 1:17, "For the Law was given through Moses; grace and truth were realized through Jesus
Christ."

Christians should pray for the educational system in America and get involved in improving it.
They should be very active in governing the education that their children are receiving, especially in
the home.

Education of the young, and ourselves, is a responsibility given to us by God.

*For further information see the Center for Education and Reform at http://edreform.com.
Also, check out the Classical Christian Schooling Network http://www.ccsnet.org
Christianity and Homosexuality
The homosexuals and lesbians have gained considerable political and social momentum in America.
They have "come out" as the term goes, left their closets, and are knocking on the doors of your
homes. Through the TV, Radio, Newspapers, and Magazines, they are preaching their doctrine of
tolerance, equality, justice, and love. They do not want to be perceived as abnormal or dangerous.
They want acceptance and they want you to welcome them with open, loving arms, approving of what
they do.
In the California State senate, several bills have been recently introduced by the pro homosexual
politicians to ensure that the practice of homosexuality is a right protected by California law. Included
in these bills are statements affecting employers, renters, and schools. Even churches would be
required to hire a quota of homosexuals with "sensitivity" training courses to be "strongly urged" in
various work places. There is even legislation that would make the state pick up the tab for the
defense of homosexuality in lawsuits, while requiring the non homosexual side to pay out of his/her
pocket.
The Christian church has not stood idle. When it has spoken out against this political immorality,
the cry of "separation of church and state" is shouted at the "religious bigots." But when the
homosexual community uses political power to control the church, no such cry of bigotry is heard.
After all, it isn't politically correct to side with Christians.

What does the Bible say?

The Bible, as God's word, reveals God's moral character and it shapes the moral character of the
Christian. There have been those who have used the Bible to support homosexuality, taken verses out
of context and reading into them scenarios that are not there. Quite simply, the Bible condemns
homosexuality as a sin. Let's look at what it says.

Lev. 18:22, "You shall not lie with a male as one lies with a female; it is an
abomination"

Lev. 20:13, "If there is a man who lies with a male as those who lie with a woman,
both of them have committed a detestable act; they shall surely be put to death. Their
bloodguiltness is upon them"

1 Cor. 6:9-10, "Or do you not know that the unrighteous shall not inherit the kingdom
of God? Do not be deceived; neither fornicators, nor idolaters, nor adulterers, nor
effeminate, nor homosexuals, 10nor thieves, nor the covetous, nor drunkards, nor
revilers, nor swindlers, shall inherit the kingdom of God."

Rom. 1:26-28, "For this reason God gave them over to degrading passions; for their
women exchanged the natural function for that which is unnatural, 27and in the same
way also the men abandoned the natural function of the woman and burned in their
desire toward one another, men with men committing indecent acts and receiving in
their own persons the due penalty of their error. 28And just as they did not see fit to
acknowledge God any longer, God gave them over to a depraved mind, to do those
things which are not proper."

With such clear statements against homosexuality, it is difficult to see how different groups can say
the Bible supports homosexuality. It doesn't. But when a group wants acceptance and the Bible is the
Christians' handbook on morality, the homosexual agenda must try to make the Bible agree with its
agenda. But it doesn't work.
Unlike other sins, this sexual sin has a severe judgment administered by God Himself. This
judgment is simple: They are given over to their passions. That means that their hearts are allowed to
be hardened by their sins. As a result, they can no longer see the error of what they are doing.
Without an awareness of their sinfulness, there will be no repentance. Without repentance, there will
be no forgiveness. Without forgiveness, there is no salvation.

Should homosexuals be allowed to marry one another?

In this politically correct climate that relinquishes morality to the relativistic whims of society,
stating that homosexuals should not marry is becoming unpopular. Should a woman be allowed to
marry another woman? Should a man be allowed to marry another man? Should they be given legal
protection and special rights to practice their homosexuality? No. No. No.
The Bible, of course, condemns homosexuality. It takes no leap of logic to discern that homosexual
marriage is also condemned. But our society does not rely on the Bible for its moral truth. Instead, it
relies more on a humanistic and relativistic moral base upon which it builds its ethics.
Homosexuality is not natural. The male and female bodies are obviously designed to couple. The
natural design is apparent. It is not natural to couple male with male and female with female. In fact,
if such couplings occurred in the animal world as a predominant practice, species would quickly
become extinct. Nevertheless, some argue that homosexuality is natural since it occurs in the animal
world. But this is problematic. In nature we see animals eating their prey alive. We see savagery,
cruelty, and extreme brutality. Yet, we do not condone such behavior in our own society. Proponents
of the natural order as a basis for homosexuality should not pick-and-choose the situations that best
fit their agendas. They should be consistent and not compare us to animals. We are not animals. We
are made in God's image.
Political protection of a sexual practice is ludicrous. I do not believe it is proper to pass laws
stating that homosexuals have 'rights.' What about pedophilia or bestiality? These are sexual
practices. Should they also be protected by law? If homosexuality is protected by law, why not those
as well?
Of course, these brief paragraphs can in no way exhaust the issue of homosexuality's moral
equity. But, the family is the basis of our culture. It is the most basic unit. Destroy it and you
destroy society.

What should be the Christian's Response to the Homosexual?

Just because someone is a homosexual does not mean that we cannot love him (or her) or pray
for him (her). Homosexuality is a sin and like any other sin, it needs to be dealt with in the only way
possible. It needs to be laid at the cross, repented of, and never done again.
As a Christian, you should pray for the salvation of the homosexual the same you would any other
person in sin. You should treat them with the same dignity as a person made in the image of God, that
you would any other person. However, this does not mean that you are to approve of their sin. Don't
compromise your witness for a politically correct opinion that is shaped by guilt and fear.
The Christian Family
The family has come under heavy attack in today's society. On television, father's are often
depicted as buffoons. Mothers are typically depicted as career minded and in control. Parents in
general are characterized as dim wits who aren't "with it."
In too many families, in order to make ends meet, the mother must work and entrust the care of
her children to strangers. Children are independent, arrogant, and disrespectful. Everywhere you look
society is offering quick and easy fixes, sexual promiscuity, "safe sex," and promises of freedom
without penalty.
Divorce destroys 50% of all marriages.
Gangs are growing.
Sexual molestations by parents of their own children is increasing as are acts of violence.
Amidst all the depressing facts there is a ray of hope: the Bible. God has given us the instruction
book for families. He has defined the family, taught us the roles of each of the family members, and
has promised to bless those who adhere to His will. Praise God! We need it!

1. What is the Family?


A. The Family is God's covenant arrangement where two people, male and female, are joined to
one flesh, "For this cause a man shall leave his father and his mother, and shall cleave to his
wife; and they shall become one flesh,” (Gen. 2:24). It is a covenant in that it is an
agreement, a promise made between the couple getting married. This covenant is before
God and is binding until death (or adultery) breaks the union.
B. The family does not consist of a homosexual or lesbian relationship where the two people of
the same sex seek to be married. This is against scripture: "And God created man in His own
image, in the image of God He created him; male and female He created them. 28 And God
blessed them; and God said to them, Be fruitful and multiply, and fill the earth, and subdue
it; and rule over the fish of the sea and over the birds of the sky, and over every living thing
that moves on the earth,” (Gen. 1:27-28).
2. Where did the Family begin?
A. The Family was instituted by God in the Garden of Eden Genesis chapter 1:26-31 and Gen.
2:18-25.
i. Gen 1:26-31 is the declaration of Man's creation with the command to be fruitful and
multiply. Concluding the section is God's declaration that it was very good.
ii. Gen. 2:18-25 is the account of Adam looking for a helper and God then making Eve from
Adam's rib and instituting marriage.
3. What is the purpose of the Family?
A. To multiply and fill the earth -- Gen. 1:28, ". . .Be fruitful and multiply, and fill the earth,
and subdue it. . ."
B. Marriage
C. Procreation -- Gen. 1:28
i. Continuing the covenant -- through procreation, the covenant between Adam and God is
continued.
ii. Fulfilling God's command to multiply and fill the earth
iii. Fulfilling God's command to subdue the earth
D. Sexual union - 1 Cor. 7:3-5
E. Sexual Fidelity and Purity - Exodus 20:14
4. The Important role of the Family in Society
A. Building Block of Society.
B. Place of learning.
C. Society reflects the family.
5. Scriptures concerning the family
A. Concerning Husband and Wife
i. The husband is the head: 1 Cor. 11:3 - But I want you to understand that Christ is the
head of every man, and the man is the head of a woman, and God is the head of Christ.
ii. The husband is the leader: Gen. 18:19 -- For I have chosen him, in order that he may
command his children and his household after him to keep the way of the Lord by doing
righteousness and justice; in order that the Lord may bring upon Abraham what He has
spoken about him.
iii. Wife is subject to husband: Col. 3:18 -- Wives, be subject to your husbands, as is fitting
in the Lord. cf., Eph. 5:22-24
iv. Husband is to love his wife: Eph. 5:25, Husbands, love your wives, just as Christ also
loved the church and gave Himself up for her;
v. Sexual duty: 1 Cor. 7:3 Let the husband fulfill his duty to his wife, and likewise also the
wife to her husband.
vi. Sexual Purity: Exodus 20:14 - You shall not commit adultery.
B. Concerning Children
i. Honor: Exodus 20:12 - Honor your father and mother.
ii. Obedience: Eph. 6:1 Children, obey your parents in the Lord, for this is right.
iii. Child rearing: Eph. 6:4 - And, fathers, do not provoke your children to anger; but bring
them up in the discipline and instruction of the Lord.
C. Polygamy
i. Permitted in the O.T.: Gen. 4:19 - And Lamech took two wives: the name of the one
was Adah, and the name of the other, Zillah.
ii. Condemned in the N.T.: 1 Tim. 3:2,12; Titus 1:6.
D. Miscellaneous Scriptures
i. Prov. 12:4, An excellent wife is the crown of her husband, but she who shames him is as
rottenness in his bones
ii. Prov. 14:1, The wise woman builds her house, but the foolish tears it down with her own
hands.
iii. Prov. 19:13, A foolish son is destruction to his father, And the contentions of a wife are
a constant dripping.

The Bible has much to say about the Family. God takes it very seriously. As Christians, we should
too. . . especially the fathers.
Christianity and Science
Science is that branch of study which seeks to observe, discover, and understand the nature and
principles that govern our the universe, our world, and ourselves. The result of this process is a
systematic categorization of knowledge with the goal of predicting and manipulating events according
to discovered natural laws.
Science has shaped our lives dramatically. Because of science we now have great medical
knowledge. We can travel in jets, automobiles, and trains over great distances. We can harness rivers,
predict storms, and use the power of the atom. By picking up a phone we can talk to almost anyone in
the world. We can see anywhere on the planet via television and even gaze upon the surface of the
moon and Mars. Like a giant flood gate that has been opened, what is flowing through its doors is a
wonderful technology of helps, advancements, relaxation, amusements, security, answered questions,
and hope. No longer must we till the land with our hands, pray for life giving rain to water our crops,
be subject to the whims of nature, and be helpless during times of sickness. In fact, science has
become for many a new god.
Before the time when science was looked to for explanations of the unknown, mankind turned to
religion. History is full of stories, writings, and the influence of religious beliefs upon entire societies.
Temples dedicated to various gods are all over the world. Even though science has not replaced
religion in all areas, it offers an intellectually justifiable reason to deny God's sovereignty. Evolution is
a good example.
When a society is powerless to control its future and is vulnerable to the elements of nature, it
turns to that which is in control of those things: god(s). In all religious systems, prayers are offered in
the hopes of appeasing and convincing a god or gods to help in time of need. But now that we can
provide irrigation to replace rain, medicine to cure diseases, weapons to protect our homes, and
television to dull our minds and hearts, the need for a deity to pray to has given way to the need to
obtain money in order to gain the benefits of technology: comfort, leisure, and security. The world
offers a new kind of religion: science. The scientists are its priests and the general populace is the
congregation.

What Does this Mean to a Christian?

To the Christian, science is merely that branch of discovery that categorizes, discovers, and
utilizes the knowledge woven into the fabric of the universe by a Sovereign, All Powerful, and
Omniscient, Creator. Science is not the end of all things, but merely one of the means by which man
may glorify God. This is because God is the creator of all that is. He has hidden the treasures of his
ominous glory in the very universe in which we exist. The power in the atom, momentum, energy,
mass, time, etc. are all creations of God and, therefore, under his authority. The more the Christian
learns of these things, the more He can glorify God. Science must be subservient to Him, not the other
way around. Science is not God's replacement. Every Christian should know that.

Do the Bible and Science Disagree?

Where other cosmologies found in religions have the world on the back of turtles, or the earth
being the result of a fight between gods, biblical revelation is quite consistent with science. This is not
to say that the Bible is vindicated by science; rather, it is science that is vindicated by the Bible.
Consider the following: (note: all quotes are from the NIV)

The Spherical Shape of the Earth - "He sits enthroned above the circle of the earth, and its
people are like grasshoppers. He stretches out the heavens like a canopy, and spreads them out like a
tent to live in,” (Isaiah 40:22).
The Hebrew language did not have a word for "sphere." Circle is quite sufficient.

The Earth is suspended in nothing - "He spreads out the northern [skies] over empty space;
he suspends the earth over nothing," (Job 26:7).
The Stars are Innumerable - "He took him outside and said, "Look up at the heavens and count
the stars -- if indeed you can count them." Then he said to him, "So shall your offspring be,” (Genesis
15:5).

The Existence of Valleys in the Seas - "The valleys of the sea were exposed and the
foundations of the earth laid bare at the rebuke of the LORD, at the blast of breath from his nostrils,”
(2 Samuel 22:16).

The Existence of Springs and Fountains in the Seas - "In the six hundredth year of Noah's
life, on the seventeenth day of the second month -- on that day all the springs of the great deep burst
forth, and the floodgates of the heavens were opened," (Genesis 7:11). See also Gen. 8:2; Prov. 8:28.

The Existence of Water Paths (Ocean Currents) in the Seas - "O LORD, our Lord, how
majestic is your name in all the earth!...When I consider your heavens, the work of your fingers, the
moon and the stars, which you have set in place,...You made him [man] ruler over the works of your
hands; you put everything under his feet...the birds of the air, and the fish of the sea, all that swim
the paths of the seas,” (Psalm 8:1,3,6,8).

The Hydrologic Cycle - "He wraps up the waters in his clouds, yet the clouds do not burst under
their weight,” (Job 26:8).
- "He draws up the drops of water, which distill as rain to the streams; the clouds pour down their
moisture and abundant showers fall on mankind," (Job 36:27-28)
- "The wind blows to the south and turns to the north; round and round it goes, ever returning on
its course. All streams flow into the sea, yet the sea is never full. To the place the streams come from,
there they return again,” (Ecclesiastes 1:6-7).

The Concept of Entropy - "In the beginning you laid the foundations of the earth, and the
heavens are the work of your hands. They will perish, but you remain; they will all wear out like a
garment. Like clothing you will change them and they will be discarded," (Psalm 102:25-26).

The Nature of Health, Sanitation, and Sickness - The listing for this section is too large for
this page. But the scriptural references are Leviticus chapters 12 through 14.

There is no God but One

Christians need to be careful not to let science usurp the place of authority and honor that belongs
to God alone. If you are a Christian, you need to keep in mind that God alone is the Lord and that He
placed the universe here. We are here for Him, so we might glorify Him and enjoy Him forever (1 Cor.
1:9). There is no God, but one (Isaiah 44:6,8). But. . .
Do you look to science as your hope, your security, and your safety net? Do you go to God only
after science has failed you, only after the medicine doesn't work, or only after your comforts are
threatened? Is television an idol to which you sacrifice your time and energy? Are the pleasures
offered by technology, which is the child of science, the fruit you seek more than God? How much of
your dependence upon God has been replaced by your dependence upon things?
Science is the creation of God and God is the Sovereign of all.
Look inside your own heart and see which is on the throne.
The Failure of the Christian Church
Christianity is supposed to be the representative of Jesus who taught love, forgiveness, sacrifice,
unity, and humility. Though that may be true to a large extent, Christianity has demonstrated an
ability to overlook many of its professed virtues and allow denominational fragmentation to weaken it.
Unfortunately, because of doctrinal "refining" on the non-essentials, a desire to be comfortable, and
apathy, the American church has, in many respects, become castrated. It spends too much time
splitting doctrinal hairs, separating, and then hiding in churches designed to keep its members
comfortable and safe. On the outside, the world is going to hell while on the inside, we are playing the
"religion game."
I am not saying that doctrinal purity is unimportant. It most certainly is. Without proper
understanding of who God is, what He has done, and what we must do, we would all be surely
damned. Our salvation depends on who Jesus is and what He did. We need to know its truth.
Therefore, as Christians, we should separate ourselves from the false doctrines that make powerless
the saving truth of Christ's sacrifice. But the motive to divide should be reserved for our dealings with
heretics. We should separate ourselves from false teachers and false doctrines, not from each other.
In the non-essentials we need to remain united as much as possible.
I recognize that denominations, to a small degree, are necessary and will never go away. But
comfort and "doctrinal purity" have robbed the Church of much of its power. Where the early
Christians had to rely on God for their every need, today creature-comforts and drive-through
churches have made us complacent and sluggish to the call of God to make disciples of every nation.
We are comfortable in America where we have the best of everything and only need to put on credit
what our whims demand and thereby avoid the dependence upon God for our needs; this makes faith
in God less of a pressing need. We have become distracted and the church is showing signs of spiritual
apathy. We have our VCR’s, air-conditioning, remote controls, and fast food. We have churches with
central air, great sound systems, well educated preachers, plush pews, and fine-tuned choirs, pianos,
and organs. We are blessed with committees, plans, and money. In fact, we have so many churches
we are guaranteed we can find one to suit any whim or preference. And all too often, the messages
are pleasant and don’t make our hearts ache for the lost or our Lord.
Doctrinal purity is a plague when it unnecessarily divides that which has been made Holy by
Christ’s blood. It may already be that doctrinal idols have invaded our churches after all, we too often
sacrifice people on the altar of doctrinal purity. Then we politely and lovingly expel fellow believers
from our churches and bless them on the way out because they baptize by immersion or don't, or
speak in tongues or don't, or believe in pretrib or don't, etc. Hurt and confused, some wander the
spiritual landscape looking for a safe haven only to fall prey to false teachers or the seductive call of
the world. Yes, we need doctrinal purity and we may even need to die for it one day, but doctrinal
purity is not our God. Confessions and creeds are not our bread and wine. We should not sacrifice the
blessing of unity for the minutia of purity. But some will say, "These doctrines are important and our
church has the truth." Perhaps. But Jesus said the world would know we were His disciples by the
love we have for one another, not the purity of our doctrine.
And what does the world see in all this? Does it see a visible church full of sacrifice, full of love, or
full of people who consider others more important than themselves? No. It sees polished televangelists
with perfect hair and smiles pulling the wool over the eyes of countless thousands of gullible people as
they ask for money. It sees the hypocrisy of moral uprightness proclaimed proudly in word and
contradicted in deed. It sees a denominationally fragmented church that can’t even clean its own
house.
And what’s more, the church has all but stopped its public proclamation against sin. It has begun to
believe the lie that the church is weak and powerless to stop the momentum of social decay. It
flounders when faced with immorality and balks at standing strong against sin!

What are the consequences of this?

We see the effects in the rise of the cults like Mormonism and Jehovah’s Witnesses who have
millions of followers going door to door faithfully and consistently spreading their damning doctrines.
Where are the Christians who oppose them? Where is the church? Is it supporting the efforts to stop
this spread of lies? Is it uniting behind a common cause? No! It leaves the work to the weary and small
who have a burden and who spend their efforts in a constant and mostly frustrating battle for the
truth. The church pats them on the back and says, "God bless. Go in peace" but leaves the depleted
warriors to fend for themselves.
We see consequences in the educational system with the rise of humanistic philosophy. Purely
secular agendas are being taught on moral, political, and social levels in schools. Homosexuality,
relativism, values clarification, and "ethical cleansing" are wiping the minds of the youth clear of
Christian values. The children sit and listen while we go to church and talk about hymnals, the organ,
and the color of carpet. May God have mercy on us.
Society needs not concern itself with the musings of our people because its conscience cannot be
pricked when so many of the bickering failures of Christianity speak louder than our words. Society is
little affected by the gospel. The secularist does not need to be wary of the church that sits idly by
and pampers its members and does not encourage them to take risks for the gospel. The secular world
is free to mock the truth, chip away at our freedoms, and claim more and more converts for itself. It is
safe from Christianity. But is Christianity safe from it?

What Should We Do?

First of all, we need to confess our sins to our Lord and Savior and repent from them. We need to
recognize our sinfulness of apathy, pride, gossip, and any idols of 'doctrinal purity' that are so
divisive. We need to forsake them, and drop to our knees, pray, confess, forgive, and go on. We need
to recognize that we must be united to be strong. But we must do this without compromising the
gospel of truth (1 Cor. 15:1-4; Rom. 5:1).
Second, we need to recognize the Great Commission as something more than a recommendation
from Jesus. It is not an option. It is a command. Jesus said, "Go therefore and make disciples of all the
nations . . . " (Matt. 28:18). Are we being fishers of men or keepers of the aquarium? Are we being
obedient or comfortable?
Third, we need to work together as much as possible to bring the gospel of truth to the lost. This
will require sacrifice, prayer, humility, and risk. We cannot easily undo the great fragmentation of the
body of Christ, but we can cross the denominational boundaries by focusing on that which unites us in
the faith: Jesus is God in flesh (Trinity), salvation by grace through faith alone, the atonement, and
forgiveness of sins in Jesus' shed blood. We need to look at the essentials and let the gospel of God
change the hearts of people.
Fourth, we need to use whatever gifts the Lord has given us for the increase of His kingdom.
Whether it is praying for the lost and for the workers in Christ, or helping support financially, teaching
the body, doing works of administration, or whatever gift you have, use it for the glory of God. Give it
to Him and ask Him to bless you by letting you use your gifts – and then do it! And don't be afraid to
fail.

Conclusion

God is a God of forgiveness, love, and power. He has forgiven us of our sins and continues to do
that by His awesome Grace (1 John 1:9). He loves us deeply and wants to commune with us and enjoy
our presence through Jesus (1 Cor. 1:9). And, His gospel is powerful, able to save the lost from their
sins (Rom. 1:16) and change this world. Pray for the work of God in your life and in the lives of others.
Make a strong effort to support and spread the gospel. Intercede prayerfully to the Father on behalf of
the church that preaches, and the lost that need to hear. Humble yourselves before God and men.
Don’t remain comfortable. Take a risk. Trust God and go!
To The Christian Church
Introduction

1. What is the great commission? p. 234


2. What is the Christian church? p. 236
3. What is the Christian Church supposed to be and do? p. 238
4. Is disunity a sign of apostasy? p. 240
5. Are Christians immune to being prideful? p. 242
6. What are some of the essentials of the Christian faith? p. 244
7. What are some of the signs of apostasy in the Church? p. 245-246
8. What are some of the signs that the Church is becoming secular? p. 249
9. Should unbelievers lead Christians in worship? Why or why not? pp. 253-254
10. Have you examined yourself to see if you're in the faith? What do you think? p. 255
11. Can you describe what the basics of the Christian faith are? p. 257
12. Why is comfort a potential problem in the Christian Church? p. 262
Introduction: Why this topic?

My motivation for writing this section is twofold: First, there is a slow apostasy that is creeping in
to many so-called Christian denominations. Many groups that claim the name of Christ are advocating
anti-Christian principles. Second, it seems that majority of Christians are not adequately trained nor
sufficiently motivated to carry out or the Great Commission: "Go therefore and make disciples of all
the nations, baptizing them in the name of the Father and the Son and the Holy Spirit, 20teaching them
to observe all that I commanded you..." (Matt. 28:19-20). To carry out this commission, Christians
need to be disciples and disciple-makers. It means knowing basic Christian doctrine, knowing the
Bible, and being able to defend the Christian faith. Of course, I'm not saying that every Christian has
to be seminary trained, memorize the New Testament, and stand on street corners shouting about
Jesus. Not at all. I am talking about the basics of knowledge of God's word as well as the basics of
action in and out of the church that leads us to do what Jesus charged us to do: make disciples.
When I teach seminars on cults, Christianity, and apologetic, I always take polls of the people I am
teaching. I ask how many can quote me scriptures defending the deity of Christ. Usually about two
people in fifty respond. When asked to explain the difference between justification and sanctification,
rarely can anyone tell me what it is. When I ask how many have shared their faith and presented the
gospel with anyone in the past year, about 1 in five say they have. These are not baby Christians.
These are "seasoned" Christians who've been in churches for years.
Apostasy means to fall away from the truth. To the degree that Christians adopt the ideas of the
world above scripture they are committing apostasy. Jesus has given us a commission and to do this
means we have to be prepared and Bible-focused. Carrying out the Great Commission means that we
have to be praying, studying, tithing, learning, being trained, being active, supporting the church,
supporting missionaries, etc. Some churches do this. Others do not.
The Great Commission (Matt. 28:19-20) is the charge of Jesus to believers, to every believer, not
just the pastor and the missionary. Perhaps you may say that you are not called to be a pastor, a
missionary, or an evangelist. That's okay. But, are you praying for those who are pastors,
missionaries, and evangelists? Are you supporting them in your tithes? Are you using whatever gifts
that you have in support of the church so that the Great Commission can be carried out?
The Great Commission is a commission of love given to us by the God of love. It is what Jesus
asked us to do. People are going to hell. Jesus wants us to help as many as possible find salvation in
Jesus. He wants us to be His disciples and then make others into disciples as well. This is what He
wants. Is this happening in your life and church? If it is not, you need to make some changes.
In America, too many Christians are comfortable with their lives, their VCR's, their remote control
TV's, their air conditioned cars, retirement funds, and their polished preachers. Our comfort is
important to us. But, it can lure us into a casual relationship with God because all our earthly needs
are met. Such casualness destroys the urgency, the intimacy of dependence upon God that excites
and motivates the believer into action when God miraculously and continuously provides our needs. I
also believe that many pastors are failing to do what the Bible says to do: equip the saints (Eph.
4:12). I suspect far too many pastors are more concerned about not offending their own
congregations with the gospel than spreading its truth -- along with what it means to mature in Christ
by picking up their crosses and following Jesus. The pastor is not there to baby sit Christians. He is
not there to simply comfort them and to make them feel warm and cozy. Nor is he there to reflect the
current social trends and morays of the secular environment. He is there to equip the saints, to train
them up to be more like Jesus (Eph. 4:12), to help them mature in Christ so that they can become a
people of action, as well as love.
The gospel is not only about being born again, but is also about picking up your cross and following
Jesus (Luke 9:23), about prayer, about supporting Christians who teach, about bearing one another's
burdens, about defending the faith, about standing up for righteousness, and much more. For too
many Christians, picking up the cross and following Jesus is too much to ask. But it is, however, easy
to drop a check in the offering plate and think that they've done their part as a Christian.
Is this too harsh?

If you think I am being too harsh, let me say that I know that there are many Christians who take
their faith seriously, are learning and applying God's word, and doing what they can to expand God's
kingdom whether it be by praying, tithing, witnesses, teaching, church work, living godly lives, etc.
Likewise, I know that there are many pastors who labor to equip their congregations and who lovingly
work to shepherd them with all sincerity and obedience to Christ. For you all, I praise God for His
miraculous work in you.
Pastors have a huge job before them. They are to preach God's word, teach the congregation,
counsel, model godliness, and equip the saints. This is difficult to do, especially when secularism is
slowly making inroads into the hearts and minds of Christians. Regarding moral issues, Christians,
statistically, are in bad shape. According to Barna Research online, of those claiming to be born again,
only 23% believe abortion should be illegal; 34% believe homosexuality is alright; 36% believe that a
man and a woman living together is okay; 37% says profanity is acceptable; only 20% believe it is
wrong to get drunk, etc. This is truly sad and dangerous. Oh sure, you may say they are not 'real'
Christians. I hope you're right. But, the statistics are real and those who are truly born again should
be out there fighting against abortion, homosexuality, drunkenness, etc., as well as praying for and
seeking revival in Christian churches.
People are going to hell. The enemy is making coverts to false gospels in the cults, humanistic
principles in schools, and moral relativism in society. Christianity is not supposed to be keepers of the
aquarium. It is supposed to be fishers of men. It is supposed to confront the world in a wise and
loving fashion. This is what the Bible says to do. To do this, the Christians need a truly Christian
world view with the desire to spread that world view everywhere.

The Christian Church needs to wake Up!

Christianity is under an ever increasing attack. Here in America, laws are being passed to reduce
and remove our religious freedoms. Prayer has been removed from schools, the 10 Commandments
removed from courtrooms. Movies and TV routinely portray Christians as ignorant bigots. Universities
constantly attack the absolutes of Christianity and some even promote Eastern Mysticism, witchcraft,
relativism, and a homosexual agenda by having representatives of these lies come in and teach!
Secular society as a whole is imposing its moral agenda upon all people, the church included, and it is
working! Christians are starting to listen to the false teaching of a fallen world and recanting on
biblical doctrines of the Trinity, the deity of Christ, of Jesus being the only way, of moral absolutes,
and of there being a Day of Judgment with the unsaved going to hell. This is the sign of apostasy
within the church!
Again, let me add that not all Christians are apathetic and worldly. There are many churches with
godly pastors who are teaching all of God's word. There are many churches out there with members
who are learning God's word, who are making converts, and who are standing up for righteousness.
It is because of people like them that the gospel is spreading throughout the world. There are more
Christians alive now than ever before. But, there are also more Muslims now than ever before, more
Mormons, more Jehovah's Witnesses, more atheists, etc., than ever before. Let's not give up nor
become discouraged. Let's support one another in prayer. Let's study to show ourselves approved to
God. Let's tithe properly. Let's witness. Let's take risks for Jesus. Let's do what He asks of us.

"All authority has been given to Me in heaven and on earth. 19Go therefore and make
disciples of all the nations, baptizing them in the name of the Father and the Son and
the Holy Spirit, 20teaching them to observe all that I commanded you; and lo, I am with
you always, even to the end of the age," (Matt. 28:18-20, NASB).
What is the Christian church?
"that He might present to Himself the church in all her glory, having no spot or wrinkle
or any such thing; but that she should be holy and blameless," (Eph. 5:27).

The Church is the "body of believers." It is comprised of those who have been saved and
redeemed by the True and Living God, based upon the sacrifice of the Lord Jesus upon the cross.
Inclusion in the Body of Christ is not by membership in a denomination, nor by baptism, nor or by
dedication. It is not received by ritual, or by ceremony, or by natural birth. It is received by faith
(Rom. 5:1; Eph. 2:8). The invisible church is the church made up of true believers. The visible church
consists of those who say they are Christian but may or may not be truly saved. Being a member of a
church on earth, guarantees nothing. Being a member of the Body of Christ, guarantees salvation.
The Christian Church does not include the Mormons, the Jehovah's Witnesses, the New Agers, the
Muslims, the Buddhists, the Atheists, etc., These groups deny the true God and/or serve false gods.
The true Christian Church is comprised only of those who have been redeemed by Christ who died on
the cross and rose again. They are justified26 by faith in Christ (Rom. 5:1). They are not saved by
false teachers, false gods, false gospels, by their works, or by their works combined with the grace of
God. They are saved by grace through faith (Eph. 2:8) and that through Jesus alone: "And there is
salvation in no one else; for there is no other name under heaven that has been given among men, by
which we must be saved," (Acts 4:112, NASB).1
The word "church" comes from the Greek ekklesia which means "assembly" or "gathering." But,
the church is more than a meeting place. It is much more than a gathering of believers who profess
the true and living God and attend weekly worship meetings. The church is the bride of Christ. It is a
living temple of the True God. It is not the building, the meeting place, an organization, or a
denomination. The church is the totality of all true believers irregardless of denominational affiliation.
The entire body of believers is the church and as such, it is the dwelling place of the Holy and Infinite
God.
Because the Church is the temple of God, it is holy because where God dwells is holiness. This
Holiness is not derived from merit of deed or ceremonies of church members (Rom. 3:28). This
Holiness is the Holiness of Jesus, God in flesh (John 1:1,14; Col. 2:9), that is given to each and every
Christian in the Body of Christ, by faith (Rom. 4:5; 5:1). Though the members of the Body of Christ
still fail and still sin in their struggle to more like Christ ( 1 John 1:7-9), their sins have all been paid for
by the risen Lord Jesus who bore their sins in His body on the cross (1 Peter 2:24). They do not need
to maintain their salvation by their deeds (Gal. 3:1-3). They have rest in Christ (Matt. 11:28). They
have been made clean by the blood of Jesus (1 John 1:7) and possess eternal life (John 10:28).
The church is the glorious mystery of God made real and revealed in scripture. It is the dwelling
place of God. Are you in the church? Do you know what the church is supposed to be?

The church

The church was purchased by God: "Be on guard for yourselves and for all the flock, among which
the Holy Spirit has made you overseers, to shepherd the church of God which He purchased with His
own blood," (Acts 20:28).
Jesus is the head of the Church: "And He put all things in subjection under His feet, and gave Him
as head over all things to the church, 23which is His body, the fulness of Him who fills all in all," (Eph.
1:22-23).

The church is the dwelling place of God: "in whom you also are being built together into a dwelling
of God in the Spirit," (Eph. 2:22).

Such a miracle as the church can result in a changed world. If we Christians can more fully
understand what the church is and the power it has because of what it is and who it is indwelt by, then

26
To be justified means to be saved. Justification is the legal declaration by God upon a sinner where God
declares the sinner righteous based on the work of Christ.
they can be more confident in seeking to stand up for the truth. As those who are indwelt by God
Himself, you can move into and through the darkness of unbelief and conquer it by faith. The
knowledge of what the church really is should result in countless believers living for Jesus in every way
possible, being ready to lay down their lives for Him in both death and in daily living. It should not
result in the mockery brought upon itself by its own failures and hypocrisy.
The church is the temple of the Holy and Infinite God. You, if you are in Christ, have God living in
you. You are in the church as much as you are the church.
What is the Christian church supposed to be?
"I write so that you may know how one ought to conduct himself in the household of
God, which is the church of the living God, the pillar and support of the truth," (1 Tim.
3:15).

The Christian church should be a reflection of Jesus' love, words, and deeds. Its goal should be to
glorify God, make Jesus known, develop godly people, and make disciples of every nation. The church
should be a group of believers, on the whole and in part, who live and teach the saving words of
Christ. To the extent that Jesus lived truth, so also should the people in the church.
The church is not supposed to be the building or the structure of a ruling body who act in the place
of God and dictate to the congregants what is and is not truth. The church is not headquartered at
Salt Lake City, Utah, or Brooklyn, New York, or Rome, or Jerusalem, or any other city. The church is
not headquartered in a central location -- except to say that the Head of the Church is Jesus who is in
heaven. The church is not a convention, a building, a series of meetings, or commitments. The
church is the living Body of Christ comprised of the redeemed in Christ. The external trappings of
celebrations, rituals, buildings, robes, hymn books, organs, pianos, chairs, pews, windows, etc. are
merely those things that give a tangible effect to the invisible reality of redemption. These externals
should not be considered the substance of the church. The substance of the church is the redeemed in
Christ.
The visible church27 is supposed to be a collection of people who are saved by the blood of Jesus
and indwelt by God Himself (John 14:23). The Christian church is comprised of believers, equipped by
God with teachers, pastors, etc. (1 Cor. 12:28), who grow in their relationship with Jesus through
prayer and the study of God's word and who actively seek to expand God's kingdom through preaching
and living the Gospel.
The Christian church is supposed to be a light to the world. It is supposed to fight against
wickedness, oppression, poverty, sin, rebellion, adultery, homosexuality, fornication, abortion, etc. It
is not supposed to sit idly by and watch the unbelievers go to hell. The church is supposed to be
active, living what is right before God and standing against sin. Yet, this resistance against
ungodliness is to be done with gentleness, love, patience, kindness, and wisdom. It is this last item,
wisdom, which is so often lacking in the church today.

Conduct yourselves with wisdom

The church should be wise. "Conduct yourselves with wisdom toward outsiders, making the most
of the opportunity," (Col. 4:5). Christians should not parade themselves on television as incessant
beggars of money, or gaudy over dressed "guides of the blind" who sit in gold chairs and weep at the
drop of a hat. They should not display a drastic misuse of charismatic gifts by praying in tongues in
public, becoming lost in uncontrollable laughter, or barking like dogs. Christians should not speak of
Jesus in one breath and then laugh at a dirty joke in the next. All Christians are in the public eye one
way or another and need to be above reproach, not hungry for money, not desirous of possessions,
and not addicted to pornography, prostitutes, alcohol, drugs, off-color humor, or taking the Lord's
name in vain. Such sin brings mockery to the name of Christ. The Christian church should be an
example of propriety, decency, self-sacrifice, servitude, and love, not a money hungry, hypocritical,
whining entity that is out of touch with reality as is so often portrayed on television.
The Christian church has great liberty and I am not condemning the right to let financial needs be
known nor am I discouraging the expression of the Holy Spirit within the church. There are thousands
of godly Christians who are very loving and giving and who honestly desire to honor and serve God.
But, the church as a whole needs to act and move in wisdom because the world is watching -- closely.
It is the begging, the charismatic chaos, and the chicanery that permeates the high-profile, public
church today that needs to be eliminated. We Christians need to clean our own house first before we
starting pointing fingers at sinners! Those Christians who are in high public positions should always be

27
The visible church is the social structure and buildings comprised of those who profess Christ. Not all in the
visible church are saved. By contrast, the invisible church is comprised of those who are truly redeemed.
mindful of what they do and should ask themselves "How will this appear to the unbeliever? Will it
stumble them?" Such was the concern of Paul in 1 Cor. 14. He did not want the church in its freedom
to stumble the unbeliever.
Individual Christians should also be mindful of what they do and say in the workforce as well.
Remember that for many unbelievers, the only time they encounter Jesus is when they seem Him
represented in Christians. Therefore, the Christian should live his life lovingly, from a pure heart, a
good conscience, and a sincere faith (1 Tim. 1:5).
The world hears us call ourselves followers of Christ but too often observes us following whatever
draws away our interests whether it is money, buildings, or experience. Is this what the church is
supposed to be and do? There is nothing wrong with having buildings or asking for money for
legitimate needs, but they must not be the reason for the church's existence and they should not be
the things looked to for security.
The Christian church is supposed to be a light in the world, a light of love, peace, wisdom, truth,
most importantly, it is supposed to bring Glory to God, equip the Christians for the work of the Great
Commission, and demonstrate godliness and holiness. That is what the church is supposed to be and
do.
The need for unity in the church
"Make my joy complete by being of the same mind, maintaining the same love, united in
spirit, intent on one purpose," (Phil. 2:2).

One of the signs of apostasy in the Christian Church is the bickering and disunity among Christians.
Jesus said that the world would know that we were His disciples by the love that we have for one
another (John 13:35). In Col. 3:14, it says that love is the perfect bond of unity. The New Testament
speaks about us being unified in Christ (Eph. 4:5). In response to Christians who following after
individuals rather than Jesus, Paul says that Christ is not divided ( 1 Cor. 1:12-13). Though Christ is
not divided, His body of believers is. Divisions in the Christian church can be a healthy and necessary
thing: "For there must also be factions among you, in order that those who are approved may have
become evident among you," (1 Cor. 11:19). But too much of a good thing isn't so good.
It is all right to have differences of opinion on the non-essential matters like worship styles, or pre-
trib rapture or post-trib rapture. These things whether you believe one or the other, do not affect
salvation. Yet far too many Christians use these non-essential differences as justification for division
and sometimes even anger. When this occurs, the love of God in our hearts is sacrificed to our pride.
Instead of saying to one another, "I am right and you are wrong," we should be saying something like,
"It is certainly possible that you are correct. Now, let's work together to glorify God and expand His
kingdom." Perhaps this is too simplistic, but at least it displays an attitude of humility that helps to
bring unity. It is the devil that wants us to fall into the self abuse of division and bickering.

Sometime apostasy means remaining united

There is a time for division in the body of Christ. When an individual or a church group is denying
clear scripture and remains unrepentant after being admonished, then it is time to break fellowship
with that group. Such is the case with the Metropolitan Community Church denomination which
openly advocates the support of homosexuality. Also, the Evangelical Lutheran church is in risk of
apostasy by entertaining the idea of accepting homosexual relationships into church as is also the case
with United Church of Christ: "The United Church of Christ set up a $500,000 scholarship fund for gay
and lesbian seminarians Friday and urged wider acceptance of homosexuals by other denominations."
(United Church Makes Gay Scholarship, CLEVELAND, Jun 16, 2000, AP Online via COMTEX). Or "The
supreme court of the United Methodist Church was asked Thursday to reconsider the denomination's
ban on gay clergy. (Church court of United Methodists asked to decide on gay clergy ban, NASHVILLE,
Tennessee, Oct 25, 2001, AP WorldStream via COMTEX). Such movements by churches to move to
accept homosexuality as acceptable in Christianity are clearly not believing God's word. If they don't
believe God's word in such a fundamental issue, how can they be trusted to understand God's word in
other issues?
Church groups like this are in open rebellion against God and His word and it would not only be
prudent, but it would be biblical to not fellowship with these groups.

What is it that unites us?

Primarily, it is the saving work of Christ that unites us. Secondarily, it is the essential doctrines
that define orthodoxy. We have, as a common heritage, the blood of Christ that has been shed for the
forgiveness of our sins. True Christians serve the true and living God and we know Jesus in a personal
and intimate way (1 Cor. 1:9). We have been redeemed by God himself. Furthermore, we have the
body of Scriptures which tell us the essentials of the faith and deviating from these essentials means
to be outside the camp of Christ. It is the essential doctrines that we must know and unite in.
Why then, for all practical purposes, do we elevate the non-essential is to the place of essentials? I
cannot see how such a huge fragmentation in the Christian Church in denominations and sects glorifies
God.
The Christian church needs to repent. We need to look at ourselves. We need to look at our
churches. We need to look at one another and decide that we will stand on the essential doctrines of
the faith and that we will stand united against the enemy. Those of us who are united by the blood of
Christ are not enemies with one another whether we be Presbyterian or Baptist or Lutheran. It may be
difficult for many of us to look lovingly into the eyes of those of a different denomination without
thinking in our hearts that they are wrong about this doctrine or that doctrine. We need to be
reminded that there is neither a Presbyterian nor a Baptist nor a Lutheran on the throne of God. All of
us I am sure, will have our theologies corrected when we stand before the throne of God. Therefore,
we need to seek to work together to further the Kingdom of God.
Ask yourself what is most important in life. Is it your relationship with God? If it is, and it should
be, should you not also be seeking the same thing that God wants? Should you not also be seeking to
love one another as Christ commanded us? Love is the perfect bond of unity.
Apostasy can begin in our own hearts when we put distance between ourselves and our brothers
and sisters in Christ because of a difference of opinion on a non-essential doctrine. Apostasy means
that we fall away from the truth. This falling away can be complete or it can be slight. Let's not to
commit apostasy in our hearts by abandoning the call of unity and love within the body of Christ.
Remember, it is the devil that wants us to fight each other so that he can be freed up to deceive the
world. If we are fighting each other then we are falling pray to His tactics.
I know that it is easy to speak these words and it is very difficult to apply them. That is true
because true love is difficult to live. But what if the world began to see the Christian Church uniting in
spite of its differences? What if the world started to see how the Christian Church started to love not
only their own church members but other church members? What do you think the world would say if
the churches' bickering stopped? What do you think the unbelievers would say if they saw us living
more and more the lovingly attitude and a sacrificial life of Christ across denominational barriers? It
would be a tremendous witness for Christ. It would be a tremendous assault on the enemy and I know
that God would use it mightily to bring others to Himself by his grace.
Apostasy begins with the individual. Apostasy begins in the heart and the mind. Whichever comes
first is not important. Whether we think something wrong and then feel it or feel something wrong and
then think it. The heart and the mind are so closely related that we must guard them both. We must
focus on the truth of God's word and let our minds be shaped by it. We must seek to have our shaped
by the love of God as we move not only to learn about Him, but also to carry out His desires.

What should we do to bring unity wherever possible?

We need to look at our own hearts and our own minds and compare them to Jesus and the
Scripture. Where ever the two are not an agreement, it is we who need to change. We need to pray
that the Lord would provide opportunities to work with other Christians across denominational lines.
We need to recognize that we have differences of opinions and worship styles and that that is okay.
But we need to lift each other up and be united in Christ.
There is pride in the Christian Church

There is in the Christian Church a great gifting in the area of pride. It is everywhere, in every
Church, in every denomination. It manifests itself in division and polite "Christian" mockery, gossip,
and condemnation of other Christians who do not believe as they do. Don't think you are so innocent.
The pride that I speak of is that which is anchored in our self-assured opinions about non-essential
doctrines. I am not speaking of the central doctrines upon which the Christian faith depends and by
which we are able to recognize and refute error. Such central doctrines as the Trinity, the deity of
Christ and the Holy Spirit, Jesus' physical resurrection, salvation by grace, etc., are the basics of the
Christian faith that unites us all. It is not these that is the problem. Rather, it is the non-essentials of
the faith where we draw the dividing line in our hearts and look down upon other Christians who are
not as wise as we and then we say, "Lord, I thank you that I am not like that Christian over there."
One Church teaches a pre-tribulation rapture and subtly implies that its doctrine is the true
doctrine, rightly divided; other options are systematically looked down upon and indirectly division in
the body of Christ is increased. Another church teaches that amillennialism is correct and that anyone
not believing in it cannot rightly understand God's word. Another church condemns the Charismatic
gifts in such a way that you are left believing that anyone who is charismatic is without maturity in the
Lord. It is not an opinion that is offered, but the "truth" that is offered at the expense of humility and
love and unity in the body of Christ.
Do these teachers who "knowingly" teach that they have the truth say that their positions are
opinions and that they are debatable and that the believer should study for himself and make up his
own mind -- even if it is contrary to the teacher's position? Do these teachers leave the listeners
believing that the grace of God is also working in others with whom they disagree in the non
essentials?
Is not God the God of all Christians? Didn't Jesus shed His blood for all Christians, even the
charismatics, even the Calvinists, even the Baptists, even for those steeped in quiet tradition and
liturgy, and even for those who weep during worship? Yes, He did.
Where is the humility of teaching about the non-essentials and saying that it is possible that
another position on them may be true? When do teachers say that other gifted teachers see things
differently...and that that is okay? Unfortunately, those who focus on the non-essentials to the point
of division in the body of Christ counter Christ's own words which speak of unity and love. Am I right
or am I wrong? Are we prideful in our hearts or not?
Pride, like humility, hides itself in its host so that it cannot be seen except my others. Pride is in the
Christian Church. We see it in the denominational divisions that are rampantly scorching the land.
Instead of uniting in humility, instead of admitting that our own sinfulness is what has resulted in our
inability to come to a unified belief in the non-essentials is simply proof that we all must be humble
before God and live according to Romans 14:1-7,

Now accept the one who is weak in faith, but not for the purpose of passing judgment
on his opinions. 2One man has faith that he may eat all things, but he who is weak eats
vegetables only. 3Let not him who eats regard with contempt him who does not eat, and
let not him who does not eat judge him who eats, for God has accepted him. 4Who are
you to judge the servant of another? To his own master he stands or falls; and stand he
will, for the Lord is able to make him stand. 5One man regards one day above another,
another regards every day alike. Let each man be fully convinced in his own mind. 6He
who observes the day, observes it for the Lord, and he who eats, does so for the Lord,
for he gives thanks to God; and he who eats not, for the Lord he does not eat, and gives
thanks to God. 7For not one of us lives for himself, and not one dies for himself; 8for if
we live, we live for the Lord, or if we die, we die for the Lord; therefore whether we live
or die, we are the Lord’s.

Christians who disagree should admit to each other that the reason we disagree is because of our
own shortsightedness, our own inability to rightly divide God's word. All of us must and should admit
that we can be wrong in these non-essentials. If we can do this, then it is necessarily true that the
other person may be correct. This is humility. But, I know, you don't believe the other person is
correct. Fine, neither does he think you are correct.
Look into your own heart. Are you so confidence about when the rapture will happen, or about
predestination or the lack thereof, or baptism for infants or not, or alter calls, or Saturday or Sunday
worship, or hymns verses praise music, or the charismatic gifts, that you will look down in your own
heart upon a brother or sister in Christ for whom the Lord has shed His precious blood? Or, do you
love them instead?
Ask God to examine your heart and see if there be any prideful or hurtful way in it.

"Search me, O God, and know my heart; Try me and know my anxious thoughts; 24And
see if there be any hurtful way in me, And lead me in the everlasting way," (Psalm
139:22-23, NASB).
Apostasy in the Christian church
"Let no one in any way deceive you, for it [Jesus' return] will not come unless the
apostasy comes first, and the man of lawlessness is revealed, the son of destruction," (2
Thess. 2:3, NASB).

Apostasy means to fall away from the truth. Therefore, an apostate is someone who known and
then rejected the truth of God. Apostasy is a rebellion against God because it is a rebellion against
truth. In the Old Testament God warned the Jewish people about their idolatry and their lack of trust
in Him. In the New Testament the epistles warn us about not falling away from the truth. Apostasy is
a very real and dangerous threat.
The verse at the top of the page tells us that there will be an apostasy that is associated with the
appearance of the antichrist. Most Christians are looking for the arrival of the antichrist, but very few
are looking for "the apostasy" that must come first. the arrival of the antichrist cannot occur until
sufficient apostasy has happened in the world. The antichrist, who is the ultimate of liars, cannot
abide in a world where the truth of God's word is taught. This is why the Bible says that the apostasy
will come first and then the antichrist will be revealed.
Therefore, we must, as Christians, ask this question "Is there an apostasy occurring in the
Christian church today?" Some would say no and others yes. But, as we look for the arrival of the
antichrist, should we not also be looking for the arrival of apostasy? And where else should we first
look but in our own house for the Bible tells us that judgment will begin in the house of the Lord ( 1
Peter 4:17).
If there is indeed on apostasy occurring in the Christian Church, we would first need to know what
is true so that we could recognize what was false. It is only after truth is established that we would
then have a measuring rod by which apostasy can be detected. Therefore, I propose the following list
of biblical truths as a sample of essential Christian and non-essential doctrines by which we might
compare other teachings and phenomena.

1. Primary Essentials (Nature and work of Christ)


A. Jesus is both God and man (John 1:1,14; Col. 2:9).
B. Jesus rose from the dead physically (John 2:19-21).
C. Salvation is by grace through faith (Rom. 5:1; Eph. 2:8-9).
D. The gospel is the death, burial, and resurrection of Jesus (1 Cor. 15:1-4; Gal. 1:8-9).
2. Secondary Essentials (Nature of God)
A. There is only one God (Isaiah 43:10; 44:6,8)
B. God exists as a Trinity of persons: Father, Son, and Holy Spirit. (See Trinity)
C. Jesus was born of the Virgin Mary
3. Primary Non-Essentials (Church ordinances and practice)
A. Male eldership
B. Fidelity in marriage in heterosexual relationships
C. The condemnation of homosexuality
4. Secondary Non-Essentials - does not affect one's relationship with God.
A. Baptism for adults or infants
B. Predestination, election, and free will
C. Communion every week, monthly, or quarterly, etc.
D. Saturday or Sunday Worship
E. Pre, mid, post trib rapture.
F. Premill, Amill, and post millenialism.
G. Continuation or cessation of the charismatic gifts

Of course, the non-essentials are debatable (which leads to denominational fragmentation). But by
way of explanation, the Primary Essentials are those doctrines that the Bible states if they are denied,
damnation follows. I have written on this in essential doctrines. For brevity, the Bible states that if
you deny Jesus is God, you are dead in your sins (John 8:24,58 cf. Exodus 3:14); that if you deny
Jesus' physical resurrection, your faith is in vain (1 Cor. 15:14, cf. John 2:19-21); that if you add
works to salvation, you are not in Christ (Gal. 3:1-3; 5:1-4); and that if you preach a gospel contrary
to what the apostles preached, you are accursed (Gal. 1:8-9, cf., 1 Cor. 15:1-4). Therefore, to deny
any of these doctrines, according to scripture, is to be outside the camp of Christ, and invited eternal
damnation. This would obviously be apostasy.
The Secondary Essentials are essentials are further clarifications of orthodoxy, but there is no
explicitly scriptural statement regarding each (that I am aware of) which states that denying them
results in damnation the way the Primary Essentials do. The Secondary Essentials deal with the nature
of God, primarily. The fact that there is one God, who is a Trinity, is clearly essential to Christian
orthodoxy, but there is no scriptural statement stating that to believe in the Trinity is necessary for
salvation. However, that does not mean that denial of the Trinity is acceptable. A person can be
saved without knowing about the Trinity. But, since the Trinity is a biblical truth, and the believer is
indwelt by the Holy Spirit who bears witness of truth, a true Christian will not openly denounce the
Trinity once he has been taught it from scripture. So, it could be said that the Secondary Essentials
are essentials to the faith as well as the Primary Essentials are.
The Primary Non-Essentials are biblical teachings that if denied do not affect one's salvation. But,
because the Bible teaches then, denying them is a sign of apostasy. The Secondary Non-Essentials do
not affect ones position with God nor do they affirm or deny biblical teaching since they are very
debatable. Having differing beliefs in these is not a sign of apostasy, just differences of opinion.
Again, I am aware that the categorization of the non-essentials is debatable, but I must draw the line
somewhere. Sadly, it is in Secondary non-essential doctrines that most denominational fragmentation
occurs. This is a sad display that most division occurs over that which is least important.
Furthermore, I believe that it is in the area of the Non-Essentials that apostasy can first be detected.

2 Thessalonians 2

As quoted above, there is a prophecy in 2 Thessalonians about a coming apostasy that is


associated with the disclosure of the anti-Christ.

"Let no one in any way deceive you, for it [Jesus' return] will not come unless the
apostasy comes first, and the man of lawlessness is revealed, the son of destruction," (2
Thess. 2:3, NASB).

Have you been looking for the coming of the anti-Christ? Are you waiting for him to pop up on the
world scene? If you are, are you also looking for the related apostasy? Most Christians are looking for
the anti-Christ but are not looking for signs of apostasy.
Since apostasy is falling away from the truth, perhaps this means that the Christian church is what
will commit apostasy. After all, you must have the truth before you can fall away from it. Or could it
be that this prophecy is about the general Christian church which is filled, sadly, with many
unbelievers? Or, is it a prophecy about the general failure of humanity to embrace the truth of God as
they adopt false beliefs? I would like to add that I believe it is not the true Christians that will ever fall
away from the truth. Instead, I believe it is the false sheep in the visible church that will be and are
part of the apostasy.
The Bible is God's word and it tells us what is right and wrong. To the degree that anyone
disagrees with the truths of God's word, to that same degree they are falling away from it. What,
then, might be some of the signs of apostasy? I've compiled a representative list of issues. You may
agree with and some, or you may not. I provide them as food for thought.

1. Denial of basic Christian doctrines such as the Trinity, the deity of Christ, the deity of
the Holy Spirit, salvation by grace, and moral absolutes as found in the Bible.
A. God's word is true. Deviation from the basics of its truth is surely apostasy.
2. Countless denominational divisions that contradict John 13:35 and 1 Cor. 1:10.
A. Of course, there are bound to be divisions in the body of Christ and differences of opinions
are permitted (Rom. 14:1-12). But, the amount of divisions in the Church is ridiculous and
contrary to Col. 3:14.
3. Ordination of homosexuals
A. Homosexuality is clearly condemned in God's word (Lev. 18:22; 1 Cor. 6:9). To ordain
homosexuals into ministry is clearly contrary to biblical truth and clearly apostasy.

4. Women elders and pastors


A. Whether people like it or not in this politically correct environment, the Bible does not
support women elders or pastors (1 Tim. 2:12-14; 3:2; Titus 1:5-7). Men are called to be
leaders in the church. The fact that women elders and pastors exist is a sign that men are
not doing their God-given job.
5. Not preaching the gospel per 1 Cor. 15:1-4.
A. The gospel is the death, burial, and resurrection of Jesus for our sins. It is not a message of
convenience or embarrassment. Do not be ashamed of the gospel (Rom. 1:16).
6. Using the Lord's name in vain, something a surprising number of Christians do.
A. God's name and title are to be used only by Christians in a reverent and respectful manner,
never in casual exclamation. Just because the sinners do it, does not mean it is okay for the
Christians.
7. Not sending out or failing to support missionaries (or cutting back unnecessarily) in
violation of Matt. 28:18-20.
A. Carrying out the Great Commission is the command of Jesus. Any church that is able to
support missionary work and does not, is in direct violation of Christ's command in the Great
Commission.
8. Marketing and merchandising
A. Those in ministry should make a living from their labor. Churches should seek to spread the
gospel best they can and selling things to do it is acceptable. But, how many trinkets and
bobbles are offered in the name of Christ that do not honor God but are merely for the
purpose of financial gain? Is the duty of the church business or the gospel? Remember how
Jesus cleansed the temple?
9. Pastors who are more concerned with growing a church than preaching the truth.
A. Whoever and wherever they are, they need to repent. Pastors must stand on the truth of
God's word, even if it costs them financially and materially.
10. Pastors who don't pray and seek God's face
A. Of course, this should be rare. But, any pastor who does not seek God's face in humility is
seeking to do a job, not a ministry, under his own power.
11. Pastors who cave in to pressures from the church in contradiction to the word of God.
A. Any pastor who does this should repent now or step down from the pulpit. Pastors are to
stand upon and for God's word, no matter what the obstacles or the cost.
12. Pastors who fail to equip their congregations according to God's word.
A. Pastors are called to equip the Christian for the work of the ministry in all aspects of life
(Eph. 4:11): apologetics, evangelism, missionary work, prayer, service, love, etc. Far too
many congregations are not being equipped with even the basics of Christianity and are
instead being taught political correctness.
13. Christians gathering teachers to themselves to make them feel good
A. Is comfort or truth the primary objective for the Christians? Are we divine in nature or
sinners saved by grace? Do we deserve to be saved or are we saved by God's free choice?
Christians who want merely to be entertained and comforted from the pulpit are still
children. They should be challenged to grow and take risks.
14. Evolution
A. Denominations that either adopt evolutionary principles or refuse to take a stand on
evolution.

Apostasy is all around us in varying degrees. As Christians, we need to be very sure that we are
clinging to the truth of God's word and resisting the inclusion of liberalism, moral relativism, and the
oncoming secularism that is all around us. We need to stand on the word of God and never be
ashamed of the truth of the Gospel: "For I am not ashamed of the gospel, for it is the power of God
for salvation to everyone who believes, to the Jew first and also to the Greek," (Rom. 1:16).
Examples of Apostasy in the Christian church
"For the time will come when they will not endure sound doctrine; but wanting to have
their ears tickled, they will accumulate for themselves teachers in accordance to their
own desires; 4and will turn away their ears from the truth, and will turn aside to myths,"
(2 Tim. 4:3-4).

Following is a list of examples of church that claim to be Christian who are adopting unbiblical
ideas. Note: individual churches do not represent denominations.

1. Changing the Bible to suit gender-neutral wording.


A. This has crept into the church periodically in the last 30 years. Due to pressure from the
secular society, some Christian based Bible publishers are offering gender-neutral Bibles.
This is sad. Is God's word any less true because it is not gender-neutral? Should Christians
change God's word to suit the unbeliever? No and no.
2. Metropolitan Community Church which openly approves of homosexuality.
3. The Evangelical Lutheran Church (on homosexuality)
A. "The Evangelical Lutheran Church in America decided Monday to undertake its first major
study on whether to endorse the morality of homosexual relationships...The Rev. Ann
Tiemeyer of Woodside, N.Y., said she supports greater acceptance of gay church members
and clergy. "It is time to send a message that we are a welcoming church, in our clergy as
well as our pews," she said," (Associated Press Online, 08/13/2001, Lutherans to Study
Homosexuality).
B. "The church currently allows homosexual clergy if they practice celibacy." (New Bishop for
Evangelical Lutherans, ELCA, Associated Press Online, 10/06/2001.)
4. The Evangelical Lutheran Church (on evolution)
A. The ELCA doesn't have an official position on creation vs. evolution, but we subscribe to the
historical-critical method of biblical interpretation, so we believe God created the universe
and all that is therein, only not necessarily in six 24-hour days, and that he may actually
have used evolution in the process of creation. (http://www.elca.org/co/faq/evolution.html)
5. The Episcopal Church (woman bishop to oust conservative priest)
A. "An Episcopal Church committee has backed efforts by Washington's acting Bishop Jane
Holmes Dixon to oust a conservative priest. A nine-member panel of clergy and laity said
Dixon followed church law when she rejected the decision of the Christ Church vestry to hire
the Rev. Samuel Edwards and ordered him to leave by May 25. Edwards refused, and
continues to conduct services and occupy the rectory," (Religion News in Brief, Associated
Press Online, 09/27/2001),
6. The Mennonite Church (on homosexuality)
A. "After nearly two decades of negotiation, the nation's two largest Mennonite denominations
overwhelmingly approved a merger Thursday that creates the 125,000-member Mennonite
Church USA....Many delegates expressed mixed feelings about the new membership
guidelines, under which the church will not recognize same-sex marriages. Individual
congregations and regional conferences will decide whether homosexuals will be allowed as
members. (Two Mennonite Groups Approve Merger, NASHVILLE, Tenn., Jul 05, 2001, AP
Online via COMTEX).
7. The Presbyterian Church United States of America (on homosexuality)
A. "In a closely watched case, the highest court of the Presbyterian Church (U.S.A.) refused to
rule Tuesday on whether a homosexual man is eligible to serve as a church elder. The court
said the dispute surrounding Wayne Osborne had become moot because the First
Presbyterian Church of Stamford, Conn., had installed another group of elders,"
(Presbyterian Court Sidesteps Ruling; Ky., Dec 04, 2001, AP Online via COMTEX).
8. The United Church of Christ (on homosexuality)
A. "The United Church of Christ set up a $500,000 scholarship fund for gay and lesbian
seminarians Friday and urged wider acceptance of homosexuals by other denominations."
(United Church Makes Gay Scholarship, CLEVELAND, Jun 16, 2000, AP Online via COMTEX).
9. The United Methodist Church (on homosexuality)
A. "The supreme court of the United Methodist Church was asked Thursday to reconsider the
denomination's ban on gay clergy. (Church court of United Methodists asked to decide on
gay clergy ban, NASHVILLE, Tennessee, Oct 25, 2001, AP WorldStream via COMTEX).

In contemporary culture, homosexuality is gaining ground and acceptance. This should never
bleed over into the Christian church. The fact that homosexuality is even in question in some
denominations is a definite sign of apostasy since homosexuality clearly contradicts the plain teaching
of scripture that homosexuality is a sin: "Or do you not know that the unrighteous shall not inherit the
kingdom of God? Do not be deceived; neither fornicators, nor idolaters, nor adulterers, nor effeminate,
nor homosexuals, 10 nor thieves, nor the covetous, nor drunkards, nor revilers, nor swindlers, shall
inherit the kingdom of God," (1 Cor. 6:9-10, NASB).
Unfortunately, too many denominations are listening to the false gospel of the world and becoming
secularized. They need to repent and stand on God's word, whether it is popular or not.
What are signs that a church is becoming secular?
"For the grace of God has appeared, bringing salvation to all men, 12instructing us to
deny ungodliness and worldly desires and to live sensibly, righteously and godly in the
present age," (Titus 2:11-12).

What does a secular church look like? Would it be easy to find one? Would we know one if we saw
one? Then again, maybe you attend a secular church and don't know it. It is certainly possible, but
how would you know? How would you recognize a church that is more secular than sacred?
Like any counterfeit, the best way to recognize the secular is to be familiar with the sacred. That is
why dedication to God's word is so important. The Christian church focuses on being fed out of God's
word and seeks to align itself with what it says. The secular church allows the ways of the world to
seep into the beliefs and practices of the Christian church thereby diluting the truth.
If we find something in the church that is contrary to scripture but is taught in the secular world,
then that church has become, in part, secularized. The more we find from the world in the church, the
more that church is secularized.
Following is a list of things that, in my opinion, are examples of secularization in the church. Of
course, this list is not exhaustive nor is it authoritative. It is my opinion. Nevertheless, it is offered as
food for thought. Is your church becoming secular? Are you?

1. Teaching that the Bible is not inspired and inerrant.


A. The Bible is the word of God (2 Tim. 3:16) and is the measure of truth and righteousness.
To claim that it is not inspired is to reduce it to the level of the Quran, or the Bhagavad-Gita,
or the Book of Mormon which are mere man-made writings and not inspired.
B. When the authority of God's word is lost, then man-made doctrines creep in. The authority
and inspiration of scripture is the anchor that keeps the church from drifting into error.
2. Using books instead of the Bible in Bible study
A. It is okay to use books that assist in Bible study, but the Bible should be the central source
of spiritual truth, not books about the Bible. If Bible studies are using guidebooks more than
the Bible itself, then the Bible has been moved to a secondary position. If Christians are
having trouble understanding God's word, then the pastor (or Bible study leader) needs to
teach them how to find its truths so they can check all things by scripture b7 themselves
(Acts 17:11).
3. Teaching that there is more than one way to God besides Jesus
A. In this world of relativism, it is not popular to claim that Jesus is the only way to be saved.
But this is what the Bible says. John 14:6, "Jesus *said to him, "I am the way, and the
truth, and the life; no one comes to the Father, but through Me." Also, Acts 4:12, "And there
is salvation in no one else; for there is no other name under heaven that has been given
among men, by which we must be saved." There is no other way to be saved. The Muslims,
the Buddhists, the Taoists, etc., cannot be saved without Jesus.
4. Being embarrassed to say that Jesus is the only way to salvation
A. Like the issue above, Jesus is the only way (John 14:6). Christians should never be
ashamed (Rom. 1:16) to speak the truth of God's saving work in Christ. For some, to be
timid and embarrassed means that one's eyes are off of God and onto people.
5. Teaching that there is no absolute right and wrong.
A. As mentioned above, moral relativism is the norm of society. We often hear, "It is true if it
is true for you." The Bible tells us that there are moral absolutes independent of what we
think is right. Exodus 20:1-17 is a list of the Ten Commandments which are moral
absolutes. The Bible teaches us there is absolute right and wrong. Without moral absolutes,
no one can say anything is right or wrong.
6. Being careful to not offend anyone at the expense of biblical truth.
A. Whether or not someone likes what the Bible says does not change the truth of the Bible.
We should not be offensive just to be offensive, but we should not be afraid to speak the
truth of God when the need arises. The gospel that offends no one is not the gospel of the
Bible.

7. Pastors preaching moralism instead of Christ centered messages


A. Moralistic preaching is preaching that does not focus on the cross of Christ. For example, we
do not try and be good because being good is nice. We try and be good because Jesus
saved us from our sins and doing what is right glorifies Him. Preaching that is not focused
on the cross is not preaching. It is a waste of time.
8. Approving of homosexuality
A. Homosexuality is being accepted as normal all over the world. It should not be accepted as
normal in the Church. Homosexuality is a sin (1 Cor. 6:9). It is wrong. But this does not
mean we are to hate homosexuals. We are to pray for them and their repentance.
Homosexuals are not to be pastors or elders in churches.
9. Approving of women elders
A. This one may offend a lot of people, but the Bible teaches that the elder is to be the husband
of one wife. This is not merely a cultural norm of the time. It is what the Bible teaches.
10. Not condemning the sins of society
A. If at all possible and according to wisdom, Christians should not be intimidated by the world
when it comes to condemning sin. Sometimes, when Christian pastors condemn a sin in the
world, like abortion, they are attacked. Truth is not silenced by complaining voices. We do
not answer to them. We answer to God.
11. Psychology as an authority on human nature.
A. The Bible tells us that people are sinners by nature, selfish, prideful, and in need of the
saving work of God. The Bible tells us what is right and wrong regarding childrearing,
criminal behavior, actions, and words. Psychology can give us insights on many things, but
if it contradicts God's word, it is wrong. We need to accept the fact that God is the authority
on man, not the psychologists.
12. Use of politically correct terms of the world from the pulpit where those terms replace
biblical values and truths
A. Is a wife or husband a "partner?" Are Christians who condemn homosexuality
"homophobes?" Is it correct to say a woman can kill the baby in her womb and call it
"abortion rights" when discussion "reproductive rights"? Is accepting false theologies called
"diversity awareness"? If such words and terms become the common vocabulary of the
pulpit without qualification and/or explanation, then the preacher is adopting the terms of
the world and not of God and by it he is slowly being seduced by the world.
13. Going to church as a social habit
A. Church is not a social club where politically correct ideas are tried and tested. It is not a
place we go to voice our opinions so that others can hear our wisdom. It is the place where
we go to learn, to be corrected, to grow, and to encounter God who is not made after our
own image.
14. Prayer as a last resort
A. To seek to accomplish things in life without God, whether it be big or small, is to say we do
not need God and to proclaim our independence from Him. To resort to prayer as a last
resort is to exclude God from the beginning of our work and to proclaim our independence
from Him. It is the world that operates without God, not the Christian.
15. Missionaries not sent out or supported
A. Missionary efforts needlessly curtailed. If a church cannot support a missionary effort
because of legitimate circumstances, that is one thing. But, if it can and it does not
specifically reach out to the world with the gospel, then it is disobeying God's word ( Matt.
28:18-20).
16. Divorce statistics as common as secular society
A. What a horrible failure this is that the Christians have the same divorce rate as the
unchurched. Undoubtedly, this is due to the secularization of the heart.
17. Evolution
A. Either not condemning evolution or not taking a stand on it.
The elder in the church

The elder is a leader in the Christian church. It is a divinely appointed office that is held by men
who are able to teach sound doctrine, refute error, be of good reputation, having believing children,
who manage their households well, etc. The elder should not be in office if he is unable to fulfill the
requirements of the office and he should not be chosen because he is a popular figure in the local
church.
Being an elder is a high calling and there are thousands of very godly men in many churches and
who seek God and serve Him as best as they can. But, it is also true that many should not be elders
who are in that position. Women, should not be elder's. Men with unbelieving children living with
them should not be elders. And, elders should be able to teach correct doctrine and refute error.
Sadly, many elders cannot do this.
Below is an outline dealing with the subject of "elder." Pastors, elders, and congregation members
should all be careful who they pick and recommend as elders. They should not let feelings of "let's be
nice" influence their decisions and choose men who are not qualified. The church needs to take the
office of elder very seriously and it needs to put the elder to the test according to the guidelines of
scripture. If he is not qualified, he should not be an elder.

1. The Term "Elder"


A. PRESBUTEROS - elder, an old man, a leader in the church. The term is used
i. of the elder of two persons (Luke 15:25, or more, John 8:9).
ii. of a person advanced in age (Acts 2:17; in Heb. 11:2).
iii. of the forefathers in Israel (Matt. 15:2; Mark 7:3,5).
iv. of members of the Sanhedrin (Matt. 16:21; 26:47).
v. of those who managed public affairs in the various cities (Luke 7:3).
vi. of those who were the heads or leaders of the tribes and families, as of the seventy who
helped Moses (Num. 11:16; Deut. 27:1). This included:
a. acting as judges in apprehending murderers (Deut. 19:12).
b. conducting inquests (Deut. 21:2).
c. settling matrimonial disputes (Deut. 22:15; 25:7).
d. If theirs was a city of refuge they also heard pleas for asylum (Joshua 20:4).
vii. of those qualified by the Holy Spirit who exercised spiritual care and oversight of the
local congregation.
B. EPISKOPOI - overseers, bishops
i. Titus equates bishop and elder in Titus 1:59.
2. Regarding the Office:
A. The office of Elder is a divinely appointed office as defined in the Pastoral Epistles.
B. Elders are apparently appointed by the laying on of hands (1 Tim. 4:14; 2 Tim. 1:6).
C. Should receive double honor in the church (1 Tim. 5:17).
D. The pastor is an elder who preaches and/or teaches (1 Tim. 5:17).
i. The pastor (elder) is to equip the body of Christ (Eph. 4:11-13).
E. Must be a man (1 Tim. 2:9-13).
i. All uses of "elder" are in the masculine except for 1 Tim. 5:2 where it means older
women.
3. The Responsibilities of Elders in the NT Church:
A. Must shepherd the flock (1 Peter 5:2).
B. Must voluntarily exercise oversight upon the flock (1 Peter 5:2).
C. Must live as examples to the flock (1 Peter 5:3).
D. Anoint and pray for the sick (James 5:14).
E. They have the tasks of teaching (1 Tim. 5:17; Titus 1:5,9).
F. They have the tasks of acting as judges (Acts 15:2,6,22-29; 16:4).

4. Qualifications for an elder


A. Must be above reproach (Titus 1:6; 1 Tim. 3:2)
B. Husband of one wife (Titus 1:6; 1 Tim. 3:2).
C. Household must be in order with children who believe (Titus 1:6 ;1 Tim. 3:4).
D. Not a new convert (1 Tim. 3:6).
E. Self controlled and temperate (Titus 1:7; 1 Tim. 3:2).
F. Honorable, hospitable, seeking good (Titus 1:7).
G. Have a good reputation (1 Tim. 3:7).
H. Not addicted to wine (1 Tim. 3:3).
I. Not greedy (1 Tim. 3:3).
J. Able to exhort (teach) sound doctrine (Titus 1:9; 1 Tim. 3:2).
K. Able to refute false teaching (Titus 1:9).
L. They must be ready to earn their own living if necessary (Acts 20:17, 33-35).
Should unbelievers lead Christians in worship in a church service?

This question should never arise in the Christian church. But, unfortunately, it needs to be
addressed because there are churches in America that have unbelievers leading Christians in worship
on Sunday morning services. This is wrong and it is a sign of apostasy in the Christian church.
First of all, worship can only be rightly performed by believers who have been justified and
sanctified by the blood of Jesus Christ (1 Pet. 2:9). Only blood-bought believers have the right to
worship God because only they have a mediator by which their worship may be accepted (1 Tim. 2:5).
The unbeliever has no mediator. Unbelievers do not have that right to praise God for His goodness
and mercy because they are in a state of rejecting it -- which is why they are unbelievers -- and they
are unclean before God! Remember how God rejected the sacrifice of Cain who was a murderer (Gen.
4:5), how much more the false and hypocritical worship of unbelievers. Jesus said to the Pharisees
and Scribes, "You hypocrites, rightly did Isaiah prophesy of you, saying, 8‘This people honors Me with
their lips, but their heart is far away from Me," (Matt. 15:7). Are the unbelievers any better off than
the Pharisees and Scribes? No, they are, like the Pharisees, under harsher judgment.
How dare the unbeliever lead the believer in worship of the true and Holy God when he or she is in
a state of rebellion against God! May it never be! "But to the wicked God says, “What right have you
to tell of My statutes, and to take My covenant in your mouth?" (Psalm 50:16). Can the unbeliever
praise God with the following words and not be a hypocrite? "Come, let us worship and bow down;
Let us kneel before the Lord our Maker. 7For He is our God, And we are the people of His pasture, and
the sheep of His hand," (Psalm 95:6). No. He cannot! To sing such praises is a mockery to God.
Therefore, they are in the same state as those who are condemned by Christ on the day of judgment:

"Many will say to Me on that day, ‘Lord, Lord, did we not prophesy in Your name, and in
Your name cast out demons, and in Your name perform many miracles?’ 23"And then I
will declare to them, ‘I never knew you; depart from Me, you who practice lawlessness,’
(Matt. 7:22-23).

Second, it is hypocritical for a pastor to have unbelieving musicians come into the church of Lord
and lead blood-bought believers in worship. Hypocrisy is saying (singing) one thing and doing
another. It is a display of behavior of falseness. And that is exactly what the pastor is encouraging
when he has unbelievers lead believers in praises to God because, in their hearts, they do not trust in
the sacrifice of Christ. Their words are singing praises of trust and adoration to God when in their
hearts they do not believe it. This is hypocrisy and the pastor is encouraging it.
Furthermore, I am sure that in the great majority of such hypocritical situations, the unbelievers
are "praising God" for money. Think about it: a pastor offers money to unbelievers to have them
come into the church of God and lead believers in worship. Worship is supposed to be an offering to
God by believers and a means of preparation of the heart so that the Christian might receive the word
of God in the sermon. How dare a pastor pay and unbeliever to do this!
Third, it was the Pharisees in the New Testament who had an outward manifestation of worship but
inwardly were unregenerate. To this, Jesus condemned them as hypocrites (Matt. 15:7). If a pastor
of a church is not informing the congregation of the fact that unbelievers are leading believers in
worship (which should never occur in the first place) then the following verses apply to the pastor:

"But woe to you Pharisees! For you pay tithe of mint and rue and every kind of garden
herb, and yet disregard justice and the love of God; but these are the things you should
have done without neglecting the others. 43"Woe to you Pharisees! For you love the front
seats in the synagogues, and the respectful greetings in the market places. 44"Woe to
you! For you are like concealed tombs, and the people who walk over them are unaware
of it," (Luke 11:42).

If the congregation is unaware of the unholy and unregenerate nature of those leading them in
worship, then they are inadvertently participating in the unholy worship of hypocrites. When the
Pharisees worshipped God, their worship was rejected and condemned. Jesus, exposed the hypocrisy
of outward worship and inward disbelief and said it "dirtied" those who followed their lead - referring to
Levitical laws of cleanness. Is this not basically the same thing happening when unbelievers lead
unsuspecting believers in worship? In this, the congregation thinks the worship leaders are "sanctified
by Christ's blood" when, in reality they are not. And, to make it worse, the "worship leaders" further
the deception on a weekly basis. Now, the pastor is the leader in the church. Woe to him who leads
the church into hypocritical worship to God and encourages sinners to sin.
Fourth, we find in scripture that only believers lead worship and are involved in true worship, never
unbelievers:

• "These things I remember, and I pour out my soul within me. For I used to go along with the
throng and lead them in procession to the house of God, with the voice of joy and thanksgiving,
a multitude keeping festival," (Psalm 42:4).
• "Ascribe to the Lord, O sons of the mighty, Ascribe to the Lord glory and strength. 2Ascribe to
the Lord the glory due to His name; Worship the Lord in holy array," (Psalm 29:1).
• "But you are a chosen race, a royal priesthood, a holy nation, a people for God’s own
possession, that you may proclaim the excellencies of Him who has called you out of darkness
into His marvelous light," (1 Pet. 2:9).

We find no place in the scriptures where unbelievers lead believers in worship. It is not in the Bible
and it should never happen in the Christian church.

What of the benefit to the unbeliever?

What about the benefit of exposing an unbeliever to the gospel by getting him into church to
participate in worship? After all, it has probably led to conversions.
The ends do not justify the means. If God so chooses to save someone in spite of a sinful
situation, that is God's business. The pastor is to make sure that the worship honors God. It is not
the place of evangelism. It is the place of worship. Do not be deceived into compromising praise and
worship to God by having unbelievers participate in it.

To the pastor

If you are a pastor who has unbelievers lead the believing congregation in worship, stop it now.
You are responsible for shepherding your church in truth, honor, and sanctification before God. You
must repent of this sin and confess it. You must dismiss the unbelievers from worship, invite them to
attend the church service, confess your sin to the elders and the church, ask forgiveness
It is far better that the worship in the church be less than perfect on the outside than have it be
stained by unbelievers with unregenerate hearts
What kind of a Christian are you?
"Test yourselves to see if you are in the faith; examine yourselves! Or do you not
recognize this about yourselves that Jesus Christ is in you— unless indeed you fail the
test?" (2 Cor. 13:5, NASB).

What kind of a Christian are you? Have you ever sat down and asked yourself that? Are you a
Christian with your heart and mind in the world and your feet in the church? Or, are you the kind that
seeks to do God's will as revealed in Scripture? Whichever one you are, the Bible tells you to examine
yourself to see if you are in the faith. Have you done that? Have you examined your beliefs, your
actions, and your motives and compared them to scripture to see what kind of a Christian you are?
Are you submitting your life to God or God to your life?
This is an important question. No one should simply take it for granted that he is saved because
he is born into a Christian family, or is a good person, or is sincere in heart and mind, or thinks that
God is so loving that He'll let everyone into heaven. These things do not make anyone a Christian nor
do they guarantee anything with God. Only faith in Christ (Rom. 5:1), trust in Him alone for the
forgiveness of sins (Acts 4:12), receiving Him (John 1:12) is what being born again is about. It means
to be changed from the inside because the Lord has changed you, saved you, and the Holy Spirit is
now in you.
What kind of a Christian are you? Are you one on the outside but not on the inside? Are you a
social Christian? Do you claim the name but not the heart of Christianity? Are you a person who
knows he or she is saved because of the work of Christ on the cross and you trust in Him alone? Or do
you think that your sincerity and good works, in cooperation with God's grace, will get you to heaven?
Are you the kind of a person who believes in God yet walks into sin, knowingly, painfully? Maybe you
listen to the world and think that homosexuality is an acceptable alternative life style. Or maybe you
think that true Christians need to remain open minded and not condemn anyone. Is premarital sex is
okay if you love each other and/or plan to get married? Do you think that adultery is just an "affair"
or is it a terrible sin against God? Your opinions reveal what you are inside: biblical, worldly, or a little
bit of both. Ultimately, though, you must examine yourself to see if you are in the faith. Basically, if
you agree with the Bible, you're fine. If you do not, then you are wrong.
Here are some questions meant to get you to think and examine yourself to help you see what kind
of a Christian you are, maybe to see if you really are one. The Holy Spirit can prompt you. He can let
you know about sin and righteousness (John 16:8). Lovingly and humbly submit yourself to Him.
Examine yourself:

• Do you believe the essentials of the Christian faith: Trinity, Jesus is God in flesh, salvation by
grace through faith, Jesus rose from the dead physically?
• Do you confess that you are a sinner before a holy God? (Rom. 3:23)
• Do you confess that you cannot please God through your own efforts? (Isa. 64:6)
• Do you acknowledge that Jesus is the only way to salvation? (John 14:6; Acts 4:7-12)
• Do you acknowledge that there is only one God in all the universe? (Isaiah 43:10; 44:6,8)
• Do you seek to do the will of the Lord? (Matt. 7:21)
• Do you know that God loves you and desires your relationship with Jesus? (1 Cor. 1:9).
• Do you acknowledge that sin causes a separation between you and God? (Isaiah 59:2)
• Do you approve of social agendas in the church? (Rom. 12:12)
• Do you believe that morals are relative? (Exodus 20:1-17)
• Do you suspect that reincarnation might be true? (Heb. 9:26).
• Do you casually look at the Bible as a guide book, not the rule of truth and faith? ( 2 Tim. 3:16)
• Do you believe that feelings are as valid as scripture to find truth? (Jer. 17:9)
• Do you believe that those who reject Christ will go to hell forever? (Matt. 25:46)
• Do you pray only when something is wrong in your life? (Phil. 4:6)
• Do you go to church only on special occasions? (Heb. 10:25)
• Do you use the Lord's name in vain? (Exodus 20:7)
• Do you regularly watch things on TV and in the movies that you shouldn't? ( Phil. 4:8)
• Are we basically good in nature or bad? (Eph. 2:3; Psalm 51)
• Is the devil a real being? (Rev. 20:110)
• Is dust collecting on your Bible?

If you have the Son in you, the Holy Spirit will bear witness of the truth of your life in Him. Jesus
said, "My sheep hear My voice, and I know them, and they follow Me; 28and I give eternal life to them,
and they shall never perish; and no one shall snatch them out of My hand," (John 10:27-28).

A call to repentance and commitment

Repentance means to turn from that which is unholy towards that which is holy. It means to turn
away from sin and move towards God. God is not where our sin is. He is not where rebellion to His
word is. He is not in the world that seeks to use the Church as a social instrument, nor with the
Christian who wants to change the church and make it more "tolerant." He is not in the hearts of the
cold who do not take His words seriously, but instead give casual regard to it only when it agrees with
their feelings and wants.
Repentance begins when we realize that our hearts are not in accord with God and we willingly
submit our wills to God's will as it is revealed in the Bible. We discover our need to do this this when
we compare ourselves to the Lord Jesus. We discover this when we compare ourselves to the word of
God.
After repentance comes commitment. Commitment to God and His word should be the heart of
every Christian. After all, Jesus said that those whom He knows hear His voice and follow Him ( John
10:27-28). That is, they follow what Jesus says and do what He does.
What kind of a Christian are you?
Do you know the basics of the Christian faith?

"Therefore leaving the elementary teaching about the Christ, let us press on to maturity,
not laying again a foundation of repentance from dead works and of faith toward God,
2
of instruction about washings, and laying on of hands, and the resurrection of the dead,
and eternal judgment,” (Heb. 6:1-2, NASB).

God tells us to grow beyond the basics of the Christian faith and to press on to maturity. Sadly,
most Christians are not very well acquainted with the basics of Christianity and so linger in the milk of
the word instead of going on to the meat. My interviews of Christians over the years has led me to
believe that about 80% of them have a faulty understanding of justification by faith and about 90% as
many couldn't defend the deity of Christ from the Bible. In fact, most Christians I've encountered do
not understand who Jesus is biblically, let alone defend the proper understanding of what Jesus did on
the cross.
Now, I am not saying that all Christians need to be masters of theology, philosophy, and the Bible.
I am saying that Christians need to grow passed the basics and onto maturity just like it says in Heb.
6:1-2.
Following is a list of questions that are basics. The average Christian who has been a believer for,
say, two years or more, should know the majority of the answers below. If you don't know them, then
click on the answers link at the end of the questions to see what the right answers are and learn.
I suggest that you get a notebook together and fill it with things about doctrines, who Jesus is,
what He did, what salvation is, etc. Print papers from this site. You don't have to have them
memorized, but know them. Understand them. Use them as foundations in your Bible studies. They
will help.
How do you fair with the questions?

1. Can you tell someone what the Great Commission is?


2. Can you tell someone what the gospel is?
3. Can you tell someone where in the Bible the gospel is defined?
4. How many ways are there to get to God?
5. Can you quote John 3:16?
6. What book and chapter of the Bible has the 10 Commandments?
7. What is the basic salvation message?
8. Must you be baptized in order to be saved?
9. Is salvation obtained by being good, by faith in Jesus alone, or both?
10. Can you tell define the term justification?
11. Can you tell define the term sanctification?
12. Can you tell someone the difference between Mormonism and Christianity?
13. Can you quote scripture and location that says we are saved by grace?
14. Can you quote scripture and location that says there is only one God?
15. Is Jesus God in flesh, a good man, or an angel who became a man?
16. Did Jesus rise from the dead in a physical body or not?
17. How many natures does Jesus have?
18. Is the Holy Spirit a person like the Father and Son, or a force like radar?
19. Can you explain what the Trinity is?
20. Has God always been God?
21. Can you tell anyone what some of the attributes of God are?
22. Is Jesus going to return or not?
23. Did Jesus bare our sins in His body on the cross or not?
24. What is salvation?
25. What is sin?
26. What is damnation?
27. What is hell?
28. What is heaven?
29. Is the Bible inspired or is it a good book full of good moral stories?
30. What are the first five books of the Old Testament?
31. How many books are in the Bible?
32. Were Adam and Eve real people or not?
33. What is the biblical reason that Adam and Eve cast out of the Garden of Eden?
34. Is Satan a real being?
35. What are some of the qualifications of an elder?
Answers to the questions on basics.

You don't need to have the scripture verses memorized, and this list isn't authoritative. So, if you
don't know everything, that's alright. But, these are the basics in Christianity and Christians should, at
the very least, be familiar with them.

1. Can you tell someone what the Great Commission is?


A. "And Jesus came up and spoke to them, saying, "All authority has been given to Me in
heaven and on earth. 19"Go therefore and make disciples of all the nations, baptizing them in
the name of the Father and the Son and the Holy Spirit, 20teaching them to observe all that I
commanded you; and lo, I am with you always, even to the end of the age," (Matt. 28:18-
20).
2. Can you tell someone what the gospel is?
A. "Now I make known to you, brethren, the gospel which I preached to you, which also you
received, in which also you stand, 2by which also you are saved, if you hold fast the word
which I preached to you, unless you believed in vain. 3For I delivered to you as of first
importance what I also received, that Christ died for our sins according to the Scriptures,
4
and that He was buried, and that He was raised on the third day according to the
Scriptures," (1 Cor. 15:1-4)
3. Can you tell someone where in the Bible the gospel is defined?
A. See above.
4. How many ways are there to get to God?
A. Only one: "Jesus *said to him, "I am the way, and the truth, and the life; no one comes to
the Father, but through Me," (John 14:6).
5. Can you quote John 3:16?
A. "For God so loved the world, that He gave His only begotten Son, that whoever believes in
Him should not perish, but have eternal life," (John 3:16).
6. What book and chapter of the Bible has the 10 Commandments?
A. Exodus 20
7. What is the basic salvation message?
A. The basic salvation message is that Jesus die on the cross for our sins and rose from the
dead (1 Cor. 15:1-4). All who would believe in Him would have everlasting life and escape
the judgment of God upon those who have sinned against Him (John 3:16-18). Receiving
salvation from God is done by faith (Rom. 5:1; Eph. 2:8).
8. Must you be baptized in order to be saved?
A. No, you don't have to be baptized to be saved (Rom. 5:1; Acts 10:44-48). See "Is Baptism
necessary for Salvation?" for more information on this.
9. Is salvation obtained by being good, by faith in Jesus alone, or both?
A. Salvation is by grace through faith, not of works (Eph. 2:8-9; Rom. 5:1).
10. Can you define the term justification?
A. To be justified is to be made righteous. It is a divine act where God declares the sinner to be
innocent of his sins. It is not that the sinner is now sinless, but that he is "declared" sinless.
This justification is based on the shed blood of Jesus, "...having now been justified by His
blood..." (Rom. 5:9).
11. Can you tell define the term sanctification?
A. Sanctification follows justification. In justification our sins are completely forgiven in Christ.
Sanctification is the process by which the Holy Spirit makes us more like Christ in all that we
do, think, and desire.
12. Can you tell someone the difference between Mormonism and Christianity?
A. Mormonism teaches that God used to be a man on another world he became a god and
brought one of his wives to this world. They produce spirit offspring who enter human
babies at birth. Mormons believe they have the potential of becoming gods of their own
worlds.
B. Christianity teaches that there is only one God in all existence (who was never a man on a
planet). God exists as a Trinity: Father, Son, Holy Spirit. Jesus is the word made flesh
(John 1:1,14) and we cannot become gods.

13. Can you quote scripture and location that says we are saved by grace?
A. Eph. 2:8, "For by grace you have been saved through faith; and that not of yourselves, it is
the gift of God."
14. Can you quote scripture and location that says there is only one God?
A. "...Before Me there was no God formed, and there will be none after Me," (Isaiah 43:10).
See also Isaiah 44:6,8.
15. Is Jesus God in flesh, a good man, or an angel who became a man?
A. Jesus is God in flesh. Col. 2:9 says, "For in Him all the fulness of Deity dwells in bodily
form." John 1:1,14 says, "In the beginning was the Word, and the Word was with God, and
the Word was God...And the Word became flesh, and dwelt among us, and we beheld His
glory, glory as of the only begotten from the Father, full of grace and truth."
16. Did Jesus rise from the dead in a physical body or not?
A. Jesus rose from the dead in the same body He died in, though it was a glorified body.
"Jesus answered and said to them, "Destroy this temple, and in three days I will raise it up."
20
The Jews therefore said, "It took forty-six years to build this temple, and will You raise it
up in three days?" 21But He was speaking of the temple of His body." And, "So also is the
resurrection of the dead. It is sown a perishable body, it is raised an imperishable body; 43it
is sown in dishonor, it is raised in glory; it is sown in weakness, it is raised in power; 44it is
sown a natural body, it is raised a spiritual body," (1 Cor. 15:42-44).
17. How many natures does Jesus have?
A. Two. Jesus is both God and man. Col. 2:9 says, "For in Him all the fullness of Deity dwells
in bodily form."
18. Is the Holy Spirit a person like the Father and Son, or a force like radar?
A. The Holy Spirit is a person like the Father and the Son. He speaks (Acts 8:29; 13:2), has a
will (1 Cor. 12:11), loves, and can be grieved (Eph. 4:30).
19. Can you explain what the Trinity is?
A. The Trinity is the teaching that God exists in three eternal, simultaneous, co-equal persons
known as the Father, the Son, and the Holy Spirit.
20. Has God always been God?
A. Yes, God has always been God. Psalm 90:2 says, "Before the mountains were born, or Thou
didst give birth to the earth and the world, even from everlasting to everlasting, Thou art
God."
21. Can you tell anyone what some of the attributes of God are?
A. He knows all things (1 John 3:20). He is all powerful (Jer. 32:17,27). He is everywhere
(Psalm 139:7-12). He is holy (Isaiah 6:3; Rev. 4:8).
22. Is Jesus going to return or not?
A. Yes, Jesus is going to return. Acts 1:10-11 says, "And as they were gazing intently into the
sky while He was departing, behold, two men in white clothing stood beside them; 11and
they also said, "Men of Galilee, why do you stand looking into the sky? This Jesus, who has
been taken up from you into heaven, will come in just the same way as you have watched
Him go into heaven."
23. Did Jesus bare our sins in His body on the cross or not?
A. "and He Himself bore our sins in His body on the cross, that we might die to sin and live to
righteousness; for by His wounds you were healed," (1 Peter 2:24).
24. What is salvation?
A. Salvation is the "saving" of a sinner from the righteous judgment of God. When someone
appeals to God and seeks forgiveness in Jesus, his sins are forgiven. He is cleansed. His
relationship with God is restored, and he is made a new creature (2 Cor. 5:17). All of this is
the work of God, not man. Salvation is a free gift (Rom. 6:3).
25. What is sin?
A. Sin is anything that is contrary to the law or will of God. For example: if you lie, you have
sinned. Why? Because God has said not to lie (Exodus 20:16). If you do what God has
forbidden, then you have sinned. In addition, if you do not do what God has commanded,
you sin (James 4:17).
26. What is damnation?
A. The righteous judgment of God upon a sinner where the sinner is cast out of the presence of
God into Hell for ever. (Matt. 25:46).

27. What is hell?


A. Hell is the future place of eternal punishment of the damned including the devil and his fallen
angels. See Matt. 11:23; 16:18; Acts 11:27; 1 Cor. 15:55; Rev. 1:18; 6:8).
28. What is heaven?
A. Heaven is the dwelling place of God and for those who go there a place of everlasting bliss.
See Matt. 23:34-37; Luke 10:20; and Rev. 22:2,20-27.
29. Is the Bible inspired or is it a good book full of good moral stories?
A. The Bible is inspired and inerrant in the original documents. See 2 Tim. 3:16.
30. What are the first five books of the Old Testament?
A. Genesis, Exodus, Leviticus, Numbers, Deuteronomy.
31. How many books are in the Bible?
A. There are 66 books in the Bible: 39 books in the OT, 27 in the NT.
32. Were Adam and Eve real people or not?
A. Yes, they were real individuals. (Gen. 2-3)
33. What is the biblical reason that Adam and Eve cast out of the Garden of Eden?
A. They disobeyed God by eating of the forbidden fruit (Gen. 3).
34. Is the devil a real being?
A. Yes, he is a real being, a fallen angel. "But Michael the archangel, when he disputed with
the devil and argued about the body of Moses, did not dare pronounce against him a railing
judgment, but said, "The Lord rebuke you," (Jude 9). (see also Rev. 12:9)
35. What are some of the qualifications of an elder?
A. Must be above reproach (Titus 1:6; 1 Tim. 3:2)
B. Husband of one wife (Titus 1:6; 1 Tim. 3:2).
C. Household must be in order with children who believe (Titus 1:6; 1 Tim. 3:4).
D. Have a good reputation (1 Tim. 3:7).
E. Able to exhort (teach) sound doctrine (Titus 1:9; 1 Tim. 3:2).
F. Able to refute false teaching (Titus 1:9).
Are you comfortable?
"Consider it all joy, my brethren, when you encounter various trials, 3knowing that the
testing of your faith produces endurance. 4And let endurance have its perfect result, that
you may be perfect and complete, lacking in nothing," (James 2:2-4, NASB).

Are you comfortable? Do you feel safe in the Lord? I hope so. You should. Comfort and peace
are great blessings from the Lord. He loves us so much that He gave His Son and sent the Holy Spirit
who is called The Comforter (John 14:26, KJV). We are secure in Him (John 10:27-28), can rest in
Him (Matt. 11:28), and don't need to be anxious for anything (Phil. 4:6). We have a great and
awesome God who has made all this possible.
However, sometimes comfort can be a stumbling block. Sometimes comfort can rob us of our
strength and dependence on God. Think of a man who is so comfortable in his life with so few
problems that he doesn't do much of anything let alone worry about anything. He relaxes and enjoys
life. He also becomes weak and dependent upon his routine and life. So too the Christian who is very
comfortable in his life, can also become weak and dependent upon the securities of life instead of the
Lord. There is nothing wrong with being comfortable, unless that comfort makes us depend on God
less and cause us to become complacent about the lost around us.
Where the early Christians had to rely on God for their every need, today in America and much of
the modern world, creature-comforts and drive-through churches have made many Christians
complacent and sluggish. Most (I hope) are saved, but it seems that far too many have settled into
the church routine: Sunday service; maybe Wednesday, too; don't share their faith much; pray when
a need arises; enjoy life; tithe occasionally; let pastors and missionaries do the hard spiritual work,
etc. In this mode, the call of God to make disciples of every nation is a faint whisper that if listened
to, can only cause inconvenience and a disruption of Christian comforts. Are you one of those
Christians? Are you so comfortable in your life that you aren't concerned about the lost, don't depend
on God, tithe infrequently, and hardly seek God's face?
God has blessed us in America. We have the best of everything and only need to put on credit
what our whims demand. We have our VCR’s, air-conditioning, remote controls, and fast food
restaurants. We have churches with central air, great sound systems, well educated preachers, plush
pews, and fine-tuned choirs, pianos, and organs. We are blessed with committees, plans, and money.
In fact, we have so many churches we are guaranteed we can find one to suit any whim or
preference. Unfortunately, all too often, the messages are pleasant and don’t make our hearts ache
for the lost or for our Lord. This is a recipe for danger. We are truly blessed. But, those blessings can
become curses if they weaken our desires to live for God and reach the lost.
God sometimes allows trials and tribulations in our lives in order to get us to look to Him. Struggle
tends to strengthen faith because in struggle we turn to God. He answers our prayers and provides
our needs and we in turn praise Him. In this process, the Holy Spirit is alive in us, working mightily,
and we sense Him teaching us, guiding us, shaping us. That is why whenever we are close to God, we
are far from sin. Whenever we are close to God, we are aware of our ungodliness. Whenever we are
close to God, we are concerned for the lost--because He is. Are you far from sin? Are you aware of
your ungodliness? Are you concerned for the lost?
Therefore, I ask you. Have you become distracted from the calling of God to grow in grace and
make disciples of all nations? I am not talking about doing your duty of going to church on Sunday
and reading your Bible occasionally. I'm talking about making headway, actively seeking God, being
willing take risks for Him, asking to be used, asking to be shaped, etc. Are you doing that? If not,
maybe you are too comfortable.
So what should we do?

Please understand that I am not advocating poverty and misery as a way of measuring the
Christian life. There is nothing wrong with being comfortable, having money, or remote control TV's,
and air conditioned cars. We should praise God for these. But, you need to ask yourself if your life
has become filled with a familiarity with comfort and with the "Christian life" that you are simply in a
routine that has unwittingly numbed you to the spiritual realities of life. If you think that maybe you
have backslidden in this way, then I have some suggestions.
First of all, pray to the Lord and ask Him to reveal your sins to you. Confess them and do your
best to repent as you continue to rely on His grace. Second, read your Bible regularly and ask the
Lord to apply to your heart what you read. Third, ask God to put a desire on your heart that is in
accordance with your spiritual giftings so that you may not only grow and edify the body of Christ, but
also to reach out to the lost. If you don't know what your spiritual gifts are, that's okay. God will
show you. Fourth, don't be afraid to take risks for the Lord. Don't be afraid to become a little un-
comfortable. Tithe. Pray. Intercede. Read the Word. Confess your sins. Speak the gospel.
Remember, our life is not about our comforts. It is about loving God, loving others, and spreading
the word of God.
Apologetics
Introduction

Apologetics is somewhat of a lonely endeavor. It is possible for a person to give a great deal of
effort to apologetic work, to defending God’s word, to answering questions, to reasoning with people
and have it all seem as though it was worthless. Discouragement is a reality to the apologist. There
are certainly victories, by God’s grace. But there are many encounters that could simply be classified
as "unprofitable."
To help you keep your eyes on the real issue of apologetics, I offer the following illustration. The
idea is to get you to understand what your job is as an apologist, as someone who answers questions
and objections, and points people to Jesus. I believe that if you understand where you are and what
your ‘job’ is, then you won’t be as overcome with discouragement as you might otherwise be.

Apologetics is like . . .

Apologetics is like a field. In the center of the field is a garden. This garden has one door and that
door is Jesus. There is one path that leads to that door. Inside the garden is eternal life in the
presence of God. Outside it, however, in the field, are rocks, boulders, thorns, thistles, valleys, hills,
and many false paths that lead nowhere.
The apologist resides in the field and points people to the true path so they can find the Garden.
The apologist seeks to remove the intellectual thorns and emotional rocks that prevent people from
finding the truth path to God. Also, there are many people who are walking false paths (cults,
philosophies, etc.) who will never reach that garden. The apologist gently guides the person, removes
the obstacles, and points in the direction of the Garden. When people arrive there, it is between them
and God on whether or not they enter.
Picture yourself as a laborer in the field. It isn’t your job to save anyone. It is your job to point the
way. You aren’t the only one in the field. Getting them to the Garden is not your job. They get there.
You simply help them.

1. What is apologetics? p. 2
2. Why is logic important in apologetics? p. 5
3. Why is prayer important in apologetics? p. 7
4. What are some of the guidelines for doing apologetics? p. 8
5. What is classical apologetics? p. 9
6. What is Presuppositional Apologetics? p. 11
7. What is Evidential Apologetics? p. 12
8. Basically, what is the Cosmological Argument? p. 13
9. Basically, what is the Teleological Argument? p. 14
An Introduction to Apologetics
The word "apologetics" comes from the Greek word "apologia," pronounced, "ap-ol-og-ee’-ah." It
means, "a verbal defense." It is used eight times in the New Testament: Acts 22:1; 25:16; 1 Cor. 9:3;
2 Cor. 7:11; Phil. 1;7,17; 2 Tim. 4:16, and 1 Pet. 3:15. But it is the last verse that is most commonly
associated with Christian apologetics.

"but sanctify Christ as Lord in your hearts, always being ready to make a
defense to everyone who asks you to give an account for the hope that is
in you, yet with gentleness and reverence," (1 Pet. 3:15).

Therefore, Christian apologetics is that branch of Christianity that deals with answering any and all
critics who oppose or question the revelation of God in Christ and the Bible. It can include studying
such subjects as biblical manuscript transmission, philosophy, biology, mathematics, evolution, and
logic. But it can also consist of simply giving an answer to a question about Jesus or a Bible passage.
The later case is by far the most common and you don’t have to read a ton of books to do that.
Apologetics can be defensive and offensive. The apologist can and should defend his reasons for
believing (1 Pet. 3:15). But, he can also go on the attack. He can seek out those who oppose
Christianity (2 Cor. 10:5). Of course, he should be prepared to do this before hand and all apologetics
is to be done with gentleness.
Apologetics can be, basically, evidential (often called "classical") or presuppositional. Evidential
apologetics deals with the evidence for Christianity: Jesus’ resurrection, the biblical manuscripts,
fulfilled prophecy, miracles, etc. Presuppositional apologetics deals with the presuppositions of those
who oppose Christianity, because presuppositions effect how a person views evidence and reason.
Some areas of debate within Christian apologetics deal with the use of evidence, reasons,
philosophy, etc. Should the apologist use only those criteria acceptable to unbelievers? Are we allowed
to use the Bible as a defense of our position or must we prove Christianity without it? Is reason alone
sufficient to prove God existence or Christianity’s truth? How much should reason and evidence be
used in light of the Scriptures teaching that it is God who opens the mind to understand? What part
does prayer, using the Bible, and the sinful nature of the unbeliever play in witnessing? How do these
factors interrelate to bring an unbeliever to faith? The questions are easy. The answers are not.
Jesus chose one highly educated religious person as an apostle. That was Paul. The rest were
fishermen, a tax collector, a doctor, etc. They were normal people of the day who were available and
willing to be used by the Lord. They were filled with the Spirit of God and they were used as vessels of
God. God uses all things for His glory. So, we do apologetics by faith.
The Lord has called every Christian to be ready to make a defense of his faith. That means you are
called to give reasonable answers to questions regarding Christianity. Now, this does not mean that
you must have a PhD or that you have to go to seminary. But it does mean that you should be willing
to at least give an answer for your beliefs. If you find you cannot, then prayerfully take it to God and
start studying.

What do you study?

You could pray and ask the Lord to teach you what He wants you to know. Ask Him to give you a
burden for something to learn. It doesn’t matter what it is. Just ask. Whatever you become interested
in is what you should learn about because it is probably something God wants you to know for later
use. It is like having tools in a tool shed. The more you have, the more you can accomplish.
Another way to find out what God wants you to study is through circumstances. Let’s say that a
Jehovah’s Witness comes to your door and debates the deity of Christ with you and you find you don’t
know how to defend it biblically. In that case, you know you need to study biblical verses that teach
Jesus is God in flesh. Or maybe a coworker asks you how you know the Bible is true? If you don’t have
a answer, pray, and start researching. Go to a Christian bookstore and get some books on it. Talk to
your pastor. You’ll learn.
Sometimes God will make a verse or subject in the Bible "come alive" to you and it might strike
you as odd or interesting. You could get a commentary and read up on it. You could ask others about
it. In so doing, you are preparing yourself through learning to be ready to answer questions and point
people to the truth. You'd be surprised how many details God can use to help you in your witness,
even through those apparently odd times when verses suddenly "come alive."

Get an Ignorance Notebook

The ignorance notebook is something I started almost 20 years ago to help me study. You can
make one yourself. It’s simple. Get a 3 ring binder and fill with blank pages. Then ask God to fill it with
what you need to know.
As in the examples above, God will put burdens on your heart, or cause you to find places where
you are lacking in knowledge, or a Bible verse will strike you. Write down what you learn in your
notebook. Date the pages as you go. You’ll be amazed at what you learn.
Basically, apologetics is equivalent to theology in sneakers. It means getting the hay down off the
loft and down to where the cows can eat it. Anyone can ‘do’ apologetics. All it takes is willingness, a
little work, and the Spirit of God in you.
Are you and apologist?

Sometimes when I do seminars, I stand in front of a group of people and introduce myself. I give a
very brief history of what got me started in apologetics and what keeps me going. Usually, those who
are there are there to learn about witnessing to Mormons or Jehovah's Witnesses, some other cult
group, Christian doctrine, evangelism, or are simply there to ask questions on different subjects.
Invariably, I introduce the term 'apologetics' to the group and define it as that field of Christian study
that defends biblical truth against anything that opposes it. Also, I state that apologetics is as varied
as there are people and subjects and that no one can master all areas. According to a person's gifts
and interests, he or she will become proficient in what interests that person as God calls him into
study.
But one of my concerns when doing seminars is what I call "The Speaker Effect." Basically, when a
group gathers to hear a speaker, it is assumed that the speaker knows his material and is very
experienced in the subject. Given the fact that public speaking is America's number one phobia, the
mere fact that a person can get up there and speak for an hour on a subject (and enjoy doing it) has a
psychological affect of distancing the learner from the teacher. The speaker is often elevated to the
status of "A Special Teacher Called of God." Actually, in my case, the speaker is just someone who
likes to blab about what he knows. I'm no different than anyone else, and that is important. People
need to realize that they are called by God to study and show themselves approved (2 Tim. 2:15).
Furthermore this "effect" tends to make people think that they can't be good apologists since they
aren't up there speaking. This is not true and I always try and motivate people to study and master
those areas that the Lord calls them to study.
Apologetics is the attempt to make a defense for the Christian faith. If you do that in any way,
then you are an apologist. In fact, you are commanded to be an apologist by Peter: "but sanctify
Christ as Lord in your hearts, always being ready to make a defense to everyone who asks you to give
an account for the hope that is in you, yet with gentleness and reverence," "but sanctify Christ as Lord
in your hearts, always being ready to make a defense to everyone who asks you to give an account for
the hope that is in you, yet with gentleness and reverence," (1 Pet. 3:15).
If God commands you to make a defense, then He is commanding you to be an apologist. So, you
are, whether you like it or not, called to be an apologist. But don't worry. God is not in the habit of
sending people to accomplish His will without equipping them.
Now, what I am going to tell you is true. I've experienced this many times. But please understand
that this is the work of the Holy Spirit, not me.
There would be times in varying situations when I would be discussing something with an
unbeliever. He would ask a difficult question that would give me pause. I'd nod, trust God, and inhale
to begin to answer. The answer would come as I began to speak. In other words, I didn't know what
to say, until I started to say it. I am reminded of Jesus' words in Mark 13:11, "And when they arrest
you and deliver you up, do not be anxious beforehand about what you are to say, but say whatever is
given you in that hour; for it is not you who speak, but it is the Holy Spirit."
I say this because I want you to learn to trust the Lord. He said He will be with us always to the
end (Matt. 28:20). I believe it. He is there when we need Him. So, you need to study and be ready
and trust the Lord to provide what you need when you need it. Trust God and Go!
Logic in Apologetics
Logic is typically very important in apologetics. To defend the faith, the Christian must use truth,
facts, and reason appropriately and prayerfully. The Christian should listen to objections and make
cogent and rational comments in direct response to the issues raised.
Logic is simply a tool in the arsenal of Christian apologetics. Logic is a system of reasoning. It is the
principles of proper thinking used to arrive at correct conclusions. Of course, some people are better at
thinking logically than others and there is no guarantee that using logic to the best of one's ability will
bring conversion of anyone. After all, logic is not what saves a person. Jesus does that and we are
justified by faith (Rom. 5:1).
Therefore, the proper use of logic in apologetics is to remove intellectual barriers that hinder a
person from accepting Jesus as Savior. Logic is not to be looked at as the answer to every problem
facing Christianity or every objection raised. Logic has its limits. It cannot guarantee wisdom. It
cannot prove or disprove inspiration or love. It cannot replace the intuition gained through
experience, the prompting of the Holy Spirit, nor the clear truth of God's word. Nevertheless, logic is
still very valuable and can be quite powerfully used by people both the saved and unsaved.

Opponents of Christianity use logic

Sometimes, an opponent of Christianity might use logic problems as a type of evidence against
God’s existence. Consider this rather basic objection:

• Proposition: God can do all things.


• Statement: Can God make something so big he cannot pick it up? If He can, then
he cannot do all things because he could not pick up the rock. If He cannot, then He
cannot do all things because he cannot make a rock so big He can’t pick it up.
• Conclusion: Since God can do all things and we have shown that there are things
he cannot do, therefore, God does not exist.

On the surface, this logic could be difficult to answer. But, all we have to do is think a bit more and
we can see that the problem asserted above is not logical to begin with. Here's the answer:

• Proposition: God cannot violate His own nature; that is, He cannot go against
what He naturally is.
• Statement: God's nature does not permit Him to lie, to not be God, etc.
• Conclusion: Therefore, the statement that God can do all things, is not true and
the conclusion raised against God is also not true.

Logic is a valuable tool in witnessing, particularly when using proofs of God's existence. Consider
the following basic approach using logic:

1. The universe exists


2. The universe cannot be infinitely old because if it were, it would have entered into a
state of entropy long ago.
A. Entropy is the second Law of thermodynamics that states that all things are
moving toward chaos and no-usable energy. In other words, everything is running down.
3. The universe is not in a state of entropy; therefore it is not infinitely old.
4. Since the universe is not infinitely old, it had a beginning.
5. The universe could not have brought itself into existence.
6. Something before the universe and greater than the universe had to bring the
universe into existence.
7. That something is God.
I suggest getting books on introduction to logic and go through what you can. Absorb as much as
possible. Also, learn to ask questions in discussions. Learn to think about what the ramifications are
of what people are saying. Look for logic flaws in their speech and your own.

Is logic a common ground between the believer and the unbeliever?

Some state that there is no common ground between the believer and the unbeliever, that the
unbeliever's initial presuppositions against the Christian God do not allow him to accurately reason
concerning God, the world, truth, or themselves. Therefore, some Christian theologians conclude,
there can be no ultimate common ground because the unsaved are unregenerate and their
presuppositions are opposed to true rationality. I believe that logic is indeed a type of common
ground. But I do not believe that it possess some innate quality that renders it above human capacity
or limitations, nor is it possessing of any ethereal, mystic qualities that somehow transcends the
blinding influence of sin. I think that logic, used properly, always vindicates the truths found in the
Bible and point to God – whether or not an unbeliever acknowledges it.
Logic belongs to God. This is so because God has invented the universe, the physical laws,
mathematics, and all other natural and true phenomenon in it. Existence has an order because God
gave it order. Logic is true, not because it is logical, but because it is a reflection of God's nature which
is order and truth. Therefore, logic, ultimately, belongs only to God and can only properly be used by
Him and, in matters pertaining to God, by the Christian.
This is not to say that an unbeliever cannot master the logic, say of mathematics, better than an
unbeliever. There are areas of knowledge common to both and God has given some people abilities not
possessed by others. However, this not an assertion that all Christians, when speaking of God, do so
flawlessly. Many Christians are very illogical when they try and defend God.
The fact is that no one can claim to have ultimately mastered logic. In a perfect world with unfallen
people, reasoning would be a marvelous adventure that would lead us to more of God's revelation and
truth. But, we don't live in a perfect world. We live in a fallen world where sin has influence not only
our bodies, emotions, and wills, but also our minds.

Is logic enough?

Is logic enough for the Christian? No, it isn't. Logic has two major flaws: First, it is only as good as
the one who is using it (though that really isn’t a flaw in logic). Second, logic doesn't save. Jesus does.
We cannot reason someone into the kingdom of God. It is the Holy Spirit who convicts of sin and
righteousness and who opens the heart to understand the truth (John 16:8).
But if that is true, then should we even bother to try and reason with unbelievers? Absolutely yes.
For several reasons:

• We are commanded by God to give an answer to unbelievers (1 Pet. 3:15) and to


reason (Isaiah 1:18).
• God can, in His sovereignty, use our witness and reasoning to bring someone into
the Kingdom. He is not limited by our inadequacies.
• Answers that are in agreement with God's word, given to unbelievers, even if
they are rejected, are still true answers. The unbeliever will be held accountable on
judgment day for rejecting those truths.

Conclusion

Logic is a tool for the Christian. It is nothing to be afraid of. In fact, if you accept the truth that
logic "belongs" to God, then you should be encouraged to use it. But, don’t let it become an idol; that
is, it is not the answer to problem. As Christians, we need to use logic, as well as evidence, prayer,
God's word, love, kindness, etc., in our efforts to win people to Jesus. Reasoning has a valuable place
in apologetics and with the believer. It is worth doing well. But, use it with love, prayer, and patience.
Prayer in Apologetics

One of the dangers of the apologist is falling into the trap of relying on his own intellectual abilities
to try and wrestle someone into the kingdom of God. I am sad to say that I have been guilty of this.
Pride hides itself in the heart so it cannot be seen. When we find ourselves relying on our
knowledge instead of God's word, mercy, and grace, then we have fallen into that trap. It is not
reason that converts, but God's Spirit. It is not logic that draws us to God but Jesus (John 12:32). It
is not evidence that convicts a person of his sins, but the Holy Spirit ( John 16:8). That is why we need
to rely on God and trust that He will use our defense of the truth for His glory and their benefit.
To ignore prayer in apologetics is to be prideful. It is the same as saying we don't need God. But
we do. We need to pray for those who are lost, pray for their minds to be opened, pray that God's
word will ring true to them, pray that our witness will be strong, and pray that the evil one will not
have a foot-hold with them or with us. We are fighting a spiritual battle and need spiritual tools.
Prayer is perhaps the most important of them all.
It is the Lord who opens the heart and mind, not you (Acts 16:14). Ask God for guidance (John
14:14). Ask for blessing in your understanding (James 1:5) and your speech (Col. 4:6). Ask the Lord
to also open their understanding to Gods word (Luke 24:45). This is what He does.
Prayer brings humility to the one praying. It admits dependence on God. If we are humble and
depended on God, we are more likely to hear His voice. Prayer means that you are seeking divine
intervention. It works power to your words. It changes your heart. It moves you closer to God.
Being a great apologist is not a badge of honor to be worn by the Christian as a demonstration of
his intellectual abilities. Rather, it is a response to the calling of God upon all Christians (1 Pet. 3:15)
that is be undertaken with love and humility: love of people and humility before God.
Never let your study and practice of apologetics replace the power, received by faith, in prayer
before the Holy Creator. Ask God to empower your words and open the hearts of those with whom
you speak.... and then study and witness to the best of your abilities.
Are there Guidelines for doing Apologetics?
Most every discipline has a set of rules and guidelines that help a person perform better. In fact,
guidelines could be produced for most any endeavor. Why should apologetics be any different?
Following are some things I have found that are very helpful in developing apologetic skills. I am
not saying that these are definitive or exhaustive in scope. Rather, these are simply the things that I
have found that have helped me. I hope they help you.
Remember, there is no method for apologetics that works in all situations. There can be no outline
approach that, if followed, will always lead a person to understanding and accepting the truth. That is
why apologetics is a combination of what you know and are. It is a fluid expression that must adapt
the obstacles in its course.
Apologetic skill is directly related to your experience and knowledge. You gain knowledge by
experiencing a situation where you defend the truth. This is "doing" apologetics. It is through this
doing that you polish what you know, discover your areas of weakness, and plan ways to improve your
abilities. You need to learn as much as you can through study, practice what you learn in real
situations, think of ways to apply what you know, mess up, and keep going. All of this is what
apologetics is and is how you get better. So, is there one single rule that will help you develop skill in
apologetics? Yes there is. Go for it! You will have success and failures.
In fact, when I teach seminars on apologetics, I can confidently state that I have probably made
more mistakes in evangelism, witnessing, apologetics, etc., than any ten people combined. My wife will
attest to that. But hey, that's okay. You don’t grow if you don’t go. Here are some guidelines.

1. Pray - It is the Lord who opens the heart and mind, not you (Acts 16:14). Ask God for
guidance (John 14:14). Ask for blessing in your understanding (James 1:5) and your speech
(Col. 4:6). Ask the Lord to also open their understanding to Gods word (Luke 24:45).
2. Memorize Scripture - Few things are as powerful when defending the faith as being able to
cite chapter and verse of a particular verse (Psalm 119:11; 2 Tim. 3:16).
3. Memorize the locations of information whether it be in cult material, secular material, or
any other source you’ve got. - It is extremely valuable to know material in different
disciplines. Of course, you cannot know everything, but you can memorize a few pertinent
facts about Mormonism, or evolution, or philosophy, or the Bible, or whatever else may be
needed. You will learn what you need as you witness.
4. Listen to what is being said to you - and respond to what is said. It is by listening that
you will then know what to say. Listen for errors in logic. Listen for motives, for hurts, for
intent. Listen.
5. Don’t interrupt - This is just common courteously. You need to earn the right to speak. Just
because you have an answer doesn’t mean it must be heard right away. When interruptions
become the norm, learning is thrown out the window.
6. Don’t be afraid to make mistakes - One of the best ways to improve, is to discover your
weaknesses. The best way to discover your weaknesses is when mistakes uncover them for
you.
7. Study what you discover you don’t know - If you don’t know something, then study it.
Get books and read. Write down what you learn.
8. Don’t be afraid to take a chance - This takes real faith. All you have to do is be available,
speak up, and take a chance in defending the Christian faith. You’ll be surprised at how well
you do. And when you mess up, don’t worry, review guideline # 6.
9. Rehearse - Perhaps the best place to do apologetics is in your head. Think of a situation, a
scenario that you need to have an answer for, and develop an answer. Practice in your mind.
Try and corner yourself and then get out of it.
10. Read Books that deal with what you need to know - The knowledge of others is
invaluable. Isaac Newton said, "If I have reached the stars, it is because I have stood on the
shoulders of giants." In other words, he learned from others.
Basically, the guidelines are common sense. All you have to do is try, don’t worry about failure,
keep going, pray, and trust God. It works.

Classical Apologetics
Classical Apologetics is that style of Christian defense that stresses rational arguments for the
existence of God and uses evidence to substantiate biblical claims and miracles. It is quite similar to
evidential apologetics and appeals to human reason and evidence. Early Classical Apologists include
Augustine, Anselm, and Thomas Acquinus. Contemporary classical apologists are Norman Geisler,
William Craig, and J. P. Moreland.
Some of the arguments relied upon for proofs of God's existence are the cosmological argument and
the teleological argument. The cosmological argument attempts to prove God exists by stating that
there has to be an uncaused cause of all things. That uncaused cause is God. The teleological
argument uses the analogy of design; that is, the universe and life exhibit marks of design. Therefore,
there must be a Designer. Other times, strict evidence is used establish Christianity's validity. Of
course, both aspects are also combined in classical apologetics.
An example of the latter might be as follows:

Allen: I do not believe Jesus rose from the dead?


Paul: Why not? The eyewitnesses stated that they saw him after his resurrection?
Allen: Sorry, I can't accept that. The Bible has been rewritten so many times it cannot be trusted.
Paul: But the manuscript evidence for the New Testament shows us that the manuscripts have been
faithfully and accurately transmitted to us. We can trust the documents and we can trust that the
Bible is reliable.
Allen: What manuscript evidence?
Paul: The fact that over 5000 supporting Greek manuscripts of the New Testament have been
discovered, when examined, shows us that they are all copied with extreme accuracy. The New
Testament alone is over 99% textually pure. In addition, another 20,000 manuscripts in other
languages also have been discovered. All these manuscripts range from the second century to after
the turn of the first millennium. They all demonstrate an amazing accuracy and consistency within the
copies. Therefore, we can trust that the New Testament which has been transmitted to us as
accurate. Since we can trust the documents, we have reliable eyewitness accounts of the resurrection
accurately recorded and transmitted to us.
Allen: Even if the New Testament is reliable, I still cannot believe Jesus rose from the dead.
Paul: But, if it is reliable and it accurately records eyewitness accounts of Jesus' resurrection, then
why won't you believe the witness? If Jesus did indeed rise from the dead, then what He said is true.

The preceding very simplistic dialogue has strengths and weaknesses but it demonstrates a way of
using evidence and logic as a defense to support the resurrection, a biblical miracle.
A variation on this could focus on prophecies and be as follows:

1. The Bible claims to be the word of God.


2. The Bible has been accurately transmitted to us through the copying method.
3. The Old Testament was written before the New Testament.
4. The Old Testament contains prophecies of Jesus fulfilled in the New Testament.
5. Jesus fulfilled the prophecies
6. This shows that the Bible is inspired.
7. Since it is inspired, it is accurate.
8. It says that God exists
9. Therefore, God exist.
No argument is without strengths and weaknesses and all Classical Apologetic approaches have
been tackled by critics. But, the critics are not left unanswered and Christians have, in turn, refuted
the refutations. This back and forth process of point-counter-point is going to continue until Jesus
returns. Nevertheless, God commands that we do our best to defend the faith and classical
apologetics is one of the means to do that.
Much of the information here on CARM can be used in a classical defense. There is documentation
for biblical manuscript evidence in the Bible section. There is a list of prophecies about Jesus also in
the Bible section and more. I recommend you go to the Apologetics Dialogues section and read a few
of them to see how different subjects can be used. If you want logical approaches, try some proofs for
God in the Atheist section. Finally, if you really want to test yourself, get on the internet, find a chat
room through AOL instant messenger or Yahoo instant messenger and go in and debate with people in
religious discussion rooms. You will learn real fast what you need to know.
Whichever you do, think of apologetics as a mosaic of skills and knowledge that God uses in the
believer to bring truth to the world. At first it is not that easy to do, but it gets easier and easier the
more you do it.
Adherents to this position have been Augustine, Anselm, Aquinas, R. C. Sproul, and Norman
Geisler.
Presuppositional Apologetics
This form of Christian apologetics deals with presuppositions. 28 A Christian presuppositionalist
presupposes God's existence and argues from that perspective to show the validity of Christian
theism.29 This position also presupposes the truth of the Christian Scriptures and relies on the validity
and power of the gospel to change lives (Rom. 1:16). From the scriptures we see that the unbeliever is
sinful in his mind (Rom. 1:18-32) and unable to understand spiritual things (1 Cor. 2:14). This means
that no matter how convincing the evidence or good the logic, an unbeliever cannot come to the faith
because his fallen nature will distort how he perceives the truth. The only thing that can ultimately
change him is regeneration. To this end, the presuppositionalist seeks to change a person's
presuppositions to be in conformity with biblical revelation.
I have found that a person's presuppositions are extremely important when discussing God and the
validity of Christianity. I always ask diagnostic questions to find out where a person is philosophically
and presuppositionally so I might better discuss Christianity. This is a very important point to focus on
because one's presuppositions will govern how one interprets facts. Please consider the following
dialogue as a realistic example of how this is works.

Allen: I am an atheist and evolutionist. Prove to me there is a God.


Paul: I do not think I can with your presuppositions.
Allen: Why not?
Paul: Because your presuppositions will not allow you to examine without bias the evidence that I
present to you for God's existence.
Allen: That is because there is no evidence for God's existence.
Paul: See? There you go. You just confirmed what I was stating.
Allen: How so?
Paul: Your presupposition is that there is no God, therefore, no matter what I might present to you to
show His existence, you must interpret it in a manner consistent with your presupposition; namely,
that there is no God. If I were to have a video tape of God coming down from heaven, you'd say it
was a special effect. If I had a thousand eye-witnesses saying they saw Him, you'd say it was mass-
hysteria. If I had Old Testament prophecies fulfilled in the New Testament, you'd say they were
forged, dated in correctly, or not real prophecies. So, I cannot prove anything to you since your
presupposition won't allow it. It is limited.
Allen: It is not limited.
Paul: Yes it is. Your presupposition cannot allow you rightly determine God's existence from evidence
-- providing that there were factual proof of His existence. Don't you see? If I DID have
incontrovertible proof, your presupposition would force you to interpret the facts consistently with your
presupposition and you would not be able to see the proof.
Allen: I see your point, but I am open to being persuaded, if you can.
Paul: Then, I must ask you, what kind of evidence would you accept that would prove God's
existence? I must see what your presuppositions are and work either with them or against them.

Presuppositional apologetics differs from Classical apologetics "in that presuppositional apologetics
rejects the validity of traditional proofs for the existence of God." 30 A pure presuppositionalist tackles
the worldview of a person and seeks to change the very foundation of how a person perceives facts.
Adherents to this position have been Cornelius Van Til, Abraham Kuyper, Greg Bahsen, John
Frame, etc.

28
A presupposition is an assumption that is taken for granted.
29
Theism is the belief that God exists and is involved in the world.
30
Geisler, Baker's Encyclopedia of Christian Apologetics, page 607.
Evidential Apologetics

Evidential Apologetics is that style of Christian defense that stresses the miracles found in the
Bible, particularly of Christ, as an evidence for the existence of God and, of course, the validity of
Christ and His words. It also uses historical evidences to support the veracity of the biblical
account(s). In this, it is very similar to Classical Apologetics which stresses reason in its approach to
evidences. Basically, evidential apologetics stresses evidence such as miracles, fulfilled prophecies,
etc. and uses reason to support them.
An example of evidential apologetics might be as follows (Note the similar argument to the classical
approach):

Allen: How do I know God exists?


Paul: One of the ways can be found in the gospel accounts where Jesus performed many miracles like
walking on water, healing the sick, etc. and then finally rising from the dead. No mere man can do
those kinds of things. There had to be something supernatural at work. Why can't that be God?
Allen: But the Bible is full of myths. It is just a bunch of stories.
Paul: Actually, they are not just myths and stories. The gospels, for example, were written by those
who either knew Jesus personally, or were under the direction of those who did. The gospels are full
of factual accounts of cities, customs, terms, locations, etc., that can all be verified historically and
archaeologically. There are many books that have verified the authenticity of the gospel accounts.
Allen: If that is true, then I am sure the gospels have been corrupted over time.
Paul: Actually, that isn't quite accurate. You see, the New Testament alone has something like
24,000 supporting biblical manuscripts and they are around 99.5% textually pure. That means that
they have been reliably transmitted to us through the centuries. We can trust them.
Allen: Still, I can't believe all those miracles and stuff.
Paul: Why not? Many eyewitnesses wrote and spoke aobut what they saw Jesus do. After the gospel
accounts were written, there were plenty of people around who had seen Jesus who could have spoken
up or written something down contradicting what the apostles wrote. But, we have no account of this
happening.
Allen: I didn't think of that.
Paul: Furthermore, since the eyewitnesses wrote about what they saw, and they saw miracles, as did
hundreds of others, and Jesus healed people, walked on water, calmed a storm by a command, and
rose from the dead, then whatever He says must be true since He backed up His words with His deeds.
Allen: That makes sense, but that doesn't mean there is a God.
Paul: True, it doesn't require that a God exist, but since Jesus spoke about God, about the need to be
right with God, etc., and since He performed many miracles including rising from the dead, then it is
safe to say that not only is there a God, but that we should listen to Jesus. This would also mean that
the Bible is the inspired word of God.
Allen: I'll have to think about what you said.

Generally, the evidential apologetics stresses data that supports the miraculous evidences of the
biblical accounts thereby authenticating the Bible and the claims and deeds of Jesus.
Adherents to this position have been B. B. Warfield, John Warwick Montgomery, Clark Pinnock, etc.
The Cosmological Argument
The Cosmological Argument attempts to prove that God exists by showing that there cannot be an
infinite number of regressions of causes to things that exist. It states that there must be a final
uncaused-cause of all things. This uncaused-cause is asserted to be God.
The Cosmological Argument takes several forms but is basically represented below.

Cosmological Argument

1. Things exist.
2. It is possible for those things to not exist.
3. Whatever has the possibility of non existence, yet exists, has been caused to exist.
A. Something cannot bring itself into existence since it must exist to bring itself into
existence which is illogical.
4. There cannot be an infinite number of causes to bring something into existence.
A. Because an infinite regression of causes ultimately has no initial cause which means
there is no cause of existence.
B. Since the universe exists, it must have a cause.
5. Therefore, there must be an uncaused cause of all things.
6. The uncaused cause must be God.

Thomas Aquinas (1224-1274) had a version of the Cosmological Argument called the Argument
from Motion. He stated that things in motion could not have brought themselves into motion but must
be caused to move. There cannot be an infinite regression of movers. Therefore, there must be an
Unmoved Mover. This Unmoved Mover is God.

Strengths of the argument

The strengths of the Cosmological Argument lie in both its simplicity and easily comprehensible
concept that there cannot be an infinite number of causes to an event. Some arguments for God's
existence require more thought and training in terms and concepts, but this argument is basic and
simple. Also, it is perfectly logical to assert that objects do not bring themselves into existence and
must, therefore, have causes.

Weaknesses of the argument

One of the weaknesses of the argument is that if all things need a cause to exist, then God Himself
must also, by definition, need a cause to exist. But this only pushes causation back and implies that
there must be an infinite number of causes which cannot be. This is paradoxical.
Also, by definition, God is uncaused.

___________________
Sources:

• Apologetics to the Glory of God, by John Frame, P&R Publishing, Phillipsburg, New Jersey, 1994.
• Baker Encyclopedia of Christian Apologetics, by Norman Geisler, Baker Book House, Grand
Rapids, MI, 1999.
• Christian Apologetics, by Norman Geisler, Baker Book House, Grand Rapids, MI, 1976.
• Dictionary of Philosophy, Edited by Dagobert D. Runes, Philisophical Library, New York, 1942.
• The New International Dictionary of the Christian Church, ed. J. D. Douglas, Zondervan, Grand
Rapids, MI, 1978.
The Teleological Argument
The teleological argument is also known as the argument from design. Quite simply, it states that
a designer must exist since the universe and living things exhibit marks of design in their order,
consistency, unity, and pattern.
A typical analogy of this is the Watchmaker which was given by William Paley (1743-1805). The
argument goes as follows. If you found a watch in an empty field, you would logically conclude that it
was designed and not the product of random formation. Likewise, when we look at life and the
universe, it is natural to conclude there is a designer see we see how perfectly the universe and life
forms operate. The eye is typically used as an example of design. It is a marvelous development. In
order for it to work there must be many different convergent parts that individually have no function
but have value only in a designed whole. It is only in the combined total do they exhibit their total
function. This function is by design.
Paley's argument is as follows:

1. Human artifacts are products of intelligent design.


2. The universe resembles human artifacts.
3. Therefore the universe is a product of intelligent design.
4. But the universe is complex and gigantic, in comparison to human artifacts.
5. Therefore, there probably is a powerful and vastly intelligent designer who created the
universe.

Strengths of the argument

This argument is simple to understand and has merit since humans are designers by nature and it is
natural to think in terms of things having purpose. It is also consistent with Rom. 1:20,

For since the creation of the world His invisible attributes, His eternal
power and divine nature, have been clearly seen, being understood
through what has been made, so that they are without excuse.

I think the teleological argument carries weight because it is consistent with Scripture. The Bible
states that we are made in God's image. Therefore, there are certain things that we will resonate to.
Even though the unbeliever suppresses the truth of God in his unrighteousness (Rom. 1:18-32), the
truth is still there.
Additionally, evolutionists have difficulty accounting for apparent design in objects like the eye, the
heart, and the brain where many different parts come together to form the whole. These individual
parts have no purpose except in the function of the whole. How can evolution account for these
detailed congruent occurrences? So far, it can't.

Weaknesses of the argument

The idea that the universe is designed is subjective. Different observations in the the natural world
can produce different theories to account for their existence. Also, this proof is built upon analogy. If
we find things in the universe that are chaotic, then by analogy, that would imply there is no designer.

___________________
Sources:

• Baker Encyclopedia of Christian Apologetics, by Norman Geisler, Baker Book House, Grand
Rapids, MI, 1999.
• Christian Apologetics, by Norman Geisler, Baker Book House, Grand Rapids, MI, 1976.
• The New International Dictionary of the Christian Church, ed. J. D. Douglas, Zondervan, Grand
Rapids, MI, 1978.
Apologetics Bibliography
For further reading on apologetics and areas related to it, please consider the following books
which I have used in researching apologetics for this section.

• Bush, L. Russ, ed., Classical Readings in Christian Apologetics. A.D. 100-1800. Grand Rapids,
Michigan: Academie Books. 1983.
• Douglas, J. D., The New International Dictionary of the Christian Church, Douglas, Zondervan,
Grand Rapids, MI, 1978.
• Frame, John. Apologetics to the Glory of God. Phillipsburg, New Jersey: P&R Publishing, 1994.
• -----, The Doctrine of the Knowledge of God, Phillipsburg, New Jersey: P&R Publishing, 1987.
• Geisler, Norman. Christian Apologetics. Grand Rapids, Michigan: Baker Book House, 1976.
• -----, Baker Encyclopedia of Christian Apologetics. Grand Rapids, MI. Baker Books; 1999.
• Grudem, Wayne, Systematic Theology: An Introduction to Biblical Doctrine, Zondervan
Publishing House, Grand Rapids, MI, 1994.
• Harrison, E. F. ed., Bakers' Dictionary of Theology, Baker Book House, Grand Rapids, MI, 1960.
• Lewis, C. S. The Problem of Pain. New York: Simon & Schuster. 1996.
• -----, Mere Christianity. New York. Macmillan Publishing Co. Inc. 1960.
• McDowell, Josh. Evidence that demands a Verdict. San Bernardino, CA. Here's Life Publishers,
Inc. 1979
• -----, More Evidence that demands a Verdict. San Bernardino, CA. Here's Life Publishers,
Inc. 1981
• McDowell, Josh & Stewart, Don. Answers to Tough Questions skeptics ask about the Christian
Faith. San Bernardino, CA. Here's Life Publishers, Inc. 1983.
• Runes, Dagobert, D., ed., Dictionary of Philosophy, Philisophical Library, New York, 1942.
• Van Till, Cornelius. Christian Apologetics. Phillipsburg, New Jersey: Presbyterian and
Reformed Publishing, Co. 1976.
Apologetics Dialogues
Introduction

These dialogues are from various sources: IRC Chat, ICQ, and AOL Instant Messenger.
For those of you who are unfamiliar with them, these are places where you can have live dialogues
with people by typing back and forth. Some of the dialogues were one-on-one discussions and others
were in public forums. I have edited them down for grammar, syntax, and continuity, yet have
retained the original dialogues.
One of the disadvantages to publishing one’s own dialogues in apologetics is the problem of
remaining unbiased. These dialogues represent, in part, my ability to defend the Christian faith.
Therefore, I want to make myself look good. Realizing this danger, I have sought to remain unbiased
-- though I admit I am grossly unable to do this in selecting which dialogues to post. Nevertheless, I
will produce dialogues, as they occur, and plan to include those where I fail in my goal to defend the
faith. The reason is simple: we can learn from failures as well as successes.
As I read through some of them, I see areas where I could improve. I also see areas where logic
was not that good, or points were missed, or plain old mistakes occurred. But that’s okay. No one
dialogues perfectly and it is easy to go back over something later and pick it apart. Nevertheless, for
those of you who are interested in reviewing real dialogues, I present this section as an aid to see how
it's done.
I hope they help.

1. What scripture is used to support the physical resurrection of Christ? p. 18


2. What verses can you use to support Jesus' divinity? pp. 18-19.
3. Why is Luke 24:39 important in the discussion about God having a body? p. 22
4. Why is baptism not necessary for salvation? pp. 25-28
5. Was Jesus' resurrection literal of symbolic? p. 29
6. Is the trinity illogical? p. 33
7. Is it right to tell people that what they believe in is false? Why or why not? pp. 38-40
8. Does Christianity have any evidence supporting it? What is it? pp. 46-48
9. Is evolution a fact? p. 54
10. What does it mean to have a relationship with Jesus? p. 57
11. What is the problem with the Muslim form of salvation? pp. 59-61
12. Why is Luke 22:42 important in a discussion with Oneness people? pp. 62-64
13. What is the problem with an atheist saying there is no evidence for God? p. 66
14. What is the problem with an atheist saying he knows there is no God? p. 67
15. What is the basic argument around using logical absolutes as a proof for God's existence? pp.
74-75
16. What are some points in arguing against a Catholic about interpreting the Bible? pp. 79-82
17. What are the mistakes the Mormons make in their definitions? pp. 83
18. What did the Mormon misunderstand about salvation and works? pp 92-94-100-102
19. Why is it that Joseph Smith did not see God the Father? pp. 95-96
Jesus' Resurrected Body, the Atonement, and Islam
This conversation dealt with the resurrected body of Christ, the atonement, and a bit on what
Muslim theology has to offer. It is brief, but it has value in the discussion of Christ.

Gil: Hey Matt


Matt: Hi
Gil: Can I ask a question?
Matt: You just did
Gil: How about another? A third one I mean.
Gil: Heheh
Matt: Now you've got it. What's up?
Gil: Okay, Well actually, I've got one specific question: Is Jesus eternally human?
Matt: He is now.
Gil: Okay. So, you believe that He is human?
Matt: Absolutely...!
Gil: As he sits next to the Father? and the Father is spirit….and the Spirit is spirit
Matt: Yes...
Gil: But the Son is human and spirit?
Matt: The son is both God and man. He has two 2 natures.
Gil: I personally believe that he was man for 33 years because in the Bible it says that God is Spirit
and not a man.
Matt: Do you go to church? If so, which one?
Gil: Reformed Church of America. My belief is unorthodox.
Matt: Okay... Bible lesson time. After the resurrection, Jesus retained the scars, right?
Gil: Yes.
Matt: He had a body of flesh and bones after the resurrection, right?
Gil: Yes, but he also walked through walls and stuff. Right? He had a supernatural body.
Matt: He did not walk through walls. He just appeared....
Gil: Well that's not a human behavior. He didn't do that before he resurrected.
Matt: It is the behavior of a resurrected body... We do not have resurrected bodies right now.
Gil: I know
Gil: but I'm saying I don't believe the resurrection body is a human body. It's a supernatural spirit
body.
Matt: If it isn't human, then what is it?
Gil: Spirit
Matt: 1 Cor. 15 says it is a resurrected body... Flesh and bones...
Gil: hmmm. Okay.
Matt: Jesus was recognized and he retained the scars of his crucifixion.
Matt: Also, consider this... Jesus prophesied that he would raise His body, the very same one that was
crucified (John 2:19-21).
Matt: Therefore, his same body was raised....
Gil: hmmm
Gil: What about when He ascended?
Matt: Same body.
Matt: If you deny his physical resurrection, you are in trouble.
Gil: But the bible says that God is Spirit not man.
Gil: I believe that he physically resurrected.
Matt: The Father is a spirit. John 4:24 speaks of God as the Father... Jesus, rose in the same body he
died in, right?
Gil: How do you know it meant the Father?
Matt: Because that is how John refers to God in his gospel. God is the Father. The word is Jesus. Ref.
John 1:1,14, 18; 6:46 for the pattern of usage.
Gil: Okay, Can I be blunt with you?
Matt: Sure.
Gil: I used to be really unsure about the trinity and how it's all set up. I read James White's Forgotten
Trinity which helped me tons...
Matt: good
Gil: But now its like it all seems like weird to me again. Two of my best friends are Muslim. And its
soooo appealing to me.
Matt: How are your sins forgiven?
Gil: Through the blood of Jesus and God's grace.
Matt: Do the Muslims have an atonement for your sins to offer you?
Gil: Yes, the mercy of Allah.
Matt: Mercy is not atonement.
Gil: Um okay.
Matt: Atonement removes sin (1 John 2:2; Heb. 9:26).
Gil: His mercy provides atonement.
Matt: Atonement means to remove, to cover the sin... in Christianity, it is by the sacrifice of God
Himself. This guarantees your forgiveness.... Jesus walked the earth, rose from the dead, walked on
water, etc.
Gil: I agree
Matt: He said He was God in flesh... He atoned..... Can Islam compare to that?
Gil: When did He say he was God in flesh? (Just so I can tell my Muslims friends the verse.)
Matt: John 8:58 is where Jesus said, "Before Abraham was, I AM."
Gil: I know about John 8:58 and John 10:30. But He never said "I am God in flesh."
Matt: What would YOU think, if you were a Jew, knew where God said I AM in the OT [Exodus 3:14]
and then you have this guy standing in front of you (who walks on water, calms storms, etc.) and then
he says to you, "Before Abraham was, I AM." What would you think Jesus meant by this?
Gil: That He is God, which would explain why they tried to stone him. But He never said it flat out.
Matt: If He was saying He was God, and He was right there in the flesh, then what was He claiming?
Gil: Okay.
Matt: Now, do you believe that Jesus rose from the dead in the same body He died in?
Gil: Yes. But as far as the ascension goes, I don't know.
Matt: Why would his body NOT be a body at his ascension? Also, in 1 Tim. 2:5, it says that Jesus is a
man.
Gil: But the Bible says God is a spirit.
Matt: In Acts 7:55-60, Stephen has a vision and sees Jesus... how would he recognize Jesus if it
wasn't in His body?
Gil: I see your points…. Interesting.
Gil: I gotta get running I will catch you later. Matt, thanks for talking to me.
Matt: Sure.
Gil: Peace and Grace

Sometimes people just get stuck on an idea and have problems letting God's word change those
ideas. We are all guilty of that one way or another. But, at least with the resurrection of Jesus, He
rose in the same body He died in.
This is important because Paul says in 1 Cor. 15:14, "And if Christ be not risen, then is our
preaching vain, and your faith is also vain." This combined with Jesus' prophecy that He would raise
His body in John 2:19-21 is proof that believing in the bodily resurrection of Christ, is one of the
essential doctrines of Christianity.
John 2:19-21, " Jesus answered and said to them, “Destroy this temple, and in three days I will
raise it up.” 20The Jews therefore said, “It took forty-six years to build this temple, and will You raise it
up in three days?” 21But He was speaking of the temple of His body."
Discussion with a Gnostic on Jesus' Resurrection
This short dialogue is an interesting one. This gentleman retained the nickname ‘Gnostic.’
Gnosticism is/was the teaching that tried to reduce Christianity to a philosophy. The Gnostics claimed
they had secret and superior knowledge. It generally taught that matter was evil and that the God of
the Bible is a lower god called the Demiurge. The Absolute Supreme Being is unknowable. Jesus could
not be God in flesh, because matter is evil. Therefore, Jesus became the Christ when the "heavenly
Christ" came upon Him at His baptism.
This dialogue is short and it was my first encounter with a Gnostic.

Gnostic: Hi
Matt: Hi. What's up?
Gnostic: Not much...You?
Matt: Not much. Why the nickname. Why 'gnostic'?
Gnostic: I am a gnostic. And you?
Matt: I’m a Christian. What do you mean by gnostic?
Gnostic: As in the "heretical movement" of the first two centuries.
Matt: So, you deny Jesus is God in flesh?
Gnostic: That is a difficult question to answer. We believe the answer to be yes and no. He was God,
but not truly manifested in the flesh.
Matt: To say that Jesus was God but not truly manifested in the flesh, is a form of docetism, I believe.
Gnostic: Docetists said that the flesh and the divine cannot mix, therefore Jesus was truly God but
only in the form of a human being.
Gnostic: Gnosticism offered two traditional answers to the question...Adoptionist Gnostics said Jesus
was a regular man, but when he was baptized by John a divine power descended on him and he was
"adopted" as the Son of God.
Gnostic: and Valentinian Gnostics accepted the idea of Jesus being truly human and truly divine, but
argued that there were actually two persons residing in the same body, a human person and a divine
person...and when Jesus died it was only his physical nature that was crucified, while his divine person
lived on.
Matt: Both of those views are in error.
Gnostic: But according to any, no Jesus was not actually an enfleshment of God, since, as St. Paul
quite rightly tells us in 1 Corinthians, flesh and blood cannot inherit the kingdom of God, nor can the
perishable inherit the imperishable.
Matt: Flesh and blood... true, an idiom for sinful flesh. However after the resurrection, Jesus said he
had flesh and bones (Luke 24:39) He had risen from the dead in his physical body... He was god in
flesh.
Gnostic: He walked through doors...his disciples couldn't recognize him on the road to Emmaus... His
post-resurrection body was different, it shifted forms, it was not like his pre-resurrection body...even
according to the canonical gospels.
Matt: He did not walk thru doors. He simply appeared in a room.
Gnostic: Can a physical body simply translocate itself?
Matt: Jesus said he would raise his body in John 2:19-21... he retained the scars after his
resurrection. it was the same body. 1 Cor. 15 speaks of a resurrected body. It is different, yet it is the
same body, only, resurrected.
Gnostic: It also speaks of two bodies, a spiritual body and a physical body.
Matt: Yes... but take it with all of the Bible... they are the same thing... like a butterfly was once a
caterpillar... they are they same life, only transformed.
Gnostic: Yes, I would agree with you absolutely that that is what Paul is ultimately teaching. I am just
using his metaphor as a jumping-off point but I would go beyond it and say that they are two different
things.
Matt: But to do that is an error.... I mean no offense. Jesus said he'd raise the same body he died in.
He did.
Gnostic: And that resurrection can only take place when the spirit is free from the flesh, free from the
pain and the pleasures of physical existence...and that separation of spirit from flesh at the crucifixion
is how a Gnostic would describe Jesus' resurrection.
Matt: He retained the scars... it was the same body... yet he was able to do 'weird' stuff.
Gnostic: Not a resurrection of a mass of flesh and sinful temptations, but an rising of the spirit up out
of the physical nature.
Matt: That isn't what Jesus said about his own body. Why would it be different for others? He is, after
all, the first-fruits of the resurrection.
Gnostic: Yes! and just as he shed physicality and arose as a "life-giving" spirit, so will we...he is the
prototype of our "resurrection", of our ascension past the flesh.
Matt: But, he rose in the same physical body he died in. John 2:19-21 prophesied that. Jesus said it.
Gnostic: No, but Gnostics never accepted most of the canonical scriptures as actually being the word
of God.
Matt: Why not?
Gnostic: Well, that is a primary difference in our beliefs.
Matt: Canonicity is another subject.
Gnostic: The life that he gives us is the capacity to move beyond the temptations and pleasures and
pain of mortal existence.
Matt: Wait a second, please.
Gnostic: Sure.
Matt: How do you handle Jesus' own words that he would raise His own body from the dead? He even
told Thomas to stick his finger in His hands and into His side... He still had the wounds... It was his
body, the same one.
Gnostic: I don't believe Jesus uttered those words. I find the statement blasphemous, just as you
would not believe that Jesus said some of the things written down in Gnostic gospels, I am sure.
Matt: It is in the Bible recorded by those who knew him. So, then, you're accusing John the apostle of
being a liar?
Gnostic: sorry I'm going to have to run. Take care, god bless.

I was surprised by the abrupt termination of this conversation. But that is often the nature of
discussions on the Internet.
Gnostic’s weak areas were his inconsistency with believing what Jesus said, casting doubt on God’s
word, and not dealing with the issue of Jesus’ physical resurrection.
My weak area was not knowing more about Gnosticism. I was familiar with it, but not very
knowledgeable. Had I known more, perhaps the conversation would have been better. By the way, the
intro to this dialogue is easy to write after the conversation – as I did my research afterwards.
Does God have a Body?
This dialogue touched on issues of God's nature and physical manifestation. I've dealt with this
before with many cult groups so I was able to recall a lot of scriptures and insert them into the
dialogue as we were talking. Memorizing verses sure does have its advantages.

Tony: Hey Matt!


Matt: hi
Tony: Are you busy?
Matt: I've got about 5 minutes... what's up?
Tony: Love your site!
Matt: thanks... it's a lot of work...
Tony: Yes I can see!!!
Matt: Shoot
Tony: Do you think God has a body? Or is he Spirit only like it says in john (I think).
Matt: The Bible says that god is spirit (John 4:24). In Luke 24:39 Jesus says that a spirit does NOT
have flesh and bones. We do not know what spirit is, but we do know what it is not.
Matt: Also, God is everywhere. If he had a body, He could not be everywhere.... A body would mean
that he has material, Matter, etc. Time is a function of the existence of Matter. Since God is outside of
time, eternal, He could not be material. Therefore, He cannot have a body because he could not them
be omnipresent.
Tony: What about Steven when he looked up and saw Jesus sitting on the right hand of the Father...?
Matt: It was a vision.
Tony: Hmmmm
Matt: Was Jesus LITERALLY sitting ON God's right hand?
Tony: God gave him a vision of himself........?
Matt: yep.
Tony: Why did God do that?
Matt: Not sure really. Good question. But God reveals Himself to those who seek Him. With Stephen
maybe it was because he was the first Christian martyr.
Matt: Now, let me ask this again. Was Jesus LITERALLY sitting ON God's right hand?
Tony: I think so. The disciples wanted to sit on his right hand.
Matt: Then does that mean that God could not use His right hand because someone was sitting on it?
Tony: Ha Ha I though you were trying to say that but wasn't sure…
Matt: What does it mean to sit ON God's right hand?
Tony: Power?
Matt: Yes, and authority.
Matt: If you someone says that God has a body, then does He have genitalia too?
Tony: If a body then I guess so....but didn't God wrestle with Abraham in the Old Testament? Or was
that just an angel of the Lord?
Matt: An angel of the Lord. Angels often manifested human form (Gen. 18:1 and following is a good
example)
Matt: God is invisible and dwells in unapproachable light ( 1 Tim. 6:16-17).
Tony: I'll read it......
Matt: Are you a Mormon?
Tony: No, I'm born again since 1980 and love the Lord!! I believe in the Trinity.
Tony: Jesus said if you seen me you seen the Father.
Matt: Heb. 1:1-3 says that Jesus is the exact representation of God.
Tony: Yes......
Matt: He so completely represented the Father, that to see Him was to see the Father.
Tony: Heavy!! What a beautiful thing for a person to accept Jesus.......
Matt: Yes, Jesus has two natures: human and divine (Col. 2:9 Phil. 2:5-8). But God the Father, does
not.
Tony: Yes human and divine.
Matt: Did you know that God was seen in the Old Testament?
Tony: Yes I have read that.... How about with Moses
Matt: Yes, in Num. 12:6-8, God says He appears to others in visions and dreams, but not Moses.
Moses beheld God's very form.
Tony: Amen
Matt: In Exo. 24:9-11, 74 people saw the God of Israel and under his feet there was a pavement of
sapphire, etc....
Matt: God was seen in human form in the O. T., right?
Tony: Yes
Matt: But Jesus said in John 6:46 that no one has ever seen the Father.
Tony: Yes, so?
Matt: So, if God was being seen in the OT, but it was not the Father, then who was it?
Tony: Hmmmmmmm
Tony: The Son!
Matt: Yep. Preincarnate Christ.
Matt: The Father does not have a body of flesh and bones. Jesus does -- at least now...
Tony: When Jesus said no one has seen the Father, was he talking bout the people right there at that
time?
Matt: Yes and those before Him.
Tony: Oh ok...
Matt: In John 5:37 Jesus says that the Pharisees had never seen the Father... He was talking specific
to them there I believe.
Matt: Besides, in 1 Tim. 6:16-17 it says that God dwells in unapproachable light whom no mans has
seen, nor CAN see.
Matt: John 14:9 is where Jesus said to see him was to see the Father. Does that mean that the Father
has two legs, two hands, a beard, and a couple eye balls? NOT!
Tony: I believe He is spirit...and worship him and in spirit and truth
Matt: Good. John 4:24.
Matt: Does all that help?
Tony: yes it does
Tony: appreciate it..........

I thought this was a nice dialogue. There was no name-calling and it flowed pretty well. Also, it's
nice have some scriptures memorized. It comes in handy.
A Christian having doubts because of school.

I found this dialogue hidden on my computer and decided to put it up. It is about a Christian who
is having doubts because of what he is learning in school.

Sam: Hi. Do you have time to talk for a few minutes?


Matt: A few. Yes.
Sam: Okay. If you are busy please let me know. I've been looking at your web site. I am a Christian.
A college student and I have q quick question.
Matt: What?
Sam: I go to a secular university and they teach that it doesn't matter what you believe and I've even
had teachers make fun of Christ. It makes me angry and confused. What can I do to be more firm in
my faith even while all those things are being taught.
Matt: Faith is only as good as who you put it in. If it doesn't matter, then ask them if
it is okay to put your faith in satanism and practice evil.
Sam: Ok
Matt: Always ask for documentation. Ask WHY they would say what they do...
Sam: yes I see...
Matt: Do you have my notebook?
Sam: No I sure don't. I saw it advertised on teh page.
Matt: well, it's basically what is on CARM. Could you email me with some of the specific questions
they ask?
Matt: I'm thinking of writing a book on this stuff.
Sam: yes I sure will what is your address.
Matt: carmorg4@hotmail.com
Sam: Ok. I will do that.
Sam: thank you for your time. i know you must be busy. Please pray for me. My name is James.
Matt: What are you having doubts about? Can you get into my voice chat area? Do you have a
microphone and speakers?
Sam: No, sorry I don't
Sam: I'm not as close to Christ as I used to be. I first placed my faith in Him 3 years ago. All the
things I hear in college and everything doesn't seem to be strengthening my faith.
Sam: I try and follow Christ the best I can.
Matt: Well, tell me what it is that is causing you to doubt.
Sam: Several things... when I first placed my faith in Christ I didn't know much about Jesus. I just
knew I needed Him. I didn't know He was God in the flesh, and I've heard people say that....a person
cant be saved unless they know specific doctrines.
Matt: Yes and no....
Sam: I didn't know that stuff when I first placed my faith in Him.
Matt: A Christian will eventually come to accept that He is God. If a person continues
to reject that, then he is not saved.
Sam: So sometimes I doubt my salvation because I didn't know those things then.
Matt: So? It's okay. You aren't saved by doctrine, per se. You're saved by Jesus.
Sam: I believe Christ is God. Sometimes I doubt. Mostly because of things I hear. But I always come
back. Does it matter if you don't know the specific time you were saved?
Matt: It doesn't not matter at all if you know the specific time you were saved. Heck,
I can't even remember the year I got saved.
Sam: yes. I've read it so many times to strengthen my faith I can quote it by memory.
Matt: Praise God.

I don't know what happened to this dialogue. I unearthed it and this was all there was. It is a
shame there wasn't more and it equally sad that secular school is so antagonistic to Christianity.
Is Baptism Necessary for Salvation?
This conversation is a good illustration of the need to know more than basic theology when
discussing the issue of whether or not baptism is necessary for salvation.
Both Mark and myself had computer Bible programs at our disposal so we were copying text into
the dialogue. This made things easier and quicker.
Also, this dialogue belies my belief in infant baptism. I'm Reformed in theology and believe in the
covenant aspect of baptism that includes infants. I do not believe it saves infants but I believe that as
circumcision was a covenant sign in the Old Testament, baptism is the covenant sign in the New.
My desire is not to convince you or anyone about this position here in this dialogue. Rather it is to
establish the fact that baptism is not a requirement for salvation.

Mark: You're from CARM right?


Matt: Yep.
Matt: I'm Matt Slick.
Mark: That's what I thought.
Mark: What do you think of water baptism regeneration?
Matt: It is a false doctrine.
Mark: Still another meaning refers to the sacrament of baptism, which is a spiritual rather than bodily
cleansing (Matt. 28:19, Rom. 6:3, 1 Pet. 3:21).
Matt: Correct. It is important, but it is not what saves us.
Mark: event from water baptism (see John 3:5, Acts 2:38, 19:2-3, 22:16, Rom. 6:3, Col. 2:11-12,
Titus 3:5, and 1 Pet. 3:21 on the subject of baptism's graces). But something instructive can be
learned about the mode of baptism and the meaning of the word 'baptizo' even if they insist on
separating Spirit baptism from water baptism. Aren't they the same?
Matt: water baptism is a sacrament - a physical manifestation of a spiritual reality. Spirit baptism is
an anointing, possession, movement of God on a person.
Mark: Isn't it essential for salvation? "Repent, and be baptized every one of you in the name of Jesus
Christ for the forgiveness of your sins; and you shall receive the gift of the Holy Spirit ," Acts 2:38;
what about this?
Mark: Acts 19:1-6 weren't they baptized immediately in this passage of scripture?
Matt: Do you believe baptism is necessary for salvation?
Mark: I don't know.
Matt: Have you got a Bible?
Mark: Yep.
Matt: Acts 10:44, They are baptized after receiving the holy spirit, right? It was after they speak in
tongues and glorify God. Tongues is a gift for the church, for church members, for the saved… then
they were baptized.
Matt: If baptism is necessary, then that means those people were filled with the HS, glorified God,
and spoke in tongues, but they were not saved.
Matt: Does that make any sense?
Mark: How do you explain, the baptism 1st in (Acts 2:41)?
Matt: Hold on. One thing at a time. Do you see the point in Acts 10:44ff?
Mark: Luke 3:3; Luke 7:29; Acts 19:4; Rom. 6:4; Eph. 4:5 -- All refer to baptism and I read Acts
10:44 but there are other passages too, that should be looked at like Matt. 3:13. Didn't Jesus give us
an example of how we should be baptized?
Matt: First of all, were the people in Acts 10:44ff, saved before baptism?
Mark: Yes, in that particular situation, but how about all these other verses. Why absolutize that
passage and not others? Like Acts 1:5, why did John the Baptist give water baptism?
Matt: If they were saved before baptism, then baptism isn't necessary for salvation, is it?
Matt: Psalm 119:151 says that the SUM of God's law is truth.
Matt: Let's tackle Acts 2:38, okay?
Mark: Okay.
Matt: What is 'the promise' spoken of in Acts 2:39?
Mark: The promise of the HS and the LORD shall call. But you still have not answered the above
passages that are in reference to baptism.
Matt: First of all, let's stick with one thing at a time. Second, that is not it.
Mark: What do you mean?
Matt: To a Jew what would be "The Promise"?
Matt: Gen. 17:7 And I will establish My covenant between Me and you and your descendants after you
throughout their generations for an everlasting covenant, to be God to you and to your descendants
after you. And I will give to you and to your descendants after you, the land of your sojournings, all
the land of Canaan, for an everlasting possession; and I will be their God.
Matt: Gal. 3:8-14, And the Scripture, foreseeing that God would justify the Gentiles by faith, preached
the gospel beforehand to Abraham, saying, All the nations shall be blessed in you. 9So then those who
are of faith are blessed with Abraham, the believer. For as many as are of the works of the Law are
under a curse; for it is written, Cursed is everyone who does not abide by all things written in the book
of the law, to perform them. Now that no one is justified by the Law before God is evident; for, The
righteous man shall live by faith. 12However, the Law is not of faith; on the contrary, He who practices
them shall live by them 13Christ redeemed us from the curse of the Law, having become a curse for us
for it is written, Cursed is everyone who hangs on a tree 14in order that in Christ Jesus the blessing of
Abraham might come to the Gentiles, so that we might receive the promise of the Spirit through faith.
Matt: Notice that in verse 8 the promise is called the Gospel being justified by faith. Verse 14
mentions it as THE PROMISE.
Matt: Acts 7:17, But as the time of the promise was approaching which God had assured to Abraham,
the people increased and multiplied in Egypt,
Matt: Acts 13:32-33, And we preach to you the good news of the promise made to the fathers, 33 that
God has fulfilled this promise to our children in that He raised up Jesus, as it is also written in the
second Psalm, Thou art My Son; today I have begotten Thee.
Mark: Verse 39 says, "for the promise is unto you, and to your children, and to all that are afar, even
as many as the Lord our God shall call."
Matt: Yes. Very good.
Matt: The Abrahamic Promise included infants, didn't it? The Abrahamic Promise is that God would
justify the gentiles by faith.
Matt: Check out Rom. 4:13-16, For the promise to Abraham or to his descendants that he would be
heir of the world was not through the Law, but through the righteousness of faith. 14For if those who
are of the Law are heirs, faith is made void and the promise is nullified; 15for the Law brings about
wrath, but where there is no law, neither is there violation. 16For this reason it is by faith, that it might
be in accordance with grace, in order that the promise may be certain to all the descendants, not only
to those who are of the Law, but also to those who are of the faith of Abraham, who is the father of us
all.
Mark: "Rise and be baptized, and wash away your sins, calling on his name" (Acts 22:16). 1 Pet.
3:21: "Baptism . . . now saves you, not as a removal of dirt from the body but as an appeal to God for
a clear conscience, through the resurrection of Jesus Christ." Luke 18:15 says, "Now they were
bringing even infants to him" (Proseferon de auto kai ta brephe), and following this are the same
words as in Matt. 19:14. The Greek word brephe means "infants"-- could you explain these passages?
Matt: We are doing Acts 2:38. One verse at a time.
Mark: Okay.
Matt: Do you see that the Abrahamic promise is still in effect?
Mark: What you mean?
Matt: The Abrahamic promise is THE PROMISE spoken of in Acts 2:39. THE PROMISE is that God
would justify the Gentiles by Faith. That is reiterated by Paul in Gal. 3:8ff, and Rom. 4:13ff.
Matt: Acts 2:38 is referred to in Acts 2:39, THE PROMISE.
Matt: Do you remember what the sign of the Abrahamic promise was?
Mark: I don't know, then how in the world can infants be saved, they do not understand to have faith?
Matt: Were infants included in the Abrahamic promise through circumcision?
Mark: Yes.
Matt: Circumcision was the covenant sign of the Abrahamic promise, right?
Mark: To be honest I don't know.
Matt: THE promise in Acts 2:39 was meant for the children too, wasn't it?
Mark: I guess...
Matt: Gen. 17:11-12, "And you shall be circumcised in the flesh of your foreskin; and it shall be the
sign of the covenant between Me and you. 12And every male among you who is eight days old shall be
circumcised throughout your generations, a servant who is born in the house or who is bought with
money from any foreigner, who is not of your descendants."
Matt: Acts 2 is full of quotes from the OT. Peter was a Jew. He was speaking to Jews. THE PROMISE
was understood by the Jews, they knew what it was. That is why Peter said it was for their children as
well. That is why entire households were baptized. Don't households have babies in them?
Mark: Yep. You see, this is theology. The groups that teach baptismal regeneration don't know this
stuff... therefore, they make mistakes.
Matt: Acts 2:38 isn't speaking about baptism being a requirement. It is a sacramental sign.
Circumcision didn't save. Baptism doesn't either.
Mark: I can see that.
Matt: It is a covenant sign and promise for God's people and their children. That is the context.
Matt: Peter wasn't saying that baptism saves... but he was drawing their attention to the significance
of the NEW covenant sign for Christians: baptism.
Mark: What about these passages? "Rise and be baptized, and wash away your sins, calling on his
name" (Acts 22:16). 1 Pet. 3:21: "Baptism . . . now saves you, not as a removal of dirt from the body
but as an appeal to God for a clear conscience, through the resurrection of Jesus Christ." Luke 18:15
says, "Now they were bringing even infants to him" (Proseferon de auto kai ta brephe), and following
this are the same words as in Matt. 19:14.
Matt: Acts 22:16, calling on Jesus name is what saves us, not water.
Matt: 1 Pet. 3:21, let's look at it in the NASB.
Mark: Okay.
Matt: "And corresponding to that, baptism now saves you not the removal of dirt from the flesh, but
an appeal to God for a good conscience through the resurrection of Jesus Christ,"
Matt: "And corresponding to that" in the Greek, is the word, 'antitupon.' It means, antitype,
representation, etc. The question is then, "Corresponding to what?". For that we need to look at the
previous verses.
Matt: 1 Pet. 3:18-20, "For Christ also died for sins once for all, the just for the unjust, in order that
He might bring us to God, having been put to death in the flesh, but made alive in the spirit; 19in which
also He went and made proclamation to the spirits now in prison, 20who once were disobedient, when
the patience of God kept waiting in the days of Noah, during the construction of the ark, in which a
few, that is, eight persons, were brought safely through the water."
Matt: What was it that saved Noah and his family? The water or the ark?
Mark: The ark.
Matt: Did they enter the ark by faith?
Mark: Yes.
Matt: Then they were saved by faith, weren't they?
Matt: That is why it says in the rest of 1 Pet. 3:21, "...not the removal of dirt from the flesh, but an
appeal to God for a good conscience through the resurrection of Jesus Christ,"
Matt: Peter is stating that it was not the water, but an appeal to God that saves. The appeal is done
by faith, right?
Mark: It looks that way.
Matt: Then baptism is actually an appeal to God, a trust in Him by faith, a dying to ourselves (Rom. 6;
Col. 3), a public declaration of our identification with Christ.
Matt: Does that make sense?
Mark: I think.
Mark: Acts 1:5, why did John the Baptist give water baptism?
Matt: In Matt. 3:13-15 it says, "Then Jesus came from Galilee to the Jordan to be baptized by John.
But John tried to deter him, saying, "I need to be baptized by you, and do you come to me?" Jesus
replied, "Let it be so now; it is proper for us to do this to fulfill all righteousness." Jesus got baptized to
fulfill all righteousness."
Matt: He came to fulfill the Law. Quite simply, Jesus was baptized so he could enter into the
Melchizedek priesthood so He could be the High Priest and offer Himself as a sacrifice for our sins.
Mark: When believers get baptized is that to show a fulfillment of righteousness?
Matt: No. When JESUS got baptized it was to fulfill righteousness.
Matt: To be consecrated as a priest, He had to be:
- washed with water (Lev. 8:6; Exodus 29:4, Matt. 3:16).
- Anointed with oil (Lev. 8:12; Exodus 29:7; Matt. 3:16).
Both of these were bestowed upon Jesus at His baptism. Additionally, He may have needed to be
30 years old - (Num. 4:3) Jesus said in John 5:39 that the Bible was about Him.
Matt: Jesus came to fulfill the Bible prophecies and typologies.
Matt: He was baptized to enter into the priesthood (Heb. 5-7).
- Exodus 29:1 - "This is what you are to do to consecrate them, so they may serve me as priests:
Take a young bull and two rams without defect."
- Exodus 29:4 - "Then bring Aaron and his sons to the entrance to the Tent of Meeting and wash
them with water."
- Exodus 29:7 - "Take the anointing oil and anoint him by pouring it on his head."
Matt: The Holy Spirit anointed Jesus at His baptism. He was 30 years old. THAT is why He was
baptized, to fulfill the Law and to be our High Priest after the order of Melchizedek so He could be a
sacrifice for our sins.
Mark: Okay, I've got to think about this for a while.
Matt: Save the transcript and read it, okay?
Mark: okay, no problem.

Baptism is a very important ordinance for the Christian. But it is not what saves us and it is not
part of salvation. We are justified by faith (Rom. 5:1), not faith and baptism.
The Resurrection of Jesus: Literal or Symbolic?
This dialogue was initiated after I was in a Christian IRC chat room. Shirley and I were in a private
chat room, at her request. It began with her sending me a link to a porn site. Of course, I did not
look and I initiated a dialogue on why should would do something like that. Our conversation
continued for a while until I discovered she lived in Belgium and was a Catholic.
Now, I in no way want to imply that her behavior is the result of Catholic teaching. I am simply
stating the facts. She said that the Catholics there do no believe the Bible is literal. I replied that it
was. What followed is this dialogue:

Shirley: So why do you take the Bible so literally? I think it should be taken symbolically.
Matt: The Bible is accurate, truthful, prophetically precise, the manuscripts copies are 99% pure, and
it is archaeologically accurate too.
Shirley: Perhaps. But there is one mistake you made. It's IMPOSSIBLE.
Matt: Why is it impossible?
Shirley: Dead is dead.
Matt: You aren't making sense... please explain.
Shirley: A dead man can not relive his life in the same body!!!
Matt: Then you mean that Jesus did not rise from the dead?
Shirley: No He didn't!!! That's just a symbol.
Matt: Then you mean to tell me that all the people who saw him alive were mistaken?
Shirley: They were hallucinating or on drugs.
Matt: So, you are presupposing that the resurrection cannot happen. Why?
Shirley: Well, ask any scientist...
Matt: Many scientists believe it could indeed happen.
Shirley: Yeah, true. But I see it as untrue until proven otherwise.
Matt: But the biblical evidence is that the tomb was empty, that Jesus appeared, showed people his
wounds, and that at least 500 others saw him. Why can you not believe the facts of what they attested
to?
Shirley: I don't believe the Bible literally. I like to see the symbolic use of it. Like 'just be good'. Look
inside yourself to find the best in yourself.
Matt: So then, you are not basing your opinion on the facts, but simply out of a desire, a whim, to
NOT believe the biblical evidence. Is that wise? Can it lead to truth?
Shirley: I hope so, or else my life would be useless.
Matt: Very true. Then you are risking a great deal by looking within yourself. You assume too much.
You assume you have the natural ability to know truth simply by looking into yourself.
Shirley: I assume all. I try to figure everything out for myself. I don't believe other peoples’ opinions.
Matt: If you assume all, then you should also assume that the people who knew Jesus were telling the
truth when they said he rose from the dead. If that is true, then you have no logical right to deny the
resurrection and it is definitely possible. If it is possible, then you need to examine the evidence. If the
evidence is there, then Jesus rose from the dead. If he did, that is utterly significant. No one else has
done that. Therefore, what Jesus said must be true.
Shirley: But the Bible has changed.
Matt: The Bible is textually reliable and it was written by eye witnesses. Jesus said he would rise from
the dead (John 2:19-21). He did.
Shirley: He said nice things, good things. If we would all live by His word, this would be a better
world.
Matt: True. But you are not believing what He said.
Shirley: Jesus said he would rise from the dead, but so does my father... (really).
Matt: Has your father walked on water? Read the hearts of people? Calmed a storm with a command?
Fed 5000 people with a few fish? Raised others from the dead? Healed people? You see, there is more
to it than just making a claim. Jesus backed it up with what He did.
Shirley: I believe what he said when he was talking about good and bad. I do not believe him when
he said he would rise from the dead.

Matt: But if He said it and you don’t believe Him, then you are calling Jesus a liar. He is the one who
walked on water, claimed to be God (John 8:58) and said He would rise from the dead (John 2:19-21).
Shirley: I do not believe he walked on water, etc. I only listen to his words, and I extract the things
from it that I find useful.
Matt: On what basis do you do this extraction of yours?
Shirley: On the basis of what I consider to be right or wrong?
Matt: Then you set yourself to be the final judge of right and wrong. Have you raised others from the
dead, walked on water, healed people, and fulfilled prophecy? Why should I believe you over Jesus?
Shirley: You shouldn't. You should believe anything you like. You should believe anything that makes
you feel better. That’s what faith is all about.
Matt: Sorry, I disagree. That is not what faith is about. Faith is only as good as who you put it in. Are
you your own savior?
Shirley: Yes, I hope so.
Matt: I'll trust Jesus, his resurrection, His truth, etc. It was nice chatting with you.

The dialogue ended there since it was late.


As you can see from this dialogue, Shirley has no reason for her method of determining truth, let
alone determining the veracity of Jesus' claims. The resurrection was attested to by eye witnesses.
The Bible is reliable and trustworthy. For her to arbitrarily deny the veracity of Jesus' resurrection is
illogical and contrary to the evidence.
Pain can make us doubt God
Pain can make us doubt God. But we must keep our eyes on Him anyways. Of course, this is easy
to say and difficult to do.
The following dialogue isn't much of an apologetic on anything except to say that any truth is
good. Sometimes God can use us in ways we do not suspect.
As I reread this dialogue, I find myself wishing I had been more compassionate with the loss and
pain of the 'other' fellow. Perhaps this is why God allows us to experience pain, so that we might more
easily and lovingly sympathize with those who are going through it.

Sam: Perhaps you could help me out...


Matt: Okay
Sam: Is there anything in the scripture stating that every person has a match? I am not aware of it.
Matt: No
Sam: I didn't think so...after all...God told Jeremiah not to marry, and there is no evidence that any of
the apostles married...am I wrong?
Matt: Paul was married as was Peter - If I remember correctly.
Sam: They were? What happened to their wives, and could you kindly refer me to scriptural evidence?
or is it extra-scriptural that it is stated. Paul never refers to his wife in his letters, if I recall.
Matt: I don't have it off the top of my head.
Sam: Any ideas where to look, perhaps a specific book?
Matt: [I have the advantage of a Bible program and was able to find the verses so I posted them for
him.] "My defense to those who examine me is this: 4Do we not have a right to eat and drink? 5Do we
not have a right to take along a believing wife, even as the rest of the apostles, and the brothers of
the Lord, and Cephas?" (1 Cor. 9:3-5).
Sam: OK, thanks... I must have forgotten it, but I have verse 5 highlighted...
Matt: Good.
Sam: I am emailing a friend concerning how he lost the love of an unbelieving woman after he was
converted, and then you popped on [to the chat program]. I figured that you could help me... many
thanks.
Matt: How is he handling it?
Sam: It’s weird. This break-up was more than a year ago, and I haven't known him to be this
distraught over anything this bad.
Matt: Love does that... Is he a Christian?
Sam: Yes... but he is going through doubts now.
Matt: About what?
Sam: Here's a quote from his email to me:

"Let this be a lesson to you all: I've lost the only person that I've ever loved with all my
heart, all my soul, and all my mind for a GOD whom I've never seen, never spoken too,
and never touched... Why should I believe? I shouldn't, should I?"

Matt: It is a difficult thing to live in pain. God the father, lost a greater love with the death of His
son.... The pain was infinite because God is infinite. Love makes us vulnerable. Faithfulness to God is a
decision -- no matter what. Sometimes God let's us go through trials for 2 reasons. 1) to show us God.
2) to show us ourselves This man has the opportunity to discover the loving sacrifice of God through
his own experience. He also has the opportunity to discover where his faith is and WHAT it is in. If it is
in God, truly, then pain won't drive him from Him.
Sam: I couldn't say it any better.
Matt: I'd be willing to call your friend and talk to him.
Sam: Thanks, but I am going to try to get our youth pastor to talk to him.
Matt: Okay.....
Sam: I appreciate it.
Matt: But just so you know, five years ago, my son was born and died ½ hour after birth. The pain
was incredible.... My wife and I wept a great deal....
Sam: I can't imagine...
Matt: But... we had our eyes on the Lord... and through it, we grew stronger in our faith....
Sam: Amen.
Matt: Sometimes God allows our heart to feel the sorrow of loss so that it will know the depths of
hope and patience that only that loss can teach once it has healed. Then we can better serve others in
their pain....... True love is other centered.... It seems as though he is becoming a little 'self' centered.
That often happens when we are hurting. His love is turning to pain and anger... and he is trying to
blame God...
Sam: I see what you mean.
Matt: The wrath spoken of in Rom. 1:18 is upon the heart. God will let people be given over to their
sinful desires.... Ask him: What is the desire of his heart... where is his focus? Is God allowing Him to
have what He wants?
Sam: That’s interesting.
Matt: as far as his lost love goes... It is a sin for him (as a Christian) to marry an unbeliever....
because it joins the holy with the unholy.
Sam: I got that.
Matt: Christians are the temple of God... they are not to join with unbelievers in marriage. God will
honor him. But, he must keep his eyes upon the Lord. No matter what.
Sam: You are right. He needs to do that. But it is difficult when he is hurting so much.
Matt: Yes it is. I can’t say I’ve done it perfectly myself. But that doesn’t make it any less true.
Sam: Yep. You got that right....

Sometimes truth is better left unsaid. I think that perhaps I was not compassionate enough with
my comments on Rom. 1:18. I think they were true, but that doesn't mean that saying it was the best
thing to do. Sometimes people just need to be encouraged, not corrected. May I learn from my
mistakes......
An Agnostic Questions the Trinity
This is a very brief dialogue, but it contains a bit of information that I think is helpful in dealing
with people who are curious about the Trinity and who think it is illogical.

Vick: Hello. I'm the Agnostic who e-mailed you yesterday. Want to have a little discussion?
Matt: Sure.
Vick: So why exactly are you a Christian, if you mind me asking? What led you to Christianity?
Matt: Well, it's because I "got saved." It was an extremely emotional and spiritual event.
Vick: What were your beliefs before then?
Matt: Before I was a Christian I was an agnostic, an evolutionists, and I did not really take God, or
the issue of God, very seriously.
Vick: I think I've given it a good deal of thought. I just came to the conclusion that Christianity,
Islam, and just about every other world religion was illogical.
Matt: Why is that?
Vick: Things like the Trinity, God having a arch enemy....that kind of stuff.
Matt: Well, let's take a look at the Trinity quickly. The Trinity is the doctrine that there is one God
who exists in three persons: Father, Son, and the Holy Spirit. They are not three gods. Now, what
about that is illogical?
Vick: Well, for starters, why? Why exist in 3 persons?
Matt: Asking why is not an issue of logic, but of curiosity.
Matt: The Trinity is not illogical. Asking "Why" really isn't the issue. Do you see my point?
Vick: Yea, I see. But I can't see why that would be essential for him to survive, or anything for that
matter
Vick: Not to mention the idea of a trinity was not popular until well after the 1st century.
Matt: We do not know if it is or it is not essential, that God be a Trinity -- though I assume it is
because that is what He is. We Christians simply state that the Trinity is the doctrine revealed in
scripture. It is what God has revealed about himself. Take for example the concept of time. Time
exists in three parts: past, present, and future. Each is not the other. But each shares the same
nature: time. So too with space which is height, width, and depth. Matter is solid, liquid, and gas.
Matt: You live in the trinity of trinities. Why would the concept of the Trinity be so difficult to accept?
The Bible says in Romans 1 that God's nature is revealed in creation. It is all around us.
Matt: Also, it does not matter if the Trinity was popular or not before or after the first century. The
issue is whether or not the Bible teaches it. The Church merely took a while to recognize what the
Bible said about God's nature. What they concluded was what we call the Trinity.
Vick: While we're on the subject of the bible, what do you have to say about errancy of the bible?
Matt: I believe the Bible is inerrant in the original manuscripts. What we have are copies of inerrant
documents. There are minor areas of textual variation worth consideration. But, the overlapping
concepts and the overlapping manuscript evidence is overwhelming. We can easily accept the Bible as
a reliable and historically accurate document.
Vick: be right back

Vick never came back and the discussion ended here.


Claims to be a god
I don't know if this dialogue was productive or not and I'm not sure I handled it properly. I tried
to get Dan to see that he wasn't been logical and that he needed forgiveness because he was a
sinner. When someone claims to be a god, there are some natural questions that arise. His answers
to them were interesting.

Dan: Hi.
Matt: Hi. Are you a Christian?
Dan: Nope nope nope
Matt: Then what?
Dan: Satanist, a good Satanist, not a devil worshipping moron.
Matt: Oh, Why?
Dan: Because I'm my own god
Matt: You're your own god?
Dan: Yup
Matt: Do you ever get sick or screw up at anything?
Dan: I haven't been sick since I went vegan and yes I do make the occasional error... my god is not
perfect.
Matt: Well, then, you aren't a very good god are you?
Dan: Not like your almighty imagination, neither like a character in a book.
Matt: Have you created anything lately, raised anyone from the dead, walked on water, or anything?
Dan: Neither.
Matt: Oh... well, that isn't very good... tell me, as a god, what are you able to do... that is godlike?
Dan: Yeah, I make stuff all the time and yes I walked on water today.... oh yeah and an amazing
feat... Define godlike.
Matt: God is able to do incredibly awesome things, like create, uncreate, heal people instantly, be all
places at once, know people's thoughts... that kind of stuff. Can you do that?
Dan: You said godlike, not like God. There's a major difference. And "he's" nowhere.
Matt: No. You said you were your own god? You said it above.
Dan: Yup.
Matt: That is what you said...
Dan: He's not material so he can't be anywhere. Yes.
Matt: You didn't' say godlike... and, you blew it... so, you're failing at being godlike.
Dan: But then you twisted my words to be the Christian god.
Matt: I did not.... If you are godlike, you should have known this before hand. From what I can see,
you aren't very good
at this god thing....
Matt: So tell me, how are you godlike?
Dan: I'm a humble god, satisfied with being average. I need no special abilities. I lead but one...
myself. So, as long as I'm at least as good as myself, I'm fine. I'm being the best god I can be.
Matt: You are humble?
Dan: Of course
Matt: if you have no special abilities, then how can you be a god?
Dan: I'm not saying all must worship me or perish. God's are just better at something than someone
else, perhaps I'm a god of wit and being a decent guitar player.
Matt: Okay, but how can you be a god, if you boast about being humble and don't have any special
abilities?
Dan: I'm a laid back god, and only the Christian god knows everything. I've never heard of another
god that was so full of himself. No roman gods claimed that.
Matt: I don't think you're a god.
Dan: Oh well, I'm not asking you to believe.
Matt: You said you were one... but can't prove it... You've got nothing.
Dan: Oh well.
Matt: Jesus had a lot more going for Himself than you do.
Dan: Doesn't bother me. Yeah, he had a fictional novel written about him. Dang! why not me?
Matt: How do you know it is fiction and not true?
Dan: Because it's complete B.S.
Matt: Really? What part?
Dan: From page 1 on. That big chunk right in there.
Matt: Got any specifics?
Dan: Every reference to god, and Jesus, and Moses, and Mary, and Joseph, and the apostles, and
everything else.
Matt: You aren't saying anything here. You aren't even offering any credible evidence or reason....
Dan: Neither are you. I don't care. I'm not looking to convert.
Matt: You admit your aren't? If you were a god, would you really make that kind of mistake, not being
able to prove anything?
Dan: Why don't you let your precious god fight for himself?
Matt: He will. He will fight quite well.
Dan: Then let him. I'm not an egotistical jerk.
Matt: You aren't? And you claim to be a god! Isn't that rather egotistical?
Dan: I'm not asking you to believe anything, and here you are pushing your beliefs on me.
Matt: I'm not pushing anything. I'm just trying to show you how illogical you are being.
Dan: Egotistical: being excessively interested in ones self: having a self centered attitude. I have
none of these problems.
Matt: oh... like saying your a god isn't egotistical?
Dan: And i don't think I'm the one being illogical. I do know who's being irrational, and it's not me.
Matt: Well, I'm going around claiming to be a god.... and you're the rational one?
Dan: Yes, because I'm not trying to get anyone to believe in anything. I said I was my own god
Matt: you're trying to get me to believe you're a god aren't you?
Dan: You know very well that I was not saying I'm some supreme being. But you use it as an excuse
to push Jesus Christ and all his bull on me
Matt: I didn't say you were a supreme being.
Dan: You're trying to get me to prove to you I am a god. I wasn't trying to convince at all.
Matt: So, you admit he existed?
Dan: nope
Matt: Do you believe Jerusalem exists? and Egypt? and Israel? and the Jews? and the Red Sea?
Dan: They all exist, but not because of the Bible.
Matt: But the Bible speaks of them... accurately, right? I mean, they are there.
Dan: All that proves is the bible was written after they all were established.
Matt: By people who were actually there.... right?
Dan: Perhaps by anti-Semitic people to get everyone to hate the Jews.
Matt: Are you guessing?
Dan: Assuming, yes.
Matt: So, all the prophecies fulfilled by Jesus, the eyewitness accounts of Jesus' miracles, etc. are all
bogus?
Dan: Yup, I've not seen them and no one will ever convince me that they really have. It's illogical.
Matt: No one will convince you no matter what the evidence is, right?
Dan: Someone's cancer goes away and OHHH IT MUST BE GOD.
Dan: A plane crashed and people lived OHHHH THAT'S DEFINITELY GOD!
Matt: no one will convince you no matter what the evidence is, right?
Dan: If I saw God with my own eyes I would believe. That's about it.
Matt: Why don't you ask Him to appear to you, then?
Dan: No... that's most certainly it
Dan: I have, and guess what hasn't happened.
Matt: guess what? I asked him too. He didn't appear....Does that mean he does not exist?
Dan: Apparently.
Matt: oh... so, if I can't see your mother, does that mean she doesn't exist?
Dan: If that's what you want to believe even though people have actually seen her as opposed to
someone I've heard of.
Matt: But you exist, you must have had a mother, to believe contrary would be illogical... to believe
other than that, by simply choice, would be contrary to evidence.
Matt: Why do you do the same with God?
Dan: Because God never did anything. An imaginary being can't do anything. He made nothing. He
said nothing.
Matt: You don't want Him. Why should He help you? You insult Him....
Dan: Why would I believe in something that contradicts every law of physics and nature? I'm not
asking him to help me with anything.
Matt: What laws does God contradict?
Dan: Everything, and immaterial being, who lives on forever and was here since forever, knows
everything, is everywhere at once. It is all illogical.
Matt: If you do not know all the Laws of nature, you cannot say that is impossible.
Dan: I wouldn't believe in a human who didn't physically exist, or a dollar that existed for all time.
Matt: Claiming to be a god is illogical.
Dan: I've never seen (nor has anyone) anything that was immaterial and eternal and was all places at
once. Claiming that there is a god is illogical
Matt: Therefore, it doesn't exist because you think it is illogical?
Dan: Yup. How can something that doesn't physically exist in some way, possibly exist?
Matt: Thoughts exist and are not physical.
Dan: So is everything we've ever known, so asking someone to believe something different than what
they've known as fact since before they were born is irrational.
Dan: But they belong to something that is physical. They are not their own entity.
Matt: Thoughts exist and are not physical.... you believe in them, right?
Dan: That's a really really bad analogy.
Matt: Why?
Dan: I pity you now
Matt: Thanks....
Dan: no problem
Matt: I think you need to study logic a bit more if you want to be a god.
Dan: Stick it! I'm not a god you're not a god. God's not a god. No one's a god
Matt: And you want him to appear to you? and you insult him like that? That isn't smart
Dan: Oh well, he enjoys sending people to horrible places of pain and suffering.... some kind loving
god huh?.... so let him do his worst.
Matt: He will.... you'll not like.
Dan: Oh well.
Matt: Not after you insult him so much... and he is a lot bigger than you. You need Jesus.
Dan: Bigger in what way?
Matt: Much bigger and stronger than you.
Dan: If he doesn't physically, exist he has no size. So I'm much bigger than him. An ant is much
larger than him.
Matt: God is loving. That is why He sent His son to die and pay for the sins of the world.
Dan: Oh yeah that's love, too.
Matt: Hey, if you slap him in the face, and reject his offer to help you, what do you want him to do?'
Dan: Yeah, that's real loving. "Go die son."
Dan: He's self centered apparently.
Matt: God HIMSELF died.... became a man... he sacrificed himself.
Dan: Follow me. Believe in me or suffer.
Matt: No... love gives. God so loved the world he GAVE his only begotten son.
Dan: Yeah right.
Matt: Yep, that is right.
Dan: If he's so mighty he could make more
Matt: He can forgive you of your sins, if you want.
Dan: I already forgave myself, i don't need him.
Matt: But if you don't want, then you'll spend eternity without Him.
Matt: Eternity is a long time to be wrong.
Dan: Thank "God" because I don't want to be around an egomaniac for eternity.
Matt: Neither do I. But I do want to be with God.
Dan: He is the biggest self centered egotistical, bitter, unforgiving, unloving S.O.B ever.
Matt: If god is real, would you want him to forgive you?
Dan: I'd believe in him, forgiveness.... I dunno.
Matt: Wow! You make him sound like Hitler. But that sure isn't the Lord I know. You've been told
some pretty weird stuff.
Dan: Sounds like god to me.
Matt: Do you want forgiveness, and cleansing, and a warm and good heart?
Matt: You can find out if He's real....
Dan: I have a good heart. I can take baths, and I'm not sorry for anything.
Matt: You'd have to humble yourself before Him....
Matt: You'd have to ask him to forgive you of your sins.... and mean it.
Matt: Jesus forgives sins.
Dan: Impossible.
Matt: Why?
Dan: I wouldn't mean it.
Matt: Are you a sinner? Have you ever done anything wrong?
Dan: "He" made me a sinner, it's his own fault.
Matt: No. You're a sinner because Adam sinned....and his children were sinners, and so on...
Matt: Ever done anything wrong?
Dan: I still don't get why you're trying to convert me. Why do you let religion take over your life and
make you further alienate people like me?
Matt: I'm trying to point you to Jesus, that's all.
Dan: You're trying (seemingly) to push me as far from Jesus as you possibly can.
Matt: Ever done anything wrong?
Dan: All the time.
Matt: If you've done wrong things, then you've offended God.
Dan: Oh well
Matt: He's the one you have to worry about.
Dan: He made mankind sinners, let him deal with it
Matt: No. He made Adam good. Adam chose to rebel. Adam, the sinner, had children. Blame Adam.
Matt: ...and you would have done better than Adam?
Dan: So Adam has the power to overcome god's will?
Matt: God let him have the freedom to sin.
Dan: So shouldn't you be worshipping Adam? You just said god made him good
Matt: I'll put my faith in Jesus.
Dan: Supposedly god knows all that will happen and has for all time, so I'm predestined to whatever
fate may await me.
Matt: Make your choice
Dan: well as I said, God already made that choice for me.
Matt: You are making it right now. Don't blame God for your anger and rebellion.
Dan: I'm not angry....
Matt: Could have fooled me.
Dan: well I'm angry that you can't accept my decision seeing as how it's mine.
Matt: Make it... and live with it for eternity.
Dan: God made it for me before time began
Matt: You made it... you are making it now... make your choice.
Dan: He likes to watch people suffer.
Matt: No he doesn't.
Matt: make your choice... and live with it for eternity.
Matt: can you change your heart away from anger, mockery, and pride?
Matt: If not, you are no god.
Dan: Great, I wanted to leave an hour ago, but I didn't want you to think you won.
Matt: Okay, then have the last word.
Dan: I don't want the last word
Matt: Okay, then let me have it. Jesus can still forgive you and give you a new heart.
Dan: bye
Matt: bye
Is it right to tell people that what they believe in is false?
This dialogue deals with the right, or lack thereof, of Christians like me calling other peoples’ beliefs
false. Is it right to do that? How do we know we are not deceived ourselves?

Tim: Hi. I have some questions. Can we talk?


Matt: Sure. What’s up?
Tim: What right do we have calling someone a heretic. I mean nobody is perfect. Nobody understands
the Bible perfectly. We are all mistaken about something about the Bible.
Matt: The Bible defines what is true and states that those who fall outside certain parameters of
doctrine are false. We are only repeating and teaching people what God has already stated about
truth.
Tim: True, but how do we know if we are right? We could be interpreting the Bible falsely. It is like the
tea pot calling the kettle black, if you get my meaning
Matt: That is true. But let me ask you. Is Jesus God in flesh?
Tim: I believe so! But there are verses in the Bible that seem to discredit this.
Matt: Such as?
Tim: Jesus saying the father is greater than him.
Matt: But how does that mean that Jesus is not God? Remember, according to Phil. 2:5-8 and Heb. 2,
Jesus became a man. He ADDED human nature to himself and became one of us. He was made for a
little while, lower than the angels.
Matt: I am married. My wife is a Christian. I am greater than she is in authority and position in the
family. Does that mean that she is not human because I am ‘greater’ than her in position?
Tim: Okay, I see what you’re saying.
Tim: But, we are saved by grace Matt and if God wants to be gracious to a Mormon or Jehovah’s
Witness, what right do we have to deny the possibility? God is gracious to whom ever he wants to be
he does not care about denominations.
Tim: Is it possible their could be a few Mormons running around who understand and are saved?
Matt: Yes, it's possible. But official Mormon doctrine is very false and leads to damnation.
Matt: This is because Mormonism teaches that God came from another planet, has a goddess wife,
and that men and women can become gods and goddesses of their own worlds.
Matt: The Bible contradicts this in Isaiah 43:5, 44:6_8, for example.
Tim: True but not all Mormons believe this. The reorganized Mormon church does not.
Matt: The RLDS church is quite different than the LDS church in many respects. But, by far, most
Mormons believe Mormon doctrine. Besides, I do not argue the position from a possibility that they
might know the truth. I argue against Mormon doctrine.
Matt: Now, God says that we are to honor and worship Him only, not any false gods (Exodus 20). Is
the Mormon God true?
Tim: No
Matt: Then they are serving a false god, right?
Tim: Yes.
Matt: Does a false god save anyone?
Tim: Nope.
Matt: Then it is proper to say that if someone believes in the Mormon god, he will not be saved, right?
Tim: True, but most Mormons don’t even know what they believe. They just go to church to feel good.
I should know I have met Mormons who don’t know that the Mormon church teaches they could
become a god.
Matt: I don’t' know about that. 99% of those I've spoken to over 20 years have known Mormon
doctrine.
Matt: God is not a man from another planet who has a goddess wife as Mormonism teaches. That is
false doctrine. It is right and proper to state so.
Tim: True. I never looked at it that way.
Matt: To ignore it, knowingly, is to not warn them about the dire consequences. That is why I have
my web site up....to teach the truth and expose error. I don't want people to go to hell.
Matt: It isn't because I hate Mormons (I like them and work with them), or Jehovah's Witnesses, etc.,
it is because I care about them......
Tim: True. But it does no good to argue. They are trained to argue and they want to argue. They do
not know what they are doing to themselves. It is so confusing because it is hard to deal with someone
who believes they are Christian when they really aren’t.
Matt: I am trained to discuss the issues with them and expose the error. Each and every Mormon is
created in God's image and is worth the struggle of truth which is why I do what I do.
Tim: True.
Matt: Besides, the more that Mormonism (and all cults) spread, the more people are damned.
Tim: True but as a Calvinist you must know the elect will be saved no matter what you do...LOL That
is what gets me about Calvinism.
Matt: True. But, I also know that God uses the preaching of His word and that he ordains the means
of saving people.
Matt: My web site makes a difference in people's lives because God ordains that it does.
Tim: But I think Calvinism is a very confusing doctrine.
Matt: It isn't confusing at all....
Matt: God is sovereign, right?
Tim: Yes.
Matt: God is in control of all things, right?
Tim: Yes.
Matt: God uses all things for His glory and for accomplishing His will, right?
Tim: Yes.
Matt: God uses preaching, teaching, evangelism, and apologetics to bring His people into salvation,
right?
Tim: Yes.
Matt: He uses the freedom we have as Christians to preach to the lost, right?
Tim: Yes.
Matt: He has said in James 5 that the prayers of the righteous make a difference with Him, right?
Tim: True.
Matt: So then, my prayers and preaching influence God, right?
Tim: Yes....to a certain degree.
Matt: Yet, God, from all eternity, has not changed, and His will is carried out.
Matt: Right?
Tim: Yes.
Matt: I am free to preach and teach and let God worry about appointing people to believing (Acts
13:48). Yet, I know what I do makes a difference.....
Tim: True.
Matt: If I am a Calvinist as you think I should be, then why do I have CARM on the net?
Tim: ...because of the reasons you gave me.
Matt: You see, God is sovereign and He still uses us.
Tim: Do you believe we are living in the so-called "end times"?
Matt: Yes, I do.
Tim: Well I am not so sure.....the human race could go on for another hundred years are so
Tim: 1000 year to God is but a day.
Matt: Maybe it will. But we must never stop evangelizing and telling the truth....even up to the day of
His glorious return. Amen?
Tim: YES.
Matt: Now, are YOU saved?
Tim: Yes I believe so....but I hope it is not a head thing and not a heart thing if you understand. I
understand the words but have I really accepted it?
Tim: You can believe something to be true and not accept it
Matt: That is true.
Matt: Let me ask you some questions, all right?
Tim: Sure.
Matt: Do you believe Jesus is God in flesh?
Tim: Yes. I know that part is true. If the Bible is true then Jesus is God

Matt: Do you believe Jesus rose from the dead in the same body He died in, though it was a glorified
body (John 2:19_21; 1 Cor. 15)?
Tim: Yes, but is that because I have been taught to believe or because I have accepted it?
Matt: I understand, but what is your state of belief right now? Do you or do you not believe it?
Tim: It changes from day to day
Matt: I understand. But what you need to do is trust what God says. Faith is only as good as who you
put it in.
Tim: True and I believe that Jesus is the way.
Matt: You need to read the Bible, study Jesus' words, pray to Him, and do your best.
Tim: I have studied other religions and they have no answer to the sin problem.
Matt: That is true. Only Jesus is the way out of sin.
Tim: Hinduism believes in reincarnation, but that does not solve the problem of sin. It makes it worse.
Matt: Yes. It does.
Tim: Only Christianity does it for you....what do you think heaven is like anyway?
Matt: It'll be great being with Jesus.
Tim: With billions of other Christians....I hope Jesus will be in more than one place at the same time
Matt: He will work it out.
Tim: I would hate to stand in line for all eternity. I can’t stand to stand in line for food at the grocery
store.
Matt: It'd be worth it waiting for Jesus, right? :)
Tim: Yes, it would.
Tim: Well, I need to go and get some sleep. Thanks for talking.
Matt: Sure. Anytime.
Tim: Bye
Matt: God bless.
Dialogue with someone who claims to be one of the two witnesses of
Revelation

Palktalk (www.paltalk.com) is an excellent voice chat system that also has text chat ability. I was
in charge in a paltalk discussion room teaching Christian theology when someone named Scott Hofstee
came in. Scott claims to be one of the two witnesses of the book of Revelation and he has made a
prophecy concerning Nov. 20, 2002. Nevertheless, he quickly became crass so I removed his voice
and text privileges in the public chat room. A few minutes later, in a private message to him, I asked
if he would like to be able to communicate in the room again and that if he did, he'd need to be polite.
He responded with the following opening line in our private text dialogue. As you read through you
will see how sad this person really is. I have blocked out his foul language. One more thing, he gave
me permission to post this whole dialogue -- as is stated in the text below.

Scott Hofstee: Take your offer you pharisaical b****rd and shove it up your ***. Any questions?
Matt Slick: So what is going to happen on Nov. 20, 2002?
Scott Hofstee: Read it cult expert and await the beginning of your end.
Scott Hofstee: http;//thedaysofthunder.faithweb.com/ahead.html
Matt Slick: Do you believe Jesus is God in flesh?
Scott Hofstee: Listen idiot stick. Jesus is not God. God is in Jesus. Big difference. Your paganism is
from the devil.
Matt Slick: So, then... the trinity is false too?
Scott Hofstee: You have no idea who Jesus is because you you have never met him.
Matt Slick: Did Jesus rise from the dead in the same body he died in?
Scott Hofstee: The trinity is error.
Matt Slick: Did Jesus rise from the dead in the same body he died in?
Scott Hofstee: LOL [this means Laugh Out Loud]
Matt Slick: I'm just asking what you believe.
Scott Hofstee: Better ask him. Can you hear his voice or do you just follow the paper Jesus? All you
know of him is what you have imagined in your bible. Unless the spirit of God is revealing you cant.
Matt Slick: Did Jesus rise from the dead in the same body he died in? Can you please answer the
question? I'm trying to politely ask you what you believe about Jesus in this regard. Did Jesus rise
from the dead in the same body he died in?
Scott Hofstee: Matt of course he did. The body was changed because even the disciple didn't
recognize him as they were walking with him. It simple.
Matt Slick: I would disagree with it being changed. It was glorified. Jesus rose in the same body He
died in. He retained the wounds in His hands, etc.
Scott Hofstee: Of course he did. But His body was glorified. But it had changed. The glory
automatically changes.
Matt Slick: Now, how are we saved from our sins?
Scott Hofstee: Through Jesus. Of course he is the way the truth and the life. No man come to the
Father but by HIM. He is the only path. Any other way and you are liar and thief belief and faith in his
sacrifice and the fathers resurrection of HIM.
Matt Slick: What do you mean, "through Jesus"? Must you be baptized to be saved?
Matt Slick: So who or what is Jesus? Is He a man right now?
Scott Hofstee: He is man in whom the fullness of the Godhead dwells bodily and to whom has been
given all authority in Heaven and Earth.
Matt Slick: Is Jesus a man right now? right NOW?
Scott Hofstee: Matt he's a male
Matt Slick: I'll take that as a yes.
Scott Hofstee: LOL
Scott Hofstee: Of course he is
Matt Slick: But, you deny that Jesus is God in flesh.
Scott Hofstee: There is only one God and its not Jesus.
Matt Slick: ...and Jesus is not God, he is a man with God "in" Him, right?

Scott Hofstee: Jesus was preexistent with the Father. He proceeded forth from HIM and created all
that is.
Matt Slick: Was Jesus an angel or something?
Scott Hofstee: An angel in the sense that He was a messenger
Matt Slick: Jesus created all that is?
Scott Hofstee: Yes. But he created all angels through the power of the father.
Matt Slick: So, Jesus was pre existent. Was he a created thing?
Scott Hofstee: He was created before ALL that was created and created all thereafter. The Father
created it through HIM because the logos was IN HIM completely.
Scott Hofstee: At the appointed time he took on humanity in order to atone for sins of mankind.
Matt Slick: Was Jesus a created thing before he became a man?
Scott Hofstee: Of course
Scott Hofstee: He appeared in the OT many times.
Matt Slick: Okay, so then, Jesus was a created thing according to you. I am just being clear.
Matt Slick: Got a question for you.
Scott Hofstee: Yes.
Matt Slick: God says in Isaiah 44:24 that He created the heavens and the earth alone. How could it
be that Jesus created all things if Jesus is not God, but a created thing?
Scott Hofstee: He proceeded forth from the bosom of the Father. Very simple. The same way the
apostles healed the sick. The power of God coursed thru them. They were a conduit. Jesus was even
much more than they.
Matt Slick: But it says that God did it alone. That would mean that Jesus didn't do it... if Jesus is not
God.
Scott Hofstee: He was the house that the Father dwelled in FULLY. Matt his power alone created it.
Jesus in Rev 3:14 described himself. It doesn't diminish him at all. It makes what he did on the cross
and in this earth even more awesome. He overcame ever obstacle, every temptation as a man like
us. That was 100 percent submitted to God
Matt Slick: It doesn't say it was His power alone that created. It says that God created alone. Isaiah
44:24, "Thus says the Lord, your Redeemer, and the one who formed you from the womb,“ I, the
Lord, am the maker of all things, stretching out the heavens by Myself, and spreading out the earth all
alone."
Scott Hofstee: The Father fully resided in HIM. So too was Jesus a complete visible image of the
invisible God.
Matt Slick: Then you have no true incarnation. You deny that the Word that WAS God and became
flesh [John 1:1,14]. Instead, you have the Father "in" Christ.
Scott Hofstee: They are ONE.
Matt Slick: Without a true incarnation, there is no true atonement. You are dead in your sins. Jesus
said, "Unless you believe that I AM" you will die in your sins," (John 8:24).
Scott Hofstee: The word is the thoughts of God. They were in Christ.
Matt Slick: So, the the thoughts of God ARE God? The things that are OF God ARE God?
Scott Hofstee: Well tell me what God looks like. Tell me his shape.
Matt Slick: God thinks thoughts. Thoughts are not Him. They are His thoughts.
Scott Hofstee: LOL
Scott Hofstee: You think he is sitting on literal throne in Heaven?
Matt Slick: You deny the incarnation and according to Jesus own words, you are lost.
Matt Slick: In Exodus 3:14, God said "And God said to Moses, “I AM WHO I AM”; and He said, “Thus
you shall say to the sons of Israel, ‘I AM has sent me to you.’"
Matt Slick: Jesus said in John 8:24 that "Unless you believe that I AM, you will die in your sins."
Scott Hofstee: No matt your Walter Martin logic don't work. A lot of people don't understand the
trinity and they re saved in your book. LOL. No where does it say that you believe in the trinity you
will be saved. LOL
Scott Hofstee: You have added to the counsel of God and begun to preach another cursed gospel.
Matt Slick: I do not believe you know God. Jesus reveals the Father (Matt. 11:27) and without the
true Jesus, you will not know the truth Father.
Scott Hofstee: Hey Matt I know the father because Jesus introduced me to HIM.
Matt Slick: Can you quickly tell me exactly what the gospel is? I'm curious.

Scott Hofstee: I was a Trinitarian like you for 34 years. Trinitarianism is pagan in its origin and you
are therefore a false teacher which only further reveals the fact that you don't know the TRUTH who
Jesus is
Matt Slick: I have a lot of info on the trinity as in www.carm.org/doctrine/trinity.htm, etc. But, can
you please tell me exactly what the gospel is?
Matt Slick: Well, I'd be glad to debate you on who Jesus is sometime. But for now, can you quickly
tell me exactly what the gospel is?
Scott Hofstee: LOL
Scott Hofstee: The gospel is contained in Jesus matt. I have presented it to you
Matt Slick: Okay, but what IS the Gospel?
Scott Hofstee: He is the way unto the Father and that happens by knowing him.
Matt Slick: Can you please tell me what the gospel is? Can you tell me what it is according to
scripture?
Scott Hofstee: He said my sheep hear my voice and another they will not follow. So to know him you
must believe He is the one in whom salvation is contained.
Matt Slick: Yes, I know, but can you quickly tell me exactly what the gospel is?
Scott Hofstee: His sinless life was a perfect sacrifice.
Matt Slick: Yes, I know all that, can you quickly tell me exactly what the gospel is?
Scott Hofstee: That was presented unto the father. He became sin. The object of the fathers wrath
toward sin that we might have life. Believe in that, then follow HIM and submit your life as a living
sacrifice and you will live the gospel, the good news. LOL
Scott Hofstee: matt one thing I know is that you know everything. One needs only to ask you.
Matt Slick: 1 Cor. 15:1-4 tells us that the gospel is the death, burial, and resurrection of Jesus for our
sins. You mentioned sacrifice, that is good. But, you didn't really get it.
Matt Slick: 2 Cor. 4:3-4 says, "And even if our gospel is veiled, it is veiled to those who are perishing,
4 in whose case the god of this world has blinded the minds of the unbelieving, that they might not
see the light of the gospel of the glory of Christ, who is the image of God."
Matt Slick: It is you who is the false teacher. You deny the incarnation. You deny that Jesus is God.
You deny that Jesus is the "I AM." You, therefore, do not know God the Father and are a false
teacher.
Matt Slick: Now, how would you like to debate whether or not Jesus is God in flesh? We can do it on
paltalk. I am sure it will draw a large crowd. Are you interested?
Scott Hofstee: Hey matt. I couldn't care less about crowds. Your the ego maniac. I'm here to talk
to you.
Matt Slick: Good. Then you won't mind it when people listen to your side of the issue as we debate it.
Scott Hofstee: All these people have heard what I have said a dozen times. I'm on here and teach
very openly. Its well known what I believe.
Matt Slick: I want to have a public discussion with you on the deity of Christ, for all to hear.
Scott Hofstee: Then drop in our room any time.
Matt Slick: In your room I am treated very poorly.
Scott Hofstee: LOL
Scott Hofstee: You are treated as I am
Matt Slick: I gave you a chance in my room, and you abused it and were very crass.
Scott Hofstee: LOL
Matt Slick: When I go into your rooms, I am mocked.
Scott Hofstee: You are an idiot thus you are mocked.
Matt Slick: When someone red dotted you in my room, I requested that it be removed. [red dot
removes speaking and text privileges in the room]
Scott Hofstee: If you weren't dead to this world it wouldn't matter
Matt Slick: Now, I challenge you to a public debate on the deity of Christ.
Scott Hofstee: I have had every manner of thing spoken about me in this room tonight.
Matt Slick: Do you accept my challenge to a debate?
Scott Hofstee: Take your challenge and shove it up your a** pee wee. Your challenge means
nothing to me.
Matt Slick: I'll take that as a no.
Scott Hofstee: then you are one perceptive ba***rd. LOL. You come into my turf.

Matt Slick: Col. 3:8 says, "But now you also, put them all aside: anger, wrath, malice, slander, and
abusive speech from your mouth."
Scott Hofstee: Don't flatter yourself into thinking I'm mad at you. Let me tell you this. The Lord is
mad at you and I'm here to tell you.
Matt Slick: You mean you are NOT mad at me and you speak so badly? So, the LORD uses cuss
words?
Scott Hofstee: You are going to have your life flipped upside down in a short time. Yeah he uses the
words b****d, a**, dung which is ***t. Toughen up buttercup. You think the Lord is like you?. You
are a *****.
Matt Slick: Well, since you won't accept my challenge to debate what you believe and since you
continue to insult me, perhaps we should continue this at another time.
Scott Hofstee: He aint anything like you. You run around with your degree and suppose you are
speaking for the Lord. You speak for yourself you arrogant *****rd.
Matt Slick: Perhaps you need to read Col. 4:5-6 and 1 Tim. 1:5. They are good scriptures.
Scott Hofstee: And I'm her to tell you your days of speaking for the Lord as though you were sent
are about done. Remember this night.
Matt Slick: Really? So, what is going to happen? Are you threatening me?
Scott Hofstee: No dip***t. The Lord is telling you. You are about done and what is the Lord
saying. He is going to bring you to your knees and humiliate you.
Matt Slick: If He brings me to my knees and humiliates me, I will be most grateful. It would be a
great blessing.
Scott Hofstee: You will have a choice to repent of being a Pharisee or harden your heart and be
destroyed. Your false humility doesn't fool the Lord. He sees your heart. He sees past your pompous
words
Matt Slick: And, YOU are God's messenger?
Scott Hofstee: I am indeed
Matt Slick: Is it not true that you claim to be one of the two witnesses of Revelation?
Scott Hofstee: It is absolutely true
Matt Slick: How do you know you are one of the two witnesses of Revelation?
Scott Hofstee: LOL
Scott Hofstee: Very easy. The Father sent me and told me thru His SON Jesus. How does a man
know anything about himself except thru Jesus. How did Jesus know who He was?
Matt Slick: Well, since you have a false Jesus, I will conclude you do not know Jesus and that you are
not speaking for God and that you are not one of the two witnesses of Revelation.
Scott Hofstee: How did Peter know who Jesus was? LOL. Bad math matt. LOL
Matt Slick: I wish I could post this dialogue on my website.
Scott Hofstee: LOL. Matt please do. I wish you would.
Matt Slick: Okay
Scott Hofstee: Matt i know a few things you don't.
Matt Slick: Such as?
Scott Hofstee: I know from where I came. I know when I am speaking to one of the Fathers
servants in a second because the Father reveals them. He has told me for a certainty you are not
HIS. You sent yourself. HE NEVER SENT YOU. Therefore you are of the devil.
Matt Slick: Thank you.
Scott Hofstee: But you have hope.
Matt Slick: My hope is in Jesus, not in you.
Scott Hofstee: You can repent of your lies and your filth.
Matt Slick: You are full of hatred and foul speech and condemnation.
Scott Hofstee: Well what a coincidence. He is my master and has me speaking to you. Well my
friend if you weren't walking in the flesh you couldn't be condemned. But because you are walking in
the flesh you are totally condemned.
Matt Slick: You know, you really are one of the more interesting cultists I've ever dialogued with.
Scott Hofstee: I'm no cultist. Your carnality is at enmity with God. Matt... I cannot fit into any or
your models of a cult because we have no cult.
Matt Slick: Look, I need to get going because this dialogue is getting no where now. I am convinced
that you are lost and self deceived.
Scott Hofstee: LOL
Matt Slick: I am convinced that you do not have Jesus and that you preach a false god.
Scott Hofstee: Matt remember my words. Your road to Damascus experience is at the door what will
you do when you meet the real Jesus and not the one of your vain imagination?
Matt Slick: I am convinced that you are filled with a false spirit.
Matt Slick: So, it was nice talking to you and very informative to have you verify from your own
mouth (fingers), your error.
Scott Hofstee: Matt. You don't get it.
Matt Slick: I will let others know that you refused to debate me and that you called me lots of names.
Scott Hofstee: We don't care about what you would consider negative publicity.
Matt Slick: You have not shown the fruit of the Spirit of God.
Scott Hofstee: I'm debating you right here you moron. You have a lengthy dialogue demonstrating
such.
Matt Slick: ....as I said... insults and name calling.
Matt Slick: I am sure we will meet again.
Scott Hofstee: Names like Jesus used
Matt Slick: Till then,
Scott Hofstee: Snake
Matt Slick: Later.
Scott Hofstee: White washed tomb
Scott Hofstee: Fool
Scott Hofstee: Hypocrite
Scott Hofstee: Matthew 23
Scott Hofstee: Its all about you
Scott Hofstee: Study it
Scott Hofstee: It foretells of your fate
Matt Slick: 2 Cor. 11:3-4, "But I am afraid, lest as the serpent deceived Eve by his craftiness, your
minds should be led astray from the simplicity and purity of devotion to Christ. 4For if one comes and
preaches another Jesus whom we have not preached, or you receive a different spirit which you have
not received, or a different gospel which you have not accepted, you bear this beautifully."
Matt Slick: bye

As you can see, this person is quite deluded. He is full of hatred and condemnation as well as a
false understanding of who Jesus is. Since He has a false god and Christ, it explains why he is so
quick to swear and demonstrate the lack of the Spirit of God.
Above, I referenced a couple verses that I though Mr. Hofstee should focus on. They are

• Col. 4:5-6, "Conduct yourselves with wisdom toward outsiders, making the most of the
opportunity. 6Let your speech always be with grace, seasoned, as it were, with salt, so that you
may know how you should respond to each person.
• 1 Tim. 1:5, "But the goal of our instruction is love from a pure heart and a good conscience and
a sincere faith."

If this person were a Christian, the Spirit of God would convict him of his errors and attempt to
bring him to a consistent behavior with scripture.
Discussion on the possibility of Jesus' resurrection.

The following dialogue was a quick one in a crowded discussion room. On the fly, it is difficult to
give adequate answers, but the more you know the better you can do. I can only hope that this
conversation went well enough to be used by the Lord to accomplish His will in this person's life.
This dialogue began because a topic was offered stating that Christians were mentally disturbed. I
jumped in by asking a question.

Matt: Why are Christians mentally disturbed?


Tom: Because they believe in things for which there is no evidence and that is irrational indeed.
Matt: But they would offer the eyewitness accounts of the Bible, the fulfilled prophecies, etc., as some
form of evidence. To say there is NO evidence is inaccurate. It would seem that you just don't like
their evidence.
Tom: You are assuming that the "eyewitness accounts" are worthy of being called "evidence."
Matt: Why not? Is not eyewitness testimony in a court valid?
Tom: I would contest that seemingly impossible happenings could be witnessed.
Matt: You can contest it, but that does not invalidate the evidence. The eyewitness accounts are
there.
Tom: So you are saying the evidence is valid?
Matt: You tell me. They claim to have eyewitness accounts recorded down.
Tom: So you are saying that if someone says they witnessed something then it must be true?
Matt: The accounts seem to be consistent with archaeological evidence of times and places.
Tom: What are you trying to prove by this assertion?
Matt: I am only trying to get you to think. Logic can help find error in thinking.
Tom: I do plenty of thinking.
Matt: To say there is NO evidence is an illogical statement, since you cannot know all evidence.
Matt: The Christians claim evidence in the eyewitness accounts of Christ. I am asking why it isn't
valid.
Tom: I am not disputing that Christ existed.
Matt: Okay.
Matt: Are you disputing the eyewitness accounts.
Tom: But what are you claiming that this evidence supports?
Matt: The evidence supports what it supports.
Tom: What proposition are you trying to establish?
Matt: If eyewitnesses see Jesus rise, is it possible that it could be true?
Tom: Hume said that no one should ever believe a miracle had taken place unless it would be a
greater miracle that the person reporting the miracle was either mistaken or lying.
Matt: Why would the eyewitnesses lie? Why didn't the Jewish community say Christ did not rise? I am
talking about the evidence. What does the evidence of the eyewitnesses suggest? Does it suggest
that Christ rose from the dead? Or, does it suggest they were all liars?
Tom: Why would anyone believe that Jesus rose from the dead, when it is clear from all our
experience that people do not do that?
Matt: You beg the question. You assume the thing you are trying to prove.
Tom: It does not suggest to me that Christ rose from the dead .
Matt: If there is a god, is it not logical to say Jesus could have risen,....since he claimed to be God.
Tom: You are presupposing that there is a god.
Matt: I said, IF there is a god. That is not a presupposition. Follow the logic.
Tom: So what if there isn't a god?
Matt: If there is no God, then the resurrection cannot happen, correct?
Matt: But, if the eyewitness reported a physical resurrection, then wouldn't that suggest that there is
a God? Again, what does the evidence suggest?
Tom: How is the supposed resurrection of a human suggestive of the existence of a divine being?
Surely there is NO logical connection there?
Matt: You asserted that it couldn't happen because it just doesn't happen. I am simply asking what
the evidence suggests.
Tom: So if it did happen, why would that suggest that God exists?
Matt: Okay, since the eyewitnesses saw him die, and saw him after three days of being dead, then
isn't that evidence of the supernatural?
Matt: A resurrection would be supernatural, right? You stated that things like that don't happen. I
am asking what the eyewitness evidence suggests. Simple.
Tom: If a dead and stinking body rose from the dead, then it would be supernatural yes. So, the so-
called eyewitness accounts are either mistaken or downright lies.
Matt: The third option is that they are accurate and true.
Tom: Why would one suppose that?
Matt: If you miss all the logical options, then how can you possibly draw accurate conclusions? Again,
I ask you, what does the evidence suggest?
Tom: So you think that it is more "logical" to assume that the eyewitness accounts of Jesus rising
from the dead are accurate and true rather to suppose that they are the products of mistakes or lies?
Matt: Mistakes are certainly possible. But, can all the witness be mistaken? That they lied is also
possible, but why die for a lie? Remember, the Christian apostles died for the resurrection of Christ.
Why would they do that if it were a lie? They had nothing to gain except being ostracized, ridiculed,
and dying for what they believed.
Tom: You are forgetting the motivation factor.
Matt: Did they not see Him die? Did they not see him after the resurrection? These are the accounts,
of eyewitnesses.
Matt: So, please offer some rational explanation for their consistent and repeated mistakes by all the
disciples, and/or their intent to lie about what they claimed they saw.
Tom: If I were to produce several eye witnesses who assured you that they had seen a giant ladybird
lumbering down the street singing "land of hope and glory" would you believe them?
Matt: You are not addressing the questions I have asked. You offered a theory. Can you back up your
theory with logic and or evidence?
Tom: The onus is not on me to disprove that which according to all our knowledge of the universe
could not possibly take place.
Matt: You offered two alternative theories to account for the resurrection. I am asking you to
somehow substantiate them. If you cannot offer some rational reason for your theories, then why do
you hold them? Is it because you presuppose that the resurrection could not occur? I suspect that
you must simply dismiss the evidence without logical reason.
Tom: I have given a rational reason as to why I think that the resurrection did not occur.
Matt: So, if that is the case, if you cannot offer something suitable, in place of it, then you have no
point at all.
Matt: What is that rational reason that the resurrection did not occur?
Tom: That all our experience of the universe tells us that dead and stinking bodies do not return to
life.
Matt: Really? "All" our experience? So, the experience of the eyewitness is not valid? You dismiss it.
On what basis, because it contradicts your presupposition?
Tom: You didn't address my point about the giant ladybird did you?
Matt: The facts should determine that, not a belief that makes an assumption as you have done.
Matt: So, you simply dismiss the eyewitness accounts because they do not agree with your
presupposition, isn't that correct?
Tom: And YOU cannot assume that "so called" eyewitness accounts of things which happened two
thousand years ago are any indication of truth.
Matt: I am not assuming. I am simply asking you to tell me what the evidence suggests. But all you
have done is dismiss the evidence. I do not consider that to be a good way to determine truth.
Tom: Ok I am dismissing the evidence because I do not consider it to be valid.
Matt: But you have not given a valid reason for invalidating the evidence.
Tom: Do YOU have a valid reason for assuming that it IS valid?
Matt: The evidence for its validity is that it is eyewitness accounts recorded, corroborated by other
writers of the New Testament, and there is no contradictory evidence of the resurrection of Christ
offered by the Romans or the Jews of that time period.
Tom: Well, excuse me if I don't roll over and accept it.
Matt: The eyewitnesses then died for their faith later, based upon the resurrection. They had watched
Christ die.
Tom: SO WHAT???
Matt: The most logical thing to assume is that He rose from the dead.
Tom: Oh it is is it?
Matt: Yes it is. Think about it. There is no contradictory evidence for the resurrection. Many people
attested to it. The Jews and the Romans left NO writings contradicting it. The Christian witnesses
ended up dying for it. Why would I NOT believe it?
Tom: You are being totally illogical. So you would take the so called eyewitness accounts of people
and hold them in greater account than all our present knowledge of how the universe
operates????????
Matt: No, it is you who is being illogical. You do not know how the universe operates. Therefore, the
resurrection is possible.
Tom: You are placing your personal need to believe these things above all rational thought.
Matt: You presuppose the resurrection could not happen and contradict the evidence. Is that logical?
Tom: You are prostituting your intellect in favor of a selfish need to believe.
Matt: You are not a good mind reader. I see that you have run out of answers and are now attacking
me personally. I think this conversation is over.
Tom: Fine by me. Goodbye.
Matt: I will return another time and perhaps we can have another conversation after you've thought
this through more carefully.
Tom: No I don't think so. I don't wish to talk to you again

This dialogue was interesting in that Tom refused to back up his denial of the resurrection with
anything other than an argument that "It just can't happen." I offered him questions he failed to
answer regarding the apostles and their willingness to die for their faith. Please note, however, that
dying for ones' faith doesn't prove anything the faith is true. But, dying for the belief in the
resurrection of Christ is a powerful commentary on the beliefs of the apostles; namely, that they all
believed in Jesus' bodily resurrection. It would take something major to move the apostles away from
their Jewish beliefs and culture which led to their deaths as martyrs. Tom offered nothing substantial
in place of this and simply said that I was being illogical. If I am illogical, then demonstrate the illogic
of my statements instead of saying that the universe operates a certain way and Jesus' resurrection
isn't possible. In so doing, he simply begs the question. That is, he assumes the thing he is trying to
prove.
I would have like to have pressed him even more on this issue of logic and evidence. Perhaps next
time in a similar discussion, I will.
Condemning words and pompous attitude

This dialogue occurred on paltalk (paltalk.com, a voice chat system), in a private text message
box. The conversation was initiated by this woman who did not like it that a person was red dotted
(speaking and writing privileges were removed) because she so consistently insults people wherever
she goes.
The person here in the dialogue sought me out and began the conversation as it is here. I tried to
respond and be respectful, but you will see that she did not return the favor.
All the names were changed, except mine.

Gale: You made trashing remarks about her .. and she was red dotted for far les .. just as I was.
Matt: Perhaps you are unaware of her. Are you familiar with her history here on paltalk?
Gale: I don't care what her history is. I care about FAIRNESS. If she is red dotted for her remarks
about you then you should be red dotted for attacking her. You put yourself at the same level as Jim
who is obviously a bigot.
Matt: So, you don't care about the facts? Just so you know, Chris always insults me and others...
every chance she gets.
Matt: She constantly does that, un provoked, every time I enter a room. She is known to be a trouble
maker. She is known to be rude and insulting.
Gale: Facts? What facts? If you can't defend your views then your views need scrutiny.
Matt: She has simply earned the need to be dotted.
Gale: I saw nothing insulting .. only her opinion .. which I happen to share .. that YOU are a fake.
Matt: Okay. So why am I a fake? A fake means "not real." Co, can you clarify?
Gale: Your views are bigoted and often unChristian .. and usually contra-Scriptural .. I've observed
you, too.
Matt: Which views are those?
Gale: You are a fake Christian. .that is, you claim to be a Christian, but your actions say otherwise.
Matt: What actions are those?
Gale: You often act in vicious unChristian manner.
Matt: Can you please be more specific?
Gale: I've seen you attack others for their views .. especially Catholics .. where is that Christian
behavior?
Matt: Can you please give me an example?
Gale: Sure
Gale: You attacking Sothe7
Matt: I am awaiting specifics...
Gale: You "pretend" to not be a Calvinist .. yet, your views pretty much run in that vein .. like you are
ashamed to wear that label.
Matt: I do not pretend to not be a Calvinist. I openly state I am and defend it on occasion.
Gale: specifics? you already know your behavior .. I'm not a tape recorder .. I usually discount
anything you say as babble .. since you have almost no understanding of Scripture.
Gale: You do pretend to be not Calvinist. I've seen you do it.
Gale: Calvinism is completely unbiblical. PROVE to me that YOU are among the "elect" ..... PROVE it.
Matt: I just stated I was a Calvinist in that room, did I not?
Matt: May I ask how old you are?
Gale: 59
Matt: I'm surprised.
Gale: PROVE to me that YOU are among the "elect" ... PROVE it.
Gale: Surprised that I do not buy in to horse crap theology?
Matt: How do you want me to prove it to you?
Gale: Any way you can.
Matt: I don't know your presuppositions. Therefore, "proofs" might not work for you.
Gale: Believing does NOT constitute proof.
Matt: Since you are hostile, I doubt anything I offered would suffice. Are you a Trinitarian?
Gale: I am not hostile. I'm just not a sucker.
Gale: I am a Catholic. There is my theology.
Matt: My apologies, but it seems to me that you are hostile to me and what I believe.
Gale: Catholics are Trinitarians.
Matt: May I ask if you are going to heaven or not?
Gale: I am never hostile. No need to be
Gale: I hope in my salvation and WORK toward it as called for in Scripture. Need the verses?
Romans 8:24. No one is "elect".
Matt: So, you do not know if you are saved? Why not?
Gale: We are ALL given sufficient grace to obtain our personal salvation.
Gale: You are NOT saved. In fact, your views have you on a path to hell. You incorporate HATE as
part of your beliefs .. HATE is a cardinal sin that DAMNS souls.
Matt: Well, 1 John 5:13 says that I can know I have eternal life. Since I have trusted in Christ alone
for my salvation and in nothing of my own effort, I trust Christ and believe that what He said is truth
-- that He gives me eternal life.
Gale: That would be fine if that were the sum total of Christ's teaching .. but, its not.
Gale: I have personally witnessed you committing acts of hate .. even tonight.
Matt: Okay, could you please tell me what acts of hate I committed tonight?
Gale: Sure. You "trashed" Chris and you failed to communicate to Jim & Pete that they have no
business red dotting souls simply because that soul either disagrees with them or defends some one or
a view they find distasteful .. that is hate by omission
Matt: Excuse me, but as soon as I entered the room, Chris said that I was a fake. She got red dotted
which was not my doing. I then said that she has "issues." So, how is that hate?
Gale: I was red dotted simply for defending Chris .. by asking why they did not red dot you
Matt: I cannot tell you why they do what they do. I can only speak for myself.
Gale: Its hate because you made mention of it in public .. you "trashed" her in front of others .. is that
a Christian act?
Gale: no .. you DAMNED well better speak out against injustice .. that is the hate you manifest.
Matt: Do you mind if I put this dialogue up on my website? I'll change your name so no one will know
it is you.
Gale: Be my guest.
Matt: Thanks, I will.
Matt: If I said she has "issues" shows that I was hateful, then is her saying that I am a fake also a
hateful thing? Yes, it sure is. It was wrong.
Matt: I am glad that you admitted that.
Gale: Your web site is probably mostly visited by those who kiss your a** .. .and they are of no
consequence to me.
Matt: As I have already said, she is very antagonistic towards me. I honestly don't know why, but she
obvious has some issues she needs to deal with.
Gale: If she is antagonistic.. Why not seek to resolve the differences in a CHRISTLIKE manner? Like
prayer ...like private conversations, like finding out why she feels the way she does.
Gale: Speaking of prayer .. would you please describe your daily prayer life for me?
Matt: I have asked her what her problem is and she simply insults me. Whenever I go into a room
where she is, she immediately begins to insult me. I was forced to put her on ignore.
Matt: She wouldn't explain to me what her problem is. So, why bother? Obviously, as I said before,
she has some issues she needs to deal with.
Gale: Keep asking .. and use prayer .. oh, I forgot .. since everything is PREDETERMINED .. prayer is
of no value .. sorry, I'm not hip to Calvinism.
Matt: You do not understand Calvinism. We believe that prayer is very important and that it has an
influence with God, per James 5:16. Perhaps you might want to study it more before you attack it.
Just a suggestion.
Gale: Haven't you ever read Paul's accounts of his ministry?
Matt: Of course I have read them.
Gale: How can prayer be important if everything is predetermined by God's will?
Matt: Let's leave the Calvinism discussion for another time, okay?
Gale: I have no use for Calvinism
Matt: You can dismiss Calvinism if you choose, but at least understand it before you do. So far, it
seems that you have a false understanding of what it is.
Gale: 4-5 years ago Sothe7 was determined to lead me OUT of Catholicism and into Calvinism .. I
attended her entrance into the Church and first Holy Communion .. so, that should alert you as to the
strength of my convictions .. I have the Truth in the full risen Jesus .. no rotted soul in hell is going to
draw me from Christ.
Matt: Well, my concern is that you know Christ and not put your trust in anything else, especially your
own works. Salvation is by grace through faith, and not by anything you add to it.
Matt: My opinion of Roman Catholicism hasn't changed. I still believe it has "issues."
Gale: My own works will determine my fate.. just as your will.
Matt: From your own words, I think you are not saved. Sorry, but that is what I believe.
Gale: Catholicism is the True Church given to us by Jesus.
Matt: The Bible says that we are saved apart from our works, Rom. 3:28.
Gale: Your opinion is of no value to me .. in fact your opinion makes YOU a hypocrite.
Matt: Then please respond to Rom. 3:28, "For we maintain that a man is justified by faith apart from
works of the Law."
Matt: Also, Gal. 2:16, "nevertheless knowing that a man is not justified by the works of the Law but
through faith in Christ Jesus, even we have believed in Christ Jesus, that we may be justified by faith
in Christ, and not by the works of the Law; since by the works of the Law shall no flesh be justified."
It seems that God's word contradicts yours. Which one do you want me to believe?
Gale: Again, if that were the sole teaching of Christ ..it would stand as the final fact .. truth is .. you
must take the entire teaching of Christ as a whole .. not snippets.
Matt: The whole does not contradict itself.
Gale: My words do not contradict the teachings of Christ.
Matt: All the cults add their works to salvation: Mormonism, Jehovah's Witnesses, etc. If you add
anything to the work of Christ (your own works), then from what I understand in scripture, you are not
saved.
Gale: I do not claim to be "saved" .. I am, as Scripture teaches .. working out my salvation in fear and
trembling.
Matt: You are seeking to be justified, in part, by your works. Because of that, you will not be saved.
You must rely on Jesus alone, not Jesus AND your works.
Gale: Jesus said to store up treasures in heaven .. now just how do we get these treasures? The
Scripture says that faith without works is DEAD .. is this Truth?
Matt: Yes, it is true. The context is that those who SAY they have faith but have not works, that kind
of faith is useless. I have written about this at http://www.carm.org/questions/faithorworks.htm.
Gale: Not interested in your web site.
Gale: You have little of the Truth and a lot of fluff.
Matt: Well, I've studied the issue and written on it. You can reject it if you want.
Gale: I suggest you read Romans ...
Matt: Romans? I love the books of Romans. I've taught it many times. In fact, here is a great verse in
Romans.
Matt: Rom. 5:1, "Therefore having been justified by faith, we have peace with God through our Lord
Jesus Christ,"
Gale: I reject your personal misunderstandings .. I have the solid 2,000 year teachings of the Church
to guide me.
Gale: Apparently you have never really read Romans. Try reading it for CONTENT.
Matt: I have read it many times and taught out of it often. I strongly suggest you read Romans
chapters 3 through 5. It will deal with your works issue in relation to justification.
Gale: Why is your response limited to single verse quotes? Where is the full understanding? Doesn't
Romans warn that we are JUDGED on our works .. good and bad?
Matt: Single verse quotes? Interesting comment. We are judged for rewards, not for salvation.
Gale: Where in Scripture does it ever say that we are judged on our faith?
Matt: It does not say we are judged on our faith.
Matt: Rom. 5:1 says, "Therefore having been justified by faith, we have peace with God through our
Lord Jesus Christ." Can you tell me what that means?
Gale: I'm not going to bother. That is a waste of my time.
Gale: oh, so, a "saved" soul who is among the "elect" can have several abortions and she has no need
to fear damnation?
Matt: Romans 6:1-3 and 1 John 2:4 refutes that error. Each speaks about NOT doing sinful things
because we are saved.
Gale: Again, you pluck out a verse here and there that seem to support your view while ignoring the
majority that disagree with you.
Matt: Then you admit that the verses I have quoted agree with what I am saying? Good.
Gale: What pomposity. Are you trying to tell me that "saved" women never have abortions?
Matt: But, I ask you what a verse means, you do not tell me, and then you say I am pompous.
Gale: Or is it that abortions are NOT sins for the "elect"?
Matt: I cannot tell you what women do or do not do.
Matt: I can only tell you what the Bible says about being saved.
Gale: I do not disagree with any verse of Scripture .. I do, however, disagree with misapplying them.
You are side stepping my direct question for the second time. Are "saved" women damned for
unrepented abortions or not?
Matt: Look, I am not going to argue the abortion women thing. I am trying to get you to tell me what
the verses mean that I have raised.
Gale: I'm not interested in cherry picking pet verses with you .. I have plenty of my own that will
confound you.
Matt: I have already told you that the Bible warns against sinning for Christians.
Gale: I did not ask you to argue .. I asked you a direct question.
Matt: Well, you can say what you want and ignore the verses and then call me names, but it isn't very
productive on your part.
Gale: What is the consequence of sin?
Matt: Romans 6:23 says that it is death.
Gale: So, it the wages of sin is death (damnation) then a "saved" woman who has an unrepented
abortion is in serious danger of damnation .. true or false?
Matt: How can someone who is saved, be damned?
Gale: You are pompous .. over and over. Well, perhaps she isn't "saved" after all.
Matt: Can you please deal with the issue and not insult me?
Gale: I'm not insulting .. stop acting pompous and I'll stop making the comment.
Matt: If you can't be nice, I'll just end our conversation.
Gale: I'm nice .. you just can not handle anyone who has the upper hand on you .. you have to run
and hide behind pomposity.
Matt: You want me to judge the salvation of a hypothetical woman in a hypothetical situation? She is
YOUR invention, you tell me.
Gale: Deal with me .. if you are strong enough .. btw .. you avoided another important direct question
of mine .. tell me about your personal daily prayer life.
Matt: This is not a discussion where I am going to allow myself to be interrogated by someone who
can't explain scripture, who calls me names, and who avoids the issues I've raised. You are mistaken
about this and need to focus on the issue, not the person. Your dislike for me is evident and you
simply want to find whatever you can to use against me.
Matt: I have suggested that you read Romans 3 through 5 because it contradicts what you have told
me about your works.
Gale: Let's see... You are relying on standard Calvinist tactic number one .. run away and declare
"victory".
Matt: I mean no offense, but since you have contradicted scripture in regards to justification, I am
forced to assume that you are not saved.
Gale: You have yet to prove to me that you are among the "elect."
Matt: Can you please stay on the topic at hand?
Gale: Sure. .I've asked you THREE questions none of which you have made any attempt at responding
to .. as for name calling .. I find that your pompous attitude reeks whether you like it or not .. btw..
I'm not alone in that assessment of you.
Matt: I am sure there are many who agree with you about me. But, that is irrelevant.
Gale: Yes, it is, so, get over the pomposity .. it does not serve you.
Matt: Could you read Romans 3 through 5 and then can we discuss it later?
Gale: While I read Romans for you .. please respond to my questions.
Matt: I have responded to your questions.
Gale: 1) prove to me that you personally are among the "elect" .. 2) how can a "saved" woman have
an unrepented abortion and go to heaven? .. 3) describe your personal daily prayer life.
Matt: I cannot prove to you that I am elect. It is something based on biblical principles that you do
not accept.
Gale: Oh, so you don't KNOW if you are among the "elect" .. you are GUESSING. You have not
responded to my questions in the least .. get off that POMPOUS, ASININE platform and act like an
adult.
Matt: I have responded here and earlier and you continue to insult me. Please, let me tell you that if
you do it just once more, I'll politely end our conversation.
Matt: If a woman is saved, will she want to have an abortion? Will she do that which is against God in
such an egregious manner? And, I will not tell you about my personal life. That is between the Lord
and myself.
Gale: "Saved" women have abortions in this country every day. You can't describe your personal daily
prayer life because you simply do not have one ... that makes you a LIAR to me
Gale: How's that for strong words? Are you ASHAMED of how little you pray?
Matt: Okay, well, I can see that you will not be polite. So, I politely now end our conversation. Please
consider how you have done in this conversation. You can review it on my site shortly.
Matt: Good bye.

(At this point I terminated the dialogue, but she continued with the following to which I did not
respond.)

Gale: You don't pray because Calvinists are arrogant and conceited and see themselves ABOVE
prayer .. admit it.
Gale: No, I'm not polite.. but, I know what the Lord expects from us and it sure as HELL isn't your
pompous, arrogant little conceit... the Lord expects us to FOLLOW HIM . .not Calvin... grow some
B***s and talk to me .. I am waiting for you to act like A BELIEVER.

Unfortunately, this person was difficult to deal with and I tried to remain polite and focused during
our conversation. She continued to be insulting so it was time to end the conversation.
She accused me of many things which reminded me of Romans 2:1,

"Therefore you are without excuse, every man of you who passes judgment, for in that you
judge another, you condemn yourself; for you who judge practice the same things."
An evolutionist says evolution is a fact
This dialogue between an evolutionist and myself was most profitable for me. It showed me some
areas where I need to study. It is also a good example of the dangers of getting involved in an area
that is not your expertise – as becomes evident. Nevertheless, I am not afraid of failure and use it as
an opportunity to learn.
When dialoging with people who know more about something than you, don’t let that stop you
from trying. They can help you learn.

Evolutionist: Do you have a Ph.D. too?


Matt: No. M.Div.
Evolutionist: Master of Divinity?
Matt: Yes.
Matt: I'm a theologian. So, I'm utterly qualified to disprove evolution.
Evolutionist: No. You are not.
Matt: Uh. Humor? Evolutionists have that don't they, or did it disappear through natural selection?
Evolutionist: We have humor, more to the point we have education. Define empirical evidence
please.
Matt: I’m a theologian. You'll have to define it for me.
Evolutionist: Okay. That in and of itself tells me you know nothing of science. I can educate. But will
you listen?
Matt: Is your presupposition that my ignorance invalidates any of my arguments?
Matt: I could presuppose that your ignorance of God negates your ability to rightly judge his
existence, thereby forcing you to arrive at erring conclusions about evolution.
Evolutionist: What is a phylogenetic relation? What is neotony, heterochrony? You cannot argue
against something scientific without knowing science. It is moronic to try to do so.
Matt: Who said I was arguing science? So far, your logic hasn't impressed me.
Evolutionist: You are arguing against evolution, an empirical science, you must know something
about it to argue against it, otherwise its called DOGMATIC PRESENTATION.
Matt: That makes sense. Since I am a theologian, as I said earlier, I am not qualified to refute
evolution. I am simply restating the truth I said earlier.
Evolutionist: Agreed, but you are trying to disprove it yes? Or present alternate hypotheses with
adequate evidence to support it. I’ll listen to that if it is the case. But it has to be scientific evidence.
Matt: Well, more or less, yes. I think evolution is a great deception.
Matt: Is evolution falsifiable?
Evolutionist: Evidence is. But in science there is the double blind test. Part of empirical evidence.
That negates false proof.
Matt: Is there any evidence at all that goes against evolution?
Evolutionist: Not yet, as a scientist I must concede that there is always evidence coming in, but none
in the last 200 years plus. Has there been any negative proof. We have had false claims. But they
have been sought out through empirical means i.e. Piltdown man
Matt: If you have studied it thoroughly, then undoubtedly, you should have encountered evidence
contrary to your belief.
Matt: So then, is evolution an absolute fact?
Evolutionist: Absolute fact yes. Law no. Evolution happens, genetic change over time within a
population but the how is theory, facts backing it. But theory, predictable at that
Matt: Macro, or micro?
Evolutionist: I have encountered many claims no evidence.
Evolutionist: Define micro and macro please. I feel your definition may be different than science’s.
Matt: Micro - change in allele frequency.
Matt: Macro - one species to another: Radical DNA restructuring.
Evolutionist: Okay, there you have it. How long does micro evolution take. How long does macro
take?
Evolutionist: We are getting somewhere here.
Matt: Micro occurs. But I believe in the genetic 'lessening' of the gene pool through time, not its
increase. Also, when I look at the eye, the heart, etc. The complexity is simply too vast.
Matt: Abiogenesis is an absurdity, mathematically. I just can't buy the change of DNA info on such
levels it is too complex.
Evolutionist: It is not. Look at an embryology book it is very explainable.
Matt: Embryonic recapitulation?
Evolutionist: No, more than phylogenetic recapitulation.
Matt: What do you mean?
Evolutionist: Okay. First how much paleo do you know for the evolution of life to modern.
Matt: Very little. I know mathematics a bit.
Evolutionist: You have to know the evolutionary trend of light reception? Ok, then we can start at the
beginning.
Matt: I know that the DNA molecule is extremely complex.
Evolutionist: Yes and you are going to quote the probability against abiogenesis.
Matt: Do you agree that abiogenesis is impossible mathematically?
Evolutionist: No I don’t. It is not impossible. See, the math is all wrong. For a thorough defense by
someone that knows more than I, go to www.talkorigins.org.
Matt: I’ve seen stuff like that.
Matt: I’ve done my own calculations. Permutations on gene sequence is functionally zero when it
comes to abiogenesis.
Evolutionist: What is the highest level of math you have had?
Matt: Some calculus.
Evolutionist: This will end in no new light being shed. I’ve heard and defended against these
arguments. I am wasting my time. Sorry.
Matt: Okay.
Matt: Are all the missing links found?
Evolutionist: No they aren’t. We will always be finding more. There is only negative evidence for a
deity.
Matt: You mean that the evidence of the eye and how it must have 'evolved' by chance all with
concurrent development before the whole can work is NOT evidence against evolution?
Matt: You’re not qualified as a theologian are you?
Evolutionist: I am a reverend of the Universal Life Church.
Matt: In other words, you're not qualified.
Evolutionist: I am a scientist. I only deal in testable facts not opinion. I have little theological training
but I have philosophical training. You have a blind argument requiring faith. Science does not require
faith.
Matt: I have evidence.
Evolutionist: Then enter it but it must be testable, passing the double blind test. If it is scientific,
publish it. You’ll be famous and change the world forever.
Matt: Oh, I see, any evidence I have must meet YOUR criteria? Okay, I can play that game too.
Evolutionist: No it must meet scientific criteria to be entered. Not mine.
Matt: When you stack the cards, you always win….or think you do.
Matt: The scientific method is not flawless. It is only as good as those who are using it. You are a
sinner. You're mind is affected by sin as is your will.
Evolutionist: But science is self-correcting, theology is not. Prove sin. Prove will.
Matt: What makes you think science is self-correcting? It has lead to survival of the fittest in society.
It is not evolutionary theory that stops the man in the alley from bashing your brains and robbing you,
it is God in his heart.
Evolutionist: Don’t even tell me that it is god. And why is he lurking in the ally anyway? That’s the
monkey juices flowing.
Evolutionist: 79% of all inmates convicted of violent crime is of a Judeo-Christian orientation.
Matt: Really? Wow. Does that also go for atheistic Russia’s political system as well as China's that
have murdered millions?
Matt: Don't you see? Presuppositions cause you and me to see things differently. You must believe in
evolution . Your god is science. You have faith in it. My faith is in God. You have faith that it will
answer all questions.
Evolutionist: No. I know it won’t answer all questions where you think that religion can. I don’t have
faith. Evolution stands with or without my beliefs. It is TESTABLE.
Matt: So is my faith.
Matt: Religion cannot answer all questions. Neither can Christianity. That is what happens when you
encounter God. You encounter areas that you cannot fathom.
Evolutionist: Ok you believe in a god, a non-provable god. Science asks where did he come from.
Your answer is that God is the only causeless cause.
Matt: Yep. Time is a function of matter, correct? Time exists when matter exists. You know this, right?
Evolutionist: We are at an impasse, good day sir.
Matt: Hold on.
Matt: God is outside of time. Therefore, he can be the uncaused cause.
Matt: That is perfectly logical.
Evolutionist: I’m going. Evolution is a fact. It is empirical evidence.

It is dialogues like this that help me understand the areas in which I need to improve. What the
heck is neotony, heterochrony? What is the scientific definition of empirical evidence? Though I’ve read
books on evolution (pro and con), I still have a lot to learn.
Discussion with a Jehovah's Witness about a relationship with Jesus

This brief dialogue with a Jehovah's Witness is representative of a problem they have with a
personal relationship with the Lord Jesus. Since they believe that Jesus is a created thing, they are
not allowed to pray to Him. This means they cannot talk to Him. The question I kept asking was how
do you have a relationship with someone you never talk to? This Jehovah's Witness tried to explain it,
but couldn't adequately explain how anyone can have a relationship with someone they don't talk to.

Matt: I have a question. Do you have devotions, personal devotions?


Ted: You mean prayer?
Matt: Where you open the Bible, read just it, pray, read.... talk to God... pray... read...
Ted: Sure, all the time. That is prayer, read, meditate get holy spirit to understand. Sometimes I get
an answer to my questions that way.
Matt: I feel for you
Ted: Likewise. And if the end comes soon I will think about you.
Matt: How so? if I am wrong, annihilation. So what? If you are wrong, eternal conscious torment.
Ted: Death is an enemy as Paul says.
Matt: Truth is the solution
Ted: I know and I am glad I have it.
Matt: Well, I have Jesus, a relationship with Him. I always have... from the first moment of my
conversion when He came to me. I've always known Him... since then.
Ted: I have a relationship with both the Father and the Son and am on a first name basis with both of
them.
Matt: And do you talk to Jesus?
Ted: No, I call on the name of Jesus by praying to the Father through him.
Matt: How do you have a relationship with someone you never talk to.
Ted: Because he uses me. The Father instructs him how to use me.
Matt: You didn't answer it.
Matt: You and I have a relationship. We communicate with each other.
Matt: I have a relationship with Jesus. I speak to him... I "hear" Him..... He speaks to me...
Matt: relationship.... You don't have that with Jesus.
Ted: Well, actually neither do you.
Matt: I speak with him regularly. I am aware of his presence.... though I admit, I sometimes ignore
it.
Matt: I know the intimacy of communion with Him.... of speaking His name to Him, of praising the
Lamb and saying Holy Holy Holy to him.
Matt: You cannot do that. You cannot pray to Him and talk to Him. I can. You cannot come to Him
and ask Him anything. I can...and I do.
Matt: I have no one telling me to do it or not. I hear Him call me in my heart into His presence.
Matt: Do you have that with Him?
Ted: The Father is the one who gives all gifts. He is the one we need to ask.
Matt: Jesus said "Come to me" (Matt. 11:27-28). Have you done that?
Ted: Yup, he lead me the Father (John 14:6).
Matt: Jesus has all authority in heaven and earth (Matt. 28:19-20). He forgives sin. Have you gone to
Him and asked him to forgive you.
Matt: The Father will honor us if we serve Jesus - John 12:26
Matt: The Father tells us to listen to Jesus - Luke 9:35; Matt. 17:5
Matt: The Father tells us to come to Jesus - John 6:45
Matt: The Father draws us to Jesus - John 6:44
Matt: If you have the Father, then you will come to Jesus, listen to Jesus, be drawn to Jesus, serve
Jesus, etc.
Matt: Is that what you do?
Ted: Once one is drawn to the Father through Jesus then he prays to the Father in Jesus' name. That
is what Jesus taught us to do. And that is what I do.
Matt: And how do you have a relationship with someone you never talk to?
Ted: Because he is used by the Father to guide me with the Holy Spirit. So when I pray to the Father
for assistance and I feel the Holy Spirit it is Jesus who is operating the spirit.
Matt: But, how do you have a relationship with someone you never talk to?
Ted: I understand that you feel you talk to Jesus and he answers you. But most Trinitarians do not
hear Jesus literally.
Matt: Please explain how you have a relationship with someone you never talk to....
Ted: Tell me how you have a relationship with someone who does not talk to you.
Matt: You can't.
Ted: Does the Father talk to you? Do you hear him audibly?
Matt: Now, please tell me how do you have a relationship with someone you never talk to...
Matt: Can you please answer my question?
Ted: I asked you one.
Matt: I answered.
Ted: I want to know if the Father speaks audibly to you.
Matt: The Father calls me into intimacy with the Son (1 Cor. 1:9). So, in order to do that, I must
speak to the Lord Jesus.
Matt: The focus is on Jesus and I think you are simply trying to avoid the issue.
Ted: But you do not hear the voice of the Father.
Ted: If you say you have a relationship with the Father (as I do) then how is that possible if he does
not talk to you.
Matt: I have asked you how you can have a relationship with someone you never talk to. Can you
please explain how that is possible?
Ted: The same way I have a relationship with the Father. I talk to him and he answers me with Jesus.

Matt: You have a relationship with the Father... He speaks to you through the word, right? You pray
to Him, right?
Matt: But, how do you have a relationship with Jesus since you never talk to Him?
Ted: I pray to the Father. He answers by having Jesus sent the Holy Spirit to me.
Ted: I have a relationship with both of them in that way.
Matt: But, how do you have a relationship with Jesus if you never talk to Him?
Ted: One cannot have a relationship with the Father independent of the Son and vice versa.
Matt: Correct. Without either, you have neither.
Ted: Then one cannot pray to Jesus without the Father.
Matt: But how do you have a relationship with Jesus, since you never talk to Him?
Matt: it isn't a relationship with Jesus at all. You don't have one.
Ted: I need to eat dinner
Ted: talk to you later!
Can the Muslim do enough good works to go to paradise?
This dialogue is very typical of discussions with Muslims. Abdul (not his real name) would not stick
to the topic. When he was faced with a difficult question, he tried to go on the attack by not
answering the question. Instead, he attempted to prove Christianity is wrong. But, he failed to
answer the main question I continued to ask him.
When dialoguing with people, it is best to try and keep them on one subject instead of letting them
lead you all around the place.

Abdul: ...download a copy of Quran


Matt: Are you going to paradise?
Abdul: Going to paradise or hell is in the hands of God. But worshipping a false God, makes u that
much closer to hell.
Matt: I asked you a question and you did not answer.
Abdul: I told you. I answered your question. I said, going to hell or heaven is in the hands of God
Matt: I didn't ask you in whose hands it is. I asked if you were going to paradise.
Abdul: I believe God has the answer for that.
Matt: I asked YOU a question. Have you done enough good works? Is your scale tilting in your favor?
Abdul: What scale ?? Where is the scale? Who has the scale? I don't have a scale, I don't see a
scale, its God who is in control and who knows the unseen. It is God who knows where I will go, hell
or heaven.
Matt: "To those who believe and do deeds of righteousness hath Allah promised forgiveness and a
great reward" (Surah 5:9). (A Surah is a chapter in the Quran)
Matt: "O you who believe! If you are careful of (your duty to) Allah, He will grant you a distinction and
do away with your evils and forgive you; and Allah is the Lord of mighty grace," (8:29, online, trans.
by M.H. Shakir).
Abdul: Yea it is Allah who will forgive. It is not in our hands.
Matt: "Then those whose balance (of good deeds ) is heavy, they will be successful. But those whose
balance is light, will be those who have lost their souls; in hell will they abide," (23:102-103).
Abdul: Where is the balance?
Matt: "And We set a just balance for the Day of Resurrection so that no soul is wronged in aught.
Though it be of the weight of a grain of mustard seed, We bring it. And We suffice for reckoners,"
(21:47).
Abdul: The balance is with God.
Matt: "They are those who deny the Signs of their Lord and the fact of their having to meet Him (in
the Hereafter): vain will be their works, nor shall We, on the Day of Judgment, give them any weight,"
online Qur'an, 18:105
Abdul: He is the one who weighs our deeds.
Matt: Is your scale tilting in your favor? Are you doing enough good or will you go to hell? Have you
been good enough?
Abdul: I will know that on the judgment day. Until then, we can just hope for the best>
Matt: That is all you have? That's it?
Abdul: I do good deeds, and leave the rest to God. I do bad deeds and seek Allah's forgiveness.
Matt: 1 John 5:13, "These things I have written to you who believe in the name of the Son of God, in
order that you may know that you have eternal life."
Matt: In Christianity, we can NEVER do enough to please God. He is far too holy and perfect for that.
Therefore HE arranges forgiveness for us, in Jesus.
Abdul: Okay, go on.
Matt: We trust in what JESUS did, NEVER in what we do.
Abdul: I am listening.
Matt: We can NEVER measure up to God.
Abdul: So what Jesus did takes you to heaven, right?
Matt: What Jesus did was pay for my sins so that I might trust in what HE did and not me. That by
faith I am saved and I am saved NOW.
Matt: Then...because I am saved... then.... I serve God with love
Abdul: Is it ok to punish an innocent person in place of the culprit?
Matt: did God punish Jesus or did man do that? God did not punish Jesus. People, by their evil deeds
killed Jesus.
Abdul: I am asking u a question. Answer me.
Matt: God simply let them do it so that we might be saved.
Abdul: Is it OK to kill your neighbor, for your deeds?
Matt: It is not right to punish an innocent person.
Abdul: Do u agree Jesus was sinless?
Matt: Yes, Jesus was sinless (1 Pet. 2:22).
Abdul: Ok.
Matt: Was Muhammad sinless?
Abdul: Is it ok, for a innocent person like Jesus to be punished for all the evil persons in the world? Is
that fair?
Matt: Wrong question. Did God punish Jesus?
Abdul: No. I am asking about your concepts.
Matt: No. God did not punish Jesus. So your question does not apply to Jesus.
Abdul: You believe that Jesus died on the cross for your sins?
Matt: It was the Pharisees and the Romans who worked together to kill Jesus. God knew what would
happen to Jesus and allowed it.
Abdul: Wrong again. It was the Jews who killed Jesus.
Matt: If I am wrong, prove it.
Abdul: God knows everything that happens, and still he lets it happen.
Matt: The Romans nailed him to the cross. There were six trials. Three before the Romans and three
before the Jews...and yes, God let it happen. And while Jesus was on the cross, somehow, someway,
He then bore our sins (1 Pet. 2:24).
Abdul: God also knew that Jews will be killed by Hitler, yet he let it be ??
Matt: Stick to the subject.
Abdul: And Hitler goes to heaven, because his sins has been forgiven because Jesus died for him.
Abdul: And the Jews go to hell, because they didn't believe in Jesus. That's not a fair God you know.
Matt: Hitler denied God. Hitler was not a Christian. He denied God. He is not saved.
Abdul: We Muslims believe that every individual is responsible to his/her own actions.
Matt: Christians believe that too.
Matt: But, how can you possibly believe you can ever do more good works than bad and please an
infinite God?
Abdul: We Muslims don't believe that someone died for our sins.
Matt: Are you good enough to do that?
Abdul: Did u know that the concept God's son dying for the sins of others is not of Christianity? Did u
know that? It existed much before Christianity. In fact 1500 years before Christianity.
Matt: You are not answering my question.
Abdul: You answer me. Why should I believe in Jesus.
Abdul: Prove to me that Jesus is God.
Matt: Are you good enough to do more good works than bad and please an infinite God?
Abdul: Have you heard about ADONIS?
Matt: You do not stay on the subject. You jump around. I have asked you a question and you do not
want to discuss it.
Matt: It is your method simply to jump around and try different attacks. I have answered you and
now request you to answer me.
Matt: Are you good enough to do more good works than bad and please an infinite God?
Matt: I am asking you this question: Are you good enough to do more good works than bad and
please an infinite God?
Abdul: As you say Jesus died for our sins 2000 years ago. What about the people before 2000 years
ago? They didn't have the opportunity to get to know Jesus. Do they go to hell then?
Matt: Are you good enough to do more good works than bad and please an infinite God?
Abdul: What about people like Abraham, people like Moses?
Matt: Are you good enough to do more good works than bad and please an infinite God?
Abdul: What about them?
Matt: Are you good enough to do more good works than bad and please an infinite God?
Abdul: Think about it and read the Quran.
Matt: If you cannot answer me, then tell me you cannot answer.
Abdul: You will get to know what is the truth.
Matt: The truth is in Jesus (John 1:17).
Matt: Are you good enough to do more good works than bad and please an infinite God? If you say
yes, then you are arrogant. If you say no, then you are lost.
Abdul: Millions across the world are learning the truth and reverting to Islam in hoards.
Matt: Well, God did say that in the last days a great deception would come on the world.
Matt: Now, I ask you again, Are you good enough to do more good works than bad and please an
infinite God?
Abdul: Well the deception is already exists.
Matt: "Then those whose balance (of good deeds ) is heavy, they will be successful. But those whose
balance is light, will be those who have lost their souls; in hell will they abide," (23:102-103).
Abdul: Your believing Jesus as God, that deception is true.
Matt: Why do you not answer my question?
Matt: "Then those whose balance (of good deeds ) is heavy, they will be successful. But those whose
balance is light, will be those who have lost their souls; in hell will they abide," (23:102-103).
Matt: I ask you again, Are you good enough to do more good works than bad and please an infinite
God?
Abdul: I answered your question long back.
Matt: No. You answered a question I did not ask.
Matt: I ask you again, are you good enough to do more good works than bad and please an infinite
God?
Abdul: We as humans can do the best to please God, by doing good deeds, but its ultimately up to
god whether he is pleased with us or not. Its all in Almighty God and we will find that out in the
judgment day.
Matt: I did not ask if you could please God by doing your best. I asked you if are you good enough to
do more good works than bad and please an infinite God?
Abdul: I told you, a human being can do his best to please his master.
Matt: Can you please answer the question I ask instead of answering ones I do not ask?
Matt: I ask you again, are you good enough to do more good works than bad and please an infinite
God?
Abdul: I believe in one God and worship him alone, that is the greatest act of good deed in the eyes
of god
Matt: I see you cannot answer my question.
Abdul: On the other hand, If I worship a man, who I think as God, then that act is the worst in the
eyes of God, and I will surely go to hell.
Matt: Again, you did not answer me.
Abdul: I have answered you again, sir.
Abdul: Its you who has closed your eyes. Open them and seek the truth
Matt: Jesus is the way, the truth, and the life (John 14:6).
Matt: I believe what Jesus said. HE alone is the way. Not Muhammad, not your works.
Matt: I have eternal life because of what Jesus did. Not because of what I do.
Matt: You do not have eternal life because you must earn it.

The dialogue simply ended here. Please notice how this Muslim did not really address the question I
asked. He gave answers to questions that were close, but not to the one I kept asking. The truth is
that he knows he is not good enough and as soon as he answers that I would then bring up the Quran
which talks about the scale and then I'd simply ask him if he is doing enough good works? Is anyone
doing enough good works to please an infinitely holy God? I don't see how that is possible.
Instead, people need the free gift of salvation in Jesus Christ who is God in flesh (John 1:1,14; Col.
2:9). Only in Jesus is their forgiveness of sins because it is impossible to please God by what we do.
Discussion on how God can be one person as flesh and spirit.

This dialogue began after I made a comment about the error of oneness theology in a chat room.
Gary responded to me and we had the following dialogue.
Please note that during the dialogue I attempted to get Gary to see how confusing his position on
the Godhead really is. Also, oneness theology teaches that God is one person, not three as in the
Trinity: Father, Son, and Holy Spirit.
I begin the dialogue here with diagnostic questions trying to determine exactly what this guy
believes. I then continued by trying to hammer on his answers looking for any inconsistency.

Matt: Is Jesus a man right now?


Gary: Yes, in the Form of the Holy Ghost, Jesus is man, Jesus is also at the same time, Sprit in
heaven
Matt: Does Jesus, right now, the person of Jesus, have holes in His wrists and feet?
Gary: In Glorified form, in heaven, yes he does
Gary: Have you never read about Thomas?
Matt: Thomas in John 20:28? Yes. And Jesus is a man, right? Therefore, he is a person, right?
Gary: Yes, Jesus is a person, a man and more.
Matt: So, if Jesus is a person, then who is the Holy Spirit?
Gary: Jesus
Matt: So, the Holy Spirit has a body of flesh and bones with holes in his feet and hands?
Gary: Holy Spirit is not a name. Holy Spirit describes an aspect of God.
Matt: In the Bible, the Holy Spirit speaks, Acts 13:2; The Holy Spirit loves (Rom. 15:30); The Holy
Spirit has a will (1 Cor. 12:11). Just like the Father.
Matt: Jesus speaks to the Father. The Father speaks about the Son and to others, while Jesus is on
earth!
Gary: The Holy Spirit and the Father are one in the same
Matt: Each person, Father, Son, and Holy Spirit have a will.
Gary: Yes, God was fully man, and fully God.
Matt: Are the Father and the Son the same person?
Gary: Yes absolutely
Matt: Okay, now, let me ask you this. A person has a will, right?
Matt: If the Son is actually the Father, then that means there is only one will, right?
Gary: I would have to study the definition of person more to adequately answer that
Matt: Faith enough.
Gary: There is God in divine nature, and God in the Form of Christ, so yes they have one will
Matt: A person has "personhood". That is, a person is self aware, speaks, says "you" "yours", "me,"
"mine," etc. Okay, if they have one will then why was the will of the Son different than that of the
Father?
Matt: "And he was withdrawn from them about a stone’s cast, and kneeled down, and prayed,
42
Saying, 'Father, if thou be willing, remove this cup from me: nevertheless not my will, but thine, be
done,'" (Luke 22:42).
Gary: Yes, absolutely. But it is like when you work out and your body screams at you, stop, I am
tired, but in your heart of hearts, you go and drive against the nature of the flesh.
Matt: You mean that Jesus' body was talking to God? It wasn't the person of Jesus that was speaking?
Gary: Not at all, mind body spirit we can have more than one will
Matt: If they are the same person, then they have one will, as you said. But, we see that the will of
the Son was different than that of the Father, hence, two persons.
Gary: I think you are confused
Matt: No, I think you are confused. You are telling me that the body of Jesus was speaking to the
Father. You are telling me that the Father who is the Son has two different wills on the same subject
at the same time.
Gary: Yes, as the son, he has the will of the flesh.
Matt: So, are you saying that the will of the flesh was talking to the Father?
Matt: Then where was Jesus? Was He not a man?
Matt: Did He not have flesh and blood?
Matt: Or was the Spirit of God in heaven and the flesh-man called Jesus on earth?
Gary: Yes, you make my point well
Matt: I see. So then, it really wasn't an incarnation was it?
Matt: It really wasn't God in flesh. It was flesh only.
Gary: Yes it sure was
Matt: If it was, how can this be since you divide Christ up into the Father in heaven and the flesh on
earth. That is not an incarnation.
Gary: NO, it was God putting on flesh, real flesh and blood.
Matt: Then was Jesus God in flesh or not?
Matt: If He was God in flesh, then who was He talking to in Heaven?
Matt: If Jesus is the Father, than was the Father in heaven and in the body of Christ at the same
time?
Matt: Was it the Father that was incarnated in the Son?
Gary: It was God in the limitations of the human form, crying out to the God in his unlimited divine
nature.
Matt: You aren't answering my question.
Matt: Was it the Father that was incarnated in the Son?
Gary: No, it, was GOD incarnated in the Son. Father, is just a hat he wears.
Matt: Then is God the Father, or the Son, or the Holy Spirit?
Gary: I am a son, and I can be a father, and more, but I am only one.
Matt: If Jesus is God, the totality of God, then who was the Father who was speaking to Jesus at the
same time that Jesus was God?
Matt: Can you answer that question?
Gary: God is Father, Son, Spirit, and so much more
Matt: But, Jesus spoke TO the Father. The Father spoke TO the Son.
Matt: Who are they?
Matt: The same? The same person?
Gary: HE is God, the same person.
Matt: The same person speaking to himself in different forms and different names?
Gary: Yes, for our benefit.
Matt: And the same person disagreeing with himself?
Gary: Yes.
Matt: So the Father who was the Son disagreed with the Son who is really the Father, right?
Matt: And the Son did the will of the FATHER which is different than the will of the Son, who are really
the same person, right?
Gary: Yes they are.
Matt: I see... So, the Father and the Son are the same person, but they have different wills?
Gary: Yes they sure could at times.
Matt: They could? So, what you are telling me is that God is really one person, who displays himself
as Father and Son at the same time, who really has ONE will, but displays two that disagree with each
other, right?
Gary: Yes. Flesh disagrees with spirit.
Matt: Wait. Was the Flesh the Father? Because you said that the Son and the Father are the same. If
they are, then the Son was a man, then the Father was flesh, right?
Gary: I think you are confused. God is the same.
Matt: No, I think YOU are confused. I am just repeating back to you what you are teaching.
Matt: You see the confusion in what is being said. That is good.
Gary: I am not teaching anything. I see one mixed up puppy chatting at me.
Matt: Yes you are teaching. You are trying to teach me that the Father and the Son and the Holy
Spirit are all the same person.
Matt: I am trying to get you to see that you aren't making any sense.

Matt: You have the Father and the Son being the same person who disagree with each other, which
really isn't "each other" it is really Himself.
Matt: You have God as flesh disagreeing with God as spirit.
Gary: Timothy 3:16, "And without controversy great is the mystery of godliness: God was manifest in
the flesh, justified in the Spirit, seen of angels, preached unto the Gentiles, believed on in the world,
received up into glory."
Gary: I think you are getting titles mixed up with God.
Matt: If Jesus was flesh disagreeing with the Father, then how can a single person have a
disagreement with himself and have a conversation with himself at the same time?
Matt: Also, appealing to mystery at this point only tells me that you can't logically explain your
position and that you see it is confusing.
Gary: I think you don't get it. I think you are trying to chop up God into the boxes you want to put
him in.
Matt: No. I am trying to get you to see that your view of God doesn't make sense.
Matt: I am not chopping God up, you are.
Matt: You have him part flesh, part spirit, part father, part son who all are not really different, but all
the same person who talk to themselves but it really isn't themselves, it is really ITSELF.
Matt: You have the flesh of God incarnate disagreeing with the Spirit of God in heaven.
Matt: But if that is true, then there is no incarnation and your view of Jesus' sacrifice is useless.
Gary: How so? Please explain?
Matt: You don't have a real incarnation. You don't have the person of God incarnating.
Gary: My view of God makes perfect sense
Matt: It makes perfect sense? hardly.
Gary: God is one God, who holds many positions, yet remains one God.
Matt: You cannot see that the Son speaks TO the Father. They have different wills, at the same time,
on the same subject: the crucifixion ordeal I mentioned earlier in Luke 22:42 in the Garden.
Gary: One person, with many hats, I do not see what is so confusing about that?
Matt: The Father has a will. The Son has a will.
Matt: They each have wills.
Matt: Jesus isn't just flesh. He is God incarnate, the Word incarnate, the Word who was WITH God
(John 1:1,14).
Matt: The Son speaks TO the Father. They converse.
Matt: They have identities
Matt: But you mix them up.
Matt: You have the Son be something that is or isn't God in flesh.
Matt: He is somehow the flesh talking to the spirit as if the flesh has its own personality that can
communicate.
Matt: Is Jesus neither or both sprit and flesh?
Matt: You divide him up into parts and have one part talk to another.
Matt: Is this what it means to be a man? to have your flesh actually SPEAK out loud to your SPIRIT
that is in a different location?
Matt: That makes no sense at all.

At this point, he stopped posting. It was probably because he was getting frustrated trying to
explain the same thing to me. But, he could not see that he wasn't making any sense at all
A Satanist judges God
This conversation with a Satanist demonstrates the difficulty with dealing with someone whose
mind is made up on an issue. All we can do attempt to answer questions and objections and pray that
the Holy Spirit will open his heart and mind.

Matt: Someone said you were interested in talking to me. No biggie. Are you a satanist?
Len: I don't know if I am a Satanist per say. But, how about you start by explaining
the book of Job to me. How is the God portrayed in Job, a merciful and just one?
Matt: God wanted the angels, Lucifer included, to come before him. God pointed out Job. Lucifer
complained, God gave him permission to afflict him. Lucifer did... Job didn't sin in rebellion....Lucifer
was wrong. God is sovereign. He can do as He wishes.
Len: Why would a merciful God give Satan permission to afflict a faithful servant?
Matt: Why not? Isn't it merciful to even let Job live who is a sinner?
Len: Is that "just" and "merciful?"
Matt: Hasn't God been offended?
Len: Job was not a sinner.
Matt: Yes he was... Let's look at this for just a minute.
Len: Job was a faithful servant.
Matt: Yes... but he is human. Humans sin. Are you saying he never sinned?
Len: As a matter of fact, if you read Job, then you know that he was one of the most faithful of God's
servants. That is why he was chosen. So God could win his little bet with Satan.
Matt: Okay. No problem there. Are you saying that Job never sinned?
Len: No. But he was a FAITHFUL servant, was he not?
Matt: Yes, but that does not mean he wasn't a sinner.
Len: He was faithful, and a "merciful" God gave him into the hands of Satan? Are you kidding me?
That is "just" and "merciful?"
Matt: Okay, let's start from the beginning. God is holy, right?
Len: Sure.
Matt: Has God ever sinned, or can He sin?
Len: Nope.
Matt: God is just, right?
Len: Nope.
Matt: Then you are accusing him of sinning, right? If He is sinless, then He is just. If He is not just, He
is a sinner. Which is it? Is He sinless and just or is He a sinner and not just?
Len: No. Sin is an act against God. He can't sin, that would be a conundrum, would it not?
Matt: Yes it would. God cannot do anything wrong, correct?
Len: Don't even give me this "whatever God does is right" bull. No, he is not always just. Sinning
and being "unjust" are two different things.
Matt: If He is not just, then you are accusing him of doing wrong.
Len: I am accusing him of wrong doing.
Matt: So you are judging God then, correct?
Len: Of course.
Matt: So you, a mere person, a sinner, is telling me that you know that God does wrong.
You are sitting there telling me that you are judging God, correct?
Len: Yes. I am. Am I gonna go to hell now? Are you going to judge me?
Matt: I'm not judging you. You are judging God. On what basis do you justify judging God?
Len: I judge God on his actions. I judge him as I would judge anyone else. If he were a
human, he would not be seen as "just and merciful." Damn right I am judging God.
Matt: So, it is not okay for me to make a judgment on you, but it is okay for you to judge God
Almighty? Well?

I do not think that I did all that well in this conversation. My goal was to move him closer to the
Lord, and it seems I failed to do that. All that we can do as Christians in situations like this is to do
our best. It is easy to look back on something like this and find better things that could have been
said. Nevertheless, even failures can be used to teach us
An Atheist Says There Is No Evidence for God
This short dialogue deals with evidence for God's existence. Though the discussion didn't really
examine any proofs for God, it dealt more with Dan's presuppositions and what evidence he would
accept as sufficient to show that God exists.

Matt: Why is it that you do not believe in God?


Dan: Because there is no evidence that he exists.
Matt: You can’t say that because you have not looked at all evidence in the world. That isn’t possible.
Dan: Let’s just say I don’t see sufficient evidence for god’s existence.
Matt: But, if a person asked you what kind of things you’d accept, within reason, as evidence for God,
what would you say? If you have nothing to offer, then you haven’t thought your position through...
and if you haven’t done that, then can you honestly lay claim to the title ‘atheist’?
Dan: Come up with a way that you would believe in unicorns, and I’ll show you a way to fake it. You
come up with an air tight way to believe in unicorns, then get back to me about the illogic of my
position.
Matt: The way to believe in unicorns is to find one, or have pictures of one, or a fossil of one, or a
bunch of people who said they saw one, and they all described, basically the same thing: a unicorn.
That would be a way.
Dan: Well, how about, if he [God] could do something that was clearly illogical, like make a square
circle, and show it to me. Then I would believe.
Matt: A square circle is a non-sequitur. It is self contradictory by definition. God cannot violate his
own nature. Besides how would you comprehend such a contradictory thing if it somehow were able
to be done? You wouldn’t know it and your proof would be useless since you couldn’t understand it.
Besides, it can’t be done anyway.
Dan: Why not?
Matt: Can you violate your own nature? Can you will yourself to be bigger than the sun?
Dan: No, but if there is a god, I’d expect him to exist outside of logic.
Matt: Perhaps, but not against logic since He created it.
Dan: If he created logic, why can’t He do things that run against it?
Matt: If God created the universe and everything in it, then he created it out of his own nature. The
design and natural laws had to originate in His mind. Therefore, it will have His characteristics woven
into it: logic, physics, etc. These are all reflections of God’s awesome creative character. Also, since
God is self-sufficient, He cannot be self contradictory. Otherwise, He could not sustain Himself.
Therefore, He cannot violate His own nature.
Dan: So? Is he limited to the things he built into the universe? Isn’t he omnipotent?
Matt: Yes....
Dan: Why can’t he act against His own universe?
Matt: He could. He could destroy the entire universe. But He chooses not to.
Dan: What a crock. Just like I could stomp the earth and crush all armies with a wave of my hand. I
just choose not to. Your argument isn’t valid.
Matt: Why? Just because God doesn’t’ choose to do something He has the power to do, it does not
mean He does not exist. After all, does it prove that you do not exist if you choose not to do
something you could do? If you choose not to clap your hands right now, does that mean you do not
exist? Of course not.
Matt: Think about this. God choosing to not exercise His will in something is the same as you choosing
not to exercise belief in a god. You could, you just don’t. Both are a lack of action. So, how can you
complain against God for not moving according to your criteria, when you choose to not move at all
and believe in Him?

At this point, the conversation ended.....I believe that Dan was incapable of finding God because he
had a false method of verifying evidence for God. He seemed to require evidence that was naturally
impossible. I attempted to show him the error in his logic.
An Atheist Says He Knows There Is No God
This atheist actually believed he knew for a fact that there was no God. I found that position to be
interesting and, quite honestly, not possible.
Also, he and I discussed faith a bit towards the end.

Atheist: As to religious arguments I haven't found one that can stand up to the logic of atheism.
Matt: Are you a strong atheist or a weak one?
Atheist: Never heard of a weak atheist
Matt: I’ll explain. A strong atheist states that there is no God. He knows there is no God. A weak
atheist, basically, 'lacks belief' in a god of any sort.
Atheist: Then I am a strong atheist.
Matt: Then you know there is no God?
Atheist: As much as knowledge can tell us yes..
Atheist: Maybe it's you who have to catch up on your atheism... Agnostic fits the description pretty
well of a weak atheist...
Matt: That is what I said...which are you?
Atheist: I am a strong. Characteristic human thought, coupled with hope is what religion boils down
to, the unexplained tried to be explained...
Matt: So, you know there is no god?
Atheist: Yes.
Matt: How can you know that?
Atheist: It's a reasonable assumption
Atheist: If you want a definitive answer.
Atheist: Does any Christian bother to look in the dictionary to what truth actually means? There is no
100% anything. Only close to it.
Matt: Then you cannot KNOW there is no God. Your strong atheism is illogical.
Atheist: Let's look at Christianity. It runs on faith. Faith is not logical. It gives credence to unicorns,
goblins and thing s that go bump in the night.
Matt: Nope. The subject is your atheism. Please don't try to change the subject.
Atheist: The subject can jump where ever.
Matt: Your atheism is illogical. You cannot know there is no God. To do that, you'd have to know All
things to know there is no God.
Atheist: I will defend, but also place in attack. Try to defend faith
Matt: One subject at a time....
Matt: You'd have to have seen all evidences to know there is no God. You cannot claim this,
therefore, your atheism is illogical.
Atheist: You can never see all evidences but that does not mean there is a god.
Matt: Correct.
Atheist: No, that means there is not enough information for a conclusion. So we make assumptions as
best we can according to our knowledge...
Matt: But you must concede that your claim to strong atheism (that you know there is no God) is not
logical.
Atheist: My knowledge of the human brain leads me to believe there is no god...
Matt: Then that means there MIGHT be a God, because you don't know all the evidence. Therefore,
you must logically be an agnostic.
Atheist: And so must you... But you picked a side.
Matt: Then it [your atheism] is not logical, but only assumptions you base your atheism on. Your
atheism is untenable.... You must admit that agnosticism is more logically viable.
Matt: If you admit that, we can discuss my faith.
Atheist: I'm not ignorant to say I don't use faith. But only the usage of faith in a situation that
remains provable
Matt: So, are you agnostic or atheist? Which is it?
Atheist: Atheist.
Matt: You've lost the argument. Sorry...

Atheist: Wrong. What your doing is a ploy. You bring me over.. but you stay the same. Either you
must move over as well or the argument is mute in the first place. One can not keep faith and call his
beliefs logical.
Atheist: For a bit I will stray over to the agnostic side. But I am willing to state instances where I
believe prove my contention that there is no god.
Matt: You have been cornered.... It is not logical for you to claim strong atheism. You have not seen
all the facts. Therefore, the possibility of God's existence is real. Therefore, you must admit that
agnosticism is more logical in this situation.
Matt: Alright, Let's talk faith.
Atheist: Alright faith. you first.
Matt: I believe God exists. I have faith that he exists.
Atheist: Proof
Matt: I have none.
Atheist: No proof with faith. So, do you always believe in things that you can not prove?
Matt: No... not at all... I have evidences, but they cannot lead to 100% proof or else all could be
forced to believe. But, if there is enough evidence, I do believe.
Atheist: So you must be agnostic in that sense as well.
Matt: No... because I make choices. Though it is possible for my faith to be proven wrong, I still rest
on the evidences and draw logical conclusions.
Atheist: Evidence....
Matt: Yes....
Atheist: Then you disregard one of the most fundamental rules of the game...
Matt: Which is?
Atheist: "Where ever knowledge is incomplete, there is a place for "faith;" but where ever knowledge
and "faith" conflict, it is "faith" which must be modified or abandoned."
Matt: Or the understanding must be reevaluated.... 'Facts' have been found to be wrong before.
Atheist: Facts are not Truths. They are reasonable assumptions. I will get a dictionary definition for
that one...
Matt: That's fine. So what about it? What kind of evidence would be sufficient for you to conclude
there is a god?
Atheist: Fact - Reality or actuality as distinguished to from conjecture or fantasy; Something known
by observation or experience to be true or real.
Matt: That's good.... Now... what would constitute evidence for God's existence?
Atheist: An instance of superiority... Something humans could not do.. OR not be able to explained
through phenomena but event then...
Matt: That's good... now... what would constitute evidence for God's existence?
Atheist: A universal movement. A stoppage of the planet. Nothing earthly.
Matt: If that were to happen, would you conclude there was a god? Couldn't it be explained in other
ways?
Atheist: I would of course doubt it at first. I would look for an explanation... And for something like
that I would probably find no reasonable explanation..
Matt: If you had could not find one, what would you conclude? Would you conclude that there is a
god? or that you simply don't have all the facts?
Atheist: You never have all of the facts... Reasonable assumptions, remember?
Matt: Then you could not safely conclude it was the hand of God, could you?
Atheist: Nope. that would be the only explanation that I could think of that would have the three
means, opportunity. (forget motive) [I did not understand him here...]
Matt: Then you couldn't know anything for sure, right? That is, if you don't have all the facts, all of
them.
Atheist: Haven't we already agreed you can never have all the facts?
Matt: What you are telling me is that you have no real way of proving or disproving God.
Matt: So then, doesn't it come down to faith based upon evidence? I have evidence....
Atheist: I have to go. Friends just arrived. Can we finish this later?
Matt: If you want....

I'm not sure how it went with this atheist. But I hope some seeds were planted.
Sickness as an argument against God's existence
We jump into this conversation after I entered an atheist chat room and joined a dialogue about
God and sickness. Vic, Phil, and Judy were the main participants. Vic and Phil are atheists and Judy is
supposed to be a Christian, though I have my doubts based on her comments. Nevertheless, this
dialogue was an attempt by me to experiment. I wanted to ask more questions than I answered and
thus avoid a defense posture in an antagonistic setting. I don't know if I succeeded. But at least I gave
it a try.
Too many times the Christian is left defending himself while the atheists gang up and demand
explanations for difficult issues. With this dialogue I thought I'd see if I could get the atheists to
explain and defend their positions by asking them to make judgments on various issues.

Vic: God cannot be good if he allows people to get sick with cancer because cancer is bad and God is
supposed to be good.
Matt: So... why is it that God is NOT supposed to make us sick or something? I mean, why not?
Maybe he has a good reason. Maybe there is a good in it.
Matt: Then again, maybe this is just what the world is like with sin in it. Besides, God doesn't owe me
or you anything.
Vic: God, who is our father, should not do bad things to us, such as give us cancer.
Matt: oh.... Exodus 4:11 is a problem for you then. It says in there that God makes the eye blind, the
ear deaf, etc.
Phil: Matt, do you really think anyone here accepts that moldy text created by smelly sheep herders?
Matt: I don't know.... is it an a priori that the texts are not reliable?
Vic: Matt, Then he is not a good God is he?
Matt: Vic, why would he not be good if he makes someone blind?
Phil: Matt, perhaps sadistic would be the right term.
Phil: Matt, is it the opposite? Given the many varied versions of God, the world-spirit, etc., why would
it be wise to take the Bible at face value?
Matt: The Bible is historically accurate, powerful, prophetic, etc.
Matt: You aren't answering my question.... I see God as good... I don't understand what he does
sometimes. But it does not change his nature. He is infinitely above me and permits things to happen.
I DO take some things on faith.
Vic: Matt, if I were to make you blind, by hitting you in the back of the head with a wooden board, we
could all agree that I had done a bad thing, right?
Matt: You.? Yes. If it were God...? Who am I to question?
Phil: Matt, that is at best a statement based on faith, and has little to do with any objective criteria.
Matt: Not true... I can look at the Bible and see its textual reliability, its spiritual truth, its accuracy,
its prophecy, its patterns, messages, and make an intelligent decision based on it. Some things within
it, I must take on faith. That is perfectly rational.
Phil: Vick: Since God is beyond any human judgment, it really doesn't matter to a Christian. God can
hurt anyone he/she/it wants to, and well, that's tough.
Matt: Phil. That is correct.
Vic: Matt, then you believe everything God does is automatically good, righteous, just? That he can do
no wrong? That if he hits someone in the nose, it is good thing, although if I were to do that, I would
have done a bad thing?
Matt: Vic, correct. God can do no wrong.
Matt: If God hit somebody in the nose, why did he do it? Was it to get his attention so he would jump
out of the way of a train?
Phil: Vic, in essence, Christians are, well, sadists.
Vic: If he cannot do wrong, he is not very omnipotent then is he.
Matt: Of course he is omnipotent, but he cannot violate his nature. Your logic is flawed. Please try and
think more clearly.
Phil: Vic, they enjoy the idea of an all-powerful God messing with them from time to time, so they can
feel special in God's eyes.
Matt: Well, if you're going to speak of God, at least do it realistically.
Phil: Suffice it to say, there is no objective criteria for proving the validity of the Bible's claims.
Matt: Perhaps, but does that mean that it is not true? or that God is not knowable? Besides, it is
historically and archaeologically accurate. That is validation.
Phil: Matt, no, but it does mean that I shall suspect individuals who tell me to believe it or else.
Vic: Matt, if God gives someone cancer, and they die from it, what was the purpose of that? Or can we
write that one off as another example of, God works in mysterious ways?
Phil: You must either accept or reject the Bible's claims at face value.
Matt: Vic, let me ask you, if there is a God and he encompasses the universe, is it possible that he
would work in ways that are beyond us? Is that possible?
Phil: Matt, if there was a world-spirit and it encompasses the universe... is it possible that it would
work in ways that are beyond us?
Matt: yes....
Matt: Vic? Well?
Vic: Matt, of course it is possible, although I would say it is unlikely. I don't see any reason why a god
would not reveal all to us, and not try to hide things from us.
Matt: If it is possible, then doesn't that mean that faith is necessary and logical at this point?
Phil: Matt, No.
Matt: Phil, why not?
Matt: Hypothetical situations is a tool of theory and learning.
Phil: Matt, hmm, because possibilities don't create probabilities, or certainties.
Matt: But logic is still logic and God, if he made logic, can be found in it...
Matt: Faith becomes logical when there is enough evidence to support it.
Phil: But logic is still logic and giant pink invisible elephants, if they made logic, can be found in it...
Judy: Matt, you cannot prove God. If you could, then I would want nothing to do with whatever "god"
you prove.
Phil: Matt, Judy is a Christian, by the way.
Judy: There you go blowing my cover again.
Phil: Or she was the last time I chatted with her. Perhaps she has given herself to the dark one these
days.
Judy: No way...even more fundie and conservative than ever.
Phil: Good, glad to see things never change.
Matt: I don't intend to prove God....
Phil: Matt, you could have fooled this crowd.
Judy: Matt, what do you intend?
Matt: I am trying to learn how atheists think so I can refute them better.
Phil: Judy, he's here to dazzle us with verbiage and interesting sentence structures.
Vic: You should be able to prove god exists. If he does, it should be entirely possible to produce a
picture of him, or setup some press conference with him
Matt: I have noticed a lot of condescension among atheists when speaking to Christians.
Phil: Matt, hmm, well, it's just good-humored banter.
Judy: Ha ha. So that's why you came here with a challenge and you were attempting to prove God's
existence by logic and hypothetical situations?
Matt: Judy, no... to see how you guys argue.... to see your logic... style, insults even.
Judy: You guys?
Matt: Yeah, you seem to be siding with them. Are you going to convert to atheism?
Judy: No. You're the Christian preaching your logic.
Matt: Do you want me to preach to you?
Phil: Matt, preaching is not allowed on this channel.
Matt: Just kidding.
Matt: You atheists reject a priori the miraculous. What can I provide that you would not, by necessity,
interpret in humanistic naturalistic presuppositions.
Matt: Your assumptions make it impossible for you to be objective and to accurate examine all the
facts.
Phil: Matt, we could say the same of you old boy.
Matt: Phil, yes you could... but at least I do not rule out the miraculous.... therefore, I am open to it.
You, however, are not. Which is more 'open' to truth then?
Vic: Matt, I do not, although I have yet to meet a Christian who has not rejected the claims of the
miraculous by Hindus a priori.
Phil: Matt, I don't think the subject of the miraculous has been brought up. Still, I am in total
agreement with Hume when he writes that the idea of proving miracles in and of itself undermines the
whole worth of miracles.
Matt: Vic, please clarify.
Matt: Phil, miracles simply are... they happen.
Phil: Matt, for you, I'm sure they do.
Matt: All knowledge cannot be ascertained by logic or experiment.
Judy: Hmmm.
Vic: Matt, are the miraculous events witnessed by Hindus the result of Aryan Gods, or the Hebrew
Gods?
Matt: They are demonic miracles. Jesus stated that the ungodly can perform miracles... but not by the
power of God.
Vic: Matt, the Hindu gods being demons then?
Matt: Vic, yes.
Phil: Vic, Well, any other god would be a demon, or a mere human creation, apparently.
Matt: Phil. That is correct.

At this point the conversation just died as often happens in chat rooms.
I don't know if I accomplished anything in this dialogue, but it is at least another step in the right
direction of trying to find ways to answer atheists.
Discussion with an obnoxious atheist
Nick is an atheist who sought me out for a dialogue. Not all atheists are as obnoxious as this one.
I tried to be civil with him but he became quite rude. Such discussions rarely accomplish anything. I
bleeped out a few of his "colorful" words. I failed to save the first few lines so it just jumps right in.

Nick: I am an Atheist.
Matt: It seems to me you already have you mind made up and trying to change it won't make any
difference....
Nick: Nothing funnier than self-righteous indignation...
Matt: You want me to be reasonable, but you have started off unreasonably.
Nick: OK .. let me re-approach.
Matt: Good idea.
Nick: Matt. I love your dialogues with Atheist on carm.org. It's an extremely intelligent site. I Love
it.
Matt: Thanks.
Nick: I may learn from it.
Matt: Good.
Nick: But I have questions.
Matt: Okay, that was better, now go ahead and attack. I'm ready.
Nick: Hope you log this one.
Matt: You want me to?
Nick: I wanna see it appear on carm.org
Nick: ;-)
Nick: Again let me repost: What makes you so sure that your Christian idea of the universe is better
than all the other religions? Where's your evidence to disprove the validity of other religions?
Matt: That is quite a question. Let me wade through it slowly.
Nick: Just try it. I know your going to wimp out on this one. ;-)
Matt: First of all, I don't begin with trying to disprove other religions. I begin with demonstrating the
validity of Christianity.
Matt: Once that is done, the other ones fall by the wayside. The Bible is quite reliable historically,
archaeologically, prophetically, etc.
Matt: It has Jesus' words and deeds in it, which are, to say the least, miraculous. In short (very
short), I believe the eyewitness accounts of His miracles....of the fulfilled prophecies...of the accurate
accounts, historically, of the Bible, etc. It is quite logically to conclude it is truthful.
Nick: What, you think there haven't been thousands of gods invented throughout human history ???
Matt: In it, Jesus said that He alone was the way, the truth, and the life.
Matt: Therefore, Jesus said all other systems are false. I believe what Jesus said. Simple.
Nick: Why do you twisted morons insist that these stupid rules your Christian masters made up from
thin air actually apply to everybody?
Matt: I think you missed the whole point.
Matt: You ignored the reasoning...
Nick: Allah created the universe. The Koran is the story (Islam)
Matt: Ah, I see... you ask for reason and when I give it to you, you ignore it and make emotional
accusations.
Nick: Shiva created the universe....mahabharata is the story (Hinduism)
Nick: Spirits created the universe...our ancestors tell the story Indians)
Nick: God created the universe......The bible is the story (Christianity)
Matt: yeah... ? so?
Nick: Pink Turtles created the universe... Told by a friend at a party (drunkenness)
Matt: Jesus said they were all wrong when He said He was the way, the truth, and the life (John
14:6). I believe Him.
Nick: Now, which one of these is valid? If one is valid, then all others must be invalid. Or none are
valid at all.
Matt: Now, please, can you tell me why I shouldn't believe what Jesus said?
Nick: For the world cannot be created more than once by different entities if it was not destroyed
first. The Koran can't be the story of creation if the Bible is.
Matt: Jesus said they were all wrong when He said He was the way, the truth, and the life (John
14:6). I believe Him. Now, please, can you tell me why I shouldn't believe what Jesus said?
Nick: Matt, I will tell you why this way, how can you reasonably reject every other religion in the
world except yours? They've all got musty old books of myths.
Matt: ... and please, one comment at a time.....
Matt: Can you please tell me why I should reject the words of Jesus who claimed to be God in flesh,
fulfilled the prophecies, rose from the dead, etc.?
Nick: Maybe because he died, because he was a minor rabble rouser and pissed off the authorities.
Then he whined like a ***** when they pinned him to a stick and left him to dry in the sun. You tell
me.
Matt: You wanted reasonable discussion. So, please give me a reasonable answer to my question.
Matt: Okay, I see you don't really have a good reason. I have given you some quick, basic reasons
for my faith and why I believe what I do and you've offered nothing of substance at all. So, please, try
again.
Nick: Matt ... it's text written by moronic peasants who believe any pie the sky is by and by.
Matt: By the way, if you can't offer something, I think I'll just end this discussion and go do
something else.
Nick: Why do you theists insist on misusing the word "faith" when you really mean "confidence"?
Matt: Come on, last chance for a reasoned comment from you with some evidence... if not, I'm going
to leave you to your own thoughts.
Nick: I just don't get it. It must be some huge mental problem where you have to project your
baseless beliefs on those who are demonstrably free of them.
Matt: Last chance....
Nick: It bothers you immensely, so you have to pretend that we are just as stupid as you.
Nick: Faith means ungrounded belief *without* evidence or reason.
Matt: You started it, please finish this....
Nick: You try and zap me with my illogic.
Matt: Later....
Nick: Well, here you are becoming just another arrogant theist *#@^#@*. You think that real
meaning only comes with god-belief. Piss off. Or try to understand.
Matt: Is that your best come-back? You began this by stating you wanted facts and reason.
Matt: I'm asking that of you and you don't give me any.
Nick: If you want to call this Jesus assertion faith, then fine, all you are proving is that you are a
solopsist. For me, the difference between this and *religious* faith, is that my sense data can be
corroborated by others. If it cannot, then I think that is a reliable indication that I am deluded in some
way. There's plenty of evidence that humans can be deluded in many different ways, so it would be
foolish of me to think that I or anyone else are beyond delusion.
Matt: Instead, you insult me.
Matt: Okay, I'll see you round....
Matt: When you are ready for a serious discussion, contact me again.
Matt: Signing off.
Nick: Matt your faith in "his" words is clearly nothing more than a psychological delusion, which your
mind has manufactured in order to give you a feeling of worth and purpose, that you should have
developed as a child.

I've noticed that a significant number of atheists who profess a desire for rational debate often
degenerate their own arguments into accusations and insults. When this happens, it means they have
nothing rational to offer. I point that out and if they don't calm down, I just leave.
On the other hand, I have had some very good conversations with a few atheists who were polite
and reasonable. We disagreed, but neither side ended up in name-calling.
One more thing, he misspelled "solipsist" which is someone who believes in solipsism: "The theory
or view that the self is the only reality."
Discussion on logical absolutes as a proof for God's existence .
We jump into this discussion with an atheist after a challenge to prove that God exists. But, I would
like to note that sometimes in the atheist discussion rooms there are bystanders who will mock, insult,
and say vile things about the Lord while the conversation between a Christian and an atheist goes on.
Of course, the insults are hurled at the Christian. This was happening here. I have edited out all of the
foul language by them. But, it did detract slightly from my ability to concentrate. This is one of the
tactics I have seen atheists use in text based chat rooms on the Internet. So, be aware of it and be
prepared. Nevertheless, here is the dialogue I had with "Bill" an atheist.

Bill: Alright, try and prove God exists..


Matt: Okay.....Are there such things as logical absolutes? For example... A cannot be both A and not
A at the same, time.
Bill: I believe so.
Matt: There are logical absolutes.
Matt: Now, in an atheistic presuppositional platform, how do you account for the existence of logical
absolutes?
Matt: Do they reside in matter? Can they be quantified, tested, put in a jar?
Bill: Well, that depends. According to Quine's holism there are no absolutes and any principle can be
held eternally by changing any number of other principles
Matt: But to say there are no absolutes is an absolute and is self-defeating.
Bill: Not really.
Matt: Yes. To say there are no absolutes is an absolute statement. It is either true or false. If it is
true, then it is false...which is logically contradiction which proves my premise to begin with.
Therefore, it can only be false and there are such things as logical absolutes.
Bill: If one rejects the principle of non-contradiction. Of course, most people wouldn't want to do that
because things get messy quickly
Matt: But on what basis do you reject the principle of non-contradiction?
Bill: Because we can arbitrarily choose to accept any statement eternally if we make enough changes
to other parts of our total system of knowledge.
Matt: If you do so based on logic, then you are using logic to defeat itself which is not logical. Then
you have nothing but relativism.
Bill: well, according to Quine all knowledge is subjective.
Matt: Is it subjective that all knowledge is subjective? If so, how can he say that since it is subjective?

Bill: No, you just don't understand. It is perfectly acceptable to say that subjectively all knowledge is
subjective. You're just dismissing the possibility because you don't like it.
Matt: Not at all. I am using logic to address it. Logic is not subjective. Subjectivity is relativism. If all
things were subjective, then nothing is true....except the notion that all things are subjective... which
means the statement itself is subjective and not absolute.
Bill: and the problem with that is?
Matt: The problem is that it is self defeating. I'm not saying that some forms of knowledge cannot be
subjective. I am saying that there ARE logical absolutes. That is the issue.
Matt: Again... do logical absolutes exist? If you say no, then you are giving me an absolute; namely,
that no logical absolutes exist.
Matt: Here is the point. Logical absolutes exist. They are, by nature, conceptual absolutes.
Matt: Conceptual absolutes exist in the mind. They do not reside in matter.
Matt: These logical absolutes cannot be quantified or tested in a lab. Yet, they exist.
Matt: In fact, scientists USE these logical absolutes as a basis for verifying their science.
Matt: The problem for the atheist is accounting for their existence.
Matt: Since the logical absolutes are conceptual, they transcend all people at all time and are absolute
in all circumstances... since they are absolute.
Matt: Conceptual Absolutes cannot be accounted for in an atheistic worldview. But they can be
accounted for in a theistic one.
Matt: The Absolute God with and absolute mind, has conceived of the logical absolutes. They are a
reflection of His mind.
Matt: At least I can offer an explanation for their existence where the atheist cannot.
Bill: The basis of science and scientific knowledge is exactly why Quine discussed the possibility of
rejecting certain aspects of knowledge and logic including the law of non-contradiction.
Matt: if the law of non-contradiction is dismissed, then it isn't a law is it? That would also mean that
we could go around contradicting ourselves all the time and that'd be fine...right?
Bill: I have not yet been convinced of the necessity for logical absolutes
Matt: If there are not logical absolutes, then you have no logical basis for your statements. It would
then be purely subjective and meaningless.
Bill: The criterion of meaning is not necessarily based on logic.
Matt: How do you know? Are you using logic to substantiate that or subjectivity?
Bill: Quine specifically spoke of the relation of knowledge to observation. If an observation conflicted
with a statement of theory or knowledge, than that statement could be held in spit of this
'contradiction' with observation by making modification to knowledge systems elsewhere, such as
modifying or even rejecting laws of knowledge if necessary.
Matt: What logic is he using to say that?
Bill:
Matt: you have yet to justify the necessity of logical absolutes
Matt: Without them, you cannot prove anything.
Bill:
Matt: you have yet to justify that statement.
Matt: Unless you can refute that statement, it is true.
Bill: You can say they exist all you want, but that doesn't mean anything.
Matt: Alright.... well, I guess, I'll see you later....

In this dialogue, I had to filter out all the insults and foul language from the other atheists that
were interjected in our conversation. Their foul behavior combined with Bill's unwillingness to
acknowledge the existence of logical absolutes, made continuing the discussion very difficult and it
became fruitless.
I suspect Bill saw the logical problem of his system; namely, that to say there are no logical
absolutes, leads to relativism. With a relativistic system, there can be no real truths. This is a
philosophically dangerous slope to rest on. Nevertheless, I suspect that with him the alternative of not
being able to account for logical absolutes as an atheist and my assertion that they reflect God's
thinking, therefore, God exists, was something he could not and would not allow to occur. Therefore, I
suspect he tried to keep the argument away from absolutes and into subjectivity. Nevertheless, this is
a good example of the need to define terms and establish the necessity of absolutes. You see, without
them, no truth can really be known.
Two Catholics say baptism is necessary for salvation .

In this dialogue, I had entered a catholic discussion room on the Internet. There were only these
two people in there and we quickly got into a nice discussion. I do not know if I accomplished
anything with this, but at least you can see how difficult it can be discussion this issue with people.

Kate: Welcome. Are you Catholic?


Matt: No. I'm Christian.
Kate: Catholics are Christian! :-)
Matt: Some are.
Kate: I could agree with that statement. Not all who claim to be Christian are Christian either...
Matt: That is very true.
Will: Hmmm
Matt: I have problems with catholic theology and never assume a catholic is saved. No offense meant.
Will: No offense taken, why though?
Matt: I find many of the Catholic additions to be troublesome.
Will: Like?
Matt: Particularly the idea that we must be baptized to be saved, and take the sacraments, etc., in
order to "keep" our salvation -- if it is possessed at all.
Kate: Well, Jesus Himself commanded Baptism.
Matt: Jesus also commanded that we love God and love our neighbors, but not doing them properly
does not mean we are unsaved.
Matt: But, Catholic doctrine, from what I understand, teaches that grace is infused into a believer. The
believer is then enabled to do good works by which God then judges his merit as to salvation.
Kate: Not quite. Salvation is a free gift of God. Let's take it one at a time.
Matt: I believe it is a free gift. But, I see the Catholic theologians teach that it is maintained by
obedience to Catholic rules and regulations, etc.
Matt: On top of that, there is prayer to Mary, purgatory, etc. I find these to be unbiblical.
Will: Acts 22:16 "And now why tarriest thou? Arise, and be baptized, and wash away thy sins, calling
on the name of the Lord."
Matt: The thing that washes away our sins is calling on Jesus, not the water. It is the blood that brings
forgiveness (Lev. 17:11; Heb. 9:22), not water.
Matt: Baptism is a covenant sign.
Kate: You must be overlooking the verse that say "baptism now saves us."
Kate: 1 Pet. 3:21, "Whereunto baptism being of the like form, now saveth you also: not the putting
away of the filth of the flesh, but the examination of a good conscience towards God by the
resurrection of Jesus Christ."
Matt: The context is important. It says "and corresponding to that baptism now saves us."
Corresponding to that" in the Greek word, "antitupon." [The KJV says "like figure]
Matt: It means a "type" or a "representation." Now, baptism corresponds to something. The question
is what does it correspond to?
Matt: The verse before it speaks about Noah being saved in the Ark and the floods destroying the
wicked.
Matt: So, does baptism correspond to the flood or to the ark? Which saved Noah?
Will: And you have to be in the ark to be saved :)
Matt: Yep, and they entered the ark by faith. And notice, Peter goes on to say that it is not the water
that saves, but an appeal.... faith!
Kate: But it is baptism. It is the outward sign, instituted by Christ to gain Grace.
Matt: They went into the ark by faith. The same faith that justified Abraham ( Rom. 4:3).
Matt: We are justified by faith (Rom. 5:1), not faith AND something else.
Matt: Otherwise we have the admonition of Gal. 3:1-3 invoked where Paul says that the Galatians
were foolish to think that they could perfect in the flesh what God had begun in the spirit.
Will: Except a man be born of WATER AND OF THE SPIRIT, he cannot enter into the kingdom of God."
Matt: That verse is in John 3 and the "water" there can easily mean the water of the womb.
Matt: Note that Nicodemus refers to entering back into his mother's womb and Jesus responds by
saying that that which is of the flesh is flesh and spirit is spirit.
Will: Yes but you must be born again of water and of the Holy Spirit.
Matt: Born of the water can mean natural birth. Read the context in John 3.
Matt: That is why Jesus said you must be "born AGAIN"
Kate: If the water in John 3 means the water of the womb... that still doesn't release one from
"baptism of the spirit" which, through the outward sign, in obedience to Our Lord, we accomplish
through Water Baptism
Will: However, it DOESN'T mean the water of the womb
Matt: You mean your opinion is that it doesn't refer to the water of the womb?
Will: And you mean that YOUR opinion is that it does
Matt: Correct. Opinions are only opinions. But the Bible says we are justified by faith ( Rom. 5:1 -
"Therefore having been justified by faith, we have peace with God through our Lord Jesus Christ" -
NASB). That settles it.
Matt: The promise is the promise given to Abraham by God to justify people by faith. Covenant!
Kate: I do not deny that baptism is a covenant sign, (read that "outward sign"), instituted by Christ,
to gain Grace.
Matt: Gal. 3:8-9 speaks of this: "And the Scripture, foreseeing that God would justify the Gentiles by
faith, preached the gospel beforehand to Abraham, saying, "All the nations shall be blessed in you."
9
So then those who are of faith are blessed with Abraham, the believer" (NASB).
Matt: We don't gain grace by something we do!!!! Otherwise it isn't grace! Rom. 11:6 says, "But if it
is by grace, it is no longer on the basis of works, otherwise grace is no longer grace."
Will: What does it mean to arise and be baptized and wash away your sins John?
Matt: That is Acts 22:16. Again, calling upon His name is what forgives us of our sins.
Will: Matt, give the quote not just the reference.
Matt: "And now why do you delay? Arise, and be baptized, and wash away your sins, calling on His
name" (Acts 22:16).
Matt: What forgives us? What washes our sins away? Water or Christ's blood?
Will: Be baptized and wash away your sins? You tell me.
Kate: To deny the requirement of baptism is to rip pages from the Bible.
Matt: Water doesn't wash our sins away. Christ's shed blood does that.
Will: But, baptism is the method Christ used to wash away the sins.
Matt: I didn't say baptism isn't important.
Matt: Did Jesus baptize anyone? No.
Matt: In 1 Cor. 1:14-17, Paul stated he came to preach the gospel (which saves according to 1 Cor.
15:1-4), NOT to baptize.
Will: Does he say that no one was baptizing?
Matt: No. But, in Acts 10:44-47, after Peter was preaching, people we speaking in tongues, and
glorifying God. THEN, they were baptized. They were already saved.
Will: Where does it say that they were already saved Matt?
Matt: Think about this. They are speaking in tongues and glorifying God after hearing the preaching of
Peter. Tongues was a gift from God to the church not to those unsaved!
Kate: Jesus didn't baptize, but He commanded the Apostles to "go forth into all nations, baptizing
them in the name of the Father, the Son and the Holy Ghost."
Matt: Paul wrote what the gospel was in 1 Cor. 15:1-4 and he stated that it is what saves. He did not
include baptism. In 1 Cor. 1, Paul said he came to preach the gospel, not to baptize.
Will: Can you please quote the verse here?
Matt: 1 Cor. 15:1-4, "Now I make known to you, brethren, the gospel which I preached to you, which
also you received, in which also you stand, 2 by which also you are saved, if you hold fast the word
which I preached to you, unless you believed in vain. 3 For I delivered to you as of first importance
what I also received, that Christ died for our sins according to the Scriptures, 4 and that He was buried,
and that He was raised on the third day according to the Scriptures."
Will: Part of the Gospel that they had to believe is being baptized. Believe AND BE BAPTIZED AND
WASH AWAY YOUR SINS
Matt: 1 Cor. 1:14-17, "I thank God that I baptized none of you except Crispus and Gaius, 15 that no
man should say you were baptized in my name. 16Now I did baptize also the household of Stephanas;
beyond that, I do not know whether I baptized any other. 17For Christ did not send me to baptize, but
to preach the gospel,"
Will: So, others baptized. He preached the Gospel to those who had to believe and be baptized
Matt: Do you read Paul's words there?
Matt: The gospel is what saves (1 Cor. 15:1-4).
Will: Yes. Did you read my reply?
Matt: Yes.
Matt: Paul came to preach the gospel, not to baptize (1 Cor. 1:14-17).
Will: Yes and part of that gospel is to believe and be baptized
Matt: No. If it were, then he would have included baptism in the gospel. He didn't.
Will: He preached baptism. Others baptized.
Matt: Baptism is an outward sign, a public declaration of unity and purpose and acceptance of/with
God. I think you need to study 1 Cor. 1:14-17 more thoroughly.
Matt: Baptism isn't what saves us. Faith in Jesus does that ( Rom. 5:1).
Will: Matt, does Rom 5:1 say that baptism does NOT save us?
Matt: No, but it tells us what does!

The conversation ended here and nothing of importance was said afterward. They were courteous
people.
Discussion with a Catholic on interpreting the Bible
The following dialogue is a good representation of the importance of presuppositions. In other
words, you will interpret things according to your presuppositions. Dan raised an excellent question
regarding how do we know who has the right interpretation for something. I really didn't focus on the
standard answer to that question dealing with interpretive methods. Instead, I tried to demonstrate
that we are right in so far as we agree with the Bible.

Dan: Hi. I am a Catholic and I know that I am in the true church because it was started by Jesus,
expanded by the Apostles who gave the power of leadership to the bishops. My question is how do you
know that your beliefs are right when other Protestants claim to do everything that you do but still
come with different believes?
Matt: I base it on the Bible.
Dan: But Luther said the same thing and he believe in the real presence of Jesus in the bread and
wine. Other Protestants do not
Matt: That's nice.
Dan: That is my question. All Protestants say they base it on the Bible, but each church comes with
something different
Matt: Purgatory? Praying to Mary? Penance? Indulgences? Are these in the Bible? No.
Dan: Is it in the Bible is not the right question. Because each church claims that different things are
in the Bible. Luther and Catholics claim Eucharist is in the Bible other churches do not. My question is
how do you know who is right?
Matt: Is it in the Bible or not? That is the issue...
Dan: Yes, but different people interpret the Bible differently. Who is right?
Matt: The Bible is right. Stick to it.
Dan: Yes. But who has the right interpretation of it?
Matt: God. Read the Bible and stick to it.
Dan: To whom does God reveal his interpretation?
Matt: To Christians
Dan: That is my problem. You read it and you say that this fragment means this. I say that it does
not mean this. Someone one else reads the same thing and he says that both of us are wrong because
it means this.
Matt: Stick to the Bible. Listen.....
Matt: Jesus said that His sheep hear HIS voice and follow HIM, right?
Dan: Yes
Matt: The Bible is God's (Jesus') word, right?
Dan: Yes
Matt: Christians will follow Jesus, right?
Dan: I cannot answer this one
Matt: Christians DON'T follow Jesus?
Dan: Yes they do
Matt: Jesus said Christians hear his voice and follow HIM, right?
Matt: Where is Jesus' voice found? Is it in sacred tradition? Is it in my words? Where is Jesus' voice
found?
Dan: In the Bible and in those who are filled with Holy Spirit.
Matt: The Holy Spirit bears witness of Jesus.
Dan: Yes
Matt: Jesus said His sheep follow HIM and listen to HIS voice... none other.
Dan: Yes
Matt: Whose voice have you been listening too?
Dan: God's.
Matt: Jesus points to Himself, right?
Dan: Yes.
Matt: Then you must study the Bible to find His voice, right? It IS His word, right?
Dan: Yes, I read it almost everyday for 4 years.
Matt: Good... now, which dominates your interpretation? The Catholic Church over it or IT over the
Catholic Church? Who's voice do you listen to?
Dan: None, I do not find any disagreement between the Bible and the church. And that is my
problem. You find it. I do not. You read the Bible I read the Bible. Who is right?
Matt: None? Then you are not listening to Jesus are you?
Dan: According to you, but not to me and other people. How can we decide?
Matt: First, get on your knees. Pray to Jesus. Ask him to reveal the truth to you... Read HIS word.
Subject yourself to IT. Because it is His voice.
Matt: Look.... I am pointing you to HIM, not a church.... A church doesn't save. Jesus does.
Dan: Most days I pray over an hour. I read the Bible then I think about it. Many times I did not
understand something. I kept on reading. Within few day or months I read something that cleared up
the thing I did not understand
Matt: That's good.
Matt: Do you pray to Mary?
Dan: No I pray through her, but I do not want to get into any of these beliefs because we will not get
anywhere.
Matt: The Bible says there is ONE mediator, not two. Praying THRU her is just a way of saying you
pray TO her...but without the obvious difficulties involved.
Dan: Do you ask other people to pray for you?
Matt: They are here with me. They are not dead. Jesus is risen, the one mediator. Others are not
my mediator. Neither is Mary. Your praying through Mary is praying TO her.
Matt: Where is this found in the Bible?
Dan: Do you ask others to pray for you.
Matt: Yes. I do.
Matt: Are they my mediator between God and me? No.
Matt: Where does it say we can pray to or through Mary?
Dan: I ask my spiritual mother to pray for me. That is all: I say Mary pray for me. Not Mary grant
me this or that.
Matt: Okay, we're done.....
Dan: You see i told you I do not want to go into that because this will happen.
Dan: But you showed one thing. That you are not open minded. When you hear something you do
not want to hear you just cover your ears.
Matt: What happened is that you deny God's word.... We cannot go further... we're done. You don't
get it.
Matt: I am trying to get you to trust God's word in totality and sufficiently.
Matt: You do not want to do that.
Matt: You want to hold on to what you have been taught by the RC church.
Matt: I do not doubt my salvation or my assurance that Mary is not my mediator.
Matt: Why? Because the Bible does not teach it. Simple.
Matt: If you deviate from the Bible, you WILL end up in error. I tried to get you to see it. You do not
want to.... You want to excuse the unbiblical practice you have. Therefore, how can I get you to see?

Matt: I cannot. It is your choice. Therefore, we are done.


Dan: So ok. You are not close-minded but you believe that you are right and every one else is
wrong. And that is my question. How come you are so sure that you are right?
Matt: I am not sure I am right about everything.
Matt: I am only right in so far as I agree with the Bible.
Dan: that what i meant
Dan: I and other Protestants who have different beliefs then you are also sure that they agree with
the Bible. How do you know that we are wrong?
Matt: Read Rom. 14:1-12. I'll wait.
Dan: Ok [nothing for a couple minutes]
Dan: and
Matt: Did you read it?
Dan: Yes
Matt: What is the point in it?
Dan: Not to judge others.
Matt: How about allow differencing opinions...? on non essentials? right?
Dan: Yes.
Matt: Good
Dan: And?
Matt: So, according to God, there is room for variation in non-essential beliefs, right?
Matt: Therefore, it is not essential that we Protestants agree on every detail, correct?
Matt: But, we ARE to agree on the essentials of the faith.
Matt: Do you know what those essentials are?
Matt: http://www.carm.org/doctrine/3essentials.htm
Dan: Yes, but for example Eucharist is not something small. If it is false I am worshiping bread
Matt: True. But you start with the basics, and then work up.
Dan: Also salvation
Matt: Yes... But, the Bible is the standard of truth isn't it?
Dan: Yes and we should disregard all teachings that do not agree with it.
Matt: If the Bible is the standard of truth, then you should make sure you agree with it as much as
possible, right?
Matt: Correct.... as much as is possible.
Dan: but you see I do not see disagreement between it and the teachings of the Catholic church.
Dan: How we can decide that I am wrong and you are right? How can we prove it?
Matt: Easy. Where in the Bible does it state we are to use Mary as a mediator, someone to pray to, or
even through?
Dan: Nowhere. But we Catholics see things pointing to it like we and you see things pointing to the
Trinity
Matt: If the teaching of prayer thru/to Mary is nowhere in the Bible, then is it safe to do it?
Dan: That is why I cannot prove to you that we can pray thru Mary because you see the same thing
differently.
Matt: But... I see that the Bible teaches me to pray to God, that there is ONE mediator, etc.
Therefore, I only have Jesus as my mediator.
Dan: So that is why I am trying to find out how we can find out who sees it as it is.
Matt: Then go to the Bible. Read it. Don't go beyond it.
Matt: We have a tendency to make it into what it isn't. I'm guilty of that too sometimes, but... if I try
and stick to it alone, I have far less chance of error.
Dan: But it is not that simple, I do not think I go beyond it. You think I do. That depends on how we
view it.
Matt: Perhaps, but the Bible does not teach prayer to or through Mary, does it?
Dan: But to me the Bible teaches that there is something like tradition and that I should listen to it.
The Bible points and tradition says that I can ask Mary for prayer. But you will disagree because you
do not see the Bible saying that we should listen to apostolic tradition
Matt: Then your tradition supercedes the Bible?
Matt: Where does it say in the voice of Jesus, that you are to pray to or through Mary?
Dan: No, as I said the Bible points to it like the Bible points to Trinity. But let’s get off this because
we will get nowhere.
Matt: You see, the Bible does teach the Trinity. http://www.carm.org/doctrine/trinity.htm. It does not
teach that you can pray to or through Mary.
Matt: This is simply an example of how you know if you are being biblical or not.
Matt: Does the Bible teach the Trinity? Yes.
Matt: Does the Bible teach prayer to or through Mary? No.
Dan: To you, because you see it differently. Who sees it like it is?
Matt: You admitted the Bible didn't teach it.
Matt: So, you know that God has not approved it.
Matt: Why do you want to hold on to this?
Dan: To me, Jesus says my body is real food and my blood is real drink. I do not see other way to
take it, but you take it other way.
Matt: I'd be glad to talk to you another time on this... but I am trying to get you to see a principle
dealing with what the Bible does and doesn't say.
Matt: Why don't you think about it and let's talk again another time?
Dan: So we will never convince each other because I see a 4 and you see a 3.
Matt: Sorry, I want to listen to the voice of Jesus, not man's tradition.
Dan: Okay, but can you think about a way to find out is it a 3 or a 4.
Dan: Thanks for the talk
Matt: You're welcome and thank you as well.

I thought this conversation went well. The place where I said "We are done" was an attempt by
me to get this person to see that we were at an roadblock, that he did not want to see beyond his own
preset ideas even if the Bible contradicted it.
I firmly believe that the Bible is where we should be looking for spiritual truth and not the traditions
of men.
Two Mormons state that 3 Gods is really 1 God .
This chat began by "Mormon" contacting me on ICQ. I responded, asked him what he believed. He
said he was a Mormon and asked me what I like to do. "Debate," I said. He asked me what I liked to
debate. I said theology and we were off. I began with a comment on how Mormonism contradicts the
Bible. The conversation progressed and later he enlisted the help of a Mormon missionary.
In my opinion, this dialogue is a good example of how Mormons cannot accept what the Bible
plainly teaches. Instead, they must reinterpret the scriptures and change word meanings to get the
Bible to agree with their theology.
In this dialogue, I was a bit direct and tried to stay on one subject attempting to get them to really
understand the difference between the Bible and Mormonism. Whether or not I accomplished that, I do
not know. I tried to remain polite and hope that I was.

Matt: As I was saying, the Bible teaches that there is only one God in all existence. Therefore, there
can be no other god or goddess as Mormonism teaches.
Mormon: You are taking things out of context...another example of what so many other people do.
You start at the end, not bothering to know what comes before
Matt: The issue is whether or not Mormonism teaches the true god. It does not. The Bible says there
are no gods created after God and none created before him. Therefore, you cannot become a god.
Mormon: Give me scripture and verse for this.
Note: I have the advantage of a computer Bible program, so I simply pasted the verses into
our conversation.
Matt: Isaiah 44:6 says, "Thus says the Lord, the King of Israel, and his Redeemer, the Lord of hosts:
‘I am the first and I am the last, and there is no God besides Me."
Matt: Also, in Isaiah 44:8, it says, "Do not tremble and do not be afraid; Have I not long since
announced it to you and declared it? And you are My witnesses. Is there any God besides Me, Or is
there any other Rock? I know of none.’"
Matt: God says He does not even know of any other god, therefore, there is no goddess wife. He says
there is no God besides Him... Period.
Mormon: Okay, what does it say in the KJV Bible? That is what I study by. I find with all the other
versions, there is much bias.
Matt: Okay, here it is. Isaiah 44:6: "Thus saith the LORD the King of Israel, and his redeemer the
LORD of hosts; I am the first, and I am the last; and beside me there is no God."
Matt: Isaiah 44:8 says, "Fear ye not, neither be afraid: have not I told thee from that time, and have
declared it? ye are even my witnesses. Is there a God beside me? yea, there is no God; I know not
any."
Matt: Also, Isaiah 43:10 says, "Ye are my witnesses, saith the LORD, and my servant whom I have
chosen: that ye may know and believe me, and understand that I am he: before me there was no God
formed, neither shall there be after me."
Mormon: The thing is, there are so many things that people just don't understand because of the fact
that they don't start at the beginning. I mean when you think about it, would you go into a grade one
class and teach them grade 12 algebra?
Matt: I understand. But what does the Bible say? Does it say there are no other gods? If so, then
there aren't any other gods.
Mormon: There are certain precepts that you have to understand before that...and I get this all the
time from people. They just come in and try to start bashing the church, not taking into consideration
what the person thinks, of his feelings. It is just unfair to treat a person's beliefs like that.
Matt: You asked and I quoted scripture to you that says there is no other God. So how do you answer
the challenge of God's word saying there is no other God?
Mormon: I commend you on the fact that you have studied this out.. I know that I don't have all the
answers. I have only been in the church for ten months, and am still learning more everyday. I will get
answers, maybe not tonight, but within the next day or so. I am sure of that. May I ask you a
question?
Matt: shoot.
Mormon: You told me when I asked you what was up that you were looking for a debate...why do
that? Why come in just to debate and try to tear down what others believe?
Matt: I debate both to learn and to teach. If you believe in a false god, then you are in trouble. I am
simply trying to help you find the real one.
Mormon: But is it RIGHT to come in and try to tear them down like that?
Matt: Yes. It is right to warn people about the danger they are in.
Mormon: Okay, well, what church do you belong to?
Matt: I go to a non-denominational church.
Matt: By the way, you still haven’t answered my question about there being only one God.
Mormonism teaches that there are many many gods, that people can become gods...
Mormon: Can you give me a standard of belief?
Matt: The standard is the Bible.
Mormon: Like I said, could you give me a couple of days to get answers, I haven't studied this topic a
lot.
Mormon: Ok, well, how about the method of baptism, how is it done?
Matt: Baptism is really immaterial when it comes to the importance of knowing who is the true God.
But I'll answer it. Could you be more specific?
Mormon: What is the method of baptism? Some do it in the name of the Father, Son, and Holy Spirit,
others do it in the name of Jesus Christ. Here is a question: Do you believe baptism is necessary?
Matt: Those who baptize "in Jesus name" are the United Pentecostal and the United Apostolic
churches and they are both cults.
Mormon: WHAT!?
Matt: Yes... The proper mode is in the name of the Father, Son, and Holy Spirit.
Mormon: Hold on….I am bringing a friend into this chat
Matt: Okay,
[LDS Missionary enters the chat]
Matt: Hi
Mormon: Hi LDS Missionary. Meet Matt. LDS served a mission, and has been in the church longer
then I and should therefore be able to help.
Matt: Fine, let's talk then...
Mormon: Ask LDS some questions.
LDS Missionary: Hello Matt. Have you two been talking a while?
Mormon: Yes, for a little while.
Matt: I’ll restate my assertion: The Bible says that there is only one God. Period. Therefore,
Mormonism is wrong.
LDS Missionary: Yes there is only one god.
Matt: …in all existence in all places in all time?
LDS Missionary: We believe there is only one God
Matt: LDS, do you believe that there is only one God in all the universe? in all times? in all places?
Mormon: We believe in God the eternal Father, and in his Son, Jesus Christ, and in the Holy Ghost,
that is the first article of faith.
LDS Missionary: Yes. He is eternal, from everlasting to everlasting
Matt: Is Jesus a god?
LDS Missionary: Yes. He is divine. He is the son of God the Eternal Father.
Matt: So then, the Father is a god, and the Son is a god. That is two gods, right?
Mormon: Jesus is a part of the Godhead, they together make up one eternal God.... inseparable so to
speak
Matt: I thought you said there was only one?
Matt: Mormon. Your church teaches that the godhead is three separate gods.
Mormon: Okay. He is a member of the Godhead.... TOGETHER they make up "God".
Matt: Your church also teaches that there is a wife to God, a goddess wife.
LDS Missionary: As Mormon said, there's the Father, Son, and the Holy Ghost. They are one in
purpose. We know very little about that doctrine or the goddess mother. But yes we believe there is a
heavenly mother.
Matt: LDS, I thought you said there was only one god in all the universe. Now you just admitted there
is more than one god. So, you have God the Father, a god the Son, a god the Holy Ghost, and a
goddess mother. That’s four gods.
LDS Missionary: Three separate personages, but one in spirit and purpose.
Matt: In Isaiah 43:10, it says, "Ye are my witnesses, saith the LORD, and my servant whom I have
chosen: that ye may know and believe me, and understand that I am he: before me there was no God
formed, neither shall there be after me." Isaiah 44:8 says, "Fear ye not, neither be afraid: have not I
told thee from that time, and have declared it? ye are even my witnesses. Is there a God beside me?
yea, there is no God; I know not any."
LDS Missionary: As a godhead, the Father, Son and Holy Ghost are eternal.
Matt: How do you explain your theology in light of the Bible's teaching that there is only one God,
period, none other than God, none before, and none after?
LDS Missionary: I wouldn’t be able to believe the way I did unless I believed in modern revelation.
We have modern prophets to reveal God's word to us.
Matt: Was God, whom you call elohim, once a man?
LDS Missionary: Again, a doctrine we know very little about.
Mormon: Amos 3:7 here! "Surely, the Lord God doeth nothing, save it shall be through his servants,
his prophets."
Matt: Luke 16:16 says, "The law and the prophets were until John: since that time the kingdom of
God is preached, and every man presseth into it."
Matt: Prophets were the Old Testament way. Now we have Jesus and no longer need prophets (Heb.
1:1-3).
Matt: Now please, back to the issue of God.
LDS Missionary: We don't teach these doctrines to everyone.
Matt: The Bible says there are no other gods, that God doesn't even know of any other (Isaiah 43:10;
44:6 - 8). Now, if Jesus, the Holy Spirit, and god's wife are all gods, that’s three others. How do you
explain God saying he doesn't even know of any others?
Matt: Don't you see? The Bible contradicts your theology. Which should I trust?
LDS Missionary: Have you ever read the book of Mormon?
Matt: Most of it....
LDS Missionary: What was your intent in reading it?
Matt: To study. Mormonism teaches us to read it and pray about it to get a testimony that it’s true.
We’re to pray about it with a sincere heart, etc., But it doesn't matter what I feel. The Bible says there
is only one God, not more than one. How do you reconcile your beliefs with that?
Mormon: Like we said, Heavenly Father, Jesus Christ, and the Holy Ghost make up ONE eternal God!

Matt: Mormon? Can I ask you something?


Mormon: Go ahead
Matt: How many gods are there in all places, all times, etc.?
Mormon: There is only God. We believe that with all our hearts. But what you are not comprehending
is that we believe that Heavenly Father, Jesus Christ, and the Holy Ghost is ONE eternal God, they are
separate personages, but they are one in purpose
Matt: Is the Father a god?
Mormon: It says in the BIBLE, that God is the same YESTERDAY, TODAY, and FOREVER,
Matt: Yes I know. Heb. 13:8, yes. Psalm 90:2 is good also.
LDS Missionary: We actually can't answer that for you. You need to start with the basics.
Matt: This is the basics! How many gods are there? This is quite basic.
LDS Missionary: Well, the question is basic. But the understanding of that doctrine is probably
beyond what any of us are ready for. Do you really believe that god has finished speaking to us, that
the Bible is all there is now? So if he talked to the people then, why wouldn't he do it now?
Matt: Yes, because he said in Heb. 1:1-3 that now He speaks to use through Jesus, not prophets.
Heb. 1:1-3 (incidentally), God, who at sundry times and in divers manners spake in time past unto the
fathers by the prophets, 2Hath in these last days spoken unto us by his Son, whom he hath appointed
heir of all things, by whom also he made the worlds; 3Who being the brightness of his glory, and the
express image of his person, and upholding all things by the word of his power, when he had by
himself purged our sins, sat down on the right hand of the Majesty on high;
Matt: Mormon, is the Father A god?
Mormon: Yes, they are all members of the Godhead. They combine to make up one eternal God
Matt: Is Jesus a god?
Mormon: Okay...let me go through this one more time, they are all members of the Godhead, and the
three combined make up one eternal God
Matt: You are stating that God is an office. The Bible states there is one God, only one.
Mormon: What do you mean by that?
Matt: You are stating that the Father is a god, that the Son is a god, and that that Holy Ghost is a
god. That is 3 gods.
Matt: Now, how many gods ARE there?
Matt: I've already shown you where the Bible says there is only one, period, that's it.
Matt: Now you have to jump around trying to explain yourselves... it isn't working. There is either
more than one god or there isn't.
Mormon: ok...They are all members of the Godhead.... Together they make up one eternal God!!!
Matt: So, three gods, make up one eternal god? How is that possible?
Mormon: No...They are members of the Godhead. They each play a vital role in God. They each have
a specific purpose. They together make one eternal God.
Note: The Mormon has redefined the word "god" to mean two different things: god as an
individual and god as an office held by three individual gods. So, when I ask how many gods
there are, Mormon says 'one.' In this he is using the term 'god' as referring to the office held by
three gods. When I ask if the Father is a god, and he says yes, he is using the word 'god' in the
individual sense. This is double-talk and he does not see what he is doing. I am repeating this
different ways with them both in an attempt to get them to see their inconsistency.
Matt: The three separate gods make one god?
LDS Missionary: You know our beliefs. We've explained them. We believe the Bible coincides with our
beliefs perfectly.
Matt: But that's like saying that the 3 person of Frank, Joe, and Mike, make up one person, not three.
It doesn't make sense.
Mormon: I not only believe that it does, but I KNOW it does...I have a firm testimony of the
truthfulness of this church, and nobody, NOBODY will ever take that away from me!
Matt: But the Bible does not teach that.
Mormon: Okay...let me use an analogy. You have a family, right?
Matt: That is plain and simple. So, how can you state that you can become a god, or that god used to
be a man, or that there is a goddess mother in heaven?
Mormon: Will you answer my question. Do you have a family?
Matt: Yes, I have a family
Mormon: Okay, Well, each member of your family is a separate individual right?
Matt: Yes
Mormon: Well...together they make up ONE FAMILY. That is how it is with the Godhead. Matt: I
understand. But I'm trying to get YOU to see the problem.
Mormon: What problem!?
Matt: In Mormonism, the Godhead is like an office held by three gods. Each god is not the other god.
They are three gods. You say the three separate gods make up on godhead.
Matt: But the Bible doesn't say there is only one Godhead. It says there is only ONE GOD!
Mormon: Ok, I am about to give up here... You aren't willing to start with the basics, you aren't
willing to LISTEN so there is nothing that I can do.
LDS Missionary: We may as well give up for now. What church do you go to Matt?
Matt: I go to a non-denominational church.
Matt: Mormon?
Mormon: I am here
Matt: Does the Bible say there is only one God?
Mormon: Yes...and together...the Father, Son and Holy Spirit makeup that one God!
Matt: Amazing.... is the Father a god? Is the Son a god? Is the Holy Ghost a god? How many gods is
that?
Mormon: Okay...together they are God!
Matt: One plus one plus one equals three.
Mormon: Key word: TOGETHER!!
Matt: So three separate gods equals one god?
Mormon: TOGETHER they make up ONE ETERNAL GOD!
Matt: So then, the three separate gods is really only one god?
Mormon: This isn't working. You aren't really listening to what we are saying!!!
Matt: Yes I am listening. I am listening carefully.
Matt: You are telling me there are three gods and that the three gods make up one god.
Mormon: With your ears you are, but not with your heart. Right now your heart is too full of
contention and the desire to argue... Well, the things of God are holy, and sacred, and I REFUSE to
argue about them
Matt: That is not logical. You are playing with words... you should be saying that the 3 separate gods
equals one godhead (according to Mormon definitions).
Matt: Don't you see? The Bible says there is only one God, not three gods. Not three gods... just
one.... Not one godhead consisting of three gods, but only ONE God.
Matt: Mormonism disagrees with the Bible.
Mormon: No, Mormonism does agree with the Bible. You are making it disagree!
Matt: You mean, that by quoting the Bible plainly, I am making it disagree with Mormonism?
Interesting. Want me to quote the verses again, and you please tell me where it is wrong?
Matt: Isaiah 43:10, "Ye are my witnesses, saith the LORD, and my servant whom I have chosen: that
ye may know and believe me, and understand that I am he: before me there was no God formed,
neither shall there be after me."
Mormon: No, that isn't what I mean. You are coming in with the attitude that we are wrong, that
there is no way we could be right, and therefore you are not really listening, or even thinking about
what we are saying.
Matt: How could I not be right when I agree with the Bible when it says, "Fear ye not, neither be
afraid: have not I told thee from that time, and have declared it? ye are even my witnesses. Is there a
God beside me? yea, there is no God; I know not any" (Isaiah 44:8)?
Matt: If the Father is a god, and the Son is a god, and the Holy Ghost is a god, and there is a goddess
mother, then what is God saying here in Isaiah 44:8?
LDS Missionary: These scriptures must be understood in the context they're meant for. The Lord had
something to teach at that particular time
Matt: God will not contradict himself later, will he? Will he teach Israel there is only one God when
there really is more than one?
Matt: Will he have Mormon prophets tells us later that we can become gods, when so long ago God
said there would be no Gods formed after Him?
Mormon: Matt will you be online tomorrow?
Matt: depends... I have a meeting to go to?
Mormon: I will get some answers and will chat again with you soon
Mormon: What time can you make it in.
Matt: Maybe around 9 p.m.
Mormon: I will have tome to study my scriptures and get some answers.
Matt: Okay... Let me help you out with some standard verses the Mormons use: 1 Cor. 8:5-6; John
10:30-34 and Psalm 82:6. Mormons like those But don’t forget to check out Gal. 4:8; Eph. 4:5; Deut.
6:4.
Mormon: Okay...until tomorrow then
Matt: This will help you get your answers... but, context is everything, and I'll be ready....
Matt: I'll try tomorrow at 9. Okay?
Mormon: Okay .bye
Matt: Cya

This was a very interesting dialogue for me because it is so typical of how Mormons redefine words
in order to retain their theological views. Because of their testimony of the truth of Mormonism and
that God is an exalted man and has a goddess wife, etc., it is absolutely essential that the Bible be
reinterpreted and words redefined. It is difficult to get them to see this is happening, especially when
they think I am the one who has done the redefining.
Nevertheless, I hope that they come to a full and complete saving and personal relationship with
Jesus.
Discussion with a Mormon on God's Nature
This dialogue is interesting because it was a pre-arranged debate between John and myself. Other
Mormons are interested in the transcripts so I will make no comments on this dialogue. I have,
however, edited out the typos, miscapitalizations, etc. to make it more readable. This is the complete
dialogue.

Matt: Hi
John Hello
Matt: How's it going?
John: It is going pretty well. I've been up a little longer than normal. You might have to forgive me if I
nod off at the keyboard.
Matt: Just so you know, my computer has been crashing spontaneously this week. If I suddenly
disappear, wait and I'll come back on line.
John: Do you want to open up with a statement or question?
Matt: You could. What are you interested in? Let's discuss subjects.
John: Would you be so kind as to refresh me on your definition of the trinity (or Godhead as I usually
refer to it)?
Matt: The orthodox doctrine of the Trinity is that there is only one single God in all the universe, in all
places, in all times. He exists in three persons. Each is not the other, but each is, in essence, God.
They all three are the ONE true and living God. They are not three separate gods.
Matt: It can be compared to time: past, present, future. Each is not the other, but each is, in essence,
time. All three make up one time.
John: I'm a little lost on the time analogy... give me a second to digest it. Let me see if I understand
your view right.
John: In the Old testament God spoke to us through prophets, men who he inspired..... that would be
"past".
Matt: No.... We can understand trinities in nature. Time is past, present, and future. Why can't we
understand God as a Trinity?
John: Then when Christ came.... God was here in person.
Matt: That is correct. The Word (John 1:1) became flesh (John 1:14), and added human nature to
himself (Phil. 2:5-8; Col. 2:9).
John: I guess when you used your analogy my mind went to Heb. 1:1-3.
Matt: Good. Jesus is the representation of the nature of the Father.
John: I think I got that.
John: Wait maybe I don't? What does it mean for him to be "the nature of the Father".... the word
"nature".... I'm not quite following.
Matt: Everything has a nature. A cat has 'cat' nature. A human has 'human' nature. God's nature, if I
might dare to proclaim, is divine, holy, pure, perfect, etc. Jesus represented that perfectly.
John: I like that, I don't know if I have ever heard it put that simply before, thanks.
John: I have just one question concerning the Holy Ghost.
Matt: okay.
John: Back to the analogy (hope I'm not pounding that to death) does the Holy Ghost more or less
represent the "future".
Matt: No....
Matt: That isn't what I was getting at.
Matt: The Trinity is three distinct 'parts' or persons. Yet there is only ONE being.
John: Maybe I'll drop the analogy for now.
Matt: Time (as the analogy goes) is three distinct 'parts', yet there is only ONE time.
Matt: The point is that the Trinity is the doctrine that there is only one God in all existence.
Mormonism states that there is more than one God in existence. They both cannot be true.
John: I understand the One being.
John: Agreed.
Matt: Okay.... but as a Mormon do you believe there is only ONE God in all the universe?
John: No.
Matt: Okay. That is a great starting point. We Christians believe the Bible teaches that there is only
one God in all existence. Therefore, we conclude that Mormonism is not true.....in a nutshell.
John: I believe that there are three, at least with whom [incomplete sentence]
John: But what if the Bible .... The Bible teaches that they (the Godhead) are separate.
Matt: If it taught that, I'd believe it. But it doesn't. It teaches that there is only one God.
John: Go with me to Revelations 4 and 5 and glance over them quickly... if you will?
Matt: Okay.
John: In verses 2 and 3 of chapter 4, who is sitting on the throne?
Matt: That's Jesus.
John: Who is then represented in verse 6 of chapter 5?
Matt: Jesus.
John: Talk to me about verse 7 of chapter 5.
Matt: In this vision, it appears the Father and the Son are there....
Matt: okay..... ?
John: Why would God represent himself as been two beings, one sitting on a throne, and one
standing, receiving a book form the other?
Matt: He is not representing Himself as two beings. Revelation is a highly symbolic book. You must at
least take that into consideration when quoting it.
Matt: Nevertheless, the Trinity is three separate persons, not three separate beings.
Matt: Therefore, to see Jesus standing at the right hand of the Father in a vision (Acts 7:55-60) is
perfectly consistent with the theological definition.
Matt: Also, you must take all of the Bible as a whole. In Isaiah chapters 43-45 the doctrine of
Monotheism (one God in existence) is clearly taught.
John: I'm an artist, not that that is important, but as such I spend much time visualizing things, I
can't picture anything but two separate beings. Sorry, I just can't.
Matt: So? Must you visualize something before it can be proven to be true?
John: Slow down a bit.... I can't keep up.
Matt: No problem.
Matt: When we encounter God, we will encounter areas of intellectual difficulty. After all, He is infinite
and NOT like us.
John: Let me just back up a second. I understand that Revelations and other such scriptures are
highly symbolic.... But, to me anyhow, the symbolism of two sends a clear message to me that there
is a distinction.
Matt: good... there is a distinction. That is part of the doctrine of the Trinity. Now add to that
distinction the doctrine from the Bible that there are not two or three gods, but only one. Then you'd
almost be there.
John: I interpret that distinction to be that they are separate.... you don't.... where does that leave
us?
Matt: Then you would conclude that they are each 'a' god. right?
John: Yes.
Matt: Okay... But when you encounter biblical passages that teach that there is only one God, what do
you do?
John: The Godhead is one in purpose, one in authority, one in power.... When one of the members of
the Godhead speaks, either the father, the Son, or the Spirit, they speak for all, as one. In fact, even
the Angels, when sent, they speak even as though they are one with the God head. In a second, I'll
generate some passages for you.
John: Pardon me a minute... sleep deprivation.
John: I'm still here.
Matt: Okay.
John: Here is one example anyhow: Gen. 31:11-13 "And the angel of God spake unto me in a dream,
saying, Jacob... I am the God of Beth-el, where thou anointedst the pillar, and where thou vowedst a
vow unto me..." see Gen. 28:16-22
Matt: Angels often speak for the Lord (Gen. 17:1ff) is a good example. But, if the Bible teaches there
is only one God, then is there only one God?
John: Unfortunately I am using a brand new Bible that I received a few months ago and have very
little marked in it... If I had had my older one I could have found some of the other quotes.... but I'm
a bit lost at the moment.... and tired.
Matt: I can help you post them... don't worry.
John: Thank you.
Matt: But, take a look at Isaiah 43:10: "Ye are my witnesses, saith the LORD, and my servant whom I
have chosen: that ye may know and believe me, and understand that I am he: before me there was
no God formed, neither shall there be after me. 11I, even I, am the LORD; and beside me there is no
saviour. The King James Version, (Cambridge: Cambridge) 1769.
Matt: LORD there is the Hebrew word of Jehovah. The word 'God' there in the Hebrew is the word
'elohim.' So, Jehovah is saying there is no elohim besides him. In other words, God is saying there is
no God besides Him.
John: When the Bible speaks of there being One God I'm hesitant right now to start throwing out a lot
of scriptures... I'm really not up for a long debate. I do not wish to dodge any scriptures either.
Matt: I understand. But let me post some quick verses that we use to show there is only one God: We
can comment on them as a whole.
Matt: Isaiah 44:6, "Thus saith the LORD the King of Israel, and his redeemer the LORD of hosts; I am
the first, and I am the last; and beside me there is no God."
Matt: Isaiah 44:8, "Fear ye not, neither be afraid: have not I told thee from that time, and have
declared it? ye are even my witnesses. Is there a God beside me? yea, there is no God; I know not
any."
Matt: These are typical verses we monotheists use to show there is only one God. Therefore, the
Trinity cannot be more than one god. It is that simple.
John: I believe in the Bible. I believe it when it teaches that God is one. I believe it when the Book of
Mormon teaches that God is one or where the Doctrine and Covenants teach that God is one.....
and...... I hope you are getting my message.
Matt: I understand. Deut. 6:4 says that God is one. But the Bible also teaches that there is only one
God, not three.
John: God is not divided. The Godhead is not divided.
Matt: The verses quoted above show that. But, in Mormonism the godhead is three separate gods.
That is divided. In the doctrine of the Trinity, there is only one God, not three. That is unity.
John: How many Apostles did Christ call?
Matt: 12.
Matt: How many God's does the Bible say exist?
John: "Neither pray I for these alone, but for them also which shall believe on me through their word;
That they all may be one; as thou, Father, art in me, and I in thee, that they also may be one in us:
that the world may believe that thou hast sent me" John 17:20-21.
Matt: Yes... that context is one in purpose. But John, you haven't addressed the verses I've quoted
that show there is only one God. What about those?
John: Simply put, I believe that when God says that he is One he means so in purpose. The Godhead
is not divided, they are one in purpose. Even as we have been commanded to be one in purpose. That
is what I believe, that is what I believe the Bible teaches. I do not know any other way of stating this.
John: I would like to give you some verses proving that the Godhead is separate in purpose.... but not
now.... I am slowly loosing it....Correction... quickly loosing it.
Matt: Then what do you do with the verses where God says that there are no other gods besides Him?
Isaiah 44:8, "Fear ye not, neither be afraid: have not I told thee from that time, and have declared it?
ye are even my witnesses. Is there a God beside me? yea, there is no God; I know not any."
John: What do you say, five more minutes and then we hang it up for now?
Matt: Okay
John: In that particular verse, as in the others.... When God, Jehovah, is talking he is talking for the
Godhead as a whole, as one..... and beside him, them...... there is no other God.
Matt: So, then, when He says that there are no other gods, he really means there ARE other gods,
right?
John: He is speaking for all three, speaking as one, for they are one, beside them there are no other.
They are the only ones to whom we must list to follow and obey. They are the only ones, Christ most
specifically, by whom we can be save. Salvation comes from no others.
Matt: That isn't what it says. It says there are no other gods. He doesn't even know of any. None
created before and none created after. I believe it.
John: I don't believe that that is what it says.
Matt: But, I just quoted exactly what it says and you disagreed with it. Don't you see what just
happened? All I did was quote different parts of the verses already place in this chat and you disagreed
with them. That means you understood them plainly and rejected them outright.
John: I disagree only with your (and most of Christendom's) interpretation of those verses.
John: I don not reject them.
Matt: But, I simply quoted them to you and you disagreed with them.....
John: Do you have any parting thoughts or statements?
Matt: I didn't interpret the verses. I quoted them. You disagreed with them. That isn't good.
Matt: Until next time.
John: Take care of yourself..... better still.... may God take care of you and bless you. A Friend....
John.
What is salvation and who is God?
This dialogue with two Mormons is a good illustration of the necessity of knowing not only what
they believe, but also what the Bible teaches, where it says it, and what it means. This takes some
practice, but practice makes perfect.

Gary: In your info on CARM, do you preach mostly against the Mormons, or is there anything we teach
you believe is good?
Matt: I teach against what goes against the Bible. The Mormons do many good things. But, they teach
a false god, therefore, they are leading people to hell. Simple....
Gary: So all our good works count for nothing?
Matt: That's right. They count for nothing in getting your sins forgiven.
Matt: The Bible says that if we could be forgiven by what we do, then Jesus died needlessly (Gal.
2:21). It says in Romans 3 that we are made right in God's eyes apart from the works of the law.
Gary: I agree with you. We can't save ourselves.
Matt: The Bible says we are saved (justified -- cleansed from all sin), by Faith (Rom. 5:1). You can do
NOTHING at ALL to contribute ANYTHING to God forgiving you. It is NOT dependent upon any of your
works.
Gary: We're saved by grace. Right, God forgives us because of what his son Jesus Christ did for us.
Matt: In Mormonism, salvation is universal resurrection. Is that what you are referring to?
Sue: We don't think that just because we do a lot of good things that we will get to heaven. It is by
grace yes, but we as Christians should do things to show our willingness, our faithfulness, etc.
Gary: He's asked us to be obedient though. That's part of it. But to be forgiven of our sins we must
repent. Do you believe that?
Matt: Yes, we are to be obedient, but the point I am trying to make is that obedience FOLLOWS
salvation. We are freely forgiven of our sins, by God, if we trust in Jesus’ ALONE. We can’t add
anything to what He did and we can’t add anything that contributes to our salvation in any way. We
are not saved by "grace through faith after all we can do." We are forgiven of our sins, completely,
and solely, by faith alone in Christ alone. THEN we go and do good works.
Gary: I believe that we must show God our good works so he knows we are sincere. Then we can be
saved.
Matt: But Gary, that isn’t what the Bible teaches. You should study Eph. 2:8-9; Rom. 6:23, Gal. 2:20-
21; and Romans chapters 3-5.
Gary: Maybe I’ll take a look sometime.
Matt: I hope so. Let me ask you something else. Do you believe that God used to be a man on
another world and that he became a god and brought one of his wives to this world?
Gary: I don't know about all that. As far as the Bible says, God is infinite and eternal. We believe that.
Matt: Gary, let's work with that.... Do you believe God has a body of flesh and bones and that he used
to be a man on another world?
Gary: Yes. But he is still infinite and eternal.
Matt: So, how can god be eternal if he had a beginning? How can he be everywhere if he has a body
of flesh and bones....
Gary: Do you have a reference to these teachings of ours you like to bring up? or did you just hear
them from someone?
Matt: Hold on. I’ll copy it from my website into here.
Matt: There are many gods, Mormon Doctrine, p. 163.
Matt: There is a mother god, Articles of Faith, by James Talmage, p. 443.
Matt: God used to be a man on another planet, Mormon Doctrine, p. 321. Joseph Smith, Times and
Seasons, vol 5, pp. 613-614; Orson Pratt, Journal of Discourses, vol 2, p. 345, Brigham Young, Journal
of Discourses, vol. 7, p. 333.
Matt: After you become a good Mormon, you have the potential of becoming a god, Teachings of the
Prophet Joseph Smith, pages 345-347, 354.
Matt: God the Father had a Father, Joseph Smith, History of the Church, vol. 6, p. 476; Heber C.
Kimball, Journal of Discourses, vol. 5, p. 19; Milton Hunter, First Council of the Seventy, Gospel
through the Ages, p. 104-105.
Matt: God resides nearest a planet called Kolob, Pearl of Great Price, pages 34-35; Mormon Doctrine,
p. 428.
Matt: God the Father has a body of flesh and bones, Doctrine and Covenants, 130:22.
Matt: God is in the form of a man, Joseph Smith, Journal of Discourses, Vol. 6, p. 3.
Matt: God is married to his goddess wife and has spirit children, Mormon Doctrine p. 516.
Sue: CARM, read Romans 8:16, you can too Todd, I think it will help out in the issue of whether or not
we are children of god
Matt: Rom. 8:16 says, "The Spirit itself beareth witness with our spirit, that we are the children of
God."
Matt: That is true. The verse is speaking to those who are already Christians.
Sue: Yes, it says right there that we are children of God!!
Matt: The children there in Romans 8 is referring to adoption.
Matt: Rom. 8:16, For you have not received a spirit of slavery leading to fear again, but you have
received a spirit of adoption as sons by which we cry out, "Abba! Father!"
Matt: Rom. 8:23, And not only this, but also we ourselves, having the first fruits of the Spirit, even we
ourselves groan within ourselves, waiting eagerly for our adoption as sons, the redemption of our
body.
Matt: The problem with the Mormons is that they do not teach the Bible. They teach 'parts' of the
Bible and when they do, they take it out of context.
Matt: The children of God stuff is dealing with adoption. That is what Romans 8 is talking about. If we
are LITERALLY children of God, why do we need to be adopted?
Matt: Hhave you read John 1:12? "It says, "But as many as received him, to them gave he power to
become the sons of God, even to them that believe on his name."
Matt: The word power there in the KJV is also the word for authority.
Matt: Either way, being a child of God is not natural. It must be by divine action based upon the work
of Christ; hence, receiving Christ.
Gary: We need to be adopted into the family of Christ. But we are already God's children.
Matt: The Bible states that we are children by adoption, not because god had relations with his
goddess wife and had children who came down and inhabited human bodies. There is no pre-
existence.
Matt: 1 Cor. 15:46 says that the natural is first, THEN the spiritual. In other words, the natural,
physical, is what comes first as far as bodies go, and then the spiritual body follows. Mormonism has it
reversed.
Gary: In one way we must be adopted. But first, we are already His literal children
Matt: So, does God literally produce children? He has a body of flesh and bones, and he has a wife,
right. Does he have genitalia?
Gary: That's very disrespectful to speak of those doctrines you know nothing about. if only your heart
were to change and be willing to really seek the truth, maybe you could find out the truth. you mock
sacred things, and the spirit cannot reside in you. you could not recognize the voice of God with your
attitude
Gary: Do you hold anything sacred?
Matt: Honestly, I mean no offense. I just don’t any other way to say it. I am curious.
Gary: I doubt that.
Matt: I’m trying to make a point. Romans 1:22-23 says, "Professing themselves to be wise, they
became fools, 23And changed the glory of the uncorruptible God into an image made like to corruptible
man."
Matt: Don't you see? Your god is in the form of a man.... completely! That is NOT the God of the
Bible.
Gary: He's an incorruptible perfected Man.
Matt: Your god is the image of a man. He was corruptible because he sinned.
Gary: More like we are in His image.
Matt: But isn’t it more than that? He has a goddess wife. They have children. They both have bodies
of flesh and bones.
Matt: Your god is merely an ex-sinner who became a god of a particular world and he has a goddess
wife.
Matt: That is NOT in the Bible. The Bible contradicts such teachings.
Matt: Because you cannot see it, you are unregenerate. You are not a Christian.
Gary: He is perfect. We are His children.
Gary: You haven’t answered my question from before.
Matt: Which question?
Gary: Do you hold anything sacred?
Matt: Yes I do... that is why I defend the truth. God is not an exalted man. He does not have a
goddess wife. I hold the truth sacred and defend it.
Gary: Does it bother you if people tear down what you believe is true?
Matt: Yes. That is why I defend Christianity.
Gary: Well, I can see that this is getting no where.
Matt: Sorry, I do not mean to be difficult. But I am just trying to get you to see.
Gary: I think it is you who cannot see.
Matt: That is always a possibility. But, at least I am trusting God’s word.
Gary: I do too.
Matt: Well, that is debatable given our conversation.
Gary: Alright, I have to go.
Matt: Okay, I hope to see you again sometime
Gary: Bye.
Matt: Bye.
Sue: Bye.
Did Joseph Smith see God the Father?
We jump right into this conversation where a Mormon stated there was an apostasy and that
Joseph Smith saw God the Father. Of course, this is a serious problem because the Bible teaches that
no one can see God the Father.

Matt: You state that there was an apostasy. Smith's vision states that God said all churches were
wrong.
Matt: The Book of Mormon says that my church is a false church.
Alex: God appeared to Joseph and told him all churches draw near with their lips but their hearts are
far from Him. None of them had the complete truth.
Matt: God the Father appeared to Joseph?
Alex: Yes. and His son Jesus Christ
Matt: Well, then we have a problem. The Bible says in 1 Tim. 6:15-17, "Which in his times he shall
shew, who is the blessed and only Potentate, the King of kings, and Lord of lords; 16Who only hath
immortality, dwelling in the light which no man can approach unto; whom no man hath seen, nor can
see: to whom be honour and power everlasting. Amen."
Matt: So who do I believe? The apostle Paul or Joseph Smith? Paul states that no man can see God
(the Father). Smith, therefore, did not.
Alex: He proceeded to restore the fullness of His gospel once again to us. But many have seen him.
Moses, Paul himself saw God the Father and the Son side by side, or that might have been Timothy.
Matt: No. No one has seen God the Father.
Matt: John 6:46 Jesus says, "Not that any man hath seen the Father, save he which is of God, he
hath seen the Father."
Matt: The people in the Old Testament were seeing Jesus, not the Father because the Father cannot
be seen.
Alex: Acts 7:55. Stephen, I believe.
Matt: Yes, he had a vision.... He had a vision of Jesus standing on the right hand of the Father. The
'right hand" means position of authority.
Matt: Jesus was in heaven, next to God's throne, probably, and Stephen was allowed to see this
vision.
Alex: Do you believe the Adam and Eve story? He spoke with God. They both did. Yes, I’d say it was a
vision. Just like Joseph smith had a vision, and many others before and after.
Matt: Adam and eve saw Jesus, the preincarnate Christ.
Matt: Jesus said that no one has ever seen the father, John 6:46, therefore, no one has. Why can’t
you believe it? Also, 1 Tim. 6:15-17 says no one can see or has seen him... Why can't you simply
believe what it says?
Alex: The Bible seems to contradict itself if it is read without the help of interpretation.
Matt: Yes, if read without the Spirit of God. Do you have the Spirit of God?
Alex: The bottom line is you can't find all the truth just by reading the Bible on your own. God has
more to help us now. And you can't say someone else's church is a cult because they choose to
interpret the Bible differently. That's not what Jesus Christ did.
Matt: Mormonism is a cult because it teaches a false god. THAT is why it is wrong and Jesus corrected
all sorts of people. He nailed the Pharisees. He stomped on the Sadducees.
Matt: Paul did the same thing later. It is perfectly biblical to tackle error and expose it. That is what I
am doing with Mormonism.
Alex: you may not choose to believe what we do, but you can leave us alone with our beliefs. let us
keep sacred the things we hold sacred
Matt: It is in error. If you die a Mormon, believing Mormon doctrine, you will go to hell. I don't want
that.
Alex: Christ denounced the Sadducees for their evil works
Matt: And their evil teachings as he did with the Pharisees also.
Alex: He called them to repentance.
Matt: yes... And God is calling you to repentance. Turn from the sin of having a false god.
Matt: Exodus 20 is where God warns people to not have false gods. Mormonism has several false
gods: a god the father, a god the son, a god the holy ghost, a goddess mother, becoming gods, etc.
Matt: There is only one God, not lots of them.
Alex: We believe in Christ and what He taught and what he's taught his witnesses, his prophets.
Anyone who accepts Christ and follows him can be saved to inherit a degree of glory.
Matt: No... you break the Commandments of God in Exodus 20 by having a false god.
Matt: God is not an exalted man from another planet who has a goddess wife.
Matt: There are not many many gods. There is only ONE God, period. Not just "of this world" but only
one in all the universe.
Alex: There's only one God in all the universe, in all eternity.
Matt: Let me ask again. You mean that there is no goddess mother?
Alex: yes there is.
Matt: btw, do you believe that Jesus is the god of this world?
Alex: Yes.
Matt: There is a mother goddess? Then how can there be only one god in all the universe and yet
there be two of them?
Alex: God is God from eternity to eternity.
Matt: Is Jesus a god too? Does Elohim have a 'dad', a father who was HIS God on the other world?
Alex: I know nothing about that. It's definitely not easily explained, or understood.
Matt: It is contradictory. In Mormonism. Jesus is A god. The Holy Ghost is A god. The Father is A god.
That is three gods. The goddess mother makes four.
Alex: By mans wisdom alone many things of spiritual nature seem contradictory
Matt: So you admit it is contradictory?
Matt: Then you do not believe based on the Bible. You believe based on feelings, your testimony...
that contradicts the Bible.
Alex: I admit what I just said: by our wisdom alone, it seems so.
Matt: Now... why in heaven should I believe your testimony when what you are telling me is self-
contradictory? Also, your belief contradicts the Bible!
Matt: Why should I believe you?
Alex: You don't have to. It'd be great if you did, but you probably won't. I know I lack a lot of wisdom
and understanding but i have faith and a testimony of the true church
Matt: You don't get it... the Bible states there is only one God, period. You state otherwise.
Matt: Your faith is in a false god and your testimony contradicts the Bible. You are on pretty shaky
ground.
Alex: Do you feel what you believe is true or do you believe against your feelings?
Matt: I don't "feel" anything is true when it comes to this. I trust the Bible. But, I often believe against
my feelings. The Bible says not to trust your feelings (Jer. 17:9).
Matt: I trust God's word. That is why I am not a Mormon and never will be one.
Alex: Ah that’s a fun scripture. Don't be led away by your natural passions
Matt: No, it says the heart is desperately deceitful, no one can trust it.
Matt: So, God says do NOT trust your heart. Yet you do.
Alex: The Spirit of God works with us to help us know the truth. We interpret it differently
Matt: He wants you to trust His word. If you don't, you WILL be led astray
Alex: I understand what you are saying but I don’t agree.
Matt: I know you don’t. If you did, you wouldn’t be a Mormon.
Alex: I’m sure we’ll talk again sometime.
Matt: I hope so.

With that the conversation ended. As you can see, the Mormons have a difficult time reconciling
the teaching of multiple Gods and the biblical teaching that there is only one. This type of conversation
happens too many times with Mormons.
Feelings, gods, and Joseph Smith
This dialogue demonstrates a major problem with witnessing to those in Mormonism. They think
that their feelings are as valid, if not more so, than biblical revelation. In fact, they often subordinate
the Bible to their "testimony", their feelings. Mormons contend that they received a confirmation from
God in their hearts that Mormonism is true. The problem with this is, that their testimony contradicts
the Bible. But, when you put feelings above scripture, it doesn't matter.

Greg: Hi. I visited your site the other day and I just had some questions.
Matt: Okay.
Greg: I was just wondering what the purpose of the site is.
Matt: To glorify God, get people out of cults, and equip Christians.
Greg: So it's not actually to teach people true beliefs about different churches?
Matt: Of course it is. What church do you go to?
Greg: Well, I joined the Church of Jesus Christ of Latter-day Saints about a year ago
Matt: Thought so... and you believe it?
Greg: Before either of us say anything more, I really don't want this to be any sort of argument and I
don't want anything said that would offend either of us
Matt: No problem.
Greg: Ok. Thanks. I really have no problem with anybody disagreeing with what I believe, but I would
hope that it's for the right reasons and I really don't think that the web site explains my church's
beliefs fully and truthfully.
Matt: I understand...
Greg: …like, when I was reading the page about what Mormons believe, a lot of the things were not
explained or taken out of context.
Matt: It was intended to be brief.... but... it is accurate....
Greg: And I mean, the KKK uses passages from the Bible to support their beliefs, but everybody
understands that they're taking them out of context
Greg: All I really wanted to say was that if you want something even shorter about what Mormons
believe, here's the 13 Articles of Faith. You should go to the LDS web site and read them. And I hope
that you would take the opportunity to better understand other's beliefs
Matt: I'm sorry... but there is a lot you do not understand about this issue...
Greg: Like what
Matt: Like God, for example.... When your articles say you believe in one God, that isn't true.
Greg: In what way?
Matt: There is a goddess mother in Mormonism, Jesus is a god as well. The Holy Spirit is another
god...that's four if you include God the Father himself. So what you are doing is misrepresenting
yourselves. You use the right words, but have the wrong meaning.
Greg: As far as the trinity, many Christians believe in that.
Matt: Nope. Not so.
Matt: Mormonism does not teach the Trinity. Mormonism teaches a triad... quite different. The Trinity
is ONE God. A triad is three gods.
Matt: There is only one god in the universe according to the Bible. But, Mormonism contradicts that.
Greg: Actually the Old Testament talks of others.
Matt: It speaks of false gods.
Greg: It's not until later in the Old Testament that God is not just the best god but the only one
Matt: No. That is not true.
Matt: Isaiah 44:6 says, "Thus says the Lord, the King of Israel, And his Redeemer, the Lord of hosts:
‘I m the first and I am the last, And there is no God besides Me."
Matt: Isaiah 44:8 says, ‘Do not tremble and do not be afraid; Have I not long since announced it to
you and declared it? And you are My witnesses. Is there any God besides Me, Or is there any other
Rock? I know of none.’"
Matt: God says there is no other God... that he doesn't even know of any others. Therefore, there is
only one God.
Greg: Well, I really don't see this going anywhere now, but all I want is you to not misrepresent the
church.
Matt: Could it be that your church is misrepresenting Christianity?
Greg: Well, I wouldn't be a member if that were any type of issue with me. I took a lot of time in
deciding.
Matt: You decided based upon what they told you and the way they told you. Sure, they are nice. But,
are they true? Can you really become a god? Is there really a goddess mother in heaven? Was God
really a man on another world at one time?
Matt: Do you believe these things?
Greg: And you know? Whatever arguments you have, there's nothing you can say that would
contradict the feeling it gives me when I read scriptures. And everything about it feels amazing to me.
Matt: But, the Bible says not to trust your feelings.
Greg: Where? And it what context?
Matt: Jer. 17:9 says, "The heart is more deceitful than all else and is desperately sick; Who can
understand it?"
Matt: Mormonism wants you to trust feelings. But God wants you to trust His word.
Greg: But what about faith?
Matt: Faith is only as good as who you place it in. Faith in something false is powerless to save.
Matt: God says to trust His word...
Matt: Acts 17:11, "Now these were more noble-minded than those in Thessalonica, for they received
the word with great eagerness, examining the Scriptures daily, to see whether these things were so."
Greg: But how do you get his word -- you don't get it written in the sky. It's a feeling.
Matt: The Word is the Bible. The Mormons have told you it is a feeling. God has told you to compare
truth to His word, the Bible. Who are you going to listen to?
Greg: I read the Bible, and I agree with it, but what is really amazing is how I feel when I read it. I
believe it is a feeling.
Matt: You are using feelings to judge that feelings are true.
Matt: God's word says that there are no other gods, that you cannot become a god... etc. Therefore,
Mormonism is wrong.
Greg: You said that Mormons have told me it is a feeling and I'm saying it's me who thinks that, even
before I had any clue what Mormons taught.
Matt: Mormons very often use the "feeling" thing as a way to get you to agree with them. They often
say, "How do you feel about this"... at least the missionaries I've spoken to have said this countless
times over the years.
Greg: Well, I don't think that religion should be totally intellectual. Is it wrong to feel love from God?
Matt: Not at all. Is it wrong to believe what God has said in the Bible?
Greg: No not at all. But how can you say it's right to feel love from God when you say it's wrong to
trust feelings.
Matt: Feeling love and trusting feelings are different.
Matt: It is good to feel love. But is it wise to trust a feeling about spiritual truth when God says the
heart is deceitful (Jer. 17:9)?
Greg: But the whole thing about other gods and everything you disagree with is so much bigger than
what God tells us about this world. In this world, right he is the only god.
Matt: Other gods? Then read this... Gal. 4:8, "However at that time, when you did not know God, you
were slaves to those which by nature are no gods."
Matt: 1 Cor. 8:5-6, "For even if there are so-called gods whether in heaven or on earth, as indeed
there are many gods and many lords, 6 yet for us there is but one God, the Father, from whom are all
things, and we exist for Him; and one Lord, Jesus Christ, by whom are all things, and we exist through
Him."
Matt: Notice how Paul says they are "so-called" gods and that they are not really gods by nature.
Greg: I really believe that God gives us what we can handle. First there were the beliefs in the Old
Testament for those people. Then Jesus came along and perfected that. And so why is it so outrageous
for Joseph Smith to perfect it even more?
Matt: Really? And you trust Joseph Smith?
Greg: Yes
Matt: Would you trust someone who said he'd done more to keep a church together than even Jesus?
Greg: Well, I would like to see where that was actually said and in what context.
Matt: Joseph Smith said, "God is in the still small voice. In all these affidavits, indictments, it is all of
the devil--all corruption. Come on! ye prosecutors! ye false swearers! All hell, boil over! Ye burning
mountains, roll down your lava! for I will come out on the top at last. I have more to boast of than
ever any man had. I am the only man that has ever been able to keep a whole church together since
the days of Adam. A large majority of the whole have stood by me. Neither Paul, John, Peter, nor
Jesus ever did it. I boast that no man ever did such a work as I. The followers of Jesus ran away from
Him; but the Latter-day Saints never ran away from me yet . . . " (History of the Church, Vol. 6, p.
408-409).
Matt: Joseph Smith said this. He boasted mightily. This is the man you are trusting to tell you spiritual
truth.
Matt: Joseph Smith also said, "A question may be asked, ‘Will mothers have their children in eternity?'
Yes! Yes! Mothers, you shall have your children." (Journal of Discourses, Vol. 6, page 10). "Eternity is
full of thrones, upon which dwell thousands of children reigning on thrones of glory, with not one cubit
added to their stature." (Journal of Discourses, Vol. 6, p. 10).
Matt: and he said, "I have always declared God to be a distinct personage, Jesus Christ a separate
and distinct personage from God the Father, and the Holy Ghost was a distinct personage and a Spirit:
and these three constitute three distinct personages and three Gods." (Teachings of Prophet Joseph
Smith p. 370).
Greg: Well, I would like to see that in the book first of all, but even if that's what he said, why is it so
horrible if Latter-day Saints have believed him more than people did in Jesus' time. What is wrong
about that?
Matt: But the Bible says in Isaiah 45:5, "I am the Lord, and there is no other; Besides Me there is no
God...."
Greg: You know, I really don't see this getting anywhere.
Matt: But Smith boasted he did more than Jesus to keep a church together. Doesn't that bother you?
Greg: You don't know with what inflection of voice he said that.
Matt: It doesn't matter, God says that pride goes before a fall. Joseph died one month later.
Greg: You definitely haven't convinced me of anything, I haven't you, so I don't see any point in this.
All I wanted was to show you a site where the beliefs are explained more
Matt: I understand. But I think you need to examine God's word more completely.
Greg: Well I think the same about you
Matt: Eternity is a long time to be wrong.
Greg: And I agree
Matt: Please, check the references and get back with me. Okay?
Greg: Ok, but I would hope that you would look through the site I gave you
Matt: I already have…..
Mormon and salvation and works

This dialogue was a friendly one. There weren't any "victories" on either side. I took over a
conversation that a friend of mine was having with this person. My goal was to point him to Jesus and
true forgiveness. Salvation isn't attained through intellectual competition but through the work of the
Holy Spirit. The question is, does the Mormon have the Holy Spirit, the true Holy Spirit? Are they
saved by their own works, by God's sacrifice alone, by faith, etc.? Can our works save us in any way?
These kinds of issues are what I was thinking while going through this dialogue.
Also, I made a mistake in the dialogue and had to correct myself in it. We all make mistakes.

Matt: Hi. What is your name?


Ken: Howdy
Matt: My friend tells me you are a Mormon apologist, true?
Ken: I try to be. I am only an amateur.
Matt: I was just telling him that it is useless to talk to you....
Ken: Why is that?
Matt: Because you believe what you believe. You wont' be converted. You can't be.
Ken: Like I told your friend when we started this exchange, I am only trying to correct his error of
LDS theology. I trust the Lord to covert his heart.
Matt: Which Lord? The LDS one is different than the one I believe in.
Ken: You believe in a nature of God that I do not.
Matt: Correct. I believe there is only one God in all existence. You do not.
Ken: The argument stands that I believe that there are three Gods who act as ONE, you believe there
are ONE God in three essences.
Matt: Not three essences, but close enough.
Matt: You are a polytheist. I am a monotheist.
Ken: Not three essences? How not?
Matt: You believe that God has a body of flesh and bones. I do not.
Ken: That is true.
Matt: Three persons, distinct persons, not flesh and bones, yet only one God. To continue.... You
believe that Jesus is the brother of us all, correct? and that there is a goddess mother in heaven....
correct? You believe this all because you think the Holy Spirit bears witness of this, right?
Ken: I believe he is our Spirit Brother. Yes, I would also accept the statement of Heavenly Mother.
Sure.
Matt: Well, that isn't what the Bible teaches..... So, you are wrong. But, I cannot convince you of
that. It is a spiritual issue. You must have the True Holy Spirit in order to know the truth. You must
first be born again or you will not have the Spirit of Truth. That is the issue. Jesus said you must be
born again. Jesus sends the Holy Spirit who bears witness of truth. But, he will not inhabit unholy
vessels. You must be cleansed of your sins. Are you cleansed? Are you cleansed of your sins?
Ken: Of course. One must be converted of the spirit. Amen. Thank the atonement of Jesus.
Matt: Remember, the Spirit of God cannot inhabit the Temple until after it had been cleansed and
sanctified.
Ken: Amen
Matt: Now I have a question for you. Do you now have eternal life, the eternal life is given by Jesus?
Ken: Eternal life is Jesus. Amen to that. If I endure to the end.
Matt: Jesus said that he gives eternal life (John 10:28).
Ken: Amen
Matt: And 1 John 5:13 says that we can know right now that we have eternal life. I ask because the
true Jesus of the Bible, who reveals the truth and gives eternal life is the true savior.
Ken: If I endure to the end.
Matt: It does not say that eternal life is dependent on whether or not you endure. 1 John 5:13 says,
"These things I have written to you who believe in the name of the Son of God, in order that you may
know that you have eternal life." It says that you may know, present tense, that you have eternal life.
Does your Jesus gives you this eternal life right now? Let me ask you, are you telling me that all of
your sins are forgiven and that you possess eternal life right now?
Ken: Give eternal life. Yes. Right now, Yes.
Matt: How do you receive this eternal life? What do you do to get it?
Ken: There are many steps, though I already know how you will respond. Like I said, I get this
everyday. Thanks for enlightening me.
Matt: I do not enlighten you. Only God can do that.
Matt: Then do you disagree with the eighth article of your Church which states that you are saved by
Grace after all you can do?
Ken: The 8th article of faith says nothing about what you just said.
Matt: That you are saved by Grace after all you can do?
Ken: But that is a good tactic anyways.
Matt: This is not a tactic. This is not a game. This is life and death.
Ken: Dum dum dum.
Matt: I cannot convince you through logic, intellectual traps, or proof texts. I am only trying to get
you to see who the real Jesus is.
Ken: Thanks for trying to scare me into believing.
Matt: I do not believe that anything I said was "scary." I was not intending to scare you into
anything. The real Jesus does not require your good works in order to make you a Christian.
Ken: Thanks. I do say though that you are a much more interesting dialogue than with your friend.
Ken: Maybe you and I should have a dialogue.
Matt: We are having a dialogue, right now.
Ken: I believe Grace, Faith and Works are an eternal principle.
Matt: I need to stand corrected. I just checked. The eighth article of your Church does not state what
I thought. It has been a long time since I looked at them.
Ken: No problem.
Matt: However, the third article says this: "We believe that through the Atonement of Christ, all
mankind may be saved, by obedience to the laws and ordinances of the Gospel."
Ken: Amen to that. "If you love me keep my commandments." Can I get an amen?
Matt: You see, one of the basic differences between Christianity and Mormonism aside from the
dealings with God and His nature, in his how we are forgiven of our sins. Yes, we are to keep his
Commandments, but we are not saved by keeping those Commandments.
Ken: Amen to that also.
Matt: This is clearly taught in the book of Romans chapters 4 and 5.
Ken: Amen to that also. Of course you would have to stand corrected if you think that we are saved
by our good works.
Matt: In fact, the third article of your Church contradicts the book of Romans and Ephesians and
Galatians which all teach justification by faith alone.
Matt: Mormon 3rd article says "We believe that through the Atonement of Christ, all mankind may be
saved, by obedience to the laws and ordinances of the Gospel. Which laws and ordinances must you
obey in order to be forgiven of your sins?
Ken: Obey, of course. Do you argue that we may sin and be saved in our sins?
Matt: I am asking which laws and ordinances on the Mormon Church you must obey in order to have
your sins forgiven. Can you please tell me?
Ken: There are two. And I believe you already know them.
Matt: You are confusing the difference between justification and sanctification.
Ken: No.
Matt: Justification is our legal declaration by God upon on us, sinners, that we are declared righteous
in His sight. Sanctification is the process we go through and our lives where the Holy Spirit makes us
more like Jesus.
Ken: I must go to bed. It is almost 11:00 here and I must go to bed. It was very good talking to you.

Matt: The question, then, in his how are you justified? how are you made righteous in God's site?
Ken: Please email me and we will continue to exchange. Keep the same questions and we will
continue this.
Matt: I lovingly request that you read Romans chapters 3-5 and Galatians 3-5
Ken: I am sorry. I would like to keep this up to but I have to go to bed. Plus, I need to spend some
time with my wife.
Matt: It is these chapters that speak of this issue. I hope that if you read them you will see that the
third article on your Church is in contradiction to God's word.
Ken: Ok, I will do that for tomorrow.
Matt: I understand you need to sleep.
Matt: Please understand that I am not here to bash on you. I believe you are lost and I do not want
you to go to hell.
Ken: You are a breath of fresh air, with sound logic.
Matt: What ever I am I am by the Grace of God. I want you to find the real and loving Jesus who can
fill your heart with His salvation.
Ken: Thanks for your concern. Of course I would say ditto.
Ken: Amen to that.
Ken: Good night.
Matt: I will be praying, to Jesus, for you and your salvation. Goodnight.

I can't say this was the greatest dialogue. I only hope the Holy Spirit opens his mind and heart to
understand the truth so He can come to a real relationship with Jesus.
40 Objections with Answers

Introduction

Most every unbeliever has a reason or reasons for not being Christian. They range from excuses to
misinformation. In an attempt to help you have some ideas for answering some of the basic
objections, the following answers are offered. This list could be much longer, but these forty basic
objections and responses will help get you started.

1. What is sin? p. 104


2. What is salvation? p. 104
3. What must a person do to be saved? p. 104
4. Is sincerity good enough to be saved? p. 105
5. What can you say to someone who says they aren't that bad a person? p. 106
6. What are some things you can say to an atheist who denies God's existence? p. 107
7. What might you say to someone who is already religious? p. 108
8. Is believing in God enough to be saved? p. 108
9. Why did God make us? p. 109
10. Is Jesus just like any other great man of history? p. 109
11. What makes Jesus so special? p. 110
12. Why did Jesus need to die so we can go to heaven? p. 111
13. What could you say to someone who trusts in his feelings? p. 112
14. Do all religions lead to the same place? p. 113
1. I am not a sinner.
A. Are you saying you are perfect? If you are, then you're the first perfect person I've ever met.
B. Are you saying you've never broken the Law of God? Have you ever lied, cheated, or stolen?
If you have, then you are a sinner whether you think so or not. The laws of God have
punishments (a law without a punishment is only a slogan). As a sinner, you are separated
from God (Isaiah 59:2). However, God loves you enough not to want you to be separated
from Him. He sent Jesus (1 John 4:10) to pay for sins on the cross. So, the only way to have
your sins forgiven is to put your trust in Jesus and the sacrifice He made.
C. The Bible says that everyone has sinned (Rom. 5:12). That means you, too.
2. What is sin?
A. Sin is doing what is wrong as well as not doing what is right. It is breaking the Law of God (1
John 3:4). In other words, it is doing what is against God's will. If He says do not lie and you
lie, then you have sinned. If He says do not steal and you steal then you have sinned. And,
according to God, sin separates you from Him (Isaiah 59:2).
B. Sin is an offense to God's character. Because God cannot lie, it is wrong for you to lie.
Because God cannot steal, it is wrong for you to steal. Right and wrong, then, is a
manifestation of the character of God. God is holy; He cannot sin. Sin offends Him personally
because it is His laws of right and wrong you are breaking. If you have offended Him then
you must find a way to "unoffend" Him. The problem is that you can't, but He can and has,
by offering His Son, Jesus Christ, on the cross as a sacrifice for sin.
3. I am too big a sinner.
A. Nobody is too big a sinner. The love of God and the sacrifice of Jesus is capable of cleansing
the worst of all sin. Even Hitler could have been saved if he would have turned to Christ. You
have sinned the same as anyone else. It is just that your s3ins are yours. They aren't too big
for God to wipe away. Sin has no power over God, only over you.
B. Let me ask you something. Do you think murder and adultery are serious sins? Yes? Well,
David, a man in the Bible who was called by God a man after His own heart (Acts 13:22),
was a murderer and an adulterer. He even tried to hide his sin from everyone. But God knew
his sins and exposed them. David repented and threw himself on the mercy of the Lord. God
forgave him and loved him. God loves you and He will forgive you if you put your trust in
Jesus and ask Him to forgive you of your sins (Rom. 10:9-10).
4. What is salvation?
A. Salvation is the forgiveness of sins. It is only accomplished through faith in Jesus as Savior.
He died on the cross for sins. If you want salvation, you need to trust in what Jesus did on
the cross. Only then can you have eternal life and be with God.
B. Salvation is saving a person from damnation. Damnation is judgment upon the sinner. This
judgment consists of God condemning the sinner to eternal punishment in hell. This is the
destination of all who reject God's provision for the forgiveness of sins. If you want salvation,
then you need to recognize that you are a sinner and ask Jesus to forgive you. He will.
5. What do I do to get saved?
A. Salvation is a free gift of God (Rom. 6:23). Jesus bore sin in His body (1 Pet. 2:24) and paid
the penalty for breaking the Law of God, which is spiritual death (eternal separation from
God). If you want salvation, you need to admit that you are a sinner and that you want
Jesus to forgive you of your sins. You must acknowledge that there is nothing you can do to
earn forgiveness. Pray and ask Him to forgive you. You need to trust in Jesus. Seek Him; He
will save you.
B. Repentance is part of salvation. Once saved, you should stop doing those things that are
displeasing to God. He will live in you and give you the ability and desire to resist sin (1 Cor.
10:13). When you are saved, expect to change -- for the better.
6. Is baptism necessary for salvation?
A. No. Faith in Jesus is sufficient for salvation. You don't have to do anything. Christ has done it
all. However, baptism is very important and all believers should be baptized. If you refuse
baptism after salvation, I would doubt your conversion.
B. There are Christian denominations that believe baptism is necessary for salvation. The
arguments used, on the surface, seem to be powerful. However, upon examination baptism
is found to occur after conversion and is not in anyway a cause or part of it. Take, for
example, Acts 10:44-47. While Peter was witnessing, the Holy Spirit fell upon all those who
were listening to the message...and they were hearing them speaking in tongues and
exalting God. Then Peter answered, "Surely no one can refuse the water for these to be
baptized who have received the Holy Spirit just as we did, can he?"
i. This passage shows that baptism happens after salvation. How do we know they were
saved? They were speaking in tongues -- which is a gift from God (1 Cor. 14) to
believers and they were exalting God. Non-believers do not exalt God. Also, Peter said
they had received the Holy Spirit. That is only for Christians and it happened before
baptism. (Note: speaking in tongues is simply a sign of salvation. It is not necessary
that a Christian speak in tongues as a proof of salvation. Not all speak in tongues (1 Cor.
12:30).
ii. Another set of verses applicable to this issue is 1 Cor. 1:17. Paul says, "For Christ did
not send me to baptize, but to preach the gospel..." The gospel is what saves and it is
explained in 1 Cor. 15:1-4. Baptism is not part of the gospel; it is something that the
believer does after salvation.
C. Baptism is only a symbol of that which saves and symbols don't save.
7. I am already good enough.
A. How good do you have to be to get to heaven? God is holy and requires holiness. Holiness is
purity. Even though you may think you are good enough, even one sin disqualifies you from
being in the presence of God. You could never be good enough. That is why you need Jesus.
B. The Bible says that there is none good enough. "There is none who does good, there is not
even one," (Rom. 3:12). Goodness is measured by God's standard not yours.
C. To say that you are good enough means that Christ did not have to die. But He did die to
save sinners. The Bible says if righteousness can come by good deeds then Christ didn't
need to die (Gal. 2:21), but He did, so being good isn't enough.
8. I am doing the best I can and I'm sincere.
A. Even if you could do far better than you are doing now you still can't do well enough because
you don't please God by being good (Gal. 2:21), but by accepting Jesus (John 1:12).
B. Sincerity is not the way to heaven. What if you are sincerely wrong? (Remember John 14;6?)
C. If you are relying on your sincerity, then are you saying that because you are sincere,
therefore you are good enough on your own to be with God." Don't you see, to appeal to
your sincerity is to appeal to pride, because you are appealing to something that is in you
and not God for your reason to go to heaven. I am sorry, sincerity is not enough. You must
have faith, in Jesus.
D. How long have you been doing your best? Has it worked so far? Has it given you eternal life?
9. I am skeptical.
A. Are you honestly looking for answers? If you are, I would be very willing to talk with you
more about Jesus, the Bible, or whatever else you want to talk about.
B. What are you skeptical about? Perhaps we can talk about some of the things that you feel
keeps you from a saving knowledge of Jesus.
10. I tried Christianity once.
A. The Bible says that once you are saved, you are never the same again, you are a new
creature (2 Cor. 5:17). If you have gone back to your old ways, then most probably you
were never saved. If, however, you were saved, then God won't let you stay in rebellion for
long. He will deal with you in whatever way is necessary to bring you back into fellowship
with Him.
B. Did you become a Christian by going to church or by asking Jesus to forgive you of your
sins? The latter makes you a Christian, the former doesn't.
11. I knew some Christians once and they wronged me.
A. Christians aren't perfect. They make mistakes like anyone else. I hope you can find it in your
heart to forgive them. I think that is what they would do for you.
B. Maybe they didn't know they wronged you. Was it something really bad or was it just a
mistake? Have you gone to them and spoken to them about it? Maybe if you were to forgive
them you would begin to understand the forgiveness God has for you. We all need to be
forgiven, don't you agree?
12. I'll take my chances.
A. With what, eternity? Eternity is a long time to be wrong. Why would you want to take a
gamble on something as important as your eternal destiny? It takes only a moment to trust
Christ for your salvation. There will be an eternity of pain and regret if you don't.
B. You don't take chances with guns do you? You don't take chances and run red lights do you?
Why would you take a chance on something that is far more important than these? Don't
take a chance on something eternal. It isn't worth it.
C. Jesus said He was the only way to God. He forgave sins, walked on water, calmed a storm
with a command, raised people from the dead, and rose from the dead Himself. No one else
in all of history has done that. If He can do all that, don't you think you should listen to Him.
13. I am not that bad a person.
A. Whether or not you feel you are bad or good is not the real issue. The Bible says that all
have sinned (Rom. 3:23). If all have sinned, good or bad, then all will suffer the judgment of
God. God does not require someone to be pretty good; He requires that he not sin at all. But
He knows that you cannot be sinless. That is why He gave His only begotten Son that
whosoever would believe in Him would not perish but have everlasting life (John 3:16).
B. The Bible says that our good works are filthy rags before God (Isaiah 64:6). It isn't saying
that we might not try to be good, it is saying that whatever good we do, it is not good
enough. It also says that there is none who does good (Rom. 3:12). The standard God seeks
is perfection. We cannot please God on our own. That is why Jesus died on behalf of sinners.
If you want to be good enough, then you must let God see you through the righteousness of
Jesus Christ. That is the only goodness that counts to God.
14. I am too old or too young.
A. You are never to old to trust in Jesus as your Savior. As long as you are alive you can call on
Him to forgive you of your sins. He is as close as the call of your heart.
B. (Granted there may be some who are too young to understand the gospel message, but here
we will address those who simply use that as an excuse.) Youth is a blessing from God. Don't
use it as an excuse to stay away from Him. If you can understand what sin is and your need
for deliverance from it, then you are not too young to receive Jesus as your Savior. He saves
everyone, young and old.
15. I can't believe in a God who would send people to hell.
A. Hell was originally created for Satan and his angels. In the future it will contain those who
join Satan in rejecting God. If you reject God's provision for the forgiveness of your sins then
you will join the Devil who rejected God from the beginning. Is that what you want?
B. Could you believe in a God who would become a human, suffer at the hands of humans, and
be killed by them, all so that His death could be the payment for their sins? That is
extremely loving. God is saving people who deserve to go to hell -- and we all deserve that.
Remember that the same God that sends people to hell also died for them. If they reject
what God has provided then what is God left to do? He would have to judge them.
C. Whether you believe in something or not does not change the fact of its existence. Jesus
spoke often of hell (Matt. 25:41-46; Mark 9:47-48; Luke 16:19-31) and warned us so we
would not go there. Would you say Jesus didn't know what He was talking about?
D. Are you implying that it is unjust for God to send people to hell? If so, then you accuse God
of injustice. Sin is wrong and it must be punished. What would you have God do to those
who oppose Him and do evil? Do you want Him to ignore that which is wrong? Do you want
Him to turn His head and not be holy and righteous?
16. I will worry about it in the next life.
A. That you may very well do, forever. Eternity is a long time to be wrong, especially about
Jesus.
B. God has warned us in the Bible that it is appointed for men to die once, then judgment (Heb.
9:27). After death you will be judged. Do you want to face eternity without the sacrifice of
Jesus Christ accounted to you? God hates sin and you have sinned. God will punish sinners if
they reject Jesus. However, He loves you. That is why He sent His Son to die for sins. If you
want eternal life, then you need to worry about it now. Eternity is a long time to be wrong,
especially about Jesus.
C. There is no next life. Reincarnation isn't true. The Bible says after death you face God (Heb.
9:27).

17. I don't want to give up what I like doing.


A. If you become a Christian, are you saying you must stop doing what you're doing now. That
means you know it is wrong. Let me ask you something. If you were to become a Christian
and God were to live in your heart and you looked back upon your life now, would you say to
yourself, "I did a lot of things I wish I hadn't done."? Probably so. The Bible speaks about
just such a thing. In Rom. 6:21 it says, "What benefit were you then deriving from the
things of which you are now ashamed, for the outcome of those things is death," (NASB).
What you are saying is that God will require you to give up certain things that you like to do.
Since God only wants what is good and right, and you say you don't want to give up what
your are doing, then you are saying you want what is wrong.
B. Will you let your pleasures get in the way of salvation? Is your life of sin worth an eternity of
pain? Jesus said, "What will it profit a man if he gains the whole world but loses his soul?"
(Mark 8:36).
18. Christianity is boring.
A. Then you haven't experienced it. No one who is a Christian will ever say that it is boring.
B. How do you know? Have you tried it? There are millions of Christians who have a lot of fun
being Christian. We just do it with a lot less sin, and therefore, a lot less problems. Maybe its
only your problems that keep you from getting bored.
C. What do you think we do all day, sit around fireplaces and read Bibles? We ski, swim, play
sports, read, have friends and problems like anybody else. Christianity is not boring. It is an
adventure.
19. I am an atheist. I don't believe in God.
A. An atheist is defined in two senses: Someone who says he believes there is no God and
someone who simply lacks belief in God. An atheist cannot say he knows there is no God
because he would have to know all things in order to know if there is or isn't a God. If he
says he believes there is no God ask him why he believes that way and begin there. If he
says he lacks belief in God, then ask what he does believe in and start there. I always get
around to the question of, "How did we get here?" Since creation and evolution are the only
options, I have something further to work with.
i. Evolution has a lot of problems with it. It seems to me that it takes a lot of faith to
believe that you developed out of ocean slime, simply by chance. At least as a Christian
I have the evidence of the resurrection of Christ from eye witnesses as recorded by
them in the gospels. Evolution or not, Jesus rose from the dead, said He was God, and
forgave sins. I'll put my faith in Him instead of evolution.
B. An agnostic says he doesn't know if there is or isn't a God. (Usually after saying this I
challenge them to explain the prophecies of the Old Testament fulfilled in the New. I state
how the Bible is unique that way and that only God can make prophecies that are 100%
accurate. Then I ask him to explain how that could be done if there is no God.)
C. If there is no God as you say, then in the end I lose nothing. But if there is a God like I say,
in the end you lose everything.
D. Why don't you believe in God? Is there any reason for you to intelligently reject His
existence? Or, do you simply desire not to believe in Him?
E. The Bible doesn't attempt to prove that God exists. It simply speaks as though He does.
Maybe I can't prove to you there is a God, but I can introduce Him to you through His Son
Jesus Christ and you can judge for yourself if the Words of Christ in the Bible convince you of
His existence.
a. (Note: We exist. How did we get here? An atheist's only option would be to
say evolution. If you study evolution, by reading Christian books that reveal and document its
many serious problems, then you will be able to weaken the atheist's belief in it. The logic is
simple: If evolution and creation are the only options, to remove one is to support the other.
Therefore, learn as much as you can about evolution. Your witness will be greatly
strengthened.
However, evolution does not explain the origin of life, only its development. Discussions on
origins of life are interesting in themselves.)

20. I am trying to be a Christian.


A. You become a Christian by simply putting your trust in Jesus and His sacrifice for you on the
cross. There is no trying involved. If you trust Jesus, if you ask Him to forgive you of your
sins and be your Savior, then you are a Christian. It is living like a Christian after you've
become one that is difficult.
B. If you believe that in order to become a Christian you must be good, then you
misunderstand or don't have a good understanding of salvation. A Christian is a Christian by
the gift of God (Rom. 6:23), not the work of man (Eph. 2:8-9). There is nothing you can do
to earn salvation or keep salvation. It is simply something God freely gives you. If you want
it, confess your sins, repent, turn to God, and trust Jesus as your only Savior. Then, and
only then, will you become a Christian.
21. I am already religious.
A. Who said God wants you to be religious? He wants a relationship with you. Religion is man's
attempt to reach God. Christianity is God reaching man. 1 Cor. 1:9 says that God wants you
to have fellowship with Jesus. He is talking about a relationship, someone you can talk to. He
doesn't want to weigh you down with a bunch of do's and don't's. He wants to extend a
loving hand to you and help you live a good clean life. But that cannot be done until the real
problem in you is done away with, and that is sin. Sin separates you from God (Isaiah 59:2).
If you want salvation instead of "religion," then go to Jesus. Seek Him. He will never let you
down.
B. I see. Where do you attend church?
22. I don't need God.
A. If you say you don't need Him, then you believe He exists. If you do, why would you say you
don't need Him. Isn't He the One who determines your destiny? Doesn't He have the
authority and power to do as He pleases and to send you to heaven or hell? It is foolish to
say you don't need the One who is your Creator, who loves you and has provided the way
for forgiveness of sin. You need God because only He can cleanse you from your sins.
B. What do you need? Are you really doing that well without God? Are you happy with the way
things are in your life? If you aren't, then you need Jesus. And even if you are happy, you
still need Him, because you can't take what makes you happy with you when you die.
23. I have things I need to do before I become a Christian.
A. Like what? Why do you need to do these things before you come to God? Are they bad
things or good? If they are bad, then you shouldn't do them. If they are good, why can't you
become a Christian and then do them?
B. Nothing you can do could be more important than your relationship with God. To put Him off
is unwise. What if you die before you become a Christian? Then you would be eternally
without hope.
C. Your statement implies you believe following God will mean you won't be able to do the
things you want to do. If that is true, then that means the things you intend to do would
displease God? Are you saying you prefer to do something God wouldn't want you to do? If
that is so, you are willfully sinning against God and putting yourself in a dangerous situation.
That is all the more reason you need His forgiveness.
24. I prefer to remain open minded.
A. Open mindedness means looking at everything honestly. Are you willing to do that with
Christianity? Do you want to see what Jesus has said and learn about what He can offer you?
B. If you say you are going to remain open minded and not accept Christianity, then in reality
you are being very closed minded. Maybe Christianity is true. Your open-mindedness could
keep you from discovering it.
25. I already believe in God.
A. Are you living your life as if that were true? Does your belief in God affect the way you live
or do you still do entirely as you please?
B. If you say you believe in God, then how do you know what He wants for you? Are you in
contact with Him? Do you just trust whatever you feel is right?
C. The Bible says the Devil believes in God (James 2:19) and he is lost. If all you do is simply
believe that God exists, then you are no better off than he is. It is not intellectual
acknowledgment of God's existence that God wants, but your accepting the sacrifice Jesus
made on behalf of sinners that pleases God. Simply believing is not enough. You must
choose to follow Him.
D. It is not that you believe; it is who you put your faith in. Who is this God you believe in? Is
He the Christian one? Is he Allah? Is he from another planet? Is he whatever you feel is
right? Is he loving? Believing in God is fine unless your god is false. The important things is
that you must believe in the true God, not a false one, and the true one is found in the Bible.
26. I'll choose God later.
A. If you won't choose Him now, what makes you think you'll choose Him later? The longer you
go without God, the harder it will be for you to come to Him. The longer you sin, the harder
your heart will become and the further from God you will be (Heb. 3:13). To wait is to invite
damnation. God calls you to repent from sin now, not later. Which will you choose?
B. If you say you will choose Him later, do you admit then that you need Him now? If so, then
why do you wait? You might die soon and then it would be too late.
27. There are too many hypocrites in the church.
A. Church is a good place for hypocrites, as well as liars and thieves. It is there where they will
be exposed to the Word of God and learn that hypocrisy is wrong. For you to judge those in
the church is to condemn yourself, because we are all hypocrites in one form or another.
Your recognition and condemnation of it tells me you know it is wrong. Is it hypocrisy to
point a finger at the church full of sinners when you yourself are one as well?
B. It has been said that you must be smaller than the thing you hide behind. Are you hiding
behind the hypocrisy of others to keep yourself out of church? You must realize that you are
responsible for yourself and God won't ask others about you on judgment day. He will come
to you and ask you to give an account for your life. The hypocrites in the church will also
stand before God, with or without you there.
C. People don't counterfeit pennies. Why do you think there are hypocrites? Because
Christianity is valuable.
28. Why are we here? Or, Why did God make us?
A. God made us so we could glorify Him and have fellowship with Him (1 John 1:1-3). He made
Adam and Eve and put them in the garden and then He walked in fellowship with them. He
gave them the greatest thing they could have, His love and presence. After they sinned, God
said, "Adam, where are you?" God sought Adam. In Exodus 25:8 God said to Moses while
Israel was in the wilderness, "And let them construct a sanctuary for Me, that I may dwell
among them." In the New Testament in John 1:14 it says, "And the Word became flesh and
dwelt (tabernacled) among us..." (1) God seeks our presence. He wants to have fellowship
with us. He made us to give us His love and enjoy His presence. But, man sinned and
separated Himself from God. That is why Christ died for sins, that our fellowship with God
would be restored.
29. What about those who have never heard the Gospel?
A. That is a good question. The Bible says that God is a just God. We know that whatever He
does is right. When it comes to those who have never heard the Gospel, He will do what is
right, whatever that is. But as for you, you have heard the gospel and He will judge you
according to how you respond. He is calling you to repentance, to turn from sin and come to
Him.
B. In Romans 2:11-16 it speaks about those who have never heard the Law of God and how
they will be judged according to the law that is written in their hearts. The Law written in
their hearts is the knowledge of right and wrong. Perhaps God's judgment of those without a
proper knowledge of Him is included there where it says that they will be judged according
to their own consciences that "bear witness, and their thoughts alternately accusing or else
defending them." All I know is that God will do what is right and the only way to have your
sins forgiven is through Jesus.
30. Jesus is only one of many great men of history.
A. Granted, Jesus was a great man of history. That is a fact. But, He is different from all the
other great men of history. How many great men of history rose from the dead, calmed a
sea, walked on water, raised others from the dead, healed sickness, and forgave sins? There
aren't any others that I know of. Do you know of any? These things make Him more than
great. They make Him special and unique.
B. You are right, Jesus was a great man. But let me ask you. If He were great, would He lie? Of
course not. If He were great, would He be insane? No. You see, Jesus said He was God (John
1:1,14; 10:30-33; 20:28; Col. 2:9; Phil. 2:5-8; Heb. 1:8). If He were lying, we shouldn't
listen to Him and we couldn't call Him great. If He were insane then we shouldn't listen to
Him and again we couldn't call Him great. If He is great, then He must be telling the truth.
And He was great, right?
C. John 1:1 says, John 1:1 Verse 14 says, "and the Word became flesh..." The Word is Jesus.
31. Why is there evil and suffering in the world?
A. The question implies that if a good God exists, then evil shouldn't, because God being all
powerful, should stop it.
B. We need to ask and answer two questions. First, what is evil? It is that which is against God.
It is anything morally bad or wrong. It is injurious, depraved, wicked. Some acceptable
examples might be murder, rape, stealing, lying, and cheating. Second, if we want God to
stop evil do we want Him to stop all evil or just some of it? In other words, if just some of it
then why? If He were to stop only part of the evil, then we would still be asking the question,
"Why is there evil in the world?".
Let's suppose that someone was about to commit murder. God would have to stop him,
maybe whisper in his ear, or if that didn't work do something a little more drastic like have
something fall on him, or stop his heart, or make his hands suddenly fall off. Anyway, God
would have to do something.
What if somebody wanted to steal? God would have to stop him too, right? Undoubtedly,
God's imagination would permit a more practical method than I have suggested, but the end
results would be the same.
What about lying? If someone were to tell a lie, then to be consistent wouldn't you want
God right there to stop that person from lying? After all, He couldn't let any evil occur could
He?
Let's take it a step further. Suppose someone thought something evil. Then, of course,
God would have to step in and prevent him from thinking anything bad at all, right? The end
result would be that God could not allow anyone to think freely. Since everyone thinks and
no one thinks only pure thoughts, God would be pretty busy and we wouldn't be able to
think. Anyway, at what point do we stop, at the murder level, stealing level, lying level, or
thinking level? As your questions implies, if you want God to stop evil, you would have to be
consistent and want Him to do it everywhere all the time, not just pick and choose. It
wouldn't work.
Evil is in this world partly because we give it its place but ultimately because God, in His
sovereignty, permits it and keeps it under His control.
Then you might say, "Couldn't He just make us perfect and that way we wouldn't sin?"

He already did that. He made a perfect angel, Satan, but he sinned. He made a perfect man,
Adam, and he sinned. He made a perfect woman, Eve, and she sinned. God knows what He
is doing. He made us the way we are for a purpose. We don't fully understand that purpose,
but He does.
C. God is sovereign; He has the right to do as He wishes. He has the right to permit evil for
accomplishing His ultimate will. How can He do that? Simple, look at the cross. It was by evil
means that men lied and crucified Jesus. Yet God in His infinite wisdom, used this evil for
good. It was on the cross that Jesus bore our sins in His body (1 Peter. 2:24) and it is
because of the cross that we can have forgiveness of sins.
D. Consider the biblical example of Joseph in the Old Testament. He was sold into slavery by his
brothers. Though they meant it for evil, God meant it for good (Gen. 50:20). God is so great
that nothing happens without His permission, and in that permission His ultimate plan
unfolds. In His plan He is able to use for good what man intends for evil. God is in control.
32. What makes Jesus so special?
A. Who He said He was. He said He was God. In John 8:58, Jesus said, "Truly, Truly, I say to
you, before Abraham was, I AM." When He said, "I AM," He was quoting from the Old
Testament in Exodus 3:14. That is where Moses was talking to God and asked Him His
name. God answered and said, "I AM." When Jesus said "I AM" He was claiming the name of
God for Himself and thereby claiming to be God. Other great men of history point to a
philosophy and teach good ideas. Only Jesus pointed to Himself, claimed to be God, and
spoke with authority that matched His claim.

B. What He did. Jesus forgave sins (Luke 5:20). He rose from the dead (Luke 24; John 2:19-
21), raised others from the dead (John 11:43-44), and He walked on water (John 6:19). No
one on earth has ever done the things Jesus did. There is no way around it. Jesus is special,
about that there can be no doubt.
33. Why did Jesus have to die in order for me to go to heaven?
A. Because the wages of sin is death (Rom. 6:23). Though Jesus never sinned (1 Peter. 2:22),
He bore our sins in His body on the cross (1 Peter. 2:24) and died. He died in our place.
Instead of God making us pay for our sins, He did it Himself by becoming one of us.
B. Two things happen when we sin: one to God and one to ourselves. When we sin, God is
offended. Why? Because it is His Law that we are breaking. Also, when we sin we are killed.
We don't die right there on the spot, we will face a death that is far more severe. Sin kills us
(Rom. 9:12) by causing eternal separation from God (Isaiah 59:2). God hates sin (Hab.
1:13) and sin must be punished. Since we are unable to please God because we are all
sinners, He made an offering that is pleasing to Him. That offering was the sacrifice of Jesus
on the cross. There was no other way. If there were, God would have done it.
34. What makes you think the Bible is the word of God?
A. Prophecy. The Old Testament was written before Jesus was ever born. The New Testament
was written by the men who knew Jesus, who walked with Him, ate with Him, and learned
from Him. In the O.T. there are prophecies concerning His birthplace (Micah 5:1-2), that He
would be born of a virgin (Isaiah 7:14), that He would be rejected by His own people (Isaiah
53:3), that He would be betrayed by a close friend (Isaiah 41:9), that He would die by
having His hands and feet pierced (Psalm 22:16-18), and that He would rise from the dead
(Psalm 16:10, 49:15). In the N.T. all these prophecies, and many more, are fulfilled by
Jesus. Now, this is the question you must answer: "If the Bible is not inspired from God,
then why does it have so many fulfilled prophecies?" How is that possible if the Bible were
not from God?
Only God knows the future, has power over it, and can look into it to tell us exactly what
will happen. In the Bible we have the finger prints of God: fulfilled prophecy!
B. Wisdom. The Bible is full of the greatest truths about man and God, sin, and salvation. The
Sermon on the Mount (Matthew 5) is beautiful in its wisdom, humility, and love. The Psalms
are incredible poetry of great depth and beauty. The N.T. epistles are great descriptions of
love, forgiveness, longsuffering, kindness, etc. Even if you don't want to become a Christian,
studying the truth God has revealed in the Bible will greatly help you in your life. (The aim is
not to merely to get the person to use the Bible as a guide to good living, but to encourage
him to read it. This way, he will at least be reading the Word of God and be that much closer
to conversion because God's Word will accomplish what He wants it to (Isaiah 55:11).
35. The New Testament was written so that it would only look like Jesus fulfilled prophecy.
A. Then what you are saying is that the N.T. writers lied about Jesus. He really didn't rise from
the dead and all those miracles about Him are really false, right?
B. I could see your point, but there is just one problem. How do you account for the writers
of the N.T. teaching about truth, love, honesty, giving, etc. all based on lies? Why would
they suffer hardships like beatings, starvation, shipwreck, imprisonments, and finally
execution for nothing but lies? What you are saying doesn't make any sense and raises more
questions than it answers.
The only logical explanation is that the fulfilled prophecies really did happen. Jesus
actually rose from the dead. He performed miracles and He forgave sins. He forgave sins
then and He can still do it now. My sins are forgiven, are yours?
36. The Bible is full of contradictions.
A. Really. Do you know of any? Could you quote me one or two?
B. (Just in case someone actually does quote what he thinks is a contradiction, it is up to you to
give a competent answer (1 Peter. 3:15). If you can't, don't worry. Simply tell him that you
will research it and get back with him, and make sure you do.
C. There are areas of Scripture that are difficult to understand. This does not mean the Bible is
untrustworthy. A very good book to have is the Encyclopedia of Bible Difficulties by Gleason
Archer, Zondervan Publishing House, (Grand Rapids Michigan).
37. How do I know which religion is right?

A. (This is a difficult question to answer because it involves discussing some principles that the
person you are witnessing to may or may not agree with. For example, does he or she agree
with you that truth is knowable, that God would attempt to communicate with His people, or
that only one religion may be right? Usually, I start by acknowledging the difficulty of coming
to an easy answer. However, I tell them that I do have an answer; I am sure it is the right
one, because it is an answer based on evidence. What kind of evidence? Prophecy and its
fulfillment (see question # 34), Jesus and His miracles, the resurrection of Christ, etc. Then I
ask that person if he or she knows of these things happening in other religions. 31 The
answer is invariably, "No." Then I point out that they have only happened in Christianity. If
any religion were true, Christianity fits the bill.

38. Religion is whatever you feel is right.


A. How do you know what you feel is right? Haven't your feelings ever turned out to be wrong?
Are you are saying that what you feel determines truth? If so, then you are putting yourself
in the place of God and looking to yourself for what you "feel" is right.
B. If religion is whatever you feel is right then that could lead to chaos. What if some people
had a religion where they felt stealing was acceptable? And what about lying and cheating?
Would you trust someone who believed in a religion that felt it was alright to steal, lie, and
cheat?
C. Hitler felt killing Jews was right. He was wrong. The Bible says that the heart is deceitful and
untrustworthy (Jer. 17:9). If you could come to know truth by what you felt, then the Bible,
which is the revelation of God, didn't need to be written. But, it has been written and it has
revealed that only God is the Source of truth, not your feelings.
D. I've never known truth to contradict itself. What if someone felt that something was right
and another person felt it was wrong? Would they both be right? If your statement is true,
then how can there be a contradiction like that if feelings determined truth?
39. All religions are different paths to the same place.
A. If all religions are different paths to the same place then why do the paths contradict each
other? Does truth contradict itself? Let's review the teachings of just three religions:
B. Buddhism is pantheistic and says there is no personal God and everyone can reach
Godlikeness on his own. Islam says that Jesus was just a prophet and not the only way to
God. Christianity says that there is a personal God and that the only way to Him is through
Jesus (John 14:6). If these three religions are, as you say, different paths to the same place,
then why do they contradict each other? Does truth contradict itself?
C. What about dinosaurs and evolution?
D. Also, you could read a couple of books: Evolution The Fossils Say No! by Duane T. Gish
(Creation Life Publishers, San Diego), and Man's Origin, Man's Destiny by A. E. Wilder-Smith.
Bethany House Publishers, (Minneapolis, Minn.). Both books will help you greatly.
E. Even if evolution were true (it isn't -- but just for the sake of argument), does that mean
there is no God? How do you know God didn't use it to get us here? (I am not teaching that
evolution is true, nor that God used it, which is called theistic evolution, I am simply
reasoning with them.) If you believe in evolution does that mean you aren't a sinner? God
won't accept the excuse that you believed in evolution and not Him.
F. Have you examined evolution to see if it is true? Evolution is not all that you are led to
believe. There are all kinds of problems in the fossil record. New theories are being raised all
31
Note: Be careful. Just because someone does not know if there has been any similar occurrences in
other religions doesn't mean that there aren't. You should point that out. However, no other religion in
the world has ever made the claims that Christianity has, and lived up to them.
the time to account for why there aren't any undisputed transitional forms found between
any species of any kind, anywhere, anytime in all the fossil record. But, you wouldn't know
these things because you haven't studied. You need to know the facts about evolution and
you need to know the facts about Jesus.
Evidence and Answers
Introduction
Without a doubt, Christianity is under attack. It has always been under attack ever since its
inception when unbelievers killed Jesus and persecuted the church. In fact, the truth of God has been
under attack since the creation of the world when the evil one entered the Garden of Eden and
contradicted God's word bringing death. In the 20th century alone, more Christians were killed for
their faith than in all other centuries combined
In this pluralistic society where cults abound (i.e., Mormonism, Jehovah's Witnesses), where
relativism has taken deep root, where the universities teach liberalism, and where evolution theory
and atheism are growing, the absolute claims of Christianity are being met with increasing hostility.
Unfortunately, the average Christian is very ill prepared to meet the present battle, let alone the more
difficult future ones. Therefore, it is necessary, not recommended, but absolutely necessary that the
Christian learn how to defend the truth of God's revelation and carry that truth into society and into
politics. If he doesn't, he will eventually lose the freedoms granted to him and become the prey of the
secular lions.
This section "Evidence and Answers" is intended to help the Christian with documentation and
answers to difficult questions so that he/she can more adequately and easily defend the truth.
Remember, we have the truth and we cannot hide it under a bushel. Christianity is not something that
we do on Sundays. It is a way of life. It is the truth that the world needs in order to obtain
forgiveness of sins and escape the coming judgment. Therefore, Christians should be determined to
know how to defend it better and how to carry it out into the world in order to "make disciples of all
nations..." (Matt. 28:19). The goal of this section, and all of CARM, is to help you do just that.
I hope that this information is helpful to you and is an encouragement to you as well. Do not let
the lies of the secular overwhelm the truth of the sacred. Stand up and fight for the truth. Be loving
and kind, but be strong and unyielding in your fight against sin and in your proclamation of the truth of
Jesus.

1. Are there textual variations in biblical manuscripts? pp. 114-115


2. What do the miracles of Christ attest to? p. 116
3. Why is the manuscript evidence for the New Testament superior to anything else of ancient
times? pp. 117-119
4. Who are some ancient historians who mention New Testament accounts? pp. 120-122
5. What kind of archaeological evidence is there supporting the Bible? pp. 123-125
6. What is a key element in determining the dates of the writings of the gospel? p. 128
7. What is some evidence of the inspiration of the Bible? pp. 131-133
8. What is the gospel of Q and why is it potentially important? pp. 143-144
9. If a non biblical book is mentioned in the Bible does is that book inspired or lost? p. 148
10. Does the Bible provide extraordinary evidence for extraordinary claims? pp. 157-158
11. What is Occam's Rasor and why is it important? p. 163
12. What is the swoon theory and why does it not work? pp. 165-166
13. Was Jesus just a myth? pp. 173-174
14. Who is Apollonius of Tyana? p. 186
15. Doesn't the religion of Mithra prove that Christianity is false? p. 187
16. Is it intolerant to say that Christianity is the only true religion? 197
17. Why would God have to die to save people from Himself? p. 199
Inerrancy and inspiration of the Bible

The Christian Church as a whole claims that the Bible is inspired and inerrant. This means that
God is the one who moved through the writers to communicate to us the words which God wanted us
to hear. This inspiration, however, is not a dictation, but a movement of God's spirit through the
writer, utilizing the personality and style of the writer. Inerrancy means that all that is written in the
inspired documents is without error. Now, there is a comment worth mentioning here. Inspiration and
inerrancy applies to the original writings, not to the copies. In other words, it is the original writings
that are without error. The copies, sadly, have copyist errors in them.
Therefore, when critics of the Bible point out apparent contradictions, what they are doing is either
failing to understand the context of the passages they are examining, or they have encountered a
scribal copying error. The fact is that there are indeed copyist errors on the biblical documents and
they account for many alleged contradictions. Remember, it is the autographs (original writings) that
are inspired and inerrant, not the copies. The copies we have now are copies of inspired documents.
The copies are not themselves "inspired"; that is, they have no guarantee of being 100% textually
pure. Does this then mean that we can't trust the Bible? Not at all. The copies are so accurate that
all of the biblical documents are 98.5% textually pure. The 1.5% that is in question is mainly nothing
more than spelling errors and occasional word omissions like the words "the", "but", etc. This reduces
any serious textual issues to a fraction of the 1.5%. Nevertheless, nothing affects doctrinal truths. In
fact, nothing in ancient history even comes close to the accuracy of the New Testament documents. If
the New Testament is disallowed, then all other documents of ancient history (Plato, Aristotle, Homer,
etc.), must also be disallowed because the biblical documents are far superior in their copying
accuracy than any other ancient literature in existence. See the chart below for further information on
this.
Nevertheless, following is a list of the types of errors that have crept into the Bible:

• Dittography - Writing twice what should have been written once.

 A good example would be writing "latter" instead of "later." "Latter" means nearest the
end. "Later" means after something else.
 Fission - Improperly dividing one word into to words.
 Example: "nowhere" into "now here."
 Fusion - Combining the last letter of one word with the first letter of the next word.
 "Look it is there in the cabinet... or Look it is therein the cabinet."
 Haplography - Writing once what should have been written twice.
 A good example would be "later" instead of "latter." "Later" means after something
else. "Latter" means nearest the end.
 Homophony - Writing a word with a different meaning for another word when both words have
the exact same pronunciation.
 Meat and meet have the exact same sound but different meanings. Also, there and their
and they're are another example.
 Metathesis - An improper exchange in the order of letters.
 Instead of writing "mast," someone writes "mats," or "cast" and cats."

Does this mean that the Bible we hold in our hand is not inspired? Not at all. Inspiration comes
from God and when He inspired the Bible, it was perfect. Our copies of the original documents are not
perfect, but they are very close to being so. The critics often mistakenly assume that even the copies
are supposed to be perfect. But when I point out that God never said the copies would be perfect,
they then ask how can the Bible be trusted at all? Quite simply, it is redundant in its facts and
information sufficiently to guarantee accuracy.
Compared to other ancient documents, the New Testament, for example, has far more textual
evidence in its favor than any other ancient writing. Please consider the chart below.
Author When Written Earliest Copy Time Span No. of Copies
Homer (Iliad) 900 BC 400 BC 500 years 643
Ceasar (The
100 - 44 BC 900 AD 1,000 years 10
Gallic Wars)
Plato
427 - 347 BC 900 AD 1,200 years 7
(Tetralogies)
Aristotle 384 - 322 BC 1,100 AD 1,400 years 49
Herodotus
480 - 425 BC 900 AD 1,300 years 8
(History)
Euripedes 480 - 406 BC 1,100 AD 1,500 years 9
New Testament 50 - 90 A.D. 130 AD 30 years 24,000
This chart was adapted from charts in Evidence that Demands a Verdict, by Josh McDowell, 1979,
pages 42 and 43.

If the Bible cannot be trusted as being reliable because it has only a small percentage of copyist
errors, then neither can the above documents be trusted that have far less textual support.
Therefore, we can see that the Bible is an ancient document that has withstood thousands of years
of transmission with remarkable accuracy and clarity. We can trust it to be what it says it is: the
word of God.
The Miracles of Jesus
The miracles of Jesus attest to who Jesus was, God in flesh (John 1:1,14; Col. 2:9). Jesus used
the Divine Name for Himself (John 8:58), the same Divine Name used by God when Moses asked God
what His name was in Exodus 3:14. However, a person claiming to be God is not that uncommon in
the world. Mental institutions are filled with those who claim to be divine. But, Jesus doesn't fit the
category of a crazy person. If anything, Jesus deserves the utmost respect and admiration for His
humility, love, teaching, and self-sacrifice. But, even though there are other great teachers in history
who have exhibited extraordinary love and compassion, none have also claimed to be God in flesh --
and then demonstrated it by performing miracles. This is why the miracles of Jesus are what verify
the claims of Jesus about Himself and about Him being the only way to salvation ( John 14:6).
The critics of Christianity must either deny Jesus' existence, the biblical documents, say that the
accounts were borrowed or made up, or deny that miracles occur at all. If their objections cannot be
reasonably maintained, then they would be forced to make decisions about Jesus' claims to be divine,
His miracles, and His physical resurrection with all the implications that they entail. This means that
people must decide what they are going to do with Jesus. Will they believe Him for who He claimed to
be or reject Him? That is what it comes down to.
In Christianity, Jesus is the creator (John 1:1-3,14; Col. 1:16-17), the Lord to whom we call (1
Cor. 1:2), the one who bore our sins in His body on the cross (1 Pet. 2:24), and the one who enables
all who call upon His name (Rom. 10:13) to be saved by faith (Rom. 5:1; Eph. 2:8-9). He is a
remarkable figure whether anyone wants to admit it or not.
Following is a partial list of some of the miracles of Jesus. It is these miracles that attest to who
Jesus is because His remarkable deeds confirm His incredible claim to be God in flesh, the only way to
salvation.

• Jesus was born of a virgin (Matt. 1:25).


• Jesus changed water into wine (John 2:6-10).
• Jesus caused the disciples to catch a large load of fish (Luke 5:4-6).
• Jesus cast out demons (Matt. 8:28-32; 15:22-28).
• Jesus healed lepers (Matt. 8:3; Luke 17:14).
• Jesus healed diseases (Matt. 4:23,24; Luke 6:17-19)
• Jesus healed the paralytic (Mark 2:3-12).
• Jesus raised the dead (Matt. 9:25; John 11:43-44).
• Jesus restored sight to the blind (Matt. 9:27-30; John 9:1-7).
• Jesus cured deafness (Mark 7:32-35).
• Jesus fed the multitude (Matt. 14:15-21; Matt. 15:32-38).
• Jesus walked on water (Matt. 8:26-27).
• Jesus healed the sick (Matt. 8:5-13; 9:22).
• Jesus forgave sins (Mark 2:5).
• Jesus calmed a storm with a command (Matt. 8:22-27; Mark 4:39).
• Jesus was transfigured (Matt. 17:1-8).
• Jesus rose from the dead (Luke 24:39; John 20:27).
• Jesus appeared to disciples after resurrection (John 20:19).
• Jesus ascended into heaven (Acts 1:9).

What is so important here is that Jesus claimed to be God (John 5:18; 8:24; 8:58 -- see Exodus
3:14) and the only way to salvation (John 14:6). If He really did rise from the dead and perform those
miracles, then what He said about Himself -- and who He claimed to be -- become vitally important.
Either the list above is a fabrication or it is not. Your presuppositions will determine which category
they fall into. You must ask yourself it the evidence for the Bible's reliability and the eyewitness
accounts recorded there is sufficient to warrant a serious consideration that what is written is true.
Manuscript evidence for superior New Testament reliability
The New Testament is constantly under attack and its reliability and accuracy are often contested
by critics. But, if the critics want to disregard the New Testament, then they must also disregard other
ancient writings by Plato, Aristotle, and Homer. This is because the New Testament documents are
better preserved and more numerous than any other ancient writing. Because the copies are so
numerous, they can be cross checked for accuracy. This process has determined that the biblical
documents are extremely consistent and accurate.
There are presently 5,686 Greek manuscripts in existence today for the New Testament. 32 If we
were to compare the number of New Testament manuscripts to other ancient writings, we find that the
New Testament manuscripts far outweigh the others in quantity.

Approximate
Number
Date Earliest Time Span Accuracy of
Author33 of
Written Copy between Copies
Copies
original & copy
died 55 or 53
Lucretius 1100 yrs 2 ----
B.C.
Pliny 61-113 A.D. 850 A.D. 750 yrs 7 ----
Plato 427-347 B.C. 900 A.D. 1200 yrs 7 ----
Demosthenes 4th Cent. B.C. 1100 A.D. 800 yrs 8 ----
Herodotus 480-425 B.C. 900 A.D. 1300 yrs 8 ----
Suetonius 75-160 A.D. 950 A.D. 800 yrs 8 ----
Thucydides 460-400 B.C. 900 A.D. 1300 yrs 8 ----
Euripides 480-406 B.C. 1100 A.D. 1300 yrs 9 ----
Aristophanes 450-385 B.C. 900 A.D. 1200 10 ----
Caesar 100-44 B.C. 900 A.D. 1000 10 ----
Livy 59 BC-AD 17 ---- ??? 20 ----
Tacitus circa 100 A.D. 1100 A.D. 1000 yrs 20 ----
Aristotle 384-322 B.C. 1100 A.D. 1400 49 ----
Sophocles 496-406 B.C. 1000 A.D. 1400 yrs 193 ----
Homer (Iliad) 900 B.C. 400 B.C. 500 yrs 643 95%
2nd Cent.
New 1st Cent. A.D. A.D. less than 100
5600 99.5%
Testament (50-100 A.D. (c. 130 years
A.D. f.)

32
Norman Geisler & Peter Bocchino, Unshakeable Foundations, (Minneapolis, MN: Bethany House Publishers,
2001) p. 256.
33
This chart was adapted from three sources: 1) Christian Apologetics, by Norman Geisler, 1976, p. 307; 2) the
article "Archaeology and History attest to the Reliability of the Bible," by Richard M. Fales, Ph.D., in The Evidence
Bible, Compiled by Ray Comfort, Bridge-Logos Publishers, Gainesville, FL, 2001, p. 163; and 3) A Ready Defense,
by Josh Mcdowell, 1993, p. 45.
As you can see, there are thousands more New Testament Greek manuscripts than any other
ancient writing. The internal consistency of the New Testament documents is about 99.5% textually
pure. That is an amazing accuracy. In addition there are over 19,000 copies in the Syriac, Latin,
Coptic, and Aramaic languages. The total supporting New Testament manuscript base is over 24,000.

Almost all biblical scholars agree that the New Testament documents were all written before the
close of the first century. If Jesus was crucified in 30 A.D., then that means that the entire New
Testament was completed within 70 years. This is important because it means there were plenty of
people around when the New Testament documents were penned who could have contested the
writings. In other words, those who wrote the documents knew that if they were inaccurate, plenty of
people would have pointed it out. But, we have absolutely no ancient documents contemporary with
the first century that contest the New Testament texts.
Furthermore, another important aspect of this discussion is the fact that we have a fragment of the
gospel of John that dates back to around 29 years from the original writing. This is extremely close to
the original writing date. This is simply unheard of in any other ancient writing and it demonstrates
that the Gospel of John is a first century document.
Below is a chart with some of the oldest extant New Testament manuscripts compared to when
they were originally penned. Compare these time spans with the next closest which is Homer's Iliad
where the closest copy from the original is 500 years later. Undoubtedly, that period of time allows for
more textual corruption in its transmission. How much less so for the New Testament documents?

Important Date Approx.


MSS
Manuscript Contents Original Time Location
Date
Papyri Written Span

p52 circa
circa John Rylands Library,
(John Rylands John 18:31-33,37-38 125 29 yrs
96 A.D. Manchester, England
Fragment)34 A.D.
Rom. 5:17-6:3,5-14; 8:15-
25, 27-35, 37-9:32; 10:1-
Chester Beatty
P46 11, 22, 24-33, 35-14:8,9-
circa Museum, Dublin & Ann
(Chester 15:9, 11-33; 16:1-23, 25- Approx.
50's-70's 200 Arbor, Michigan,
Beatty 27; Heb.; 1 & 2 Cor., Eph., 150 yrs
A.D. University of Michigan
Papyrus) Gal., Phil., Col.; 1 Thess.
library
1:1,9-10; 2:1-3; 5:5-9, 23-
28
P66 circa
John 1:1-6:11,35-14:26; Approx.
(Bodmer 70's 200 Cologne, Geneva
fragment of 14:29-21:9 130 yrs
Papyrus) A.D.
circa Barcelona, Fundacion
67 Approx.
P Matt. 3:9,15; 5:20-22, 25-28 200 San Lucas Evangelista,
130 yrs
A.D. P. Barc.1

If the critics of the Bible dismiss the New Testament as reliable information, then they must also

"Deissmann was convinced that p52 was written well within the reign of Hadrian (A.D. 117-38) and perhaps even
34

during the time of Trajan (A.D. 98-117)" (Footnote #2 found on pg. 39 of The Text of the New Testament, by
Bruce M. Metzger, 2nd Ed. 1968, Oxford University Press, NY, NY). Bruce Metzger has authored more than 50
books. He holds two Masters Degrees, a Ph.D. and has been awarded several honorary doctorates. "He is past
president of the Society of Biblical Literature, the International Society fo New Testament Studies, an the North
American Patristic Society." -- From, The Case for Christ, by Lee Strobel, Zondervan Publishers, 1998, Grand
Rapids, MI: pg. 57.
dismiss the reliability of the writings of Plato, Aristotle, Caesar, Homer, and the other authors
mentioned in the chart at the beginning of the paper. On the other hand, if the critics acknowledge
the historicity and writings of those other individuals, then they must also retain the historicity and
writings of the New Testament authors; after all, the evidence for the New Testament's reliability is far
greater than the others. The Christian has substantially superior criteria for affirming the New
Testament documents than he does for any other ancient writing. It is good evidence on which to
base the trust in the reliability of the New Testament.
Illustration of Bible text manuscript tree and variant readings
The following diagram illustrates manuscript corruptions in the biblical texts that are produced, for
whatever reason, and copied down to later manuscripts. The purpose of the illustration is to show how
errors are copied down from one manuscript to another, how they are counted, and how we can
determine which is the correct reading.
In this example, of the 26 existing manuscripts (represented by solid black and red sheets) nine of
them have a textual problem where a phrase was incorrectly copied. Therefore, in this illustration, we
would have a total of nine variants in 26 manuscripts. But, it is really only one.
However, manuscripts can be categorized in family trees by analyzing their location of discovery,
jars found in, type of papyri written on, type of ink used, style of writing, etc. Therefore, daughter
manuscripts can be matched very accurately to father manuscripts.
In this example we see that the word "only" was omitted from a 3rd century document and copied
in subsequent, daughter documents. All we need to do is to take a look at the manuscripts and even
though we see nine variants here, actually we can tell that there is only one which has been copied.
Also, we can accurately determine which is the correct reading by looking at the father document from
the 2nd century.
With this type of method, the New Testament documents can be reconstructed with an incredible
accuracy. Furthermore, the New Testament is approximately 99.5% textually pure. This means that
of all the manuscripts in existence they agree completely 99.5% of the time. Of the variants that
occur, mostly are easily explainable and very few have any effect on the meaning of passages. In all,
no New Testament doctrine is affected by any variant reading.
Non biblical accounts of New Testament events and/or people

1. Flavius Josephus (AD 37?-101?, a Jewish historian) mentions John the Baptist and
Herod - Antiquities, Book 18, ch. 5, par. 2
A. "Now some of the Jews thought that the destruction of Herod's army came from God, and
that very justly, as a punishment of what he did against John, that was called the Baptist:
for Herod slew him, who was a good man, and commanded the Jews to exercise virtue, both
as to righteousness towards one another, and piety towards God, and so to come to
baptism; for that the washing [with water] would be acceptable to him, if they made use of
it, not in order to the putting away [or the remission] of some sins [only], but for the
purification of the body; supposing still that the soul was thoroughly purified beforehand by
righteousness."
B. Note: There is dispute as to the reliability of the Josephus accounts. However, there is no
textual/manuscript reason for doubting them since the extant Greek manuscripts all agree
with the texts in question; namely, the quotes shown on this page. However, the reason the
quotes are in doubt is because of the text in italics in the various quotes; they seem a little
too favorable regarding Christ. Also, it appears that the writings of Josephus were
transmitted to us through the Christian community.
2. Flavius Josephus (AD 37-101?) mentions Jesus – Ant., Book 18, ch. 3, par. 3.
A. Now there was about this time Jesus, a wise man, if it be lawful to call him a man; for he
was a doer of wonderful works, a teacher of such men as receive the truth with pleasure. He
drew over to him both many of the Jews and many of the Gentiles. He was [the] Christ. And
when Pilate, at the suggestion of the principal men amongst us, had condemned him to the
cross, (9) those that loved him at the first did not forsake him; for he appeared to them
alive again the third day; (10) as the divine prophets had foretold these and ten thousand
other wonderful things concerning him. And the tribe of Christians, so named from him, are
not extinct at this day.
i. There is debate among scholars as to the authenticity of this quote since it is so
favorable to Jesus. For an examination of this please see Regarding the quotes from the
historian Josephus about Jesus.
3. Flavius Josephus (AD 37?-101?) mentions James, the brother of Jesus - Antiquities,
Book 20, ch. 19.
A. "Festus was now dead, and Albinus was but upon the road; so he assembled the sanhedrim
of judges, and brought before them the brother of Jesus, who was called Christ, whose name
was James, and some others, [or, some of his companions]; and when he had formed an
accusation against them as breakers of the law, he delivered them to be stoned: but as for
those who seemed the most equitable of the citizens, and such as were the most uneasy at
the breach of the laws, they disliked what was done."
4. Flavius Josephus (AD 37?-101?) mentions Ananias the High Priest who was mentioned
in Acts 23:2
A. Now as soon as Albinus was come to the city of Jerusalem, he used all his endeavors and
care that the country might be kept in peace, and this by destroying many of the Sicarii. But
as for the high priest, Ananias (25) he increased in glory every day, and this to a great
degree, and had obtained the favor and esteem of the citizens in a signal manner; for he
was a great hoarder up of money
B. Acts 23:2, "And the high priest Ananias commanded those standing beside him to strike
him [Paul] on the mouth."
5. Tacitus (A.D. c.55-A.D. c.117, Roman historian) mentions "christus" who is Jesus -
Annals
A. "Consequently, to get rid of the report, Nero fastened the guilt and inflicted the most
exquisite tortures on a class hated for their abominations, called Christians by the populace.
Christus, from whom the name had its origin, suffered the extreme penalty during the reign
of Tiberius at the hands of one of our procurators, Pontius Pilatus, and a most mischievous
superstition, thus checked for the moment, again broke out not only in Judaea, the first
source of the evil, but even in Rome, where all things hideous and shameful from every part
of the world find their centre and become popular."
i. Ref. from http://classics.mit.edu/Tacitus/annals.mb.txt
6. Thallus Circa AD 52, eclipse of the sun. Thallus wrote a history of the Eastern Mediterranean
world from the Trojan War to his own time. His writings are only found as citations by others.
Julius Africanus who wrote about AD 221 mentioned Thallus' account of an eclipse of the sun.
A. "On the whole world there pressed a most fearful darkness; and the rocks were rent by an
earthquake, and many places in Judea and other districts were thrown down. This darkness
Thallus, in the third book of his History, calls, as appears to me without reason, an eclipse of
the sun."
i. Is this a reference to the eclipse at the crucifixion? Luke 23:44-45, "And it was now
about the sixth hour, and darkness fell over the whole land until the ninth hour, 45 the
sun being obscured; and the veil of the temple was torn in two."
ii. The oddity is that Jesus' crucifixion occurred at the Passover which was a full moon. It
is not possible for a solar eclipse to occur at a full moon. Note that Julius Africanus
draws the conclusion that Thallus' mentioning of the eclipse was describing the one at
Jesus' crucifixion. It may not have been.
iii. Julius Africanus, Extant Writings, XVIII in the Ante–Nicene Fathers, ed. by Alexander
Roberts and James Donaldson (Grand Rapids: Eerdmans, 1973), vol. VI, p. 130. as cited
in Habermas, Gary R., The Historical Jesus: Ancient Evidence for the Life of Christ,
(Joplin, MO: College Press Publishing Company) 1996.
7. Pliny the Younger mentioned Christ. Pliny was governor of Bithynia in Asia Minor. Pliny
wrote ten books. The tenth around AD 112.
A. "They (the Christians) were in the habit of meeting on a certain fixed day before it was light,
when they sang in alternate verses a hymn to Christ, as to a god, and bound themselves by
a solemn oath, not to any wicked deeds, but never to commit any fraud, theft or adultery,
never to falsify their word, nor deny a trust when they should be called upon to deliver it up;
after which it was their custom to separate, and then reassemble to partake of food—but
food of an ordinary and innocent kind."
i. Pliny, Letters, transl. by William Melmoth, rev. by W.M.L. Hutchinson (Cambridge:
Harvard Univ. Press, 1935), vol. II, X:96 as cited in Habermas, Gary R., The Historical
Jesus: Ancient Evidence for the Life of Christ, (Joplin, MO: College Press Publishing
Company) 1996.
8. The Talmud
A. "On the eve of the Passover Yeshu was hanged. For forty days before the execution took
place, a herald went forth and cried, "He is going forth to be stoned because he has
practiced sorcery and enticed Israel to apostasy. Any one who can say anything in his favor,
let him come forward and plead on his behalf." But since nothing was brought forward in his
favor he was hanged on the eve of the Passover!"
i. Gal. 3:13, "Christ hath redeemed us from the curse of the law, being made a curse for
us: for it is written, Cursed is every one that hangeth on a tree."
ii. Luke 22:1, "Now the Feast of Unleavened Bread, which is called the Passover, was
approaching. 2And the chief priests and the scribes were seeking how they might put
Him to death; for they were afraid of the people."
iii. This quotation was taken from the reading in The Babylonian Talmud, transl. by I.
Epstein (London: Soncino, 1935), vol. III, Sanhedrin 43a, p. 281 as cited in Habermas,
Gary R., The Historical Jesus: Ancient Evidence for the Life of Christ, (Joplin, MO:
College Press Publishing Company) 1996.

9. Lucian (circa 120-after 180) mentions Jesus. Greek writer and rhetorician.
A. "The Christians, you know, worship a man to this day—the distinguished personage who
introduced their novel rites, and was crucified on that account. . . . You see, these misguided
creatures start with the general conviction that they are immortal for all time, which explains
the contempt of death and voluntary self-devotion which are so common among them; and
then it was impressed on them by their original lawgiver that they are all brothers, from the
moment that they are converted, and deny the gods of Greece, and worship the crucified
sage, and live after his laws. All this they take quite on faith, with the result that they
despise all worldly goods alike, regarding them merely as common property."
i. Lucian, The Death of Peregrine, 11–13, in The Works of Lucian of Samosata, transl. by
H.W. Fowler and F.G. Fowler, 4 vols. (Oxford: Clarendon, 1949), vol. 4, as cited in
Habermas, Gary R., The Historical Jesus: Ancient Evidence for the Life of Christ, (Joplin,
MO: College Press Publishing Company) 1996.
ii. Though Lucian opposed Christianity, he acknowledges Jesus, that Jesus was crucified,
that Christians worship him, and that this was done by faith.

___________________
Sources

• McDowell, Josh, Evidence that Demands a Verdict, San Bernardino, CA, Here's Life Publishers,
Inc. 1979.
• Habermas, Gary R., The Historical Jesus: Ancient Evidence for the Life of Christ, (Joplin, MO:
College Press Publishing Company) 1996.
• Encarta on the Web at http://encarta.msn.com .
Archaeological Evidence verifying biblical cities

There is very little doubt in anyone's mind about the reality of so many of the Old and New
Testament cities mentioned in the Bible. Therefore it is hardly necessary to document their existence.
Nevertheless, following is a partial list of some of the cities mentioned in the Bible that have been
found and excavated by archaeologists. This is simply more evidence that the Bible describes actual
locations that can be verified. This means that at the very least, the Bible accurately reflects the
locations and cities of ancient times.
Remember, this is only a partial list. There are hundreds of biblical cities that have been verified in
archaeological digs.

1. Arad
A. Num. 21:1, "When the Canaanite, the king of Arad, who lived in the Negev, heard that
Israel was coming by the way of Atharim, then he fought against Israel, and took some of
them captive."
B. Num. 33:40, "Now the Canaanite, the king of Arad who lived in the Negev in the land of
Canaan, heard of the coming of the sons of Israel."
i. "Arad 30 km NE of Beersheba, excavated from 1962 to 1974 by Y. Aharoni and R. B. K.
Amiran." (The New Bible Dictionary, (Wheaton, Illinois: Tyndale House Publishers, Inc.;
1962.)
ii. "The site consists of an upper mound or acropolis, where excavation has revealed an
Iron Age (post thirteenth century b.c." (Achtemeier, Paul J., Th.D., Harper’s Bible
Dictionary, (San Francisco: Harper and Row, Publishers, Inc.) 1985.
iii. The remains of a Hebrew temple were uncovered at Arad, (Horn, Siegfried H., Biblical
Archaeology: a Generation of Discovery; Andrews University, Berrien Springs, Michigan;
1985. p.45-46.)
2. Bethel
A. Amos 7:12-13, "Then Amaziah said to Amos, "Go, you seer, flee away to the land of Judah,
and there eat bread and there do your prophesying! 13 "But no longer prophesy at Bethel,
for it is a sanctuary of the king and a royal residence."
i. "W. F. Albright made a trial excavation at Bethel in 1927. Albright then mounted a full
excavation in 1934. His assistant that year, J. L. Kelso, continued the excavation in
1954, 1957, and 1960." (Achtemeier, Paul J., Th.D., Harper’s Bible Dictionary, (San
Francisco: Harper and Row, Publishers, Inc.; 1985.)
3. Capernaum
A. Matt. 17:24, "And when they had come to Capernaum, those who collected the two-
drachma tax came to Peter, and said, "Does your teacher not pay the two-drachma tax?"
i. "Identified since 1856 with Tell Hum, Capernaum has been sporadically excavated for
the past 130 years." (Achtemeier, Paul J., Th.D., Harper’s Bible Dictionary, (San
Francisco: Harper and Row, Publishers, Inc.; 1985.)
4. Chorazin
A. Matt. 11:21, "Woe to you, Chorazin! Woe to you, Bethsaida! For if the miracles had
occurred in Tyre and Sidon which occurred in you, they would have repented long ago in
sackcloth and ashes."
i. "Excavations of the now deserted town indicate that it once covered an area of twelve
acres and was built on a series of terraces with the basalt stone local to this
mountainous region." (Achtemeier, Paul J., Th.D., Harper’s Bible Dictionary, (San
Francisco: Harper and Row, Publishers, Inc.; 1985.)
5. Dan
A. Judges 18:29, "And they called the name of the city Dan, after the name of Dan their
father who was born in Israel; however, the name of the city formerly was Laish."
i. "The excavation of Dan began in 1966 under the direction of Avraham Biran." (Horn,
Siegfried H., Biblical Archaeology: a Generation of Discovery; Andrews University,
Berrien Springs, Michigan; 1985. p. 42)
ii. "Formerly called Laish, it is mentioned in the execration texts, the eighteenth-century
b.c. Mari tablets, and the records of the Egyptian pharaoh Thutmose III. It is identified
with Tel Dan (modern Tell el-Qadi) covering about 50 acres in the center of a fertile
valley near one of the principal springs feeding the Jordan River...Tel Dan has been
excavated by A. Biran since 1966. The earliest occupation, probably the full extent of the
tell, goes back to about the middle of the third millennium b.c." (Achtemeier, Paul J.,
Th.D., Harper’s Bible Dictionary; San Francisco: Harper and Row, Publishers, Inc.;
1985.)
6. Ephesus
A. Eph. 1:1, "Paul, an apostle of Christ Jesus by the will of God, to the saints who are at
Ephesus, and who are faithful in Christ Jesus."
i. "Austrian archaeologists in this century [2oth] have excavated the 24,000-seat theater
and the commercial agora, as well as many other public buildings and streets of the first
and second centuries a.d., so that the modern visitor can gain some impression of the
city as known by Paul. (Achtemeier, Paul J., Th.D., Harper’s Bible Dictionary, (San
Francisco: Harper and Row, Publishers, Inc.; 1985.)
7. Gaza
A. Acts 8:26, "But an angel of the Lord spoke to Philip saying, "Arise and go south to the road
that descends from Jerusalem to Gaza."
i. Gaza was was excavated by W. J. Phythian-Adams in 1922. (Achtemeier, Paul J., Th.D.,
Harper’s Bible Dictionary, (San Francisco: Harper and Row, Publishers, Inc.; 1985.)
8. Gezer
A. Joshua 16:10, "But they did not drive out the Canaanites who lived in Gezer..."
i. R.A.S. MacAlister "directed the Palestine Exploration Fund for many years and conducted
extensive excavations at Gezer (1902–1909). (Douglas, J. D., Comfort, Philip W. &
Mitchell, Donald, Editors, Who’s Who in Christian History, (Wheaton, Illinois: Tyndale
House Publishers, Inc.; 1992.)
9. Hazor
A. Joshua 11:1, "Then it came about, when Jabin king of Hazor heard of it, that he sent to
Jobab king of Madon and to the king of Shimron and to the king of Achshaph."
B. Jer. 49:48, "Concerning Kedar and the kingdoms of Hazor, which Nebuchadnezzar king of
Babylon defeated. Thus says the Lord, "Arise, go up to Kedar and devastate the men of the
east."
i. "This large Canaanite and Israelite city in upper Galilee was excavated under Yigael
Yadin's direction from 1955 to 1958 and from 1968 to 1970." (Horn, Siegfried H.,
Biblical Archaeology: a Generation of Discovery; Andrews University, Berrien Springs,
Michigan; 1985. p. 40.)
10. Hesbon
A. Josh. 12:2, "Sihon king of the Amorites, who lived in Heshbon, and ruled from Aroer, which
is on the edge of the valley of the Arnon..."
i. Excavations were undertaken by Andrews University from 1968 to 1976. (Achtemeier,
Paul J., Th.D., Harper’s Bible Dictionary, (San Francisco: Harper and Row, Publishers,
Inc.; 1985.)
11. Jericho
A. Num. 22:1, "Then the sons of Israel journeyed, and camped in the plains of Moab beyond
the Jordan opposite Jericho."
i. "Jericho was the oldest inhabited and fortified city ever excavated." (Horn, Siegfried H.,
Biblical Archaeology: a Generation of Discovery; Andrews University, Berrien Springs,
Michigan; 1985. p. 37)
ii. "The city of OT times is represented today by a mound 70 feet high and 10 acres in
area...The ancient city was excavated by C. Warren (1867), E. Sellin and C. Watzinger
(1907-09), J. Garstang (1930-36), and K. Kenyon (1952-58)." (Achtemeier, Paul J.,
Th.D., Harper’s Bible Dictionary; San Francisco: Harper and Row, Publishers, Inc.;
1985.)
iii. "The first scientific excavation there (1907-9) was by Sellin and Watzinger (Jericho,
1913)." (The New Bible Dictionary; Wheaton, Illinois: Tyndale House Publishers, Inc.;
1962.).

12. Joppa
A. Acts 9:38, "And since Lydda was near Joppa, the disciples, having heard that Peter was
there, sent two men to him, entreating him, "Do not delay to come to us."
i. "During excavations of the site of ancient Joppa a thirteenth-century b.c. citadel gate
was uncovered..." (Achtemeier, Paul J., Th.D., Harper’s Bible Dictionary, (San Francisco:
Harper and Row, Publishers, Inc.; 1985.)
13. Nineveh
A. 2 Kings 19:36, "So Sennacherib king of Assyria departed and returned home, and lived at
Nineveh."
B. Jonah 1:1-2, "The word of the Lord came to Jonah the son of Amittai saying, 2 "Arise, go to
Nineveh the great city, and cry against it, for their wickedness has come up before Me."
i. Excavated in from 1845 to 1857 by Austen H. Layard. (Douglas, J. D., Comfort, Philip
W. & Mitchell, Donald, Editors, Who’s Who in Christian History, (Wheaton, Illinois:
Tyndale House Publishers, Inc.; 1992.)
14. Shechem
A. Gen. 12:6, "And Abram passed through the land as far as the site of Shechem, to the oak of
Moreh. Now the Canaanite was then in the land."
B. Gen. 33:18, "Now Jacob came safely to the city of Shechem, which is in the land of Canaan,
when he came from Paddan-aram, and camped before the city."
i. "Excavations were carried out at Shechem, first by Austrian-German expeditions in 1913
and 1914, and again from 1926 to 1934, under several directors, and then by an
American expedition from 1956 to 1972....Excavation of the sacred area revealed a
courtyard sanctuary and a later fortress temple dedicated to El-berith "the god of the
covenant." This temple, which was destroyed by Abimelech, the son of the judge
Gideon (Judges 9) has provided us with a date of the judges period." (Horn, Siegfried
H., Biblical Archaeology: a Generation of Discovery; Andrews University, Berrien
Springs, Michigan; 1985. p. 40)
ii. Most recently a structure identified as an Israelite altar has been excavated on the
northeastern slope of Mt. Ebal. Dating to the 13th to 12th centuries B.C., considered to
be the time of Joshua, the altar suggest the possibility that it may be the altar built by
Joshua and described in Deuteronomy 27, 28." (Horn, Siegfried H., Biblical Archaeology:
a Generation of Discovery; Andrews University, Berrien Springs, Michigan; 1985. p. 40)
15. Susa
A. Neh. 1:1, "The words of Nehemiah the son of Hacaliah. Now it happened in the month
Chislev, in the twentieth year, while I was in Susa the capitol,
B. Esther 1:2, "Now it took place in the days of Ahasuerus, the Ahasuerus who reigned from
India to Ethiopia over 127 provinces, 2 in those days as King Ahasuerus sat on his royal
throne which was in Susa the capital,
i. Escavations were conducted by Marcel Dieulafoy from 1884 to 1886 (Douglas, J. D.,
Comfort, Philip W. & Mitchell, Donald, Editors, Who’s Who in Christian History,
(Wheaton, Illinois: Tyndale House Publishers, Inc.; 1992.)
The writings of Josephus mention many biblical people and places

Flavius Josephus (37-101 A.D.) was a Jewish priest at the time of the Jewish Revolt of A.D. 66. He
was captured by the Romans, imprisoned, set free and then retired to Rome where he wrote a history
of the Revolt called the "Jewish War." Later he wrote "Antiquities" as a history of the Jews.
Following is a brief listing of some people and places mentioned by Josephus that correspond to
biblical references. They demonstrate that the Bible is not alone in its description of people, events,
and places.

1. Antipas mentioned
A. 17:8:1, "And now Herod altered his testament upon the alteration of his mind; for he
appointed Antipas, to whom he had before left the kingdom, to be tetrarch of Galilee and
Berea, and granted the kingdom to Archelaus."
i. Rev. 2:13, "‘I know where you dwell, where Satan’s throne is; and you hold fast My
name, and did not deny My faith, even in the days of Antipas, My witness, My faithful
one, who was killed among you, where Satan dwells."
2. Herod is mentioned numerous times
A. 17:8:1, "And now Herod altered his testament upon the alteration of his mind..."
B. 18.5.3, "Whereupon he ordered the army to march along the Great Plain, while he himself,
with Herod the tetrarch, and his friends, went up to Jerusalem to offer sacrifice to God, an
ancient festival of the Jews being then just approaching."
C. See also, 18:2:1,2,3; 18:4:3,5,6; 18:5:1,2,3; 18:7:2, etc.
i. Luke 3:1, "Now in the fifteenth year of the reign of Tiberius Caesar, when Pontius Pilate
was governor of Judea, and Herod was tetrarch of Galilee, and his brother Philip was
tetrarch of the region of Ituraea and Trachonitis, and Lysanias was tetrarch of Abilene."
3. The Galatians are mentioned
A. 17:8:3, "First of all went his guards, then the band of Thracians, and after them the
Germans; and next the band of Galatians, every one in their habiliments of war."
B. 12:10:6, "And when he was dead, the people bestowed the high priesthood on Judas; who,
hearing of the power of the Romans, d and that they had conquered in war Galatia, and
Iberia, and Carthage, and Lybia."
i. Gal. 1:2, "and all the brethren who are with me, to the churches of Galatia."
4. Jericho mentioned
A. 17:8:2, "...when Salome and Alexas gathered the soldiery together in the amphitheater at
Jericho..."
i. Num. 22:1, "And the children of Israel journeyed, and encamped in the plains of Moab
beyond the Jordan at Jericho."
5. Jerusalem is mentioned
A. 20:9:2, "Now as soon as Albinus was come to the city of Jerusalem..."
i. Matt. 21:10-11, "And when He had entered Jerusalem, all the city was stirred, saying,
"Who is this?" 11And the multitudes were saying, "This is the prophet Jesus, from
Nazareth in Galilee."
6. Jesus is mentioned
A. 18:3:3, "Now there was about this time Jesus, a wise man, if it be lawful to call him a man;
for he was a doer of wonderful works, a teacher of such men as receive the truth with
pleasure. He drew over to him both many of the Jews and many of the Gentiles. He was
[the] Christ. And when Pilate, at the suggestion of the principal men amongst us, had
condemned him to the cross, those that loved him at the first did not forsake him; for he
appeared to them alive again the third day; as the divine prophets had foretold these and
ten thousand other wonderful things concerning him. And the tribe of Christians, so named
from him, are not extinct at this day."
B. 20:9:1, "Festus was now dead, and Albinus was but upon the road; so he assembled the
Sanhedrim of judges, and brought before them the brother of Jesus, who was called Christ,
whose name was James, and some others, [or, some of his companions]; and when he had
formed an accusation against them as breakers of the law, he delivered them to be stoned:
but as for those who seemed the most equitable of the citizens, and such as were the most
uneasy at the breach of the laws, they disliked what was done;"
i. For information on the Testimonium Flavanium (the quotes of Josephus about Jesus)
please see Regarding the quotes from the historian Josephus about Jesus
7. Judea is mentioned
A. 20:9:1, "AND now Caesar, upon hearing the death of Festus, sent Albinus into Judea, as
procurator."
B. See also, 20:1:1; 20:5:1,2,3; 20:6:1,2; 20:7:1,2; 20:8:5,10; 20:11:1; etc.
i. Matt. 2:1, "Now after Jesus was born in Bethlehem of Judea in the days of Herod the
king, behold, magi from the east arrived in Jerusalem..."
8. John the Baptist is mentioned
A. 18.5.2 Now some of the Jews thought that the destruction of Herod's army came from God,
and was a very just punishment for what he did against John called the Baptist [the dipper].
For Herod had him killed, although he was a good man and had urged the Jews to exert
themselves to virtue, both as to justice toward one another and reverence towards
i. Matt. 3:1-2, "Now in those days John the Baptist *came, preaching in the wilderness of
Judea, saying, 2"Repent, for the kingdom of heaven is at hand."
9. Pontius Pilate is mentioned
A. 18:3:1, "But now Pilate, the procurator of Judea, removed the army from Cesarea to
Jerusalem, to take their winter quarters there, in order to abolish the Jewish laws."
B. See also, 18:3:1,2,3; 18:4:1,2,5, etc.
i. Luke 3:1, "Now in the fifteenth year of the reign of Tiberius Caesar, when Pontius Pilate
was governor of Judea, and Herod was tetrarch of Galilee, and his brother Philip was
tetrarch of the region of Ituraea and Trachonitis, and Lysanias was tetrarch of Abilene."
10. Sadducees mentioned
A. 20:9:1, "But this younger Ananus, who, as we have told you already, took the high
priesthood, was a bold man in his temper, and very insolent; he was also of the sect of the
Sadducees."
i. Matt. 16:1, "And the Pharisees and Sadducees came up, and testing Him asked Him to
show them a sign from heaven."
11. The Samaritans are mentioned
A. 18:4:1, "But the nation of the Samaritans did not escape without tumults."
i. Luke 10:33, ""But a certain Samaritan, who was on a journey, came upon him; and
when he saw him, he felt compassion."
12. Tiberius Ceasar is mentioned
A. 18.6.4, "And now Agrippa was come to Puteoli, whence he wrote a letter to Tiberius Caesar,
who then lived at Capreae, and told him that he was come so far in order to wait on him,
and to pay him a visit; and desired that he would give him leave to come over to Caprein."
i. Luke 3:1, "Now in the fifteenth year of the reign of Tiberius Caesar, when Pontius Pilate
was governor of Judea, and Herod was tetrarch of Galilee, and his brother Philip was
tetrarch of the region of Ituraea and Trachonitis, and Lysanias was tetrarch of Abilene."

There are many other such references. But what they do is help to establish that the Bible was not
written in isolation. It was written in the context of ancient Israel when and where it claims to have
been written. This is important when authenticating the Bible and this is why external references are
sometimes useful. In this case, Josephus who was a contemporary of the disciples, is referenced in
support of biblical accuracy.
When were the gospels written and by whom?
Dating the gospels is very important. If it can be established that the gospels were written early,
say before the year 70 A.D., then we would have good reason for believing that they were written by
the disciples of Jesus Himself. If they were written by the disciples, then their reliability, authenticity,
and accuracy are better substantiated. Also, if they were written early, this would mean that there
would not have been enough time for myth to creep into the gospel accounts since it was the
eyewitnesses to Christ's life that wrote them. Furthermore, those who were alive at the time of the
events could have countered the gospel accounts and since we have no contradictory writings to the
gospels, their early authorship as well as apostolic authorship becomes even more critical.

Destruction of the temple in 70 A.D., Luke and Acts

None of the gospels mention the destruction of the Jewish temple in 70 A.D. This is significant
because Jesus had prophesied concerning the temple when He said "As for these things which you are
looking at, the days will come in which there will not be left one stone upon another which will not be
torn down," (Luke 21:5, see also Matt. 24:1; Mark 13:1). This prophecy was fulfilled in 70 A.D. when
the Romans sacked Jerusalem and burned the temple. The gold in the temple melted down between
the stone walls and the Romans took the walls apart, stone by stone, to get the gold. Such an obvious
fulfillment of Jesus' prophecy most likely would have been recorded as such by the gospel writers who
were fond of mentioning fulfillment of prophecy if they had been written after 70 A.D. Also, if the
gospels were fabrications of mythical events then anything to bolster the Messianic claims -- such as
the destruction of the temple as Jesus said -- would surely have been included. But, it was not
included suggesting that the gospels (at least Matthew, Mark, and Luke) were written before 70 A.D.
Similarly, this argument is important when we consider the dating of the book of Acts which was
written after the gospel of Luke by Luke himself. Acts is a history of the Christian church right after
Jesus' ascension. Acts also fails to mention the incredibly significant events of 70 A.D. which would
have been extremely relevant and prophetically important and garnered inclusion into Acts had it
occurred before Acts was written. Remember, Acts is a book of history concerning the Christians and
the Jews. The fact that the destruction of Jerusalem and the temple is not recorded is very strong
evidence that Acts was written before A.D. 70. If we add to this the fact that acts does not include
the accounts of "Nero's persecution of the Christians in A.D. 64 or the deaths of James (A.D. 62), Paul
(A.D. 64), and Peter (A.D. 65)," 35 and we have further evidence that it was written early
If we look at Acts 1:1-2 it says, "The first account I composed, Theophilus, about all that Jesus
began to do and teach, 2 until the day when He was taken up, after He had by the Holy Spirit given
orders to the apostles whom He had chosen." Most scholars affirm that Acts was written by Luke and
that Theophilus (Grk. "lover of God") "may have been Luke’s patron who financed the writing of Luke
and Acts."36 This means that the gospel of Luke was written before Acts.

"At the earliest, Acts cannot have been written prior to the latest firm chronological marker
recorded in the book—Festus’s appointment as procurator (24:27), which, on the basis of
independent sources, appears to have occurred between A.D. 55 and 59." 37
"It is increasingly admitted that the Logia [Q] was very early, before 50 A.D., and Mark
likewise if Luke wrote the Acts while Paul was still alive. Luke's Gospel comes (Acts 1:1) before
the Acts. The date of Acts is still in dispute, but the early date (about A.D. 63) is gaining
support constantly." 38

For clarity, Q is supposedly one of the source documents used by both Matthew and Luke in
writing their gospels. If Q actually existed then that would push the first writings of Christ's words and

35
McDowell, Josh, A Ready Defense, Thomas Nelson Publishers; Nashville, Tenn., 1993, p. 80.
36
Walvoord, John F., and Zuck, Roy B., The Bible Knowledge Commentary, (Wheaton, Illinois: Scripture Press
Publications, Inc.) 1983, 1985.
37
Mays, James Luther, Ph.D., Editor, Harper’s Bible Commentary, (New York: Harper and Row, Publishers, Inc.)
1988.
38
Robertson, A.T., A Harmony of the Gospels, Harper & Row; New York` 1950. pp. 255-256.
deeds back even further lessening the available time for myth to creep in and adding to the validity
and accuracy of the gospel accounts. If what is said of Acts is true, this would mean that Luke was
written at least before A.D. 63 and possibly before 55 - 59 since Acts is the second in the series of
writings by Luke. This means that the gospel of Luke was written within 30 years of Jesus' death.

Matthew

The early church unanimously held that the gospel of Matthew was the first written gospel and was
penned by the apostle of the same name (Matt. 10:2). Lately, the priority of Matthew as the first
written gospel has come under suspicion with Mark being considered by many to be the first written
gospel. The debate is far from over.
The historian Papias mentions that the gospel of Matthew was originally in Aramaic or Hebrew and
attributes the gospel to Matthew the apostle. 39

"Irenaeus (ca. a.d. 180) continued Papias’s views about Matthew and Mark and added his belief
that Luke, the follower of Paul, put down in a book the gospel preached by that apostle, and
that John, the Beloved Disciple, published his Gospel while residing in Asia. By the time of
Irenaeus, Acts was also linked with Luke, the companion of Paul." 40

This would mean that if Matthew did write in Aramaic originally, that he may have used Mark as a
map, adding and clarifying certain events as he remembered them. But, this is not known for sure.
The earliest quotation of Matthew is found in Ignatius who died around 115 A.D. Therefore,
Matthew was in circulation well before Ignatius came on the scene. The various dates most widely
held as possible writing dates of the Gospel are between A.D. 40 - 140. But Ignatius died around 115
A.D. and he quoted Matthew. Therefore Matthew had to be written before he died. Nevertheless, it is
generally believed that Matthew was written before A.D. 70 and as early as A.D. 50.

Mark

Mark was not an eyewitness to the events of Jesus' life. He was a disciple of Peter and
undoubtedly it was Peter who informed Mark of the life of Christ and guided him in writing the Gospel
known by his name. "Papias claimed that Mark, the Evangelist, who had never heard Christ, was the
interpreter of Peter, and that he carefully gave an account of everything he remembered from the
preaching of Peter."41 Generally, Mark is said to be the earliest gospel with an authorship of between
A.D. 55 to A.D. 70.

Luke

Luke was not an eyewitness of the life of Christ. He was a companion of Paul who also was not an
eyewitness of Christ's life. But, both had ample opportunity to meet the disciples who knew Christ and
learn the facts not only from them, but from others in the area. Some might consider this damaging
to the validity of the gospel, but quite the contrary. Luke was a gentile convert to Christianity who
was interested in the facts. He obviously had interviewed the eyewitnesses and written the Gospel
account as well as Acts.

"The first account I composed, Theophilus, about all that Jesus began to do and teach, 2 until
the day when He was taken up, after He had by the Holy Spirit given orders to the apostles
whom He had chosen. 3 To these He also presented Himself alive, after His suffering, by many
convincing proofs, appearing to them over a period of forty days, and speaking of the things
concerning the kingdom of God," (Acts 1:1-3).

39
Douglas, J. D., Comfort, Philip W. & Mitchell, Donald, Editors, Who’s Who in Christian History, Wheaton, Illinois:
Tyndale House Publishers, Inc.; 1992.
40
Achtemeier, Paul J., Th.D., Harper’s Bible Dictionary, (San Francisco: Harper and Row, Publishers, Inc.; 1985
41
Douglas, J. D., Comfort, Philip W. & Mitchell, Donald, Editors, Who’s Who in Christian History, (Wheaton, Illinois:
Tyndale House Publishers, Inc.; 1992.
Notice how Luke speaks of "them," of those who had personal encounters with Christ. Luke is
simply recounting the events from the disciples. Since Luke agrees with Matthew, Mark, and John and
since there is no contradictory information coming from any of the disciples stating that Luke was
inaccurate, and since Luke has proven to be a very accurate historian, we can conclude that Luke's
account is very accurate.
As far as dating the gospel goes, Luke was written before the book of Acts and Acts does not
mention "Nero's persecution of the Christians in A.D. 64 or the deaths of James (A.D. 62), Paul (A.D.
64), and Peter (A.D. 65)."42 Therefore, we can conclude that Luke was written before A.D. 62. "Luke's
Gospel comes (Acts 1:1) before the Acts. The date of Acts is still in dispute, but the early date (about
A.D. 63) is gaining support constantly." 43

John

The writer of the gospel of John was obviously an eyewitness of the events of Christ's life since he
speaks from a perspective of having been there during many of the events of Jesus' ministry and
displays a good knowledge of Israeli geography and customs.
The John Rylands papyrus fragment 52 of John's gospel dated in the year 125-135 contains
portions of John 18, verses 31-33,37-38. This fragment was found in Egypt. It is the last of the
gospels and appears to have been written in the 80's to 90's. Most scholars say it was written in the
early 90's. This means that the time span between the original writing of John and its earliest copy
(fragment) is approximately 35-45 years.
Of important note is the lack of mention of the destruction of the Jewish temple in 70 A.D. But this
is understandable since John was not focusing on historical events. Instead, he focused on the
theological aspect of the person of Christ and listed His miracles and words that affirmed Christ's
deity.

Though there is still some debate on the dates of when the gospels were written, they were most
assuredly completed before the close of the first century and written by eyewitnesses or under the
direction of eyewitnesses.

42
McDowell, Josh, A Ready Defense, Thomas Nelson Publishers; Nashville, Tenn., 1993, p. 80.
43
Robertson, A.T., A Harmony of the Gospels, Harper & Row; New York` 1950. pp. 255-256.
Evidence of biblical inspiration

Is the Bible inspired? Christians claim it is. If that is so, then where is the evidence for its
inspiration? Simply saying it is inspired isn't enough. Let's see some facts.
First of all, we must understand the inspiration of the Bible is in reference to the original
documents, not the copies. Christianity holds that the original writings, the autographs, were without
error in everything they address. It is not the copies that are inspired. What we have are copies of
inspired documents and the truth is, some copying errors have woven themselves into some of the
biblical copies. However, this does not mean that the Bible is not trustworthy.
Textually speaking only 1/1000th of the Bible has any textual variation in the copies. That means
that the Bible as a whole is around 98.5% textually pure. The New Testament is about 99.5%
textually pure. Furthermore, there is enough redundancy in the copies that have been unearthed via
archaeology, that we can reconstruct the Bible to almost 100% accuracy. It is a remarkably well
preserved series of different books. For more information on this, please see Manuscript evidence for
superior New Testament reliability.
In practical terms, this means that because of various copying errors, usually numbers, word order,
spelling, and punctuation, we have Bible Difficulties; hence, the section on CARM dealing with many of
them. Therefore, even though we can see some surface issues, we can still very easily see evidence of
its inspiration.

Prophecy

One of the greatest proofs of the Bible's inspiration is prophecy. Following are some of the
prophecies of both secular and religious fulfillment. In other words, the first section deals with
prophecies of the secular world. The second part deals with prophecies about Jesus.

1. Secular Prophecies
A. In Daniel 2 four kingdoms are described in the interpretation of the dream of
Nebuchadnezzar who was the king of Babylon. The four were the Babylon, Medo-Persia,
Greek and the Roman empire (Dan. 2:39-43). These four kingdoms occurred just as
prophesied.
B. The following cities were prophesied to be destroyed and never rebuilt which has come true
since they have not yet been rebuilt. Nineveh (Nah. 1:10; 3:7,15; Zeph. 2:13-14), Babylon
(Isaiah 13:1-22), and Tyre (Ezek. 26:).
C. Daniel 12:4 prophesied that knowledge would increase as well as the ability to travel great
distances. Of course, this has occurred given our present rise in technology.
2. Messianic Prophecies
A. Jesus would be born of a virgin Isaiah 7:14, "Therefore the Lord himself will give you a sign:
The virgin will be with child and will give birth to a son, and will call him Immanuel."
i. Fulfilled in Matt. 1:18,25, "This is how the birth of Jesus Christ came about: His mother
Mary...was found to be with child through the Holy Spirit... But he had no union with her
until she gave birth to a son. And he gave him the name Jesus."
B. Jesus' place of birth in Bethlehem Micah 5:2, "But you, Bethlehem Ephrathah, though you
are small among the clans of Judah, out of you will come for me one who will be ruler over
Israel, whose origins are from of old, from ancient times."
i. Fulfilled in Matt. 2:1, "After Jesus was born in Bethlehem in Judea, during the time of
King Herod, Magi from the east came to Jerusalem."
C. Jesus would be preceded by a messenger Isaiah 40:3, "A voice of one calling: 'In the desert
prepare the way for the LORD; make straight in the wilderness a highway for our God.'"
i. Fulfilled in Matt. 3:1-2, "In those days John the Baptist came, preaching in the Desert of
Judea and saying, 'Repent, for the kingdom of heaven is near.'"
D. Rejected by His own people Isaiah 53:3, "He was despised and rejected by men, a man of
sorrows, and familiar with suffering. Like one from whom men hide their faces he was
despised, and we esteemed him not."
i. Fulfilled in John 7:5, "For even his own brothers did not believe in him," and John 7:48,
"Have any of the rulers or the Pharisees believed in Him?"
E. Jesus' side pierced Zech. 12:10, "And I will pour out on the house of David and the
inhabitants of Jerusalem a spirit of grace and supplication. They will look on me, the one
they have pierced, and they will mourn for him as one mourns for an only child, and grieve
bitterly for him as one mourns for an only son."
i. Fulfilled in John 19:34, "Instead, one of the soldiers pierced Jesus' side with a spear,
bringing a sudden flow of blood and water."
F. Jesus would be crucified Psalm 22:1; 22:11-18, "For the director of music. To the tune of
"The Doe of the Morning." A psalm of David. My God, my God, why have you forsaken
me?...Do not be far from me, for trouble is near and there is no one to help. Many bulls
surround me; strong bulls of Bashan. Dogs have surrounded me; a band of evil men has
encircled me, they have pierced my hands and my feet. I can count all my bones; people
stare and gloat over me. They divide my garments among them and cast lots for my
clothing."
i. Fulfilled in John 19:23-24, "When the soldiers crucified Jesus, they took his clothes,
dividing them into four shares, one for each of them, with the undergarment remaining.
This garment was seamless, woven in one piece from top to bottom. Let's not tear it,
they said to one another. "Let's decide by lot who will get it." This happened that the
scripture might be fulfilled which said, "They divided my garments among them and cast
lots for my clothing." So this is what the soldiers did."

Other information

The Bible also contains information about physical phenomena that is particularly unusual.

1. The Shape of the Earth


A. "He sits enthroned above the circle of the earth, and its people are like grasshoppers. He
stretches out the heavens like a canopy, and spreads them out like a tent to live in, " (Isaiah
40:22, NIV).
i. This may or may not be construed to support the spherical shape of the earth. The
horizon is a circle and a circle is flat.
2. The Earth is suspended in nothing
A. "He spreads out the northern [skies] over empty space; he suspends the earth over
nothing," (Job. 26:7, NIV).
i. This is particularly interesting considering that the cosmology of other cultures at that
time did not have the earth suspended in nothing, but rather upon pillars, or people, or
animals.
3. The Existence of Valleys in the Seas
A. "The valleys of the sea were exposed and the foundations of the earth laid bare at the
rebuke of the LORD, at the blast of breath from his nostrils," (2 Sam. 22:16, NIV).
4. The Existence of Springs and Fountains in the Seas
A. "In the six hundredth year of Noah's life, on the seventeenth day of the second month -- on
that day all the springs of the great deep burst forth, and the floodgates of the heavens
were opened," (Genesis 7:11, NIV). See also Gen. 8:2; Prov. 8:28.
5. The Existence of Water Paths (Ocean Currents) in the Seas
A. "O LORD, our Lord, how majestic is your name in all the earth!...When I consider your
heavens, the work of your fingers, the moon and the stars, which you have set in
place,...You made him [man] ruler over the works of your hands; you put everything under
his feet...the birds of the air, and the fish of the sea, all that swim the paths of the seas,"
(Psalm 8:1,3,6,8, NIV).
6. The Hydrologic Cycle
A. "He wraps up the waters in his clouds, yet the clouds do not burst under their weight, " (Job.
26:8, NIV).
B. "He draws up the drops of water, which distill as rain to the streams; the clouds pour down
their moisture and abundant showers fall on mankind," (Job. 36:27-28, NIV)
C. "The wind blows to the south and turns to the north; round and round it goes, ever returning
on its course. All streams flow into the sea, yet the sea is never full. To the place the
streams come from, there they return again,” (Ecc. 1:6-7, NIV).

Though nothing in the above lists "prove" biblical inspiration, they are strong evidence that it is
indeed inspired. Add to them that millions of people all over the world testify to having an encounter
with the God of the Bible, the seemingly powerful nature of the words of the Bible, the changed lives of
countless people and you have further, though more subjective, evidence that the Bible is the inspired
word of God.
Can we trust the New Testament as a historical document?

Many people do not believe that the Bible is a reliable document of history. To choose this,
however, the Bible is very trustworthy as a historical document. If we were to look at a chart that
compared the biblical documents with other ancient documents, we would see that the Bible is in a
class by itself regarding the number of ancient copies and their reliability. Please consider the chart
below

Approximate Time
Date Earliest Number Accuracy of
Author44 Span between
Written Copy of Copies Copies
original & copy
died 55 or
Lucretius 1100 yrs 2 ----
53 B.C.
Pliny 61-113 A.D. 850 A.D. 750 yrs 7 ----
427-347 B.C
Plato 900 A.D. 1200 yrs 7 ----
.
4th Cent.
Demosthenes 1100 A.D. 800 yrs 8 ----
B.C.
480-425
Herodotus 900 A.D. 1300 yrs 8 ----
B.C.
Suetonius 75-160 A.D. 950 A.D. 800 yrs 8 ----
460-400
Thucydides 900 A.D. 1300 yrs 8 ----
B.C.
480-406
Euripides 1100 A.D. 1300 yrs 9 ----
B.C.
450-385
Aristophanes 900 A.D. 1200 10 ----
B.C.
Caesar 100-44 B.C. 900 A.D. 1000 10 ----
59 BC-AD
Livy ---- ??? 20 ----
17
circa 100
Tacitus 1100 A.D. 1000 yrs 20 ----
A.D.
384-322
Aristotle 1100 A.D. 1400 49 ----
B.C.
496-406
Sophocles 1000 A.D 1400 yrs 193 ----
B.C.
Homer (Iliad) 900 B.C. 400 B.C. 500 yrs 643 95%
2nd Cent.
1st Cent.
New A.D.
A.D. (50- less than 100 years 5600 99.5%
Testament (c. 130
100 A.D.
A.D. f.)

44
This chart was adapted from three sources: 1) Christian Apologetics, by Norman Geisler, 1976, p. 307; 2) the
article "Archaeology and History attest to the Reliability of the Bible," by Richard M. Fales, Ph.D., in The Evidence
Bible, Compiled by Ray Comfort, Bridge-Logos Publishers, Gainesville, FL, 2001, p. 163; and 3) A Ready Defense,
by Josh Mcdowell, 1993, p. 45.
It should be obvious that the biblical documents, especially the New Testament documents, are
superior in their quantity, time span from original writing, and textual reliability. The question is not
whether or not they have been reliably transmitted to us. The question is whether or not the biblical
documents record actual historical accounts. They do.

The Bible is a book of History

It could be said that the Bible is a book of history -- and it is. The bible describes places, people,
and events in various degrees of detail. It is essentially an historical account of the people of God
throughout thousands of years. If you open to almost any page in the Bible you will find a name of a
place and/or a person. Much of this can be verified from archaeology. Though archaeology cannot
prove that the Bible is the inspired word of God, it has the ability to prove whether or not if some
events and locations described therein are true or false. So far, however, there isn't a single
archaeological discovery that disproves the Bible in any way.
Nevertheless, many used to think that the Bible had numerous historical errors in it such as Luke's
account of Lysanias being the tetrarch of Abiline in about 27 AD (Luke 3:1). For years scholars used
this "factual error" proved Luke was wrong because it was common knowledge that Lysanias was not a
tetrarch, but the ruler of Chalcis about 50 years earlier than what Luke described. But, an
archaeological inscription was found that said Lysanias was the tetrarch in Abila near Damascus at the
time that Luke said. It turns out that there had been two people named Lysanias and Luke had
accurately recorded the facts.
Also, the walls of Jericho have been found, destroyed just as the Bible says. Many critics doubted
that Nazareth ever existed, yet archaeologists have found a first-century synagogue inscription at
Caesarea verified its existence. Finds have verified Herod the Great and his son Herod Antipas. The
remains of the Apostle Peter's house have been found at Capernaum. Bones with nail scars through
the wrists and feet have been uncovered as well demonstrating the actuality of crucifixion. The High
Priest Caiaphas' bones have been discovered in an ossuary (a box used to store bones).
There is, of course, a host of archaeological digs that corroborate biblical records such as
Bethsaida, Bethany, Caesarea Philippi, Capernaum, Cyprus, Galatia, Philippi, Thessalonica, Berea,
Athens, Corinth, Ephesus, Rome, etc. For more on this see, Archaeological Evidence verifying biblical
events and places.

1. An inscribed stone was found that refers to Pontius Pilate, named as Prefect of Judaea.’ (The
New Bible Dictionary, (Wheaton, Illinois: Tyndale House Publishers, Inc.; 1962.)
A. Luke 3:1, "Now in the fifteenth year of the reign of Tiberius Caesar, when Pontius Pilate was
governor of Judea..."
2. "A decree of Claudius found at Delphi (Greece) describes Gallio as proconsul of Achaia in ad 51,
thus giving a correlation with the ministry of Paul in Corinth (Acts 18:12)." ( The New Bible
Dictionary)
A. Acts 18:12, "But while Gallio was proconsul of Achaia, the Jews with one accord rose up
against Paul and brought him before the judgment seat."
3. Excavations have revealed a text naming a benefactor Erastus which may be a reference relating
to the city-treasurer of Rom. 16:23. (The New Bible Dictionary)
A. Rom. 16:23, "Gaius, host to me and to the whole church, greets you. Erastus, the city
treasurer greets you, and Quartus, the brother."
4. At Ephesus parts of the temple of Artemis have been uncovered as is mentioned in Acts 19:28-
41. (The New Bible Dictionary)
A. Acts 19:28, "And when they heard this and were filled with rage, they began crying out,
saying, "Great is Artemis of the Ephesians."

5. "It is known that Quirinius was made governor of Syria by Augustus in AD 6. Archaeologist Sir
William Ramsay discovered several inscriptions that indicated that Quirinius was governor of Syria
on two occasions, the first time several years prior to this date...archaeology has provided some
unexpected and supportive answers. Additionally, while supplying the background behind these
events, archaeology also assists us in establishing several facts. (1) A taxation-census was a
fairly common procedure in the Roman Empire and it did occur in Judea, in particular. (2) Persons
were required to return to their home city in order to fulfill the requirements of the process. (3)
These procedures were apparently employed during the reign of Augustus (37 BC–AD 14), placing
it well within the general time frame of Jesus’ birth." 45

6. "The historical trustworthiness of Luke has been attested by a number of inscriptions. The
‘politarchs’ of Thessalonica (Acts 17:6,8) were magistrates and are named in five inscriptions
from the city in the 1st century ad. Similarly Publius is correctly designated proµtos (‘first man’)
or Governor of Malta (Acts 28:7). Near Lystra inscriptions record the dedication to Zeus of a
statue of Hermes by some Lycaonians, and near by was a stone altar for ‘the Hearer of Prayer’
(Zeus) and Hermes. This explains the local identification of Barnabas and Paul with Zeus (Jupiter)
and Hermes (Mercury) respectively (Acts 14:11). Derbe, Paul’s next stopping-place, was
identified by Ballance in 1956 with Kaerti Hüyük near Karaman (AS 7, 1957, pp. 147ff.). Luke’s
earlier references to *Quirinius as governor of Syria before the death of Herod I (Luke 2:2) and to
*Lysanias as tetrarch of Abilene (Luke 3:1) have likewise received inscriptional support." (The
New Bible Dictionary.)

There are many such archaeological verifications of biblical events and places. Is the Bible
trustworthy? Absolutely! Remember, no archaeological discovery has ever contradicted the Bible.
Therefore, since it has been verified over and over again throughout the centuries, we can continue to
trust it as an accurate historical document.

45
(Habermas, Gary R., The Historical Jesus: Ancient Evidence for the Life of Christ, (Joplin, MO: College Press
Publishing Company) 1996.)
Wasn't the New Testament written hundreds of years after Christ?

Though some say that the New Testament was written 100-300 years after Christ died, the truth is
that it was written before the close of the first century by those who either knew Christ personally, had
encountered him, or were under the direction of those who were His disciples.
In the article When were the gospels written and by whom?, I demonstrated that Matthew, Mark,
and Luke were all written before 70 A.D. Basically, the book of Acts was written by Luke. But Luke
fails to mention the destruction of Jerusalem in 79. A.D., nor does he mention the deaths of James
(A.D. 62), Paul (A.D. 64), and Peter (A.D. 65). Since Acts is a historical document dealing with the
church, we would naturally expect such important events to be recorded if Acts was written after the
fact. Since Acts 1:1-2 mentions that it is the second writing of Luke, the gospel of Luke was written
even earlier. Also, Jesus prophesied the destruction of the temple in the gospels: "As for these things
which you are looking at, the days will come in which there will not be left one stone upon another
which will not be torn down," (Luke 21:5, see also Matt. 24:1; Mark 13:1). Undoubtedly, if Matthew,
Mark, and Luke were written after the destruction of the Temple, they would have included the
fulfillment of Christ's prophecy in them. Since they don't, it is very strong indication that they were
written before 70 A.D.
The gospel of John is supposed to have been written by John the apostle. It is written from the
perspective of a first hand witness of the events of Christ's life. The John Rylands papyrus fragment 52
of John's gospel dated in the year 135 contains portions of John 18:31-33, 37-38. This fragment was
found in Egypt and a considerable amount of time is needed for the circulation of the gospel before it
reached Egypt. It is the last of the gospels and appears to have been written in the 80's to 90's.
Of important note is the lack of mention of the destruction of the Jewish temple in 70 A.D. But this
is understandable since John does not mention Jesus' prophecy of the destruction of the Temple. He
was not focusing on historical events. Instead, he focused on the theological aspect of the person of
Christ and listed His miracles and words that affirmed Christ's deity. This makes perfect sense since
he already knew of the previously written gospels.
Furthermore, 1, 2, and 3 John all contain the same writing style as the gospel of John and the book
of Revelation which is supposed to have been written in the late 80's or early 90's.

Paul's Writings:
Romans, 1 & 2 Corinthians, Galatians, Ephesians, Philippians, Colossians,
1 & 2 Thessalonians, 1 & 2 Timothy, Titus, Philemon

Paul the Apostle was a convert to Christianity. The book of Acts speaks of his conversion in Acts
9. Since Acts was written before 70 A.D. and Paul wrote the Pauline Epistles and we know that Paul
died in 64 A.D., the Pauline Epistles were all written before that date. Furthermore, in 1 Cor. 15:3-4 is
an early creed of the Christian church where Paul mentions that Jesus had died and risen. "For I
delivered to you as of first importance what I also received, that Christ died for our sins according to
the Scriptures, 4and that He was buried, and that He was raised on the third day according to the
Scriptures," (1 Cor. 15:3-4). Notice that he says he received this information. From whom did he
receive it? Most probably the apostles since he had a lot of interaction with them. This means that
Paul received the gospel account from the eyewitnesses. They were, of course contemporaries and
since they all died before the turn of the century. Therefore, their writings were completed within the
lifetime of the apostles of Jesus.

Hebrews

It is not known for sure who wrote the book of Hebrews. Authorship has been proposed for Paul,
Barnabas (Acts 4:36), Apollos (Acts 18:24), etc. The only geographical area mentioned is Italy (Heb.
13:24). The latest possible date for the writing of Hebrews is A.D. 95 but could have been written as
early as A.D. 67. The book of Hebrews speaks of the sacrifice by the High Priest in the present tense
(Heb. 5:1-3; Heb. 7:27) possibly signifying that the destruction of the Jerusalem Temple in 70 A.D.
had not yet happened.
James

This epistle claims to have been written by James, "James, a bond-servant of God and of the Lord
Jesus Christ, to the twelve tribes who are dispersed abroad, greetings," (James 1:1). The question is,
"Which James?" Is it James, the son of Zebedee (Matt. 10:2-3); James, the son of Alphaeus (Matt.
10:2-3), or the most commonly and accepted James who was the brother of Jesus? "Is not this the
carpenter’s son? Is not His mother called Mary, and His brothers, James and Joseph and Simon and
Judas? 56And His sisters, are they not all with us?" (Matt. 13:55). Notice the context of the verses
suggests immediate family since it mentions Jesus' Mother, brothers, and sisters. Also, see Gal. 1:19
which says "Then three years later I went up to Jerusalem to become acquainted with Cephas, and
stayed with him fifteen days. 19But I did not see any other of the apostles except James, the Lord’s
brother." It is probable that James didn't believe in Jesus as the Messiah until Jesus appeared to him
after His resurrection as is mentioned in 1 Cor. 15:7, "then He appeared to James, then to all the
apostles." James was martyred by the order of the high priest Ananus after the death of the
"procurator Festus in A.D. 61 (Josephus, Ant. 20. 9)." Therefore, the epistle of James was written
before A.D. 61.46

1 and 2 Peter

Both epistles clearly state that they were authored by Peter, an eyewitness of Jesus' life and post
resurrection appearances. Though there has been some who have doubted the authorship of these
two epistles, the clear opening statements of each epistle tell us Peter was the author. "Peter, an
apostle of Jesus Christ, to those who reside as aliens, scattered throughout Pontus...", (1 Pet. 1:1) and
"Simon Peter, a bond-servant and apostle of Jesus Christ, to those who have received a faith of the
same kind as ours..." (2 Pet. 2:1). It certainly seems most logical that Peter is indeed the author of
the letters that bear his name.
Peter died at Rome during Nero's persecution of Christians around 64 AD so the epistles were
obviously written before that time.

1, 2, 3 John

The writer of 1 John does not identify himself in the letter. The writer of 2 and 3 John refers to
himself as "the elder," (2 John 1; 3 John 1). Regarding the first epistle, authorship can reasonably be
determined to be that of John the Apostle. The opening of John is written from the perspective of
someone who was there with Jesus (John 1:1-4). Also, "Eusebius (Ecclesiastical History, 3.39) says of
Papias, a hearer of John, and a friend of Polycarp, 'He used testimonies from the First Epistle of John.
Irenaeus, according to Eusebius (Ecclesiastical History, 5.8), often quoted this Epistle. So in his work
Against Heresies (3.15; 5, 8) he quotes from John by name, 1 John 2:18...Clement of Alexandria
(Miscellanies, 2.66, p. 464) refers to 1 Jn 5:16, as in John’s larger Epistle.'" 47 "In the earliest canonical
lists, dating from the end of the second century, 1 John already appears. Indeed, 1 John is quoted as
authoritative by Bishop Polycarp of Smyrna [a disciple of John the apostle] before the middle of the
second century. The attestation of 2 John is almost as good. There is no second-century reference to 3
John, but that is not surprising, since it deals with a specific, local issue." 48 Furthermore, the style of
the three epistles is very similar to that of the gospel of John. 1 John mentions the "word of life"
(1 John 1:1) as does the gospel of John 1:1, etc.
It appears that the epistles were written after the Gospel of John since the epistles seem to assume
a knowledge of the gospel facts.
Date of writing varies from A.D. 60 to the early 90's. 49

Jude

46
The New Bible Dictionary, (Wheaton, Illinois: Tyndale House Publishers, Inc.) 1962.
47
Jamieson, Robert; Fausset, A.R.; and Brown, David, Commentary Critical and Explanatory on the Whole Bible,
(Oak Harbor, WA: Logos Research Systems, Inc.) 1998.
48
Achtemeier, Paul J., Th.D., Harper’s Bible Dictionary, (San Francisco: Harper and Row, Publishers, Inc.) 1985.
49
Walvoord, John F., and Zuck, Roy B., The Bible Knowledge Commentary, (Wheaton, Illinois: Scripture Press
Publications, Inc.) 1983, 1985.
Jude identifies himself as the brother of James (Jude 1). It is most likely that Jude, in true
Christian humility, does not want to equate himself as the brother of Jesus as he is traditionally held to
be and seems to be supported by scripture: "Is not this the carpenter’s son? Is not His mother called
Mary, and His brothers, James and Joseph and Simon and Judas?" (Matt. 13:55).50 Instead, he
mentions himself as a servant of Jesus, as James has also done.
The date of writing seems to be anywhere from A.D. 68 to the early 90's. Remember that if Judas
was a brother of Jesus, he was born around after Jesus which would mean the later the writing date,
the older was Judas. There is no mention of the destruction of Jerusalem which could have been
naturally included in the writing considering that Jude mentions judgments from God upon believers
and unbelievers alike (Jude 5-12). Nevertheless, it appears that Jude may have quoted from James.
Jude 17-18 says, "But you, beloved, ought to remember the words that were spoken beforehand by
the apostles of our Lord Jesus Christ, 18that they were saying to you, "In the last time there shall be
mockers, following after their own ungodly lusts." Compare this to 2 Pet. 3:3, "Know this first of all,
that in the last days mockers will come with their mocking, following after their own lusts." If this is a
quote, it would place the epistle after the writing of 2 Peter. 51

Revelation

The author of the Book of Revelation is John. "The Revelation of Jesus Christ, which God gave Him
to show to His bond-servants, the things which must shortly take place; and He sent and
communicated it by His angel to His bond-servant John," (Rev. 1:1). "Justin Martyr (Dialogue with
Trypho, p. 308) (A.D.. 139–161) quotes from the Apocalypse, as John the apostle’s work."52
Revelation was probably written at the end of John the Apostle's life. Some hold to the 90's and it
is the last book written in the New Testament.

Conclusion

Though this information is basic, it supplies enough evidence to support the apostolic authorship of
the New Testament documents. The debate on the dating of the books may never be absolutely
settled, but as scholarship and archaeology advance, confirmation of early authorship of the New
Testament continues to be validated.

50
This is not Judas Iscariot who betrayed Jesus - "Judas (not Iscariot) *said to Him, "Lord, what then has
happened that You are going to disclose Yourself to us, and not to the world?" (John 14:22). Also, Clement of
Alexandria [Adumbrations, in Epistle of Jude, p. 1007] says, "Jude, through reverential awe, did not call himself
brother, but servant, of Jesus Christ, and brother of James." Jamieson, Robert; Commentary Critical and
Explanatory on the Whole Bible.
51
Jamieson, Robert; Commentary Critical and Explanatory on the Whole Bible.
52
ibid.
Hasn't the Bible been rewritten so many times that we can't trust it
anymore?

This is a common misconception. Some people think that the Bible was written in one language,
translated to another language, then translated into yet another and so on until it was finally
translated into the English. The complaint is that since it was rewritten so many times in different
languages throughout history, it must have become corrupted. The "telephone" analogy is often used
as an illustration. It goes like this. One person tells another person a sentence who then tells another
person, who tells yet another, and so on and so on until the last person hears a sentence that has little
or nothing to do with the original one. The only problem with this analogy is that it doesn't fit the
Bible at all.
The fact is that the Bible has not been rewritten. Take the New Testament, for example. The
disciples of Jesus wrote the New Testament in Greek and though we do not have the original
documents, we do have around 6,000 copies of the Greek manuscripts that were made very close to
the time of the originals. These various manuscripts, or copies, agree with each other to almost 100
percent accuracy. Statistically, the New Testament is 99.5% textually pure. That means that there is
only 1/2 of 1% of all the copies that do not agree with each other 100%. But, if you take that 1/2 of
1% and examine it, you find that the majority of the "problems" are nothing more than spelling errors
and very minor word alterations. For example, instead of saying Jesus, a variation might be "Jesus
Christ." So the actually amount of textual variation of any concern at all is extremely low. Therefore,
we can say that we have an extremely accurate compilation of the original documents.
So when we translate the Bible, we do not translate from a translation of a translation of a
translation. We translate from the original language into our language. It is one step, not a series of
steps that leads to corruption. It is one translation step from the original to the English or to whatever
language a person needs to read it in. So we translate into Spanish from the same Greek
manuscripts. Likewise we translate into the German from those same Greek manuscripts as well. This
is how it is done for each and every language we translate the Bible into. We do not translate from
the Greek to the English, to the Spanish, and then to the German. It is from the Greek to the English.
It is from the Greek into the Spanish. It is from the Greek into the German. Therefore, the
translations are very accurate and trustworthy in regards to what the Bible originally said.

Comparison Chart

The following chart represents a compilation of various ancient manuscripts, their original date of
writing, the earliest copy, the number of copies in existent, and the time span between the originals
and the copies. If the Bible is singled out to be criticized as unreliable then all the other writings listed
below must also be discarded.

Approximate Time
Date Earliest Number Accuracy of
Author53 Span between
Written Copy of Copies Copies
original & copy
died 55 or
Lucretius 1100 yrs 2 ----
53 B.C.
Pliny 61-113 A.D. 850 A.D. 750 yrs 7 ----
427-347 B.C
Plato 900 A.D. 1200 yrs 7 ----
.

53
This chart was adapted from three sources: 1) Christian Apologetics, by Norman Geisler, 1976, p. 307; 2) the
article "Archaeology and History attest to the Reliability of the Bible," by Richard M. Fales, Ph.D., in The Evidence
Bible, Compiled by Ray Comfort, Bridge-Logos Publishers, Gainesville, FL, 2001, p. 163; and 3) A Ready Defense,
by Josh Mcdowell, 1993, p. 45.
4th Cent.
Demosthenes 1100 A.D. 800 yrs 8 ----
B.C.
480-425
Herodotus 900 A.D. 1300 yrs 8 ----
B.C.
Suetonius 75-160 A.D. 950 A.D. 800 yrs 8 ----
460-400
Thucydides 900 A.D. 1300 yrs 8 ----
B.C.
480-406
Euripides 1100 A.D. 1300 yrs 9 ----
B.C.
450-385
Aristophanes 900 A.D. 1200 10 ----
B.C.
Caesar 100-44 B.C. 900 A.D. 1000 10 ----
Livy 59 BC-AD 17 ---- ??? 20 ----
circa 100
Tacitus 1100 A.D. 1000 yrs 20 ----
A.D.
384-322
Aristotle 1100 A.D. 1400 49 ----
B.C.
496-406
Sophocles 1000 A.D. 1400 yrs 193 ----
B.C.
Homer (Iliad) 900 B.C. 400 B.C. 500 yrs 643 95%
2nd Cent.
1st Cent.
New A.D.
A.D. (50-100 less than 100 years 5600 99.5%
Testament (c. 130
A.D.
A.D.)

As you can see, the New Testament documents are very accurate. Therefore, when the scholars
translate from the Greek into the English (or into any other language), we can trust that what is
translated is accurate and reliable.
Since the New Testament writers were biased, can we trust their
testimony?

Yes, we can trust their testimony. Being biased about something does not mean that you cannot
tell the truth. Take for example the case of a robbery. The robber shoots and wounds two employees,
escapes, but is later apprehended. At the trial the employees, who have recovered from their injuries,
are brought in to testify. Both of these witnesses are biased in that they want to see the perpetrator
properly punished. But, under oath their testimony is accepted as perfectly valid -- providing there
aren't obvious problems. So, being biased does not automatically mean that the testimony they give
is not true.
The New Testament writers were certainly biased, but their bias was towards honesty and truth,
not deceit. Their intention was to accurately record and testify to the events that they had seen.
Remember, the disciples were followers of Jesus who taught them to love, to be kind, faithful, and
honest. And this wasn't all. Jesus warned against hypocrisy (Matt. 6:1, and against bearing false
witness (Matt. 19:18). The whole life of Jesus was based on integrity, character, faithfulness,
truthfulness, love, and sacrifice. This is what the disciples learned from Jesus and this is what they
taught in their writings. So, if they learned anything from Jesus it was to live in truth for this is
exactly what Jesus said, "Sanctify them in the truth; Thy word is truth. 18"As Thou didst send Me into
the world, I also have sent them into the world. 19"And for their sakes I sanctify Myself, that they
themselves also may be sanctified in truth," (John 17:17-19).
Furthermore, the fact is that there were plenty of people around who could have discounted what
the apostles had written if what they wrote was inaccurate. Yet, we find no evidence of any such thing
in any writings of the time. Yes, the disciples were biased. But to what? To lying? To exaggerating?
Or were they biased towards the truth of who Jesus is and what He had done?
Of course, just because eyewitnesses wrote about Jesus rising from the dead does not mean it
actually happened. This is true, but why would the disciples lie about this? Why would they risk their
lives, their families, their cultural ties, and even end up dying for it all if they knew it was all a lie
developed out of their "bias"? It doesn't make sense. But what does make sense is that the disciples
were telling the truth.
What is the gospel of Q and does it prove the Gospels are false?

Q comes from the German "quelle" meaning "source." Some biblical scholars have proposed that
there was a document prior to the writing of the gospels which was used by the writers of Matthew and
Luke as a source of information.54 They have called this hypothetical document "Q." It is hypothetical
because there is no proof that the document existed. Nevertheless, this proposal has gained some
acceptance in scholarly circles due to the very close similarities and identical written accounts found in
both Matthew and Luke. It is reasoned that the very similar accounts must be taken from a common
source.
Since Matthew was probably originally written in Hebrew or Aramaic according to the historian
Papias, and all we have is the Greek texts, some conclude that a translation of the Hebrew gospel of
Matthew into Greek would have resulted in translations slightly different from the Luke accounts. But
since some of the accounts are identical, it is proposed that Matthew and Luke shared a common
reference source. This is perfectly reasonable and we do see differences in translations as well as
identical wording. Is this the result of an unknown document known as Q? Perhaps, but there is no
way to be sure since it is possible that one copied from another or copied from Mark.
Following is a small sample chart of some of the sayings in Matthew and Luke that are identical as
is demonstrated by being underlined. The text is taken from the NASB.

Matt. 3:7-10, But when he saw many of the Luke 3:7-9, "He therefore began saying to the
Pharisees and Sadducees coming for baptism, he multitudes who were going out to be baptized by
said to them, "You brood of vipers, who warned him, "You brood of vipers, who warned you to flee
you to flee from the wrath to come? 8"Therefore from the wrath to come? 8"Therefore bring forth
bring forth fruit in keeping with repentance; 9and fruits in keeping with repentance, and do not
do not suppose that you can say to yourselves, begin to say to yourselves, ‘We have Abraham for
‘We have Abraham for our father’; for I say to our father,’ for I say to you that God is able from
you, that God is able from these stones to raise these stones to raise up children to Abraham.
up children to Abraham. 10"And the axe is already 9
"And also the axe is already laid at the root of
laid at the root of the trees; every tree therefore the trees; every tree therefore that does not bear
that does not bear good fruit is cut down and good fruit is cut down and thrown into the fire."
thrown into the fire.

Matt. 23:37, "O Jerusalem, Jerusalem, who kills Luke 13:34, "O Jerusalem, Jerusalem, the city
the prophets and stones those who are sent to that kills the prophets and stones those sent to
her! How often I wanted to gather your children her! How often I wanted to gather your children
together, the way a hen gathers her chicks under together, just as a hen gathers her brood under
her wings, and you were unwilling. her wings, and you would not have it!

Matt. 11:2-6 Luke 7:18-23

Matt. 8:18-22 Luke 9:57-62

Matt. 17:19-20 Luke 17:5-6

Matt. 25:14-30 Luke 19:11-27

...etc.

54
Achtemeier, Paul J., Th.D., Harper’s Bible Dictionary, (San Francisco: Harper and Row, Publishers, Inc., 1985.
As I said above, another theory is that Matthew and Luke copied from Mark. Please consider the
following quote which deals with both the Q theory and the Markan Source theory.

"It is plain as a pikestaff that both our Matthew and Luke used practically all of Mark and
followed his general order of events. For this reason Mark has been placed first on the pages
where this Gospel appears at all. But another thing is equally clear and that is that both
Matthew and Luke had another source in common because they each give practically identical
matter for much that is not in Mark at all. This second common source for Matthew and Luke
has been called Logia because it is chiefly discourses. It is sometimes referred to as "Q."55

The above quote from A.T. Robertson, one of the foremost Greek scholars of the 20th century, is a
good reflection of the position of many scholars. It may be that there was a common source for
Matthew and Luke, either Q or Mark, or a combination of both.

If Q is true, are the gospels inspired?

Some people say that the Q theory invalidates the inspiration of the gospels since it would mean
that the writers copied their material from one another and were then not inspired of God. But this
does not invalidate inspiration at all. If Q is an actual source it does not invalidate the validity of the
gospels. Why would copying from an earlier source invalidate the Gospels or say they were not
inspired? Can God not inspire a writer as he copies from another document? Of course.
If anything, the existence of Q would mean that the time between the actual events and their
written record is lessened. In other words, Q would have to precede Matthew and Luke. This would
mean that there is a source even earlier than the gospels which only adds to the validity of the
accuracy of the gospels since it shortens the time between the event and the record.

If Mark was a source of Matthew and Luke, then...

Furthermore, if Mark was used as a source for Matthew and Luke, how would he have known what
to write about if he wasn't an eyewitness? This isn't a problem because Mark was a disciple of Peter
and Peter was in the inner circle with Jesus. Therefore, Mark received his information from Peter who
was an eyewitness. Also, Matthew, who was a disciple, agrees with the account of Mark's gospel
concerning Jesus and the events surrounding Him as is evidenced in his account. There is no
disagreement between them, only confirmation and verification. So, we have confirmation of Mark's
accuracy.

_______________
For further reading the possibility of Q, please consider

• Richard A. Edwards, A Theology of Q: Eschatology, Prophecy, and Wisdom (Philadelphia, PA:


Fortress Press 1976).
• John S. Kloppenborg Verbin, Excavating Q: The History and Setting of the Sayings Gospel
(Minneapolis, MN: Fortress Press 2000).

55
Robertson, A.T., A Harmony of the Gospels, Harper & Row; New York` 1950. p. 255.
Why isn't there other evidence of the massacre of the babies?

"Then when Herod saw that he had been tricked by the magi, he became very enraged, and
sent and slew all the male children who were in Bethlehem and in all its environs, from two
years old and under, according to the time which he had ascertained from the magi," (Matt.
2:16).

If Herod really did slaughter all the male babies in Jerusalem, why isn't there any mention of it in
historical accounts outside the Bible such as the Jewish historian Josephus or some other Roman
historians? Since we find none, doesn't that mean that it didn't happen or at the least cast doubt upon
the validity of the event? After all, killing a town full of babies being slaughtered is something that
would have been recorded.
First of all, not having any evidence outside the Bible of the slaughter of the babies, does not mean
it didn't happen, especially since the Bible does record it and the Bible has already been proven to be
historically accurate.
Second, Bethlehem as far as the Romans was concerned, was an insignificant and very small town
located about five miles south of Jerusalem at around 2500 feet elevation. It probably had a
population of no more than 500 - 600 people. Micah 5:2 it says, "But as for you, Bethlehem
Ephrathah, too little to be among the clans of Judah, from you One will go forth for Me to be ruler in
Israel. His goings forth are from long ago, from the days of eternity.” Notice that Micah (written
around 500 B.C.) prophecies that from Bethlehem, a small town, Jesus will be born. If there were as
many as 600 people in Bethlehem, how many children would have been under the age of two? Ten,
twenty, thirty? Whatever the number, it would not have been hundreds. It would have been relatively
few. Add to this the fact that Herod was known for committing horrendous crimes against people and
you could see why this event in an insignificant village in the Jewish area, would be ignored.

"But it is not surprising that he [Josephus] and other secular historians overlooked the death of
a few Hebrew children in an insignificant village, for Herod’s infamous crimes were many. He
put to death several of his own children and some of his wives whom he thought were plotting
against him. Emperor Augustus reportedly said it was better to be Herod’s sow than his son, for
his sow had a better chance of surviving in a Jewish community." 56

Third, there were more "important" things happening in the Roman Empire which would occupy the
details of historical writers. Take a look at the chart below and notice that at the time of Christ, some
major events were taking place. Undoubtedly, Roman historians would have focused on issues more
appropriate to the Empire.

56
Walvoord, John F., and Zuck, Roy B., The Bible Knowledge Commentary, (Wheaton, Illinois: Scripture Press
Publications, Inc.; 1983, 1985.
Year Event Roman Empire Israel
20 B.C. - Herod begins remodeling of the
Temple
12 B.C. - Beginning of war between the
Pannonians and the Romans.
9 B.C. - Pannonians are defeated.
7 B.C. - Rome is divided into 14 regions.
- Herod executes his son.
4 B.C. - Herod burns alive 40 Jews who
- Herod dies. destroyed a golden eagle.
- Possible date of the slaughter of the babies
3 B.C. - Archelaus (Herod's son) kills 3000
Jews in the Temple
(Note that the chronology of Jesus' birth is probably 4 years too late.
-0-
Therefore, Jesus was probably born around 4 B.C.)
1 A.D. - War in Germany
2 A.D. - Peace made with Persia
3 A.D. - Roman decree permitting Jews to
follow their religious customs
4 A.D. - Herod dies
- Tiberius subdues Germany
6 A.D. - Pannonians revolt.
- Judea is absorbed into the Roman
- Herod Archelaus deposed by
Empire
Augustus

We must remember that the Bible has demonstrated itself to be reliable and accurate countless
times. It may very well be that some inscription is waiting to be uncovered which will, like many
inscriptions in the past, validate yet another biblical event. In the meantime, we can trust the Bible to
be the accurate document of historical record that it is.
Why isn't there any record of millions of Jews wandering in the desert?
There has been a lot of speculation on the route of the Exodus and why the traditional site hasn't
yielded any archaeological evidence. After all, if two million people wander in a desert for forty years,
you'd think that at least something would be found to support it. But, nothing at all has been
unearthed in the Sinai Peninsula supporting the biblical account of the Exodus. Various explanations
for this range from the idea that it is naturally difficult to find any archeological evidence in a desert of
sand, to the explanation that the traditional site is the wrong one.
First of all, we must understand that no archaeological find has ever contradicted the Bible.
Archaeology has only confirmed what the Bible says. As has been the case with so many other things
in the Bible, as archaeology progresses, they will most certainly uncover evidence in the future.
Again, the Bible has yet to be proven wrong by archaeology.
Second, lack of evidence doesn't mean there wasn't an Exodus. However, this is a slippery slope
since having a lack of evidence for an ice cream factory on Jupiter doesn't mean that there is one.
What we need is evidence and it is fair to say that there should be some evidence for the wanderings
of two million people for forty years in a desert.
Third, and I think this is the most probably explanation, it may
be that the traditional site of Mt. Sinai is incorrect. Gal. 4:25 says
"Now this Hagar is Mount Sinai in Arabia, and corresponds to the
present Jerusalem, for she is in slavery with her children." Present
theories dealing with Mt. Sinai's location have it in the Sinai
Peninsula, yet the Bible says it was in Arabia. The map to the right
shows the traditionally accepted route (in the dotted line) and the
currently accepted location of Mt. Sinai. The problem is that there
has been absolutely no archaeological evidence unearthed at that
site to verify the Exodus. The route in the dashed line shows an
alternate path that is consistent with Paul's description in Gal.
4:25. This would have Mt. Sinai be in Arabia, which is now Saudi
Arabia, instead of the traditionally accepted Sinai Peninsula.
In a recent book titled "In search of the Mountain of God," by
Bob Cornuke and David Halbrook (Broadman and Holman, 2000), Bob Cornuke (a Christian) recounts
his story of going into Saudi Arabia with his friend Larry Williams (a non-Christian commodities
trader). They uncovered evidence of an alternate site where the real Mt. Sinai might be. Bob Cornuke
was a police officer, swat team member, and crime scene investigator in Southern California and is
the President of the Bible Archaeology Search and Exploration (BASE) Institute BaseInstitute.org. He
and Mr. Williams have produced a video and book (available on that site) where they claim to have
found evidence in Saudi Arabia to support that Mt. Sinai is located within its borders. Now, I must
admit that this has not been verified by any "official" archaeologists, but the video, which I have seen,
does raise some interesting possibilities.
Mr. Cornuke and Williams claim to have simply let the Bible guide them as they attempted to locate
the actual route of the Jews of the Exodus. Through trial and error over several weeks, they followed
what they believed was the route as is laid out by the Bible and they found the items described in
Exodus 13 - 19 including, springs, a split rock, an altar, an underwater land bridge at the end of the
Sinai Peninsula where the people of Israel could have crossed, and much more. The present location
of Mt. Sinai, according to the locals in their account, is known as Jabal Al Laws as is traditionally known
by them as the mount of Moses. The Saudis have the area fenced off with warning signs in Arabic and
English telling people not to enter. If this is so, why would the Saudis not want anyone to know about
the place? It might be because if Mt. Sinai is located in Muslim territory then one of the most holy
places of the Jewish and Christian religions it could pose serious political problems.
I must admit that this is speculative at present and it has not been verified. But the video was
compelling. Whether or not this is a valid option is yet to be determined and it is supportive of the
idea that the traditional location of Exodus route might indeed be incorrect, as Gal. 4:25 seems to
suggest.
Do the lost books of the Bible prove that the Bible has been altered?

There is much talk these days about lost books of the Bible. Sometimes people claim that the Bible
was edited to take out reincarnation, or the teaching of higher planes of existence, or different gods,
or ancestor worship, or "at-one-ment" with nature, anything that disagreed with what the people in
power didn't like. But, none of this is true. The "lost books" were never lost. These so called lost
books were already known by the Jews and the Christians and were not considered inspired. They
weren't lost nor were they removed from the Bible because they were never in the Bible to begin with.
These so called lost books were not included in the Bible for several reasons. They lacked apostolic
or prophetic authorship; they did not claim to be the Word of God; they contain unbiblical concepts
such as prayer for the dead in 2 Macc. 12:45-46; or have some serious historical inaccuracies. These
books were never authoritative, inspired, or authentically written by either the Jewish Prophets or the
Christian Apostles.
Nevertheless, in spite of these problems the Roman Catholic church has added certain books to the
canon of scripture. In 1546, largely due in response to the Reformation, the Roman Catholic church
authorized several more books as scripture known as the apocrypha. The word apocrypha means
hidden. It is used in a general sense to describe a list of books written by Jews between 300 and 100
B.C. More specifically, it is used of the 7 additional books accepted by the Catholic church as being
inspired. The entire list of books of the apocrypha are: 1 and 2 Esdras, Tobit, Judith, the Rest of
Esther, the Wisdom of Solomon, Sirach, (also titled Ecclesiasticus), Baruch, The Letter of Jeremiah,
Song of the Three Young Men, Susanna, Bel and the Dragon, The Additions to Daniel, The Prayer of
Manasseh, and 1 and 2 Maccabees. The books accepted as inspired and included in the Catholic Bible
are Tobit, Judith, 1 and 2 Maccabees Wisdom of Solomon Sirach (also known as Ecclesiasticus), and
Baruch
The Pseudepigraphal books are "false writings." They are a collection of early Jewish and
"Christian" writings composed between 200 BC and AD 200. However, they too were known and were
never considered scripture. A list of these would be the Epistle of Barnabas, the First Epistle of
Clement to the Corinthians, the Second Epistle of Clement to the Corinthians, the The letter of the
Smyrnaeans or the Martyrdom of Polycarp, the The Shepherd of Hermas, the The Book of Enoch, the
Gospel of Thomas (140-170 AD), the The Psalms of Solomon, the The Odes of Solomon, the The
Testaments of the twelve Patriarchs, the Second Baruch, the Third Baruch, the The Books of Adam and
Eve.
The Deuterocanonical (apocrypha) books are those books that were included in the Greek
Septuagint (LXX) but not included in the Hebrew Bible. The recognized deuterocanonical books are 1
Esdras (150-100 BC), 2 Esdras (100 AD), Tobit (200 BC), Judith (150 BC), the Additions to Esther
(140-130 BC), the Wisdom of Solomon (30 BC), Ecclesiasticus (Sirach) (132 BC), Barach (150-50
BC), the Letter of Jeremiah (300-100 BC), the Susanna (200-0 BC), Bel and the Dragon (100 BC), the
Additions to Daniel (Prayer of Azariah (200-0 BC), the Prayer of Manassesh (100-0 BC), 1 Maccabees
(110 BC), and 2 Maccabees (110-170 BC).57
These pseudepigraphal and deuterocanonical books were never considered scripture by the
Christian church because they were not authoritative, inspired, written by either Prophets or Apostles,
nor do they have the power of the word of the books of the existing Bible. Therefore, since the books
are not lost and were never part of the Bible to begin with, they have no bearing on the validity of the
Bible.

57
Achtemeier, Paul J., Th.D., Harper’s Bible Dictionary, (San Francisco: Harper and Row, Publishers, Inc.) 1985.
Is there non-biblical evidence of a day of darkness at Christ's death?

In Luke 23:44-46 there is the record of darkness falling upon the land during Christ's crucifixion.
"And it was now about the sixth hour, and darkness fell over the whole land until the ninth hour, 45the
sun being obscured; and the veil of the temple was torn in two. 46And Jesus, crying out with a loud
voice, said, "Father, into Thy hands I commit My spirit." And having said this, He breathed His last." Is
there any non-biblical evidence of the day of darkness mentioned at Christ's death? The answer is
yes, there is.

"Circa AD 52, Thallus wrote a history of the Eastern Mediterranean world from the Trojan War
to his own time. This work itself has been lost and only fragments of it exist in the citations of
others. One such scholar who knew and spoke of it was Julius Africanus, who wrote about AD
221...In speaking of Jesus’ crucifixion and the darkness that covered the land during this event,
Africanus found a reference in the writings of Thallus that dealt with this cosmic report.
Africanus asserts: 'On the whole world there pressed a most fearful darkness; and the rocks
were rent by an earthquake, and many places in Judea and other districts were thrown down.
This darkness Thallus, in the third book of his History, calls, as appears to me without reason,
an eclipse of the sun.'" 58

One might wonder why other historians of the time did not also mention the darkness. First of all,
the darkness was localized so it would not be a widespread phenomenon that other historians would
naturally record. Second, other historians like Pliny, Tacitus, and Josephus, generally were focusing on
events that could be verified and were not based in the miraculous. The fact that Thallus mentions the
darkness tells us that something did happen and that there is extrabiblical citation for the event.

58
Julius Africanus, Extant Writings, XVIII in the Ante–Nicene Fathers, ed. by Alexander Roberts and James
Donaldson (Grand Rapids: Eerdmans, 1973), vol. VI, p. 130. as cited in Habermas, Gary R., The Historical Jesus:
Ancient Evidence for the Life of Christ, (Joplin, MO: College Press Publishing Company) 1996.
Regarding the quotes from the historian Josephus about Jesus

Flavius Josephus was a Jewish priest at the time of the Jewish Revolt of A.D. 66. He was captured
by the Romans, imprisoned, set free and then retired to Rome where he wrote a history of the Jewish
Revolt called the "Jewish War." Later he wrote "Antiquities" as a history of the Jews. It is in
Antiquities that he mentions Christ. The mention is called the Testimonium Flavianum (Ant. 18.63-64;
see below). Josephus was born in Jerusalem around 37 A.D. He died around the year 101.
The problem with the copies of Antiquities is that they appear to have been rewritten in favor of
Jesus as they are very favorable, some say too favorable to have been written by a Jew. Add to this
that the Christians were the ones who kept and made the copies of the Josephus documents
throughout history and you have a shadow of doubt cast upon the quotes.
However, all is not lost. First of all, there is no proof that such insertions into the text were ever
made. They may be authentic. The Testimonium is found in every copy of Jesusphus in existence.
Second, Josephus mentions many other biblically relevant occurrences that are not in dispute (see
outline below). This adds validity to the claim that Josephus knew about Jesus and wrote about Him
since he also wrote about other New Testament things. Nevertheless, though there may be some
Christian insertions into the text, we can still reconstruct what may have been the original writing.
Two researchers (Edwin Yamauchi and John P. Meier) 59 have constructed a copy of the
Testimonium with the probable insertions in brackets and underlined. The following paragraph is
Yamauchi's:

“About this time there lived Jesus, a wise man [if indeed one ought to call him a man.] For he
was one who wrought surprising feats and was a teacher of such people as accept the truth
gladly. He won over many Jews and many of the Greeks. [He was the Christ.] When Pilate,
upon hearing him accused by men of the highest standing amongst us, had condemned him to
be crucified, those who had in the first place come to love him did not give up their affection for
him. [On the third day he appeared to them restored to life, for the prophets of God had
prophesied these and countless other marvelous things about him.] And the tribe of the
Christians, so called after him, has still to this day not disappeared.”

Though this may be a correct assessment of the Testimonium, we should note that an Arabic
version (10th Century) of the Testimonium (translated into English) is in basic agreement with the
existing Josephus account:

"At this time there was a wise man who was called Jesus. And his conduct was good, and he
was known to be virtuous. And many people from among the Jews and the other nations
became his disciples. Pilate condemned him to be crucified and to die. And those who had
become his disciples did not abandon his discipleship. They reported that he had appeared to
them after his crucifixion and that he was alive; accordingly, he was perhaps the Messiah
concerning whom the prophets have recounted wonders." 60

The Arabic version was copied from a Greek version. What is not known is which one? But if you
notice the comparison below, if the Arabic version was a direct translation of the Greek, then why the
differences? Nevertheless, what is important in the Arabic Version is that the resurrection of Christ is
maintained.

59
Edwin Yamauchi, “Jesus Outside the New Testament: What is the Evidence?” in Jesus Under Fire: Modern
Scholarship Reinvents the Historical Jesus. Edited by Michael J. Wilkins and J. P. Moreland ( Zondervan, 1995):
212-4. And 2) John P. Meier, “Jesus in Josephus: A Modest Proposal,” Catholic Biblical Quarterly 52 (1990): 76-
103.
60
Arabic summary, presumably of Antiquities 18.63. From Agapios' Kitab al-'Unwan ("Book of the Title," 10th c.).
See also James H. Charlesworth, Jesus Within Judaism.
(http://ccat.sas.upenn.edu/~humm/Topics/JewishJesus/josephus.html)
Greek Version Arabic Version
“About this time there lived Jesus, a wise man "At this time there was a wise man who was
[if indeed one ought to call him a man.] called Jesus.
For he was one who wrought surprising feats And his conduct was good, and he was known
and was a teacher of such people as accept to be virtuous.
the truth gladly.
He won over many Jews and many of the And many people from among the Jews and
Greeks. [He was the Christ.] the other nations became his disciples.
When Pilate, upon hearing him accused by Pilate condemned him to be crucified and to
men of the highest standing amongst us, had die. And those who had become his disciples
condemned him to be crucified, those who did not abandon his discipleship.
had in the first place come to love him did not
give up their affection for him.
[On the third day he appeared to them They reported that he had appeared to them
restored to life, for the prophets of God had after his crucifixion and that he was alive;
prophesied these and countless other accordingly, he was perhaps the Messiah
marvelous things about him.] And the tribe of concerning whom the prophets have
the Christians, so called after him, has still to recounted wonders."
this day not disappeared.”

To summarize, the Testimonium Flavianum cannot be so easily dismissed as pure Christian


interpolation (insertion into the text). Though it seems probable that interpolation did occur, we
cannot be sure what was added. Also, the Arabic version contains very similar information as the
Greek one regarding Jesus in His resurrection.
Even if both versions have been tampered with, the core of them both mentions Jesus as a
historical figure who was able to perform many surprising feats, was crucified, and that there were
followers of Jesus who were still in existence at the time of its writing.
1 Cor. 15:3-4 demonstrates a creed too early for legend to corrupt.

One of the criticisms raised against the historic validity of Jesus, His crucifixion, and resurrection, is
that after Jesus' time, legend crept in to the stories about Him and corrupted the true accounts of His
life. If that is so, then the earlier we can find information concerning the fundamental events of
Christ's crucifixion, the less likely error and legend would have crept into the story and the more
believable it will be.
1 Cor. 15:3-4 is considered by many scholars to be an extremely early creed of the Christian
church. A creed is a statement of belief. In 1 Cor. 15:3-4 we see that Paul says he received this
information. It reads,

"For I delivered to you as of first importance what I also received, that Christ died for our sins
according to the Scriptures, 4and that He was buried, and that He was raised on the third day
according to the Scriptures, 5and that He appeared to Cephas, then to the twelve," (1 Cor.
15:3-5).

If we were to take a chronological look at some important events and their dates related to this
subject we find that the time period between the event and the record is very small.

EVENT DATE DOCUMENTATION


Jesus' 30
Crucifixion A.D.
32 Strobel, Lee, The Case for Christ, (Grand Rapids, Mi:
Paul's A.D. Zondervan), 1998, p. 35
conversion 34-37 The New Bible Dictionary, (Wheaton, Illinois: Tyndale House
A.D. Publishers, Inc.) 1962.
Paul's first The New Bible Dictionary, Wheaton, Illinois: Tyndale House
visit to Publishers, Inc.) 1962.
37-38
Jerusalem
A.D. The Chronological Bible, (Nashville, TN: Regal Publishers),
since
conversion 1977, p. 1429.

Achtemeier, Paul J., Th.D., Harper’s Bible Dictionary, (San


Francisco: Harper and Row, Publishers, Inc.) 1985.
Writing of 54-55
1 Corinthians A.D. Walvoord, John F., and Zuck, Roy B., The Bible Knowledge
Commentary, (Wheaton, Illinois: Scripture Press Publications,
Inc.) 1983, 1985.

If the Crucifixion was in 30 A.D., Paul's Conversion was as early as 34 A.D., and his first meeting in
Jerusalem was around 37 A.D., then we could see that the time between the event of Christ's
crucifixion and Paul receiving the information about His death, burial, and resurrection (in Jerusalem)
would be as short as seven years (five if we use the earlier date). That is a very short period of time
and hardly long enough for legend to creep in and corrupt the story. This is especially important since
the apostles were alive and spoke with Paul. They were eyewitness accounts to Christ's death, burial,
and post death appearances. Paul himself had seen the Lord Jesus prior to His death and after His
resurrection (Acts 9). Paul's account agreed with the other Apostles' account and Paul wrote it down in
1 Cor. 15 around the year 54.
So, since 1 Corinthians was written as early as 54 A.D., that would mean that from the event
(Jesus' death, burial, and resurrection) to writing it down is 24 years. That is a very short period of
time. Remember, there were plenty of Christians around who could have corrected the writings of
Paul if he was in error. But we have no record at all of any corrections or challenges to the death,
burial, and resurrection of Christ from anyone: Roman, Jew, or other Christians.
We must note here that some critics of the Bible claim that there is no extra biblical -evidence of
Christ (not true) and that because of it, He didn't exist. The sword cuts both ways. If they can say
that Jesus' events aren't real because there is no extra-biblical evidence mentioning them, then we can
also say that since there are no extra biblical-accounts refuting the death, burial, and resurrection of
Christ, then it must be true. In other words, lack of extra-biblical writings does not prove that Christ
did not live and did not die.
Furthermore, Paul corroborated the gospel accounts (He wrote before the gospels were written)
and verified several things:

• Jesus was born in as a Jew (Gal. 4:4),


• Jesus was betrayed (1 Cor. 11:23)
• and Jesus was crucified (Gal. 3:1; 1 Cor. 2:2; Phil. 2:8).
• Jesus was buried in rose again (1 Cor. 15:4; Rom. 6:4).

Obviously, Paul considered Jesus was a historical figure, not a legend or a myth. Furthermore, Paul
was a man of great integrity who suffered much for his faith. He was not the kind of person to simply
believe tall tales. After all, he was a devout Jew (a Pharisee) and a heavy persecutor of the Church.
Something profound had to happen to him to get him to change his position, abandon the Jewish faith
and tradition, suffer persecutions, whippings, jail, etc. The most likely event that fits the bill is that
Jesus died, was buried, and rose again from the dead, and appeared to Paul, just as Luke said in Acts
9.
Extraordinary claims require extraordinary evidence.

The phrase "Extraordinary claims require extraordinary evidence" was popularized by Carl Sagan
(1934 - 1996), a well-known astronomer and author who hosted a TV series called "Cosmos,"
published hundreds of scientific articles, and was professor of astronomy at Cornell University in New
York. The statement is self explanatory; if someone makes an extraordinary claim, there better be
extraordinary evidence to back it up. If, for example, someone made the claim that an alien race has
made contact with earth, we would need sufficient evidence to verify the claim, such as an alien space
craft, or an actual alien. The extraordinary claim would need extraordinary evidence.
At the heart of "extraordinary claims require extraordinary evidence" is a healthy and normal
skepticism. There are far too many charlatans and con-men in the world who make extraordinary
claims without evidence to back them up. Unfortunately, too many people lack the necessary
skepticism and critical thinking skills to help them avoid being duped by con artists and wild theories.
Personally, except for a few qualifications, I agree with the sentiment of the statement "extraordinary
claims require extraordinary evidence." Those qualifications follow.

Presuppositions

Requiring extraordinary evidence for extraordinary claims sounds good on the surface. But, it is
subjective. The fact is that a person's presuppositions strongly affect how and to what degree the
statement is applied. In Jesus' resurrection, for example, Christians presuppose that God exists and
that He could easily have raised Jesus from the dead. The evidence of fulfilled prophecy, eyewitness
records, and changed lives of the disciples is enough to convince many people who believe in God that
Jesus rose from the dead. This is a logical conclusion based on the presupposition and the evidence.
Atheists, on the other hand, would negate the resurrection by default since their presupposition
that there is no God61 would require that God involved event cannot occur. Therefore, for an atheist
the extraordinary evidence would have to be "exceptionally" extraordinary in order to overcome his
atheistic presuppositional base. In other words, evidence would need to be presented that was rock
solid and irrefutable.
This is why the skeptic must require "extraordinary evidence." It enables him to retain his
presupposition should the extraordinary level of the evidence not be met. Therefore, requiring
extraordinary evidence effectively stacks the deck against the claim.

What would qualify as extraordinary evidence?

When debating skeptics, I often ask them to tell me what would qualify as extraordinary evidence
for the resurrection of Jesus. Generally, nothing sensible is offered. Normal evidence would be written
accounts. Extraordinary evidence would be a film, but we know that this extraordinary evidence is not
reasonable. Therefore, can the requirement that extraordinary claims (Christ's resurrection) require
extraordinary evidence apply to Jesus' resurrection? It would seem not. Since Jesus' resurrection is
alleged to be a historical event, then it seems logical that normal historical evidence and normal
historical examination of that evidence would be all that we could do. The resurrection is supposed to
be an event of history and since it claims historical validity, then typical criteria for examining
historical claims should be applied.

What criteria do they use to determine what is extraordinary evidence?

61
I am aware of the different atheistic positions, i.e., lack of belief, belief there is no God, etc., but for simplicity
in the illustration I am using the "belief there is no God," atheistic position.
The reality is that there is no precise scientific method for determining the validity of historic
events. There is a degree of subjectivity involved. Different people will claim different requirements
for validating ancient phenomena based upon their presuppositions and the type of evidence involved.
Also, since ancient events dealing with human history and claims cannot be observed or repeated, we
must look at the evidence differently. This makes the application of "extraordinary claims require
extraordinary evidence" somewhat subjective and invalid for determining ancient phenomena.

Is the criteria for extraordinary evidence reasonable?

The skeptic often requires "proof" that God exists, or "absolute proof" that Jesus rose from the dead. I
have heard many atheists, for example, say that the only proof they would accept of Jesus'
resurrection would be if it could be tested using the scientific method. Of course, we know that is
impossibility since the scientific method means observation, experimentation, and repetition and we
can't apply that to an event that occurred 2000 years ago. Atheists know this and that is why they
require it. When the Christian fails to produce a scientific method or scientific evidence, the atheist
feels safe in his position.
However, the requirement for absolute proof ignores the fact that there is a category of "sufficient
evidence." In logic, there is deduction and induction. Deduction is drawing a conclusion based on
facts. It is reasoning from the general to the specific. Induction is process of drawing general
principles from specific facts. It is from the specific to the general. Often times, we use deductive and
inductive reasoning to arrive at conclusions about events in history. In so doing, there is no
requirement of "extraordinary evidence." The evidence is simply examined contextually; that is, it is
examined according to the genre in which it fits. This is what I mean...
We do not apply the logical requirement of establishing a fact that is thus established through
experimentation and repetition to the subject of Napoleon's existence. The genre, history, does not fit
that methodology. Yet, the skeptic will sometimes require that experimentation and repetition be
applied to Jesus' resurrection, thereby, misapplying evidential and logical analysis.
Furthermore, we cannot ascertain all things with absolute certainty. We cannot, for example,
prove that Alexander the Great (356-323 B.C.) ever lived by observing him. But, we have ancient
writings from eyewitnesses concerning his existence. Skeptics readily believe in Alexander the Great
without involving the scientific method and without requiring "extraordinary evidence."
However, a skeptic might say that Alexander the Great never claimed to have risen from the dead
and that normal evidence would be sufficient to determine his existence with a reasonableness of
probability. But, Alexander the Great, according to history, performed an extraordinary feat. By the
age of 33 he had conquered the known world. That is indeed an extraordinary event in history. So, I
ask, "Where is the extraordinary evidence to back that extraordinary claim up?" Has any skeptic in
Christ's resurrection, equally applied the principle of "extraordinary claims require extraordinary
evidence," to Alexander the Great's conquest of the known world? If not, then this brings us full circle
to the issue concerning presuppositions. With an atheist, for example, the presupposition that God
does not exist means that the extraordinary claim of Christ's resurrection requires extraordinary
evidence but Alexander the Great's world conquest does not, yet both are extraordinary claims of
history.

Conclusion
If it is true about Alexander the Great, no big deal. It won't have any effect on anyone and it won't
change anything in anyone's life outside of just having the information that he conquered the known
world by age 33. But, if it is true about Jesus, then that is completely different. Jesus claimed to be
divine and He had a message for people about heaven and hell and that salvation is only through Him. Such a
dead. This would have a profound effect on people and it can make them uncomfortable. Therefore,
people will not want it to be true and will desperately try to hold onto their presuppositions; hence, the
claim that extraordinary claims require extraordinary evidence.
Nevertheless, when defending the Bible and dealing with the claim that "extraordinary claims
require extraordinary evidence," address the following issues:

1. Will their presuppositions allow unbiased examination of the evidence?


2. What would qualify as extraordinary evidence?
3. What criteria is used to determine what is extraordinary evidence?
4. Are the criteria for extraordinary evidence reasonable?

Hopefully, a healthy dialogue can be had by both parties.


Does the Bible provide extraordinary evidence for Jesus' Resurrection?

If you read the paper on Extraordinary claims require extraordinary evidence, then you read the
concluding points about the validity and weaknesses of the position and addressing four major points:

1. Will their presuppositions allow unbiased examination of the evidence?


2. What would qualify extraordinary evidence?
3. What criteria is used to determine what is extraordinary evidence?
4. Are the criteria for extraordinary evidence reasonable?

Nevertheless, does the Bible actually provide extraordinary evidence for extraordinary claims? I
believe so. But, instead of providing a list of various claims and evidences, I want to focus on the
most important one in the Bible: Christ's resurrection.
Basically, does the extraordinary claim that Jesus rose from the dead have any extraordinary
evidence to back it up? But, since this is a subject of history, we cannot apply the methods of
experimentation and repetition to see if it happened. We don't have any film. All we have is the
evidence presented in the Bible, a document of history. Like the extraordinary Alexander the Great
who conquered the known world by the age of 33, the resurrection of Jesus is also a historic event.
Following is a chart that categorizes some Biblical facts into two categories. I admit this is a bit
subjective, but I think that my analysis is sound. Afterwards, I will briefly comment on each one.

Extraordinary claim: Jesus physically rose from the dead


Extraordinary evidence Not so extraordinary evidence
1. Textual reliability of the ancient document 5. Accounts written by eyewitnesses
2. Retention of crucifixion wounds post event 6. No counter historic information
3. Post death appearances to many people 7. Jesus' body is gone from the tomb
4. Prophetic fulfillment 8. Changed lives

1. Textual reliability of the ancient document


A. The New Testament documents are 99.5% textually pure. This is indeed an
extraordinary fact since all other ancient documents do not even approach this level of
accuracy.
2. Retention of crucifixion wounds post event
A. This would indeed be an extraordinary evidence of a resurrection to see the actual holes
in Jesus' hands and side after he had died on the cross.
i. John 20:27, "Then He *said to Thomas, "Reach here your finger, and see My
hands; and reach here your hand, and put it into My side; and be not
unbelieving, but believing."
3. Post death appearances to many people
A. It is indeed extraordinary to have someone who has died in public at an execution to
appear to many people afterwards.
i. John 20:26, "And after eight days again His disciples were inside, and Thomas
with them. Jesus *came, the doors having been shut, and stood in their midst,
and said, "Peace be with you."
4. Prophetic fulfillment
A. Fulfilling prophecies made hundreds of years earlier about Jesus birth, death, crucifixion,
and resurrection is indeed extraordinary.
5. Accounts written by eyewitnesses
A. It is perfectly ordinary to have people write about what they saw. History is full of such
accounts.
6. No counter historic information
A. There is no contradictory historical information concerning Jesus' resurrection. This
doesn't prove anything, but when the gospels were written, people contemporary to the
described events (Jews, Romans, etc.), could have easily written something refuting or
correcting the resurrection account. No such writings exist. This isn't extraordinary, but
it is important.
7. Jesus body is gone from the tomb
A. It is not extraordinary for a body to disappear from a tomb if we realize that it could
have been stolen.

We can see that there is sufficient reasons to believe that the Bible does indeed provide
extraordinary evidence for an extraordinary claim; namely, the resurrection of Jesus.
It is improbable that Jesus rose from the dead

When someone says that it is improbable that Jesus rose from the dead, he is speaking logically.
The fact is that probability strongly works against Jesus rising from the dead. After all, how many
people have risen from the dead in this century? If it had happened, would not the news have reported
it? Would not the doctors have known about it? Anyone rising from the dead would be a noteworthy
event. So, on one hand, it is true that it is improbable that Jesus rose from the dead. However, on the
other hand is not.
If there were no God in the universe and if all things followed the natural laws that we know and
universe then indeed it would be highly improbable that anyone would rise from the dead. But if there
were a God who controls the natural laws and is in fact the author of those laws, then it would be easy
for him to raise someone from the dead. The issue of improbability cannot be examined without
examining the concept of whether or not God exists. After all, if he does exist the resurrection of Christ
is certainly possible. So we see that someone's presuppositions about the existence of God will affect
whether or not he or she can accept the idea that Jesus can rise from the dead. Even though
statistically it is not normal that anyone would rise from the dead, the statistical improbability does not
mean that it is impossible.
But we see in the New Testament eyewitness accounts of people seeing Jesus after He was
crucified, died, and buried. Take, for example, the following accounts of Jesus appearing after His
death and burial.

• John 20:25-28, "The other disciples therefore were saying to him, “We have seen the Lord!”
But he said to them, “Unless I shall see in His hands the imprint of the nails, and put my finger
into the place of the nails, and put my hand into His side, I will not believe.” 26And after eight
days again His disciples were inside, and Thomas with them. Jesus came, the doors having
been shut, and stood in their midst, and said, “Peace be with you.” 27Then He *said to Thomas,
“Reach here your finger, and see My hands; and reach here your hand, and put it into My side;
and be not unbelieving, but believing.” 28Thomas answered and said to Him, “My Lord and my
God!”
• Luke 24:39, "See My hands and My feet, that it is I Myself; touch Me and see, for a spirit does
not have flesh and bones as you see that I have."

Of course, simply quoting the Bible is not sufficient for skeptics who cannot or will not believe in
the resurrection of Christ. But it is difficult to blame them because someone rising from the dead is
indeed improbable. In fact, they would say that such an extraordinary claim would require
extraordinary evidence. This is not unreasonable if applied fairly and consistently to the context of
history. I have written on this in the paper "Extraordinary claims require extraordinary evidence."
Does the New Testament provide extraordinary evidence for the resurrection of Christ? Quite
frankly, yes it does. It does in that the eyewitness accounts which were written down by the apostles
of Christ, were preserved on an extraordinary good level. There is absolutely no comparable ancient
document or documents that even approaches the accuracy and reliability of the New Testament
documents. This is indeed extraordinary. To see more on this, please read "Does the Bible provide
extraordinary evidence for Jesus' resurrection?"
Just because something is improbable, does not mean that it is impossible. Given that God exists
in that he is involved in human history, and that Jesus performed many miracles, walked on water,
and raised others from the dead, it is not improbable to conclude that he has risen from the dead. In
fact, in light of the eyewitness accounts that have been accurately transmitted to us, it is perfectly
reasonable to trust in his resurrection.
The Christians were mistaken about Jesus' resurrection

Sometimes critics of Christianity say that Jesus' disciples were mistaken about His resurrection.
They say that because no one can rise from the dead, then the disciples were wrong when they said
that Jesus rose from the dead. First of all, they are assuming something that may not be true. After
all, if there is a God, then why can't a resurrection happen? But, when I ask them to explain how it
was possible to be mistaken about something like a person rising from the dead according to the
gospel accounts, I don't get any answers except, "Well, they were wrong."
It is true that the disciples made mistakes. After all, they were only human. But, how could they
be mistaken about something as serious and as monumental as Jesus rising from the dead? Is it likely
that they simply goofed, that somehow after seeing Jesus die on the cross, and after fleeing and going
into hiding that the figure that appeared before them in the closed room that looked like Jesus and
sounded like Jesus and had holes in His hands and feet really wasn't Jesus? Were the women who saw
the empty tomb also mistaken when they looked into it and saw that the body wasn't there? Was the
apostle John mistaken when he said that Jesus appeared before Thomas and said, "Reach here your
finger, and see My hands; and reach here your hand, and put it into My side; and be not unbelieving,
but believing," (John 20:27). Was it Jesus or not? If not, then who was it? Did the disciples make up
the story? Did the apostle John lie when he wrote the account? If so, where are the records refuting
this preposterous notion? There aren't any.
Is it possible that all the disciples were mistaken about the same thing at the same time especially
when they believed that Jesus had died and was still dead? What would cause them all to switch from
believing that when you're dead you're dead to believing that Jesus died and rose from the dead? Was
Paul the apostle also mistaken when he was riding along the road to Damascus and claims to have
encountered Jesus? Remember, Paul was a persecutor the Christians. He had authority to arrest the
Christians and imprison them. He was a devout Jew and quite powerful in the Jewish religious
system. How is it that he changed his mind so drastically and claims to have seen the risen Jesus ( 1
Cor. 9:1)? Was he also simply mistaken? If so, how? What did he see on the road to Damascas that
changed his life if not something incredible?
Is all of Christianity a big "oops"? Might we meet the disciples in the afterlife and have them say to
us, "Uh, remember that resurrection thing about Jesus we wrote about? Well, we goofed. It really
didn't happen. We mistook the empty tomb -- never did find His body -- the prophecies of the Old
Testament about Him rising (Psalm 16:10), the prophecy of Jesus saying He'd rise (John), the
accounts of the women saying that they had seen Him risen, the appearances of a man who looked
like Jesus and who had holes in his hands and feet and appeared to us in closed rooms, the conversion
of Paul -- that was weird -- oh, and all those miracles He did and those that we then did afterwards,
too, well, that was all a big mistake. Also, it was a big mistake going around Israel and all of the
Mediterranean proclaiming Jesus had risen from the dead while we suffered persecution and
death...yeah, it was all a big mistake. Hope there are no hard feelings."
Is it rational to think that the disciples were simply mistaken about something as serious as stating
that Jesus had risen from the dead? How do you mistake someone rising from the dead? What would
have to happen for numerous people to change their minds about someone coming back to life? Or is
it more rational to simply conclude that the disciples weren't mistaken and that Jesus actually did rise
from the dead?
The New Testament writers conspired together to gain power and
influence

It is certainly possible that the New Testament writers worked together and concocted a plan to
use a good man named Jesus, who had recently died, in order to gain power and influence for
themselves. But just because something is possible does not mean that it is a reality. It is possible
that there is an ice cream factory on Jupiter, but that does not mean that one exists. When we look at
the New Testament claims of Christ do we see what looks like an elaborate deception concocted by
several people? Or do we see that their behavior is more consistent with the idea that Jesus actually
did do miracles and rise from the dead? It is the latter explanation that best fits the facts.
Following is a list of reasons why the conspiracy theory does not work.

It would require great coordination of events and writing over a long period of time.

First of all, in order for this conspiracy to work several people would have needed to get together
and write documents that were not only inspirational but reflected accurate historical accounts, could
stand up to cross examination, and agreed with each other sufficiently to avoid being exposed as a
fraud. After all, if their stories and writings were contradictory, their conspiracy would fall apart. This
means that there had to be large and sophisticated collusion and careful, deliberate fabrication over a
long period of time since the New Testament documents were written over approximately a 50 year
span. The writers would have to be very careful about who was named and what places were
mentioned. Why? Because the accounts dealt with actual places and people and they would have to
make sure it was all correct.
If these people wanted to gain power and influence by concocting a plan as grandiose as this, is it
logical to say that they agreed to make up a story about this person Jesus, who was known to many
people, and say things about Him that were not true, and then get people to believe that He had risen
from the dead? Does it make sense that they would go against not only the Jewish system but also
that of the Roman Empire, all so that they could try and gain power and influence in an area already
dominated by two powerful cultures, the Jewish and Roman? Or is it more logical to say that they
didn't conspire to deceive, but simply wrote and testified to what they saw? Doesn't it make more
sense to say that they wrote what they knew, recorded the facts, the places, and the events and that
it was all true and that that explains the New Testament documents better than anything else?

It would mean that the NT writers wrote about truth based on a lie

The writers of the New Testament used the words "true" and "truth" 170 times. They lived for the
truth of what they believed and they died for it as well. They wrote about truth (Rom. 9:1; 2 Cor.
4:2), honesty (Luke 8:15), love (1 Cor. 13:4-8), integrity (2 Cor. 7:2), compassion (Col. 3:12),
forgiveness (Col. 3:13), etc., and it was all based upon their love for and dedication to the truth of
Jesus. They spoke against hypocrisy (Rom. 12:9), lying (Col. 3:9), jealousy (James 3:13), and selfish
ambition (James 3:16). In fact, they lived according to their words. Does it really make sense to say
that the NT writers deliberately conspired to misrepresent the truth and then go to great depths, even
to suffer beatings and death, all while they were continuously telling people to believe in a lie? Add to
this how they knew they would be persecuted for this alleged conspiracy of lies and we have serious
problems explaining their behavior. It would make far more sense to simply acknowledge that they
were telling the truth and that it was not a conspiracy to deceive.

It would mean that the conspiracy would have to survive cross examination

For the conspiracy to work, it would have to face cross examination. Remember, the gospels were
written as historical documents mentioning places, people, and events. There certainly were many
people who were still alive and who could verify and/or deny the miraculous events concerning Jesus.
If you want to make a conspiracy work, you don't offer verifiable facts. Instead, you make up stories
that cannot be verified but sound good. This is what Joseph Smith did when he began Mormonism.
Nothing of his great cities and civilizations in the Book of Mormon have been verified since 1830 when
he published his book of Mormon. Smith's religion isn't based on historical fact with verifiable
locations and events. Instead, it is based on a story that cannot be verified. This is not the case with
the New Testament books. The Gospels contained records of Jesus performing many miracles and
eventually rising from the dead in Jerusalem. He was crucified at the hands of the Romans who were
urged by the Jewish Sanhedrin. This was verifiable at the time especially since names and places are
listed in the gospels and epistles. All anyone would have to do is contact those people (or check the
court records) and go to those places to verify the accounts.
If it was all a conspiracy, then where are the contradictory accounts refuting what the New
Testament writers claimed? The problem is that there are no contradictory documents known
anywhere that attempt to refute the claims recorded in the Gospels. In other words, there is no
contradictory evidence even though there were plenty of people around who could have written
material contrary to the claims of the New Testament. After all, Pontius Pilate was named ( Matt. 27:2),
as was Herod, king of Judea (Luke 1:5), the high priest Caiaphas (Matt. 26:3), Elizabeth (Luke 1:57),
Mary (Matt. 1:25), John the Baptist (Matt. 3:1), Paul the apostle a convert from Judaism (Acts 9), etc.
Locations were cited: Damascas (Acts 9:10), Cyprus and Cyrene (Acts 11:20), Jerusalem (Matt.
16:21), etc. Also, claims of Old Testament prophetic fulfillment were made (see Prophecy, the Bible,
and Jesus) and all people had to do was read the Old Testament to check. In other words, there were
plenty of people, most of whom were still alive, and places to go to and check in order to expose the
conspiracy. But we find no contrary evidence or writings concerning the miraculous events of Jesus
life, death, and resurrection.
If there is no contrary evidence, no contrary writings, then does it make sense that it was all a
conspiracy? Of course not. If it was a conspiracy, then where is the evidence for it?

It would mean the conspirators would have to face persecution

Undoubtedly, if the writers of the New Testament documents wanted to gain power and influence
by writing about a new religious system that would go against the culture of Judaism as well as that of
the Roman Empire, they most assuredly knew they would face persecution. We have to remember
that the culture of the time was not beset with litigation and polite procedures. People often reacted
irrationally and would spontaneously try to kill people (John 8:59). It also means that those who
wrote the New Testament faced certain social, economic, and theological pressures.
In the Jewish culture the religion was intimately interwoven into the social and economic fabric.
Anyone who would go against that system would knowingly risk starvation, mockery, beatings,
ridicule, loss of family and friends, etc. This is not something to be considered lightly. Perhaps a
single demented individual might consider doing such a thing, but how is it possible to get Matthew,
Mark, Luke, John, Paul, Peter, James, Jude, Timothy, Apollos, etc. to all join in the charade, risk loss of
family, reputation, economic stability, be persecuted and maybe even face death? Is this something
that is rational to consider? Should we believe that they were all working together to deceive people
so they could gain power, fame, and influence? It is simply extremely unlikely and full of problems as
a theory.

It would have to explain Paul's Conversion

How did the Christian conspirators persuade Paul who was a devout Jew, educated in Jerusalem at
the school of Gamaliel, (Acts 22:3), a Pharisee of Pharisees (Acts 22:3), and who was given letters of
authority by the Jews to go out and arrest Christians (Acts 9:1-2), to become a Christian and thereby
give up everything he had come to believe and stand for? Remember, Paul was a heavy persecutor of
Christianity: "And Saul was in hearty agreement with putting him [Stephen] to death. And on that
day a great persecution arose against the church in Jerusalem; and they were all scattered throughout
the regions of Judea and Samaria, except the apostles," (Acts 8:1).
The most logical reason for Paul's conversion is that Jesus actually appeared to him on the Road to
Damascus in Acts 9. It would take something pretty severe to cause Paul to abandon everything he
had been taught his whole life and to not only convert, but to also advocate, and teach about the risen
Lord Jesus -- and he did this for years before he was finally killed for his faith. So, how would the
conspiracy theory account for Paul's incredible conversion and life long pursuit of proclaiming Jesus as
Lord and Savior? If an adequately plausible explanation cannot be offered, then the simplest one is
best; namely, that Jesus appeared to Paul on the road to Damascus and Paul was then converted.

Occam's Razor

There is a principle known as Occam's Razor. This principle states that generally the simplest
explanation is the best. When we examine the facts about the New Testament claims is it simpler to
say that the New Testament writers conspired over many decades to write about actual places and
people in such a way so as to convincingly deceive thousands of people into believing that Jesus was
the Messiah, fulfilled Old Testament prophecy, healed the sick, cured diseases, claimed to be divine,
raised Lazarus from the dead, was crucified by Romans after enduring the religious court of the
Sanhedrin, was buried, and rose from the dead or that it simply all happened and they recorded it?
Which is the simpler explanation? Which requires greater faith?

Did the conspirators get what they were after?

Finally, if power and influence were sought by the New Testament writers, did they attain it? At
best, what they have gained by such an elaborate hoax would have been influence in a small group of
people who were outcasts in Israel as well as Rome. Remember, to get followers into Christianity
meant that you went against not only the Jewish system but also the Roman system, not to mention
being able to concoct a story that could stand scrutiny. Obviously the odds are extremely against such
a thing.
Did they get what they were after? They were outcasts in their own society. They were beaten,
ridiculed, accused of debauchery, jailed, beaten, and executed. If it was all a conspiracy, did they get
the influence and power they were after? It doesn't seem so. Instead, it simply makes more sense to
believe the New Testament than to say it was all a hoax.
Jesus was a magician who made people hallucinate about His miracles

All sorts of excuses and challenges have been offered to contradict or explain away the miraculous
accounts of Jesus' life. Among the weaker challenges offered is that Jesus was some sort of a
magician who was able to get people to hallucinate about His miracles. In other words, countless
people were all seeing Jesus do things that were not really happening and it was Jesus who was
perpetrating this deception upon them. Let's take the account of where Jesus feeds the five thousand
with five loaves and two fish (Matt. 14:19-21). Though it is certainly possible to have one person
hallucinate about this, how do the critics account for five thousand people hallucinating about the
same thing at the same time? Or what about the resurrection? How do the critics explain the
accounts of Jesus appearing to the disciples with holes in His feet and hands? How did Jesus get
numerous people to believe a lie about His resurrection (a mass hallucination?) after the Romans, who
were experts at executions, not only flogged Him severely, beat Him, and hung Him on a cross for six
hours and then pierced His side where water and blood came out? How did Jesus do that?
Some have alleged that Jesus went to the Far East and learned many "tricks" and techniques for
influencing people as well as controlling His bodily functions so as to appear dead. Of course, this kind
of theory lacks any evidence at all and is nothing more than conjecture and guesswork. Besides, the
Bible says in Luke 2:51 that Jesus from a very young age continued in subjection to His parents. This
means that in that culture, Jesus was obligated to stay with His earthly parents and care for them in
accordance to the Ten Commandments which stated that He was to honor His mother and father. His
obligation was to be there and care for them in their old age, not abandoning them for some journey
to the far east in order to learn techniques of mind control.
Hallucinations are misperceptions, false interpretations of reality. It is certainly possible for a
single person to have a hallucination about something. But, how do you get two, three, or four people
to misperceive reality and claim to see the same thing at the same time -- like Jesus'
resurrection? That is very difficult to do. In fact, have you ever heard of a group of people
succumbing to a mass hallucination and all of them believe the same thing?
But then, some might say that Jesus was able to hypnotize people which would account for the
mass hallucination. But you must remember that if Jesus were hypnotizing people, then He would
have had to do it over and over again in different circumstances (in homes, in temples, in open fields,
in boats, from the cross, etc), with hostile audiences (Pharisees, Sadducees, etc.), as well as those
who were already believing Him. If Jesus was so good a hypnotizing people and getting them to
believe things that weren't true (which makes Him a deceiver), then why did He not fool people and
escape the sentence of being beaten and crucified? Or is it all part of the incredibly great hoax that
Jesus somehow managed to accomplish on hundreds and hundreds of people.
Also, did Jesus teach His disciples how to do mystical and/or mind control techniques? If that is so,
then where is the evidence? Merely claiming that Jesus could do it, does not mean that it is true.
There must be some compelling evidence to support the claim. Simply stating that miracles cannot
happen and this must mean that Jesus was a magician or some sort, is begging the question. In other
words, the critics assume to be true the thing they are trying to prove; namely, that miracles cannot
happen. They then base conclusions upon that assumption which cannot be proven at all.
In order to maintain the theory that Jesus was a master magician who caused people to hallucinate
it would seem that the person holding that position must himself be hallucinating.
Jesus only appeared to have died on the cross - Swoon Theory

The swoon theory is the theory that Jesus never really died on the cross but that He was crucified
and came very close to death. It further states that after He was taken down from the cross and laid
in the tomb, after three days the coolness of the tomb revived Him and He managed to, roll away the
stone, out of the tomb and appear to the disciples making them think He'd risen from the dead.
The swoon theory has been thoroughly refuted by many people and very few continue to bring it
up as a possibility. Nevertheless, following is an outline of why the Swoon theory can't work.
Basically, it is because Jesus' ordeal was far too serious to permit Him to survive.

1. Six trials - three before Jewish officials (Annas, John 18:12-14; Caiaphas Matt 26:57-68; the
Sanhedrin, Matt. 27:1-2), and three before Roman officials (Pilate, John 18:28-38; Herod, Luke
23:6-12; Pilate, John 18:39-19:6).
A. In these trials, Jesus was beat on the face (Matt 26:67).
2. Scourging
A. Scourging was done with a flagrum, a short whip with several
leather strips which were either embedded with pieces of metal
and glass or small metal balls were tied to the ends of the leather
strips. The victim was either tied to a post or tied bent over an
object with his back exposed. The person inflicting the blows had
been trained on how to properly administer the beating so as to
assure the most painful and damaging punishment. The
whipping consisted of 39 lashes. Each lash was administered and
pulled across the back so as to rip the back open. Often the back
muscles were so badly shredded that the skeletal structure was exposed. People very
often died from this punishment alone.
Jesus suffered 39 such lashes. Undoubtedly, his back was very badly beaten and
bloody.
3. Crown of thorns
A. A crown of thorns was placed on the head of Jesus (Matt. 27:29). There are different
thorn bushes growing in the region with thorns being very short to quite long. Even
short thorns can tear the scalp. The crown was woven and then pressed down around
the head ripping the skin. Bleeding would then occur.
4. Purple Robe
A. Wearing a purple robe (John 19:5) may not seem like a physically harmful thing to do.
But, when you consider that Jesus had just undergone a terrible scourging and that His
back had been ripped open and was quite bloody and raw, the robe on His back would
cause additional pain by rubbing against it. Additionally, as the blood began to congeal,
it would congeal into the fabric of the robe. When the robe was ripped off, more
excruciating pain would result.
5. Crucifixion
A. The arms are pulled apart and nails driven through the wrist into a cross beam which is
raised in place. This dislocates the shoulders. The nails in the wrists sever the median
nerve resulting in a burning pain as well as paralysis in the hand. To breathe Jesus had
to press up on the nails in His feet, scraping His raw back on the wood. The body
gradually drains of blood causing the heart to beat faster and faster. Dehydration is
occurring. The breathing becomes more labored and intense as well as frequent adding
to the agony. The blood loss results in extreme thirst as the body craves water to
restore the lost blood. Jesus said, "I thirst" (John 19:28). The heart beats so hard trying
to compensate for the loss of oxygen (due to the lack of blood) in the body that it
eventually ruptures. At this point the chest cavity fills with fluid. The soldier pierced
Jesus' side and out came blood and water, signifying that the heart had stopped beating
and the blood was settling in the chest cavity. Jesus was dead.

6. Burial
A. Jesus' body was wrapped in linen (Matt. 27:59). This wrapping was done tightly around
the whole body from head to toe. We see from the resurrection of Lazarus, that Lazarus
had to be unbound (John 12:44) since help was needed to get out of the linens.
7. Three days without medical attention in the cold tomb
A. The tomb was cold and Jesus laid in it for three days without medical attention.
8. Moving the stone
A. A "large stone" had been placed over the entrance to the tomb (Matt. 27:60). Unless
Jesus had some help, which isn't mentioned, He would have had to move the large
stone. The stone had to be large enough to cover an entrance big enough for people to
walk into. Even if they ducked to get in, the stone was large enough that it would take
more than one person to move it in place.
9. Presence of the guards
A. The Romans guards on the tomb were given the job to guard the tomb. Since there had
been rumors that the body of Jesus might be stolen, they were ready to meet the
challenge. In Roman society, if the prisoner of the guard escaped, the guard would then
take the prisoner's place in punishment. The guards had a strong motivation to not let
anyone take the body of Jesus.
10. Walked on pierced feet to get to the disciples.
A. Jesus appeared to several people after His resurrection. Does this mean He walked on
feet with holes that had been made by nails several inches long?

The swoon theory falls apart quickly when you consider that Jesus had undergone six trials, been
beaten, then scourged with 39 lashes that left His back raw, exposed, and bloody, had a crown of
thorns forced upon His head, ripping His scalp, been crucified with nails in the hands and feet, hung
there for six hours bleeding and dehydrating, had his side pierced with a spear which emitted blood
and water, was left in a tomb for three days, and was tightly wrapped up. Was anyone in this
condition able to revive, get himself out of the tight wrappings, walk on pierced feet, and single
handedly move a large stone with hands that were unusable due to the wrist piercings which severed
the median nerve in the hands and paralyzed them, and then some how got by the armed guards
given the charge of watching the grave side? Are we to further believe that Jesus managed to walk a
long distance on feet which had been pierced through and appear to the disciples as a victorious
conqueror of death? It makes no sense. In fact, it would take more to believe this ridiculous
conjecture than it would to believe that Jesus rose from the dead.
The Disciples stole Jesus' body and faked His resurrection.

This possibility has been raised by critics ever since Jesus rose from the dead. But it has never
taken root except in some Jewish circles because the New Testament account does not support a faked
resurrection theory. Nevertheless, in order for the disciples to have faked the resurrection of Jesus,
several conditions must have been met. Let's take a look at them and analyze them.

1. The disciples would need to concoct an elaborate plan.


A. The disciples would have to have a plan. You can't just walk to a tomb guarded by Roman
soldiers and ask for Jesus' body. So, in order to fake Jesus' resurrection the disciples
would have to obtain and dispose of the body of Jesus without any hostile witnesses
seeing them do this. This would mean that the guards in front of Jesus' tomb would need
to be bribed (discussed later). It would further mean that several people would have to
be involved in carrying the body of Jesus to an area where it could be disposed of. A
single person would not be able to carry another human body a long distance. Therefore,
these several people would have to agree to steal the body of Jesus and risk arrest by the
guards and the Jewish leaders.
Furthermore, this plan would also have to include other people outside the circle of the
disciples since such an "impossible" occurrence as a resurrection would be more
convincing if others who were not biased followers of Jesus said that they saw Jesus after
the crucifixion. This means that the disciples would have to convince a lot of people to go
against the Jewish religious leaders, thereby risking their own economic and social
security, as well as risk bringing conflict into the region since the Jews who sent Jesus to
the cross, could easily persecute these new apparent converts. Additionally, this would
bring further attention of the Romans to the issue thereby escalating tension which was
not something the Jewish people wanted.
One more thing, it would be very obvious to the disciples that to continue claiming
Jesus rose from the dead would bring the harsh attention of the religious leaders upon
them. Remember, the Jewish leaders knew who Jesus' disciples were. Therefore, easy
attention could be focused on them in the form of persecution. Unlike others, the disciples
would be easy targets. Since the Jewish leaders had just sentenced Jesus to die a horrible
death on the cross, what would stop them from continuing with the disciples who would
then start proclaiming Jesus had risen from the dead? The disciples had to know what
they were getting into. They were risking their families and their own lives.
In all, concocting an elaborate plan to deceive many people has too many difficult
variables in it to overcome. It would simply make more sense to assert that the reason
the disciples proclaimed the resurrection of Jesus is because they actually saw the
resurrected Jesus.
2. A sufficient motive would have to be offered to account for the disciples' intended
deception?
A. Remember, we have many people in the Bible who said that Jesus rose from the dead. Did
these people all agree to lie? If so, why would they do that? What would motivate various
people, who have differences of opinions, differences in needs and desires, to all agree to
testify to something false? Could it be that they were dissatisfied with the Roman Empire
ruling over the Jewish nation? But what would they accomplish by proclaiming Jesus'
resurrection? Did they think that the Roman Empire would suddenly leave Israel because
of that? Not a chance.
Or perhaps the people were tired of the hypocrisy of the Jewish religious leaders and it
motivated them to claim Jesus rose from the dead in order to undermine their authority.
But if this is the case then we have an inconsistency between motive and behavior
because people who would be upset with someone else's hypocrisy are not likely to
proclaim such an incredible lie as a resurrection -- thereby being even bigger hypocrites
than the leaders. Does this make sense? Also, since Jesus taught love, truth, and self
sacrifice, such deceptive actions would be in direct contradiction with the teachings of the
One they were following.
At best all anyone can do is guess about what the disciples may have been thinking or
what might have motivated them to devise an elaborate deception. Guessing is all that
can be done. But we would need to ask if any proposed motives of the disciples could be
harmonized with the facts of their preaching and teaching about truth, long-suffering,
patience, kindness, and love. No one can read their hearts or their minds and insert into a
scenario 2000 years old the motivations of people long gone. It is best to simply let the
facts speak for themselves. They lived, suffered, proclaimed, and died for the truth of the
resurrection.
3. The guards at the tomb would need to be bribed.
A. The problem of bribing works both ways. The disciples could have bribed the guards to not
say anything about them taking the body of Jesus. But, the Jews could also have bribed
the guards into saying that the disciples stole the body of Jesus. In fact, the only bribing
we see in the gospel accounts of the guards is done by the Jewish leaders. Matt. 28:11-
15 says, "Now while they were on their way, behold, some of the guard came into the city
and reported to the chief priests all that had happened. 12And when they had assembled
with the elders and counseled together, they gave a large sum of money to the soldiers,
13
and said, "You are to say, ‘His disciples came by night and stole Him away while we were
asleep.’ 14"And if this should come to the governor’s ears, we will win him over and keep
you out of trouble." 15And they took the money and did as they had been instructed; and
this story was widely spread among the Jews, and is to this day."
The bribe would be very necessary since the guards were at risk of their lives if they
had failed at their duty. It was the custom of the Roman military that if a prisoner
escaped, then the guard(s) who was in charge of guarding the prisoner would take the
prisoner's place. This is a very strong motivation to make sure that they carried out their
duty, bribe or not. In Acts 16:25-30 when Paul was in prison, there was an earthquake
that opened the cell doors which would have allowed the prisoners to escape. When the
jailor saw this he intended to kill himself. Undoubtedly, this is because he knew he would
have to take their place should they escape. But Paul called out and said in verse 28, "Do
yourself no harm, for we are all here!" This shows that the guard did not want to take the
place of the prisoners. In fact, in Acts 12:18-19 we read of how Herod ordered guards to
be executed who had allowed Peter to escape.
Therefore, for the guards to risk their lives, a large bribe would be necessary. Who was
more likely to have enough money to bribe the guards, the religious leaders or the
disciples? Also, who had a greater motive, the disciples who wanted to have Jesus rise
from the dead (risking further persecution), or the Jews who wanted to complete their
attempt to be rid of Jesus?
4. The body of Christ would need to be disposed of to prevent disproof of his
resurrection.
A. If the scenario of an elaborate plan with bribed guards and collusion on the part of many
non followers of Jesus were to be effective, the body of Jesus would need to be disposed
of. If the disciples could get a hold of His body and get away from the population, it would
not be difficult at all to bury it someplace. It would then be necessary that the disciples
promised that they would never disclose the location. This is a possibility but it would
mean that the disciples were liars and thieves. Is this basis for their faith consistent with
their writings about truth, honesty, etc., combined with their dedication to their assertion
of Jesus' resurrection that cost them their lives?
5. Various witnesses would need to be arranged
A. As I have already stated above, many people would be to be coached into lying about
seeing the risen Lord. Is this probable for so many Jews who grew up under the idea that
lying was a sin? Perhaps. But, is it easy to convince people to lie about an event that
they know would bring them economic, familial, social, and religious difficulties? The
answer, of course, is no it is not. The Jewish people were living under Roman rule. The
Romans served both as oppressors and protectors. They were oppressors and that they
forced many of their own rules upon the Jewish people. On the other hand, they protected
the land of Israel from hostile nations surrounding them. Friction in the region is not
something people would want to have, especially if they have families with children and
parents to take care of and to love. Does it really makes sense that so many people would
agree to such a great lie for such a great consequence?
The Apostle Paul

But what about Paul the Apostle? Did the disciples plan on converting one of their greatest enemies
into a Christian? How did they get Paul to agree to the conversion and in so doing convince Paul to
give up everything he had stood for and worked for his entire life in order to be ostracized,
condemned, persecuted, shipwrecked, beaten, and finally martyred by both the Romans and the
Jewish leadership? Does a faked resurrection account for such a bold and profound conversion of
someone who had been seeking to destroy the very Christians that he later proclaimed? Remember,
Paul claimed to have seen the Lord on the road to Damascus (Acts 9; 1 Cor. 9:1)? What would
motivate him to give up everything and to proclaim Christ's resurrection? What would he have to gain?
Power? Money? Fame?
If Paul wanted power, then perhaps it could be said he achieved it since he wrote much of the New
Testament and had great influence in the Christian Church. But, power is not what he demonstrated
over anyone. The New Testament does not demonstrate any wielding of power. Some of Paul's
writings are the greatest testimonies to truth, love and wisdom that have ever been written. Are the
words of Paul in 1 Cor. 13, or Col. 3, and the entire book of Romans the words of one man who knew
that everything he was writing and teaching was based on a lie just so he could get power? It just
doesn't make sense to say so.
If it was money Paul was after, then why did he preach without charge ( 2 Cor. 11:7)? Why did he
often go without food (2 Cor. 11:27)? Why did he have odd jobs in order to make a living (Acts
18:3)? It does not make sense to say that he was in it for the money.
If it was fame that he was after, then he certainly attained it. Paul the apostle is still a famous
person throughout all of Western civilization. But we cannot know if this was a motive or not since we
cannot ask him. What we can do is read what he wrote and do our best to discern his motives there.
It would be up to the reader to read his epistles and see if the quest for fame is woven into his words.
Personally, I see no such thing when I read his works. I see a man who preached Jesus and Him
crucified and risen from the dead.

Conclusion

It is very unlikely that the disciples faked the resurrection of Christ. In summation, this is why:

1. They would need an elaborate plan involving many unpredictable elements: guards, other
witnesses, etc.
2. There is a large problem in developing a motive to deceive that would be greater than the
consequences of that deception. Remember, the disciples would be risking their security,
safety, families, and their lives for their beliefs.
3. The guards at the tomb would have to be bribed, but the only bribing we see is from the
Jewish leaders (Matt. 28:11-15) who had a very strong motivation to finish what they had
started with Jesus.
4. Various witnesses not involved with the disciples would have to be obtained in order to
validate the story. But this means that a strong incentive would have to be offered to the
additional witnesses since their story would likely get them in deep trouble with the Jewish
leadership.
5. The apostle Paul. He is a wild card. What illegitimate thing would motivate him to proclaim
the resurrection of Jesus when it didn't happen? Remember, he was a heavy persecutor of
the church. Something happened to change him. According to him, it was the appearance
of the risen Lord Jesus.
There are no non-biblical accounts of the resurrection

First of all, saying that there are no non biblical accounts of the resurrection does not invalidate the
resurrection. The New Testament documents, particularly the Gospels, were written by eyewitnesses
or under the direction of eyewitnesses before the death of the apostles. Therefore there were plenty of
people around who could have contested the post crucifixion appearances of Christ. We must first
understand that the Gospels are historical documents and they are reliable ones.
Second, it is not accurate to say that there are no extra biblical accounts of the resurrection of
Christ. There are other historians who have written about this. However, the problem with most of
them is that they were not contemporaries of Jesus. They were written well after the fact. This,
therefore, tends to invalidate the reliability of these extra biblical accounts according to the critics. But
if the extra biblical accounts are not valid because they were written after the fact by non
eyewitnesses, then that indirectly supports the gospel accounts which were written by the
eyewitnesses, by those who knew Jesus, and encountered him after his resurrection.
Third, how do you have witnesses to the resurrection? Even the disciples didn't see Him rise from
the dead. Instead, they saw Him after He had risen, as was evidenced by the wounds in His hands and
side when He appeared to them (John 20:27). He appeared to those who most needed to see Him.
They were the ones who had spent years with Him, watching Him do miracles, watching Him heal the
sick, and teaching great wisdom and love. After Jesus died, their faith in Him had been shattered. It
was necessary that Jesus appear to them in order to establish the truth of who He said He was;
namely, God in flesh (John 8:24,58; 10:30-33).
Fourth, Jesus would have to appear only to those who had seen Him before His crucifixion since
appearing to someone who had never seen Him nor knew that He died, would prove nothing. This
means that the ones whom Jesus would appear to were those who were following Him in the area of
Israel. This further means that at best, other records of His resurrection would have to be hearsay,
written well after the fact, by those who did not know Jesus.
Fifth, we do have non-biblical accounts of the resurrection of Jesus.

• Now there was about this time Jesus, a wise man, if it be lawful to call him a man; for he was a
doer of wonderful works, a teacher of such men as receive the truth with pleasure. He drew
over to him both many of the Jews and many of the Gentiles. He was [the] Christ. And when
Pilate, at the suggestion of the principal men amongst us, had condemned him to the cross, (9)
those that loved him at the first did not forsake him; for he appeared to them alive again the
third day; (10) as the divine prophets had foretold these and ten thousand other wonderful
things concerning him. And the tribe of Christians, so named from him, are not extinct at this
day.
 There is debate among scholars as to the authenticity of this quote since it is so
favorable to Jesus. For more information on this, please see Regarding the quotes from
the historian Josephus about Jesus

No sign would be given

Jesus typically would not demonstrate anything miraculous to those who refused to believe in Him.
It is, therefore, consistent with Jesus' method to demonstrate Himself to those who were in need of
Him and who did not mock Him and doubt Him. Like it or not, this is how He operated. It would be
logical to assume that He would deal in the same manner after His resurrection and only appear to
those who knew Him and followed Him. For verification of Jesus' denial to those who doubted him,
please note the following quotes.

• "And the Pharisees came out and began to argue with Him, seeking from Him a sign from
heaven, to test Him. 12And sighing deeply in His spirit, He *said, "Why does this generation
seek for a sign? Truly I say to you, no sign shall be given to this generation." 13And leaving
them, He again embarked and went away to the other side," (Mark 8:11-13).
• "Then some of the scribes and Pharisees answered Him, saying, "Teacher, we want to see a
sign from You." 39But He answered and said to them, "An evil and adulterous generation craves
for a sign; and yet no sign shall be given to it but the sign of Jonah the prophet; 40for just as
Jonah was three days and three nights in the belly of the sea monster, so shall the Son of Man
be three days and three nights in the heart of the earth," (Matt. 12:38-40).
• "And the Pharisees and Sadducees came up, and testing Him asked Him to show them a sign
from heaven. 2But He answered and said to them, "When it is evening, you say, ‘It will be fair
weather, for the sky is red.’ 3"And in the morning, ‘There will be a storm today, for the sky is
red and threatening.’ Do you know how to discern the appearance of the sky, but cannot
discern the signs of the times? 4"An evil and adulterous generation seeks after a sign; and a
sign will not be given it, except the sign of Jonah." And He left them, and went away," (Matt.
16:1).
• "For the heart of this people has become dull, and with their ears they scarcely hear, and they
have closed their eyes lest they should see with their eyes, and hear with their ears, and
understand with their heart and return, and I should heal them," (Matt. 13:15).

Jesus plainly taught that He would not "perform" for those who denied Him. He did, however, do
public miracles in order to validate who He was, God in flesh (John 1:1,14; 8:24; 8:58). This great
truth is a matter of faith and is not something proven with a calculator or a camera. Jesus claimed to
be the Son of God which, in that culture, meant to claim equality with God ( John 5:18). Jesus said
that "Before Abraham was, I AM" (John 8:58) a statement that infuriated the Jews who were familiar
with God's self description to Moses in Exodus 3:14 when He said, "I AM that I AM. Thus you shall say
to the sons of Israel, I AM has sent me to you." It is a claim made real by evidence. The evidence
was His miracles.

Conclusion

The real issue of the resurrection deals with its evidence. This evidence consists of the testimony of
many people who stated that they had seen Jesus after His crucifixion and death. The same people
who testified of the resurrection of Christ also gave up their social and economic security and put their
lives on the line in order proclaim that Jesus had risen. Does it make any sense at all to say that they
knew Jesus did not rise from the dead and had concocted an elaborate plan in order to deceive a great
many people into believing that Jesus had risen? Why would they do that? Does it also make any
sense that they would continue in this lie while being persecuted, ostracized from family and friends,
beaten, imprisoned, and finally killed for what they believed? It makes more sense to believe that
their actions were consistent with their teaching. In other words, they taught about self-sacrifice,
dedication to truth, love, peace, etc., and they based it all on the risen Lord. It was based upon the
truth that they had seen.
Miracles cannot happen
Before we can decide whether or not miracles can happen, we must first define what a miracle is.
Basically, a miracle is an event that cannot be normally explained through the laws of nature. In the
context of Christianity, miracles are the product and the work of God who created the natural laws as
well as the universe. However, vital to the discussion of whether or not miracles can occur is the
issue of a person's presuppositions. If someone believes that there is no God and also believes in what
is called naturalism - that all things in the universe are subject to natural physical laws - then miracles
are defined out of existence. That is, the universe is defined in such a way as to make miracles
impossible. Therefore, if someone says that miracles cannot happen, then it is most probable that they
deny the existence of God and/or believe in naturalism along with its companion, evolution.
On the other hand, if someone believed that there was a God and that God is involved in the world,
then it is easy to acknowledge that miracles can occur. If God created the universe as the Bible states
(Gen. 1), why can't God also intervene in our world and perform miracles? Take the resurrection of
Jesus, for example. With an atheistic, naturalistic presupposition the resurrection of Christ could not
occur since people simply do not rise from the dead, no matter what is said. Therefore, the account of
Jesus' resurrection must be wrong. Either the Bible is untrustworthy, the witnesses collaborated on a
lie, Jesus never died, He only appeared dead, His body was stolen to make it look like He'd risen, or
someone else died in His place. Either way, the non-God, non-miraculous presupposition would not
allow the skeptic to believe in the resurrection of Jesus, especially after three days of being in the
tomb. It just could not have happened, no matter what. The problem is that with this kind of
presupposition, objectivity can be thrown out the window. This is especially ironic since many atheists
consider the Christians to be the ones who lack objectivity.

Weigh the Evidence

If someone believed that miracles were possible because he believed that God exists, then all he
needs to do is look at the Bible, weigh the evidence and decide to believe or not believe in miracles --
like Jesus' resurrection. From the accounts of the eyewitness testimonies in the Gospels we can see
many people believed that Jesus rose from the dead. After all, the Romans, who were expert at
crucifixion, killed Jesus, and put guards on the tomb. Yet, the tomb was found empty. The disciples
who were in hiding, suddenly started proclaiming that Jesus had risen. These same disciples risked life
and limb in order to teach that Jesus had risen. Why would they do that for a lie that would cost them
everything, even their lives…unless it really happened?
Since Christians do not have a presupposition that excludes the miraculous, we are able to look at
the resurrection of Christ as recorded in the Bible, weigh the evidence, and make a choice to believe or
not believe. Of course, Christians by default believed in the resurrection of Christ.

Logic

Finally, it would be illogical to state that miracles cannot occur because in order to logically state
that miracles cannot occur, a person must either know all things in the universe to know that miracles
cannot occur, or he must have a logical proof why miracles cannot occur. Furthermore, it is not
enough to state that there is no evidence for the miraculous since a person's experiential base is
limited. It may very well be that miracles have occurred and this person is simply not aware of it.
Therefore, at best someone could simply say "I do not believe that miracles occur because (insert
reason)." At least this leaves open the possibility that they may occur. And if they might indeed occur,
why not have the possibility that Jesus who claimed to be God (John 8:58 with Exodus 3:14), who
fulfilled Old Testament prophecies (i.e., Psalm 22:11-18; Isaiah 7:14; 9:6, Micah 5:1-2, etc.), who
predicted his own death and resurrection (John 2:19-21; ), appeared to people after His public
execution (Luke 24:39; John 20:25-28), did indeed actually rise from the dead? Since the eyewitness
accounts have been accurately transmitted to us, would it not be logical to believe the witnesses who
described what they saw? It would seem so.
Can miracles occur? Yes, they can because there is a God in the universe.
Was Jesus just a myth?

Was Jesus simply a mythical figure, a fabrication by religious zealots who wanted a symbol to rally
behind for whatever reason they needed at the time? Or, was Jesus an actual person who lived in
Israel 2000 years ago? Most often, those who deny Jesus as a historic figure denounce the New
Testament writings, particularly the gospels, as fabrications or highly embellished stories passed down
through the years. They must do this. Otherwise, they would have to acknowledge that Jesus lived.
In reality, a person must ignore a great deal of evidence establishing the historic accuracy of the
gospels. In other words, the Bible alone is sufficient evidence that Jesus lived, whether or not the
critics want to admit it. But making this claim doesn't establish it as fact. So, let's look at reasons
why Jesus is not a mythical creation, but an actual man who lived in Israel.

The Gospels as history - date of authorship

According to the Christian church, the four Gospels were written by the apostles and/or those
under the direction of the apostles of Jesus. That means that they were written under direction of
eyewitnesses of the actual events. Also, none of the gospels mention the destruction of the Jewish
temple in 70 A.D. This is significant because Jesus had prophesied concerning the temple when He
said "As for these things which you are looking at, the days will come in which there will not be left
one stone upon another which will not be torn down," (Luke 21:5, see also Matt. 24:1; Mark 13:1). If
the gospels had been written after that date and if they were fabrications, then surely they would have
contained the account of the fall of Jerusalem and the destruction of the temple which are known
historical facts. Yet, Matthew, Mark, and Luke contain no such information. Luke was written before
Acts (Acts 1:14). The book of Acts, a history of the Christian church, which doesn't mention the fall of
Jerusalem either, nor does it record the deaths of Paul, James, and Peter which all happened in the
early 60's. This means that Acts was written at least by A.D. 62 and Luke was written before that.
Therefore, the time between the events and the writings is around 30 years. This further means that
the eyewitnesses were around who could have corrected any statements written in the gospels. Yet,
we have absolutely no corrective or contradictory writings from that time, from anyone, denying the
accounts of the gospels. For more information on this please see When were the gospels written and
by whom?
Therefore, we can conclude that the gospels were written well before the close of the first century.

The Gospels as history - historical content

The gospels do not have the sense of myth. If anything, they are written as eyewitness accounts.
Consider the first four verses of the gospel of Luke which clearly states that it is a researched
document.

Luke 1:1-4, "Inasmuch as many have undertaken to compile an account of the things
accomplished among us, just as those who from the beginning were eyewitnesses and servants
of the word have handed them down to us, it seemed fitting for me as well, having investigated
everything carefully from the beginning, to write it out for you in consecutive order, most
excellent Theophilus; so that you might know the exact truth about the things you have been
taught."

This is not how myths are made. This is how you uncover evidence and record it. Luke examined
the witnesses, interviewed them, and checked out the facts. In Luke 2:1-2, we have historically
verifiable information: "Now it came about in those days that a decree went out from Caesar
Augustus, that a census be taken of all the inhabited earth. 2This was the first census taken while
Quirinius was governor of Syria." Also, see Luke 3:1-2, "Now in the fifteenth year of the reign of
Tiberius Caesar, when Pontius Pilate was governor of Judea, and Herod was tetrarch of Galilee, and his
brother Philip was tetrarch of the region of Ituraea and Trachonitis, and Lysanias was tetrarch of
Abilene, 2in the high priesthood of Annas and Caiaphas, the word of God came to John, the son of
Zacharias, in the wilderness." We clearly see numerous historical statements that have been verified
through archaeology. This is precise record keeping, not extravagant additions. In fact, "Sir William
Ramsey has shown that in making reference to 32 countries, 54 cities, and 9 islands he made no
mistakes!"62 Sir William Ramsey (1851-1939) was a classical scholar and archaeologist. He taught
at Oxford England, Aberdeen. He authored several scholarly books dealing with archaeology and had a
major influence upon it as a science.
Nevertheless, there are many verifiable things found in the gospel accounts.

1. Herod, king of Judea, (Matt. 14:1; Luke 1:5).


2. Herodias, the wife of Herod's brother Philip, (Matt. 14:3).
3. Pool of Bethesda, (John 5:115).
4. Pool of Siloam, (John 9:7), etc.

There are many many more citations verified by archaeology that demonstrate the accuracy of the
gospels. When they mention events dealing with rulers, places, events like a census, who was
governor, etc., they are all accurate historically.

The Gospels as history - accuracy of transmission

A very important issue is whether or not the Gospels have been accurately transmitted from the
original writings down to the copies that we have today. Yes, they have been accurately transmitted to
us. The truth is that the New Testament documents are 99.5% textually pure. This means that only
1/2 of 1% of all the copies in existence has any question about the text. If this is compared to any
other ancient writing, the New Testament comes out way ahead. See Manuscript evidence for
superior New Testament reliability for more information on this.
Nevertheless, the accuracy is really even greater than 99.5%. The reason is because many of the
copies that have spelling errors, minor word omissions and additions, etc., are copied and those copies
contain those various minor errors. So, for example, if one manuscript has "Jesus Christ" and it is
copied only as "Jesus", then the following manuscripts will contain only the word "Jesus" where other
manuscripts might contain "Jesus Christ." All that is needed is to compare the copies, see which of
them is the oldest, which fits into the lineage of copies that is correct, or has an error, etc., and the
mistake is usually very easily cleared up by comparing copies. Therefore, we can know what the
original said in almost every case.
Also, if 10 copies contain the same copying error, then the total number of copies with error
increases. But in reality by tracing the text back through parent manuscripts that are uncovered to
archeology, we can uncover manuscripts that shed light on which rendering is correct.
Following is a partial chart detailing various copies of various ancient writings. It should be obvious
that the New Testament documents are extremely well preserved. Therefore, if the New Testament
cannot be considered reliable, then neither can any of the other writings listed below.

Approximate Time
Date Earliest Number Accuracy of
Author Span between
Written Copy of Copies Copies
original & copy
Plato 427-347 B.C. 900 A.D. 1200 yrs 7 ----
Caesar 100-44 B.C. 900 A.D. 1000 10 ----
Aristotle 384-322 B.C. 1100 A.D. 1400 49 ----
Homer 900 B.C. 400 B.C. 500 yrs 643 95%
New 1st Cent. A.D. approx. 5600
less than 100 years 99.5%
Testament (50-100 A.D. c. 130 A.D. (in Greek)

Geisler, N. L., & Howe, T. A. 1992. When critics ask : A popular handbook on Bible difficulties . Victor Books:
62

Wheaton, Ill., p. 384.


Miscellaneous Information

Noted Oxford expert on literature and myths, C. S. Lewis, said, "I have been reading poems,
romances, vision-literature, legends, myths all my life. I know what they are like. I know that not one
of them is like this [the Gospels]."63
Regarding the gospel of Mark. "A date before a.d. 50 leaves no time for mythological
embellishment of the records. They would have to be accepted as historical." 64
"New Testament books appeared within the lifetime of eyewitnesses and contemporaries. Luke was
written by about 60, only twenty-seven years after Jesus’ death, before Acts in 60–62 (see Hemer,
all). First Corinthians was written by 55–56, only twenty-two or twenty-three years after Jesus’ death
(cf. 1 Cor. 15:6-8). Even radical New Testament scholar John A. T. Robinson dates basic Gospel
records between 40 and 60...there is no time or way for a legend to develop while the eyewitnesses
were still alive to refute the story." 65

Conclusion

There is no reason to doubt the reality of Jesus as a historic figure. The gospel accounts are four
different accounts from four different people. They were penned by either eyewitnesses or under the
direction of the eyewitnesses. These same gospels were distributed throughout the region very quickly
and we have no account anywhere on any of the contemporaries attempting to refute any of the facts
written in them -- including those accounts dealing with the miracles of Jesus.
In order for Jesus to be a myth, it would have to be shown that the gospel accounts were highly
embellished and inaccurately copied and transmitted. But, considering that there are other, non
biblical accounts mentioning Jesus, it would be very difficult for anyone to demonstrate that He never
lived.

63
C. S. Lewis, Christian Reflections (Grand Rapids: Eerdmans, 1967), 154–55. cited in Geisler, N. L., & Saleeb, A.
2002. Answering Islam : The crescent in light of the cross (2nd ed.) . Baker Books: Grand Rapids, Mich., p. 244.
64
Geisler, N. L. 1999. Baker encyclopedia of Christian apologetics. Baker reference library . Baker Books: Grand
Rapids, Mich., p. 188.
65
Ibid, p. 518
Did Jesus really die on the cross?

Yes, Jesus really did die on a cross. The scriptures teach this in numerous places. Following are two
of them:

• "And as they were coming out, they found a man of Cyrene named Simon, whom they pressed
into service to bear His cross," (Matt. 27:32).
• "Therefore the soldiers did these things. But there were standing by the cross of Jesus His
mother, and His mother’s sister, Mary the wife of Clopas, and Mary Magdalene," (John 19:25).

There is dispute on the Greek word for "cross" which is "stauros." It can mean an upright stake or
a cross. Therefore, some groups like the Jehovah's Witnesses, say that Jesus died on a vertical stake.
But then, others simply deny that Jesus was crucified at all and they claim He died of old age or some
other natural sickness. Both of these conjectures are in error according to the New Testament.
First of all, there is a verse in the New Testament that demonstrates Jesus did not die on a vertical
stake without a cross bar. If Jesus died on a vertical stake, then His hands would have been placed
together over His head, one on top of another. It would be very easy to then take a single nail and
drive it through both wrists at the same time. This is how crucifixion was done when victims were
crucified in this manner. However, if a person was crucified on a cross, then two nails were required,
one for each wrist, since the hands would be spread apart.
If we look at John 19:25 we see that the plural word "nails" is used in reference to Jesus' hands.

"The other disciples therefore were saying to him, "We have seen the Lord!" But he said to them,
"Unless I shall see in His hands the imprint of the nails, and put my finger into the place of the nails,
and put my hand into His side, I will not believe," (NASB).

The plural form "nails" is used. This means that more than one nail was used upon Jesus' hands.
Therefore, we can conclude that the most logical explanation for the plural use of nails is that there is
at least two nails, one for each hand that was stretched out from left to right as would be done if Jesus
were crucified on a cross. This would mean that the torture stake or the vertical stake theory would
be invalid.

Did Jesus really die on the cross?

According to the gospel accounts, Jesus went through six different trials before He was condemned
to die on a cross. Three of the trials were before Gentiles and three were before the Jews. He was
repeatedly put before people to be tried and the whole time He was under heavy guard. They knew
who Jesus was. So, this excludes the possibility of mistaken identity. Furthermore, it is highly unlikely
that the Romans would have crucified a man by mistake. Remember, Jesus had been performing
many miracles and was quite well known in the area. Since the Roman soldiers had Him in their
possession during the trials, during the beatings, and finally on the way to the cross, the most logical
conclusion is that they did not make a mistake and crucified someone else instead of Jesus. Therefore,
we can logically conclude that Jesus really did die on a cross.
Did Jesus rise from the dead?
This is one of the most important questions of all of history. If Jesus arose from the dead, then
what he said about himself is true. If he did not arise from the dead, then what he said about himself
is not true. Jesus claimed to be the only way to God (John 14:6). He claimed to be able to forgive sins
(Luke 5:20). He also claimed to be divine (John 8:24; 8:58 with Exodus 3:14). Therefore, his
extraordinary claims are tied to his resurrection.
Of course, it is one thing to "say" that Jesus arose from the dead; it is another thing to prove it.
But the problem is we cannot prove that he rose from the dead. The reason is because the documents
that describe Him are 2000 years old. At best, all we can do is look at those documents to determine if
they are reliable and accurate. If they are, then we simply need to look at what they say in order to
see if they support His resurrection or not. It is then up to the individual to accept or reject the claims
thus presented.
This becomes an important issue because a person's presuppositions will govern how he interprets
the data. If a person presupposes that God does not exist, or that miracles cannot happen, then it
would be virtually impossible to convince such a person that the resurrection of Jesus occurred. On the
other hand, if a person presupposes that there is a God and that miracles can happen, then it would be
easier to convince a person that the resurrection of Jesus did happen. So, what are your
presuppositions? Do your presuppositions allow you to objectively look at the evidence in order to
make an "unbiased" conclusion about it?
In my paper Can we trust the New Testament as a historical document? I cover the information
necessary to validate the New Testament documents as being accurate and reliably transmitted to us
today. If you are interested in reading a more detailed examination of this, please click on the link
above. Nevertheless, I will review the information in brief here.

The New Testament is reliable

First of all, the New Testament documents have a greater reliability to them in any other set of
ancient documents in existence. The New Testament documents are 99.5% textually pure. This means
that there is less than 1/2 of 1% of all the 26,000 copies we have of the various documents included in
the New Testament. Added to this the incredible redundancy of copies, and their almost 100 percent
agreement, and we can easily conclude that the transmission of the documents to us has been
extremely reliable. If anyone were to dismiss the New Testament documents by saying that they are
corrupted, then he must also throw out all other ancient documents including those of the writings of
Plato, Socrates, Aristotle, and many others because none of them come close to approaching either
the number of manuscripts in existence, their reliability, or the accuracy of the copies that exists in the
New Testament documents.
Second, since the documents are reliable then we must look at them to determine whether or not
the content supports the resurrection of Jesus. At this point, it becomes very easy to demonstrate this
since the Bible definitely teaches that Jesus rose from the dead. There are many scriptures that teach
Jesus' resurrection. Following are three of them.

• Luke 24:56, "and as the women were terrified and bowed their faces to the ground, the men
said to them, "Why do you seek the living One among the dead? 6 "He is not here, but He has
risen..."
• John 2:19-21, "Jesus answered and said to them, "Destroy this temple, and in three days I will
raise it up." 20The Jews therefore said, "It took forty-six years to build this temple, and will You
raise it up in three days?" 21But He was speaking of the temple of His body. 22When therefore
He was raised from the dead, His disciples remembered that He said this; and they believed the
Scripture, and the word which Jesus had spoken."
• 1 Cor. 15:3-4, "For I delivered to you as of first importance what I also received, that Christ
died for our sins according to the Scriptures, 4and that He was buried, and that He was raised
on the third day according to the Scriptures."

Did Jesus rise from the dead? According to the Bible, yes He did.
Didn't Jesus simply rise in a non-physical, spirit form?
Some people say that Jesus' resurrection was not physical, but a spirit form. Groups like the
Jehovah's Witnesses hold to this. But, the Bible teaches that the resurrection of Jesus was physical,
not simply spiritual. In John 2:19-21, Jesus said, "Destroy this temple and in three days I will raise it
up." The Jews therefore said, 'It took forty-six years to build this temple, and will you raise it up in
three days?' But He was speaking of the temple of his body." Notice that Jesus was speaking of His
body. The apostle John then comments that Jesus was prophesying that His physical body would be
raised from the dead. Is this what happened?
After Jesus' resurrection He appeared to Thomas. "Then He said to Thomas, 'Reach here your
finger, and see My hands; and reach here your hand, and put it into My side; and be not unbelieving,
but believing,'" (John 20:25). Notice that Jesus still retained the hole in His side where he was pierced
during the crucifixion: "But one of the soldiers pierced his side with a spear, and immediately there
came out blood and water" (John 19:34). This clearly shows that Jesus retained the wounds in His
body. The only way this is possible is if He was raised from the dead physically.
But some people refer to 1 Cor. 15:44 that says, "it is sown a natural body, it is raised a spiritual
body. If there is a natural body, there is also a spiritual body." They conclude that Jesus was raised in
a spiritual form, not a physical one. But, the truth is that He was raised physically. His body was a
glorified body. It was the same body, but it was slightly different. That is, His body was raised
physically, but it had been glorified. It had been changed. If we look at the context of the verse we
see that Paul is repeatedly stating that the thing that is sown, is also the thing that is raised. "So also
is the resurrection of the dead. It is sown a perishable body, it is raised an imperishable body; it is
sown in dishonor, it is raised in glory; it is sown in weakness, it is raised in power; it is sown a natural
body, it is raised a spiritual body. If there is a natural body, there is also a spiritual body," (1 Cor.
15:42-44).

Flesh and blood cannot inherit the kingdom of God

The Bible says that flesh and blood cannot inherit the kingdom of God ( 1 Cor. 15:50). If this is so,
then how could a physical body have been raised? The answer is simple. After His resurrection Jesus
said, "Touch me and see, for a spirit does not have flesh and bones as you see I have" (Luke 24:39).
You must note that Jesus did not say, "flesh and blood." He said, "flesh and bones." This is because
Jesus’ blood was shed on the cross. The life is in the blood and it is the blood that cleanses from sin:
"For the life of the flesh is in the blood, and I have given it to you upon the altar to make atonement
for your souls; for it is the blood that makes atonement for the soul," (Lev. 17:11). See also, Gen.
9:4; Deut. 12:23; and John 6:53-54. Jesus was pointing out that He was different. He had a body, but
not a body of flesh and blood. It was flesh and bones. I am of the opinion that Jesus' body had no
functioning blood in it. Remember, after the resurrection He still retained the wounds in His hands,
feet, and side. But, His blood was the thing that cleanses us of our sins: "but if we walk in the light
as He Himself is in the light, we have fellowship with one another, and the blood of Jesus His Son
cleanses us from all sin," (1 John 1:7). His body was raised, but it had no blood flowing through its
veins. It was a glorified, physical body.
This explains why Paul said in Col. 2:9, "For in Him all the fullness of Deity dwells in bodily form."
And also, 1 Tim. 2:5 that says, "For there is one God, and one mediator also between God and men,
the man Christ Jesus." In these two verses, written after Jesus' resurrection and ascension, Jesus is
said to be in bodily form and also to be a man. How could he be in bodily form and be a man if He
does not have a body of flesh and bones?
Therefore, Jesus rose from the dead in a physical body.
If Jesus is God in flesh, why did He not inherit original sin?

If all people have original sin and Jesus was a human being, then didn't Jesus need to have had a
sin nature?
Before we can answer this question, we need to know what the term "original sin" means. This is a
term used to describe the effect of Adam's sin on his descendants (Rom. 5:12-32). Specifically, it is
our inheritance of a sinful nature from Adam. The sinful nature originated with Adam and is passed
down from parent to child. We are by nature children of wrath (Eph. 3:2). So, if we inherit our sinful
nature from our parents, then Jesus, who had Mary as a parent, must have had a sin nature. Right?
Not necessarily. I believe that the sin nature is passed down through the father. Let me explain.
Some Bible commentators, with whom I agree, hold the position that the sin nature is passed down
through the father. Support for this position is found in the fact that sin entered the world through
Adam, not Eve. Remember, Eve was the one who sinned first. However, sin did not enter the world
through her. It entered through Adam. Rom. 5:12 says, "Therefore, just as through one man sin
entered into the world, and death through sin, and so death spread to all men, because all sinned."
The concept behind this is called Federal Headship. This means that a person (a father) represents
his descendants. We see this concept taught in Heb. 7:9-10, "And, so to speak, through Abraham
even Levi, who received tithes, paid tithes, 10for he was still in the loins of his father when Melchizedek
met him." We see in Hebrews that Levi, a distant descendant of Abraham, is said to have paid tithes
to Melchizedek when Abraham was the one offering the tithes, not Levi. What this means is that there
is biblical support for the idea that the sin nature was passed down through the father. Since Jesus
had no literal, biological father, the sin nature was not passed down to Him. However, since He had a
human mother, he was fully human but without original sin. Jesus has two natures: God and man.
Col. 2:9 says, "For in Him dwells all the fullness of deity in bodily form." Jesus received His human
nature from Mary, but He received His divine nature through God the Holy Spirit. Therefore, Jesus is
both God and man. He was sinless, had no original sin, and was both fully God and fully man.
Can't all Jesus' miracles be explained naturally?
Can Jesus' miracles be explained naturally? It is certainly possible that some might be explained
with non-divine answers, but can all of them? I don't see how.
It could be said that the gospel accounts were simply altered to make it look like Jesus was
performing miracles which never happened. This is a possibility, but it is not very probable. Briefly,
the eyewitnesses of Jesus' miracles were still around when the gospels were written and could have
easily refuted such claims. Yet, we have no record of any such refutations. The disciples died for what
they believed. Remember, this is not simply dying for a principle(s) or philosophy like Buddhism.
They died for their belief in the risen Lord Jesus who claimed to be God and performed miracles in
front of their very eyes. This is far different than believing in something that wasn't tangible or was
merely a belief for the sake of believing and being good. Therefore, I will not address the idea that the
disciples were deceivers in their attempt to recount Christ's work. For further reading on this, please
see Can we trust the New Testament as a historical document? and Since the NT writers were biased,
can we trust what they wrote?
In the following outline, I have stated a miracle, then offered a potential explanation, and then
rebutted the explanation.

1. Jesus was born of a virgin (Matt. 1:25).


A. It could be said that Jesus was born normally, and myth crept into the story of Christ's
birth in order to make Him seem special. After all, how do you verify a virgin birth?
i. But, Mary, the mother of Jesus was probably still around when the gospels were
written. As was James, Jesus' brother. If the gospel accounts of Jesus' virgin
birth were fictitious, certainly those who "knew" the situation would have refuted
it. Yet, we have no account of any such refutation.
2. Jesus changed water into wine (John 2:6-10).
A. It is possible that Jesus switched the water for wine or had some help in doing it.
i. There is no indication in the account of John that would lead anyone to believe
that this was the case. Given that the six jars of wine were very heavy
(minimum of 160 pounds each, maximum 240 pounds each), Jesus would have
had help to do this. But, if that is so, who was it and why? Did Jesus secretly
arrange for a large supply of wine to be delivered to a party long after it had
begun? Again, there is no evidence of this.
B. It is possible that the members of the party were simply mistaken about the wine
running out.
i. This is possible, but we have the wine steward tasting the wine and commenting
on how good it is. His speech displays clarity of thought so he was not drunk.
Not being drunk, he was easily able to recognize the quality of the new wine.
Therefore, it is very unlikely that this was a mistake regarding the water for wine.
3. Jesus caused the disciples to catch a large load of fish (Luke 5:4-6).
A. The only explanation I can come up with to account for the time when Jesus instructed
the disciples to cast their net into the water and they caught a large amount of fish even
though they had been fishing all night long and caught nothing is that from shore, Jesus
was somehow able to see into the water and see the fish swimming there.
i. The problem with this is that Jesus told them to go out into the deep water.
Deep water is far from shore and it is basically impossible for Him to have seen
so far out into the water at any depth. Remember, many of the disciples had
been life long fisherman and they knew how to catch fish. If all it took was to
look into the water to see fish, they would have long ago used that method.
4. Jesus cast out demons (Matt. 8:28-32; 15:22-28).
A. It is possible that demons were never cast out of anyone. It is possible that the people
were pretending to be sick and then appeared cured after Jesus did whatever He did to
cure them.
i. Though this is possible, it is merely conjecture. It isn't as though this is a
repeatable experiment we can do in a laboratory. After all, the existence of
demonic forces is something that must be taken on faith.
5. Jesus healed diseases (Matt. 4:23,24; 8:3 Luke 6:17-19; 17:14).
A. Jesus had knowledge of herbs and roots that when applied to various ailments cured
people. Therefore, it would not be miraculous.
i. This is possible, but where is the evidence of them doing that? How can Jesus
cure leprosy with herbs, or heal a withered hand, or raise the dead using herbs
and roots. Sure, maybe, just maybe some herbs and roots were applied to basic
ailments, but such an explanation cannot account for many of the miracles
accounted to Jesus.
B. Jesus faked the healings
i. I am not sure how it would be possible to fake the healing of leprosy and
disease. The people of the time knew what the disease was and what it looked
like. People's fingers and hands would fall off from leprosy. How it would be
possible to restore hands and feet and fingers and have it all be a trick would be
an amazing thing to do. I cannot think of any way to fake such a thing especially
since so many of the lepers were known by the people around them and cures
would have been obvious.
6. Jesus healed the paralytic (Mark 2:3-12).
A. In order to make it look like Jesus healed a paralytic, it would require that the paralytic
be willing to appear paralyzed in order to fool those around him. This is possible since
Jesus could have had enough time to employ the individual.
i. In the account of Mark, the paralytic is let down through the roof because there
were so many people that they could not bring him in on a stretcher. Jesus then
healed the man. If the paralytic was in the employ of Jesus in some way, the
men who lowered him through the roof must also have been in His employ since
they helped to accomplish the ruse -- if that is what it was. But, simply stating
that this is a possibility does not mean that it is a reality. All the text says is that
there was a paralytic who was let down through a roof and Jesus healed him.
There is no information that would lead us to believe that collusion was occurring.
7. Jesus raised the dead (Matt. 9:25; John 11:43-44).
A. Those who were dead were really only appearing to be dead. Given that the people of
the time were not aware of many of the medical intricacies that can lead to people
looking dead when they were not really dead.
i. This explanation is certainly possible, but is it really that likely? People in ancient
times were far more familiar with death than we are. Our people die in hospitals
away from the families. Undoubtedly, we are less familiar with death then they
were. It is more probable that they knew when people were actually dead,
especially since the dead were left in stasis for several days as they were washed
and prepared for burial.
ii. The John 11 account of Lazarus' resurrection
8. Jesus restored sight to the blind (Matt. 9:27-30; John 9:1-7).
A. The blind were not really blind but were working with Jesus in order to make it look like
He was able to perform miracles.
i. This is a possibility, but it has no basis or evidence. Furthermore, how can
anyone account for the man born blind in John 9:1-7. He was known, from birth,
to be blind and yet he was healed by Jesus. How can anyone account for this
other than to say that the man was healed?
9. Jesus cured deafness (Mark 7:32-35).
A. The deaf person was not really deaf. It was a trick, a previously arranged setup to make
Jesus look good.
i. If this is the case, where is the evidence? Simply saying this is what happened
doesn't make it so.
10. Jesus fed the multitude (Matt. 14:15-21; 15:32-38).
A. The disciples had previously arranged a large stash of food sufficient to feed a great
many people.
i. This is a possibility but we see no evidence of it. Also, it means that the account
is, basically, a lie which doesn't fit the character of Jesus and the disciples who
wrote so much about integrity.
B. The people had already brought their own food and were sharing it with each other at
the urging of Jesus so it was written to make it look like He'd done a miracle.
i. This, of course, has no evidence for it either. The account simply states that
Jesus fed the multitude with just a few fish and some bread. What would be
wrong with simply writing the truth, if it were true, that everyone had brought
food? Besides, that isn't what it says.
11. Jesus walked on water (Matt. 14:22-24).
A. There was either a ledge Jesus was walking on near shore or He was in a low profile
boat in which He was standing. This way it only appeared that He was walking on
water.
i. Matt. 14:24 says that the boat was many stadia away. A stadia is about 600
feet. So, they were way out on the lake when the storm hit. How could Jesus
have gotten out into the middle of the lake during a storm and manage to find a
ledge to stand on that happened to be close to the disciples' boat? It is
extremely unlikely.
ii. If Jesus was in a low profile boat out in the middle of a lake during a storm, it
would have sunk long before He got out to them. So, this wouldn't work as an
explanation.
12. Jesus calmed a storm with a command (Matt. 8:22-27; Mark 4:39).
A. It was merely a coincidence. Jesus grew up around the area and knew when storms
were coming and going. He simply knew what to look for, waited, and then commanded
the storm to be quiet at the right moment.
i. If Jesus, who was a carpenter, knew when storms were coming and going, then
why didn't the disciples who also grew up in the area and who were fishermen
also know this? If they did, then they would have been very unimpressed by
Jesus' command. In fact, they would have thought He was pretending to be able
to command the storm to stop when in reality He couldn't. This would cause
them to doubt Him, not to believe in Him more as the account suggests.
13. Jesus rose from the dead (Luke 24:39; John 20:27).
A. The disciples stole Jesus' body and lied about His resurrection.
i. This is unlikely since the guards were there in front of the tomb. Also, the
disciples later died for their belief in the risen Lord. Add to this the various
persecutions they received during their lives and it doesn't make sense that they
endured so much pain and suffering for what they knew was a lie.
ii. Also, what about the apostle Paul? He claims to have seen the risen Lord as
well. Was he, a heavy persecutor of the church, conned by the disciples into
joining with them, losing his place in Jewish culture and society, also suffering
persecution and martyrdom all for what he knew was a lie as well? It makes no
sense.
B. Jesus never died in the first place.
i. This is sometimes called the swoon theory that states that Jesus almost died.
But it does not fully consider the severe trauma that Jesus had undergone before
He got to the cross, let alone the actual crucifixion itself which was incredibly
painful. Also, the Romans were experts at killing by crucifixion. The evidence of
the water and blood coming out of Jesus' side after being pierced is evidence
enough that Jesus had died since that is a sign of blood flow having stopped.
14. Jesus appeared to disciples after resurrection (John 20:19).
A. This was because Jesus had never died. He almost died.
i. This is sometimes called the swoon theory that states that Jesus almost died.
But it does not fully consider the severe trauma that Jesus had undergone before
He got to the cross, let alone the actual crucifixion itself which was incredibly
painful. Also, the Romans were experts at killing by crucifixion. The evidence of
the water and blood coming out of Jesus' side after being pierced is evidence
enough that Jesus had died since that is a sign of blood flow having stopped.
B. Someone else who looked like Jesus died in His place.
i. This is an unsubstantiated and completely fictional fabrication. There is no
evidence of this at all. Besides, the Jews and Romans knew exactly who Jesus
was, along with the disciples. They'd know if a "fake" was taking Jesus' place.
C. The disciples lied. Jesus never appeared to them.
i. This has been answered here. Since the NT writers were biased, can we trust
what they wrote?
15. Jesus ascended into heaven (Acts 1:9).
A. Only the disciples saw this. Therefore, they fabricated the ascension.
i. It is possible that they lied, but then we are still stuck with explaining why they
would lie, why they would continue in the lie, why they would preach and teach
honesty and truth based upon a lie, why they would suffer persecution for a lie,
and why they would die for a lie. It just doesn't make sense.
Are the New Testament themes found in the Old Testament?
The concepts in the New Testament were not derived out of thin air. Amos 3:7 says, "Surely the
Lord God does nothing unless He reveals His secret counsel to His servants the prophets." What is
mentioned in the New Testament is revealed in the Old Testament either clearly or in types and
figures. Gen. 22 is a great example of the sacrifice of Jesus, the Son in Typology represented by the
sacrifice of Isaac.
Some critics of Christianity state that Christianity borrowed its concepts from pagan sources like
Mitrha, Osiris, Apollonius, etc. Admittedly, there are similarities in some pagan religions with
Christianity, but that does not mean Christian writers borrowed from them any more than similarities
between Communism and Democracy mean one is from another. Similarities abound in many
religions. Hinduism has moral statements similar to Christianity as does Taoism. But they are
unrelated to each other.
There are, however, several reasons working against the idea that the people who wrote the New
Testament copied ideas from pagan myths. First of all, the writers of the New Testament were Jews.
As Jews they would have nothing to do with paganism in any form. They knew specifically that Jesus
fulfilled the Old Testament which is why they followed Him. Second, the Old Testament has almost all
the New Testament themes from which a devout Jew would refer when writing the New Testament.
Third, there is no proof at all that the New Testament writers borrowed from pagan sources and
incorporated them into the New Testament. It is up to the critics to supply reasonable evidence for
this if they want to hold the position. Just saying it happened doesn't mean anything. Fourth, so what
if there are similarities? What does it prove? If two writers in the same city both write similar articles
about the President of the U.S., does it mean one used another's concepts? Not at all. Similarities
happen all the time when dealing with similar subjects. Besides, it makes sense that common themes
would be around an area at the same time in history when all nations served various gods.
Undoubtedly, some similarities will occur, but that doesn't mean one was borrowed from another.
Finally, there is another possibility worth examining. The concepts of redemption, the incarnation,
resurrection, etc., are prophesied in the Old Testament and these documents were around for
hundreds and hundreds of years. It is quite possible that if any borrowing was done, it was done by
the pagans who incorporated Old Testament concepts since these documents existed prior to many of
these pagan myths.
Nevertheless, following is a chart that exemplifies many of the themes that were revealed in the
Old Testament and fulfilled in the New. It is easy to see that there is no need at all for the Christians
to borrow from any source outside the Old Testament.

John 5:39, "You search the Scriptures, because you think that in them you have eternal life;
and it is these that bear witness of Me."
New Testament themes found in the Old Testament

Old Testament New Testament


Theme
Reference fulfilled in Jesus
Ascension of Jesus to
Ps. 110:1 Matt 26:64; Acts 7:55-60; Eph. 1:20
the right hand of God
Atonement by blood Lev. 17:11 Heb. 9:22
Exodus 40:12-15; Lev. 16:4; Matt. 3:16; 28:19; Col. 2:11-12; Heb.
Baptism
Gen. 17:10; Ezek. 36:25 10:22
Begotten Son, Jesus Psalm 2:7 Acts 13:33; Heb. 1:5
Creative work Gen. 1; 1:26 John 1:1-3; Col. 1:16-17
Crucifixion Psalm 22:11-18; Zech. 12:10 Luke 23:33-38
Damnation and
Dan. 12:2 Matt. 25:46
Salvation
Eternal Son Micah 5:1-2; Psalm 2:7 Heb. 1:5; 5:5
First and Last Isaiah 41:4; 44:6; 48:12 Rev. 1:8,17; 22:13
John 1:1,14; 20:28; Col. 2:9; Matt.
God among His people Isaiah 9:6; 40:3
3:3
1)Ex 3:14; 2)Ps. 45:6 Isaiah 1)John 8:58; 1:1,14; 2)Heb. 1:8; Col.
Incarnation of God
9:6; Zech. 12:10 2:9; Heb. 1:1-3
Monotheism Isaiah 43:10; 44:6,8; 45:5 John 10:30; Eph. 4:5
Only Begotten Son Gen. 22:2. See Typology John 3:16; Heb. 11:7
Priesthood of Jesus Psalm 110:4 Heb. 6:20; 7:25
Resurrection of Christ Psalm 16:9-10; 49:15; Is. 26:19 John 2:19-21
Return of Christ Zech. 14:1-5; Mic. 1:3-4 Matt. 16:27-28; Acts 1:11; 3:20
Sacrifice of the Son Gen. 22. See Typology Heb. 9:27
Salvation by grace 1)Gen. 12:3; 2)15:6; Hab. 2:4 1)Gal. 3:8-11; 2)Rom. 4:9
Sin offering Ex. 30:10; Lev. 4:3 Rom. 8:3; Heb. 10:18; 13:11
Sin offering made
Ex. 29:14 Heb. 13:12-13
outside the camp
Sin offering without Ex. 12:5; Lev. 22:20; Deut.
Heb. 9:14
defect 17:1
Son of God Psalm 2:7 John 5:18
Substitutionary Matt. 20:28; 1 Pet. 2:24; 2 Cor. 5:21;
Isaiah 53:6-12; Lev. 6:4-10,21
Atonement 1 Pet. 3:18;
1)Gen. 1:1,26; Job 33:4; 2)
Gen. 17:1; 18:1; Ex. 6:2-3;
1)John 1:1-3; 2)John 1:18; 6:46;
Trinity 24:9-11; 33:20; Num. 12:6-8;
3)Matt. 28:19; 2 Cor. 13:14
Psalm 104:30; 23)Gen. 19:24
with Amos 4:10-11; Is.48:16
Virgin Birth Isaiah 7:14 Matt. 1:25
Matt. 2:2,11; 14:33; 28:9; John 9:35-
Worship of Jesus Psalm 97:7
38; Heb. 1:6
Apollonius of Tyana also did miracles and rose. What about him?

Apollonius of Tyana (a city south of Turkey) is sometimes offered as a challenge to the uniqueness
of Jesus Christ. It is said that Apollonius, who lived in the first century, also performed miracles, had
disciples, died, and appeared after his death the same as Jesus. Therefore, critics conclude, what
Jesus did isn't unique. Some even say that this is evidence that the Christian account of Christ's
healings, miracles, and post death appearances were merely copied from the accounts of Apollonius.
Are these accusations supportable? No, they aren't.
First of all, the accounts of Apollonius were written well after he is supposed to have lived by a man
named Philostratus (170 - 245 A.D.). This is long after the New Testament was written. Therefore the
written accounts of Apollonius were not written by eyewitnesses as were the gospels. If critics want to
maintain that the New Testament is full of myth and must be discredited, then so must the accounts of
Apollonius since the writings are written several generations after the fact. By contrast the New
Testament was written by the eyewitnesses of Jesus' life. Logically, it is the New Testament accounts
that are far more reliable than those of Apollonius. Also, this would mean that if any borrowing was
done, it was done by Philostratus, not by the gospel writers.
Second, the eyewitness accounts of the New Testament writers were written before the close of the
first century. For example, we know that Matthew, Mark, Luke, and Acts do not contain the account of
the fall of Jerusalem which occurred in 70 A.D. This fall included the destruction of the Jerusalem
temple which was prophesied by Jesus in Matt. 24:1, Mark 13:1, and Luke 21:5. Such an incredibly
major event in Jewish history would surely have been included in Acts and the synoptic gospels
(Matthew, Mark, and Luke) if they were written after 70 A.D. since they would verify Jesus' predictive
abilities. But, it is not included. Therefore, it is safe to say that they were written by the eyewitnesses
of Jesus' life, unlike the accounts of Apollonius.
Third, Philostratus is the only source for the accounts of Apollonius where the Bible is multi-
sourced. In other words, we have different writers writing about Jesus. Matthew, Mark, Luke, John,
Paul, etc., are different writers who's epistles were gathered by the Church and assembled into the
Bible. That means that there is no verification for Apollonius other than the single writing of
Philostratus.
Fourth, Philostratus was commissioned by an empress to write a biography of Apollonius in order to
dedicate a temple to him. This means that there was a motive for Philostratus to embellish the
accounts in order satisfy the requirement of the empress. 66
It is not likely in the slightest that the gospels borrowed from Apollonius. It is most probably the
other way around, especially since Philostratus had a motive to satisfy the empress who had
commissioned him to write a biography of the man for whom a temple had been constructed.

66
Strobel, Lee, The Case for Christ; Zondervan, Grand Rapids, Michigan, 1998, p. 120.
Doesn't the religion of Mithra prove that Christianity is false?
Some critics of Christianity teach that the Christian religion was not based upon divine revelation
but that it borrowed from pagan sources, Mithra being one of them. They assert that the figure of
Mithra has many commonalities with Jesus, too common to be coincidence.
Mithraism was one of the major religions of the Roman Empire which was derived from the ancient
Persian god of light and wisdom. The cult of Mithraism was quite prominent in ancient Rome,
especially among the military. Mithra was the god of war, battle, justice, faith, and contract.
According to Mithraism, Mithra was called the son of God, was born of a virgin, had disciples, was
crucified, rose from the dead on the third day, atoned for the sins of mankind, and returned to
heaven. Therefore, the critics maintain that Christianity borrowed its concepts from the Mithra cult.
But is this the case? Can it be demonstrated that Christianity borrowed from the cult of Mithra as it
developed its theology?
First of all, Christianity does not need any outside influence to derive any of its doctrines. All the
doctrines of Christianity exists in the Old Testament where we can see the prophetic teachings of Jesus
as the son of God (Zech. 12:10), born of a virgin (Isaiah 7:14), was crucified (Psalm 22), the blood
atonement (Lev. 17:11), rose from the dead (Psalm 16:10), and salvation by faith (Hab. 2:4). Also,
the writers of the gospels were eyewitnesses (or directed by eyewitnesses as were Mark and Luke)
who accurately represented the life of Christ. So, what they did was write what Jesus taught as well
as record the events of His life, death, and resurrection. In other words, they recorded history, actual
events and had no need of fabrication or borrowing.
There will undoubtedly be similarities in religious themes given the agrarian culture. Remember,
an agriculturally based society, as was the people of the ancient Mediterranean area, will undoubtedly
develop theological themes based upon observable events, i.e., the life, death, and seeming
resurrection of life found in crops, in cattle, and in human life. It would only be natural for similar
themes to unfold since they are observed in nature and since people created gods related to nature.
But, any reading of the Old Testament results in observing the intrusion of God into Jewish history as
is recorded in miracles and prophetic utterances. Add to that the incredible archaeological evidence
verifying Old Testament cities and events and you have a document based on historical fact instead of
mythical fabrication. Furthermore, it is from these Old Testament writings that the New Testament
themes were developed.

Old Testament New Testament


Theme
Reference fulfilled in Jesus
Ascension of Jesus to
Ps. 110:1 Matt 26:64; Acts 7:55-60; Eph. 1:20
the right hand of God
Atonement by blood Lev. 17:11 Heb. 9:22
Begotten Son, Jesus is Psalm 2:7 Acts 13:33; Heb. 1:5
Crucifixion Psalm 22:11-18; Zech. 12:10 Luke 23:33-38
Eternal Son Micah 5:1-2; Psalm 2:7 Heb. 1:5; 5:5
God among His people Isaiah 9:6; 40:3 John 1:1,14; 20:28; Col. 2:9;
Incarnation of God 1)Ex 3:14; 2)Ps. 45:6 Isa 9:6 1)John 8:58; 1:1,14; 2)Heb. 1:1-8
Only Begotten Son Gen. 22:2. See Typology John 3:16; Heb. 11:7
Psalm 16:9-10; 49:15; Is.
Resurrection of Christ John 2:19-21
26:19
Sin offering Ex. 30:10; Lev. 4:3 Rom. 8:3; Heb. 10:18; 13:11
Son of God Psalm 2:7 John 5:18
Substitutionary Isaiah 53:6-12; Lev. 6:4- Matt. 20:28; 1 Pet. 2:24; 2 Cor. 5:21;
Atonement 10,21 1 Pet. 3:18;
Virgin Birth Isaiah 7:14 Matt. 1:25
As you can see, there is no need for any of the Christian writers to borrow from anything other
than the Old Testament source in order to establish any Christian doctrine concerning Jesus. If the
argument that pagan mythologies predated Christian teachings and therefore Christianity borrowed
from them is true, then it must also be truth that the pagan religions borrowed from the Jewish
religion because it is older than they are! Given that all of the Christian themes are found in the Old
Testament and the Old Testament was begun around 2000 B.C. and completed around 400 B.C., we
can then conclude that these pagan religions actually borrowed from Jewish ideas found in the Old
Testament. Think about it, the idea of a blood sacrifice and a covering for sin is found in the first three
chapters of Genesis when God covered Adam and Eve with animal skins and prophesied the coming of
the Messiah.
Furthermore, those who wrote about Jesus in the New Testament were Jews (or under the
instruction of Jews) who were devoted to the legitimacy and inspiration of the Old Testament
scriptures and possessed a strong disdain for pagan religions. It would have been blasphemous for
them to incorporate pagan sources into what they saw as the fulfillment of the sacred Old Testament
scriptures concerning the Messiah. Also, since they were writing about Jesus, they were writing based
upon what He taught: truth, love, honesty, integrity, etc. Why then would they lie and make up
stories and suffer great persecution, hardships, ridicule, arrest, beatings, and death all for known lies
and fabrications from paganism? It doesn't make sense.
At best, Mithraism only had some common themes with Christianity (and Judaism) which were
recorded in both the Old and New Testaments. What is far more probable is that as Mithraism
developed, it started to adopt Christian concepts.

"Allegations of an early Christian dependence on Mithraism have been rejected on many


grounds. Mithraism had no concept of the death and resurrection of its god and no place for
any concept of rebirth -- at least during its early stages...During the early stages of the cult,
the notion of rebirth would have been foreign to its basic outlook...Moreover, Mithraism was
basically a military cult. Therefore, one must be skeptical about suggestions that it appealed to
nonmilitary people like the early Christians." 67

What is more probable is that with the explosive nature of the Christian church in the 1st and 2nd
century, other cult groups started to adapt themselves to take advantage of some of the teachings
found in Christianity.

"While there are several sources that suggest that Mithraism included a notion of rebirth, they
are all post-Christian. The earliest...dates from the end of the second century A.D." 68

Therefore, even though there are similarities between Christianity and Mithraism, it is up to the
critics to prove that one borrowed from the other. But, considering that the writers of the New
Testament was written by Jews who shunned pagan philosophies and that the Old Testament has all of
the themes found in Christianity, it is far more probable that if any borrowing was done, it was done
by the pagan religions that wanted to emulate the success of Christianity.

67
R. Nash, Christianity and the Hellenistic World" as quoted in Baker's Encyclopedia of Christian Apologetics,
Norman Geisler; Baker Books, Grand Rapids, Mich.; 1999, p. 492.
68
Wilson, Bill, compiled by; The Best of Josh McDowell: A Ready Defense; Nashville, Tenn., Thomas Nelson
Publishers; 1993, p. 167.
Why believe in Christianity over all other religions?

Critics often ask why Christianity is any better than any other religion in the world. After all, of
all the religions that exist how can it be that only Christianity is true? If God exists, why can't God use
different religions? Don't all paths lead to God? These kinds of questions are asked all the time of
Christians and unfortunately the answers aren't always very good ones particularly when dealing with
people who have a relativistic truth base and don't believe in absolutes.
Therefore, in an attempt to demonstrate why Christianity is true and all other religious systems are
false, I've prepared the following list of reasons for Christianity's superiority.

There are such things as absolute truths

If truth is relative, then the statement that truth is relative is an absolute truth and would be self
defeating statement by proving that truth is not relative. But, if truth is absolute, then the statement
"truth is absolute" is true and not self defeating. It is true that truth exists. It is true that truth will
not contradict itself as we have just seen. In fact, it is absolutely true that you are reading this paper.
If we can see that there is such a thing as truth in the world, then we could also see that there can
be spiritual truth as well. It is not absurd to believe in spiritual absolutes anymore than physical or
logical absolutes. Even the statement that all religions lead to God is a statement held to be a spiritual
absolute by many people. This simply demonstrates that people do believe in spiritual truth. Why?
Because truth exists. However, not all that is believed to be true actually is true. Therefore, all belief
systems cannot be true since they often contradict each other in profound ways.

Religions contradict each other; therefore, they cannot all be true.

Mormonism teaches that there are many gods in existence and that you can become a god.
Christianity teaches that there is only one God and you cannot become a god. Islam teaches that
Jesus is not God in flesh where Christianity does. Jesus cannot be both God and not God at the same
time. Some religions teach that we reincarnate while others do not. Some teach there is a hell and
others do not. They cannot all be true. If they cannot all be true, it cannot be true that all religions
lead to God. Furthermore, it means that some religions are, at the very least, false in their claims to
reveal the true God (or gods). Remember, truth does not contradict itself. If God exists, He will not
institute mutually exclusive and contradictory belief systems in an attempt to get people to believe in
Him. God is not the author of confusion (1 Cor. 14:33). Therefore, it is reasonable to believe that
there can be an absolute spiritual truth and that not all systems can be true regardless of whether or
not they claim to be true. There must be more than a mere claim.

Fulfilled Prophecy concerning Jesus

Though there are other religions that have prophecies in them, none are 100% accurate as is the
Bible and none of them point to someone like Jesus who made incredible claims and performed
incredible deeds. The Old Testament was written hundreds of years before Jesus was born. Yet, the
Old Testament prophesied many things about Jesus. This is undoubtedly evidence of divine influence
upon the Bible.
Please consider some of the many prophecies of Jesus in the following chart.
Old Testament New Testament
Prophecy
Prophecy Fulfillment
Born of a virgin Isaiah 7:14 Matt. 1:18,25
Born at Bethlehem Micah 5:2 Matt. 2:1
He would be preceded by a Messenger Isaiah 40:3 Matt. 3:1-2
Rejected by His own people Isaiah 53:3 John 7:5; 7:48
Betrayed by a close friend Isaiah 41:9 John 13:26-30
His side pierced Zech. 12:10 John 19:34
Psalm 22:1, Luke 23:33;
Crucifixion
Psalm 22:11-18 John 19:23-24
Resurrection of Christ Psalm 16:10 Acts 13:34-37

Fulfillment of prophecy can have different explanations. Some state that the NT was written and
altered to make it look like Jesus fulfilled OT prophecy (but there is no evidence of that). Others state
that the prophecies are so vague that they don't count (but many of the prophecies are not vague at
all). Of course, it is possible that God inspired the writers and Jesus, who is God in flesh, fulfilled
these prophecies as a further demonstration of the validity of Christianity.

The Claims and Deeds of Christ

Christianity claims to be authored by God. Of course, merely making such a claim does not make it
true. Anyone can make claims. But, backing up those claims is entirely different. Jesus used the
Divine Name for Himself (John 8:58), the same Divine Name used by God when Moses asked God
what His name was in (Exodus 3:14). Jesus said that He could do whatever He saw God the Father do
(John 5:19), and He claimed to be one with God the Father (John 10:30; 10:38). Likewise, the
disciples also called Him God (John 1:1,14; John 10:27; Col. 2:9). By default, if Jesus is God in flesh,
then whatever He said and did would be true. Since Jesus said that He alone was the way, the truth,
and the life and that no one can find God without Him (John 14:6), this all becomes incredibly
important.
Again, making a claim is one thing. Backing it up is another. Did Jesus also back up His words
with His deeds? Yes, He did.

• Jesus changed water into wine (John 2:6-10).


• Jesus cast out demons (Matt. 8:28-32; 15:22-28).
• Jesus healed lepers (Matt. 8:3; Luke 17:14).
• Jesus healed diseases (Matt. 4:23,24; Luke 6:17-19)
• Jesus healed the paralytic (Mark 2:3-12).
• Jesus raised the dead (Matt. 9:25; John 11:43-44).
• Jesus restored sight to the blind (Matt. 9:27-30; John 9:1-7).
• Jesus restored cured deafness (Mark 7:32-35).
• Jesus fed the multitude (Matt. 14:15-21; Matt. 15:32-38).
• Jesus walked on water (Matt. 8:26-27).
• Jesus calmed a storm with a command (Matt. 8:22-27; Mark 4:39).
• Jesus rose from the dead (Luke 24:39; John 20:27).
• Jesus appeared to disciples after resurrection (John 20:19).

The eyewitnesses recorded the miracles of Jesus and the gospels have been reliably transmitted to
us. Therefore, we can believe what Jesus said about Himself because Jesus performed many
convincing miracles in front of people who testified and wrote about what they saw Him do.
Christ's resurrection

Within Christianity, the resurrection is vitally important. Without the resurrection, our faith is
useless (1 Cor. 15:14). It was the resurrection that changed the lives of the disciples. After Jesus was
crucified, the disciples ran and hid. But when they saw the risen Lord, they knew that all that Jesus
had said and done proved that He was indeed God in flesh, the Savior.
No other religious leader has died in full view of trained executioners, had a guarded tomb, and
then risen three days later to appear to many people. This resurrection is proof of who Jesus is and
that He did accomplish what He set out to do: provide redemption for mankind.
Buddha did not rise from the dead. Muhammad did not rise from the dead. Confucius did not rise
from the dead. Krishna did not rise from the dead, etc. Only Jesus has physically risen from the dead,
walked on water, claimed to be God, and raised others from the dead. He has conquered death. Why
trust anyone else? Why trust anyone who can be held by physical death when we have a Messiah who
is greater than death itself?

Conclusion

Why should anyone trust in Christianity over Islam, Buddhism, Mormonism, or anything else? It is
because there are absolute truths, because only in Christianity is there accurate fulfilled prophecies of
a coming Messiah. Only in Christianity do we have the extremely accurate transmission of the
eyewitness documents (gospels) so we can trust what was originally written. Only in Christianity do
we have the person of Christ who claimed to be God, performed many miracles to prove His claim of
divinity, who died and rose from the dead, and who said that He alone was the way the truth and the
life. All this adds to the legitimacy and credibility of Christianity above all other religions -- all based
on the person of Jesus. If follows that if it is all true about what Jesus said and did, then all other
religions are false because Jesus said that He alone was the way, the truth, and the life and that no
one comes to the Father except through Him (John 14:6). It could not be that Jesus is the only way
and truth and other religions also be the truth.
Either Jesus is true and all other religions are false or other religions are true and Jesus is false.
There are no other options. I choose to follow the risen Lord.
If God is all powerful and loving, why is there suffering in the world?

It is often asked why is there suffering in the world if God is all powerful and loving. Why doesn't
He stop it? Can He or is He weaker than we think? Suffering can fall into three simple categories:
emotional, mental, and physical suffering. But, there are a variety of causes for suffering: morally
corrupt (evil) people, disease, earthquakes, floods, famine, etc.
There are different explanations for why God allows suffering, but none of them can satisfy
everyone. Therefore, I will simply list various reasons offered to account for suffering and evil in the
world.

1. Free will
A. God has given us freedom of choice. Having this freedom means that we can rebel
against God and make choices that are contrary to His desires. Since we can say that
evil is anything contrary to God's perfect and holy will, then anyone who chooses
anything contrary to God's perfection is committing evil. But this is the risk of being able
to have freedom of choice. Evil and suffering are the result of making bad free choices.
B. But how could this account for natural disasters and sickness that brings suffering?
Biblically, Adam represented not only all of his descendents, but he was also the head of
the created order since he was given dominion over the earth. Therefore, when he fell,
sin entered into the world (Rom. 5:12) and with it the effects of being fallen spread to
the earth as well as to humanity.
2. God cannot stop evil and suffering because He is powerless
A. Of course, this does not stand up to biblical truth. God allows evil to occur partly for
reasons we do know and partly for those we do not. We know that God uses evil to
discipline people (Prov. 3:11) and to teach them (Prov. 15:32). But we cannot know all
the reasons that God has for allowing evil and suffering in the world. It is not logically
necessary that since God has not stopped evil and suffering in the world, that He
cannot. God could be using suffering for His divine plan, in order to teach, for discipline,
because people are free, etc. The existence of suffering does not at all mean that God
cannot stop all of it. It means that He simply has chosen not to do so.
3. How much evil should be stopped?
A. The question of stopping evil means that if God is to stop evil, then He must stop all evil.
This means that the murderer must be stopped along with the thief. But it also means
that thinking evil, which is in rebellion against God, must also be stopped as well; that
is, if all evil is to be stopped. Therefore, for God to stop evil and suffering may very well
mean that He must remove the ability for people to freely choose what they want to do.
So, if God is going to stop evil, is He required to stop all of it or just some of it? If only
some of it, then the question would still stand. If He stops all of it, would we be free?
4. Prevention of further evil
A. It is possible that human suffering (cancer, disease, etc.) can be a means that God uses
to remove the person from further suffering, worse suffering, or future suffering. Of
course, this not seem to be a very good option because if God or intending to stop
further suffering, why would He use suffering to stop it? Also, what about floods and
earthquakes that cause suffering? How would they fit into God decreasing or stopping
suffering except perhaps by people's deaths which ends suffering? This is difficult to
answer. Though it may be that God might use some suffering to prevent even greater
suffering, this explanation cannot answer all issues concerning it.
5. For the greater plan
A. Undoubtedly, God has a plan. Since God knows all things He is not surprised by the
presence of evil and sin in the world that brings about suffering. But if God knows all
things from all eternity, then He is perfectly capable of using suffering in the world in His
greater plan. The best and simplest example of this is the suffering of Christ at the
hands of evil men. It is by Christ's suffering and death on across that we are able to be
redeemed. It was God's plan from all eternity that Christ die for our sins yet Christ was
crucified by evil people (Acts 4:27-28). This means that God had incorporated into His
divine plan the reality of evil and suffering in order to accomplish His will. Of course,
this does not mean that God is the author of evil, but it does mean that God is above it
all and can use it to accomplish a greater good. If this is true on a large-scale, why
cannot it also be true on a smaller one in each of our individual lives?
6. For discipline and instruction
A. The Bible tells us that God disciplines those whom He loves (Heb. 12:6) and that no true
child of God is without discipline and instruction. It is obvious that the result of our
rebellion against God brings suffering and it is also true that we can learn through our
suffering that such rebellion is bad. We then could glorify God during and after our
suffering by proclaiming the truth of His word that urges us to follow God and His ways.
Sometimes we learn our greatest lessons after having suffered the consequences of
our actions -- and this is good. If we see that there are consequences through the acts
of suffering in this world, it is logical to conclude that there will be suffering in the next
as a consequence of our rebellion now. This could easily lead us to conclude that we
need to be delivered from our rebellion against God. Of course, Jesus is the answer to
this.
7. It is the result of sin
A. Biblically speaking, pain and suffering are the results of sin in the world. Adam, who
represented all humanity as well as creation, rebelled against God and brought suffering
into the world (Rom. 5:12). Sin is more than simple rebellion and breaking of God's law.
It is permeating throughout all of God's creation bringing imbalance, famine,
earthquakes, disease, etc. This does not mean that God created evil. Instead, it is God
who is allowing evil and suffering to continue for His divine plan.
8. To serve as a warning
A. Evil and suffering in the world can serve as a warning against breaking God's law and
then people can see the necessity of following God's truth. God's ways are right and
good and following them leads to security and safety. The consequences of disobeying
God's word are manifested in suffering. Therefore, suffering in the world easily serves as
a demonstration of the need to follow God's words thereby vindicating what God has
said
9. To make a point
A. It is possible that God is simply allowing evil and suffering in the world to prove that
rebellion against Him brings pain and suffering. God may be allowing sin to take its
natural course in the world so that on the Day of Judgment God can say "Do you see
what rebellion against my words brings?" This may seem overly simplistic but it may
prove to be one of the reasons that God allows pain and suffering. After all, did He not
make us in His image and give us the freedom to choose? And in our freedom have we
not rebelled? Yes, we have. Should God then make us robots or restrict our freedom so
much that we have no choices at all? Of course not. But since we are limited in our
knowledge and have used our freedom to rebel, God allows us to have what we desire
and in the end, our sins will prove that God's way is the right way.
10. To serve as a means to bring the Son
A. The death of the Son is the means by which God has redeemed those who would receive
Jesus. This death cannot occur if Jesus were not a man. In order to be a man he had to
be born as one. But since Jesus was sinless, death has no power over Him. Therefore, in
order to die and in order to redeem us, His death must be at the hands of evil people.
But, without sin, suffering, and evil in the world, Jesus could not have been sent to the
cross. So, it could be said that suffering in the world is necessary in order to bring about
the cross which in turn demonstrates the great and awesome love of God. Jesus said
that the greatest act of love is to lay one's life down for another (John 15:13). If God is
love (1 John 4:8) and love gives (John 3:16), can it be that God must demonstrate the
greatest act of love? If so, it can only be done through suffering in the world.
11. We don't know.
A. Biblically speaking, pain and suffering are the results of sin in the world. Adam, who
represented all humanity as well as creation, rebelled against God and brought suffering
into the world. This sin is more than simple rebellion and breaking of God's law. It is an
offense against a holy God. Sin is permeating throughout all of God's creation bringing
imbalance, famine, earthquakes, disease, etc. This is not have God created things but it
is God who is allowing them to continue for his divine plan. Ultimately, we can't know
all the reasons why God allows suffering, we just know that He does.

What does the Bible tell us that God has done about evil? It tells us that he sent to his son Jesus to
die for our sins and to deliver us from pain and suffering. Ultimately, God is allowing evil in the world
for a purpose; otherwise, He would not let it exist. Therefore, we must trust Him that He knows what
He is doing.
A loving God would never send anyone to hell

The idea of a loving God sending people to hell for eternity is not easy to accept. Why would God,
who is full of mercy and grace, send people to a place of torment for ever and ever for not trusting in
Jesus even though they are nice people, or never heard of Jesus, or were sincerely trying to find God?
Is that fair? Is that right?
When people ask these questions, they are appealing to what they perceive as fairness. They are
looking at the issue from their human perspective. But this perspective is not necessarily the right
one. If God exists, and He does, then it is He who is the One who says what is right and fair, not us.
So, we need to see what the Bible says about what is right regarding sin and salvation and make a
decision afterwards.
The Bible tells us that God is holy, "You shall be holy, for I am holy," (1 Pet. 1:16). Holiness is
incorruptibility, perfection, purity, and the inability to sin, all of which are possessed by God alone.
Holiness is the very nature of God's character. His character is perfect, without flaw, and He is the
standard of all that is right and good.
The Bible also says that God is infinite, "Great is our Lord, and abundant in strength; His
understanding is infinite," (Psalm 147:5). If God's understanding is infinite, then God is infinite in
nature.
The Bible tells us that God is love. "And we have come to know and have believed the love which
God has for us. God is love, and the one who abides in love abides in God, and God abides in him," (1
John 4:16). God cares about us and seeks our well being and security. His thoughts about us are
infinite and His love is too. This is why God does not desire that anyone go to hell, but that all come
to repentance (2 Pet. 3:9).
The Bible tells us that God is righteous. "God is a righteous judge," (Psalm 7:9). His righteousness
is part of His character just as are mercy and love. Righteousness deals with justice and justice deals
with the Law. This means that God will always do that which is right and He does so according to the
righteous Law that He has set forth. God cannot do anything wrong. God must do that which is right;
otherwise He would not be righteous.
Jesus said that "out of the abundance of the heart, the mouth speaks," (Matt. 12:34), so too with
God. He speaks out of the abundance of His heart. God spoke the universe into existence, "Let their
be light," (Gen. 1:3) and He also spoke forth the Law (Exodus 20 - the Ten Commandments, etc.).
Therefore, the Law of God is a reflection of God's character, because it comes out of what He is, holy,
perfect, righteous, and good. Therefore, the Law is a standard of perfection. It is perfect and if we do
not keep it perfectly, then we have offended the God who gave it; after all, it is a reflection of His
character. To break God's Law is to offend (sin against) God. Since it is law, there is punishment
because there is no Law that is a law without a punishment. This means that when we break the Law
of God, we fall under the judgment of the Law of God. Since He is infinite, our offense against Him is
takes on an infinite quality because we have offended an infinitely holy and righteous God.

Must God punish?

Yes, God must punish those who break His law because it is the right thing to do. Just as a parent
should punish a child for doing something wrong (intentionally), so God must punish those who do
wrong. You see, if God did not punish the person who does wrong, then He would be unjust and
unrighteous. He would be breaking His own law -- which He cannot do. But, someone might say that
the punishment of a parent on a child is temporary whereas God's punishment is eternal. Why the
difference? The answer is two fold. First, God is infinite and a parent is not. Second, God is the
standard of all righteousness and the parent is not.
Because God is infinite, when we sin, we are offending an infinite God. This is incredibly
significant. The reason sin is so bad is not so much because of the one committing the sin, but
because of the One who is offended. In other words, sin is so incredibly bad because it takes on a
horrible quality by the very fact of who it is against: an infinitely pure, holy, and righteous God.
A parent is not the standard of righteousness. God is. A parent is (or should be) using the
righteous standard of God in raising children. Therefore, though a parent's punishment is temporary
because it is instruction and correction, the punishment of God is eternal because our sin is against an
eternal God. There is a big difference.

Can we please God on our own?

Is it possible to earn one's place before God by what we do (being good, etc.)? Is it possible for a
finite being to please an infinite one? If so, then that means a sinner who has offended an infinite
God, is able please God by his efforts. But, if he is a sinner, then aren't those "good" things he does
also touched by sin since they are motivated out of the heart of a sinner? Yes. This is what the Bible
declares since it says that our hearts are deceitful and not to be trusted (Jer. 17:9; Mark 7:21-23).
But then someone might say that if the person is sincere when he does the good works, then that
should be acceptable to God. But, saying it should be acceptable doesn't mean it is. Remember,
according to the Bible we cannot trust our own hearts (Jer. 17:9). This means that we cannot even
trust our own sincerity.
God is the judge, not us. If we could please God by our efforts or sincerity, then it would mean
that a finite person can appease an infinite God by doing good works. It further means that sincerity
becomes a meritorious condition of the heart. It would be like saying, "God, I am worthy to be with
you because of the good works I have done and the good and sincere condition of my heart." Can any
mortal who has fallen into sin ever do anything good enough to please an infinite God? The answer is
no. Gal. 2:21 says, "I do not nullify the grace of God; for if righteousness comes through the Law,
then Christ died needlessly.” In other words, if we could get to heaven by what we do, then Jesus
didn't need to die on the cross. Therefore, God has established that our works and sincerity cannot be
good enough.
Finally, for those who still maintain that we can please God by our efforts, we must ask how many
good works must he perform in order to undo an offense against an infinitely holy God? Is there a
standard by which we can judge which sin requires how many goods works to cancel out? There is
none. Therefore, he is left in a predicament. Since God must punish the sinner for offending Him
(breaking His holy and righteous law), and our works cannot undo the offense against God, then how
are we going to escape so great a righteous judgment?

The way of escape

The only way to escape the righteous judgment of God is to trust in the provision He has made.
This provision is found in Jesus. "For God so loved the world, that He gave His only begotten Son,
that whoever believes in Him should not perish, but have eternal life," (John 3:16). Jesus is the only
way to salvation (John 14:6). Jesus is also God in flesh (John 1:1,14; Col. 2:9). Therefore, Jesus' life
is of infinite value. This means that His sacrifice is sufficient to cleanse you of your sins. It is capable
of satisfying the infinitely righteous standard of God that is required to match His infinite holiness.
Jesus' sacrifice is the only provision acceptable to God the Father. If you want to escape the
eternal judgment of God, you must put your trust in Jesus and what He did on the cross and in nothing
else. Without Him, there is no hope of escape on the Day of Judgment. How do you do this? You
receive Jesus (John 1:12). You trust in Him alone. You can ask Jesus to forgive you of your sins (John
14:14). Trust Him alone.
It is intolerant to say that Christianity is the only true religion.

Yes it is intolerant. In fact, it is very intolerant to say that Jesus is the way the truth and the life
and that no one can get to God except through Him as Jesus Himself said in John 14:6. It is also
intolerant to state that there is no other name under heaven other than Jesus by which a person can
be forgiven of his sins as Peter said in Acts 4:12. It is intolerant to say that there is only one true God
as Jesus said in John 17:3. It is also intolerant to say that trying to enter into heaven by any other
way than Jesus is to be a thief and a robber as Jesus said in John 10:1. Jesus was intolerant when He
said that He is the one who reveals God to people (Luke 10:22). Jesus was even more intolerant of
religious hypocrisy when He condemned the religious know-it-alls and called them hypocrites and
deceivers (Matt. 23:25-26). Jesus was extremely intolerant of the buying and selling in the temple
when He drove the people out of it by force and overturned their money tables (John 2:13-16). Jesus
was intolerant of hatred when He said "love your enemies" (Luke 6:27). Jesus was intolerant of
ignorance when He taught the people truth (Matt. 5). Jesus was intolerant of prejudice when He gave
the parable of the Good Samaritan (Luke 10:30-37).
Yes, Christianity is intolerant because its founder, Jesus, was intolerant. Christianity is intolerant of
false gods and false gospels. Why is it so intolerant, because it is shaped after Jesus. It is intolerant
because there is a hell and Jesus, who is God in flesh (John 1:1,14; Col. 2:9), who died for our sins (1
Pet. 2:24), has made the only way to forgiveness a reality: through Him alone (John 14:6) and
without Him comes damnation.
On the other hand, Christianity is very tolerant. It teaches to be very forgiving (Matt. 18:21-22),
to be patient and kind (Gal. 5:22-23), and to be honest and wholesome (Phil. 4:6-8). Jesus taught us
to love and to heal and to be examples of kindness and truth in the world. Jesus was intolerant of
religious hypocrisy and bigotry. He was very intolerant of false teachers. He was intolerant of pride,
rebellion, sin, covetousness, adultery, lying, cheating, stealing, fornicating, and murder. He was
intolerant of husbands treating their wives poorly. He was intolerant of pain and was saddened by
suffering. Yet, at the same time He demonstrated the greatest love and patience with those who were
guilty of all these things. Why? Because He is God in flesh, incarnate love, incarnate righteousness,
incarnate humility.
The whole issue of whether or not Christianity is intolerant lies in who Jesus is, what He claimed,
and what He did. If what Jesus said and did is true, then Christianity isn't intolerant. It is simply true
and it is the world that is intolerant of that truth.
Likewise, it is true that Jesus lived. It is true that Jesus walked on water (Matt. 8:26-27). It is
true that Jesus healed the sick (Matt. 8:5-13). It is true that Jesus calmed a storm with a command
(Mark 4:39). It is true that Jesus raised the dead (Matt. 9:25; John 11:43-44). It is true that Jesus
claimed to be God (John 5:18; 8:24; 8:58 -- see Exodus 3:14). It is true that Jesus was killed on a
cross (Luke 24:20). It is true that Jesus rose from the dead (Luke 24:39; John 20:27). These are not
feeble claims made by crazy people who wanted to gain power and fame. These are the claims of
Christ Himself and of those who followed Him and suffered for Him and died for Him.
Either it is all true or it is not. Either Jesus performed miracles or He did not. Either Jesus rose
from the dead or He did not. Based solely and completely on who Jesus is and what He did,
Christianity is the truth and by necessity all other religions that disagree with Jesus are wrong.
Truth is, by nature, intolerant of falsehood. If Christianity is not true, then Jesus was not God,
then Jesus did not do miracles, then Jesus did not heal the sick, then Jesus did not walk on water, then
Jesus did not die and rise from the dead after three days. But, if He did do these things, then
Christianity alone is true since in all the religions in the world, only Christianity has the person of Jesus
and Jesus said that He alone was the way, the truth, and the life ( John 14:6).
Christianity is only as intolerant as Jesus is true
Why did animals have to die for the sins of Adam and Eve and others?

In the Old Testament, animal sacrifices were a representation of the true future sacrifice of Jesus.
These Old Testament animal sacrifices were not able to cleanse anyone of their sins (Heb. 10:4). Yet,
they were offered as a predictive representation. In other words, they were a type, a representation,
a picture of the final and real sacrifice that was to occur when Jesus died on the cross bearing our sins
in His body (1 Pet. 2:24).
In the Garden of Eden after Adam and Eve sinned, God covered them with animal skins ( Gen.
3:21). This was the beginning of the animal sacrifice system and it was instituted by God. The
amazing thing is that it is also God who is the one who fulfilled this sacrificial requirement by becoming
one of us (John 1:1,14), bearing our sins in His body on the cross (1 Pet. 2:24), and cleansing us of
our sins (1 John 1:7). All of the Old Testament sacrifices pointed ahead to the real one offered by
Jesus. This way, the Old Testament saints could, by faith, trust in God's provision: "The just shall live
by faith," (Hab. 2:4), and "Then he [Abraham] believed in the Lord; and He reckoned it to him as
righteousness," (Gen. 15:6).
So, it really wasn't that the animals were dying for anyone's sins. They simply were a type of the
true sacrifice made by Christ.
Why would God have to die to save people from Himself?

Why would God have to die to save people from Himself, because there is no other way to save
anyone from their sins.
Because God is holy and righteous, He must punish anyone who breaks His law. Since He is the
one who administers the punishment, we then need to be saved from His righteous judgment. But,
since we are not capable of pleasing God by our mere works, the only one left who can cleanse us is
God Himself.
The one offended is the one who must pardon. If I offend you, I do not ask forgiveness from your
neighbor. I have to ask you. The same goes with God. If we sin against Him we have to go to Him to
be forgiven. But, simply asking for forgiveness isn't enough. The reason is because when we sin, we
sin against a holy and infinite God. But, God is also righteous and He must do what is right.
Therefore, it is right to punish those who defy Him. But the problem is that since God is infinite and
we are not, we cannot do enough to please an infinite God.

An analogy

Let's say I am at your house or apartment with my wife. We are talking about church and in my
zeal I accidentally knock over your lamp. Now, this lamp is special. A dear friend gave it to you and it
has great sentimental value, and besides, you need a light in your room. After a moment or two you
realize that the damage is done and decide to forgive. You say to me, "That is alright, Matt. I forgive
you for breaking the lamp, but give me ten dollars."
Is asking for ten dollars after you've just forgiven me, true forgiveness? Certainly not! When God
forgives our sins, He says He will remember them no more (Jer. 31:34). Forgive and forget are similar
in spelling and similar in meaning. If you forgive me can you demand payment from the one forgiven?
No, because a forgiven debt does not exist.
Let's say that instead of asking me for ten dollars you turn to my wife and say, "Matt broke my
lamp. You give me ten dollars for it."
I ask you again. Is that true forgiveness? No. You are simply transferring the debt to someone
who was not involved in the original offense.
But, we have a problem. The lamp needs to be replaced. In true forgiveness, then, who pays for
its replacement? (Think about this a bit before you go on to read the answer.) Who pays? You do!
You're the only one left. Remember, if you've forgiven me the debt, how can you demand payment?
Now, who was my offense against? You. Who forgives? You do. Who pays? You do.
When we sin, who do we sin against? God. Who forgives? God. Who pays? God! Did you get that?
God pays! How does He do that? Simple. 2000 years ago on a hill outside the city of Jerusalem He
bore our sins in His body and died on the cross (1 Pet. 2:24). He took our punishment: "Surely our
griefs He Himself bore, and our sorrows He carried... He was pierced through for our transgressions,
He was crushed for our iniquities; the chastening for our well-being fell upon Him..." (Isaiah 53:4-5).
God is just. God is merciful. God is gracious. In the justice of God, He took our place. In the mercy
of God we don't get punished. In the grace of God, He gives us eternal life.
Even though we are unworthy of salvation, even though we are unworthy of God's love, even
though we are unworthy of mercy, even though we are worthy of wrath, God saved us. He did so not
because of who we are, but because of who He is, not because of what we do, but because of what He
did. God is love (1 John 4:16). God is holy (1 Peter 1:16). God is good (Psalm 34:8). We could never
fathom the depths of His purity and kindness (Rom. 11:33). We could never, through our own efforts,
attain Him. There is only one thing left for us. We must worship Him, love Him, and serve Him. He
alone is worthy. Blessed be the name of the Lord.
If God is not the author of confusion, what about the Tower of Babel?

This isn't a difficult issue at all. On one hand, God is not the author of confusion: "For God is not a
God of confusion but of peace, as in all the churches of the saints," (1 Cor. 14:33). The context of this
verse is dealing with the gift of tongues as were spoken in Christian churches in its early years.
Foreigners would attend these churches and hear their own languages being spoken. There would
often be interpretations of these tongues. Also, Christians would be over eager in their use of various
tongues and this would often lead to confusion as people did not do things in order. Therefore, in the
immediate verses prior to (1 Cor. 14:33), Paul had just given instruction on the proper use of the
tongues in the church, a use which stated order and sequence. The goal was not to produce confusion
among the hearers so that they would not understand the gospel. Instead, it was to produce an
orderly service of worship.
The context of the Tower of Babel is quite different. The people of the earth were attempting to
build a tower that would "...reach into heaven, and let us make for ourselves a name; lest we be
scattered abroad over the face of the whole earth." (Gen. 11:4). The sin of the people was their great
pride. They were seeking to remain one group in one location under their own efforts. Ultimately,
this was a defiance of God's proclamation to fill the earth (Gen. 9:1). God wanted them to spread
out. "So the Lord scattered them abroad from there over the face of the whole earth; and they
stopped building the city. 9Therefore its name was called Babel, because there the Lord confused the
language of the whole earth; and from there the Lord scattered them abroad over the face of the
whole earth," (Gen. 11:6). Therefore, there is no contradiction since each is a different context and a
different subject.

If babies die when they go to heaven, why is abortion wrong?

There is debate on whether or not all babies go to heaven when they die. But, for the sake of
argument, let's say that all babies go to heaven when they die. If that is so, then why would abortion
be wrong since it would be sending the person to heaven?
The reason abortion is wrong is because it is taking the life of the unborn child who has committed
no wrong. In other words, the child is not being put to death for a sin that it has committed. It is
simply being put to death to make someone else's life more convenient. 1 It is God who gives life and
takes it away. There is an exception though where God makes allowance for capital punishment. "For
it [governing authorities] is a minister of God to you for good. But if you do what is evil, be afraid; for
it does not bear the sword for nothing; for it is a minister of God, an avenger who brings wrath upon
the one who practices evil," (Rom. 13:4). This latter condition is, of course, carried out under due
process of law and is reserved for those who "practice evil," i.e., murderers, rapists, kidnappers, etc.
The unborn do not fall under this category deserving capital punishment. They are simply unborn,
human lives. When people have abortions they are taking into their own hands the taking life that is
not permitted by God.
Whether or not aborted babies go to heaven is not the issue because the ends do not justify the
means. God is the taker of life, not man.
Bible Difficulties
Introduction

There are difficulties in the Bible due to copyist errors and lack of understanding of biblical culture and
context. It is important to be familiar with some of them and to have answers.

1. Are the copies of the original bible documents inspired? p. 202


2. Where did Cain get his wife? p. 205
3. Should you make graven images or not? p. 209
4. Does Isaiah 7:14 support the virgin birth? Why or why not? p. 222
5. Why did God not destroy Nineveh when He said He was going to? p. 225
6. Why are there different genealogies for Jesus in Matthew 1 and Luke 3? p. 226
7. Was John the Baptist really Elijah? p. 238
8. How did Judas die, by hanging or falling down? p. 244
9. Can you be forgiven of all sins or not? p. 247
10. Did Jesus or Simon of Cyrene carry the cross? p. 252
11. What are the last words of Jesus? p. 254
12. Is the ending of Mark really scripture? p. 258
13. Shall we obey God's Law or human law? p. 261
14. Are we saved by faith or by baptism? p. 263
15. Are we saved by grace or works? p. 267
Introduction to Bible Difficulties and Bible Contradictions
Bible difficulties, or apparent Bible contradictions, exist. The opponents of Christianity often use
them in their attempts to discredit Christianity. Sometimes these attacks undermine the faith of
Christians who either don't understand the issues or don't have the resources to deal with them.
Opponents of Christianity will cite what they consider a Bible contradiction or difficulty by
comparing one verse to another (or more) that seems to disagree with the first. In doing this, several
verses are often referenced as being contradictory or problematic. Therefore, to make this section of
CARM easy to use, it is arranged by verse for easy lookup. Since many of the same "difficulties" deal
with one verse in opposition to another or even several others, I have listed all the verses addressed in
the same answer. This makes the initial list look larger than it really is. For example, how many
animals did Noah bring into the ark? Genesis 6:19-20 says two while Gen. 7:2-3 mentions seven.
Therefore, both verses are listed and both links point to the same answer.

The Originals are Inspired, not the copies.

What a lot of Christians don't know is that the autographs (original writings) are inspired, not the
copies. The autographs are the original writings, the original documents penned by the biblical
writers. The copies are copies of inspired documents. The copies are not themselves "inspired"; that
is, they have no guarantee of being 100% textually pure. But don't worry, the Bible manuscripts are
98.5% textually pure and only are very small amount of information is in question due to repetitive
facts, instructions, and information found elsewhere in the Bible. Nevertheless, through the copying
method over the years, various textual problems have arisen. Following is a list of the types of errors
that have occurred in copying the manuscripts. I've used English as examples instead of going into
the original languages for examples.

1. Dittography - Writing twice what should have been written once.


A. A good example would be writing "latter" instead of "later." "Latter" means nearest the
end. "Later" means after something else.
2. Fission- Improperly dividing one word into to words.
A. Example: "nowhere" into "now here."
3. Fusion - Combining the last letter of one word with the first letter of the next word.
A. "Look it is there in the cabinet... or Look it is therein the cabinet."
4. Haplography - Writing once what should have been written twice.
A. A good example would be "later" instead of "latter." "Later" means after something
else. "Latter" means nearest the end.
5. Homophony - Writing a word with a different meaning for another word when both words
have the exact same pronunciation.
A. Meat and meet have the exact same sound but different meanings. Also, there and
their and they're are another example.
6. Metathesis - An improper exchange in the order of letters.
A. Instead of writing "mast," someone writes "mats," or "cast" and cats."

Does this mean that the Bible we hold in our hand is not inspired? Not at all. Inspiration comes
from God and when He inspired the Bible, it was perfect. Our copies of the original documents are not
perfect, but they are very close to being so. The critics often erringly assume that even the copies are
supposed to be perfect. But when I point out that God never said the copies would be perfect, they
then ask how can the Bible be trusted at all? Quite simply, it is redundant in its facts and information.
The copyist errors present no problems doctrinally.
Compared to other ancient documents, the New Testament, for example, has far more textual
evidence in its favor than any other ancient writing. Please consider the chart below.
Author When Written Earliest Copy Time Span No. of Copies
Homer (Iliad) 900 BC 400 BC 500 years 643
Ceasar (The
100 - 44 BC 900 AD 1,000 years 10
Gallic Wars)
Plato
427 - 347 BC 900 AD 1,200 years 7
(Tetralogies)
Aristotle 384 - 322 BC 1,100 AD 1,400 years 49
Herodotus
480 - 425 BC 900 AD 1,300 years 8
(History)
Euripedes 480 - 406 BC 1,100 AD 1,500 years 9
New Testament 50 - 90 A.D. 130 AD 30 years 24,000
This chart was adapted from charts in Evidence that Demands a Verdict, by Josh McDowell, 1979,
pages 42 and 43.

If the Bible cannot be trusted as being reliable because it has only a small percentage of copyist
errors, then neither can the above documents be trusted that have far less textual support.
Therefore, we can see that the Bible is an ancient document that has withstood thousands of years
of transmission with remarkable accuracy and clarity. We can trust it to be what it says it is: the
word of God.

Don't Gen. 1 and 2 present contradictory creation accounts?


1. Genesis 1
- Day one - heavens and earth are created. "Let there be light." Day and Night. - Day two -
Atmospheric waters separated from earth waters.
- Day three - Land appears separating the seas. Vegetation is made.
- Day four - Sun, moon, stars are made.
- Day five - Sea life and birds are made.
- Day six - Land animals, creeping things, and man (male and female) are made.
2. Genesis 2
States heaven and earth were created. No plant yet on earth, no rain yet, no man. But, a mist
rose watering the surface of the ground. Then the Lord formed man from dust of the ground and
breathed into his nostrils the breath of life. Then God made Eve.

There is no contradiction between Genesis 1 and 2. Genesis 1 is a detailed explanation of the six
days of creation, day by day. Genesis two is a recap and a more detailed explanation of the sixth day,
the day that Adam and Eve were made. The recap is stated in Gen. 2:4, "This is the account of the
heavens and the earth when they were created, in the day that the Lord God made earth and heaven."
Then, Moses goes on to detail the creation of Adam and Eve as is seen in verses 7 thru 24 of Gen. 2.
Proof that it is not a creative account is found in the fact that animals aren't even mentioned until after
the creation of Adam. Why? Probably because their purpose was designated by Adam. They didn't
need to be mentioned until after Adam was created.
How many Gods are there, one or many?
Deuteronomy 6:4; Isaiah 43:10; 44:6,8
and Genesis 1:26; 3:22; 11:7; 1 Cor. 8:5; I John 5:7

1. One God
A. (Deuteronomy 6:4) - "Hear, O Israel! The Lord is our God, the Lord is one!" (Isaiah
43:10) - ". . . Before Me [YHWH] there was no God formed, and there will be none after
Me."
B. (Isaiah 44:6) - ". . . there is no God besides Me."
C. (Isaiah 44:8) - ". . . And you are My witnesses. Is there any God besides Me, or is there
any other Rock? I know of none."
2. Many gods
A. (Genesis 1:26) - "Then God said, "Let Us make man in Our image, according to Our
likeness . . ."
B. (Genesis 3:22) - "And the LORD God said, "The man has now become like one of us,
knowing good and evil . . ."
C. (Genesis 11:7) - "Come, let us go down and confuse their language so they will not
understand each other."
D. (1 Corinthians 8:5) - "For even if there are so-called gods, whether in heaven or on
earth (as indeed there are many "gods" and many "lords")."
E. (1 John 5:8) - "For there are three that bear witness, the Spirit and the water and the
blood; and the three are in agreement."

The Bible tells us that there is only one God in all existence (Isaiah 43:10; 44:6,8). However, it
also mentions "other gods." For example there is Adrammelech and Anammelech (2 Kings 17:31),
Asherah (1 Kings 18:19), Baal (Judges 3:1), Chemosh (Num. 21:29), Dagon (1 Sam. 5:2), Molech
(Lev. 18:21; 20:2-5), etc. The Bible is not contradicting itself. When the Bible speaks of other gods it
is speaking of false gods that have no true existence. Gal. 4:8 says, "Formerly, when you did not
know God, you were slaves to those who by nature are not gods." See also, Isaiah 37:19 and
Jeremiah 2:11. God tells us that he alone is the true God and that all of the invented gods of man do
not exist except in their own minds. So, we can see that the Bible does not contradict itself regarding
how many gods there are in existence. There is only one.
Note: in the verses in Genesis that have God saying "Let us make..., Let us go down . . . , etc." are
clues to the Trinitarian nature of God. God is a Trinity of persons: a Father, the Son, and the Holy
Spirit. There are not three Gods, but one. There are those who insist that the Trinity is polytheistic.
But it is not. Trinitarians believe in a single being who is God.
Doesn't God saying "Adam where are you?" show
God didn't know something?
Genesis 3:9

(Genesis 3:9) - "Then the Lord God called to the man, and said to him, "Where are you?"

The context of this verse is immediately after Adam had sinned. "And they heard the sound of the
Lord God walking in the garden in the cool of the day, and the man and his wife hid themselves from
the presence of the Lord God among the trees of the garden. 9Then the Lord God called to the man,
and said to him, "Where are you?" 10And he said, "I heard the sound of Thee in the garden, and I was
afraid because I was naked; so I hid myself," (Gen. 3:8-10). Does this mean that God didn't know
something? Not at all. Someone can easily ask a question to which they know the answer. Usually,
the reason to do that is to point something out or teach something. Obviously, Adam and Eve had
sinned. God asked, "Adam where are you?" Where was Adam? He was hiding from God. Adam's sin
had destroyed his fellowship with the Lord. God knew this and He was pointing it out to Adam. It is a
question we need to ask of ourselves? Where am I? Am I in fellowship with God or in rebellion
against Him?

Where did Cain get his wife?


Genesis 4:17

(Gen. 4:17) - "Then Cain went out from the presence of the Lord, and settled in the land of Nod, east
of Eden. 17And Cain had relations with his wife and she conceived, and gave birth to Enoch; and he
built a city, and called the name of the city Enoch, after the name of his son," (NASB).

We see in the Bible in Genesis 3 where Adam and Eve were cursed and sent out of the Garden of
Eden. In Genesis 4, Cain kills Abel. In Genesis 4:17 above we see that Cain had relations with his
wife. Where did he get his wife? The answer is simple. Cain married one of his sisters.
Genesis 5:4 says, "Then the days of Adam after he became the father of Seth were eight hundred
years, and he had other sons and daughters." We see that Adam and Eve had many sons and
daughters. The genetic lineage of Adam and Eve was perfect so marrying a sister wasn't going to
cause birth defects. It wasn't until much later, during the time of Moses, that incest was forbidden as
the genetic pool became less and less able to stand interbreeding. "‘No one is to approach any close
relative to have sexual relations. I am the LORD," (Lev. 18:6).
Did people really live hundreds of years according to Genesis?
Genesis 5

1. Adam lived 930 years (Gen. 5:5) - "And all the days that Adam lived were nine hundred and
thirty years: and he died."
2. Seth lived 912 years (Gen. 5:8) - "And all the days of Seth were nine hundred and twelve
years: and he died."
3. Methuselah lived 969 years (Gen. 5:27) - "And all the days of Methuselah were nine
hundred sixty and nine years: and he died."

After the fall, the genetic line of Adam and his descendents was very pure, so their health would
have been incredible. Living that long would not have been a problem. Also, some theologians think
that there was a canopy of water that engulfed the entire earth and that it was released at the time of
the flood. "In the six hundredth year of Noah's life , in the second month, on the seventeenth day of
the month, on the same day all the fountains of the great deep burst open, and the floodgates of the
sky were opened," (Gen. 7:11). The "floodgates of the sky" are sometimes alluded to as great
amounts of water suspended in the sky. Also, no rain is recorded in the Bible until after the flood
which seems to support this idea. This canopy, if it is true, might have provided some sort of
protection from the sun's harmful rays. We can't know for sure and it is only a theory. Nevertheless,
after the flood, the lifespan of people on earth was drastically reduced. "Then the LORD said, "My
Spirit shall not strive with man forever, because he also is flesh; nevertheless his days shall be one
hundred and twenty years," (Gen. 6:3). Whether or not this reduced canopy had any affect on human
lifespan may never be known.

Does the Lord change or not?


Malachi 3:6; and Genesis 6:6,7; Exodus 32:14; Jonah 3:10

1. God does not change


A. (Malachi 3:6) - "For I, the Lord, do not change; therefore you, O sons of Jacob, are not
consumed."
2. God does change
A. (Genesis 6:6,7) - "And the Lord was sorry that He had made man on the earth, and He was
grieved in His heart. 7And the Lord said, "I will blot out man whom I have created from the
face of the land, from man to animals to creeping things and to birds of the sky; for I am
sorry that I have made them."
B. (Exodus 32:14) - "So the Lord changed His mind about the harm which He said He would do
to His people."
C. (Jonah 3:10) - "When God saw their deeds, that they turned from their wicked way, then
God relented concerning the calamity which He had declared He would bring upon them. And
He did not do it."

When God says that He does not change, He is speaking about His nature and character. But this
does not mean that He cannot change how He works with people throughout history.
When we see God changing His mind, we are seeing it from a human perspective. Since God
knows all things from all eternity, He as always known the ultimate plan that He would carry out; even
the plan to "change His mind." As we have seen in Jonah's account of Nineveh. They repented and
God relented from the destruction that was to come upon the inhabitants. Of course, God knew this
would happen and instituted the warning to them in order to bring about their repentance. There is no
mystery here.
How many kinds did Noah bring into the ark, two or seven?
Genesis 7:2-3 and Genesis 6:19-20

1. Two (Genesis 6:19-20) - "And of every living thing of all flesh, you shall bring two of every
kind into the ark, to keep them alive with you; they shall be male and female. 20Of the birds
after their kind, and of the animals after their kind, of every creeping thing of the ground after
its kind, two of every kind will come to you to keep them alive."
2. Seven (Genesis 7:2-3) - "You shall take with you of every clean animal by sevens, a male and
his female; and of the animals that are not clean two, a male and his female; 3also of the birds
of the sky, by sevens, male and female, to keep offspring alive on the face of all the earth."

Genesis 6:19-20 simply instructs Noah to preserve two of every kind. Genesis 7:2-3 is additional
information where seven of the clean animals were to be taken and two of every other kind. The
reason for this is that the extra animals were for sacrifice. "Then Noah built an altar to the LORD, and
took of every clean animal and of every clean bird and offered burnt offerings on the altar," (Gen.
8:20).
Logically, to have seven pairs also means that there are two pairs, since the two are included in the
seven. If one verse said take only one pair and another verse said seven pairs, that would be a
contradiction.

Who purchased Joseph, the Ishmaelites or the Midianites?


Genesis 37:28, 37:36 and 39:1

1. Ishmaelites (Genesis 37:28) - "Then some Midianite traders passed by, so they pulled him up
and lifted Joseph out of the pit, and sold him to the Ishmaelites for twenty shekels of silver.
Thus they brought Joseph into Egypt."
2. Midianites (Genesis 37:36) - "Meanwhile, the Midianites sold him in Egypt to Potiphar,
Pharaoh’s officer, the captain of the bodyguard."
3. Ishamaelites (Genesis 39:1) - "Now Joseph had been taken down to Egypt; and Potiphar, an
Egyptian officer of Pharaoh, the captain of the bodyguard, bought him from the Ishmaelites,
who had taken him down there."

According to Achtemeier, Paul J., Th.D., Harper’s Bible Dictionary, (San Francisco: Harper and Row,
Publishers, Inc.) 1985, the term "Ishamelite" was synomous with the term "Midianites." They were
probably references to the same general group known to have decended from Abraham. Ishmael was
born to Abraham through Hagar (Genesis 16), the hand maiden. The Midianites were descendants of
Midian, a son of Abraham and his concubine Keturah (Genesis 25:1-2). Additionally, "The term
‘Midianite’ probably identified a confederation of tribes that roamed far beyond this ancestral
homeland, a usage that explains the biblical references to Midianites in Sinai, Canaan, the Jordan
Valley, Moab, and Transjordan’s eastern desert. (Achtemeier, Paul J., Th.D., Harper’s Bible Dictionary)
God kills a man for spilling his seed on the ground.
Genesis 38:9

(Genesis 38:9) - "And Onan knew that the offspring would not be his; so it came about that
when he went in to his brother’s wife, he wasted his seed on the ground, in order not to give
offspring to his brother."

Why did God kill Onan for spilling his seed on the ground? The reason God did this is not because
Onan wasted his seed on the ground, but because Onan refused to perform his familial duties of
producing offspring for his brother's. This was a great offense at the time. Now, we must realize that
the culture was very different than ours is today. In that culture, when a man died and left no
children, the next of kin was sometimes obligated to "go into" the wife and produce children. These
children were then considered to be the descendents of the original late husband and would be raised
as such. This way, the offspring would be able to take care of the mother, provide more people for the
community, and thereby raise their own children, continuing the name of that family. Onan knew this
and refused to take part in furthering the honor and name of the brother's wife and thereby also
risking provision for her in the future. To this, God was very displeased and took Onan's life.

Has anyone seen God or not?


Exodus 24:9-11, Exodus 33:11; Exodus 6:2-3, John 1:18

1. Has seen
A. (Gen. 17:1) – “Now when Abram was ninety-nine years old, the LORD appeared to Abram
and said to him, "I am God Almighty ; Walk before Me, and be blameless;
B. (Gen. 18:1) Now the LORD appeared to him by the oaks of Mamre, while he was sitting at
the tent door in the heat of the day.”
C. (Exodus 6:2-3) – “God spoke further to Moses and said to him, "I am the LORD; 3and I
appeared to Abraham, Isaac, and Jacob, as God Almighty , but by My name, LORD, I did not
make Myself known to them.”
D. (Exodus 24:9-11) – “Then Moses went up with Aaron, Nadab and Abihu, and seventy of the
elders of Israel, 10and they saw the God of Israel; and under His feet there appeared to be a
pavement of sapphire, as clear as the sky itself. 11Yet He did not stretch out His hand against
the nobles of the sons of Israel; and they saw God, and they ate and drank.”
E. (Num. 12:6-8) – “He said, "Hear now My words: If there is a prophet among you, I, the
LORD, shall make Myself known to him in a vision. I shall speak with him in a dream. 7"Not
so, with My servant Moses, He is faithful in all My household; 8With him I speak mouth to
mouth, Even openly, and not in dark sayings, And he beholds the form of the LORD. Why
then were you not afraid To speak against My servant, against Moses?"
F. (Acts 7:2), "And he [Stephen] said, "Hear me, brethren and fathers! The God of glory
appeared to our father Abraham when he was in Mesopotamia, before he lived in Haran..."
2. Has not seen
A. (Exodus 33:20) – “But He [God] said, "You cannot see My face, for no man can see Me and
live !"
B. (John 1:18) – “No one has seen God at any time; the only begotten God who is in the
bosom of the Father, He has explained Him.”
C. (John 5:37) – “"And the Father who sent Me, He has testified of Me. You have neither heard
His voice at any time nor seen His form.”
D. (John 6:46) - "Not that anyone has seen the Father, except the One who is from God; He
has seen the Father.”
E. (1 Tim. 6:15-16) – “He who is the blessed and only Sovereign, the King of kings and Lord of
lords, 16who alone possesses immortality and dwells in unapproachable light, whom no man
has seen or can see. To Him be honor and eternal dominion! Amen.”
It is evident above that God was seen. But, considering the "can't-see-God" verses, some would
understandably argue that there would be a contradiction. Once explanation is that the people were
seeing visions, or dreams, or the Angel of the LORD (Num. 22:22-26; Judges 13:1-21) and not really
God Himself. But the problem is that the verses cited above do not say vision, dream, or Angel of the
LORD. They say that people saw God (Exodus 24:9-11), that God was seen, and that He appeared as
God Almighty (Exodus 6:2-3).
At first, this is difficult to understand. God Almighty was seen ( Exodus 6:2-3) which means it was
not the Angel of the Lord, for an angel is not God Almighty, and at least Moses saw God, not in a
vision or dream, as the LORD Himself attests in Num. 12:6-8. If these verses mean what they say,
then we naturally assume we have a contradiction. Actually, the contradiction exists in our
understanding, not in the Bible--which is always the case with alleged biblical contradictions.
All you need to do is accept what the Bible says. If the people of the OT were seeing God, the
Almighty God, and Jesus said that no one has ever seen the Father (John 6:46), then they were seeing
God Almighty, but not the Father. It was someone else in the Godhead. I suggest that they were
seeing the Word before He became incarnate. In other words, they were seeing Jesus.
If God is a Trinity, then John 1:18 is not a problem either because in John chapter one, John
writes about the Word (Jesus) and God (the Father). In verse 14 it says the Word became flesh. In
verse 18 it says no one has seen God. Since Jesus is the Word, God then, refers to the Father. This is
typically how John writes of God: as a reference to the Father. We see this verified in Jesus own
words in John 6:46 where He said that no one has ever seen the Father. Therefore, Almighty God was
seen, but not the Father. It was Jesus before His incarnation. There is more than one person in the
Godhead and the doctrine of the Trinity must be true.

Should you make graven images or not?


Exodus 20:4-5, Leviticus 26:1; Deuteronomy 5:8; 27:15
and Exodus 25:18; 37:7-8

1. Shall not make graven images


A. (Exodus 20:4-5) - "You shall not make for yourself an idol, or any likeness of what is in
heaven above or on the earth beneath or in the water under the earth. 5"You shall not
worship them or serve them; for I, the Lord your God, am a jealous God, visiting the
iniquity of the fathers on the children, on the third and the fourth generations of those
who hate Me."
B. (Leviticus 26:1) - "You shall not make for yourselves idols, nor shall you set up for
yourselves an image or a sacred pillar, nor shall you place a figured stone in your land
to bow down to it; for I am the Lord your God."
C. (Deuteronomy 5:8) - "You shall not make for yourself an idol, or any likeness of what is
in heaven above or on the earth beneath or in the water under the earth."
D. (Deuteronomy 27:15) - "Cursed is the man who makes an idol or a molten image, an
abomination to the Lord, the work of the hands of the craftsman, and sets it up in
secret.’ And all the people shall answer and say, ‘Amen.’"
2. Shall make graven images
A. (Exodus 25:18) - "And you shall make two cherubim of gold, make them of hammered
work at the two ends of the mercy seat."
B. (Exodus 37:7-8) - "And he made two cherubim of gold; he made them of hammered
work, at the two ends of the mercy seat; 8one cherub at the one end, and one cherub at
the other end; he made the cherubim of one piece with the mercy seat at the two
ends." See also, Exodus 26:1,31; 36:8; 1 Kings 6:23-35.

The context of the "Thou shall not make a graven image" passages is dealing with worship of false
things. Exodus 20:4 states that no one is to make an image of what is in heaven so that you may not
worship them or bow down to them (20:5). This is reiterated in Leviticus 26:1. The Deuteronomy
passages, contextually, are dealing with the same thing: an admonition against worshipping a false
image. God does not want people bowing down before idols and worshiping false gods.
The instruction by God to make cherubim, which are angels in heaven, is not for the purpose of
worship at all. Instead, it is a representation of the heavenly realm where God dwells and the angels
are about the throne (1 Samuel 4:4; Hebrews 9:5). The Cherubim were placed on the Ark of the
Covenant, in the Holy of Holies in the temple (2 Chron. 3:10). There, they would never become
objects of worship because they were not public artifacts to which the general populace would become
familiar and thereby risk falling into idol worship.

Should we keep the Sabbath or not?


Exodus 20:8; 23:12; 31:15; Deuteronomy 5:12; Leviticus 26:2
and Romans 14:5; Colossians 2:16

1. Keep the Sabbath


A. (Exodus 20:8) - "Remember the sabbath day, to keep it holy. 9 "Six days you shall labor
and do all your work,
B. (Exodus 23:12) - "Six days you are to do your work, but on the seventh day you shall
cease from labor in order that your ox and your donkey may rest, and the son of your
female slave, as well as your stranger, may refresh themselves."
C. (Exodus 31:15) - "For six days work may be done, but on the seventh day there is a
sabbath of complete rest, holy to the Lord; whoever does any work on the sabbath day
shall surely be put to death."
D. (Deuteronomy 5:12) - "Observe the sabbath day to keep it holy, as the Lord your God
commanded you."
E. (Leviticus 26:2) - "You shall keep My sabbaths and reverence My sanctuary; I am the
Lord."
2. Don't keep the Sabbath
A. (Romans 14:5) - "One man regards one day above another, another regards every day
alike. Let each man be fully convinced in his own mind."
B. (Colossians 2:16) - "Therefore let no one act as your judge in regard to food or drink or
in respect to a festival or a new moon or a Sabbath day."

It was the custom of the Jews to come together on the Sabbath, which is Saturday, cease work,
and worship God. Of the 10 commandments listed in Exodus 20:1-17, only nine of them were
reinstituted by in the New Testament. (Six in Matthew 19:18, murder, adultery, stealing, false witness,
honor parents, and worshiping God; Romans 13:9, coveting. Worshiping God properly covers the first
three commandments) The one that was not reaffirmed was the one about the Sabbath. Instead, Jesus
said that He is the Lord of the Sabbath (Matt. 12:8).
In creation God rested on the seventh day. But, since God is all powerful, He doesn’t get tired. He
doesn’t need to take a break and rest. So, why did does it say that He rested? The reason is simple:
Mark 2:27 says, "The Sabbath was made for man, and not man for the Sabbath." In other words, God
established the Sabbath as a rest for His people, not because He needed a break, but because we are
mortal and need a time of rest, of focus on God. In this, our spirits and bodies are both renewed.
The O.T. system of Law required keeping the Sabbath as part of the overall moral, legal, and
sacrificial system by which the Jewish people satisfied God’s requirements for behavior, government,
and forgiveness of sins. The Sabbath was part of the Law in that sense. In order to "remain" in favor
with God, you had to also keep the Sabbath. If it was not kept, then the person was in sin and would
often be punished (Ezekiel 18:4; Rom. 6:23; Deut. 13:1-9; Num. 35:31; Lev. 20:2, etc.).
But with Jesus’ atonement, and justification by faith ( Rom. 5:1), we no longer are required to keep
the Law and hence the Sabbath which was only a shadow of things to come ( Col. 2:16-17). We are not
under Law, but grace (Rom. 6:14-15). The Sabbath is fulfilled in Jesus because in Him we have rest
(Matt. 11:28). We are not under obligation to keep the Law and this goes for the Sabbath as well.
Do the sons bear the sins of the fathers or not?
Exodus 20:5, Deuteronomy 5:9 and
Deuteronomy 24:16; Ezekiel 18:20

1. Yes they do
A. (Exodus 20:5) - "You shall not worship them or serve them; for I, the Lord your God,
am a jealous God, visiting the iniquity of the fathers on the children, on the third and
the fourth generations of those who hate Me,"
B. (Deuteronomy 5:9) - "You shall not worship them or serve them; for I, the Lord your
God, am a jealous God, visiting the iniquity of the fathers on the children, and on the
third and the fourth generations of those who hate Me,"
C. (Exodus 34:6-7) - "Then the Lord passed by in front of him and proclaimed, "The Lord,
the Lord God, compassionate and gracious, slow to anger, and abounding in
lovingkindness and truth; 7who keeps lovingkindness for thousands, who forgives
iniquity, transgression and sin; yet He will by no means leave the guilty unpunished,
visiting the iniquity of fathers on the children and on the grandchildren to the third and
fourth generations."
D. (1 Cor. 15:22) - "For as in Adam all die, so also in Christ all shall be made alive."
2. No they don't

A. (Deuteronomy 24:16) - "Fathers shall not be put to death for their sons, nor shall
sons be put to death for their fathers; everyone shall be put to death for his own
sin."
B. (Ezekiel 18:20) - "The person who sins will die. The son will not bear the
punishment for the father’s iniquity, nor will the father bear the punishment for the
son’s iniquity; the righteousness of the righteous will be upon himself, and the
wickedness of the wicked will be upon himself."

Exodus 20:5 is, of course, among the Ten Commandments. The Ten Commandments are arranged
in covenant form. The Suzerain-Vassal treaty pattern of the ancient near east is followed in the Ten
Commandments. This arrangement included an introduction of who was making the covenant ( Exodus
20:2), what the covenant maker had done (20:2), laws (20:3-17), rewards (20:6,12), and
punishments (20:5,7). Covenantally, when a father misleads his family, the effects of that misleading
are often felt for generations. This is because the father is being covenantally unfaithful and God has
stipulated that there are punishments to breaking the covenant with God. That is the case with these
verses that deal with the sins visited upon the children. If a father rejects the covenant of God and
takes his family into sin and rejects God, the children will suffer the consequences, often for several
generations. Whether or not this is fair is not the issue. Sin is in the world consequences of sin
effected many generations.
On the other hand, Deuteronomy 24:16 is dealing with legal matters as the context 24:6-19
shows. Ezekiel 18:20 is merely recounting the Law of the Pentateuch. Therefore, the context of
second set of verses is dealing with the legality aspect within the Jewish court system. The previous
set of verses deal with God visiting upon the descendents of the rebellious the consequences of the
rebellious fathers' sins.

Federal Headship

As a further note on this issue, there is a concept in the Bible called Federal Headship. This means
that the male, the father, represents the family. We see this in the garden of Adam and Eve. He was
the first one to eat of the fruit; she was the first one to sin. However, the Bible states that sin entered
the world through Adam (Rom. 5), not Eve. This is because Adam was the Federal Head of all
mankind. Furthermore we see in the Hebrews 7:7-10 the following:

"But without any dispute the lesser is blessed by the greater. 8And in this case mortal
men receive tithes, but in that case one receives them, of whom it is witnessed that he
lives on. 9And, so to speak, through Abraham even Levi, who received tithes, paid
tithes, 10for he was still in the loins of his father when Melchizedek met him."

In the verses in Hebrews we see that Levi, who was a descendant of Abraham, paid tithes to
Melchizedek while still in the loins, "seed," of his father Abraham, even though Levi was not yet alive.
In other words, Abraham, the father, represented his descendants. As Abraham paid tithes, so also
did Levi. Therefore, we can see the concept of Federal Headship represented in the Bible in both the
Old and New Testaments. We can conclude that God will visit the inequities of the fathers upon the
descendents because the fathers have failed to be covenantally faithful. Yet, we see in the other
verses a declaration of legality in dealing with people. There is no contradiction.

Did or did not Saul know who David was?


1 Samuel 16:19-23 and 1 Samuel 17:55-58

1. Yes, Saul knew David (1 Samuel 16:19-23) - "So Saul sent messengers to Jesse, and said,
"Send me your son David who is with the flock." 20And Jesse took a donkey loaded with bread
and a jug of wine and a young goat, and sent them to Saul by David his son. 21Then David
came to Saul and attended him, and Saul loved him greatly; and he became his armor bearer.
22
And Saul sent to Jesse, saying, "Let David now stand before me; for he has found favor in my
sight." 23So it came about whenever the evil spirit from God came to Saul, David would take the
harp and play it with his hand; and Saul would be refreshed and be well, and the evil spirit
would depart from him," (NASB).
2. Saul asked "Whose son are you?" (1 Samuel 17:55-58) - "Now when Saul saw David going
out against the Philistine, he said to Abner the commander of the army, "Abner, whose son is
this young man?" And Abner said, "By your life, O king, I do not know." 56And the king said,
"You inquire whose son the youth is." 57So when David returned from killing the Philistine,
Abner took him and brought him before Saul with the Philistine’s head in his hand. 58And Saul
said to him, "Whose son are you, young man?" And David answered, "I am the son of your
servant Jesse the Bethlehemite," (NASB).

Several possible explanations exist to explain this problem ranging from textual error to the
method of the ancient writers to write about themes and then come back to fill in the gaps later. I
think the best explanation can be found by looking at the context.
There is no contradiction. Saul was simply asking David whose son he was. He knew David but
probably didn't know who was David's father. Since David has just saved Israel, Saul wanted to know
who his father was, probably to show the father proper respect for his son David.
In 1 Samuel 16, the Spirit of the Lord had left Saul (16:14) and an evil spirit came and afflicted
Saul (16:23). David came and then played for Saul to sooth him. Chapter 17 begins the well known
story of David and Goliath with no mention of how much time passes between David playing the harp
and Goliath's challenge. It may very well have been many months or even years. Nevertheless, David
was the youngest of the sons of Jesse (17:14), who was a youth (17:33), and who tended the flocks
(17:15). David is then known as a young musician and a sheep herder, not a warrior as were his
three oldest brothers (17:13-14). Saul and David have conversations about David doing battle with
Goliath and Saul offers David his armor (17:38). David refuses the armor and goes out to kill Goliath.
Saul then asks Abner, "Whose son is this young man?" And Abner said, "By your life, O king, I do not
know," (17:55). In verse 58 Saul says, "Whose son are you, young man?" And David answered, "I am
the son of your servant Jesse the Bethlehemite."
Who killed Goliath, David or Elhanan?
1 Samuel 17:50 and 2 Samuel 21:19

1. David did (1 Samuel 17:50) - "Thus David prevailed over the Philistine with a sling and a
stone, and he struck the Philistine and killed him; but there was no sword in David’s hand."
2. Elhanan did (2 Sam. 21:19)- "And there was war with the Philistines again at Gob, and
Elhanan the son of Jaare-oregim the Bethlehemite killed Goliath the Gittite, the shaft of whose
spear was like a weaver’s beam."

The answer lies in two areas. 1 Chronicles 20:5 says, "And there was war with the Philistines
again, and Elhanan the son of Jair killed Lahmi the brother of Goliath the Gittite, the shaft of whose
spear was like a weaver’s beam." This is the correct answer; namely, that Elhanan killed Goliath's
brother.
Second, it appears there was a copyist error in 2 Samuel 21:19. According to Gleason Archer's
Encyclopedia of Bible Difficulties on page 179, it says,

1. The sign of the direct object, which in Chronicles comes just before "Lahmi," was '-t;
the copyist mistook it for b-t or b-y-t ("Beth") and thus got Bet hal-Lahmi ("the Bethlehemite")
out of it.
2. He misread the word for "brother" ('-h) as the sign of the direct object ('-t) right
before g-l-y-t ("Goliath"). Thus he made "Goliath" the object of "killed" (wayyak), instead of
the "brother" of Goliath (as the Chronicles passage does).
3. The copyist misplaced the word for "weavers" ('-r-g-ym) so as to put it right after
"Elhanan" as his patronymic (ben Y-'-r-y'-r--g-ym, or ben ya 'arey 'ore -gim -- "the son of the
forests of weavers" -- a most unlikely name for anyone's father!). In Chronicles the 'ore grim
("weavers") comes right after menor ("a beam of ") -- thus making perfectly good sense.

Therefore, we see that 2 Samuel 21:19 had a copyist error and 1 Chronicles 21:5 is the correct
information.

Who killed Saul, Saul or the Amalekite?


1 Samuel 31:4 and 2 Samuel 1:8-10

1. Saul did (1 Samuel 31:4) - "Then Saul said to his armor bearer, "Draw your sword and pierce
me through with it, lest these uncircumcised come and pierce me through and make sport of
me." But his armor bearer would not, for he was greatly afraid. So Saul took his sword and fell
on it."
2. The Amalekite did (2 Samuel 1:8-10) - "And he said to me, ‘Who are you?’ And I answered
him, ‘I am an Amalekite.’ 9"Then he said to me, ‘Please stand beside me and kill me; for agony
has seized me because my life still lingers in me.’ 10"So I stood beside him and killed him,
because I knew that he could not live after he had fallen. And I took the crown which was on
his head and the bracelet which was on his arm, and I have brought them here to my lord."
1 Samuel 31:4 gives what actually happened while 2 Samuel 1:8-10 only gives what the Amalekite
said happened. Most probably, the Amalekite took the opportunity to benefit from the King's death,
gathered his crown and bracelet and then brought them to David. Unfortunately for the Amalekite,
David said in 2 Samuel 1:13-16, "And David said to the young man who told him, "Where are you
from?" And he answered, "I am the son of an alien, an Amalekite." 14Then David said to him, "How is it
you were not afraid to stretch out your hand to destroy the Lord’s anointed?" 15And David called one of
the young men and said, "Go, cut him down." So he struck him and he died. 16And David said to him,
"Your blood is on your head, for your mouth has testified against you, saying, ‘I have killed the Lord’s
anointed.'"
The Amalekite probably thought he'd benefit from bringing the King's possessions to David, but his
plan backfired.

Did Michal have any children or not?


(2 Samuel 6:23 and 2 Samuel 21:8)

1. No children (2 Samuel 6:23) - "And Michal the daughter of Saul had no child to the day of her
death."
2. Five sons (2 Sam. 21:8) - "But the king took the two sons of Rizpah the daughter of Aiah,
whom she bare unto Saul, Armoni and Mephibosheth; and the five sons of Michal the daughter
of Saul, whom she brought up for Adriel the son of Barzillai the Meholathite" (KJV).
 (2 Samuel 21:8, NASB) - "So the king took the two sons of Rizpah the daughter of Aiah,
Armoni and Mephibosheth whom she had born to Saul, and the five sons of Merab the
daughter of Saul, whom she had born to Adriel the son of Barzillai the Meholathite."
 (2 Samuel 21:8, NIV) - "But the king took Armoni and Mephibosheth, the two sons of
Aiah’s daughter Rizpah, whom she had borne to Saul, together with the five sons of
Saul’s daughter Merab, whom she had borne to Adriel son of Barzillai the Meholathite."

Saul had two daughters: Merab and Michal. 1 Samuel 14:49, says "Now the sons of Saul were
Jonathan, and Ishui, and Melchishua: and the names of his two daughters were these; the name of
the firstborn Merab, and the name of the younger Michal." Since 2 Samuel 6:23 states that Michal had
no children, we can conclude that this is a copyist error that should have read Merab. "Many scholars
substitute Merab for Michal in 2 Sam. 21:8, regarding it as an ancient scribal error, saying that after
her death her sons were hanged to atone for Saul’s slaughter of the Gibeonites, a breaking of Israel’s
covenant."69

How many charioteers were killed, 700 or 7000?


2 Samuel 10:18 and 1 Chronicles 19:18

1. 700 charioteers (2 Samuel 10:18) - "But the Arameans fled before Israel, and David killed
700 charioteers of the Arameans and 40,000 horsemen and struck down Shobach the
commander of their army, and he died there."
2. 7,000 charioteers (1 Chronicles 19:18) - "And the Arameans fled before Israel, and David
killed of the Arameans 7,000 charioteers and 40,000 foot soldiers, and put to death Shophach
the commander of the army."

This is most probably a copyist error. Notice how the number is off by a single zero; that is, by a
single notation of a digit. According to "Alledged Discrepencies of the Bible," page 382, regarding the
characters used to designate numbers, "Nun final , was mistaken for dotted Zayin ," would account
for the copyist error in the text. Most probably, the correct number is 7,000 charioteers.

69
The New Bible Dictionary, (Wheaton, Illinois: Tyndale House Publishers, Inc.) 1962.
Who incited David to count the fighting men of Israel? God or Satan?
2 Samuel 24:1 and 1 Chronicles 21:1

1. God did (2 Samuel 24:1) - "Now again the anger of the Lord burned against Israel, and it
incited David against them to say, "Go, number Israel and Judah."
2. Satan did (1 Chronicles 21:1) - "Then Satan stood up against Israel and moved David to
number Israel."

Is this a contradiction? Not at all. In 2 Samuel 24:1, God incited David to number Israel because
God was angry with David. Ultimately, God wanted to teach David not to trust in his number of
fighting men, but to trust in Him. So, He moved to let David count the fighting men of Israel. He
used Satan to do it which is why in 1 Chronicles 21:1, it says Satan moved David to count the men.
Both are true. God most probably either sent Satan or allowed Satan to do incite David.
God's authority extends even over Satan. God can use Satan to accomplish His ultimate will. We
see this in the crucifixion of Christ where evil men brought Jesus to death. Yet, at the same time, it
was the predetermined plan of God that this be done.

"For truly in this city there were gathered together against Thy holy servant Jesus,
whom Thou didst anoint, both Herod and Pontius Pilate, along with the Gentiles and the
peoples of Israel, 28to do whatever Thy hand and Thy purpose predestined to occur,"
(Acts 4:27-28, NASB).

Furthermore, we see in Job that God allowed Satan to test Job and demonstrate Job's character
(Job 1:8-13). We see in John 13:25-27 that Satan entered into Judas to betray Jesus, but it was the
plan of God that Jesus be betrayed as Acts 4:27-28 above tells us.
God allows the evil one to work His evil yet that word is ultimately used for the glory of God. All
this is done without God sinning and it demonstrates God's absolute sovereignty over all creation.

How many fighting men were found in Judah and Israel?


2 Samuel 24:9 and 1 Chronicles 21:5

1. Five hundred thousand (2 Samuel 24:9) - "And Joab gave the number of the registration of
the people to the king; and there were in Israel eight hundred thousand valiant men who drew
the sword, and the men of Judah were five hundred thousand men."
2. Four hundred and seventy thousand (1 Chronicles 21:5) - "And Joab gave the number of
the census of all the people to David. And all Israel were 1,100,000 men who drew the sword;
and Judah was 470,000 men who drew the sword."

Israel Judah
2 Sam 24:9 800,000 "valiant" men 500,000
1 Chron. 21:5 1,100,000 men 470,000 men

Regarding Israel's number difference:


The solution to the difference in counts for Israel seems to be answered in the Hebrew word for
"valiant," which is "chayil" found in 2 Samuel 24:9. It means, "men of valor, army, host, etc." It
seems to mean that the men numbered in 2 Samuel 24:9 were those with battle experience where the
men of 1 Chronicles 21:5 were not. It was most probably true that there were an additional 300,000
men ready for battle who had not yet experienced it. Therefore, 2 Sam. 24:9 numbers only the
experienced men, where 1 Chronicles 21:5 numbers all men of battle ready age.

Regarding Judah's number difference:


The solution seems to provided for us in the following verse six which says, "But he did not number
Levi and Benjamin among them, for the king’s command was abhorrent to Joab," (NASB). Verse six
states that the numbering process had not yet been completed since the tribes of Levi and Benjamin
had not been numbered.

God sent his prophet to threaten David with how many years of famine?
2 Samuel 24:13 and 1 Chronicles 21:12

1. Seven years (2 Samuel 24:13) - "So Gad came to David and told him, and said to him, "Shall
seven years of famine come to you in your land? Or will you flee three months before your foes
while they pursue you? Or shall there be three days’ pestilence in your land? Now consider and
see what answer I shall return to Him who sent me," (NASB).
2. Three years (1 Chronicles 21:11-12) - "So Gad came to David and said to him, "Thus says the
Lord, ‘Take for yourself 12either three years of famine, or three months to be swept away before
your foes, while the sword of your enemies overtakes you, or else three days of the sword of
the Lord, even pestilence in the land, and the angel of the Lord destroying throughout all the
territory of Israel.’ Now, therefore, consider what answer I shall return to Him who sent me,"
(NASB).

There are two possible explanations for the discrepancy. First, it is possible that the duration of
the famine was reduced from seven to three years after David prayed for mercy from the Lord.
Thought the text does not specifically state this, we can infer it from the differences between the
accounts. In 2 Samuel 24:13, David is asked which of the three options that he could take. In 1
Chronicles 21:11-12, there is no question asked. David is told to make a choice. Therefore, it may be
that the Prophet Gad first asked David "Shall seven years of famine come to you in your land?" and
later told David to make a choice when he said, "take for yourself either three years of famine..."
The second, it is probably a copyist error and the better preserved text renders the famine as three
years

How many stalls of horses did Solomon have, 4,000 or 40,000?


1 Kings 4:26 and 2 Chronicles 9:25

1. 40,000 (1 Kings 4:26) - "And Solomon had 40,000 stalls of horses for his chariots, and 12,000
horsemen."
2. 4,000 (2 Chron. 9:25) - "Now Solomon had 4,000 stalls for horses and chariots and 12,000
horsemen, and he stationed them in the chariot cities and with the king in Jerusalem."

There are two possible explanations for this discrepancy. 1) a copyist error. 2) the difference is
due to time; that is, one account is at the beginning of Solomon's reign ( 1 Kings 4:26), and the other
at the end (2 Chron. 9:25). I believe the most probable is a copyist error since we can see that
Chronicles does have copyist errors in other areas. Therefore, it is probable that the same thing
occurred here.
"In general it can be said that the books of Chronicles furnish approximate numerical estimates in
the form of round numbers, frequently designed, as has been remarked, to express the magnitude of
the occasion....Some estimates in Chronicles which appear to be particularly inflated can be corrected
or scaled down by reference to the books of Samuel and Kings...However, it is not always the case
that the figures in Chronicles exceed their counterparts in Samuel and Kings." 70 The correct answer is
probably 4,000 since 40,000 seems extraordinarily large. Furthermore, it seems likely that a single
"10's" place was copied incorrectly accounting for the discrepancy.

How many supervisors were there? 3,300 or 3,600?


1 Kings 5:16 and 2 Chronicles 2:2

1. 3,300 (1 Kings 5:16) - "besides Solomon’s 3,300 chief deputies who were over the project and
who ruled over the people who were doing the work."
2. 3,600 (2 Chronicles 2:2) - "So Solomon assigned 70,000 men to carry loads, and 80,000 men to
quarry stone in the mountains, and 3,600 to supervise them."

Either this is a copyist error or the people are being counted differently. If the people are being
counted differently, it is interesting to note that the total number of supervisors is the same when we
add the numbers from 1 Kings 5:16 and 1 Kings 9:23 together and also add 2 Chronicles 2:2 and 2
Chronicles 8:10 together.

Verse Verse Total


"besides Solomon’s 3,300 chief "These were the chief officers who were
deputies who were over the project over Solomon’s work, five hundred and
and who ruled over the people who fifty, who ruled over the people doing
1 Kings were doing the work," the work,"
(1 Kings 5:16) (1 Kings 9:23).
3,300 + 550 3,850
"So Solomon assigned 70,000 men
"And these were the chief officers of
to carry loads, and 80,000 men to
King Solomon, two hundred and fifty
quarry stone in the mountains, and
2 Chronicles who ruled over the people,"
3,600 to supervise them,"
(2 Chronicles 8:10).
(2 Chronicles 2:2).
3,600 + 250 3,850

However, the interesting problem is that in both places, the numbers are different, yet the totals
are the same.
A comment worth noting is "In general it can be said that the books of Chronicles furnish
approximate numerical estimates in the form of round numbers, frequently designed, as has been
remarked, to express the magnitude of the occasion....Some estimates in Chronicles which appear to
be particularly inflated can be corrected or scaled down by reference to the books of Samuel and
Kings...However, it is not always the case that the figures in Chronicles exceed their counterparts in
Samuel and Kings." 71

70
Harrison, R. K., Introduction to the Old Testament, (Grand Rapids, Michigan: Eerdmans Publishing Company)
1969, page. 1165.
Harrison, R. K., Introduction to the Old Testament, (Grand Rapids, Michigan: Eerdmans Publishing
71

Company) 1969, page. 1165.


How many baths, 2000 or 3,000?
1 Kings 7:26 and 2 Chronicles 4:5

1. 2,000 baths (1 Kings 7:26) - "And it was a handbreadth thick, and its brim was made like the
brim of a cup, as a lily blossom; it could hold two thousand baths."
2. 3,000 baths (2 Chronicles 4:5) - "And it was a handbreadth thick, and its brim was made like
the brim of a cup, like a lily blossom; it could hold 3,000 baths."

It appears that the difference in numbers is do to a copyist error. The characters used for the
numbers 2,000 and 3,000 are similar. = 2,000 and = 3,000. A tired copyist could easily mistake
one for another.72

How old was Ahaziah when he began to rule over Jerusalem?


2 Kings 8:26 and 2 Chronicles 22:2

1. Twenty-two (2 Kings 8:26) - "Ahaziah was twenty-two years old when he became king, and
he reigned one year in Jerusalem. And his mother’s name was Athaliah the granddaughter of
Omri king of Israel," (NASB).
2. Forty-two (2 Chron. 22:1) - "Ahaziah was forty-two years old when he became king, and he
reigned one year in Jerusalem. And his mother’s name was Athaliah, the granddaughter of
Omri," (NASB). Note: the NASB corrects the copyist error and inserts 22 years. It has,
however, a note saying the Hebrew states 42 years. For clarity purposes, I quoted the NASB
and kept the original Hebrew number of 42.

The correct age of Ahaziah when he began to rule over Jerusalem is 22. 2 Kings 8:17 tells us that
Ahaziah's father Joram ben Ahab was thirty-two when he became king and he died eight years later, at
the age of forty. Therefore, Ahaziah could not have been forty-two at the time of his father's death at
age forty." (Encyclopedia of Bible Difficulties, page. 206-207.)
The discrepency in ages is due to a copyist error. We can see that the difference in ages is 20
years. The system of number notation used by the Jews at the time of Ezra consisted of horizontal
hooks that represented decades. would equal the number 14 where would be 24. If one or
both of the hooks were smudged or flaked off of a papyrus, then the dates would be off by ten years
or a factor of ten.
The fact that this is a copyist error does not invalidate the inspiration or authority of Scripture.
Remember, God inspired the originals. They were without error. The copies have problems, though
very very few. The copies are copies of inspired documents and, unfortunately, some copyist errors
did creep into the manuscripts. However, they do not affect any doctrinal areas and are very rare.

72
Alleged Discrepancies of the Bible, page 382.
How long did Jehoiachin rule over Jerusalem?
2 Kings 24:8 and 2 Chronicles 36:9

1. Three months (2 Kings 24:8) - "Jehoiachin was eighteen years old when he became king, and
he reigned three months in Jerusalem; and his mother’s name was Nehushta the daughter of
Elnathan of Jerusalem," (NASB).
2. Three months and ten days (2 Chronicles 36:9) - "Jehoiachin was eight years old when he
became king, and he reigned three months and ten days in Jerusalem, and he did evil in the
sight of the Lord," (NASB).

The discrepancy in duration is due to a copyist error. We can see that the difference time is 10
days. The system of number notation used by the Jews at the time of Ezra consisted of horizontal
hooks that represented decades, or "tens." would equal the number 14 where would be 24.
If one or both of the hooks were smudged or flaked off of a papyrus, then the dates would be off by a
value of ten. Therefore, most probably, the correct value was three months and ten days since the
hook could easily have flaked off in a copy.
Does this mean the Bible is not trustworthy? Not at all. Inspiration is ascribed to the original
writings and not to the copies. Scribes made errors. However, the errors were very infrequent and
from other information in the Bible, we can easily ascertain what the correct age is.

Who was King Abijah's mother?


2 Chronicles 11:20 and 2 Chronicles 13:2

1. Maacah (2 Chronicles 11:20) - "And after her he took Maacah the daughter of Absalom, and
she bore him Abijah, Attai, Ziza, and Shelomith."
2. Micaiah (2 Chronicles 13:2) - "He reigned three years in Jerusalem; and his mother’s name
was Micaiah the daughter of Uriel of Gibeah. And there was war between Abijah and
Jeroboam."

This is a very simple answer. Maacah was simply another spelling for Micaiah. They are one and
the same.73

How old was Jehoiachin when he became king?


2 Chronicles 36:9 and 2 Kings 24:8
1. Eight years old 2 Chron. 36:9, "Jehoiachin was eight years old when he became king, and he
reigned three months and ten days in Jerusalem, and he did evil in the sight of the Lord."
2. Eighteen years old 2 Kings 24:8 "Jehoiachin was eighteen years old when he became king,
and he reigned three months in Jerusalem; and his mother’s name was Nehushta the daughter
of Elnathan of Jerusalem."

The correct age of Jehoiachin was 18, not 8. Obviously, Jehoiachin was 18 when he began his rule
since it says he did evil in the site of the Lord which suggests maturity and responsibility.
The discrepancy in ages is due to a copyist error. We can see that the difference in ages is 10
years. The system of number notation used by the Jews at the time of Ezra consisted of horizontal

73
Alleged Discrepancies of the Bible, page 317. and, Achtemeier, Paul J., Th.D., Harper’s Bible Dictionary, (San
Francisco: Harper and Row, Publishers, Inc.) 1985.
hooks that represented values of ten. would equal the number 14 where would be 24. If
one or both of the hooks were smudged or flaked off of a papyrus, then the dates would be off by
values of 10 years.
Does this mean the Bible is not trustworthy? Not at all. Inspiration is ascribed to the original
writings and not to the copies. Scribes made errors. However, the errors were very infrequent and
from other information in the Bible, we can easily ascertain what the correct age is.

Why are the statistics in Ezra 2 and Nehemiah 7 different?


Ezra 2 and Nehemiah 7

Ezra 2 and Nehemiah 7 are listings of numbered people from different families. The chapters
represent the statistics of the same families. But they are not identical. I have produced a grid below
that lists the families that do not match. If you would like to see the grid comparing all families in
Ezra 2 and Nehemiah 7, go here.
Of 39 entries (verses), 17 do not match. They are listed below.

Ezra 2 Nehemiah 7 Difference


5 10
the sons of Arah, 775 the sons of Arah, 652 123
6 11
the sons of Pahath-moab of the the sons of Pahath-moab of the 6
sons of Jeshua and Joab, 2,812 sons of Jeshua and Joab, 2,818
8 13
the sons of Zattu, 945 the sons of Zattu, 845 100
10 15
the sons of Bani, 642 the sons of Binnui, 648 6
11 16
the sons of Bebai, 623 the sons of Bebai, 628 5
12 17
the sons of Azgad, 1,222 the sons of Azgad, 2,322 1,100
13 18
the sons of Adonikam, 666 the sons of Adonikam, 667 1
14 19
the sons of Bigvai, 2,056 the sons of Bigvai, 2,067 11
15 20
the sons of Adin, 454 the sons of Adin, 655 201
17 23
the sons of Bezai, 323 the sons of Bezai, 324 1
28 32
the men of Bethel and Ai, 223 the men of Bethel and Ai, 123 100
33 37
the sons of Lod, Hadid, and Ono, the sons of Lod, Hadid, and Ono, 4
725 721
35 38
the sons of Senaah, 3,630. the sons of Senaah, 3,930. 300
41 44
The singers: the sons of Asaph, The singers: the sons of Asaph, 20
128 148
42 45
The sons of the gatekeepers: the The gatekeepers: the sons of 1
sons of Shallum, the sons of Ater, Shallum, the sons of Ater, the
the sons of Talmon, the sons of sons of Talmon, the sons of
Akkub, the sons of Hatita, the Akkub, the sons of Hatita, the
sons of Shobai, in all 139 sons of Shobai, 138
59
and 60 list several names with 61
and 62 list numerous names 10
one total of 652 with one total of 642
65 67
besides their male and female besides their male and their 45
servants, who numbered 7,337; female servants, of whom there (singers)
and they had 200 singing men were 7,337; and they had 245
and women. male and female singers.
17 discrepancies
It is obvious from the above table, that there were many statistical differences between Ezra and
Nehemiah. Though most of them are identical, some do not match. Why? The answer is simple.

Ezra was written no later than 450 B.C. 74 Nehemiah should be dated during the reign of Artexerxes
1 (464-423 B.C.).75 According to the book, "Talk Thru the Bible," Ezra was written about 538-516
B.C.76 where Nehemiah was written around 444-425 B.C.77 Therefore, the dates of writing are different
and the statistical differences can easily be accounted for by considering that during the difference of
years, people died, families grew, etc.
I must note that there is a small percentage of copyist errors in numbers throughout the Hebrew
Scriptures. It is certainly possible that some of the numbers differ due to copyist mistakes.
Nevertheless, the difference in dates of writing can certainly account for the difference in numbers.

If Job was blameless, why did God allow Satan to afflict him?
Job 1:1 and Job 1:12

1. (Job 1:1) - "There was a man in the land of Uz, whose name was Job, and that man was
blameless, upright, fearing God, and turning away from evil."
2. (Job 1:12) - "Then the Lord said to Satan, "Behold, all that he has is in your power, only do not
put forth your hand on him."

When the Bible says that Job was blameless, it does not mean that he was absolutely sinless. It
means that he was a God-fearing man who sought to do what was right before the Lord. Job's
awareness of his own sins is acknowledged by the fact that he sacrificed animals to the Lord as
atonement for his sins in chapter 1.
As the story goes, the "sons of God", angels, presented themselves before God. Satan was there
and a conversation ensued about Job's goodness. Satan challenges God by stating that Job will
denounce God if afflicted. God gives permission to Satan to afflict Job. Of course, Job doesn't
denounce God. So, the question is why would God allow Satan to do this?
The reason is so that God may be vindicated at His word and so that we might understand that
trials and tribulations will come to those who are godly. In the former, we see the righteousness of
God. After all, none are righteous before God (Rom. 3:10-12). In the latter we see the perfection of
Job's faith (James 1:2-4).

Does God hate people or love them?


John 3:16; Romans 5:8; 1 John 4:7-8
and Psalm 5:5; 11:5; Proverbs 6:16-19; Hosea 9:15

1. God loves
A. (John 3:16) - "For God so loved the world, that He gave His only begotten Son, that
whoever believes in Him should not perish, but have eternal life."
B. (Rom. 5:8) - "But God demonstrates His own love toward us, in that while we were yet
sinners, Christ died for us."
C. (1 John 4:7-8,16)- "Beloved, let us love one another, for love is from God; and everyone
who loves is born of God and knows God. 8The one who does not love does not know God,
for God is love...16God is love, and the one who abides in love abides in God... "
2. God hates

74
Harrison, R. K., Introduction to the Old Testament, (Grand Rapids, Michigan: Eerdmans Publishing Company)
1969, page 1143.
75
Harrison, R. K. page 1146.
76
Wilkinson, Bruce and Boa, Kenneth, Talk Thru the Bible, (New York: Thomas Nelson Publishers) 1983, page 116
77
Wilkinson, Bruce and Boa, Kenneth, page 123.
A. (Psalm 5:5) - "The boastful shall not stand before Thine eyes; Thou dost hate all who do
iniquity."
B. (Psalm 11:5) - "The Lord tests the righteous and the wicked, And the one who loves violence
His soul hates."
C. (Proverbs 6:16-19) - "There are six things which the Lord hates, yes, seven which are an
abomination to Him: 17Haughty eyes, a lying tongue, and hands that shed innocent blood,
18
A heart that devises wicked plans, feet that run rapidly to evil, 19A false witness who utters
lies, and one who spreads strife among brothers."
D. (Hosea 9:15) - "All their evil is at Gilgal; indeed, I came to hate them there! Because of the
wickedness of their deeds I will drive them out of My house! I will love them no more; All
their princes are rebels."

God both loves and hates. His nature is love (1 John 4:8), but He is also righteous (Psalm 7:9)
and holy (Isaiah 6:3). The very fact that He does not incinerate all of humanity for its sin against Him
is due to his loving kindness. God doesn't owe anyone anything. We are sinners and as such, we
have offended Him because we have broken His laws -- and His laws are a reflection of His character.
But, God in His great mercy, sent His Son to die for our sins so that we might have eternal life by
receiving Christ as Savior (John 1:12; Rom. 10:9-10).
Does God hate? Yes. Does God love? Yes. This is not a contradiction. This is simply the truth.

Isaiah 7:14, in Hebrew means maiden, not virgin.


Therefore, it is not a prophecy.
"Therefore the Lord Himself will give you a sign: Behold, a virgin will be
with child and bear a son, and she will call His name Immanuel," (Isaiah
7:14).

Isaiah 7:14 says that a virgin will bear a son. The problem is dealing with the Hebrew word for
virgin, which is "almah." According to the Strong's Concordance it means, "virgin, young woman 1a)
of marriageable age 1b) maid or newly married." Therefore, the word "almah" does not always mean
virgin. The word "occurs elsewhere in the Old Testament only in Genesis 24:43 (”maiden“); Exodus
2:8 (”girl“); Psalm 68:25 (”maidens“); Proverbs 30:19 (”maiden“); Song of Songs 1:3 (”maidens“);
6:8 (”virgins“)."78 Additionally, there is a Hebrew word for virgin: bethulah. If Isaiah 7:14 was meant
to mean virgin instead of young maiden, then why wasn't the word used here?
The LXX is a translation of the Hebrew Scriptures into Greek. This translation was made around
200 B.C. by 70 Hebrew scholars. In Isaiah 7:14, they translated the word "almah" into the Greek
word "parthenos." According to A Greek-English Lexicon of the New Testament and Other Early
Christian Literature,79 parthenos means "virgin." This word is used in the New Testament of the Virgin
Mary (Matt. 1:23; Luke 1:27) and of the ten virgins in the parable (Matt. 25:1, 7, 11). If the Hebrews
translated the word into the Greek word for virgin, then they understood what the Hebrew text meant
here.
Why would the Isaiah choose to use the word almah and not bethulah? It was probably because he
wanted to demonstrate that the virgin would also be a young woman. Is it still a prophecy? Of
course.

78
Walvoord, John F., and Zuck, Roy B., The Bible Knowledge Commentary, (Wheaton, Illinois: Scripture Press
Publications, Inc.) 1983, 1985.
79
Bauer, Walter, Gingrich, F. Wilbur, and Danker, Frederick W., A Greek-English Lexicon of the New Testament
and Other Early Christian Literature, (Chicago: University of Chicago Press) 1979.
Is the Lord good or bad to people?
Psalm 145:9; Lamentations 3:38 and
Isaiah 45:7;Jeremiah 18:11; Ezekiel 20:25,26

1. Good
A. (Psalm 145:9) - "The Lord is good to all, And His mercies are over all His works."
B. (Lamentations 3:38) - "Is it not from the mouth of the Most High That both good and ill go
forth?"
2. Bad
A. (Isaiah 45:6-7) - "I am the Lord, and there is no other, 7The One forming light and creating
darkness, Causing well-being and creating calamity; I am the Lord who does all these."
B. (Jerermiah 18:11) - “So now then, speak to the men of Judah and against the inhabitants of
Jerusalem saying, ‘Thus says the Lord, “Behold, I am fashioning calamity against you and
devising a plan against you. Oh turn back, each of you from his evil way, and reform your
ways and your deeds.”’
C. (Ezekiel 20:26) - "and I pronounced them unclean because of their gifts, in that they caused
all their first-born to pass through the fire so that I might make them desolate, in order that
they might know that I am the Lord."

God's nature is good. He is not evil. He cannot sin and He can do no wrong. All that the Lord
does is right and just. But, that does not mean that we always understand what God does or why He
does it. After all, His thoughts are not our thoughts and His ways are not our ways. So, does God do
good or bad to people?
First of all, good and bad are relative. He is good to all in that He gives them rain, air, food, and
life (Matt. 5:43-48). He is good to all in that He provided His Son as a sacrifice for sin so that we
could escape the judgment to come (John 3:16-17). But, when He sends a plague to wipe out a crop
(Pharaoh in Egypt), is that good or bad? From our perspective, it would seem bad to allow such a
terrible thing to happen, let alone cause it to happen. But since God is not bad, what He does, though
tough to understand sometimes, is right. For example, it was right to send the plague upon the
Egyptians. They were holding the Jewish people prisoner. Was it loving to the Egyptians? Not
really? Was it the right thing to do? Absolutely.
God moves through history carrying out various judgments. Doing so does not mean He is not
good or loving. Consider a judge who is a very kind and forgiving man. When a criminal is found
guilty, he must pass the judgment upon him, even if that punishment is harmful to the criminal. Does
it mean that the judge is not loving or any less loving? Not at all. It means that the judge has acted
righteously, according to the Law.
So too with God. He is right and just. He acts according to Law. The Laws that He has given are a
reflection of His holy and righteous character. That is why it is wrong to lie, steal, etc. To sin against
God is to incur His wrath since that sin is an affront to His holy character. The very fact that He so
often withholds His judgment upon us is a very loving and good thing to do. However, when He does
allow judgment to come through, He is just as good and loving. But, He is exercising His
righteousness for a purpose. In His sovereign will to carry history to its designed conclusion ( Acts
4:28), He delivers righteous judgment to those who oppose Him in sinfulness. This is good and right to
do.
Does God bring good and bad upon people? Yes He does. The problem is our perspective. To us it
sometimes appears as bad. To God it is righteous.
Did Coniah have children or not?
Jeremiah 22:28-30 and Matt. 1:12

1. Childess (Jeremiah 22:28-30) - “Is this man Coniah a despised, shattered jar? Or is he an
undesirable vessel? Why have he and his descendants been hurled out And cast into a land that
they had not known? 29“O land, land, land, hear the word of the Lord! 30“Thus says the Lord,
‘Write this man down childless, A man who will not prosper in his days; For no man of his
descendants will prosper Sitting on the throne of David Or ruling again in Judah.’”
2. Had a son (Matthew 1:12) - "And after the deportation to Babylon, to Jeconiah was born
Shealtiel; and to Shealtiel, Zerubbabel."

Coniah is a shortened form of the spelling of Jehoachin who is also called "Joiachin, Jeconiah,
Jechoniah, and Coniah), one of the last two kings of Judah." 80 Matthew 1:12 states that Jeconiah bore
Shealtiel. If Jeremiah 22:30 is true, then how could a childless man have a child?
Quite simply, the context tells us what is meant by Jeremiah's term "childless." He tells us that
none of his descendants will proper sitting on the throne of David. In Matthew's genealogy, Joconiah
is included. But, Matthew gives the legal line through Solomon down to Joseph. Luke gives the
biological lineage from Mary through Nathan, brother of Solomon, upwards. Therefore, no descendent
of Coniah (Jeconiah) has ever sat on the thrown of David.

Is the Lord omnipotent or not?


Jeremiah 32:27 and Judges 1:19

1. (Jeremiah 32:27) - ""Behold, I am the Lord, the God of all flesh; is anything too difficult for
Me?"
2. (Judges 1:19) - "Now the Lord was with Judah, and they took possession of the hill country;
but they could not drive out the inhabitants of the valley because they had iron chariots."
3. (Titus 1:2) - "in the hope of eternal life, which God, who cannot lie, promised long ages ago,"

In Jeremiah 32:27 God is speaking about His might and sovereignty. God can do anything He
wants to do. In Judges 1:19, the Lord was indeed with Judah, but the fact that Judah could not drive
out the inhabitants of the Land does not mean that God couldn't do it. God often uses people and
takes their failures into account when carrying out His ultimate plan.
Titus 1:2 tells us that God cannot lie. Of course, this is not a contradiction since God cannot violate
His own nature of purity. So when God says he can do anything, He does not mean that He can do
those things that are contrary to what He is.

80
Achtemeier, Paul J., Th.D., Harper’s Bible Dictionary, (San Francisco: Harper and Row, Publishers, Inc.) 1985.
Did God destroy Nineveh or not?
Jonah 3:4 and Jonah 3:10

1. God will destroy Nineveh (Jonah 3:4) - "Then Jonah began to go through the city one day’s
walk; and he cried out and said, "Yet forty days and Nineveh will be overthrown."
2. God won't destroy Nineveh (Jonah 3:10) - "When God saw their deeds, that they turned
from their wicked way, then God relented concerning the calamity which He had declared He
would bring upon them. And He did not do it."

The solution to this apparent contradiction is very simple. When we examine the Bible, we must
look at all of what it says on a subject. If we focus on a small area we will not have the full picture
and cannot see the answer that is so often before us. God says in Jeremiah 18:8, "if that nation
against which I have spoken turns from its evil, I will relent concerning the calamity I planned to bring
on it." We see that the Lord has said He will not bring judgment if that nation turns from its sin.
Nineveh did turn from its sin after Jonah gave them the warning from God.

"Then the people of Nineveh believed in God; and they called a fast and put on
sackcloth from the greatest to the least of them. 6When the word reached the king of
Nineveh, he arose from his throne, laid aside his robe from him, covered himself with
sackcloth, and sat on the ashes. 7And he issued a proclamation and it said, "In Nineveh
by the decree of the king and his nobles: Do not let man, beast, herd, or flock taste a
thing. Do not let them eat or drink water. 8"But both man and beast must be covered
with sackcloth; and let men call on God earnestly that each may turn from his wicked
way and from the violence which is in his hands. 9"Who knows, God may turn and
relent, and withdraw His burning anger so that we shall not perish?" (Jonah 3:5-9).

Therefore, we that because Nineveh repented, God turned from the destruction He had earlier
declared.

Does the Lord change or not?


Malachi 3:6; and Genesis 6:6,7; Exodus 32:14; Jonah 3:10

1. God does not change


A. (Malachi 3:6) - "For I, the Lord, do not change; therefore you, O sons of Jacob, are not
consumed."
2. God does change
A. (Genesis 6:6,7) - "And the Lord was sorry that He had made man on the earth, and He was
grieved in His heart. 7And the Lord said, "I will blot out man whom I have created from the
face of the land, from man to animals to creeping things and to birds of the sky; for I am
sorry that I have made them."
B. (Exodus 32:14) - "So the Lord changed His mind about the harm which He said He would do
to His people."
C. (Jonah 3:10) - "When God saw their deeds, that they turned from their wicked way, then
God relented concerning the calamity which He had declared He would bring upon them. And
He did not do it."

When God says that He does not change, He is speaking about His nature and character. But this
does not mean that He cannot change how He works with people throughout history.
When we see God changing His mind, we are seeing it from a human perspective. Since God
knows all things from all eternity, He as always known the ultimate plan that He would carry out; even
the plan to "change His mind." As we have seen in Jonah's account of Nineveh. They repented and
God relented from the destruction that was to come upon the inhabitants. Of course, God knew this
would happen and instituted the warning to them in order to bring about their repentance. There is no
mystery here.

Why are there different genealogies for Jesus in


Matthew 1 and Luke 3?
Matthew 1:16 - Luke 3:23

Both Matthew 1 and Luke 3 contain genealogies of Jesus. But there is one problem. They are
different. Luke's Genealogy starts at Adam and goes to David. Matthew's Genealogy starts at
Abraham and goes to David. When the genealogies arrive at David, they split with David's sons:
Nathan (Mary's side) and Solomon (Joseph's side).
There is no discrepancy because one genealogy is for Mary and the other is for Joseph. It was
customary to mention the genealogy through the father even though it was clearly known that it was
through Mary.

Adam, the father of Seth, the father of Enosh, the father of Cainan, the father of
Mahaleleel, the father of Jared, the father of Enoch, the father of Methuselah, the father of
Lamech, the father of Noah, the father of Shem, the father of Arphaxad, the father of
Cainan, the father of Shelah, the father of Heber, the father of Peleg, the father of Reu, the
father of Serug, the father of Nahor, the father of Terah, the father of
Abraham, the father of Isaac, the father of Jacob, the father of Judah, the father of Perez,
the father of Hezron, the father of Ram, the father of Admin, the father of Amminadab, the
father of Nahshon, the father of Salmon, the father of Boaz, the father of Obed, the father
of Jesse -- the father of
David
Nathan Solomon
Mattatha Rehoboam
Menna Abijah
Melea Asa
Eliakim Jehoshaphat
Jonam Joram
Joseph Uzziah
Judah Jotham
Simeon Ahaz
Levi Hezekiah
Matthat Manasseh
Jorim Amon
Eliezer Josiah
Joshua Jeconiah
Er Shealtiel
Elmadam Zerubbabel
Cosam Abihud
Addi Eliakim
Melchi Azor
Neri Zadok
Shealtiel Achim
Zerubbabel Eliud
Rhesa Eleazar
Joanan Matthan
Joda Jacob
Josech Joseph
Semein
Mattathias
Maath
Naggai
Hesli
Nahum
Amos
Joseph Adopted Jesus
Mattathias as his own son giving him
all legal rights involving heirship.
Joseph
Jannai
Melchi
Levi
Matthat
Eli
supposedly of Joseph (Mary)
JESUS
Where did the devil take Jesus first, the pinnacle or somewhere else?
Matthew 4:5,8 and Luke 4:5,9

1. (Luke 4:5) - "And he led Him up and showed Him all the kingdoms of the world in a moment of
time."
2. (Matt. 4:5) - "Then the devil took Him into the holy city and had Him stand on the pinnacle of
the temple,"
3. (Luke 4:9) - "And he led Him to Jerusalem and had Him stand on the pinnacle of the temple..."
4. (Matt. 4:8) - "Again, the devil took Him to a very high mountain and showed Him all the
kingdoms of the world and their glory."

There is no contradiction or difficulty here. All we need to do is put the verses from each Gospel
side by side to see the order of events. When we do that, we see that Jesus was led to high places
three times. Take a look at the chart below.
We quickly see that Luke 4:5 is the first mention of Jesus being taken anywhere and it simply says,
"And he [the devil] led Him [Jesus] up and showed Him all the kingdoms of the world in a moment of
time." It isn't until later in Luke 4:9 that Jesus is lead to the pinnacle on top of the temple. Therefore,
the answer is simple. Jesus was "led up" and was shown the kingdoms of the world. Later He was led
to the pinnacle.
To see the entire context of each of the verses, in order, and compared to each other, please see
the grid below.

Event Matthew 4:1-11 Luke 4:1-13


Jesus in 1 Then Jesus was led up by the 1 Jesus, full of the Holy Spirit, returned from
wilderness Spirit into the wilderness to be the Jordan and was led around by the Spirit
tempted by the devil. in the wilderness
40 day 2 And after He had fasted forty 2 for forty days, being tempted by the devil.
fast days and forty nights, He then And He ate nothing during those days, and
became hungry. when they had ended, He became hungry.
turn 3 And the tempter came and said 3 And the devil said to Him, "If You are the
stones to to Him, "If You are the Son of God, Son of God, tell this stone to become bread."
bread command that these stones
become bread."
"not by 4 But He answered and said, "It is 4 And Jesus answered him, "It is written,
bread written, 'MAN SHALL NOT LIVE ON 'MAN SHALL NOT LIVE ON BREAD ALONE.'"
alone" BREAD ALONE, BUT ON EVERY
WORD THAT PROCEEDS OUT OF
THE MOUTH OF GOD.'"
devil 5 And he led Him up and showed Him all the
showed kingdoms of the world in a moment of time.
Jesus the
kingdoms
devil to 6 And the devil said to Him, "I will give You
give all this domain and its glory; for it has been
domain to handed over to me, and I give it to
Jesus whomever I wish.
false 7 "Therefore if You worship before me, it
worship shall all be Yours."
true 8 Jesus answered him, "It is written, 'YOU
worship SHALL WORSHIP THE LORD YOUR GOD AND
SERVE HIM ONLY.' "
devil took 5 Then the devil took Him into the 9 And he led Him to Jerusalem and had Him
Jesus to holy city and had Him stand on the stand on the pinnacle of the temple, and said
pinnacle pinnacle of the temple, to Him, "If You are the Son of God, throw
of temple 6 and said to Him, "If You are the Yourself down from here;
Son of God, throw Yourself down; 10 for it is written, 'HE WILL COMMAND HIS
throw for it is written, 'HE WILL ANGELS CONCERNING YOU TO GUARD YOU,'
yourself COMMAND HIS ANGELS 11 and, 'ON their HANDS THEY WILL BEAR
down CONCERNING YOU'; and 'ON their YOU SO THAT YOU WILL NOT STRIKE YOUR
HANDS THEY WILL BEAR YOU UP, FOOT AGAINST A STONE."
SO THAT YOU WILL NOT STRIKE
YOUR FOOT AGAINST A STONE.'"
don't test 7 Jesus said to him, "On the other 12 And Jesus answered and said to him, "It is
God hand, it is written, 'YOU SHALL NOT said, 'YOU SHALL NOT PUT THE LORD YOUR
PUT THE LORD YOUR GOD TO THE GOD TO THE TEST.' "
TEST.' "
took 8 Again, the devil took Him to a
Jesus to very high mountain and showed
high Him all the kingdoms of the world
mountain and their glory;
devil to 9 and he said to Him, "All these
give things I will give You, if You fall
domain down and worship me."
worship 10 Then Jesus said to him, "Go,
and serve Satan! For it is written, 'YOU SHALL
God only WORSHIP THE LORD YOUR GOD,
AND SERVE HIM ONLY.' "
11 Then the devil left Him; and 13 When the devil had finished every
behold, angels came and began to temptation, he left Him until an opportune
minister to Him. time.
Where did Jesus first meet Simon Peter and Andrew?
Matthew 4:18-19 and John 1:42-43

There is no contradiction here at all. The chronology of events becomes evident when viewing
the grid below. In John 1:35ff, John the Baptist was with his disciples. He tells them that Jesus is the
"Lamb of God" (v. 36) and they stay with Jesus a while (v. 37). In John 1:40-42, Andrew goes and
gets Simon-Peter and they follow Jesus. Why? Because Andrew had spent time with Jesus and told
Simon that Jesus was the Messiah (v. 41). Later, in Matt. 4:18, Jesus was walking by the Sea of
Galilee and he sees Simon-Peter and Andrew as they were fishing. He had not asked them to follow
Him until this point in verse 4:19. They do (4:20). There is no contradiction.

Event Matthew 4:18-20 John 1:35-42


John Baptist 35 Again the next day John was
sees Jesus standing with two of his disciples,
"Lamb of 36 and he looked at Jesus as He
God" walked, and said, "Behold, the
Lamb of God!"
John tells his 39 He said to them, "Come, and
disciples to you will see." So they came and
come see the saw where He was staying; and
lamb. they stayed with Him that day, for
it was about the tenth hour.
Andrew 40 One of the two who heard John
followed speak and followed Him, was
Jesus. Andrew, Simon Peter's brother.
First, Andrew 41 He found first his own brother
got Simon and said to him, "We have
Simon/Peter found the Messiah " (which
translated means Christ ).
Andrew 42 He brought him to Jesus. Jesus
brings looked at him and said, "You are
Simon to Simon the son of John; you shall
Jesus be called Cephas " (which is
translated Peter ).
Jesus by 18 Now as Jesus was walking by
sea of the Sea of Galilee, He saw two
Galilee, brothers, Simon who was called
sees Peter, and Andrew his brother,
Andrew casting a net into the sea; for
they were fishermen.
Jesus says to 19 And He said to them, "Follow
Andrew and Me, and I will make you fishers
Simon, of men."
"Follow 4:20 Immediately they left their
Me..." nets and followed Him.
Who did Jesus tell the Lord's Prayer to?
Matthew 5:1; 6:9-13; 7:28 and Luke 11:1-4

I fail to see even why these verses are sometimes viewed as a problem. Matthew 5 begins the
beatitudes and later in them, Jesus teaches the multitude how to pray in Matt. 6:6-13. In another
instance (Luke 11:1-4), the disciples ask Him to teach them and He does. There is certainly no time
conflict at all. Jesus can teach the same thing more than once.

Matthew 5:1-3; 6:9-13 Luke 11:1-4


1 When Jesus saw the crowds, He went up 1 It happened that while Jesus was
on the mountain; and after He sat down, praying in a certain place, after He had
His disciples came to Him. finished, one of His disciples said to
2 He opened His mouth and began to Him, "Lord, teach us to pray just as
teach them, saying, John also taught his disciples."
3 "Blessed are the poor in spirit, for theirs
is the kingdom of heaven, etc.

--- Matt. 5:1 begins the beatitudes that go


through to Matt. 7:29
9 "Pray, then, in this way: 'Our Father who 2 And He said to them, "When you pray,
is in heaven, Hallowed be Your name. say: 'Father, hallowed be Your name.
10 'Your kingdom come. Your will be done, Your kingdom come.
On earth as it is in heaven. 3 'Give us each day our daily bread.
11 'Give us this day our daily bread. 4 'And forgive us our sins, For we
12 'And forgive us our debts, as we also ourselves also forgive everyone who is
have forgiven our debtors. indebted to us. And lead us not into
13 'And do not lead us into temptation, but temptation.' "
deliver us from evil. [For Yours is the
kingdom and the power and the glory
forever. Amen.]'
Should or should we not let our good works be seen?
Matthew 5:16; I Peter 2:12 and Matthew 6:1-4; Matthew 23:3,5

1. Show good works


A. (Matthew 5:16) - "Let your light shine before men in such a way that they may see your
good works, and glorify your Father who is in heaven."
B. (1 Peter 2:12) - "Keep your behavior excellent among the Gentiles, so that in the thing in
which they slander you as evildoers, they may because of your good deeds, as they observe
them, glorify God in the day of visitation."
2. Don't show good works
A. (Matthew 6:1-4) - "Beware of practicing your righteousness before men to be noticed by
them; otherwise you have no reward with your Father who is in heaven. 2So when you give
to the poor, do not sound a trumpet before you, as the hypocrites do in the synagogues and
in the streets, so that they may be honored by men. Truly I say to you, they have their
reward in full. 3"But when you give to the poor, do not let your left hand know what your
right hand is doing, 4so that your giving will be in secret; and your Father who sees what is
done in secret will reward you."
B. ( Matthew 23:3,5) - "therefore all that they tell you, do and observe, but do not do
according to their deeds; for they say things and do not do them...But they do all their
deeds to be noticed by men; for they broaden their phylacteries and lengthen the tassels of
their garments."

As with any piece of literature, to best understand its statements you must read them in context.
The Bible is no different. Matthew 5:16 is in the context of the beatitudes (Matt. 5:1 - 7:29) where
Jesus is teaching proper, good, and moral behavior. Disciples of Jesus are to be lights; that is, doers
of good. 1 Peter 2:12 is where Peter is admonishing the Christians to live godly and holy lives before
the unbelievers. We Christians live in the world among unbelievers and they are going to see how we
live. Peter is telling us to act properly with unbelievers so that false accusations will not stand against
us because we have lived with integrity among them.
By contrast, Jesus in Matthew 6:1-4, which is still in the same beatitudes where He told people to
let their light shine before people (5:16) is stating that if the motive of doing something good is to be
noticed by people, then don't do that. Don't boast about how "good" you are before people. That is
wrong. There is nothing wrong with doing good works that will be seen before people, after all, we live
among unbelievers. But, when you do good things, don't do them for the purpose of drawing attention
to how "good" you are.
Likewise in Matthew 23:3,5 Jesus is addressing the crowds and teaching them about the hypocrisy
of the Scribes and Pharisees and how they do their deeds in order to be noticed and admired. Jesus
condemned this as is right.
People are supposed to notice your good works because your good character permeates them, not
because of your attempt to have them see how "good" and "great" you are. The former is humility.
The latter is prideful and wrong. The context of each verse tells us this.
Can we call someone a fool or not?
Matthew 5:22 and Matthew 23:17; Psalm 14:1

1. Do not call someone a fool


A. (Matthew 5:22) - "But I say to you that everyone who is angry with his brother shall be
guilty before the court; and whoever says to his brother, 'You good-for-nothing,' shall be
guilty before the supreme court; and whoever says, 'You fool,' shall be guilty enough to go
into the fiery hell."
2. Calling someone a fool
A. (Psalm 14:1) - "The fool has said in his heart, "There is no God." They are corrupt, they
have committed abominable deeds; There is no one who does good"
B. (Matthew 23:17) - "You fools and blind men! Which is more important, the gold or the
temple that sanctified the gold?"

When Jesus said in Matthew 5:22 that you should not call anyone a fool, contextually, He was
speaking of those who were unrighteously angry. That is why Jesus mentions anger in this verse.
There is a righteous anger which is not sinful (Eph. 4:26 - "Be angry and do not sin . . ." ) as well as
unrighteous anger that is sinful (James 1:20 - "for the anger of man does not achieve the
righteousness of God."). When God is angry with someone, He is always righteous in His anger.
Jesus, being God in flesh (John 1:1,14; 20:28; Col. 2:9;), can righteously be angry with people and
pronounce upon them the foolishness of their deeds which He did (Matt. 23:17). Also, undoubtedly,
Jesus knew Psalm 14:1 which says, "The fool has said in his heart, "There is no God . . ." Jesus didn't
forget the well known verse and God is not wrong for calling someone a fool, especially when it is true.
So, we see that the condemnation by Jesus in regards to calling someone a fool is in the context of
doing it out of unrighteous anger which does not fit the later citations of Jesus labeling the hypocritical
Pharisees as fools.

Who brought the Centurion's request to Jesus?


Matthew 8:5-13 and Luke 7:2-10

It clearly states that the Centurion came to Jesus in Matt. 8:5. But it also says that the Jewish
elders came to Jesus also. The order of events seems to be that the Centurion first sent the Jewish
elders (Luke 7:3). Jesus then agreed to go. Then the Centurion came to Jesus (Matt. 8:5). Jesus
walked everywhere he went. Centurions commanded hundred-man groups in the Roman legion. "Such
men were prestigious members of a relatively small class governing the military." 81 Therefore, the
centurion most probably had a horse upon which to ride to and from Jesus. If this is so, then he
probably returned to his home, checked on the servant and then sent friends (Luke 7:6) to speak to
Jesus saying that the Centurion was not worthy for Jesus to even enter his home. Jesus continued
on. Then, as Jesus neared the home, the Centurion himself approached Jesus (Matt. 8:8) to tell Jesus
that he was not worthy for Him to enter his house.

81
Achtemeier, Paul J., Th.D., Harper’s Bible Dictionary, (San Francisco: Harper and Row, Publishers, Inc.) 1985.
Event Matthew 8:5-13 Luke 7:2-10
The slave is 2 And a certain centurion’s slave, who was
sick highly regarded by him, was sick and about
to die.
Jewish elders 3 And when he heard about Jesus, he sent
entreat Jesus some Jewish elders asking Him to come and
save the life of his slave.
4 And when they had come to Jesus, they
earnestly entreated Him, saying, "He is
worthy for You to grant this to him;
5 for he loves our nation, and it was he who
built us our synagogue."
Centurion 5 And when He had entered Capernaum,
approaches a centurion came to Him, entreating Him,
Jesus 6 and saying, "Lord, my servant is lying
paralyzed at home, suffering great pain."
Jesus goes 7 And He *said to him, "I will come and
heal him."
Centurion 6 Now Jesus started on His way with them;
sent friends and when He was already not far from the
to Jesus house, the centurion sent friends, saying to
Him, "Lord, do not trouble Yourself further,
for I am not worthy for You to come under
my roof;
7 for this reason I did not even consider
myself worthy to come to You, but just say
the word, and my servant will be healed.
Centurion 8 But the centurion answered and said,
says he is not "Lord, I am not worthy for You to come
worthy under my roof, but just say the word, and
my servant will be healed.
the Centurion 9 "For I, too, am a man under authority, 8 "For I, too, am a man under authority,
speaks of with soldiers under me; and I say to this with soldiers under me; and I say to this
authority one, ‘Go!’ and he goes, and to another, one, ‘Go!’ and he goes; and to another,
‘Come!’ and he comes, and to my slave, ‘Come!’ and he comes; and to my slave, ‘Do
‘Do this!’ and he does it." this!’ and he does it."
Jesus' words 10 Now when Jesus heard this, He 9 Now when Jesus heard this, He marveled
and the marveled, and said to those who were at him, and turned and said to the
servant is following, "Truly I say to you, I have not multitude that was following Him, "I say to
healed. found such great faith with anyone in you, not even in Israel have I found such
Israel. great faith."
11 And I say to you, that many shall
come from east and west, and recline at
the table with Abraham, and Isaac, and
Jacob, in the kingdom of heaven;
12 but the sons of the kingdom shall be
cast out into the outer darkness; in that
place there shall be weeping and gnashing
of teeth."
13 And Jesus said to the centurion, "Go 10 And when those who had been sent
your way; let it be done to you as you
have believed." And the servant was returned to the house, they found the slave
healed that very hour." in good health.

Was the taxman named Matthew or Levi?


Matthew 9:9 and Mark 2:14; Luke 5:27

1. Matthew (Matthew 9:9) - "And as Jesus passed on from there, He saw a man, called Matthew,
sitting in the tax office; and He *said to him, "Follow Me!" And he rose, and followed Him."
2. Levi (Mark 2:14) - "And as He passed by, He saw Levi the son of Alphaeus sitting in the tax
office, and He *said to him, "Follow Me!" And he rose and followed Him."
3. Levi (Luke 5:27) - "And after that He went out, and noticed a tax-gatherer named Levi, sitting
in the tax office, and He said to him, "Follow Me."

The answer is very simple. Both are true because Matthew and Levi are the same person. Matthew
is the Greek name and Levi was the Hebrew name. As a tax-collector, Matthew worked for the
Romans who spoke Greek. He gathered taxes from the Jews who spoke Hebrew. We see, as an
example, Peter also being called Simon (Matt. 16:16).

Was Jairus' daughter alive or dead when he came to Jesus?


Matthew 9:18 and Mark 5:23

1. Daughter was dead (Matthew 9:18) - "While He was saying these things to them, behold,
there came a synagogue official, and bowed down before Him, saying, "My daughter has just
died; but come and lay Your hand on her, and she will live."
2. Daughter at point of death (Mark 5:23) - "And one of the synagogue officials named Jairus
*came up, and upon seeing Him, *fell at His feet, 23and *entreated Him earnestly, saying, "My
little daughter is at the point of death; please come and lay Your hands on her, that she may
get well and live." 24And He went off with him; and a great multitude was following Him and
pressing in on Him."

This is a difficult discrepancy. There are no textual variations in any of the Greek manuscripts so
there is no copyist error. The best explanation I can offer is that both statements were uttered by
Jairus. Jairus probably said his daughter was dying and then finally admitted that she was dead. He
probably had traveled, in desperation, to find Jesus and said that his daughter was ill. Most probably,
someone informed in that his daughter had died and then he told Jesus this.
The problem with this explanation is that it is not based on anything revealed in the text. Instead,
it is conjecture. For now, this is the best explanation I can come up with.
Does the lack of an airtight explanation mean that the Bible is to be discarded? Not at all. We do
not discard physics because we cannot understand something in it. Nor do we abandon science
because of some unexplainable phenomena. As with so many other "discrepancies" in the Bible, given
time, archaeological discover of more manuscripts, etc., more and more biblical difficulties clear up.
When did John find out Jesus was the Messiah?
Matthew 11:2-3; Luke 7:18-22 John 1:29-34,36

1. In prison (Matthew 11:2) - "Now when John in prison heard of the works of Christ, he sent
word by his disciples, 3and said to Him, "Are You the Expected One, or shall we look for
someone else?"
2. In prison (Luke 7:18-21) - "And the disciples of John reported to him about all these things.
19
And summoning two of his disciples, John sent them to the Lord, saying, "Are You the
Expected One, or do we look for someone else?" 20And when the men had come to Him, they
said, "John the Baptist has sent us to You, saying, ‘Are You the Expected One, or do we look for
someone else?’" 21At that very time He cured many people of diseases and afflictions and evil
spirits; and He granted sight to many who were blind."
3. While baptizing (John 1:29-31) - "The next day he *saw Jesus coming to him, and *said,
"Behold, the Lamb of God who takes away the sin of the world! 30"This is He on behalf of whom
I said, ‘After me comes a Man who has a higher rank than I, for He existed before me.’ 31"And I
did not recognize Him, but in order that He might be manifested to Israel, I came baptizing in
water."

Matthew, Mark, and Luke are known as the synoptic gospels. Synoptic means "similar." They are
very similar in their approach to writing about Jesus. They are more chronological than John is and
contain a majority of common information. The intent of the Gospel of John was not to present a
detailed chronological account of Jesus' life. Rather, the intent was to demonstrate that Jesus is the
Son of God (John 20:31). John is thematic in emphasis and he brings out those issues that
demonstrate Christ's divine nature (John 1:1,14;8:24,58; 10:30-33; 20:28).

When we read the accounts of the Synoptic Gospels, we see that they mesh very well. John, on
the other hand, ignores the details of the chronology and simply presents Jesus as the Christ, picking
and choosing the issues in Christ's life in order to demonstrate his goal. For example, John doesn't
even mention that John the Baptists was arrested as do the other gospels (Matt. 4:12; Mark 1:14;
Luke 3:19-20). Why? It isn't the intent of John to present the chronological events about John the
Baptist. It is meant to convince people that Jesus is the Son of God (John 20:31). Therefore, what we
see in John is a synopsis of the prophetic revelation of God regarding Jesus as the Lamb of God who
takes away the sins of the world. John, therefore, summarizes Jesus' baptism account in order to
portray Him as the Christ.
Strictly speaking, if we were to force a chronological harmony of Matthew, Luke, and John, we
would see something like the chart below.

Matthew Luke John 1


Event
Jesus comes Matt. 3:13, Then "The next day he *saw Jesus coming to him, and
to John. Jesus *arrived from *said, "Behold, the Lamb of God who takes away
Galilee at the Jordan the sin of the world! 30"This is He on behalf of
John says coming to John, to whom I said, ‘After me comes a Man who has a
Jesus is the be baptized by him. higher rank than I, for He existed before me.’
31
Lamb of God. "And I did not recognize Him, but in order that He
might be manifested to Israel, I came baptizing in
water." 32And John bore witness saying, "I have
beheld the Spirit descending as a dove out of
heaven, and He remained upon Him. 33"And I did
not recognize Him, but He who sent me to baptize
in water said to me, ‘He upon whom you see the
Spirit descending and remaining upon Him, this is
the one who baptizes in the Holy Spirit.’ 34"And I
have seen, and have borne witness that this is the
Son of God."

Jesus is 3:16 And after being


baptized baptized, Jesus went
up immediately from
the water; and
behold, the heavens
were opened, and he
saw the Spirit of God
descending as a
dove, and coming
upon Him,
John is 4:12 Now when He 3:20 he added
arrested heard that John had this also to
been taken into them all, that
custody, He withdrew he [Herod]
into Galilee; locked John up
in prison.
John in 11:2 "Now when 7:18 "And the
prison heard John in prison heard disciples of
of Christ's of the works of John reported
works Christ, he sent word to him about
by his disciples, all these
things.
"Are you the 11:3 and said to 7:19 And
One? Him, "Are You the summoning
Expected One, or two of his
shall we look for disciples, John
someone else?" sent them to
the Lord,
saying, "Are
You the
Expected One,
or do we look
for someone
else?" 20And
when the men
had come to
Him, they said,
"John the
Baptist has
sent us to You,
saying, ‘Are
You the
Expected One,
or do we look
for someone
else?’"
Was John the Baptist really Elijah?
Matthew 11:13-14 and John 1:19-21

1. Yes, he was Elijah (Matthew 11:13-14) - "For all the prophets and the Law prophesied until
John. 14"And if you care to accept it, he himself is Elijah, who was to come."
2. No, he was not Elijah (John 1:19-21) - "And this is the witness of John, when the Jews sent
to him priests and Levites from Jerusalem to ask him, "Who are you?" 20And he confessed, and
did not deny, and he confessed, "I am not the Christ." 21And they asked him, "What then? Are
you Elijah?" And he *said, "I am not." "Are you the Prophet?" And he answered, "No."

The teaching of reincarnation is against the Lld Testament. Therefore, Jesus was not teaching that
John the Baptist was Elijah reincarnated. So, what did Jesus mean when He said that John the Baptist
was Elijah? We see in Malachi 4:5 this prophecy, "Behold, I am going to send you Elijah the prophet
before the coming of the great and terrible day of the Lord." Jesus is referring to the prophecy
concerning Elijah. We see that the coming of Elijah was in the spirit of Elijah, which is so stated in
Luke 1:13-17. The context is when Zecharias, John's father-to-be, was performing his priestly duties
in the temple (Luke 1:8ff). An angel of the Lord appeared to Zacharias and said,

"Do not be afraid, Zacharias, for your petition has been heard, and your wife Elizabeth
will bear you a son, and you will give him the name John. 14"And you will have joy and
gladness, and many will rejoice at his birth. 15"For he will be great in the sight of the
Lord, and he will drink no wine or liquor; and he will be filled with the Holy Spirit, while
yet in his mother’s womb. 16"And he will turn back many of the sons of Israel to the Lord
their God. 17"And it is he who will go as a forerunner before Him in the spirit and power
of Elijah, to turn the hearts of the fathers back to the children, and the disobedient to
the attitude of the righteous; so as to make ready a people prepared for the Lord,"
(Luke 1:13-17).

So, we see that John the Baptist was in the spirit of Elijah, but not actually Elijah reincarnated.
There is, however, a little more information that might prove interesting. Elijah wore, most
probably, a camel's hair girdle. "And they answered him, "He was a hairy man with a leather girdle
bound about his loins." And he said, "It is Elijah the Tishbite," ( 2 Kings 1:8). According to the
Treasury of Scripture Knowledge, in reference to 2 Kings 1:8, Elijah . . .

"he wore a rough garment, either made of camels’ hair, as that of John Baptist, or of a
skin, dressed with the hair on. Sir J. Chardin informs us, in a MS. note on this place,
cited by Mr. Harmer, that the eastern dervishes and fakeers are clothed just as Elijah
was, with a hairy garment, girded with a leathern girdle."

Concerning John the Baptist, it says in Matthew 3:4, "Now John himself had a garment of camel’s
hair, and a leather belt about his waist; and his food was locusts and wild honey." It may be that
Zecharias, who had access to the temple and things in the temple, may have acquired Elijah's camel
hair garment and given it to John the Baptist to wear. This is speculation, but it is an interesting
possibility.
How long was Jesus in the tomb?
Matthew 12:40 and Matthew 28:1; Mark 16:2; Luke 24:1; John 20:1

1. Three days and three nights


A. (Matthew 12:40) - "for just as Jonah was three days and three nights in the belly of the sea
monster, so shall the Son of Man be three days and three nights in the heart of the earth."
2. Less than three days and three nights
A. (Matt. 28:1) - "Now after the Sabbath [SABBATHS -PLURAL], as it began to dawn toward
the first day of the week, Mary Magdalene and the other Mary came to look at the grave."
B. (Mark 16:2) - "And very early on the first day of the week, they *came to the tomb when
the sun had risen."
C. (Luke 24:1) - "But on the first day of the week, at early dawn, they came to the tomb,
bringing the spices which they had prepared."
D. (John 20:1) - "Now on the first day of the week Mary Magdalene *came early to the tomb,
while it *was still dark, and *saw the stone already taken away from the tomb."

The Jewish day was measured from sun down to sun down. If Jesus was in the grave for three 24
hour periods, then He could not have been raised on the third day because the third day had not yet
been completed. He would have to be raised on fourth day for three 24 hour periods to have been
completed and that wouldn't make sense to then say He was raised on the third day. So, what is
going on?

DAY 1 DAY2 DAY 3


THU THU FRI FRI SAT SAT SUN SUN
starts at ends at starts at ends at starts at ends at starts at ends at
sundown sundown sundown sundown sundown sundown sundown sundown
on Wed. on Thu.. on Fri. on Sat.
Night Day Night Day Night Day Night Day
Crucifixion Sabbath He rose

The solution is simple when we learn that according to Jewish custom any part of a day, however
small, is included as part of a full day. 82 "Since the Jews reckoned part of a day as a full day, the
“three days and three nights” could permit a Friday crucifixion." 83 This phenomena is exemplified in
scripture in the book of Esther. "Go, assemble all the Jews who are found in Susa, and fast for me; do
not eat or drink for three days, night or day. I and my maidens also will fast in the same way," (Esther
4:16 ). Then, in Esther 5:1 it says, "Now it came about on the third day that Esther put on her royal
robes and stood in the inner court of the king’s palace in front of the king’s rooms, and the king was
sitting on his royal throne in the throne room, opposite the entrance to the palace." We can see that
even though the three days and nights had not been completed, Esther went in to see the King on the
third day even though she said to fast for three days and nights. We see that "on the third day" is

82
Jamieson, Robert; Fausset, A.R.; and Brown, David, Commentary Critical and Explanatory on the
Whole Bible, (Oak Harbor, WA: Logos Research Systems, Inc.) 1998.
83
And also, Walvoord, John F., and Zuck, Roy B., The Bible Knowledge Commentary, 1983, 1985,
equivalent to "after three days."
Additionally, Mark 8:31 says, "And He began to teach them that the Son of Man must suffer
many things and be rejected by the elders and the chief priests and the scribes, and be killed, and
after three days rise again." Yet, 1 Cor. 15:4 says, "and that He was buried, and that He was raised
on the third day according to the Scriptures." Also, Luke 24:5-7, "and as the women were terrified
and bowed their faces to the ground, the men said to them, "Why do you seek the living One among
the dead? 6"He is not here, but He has risen. Remember how He spoke to you while He was still in
Galilee, 7saying that the Son of Man must be delivered into the hands of sinful men, and be crucified,
and the third day rise again." Here we can see that "after three days" is equivalent to mean "on the
third day."
Therefore, we can see that because of the Jewish usage of counting any part of a day as the whole
of the day, the term "three days and nights" is idiomatic and not literal.

Another possible solution

‘In the first month, on the fourteenth day of the month at twilight is the Lord’s Passover.
6
‘Then on the fifteenth day of the same month there is the Feast of Unleavened Bread to
the Lord; for seven days you shall eat unleavened bread. 7‘On the first day you shall
have a holy convocation; you shall not do any laborious work," (Lev. 23:5-7).

The verses above tell us that the Passover occurred on the 14th day of the first month of the Jewish
Calendar year; this corresponds to our months of March-April. It is possible, then, that this Passover
could have occurred during the week with the Saturday Sabbath following. Since Lev. 23:5-7 tells the
people to rest on the first day (not the last day Saturday), this is a type of Sabbath occurrence.
Therefore, perhaps the following chart could represent a Thursday crucifixion and a subsequent set of
three "night and days" before the Sunday resurrection.

Day 1 Day 2 Day 3


13th of Nisan 14th of Nisan 15th of Nisan 16th of Nisan
THU THU FRI FRI SAT SAT SUN SUN
starts at ends at starts at ends at starts at ends at starts at ends at
sundown sundown sundown sundown sundown sundown sundown sundown
on Wed. on Thu.. on Fri. on Sat.
Night Day Night Day Night Day Night Day
Passover/Crucifixion Sabbath He rose

Something worth mentioning concerning this is that in the Greek in Matthew. 28:1, it says "Now
after the Sabbaths [PLURAL], as it began to dawn toward the first day of the week, Mary Magdalene
and the other Mary came to look at the grave." It is possible that there may have been two "sabbaths"
during that week. The first may have been the Passover related "Sabbath" and the second may have
been the Saturday Sabbath. If we look at Lev. 23:5-7 quoted above it says "On the first day you shall
have a holy convocation; you shall not do any laborious work."
Another clue that this may be the case is found in a comparison of two more verses that I have
arrange in a before-and-after-the-Sabbath pattern.

"And they returned and prepared "And when the Sabbath was
spices and perfumes. And on the over, Mary Magdalene, and Mary
Sabbath they rested according to the mother of James, and
the commandment," SABBATH Salome, bought spices, that
(Luke 23:56 ). they might come and anoint
Him,"
(Mark 16:1).
I would need to do more research on this, but it is possible that this arrangement can also be an
answer to the three nights and three days scenario.

After how many days did Jesus take the three men up the mountain?
Matthew 17:1; Mark 9:2 and Luke 9:28

1. Six Days later


A. (Matthew 17:1) - "And six days later Jesus *took with Him Peter and James and John
his brother, and *brought them up to a high mountain by themselves."
B. (Mark 9:2) - "And six days later, Jesus *took with Him Peter and James and John, and
*brought them up to a high mountain by themselves. And He was transfigured before
them;"
2. Eight Days later

A. (Luke 9:28-29) - "And some eight days after these sayings, it came about that He took
along Peter and John and James, and went up to the mountain to pray. 29And while He
was praying, the appearance of His face became different, and His clothing became
white and gleaming."

In the Greek in both Matthew 17:1 and Mark 9:2, it says, "And after six days..." The word "after"
in Greek is "meta." According to the Enhanced Strong’s Lexicon "meta" means, "with, after, or
behind." In Luke 9:28, it says something different. It says "And some eight days after these
sayings . . ." (NASB). The Greek word "some" is "hosei" which means "about" or "nearly." Other
translations render it the same way.

• "About eight days after Jesus said this . . ." (Luke 9:28, NIV).
• ". . .about an eight days after these sayings . . ." (Luke 9:28, KJV).
• ". . .about eight days after these sayings . . ." (Luke 9:28, NKJV).
• "Now about eight days after these sayings . . ." (Luke 9:28, RSV).
• ". . . about eight days after these sayings . . ." (Luke 9:28, 1901 AS)

Luke 9:28 is an approximation evidenced by it saying "about eight days after . . ." Matthew 17:1
and Mark 9:2 are more precise. They say "after six days." Logically, eight days is after six days, so
there is no logical contradiction. But, the key lies in Luke saying "about eight days later." Luke was
giving an approximation. Matthew and Mark were more precise.

Who made the request to sit beside Jesus in His kingdom?


Matthew 20:20-21 and Mark 10:35-37

1. (Matthew 20:20-21) - "Then the mother of the sons of Zebedee came to Him with her sons,
bowing down, and making a request of Him. 21And He said to her, "What do you wish?" She
*said to Him, "Command that in Your kingdom these two sons of mine may sit, one on Your
right and one on Your left."
2. (Mark 10:35-37) - "And James and John, the sons of Zebedee, come unto him, saying, Master,
we would that thou shouldest do for us whatsoever we shall desire. 36And he said unto them,
What would ye that I should do for you? 37They said unto him, Grant unto us that we may sit,
one on thy right hand, and the other on thy left hand, in thy glory."

Both are correct. Most probably, the mother first approached Jesus and asked Him about her sons.
Later, they approached Jesus with the same question. Neither statement excludes the possibility of
the other.

Did Jesus cleanse the Temple on the first or second day?


Matthew 21:12; Mark 11:11-12,15-16; Luke 19:45

1. (Matthew 21:12) - "And Jesus entered the temple and cast out all those who were buying and
selling in the temple, and overturned the tables of the moneychangers and the seats of those
who were selling doves."
2. (Mark 11:11-12,15-16) - "And He entered Jerusalem and came into the temple; and after
looking all around, He departed for Bethany with the twelve, since it was already late. 12And on
the next day, when they had departed from Bethany, He became hungry. 15And they *came to
Jerusalem. And He entered the temple and began to cast out those who were buying and
selling in the temple, and overturned the tables of the moneychangers and the seats of those
who were selling doves; 16and He would not permit anyone to carry goods through the temple."
3. (Luke 19:45 ) - " And He entered the temple and began to cast out those who were selling."

There are two possible explanations. The first is that Matthew and Mark purposely arranged their
material in different patterns. Matthew is more topical in his presentation of the material than is
Mark. Therefore, it may be that Matthew condensed the information into a thematic arrangement.
The other explanation is that Jesus cleansed the temple twice. This is possible, but is disputed.
For more information on this, go to the Chronology of the Temple Cleansing and see a possible
arrangement of the gospels.

Did the cock crow once or twice before Peter's third denial?
Matthew 26:34-35,74-75; Luke 22:34,60-62;
John 13:38 and Mark 14:30

1. Denies before cock crows


A. (Mathew 26:34,74-75) - "Jesus said to him, "Truly I say to you that this very night, before a
cock crows, you shall deny Me three times . . . 74Then he began to curse and swear, "I do
not know the man!" And immediately a cock crowed. 75And Peter remembered the word
which Jesus had said, "Before a cock crows, you will deny Me three times." And he went out
and wept bitterly."
B. (Luke 22:34,60-62) - "And He said, "I say to you, Peter, the cock will not crow today until
you have denied three times that you know Me . . . . 60But Peter said, "Man, I do not know
what you are talking about." And immediately, while he was still speaking, a cock crowed.
61
And the Lord turned and looked at Peter. And Peter remembered the word of the Lord, how
He had told him, "Before a cock crows today, you will deny Me three times." 62And he went
out and wept bitterly."
C. (John 13:38) - "Jesus *answered, "Will you lay down your life for Me? Truly, truly, I say to
you, a cock shall not crow, until you deny Me three times."
2. before cock crows twice
A. (Mark 14:30) - "And Jesus *said to him, "Truly I say to you, that you yourself this very
night, before a cock crows twice, shall three times deny Me..."

If a cock crows a second time, then it has crowed once before. The problem is that in Mark, after
Peter denies the Lord for the third time (Mark 14:71), immediately a cock crows a second time (v.
72). The other gospels tell us that after Peter's third denial a cock then crows. How do we reconcile
this difficulty?
Mark does not mention when the cock crowed the first time. Therefore, it is possible that after
Peter's third denial, the cock then crowed twice; that is, two times in a row. This is logically possible.

Peter's Denial of Christ


Mathew 26:34-35, Mark 14:30, Luke 22:34, John 13:38
Event
69-75 66-72 55-62 & 18:25-27
Jesus 34 Jesus said to him, "Truly I 30 And Jesus *said to him, 34 And He said, "I say to you, 38 Jesus *answered, "Will
predicts say to you that this very "Truly I say to you, that you Peter, the cock will not crow you lay down your life for
Peter's night, before a cock crows, yourself this very night, today until you have denied Me? Truly, truly, I say to
denial you shall deny Me three before a cock crows twice, three times that you know Me." you, a cock shall not crow,
times." shall three times deny Me." until you deny Me three
35 Peter *said to Him, "Even times.
if I have to die with You, I will
not deny You." All the
disciples said the same thing
too.
first 69 Now Peter was sitting 66 And as Peter was below in 55 And after they had kindled a 18:25a Now Simon Peter
inquiry outside in the courtyard, and the courtyard, one of the fire in the middle of the was standing and warming
a certain servant-girl came to servant-girls of the high priest courtyard and had sat down himself. They said therefore
him and said, "You too were *came, together, Peter was sitting to him, "You are not also one
with Jesus the Galilean." 67 and seeing Peter warming among them. of His disciples, are you?"
himself, she looked at him, 56 And a certain servant-girl,
and *said, "You, too, were seeing him as he sat in the
with Jesus the Nazarene." firelight, and looking intently at
him, said, "This man was with
Him too."
1st 70 But he denied it before 68 But he denied it, saying, "I 57 But he denied it, saying, 25b He denied it, and said, "I
denial them all, saying, "I do not neither know nor understand "Woman, I do not know Him." am not."
know what you are talking what you are talking about."
about." And he went out onto the
porch.
second 71 And when he had gone out 69 And the maid saw him, 58a And a little later, another 26 One of the slaves of the
inquiry to the gateway, another and began once more to say saw him and said, "You are one high priest, being a relative
servant-girl saw him and to the bystanders, "This is of them too!" But Peter said, of the one whose ear Peter
*said to those who were one of them!" "Man, I am not!" cut off, *said, "Did I not see
there, "This man was with you in the garden with Him?"
Jesus of Nazareth."
2nd 72 And again he denied it 70a But again he was denying 58b But Peter said, "Man, I am
denial with an oath, "I do not know it. not!"
the man."
third 73 And a little later the 70b And after a little while the 59 And after about an hour had
inquiry bystanders came up and said bystanders were again saying passed, another man began to
to Peter, "Surely you too are to Peter, "Surely you are one insist, saying, "Certainly this
one of them; for the way you of them, for you are a man also was with Him, for he is
talk gives you away." Galilean too." a Galilean too."
3rd 74 Then he began to curse 71 But he began to curse and 60a But Peter said, "Man, I do
denial and swear, "I do not know the swear, "I do not know this not know what you are talking
man!" man you are talking about!" about."
cock 74 And immediately a cock 72a And immediately a cock 60b And immediately, while he 27 Peter therefore denied it
crowed crowed. crowed a second time. was still speaking, a cock again; and immediately a
crowed. cock crowed.
Peter 75 And Peter remembered the 72b And Peter remembered 61 And the Lord turned and
weeps word which Jesus had said, how Jesus had made the looked at Peter. And Peter
"Before a cock crows, you will remark to him, "Before a cock remembered the word of the
deny Me three times." And he crows twice, you will deny Me Lord, how He had told him,
went out and wept bitterly. three times." And he began to "Before a cock crows today, you
weep. will deny Me three times."
62 And he went out and wept
bitterly.

Event Matthew 26 Mark 14 Luke 12 John 13 & 18

How did Judas die, by hanging or falling down?


Matthew 27:3-8 and Acts 1:16-19

1. By hanging (Matthew 27:3-8) - "Then when Judas, who had betrayed Him, saw that He had
been condemned, he felt remorse and returned the thirty pieces of silver to the chief priests
and elders, 4saying, "I have sinned by betraying innocent blood." But they said, "What is that to
us? See to that yourself!" 5And he threw the pieces of silver into the sanctuary and departed;
and he went away and hanged himself. 6And the chief priests took the pieces of silver and said,
"It is not lawful to put them into the temple treasury, since it is the price of blood." 7And they
counseled together and with the money bought the Potter’s Field as a burial place for strangers.
8
For this reason that field has been called the Field of Blood to this day."
2. by falling (Acts 1:16-19) - "Brethren, the Scripture had to be fulfilled, which the Holy Spirit
foretold by the mouth of David concerning Judas, who became a guide to those who arrested
Jesus. 17"For he was counted among us, and received his portion in this ministry." 18(Now this
man acquired a field with the price of his wickedness; and falling headlong, he burst open in
the middle and all his bowels gushed out. 19And it became known to all who were living in
Jerusalem; so that in their own language that field was called Hakeldama, that is, Field of
Blood.)"

There is no contradiction here at all. A contradiction occurs when one statement excludes the
possibility of another yet both are supposed to be true. In fact, what happened here is that Judas
went and hung himself and then fell down. In other words, the rope or branch of the tree probably
broke and his body fell down and his bowels spilled out.

Was the vinegar given to Jesus on the cross mingled with gall or myrrh?
Matthew 27:34 and Mark 15:23

1. Gall (Matthew 27:34) - "they gave Him wine to drink mingled with gall; and after tasting it, He
was unwilling to drink."
2. Myrhh (Mark 15:23) - "And they tried to give Him wine mixed with myrrh; but He did not take
it."

Gall is bile secreted by the liver. Biblically, it is used to denote bitterness of spirit (Acts 8:23;
Lam. 3:19). Myrrh is an aromatic gum that grows in Arabia, Abyssinia, and India. It was used to
sweeten the smell and taste of various foods. It was also used in embalming (John 19:39).
According to "Alleged Discrepancies of the Bible" by Haley, page 28, there were two times that
Jesus was offered the vinegar. "The first time, the wine drugged with bitter narcotics, the effect of
which would be to stupefy him, he did not receive. Afterward, some drink free from drugs was given
him, which he accepted." In other words, they were of two different times and different things were
offered. Though it is a hopeful explanation, it does not fit the text. Please take a look at the table
where I have put the verses in order from Matthew and Mark and included the two times Jesus was
offered drink.

Matthew Mark
At the beginning of the crucifixion . . .
27:34 - "they gave Him wine to drink mingled 15:23 "And they tried to give Him wine mixed
with gall; and after tasting it, He was with myrrh; but He did not take it."
unwilling to drink." (Greek "wine" is "oinos" which means simply,
(Greek "wine" is "ozos" which is a mixture of wine.)
sour wine or vinegar and water.)
. . . several hours pass . . .
27:48, "And immediately one of them ran, 15:36 "And someone ran and filled a sponge
and taking a sponge, he filled it with sour with sour wine, put it on a reed, and gave Him
wine, and put it on a reed, and gave Him a a drink."
drink." (Greek "sour wine" is "ozos", or vinegar).
(Greek "sour wine" is "ozos", or vinegar as
above).

Possible solutions

Most probably, both gall and myrrh were added to the vinegar. The text does not explicitly state
this, nor does it exclude the possibility. Nevertheless, "The ancients used to infuse myrrh into wine to
give it a more agreeable fragrance and flavour." 84 This means that it is quite possible that the vinegar
already had myrrh in it, as would be expected among Roman soldiers, and gall was later added. Each
writer focused on a different aspect.
Furthermore, these verses do not necessitate a contradiction if we understand a contradiction to be
the condition when the statement of one verse negates the possibility of the other being true.
Technically, the inclusion of both gall and vinegar is very possible.
A second explanation could be that that Matt. 27:34 and Mark 15:23 are describing different
events. Textual evidence supporting this would be that different Greek words are used: "ozos" in
Matthew for "wine" while Mark has the Greek "oinos" as wine. That, combined with different Greek
words "chole" for "gall" in Matthew and "smurna" for "myrrh" in Mark, may be a clue that these are
different events. However, this explanation, though possible, is not very likely since the context of the
verses strongly suggest that each is a description of the same event.
Third, there may be an unknown copyist error. I say unknown because I could find no record of
any textual variation in Matthew and Mark regarding these verses in question. This does not mean
that we might not find one in the future which could shed more light on the issue. However, the
weakness with this explanation is that it is an argument of silence and is not favored.
I conclude that the most likely explanation is the first one, that both myrrh and gall were present in
the vinegar.

84
Enhanced Strong’s Lexicon, (Oak Harbor, WA: Logos Research Systems, Inc.) 1995.
What did Jesus do after encountering John the Baptist?
Mark 1:7-13; John 1:27-40

After His baptism, Jesus spent some time with the disciples and then went into went into the
desert. John the Apostle's account of the baptism of Jesus is not a focus on chronological events.
Instead, it is focusing on the ministerial aspect of Christ's mission. John focuses on the issue of
baptism and the commission of Christ and the blessing of the Father. Mark simply states that Jesus
went into the wilderness after His baptism. Each account is about the same thing, but each addresses
the issue in a very different manner and extracts different information from the events. Mark is
succinct and mentions events several more events than John. There is no contradiction because there
is no conflict in what is said.
For more information see the Chronology of Jesus' Baptism and Temptation.

Mark 1:7-13 John 1:27-40


7 And he was preaching, and saying, 27“It is He who comes after me, the thong of whose
“After me One is coming who is mightier sandal I am not worthy to untie.”
than I, and I am not fit to stoop down and 28 These things took place in Bethany beyond the
untie the thong of His sandals. Jordan, where John was baptizing.
8 “I baptized you with water; but He will
baptize you with the Holy Spirit.”
29 The next day he *saw Jesus coming to him, and
*said, “Behold, the Lamb of God who takes away the
sin of the world!
30 “This is He on behalf of whom I said, ‘After me
comes a Man who has a higher rank than I, for He
existed before me.’ 31 “And I did not recognize Him,
but in order that He might be manifested to Israel, I
came baptizing in water.”
9 And it came about in those days that 32 And John bore witness saying, “I have beheld the
Jesus came from Nazareth in Galilee, and Spirit descending as a dove out of heaven, and He
was baptized by John in the Jordan. remained upon Him.
33 “And I did not recognize Him, but He who sent me
to baptize in water said to me, ‘He upon whom you see
the Spirit descending and remaining upon Him, this is
the one who baptizes in the Holy Spirit.’
34 “And I have seen, and have borne witness that this
is the Son of God.”
35 Again the next day John was standing with two of
his disciples,
36 and he looked upon Jesus as He walked, and *said,
“Behold, the Lamb of God!”
37 And the two disciples heard him speak, and they
followed Jesus.
10 And immediately coming up out of the
water, He saw the heavens opening, and
the Spirit like a dove descending upon
Him;
11 and a voice came out of the heavens:
“Thou art My beloved Son, in Thee I am
well-pleased.”
12 And immediately the Spirit *impelled 38 And Jesus turned, and beheld them following, and
Him to go out into the wilderness. *said to them, “What do you seek?” And they said to
13 And He was in the wilderness forty days Him, “Rabbi (which translated means Teacher), where
being tempted by Satan; and He was with are You staying?”
the wild beasts, and the angels were 39 He *said to them, “Come, and you will see.” They
ministering to Him. came therefore and saw where He was staying; and
they stayed with Him that day, for it was about the
tenth hour.
40 One of the two who heard John speak, and followed
Him, was Andrew, Simon Peter’s brother.

Can you be forgiven of all sins or not?


Matthew 12:31; Mark 3:29 and Acts 13:39; Titus 2:13-14; 1 John 1:9

1. All sins not forgiven


A. (Matthew 12:31) - "Therefore I say to you, any sin and blasphemy shall be forgiven men,
but blasphemy against the Spirit shall not be forgiven."
B. (Mark 3:29) - "but whoever blasphemes against the Holy Spirit never has forgiveness, but is
guilty of an eternal sin. 32And whoever shall speak a word against the Son of Man, it shall be
forgiven him; but whoever shall speak against the Holy Spirit, it shall not be forgiven him,
either in this age, or in the age to come."
2. All sins forgiven
A. (Acts 13:39) - "and through Him everyone who believes is freed from all things, from which
you could not be freed through the Law of Moses."
B. (Titus 2:13-14) - "looking for the blessed hope and the appearing of the glory of our great
God and Savior, Christ Jesus; 14who gave Himself for us, that He might redeem us from
every lawless deed and purify for Himself a people for His own possession, zealous for good
deeds."
C. (1 John 1:9) - "If we confess our sins, He is faithful and righteous to forgive us our sins and
to cleanse us from all unrighteousness."

When interpreting the Bible context is everything. Anyone can take a verse out of context in one
place and compare it to another verse out of context in another place. Such is often the case when
critics quote these types scriptures listed above as criticism of the Bible.
The answer is simple. We are to take all of God's word into account, in context, when developing a
teaching. Therefore, the Bible teaches us that blasphemy against the Holy Spirit is unforgivable. All
other sins are forgivable. However, this does not mean that all sins will be forgiven. It means that all
these other sins can be forgiven. We are forgiven only when we trust in Christ and Him alone for the
forgiveness of our sins. Those who have committed blasphemy of the Holy Spirit will never seek Christ
because the Holy Spirit will not work on them.
For more information, please see the paper on blasphemy of the Holy Spirit.

Did Jesus tell His disciples to take a staff or not?


Mark 6:8 and Matthew 10:9-10; Luke 9:3

1. Can take a staff


A. (Mark 6:8) - "and He instructed them that they should take nothing for their journey, except
a mere staff; no bread, no bag, no money in their belt."
2. Cannot take a staff
A. (Matthew 10:9-10) - "Do not acquire gold, or silver, or copper for your money belts, 10or a
bag for your journey, or even two tunics, or sandals, or a staff; for the worker is worthy of
his support."
B. (Luke 9:3) - "And He said to them, "Take nothing for your journey, neither a staff, nor a
bag, nor bread, nor money; and do not even have two tunics apiece."

There are no textual variants in these verses in the Greek manuscripts that would bear upon this
issue regarding the staff. The word for "take" in both Mark 6:8 and Luke 9:3 is "airo." In Matthew
10:9, the word for "acquire" is "ktaomai." Each account is preceded by the raising of the dead girl so
the context of each is the same. It is possible that there is a textual variation not yet known. But,
this is not an acceptable explanation I would take since it is based a lack of proof. How then can this
difficulty be reconciled?
The best explanation I can offer is that Matthew clarifies the issue by telling the disciples to not
acquire anything more than what they already had. It is possible that the "take" of Mark and Luke can
be interpreted to mean acquire, but I think this is a weak explanation. Nevertheless, each verse is
saying that the disciples are to go as they are, to not take provisions. Just trust the Lord.
I must note that I am not completely satisfied with this explanation and I wait further clarification
should it arise.

How many blind men did Jesus encounter when leaving Jericho?
Matthew 20:29-30; Mark 10:46-47; Luke 18:35

1. Two blind men (Matthew 20:29-30) - "And as they were going out from Jericho, a great
multitude followed Him. 30And behold, two blind men sitting by the road, hearing that Jesus was
passing by, cried out, saying, "Lord, have mercy on us, Son of David!"
2. One blind man (Mark 10:46-47) - "And they *came to Jericho. And as He was going out from
Jericho with His disciples and a great multitude, a blind beggar named Bartimaeus , the son of
Timaeus, was sitting by the road. 47And when he heard that it was Jesus the Nazarene, he
began to cry out and say, "Jesus, Son of David, have mercy on me!"
3. One blind man (Luke 18:35,38) - "And it came about that as He was approaching Jericho, a
certain blind man was sitting by the road, begging... 38And he called out, saying, "Jesus, Son of
David, have mercy on me!"

There is no contradiction. Matthew 20:29-30 makes it clear that as Jesus was leaving Jericho that
there were two blind men sitting by the road. If there are two blind men, then there is certainly at
least one there as well. The one focused on was Bartimeaus. Both called out for healing (Matthew
20:29-30). But, Mark and Luke focus on Bartimeaus' account maybe because he was the loudest and
most determined, a point worth focusing on for spiritual reasons because God wants us to be
persistent in laying our needs before Him.

Were one or two animals brought to Jesus?


Matthew 21:2-7 and Mark 11:2-7; Luke 19:30

1. Donkey and colt (Matthew 21:2-7) - "Go into the village opposite you, and immediately you
will find a donkey tied there and a colt with her; untie them, and bring them to Me. 3“And if
anyone says something to you, you shall say, ‘The Lord has need of them,’ and immediately he
will send them.” 4Now this took place that what was spoken through the prophet might be
fulfilled, saying, 5“Say to the daughter of Zion, ‘Behold your King is coming to you, gentle, and
mounted on a donkey, even on a colt, the foal of a beast of burden.’” 6And the disciples went
and did just as Jesus had directed them, 7and brought the donkey and the colt, and laid on
them their garments, on which He sat."
2. A colt (Mark 11:2-7) - "Go into the village opposite you, and immediately as you enter it, you
will find a colt tied there, on which no one yet has ever sat; untie it and bring it here. 3"And if
anyone says to you, ‘Why are you doing this?’ you say, ‘The Lord has need of it’; and
immediately he will send it back here." 4And they went away and found a colt tied at the door
outside in the street; and they *untied it. 5And some of the bystanders were saying to them,
"What are you doing, untying the colt?" 6And they spoke to them just as Jesus had told them,
and they gave them permission. 7And they *brought the colt to Jesus and put their garments
on it; and He sat upon it."
3. A colt (Luke 19:30) - "Go into the village opposite you, in which as you enter you will find a
colt tied, on which no one yet has ever sat; untie it, and bring it here."

There is no contradiction. Matthew 21:2-7 tells us that there was both a donkey and a colt. Mark
and Luke focus on the colt only and mention that no one had ever sat upon it. Mark and Luke are
focusing on this detail while Matthew focuses on the prophetic fulfillment ( Matthew 21:4-5). Logically,
if there are two animals, then there is also, at least, one animal. To say there was one does not mean
there weren't two.
Zechariah 9:9 is the scripture that Matthew refers to. It says, "Rejoice greatly, O daughter of Zion!
Shout in triumph, O daughter of Jerusalem! Behold, your king is coming to you; he is just and
endowed with salvation, humble, and mounted on a donkey, even on a colt, the foal of a donkey." We
can see that Matthew is simply including both animals as was prophesied in Zechariah.

Did the tree that Jesus cursed wither immediately or overnight?


Matthew 21:19 and Mark 11:14,20

1. Immediately (Matthew 21:19) - "Seeing a lone fig tree by the road, He came to it and found
nothing on it except leaves only; and He said to it, "No longer shall there ever be any fruit from
you." And at once the fig tree withered.
2. Overnight (Mark 11:14,20) - "He said to it, "May no one ever eat fruit from you again!" And
His disciples were listening. . . 20As they were passing by in the morning, they saw the fig tree
withered from the roots up."

This "contradiction" is one of the more problematic. If you look at the chart below you will see that
that the areas in blue are difficult to reconcile if we look at them in a strictly chronological manner.
One answer I have discovered says that Matthew writes thematically, often grouping topics together
without a detailed focus on chronology, where Mark does not. In other words, Matthew simply
compressed the event to a single instance in order to make the point about the necessity of bearing
fruit. Therefore, Matthew's account is not meant to be chronological, but thematic and that it is Mark's
account that presents the chronological order. This is certainly a possible solution. However, the
weakness of this answer is in Matthew's use of the phrase "at once the fig tree withered" in 21:19. If
Matthew's account is not intended to be chronologically precise, that is one thing that I can accept
since it does indeed occur in Matthew. But, the phrase which tells us the fig tree withered "at once" is
stating it was immediate. This doesn't leave much room for the fig tree to wither overnight. But, if
Matthew were compressing the incident, then the immediacy would be evident.
Another possibility is that there are two different fig trees that were cursed. I don't think this is a
viable option if you look at the chart below. But it is a possibility.
Jesus was obviously using the Fig Tree as a symbol as He taught. His condemnation of the fig tree
for not bearing fruit is actually a condemnation upon the Jewish leadership and their spiritual
deadness. Note the context of Christ cleansing the temple (Mark 11:15-17) and His quote from Isaiah
56:7 and Jer. 7:11. The fig tree was used elsewhere in scripture symbolically for leaders (Judges
9:10-11), fortifications (Nahum 3:12), in parables (Matt. 24:32ff), etc. If Jesus were using the Fig
Tree as a symbol, then either Matthew or Mark could easily take liberties in focusing on the issue of
the meaning of the story rather than recounting a chronological event.
Nevertheless, the account does pose a challenge to the doctrine of inerrancy. But, does this mean
that the Bible is not trustworthy or not inspired? Not at all. We do not give up on physics or
mathematics or astronomy if we find things we cannot easily explain. The Bible is a wondrous and
beautiful book and it will always stand the test of time. As archaeology continues and older and better
manuscripts are found, then, as has happened in the past, more and more biblical discrepancies will be
answered.

Matthew Mark
11:12 On the next day, when they had
left Bethany, He became hungry.
Curse of the fig 11:13 Seeing at a distance a fig tree in
tree leaf, He went to see if perhaps He would
find anything on it; and when He came to
it, He found nothing but leaves, for it was
not the season for figs.
11:14 He said to it, "May no one ever eat
"May no one ever
fruit from you again!" And His disciples
eat fruit from you
were listening.
again!"
at the temple 21:12 And Jesus entered the temple 11:15 Then they came to Jerusalem. And
and over turns and drove out all those who were He entered the temple and began to drive
tables buying and selling in the temple, and out those who were buying and selling in
overturned the tables of the money the temple, and overturned the tables of
changers and the seats of those who the money changers and the seats of
were selling doves. those who were selling doves;
21:13 And He said to them, "It is 11:16 and He would not permit anyone to
written, 'MY HOUSE SHALL BE CALLED carry merchandise through the temple.
A HOUSE OF PRAYER '; but you are 11:17 And He began to teach and say to
making it a ROBBERS' DEN." them, "Is it not written, 'MY HOUSE SHALL
BE CALLED A HOUSE OF PRAYER FOR ALL
THE NATIONS'? But you have made it a
ROBBERS' DEN."
they wanted to 21:14 And the blind and the lame came 11:18 The chief priests and the scribes
kill Him. to Him in the temple, and He healed heard this, and began seeking how to
them. destroy Him; for they were afraid of Him,
21:15 But when the chief priests and for the whole crowd was astonished at His
the scribes saw the wonderful things teaching.
that He had done, and the children who
were shouting in the temple, "Hosanna
to the Son of David ," they became
indignant
21:16 and said to Him, "Do You hear
what these children are saying ?" And
Jesus said to them, "Yes ; have you
never read, 'OUT OF THE MOUTH OF
INFANTS AND NURSING BABIES YOU
HAVE PREPARED PRAISE FOR
YOURSELF'?"
out of 21:17 And He left them and went out of 11:19 When evening came, they would go
the city the city to Bethany, and spent the night out of the city.
there.
Next Day (Matthew) (Mark)
in the morning 21:18 Now in the morning, when He 11:20 As they were passing by in the
was returning to the city, He became morning, they saw the fig tree withered
hungry. from the roots up.
No longer shall 21:19 Seeing a lone fig tree by the
there ever be any road, He came to it and found nothing
fruit from you." on it except leaves only; and He said to
it, "No longer shall there ever be any
fruit from you." And at once the fig tree
withered.
21:20 Seeing this, the disciples were
amazed and asked, "How did the fig
tree wither all at once?”
the tree withered 11:21 Being reminded, Peter said to Him,
"Rabbi, look, the fig tree which You cursed
has withered."
faith, mountain 21:21 And Jesus answered and said to 11:22 And Jesus answered saying to
be moved them, "Truly I say to you, if you have them, "Have faith in God.
faith and do not doubt, you will not only 11:23 "Truly I say to you, whoever says
do what was done to the fig tree, but to this mountain, 'Be taken up and cast
even if you say to this mountain, 'Be into the sea,' and does not doubt in his
taken up and cast into the sea,' it will heart, but believes that what he says is
happen. going to happen, it will be granted him.
all things you ask 21:22 "And all things you ask in prayer, 11:24 "Therefore I say to you, all things
believing.... believing, you will receive." for which you pray and ask, believe that
you have received them, and they will be
granted you.

Who arrested Jesus?


Matthew 26:47; Mark 14:43; Luke 22:47; John 18:12

1. A multitude
A. (Matt. 26:47,50) - "And while He was still speaking, behold, Judas, one of the twelve, came
up, accompanied by a great multitude with swords and clubs, from the chief priests and
elders of the people . . And Jesus said to him, "Friend, do what you have come for." Then
they came and laid hands on Jesus and seized Him."
B. (Mark 14:43) - "And immediately while He was still speaking, Judas, one of the twelve, came
up, accompanied by a multitude with swords and clubs, from the chief priests and the
scribes and the elders."
C. (Luke 22:47) - "While He was still speaking, behold, a multitude came, and the one called
Judas, one of the twelve, was preceding them; and he approached Jesus to kiss Him."
2. Roman cohort, commander, and Jewish officers.
A. (John 18:12) - "So the Roman cohort and the commander, and the officers of the Jews,
arrested Jesus and bound Him,"
This is obviously no biblical contradiction. Matthew, Mark, and Luke tell us that a multitude
approached Jesus in the garden and they seized Him. John tells us that the arresting party was "the
Roman cohort and the commander, and the officers of the Jews..." Obviously, the multitude included
those who were actually doing the arresting.

Did Jesus or Simon of Cyrene carry the cross?


Matthew 27:31-32; Mark 15:20-21; Luke 23:26 and John19:17

1. Simon of Cyrene
A. (Matthew 27:31-32) - "And after they had mocked Him, they took His robe off and put His
garments on Him, and led Him away to crucify Him. 32And as they were coming out, they
found a man of Cyrene named Simon, whom they pressed into service to bear His cross."
B. (Mark 15:20-21) - "And after they had mocked Him, they took the purple off Him, and put
His garments on Him. And they *led Him out to crucify Him. 21And they pressed into
service a passer-by coming from the country, Simon of Cyrene (the father of Alexander and
Rufus), to bear His cross."
C. (Luke 23:26) - "And when they led Him away, they laid hold of one Simon of Cyrene, coming
in from the country, and placed on him the cross to carry behind Jesus."
2. Jesus
A. (John 19:17) - "They took Jesus therefore, and He went out, bearing His own cross, to the
place called the Place of a Skull, which is called in Hebrew, Golgotha."

The answer to this discrepancy is very simple when we look at the context. Jesus had undergone a
very physically traumatic few hours. Please consider the following verses placed in order of His ordeal
and pay attention to the physical trauma he received before He ever arrived at the cross.

• Sweat like blood "And being in agony He was praying very fervently; and His sweat became
like drops of blood, falling down upon the ground," (Luke 22:44).
• He was struck "And when He had said this, one of the officers standing by gave Jesus a blow,
saying, "Is that the way You answer the high priest?" 23Jesus answered him, "If I have spoken
wrongly, bear witness of the wrong; but if rightly, why do you strike Me?" (John 18:22-23).
• He was beaten with fists "And some began to spit at Him, and to blindfold Him, and to beat
Him with their fists, and to say to Him, "Prophesy!" And the officers received Him with slaps in
the face," (Mark 14:65).
• He was scourged "Then he released Barabbas for them; but after having Jesus scourged, he
delivered Him to be crucified," (Matt. 27:26).
• Crown of thorns on head and beaten "And after weaving a crown of thorns, they put it on
His head, and a reed in His right hand; and they kneeled down before Him and mocked Him,
saying, "Hail, King of the Jews! 30"And they spat on Him, and took the reed and began to beat
Him on the head. 31And after they had mocked Him, they took His robe off and put His
garments on Him, and led Him away to crucify Him," (Matt. 27:29-31).

The accused would carry the cross beam to the place of crucifixion. Jesus simply couldn't carry it
very far after all the physical trauma He had just gone through. He collapsed. That is when the
Romans drafted Simon of Cyrene to carry the cross the rest of the way.
At what hour was Jesus crucified?
Mark 15:25 and John 19:14-16

1. The third hour (Mark 15:25) - "And it was the third hour when they crucified Him."
2. The sixth hour (John 19:14-15) - "Now it was the day of preparation for the Passover; it was
about the sixth hour. And he *said to the Jews, "Behold, your King!" 15They therefore cried out,
"Away with Him, away with Him, crucify Him!" Pilate *said to them, "Shall I crucify your King?"
The chief priests answered, "We have no king but Caesar."

Most probably, John was using the Roman measurement of time when dealing with the crucifixion.
Matthew, Mark, and Luke, for the most part, used the Hebrew system of measuring a day: from
sundown to sunup. The Roman system was from midnight to midnight. "John wrote his gospel in
Ephesus, the capital of the Roman province of Asia, and therefore in regard to the civil day he would
be likely to employ the Roman reckoning. (Encyclopedia of Bible Difficulties, by Gleason Archer, page
364.)

What was written on the sign on the cross?


Matthew 27:37; Mark 15:26; Luke 23:38; John 19:19

1. (Matthew 27:37) - "And set up over his head his accusation written, THIS IS JESUS THE KING
OF THE JEWS."
2. (Mark 15:26) - "And the superscription of his accusation was written over, THE KING OF THE
JEWS."
3. (Luke 23:38) - "And a superscription also was written over him in letters of Greek, and Latin,
and Hebrew, THIS IS THE KING OF THE JEWS."
4. (John 19:19) - "And Pilate wrote a title, and put it on the cross. And the writing was, JESUS OF
NAZARETH THE KING OF THE JEWS. 20This title then read many of the Jews: for the place
where Jesus was crucified was nigh to the city: and it was written in Hebrew, and Greek, and
Latin."

The Inscription above the cross of Christ was written in four different languages: Greek, Hebrew,
Aramaic, and Latin. We see this when we look at Luke 23:38 and John 19:20. Luke 23:38 clearly
states it was written in Greek, Latin, and Hebrew. In John the word for "Hebrew" is hebraisti. "John
19:20 uses for this the adverbial form Hebraisti, which in gospel usage did not mean "in Hebrew" but
in the Jewish dialect of Aramaic. We know this because wherever Hebraisti is used elsewhere, as in
John 5:2; 19:13,17; 20:16, the word is given in its Aramaic form, transcribed into Greek letters," 85

What are the last words of Jesus?


Matthew 27:46; Mark 15:34; Luke 23:46; John 19:30

1. (Matthew 27:46) - "And about the ninth hour Jesus cried out with a loud voice, saying, "Eli, Eli,
lama sabachthani?" that is, "My God, My God, why hast Thou forsaken Me?"
2. (Mark 15:34) - "And at the ninth hour Jesus cried out with a loud voice, "Eloi, Eloi, lama
sabachthani?" which is translated, "My God, My God, why hast Thou forsaken Me?"
3. (Luke 23:46) - "And Jesus, crying out with a loud voice, said, "Father, into Thy hands I commit
My spirit." And having said this, He breathed His last."
4. (John 19:30) - "When Jesus therefore had received the sour wine, He said, "It is finished!" And
He bowed His head, and gave up His spirit."

A simple arrangement of the various gospel texts on the Crucifixion Chronology page shows that
the last thing Jesus said was "Father, into Thy hands I commit My spirit," (Luke 23:46) which also
states that He breathed his last after saying that.

Were the women close or far from the cross?


Matthew 27:55-56; Mark 15:40; Luke 23:49; John 19:25

1. At a distance from the cross


A. (Matthew 27:55-56) - "And many women were there looking on from a distance, who had
followed Jesus from Galilee, ministering to Him."
B. (Mark 15:40) - "And there were also some women looking on from a distance, among whom
were Mary Magdalene, and Mary the mother of James the Less and Joses, and Salome."
C. (Luke 23:49) - "And all His acquaintances and the women who accompanied Him from
Galilee, were standing at a distance, seeing these things."
2. Close to the cross
A. (John 19:25) - "Therefore the soldiers did these things. But there were standing by the cross
of Jesus His mother, and His mother’s sister, Mary the wife of Clopas, and Mary Magdalene."

This is no contradiction at all. Quite simply, at one point in the crucifixion chronology, the women
were standing at a distance and then later, they were standing closer. You can check the chronology
at the Crucifixion Chronology page.

85
Encyclopedia of Bible Difficulties, page 346.
How many men or angels appeared at the tomb?
Matt 28:2; Mark 16:5; Luke 24:4; John 20:1-2,12

1. An angel of the Lord on the stone (Matthew 28:1-2) - "Now after the Sabbath, as it began
to dawn toward the first day of the week, Mary Magdalene and the other Mary came to look at
the grave. 2And behold, a severe earthquake had occurred, for an angel of the Lord descended
from heaven and came and rolled away the stone and sat upon it."
2. A young man (Mark 16:5) - "And entering the tomb, they saw a young man sitting at the
right, wearing a white robe; and they were amazed."
3. Two men (Luke 24:4) - "And it happened that while they were perplexed about this, behold,
two men suddenly stood near them in dazzling apparel."
4. Two angels (John 20:1-2,12) - "Now on the first day of the week Mary Magdalene *came
early to the tomb, while it *was still dark, and *saw the stone already taken away from the
tomb. 2And so she *ran and *came to Simon Peter, and to the other disciple whom Jesus loved,
and *said to them, "They have taken away the Lord out of the tomb, and we do not know
where they have laid Him. . 12and she *beheld two angels in white sitting, one at the head, and
one at the feet, where the body of Jesus had been lying."

There is no discrepancy at all. An angel of the Lord moved the stone and was sitting upon it outside
(Matthew 28:2). The two men (Luke 24:4) were angels (John 20:12). Mark 16:5 presents the only
potential issue and it isn't one at all. If there were two angels in the tomb, then there was at least
one. This one was on the right. Therefore, we see that there was one angel outside and two on the
inside of the tomb.

What did the angels tell Mary?


Matt 28:6-7; Mark 16:6-7; Luke 24:5-7; John 20:13

1. (Matthew 28:6-7) - "He is not here, for He has risen, just as He said. Come, see the place
where He was lying. 7"And go quickly and tell His disciples that He has risen from the dead;
and behold, He is going before you into Galilee, there you will see Him; behold, I have told
you."
2. (Mark 16:6-7) - "And he *said to them, "Do not be amazed; you are looking for Jesus the
Nazarene, who has been crucified. He has risen; He is not here; behold, here is the place where
they laid Him. 7"But go, tell His disciples and Peter, ‘He is going before you into Galilee; there
you will see Him, just as He said to you.’"
3. (Luke 24:5-7) - "and as the women were terrified and bowed their faces to the ground, the
men said to them, "Why do you seek the living One among the dead? 6"He is not here, but He
has risen. Remember how He spoke to you while He was still in Galilee, 7saying that the Son of
Man must be delivered into the hands of sinful men, and be crucified, and the third day rise
again."
4. (John 20:13) - "And they *said to her, "Woman, why are you weeping?" She *said to them,
"Because they have taken away my Lord, and I do not know where they have laid Him."

The different gospel writers recalled, or were told, that which was said. Just like real witnesses
who all view the same event, each says a slightly different version of the same thing. If each sentence
were identical, then the charge of collusion would be raised. But the fact that each writer records the
words of the angels in a slightly different manner suggests that they really did witness these events, or
heard of them, and recorded them. The slight differences are proof that there were different people
seeing the event. In other words, it really happened and each person recalled a slightly, but non
contradictory, account of the angel's words. Following is a suggested arrangement of words that
might help to harmonize the words of the angels.

Matthew 28:6-7 Mark 16:6-7 Luke 24:5-7 John 20:13


women "and as the women were
were terrified and bowed their
afraid faces to the ground, the
men said to them,
Two "And they *said to her,
angels "Woman, why are you
spoke weeping?" She *said to
them, "Because they have
taken away my Lord, and I
do not know where they
have laid Him."
Two or "Why do you seek the living
one angel One among the dead?
spoke
One angel "And he *said to
spoke them, "Do not be
amazed; you are
looking for Jesus the
Nazarene, who has
been crucified. He has
risen;
"He is not here, for He is not here; 6
"He is not here, but He has
He has risen, just as risen.
He said.
Come, see the place behold, here is the
where He was lying. place where they laid
Him.
Remember how He spoke to
you while He was still in
Galilee, 7saying that the Son
of Man must be delivered
into the hands of sinful men,
and be crucified, and the
third day rise again."
7
"But go, tell His
disciples and Peter,
‘He is going before
you into Galilee; there
you will see Him, just
as He said to you.’"
7
"And go quickly and
tell His disciples that
He has risen from
the dead; and
behold, He is going
before you into
Galilee, there you
will see Him; behold,
I have told you."
Did or did not the women tell what happened?
Matt 28:8; Mark 16:8; Luke 24:9; John 20:18

1. They told what happened


A. (Matthew 28:8) - "And they departed quickly from the tomb with fear and great joy and ran
to report it to His disciples."
B. (Luke 24:9) - "and returned from the tomb and reported all these things to the eleven and
to all the rest."
C. (John 20:18) - "Mary Magdalene *came, announcing to the disciples, "I have seen the Lord,"
and that He had said these things to her."
2. They said nothing
A. (Mark 16:8) - "And they went out and fled from the tomb, for trembling and astonishment
had gripped them; and they said nothing to anyone, for they were afraid."

The best explanation is that the women initially said nothing ( Mark 16:8) and then later told the
disciples what they saw (Matthew 28:8; Luke 24:9). It would make sense that they were frightened
and didn't know what to do or say. But then later, of course, they spoke up. The John 20:18 account
is chronologically later than the other references and is not relevant in this difficulty.
Please see the Resurrection Chronology page to see how the verses fit together.

Who saw Jesus first?


Matt 28:9; Mark 16:9; Luke 24:15-18; John 20:14; 1 Cor 15:3-5

1. The three women (Matthew 28:9) - "And behold, Jesus met them and greeted them. And
they came up and took hold of His feet and worshiped Him."
2. Mary Magdalene (Mark 16:9) - "Now after He had risen early on the first day of the week, He
first appeared to Mary Magdalene, from whom He had cast out seven demons."
3. The Disciples (Luke 24:15-18) - "And it came about that while they were conversing and
discussing, Jesus Himself approached, and began traveling with them. 16 But their eyes were
prevented from recognizing Him. 17And He said to them, "What are these words that you are
exchanging with one another as you are walking?" And they stood still, looking sad. 18And one
of them, named Cleopas, answered and said to Him, "Are You the only one visiting Jerusalem
and unaware of the things which have happened here in these days?"
4. Mary (John 20:14) - "When she had said this, she turned around, and *beheld Jesus standing
there, and did not know that it was Jesus."
5. Cephas and the twelve (1 Corinthians 15:3-5) - "For I delivered to you as of first importance
what I also received, that Christ died for our sins according to the Scriptures, 4and that He was
buried, and that He was raised on the third day according to the Scriptures, 5and that He
appeared to Cephas, then to the twelve."

This is no contradiction at all. The answer is simple. The first one to see Jesus after His
resurrection was Mary Magdalene just as it says (Mark 16:9). Then the others saw Him afterward.
The context of the other verses don't present any problem at all.
Please see the Resurrection Chronology page to see how the verses fit together.
Is the ending of Mark really scripture?
Mark 16:9-20

There is dispute over Mark 16:9-20 and whether or not it should be included in the New
Testament. It is found in many old manuscripts but is omitted in two of the earliest complete copies of
the Bible known as the Vaticanus (350 AD) and Sinaiticus (375 AD). Additionally, there is another
ending to Mark in some old manuscripts that is substituted for 9-20. The alternate ending reads as
follows:

"And they promptly reported all these instructions to Peter and his companions. And
after that, Jesus Himself sent out through them from east to west the sacred and
imperishable proclamation of eternal salvation."

Some scholars have asserted that the ending is in a different style than the rest of the gospel and
that it contains 16-22 "non-marcan" words used in a "non-marcan" sense. It seems to suggest that
Jesus appeared in a different form (v. 12) which could be problematic since Jesus rose in the same
body He died in (John 2:19-21). Also, Mark 16:16 can be interpreted to mean that baptism is part of
salvation. It isn't as is testified by verses that teach justification by faith Rom. 5:1; 6:23; Eph. 2:8-9,
etc.). Whichever the case, the dispute is not settled and may never be.
I am not here trying to undermine the authority of God's word nor state that Mark 16:9-20 is not
authentic. But, the fact remains that these 12 verses are under dispute and it is necessary to spotlight
this issue when dealing with the historic reliability and inspiration of the New Testament manuscripts.

The text of Mark 16:9-20

"Now after He had risen early on the first day of the week, He first appeared to Mary Magdalene,
from whom He had cast out seven demons. 10She went and reported to those who had been with Him,
while they were mourning and weeping. 11And when they heard that He was alive, and had been seen
by her, they refused to believe it.
12
And after that, He appeared in a different form to two of them, while they were walking along on
their way to the country. 13And they went away and reported it to the others, but they did not believe
them either.
14
And afterward He appeared to the eleven themselves as they were reclining at the table; and He
reproached them for their unbelief and hardness of heart, because they had not believed those who
had seen Him after He had risen. 15And He said to them, "Go into all the world and preach the gospel
to all creation. 16"He who has believed and has been baptized shall be saved; but he who has
disbelieved shall be condemned. 17"And these signs will accompany those who have believed: in My
name they will cast out demons, they will speak with new tongues; 18they will pick up serpents, and if
they drink any deadly poison, it shall not hurt them; they will lay hands on the sick, and they will
recover."
19
So then, when the Lord Jesus had spoken to them, He was received up into heaven, and sat down
at the right hand of God. 20And they went out and preached everywhere, while the Lord worked with
them, and confirmed the word by the signs that followed," (NASB).
Are we supposed to hate or not?
Luke 14:26 and 1 John 3:15

1. You must hate - (Luke 14:26) - "If anyone comes to Me, and does not hate his own father
and mother and wife and children and brothers and sisters, yes, and even his own life, he
cannot be My disciple."
2. You must not hate - (1 John 3:15) - "Everyone who hates his brother is a murderer; and you
know that no murderer has eternal life abiding in him."

In Luke 14:26, Jesus is drawing a comparison of importance by exaggerating a relationship. He is


saying that it is far more important to love Him than anyone else, including your own parents. Of
course, He is not telling people to hate their parents. He is saying that by comparison to Him, you
must love Him more than all else.
In 1 John 3:15, John is writing to the church about abiding in the love of Christ. In fact, in 1 John,
the word "abide" occurs 16 times in the NASB and the apostle continually refers to abiding in Christ ( 1
John 2:4,24,28; 3:6,24, etc.)
Therefore, we see that a true Christian will love the Lord Jesus supremely and in so doing he will
not abide in hatred towards his brother.

Did anyone ascend into heaven before Jesus or not?


Genesis 5:24; 2 Kings 2:11 and John 3:13

1. Yes
A. (Genesis 5:24) - "And Enoch walked with God; and he was not, for God took him."
B. (2 Kings 2:11) - "Then it came about as they were going along and talking, that behold,
there appeared a chariot of fire and horses of fire which separated the two of them. And
Elijah went up by a whirlwind to heaven."
C. (Hebrews 11:5) - "By faith Enoch was taken up so that he should not see death; and he was
not found because God took him up; for he obtained the witness that before his being taken
up he was pleasing to God."
2. No
A. (John 3:13) - "And no one has ascended into heaven, but He who descended from heaven,
even the Son of Man"

The answer is simple when you understand the Jewish concept of the universe. They believed in
three heavens. The first was the atmosphere where the trees, clouds, and birds are. The second is
the realm of the stars, planets, sun, and moon. And the third heaven was the very dwelling place of
God. In the Old Testament in Genesis 5 and 2 Kings 2 above, they were not taken into the very
dwelling place of God, but into the heavens; that is, they were taken up into the sky. Exactly where is
in debate. It isn't until after the crucifixion and resurrection of Christ were those who had died before,
in faith, taken into the highest heaven. Ephesians 4:8 is often referred to as a supporting scripture for
this belief. It says, "When He ascended on high, He led captive a host of captives, and He gave gifts
to men." Furthermore, when Paul said he was caught up to the third heaven (2 Cor. 12:2), he was
referring to the very dwelling place of God.
Biblical demonstration of the three heavens

1. First heaven - Earth Atmosphere


A. (Deuteronomy 11:17) - "Then the LORD's anger will burn against you, and he will shut
the heavens so that it will not rain and the ground will yield no produce...."
B. (Deuteronomy 28:12) - "The LORD will open the heavens, the storehouse of his bounty,
to send rain on your land in season and to bless all the work of your hands."
2. Second Heaven - Outer Space
A. (Psalm 19:4,6) - "In the heavens he has pitched a tent for the sun... It rises at one end
of the heavens and makes its circuit to the other;..."
B. (Jeremiah 8:2) - They will be exposed to the sun and the moon and all the stars of the
heavens which they have loved and served...."
3. Third Heaven - Where God dwells
A. (1 Kings 8:27) - "But will God really dwell on earth? The heavens, even the highest
heaven, cannot contain you."

(Psalm 2:4) - The One enthroned in heaven laughs; The LORD scoffs at them."

Was Jesus' witness of Himself true or not?


John 8:14 and John 5:31

1. Not true (John 5:31) - "If I alone bear witness of Myself, My testimony is not true. 32"There is
another who bears witness of Me, and I know that the testimony which He bears of Me is true."
2. Is true (John 8:14) - "Jesus answered and said to them, "Even if I bear witness of Myself, My
witness is true; for I know where I came from, and where I am going; but you do not know
where I come from, or where I am going."

In John 5:31, the context is Jesus speaking about how He depends upon the Father and how He is
seeking the will of the Father. John 5:30-32 says, "I can do nothing on My own initiative. As I hear, I
judge; and My judgment is just, because I do not seek My own will, but the will of Him who sent Me.
31
"If I alone bear witness of Myself, My testimony is not true. 32"There is another who bears witness of
Me, and I know that the testimony which He bears of Me is true." The word "alone" is not in the Greek
but is included in the NASB translation, though not the NIV, the KJV. Contextually, Jesus is not
speaking as one alone, but as one dependent on the Father and that His judgments are true because
He does the will of the Father. Jesus is reflecting on the Old Testament law that didn't allow the
testimony of one person to condemn another to death. Two witnesses were needed to establish the
fact:

"One witness shall not rise up against a man for any iniquity, or for any sin, in any sin
that he sinneth: at the mouth of two witnesses, or at the mouth of three witnesses,
shall the matter be established," (Deut. 19:15). and Matthew 8:16 says, ". . . in the
mouth of two or three witnesses every word may be established." See also 2 Cor. 13:1;
Heb. 10:28.

In John 8:14, Jesus says, "IF" (kan, in the Greek) I bear witness of Myself, My witness is true. But
He was speaking of being the light of the word, v. 12, and the Pharisees accused Him of bearing
witness of Himself. Jesus was simply telling the truth that if He did, it would be true.
Who did Jesus see first upon His arrest, Annas or Caiaphas?
Matthew 26:57; John 18:19,24

1. Led to Caiaphas (Matthew 26:57) - "And those who had seized Jesus led Him away to
Caiaphas, the high priest, where the scribes and the elders were gathered together."
2. Led from Annas to Caiaphas (John 18:19,24) - "The high priest therefore questioned Jesus
24
about His disciples, and about His teaching . . . Annas therefore sent Him bound to Caiaphas
the high priest."

At the time of Jesus there were two high priests: Annas and Ciaphas. Luke 3:2 says, "in the high
priesthood of Annas and Caiaphas, the word of God came to John, the son of Zacharias, in the
wilderness." Also, Acts 4:5-6 says, "And it came about on the next day, that their rulers and elders
and scribes were gathered together in Jerusalem; 6and Annas the high priest was there, and Caiaphas
and John and Alexander, and all who were of high-priestly descent." John 18:19 says that the high
priest questioned Jesus and then in verse 24 it says that Annas (supposedly the high priest of verse
19), sent Jesus to Caiaphas who was, as we have seen, also High Priest. Furthermore, when Matthew
26:57 says that Jesus was led to Caiaphas, it is not denying that He first saw Annas.
Therefore, Annas saw Jesus first and then He was sent to see Caiaphas.

Shall we obey God's Law or human law?


Acts 5:29 and Romans 13:1; I Peter 2:13

1. Obey God
A. (Acts 5:29) - "But Peter and the apostles answered and said, We must obey God rather than
men."
2. Obey Man
A. (Romans 13:1) - "Let every person be in subjection to the governing authorities. For there is
no authority except from God, and those which exist are established by God."
B. (1 Peter 2:13) - "Submit yourselves for the Lord’s sake to every human institution, whether
to a king as the one in authority,"

The simple answer is that Christians are to obey human law except where that human law violates
God's Law. Our supreme duty is to obey God. Since God tells us to also obey human laws, we
should. But, when they come in conflict, we are to "obey God rather than men."

When Paul saw the light, did all fall to the ground or not?
Acts 9:3-4 and Acts 26:13-14

1. (Acts 9:3-4) - "And it came about that as he journeyed, he was approaching Damascus, and
suddenly a light from heaven flashed around him; 4and he fell to the ground, and heard a voice
saying to him, "Saul, Saul, why are you persecuting Me?"
2. (Acts 26:13-14) - "at midday, O King, I saw on the way a light from heaven, brighter than the
sun, shining all around me and those who were journeying with me. 14"And when we had all
fallen to the ground, I heard a voice saying to me in the Hebrew dialect, ‘Saul, Saul, why are
you persecuting Me? It is hard for you to kick against the goads.’

Of course, there is no contradiction here at all. If all fell to the ground, then Paul who was with the
group of men, fell to the ground also. Just because one verse says all and the other mentions only
Paul does not mean there is a problem.
Did the men with Paul hear the voice or not?
Acts 9:7 and Acts 22:9

1. They heard the voice


A. (Acts 9:7, NASB) - "And the men who traveled with him stood speechless, hearing the voice,
but seeing no one."
B. (Acts 9:7, NIV) - "The men traveling with Saul stood there speechless; they heard the sound
but did not see anyone."
C. (Acts 9:7, KJV) - "And the men which journeyed with him stood speechless, hearing a voice,
but seeing no man"
2. They did not hear the voice
A. (Acts 22:9, KJV) - "And they that were with me saw indeed the light, and were afraid; but
they heard not the voice of him that spake to me."
B. (Acts 22:9, NASB) - "And those who were with me beheld the light, to be sure, but did not
understand the voice of the One who was speaking to me."
C. (Acts 22:9, NIV) "My companions saw the light, but they did not understand the voice of him
who was speaking to me."

This is an interesting difficulty to tackle. As you can see from the different translations above, at
attempt has been made to harmonize the difficulty by translating Acts 22:9 as "did not understand the
voice," (NASB & NIV) where the KJV states "... they heard not the voice.." Literally, the Greek in 22:9
says, "they did not hear the sound." So, did they or did they not hear the sound?
Various explanations have been offered but the most common is summed up in the following
quotes.

• "Literally, that clause in 22:9 may be translated, “They did not hear the sound.” The
NIV correctly translates the verse, because the verb “to hear” with the genitive case may mean
“to hear a sound” and with the accusative case “to hear with understanding.” The genitive case
is employed in 9:7, and the accusative is used in 22:9. So the travelers with Saul heard the
sound (9:7) but did not understand what Christ said (22:9)."86

• Thus in Acts 9:7, “hearing the voice,” the noun “voice” is in the partitive genitive
case [i.e., hearing (something) of], whereas in 22:9, “they heard not the voice,” the
construction is with the accusative. This removes the idea of any contradiction. The former
indicates a hearing of the sound, the latter indicates the meaning or message of the voice (this
they did not hear). “The former denotes the sensational perception, the latter (the accusative
case) the thing perceived” (Cremer). 87

86
Walvoord, John F., and Zuck, Roy B., The Bible Knowledge Commentary, (Wheaton, Illinois: Scripture Press
Publications, Inc.) 1983, 1985.
87
Vine, W. E., Vine’s Expository Dictionary of Old and New Testament Words, (Grand Rapids, MI: Fleming H.
Revell) 1981.
Have all people sinned or not?
Romans 3:23; Romans 3:10; Psalm 14:3 and Job 1:1; Genesis 7:1; Luke 1:6

1. All have sinned


A. (Romans 3:9-10) - "What then? Are we better than they? Not at all; for we have already
charged that both Jews and Greeks are all under sin; 10as it is written, "There is none
righteous, not even one."
B. (Romans 3:23) - "for all have sinned and fall short of the glory of God."
C. (Psalm 14:3) - "They have all turned aside; together they have become corrupt; there is no
one who does good, not even one."
2. All have not sinned
A. (Job 1:1) - "There was a man in the land of Uz, whose name was Job, and that man was
blameless, upright, fearing God, and turning away from evil."
B. (Genesis 7:1) - "Then the Lord said to Noah, "Enter the ark, you and all your household; for
you alone I have seen to be righteous before Me in this time."
C. (Luke 1:5-6) - "In the days of Herod, king of Judea, there was a certain priest named
Zacharias, of the division of Abijah; and he had a wife from the daughters of Aaron, and her
name was Elizabeth. 6And they were both righteous in the sight of God, walking blamelessly
in all the commandments and requirements of the Lord."

The Bible clearly teaches that all people have sinned -- except Jesus (1 Peter 2:22). Romans 3:23
clearly condemns all under sin. But when it mentions people like Job, Noah, Zacharias, and Elizabeth
as people who were "blameless" and "righteous" it is not saying that they are not sinners. It is saying
that they were godly people, who kept the commandments of God and in that sense, they were
righteous. But of course, we realize that no one can keep the commandments of God perfectly which
is why all people are deserving of damnation (Eph. 2:3) and why we need a savior. If righteousness
can come through the Law, then Christ died needlessly (Gal. 2:21).

Are we saved by faith or by baptism?


Romans 5:1; Ephesians 2:8-9 and Acts 2:38; 22:16; 1 Peter 3:21

1. Saved by faith
A. (Romans 5:1) - "Therefore having been justified by faith, we have peace with God through
our Lord Jesus Christ."
B. (Ephesians 2:8-9) - "For by grace you have been saved through faith; and that not of
yourselves, it is the gift of God; 9 not as a result of works, that no one should boast."
2. Saved by baptism
A. (Acts 2:38) - "And Peter said to them, "Repent, and let each of you be baptized in the name
of Jesus Christ for the forgiveness of your sins; and you shall receive the gift of the Holy
Spirit."
B. (Acts 22:16) - "And now why do you delay? Arise, and be baptized, and wash away your
sins, calling on His name."
C. (1 Peter 3:21) - "And corresponding to that, baptism now saves you—not the removal of dirt
from the flesh, but an appeal to God for a good conscience—through the resurrection of
Jesus Christ,"

There is much debate within Christianity as to whether or not baptism is necessary for salvation. I
cannot here exhaustively examine this issue, but I can affirm that baptism is not necessary for
salvation. The scriptures teach that justification is by faith (Rom. 5:1). It also teaches that baptism is
a necessary result of becoming a disciple of Christ (Matt. 28:18-19). Even 1 Peter 3:21 above says
that the baptism mentioned is not one dealing with water, but an appeal to God.
God works covenantally. A covenant is a pact or agreement between two or more parties. The New
Testament and Old Testaments are New and Old Covenants. The word "testament" comes from the
Latin testamentum which means covenant. So, the Bible is a covenant document. If you don't
understand covenant you cannot understand, in totality, the issue of baptism because baptism is a
covenant sign. Covenant signs do not save. The things they represent are what save.
Regeneration occurs by faith (Rom. 5:1). Afterwards, baptism is administered as an outward
representation of an inward reality. For example, it represents the reality of the inward washing of
Christ's blood upon the soul. That is why it is used in different ways. It is said to represent the death
of the person (Rom. 6:3-5), the union of that person with Christ (Gal. 3:27), the cleansing of that
person's sins (Acts 22:16), the identification with the one "baptized into" as when the Israelites were
baptized into Moses (1 Cor. 10:2), and being united in one church (1 Cor. 12:13). Also, baptism is one
of the signs and seals of the Covenant of Grace that was instituted by Jesus.
Baptism is not a requirement of salvation, but it is so closely tied to it that some people erringly
think it is the actual thing that saves. It isn't. Faith in Christ is what saves.
For a more complete analysis of this issue, please see Is Baptism Necessary for Salvation?

God hardened Pharaoh's heart. Is that right?


Romans 9:17-18

(Romans 9:17-18) - "For the Scripture says to Pharaoh, "For this very purpose I raised you up,
to demonstrate My power in you, and that My name might be proclaimed throughout the whole
earth." 18So then He has mercy on whom He desires, and He hardens whom He desires."

Romans 9:9-23 is some of the most controversial scripture in the Bible. When reading through it,
one quickly finds the sovereignty of God in distinction to the free will of man. What is going on?
Basically, God has the right to do with His creation as He wills. We see from the Word that God is
in control, "For truly in this city there were gathered together against Thy holy servant Jesus, whom
Thou didst anoint, both Herod and Pontius Pilate, along with the Gentiles and the peoples of Israel, 28
to do whatever Thy hand and Thy purpose predestined to occur," (Acts 4:27-28). In other words, God
is in control. God can also move peoples' hearts (Prov. 21:1) and directs history to where He wants it
to go.
God raised up Pharaoh for a purpose: to demonstrate His power. How was this done? It was done
by working miracles through Moses and delivering the Israelites. Does God have the right and ability
to harden whom He desires in the process of accomplish His will? Absolutely for that is what it says in
9:18. Does this make God wrong in anyway? Not at all. God can do no wrong. Pharaoh was a sinner
who deserved the righteous judgment of God. Some say that God simply strengthened Pharaoh's
heart towards its natural tendency. Others maintain that God actively hardened his the heart.
Whichever the case, Pharaoh rejected the true and living God and God used him for His own purpose.

Is the Lord a God of Peace or of war?


Romans 15:33; Isaiah 2:4 and Exodus 15:3; Joel 3:9-10

1. God of Peace
A. (Isaiah 2:4) - "And He will judge between the nations, and will render decisions for many
peoples; and they will hammer their swords into plowshares, and their spears into pruning
hooks. Nation will not lift up sword against nation, And never again will they learn war."
B. (Romans 15:33) - "Now the God of peace be with you all. Amen."
2. God of War
A. (Exodus 15:3) - "The Lord is a warrior; the Lord is His name."
B. (Joel 3:9-10) - "Proclaim this among the nations: Prepare a war; rouse the mighty men! Let
all the soldiers draw near, let them come up! Beat your plowshares into swords, 10And your
pruning hooks into spears; Let the weak say, “I am a mighty man.”

Anyone can take verses out of context and compare them to other verses out of context and get a
"contradiction." But, context is sacrificed in this manner and along with it, truth is lost. In Isaiah 2:4,
God is giving a prophet announcement of a future time when He will be the one who settles disputes
and there will be no more war. In Romans 15:33, it is simply said that God is a God of peace. He is.
Yet we have the verses that show God's judgmental side. In Exodus 15:3 we see God as a warrior.
But the context is the destruction of the Egyptian Army. As we all know, Egypt had enslaved the
Israelite nation and God simply became their warrior and delivered them. In Joel 3:9-10 we see a
prophetic statement as the book of Joel clearly is prophetic in nature. In other words, there will come
a time when it is required to fight.
There is no contradiction for God to be both the Lord who battles unrighteousness and also loves
peace. This is just as true with people, who are really peaceful by nature, but will fight when the time
requires it.

Will wisdom stand or not?


1 Corinthians 1:19 and Proverbs 4:7

1. (Proverbs 4:7) - "The beginning of wisdom is: Acquire wisdom; and with all your acquiring, get
understanding.
2. (1 Corinthians 1:19) - "For it is written, "I will destroy the wisdom of the wise, and the
cleverness of the clever I will set aside."

Context is vital here. Proverbs 4:7 is simply stating that a person, on the human level, needs to
acquire wisdom. It is good to seek to be wise. Solomon is teaching his son, who is a believer in God
and His word, to avoid foolishness and seek wisdom. Of course, true wisdom is acquired from God's
word. The context of 1 Corinthians 1:19 deals with comparing the wisdom of God with the wisdom of
man. Paul writes that the wisdom of man is foolishness compared to the wisdom of God. It will be
destroyed. In other words, the unbelievers who try and defy God or proclaim that God is not true or
that the Bible is not true will all be proven false. This will either happen in this life or on the Day of
Judgment.

Is baptism for the dead really Christian?


1 Corinthians 15:29

Just north of Corinth was a city named Eleusis. This was the location of a pagan religion where
baptism in the sea was practiced to guarantee a good afterlife. This religion was mention by Homer in
Hymn to Demeter 478-79.88 The Corinthians were known to be heavily influenced by other customs.
After all, they were in a large economic area where a great many different people frequented. It is
probable that the Corinthians were being influenced by the religious practices found at Eleusis where
baptism for the dead was practiced.
Paul used this example from the pagans in 1 Cor. 15:29, when he said, "...if the dead are not
raised, then why are they baptized for the dead?" Paul did not say we. This is significant because the
Christian church was not practicing baptism for the dead, but the pagans were.
Paul's point was simple. The resurrection is a reality. It is going to happen when Jesus returns.
Even the pagans believe in the resurrection, otherwise, why would they baptize for the dead?

88
Bible Knowledge Commentary on 1 Cor. 15:29. Dallas Seminary Faculty.
How many children did Abraham have, one or two?
Genesis 22:2; Hebrews 11:17 and Galatians 4:22

1. One son
A. (Genesis 22:2) - "And He said, "Take now your son, your only son, whom you love, Isaac,
and go to the land of Moriah; and offer him there as a burnt offering on one of the
mountains of which I will tell you."
B. (Hebrews 11:17) - "By faith Abraham, when he was tested, offered up Isaac; and he who
had received the promises was offering up his only begotten son."
2. Two sons
A. (Galatians 4:22) - "For it is written that Abraham had two sons, one by the bondwoman and
one by the free woman."

The answer to this apparent contradiction is found in understanding the typological representation
of Isaac, Abraham's second born son, as a type of Christ. Abraham had Ishmael by the handmaiden
Hagar. But Isaac was the child of promise, not Ishmael: "But God said to Abraham, "Do not be
distressed because of the lad and your maid; whatever Sarah tells you, listen to her, for through Isaac
your descendants shall be named,'" (Gen. 21:12).
If you look at the chart below, you will see the similarities between Isaac and Jesus. In other
words, Isaac was a prophetic representation of Jesus. This is why Jesus said, "Your father Abraham
rejoiced to see My day, and he saw it and was glad," (John 8:56). Abraham had, in a very real sense,
seen the gospel presentation in the offering of his son, his "only begotten." So, we see here that the
term "only begotten" is in reference to the unique son of God and Isaac was acting out the sacrifice of
Christ, prophetically.

ISAAC JESUS
Only begotten Son Genesis 22:2 John 3:16
Offered on a mountain, hill 22:2 Matt. 21:10
Took donkey to place of sacrifice 22:3 Matt. 21:2-11
Two men went with him. 22:3 Mark 15:27; Luke 23:33
Three day journey. Jesus: three days in the
grave 22:4 Luke 24:13-21
Son carried wood on his back up hill 22:6 John 19:17
God will provide for Himself the lamb/Jesus 22:8 John 1:29
Son was offered on the wood/cross 22:9 Luke 23:33
Ram in thicket of thorns/crown of thorns 22:13 John 19:2
The seed will be multiplied/believers 22:17 John 1:12; Isaiah 53:10
Abraham went down, Son didn't, "not
mentioned." 22:19 Luke 23:46
Eph. 5:22-32;
Servant gets bride for son 24:1-4 Rev. 21:2,9; 22:17
The bride was a beautiful virgin 24:16 2 Cor. 11:2
Servant offered ten gifts to bride* 24:10 Rom. 6:23; 12; 1 Cor. 12

Also, Abraham had six other sons besides Ishmael and Isaac through his wife Kerturah whom he
married after Sarah died. "Now Abraham took another wife, whose name was Keturah. 2And she bore
to him Zimran and Jokshan and Medan and Midian and Ishbak and Shuah," (Gen. 25:1-2). Obviously,
this was known by the writers of Genesis as well as Hebrews and Galatians.
Do we bear one another's burdens or not?
Galatians 6:2 and Galatians 6:5

1. (Galatians 6:2) - "Bear one another’s burdens, and thus fulfill the law of Christ."
2. (Galatians 6:5) - "For each one shall bear his own load."

In Galatians 6:2, the word for "burdens" is baros. It means "heaviness, weight, burden,
trouble."89 In Galatians 6:5, the word for load is "phortion" and means "of burdensome rites." 90 The
first verse is speaking of helping one another through troubles and the second is speaking of the
responsibility each person has in those troubles and sins.
"The Christian does in fact test himself by carrying his own load. This does not contradict verse 2
because the reference there is to heavy, crushing, loads (baros)—more than a man could carry without
help. In this verse a different Greek word (phortion) is used to designate the pack usually carried by a
marching soldier. It is the "burden" Jesus assigns to His followers (cf. Matt. 11:30). There are certain
Christian responsibilities or burdens each believer must bear which cannot be shared with others.
Jesus assured His disciples that such burdens were light." 91

Are we saved by grace or works?


Ephesians 2:8,9; Romans 3:20,28; Galatians 2:16 and James 2:24; Matthew 19:16-
21

1. Saved by grace
A. (Ephesians 2:8-9) - "For by grace you have been saved through faith; and that not of
yourselves, it is the gift of God; 9not as a result of works, that no one should boast."
B. (Rom. 3:20,28) - "because by the works of the Law no flesh will be justified in His sight; for
through the Law comes the knowledge of sin... 28For we maintain that a man is justified by
faith apart from works of the Law."
C. (Galatians 2:16) - "nevertheless knowing that a man is not justified by the works of the Law
but through faith in Christ Jesus, even we have believed in Christ Jesus, that we may be
justified by faith in Christ, and not by the works of the Law; since by the works of the Law
shall no flesh be justified."
2. Saved by works
A. (James 2:24) - "You see that a man is justified by works, and not by faith alone."
B. (Matthew 19:1617) - "And behold, one came to Him and said, "Teacher, what good thing
shall I do that I may obtain eternal life?" 17And He said to him, "Why are you asking Me
about what is good? There is only One who is good; but if you wish to enter into life, keep
the commandments."

God does not want a faith that is empty and hypocritical. James 2 is talking about those who "say"
that they have faith but have no works. Therefore, people cannot tell if they are true believers or not,
because there is no fruit. That kind of a faith is useless and is not a saving faith. True faith results in
true works.
In Matthew 19:16-17, Jesus was speaking to a Lawyer who was self-righteous since he wanted to
put Jesus to the test (Luke 10:25). He asked what he must do in order to obtain eternal life and Jesus
responded with the requirements of keeping the commandments. If a person keeps all of the
commandments, it would seem that they could obtain eternal life. However, nobody can keep all of
the commandments. Therefore, Jesus' comments to this man show this man that justification can only
be by faith since no one can keep all of the commandments. This is why it says in Eph. 2:8 that we
are saved by grace to faith. Also, Romans 3:20,28 and Galatians 2:16 tells us that no one is justified
89
Enhanced Strong’s Lexicon, (Oak Harbor, WA: Logos Research Systems, Inc.) 1995.
90
Ibid.
91
The Bible Knowledge Commentary, (Wheaton, Illinois: Scripture Press Publications, Inc.) 1983, 1985.
in this site of God by the law; that is, by the works that he can do.
There is no contradiction at all when we examine the contexts. We are justified by faith but that
faith must be alive (James 2). The Law cannot save us because we are incapable of keeping it
(Matthew 19:16-17). Therefore, salvation is by faith through grace.

Is Jesus or God the creator of all things?


Col. 1:15-17 and Isaiah 44:24

1. God created all alone (Isaiah 44:24) - "Thus says the Lord, your Redeemer, and the one who
formed you from the womb, "I, the Lord, am the maker of all things, stretching out the
heavens by Myself, and spreading out the earth all alone."
2. All things created by/through Jesus (Colossians 1:16-17) - "For by Him all things were
created, both in the heavens and on earth, visible and invisible, whether thrones or dominions
or rulers or authorities—all things have been created by Him and for Him. 17And He is before all
things, and in Him all things hold together."

There is no difficulty here at all when we realize that the Trinity is involved. The Trinity is the doctrine
that God is three persons: Father, Son, and Holy Spirit. The Son is the Word, which was God and
with God (John 1:1), that became flesh in Jesus (John 1:14). Since Jesus is the second person of the
Trinity, and He has two natures, divine and human (Col. 2:9), we can then have Jesus being the
creator and God being the creator alone. In other words, Jesus is God and God created all things
alone.

Did Joseph worship at the head of the bed or leaning on a staff?


Genesis 47:31 and Hebrews 11:21

1. Head of bed (Genesis. 47:31) - "And he said, "Swear to me." So he swore to him. Then Israel
bowed in worship at the head of the bed."
2. Top of staff (Heb. 11:21) - "By faith Jacob, as he was dying, blessed each of the sons of
Joseph, and worshiped, leaning on the top of his staff."

This is no contradiction at all. Jacob asked his son Joseph to carry his bones out of Egypt to be
buried in Israel. Joseph agreed and then at the head of the bed, he leaned on the top of his staff and
worshipped.
A contradiction occurs when one statement makes the other impossible. This is not the case here.
Both statements are true.
Does the earth abide forever or not?
Psalm 104:5; Ecclesiastes 1:4 and Isaiah 65:17; 2 Peter 3:10

1. Abides forever
A. (Psalm 104:5) - "He established the earth upon its foundations, So that it will not totter
forever and ever."
B. (Ecclesiastes 1:4) - "A generation goes and a generation comes, but the earth remains
forever."
2. Does not abide forever
A. (Isaiah 65:17) - "For behold, I create new heavens and a new earth; And the former things
shall not be remembered or come to mind."
B. (2 Peter 3:10) - "But the day of the Lord will come like a thief, in which the heavens will
pass away with a roar and the elements will be destroyed with intense heat, and the earth
and its works will be burned up."

The context of the Ecclesiastes passage is revealed in the previous verse (3) which says, "What
advantage does man have in all his work which he does under the sun?" In other words, the
perspective of Ecclesiastes is from a completely human standpoint. The same is occurring in the
Psalms passage, a description from a human perspective. Therefore, the writers will see the earth
abiding forever because that is exactly how it appears. But, in Isaiah 65:17 and 2 Peter 3:10 the
contexts are altogether different. They are speaking of the time in the future when the new heavens
and new earth will be made. Take a look at 2 Peter.

"But the day of the Lord will come like a thief, in which the heavens will pass away with
a roar and the elements will be destroyed with intense heat, and the earth and its works
will be burned up. 11Since all these things are to be destroyed in this way, what sort of
people ought you to be in holy conduct and godliness, 12looking for and hastening the
coming of the day of God, on account of which the heavens will be destroyed by
burning, and the elements will melt with intense heat! 13But according to His promise we
are looking for new heavens and a new earth, in which righteousness dwells," (2 Peter
3:10-12).

Since sin is in the world, the earth and all its works will be destroyed and cleansed and a new
heavens and new earth will replace them.

Does God tempt people or not?


Genesis 22:1 and James 1:13

1. God tempted Abraham (Genesis 22:1) - "Now it came about after these things, that God tested
Abraham, and said to him, "Abraham!" And he said, "Here I am."
A. The KJV says, "And it came to pass after these things, that God did tempt Abraham, and
said unto him, Abraham: and he said, Behold, here I am"
2. God tempts no one (James 1:13) - "Let no one say when he is tempted, “I am being tempted
by God”; for God cannot be tempted by evil, and He Himself does not tempt anyone."

According to the Enhanced Strong’s Lexicon, the word "test" (NASB), "tempt" (KJV) is nacah. It
means "1) to test, try, 2) to attempt, assay, try, 3) to test, try, prove, tempt." This is why the KJV
translated it as "tempt" and NASB, NIV, NKJV, ESV, and RSV translate it as "test." Therefore, it was a
test that God offered to Abraham, not a temptation to sin.
Jude 14 quotes the book of Enoch. Is it scripture?
Jude 14

"And about these also Enoch, in the seventh generation from Adam, prophesied, saying,
"Behold, the Lord came with many thousands of His holy ones" (Jude 14).

There is debate whether or not Jude was actually quoting the apocraphal book of Enoch or
something else. This debate aside, if this is a quote from the book of Enoch, it does not affect the
doctrine of inspiration nor does it mean that the early church removed the book of Enoch because of
its internal inconsistencies. First of all, the book of Enoch was not considered scripture by the
Christian Church. There was some discussion on its canonicity by a few people, but the Christian
Church did not include it in the Bible. Second, Jude only quoted something that was true in Enoch and
it does not mean that Enoch was inspired. In fact, Paul quotes Epimenides in Titus 1:12 but that does
not mean that Epimenides was inspired.
Cleansing of the Temple Chart

Event Matthew 21:1-22 Mark 11:1-18 Luke 19:28-38,44-48


They came 1 And when they had approached Jerusalem and 1 And as they *approached 28 And after He had said these
to had come to Bethphage, to the Mount of Olives, Jerusalem, at Bethphage and things, He was going on ahead,
Jerusalem then Jesus sent two disciples, Bethany, near the Mount of Olives, ascending to Jerusalem.
He *sent two of His disciples,
Retrieving 2 saying to them, "Go into the village opposite 2 and *said to them, "Go into the 29 And it came about that when He
the colt you, and immediately you will find a donkey tied village opposite you, and approached Bethphage and Bethany,
there and a colt with her; untie them, and bring immediately as you enter it, you will near the mount that is called Olivet,
them to Me. find a colt tied there, on which no He sent two of the disciples,
3 "And if anyone says something to you, you one yet has ever sat; untie it and 30 saying, "Go into the village
shall say, ‘The Lord has need of them,’ and bring it here. opposite you, in which as you enter
immediately he will send them." 3 "And if anyone says to you, ‘Why you will find a colt tied, on which no
4 Now this took place that what was spoken are you doing this?’ you say, ‘The one yet has ever sat; untie it, and
through the prophet might be fulfilled, saying, Lord has need of it’; and bring it here.
5 "Say to the daughter of Zion, ‘Behold your King immediately he will send it back 31 "And if anyone asks you, ‘Why are
is coming to you, gentle, and mounted on a here." you untying it?’ thus shall you speak,
donkey, even on a colt, the foal of a beast of 4 And they went away and found a ‘The Lord has need of it.’"
burden.’" colt tied at the door outside in the 32 And those who were sent went
6 And the disciples went and did just as Jesus street; and they *untied it. away and found it just as He had told
had directed them, 5 And some of the bystanders were them.
saying to them, "What are you 33 And as they were untying the colt,
doing, untying the colt?" 6 And they its owners said to them, "Why are
spoke to them just as Jesus had told you untying the colt?"
them, and they gave them 34 And they said, "The Lord has need
permission. of it."
garments 7 and brought the donkey and the colt, and laid 7 And they *brought the colt to 35 And they brought it to Jesus, and
on colt on them their garments, on which He sat. Jesus and put their garments on it; they threw their garments on the
and He sat upon it. colt, and put Jesus on it.
garments 8 And most of the multitude spread their 8 And many spread their garments 36 And as He was going, they were
on ground garments in the road, and others were cutting in the road, and others spread leafy spreading their garments in the road
branches from the trees, and spreading them in branches which they had cut from
the road. the fields.
Hosanna! 9 And the multitudes going before Him, and 9 And those who went before, and 37 And as He was now approaching,
those who followed after were crying out, saying, those who followed after, were near the descent of the Mount of
"Hosanna to the Son of David; blessed is He who crying out, "Hosanna! Blessed is He Olives, the whole multitude of the
comes in the name of the Lord; Hosanna in the who comes in the name of the Lord; disciples began to praise God joyfully
highest!" 10 Blessed is the coming kingdom of with a loud voice for all the miracles
our father David; Hosanna in the which they had seen,
highest!" 38 saying, "Blessed is the King who
comes in the name of the Lord;
Peace in heaven and glory in the
highest!"
entered 10 And when He had entered Jerusalem, all the 11 And He entered Jerusalem and
Jerusalem city was stirred, saying, "Who is this?" came into the temple; and after
11 And the multitudes were saying, "This is the looking all around, He departed for
prophet Jesus, from Nazareth in Galilee." Bethany with the twelve, since it
was already late.
Jesus 12 And Jesus entered the temple and cast out all 45 And He entered the temple and
cleanses those who were buying and selling in the temple, began to cast out those who were
the Temple and overturned the tables of the moneychangers selling,
and the seats of those who were selling doves.
Jesus 13 And He *said to them, "It is written, ‘My 46 saying to them, "It is written,
speaks to house shall be called a house of prayer’; but you ‘And My house shall be a house of
the people are making it a robbers‘ den." prayer,’ but you have made it a
14 And the blind and the lame came to Him in robbers‘ den."
the temple, and He healed them.
15 But when the chief priests and the scribes
47 And He was teaching daily in the
saw the wonderful things that He had done, and
temple; but the chief priests and the
the children who were crying out in the temple
scribes and the leading men among
and saying, "Hosanna to the Son of David," they
the people were trying to destroy
became indignant,
Him,
16 and said to Him, "Do You hear what these are
48 and they could not find anything
saying?" And Jesus *said to them, "Yes; have
that they might do, for all the people
you never read, ‘Out of the mouth of infants and
were hanging upon His words.
nursing babes Thou hast prepared praise for
Thyself’?"
leaves city 17 And He left them and went out of the city to
Bethany, and lodged there.
Jesus 18 Now in the morning, when He returned to the 12 And on the next day, when they
curses the city, He became hungry. had departed from Bethany, He
fig tree 19 And seeing a lone fig tree by the road, He became hungry.
came to it, and found nothing on it except leaves 13 And seeing at a distance a fig
only; and He *said to it, "No longer shall there tree in leaf, He went to see if
ever be any fruit from you." And at once the fig perhaps He would find anything on
tree withered. it; and when He came to it, He
20 And seeing this, the disciples marveled, found nothing but leaves, for it was
saying, "How did the fig tree wither at once?" not the season for figs.
21 And Jesus answered and said to them, "Truly 14 And He answered and said to it,
I say to you, if you have faith, and do not doubt, "May no one ever eat fruit from you
you shall not only do what was done to the fig again!" And His disciples were
tree, but even if you say to this mountain, ‘Be listening.
taken up and cast into the sea,’ it shall happen.
22 "And all things you ask in prayer, believing,
you shall receive."
Jesus 15 And they *came to Jerusalem.
cleanses And He entered the temple and
the Temple began to cast out those who were
buying and selling in the temple,
and overturned the tables of the
moneychangers and the seats of
those who were selling doves;
16 and He would not permit anyone
to carry goods through the temple.
17 And He began to teach and say
to them, "Is it not written, ‘My house
shall be called a house of prayer for
all the nations’? But you have made
it a robbers‘ den."
18 And the chief priests and the
scribes heard this, and began
seeking how to destroy Him; for
they were afraid of Him, for all the
multitude was astonished at His
teaching.
Crucifixion Chronology Chart

Event Matthew 27:33-56 Mark 15:22-41 Luke 23:33-49 John 19:17-30


took him 33 And when they had come to 22 And they *brought Him to 33 And when they came to the 17 They took Jesus therefore,
to the a place called Golgotha, which the place Golgotha, which is place called The Skull, there and He went out, bearing His
Place of means Place of a Skull, translated, Place of a Skull. they crucified Him and the own cross, to the place called
the Skull criminals, one on the right and the Place of a Skull, which is
the other on the left. called in Hebrew, Golgotha.
First 34 But Jesus was saying, (23 The soldiers therefore,
Words of "Father, forgive them; for they when they had crucified Jesus,
Christ do not know what they are took His outer garments and
doing." And they cast lots, made four parts, a part to
dividing up His garments every soldier and also the
Cast lots
among themselves. tunic; now the tunic was
for His
35 And the people stood by, seamless, woven in one piece.
clothes
looking on. And even the 24 They said therefore to one
rulers were sneering at Him, another, "Let us not tear it, but
saying, "He saved others; let cast lots for it, to decide whose
Him save Himself if this is the it shall be"; that the Scripture
Christ of God, His Chosen might be fulfilled, "They
One." divided My outer garments
among them, and for My
clothing they cast lots.")
wine 34 they gave Him wine to drink 23 And they tried to give Him 36 And the soldiers also
mixed mingled with gall; and after wine mixed with myrrh; but mocked Him, coming up to
with gall tasting it, He was unwilling to He did not take it. Him, offering Him sour wine,
and drink.
myrrh
sign says 35 And when they had crucified 24 And they *crucified Him, 37 and saying, "If You are the 18 There they crucified Him,
King of Him, they divided up His and *divided up His garments King of the Jews, save and with Him two other men,
the Jews garments among themselves, among themselves, casting Yourself!" 38 Now there was one on either side, and Jesus
casting lots; lots for them, to decide what also an inscription above Him, in between. 19 And Pilate
36 and sitting down, they each should take. 25 And it "THIS IS THE KING OF THE wrote an inscription also, and
began to keep watch over Him was the third hour when they JEWS." put it on the cross. And it was
there. crucified Him. written, "JESUS THE
37 And they put up above His 26 And the inscription of the NAZARENE, THE KING OF THE
head the charge against Him charge against Him read, "THE JEWS."
which read, "THIS IS JESUS KING OF THE JEWS."
THE KING OF THE JEWS."
sign 20 Therefore this inscription
above many of the Jews read, for the
head in place where Jesus was
Hebrew crucified was near the city; and
Latin and it was written in Hebrew, Latin,
Greek and in Greek.
21 And so the chief priests of
the Jews were saying to Pilate,
"Do not write, ‘The King of the
Jews’; but that He said, ‘I am
King of the Jews.’"
22 Pilate answered, "What I
have written I have written."
thieves 38 At that time two robbers 27 And they *crucified two
on right *were crucified with Him, one robbers with Him, one on His
and left on the right and one on the right and one on His left.
left. 28 [And the Scripture was
fulfilled which says, "And He
was numbered with
transgressors."]
hurling 39 And those passing by were 29 And those passing by were 39 And one of the criminals
insults hurling abuse at Him, wagging hurling abuse at Him, wagging who were hanged there was
their heads, their heads, and saying, "Ha! hurling abuse at Him, saying,
40 and saying, "You who are You who are going to destroy "Are You not the Christ? Save
going to destroy the temple and the temple and rebuild it in Yourself and us!"
rebuild it in three days, save three days, 40 But the other answered,
Yourself! If You are the Son of 30 save Yourself, and come and rebuking him said, "Do
God, come down from the down from the cross!" you not even fear God, since
cross." 31 In the same way the chief you are under the same
41 In the same way the chief priests also, along with the sentence of condemnation?
priests also, along with the scribes, were mocking Him 41 "And we indeed justly, for
scribes and elders, were among themselves and we are receiving what we
mocking Him, and saying, saying, "He saved others; He deserve for our deeds; but
42 "He saved others; He cannot cannot save Himself. this man has done nothing
save Himself. He is the King of 32 "Let this Christ, the King of wrong."
Israel; let Him now come down Israel, now come down from 42 And he was saying, "Jesus,
from the cross, and we shall the cross, so that we may see remember me when You come
believe in Him. and believe!" And those who in Your kingdom!"
43 "He trusts in God; let Him were crucified with Him were
deliver Him now, if He takes casting the same insult at
pleasure in Him; for He said, ‘I Him.
am the Son of God.’"
44 And the robbers also who
had been crucified with Him
were casting the same insult at
Him.
Second 43 And He said to him, "Truly
Words of I say to you, today you shall
Christ be with Me in Paradise."
sixth 45 Now from the sixth hour 33 And when the sixth hour 44 And it was now about the
hour darkness fell upon all the land had come, darkness fell over sixth hour, and darkness fell
until the ninth hour. the whole land until the ninth over the whole land until the
hour. ninth hour,
Third (or 46 And about the ninth hour 34 And at the ninth hour Jesus
Fourth) Jesus cried out with a loud cried out with a loud voice,
Words of voice, saying, "Eli, Eli, lama "Eloi, Eloi, lama sabachthani?"
Christ. sabachthani?" that is, "My God, which is translated, "My God,
My God, why hast Thou My God, why hast Thou
forsaken Me?" forsaken Me?"
calling for 47 And some of those who were 35 And when some of the
Elijah standing there, when they bystanders heard it, they
heard it, began saying, "This began saying, "Behold, He is
man is calling for Elijah." calling for Elijah."
another 48 And immediately one of 36 And someone ran and filled
drink them ran, and taking a sponge, a sponge with sour wine, put
he filled it with sour wine, and it on a reed, and gave Him a
put it on a reed, and gave Him drink, saying, "Let us see
a drink. whether Elijah will come to
49 But the rest of them said, take Him down."
"Let us see whether Elijah will
come to save Him."
three 25 Therefore the soldiers did
women these things. But there were
standing standing by the cross of Jesus
by the His mother, and His mother’s
cross sister, Mary the wife of Clopas,
and Mary Magdalene.
26 When Jesus therefore saw
His mother, and the disciple
Fourth whom He loved standing
(or nearby, He *said to His
Third) mother, "Woman, behold, your
Words of son!"
Christ. 27 Then He *said to the
disciple, "Behold, your
mother!" And from that hour
the disciple took her into his
own household.
28 After this, Jesus, knowing
that all things had already
been accomplished, in order
that the Scripture might be
Fifth
fulfilled, *said, "I am thirsty."
Words of
29 A jar full of sour wine was
Christ.
standing there; so they put a
sponge full of the sour wine
upon a branch of hyssop, and
brought it up to His mouth.
Sixth 30a When Jesus therefore had
Words of received the sour wine, He
Christ. said, "It is finished!"
45 the sun being obscured;
and the veil of the temple was
Seventh torn in two.
Words of 46a And Jesus, crying out with
Christ. a loud voice, said, "Father,
into Thy hands I commit My
spirit."
50 And Jesus cried out again
Jesus 37 And Jesus uttered a loud 46bAnd having said this, He 30b And He bowed His head,
with a loud voice, and yielded
dies cry, and breathed His last. breathed His last. and gave up His spirit.
up His spirit.
51 And behold, the veil of the 38 And the veil of the temple
Veil is
temple was torn in two from was torn in two from top to
torn,
top to bottom, and the earth bottom.
earth
shook; and the rocks were
shakes
split,
52 and the tombs were opened;
tombs and many bodies of the saints
opened who had fallen asleep were
and raised;
people 53 and coming out of the tombs
rise from after His resurrection they
dead entered the holy city and
appeared to many.
54 Now the centurion, and 39 And when the centurion, 47 Now when the centurion
those who were with him who was standing right in saw what had happened, he
keeping guard over Jesus, when front of Him, saw the way He began praising God, saying,
Centurion they saw the earthquake and breathed His last, he said, "Certainly this man was
said... the things that were happening, "Truly this man was the Son innocent."
became very frightened and of God!"
said, "Truly this was the Son of
God!"
55 And many women were 40 And there were also some 48 And all the multitudes who
there looking on from a women looking on from a came together for this
distance, who had followed distance, among whom were spectacle, when they observed
Jesus from Galilee, ministering Mary Magdalene, and Mary the what had happened, began to
women to Him, mother of James the Less and return, beating their breasts.
looking 56 among whom was Mary Joses, and Salome. 49 And all His acquaintances
from Magdalene, along with Mary the 41 And when He was in and the women who
distance mother of James and Joseph, Galilee, they used to follow accompanied Him from
and the mother of the sons of Him and minister to Him; and Galilee, were standing at a
Zebedee. there were many other women distance, seeing these things.
who had come up with Him to
Jerusalem.
Total statistics of population from Ezra 2 and Nehemiah 7

Ezra 2 Nehemiah 7 Difference


3 8
the sons of Parosh, 2,172 the sons of Parosh, 2,172 0
4 9
the sons of Shephatiah, 372 the sons of Shephatiah, 372 0
5 10
the sons of Arah, 775 the sons of Arah, 652 123
6 11
the sons of Pahath-moab of the sons of the sons of Pahath-moab of the sons of 6
Jeshua and Joab, 2,812 Jeshua and Joab, 2,818
7 12
the sons of Elam, 1,254 the sons of Elam, 1,254 0
8 13
the sons of Zattu, 945 the sons of Zattu, 845 100
9 14
the sons of Zaccai, 760 the sons of Zaccai, 760 0
10 15
the sons of Bani, 642 the sons of Binnui, 648 6
11 16
the sons of Bebai, 623 the sons of Bebai, 628 5
12 17
the sons of Azgad, 1,222 the sons of Azgad, 2,322 1,100
13 18
the sons of Adonikam, 666 the sons of Adonikam, 667 1
14 19
the sons of Bigvai, 2,056 the sons of Bigvai, 2,067 11
15 20
the sons of Adin, 454 the sons of Adin, 655 201
16 21
the sons of Ater of Hezekiah, 98 the sons of Ater, of Hezekiah, 98 0
22
the sons of Hashum, 328 n/a
17 23
the sons of Bezai, 323 the sons of Bezai, 324 1
18
the sons of Jorah, 112 n/a
24
the sons of Hariph, 112 n/a
25
the sons of Gibeon, 95 n/a
19
the sons of Hashum, 223 n/a
20
the sons of Gibbar, 95 n/a
21 26
the men of Bethlehem, 123 the men of Bethlehem and Netophah,
188
22
the men of Netophah, 56 n/a
23 27
the men of Anathoth, 128 the men of Anathoth, 128 0
24 28
the sons of Azmaveth, 42 the men of Beth-azmaveth, 42 0
25 29
the sons of Kiriath-arim, Chephirah, and the men of Kiriath-jearim, Chephirah, 0
Beeroth, 743 and Beeroth, 743
26 30
the sons of Ramah and Geba, 621 the men of Ramah and Geba, 621 0
27 31
the men of Michmas, 122 the men of Michmas, 122 0
28 32
the men of Bethel and Ai, 223 the men of Bethel and Ai, 123 100
29 33
the sons of Nebo, 52 the men of the other Nebo, 52 0
30
the sons of Magbish, 156 n/a
31 34
the sons of the other Elam, 1,254 the sons of the other Elam, 1,254 0
32 35
the sons of Harim, 320 the sons of Harim, 320 0
34 36
the men of Jericho, 345 the men of Jericho, 345 0
33 37
the sons of Lod, Hadid, and Ono, 725 the sons of Lod, Hadid, and Ono, 721 4
35 38
the sons of Senaah, 3,630. the sons of Senaah, 3,930. 300
36 39
The priests: the sons of Jedaiah of the The priests: the sons of Jedaiah of the 0
house of Jeshua, 973 house of Jeshua, 973
37 40
the sons of Immer, 1,052 the sons of Immer, 1,052 0
38 41
the sons of Pashhur, 1,247 the sons of Pashhur, 1,247 0
39 42
the sons of Harim, 1,017 the sons of Harim, 1,017 0
40 43
The Levites: the sons of Jeshua and The Levites: the sons of Jeshua, of 0
Kadmiel, of the sons of Hodaviah, 74 Kadmiel, of the sons of Hodevah, 74
41 44
The singers: the sons of Asaph, 128 The singers: the sons of Asaph, 148 20
42 45
The sons of the gatekeepers: the sons of The gatekeepers: the sons of Shallum, 1
Shallum, the sons of Ater, the sons of the sons of Ater, the sons of Talmon, the
Talmon, the sons of Akkub, the sons of sons of Akkub, the sons of Hatita, the sons
Hatita, the sons of Shobai, in all 139 of Shobai, 138
verses 43 through 58 give numerous names verses 46 through 60 give numerous 0
with one total of 392 names with one total of 392
verses 59 through 60 list several names verses 61 through 62 list numerous names 10
with one total of 652 with one total of 642
64 66
The whole assembly numbered 42,360 The whole assembly together was 0
42,360,
65 67
besides their male and female servants, besides their male and their female 45
who numbered 7,337; and they had 200 servants, of whom there were 7,337; and (singers)
singing men and women. they had 245 male and female singers.
66 68
Their horses were 736; their mules, 245; Their horses were 736; their mules, 0
245;
67 69
their camels, 435; their donkeys, 6,720. their camels, 435; their donkeys, 6,720. 0
69 70
According to their ability they gave to the And some from among the heads of n/a
treasury for the work 61,000 gold fathers’ households gave to the work. The (subjects are
drachmas, and 5,000 silver minas, and 100 governor gave to the treasury 1,000 gold different)
priestly garments. drachmas, 50 basins, 530 priests’
garments.
71
And some of the heads of fathers’ n/a
households gave into the treasury of the
work 20,000 gold drachmas, and 2,200
silver minas.
72
And that which the rest of the people n/a
gave was 20,000 gold drachmas and 2,000
silver minas, and 67 priests’ garments.
Resurrection Chronology Chart

Event Matt. 28:1-15 Mark 16:1-20 Luke 24:1-12 John 20:1-18


First day of 1 Now after the 1 And when the Sabbath was over, Mary 1 But on the first day of 1a Now on the first day of the
the week Sabbath, as it began to Magdalene, and Mary the mother of the week, at early dawn, week Mary Magdalene *came
dawn toward the first James, and Salome, bought spices, that they came to the tomb, early to the tomb, while it
day of the week, Mary they might come and anoint Him. bringing the spices which *was still dark,
(Sunday)
Magdalene and the 2 And very early on the first day of the they had prepared.
other Mary came to look week, they *came to the tomb when the
at the grave. sun had risen.
2 And behold, a severe 3 And they were saying to one another,
earthquake had "Who will roll away the stone for us from
occurred, for an angel the entrance of the tomb?"
of the Lord descended
from heaven and came
and rolled away the
stone and sat upon it.
3 And his appearance
was like lightning, and
his garment as white as
snow;
4 and the guards shook
for fear of him, and
became like dead men
the stone 4 And looking up, they *saw that the 2 And they found the 1b and *saw the stone already
was stone had been rolled away, although it stone rolled away from taken away from the tomb
moved was extremely large. the tomb,
Mary 2 And so she *ran and *came
Magdalene to Simon Peter, and to the
ran and other disciple whom Jesus
told Peter loved, and *said to them,
"They have taken away the
Lord out of the tomb, and we
do not know where they have
laid Him."
Peter at 3 Peter therefore went forth,
tomb first 12 [But Peter arose and and the other disciple, and
ran to the tomb; stooping they were going to the tomb.
and looking in, he *saw 4 And the two were running
the linen wrappings only; together; and the other disciple
and he went away to his ran ahead faster than Peter,
home, marveling at that and came to the tomb first;
which had happened.] 5 and stooping and looking in,
he *saw the linen wrappings
lying there; but he did not go
in.
6 Simon Peter therefore also
*came, following him, and
entered the tomb; and he
*beheld the linen wrappings
lying there,
7 and the face-cloth, which had
been on His head, not lying
with the linen wrappings, but
rolled up in a place by itself.
8 So the other disciple who had
first come to the tomb entered
then also, and he saw and
believed.
9 For as yet they did not
understand the Scripture, that
He must rise again from the
dead.
10 So the disciples went away
again to their own homes.
three 5 And entering the tomb, they saw a 3 but when they entered, 11 But Mary was standing
women at young man sitting at the right, wearing a they did not find the body outside the tomb weeping; and
the tomb white robe; and they were amazed. of the Lord Jesus. so, as she wept, she stooped
(Mk 16:1) and looked into the tomb;
two men 4 And it happened that 12 and she *beheld two angels
(angels) while they were in white sitting, one at the
appeared perplexed about this, head, and one at the feet,
behold, two men where the body of Jesus had
suddenly stood near them been lying.
in dazzling apparel;
Angel 5And the angel 6 And he *said to them, "Do not be 5 and as the women were 13 And they *said to her,
spoke answered and said to amazed; you are looking for Jesus the terrified and bowed their "Woman, why are you
the women, "Do not be Nazarene, who has been crucified. He faces to the ground, the weeping?" She *said to them,
afraid; for I know that has risen; He is not here; behold, here is men said to them, "Why "Because they have taken
you are looking for the place where they laid Him. do you seek the living away my Lord, and I do not
Jesus who has been 7 "But go, tell His disciples and Peter, One among the dead? know where they have laid
crucified. ‘He is going before you into Galilee; 6 "He is not here, but He Him."
6 "He is not here, for He there you will see Him, just as He said to has risen. Remember how
has risen, just as He you.’" He spoke to you while He
said. Come, see the was still in Galilee,
place where He was 7 saying that the Son of
lying. Man must be delivered
7 "And go quickly and into the hands of sinful
tell His disciples that He men, and be crucified,
has risen from the and the third day rise
dead; and behold, He is again."
going before you into
Galilee, there you will
see Him; behold, I have
told you."
women left 8 And they departed 8 And they went out and fled from the 8 And they remembered
tomb quickly from the tomb tomb, for trembling and astonishment His words,
with fear and great joy had gripped them; and they said nothing 9 and returned from the
and ran to report it to to anyone, for they were afraid. tomb and reported all
His disciples. these things to the eleven
and to all the rest.
they see 9 And behold, Jesus met 14 When she had said this, she
Jesus them and greeted them. turned around, and *beheld
And they came up and Jesus standing there, and did
took hold of His feet and not know that it was Jesus.
worshiped Him.
Jesus 10 Then Jesus *said to 15a Jesus *said to her,
speaks to them, "Do not be afraid; "Woman, why are you
them go and take word to My weeping? Whom are you
brethren to leave for seeking?"
Galilee, and there they
shall see Me."
Jesus and 15bSupposing Him to be the
Mary gardener, she *said to Him,
Magdalene "Sir, if you have carried Him
speak. away, tell me where you have
She clings laid Him, and I will take Him
to Jesus. away."
16 Jesus *said to her, "Mary!"
She *turned and *said to Him
in Hebrew, "Rabboni!" (which
means, Teacher).
17 Jesus *said to her, "Stop
clinging to Me, for I have not
yet ascended to the Father; but
go to My brethren, and say to
them, ‘I ascend to My Father
and your Father, and My God
and your God.’"
18 Mary Magdalene *came,
announcing to the disciples, "I
have seen the Lord," and that
He had said these things to
her.
They 11Now while they were See comments on Mark 16:9-20 10 Now they were Mary
returned on their way, behold, 9 [Now after He had risen early on the Magdalene and Joanna
and some of the guard came first day of the week, He first appeared and Mary the mother of
reported into the city and to Mary Magdalene, from whom He had James; also the other
what they reported to the chief cast out seven demons. women with them were
saw. priests all that had 10 She went and reported to those who telling these things to the
happened. had been with Him, while they were apostles.
12 And when they had mourning and weeping. 11 And these words
assembled with the 11 And when they heard that He was appeared to them as
elders and counseled alive, and had been seen by her, they nonsense, and they
together, they gave a refused to believe it. would not believe them.
large sum of money to 12 And after that, He appeared in a 12 [But Peter arose and
the soldiers, different form to two of them, while they ran to the tomb; stooping
13 and said, "You are to were walking along on their way to the and looking in, he *saw
say, ‘His disciples came country. the linen wrappings only;
by night and stole Him 13 And they went away and reported it and he went away to his
away while we were to the others, but they did not believe home, marveling at that
asleep.’ them either. which had happened.]
14 "And if this should 14 And afterward He appeared to the
come to the governor’s eleven themselves as they were reclining
ears, we will win him at the table; and He reproached them for
over and keep you out their unbelief and hardness of heart,
of trouble." because they had not believed those who
15 And they took the had seen Him after He had risen.
money and did as they 15 And He said to them, "Go into all the
had been instructed; world and preach the gospel to all
and this story was creation.
widely spread among 16 "He who has believed and has been
the Jews, and is to this baptized shall be saved; but he who has
day. disbelieved shall be condemned.
17 "And these signs will accompany
those who have believed: in My name
they will cast out demons, they will
speak with new tongues;
18 they will pick up serpents, and if they
drink any deadly poison, it shall not hurt
them; they will lay hands on the sick,
and they will recover."
19 So then, when the Lord Jesus had
spoken to them, He was received up into
heaven, and sat down at the right hand
of God.
20 And they went out and preached
everywhere, while the Lord worked with
them, and confirmed the word by the
signs that followed.] [And they promptly
reported all these instructions to Peter
and his companions. And after that,
Jesus Himself sent out through them
from east to west the sacred and
imperishable proclamation of eternal
salvation.]
Chronology of Jesus' Baptism and Temptation Chart

Event Matt. 3:13-4:11 Mark 1:9-13 Luke 3:21-4:18


Baptism of Jesus
Jesus is 13 Then Jesus *arrived from Galilee at the Jordan 9 And it came about in those 21 Now it came about when all the
baptized coming to John, to be baptized by him. days that Jesus came from people were baptized, that Jesus also
14 But John tried to prevent Him, saying, "I have Nazareth in Galilee, and was was baptized, and while He was
need to be baptized by You, and do You come to baptized by John in the Jordan. praying, heaven was opened,
me?"
15 But Jesus answering said to him, "Permit it at
this time; for in this way it is fitting for us to
fulfill all righteousness." Then he *permitted
Him.
the Dove and 16 And after being baptized, Jesus went up 10 And immediately coming up 22 and the Holy Spirit descended
words from immediately from the water; and behold, the out of the water, He saw the upon Him in bodily form like a dove,
the Father heavens were opened, and he saw the Spirit of heavens opening, and the Spirit and a voice came out of heaven,
God descending as a dove, and coming upon like a dove descending upon "Thou art My beloved Son, in Thee I
Him, Him; am well-pleased."
17 and behold, a voice out of the heavens, 11 and a voice came out of the
saying, "This is My beloved Son, in whom I am heavens: “Thou art My beloved
well-pleased." Son, in Thee I am well-pleased.”

Temptation of Jesus
Jesus goes to 4:1 Then Jesus was led up by the Spirit into the 12 And immediately the Spirit 4:1 And Jesus, full of the Holy Spirit,
the desert wilderness to be tempted by the devil. *impelled Him to go out into the returned from the Jordan and was
wilderness. led about by the Spirit in the
wilderness
40 day fast 2 And after He had fasted forty days and forty 13 And He was in the wilderness 2 for forty days, being tempted by
nights, He then became hungry. forty days being tempted by the devil. And He ate nothing during
Satan; and He was with the wild those days; and when they had
beasts, and the angels were ended, He became hungry.
ministering to Him.
turn 3 And the tempter came and said to Him, "If You 3 And the devil said to Him, "If You
stones are the Son of God, command that these stones are the Son of God, tell this stone to
to bread become bread." become bread."
not by bread 4 But He answered and said, "It is written, ‘Man 4 And Jesus answered him, "It is
alone shall not live on bread alone, but on every word written, ‘Man shall not live on bread
that proceeds out of the mouth of God.’" alone.’"
showed Jesus 5 And he led Him up and showed
the kingdoms Him all the kingdoms of the world in
a moment of time.
devil to give 6 And the devil said to Him, “I will
domain to give You all this domain and its
Jesus glory; for it has been handed over to
me, and I give it to whomever I
wish.
false worship 7 “Therefore if You worship before
me, it shall all be Yours.”
true worship 8 And Jesus answered and said to
him, “It is written, ‘You shall worship
the Lord your God and serve Him
only.’”
devil took 5 Then the devil *took Him into the holy city; and 9 And he led Him to Jerusalem and
Jesus to he had Him stand on the pinnacle of the temple, had Him stand on the pinnacle of the
pinnacle of 6 and *said to Him, "If You are the Son of God temple, and said to Him, “If You are
temple throw Yourself down; for it is written, ‘He will the Son of God, throw Yourself down
give His angels charge concerning You’; and ‘On from here;
their hands they will bear You up, Lest You strike 10 for it is written, ‘He will give His
throw yourself
Your foot against a stone.’" angels charge concerning You to
down
guard You,’
11 and, ‘On their hands they will
bear You up, Lest You strike Your
foot against a stone.’”

don't test God 7 Jesus said to him, "On the other hand, it is 12 And Jesus answered and said to
written, ‘You shall not put the Lord your God to him, “It is said, ‘You shall not put the
the test.’" Lord your God to the test.’”
took Jesus to 8 Again, the devil *took Him to a very high
high mountain mountain, and *showed Him all the kingdoms of
the world, and their glory;
devil to give 9 and he said to Him, "All these things will I give
domain You, if You fall down and worship me."
worship and 10 Then Jesus *said to him, "Begone, Satan! For
serve God it is written, ‘You shall worship the Lord your
only God, and serve Him only.’"
the devil 11 Then the devil *left Him; and behold, angels 13 And when the devil had finished
leaves came and began to minister to Him. every temptation, he departed from
Him until an opportune time.
Bibliography

• The New American Standard Bible, (La Habra, California: The Lockman Foundation) 1977.
• Aland, Kurt, ed. Synopsis of the Four Gospels, 7th ed., (Stuttgart, Germany: German Bible
Society) 1984.
• Bush, L. Russ, ed., Classical Readings in Christian Apologetics. A.D. 100-1800. Grand Rapids,
Michigan: Academie Books. 1983.
• Douglas, J. D., The New International Dictionary of the Christian Church, Douglas, Zondervan,
Grand Rapids, MI, 1978.
• Frame, John. Apologetics to the Glory of God. Phillipsburg, New Jersey: P&R Publishing, 1994.
• -----, The Doctrine of the Knowledge of God, Phillipsburg, New Jersey: P&R Publishing, 1987.
• Geisler, Norman. Christian Apologetics. Grand Rapids, Michigan: Baker Book House, 1976.
• -----, Baker Encyclopedia of Christian Apologetics. Grand Rapids, MI. Baker Books; 1999.
• Grudem, Wayne, Systematic Theology: An Introduction to Biblical Doctrine, Zondervan
Publishing House, Grand Rapids, MI, 1994.
• Harrison, E. F. ed., Bakers' Dictionary of Theology, Baker Book House, Grand Rapids, MI, 1960.
• Lewis, C. S. The Problem of Pain. New York: Simon & Schuster. 1996.
• -----, Mere Christianity. New York. Macmillan Publishing Co. Inc. 1960.
• McDowell, Josh. Evidence that demands a Verdict. San Bernardino, CA. Here's Life Publishers,
Inc. 1979
• -----, More Evidence that demands a Verdict. San Bernardino, CA. Here's Life Publishers,
Inc. 1981
• McDowell, Josh & Stewart, Don. Answers to Tough Questions skeptics ask about the Christian
Faith. San Bernardino, CA. Here's Life Publishers, Inc. 1983.
• Runes, Dagobert, D., ed., Dictionary of Philosophy, Philisophical Library, New York, 1942.
• Van Till, Cornelius. Christian Apologetics. Phillipsburg, New Jersey: Presbyterian and
Reformed Publishing, Co. 1976.
Oneness Pentecostal
Introduction

Oneness Pentecostal theology is growing. It denies the Trinity and adds baptism to salvation.
Oneness people are clever in how they use scripture to deny basic Christian theology, so be careful
when dealing with them.

1. What does Oneness theology teach? pp. 2-3


2. What are the qualities and attributes of being a person? p. 4
3. Is Baptism Necessary for Salvation? Why or why not? p. 7-10
4. What are the textual issues with Mark 16:9-20? p. 11-12
5. What are some alternative theories about the meaning of John 3:5? p. 19
6. Does Acts 2:38 teach baptism is necessary for salvation? Why or why not? pp. 21-24
7. What does Acts 4:7-10 tell us about the phrase "in Jesus' name"? p. 25
8. Is speaking in tongues a necessary sign of salvation? p. 26
9. Did Jesus pray to Himself? p. 27
10. Was God seen in the Old Testament? p. 29
11. What are some questions you can ask Oneness people? p. 33
What is Oneness Pentecostal theology?

Oneness Pentecostal theology affirms that there exists only one God in all the universe. It affirms
the deity of Jesus and the Holy Spirit. However, Oneness theology denies the Trinity. The Trinity is the
doctrine that there is one God who manifests Himself as three distinct, simultaneous persons. The
Trinity does not assert that there are three gods, but only one. This is important because many
groups who oppose orthodoxy, will accuse Trinitarians of believing in three gods. But this is not so.
The doctrine of the Trinity is that there is one God in three persons.
Oneness theology denies the Trinity and teaches that God is a single person who was "manifested
as Father in creation and as the Father of the Son, in the Son for our redemption, and as the Holy
Spirit in our regeneration." 92 Another way of looking at it is that God revealed himself as Father in the
Old Testament, as the Son in Jesus during Christ’s ministry on earth, and now as the Holy Spirit after
Christ’s ascension. In addition, oneness theology also maintains that baptism is a necessary part of
salvation; that is, in order to be saved, one must be baptized, by immersion. If you are not baptized
you cannot be saved. However, not only must baptism be by immersion, it must also be administered
with the formula "In Jesus’ name" rather than the formula "In the name of the Father, the Son, and
the Holy Spirit" which is mentioned in Matt. 28:19. Finally, this baptism must be administered by a
duly ordained minister on a church that maintains oneness theology: United Pentecostal, United
Apostolic, etc.
Oneness churches also teach that speaking in tongues is a necessary manifestation of the Holy
Spirit. Since a person cannot be saved without the Holy Spirit (Rom. 8:9), it follows that only those
who have spoken in tongues are really saved. There is, therefore, an emphasis that Oneness church
members speak in tongues to "demonstrate" that they are saved and have the truth.
Oneness groups are decidedly Arminian in the doctrine of salvation. They deny predestination and
maintain that it is completely up to the individual to decide whether or not he wants to be saved. They
also teach that it is possible to lose one's salvation.
There is within the Oneness movement an attempt to represent themselves in a modest and holy
manner. This is to be commended. However, sometimes it tends to become legalistic in that women
are required to abstain from wearing makeup and pants. They also must have their heads covered.
Likewise, men should be well dressed, preferably in ties (this has been my experience with them).
Such practices are not wrong in themselves, and are good examples of propriety. However, when they
become requirements for acceptance in a church, it is legalistic. Legalism leads to bondage and the
requirements of keeping the law to maintain salvation. It then becomes a means by which a person's
spirituality is judged. Oneness churches strongly imply that if you go to movies, or have a TV, or wear
makeup, etc., then you are not "really" a Christian.
I am not saying that the Oneness Theology necessarily leads to legalism, but it seems to be quite
evident that it has taken over much of Oneness practice.

92
http://www.upci.org/about/index.asp
What does Oneness Pentecostal teach?
Oneness Pentecostal people are many and varied. The two main groups that hold to Oneness
theology are the United Pentecostal Church International (the largest) and the United Apostolic
church. There are others like the Assemblies of the Lord Jesus Christ and the Bible Way Churches of
Our Lord Jesus Christ as well as a host of independent Oneness churches scattered throughout the
United States.
The following points of doctrine are generally held to by the Oneness Pentecostal groups.

1. Within Orthodoxy
A. There is only one God in all existence
B. The Bible is God's inerrant word
C. Jesus was born of a virgin
D. Jesus had two natures.
E. Justification by faith

F. Baptism must be by immersion. 93

G. The elements of communion are bread and wine and are only for believers.

H. Foot-washing (John 13:4-5), is a divine institution to be practiced by church members. 94

I. Abstain from joining secret societies (James 5:12; 2 Cor. 6:14-18).


J. There will be a future rapture of the Church where the Christians will be transformed (1
Thess. 4:13-17; 1 Cor. 15:51-54; Phil. 3:20-21).
2. Outside of Orthodoxy
A. Denies the doctrine of the Trinity.
B. Denies justification by faith alone by stating that baptism is also required.
C. Jesus is God the Father.
D. Jesus is the Holy Spirit.
E. The name of God is "Jesus."
F. Baptism is necessary for salvation.
G. Denies pre-existence of the Word as the Son. Teaches that the He existed as the Father.
H. Being born again means repentance, baptism, and speaking in tongues.
I. Baptism must be administered by an ordained Oneness minister to be valid.
J. Baptism must be administered with the phrase, "In the name of Jesus" instead of the
phrase, "In the name of the Father, the Son, and the Holy Spirit" (Matt. 28:19).
K. Speaking in tongues is a necessary requirement to demonstrate that a person has been
baptized in the Holy Spirit, and is, therefore, saved. It is claimed to be the initial sign of
the infilling of the Holy Ghost.
L. Restitution of all things, though the devil and the angels will not be restored.

M. Women may be pastors.95

N. Only Oneness people will go to heaven.

93
Orthodoxy allows for sprinkling as well within Christianity.
94
Many Christian churches practice foot-washing. But it is not a required practice according to the Bible.
95
Many Oneness people deny that women can be pastors, but the UPCI (United Pentecostal Church International)
does. Also, there are many Trinitarian churches that practice women ordination and eldership. But generally
speaking, women are not to hold these positions. If you are interested in more on this issue, please see Can
women be pastors and elders?
Oneness and the word "person"

Oneness theology denies the Trinity doctrine and claims that there is one person in the Godhead
who has manifested himself in three different forms: the Father, the Son, and the Holy Spirit. These
"forms" are not three separate persons, but one person who occupied consecutive modes. The Trinity,
on the other hand, is the teaching that there is one God who exists in three separate, simultaneous,
persons. Please note, though, this is not saying there are three gods.
In defending the doctrine of the Trinity and in examining the Oneness doctrine regarding the
Godhead, it is first necessary to define the terms that are used. Since the Trinity doctrine states there
are three persons in one God, and Oneness Pentecostal theology states there is only one person, we
first need to know what a "person" is before we try to discover whether or not God is three persons or
one. Therefore, what qualifies someone as having ”personhood"?
I offer the following analysis as an attempt to adequately define personhood. After the outline, I
will attempt to show that the definition and/or characteristics of personhood can be applied to both the
Father and the Son in a context that shows they both existed as persons at the same time, thereby
proving Oneness theology is incorrect.

What are the qualities and attributes of being a person?

1. A person exists and has identity.


2. A person is aware of his own existence and identity.
A. This precludes the condition of being unconscious.
3. A self aware person will use such a statement as "I am", "me", "mine", etc.
4. A person can recognize the existence of other persons.
A. This is true provided there were other persons around him or her.
B. Such recognition would include the use of such statements as "you are", "you", "yours",
etc.
5. A person possesses a will.
A. A will is the capability of conscious choice, decision, intention, desire, and or purpose.
6. A single person cannot have two separate and distinct wills at the same time on the exact
same subject.
A. Regarding the exact same subject, a person can desire/will one thing at one moment
and another at a different moment.
B. Separate and simultaneous wills imply separate and simultaneous persons.
7. A person has the ability to communicate -- under normal conditions.
8. Persons do not need to have bodies.
A. God the Father possesses personhood without a body, as do the angels.
B. Biblically speaking, upon death we are "absent from the body and home with the Lord"
(2 Cor. 5:8).

God qualifies as having personhood in that He exists, is self aware, has identity, uses terms such
as "Me", "I AM", "My", and possesses a will.
The question now becomes whether or not there are more than one "persons" in the Godhead.

"Let this cup pass from Me."

"And he was withdrawn from them about a stone’s cast, and kneeled down, and prayed,
42
Saying, 'Father, if thou be willing, remove this cup from me: nevertheless not my will, but
thine, be done,'" (Luke 22:42).

"And he went a little further, and fell on his face, and prayed, saying, 'O my Father, if it be
possible, let this cup pass from me: nevertheless not as I will, but as thou wilt,'" (Matt. 26:39).
In both Luke 22:42 and Matt. 26:39 (which are parallel passages), the context is Jesus in the
Garden of Gethsemane, right before His betrayal. He was praying to the Father about the ordeal He
was about to undergo. Several points are worth bringing out here.
First, in this passage, Jesus addresses the Father. He says, "Oh my Father..." Note that Jesus says
"my" and "Father." These two words designate a "me and you" relationship.
Second, "If it be possible" is Jesus expressing a desire, a hope. What is that hope or desire? It is
that "this cup pass from me." The cup Jesus is speaking of is the immanent ordeal of betrayal,
scourging, and crucifixion. Jesus did not want to go through this. He was expressing His desire. It was
His will not to undergo the severe ordeal ahead of Him. If this was not so, He would not have
expressed the desire to have the cup pass from Him.
Third, in Matt. 26:39, Jesus says, "Nevertheless, not my will, but thine, be done." In Luke 22:42 he
says, "Nevertheless not as I will, but as thou wilt." With this, Jesus is expressing His will and
contrasting it to the will of the Father. Yet, He is stating that even though He does not want to undergo
what lay ahead, "Nevertheless," He would submit to the will of the Father.
This shows that the person of Jesus had a separate and different will than the Father. Since we
have two separate simultaneous wills, we have two separate and simultaneous persons and Oneness
Pentecostal theology is incorrect.

Questions to ask the Oneness person:

1. Is Jesus His own Father?


2. If Jesus' will and the Father's will were identical (in an attempt to demonstrate that there is
only one will), then why did Jesus express the desire to escape the cup but resigns Himself not
to His own will, but the will of the Father?
3. Was Jesus praying to Himself at this point?
4. Was Jesus saying, "Not My will, but My will be done?" if there is only one person and one will
involved?
Another look at Jesus, the Father, and two wills

Oneness theology teaches that there is only one person in the Godhead whose name is Jesus.
Jesus is the Father and the Holy Spirit. Regarding His incarnation, oneness people say that He was in
heaven at the same time that He was on earth. Logically this would mean that even though Jesus was
both in heaven and on earth, He was still one person, not three as the Trinity position holds.
Unfortunately, the oneness position presents a serious problem.
In the Garden of Gethsemane (Luke 22:42), Jesus prayed to the Father saying, "Father, if You are
willing, remove this cup from Me; yet not My will, but Yours be done." See also, ""And he went a little
further, and fell on his face, and prayed, saying, 'O my Father, if it be possible, let this cup pass from
me: nevertheless not as I will, but as thou wilt,'" (Matt. 26:39).
We have two wills, one of the son, one of the Father, at the same time. If this is so, then how can
one person can have two separate and opposing wills on the same subject at the same time?
The response is generally that Jesus was fully a man and that in his humanity he was not the
everlasting Father. But if this is so, then what was Jesus if not God incarnate? If He is not fully God
incarnate, then the atonement is void since it isn't God making the sacrifice but a mere man.
Sometimes oneness people say that Jesus had another existence outside His existence as a man
because he also was existing as the Father. But this implies that there are two beings since each has
its own existence different than the other. Furthermore, the Oneness position would have a will of the
Father and the Son at the same time that are in opposition to each other -- yet they are one person!
If the oneness people state that Jesus' flesh was at odds with His own presence as the Father in
heaven, then again we have no true incarnation.
The problem with the oneness position is serious and the fact that Jesus' will was separate from the
Father's demonstrates that the Father and the Son are different persons within the Godhead. The
oneness people are very wrong.

Questions:

1. If it was the flesh side of Jesus speaking to the divine side of Jesus in heaven, then that denies
the true incarnation of God in Christ and invalidates the atonement.
Is Baptism Necessary for Salvation?
Oneness Pentecostal theology states that baptism is necessary for salvation. It asserts that
without it, a person cannot be saved. Is baptism necessary for salvation? No. It is not. The Oneness
theologians are in error. Nevertheless, disagreeing with them does not make it so, particularly when
we have verses like the following:

• John 3:5, "Verily, verily, I say unto thee, except a man be born of water and of the Spirit, he
cannot enter into the kingdom of God."
• Acts 2:38, "Then Peter said unto them, Repent, and be baptized every one of you in the name
of Jesus Christ for the remission of sins, and ye shall receive the gift of the Holy Ghost."
• Acts 22:16, "And now why tarriest thou? arise, and be baptized, and wash away thy sins,
calling on the name of the Lord."
• 1 Pet. 3:21, "The like figure whereunto even baptism doth also now save us (not the putting
away of the filth of the flesh, but the answer of a good conscience toward God,) by the
resurrection of Jesus Christ."

The problem with baptismal regeneration (the belief that baptism is part of salvation and necessary
for it) is that it contradicts other scriptures that state we are justified by faith. Justification is God's
declaration upon a sinner that the person is declared righteous in God's site. In other words, only
Christians are justified; only Christians are saved. Please consider the following verses:

• Rom. 4:3, "For what saith the scripture? Abraham believed God, and it was counted unto him
for righteousness."
• Rom. 5:1, "Therefore being justified by faith, we have peace with God through our Lord Jesus
Christ."
• Gal. 3:8, "And the scripture, foreseeing that God would justify the heathen through faith,
preached before the gospel unto Abraham, saying, In thee shall all nations be blessed."
• Eph. 2:8, "For by grace are ye saved through faith; and that not of yourselves: it is the gift of
God."
• See also Rom. 4:5; 9:30; John 5:24; Gal. 2:16; Gal. 3:11-14; and Phil. 3:9.

There are other verses, but these are sufficient to show that we are made right in God's eyes --
justified, forgiven -- by faith, not by faith and baptism. If baptism were necessary for salvation, then
these verses would state that we are justified by faith and baptism. But they don't. In fact, that is
not what Paul says that the gospel is, and it is the gospel that saves us.

The Gospel is what saves

"Now, brothers, I want to remind you of the gospel I preached to you, which you received and
on which you have taken your stand. By this gospel you are saved, if you hold firmly to the
word I preached to you. Otherwise, you have believed in vain. For what I received I passed on
to you as of first importance: that Christ died for our sins according to the Scriptures, that he
was buried, that he was raised on the third day according to the Scriptures, " (1 Cor. 15:1-4).

The gospel is defined as the death, burial, and resurrection of Jesus for our sins. Baptism is not
mentioned here.
Paul said that he came to preach the gospel, not to baptize: "I am thankful that I did not baptize
any of you except Crispus and Gaius, so no one can say that you were baptized into my name. (Yes, I
also baptized the household of Stephanas; beyond that, I don't remember if I baptized anyone else.)
For Christ did not send me to baptize, but to preach the gospel..." (1 Cor. 1:14-17). If baptism is
necessary for salvation then why did Paul downplay it and even exclude it from the description of what
is required for salvation? It is because baptism isn't necessary for salvation.
Additionally, in Acts, Peter was preaching the gospel, people got saved, and then they were
baptized. Acts 10:44-46 says,

"While Peter yet spake these words, the Holy Ghost fell on all them which heard the word.
45
And they of the circumcision which believed were astonished, as many as came with Peter,
because that on the Gentiles also was poured out the gift of the Holy Ghost. 46For they heard
them speak with tongues, and magnify God..."

These people were saved. The gift of the Holy Spirit was on the Gentiles and they were speaking in
tongues. This is significant because tongues is a gift given to believers, see 1 Cor. 14:1-5. Also,
unbelievers don't praise God. They can't because praise to the true God is a deep spiritual matter that
is foreign to the unsaved (1 Cor. 2:14). Therefore, the ones in Acts 10:44-46 who are speaking in
tongues and praising God are definitely saved and they are saved before they are baptized. This
simply isn't an exception. It is a reality.

Let's Suppose...

Another way of making this clear is to use an illustration. Let's suppose that a person, under the
conviction of the Holy Spirit (John 16:8), believed in Jesus as his savior (Rom. 10:9-10; Titus 2:13),
and has received Christ (John 1:12) as Savior. Is that person saved? Of course he is. Let's further
suppose that this person confesses his sinfulness, cries out in repentance to the Lord, and receives
Jesus as Savior and then walks across the street to get baptized at a local church. In the middle of the
road he gets hit by a car and is killed. Does he go to heaven or hell? If he goes to heaven then
baptism isn't necessary for salvation. If He goes to hell, then trusting in Jesus, by faith, isn't enough
for salvation. Doesn't that go against the Scriptures that say that salvation is a free gift ( Rom. 6:23)
received by faith (Eph. 2:8-9)? Yes it does.
Saying that baptism is necessary for salvation is dangerous because it is saying that there is
something we must do to complete salvation. That is wrong! See Gal. 2:21; 5:4.
Alright, so this sounds reasonable. But still, what about those verses that seem to say that
baptism is part of salvation? I'll address those now. But, because this subject can become quite
lengthy, in fact, sufficient for a book in itself, I'll only address a few verses and then only briefly.

Baptism Verses

John 3:5, "Jesus answered, ‘I tell you the truth, no one can enter the kingdom of God unless he is
born of water and the Spirit.'"
Some say that water here means baptism. But that is unlikely since Christian baptism hadn't yet
been instituted. If this verse did mean baptism, then the only kind that it could have been at that point
was the baptism of repentance administered by John the Baptist (Mark 1:4). If that is so, then baptism
isn't necessary for salvation because the baptism of repentance is no longer practiced.
It is my opinion that the water spoken of here means the water of the womb referring to the
natural birth process. Jesus said in verse three that Nicodemus needed to be born "again." This meant
that he had been born once--through his mother. Nicodemus responds with a statement about how he
can't enter again into his mother's womb to be born. Then Jesus says that he must be born of water
and the Spirit. Then in verse 6 He says that "flesh gives birth to flesh, but the Spirit gives birth to
spirit.." The context seems to be discussing the contrast between the natural and the spiritual birth.
Water, therefore, could easily be interpreted there to mean the natural birth process.
I would like to add that there are scholars who agree with the position and some who do not.
Some believe that the water refers to the Word of God, the Bible, and others claim it means the Holy
Spirit. You decide for yourself.

Acts 2:38, "Peter replied, ‘Repent and be baptized, every one of you, in the name of Jesus Christ
for the forgiveness of your sins. And you will receive the gift of the Holy Spirit.‘"
This verse is a tough one. It seems to say that baptism is part of salvation. But we know, from
other scriptures that it isn't, lest there be a contradiction. What is going on here is simply that
repentance and forgiveness of sins are connected. In the Greek, "repent" is in the plural and so is
"your" of "your sins." They are meant to be understood as being related to each other. It is like saying,
"All of you repent, each of you get baptized, and all of you will receive forgiveness." Repentance is a
mark of salvation because it is granted by God (2 Tim. 2:25) and is given to believers only. In this
context, only the regenerated, repentant person is to be baptized. Baptism is the manifestation of the
repentance, that gift from God that is the sign of the circumcised heart. That is why it says, repent and
get baptized. In other words, the phrase "each of you get baptized in the name of Jesus Christ" is
parenthetical since it is in the singular and "repent" is in the plural as is "your" of "your sins."
Therefore, "repent" must go with the purpose of forgiveness of sins. Also, this concept fits with Peter’s
statement in Acts 10:43 where the same phrase "sins may be forgiven" is used. There it is granted on
the basis of faith alone.
Also, consider this from The Bible Knowledge Commentary: "The preposition used here is eis
which, with the accusative case, may mean "on account of, on the basis of." It is used in this way in
Matt. 12:41; and Mark 1:4. Though it is possible for this construction to mean "on the basis of," this is
not its normal meaning; eis with the accusative case usually describes purpose or direction." 96

1 Pet. 3:21, "and this water symbolizes baptism that now saves you also -- not the removal of dirt
from the body but the pledge of a good conscience toward God. It saves you by the resurrection of
Jesus Christ."
This is the only verse that says that baptism saves. But, the NIV translation of the verse is
unfortunate. A better translation is found in the NASB which says, "and corresponding to that, baptism
now saves you." The key word in this section is the Greek antitupon. It means "copy," "type,"
corresponding to," "a thing resembling another," "its counterpart," etc. Baptism is a representation, a
copy, a type of something else. The question is "Of what is it a type?", or "Baptism corresponds to
what?". The answer is found in the previous verse, verse 20: "who once were disobedient, when the
patience of God kept waiting in the days of Noah, during the construction of the ark, in which a few,
that is, eight persons, were brought safely through the water. 21And corresponding to that, baptism
now saves you" (NASB).
What does baptism correspond to? Is it the flood? Or, is it the ark? What was it that saved Noah
and his family? Was it the water or the ark? Obviously, it was the Ark. Noah built and entered the ark
by faith and was saved (Heb. 11:7). The flood waters destroyed the ungodly. Peter, when referring to
the flood waters, refers to them as the means of destruction of the ungodly (2 Pet. 2:5; 3:6). It was
the Ark that saved. Noah entered the ark by faith. Baptism here, in my opinion, refers to the Ark, not
the waters. That is why the rest of the verse says, "not the removal of dirt from the body but the
pledge of a good conscience toward God" which is consistent with what Paul said in Col. 2:11-12 where
He equates baptism with being circumcised of heart.

Acts 22:16, "And now what are you waiting for? Get up, be baptized and wash your sins away,
calling on his name."
Is the washing away of sins done by baptism, the representation of the circumcised heart ( Col.
2:11-12) which means you are already saved, or is it by the blood of Christ (Heb. 9:14; Rom. 5:9;
Eph. 1:7)? Obviously it is the blood of Jesus and the washing here refers to the calling on Jesus'
name.
Baptism is a picture of God’s inner work of washing away sin (cf. 1 Cor. 6:11; 1 Pet. 3:21).

Rom. 6:4, "We were therefore buried with him through baptism into death in order that, just as
Christ was raised from the dead through the glory of the Father, we too may live a new life."
Because the believer is so closely united to Christ it is said that the symbol of baptism is our
death, burial, and resurrection. Obviously we did not die -- unless, of course, it is a figurative usage.

Titus 3:5, "he saved us, not because of righteous things we had done, but because of his mercy.
He saved us through the washing of rebirth and renewal by the Holy Spirit."
The washing of rebirth can only be that washing of the blood of Christ that cleanses us. It is not
the symbol that saves, but the reality. The reality is the blood of Christ.

96
Walvoord, John F., and Zuck, Roy B., The Bible Knowledge Commentary, (Wheaton, Illinois: Scripture Press
Publications, Inc.) 1983, 1985, [Online] Available: Logos Library System.
Gal. 3:27, "for all of you who were baptized into Christ have clothed yourselves with Christ."
This is speaking of the believer's union with Christ. It is an identification with, a joining to, a
proclamation of loyalty to, etc. In 1 Cor. 10:2 the Israelites were baptized into Moses. That means
they were closely identified with him and his purpose. The same thing is meant here.

More on Baptism

It is an outward representation of an inward reality. For example, it represents the reality of the
inward washing of Christ's blood upon the soul. That is why it is used in different ways. It is said to
represent the death of the person (Rom. 6:3-5), the union of that person with Christ (Gal. 3:27), the
cleansing of that person's sins (Acts 22:16), the identification with the one "baptized into" as when the
Israelites were baptized into Moses (1 Cor. 10:2 ), and being united in one church (1 Cor. 12:13).
Also, baptism is one of the signs and seals of the Covenant of Grace that was instituted by Jesus. It is
in this sense a sacrament. A sacrament is a visible manifestation of something spoken. It is also said
to be a visible sign of an inward grace. For example, the communion elements of bread and wine are
called the sacrament of communion. When we take communion we are partaking of the sacrament.
The Covenant of Grace is the covenant between God and Man where God promises to Man eternal
life. It is based upon the sacrifice of Jesus on the cross and the condition is faith in Jesus Christ. As the
Communion Supper replaced Passover, baptism, in like manner, replaces circumcision. "They
represent the same spiritual blessings that were symbolized by circumcision and Passover in the old
dispensation." 97
Circumcision was the initiatory rite into the Abrahamic covenant; it did not save. A covenant is a
pact or agreement between two or more parties and that is exactly what the Abrahamic covenant was.
God said to Abraham, "I will establish my covenant as an everlasting covenant between me and you
and your descendants after you for the generations to come, to be your God and the God of your
descendants after you" (Gen. 17:7, NIV). God later instructed Abraham to circumcise not only every
adult male, but also 8 day old male infants as a sign of the covenant ( Gen. 17:9-13). If the children
were not circumcised, they were not considered to be under the promissory Abrahamic covenant. This
is why Moses' wife circumcised her son and threw the foreskin at Moses' feet. (Exo. 4:24-25). She
knew the importance of the covenant between God and her children. But at the same time we must
understand that circumcision did not guarantee salvation to all who received it. It was a rite meant
only for the people of God, who were born into the family of God (who were then the Jews).
If you understand that baptism is a covenant sign, then you can see that it is a representation of
the reality of Christ circumcising our hearts (Rom. 2:29; Col. 2:11-12). It is our outward proclamation
of the inward spiritual blessing of regeneration. It comes after faith which is a gift of God ( Rom. 13:3)
and the work of God (John 6:28).

Conclusion

Baptism is not necessary for salvation. It is the initiatory sign and seal into the covenant of grace.
As circumcision referred to the cutting away of sin and to a change of heart (Deut. 10:16; 30:6; Jer.
4:4; 9:25,26; Ez. 44:7,9) baptism refers to the washing away of sin (Acts 2:38; 1 Pet. 3:21; Titus
3:5) and to spiritual renewal (Rom. 6:4; Col. 2:11-12). The circumcision of the heart is signified by the
circumcision of the flesh, that is, baptism (Col. 2:11-12).
One last thought: If someone maintains that baptism is necessary for salvation, is he adding a
work, his own, to the finished work of Christ? If the answer is yes, then that person would be in
terrible risk of not being saved. If the answer is no, then why is baptism maintained as being
necessary the same way as the Jews maintained that works were necessary?

97
Berkhoff, Lewis, Systematic Theology, 1988, p. 620.
Baptism and Mark 16:16

"He who believes and is baptized will be saved; but he who does not believe will be
condemned," (Mark 16:16).

This verse is frequently used by baptismal regenerationists to show that baptism is necessary for
salvation. It says he who believes and is baptized will be saved. Therefore, they conclude that
baptism is a necessary part of becoming saved. But, does this verse prove that baptism is necessary
for salvation? Not at all.
Mark 16:16 does not say that baptism is a requirement for salvation. Let me show you why. I
could easily say that he who believes and goes to church will be saved. That is true. But it is belief
that saves, not belief and going to church. Likewise, if you believe and read your Bible, you'll be
saved. But it isn't reading your Bible that saves you. Rather, belief in Christ, in His sacrifice, is what
saves. As I've stated in other papers on this subject, there are numerous verses that clearly
demonstrate that justification is by faith (Rom. 5:1; Eph. 2:8; Phil. 3:9; etc.). Belief in what God has
done, not what man can do, is what results in salvation. Baptism is simply a public demonstration of
the inner work of regeneration. This is why the rest of the verse says, "...but he who does not to
believe will be condemned." Mark 16:16 focuses on the issue of belief, not baptism.

A textual issue with Mark 16:9-20

What I will share here may not be very popular with some readers. Therefore, I need to say
upfront that I believe in the absolute inspiration and authority of the Bible. It is the word of God and
what it says is authoritative. However, the simple fact is that there are textual variations within the
biblical manuscripts. The originals are what are inspired, not the copies. We have copies of inspired
documents. These copies are not perfect, but they are very close to it.
Again, I am not saying the Bible is untrustworthy. It is 98.5% textually pure. The remaining 1.5%
of textual variation are almost entirely of insignificant spelling errors and minor word omissions or
additions that do not change the meaning of the text. However, Mark 16:9-20 is a significant textual
variant. Many scholars, Christian scholars, consider the ending of Mark to lack authenticity. Please
consider the following evidence.

1. Mark 16:9-20 doesn't appear in many of the oldest ancient manuscripts.

A. “The last twelve verses of Mark (16:9-20) are lacking in the two earliest parchment codices,
B and Aleph, in the Old Latin manuscript k,, the Sinaitic Syriac, many manuscripts of the Old
Armenian version, the Adysh and Opiza manuscripts of the Old Georgian version, and a
number of manuscripts of the Ethiopic version. Clement of Alexandria, Origen, and
Ammonius show no knowledge of the existence of these verses; other Church Fathers state
that the section is absent from Greek copies of Mark known to them (e.g. Jerome, Epist.
cxx. 3, ad hedibiam,)...The original form of the Eusebian sections makes no provision for
numbering sections after 16:8. Not a few manuscripts which contain the passage have
scholia stating that older Greek copies lack it (so, for example, MSS. 1, 20,22, &c.), and in
other witnesses the passage is marked with asterisks or obeli, the conventional sigla used by
scribes to indicate a spurious addition to a literary document." 98

2. There are other endings to Mark.

98
"The Text of the New Testament," by Bruce Metzger (Professor of New Testament Language and Literature,
Princeton Theological Seminary), 2nd ed., Oxford University Press, New York, 1968, p. 226.
A. Another ending is found in L, Psi, 099, 0112, and minuscules 274mg 579, k, Syrh and more
is as follows:
i. "But they reported briefly to Peter and those with him all that had been told. And after
this Jesus himself sent out by means of them, from east to west, the sacred and
imperishable proclamation of eternal salvation."
B. Apparent, theological error.
i. Mark 16:12 says, "And after that, He appeared in a different form to two of them, while
they were walking along on their way to the country." This verse may be problematic.
Jesus rose in the same body that he died in (John 2:19), though it was a glorified body.
This is problematic because it suggests "a different form." Jesus did not appear in a
different form. He appeared in the same body he rose in.
C. Evidence against the Mark authorship.
i. There are 17 non-marcan words used in a non-marcan sense in these verses.

This information about the ending of Mark is not intended to cast doubt upon God's word. But
the fact is that the ending is under a large cloud of doubt as to its authenticity. I would not use it
as a defense for baptismal regeneration.
Baptism and Roman 6:3-5

Romans 6:3-5 is often used as a proof text for the claim that baptism is essential for salvation. It
is a strong comparison between our baptism and Christ's death, burial, and resurrection. On the
surface, one could conclude that from these verses, that baptism is part of salvation.

"Or do you not know that all of us who have been baptized into Christ Jesus have been
baptized into His death? 4Therefore we have been buried with Him through baptism into death,
in order that as Christ was raised from the dead through the glory of the Father, so we too
might walk in newness of life. 5For if we have become united with Him in the likeness of His
death, certainly we shall be also in the likeness of His resurrection,"

Is this section of scripture teaching us that baptism is necessary for salvation? No, it is not. First,
we know from the rest of scripture that salvation is by faith, not by faith and something we do Rom.
3:28-30. Second, we can see from other scriptures that baptism follows faith. Take a look at Acts
16:30-33 where the Jailer specifically asks what he must do to be saved and where baptism fits in.

"and after he brought them out, he said, "Sirs, what must I do to be saved?" 31And they said,
"Believe in the Lord Jesus, and you shall be saved, you and your household." 32And they spoke
the word of the Lord to him together with all who were in his house. 33And he took them that
very hour of the night and washed their wounds, and immediately he was baptized, he and all
his household," (Acts 16:30-33, NASB).

If baptism were part of salvation, then Paul should have said, "Believe and be baptized and you will
be saved." But, he did not. Also, consider Acts 10:44-46.

"While Peter was still speaking these words, the Holy Spirit came on all who heard the
message. The circumcised believers who had come with Peter were astonished that the gift of
the Holy Spirit had been poured out even on the Gentiles. For they heard them speaking in
tongues and praising God. Then Peter said, ‘Can anyone keep these people from being baptized
with water? They have received the Holy Spirit just as we have.' So he ordered that they be
baptized in the name of Jesus Christ. Then they asked Peter to stay with them for a few days,"
(NIV).

These people were saved. The gift of the Holy Spirit was on the Gentiles and they were speaking in
tongues. This is significant because tongues is a gift given to believers, see 1 Cor. 14:1-5. Also,
unbelievers don't praise God. They can't because praise to the true God is a deep spiritual matter that
is foreign to the unsaved (1 Cor. 2:14). Therefore, the ones in Acts 10:44-46 who are speaking in
tongues and praising God are definitely saved and they are saved before they are baptized. This isn't
an exception. It is a reality. This proves that baptism is not necessary for salvation.
What is Romans 6:3-5 saying?

"Or do you not know that all of us who have been baptized into Christ Jesus have been
baptized into His death? 4Therefore we have been buried with Him through baptism into death,
in order that as Christ was raised from the dead through the glory of the Father, so we too
might walk in newness of life. 5For if we have become united with Him in the likeness of His
death, certainly we shall be also in the likeness of His resurrection,"

The phrase "baptized into" occurs five times in the NT in four verses as found in the KJV and the
NASB..

5. Rom. 6:3, "Or do you not know that all of us who have been baptized into Christ
Jesus have been baptized into His death?"
6. 1 Cor. 10:2, "and all were baptized into Moses in the cloud and in the sea."
7. 1 Cor. 12:13, "For by one Spirit we were all baptized into one body, whether
Jews or Greeks, whether slaves or free, and we were all made to drink of one Spirit."
8. Gal. 3:27, "For all of you who were baptized into Christ have clothed yourselves
with Christ."

To be baptized "into Christ," "into His death," "into Moses," and "into one body" is to be publicly
identified with the thing you are being baptized into. The focus is not the baptism itself, but on the
thing the baptism represents. In the case of Rom. 6:3-5, being baptized into Christ is a public
identification with Christ's death, burial, and resurrection which is said to be the gospel that saves in 1
Cor. 15:1-4. Baptism then is a public statement proclaiming that the person is trusting in the sacrifice
of Christ.
Baptism by immersion is a perfect symbol for this work of Christ with which the Christian is
identifying himself. As Christ died and was raised to a new life, so to the Christian, in Christ, is said to
have died (Rom. 6:11; Col. 3:3) and has a new life. This new life of regeneration is by faith, the
internal work. Baptism is the external work of identification with Christ. This is why the reference to
baptism in the Bible is dealing more with "our union and identification with Christ than to our water
baptism."99

• Baptism is being identified as a disciple (Matt. 28:18-9).


• Baptism may be compared to a new birth (John 3:5).
• Baptism is compared to Jesus' death and resurrection (Rom. 6:3-5).
• Baptism is compared to Israel's Exodus and passing through the Red Sea (1 Cor. 10:2).
• Baptism is compared to Noah's escaping the flood waters by entering the ark (1 Pet. 3:21).

In each of the references above, baptism is identification with something. When people were
baptized into John the Baptist's baptism of repentance, it wasn't the baptism that granted them
repentance or made repentance real. Repentance is something that happens internally and is the work
of God (2 Tim. 2:25). To participate in John's baptism was to publicly proclaim that the person being
baptized was accepting John's message or repentance. Hence, it was called a baptism of repentance.
It wasn't the baptism that brought repentance; rather, baptism was the result of repentance. The
person had to first decide to repent, and then become baptized as a proclamation of his decision.
Likewise, the Christian must first decide to repent, to receive Christ (John 1:12), to rely on the
sacrifice of Christ, by faith, and then participate in the public proclamation of identifying with Christ's
work.

It is identification with the death, burial, and resurrection of Christ that baptism represents. Jesus'
shed blood is what cleanses us from our sins (Heb. 9:22), not being washed with water. It is Christ's

99
Enhanced Strong’s Lexicon, (Oak Harbor, WA: Logos Research Systems, Inc.) 1995.
death that is the payment for sin. Jesus' burial is the proof that He, in fact, died. Jesus' resurrection
is the proof of God the Father's acceptance of the sacrifice of Christ and that death is conquered.
Again, for a Christian to be baptized is to make a public proclamation that he is trusting in Christ's
work, that he is naming himself with Christ and trusting what Christ has done. This is why it says in
Rom. 6:11, "Even so consider yourselves to be dead to sin, but alive to God in Christ Jesus," (NASB).
Why? Because "I have been crucified with Christ; and it is no longer I who live, but Christ lives in me;
and the life which I now live in the flesh I live by faith in the Son of God, who loved me, and delivered
Himself up for me," (Gal. 2:20). It is on the cross that Jesus paid for our sins, not in His baptism and
not in our baptism. It is our identification with Him, being counted "in Christ" that allows us to say we
have been crucified with Christ so that we can say we are dead to sin. We are not dead to sin by our
baptism. Rather, we are dead to sin, by faith, in what Jesus did in His sacrifice.

Conclusion

Romans 6:3-5 speaks to us of Christ's work and our public identification with it. In that ancient
world of religious plurality in Roman gods, in the strict Laws of the Jewish system, and in the gods of
different cultures, to be baptized was to make a bold statement of commitment to Christ as the risen
Lord. It was not the water that saved, but faith in Christ and His work.
Baptism and Gal. 3:27

Gal. 3:27 is often used by the baptismal regenerationists to support the idea that you must be
baptized to be saved. They maintain that baptism is the place where a person "puts on Christ," where
he is "clothed with Christ" and that it means that baptism saves. They teach that being immersed in
the baptismal water is the place and time of deliverance from sins. This is simply not true.
Gal. 3:27 cannot be understood alone. It must be examined in context.

"Therefore the Law has become our tutor to lead us to Christ, that we may be justified by faith.
25
But now that faith has come, we are no longer under a tutor. 26For you are all sons of God
through faith in Christ Jesus. 27For all of you who were baptized into Christ have clothed
yourselves with Christ. 28There is neither Jew nor Greek, there is neither slave nor free man,
there is neither male nor female; for you are all one in Christ Jesus. 29And if you belong to
Christ, then you are Abraham’s offspring, heirs according to promise," (Gal. 3:24-29).

In Roman society, children were often committed to the care of trusted slaves. This would often
happen when the child was between six or seven, and it would last until puberty. "These slaves were
severe disciplinarians and were charged with guarding the children from the evils of society and giving
them moral training. This was like the Law’s function until Christ came and people could be justified by
faith in Him." 100 The Law was a harsh master to the Jews. It was very difficult to keep. This is why
the Law points to Christ by showing us our inability to keep the Law and by showing us that we must
rely on faith instead. That is why justification is by faith (vv. 24-26), because we cannot attain
justification by Law (Rom. 3:28-30; Phil. 3:9).

"For all of you who were baptized into Christ have clothed yourselves with Christ," (Gal. 3:27).

In Roman society when a child who had been under the care of a tutor and reached a matured
enough age, he was given a special robe, or toga. It was symbolic of his full rights in the family. 101
Therefore, being "clothed with Christ" is a phrase meaning that the Christian moved out from the Law
and into the gospel of grace and can enjoy full acceptance before God the Father. It is not saying that
baptism is what saves us from our sins.

100
Walvoord, John F., and Zuck, Roy B., The Bible Knowledge Commentary, (Wheaton, Illinois: Scripture Press
Publications, Inc.) 1983, 1985, on Gal. 3:24.
101
ibid, on Gal. 3:27.
Baptism and 1 Pet. 3:21
1 Pet. 3:21 says, "and this water symbolizes baptism that now saves you also -- not the removal
of dirt from the body but the pledge of a good conscience toward God. It saves you by the resurrection
of Jesus Christ." This is the only verse that says that baptism saves. Is it teaching that we must be
baptized to be saved? No. But, but to rightly understand it, we need to look at its context.

"For Christ also died for sins once for all, the just for the unjust, in order that He might bring us
to God, having been put to death in the flesh, but made alive in the spirit; 19 in which also He
went and made proclamation to the spirits now in prison, 20 who once were disobedient, when
the patience of God kept waiting in the days of Noah, during the construction of the ark, in
which a few, that is, eight persons, were brought safely through the water. 21 And
corresponding to that, baptism now saves you—not the removal of dirt from the flesh, but an
appeal to God for a good conscience — through the resurrection of Jesus Christ, 22 who is at
the right hand of God, having gone into heaven, after angels and authorities and powers had
been subjected to Him," (1 Pet. 3:18-22, NASB).

The above translation in verse 21 from the NASB is a good translation. "And corresponding to
that, baptism now saves you." The key word in this section is the Greek antitupon. It means "copy,"
"type," "corresponding to," "a thing resembling another," "its counterpart," etc. It is what the NIV
translates as "symbolizes," the NASB as "corresponding to that," and the KJV as "like figure."
Baptism, then, is a representation, a copy, a type of something else. The question is "Of what is it a
type?", or "baptism corresponds to what?"
If we look at the context, an interesting possibility arises, though I will admit, not the favored
interpretation among scholars. What does baptism correspond to? Is it the flood? Or, is it the ark?
What was it that saved Noah and his family, the flood or the ark? Obviously, it was the Ark. Noah built
and entered the ark by faith and he was saved (Heb. 11:7). The flood waters destroyed the ungodly.
Also, Peter consistently refers to the flood waters as the means of destruction of the ungodly ( 2 Pet.
2:5; 3:6), not the salvation of Noah and his family. Rather, it was the Ark that saved, the ark that
Noah entered faith. It may very well be that baptism refers to the Ark, not the waters. That is why
the rest of the verse says, "not the removal of dirt from the body but the pledge of a good conscience
toward God" which is consistent with what Paul said in Col. 2:11-12 where He equates baptism with
being circumcised of heart.
The problem with this interpretation is that it doesn't seem to fit the "water for water typology." It
would seem more natural to equate the water of baptism with the water of the flood. Furthermore, if
we were to look at the flood waters as the thing that removed evil from the land, we could say that
"correspondingly," the waters of baptism remove removes the sin from our hearts. Though this
reading seems a bit more natural, it too has problems.
The water of baptism is not what saves us, the sacrifice of Christ does which we receive by faith.
We read numerous verses about justification by faith (Rom. 5:1), salvation by faith (Eph. 2:8), etc.,
not justification "by faith and baptism," or salvation "by faith and baptism." 102 The fact is that salvation
is received by faith. Peter, not wanting to declare that baptism itself is what saves us, quickly adds,
"not the removal of dirt from the flesh, but an appeal to God for a good conscience." Water baptism,
then, must accompany the work of the Holy Spirit in the person. Peter's explanatory comment shows
us that the act of physical baptism is not what saves, but the "baptism of appeal to God." This appeal
to God is by faith the same as Noah's faith in God led him to build the Ark, enter it, and remain in it.
It was the Ark that saved Noah, not the flood waters.
The flood was for Noah a type of baptism even as the passage through the Red Sea was a type of
baptism for the Israelites.

Mark 16:16 says, "He who believes and is baptized will be saved; but he who does not believe will be
102

condemned." Please see the article on Baptism and Mark 16:16 for an examination of this verse.
"I want you to know, brethren, that our fathers were all under the cloud, and all passed
through the sea, 2and all were baptized into Moses in the cloud and in the sea, 3and all ate the
same supernatural food 4and all drank the same supernatural drink. For they drank from the
supernatural Rock which followed them, and the Rock was Christ," (1 Cor. 10:1-4).

The "baptisms" of both Noah and the Israelites served as types of a transition; that is, they moved
people from the old world to the new, from the old covenant to the new covenant. It is not the water
that saves, but the spiritual thing associated with that water that saves. For Noah it was faith in God.
For Moses it too was faith in God.
But some may say that the work of the Holy Spirit and the act of baptism are simultaneous, that
the Holy Spirit works in and through baptism to bring regeneration. But this cannot be the case since
the Bible tells us that salvation is by faith (Rom. 5:1; Eph. 2:8). Besides, we have a clear instance in
scripture where people are saved before their baptism.

Acts 10:44-48

"While Peter was still speaking these words, the Holy Spirit fell upon all those who were
listening to the message. 45And all the circumcised believers who had come with Peter were
amazed, because the gift of the Holy Spirit had been poured out upon the Gentiles also. 46For
they were hearing them speaking with tongues and exalting God. Then Peter answered,
47
"Surely no one can refuse the water for these to be baptized who have received the Holy
Spirit just as we did, can he?" 48And he ordered them to be baptized in the name of Jesus
Christ. Then they asked him to stay on for a few days," (Acts 10:44-48).

In these verses we see that Peter had been preaching the gospel and the Holy Spirit fell upon the
listeners. In verse 45 we read that "the gift of the Holy Spirit had been poured out upon the Gentiles
also." This gift manifested itself in speaking in tongues. This is significant because tongues is a sign-
gift given to believers, see 1 Cor. 14:1-5. Also, verse 46 says they were "exalting God." Unbelievers
don't praise God. They can't because praise to the true God is a deep spiritual matter that is foreign to
the unsaved (1 Cor. 2:14). Therefore, the ones in Acts 10 who are speaking in tongues and praising
God are definitely saved because they are moving in the Holy Spirit, speaking in tongues, and
glorifying God. It is the Holy Spirit who gives charismatic spiritual gifts to the church ( 1 Cor. 12:27-
28), not to unbelievers. Now, please notice that it was after this movement of the Holy Spirit that the
believers are baptized. If baptism is necessary for salvation, then how is it that the people were
speaking in tongues and exalting God before they were baptized?
If you were to say that the Holy Spirit was simply working upon and through those not yet saved,
then remember that tongues and praise to God are for the church, not the unbelievers. The church
consists of people who are saved, not unsaved. If they were not saved until they were baptized, then
they were not in the body of Christ and would not have moved in the charismatic gifts. Therefore,
they were regenerate before they were baptized. This simply isn't an exception. It is a reality.

Conclusion

1 Pet. 3:21 is not teaching us that baptism is what saves us. Rather, it is showing us that the
water symbolizes a spiritual cleansing through the power of the Holy Spirit gained through Christ’s
victory over death. It is the person's appeal to God that saves the soul, not the washing of water upon
the body.
Baptism and John 3:5

"Truly, truly, I say to you, unless one is born again, he cannot see the kingdom of God."
4
Nicodemus *said to Him, "How can a man be born when he is old? He cannot enter a second
time into his mother's womb and be born, can he?" 5Jesus answered, "Truly, truly, I say to you,
unless one is born of water and the Spirit, he cannot enter into the kingdom of God. 6"That
which is born of the flesh is flesh, and that which is born of the Spirit is spirit. 7"Do not marvel
that I said to you, 'You must be born again.' 8"The wind blows where it wishes and you hear the
sound of it, but do not know where it comes from and where it is going; so is everyone who is
born of the Spirit," (John 3:3-8).

There are five basic interpretations to this section of scripture in reference to water.

6. The water refers to the natural birth.


7. The water refers to the Word of God.
8. The water refers to the Holy Spirit.
9. The water refers to the ministry of John the Baptist.
10. The water refers to the water of baptism as a requirement for salvation.

The first option looks to the context of Jesus' words dealing with being born "again" ( 3:3).
Nicodemus responds by mentioning the experience of being born from the womb (v. 4). Jesus then
speaks of water and the Spirit and then says, "That which is born of the flesh is flesh, and that which
is born of the Spirit is spirit" (3:6). However, this view is not the most commonly held view.
The second option holds that the water is referring to the Word of God. Eph. 5:26 says, "that He
might sanctify her, having cleansed her by the washing of water with the word." Some believe that the
washing of water is done by means of the Word of God.
The third view says that the water refers to the Holy Spirit. Perhaps Nicodemus was reminded of
Ezek. 36:25-27, "Then I will sprinkle clean water on you, and you will be clean; I will cleanse you from
all your filthiness and from all your idols. 26"Moreover, I will give you a new heart and put a new spirit
within you; and I will remove the heart of stone from your flesh and give you a heart of flesh. 27"And I
will put My Spirit within you and cause you to walk in My statutes, and you will be careful to observe
My ordinances." Certainly, Jesus' own words are applicable here when He says in John 7:37-39, "Now
on the last day, the great day of the feast, Jesus stood and cried out, saying, "If any man is thirsty, let
him come to Me and drink. 38"He who believes in Me, as the Scripture said, 'From his innermost being
shall flow rivers of living water.'" 39But this He spoke of the Spirit, whom those who believed in Him
were to receive; for the Spirit was not yet given, because Jesus was not yet glorified."
The fourth view holds that the water is in reference to the water baptism of repentance taught by
John the Baptist. Matt. 3:1-6 describes John's ministry in the desert, his teaching about repentance,
and baptizing people into that repentance. Contextually, the first chapter of John mentions John the
Baptist in verses 6-8 and 19-36. Certainly, contextually, John and his ministry is in view here. If this is
the case, then Jesus would have been speaking of the "baptism" (the initiatory ordinance) of
repentance preached by John the Baptist.
The fifth view is the one held by the International Church of Christ; namely, that the water is
referring to baptism and that it is essential to salvation.
Does John 3:5 teach that baptism is essential to salvation?

As you can see, there are different interpretations to John 3:5. But, to simply say that John 3:5
does not teach the necessity of baptism isn't enough. Some sort of proof must be offered. The proof is
found in God's word that has no contradictions. Clearly, salvation is by faith. For example, Rom. 5:1
states that we are justified (declared righteous) by faith. It does not say faith and baptism. If baptism
were part of salvation, then it would say we were justified by faith and baptism. But it does not. If
justification is by faith, then it is by faith. Baptism is not faith. It is a ceremony. Furthermore, please
consider the following verses when declare how we are saved.

9. Rom. 3:22, "even the righteousness of God through faith in Jesus Christ for all those
who believe; for there is no distinction."
10. Rom. 3:26, "for the demonstration, I say, of His righteousness at the present time, that
He might be just and the justifier of the one who has faith in Jesus."
11. Rom. 3:28, "For we maintain that a man is justified by faith apart from works of the
Law."
12. Rom. 4:5, "But to the one who does not work, but believes in Him who justifies the
ungodly, his faith is reckoned as righteousness."
13. Rom. 5:1, "Therefore having been justified by faith, we have peace with God through
our Lord Jesus Christ,"
14. Gal. 3:8, "And the Scripture, foreseeing that God would justify the Gentiles by faith,
preached the gospel beforehand to Abraham."
15. Gal. 3:24, "Therefore the Law has become our tutor to lead us to Christ, that we may be
justified by faith."
16. Eph. 2:8, "For by grace you have been saved through faith; and that not of yourselves,
it is the gift of God."

Additionally, Paul tells us that the gospel is what saves us and that the gospel is the death, burial,
and resurrection of Jesus, (1 Cor. 15:1-4). Baptism is not included in the description of the gospel.
This explains why said he came to preach the gospel, not to baptize: "I am thankful that I did not
baptize any of you except Crispus and Gaius, so no one can say that you were baptized into my name.
(Yes, I also baptized the household of Stephanas; beyond that, I don't remember if I baptized anyone
else.) For Christ did not send me to baptize, but to preach the gospel..." (1 Cor. 1:14-174). If baptism
is necessary for salvation then why did Paul downplay it and even exclude it from the description of
what is required for salvation? It is because baptism isn't necessary for salvation. Therefore, John 3:5
must be interpreted in a manner consistent with the rest of scripture.
Another way of making this clear is to use an illustration. Let's suppose that a person, under the
conviction of the Holy Spirit (John 16:8), believed in Jesus as his savior (Rom. 10:9-10; Titus 2:13),
and has received Christ (John 1:12) as Savior. Is that person saved? Of course he is. Let's further
suppose that this person who confesses his sinfulness, cries out in repentance to the Lord, and
receives Jesus as Savior, then walks across the street to get baptized at a local church. In the middle
of the road he gets hit by a car and is killed. Does he go to heaven or hell? If he goes to heaven then
baptism isn't necessary for salvation. If He goes to hell, then trusting in Jesus, by faith, isn't enough
for salvation. Doesn't that go against the Scriptures that say that salvation is a free gift ( Rom. 6:23)
received by faith (Eph. 2:8-9)? Yes it does. Baptism is not necessary for salvation and John 3:5 cannot
teach that it is.
Baptism and Acts 2:38
Acts 2:38 is one of the more controversy verses in the Bible regarding baptism and whether or not
it is the requirement for salvation. On the surface it seems to support it. But upon closer examination,
we will see that it does not teach baptismal regeneration: that baptism saves.
First of all, rarely is doctrine ever made from a single verse. We need to look at all of what God's
words says about a subject in order to accurately understand what it teaches. I will briefly tackle of
this verse in the following manner.

• Examination of the verse's syntax, grammar and structure.


• Examine other verses dealing with the forgiveness of sins.
• Examine the verse in its covenant context.

Grammar and Structure of Acts 2:38

In Acts 2:38 the main verb is metanoesate (change mind), the aorist direct imperative (a
command) of metanoeo which means to repent (change mind). This refers to that initial repentance of
the sinner unto salvation. The verb translated "be baptized" is in the indirect passive imperative (a
command to receive; hence, passive voice in Greek 103) of baptizo, which does not give it the same
direct command implied in "repent." The preposition "for" in the phrase "for the remission of sins" in
Greek is "eis," unto or into, and it is in the accusative case (direct object). It can mean "for the
purpose of identifying you with the remission of sins." It is the same preposition we find in 1 Cor. 10:2
in the phrase "and were baptized unto Moses." Note that both contexts are dealing with baptism and
identification. These people were baptized or spiritually identifying themselves with the purposes and
vision of Moses. Repentance, therefore, is presented as identifying an individual with the remission of
his sins, even as baptism following repentance provides an external identification visible by others.
Repentance is something that concerns an individual and God while baptism involves others. That is
why baptistheto (let be immersed) is in the passive voice indicating that one does not baptize himself,
but is baptized by another usually in the presence of others. Repentance, however, is an act taking
place within a person's heart as the Holy Spirit moves in the sinner.
But, all this Greek stuff may be confusing. Let me break it down. All people are commanded to
repent for their sins. This is
what believers have already done by
becoming Christians. Baptism,
then, is the outward identification
with being a Christian for those
who have already repented.
Also, as the Israelites were
"baptized into Moses," (1 Cor.
10:2), so too, Christians are
baptized into Jesus. That is, they are
identifying themselves, publicly, with Christ. Likewise, in Rom. 6:1-5 where baptism is related to
death, burial, and resurrection, it is again and identification with Christ in His death, burial, and
resurrection. That is why it is said of Christians that we have died to sin ( Rom. 6:2, 11; Gal. 2:20;
Col. 2:20; Col. 3:3; 1 Pet. 2:24).
This verse is not demonstrating that baptism is essential for salvation, but that baptism is the thing
which we receive, in order to publicly identify ourselves completely and totally with Christ as a
manifestation of the inward work God has done within us.

103
Active voice is "I hit the ball." Passive voice is "The ball hit me." Middle voice is "I was hit by the ball." In
active voice, "I" performed the action. In passive voice, "I" received the action. In middle voice, "I" did
something to myself.
Other verses dealing with salvation

Justification is the work of God where the righteousness of Jesus is reckoned to the sinner so the
sinner is declared, by God, as being righteous under the Law (Rom. 4:3; 5:1,9; Gal. 2:16; 3:11). This
righteousness is not earned or retained by any effort of the saved. Justification is an instantaneous
occurrence with the result being eternal life. It is based completely and solely upon Jesus' sacrifice on
the cross (1 Pet. 2:24) and is received by faith alone (Rom. 4:5; 5:1; Eph. 2:8-9). No works are
necessary whatsoever to obtain justification. Otherwise, it is not a gift ( Rom. 6:23). Therefore, we are
justified by faith (Rom. 5:1).
Nowhere in the Bible does it state that we are justified by grace and baptism or faith and baptism
or faith and anything else. On the contrary, baptism is excluded from the gospel message. Paul said
that he came to preach the gospel, not to baptize: "I am thankful that I did not baptize any of you
except Crispus and Gaius, so no one can say that you were baptized into my name. (Yes, I also
baptized the household of Stephanas; beyond that, I don't remember if I baptized anyone else.) For
Christ did not send me to baptize, but to preach the gospel..." (1 Cor. 1:14-17).
Likewise, Paul told us exactly what the gospel that saves is. He said in 1 Cor. 15:1-4, "Now I make
known to you, brethren, the gospel which I preached to you, which also you received, in which also
you stand, 2by which also you are saved, if you hold fast the word which I preached to you, unless you
believed in vain. 3For I delivered to you as of first importance what I also received, that Christ died for
our sins according to the Scriptures, 4and that He was buried, and that He was raised on the third day
according to the Scriptures." Note that Paul state and that the gospel is what saints and he did not
include baptism in the definition of the gospel.”
So, we must ask if baptism is necessary for salvation, then why did Paul downplay it and even
exclude it from the description of what is required for salvation? It is because baptism isn't necessary
for salvation.
Further proof that baptism is not a requirement of salvation can be found in Acts 10:44-46. Peter
was preaching the gospel, people became saved, and then they were baptized. Acts 10:44-46 says,

"While Peter was still speaking these words, the Holy Spirit came on all who heard the
message. The circumcised believers who had come with Peter were astonished that the gift of
the Holy Spirit had been poured out even on the Gentiles. For they heard them speaking in
tongues and praising God. Then Peter said, ‘Can anyone keep these people from being baptized
with water? They have received the Holy Spirit just as we have.' So he ordered that they be
baptized in the name of Jesus Christ. Then they asked Peter to stay with them for a few days,"
(NIV).

These people were saved. The gift of the Holy Spirit was on the Gentiles and they were speaking in
tongues. This is significant because tongues is a gift given to believers, see 1 Cor. 14:1-5. Also,
unbelievers don't praise God. They can't because praise to the true God is a deep spiritual matter that
is foreign to the unsaved (1 Cor. 2:14). Therefore, the ones in Acts 10:44-46 who are speaking in
tongues and praising God are definitely saved and they are saved before they are baptized. This isn't
an exception. It is a reality. This proves that baptism is not necessary for salvation and that Acts 2:38
is not teaching its necessity either. But, if it isn't saying that, then why is baptism mentioned here?

Biblical Covenant Context

A covenant is a pact or agreement between two or more parties. Very often, covenants have
visible signs to represent them. The elements of bread and wine in the communion support are good
examples of this. Circumcision was both a covenant sign and and the initiatory rite into the Abrahamic
covenant (Gen. 17:10). But this covenant sign did not save anyone.
God said to Abraham, "I will establish my covenant as an everlasting covenant between me and
you and your descendants after you for the generations to come, to be your God and the God of your
descendants after you," (Gen. 17:7, NIV). God later instructed Abraham to circumcise not only every
adult male, but also eight day old male infants as a sign of the covenant ( Gen. 17:9-13). If the
children were not circumcised, they were not considered to be under the promissory Abrahamic
covenant. This is why Moses' wife circumcised her son and threw the foreskin at Moses' feet after
Moses failed to circumcise him, (Exo. 4:24-25). She knew the importance of the covenant between
God and her children. But at the same time we must understand that circumcision did not guarantee
salvation to those who received it. It was a rite meant only for the people of God, who were born into
the family of God (who were then the Jews). It was an outward sign of the covenant promise. To
reject it was to reject the covenant. But, accepting it did not guarantee salvation.

Another theological debate at risk here

There is debate within Christianity on the nature of baptism and to whom it may be administered.
I am not here trying to convince anyone of the proper objects of baptism whether it is infant baptism
or adult only baptism. I only present the following information as a proof that baptism is a covenant
sign, and not essential to salvation.
In the New Testament, circumcision is mentioned many times. But with respect to baptism it is
specifically mentioned in Col. 2:11-12: "In him you were also circumcised, in the putting off of the
sinful nature, not with a circumcision done by the hands of men but with the circumcision done by
Christ, having been buried with him in baptism and raised with him through your faith in the power of
God, who raised him from the dead," (NIV). In these verses, baptism and circumcision are related.
The extent of that relationship is still being debated. Nevertheless, Paul also says in Rom. 2:29, "But
he is a Jew who is one inwardly; and circumcision is that which is of the heart, by the Spirit, not by the
letter; and his praise is not from men, but from God." As you can see, for the Christian, circumcision
is of the heart. And because it is, we Christians are now included the Abrahamic covenant where
before, we, the Gentiles, were not. "Remember that you were at that time separate from Christ,
excluded from the commonwealth of Israel, and strangers to the covenants of promise, having no
hope and without God in the world," (Eph. 2:12, NASB). In Gal. 3:8, Paul calls the promise of the
Abrahamic covenant, the gospel. He says, "And the Scripture, foreseeing that God would justify the
Gentiles by faith, preached the gospel beforehand to Abraham, saying, 'All the nations shall be blessed
in you, 9So then those who are of faith are blessed with Abraham, the believer.'" (Gal. 3:8-9). So,
Paul calls the Abrahamic covenant, the gospel. The sign of this Abrahamic covenant was
circumcision.
Here is the catch. Since the Abrahamic covenant is still valid (we are justified by faith -- Gal. 3:8),
then is there a covenant sign for us today? I think the answer is a resounding, yes. I believe that
baptism replaces the Old Testament covenant sign of circumcision because 1) there was a New
Covenant in the communion supper (Luke 22:20), and 2) in circumcision there was the shedding of
blood, but in baptism no blood is shed. The covenant sign has changed now that the Law has been
fulfilled in Christ. If you understand that baptism is a covenant sign, then you can see that it is a
representation of the reality of Christ circumcising our hearts (Rom. 2:29; Col. 2:11-12). It is our
outward proclamation of the inward spiritual blessing of regeneration, of "heart-circumcision." It
comes after faith which is a gift of God (Rom. 13:3) and the work of God (John 6:28). Again, baptism
is the covenant sign of our covenant with God.

Acts 2:39 and "The Promise"

This would explain why Peter in verse 39 of Acts 2 says, "For the promise is for you and your
children, and for all who are far off, as many as the Lord our God shall call to Himself." What promise
is Peter speaking of when he says "the promise"? Notice that he does not say "this promise" but "the
promise." If Peter was referring to baptism as the promise he would have said "this promise."
Instead, he used a phrase "the promise." This is significant.
The phrase "the promise" occurs in 26 Bible verses in the New Testament. It is used in reference
to several different topics.

9. The Holy Spirit, (Luke 24:49; Acts 2:33; Gal. 3:14).


10. God's promise to Abraham to multiply his descendents in Egypt, physical as well
as spiritual, (Acts 7:17; Heb. 6:13, 15, 17).
11. The promise of the Messiah, (Acts 13:32; Acts 26:6-7; Rom. 4:13,14,16; Gal.
3:17,19,22; Eph. 3:6; 2 Tim. 1:1).
12. The promise of eternal redemption (Heb. 9:15; 1 John 2:25).
13. The promise that Sarah would have a child (Rom. 4:20; Gal. 4:23).
14. The promise that through Isaac, the world would be blessed, (Rom. 9:8).
15. The promise of Jesus' return (2 Pet. 3:4).
16. The promise to kill Paul by Paul's adversaries (Acts 22:21).

But, we are most interested in its context in Acts 2 which begins with the outpouring of the Holy
Spirit (Acts 2:1-13). Peter then preaches a sermon and quotes many OT scriptures (Acts 2:14-35). In
verse 2:22, Peter specifically says, "Men of Israel, listen to these words..." Peter is speaking to the
Jews. It was to the Jews that "the promise" of the outpouring of the Spirit was given. Peter is
speaking covenant language of God as He quotes the OT. Since Peter quotes Joel 2:28-32 in Acts
2:17-18, we can easily see what Peter is talking about when speaking of "the promise" in Acts 2:39.

"And it shall be in the last days,’ God says, ‘that I will pour forth of My Spirit upon all mankind;
and your sons and your daughters shall prophesy, and your young men shall see visions, and
your old men shall dream dreams, Even upon My bondslaves, both men and women, I will in
those days pour forth of My Spirit," (Acts 2:17-18).
See also, "For I will pour out water on the thirsty land, and streams on the dry ground; I
will pour out My Spirit on your offspring, and My blessing on your descendants," (Isa. 44:3).

Peter states in Acts 2:38, "Repent, and let each of you be baptized in the name of Jesus Christ for
the forgiveness of your sins; and you shall receive the gift of the Holy Spirit." Peter is clearly speaking
of the promise of God to grant the Holy Spirit in a new and better way. But is he saying that people
become saved by baptism in water or that baptism is part of salvation? Not at all. Peter is simply
speaking covenantally about the covenant sign. Baptism! Consider this proof, from Peter, that people
are saved before baptism.

"While Peter was still speaking these words, the Holy Spirit fell upon all those who were
listening to the message. 45And all the circumcised believers who had come with Peter were
amazed, because the gift of the Holy Spirit had been poured out upon the Gentiles also. 46For
they were hearing them speaking with tongues and exalting God. Then Peter answered,
47
"Surely no one can refuse the water for these to be baptized who have received the Holy
Spirit just as we did, can he?" 48And he ordered them to be baptized in the name of Jesus
Christ. Then they asked him to stay on for a few days," (Acts 10:44-48).

Notice that Peter had been preaching the gospel and the Holy Spirit fell upon the people. In verse
45 we see that "the gift of the Holy Spirit had been poured out upon the Gentiles also." These people
were saved. The gift of the Holy Spirit was on the Gentiles and they were speaking in tongues. This is
significant because tongues is a gift given to believers, see 1 Cor. 14:1-5. Also, unbelievers don't
praise God. They can't because praise to the true God is a deep spiritual matter that is foreign to the
unsaved (1 Cor. 2:14). Therefore, the ones in Acts 10:44-48 who are speaking in tongues and praising
God are definitely saved and they are saved before they are baptized. This simply isn't an exception. It
is a reality.

Conclusion

Acts 2:38 so closely ties repentance and baptism because it is contextually covenant language and
covenant concept. It is not stating that you must be baptized in order to be saved. It is saying that
baptism is the complete and total covenantal identification with Christ in His death, burial, and
resurrection. It is not the covenant representation (baptism) of what Christ did that saves us, but the
reality of His sacrifice which we receive by faith (Rom. 5:1; Gal. 3:8). That is why we can see in Acts
10:44-48 a group of people who are saved before they are baptized.
Baptism is not what saves. It is not part of salvation. It is something someone does who is
already saved.
Must baptism be "in Jesus' name"?

Oneness Pentecostal theology maintains that baptism must be by immersion using the formula "in
Jesus name" and not the formula "in the name of the Father, the Son, and the Holy Spirit" as is found
in Matt. 28:19. They reject the Trinitarian formula because they reject the Trinity. To support their
method, they cite various Bible verses that reference baptizing in Jesus' name and claim that this is
proof for their doctrine. Following are some of the Bible references they quote.

• Acts 2:38," Then Peter said unto them, Repent, and be baptized every one of you in the
name of Jesus Christ for the remission of sins, and ye shall receive the gift of the Holy
Ghost."
• Acts 8:16, "For as yet he was fallen upon none of them: only they were baptized in the
name of the Lord Jesus."
• Acts 10:48, "And he commanded them to be baptized in the name of the Lord. Then prayed
they him to tarry certain days."
• Acts 19:5, "When they heard this, they were baptized in the name of the Lord Jesus."

The phrase, "in the name of the Lord" is not a reference to a baptismal formula, but a reference to
authority. It is similar to hearing someone say, "Stop in the name of the Law!". We understand that
the "name of the Law" means by the authority of the Law. It is the same with baptism "in Jesus'
name." Consider the following:

"And when they had placed them in the center, they began to inquire, "By what power, or in
what name, have you done this?" 8 Then Peter, filled with the Holy Spirit, said to them,
"Rulers and elders of the people, 9 if we are on trial today for a benefit done to a sick man, as
to how this man has been made well, 10 let it be known to all of you, and to all the people of
Israel, that by the name of Jesus Christ the Nazarene, whom you crucified, whom God
raised from the dead — by this name this man stands here before you in good health" (Acts
4:7-10). (See also Acts 4:17-18; 5:28; 5:40-41; 8:12; 9:27-28.)

We can see that the phrase is used in the Bible as an expression of authority. This is also verified
in Acts 16:18 which says, "And this did she many days. But Paul, being grieved, turned and said to the
spirit, I command thee in the name of Jesus Christ to come out of her. And he came out the same
hour." We also see that when people were being baptized that they did it calling on Jesus' name ( Acts
22:16); that is, they were calling upon Jesus who has all authority in heaven and earth ( Matt. 28:18).
The church is supposed to "call upon the name of the Lord Jesus" (1 Cor. 1:2) because it is by His
authority (John 1:12) that we Christians have the hope and right of forgiveness of sins and adoption as
His children (Rom. 8:15).
Therefore, the Oneness Pentecostal people are simply in error by demanding that baptism be done
with the formula "In Jesus name." Instead, it should be done as Jesus commanded:

"Go ye therefore, and teach all nations, baptizing them in the name of the Father, and of the
Son, and of the Holy Ghost" (Matt. 28:19).
Is speaking in tongues a necessary sign of salvation?

The Oneness Pentecostal people teach that speaking in tongues is a necessary manifestation of the
Holy Spirit and without it a person is not truly saved. 104 It is so important to them that one ex-
oneness person told me that her church had altar calls for people to come up and receive the gift of
the Holy Spirit so they could speak in tongues. She said she never saw an alter call for people to come
up and receive Jesus as savior. But, this is only one example and may not be typical.
We see in Acts that many people who became Christians immediately spoke in tongues ( Acts 2:4;
10:46; 19:6). But is it a necessary sign of salvation? No. It isn't. Consider the following verses in 1
Cor. 12.

• 1 Cor. 12:7-11, "But the manifestation of the Spirit is given to every man to profit withal. 8For
to one is given by the Spirit the word of wisdom; to another the word of knowledge by the
same Spirit; 9 To another faith by the same Spirit; to another the gifts of healing by the same
Spirit; 10 To another the working of miracles; to another prophecy; to another discerning of
spirits; to another divers kinds of tongues; to another the interpretation of tongues. 11But all
these worketh that one and the selfsame Spirit, dividing to every man severally as he will."
• 1 Cor. 12:29-30, "Are all apostles? are all prophets? are all teachers? are all workers of
miracles? 30Have all the gifts of healing? do all speak with tongues? do all interpret?"

We can see that the Holy Spirit gives gifts as He desires. He distributes them upon His people in
the church as He wills (1 Cor. 12:11). It states in 1 Cor. 12:7-11 that different people have different
gifts and we see in verses 29-31 that not all speak in tongues. Now, the Oneness person will state
that all are supposed to speak in tongues, but that not all do. They maintain that speaking in tongues
is a necessary sign gift of true salvation and that a true believer will speak in tongues. But, that is
not what the text says.
Paul asks the questions: Are all apostles? No. Are all prophets? No. Are all teachers? No. In
other words, within the body of Christ, different people are called by God to have different gifts. If
someone states that all are supposed to speak in tongues, but that not all do, then are all supposed to
be apostles as well but not all are? Are all called to be prophets? Are all called to be teachers? No.
Likewise, not all are called by God to speak in tongues.
It is simple. Not all speak in tongues because God doesn't give the gift to everyone. Speaking in
tongues is not the sign of salvation, but a sign. If anything, the fruit of the indwelling Spirit of God is
listed in Gal. 5:22-23 is the sign of salvation. I would ask the Oneness person if the following fruit are
what the true signs of salvation in his or her life rather than speaking in tongues:

"But the fruit of the Spirit is love, joy, peace, patience, kindness, goodness, faithfulness,
23
gentleness, self-control; against such things there is no law" (Gal. 5:22-23).

So, again I ask. What is the biblical sign of salvation, tongues or the fruit of the Spirit?

104
This paper is not dealing with the issue of whether or not the charismatic gifts are still in operation.
Who did Jesus pray to?

Oneness theology states that the Father was in the Son and that the person of Jesus was also the
person of the Father. Of course, when we see instances in the Bible where Jesus prays to the Father,
we naturally wonder how this can be if they are they same person. But, according to Oneness
theology, Jesus was praying to the Father, the true divinity though it was Jesus the man who was
praying. But, how can this be? Was Jesus praying to Himself (since God is only one person) and
making it appear that He was praying to someone else?
Oneness theology correctly states that Jesus has two natures. He was both God and man while He
walked this earth. 105 But it states that the human part of Jesus was praying to the divine essence of
God as the Father. What they do is divide Jesus into two parts and have the human nature address
the divine nature.
The problem with this is that it threatens the incarnation of the Word made flesh as a complete and
single person. Jesus was both God and man in one person. He had a will. He ate. He slept, etc. He
was a man. He needed to be a human in order to bear the sins of people. He needed to be God in
order to offer a sacrifice to God the Father sufficient to cleanse us of our sins. No mere man could do
this. But the fact is, Jesus was one person -- and still is. 106 Jesus was both God and man at the same
time in the form of a single person.
The Oneness explanation risks the error of Nestorianism 107 which stated that Jesus was two
separate persons: a human person and a divine person in the form of one man. No where in the Bible
does it state that Jesus was two persons. Rather, we find scriptures where Jesus refers to Himself as
"I" and "Me" and "mine" not "us" or "our." The Oneness position is simply in error.
Jesus was not praying to Himself. We see in Scripture, Jesus praying to the Father (John 17). We
see Him addressing another person who is called God. We see Jesus saying, "Not my will, but your
will be done" (Luke 22:42) when He addressed God the Father. In other words, they had separate
wills -- at the same time. He was not praying to Himself, or an extension of Himself, of a part of
Himself. He was praying to the person of the Father.
Furthermore, according to Oneness theology, Jesus would have had to exist at the same time as
the Father if Jesus the man was praying to the Father. If this is so and Jesus was addressing the
Father, then we have two simultaneous persons. But in Oneness theology, this is a problem since God
is only one person who occupies consecutive modes. How then could the "mode" of the Father and the
"mode" of the Son be in existence at the same time if Oneness theology is correct? They cannot,
which is another reason why Oneness theology is wrong.

105
Actually, Jesus is still God and man. Col. 2:9 states that in Him dwells all the fullness of the Godhead bodily.
Note, the verse says "dwells", not dwelt. Dwells is in the present tense, not the past. Therefore, in Jesus, right
now, dwells the fullness of the Godhead.
106
See my article Oneness and the word "person" that deals with the nature of personhood.
107
This heresy is attributed to Nestorius who was a preacher at Antioch and the Bishop of Constantinople around
428 A.D. However, Nestorius did not preach that Jesus was two persons in one body, but his name has become
attached to this error.
Jesus' resurrection and ascension

One of the problems with oneness theology is in dealing with the resurrection of Jesus. Oneness
Pentecostal theology states that Jesus had two natures while he was walking the earth. That is, he was
both God and man. This is correct theology in so far as it states that within the one person of the Son,
there are two natures: God and man. But it also states that God is now in the "form" of the Holy
Spirit. What happened to Jesus' body after the ascension? Where is it? Is it alive? Is it in a coma?
Was it dissolved? Does it still exist? Is Jesus still a person? If so, how can Jesus, the person with
flesh and bones, also be the Holy Spirit?
In Trinitarian theology the second person of the Trinity became flesh (John 1:1, 14). In other
words, according to Phil. 2:5-8, Jesus added to himself human nature. Likewise in Col. 2:9 it states,
"For in Him dwells all the fullness of the Godhead bodily." 1 Tim. 2:5 says that "there is one mediator
between God and man, the man Christ Jesus." We know that Jesus rose from the dead in the same
body He died in since He prophesied He would raise His body (John 2:19-21); He retained the scars of
His ordeal after the resurrection (John 20:27); and He was seen as a man after the resurrection (Luke
24:39).

• "Behold my hands and my feet, that it is I myself: handle me, and see; for a spirit hath not
flesh and bones, as ye see me have," (Luke 24:39).
• "Then saith he to Thomas, Reach hither thy finger, and behold my hands; and reach hither
thy hand, and thrust it into my side: and be not faithless, but believing," (John 20:27).

As you can see from the above verses, Jesus retained His physical nature after His resurrection --
along with His scars. This is why it says in Col. 2:9 that in Him dwells all the fullness of the Godhead
bodily. Notice that the verb "dwells" is in the present tense. That is, right now Jesus has a body of
flesh and bones. He is physical. He is in heaven. He is a man, the Godman.
Nevertheless, some deny Jesus' resurrection by stating that the Bible says flesh and blood cannot
inherit the kingdom of God and therefore Jesus did not rise in the same body he died in. But, we need
to realize that the Bible says, "flesh and blood cannot inherit the kingdom of God" (1 Cor. 15:50), not
flesh and bones as Jesus said He had (Luke 24:39). Is this important? Yes it is. Jesus’ blood was the
sacrifice that cleanses us from sin (Lev. 17:11; Heb. 9:22). I suspect that Jesus' resurrected body did
not have any blood in it. It was shed from His body on the cross.

Jesus' Ascension and Return

The Bible tells us that Jesus ascended into the sky (Acts 1:9-11). When He did this He was still in
physical form as I've demonstrated above. In addition, the Bible tells us that Jesus will return in the
same manner.

"And when he had spoken these things, while they beheld, he was taken up; and a cloud
received him out of their sight. 10And while they looked steadfastly toward heaven as he went
up, behold, two men stood by them in white apparel; 11Which also said, Ye men of Galilee, why
stand ye gazing up into heaven? this same Jesus, which is taken up from you into heaven, shall
so come in like manner as ye have seen him go into heaven," (Acts 1:9-11).

Since the Bible teaches us that Jesus is in bodily form now (Col. 2:9), then how does the Oneness
Pentecostal person maintain that God is in the form of the Holy Spirit? Also, when Jesus returns, will
He return in His body? Will God's form then revert to the form of the Son at His return according to
Oneness?
I do not think Oneness Pentecostal theology is correct for many reasons. But here, with this issue
of Jesus' resurrection and ascension, I see their theology denying the incarnation of God in flesh right
now. After all, it says in Col. 2:9 that Jesus is in bodily form now. Oneness denies that since God,
according to its theology, is now supposed to be in the form of the Holy Spirit

God was seen in the Old Testament. Who was it?


Oneness Pentecostal theology teaches that God exists as one person who revealed Himself in three
modes, or forms. In the Old Testament, God revealed himself as the Father. When Jesus was on
earth, the revelation was as the Son. Now, God is in the mode of the Holy Spirit. Basically, oneness
teaches three consecutive modes of God: the Father who became the Son who became the Holy Spirit.
There are variations on this model within Oneness churches, but I will focus on this model here.
However, in the Old Testament there are numerous places where God is seen. In some places this
is the angel of the Lord. In others it is a vision or a dream. But, there are instances in the Old
Testament where God is seen and it is not an angel, a vision, or a dream. Of course, this can raise
some warning flags for Trinitarians as well as Oneness people. But when you look at the totality of
Scripture, you'll find that the Trinitarians have an easy answer where the Oneness Pentecostal people
do not.
Basically, in those places where God was seen in the Old Testament, it was the person of Jesus;
that is, it was the pre-incarnate Word that was seen. It was not the person of the Father that
appeared in the OT because Jesus said that no one has ever seen the Father (John 6:46). Yet, God
Almighty was seen (Exo. 6:2-3). For the Oneness people, this is a problem since God was in the mode
of the Father in the Old Testament and to them, it had to be the Father.

• John 6:46, "Not that any man hath seen the Father, save he which is of God, he hath seen the
Father."
• Exo. 6:2-3, "And God spake unto Moses, and said unto him, I am the LORD: 3And I appeared
unto Abraham, unto Isaac, and unto Jacob, by the name of God Almighty, but by my name
JEHOVAH was I not known to them."
• Num. 12:6-8, "And he said, Hear now my words: If there be a prophet among you, I the LORD
will make myself known unto him in a vision, and will speak unto him in a dream. 7My servant
Moses is not so, who is faithful in all mine house. 8With him will I speak mouth to mouth, even
apparently, and not in dark speeches; and the similitude of the LORD shall he behold:
wherefore then were ye not afraid to speak against my servant Moses?"
• See also, Exo. 24:9-11; Gen. 17:1; 18:1; 19:24 with Amos 4:10-11; Acts 7:2.

As you can see from the above scriptures, God Almighty was seen, but it was not God the Father.
How then can Oneness theology be correct if God was in the mode of the Father in the Old Testament,
that God was seen, and yet Jesus said the Father was not seen? The only answer the Oneness people
can give is that God appeared as an angel, or in a vision or dream. But if that is so, then is an angel
God Almighty? Look at Exo. 6:2-3. God identifies Himself as Jehovah (His self given name) and states
He appeared to Abraham, Isaac, and Jacob as God Almighty. Was that an angel? Was it an angel who
said, "I am God Almighty"? No. It was God Almighty. Or look at Num. 12:6-8 where God Himself
declares that He does not appear to Moses in a vision or a dream, but that Moses beholds His very
form. This negates the possibility, at least in this scriptural occurrence that God appeared to Moses in
a vision or dream.
God was seen in the Old Testament, but it was not the Father. It was the Son, the pre-incarnate
Christ. Therefore, the Son existed at the same time as the Father in the Old Testament and Oneness
theology is shown to be incorrect because we have both the Father and Son existing at the same
time.
Isaiah 9:6, Is Jesus the Everlasting Father?

“For a child will be born to us, a son will be given to us; And the government will rest on His
Shoulders; and His name will be called Wonderful Counselor, Mighty God, Eternal Father, Prince
of Peace,” (Isaiah 9:6).

Oneness Pentecostal believers deny the Trinity and teach that Jesus, the Father, and the Holy Spirit
are all one person. They sometimes quote Isaiah 9:6 in their attempt to prove their position.
However, Isaiah 9:6 cannot be used to disprove the trinity nor bolster their oneness doctrine.
When Isaiah 9:6 says that Jesus' name will be called Wonderful Counselor, Mighty God, Eternal
Father, etc., it is not saying that Jesus is the eternal Father, but that he has the characteristics of
God. In other words, Jesus has all the attributes of God, including eternality.
In the ancient Jewish culture, names had meanings. We can better understand this by noting
American Indian names such as "Running wolf" or "Fighting Bear." The same with Jewish names.
They had meanings. Isaac, for example, means "laughter." Noah means "rest" or "peace." So, when
Isaiah is speaking of the name of the coming Messiah and says his name will be Mighty God, Eternal
Father, etc, it is telling us about the characteristics of the Messiah to come in a prophetic manner.
If Jesus' name is "Eternal Father," then why don't we call Jesus "Eternal Father"? For that matter,
why don't we call his name "Wonderful counselor," or "Mighty God," or "Prince of Peace"? The text
speaks of a name, yet has four things revealed in the name. Again, this shows us that it is the
characteristics of the then-coming Messiah. The fact that the Messiah would be divine is verified in
Heb. 1:3, when it says, "And He [Jesus] is the radiance of His [God] glory and the exact
representation of His nature, and upholds all things by the word of His power..." This also explains
why Jesus said, "...He who has seen Me has seen the Father," (John 14:8). It was because Jesus so
precisely represented God the Father as His prophesied name reveals.
Furthermore, the oneness Pentecostal people assert that God's name is really "Jesus." But, if that
is true, and if Jesus is the eternal Father as they claim, then why don't they call Jesus "The Eternal
Father" as His name? Does it also mean that the mode that God is in right now is that of the Father
since His name is "Eternal Father" implying He is always the Father. If that is taken literally, then God
is the eternal Father, and the true person of the Godhead is the Father, not the Son as the Oneness
people assert.
The oneness Pentecostal theology is incorrect and improperly describes the true and living God.
What is the real gospel message?
The real gospel message is that salvation is by grace through faith (Rom. 6:23), not faith and
something you do like baptism or faith and speaking in tongues, or faith and going to a oneness
churches, etc. True salvation is freedom from the requirement of keeping any part of the Law to get
or maintain salvation. True salvation is receiving Christ (John 1:12), being in the body of Christ, and
being redeemed by the blood of the Lamb. It means that we are not obligated to keep a moral code
such as women should not wear pants, shorts, makeup, or jewelry. It does not mean that we are in
danger of losing our salvation if we go to movies, own TV's, swim in public, or wear facial hair as the
Oneness Churches emphasis.108 We have been set free from the Law. Of course, this does not mean
that we are free to sin. Not at all, for how shall we who have died to sin still live in it ( Rom. 6:1-2)?
But after studying Oneness Pentecostal theology, I have concluded that it is not Christian since it
denies the the true God by denying the Trinity and denies salvation by grace by adding a
requirement: baptism. In addition, there is the legalistic expectation of a moral code that must be
obeyed lest they be in danger of forfeiting the forgiveness of sins which Christ so lovingly bought and
freely gives. This is works righteousness.
As is the case in cult groups with false gods, false gospels follow. Such is the case with Oneness
Pentecostal theology. It teaches a false god and a false gospel. The god of Oneness theology is not a
Trinity. Its gospel is faith plus works: baptism, speaking in tongues as a necessary sign, and
obedience to a moral code as a demonstration of salvation. Are we to obey God's word? Absolutely
yes! Are we free to go and sin? No. We are obligated to obey God, but not as a requirement to be
saved or keep salvation. We are saved by faith (Rom. 5:1) and because we are saved, then
obedience follows. Keeping moral laws does not keep us in the faith. Being baptized is not what saves
us.

The true gospel is defined by Paul in 1 Cor. 15:1-4 as that which saves us. Notice that baptism is
not mentioned here:

"Now, brothers, I want to remind you of the gospel I preached to you, which you received and
on which you have taken your stand. By this gospel you are saved, if you hold firmly to the
word I preached to you. Otherwise, you have believed in vain. For what I received I passed on
to you as of first importance: that Christ died for our sins according to the Scriptures, that he
was buried, that he was raised on the third day according to the Scriptures, " (NASB).

Faith is only as good as who it is placed in. It is, therefore, essential that we rightly understand
God as He has revealed Himself, a Trinity: Father, Son, and Holy Spirit. We see the Son speak to the
Father and pray to Him (John 17). We know that God's word says that the Son bore our sins in His
body on the cross (1 Pet. 2:24) and purchased us with His own blood (Acts 20:28). We are justified
apart from the works of the Law "because by the works of the Law no flesh will be justified in His
sight . . .” (Rom. 3:20), and "for we maintain that a man is justified by faith apart from works of the
Law," (Rom. 3:28), and "For what does the Scripture say? ‘And Abraham believed God, and it was
reckoned to him as righteousness'" (Rom. 4:3), and "Therefore, having been justified by faith... "
(Rom. 5:1), and "But to the one who does not work, but believes in Him who justifies the ungodly, his
faith is reckoned as righteousness" (Rom. 4:5).
Those in Oneness theology need to come to the true Trinitarian God and Savior, renounce the
addition of baptism to salvation and rely completely and totally on God's saving grace by faith alone.
Then, they need to look at the fruit of the Spirit (Gal. 5:22-23) instead of speaking in tongues as an
evidence of their salvation.

May Jesus be glorified.

108
I am not saying that they do not have the right to practice holiness as they perceive it. I am addressing the
issue of the externalities as a necessary sign of being saved.
Witnessing to those who are in oneness churches

The Oneness Pentecostal people, as a whole, absolutely believe they have the truth and that the
Trinity doctrine is pagan in origin. With this in mind, when you speak to them, you'll encounter an
attitude that they are absolutely right. It is very difficult to break through this and often takes a lot of
time and effort. Think about it. If you are very convinced of your position in contradiction to theirs,
how would you feel if they tried to convince you that you were wrong? It'd be very difficult to do.
Nevertheless, it is important to keep a comfortable and humble dialogue going with them. Pride is
not the answer to error. God's love must come through. So, when witnessing to them, or anyone,
you need to show love and respect. They need to know the same love of Jesus in you that they claim
they have from God. Jesus said that the world will know that we are His disciples by the love we have
for one another, (John 13:35). Of course, being loving means being truthful with them. If you don't
know something, admit it. If you aren't sure about something, that's okay. Be truthful.
You also need to demonstrate to them the love of Jesus in your life. You need to emphasize that
Jesus Christ is the center of your faith. Oneness Pentecostals believe that they are the only ones who
have the true love of God. In that sense, they believe that they are a privileged group of people
enjoying a special knowledge and special relationship with God. That is why they emphasize so much
the need to follow their doctrines, be baptized in their churches, by their ministers, according to their
understanding of the gospel. They honestly believe it. So, you need to be patient, kind, and truthful
with them in your witness.
Also, and this is quite common, correct any misunderstanding of the doctrine of the Trinity. So
often, opposing groups have a misguided concept of the Trinity and end up attacking a straw man
argument. Of course, to correct any misunderstanding, you must first understand it yourself. So, if
you're rusty, read up on the Trinity. Then, when you think you've got it down, you can share what you
know with them. Show scriptures that demonstrate the Trinitarian nature of God.
You will need to demonstrate some of the problems with the Oneness Pentecostal position -- and
they exist. In order to do that, you will need to read more about them and the issues here on this
website as well as other websites and books on the subject. Unfortunately, witnessing to those in
opposing theological beliefs often means that you have to study theology. It isn't that tough. It just
takes some work and practice. But it is always worth it.
Do you know what they believe? You cannot refute error if you do not understand what you are
talking about with them. Personally, I like dialoguing with them on the internet to learn what they
believe. Reading Oneness material is helpful but it doesn't answer all the questions that I have.
Bouncing things off of them gives me a feel for how they think as well as what they think. That helps
me witness to them a lot.
Finally, pray. You must ask the Lord to bless your efforts and bring fruition to the seeds planted.
It is God who bears the fruit of truth.

"So then neither the one who plants nor the one who waters is anything, but God who causes
the growth" (1 Cor. 3:7).
Questions to ask Oneness Pentecostal believers

The following questions are not "stoppers." That is, they are not questions to ask Oneness people
so you can "stump them for sure." Instead, they are questions to ask to generate conversation. It is
during the conversation that real witnessing occurs.
Of course, I have found some of these questions to be more difficult than others for the Oneness
person to adequately answer. In fact, two of them no Oneness Person has adequately answered at
all. Which are they? Try them out.

1. Is Jesus His own Father?


2. If Jesus' will and the Father's will were identical, then why did Jesus express the desire to escape
the cup but resigns Himself not to His own will, but the will of the Father? See my article on this.
3. Was Jesus praying to Himself in the Garden of Gethsemane?
4. If Jesus was praying to the divine side of Himself, then isn't He still praying to Himself?
5. Why was Jesus not saying, "Not My will, but My will be done?" if there is only one person and one
will involved when He was praying in Luke 22:42 & Matt. 26:39.
6. If baptism is essential for salvation, then what happens to someone who repents of sin, accepts
Jesus as Savior, walks across the street to get baptized but is killed by a car. Does he go to
heaven or hell?
A. If he goes to heaven, then baptism isn't a requirement is it?
B. If he goes to hell, then faith in Christ isn't sufficient to save him is it?
7. Since the Bible teaches us that Jesus is in bodily form now (Col. 2:9), then how does the Oneness
Pentecostal person maintain that God is in the form of the Holy Spirit? Also, when Jesus returns,
will He return in His body? Will God's form then revert to the form of the Son at a later date?
8. If God is only one person, why did Jesus say in John 14:23, "If a man love me, he will keep my
words: and my Father will love him, and we will come unto him, and make our abode with him."
If God is only one person, why does Jesus say, "we"?
9. Oneness theology teaches that God was in the mode of the Father in the Old Testament. God
was seen in the OT (not as a vision or a dream or an angel in the following verses: Exo. 6:2-3;
Gen. 19:24; Num. 12:6-8). But, Jesus said no one has seen the Father (John 6:46). If they
were seeing God Almighty (Exo. 6:2-3) but it wasn't the Father, then who was it?
Answers and response to Questions to ask Oneness Pentecostal believers

Via email, I received answers to the questions in the paper, Questions to ask Oneness Pentecostal
believers. I have reproduced the answers given and responded to them accordingly. The original
questions are in bold. His responses are underlined. My responses follow his.

1. Is Jesus His own Father?


A. The response given was "Yes, Jesus is his own Father." Of course, this is an illogical position
to hold. I am my Father's son, therefore, I cannot be my own Father. But, seeing the illogic
of this position, the following comment was offered after several scriptures were quoted.
B. "Therefore Jesus is the Father (in relation to His deity), and the Son of the deity (in relation
to deity working through Humanity)." The problem with his statement found in rightly
understanding what the Bible teaches concerning the Son. Jesus as a single person has two
natures. This is called the hypostatic union; that is, in the one person of Christ are two
natures: divine and human. The oneness position effectively divides the one person of
Christ into two persons, the Father and the Son, by splitting Jesus into two separate, not
unified, parts. Jesus is either divine or He is not. He is either the God-man in one person,
or He is not. We cannot have Jesus be his own Father in respect to his deity and the son in
respect to his humanity. For a son to be his own father is illogical and the oneness position
must backpeddle and erringly divide the natures of Christ into two persons, not one.
2. If Jesus' will and the Father's will were identical, then why did Jesus express the desire
to escape the cup but resigns Himself not to His own will, but the will of the Father?
A. "If you are posing this argument to try to make two wills within the godhead (which in this
case are contrary to one another) then you are promoting outright polythiesm." This
statement reveals a lack of understanding not only of the Trinity, but also of logic. The
Trinity is the doctrine that there is one God in three persons. Each person has a will. This
does not necessitate the existence of three gods and this issue has been thoroughly
discussed throughout Christian history. Nevertheless, we see in scripture that only one God
is proclaimed and yet the Father, the Son, and the Holy Spirit each have wills and are each
called God. A will denotes identity and self awareness. The Father has a will and the Son
has a will. Are the two wills really one will? Of course not. In addition, it is the mistake of
the oneness to accuse the Trinitarians of being polytheists, an unfortunate and erring attack
that only demonstrates the ignorance of the doctrine of the Trinity, the thing they are
attacking.
3. Was Jesus praying to Himself in the Garden of Gethsemane?
A. "Since Jesus is God, is He the SAME God He was praying to or was He praying to a different
God?” The person said he had adequately answered this question in his previous comments
to question number two. Of course, he hadn't answered it adequately at all. He then poses
the above question. Again, this kind of question further demonstrates a lack of
understanding of the doctrine of the Trinity. The answer is simple. The person of the Son
was praying to the person of the Father. It was not one god praying to another god.
B. I would hope that if someone wished to attack the doctrine of the Trinity that he would at
least accurately represent it in his attacks. To misrepresent it is to attack a straw man.
4. If Jesus was praying to the divine side of Himself, then isn't He still praying to Himself?
A. "As I have answered above, even if we say "Yes" that is not a Biblical problem." But this is
precisely the problem. It would be like saying that the person of the human side of Jesus
was praying to His divine side. If that were the case, then we have two beings in the person
of Christ which would be ludicrous.
5. Why was Jesus not saying, "Not My will, but My will be done?" if there is only one
person and one will involved when He was praying in Luke 22:42 & Matt. 26:39.
A. "Once again, Jesus was speaking in His humanity...In His deity His will was one and the
same with God, because He is God. In His humanity He had a human will, that He submitted
to God." This seems to be only a confusing answer at best and does not answer the
question. Who is "God" in his answer if God, to the oneness people is at that time, Jesus?
We either have Jesus praying to Himself or we have Jesus the Son, praying to the person of
the Father. The oneness position makes no sense.
6. If baptism is essential for salvation, then what happens to someone who repents of sin,
accepts Jesus as Savior, walks across the street to get baptized but is killed by a car.
Does he go to heaven or hell?
A. There really wasn't much of an answer given. He simply tried to state that baptism is
necessary in order to be saved. He also wrote about infants who die and verbal
acknowledgement of God when becoming a Christian. But, he did get around to saying that
baptism is not an option and then ended with saying that the hypothetical position I
proposed would never happen. In other words, he didn't answer it.
B. It seems quite obvious to me that he sees the problem that I posed in the original question:
If he goes to heaven, then baptism isn't a requirement is it? If he goes to hell, then
faith in Christ isn't sufficient to save him is it? To this, he did not respond and I believe
it was because it demonstrates the error of his position and there is no way to answer it
except to say that it wouldn't happen.
7. Since the Bible teaches us that Jesus is in bodily form now (Col. 2:9), then how does
the Oneness Pentecostal person maintain that God is in the form of the Holy Spirit?
Also, when Jesus returns, will He return in His body? Will God's form then revert to the
form of the Son at a later date?
A. "This is a prime example of how you have not only a misunderstanding of the oneness
position, but also your own theology of Trinitarianism." The individual did not really answer
the question. Instead, he made statements like "God is still Spirit and He is operating
through a human body. Scripture confirms that Jesus operates in more than just a human
body form, and "Jesus making it explicitly clear that He is the Holy Spirit in the form of a
human body (dwelling with them)." Nevertheless, this person went on to say that Jesus
would return in His body. But, to be perfectly honest, I really did not understand what this
person was getting at. When speaking with oneness people about this, I've often ended the
conversation feeling rather confused. It could simply be my lack of ability to understand that
particular position, but it could also be that their position just doesn't make sense.
8. If God is only one person, why did Jesus say in John 14:23, "If a man love me, he will
keep my words: and my Father will love him, and we will come unto him, and make our
abode with him." If God is only one person, why does Jesus say, "we"?
A. "If you are trying to use Jesus' use of "We" to imply literally more than one, then you are
promoting two Spirits (three counting the Holy Spirit). The Bible says there is only ONE Spirit
(Ephesians 4:4,5) Not two not three. ONE." and "He was simply speaking in simple
language easier for the listener to understand." Again, this person erringly inserts into the
discussion something not held by Trinitarians; namely, that God is three spirits. This is
something that repeatedly arises in discussions with oneness people. They continually
misrepresent the doctrine of the Trinity. Furthermore, to say that Jesus was simply using
language they could understand really ignores what Jesus was actually saying.
9. Oneness theology teaches that God was in the mode of the Father in the Old
Testament. God was seen in the OT (not as a vision or a dream or an angel in the
following verses: Exo. 6:2-3; Gen. 19:24; Num. 12:6-8). But, Jesus said no one has
seen the Father (John 6:46). If they were seeing God Almighty (Exo. 6:2-3) but it
wasn't the Father, then who was it?
A. "Once again, you are demonstrating your lack of understanding of Oneness theology, and
your own theology." I certainly may not be understanding oneness theology completely, but
I do understand my own far better than this gentleman as I have asserted earlier in this
paper. Typically, oneness people misrepresent the Trinity doctrine and when I correct them,
they tell me I am wrong. This is because it is easier for them to attack a strawman
argument rather than the real thing.
He then states "In fact let me go ahead and turn the argument around on you. The Bible
actually states that no one has seen GOD at any time...Your own theology teaches that Jesus
is God. If Jesus is God, then why does the Bible say that no one has seen God?" The answer
is simple, God was seen in the OT many times (Gen. 17:1; 18:1; Exod. 24:9-11; Num.
12:6-8). But, according to Jesus in John 6:46, it was not the Father. Therefore, they were
seeing God the Son, the preincarnate Word.
Finally, the question I posed in no. 9 above is the result of attending a United Pentecostal
Convention (A Oneness group) and speaking to five UPC pastors who acknowledged the
modal view that the Father became the Son who became the Holy Spirit. Since then, I have
heard differing views from Oneness people on the modes of the Father, Son, and Holy Spirit
through biblical history. It is certainly possible that this gentleman retains a different view
than the UPC or a similar one. Either way, I used the Plurality Study as a means of refuting
their position to which all five UPC pastors admitted they had no answer.
Universalism
Introduction

Universalism is a philosophy, a way to trying to convince people that all people will eventually be
saved no matter what they do here on earth. Is it true? No. But, that hasn’t stopped many
universalists from attacking the historic Christian position that God will damn people to hell. To them,
such a teaching is wrong and they want everyone to know it.

Be careful of this attempted distortion of scripture that seeks to submit God’s holiness to human
preferences.

1. What is the basic teaching of universalism? p. 37


2. Can a Christian be a universalist? p. 42
3. What is one of the dangers of universalism? p. 43
4. How does Matt. 25:46 disprove universalism? p. 44
5. How does Mark 3:28-29 disprove universalism? pp. 46-48
6. God wants all to be saved, but does He arrange that all are? pp. 51-53
7. What do Greek dictionaries say about ‘aionion’? pp. 61-62
8. What are some scriptures that say not all are saved? p. 65
9. Is the unforgivable sin forgivable? pp. 66-69
10. Does God hate anyone? p. 70
11. Will the demonic forces ever be saved? p. 75
12. Does eternal punishment deny God's justice? p. 78
13. What is a danger of universalism? p. 79
Universalism

Universalism is the teaching that God, through the atonement of Jesus, will ultimately bring
reconciliation between God and all people throughout history. This reconciliation will occur regardless
of whether they have trusted in or rejected Jesus as savior during their lifetime. This universal
redemption will be realized in the future where God will bring all people to repentance. This
repentance can happen while a person lives or after he has died and lived again in the millennium or
some future state. Additionally, a few universalists even maintain that Satan and all demons will
likewise be reconciled to God.
Nevertheless, both facets of universalistic belief are in error. People will suffer eternal damnation
(Rev. 14:11) and the demonic forces have no redeemer. But, it is important to note that holding to
universalism in itself does not make one a non-Christian. Universalism alone is simply a non-essential
theological error held by some people.
However, there are those within the universalist camp who also deny the doctrine of the Trinity
and, thereby, the incarnation of the Word of God as God the Son. They also deny the personhood and
deity of the Holy Spirit. Usually, these denials are held by Unitarian Universalists, though others who
are not of the Universalist camp also deny the Trinity. Those who deny these essentials cannot be
classified as Christians.
To deny the deity of Christ is to deny one of the essential doctrines of salvation. In this sense,
those universalists who deny the deity of Christ are in a false religious belief system. Of course, when
one essential doctrine is denied, many other historic biblical doctrines are also denied and salvation is
void because the object of faith is false.
There is no official "Universal Salvation Church" denomination but there is a Unitarian Universalist
Association (UUA). The UUA can be classified as non-Christian because it denies the deity of Christ,
the personhood of the Holy Spirit, etc.
It is not possible to categorize all of universalists into one tidy doctrinal category. Its adherents
vary in belief. Some are Arian (God is one person, Jesus is a creation). Some are Trinitarian. Others
even lean toward new age concepts of man's divinity.
So, universalism is not really a doctrine that identifies a group. Rather, it is a doctrine of different,
even contradictory groups, who all claim universalism.

The problem with words

The cults are particularly guilty of using biblical words with non-biblical definitions. This is
absolutely necessary among them in order to maintain some sort of internal consistency of theology.
So too, with many universalists. Hell can mean non-existence, after-life consciousness, or this present
life on earth. Some universalists believe that all punishment is accomplished here on earth, while
others believe it is future event with a loss of rewards, and not a physical punishment. The
punishment in both groups is corrective and limited. It will last only as long, and only be as severe, as
it takes to accomplish its corrective purpose, which is to bring all mankind to a state of holiness and
happiness in obedience to God. Of course, the problem with this is that it strongly suggests that a
person is made worthy to be with God through his own sufferings and corrections in the afterlife.
In universalism, the word "eternal" means "without end" when it comes to salvation, but not when
referring to damnation, even though the same word is used for both and in the same context (Matt.
25:46). Universalists divide history and the future into different "eons" or "ages" and assert that
punishment is "age-lasting," not eternal. The term "Son of God" is claimed by all groups as an
accurate description of Jesus, yet to some it means a created being and to others it means God in
flesh. Therefore, determining which belief is held by which universalist is often difficult and it requires
digging.
Misrepresentation

Universalists often use the most negative terms to represent historic positions they disagree with.
For example, regarding the damnation of the unsaved, instead of saying that historic Christianity
teaches that those who reject Christ will suffer eternal damnation, they frequently say that historic
Christianity teaches that "God can't save everyone and wants to torture most of humanity forever."
Or, it is often implied that God will not torture people forever because "God is not sadistic enough to
send people to hell." Such emotionally slanted words reveal a hostile bias against historic doctrines
and is an unfair description of those beliefs. It is a surprisingly common tactic among universalists
which demonstrates their lack of objectivity and sheds an automatic cloud of doubt upon their
observations.

Conclusion

As you can see, universalism covers a wide range of beliefs. Under its umbrella can dwell the real
Christian as well as the false prophet. Though belief in universalism, in and of itself, does not void
salvation, it has the potential danger of allowing false teachers to abide alongside true believers.
Therefore, to determine if a universalist is Christian, you must delve further into other areas of his
belief.
Christian Universalism
Christian universalism is the teaching that all of mankind will ultimately be saved through Jesus
whether or not faith is professed in him in this life. It claims that God's qualities of love, sovereignty,
justice, etc., require that all people be saved and that eternal punishment is a false doctrine.
Salvation is not from hell, but from sin.
There are two main camps in Christian Universalism:

1. Those who teach that the unrepentant will be punished in a future state, and that their
punishment will be proportional to the degree of sin committed in the mortal state. They
generally hold that the punishment is moral and not physical. There is no hell. They do not
maintain that salvation is merited through these sufferings.
2. Those who teach that all the punishment for sin occurs in this life and that God's discipline in
our lives is for the purpose of purifying us, though this purification is not our merit for
salvation. In eternity, there will be a loss of reward for those who did not trust in Christ in this
lifetime.

Christian Universalists claim to hold many of the tenants of historic Christianity: Trinity, deity of
Christ, deity of the Holy Spirit, salvation by grace, etc. As always, it is necessary to inquire and ask
what is meant by the terms they use because the diversity that exists in universalist beliefs warrants
further examination. Nevertheless, the Christian universalists claim to affirm:

1. The inspiration and inerrancy of the Bible.


A. From what I have seen here, they are orthodox.
2. There is only one God.
A. From what I have encountered, most universalists who claim the title "Christian
universalists" do not accept the standard doctrine of the Trinity, but lean more towards
either Arianism (God is one person, Jesus is created) to modalism (God takes different
forms in history).
3. Jesus is the Son of the Living God
A. Many cult groups say the same thing. What they mean by the phrase is what is
important. The Christian Universalists tend to say the Son is a manifestation, an
image, a representation of God's essence, yet he is not equal to the Father. Therefore,
they are denying His true deity. Of course, not all Christian Universalists deny this.
B. Some hold that Jesus is not God but that He is divine. This is perplexing since divinity
is a quality of God, not angels or men.
4. Jesus' Resurrection

A. Most Christian Universalists affirm the physical resurrection of Jesus. But, some claim
he did not rise from the dead physically, but was assumed into heaven to dwell with
God. "The Crucified is living forever with God, as our hope. Resurrection does not mean
either a return to life in space and time or a continuation of life in space and time but
the assumption into that incomprehensible and comprehensive last and first reality
which we call God."109

i. If, by the above quote, the physical resurrection of Jesus is denied, as it seems it
is, then anyone who holds to that position is indeed a non-Christian since it denies
one of the essential doctrines of Christianity.
5. The Holy Spirit is God's presence
A. There is a surprisingly common denial of the personhood of the Holy Spirit.
(personhood is self-awareness, a will, the ability to speak, etc.). This is a serious error
on the part of those who hold to it. But to be fair, many universalists affirm the Holy

Quoted from "On Being a Christian: Twenty Propositions, By Hans Kung, "The Christian Challenge", pages 313-
109

316, 1979) as found at http://www.auburn.edu/~allenkc/challenge.html. #10.


Spirit as the third person in the Godhead.

6. There is no salvation without accepting Jesus as Savior


A. This statement is problematic for two reasons:
i. Since to many universalists, Jesus is not truly God by nature, they have an
improper object of faith (denying the Trinitarian nature of God and the deity of
Christ). Their faith, then, is useless since they have violated the command to
worship no other God (Exodus 20) and are worshiping a false god. The Jesus they
believe in, is not the real one.
ii. There is a second chance theology at work here where people who have rejected
Jesus in this life can come to faith in the next life, even though he has flatly
rejected Jesus' sacrificial atonement.
7. Some Universalists believe...
A. in consciousness after death, others do not.
B. in limited punishment of sinners in a type of hell that is not of fire, but of some moral
chastising.
C. that punishment in the afterlife was for a limited period during which the soul was
purified and prepared for eternity in the presence of God.

Conclusion
Christian Universalism is meshed with many other unorthodox and erroneous teachings. To
determine if a "Christian Universalist" really is Christian, you must ask more pointed questions of
individuals. This belief system should be avoided.
Can a Christian be a universalist?
The answer to this question is very easy. Yes. A Christian can be a universalist. Universalism in
itself does not make a person a non-Christian. Universalism is the teaching that all people eventually
will be reconciled to God. This is not one of the doctrines taught in scripture as a requirement for
salvation.
The essential doctrines of Christianity have been debated for centuries. These doctrines include the
physical resurrection of Jesus, salvation by grace, and that Jesus is God in flesh. With the last, many
conclude that the doctrine of the Trinity is also essential. Though the doctrine of the Trinity is not
explicitly stated as a requirement in God's word, it is logical to conclude that the true believer will
accept the Trinity teaching. The Trinity, then, becomes a test to see if someone is in faith. However, I
will admit that God casts his net further then we do and I do not believe that God will condemn all
people for their lack of the proper understanding of the Trinity doctrine. On the other hand, I believe
that those who openly reject it are not of God.
There are many doctrines that are not essential to salvation and it is these that lead us into the
differences of denominational beliefs. Where one group believes that baptism must be by immersion,
another teaches that sprinkling is acceptable. One denomination says that the charismatic spiritual
gifts have ceased, yet another that they continue. These types of the differences, and many others, do
not affect whether or not a person is saved. They are merely differences of opinion on the non-
essentials. In this, the Scriptures give us a great deal of leeway. See Romans 14.
A Christian can be a universalist, but not all universalists are Christian. It is not whether you accept
or deny universalism that makes you a true believer. Rather, it is faith in and acceptance of the true
and living God and Savior, Jesus. We have agreement in the essential doctrines that unite all
Christians and all denominations under one true God and we are allowed to have differences of
opinions.
Universalism and the Cults

Universalism teaches that all people will ultimately be saved no matter what they believe here on
earth. You could deny God, hate Him, blaspheme against Him, join a satanist group and murder people
and still go to heaven.
Bible based, non-Christian cults are those groups that claim to be Christian, use the Bible, yet
have redefined God, Jesus, and the gospel sufficiently to make salvation of no effect. Part of CARM's
purpose is to refute error and teach the truth so that people will not go to hell. Universalism is
definitely an error that needs to be address.
Let me take Mormonism, for example. Is Mormonism wrong? Is it dangerous to the soul? Does it
lead to damnation? The universalist would have to say no, even though Mormonism teaches that god
came from another planet, has a goddess wife, and that we can become gods (obvious false teaching),
universalists teach that Mormons go to heaven. Of course universalists who claim to be Christain
might assert that Mormon theology is wrong. But, they would also maintain that in the after life, they
would be able to repent and follow the true God. Logically, then, we could make the case that the
universalist would encourage the spread of Mormonism since it has good morals.
Mormonism is not Christian. It is false. It is a compilation of lies from the devil. It damns people
for believing in a false god, false gospel, and a system of works righteousness that is supposed to help
them become saved. But, to the universalist, such heresy amounts only to a goof, an error in
judgment, with the ultimate result being heaven. It makes no difference if a person is a universalist, a
Mormon, a Jehovah's Witness, or an orthodox Christian since they are all going to heaven according to
universalism.
To a universalist, there would be little or no need to refute Mormonism. Why? Because what is the
universalist going to warn him about? Damnation in hell? Not at all. Rather, he'd have to threaten
him with heaven! Instead, the universalist is more concerned with converting someone to "truth of
universalism." In so doing, they endanger the souls of all who they contact.
Let me illustrate this with a Universalist witnessing to a Mormon.

Universalist: "Listen here Mormon, if you continue to believe that you can become a god, that
Satan and Jesus are literal brothers, that God has a body of flesh and bones and has a goddess wife,
and that you can become a god of your own world, you know what is going to happen to you? You're
going to heaven! So there!"
Mormon: "Sounds good to me."

So, where is the power of universalism to correct Satan's lies? Does it carry a warning for those
who serve false gods except to say that it isn't nice to believe such things as Mormon doctrines? But,
so what? It doesn't matter. Mormonism, and other cults, would lead to heaven.
Is there orthodoxy in Universalism? Does it have the power and right to refute the errors of the
cults? I don't see how. But, I do see that it is dangerous.
Matt. 25:46 and Universalism

"And these will go away into eternal punishment, but the righteous into eternal life,"
(Matt. 25:46).

The universalists do not believe in eternal punishment. Universalists teach that all will eventually
be saved through the atonement of Jesus. Therefore, when the Bible speaks of eternal punishment
and hell fire, etc., the universalist interprets it to mean an inner sorrow due to loss of reward and/or
they maintain that the word "eternal" does not mean "without end."
In Greek, the word "eternal" is the word " ", or "aionion." This word occurs in two places in
Matt. 25:46: Let's look at it again in a Greek Interlinear form:

The exact same word " ," "aionion" is used to describe the duration of punishment as well
as of the life of the righteous - those who are saved. The same word describes both conditions. If it
means one thing in the first part of this sentence, then it means the same thing in the second part
since they are both in the same context and both are describing time-duration of the states of the
unsaved and the saved. If the punishment is eternal, then so is the life. Likewise, if, as the
universalist says, the punishment is not eternal, then neither is the life. You can't pick and choose
how the word is applied in this verse to suit your own theology.
But the universalists do just that. They are want to have Jesus say that eternal life is forever but
eternal punishment is not -- even though Jesus used the same word, in the same breath, to describe
them both. It just doesn't fit.

Let's translate it the universalist way....

The universalists are fond of translating Bible verses and transliterating a particular word. So, I
will use their style in the following translation:

And these will go away into aionion punishment, but the righteous into aionion life. 110

Or, to take a little liberty, it could be translated as,

And these will go away into non "aionion" punishment, but the righteous into "aionion" life."

I inserted the word "non" here to reflect what the universalists intend the word "aionion" to mean
when describing punishment. But notice, it isn't there when describing life because the Universalist
believes that the life of the righteous is without end: eternal. This is the kind of thing the universalist
must do in order to justify his position. It is clearly false and demonstrates an intrusion into the text
of a theological perspective. This is something Jehovah's Witnesses do when they "translated" the
Bible. They changed words to make them agree with their theology.
Nevertheless, another translation according to universalist presuppositions might be:

"And these will go away into non-eternal punishment, but the righteous into eternal life."

110
As a comment, with this type of translation, it is easy to confuse what the text is really saying because the
reader is not familiar with the Greek word "aionion." The Universalists often do this: partially translate a verse
leaving a transliterated Greek word or two in place of English words. They can then tell you what the word "really
means." This can be misleading.
But, the universalists state that "aionion" is an age, a period of time that can have a finish. They
would then answer this objection and say that punishment is for a time and so is life, but that both of
these are for an "aionion" period and after each period is another. In the case of the aionion
punishment, it would end and then after that, they would have eternal life. Likewise those possessing
eternal life already in the aionion "age" will continue to have it in the next age. The only problem is
that that isn't what the text is saying.
The universalists have constructed a multi-age scenario to fit their perspective. In so doing, they
have allowed for salvation after death, another teaching that is unbiblical. Heb. 9:27 says, "And
inasmuch as it is appointed for men to die once and after this comes judgment," (NASB). The
judgment comes from God and is upon the sinner. The universalist would have some sort of a
judgment that leads to punishment that ends and then eternal salvation in the afterlife. In so
teaching, they have ignored the translations of countless scholars and adopted those interpretations
that agree with them in order to suit their theological bias.
I hope you can see the inconsistency of translating and interpreting Matt. 25:46 any other way
than stating that the punishment is eternal as is the life of the righteous.
Mark 3:28-29 and Universalism
The universalist states that there is no unforgivable sin because all people who have ever lived will
ultimately be reconciled to God; in other words, all sins from all people who have ever lived will be
forgiven. However, if there were a sin that will never be forgiven, then Universalism would be proven
wrong.
Mark 3:28-29 are important verses in showing that there is an unforgivable sin.

"Truly I say to you, all sins shall be forgiven the sons of men, and whatever blasphemies they
utter; 29 but whoever blasphemes against the Holy Spirit never has forgiveness, but is guilty of
an eternal sin," (NASB).

The Greek Interlinear of Mark 3:28-29 is as follows:

Verses 28 and 29 are in contrast to each other. Verse 28 says that all sins shall be forgiven.
Verse 29 clarifies the statement and flatly says that there is a sin that "never has forgiveness, but is
guilty of an eternal sin." This sin is Blasphemy of the Holy Spirit -- which is stating that Jesus did His
miracles by the power of the devil.
Verse 29 has the contrasting preposition "but", Greek "de." The use of the word "but" is showing
that there is a contrast, or an exception to the previous statement. All sins are forgivable, but
blasphemy of the Holy Spirit is not. That is why the word "but" is there, to show that there is a
qualification, an exception to the first statement.
So, how do the universalists answer this verse? They do it in two ways. First, they say that the
word "aiona", "age" can mean a period of time that ends. Now, this is true sometimes and not others.
There are verses that use the word "aiona" that can refer to non-eternal duration (John 8:35; Luke
1:55) and there are verses that use it and mean eternal duration (Matt. 21:19; Mark 11:14; John
4:14; 6:51; 6:58, etc.). It is the context that determines the meaning of the word. But it is big
mistake to think that "aiona" always means a finite time.
It is a conjecture on the Universalists part that the word "age", in Mark 3:29, means a definite
period of time that will end. But that isn't the case. As I said, Jesus is contrasting the second
statement (unforgivable sin) with the first statement (forgivable sin).
Second, the Universalist will go to Matt. 12:32 which says,

“And whoever shall speak a word against the Son of Man, it shall be forgiven him; but whoever
shall speak against the Holy Spirit, it shall not be forgiven him, either in this age, or in the age
to come."

They state that "the age to come" is the 1000 year reign of Christ which will end. Therefore,
blasphemy of the Holy Spirit will be forgiven after the 1000 year reign. They then state that Mark
3:29 must be interpreted in light of Matt. 12:32.
Granted, we must look at all the verses on a subject in order to get an accurate understanding
what is said. But, the logic of the Universalist is wrong. Here is why.

1. Mark 3:29 states that blasphemy of the Holy Spirit (BHS) will never be forgiven.
A. This verse clearly states the impossibility of forgiveness of this sin.
2. Matt 12:32 states that blasphemy of the Holy Spirit will not be forgiven "in this age or the
age to come."
A. If Matt 12:32 is interpreted to mean that BHS will be forgiven, then that contradicts
Mark 3:29 which states it will not be forgiven.
B. If Matt 12:32 is interpreted to mean that BHS will not be forgiven, then it does not
contradict Mark 3:29.
3. Therefore, the only way to harmonize both verses is to say that Blasphemy of the Holy Spirit
is unforgivable.
4. If Blasphemy of the Holy Spirit is unforgivable, then Universalism is wrong.

A comparison of translations

Instead of trying to define the Greek text ad nauseum explaining how and why words are
translated, I've simply supplied ten Bible translations along with a commonly used Universalist
translation of these two verses.

Bible
Mark 3:28-29
Version
"Truly I say to you, all sins shall be forgiven the sons of men, and whatever
NASB blasphemies they utter; 29 but whoever blasphemes against the Holy Spirit never has
forgiveness, but is guilty of an eternal sin"
I tell you the truth, all the sins and blasphemies of men will be forgiven them. 29 But
NIV whoever blasphemes against the Holy Spirit will never be forgiven; he is guilty of an
eternal sin."

"Assuredly, I say to you, all sins will be forgiven the sons of men, and whatever
NKJV blasphemies they may utter; 29"but he who blasphemes against the Holy Spirit never
has forgiveness, but is subject to eternal condemnation"

Verily I say unto you, All sins shall be forgiven unto the sons of men, and blasphemies
KJV wherewith soever they shall blaspheme: 29But he that shall blaspheme against the Holy
Ghost hath never forgiveness, but is in danger of eternal damnation:

"Truly, I say to you, all sins will be forgiven the sons of men, and whatever blasphemies
RSV they utter; 29 but whoever blasphemes against the Holy Spirit never has forgiveness, but
is guilty of an eternal sin"

Verily I say unto you, All their sins shall be forgiven unto the sons of men, and their
1901 ASV blasphemies wherewith soever they shall blaspheme: 29but whosoever shall blaspheme
against the Holy Spirit hath never forgiveness, but is guilty of an eternal sin:

"Believe me, all men's sins can be forgiven and all their blasphemies. But there can
Phillips
never be any forgiveness for blasphemy against the Holy Spirit. That is an eternal sin."

Verily I say unto you, that all sins shall be forgiven to the sons of men, and all the
injurious speeches [with] which they may speak injuriously; 29 but whosoever shall
Darby
speak injuriously against the Holy Spirit, to eternity has no forgiveness; but lies under
the guilt of an everlasting sin;
`I tell you the truth. All wrong things that people do and say about anyone will be
BWE forgiven. 29 But people who say wrong things against the Holy Spirit will never be
forgiven. A person who does that will be punished for ever.'

"In solemn truth I tell you that all their sins may be pardoned to the sons of men, and
all their blasphemies, however they may have blasphemed; 29but whoever blasphemes
Weymouth
against the Holy Spirit, he remains for ever unabsolved: he is guilty of a sin of the
Ages."
28 "Verily, I am saying to you that all shall be pardoned the sons of mankind, the
Concordant
penalties of the sins and the blasphemies, whatsoever they should be blaspheming, 29
(Universalist
yet whoever should be blaspheming against the holy spirit is having no pardon for the
translation)
eon, but is liable to the eonian penalty for the sin" --

First of all, the ten translations above, all done by very reputable scholars, all say the same thing:
Blasphemy of the Holy Spirit will not be forgiven -- ever!
If you notice, the Concordant version (done by Universalists) did not translate the Greek words
"eon" (age) and "eonian" (eternal) into English, but left them transliterated. In fact, the word in Greek
is not "eon" but "aiona."
All other words are in plain English accept for these two. Why? I believe it is because they wanted
to influence the way the text sounds and is interpreted. By not translating the words, and by telling
you that the word "eon" only means a duration of time with an ending, then, the universalists can get
you to accept the idea that Blasphemy of the Holy Spirit is forgivable and that their theology is
correct. There is just one problem. It isn't.
1 Tim. 4:10 and universalism

• "For it is for this we labor and strive, because we have fixed our hope on the
living God, who is the Savior of all men, especially of believers" (NASB).
• "(and for this we labor and strive), that we have put our hope in the living God,
who is the Savior of all men, and especially of those who believe" (NIV).
• "For therefore we both labour and suffer reproach, because we trust in the living
God, who is the Saviour of all men, especially of those that believe" (KJV).

Much is made out of 1 Tim. 4:10 by the Universalist to claim that Jesus will redeem all people
whether or not they accept or reject Christ as Savior here on earth. Eventually, they say, all people
will repent (either here or in the after-life) and come to a saving relationship with God. 1 Tim. 4:10 is
used as proof. Unfortunately, the verse does not prove what the Universalists hope it does. Can God
be called the Savior of all men and yet not redeem all? Yes.
All people are, by nature, born under wrath (Eph. 2:3) and should go to hell. Why? Because God
is holy and we are sinners. Yet, we have hope in Christ. The Christian is saved by faith (Eph. 2:8)
and will join the Lord in heaven. But, the unbeliever is under judgment. John 3:18 says, "He that
believeth on him is not condemned: but he that believeth not is condemned already, because he hath
not believed in the name of the only begotten Son of God" (KJV). Why does God not simply destroy
them as is His right? Because of the Christians! Because God is being patient with the unbeliever,
allowing them to enjoy the blessings of life in this world without the rightful condemnation of God
falling upon them. This is what the Bible states:

"What if God, although willing to demonstrate His wrath and to make His power known,
endured with much patience vessels of wrath prepared for destruction? 23 And He did so in
order that He might make known the riches of His glory upon vessels of mercy, which He
prepared beforehand for glory," (Rom. 9:22-23, NASB).

As you can see, God is patient with the unregenerate. They receive a delayed judgment because of
God's love for the believer. In this sense, Jesus is the Savior of the world because He holds back His
judging hand from all who rightly and immediately deserve it. Judgment is delayed. This is a blessing
received from God upon the unbeliever. In fact, God often blesses the unbeliever because of the
presence of a believer.

"And it came about that from the time he made him overseer in his house, and over all that he
owned, the Lord blessed the Egyptian’s house on account of Joseph; thus the Lord’s blessing
was upon all that he owned, in the house and in the field," (Gen. 39:5, NASB).

Consider also Matt. 13:24-30 and the parable of the wheat and the tares. In it Jesus compares the
world to a field. He later interprets it by stating that "the good seed, these are the sons of the
kingdom; and the tares are the sons of the evil one," (Matt. 13:38). But in Matt. 13:20-30 Jesus
states that the tares are not dealt with right away because the wheat is there among them. "But he
*said, ‘No; lest while you are gathering up the tares, you may root up the wheat with them. 30‘Allow
both to grow together until the harvest," (NASB).
So, can it be said that the tares were saved from judgment? Yes...temporarily. The unbeliever
enjoys a delayed judgment. But with the Christian, Jesus is especially their Savior and judgment is
permanently removed from them.
All are made savable by Jesus' sacrifice

Another way in which Jesus is the savior of all men is that He has made all people savable.
Without Jesus' sacrifice, none could ever be saved. Since Jesus, who is the word made flesh (John
1:1,14), atoned for sin, all people are now redeemable. He is the Savior of all, but especially of
believers. That is, all are now redeemable due to the sacrifice of Christ, but redemption is specifically
applied to those who trust in Christ.

Is God the Savior?

“For it is for this we labor and strive, because we have fixed our hope on the living God, who is
the Savior of all men, especially of believers,” (1 Tim. 4:10, NASB).

1 Tim. 4:10 is referring to God in particular and not necessarily Jesus in particular. Does the title
"God" include Jesus? Of course, since Jesus is God in flesh (Col. 2:9), the Savior. God, who is a
Trinity, is called Savior in Psalm 106:21; Isaiah 43:3; Luke 1:47; 1 Tim. 1:1; 2:3; Titus 1:3-4; and
Titus 2:10. It is obvious that the term refers to God in the generic sense of being the Savior of all
men since He brings salvation to all though it is not accepted by all. This is why it says that God (not
Jesus) is the Savior of all men, especially of believers. How is it especially to believers? Simple. It is
especially and specifically realized only by those who are believers.
Furthermore, only Jesus is the mediator between God and men (1 Tim. 2:5) and He mediates only
between the saved and God. He does not mediate His atoning work for the unredeemed. His being
Savior is generic for all, but specific for the saved.
Therefore, this verse does not necessitate that all will be redeemed.
1 Tim. 2:4 and 2 Pet. 3:9. Is it God's will that all people be saved?

• "This is good and acceptable in the sight of God our Savior, 4who desires all men to be
saved and to come to the knowledge of the truth," (1 Tim. 2:3-4).
• "The Lord is not slow about His promise, as some count slowness, but is patient toward
you, not wishing for any to perish but for all to come to repentance," (2 Pet. 3:9).

To begin with, the answer to the question "Is it God's will that all people be saved?" must be "Yes,"
because that it what the Bible says. But does that mean that all will be saved? The universalists
believe so. They will appeal to 1 Tim. 2:3-4 and 2 Pet. 3:9 combined with other verses where God
says He will accomplish what He desires. They then say that since God says He will accomplish all His
desires and He desires all to be saved, then all will be saved.

• ". . . and I [God] will accomplish all My good pleasure," (Isaiah 46:10).
• "But our God is in the heavens; He does whatever He pleases," (Psalm 115:3).
• "Whatever the Lord pleases, He does," (Psalm 135:6).

This is a straight-forward approach, but it is also very simplistic and incomplete because it does not
take into account all of scripture relating to the subject of God's desire. Instead, the Universalists
"proof-text" their way into a foregone conclusion by picking and choosing certain scriptures and
combining them to form an implied conclusion. This is how error is born and it is not good theology.
Instead, the proper thing to do is to look at all of scripture on a topic and draw conclusions from the
whole, not the part. I will gather a broader scope of scripture dealing with this subject and attempt to
show that God's desire is not always accomplished and, therefore, the claim of the universalist that the
above verses prove that all will be saved, is in error.
To begin with, are God's desires always accomplished? No, they are not. God's desire is that
people do what is right and not sin: "To do righteousness and justice is desired by the Lord rather
than sacrifice," (Prov. 21:3). But people still sin in spite of God's stated desire. Was it the desire of
God that Adam and Eve rebel? No. Was it God's will that David commit adultery? No. Yet, they did
the very thing God did not want. God desires that all people repent (Acts 17:30); but not all do.
Clearly, God's will is not always done.
In theology, when examining this issue of God's will and His allowance of sin, we distinguish
between what is called God's perfect will and His permissive will. In His perfect will, He desires that all
refrain from sin. But in His permissive will, He allows sin to exist. In this sense, He has two wills
regarding sin. He desires that sin not exist because it is contrary to His nature, yet He wills that it
does by making provision for it in His sovereign plan. This does not mean that God brought sin into
existence. It means that He simply permitted it by allowing the fall. He then uses it, and other sins,
for His glory and purpose. Please recall the account of Joseph's brothers who sinned by selling him
into slavery and then lying to their father about it. After many years when the family was reunited,
Joseph said, "And as for you, you meant evil against me, but God meant it for good in order to bring
about this present result, to preserve many people alive" (Gen. 50:20). God meant it for good? How
could that be if God is only passively allowing things to occur? Here, Joseph states that God had a
purpose in their sin. Though God does not want sin, He made provision for it in His divine plan.
Consider also how evil people conspired against Jesus to bring Him to death. Was this God's plan that
they do this?

"For truly in this city there were gathered together against Thy holy servant Jesus, whom Thou
didst anoint, both Herod and Pontius Pilate, along with the Gentiles and the peoples of Israel, 28
to do whatever Thy hand and Thy purpose predestined to occur," (Acts 4:27-28).

Do you see how God predestined Herod and Pontius Pilate to carry out His will? Didn't they sin in
condemning Jesus? Yes! Did God predestine them to do what He planned? Yes! Did God make them
sin? No, for God does not tempt anyone (James 1:13). Yet, God, in His sovereignty predestined them
to do what they did.
God is in control of history and it goes where He directs it. Though He does not desire that people
sin, He makes room for it. Therefore, we can plainly see that God can desire one thing and even
ordain another by giving it a place in His sovereign plan.
So, how can anyone assert that based on 1 Tim. 2:3-4 and 2 Pet. 3:9 combined with Isaiah 46:10;
Psalm 115:3, and Psalm 135:6 above, that all will be saved because that is God's desire and God's will
is always done? They cannot. God can desire all be saved, but not ordain that all are by making
provision in His plan for their damnation: "The Lord has made everything for its own purpose, even
the wicked for the day of evil," (Prov. 16:4).
Again, simply because God states that He desires all people to be saved does not mean that all will
be saved. This is particularly important when we notice that God elects people (Matt. 24:24,31; Mark
13:20; Rom. 8:33), predestines them (Rom. 8:29-30; Eph. 1:1-11), appoints them to eternal life
(Acts 13:48), and grants that they believe (Phil. 1:29). We must ask why doesn't God elect all,
predestine all, appoint all, or grant that all believe when He has the power to do so? Is it because God
is incapable of carrying out His will? Or is there something greater than God at work? Of course not.
God is in absolute control. He can desire one thing (that people not sin), yet ordain another (plan that
sin exist in the world). Likewise, God can desire that all be saved, yet not ordain that they are.

What Does God Want?

Some will object to the claim that God sometimes wants one thing and yet does another. They
would assert that this would be a contradiction. But it is not since God has obviously done this. Is it a
contradiction when a judge wants to show mercy to all people but orders that criminals be punished?
Is it a contradiction when the judge says to a murderer, "You shall not murder!" but, according to the
law, sentences him to death? No. Though the desire and action be different, there is no contradiction
at all. The judge is under obligation to keep the Law. . . and so is God. He must remain true to His
revealed Law which is a reflection of His divine character. After all, God is just and must punish sin.
To further illustrate the point that God can ordain something different than what He desires, please
consider the scriptures below.

What God Desires What God Arranges


• "This is good and acceptable in the • "just as it is written, 'God gave them
sight of God our Savior, 4who a spirit of stupor, eyes to see not
desires all men to be saved and and ears to hear not, down to this
to come to the knowledge of the very day,'" (Rom. 11:8).
truth," (1 Tim. 2:3-4).
• "The Lord is not slow about His • "And He was saying to them, "To you
promise, as some count slowness, has been given the mystery of the
but is patient toward you, not kingdom of God; but those who are
wishing for any to perish but outside get everything in parables,
for all to come to repentance," 12
in order that while seeing, they
(2 Pet. 3:9). may see and not perceive; and
while hearing, they may hear and
not understand lest they return
and be forgiven," (Mark 4:11-12).
We can see that God says He does not wish any to perish. But, we can also see that God gave
Israel eyes to not see with and ears to not hear with. Likewise, Jesus, who is God in flesh, purposely
spoke to people in parables so they would not perceive and repent (Mark 4:11-12). If God wants all
saved, why would He arrange it so people were blinded and prevented from seeing? Some will say
that the people did this to themselves. But that is not what the text says. Clearly, God is the one
performing the actions in preventing them to see. In this case, He desires one thing and does
another.
Does God want pain and suffering in the world? The obvious answer is no. God created Adam and
Eve and put them in a perfect world without pain, without suffering, and without loss. That is God's
desire because that is the way God made things in the beginning. Yet, we have pain, suffering, and
loss in the world. Why? Because that is the nature of our sinful system -- and God permitted it. But
are we to say that God is not in control? Again, no. In fact, God causes some of the very things we
believe He does not want.

We can see that God says He does not wish any to perish. But, we can also see that God gave
Israel eyes to not see with and ears to not hear with. Likewise, Jesus, who is God in flesh, purposely
spoke to people in parables so they would not perceive and repent (Mark 4:11-12). If God wants all
saved, why would He arrange it so people were blinded and prevented from seeing? Some will say
that the people did this to themselves. But that is not what the text says. Clearly, God is the one
performing the actions in preventing them to see. In this case, He desires one thing and does
another.
Does God want pain and suffering in the world? The obvious answer is no. God created Adam and
Eve and put them in a perfect world without pain, without suffering, and without loss. That is God's
desire because that is the way God made things in the beginning. Yet, we have pain, suffering, and
loss in the world. Why? Because that is the nature of our sinful system -- and God permitted it. But
are we to say that God is not in control? Again, no. In fact, God causes some of the very things we
believe He does not want.
Objections Answered to the paper,
“Is it God's will that all people be saved?”

Someone posted a rebuttal to my paper. For the purpose of Clarification, I have color-coded this
paper to make it clearer. My original paper's quote is level “1.” The rebuttal is level “A.” My
response is the third level “i”. I've not responded to every objection because some of them weren't
that important and tended to miss the main point of the immediate context.
It is obvious that the person responded to my paper as he was reading instead of covering the
whole thing and then responding. This accounts for his comments which I covered in the paper later,
and sometimes immediately after his comments.
"[sic]" means the error in either syntax or spelling is original to the author.

1. To begin with, are God's desires always accomplished? No, they are not. God's desire is that
people do what is right and not sin: "To do righteousness and justice is desired by the Lord rather
than sacrifice," (Prov. 21:3).
A. "God would rather have people obey him out of their own love/will then to obey him out of
necessity as a type of "fire insurance". Are we of our own will able to save ourselves?"
i. I fail to see how the above comment has anything to do with the point I was making in
the paper at that time. It isn't a refutation. At best, it is only an opinion.
2. But people still sin in spite of God's stated desire.
A. "I suspect that Matt see's [sic] sin as the foremost problem in the world today, what if God
in fact uses it as the solution. In fact most fundamentalist Christians [sic] all they seem to
discuss is sin and not the solution. They discuss what Jesus did for us and then carry on
discussing how not to SIN."
i. I am completely amazed here. Of course God uses people's sin. I said in my paper, "It
means that He simply permitted it by allowing the fall. He then uses it, and other sins,
for His glory and purpose." Apparently, he didn't read it too clearly.
B. "How could God accept creating a species with whom he would consign 90+ percent to an
"everlasting" torture. Is that Good? To say that God's will is not always done is to take the
view that God could be "surprised" [sic] by the actions of his creatures."
i. Is this a refutation? Not at all. It is only a question. To ask it is to be guilty of a non-
answer. God can do what He wants. He can damn all if He chooses and be perfectly
just and right in so doing without changing His loving character one bit. It isn't a
context of who gets the most souls. Besides, God orders that people refrain from sin,
yet everyone sins. Does that make God a failure? Hardly. As I stated in my paper, it
is part of God's sovereign plan.
ii. "Enter by the narrow gate; for the gate is wide, and the way is broad that leads to
destruction, and many are those who enter by it. 14"For the gate is small, and the way is
narrow that leads to life, and few are those who find it," (Matt. 7:13-14). See also, Luke
13:22-27.
3. In theology, when examining this issue of God's will and His allowance of sin, we distinguish
between what is called God's perfect will and His permissive will. In His perfect will, He desires
that all refrain from sin. But in His permissive will, He allows sin to exist. In this sense, He has
two wills regarding sin. He desires that sin not exist because it is contrary to His nature, yet He
wills that it does by making provision for it in His sovereign plan. This does not mean that God
brought sin into existence. It means that He simply permitted it by allowing the fall.
A. "This of course depends on your definition of SIN. Matt as well as most ET's make SIN sound
like an object. A sticky, ooze, tar like substance not particularly well defined, somewhat dark
but basically bad (stay away from it)."
i. This is ludicrous. This "refutation" is far from substantial. Sin is breaking God's law and
if this person had read my definition of it on my site, he would not have stated such an
absurdity.
4. Do you see how God predestined Herod and Pontius Pilate to carry out His will? Didn't they sin in
condemning Jesus? Yes! Did God predestine them to do what He planned? Yes! Did God make
them sin? No, for God does not tempt anyone (James 1:13). Yet, God, in His sovereignty
predestined them to do what they did.
A. "If God predestined them, and his will was for Jesus to be crucified, did Herod and Pilate
really truly sin. Did they truly fall short of God's expectations in this situation? Did Judas
truly fall short of God's desire for him, Jesus told Judas to turn him over to the authorities.
He could have convinced him otherwise or said nothing at all. God does not tempt anyone?
Did he not tempt Judas? Did God not tempt Adam & Eve, he allowed Satan in the garden,
what is the natural conclusion to those conditions. God himself may not tempt, but he allows
the tempting to occur."
i. The obvious answer is yes, they really sinned. Again, this response isn't a refutation of
anything. It has no value with no point to it. Again, God allows sin to exist in the world
as part of His permissive will in His sovereign plan.
5. Again, simply because God states that He desires all people to be saved does not mean that all
will be saved.
A. "Likewise it doesn't automatically mean that they won't all be saved."
i. It is nice to see that this person admitted one of the points of my paper.
6. This is particularly important when we notice that God elects people (Matt. 24:24,31; Mark
13:20; Rom. 8:33), predestines them (Rom. 8:29-30; Eph. 1:1-11), appoints them to eternal life
(Acts 13:48), and grants that they believe (Phil. 1:29).
A. "With regards to the people that were not "predestined", "appointed", "elected" etc: what
gave God the right to choose one for eternal life and another for eternal damnation. You say
he decided, presumably before they were made, where they would spend eternity. What
function or purpose do those that God not predestine serve?"
i. "What gave God the right...?" This is an unfortunate question. God, by His nature of
being God, has the right to do whatever He wants. Who is this person to question God?
"On the contrary, who are you, O man, who answers back to God? The thing molded will
not say to the molder, "Why did you make me like this," will it?'" (Rom. 9:20). God
does this after the counsel of His own will (Eph. 1:1-11) which I so stated in my paper
and which, it seems again, was not sufficiently read by this person.
7. We can see that God says He does not wish any to perish. But, we can also see that God gave
Israel eyes to not see with and ears to not hear with. Likewise, Jesus, who is God in flesh,
purposely spoke to people in parables so they would not perceive and repent (Mark 4:11-12). If
God wants all saved, why would He arrange it so people were blinded and prevented from seeing?
Some will say that the people did this to themselves. But that is not what the text says. Clearly,
God is the one performing the actions in preventing them to see. In this case, He desires one
thing and does another.
A. "The parables were spoken so that those of a religious and rigid mind would not understand.
Jesus wanted the lay person to understand but not the religious. Matt 11:25"
i. Apparently, this critic of my paper did not read it carefully at all. I already answered
this objection in the paper. Mark 4:11-12 says, "And He was saying to them, "To you
has been given the mystery of the kingdom of God; but those who are outside get
everything in parables, 12in order that while seeing, they may see and not
perceive; and while hearing, they may hear and not understand lest they return
and be forgiven," (Mark 4:11-12). As you can see, it was not to strengthen their
blindness. Jesus actually stated that He spoke in parables in order that they might
not see. Instead of addressing this verse, the objector quotes Matt. 11:25 which says,
"At that time Jesus answered and said, "I praise Thee, O Father, Lord of heaven and
earth, that Thou didst hide these things from the wise and intelligent and didst reveal
them to babes." What confuses me in this person's response is why he would quote a
verse that further proves my point. As you can see above, the Father is the one hiding
the truth from the "wise" on earth... etc. Again, I fail to see any point of refutation of
my paper here.
8. Does God want pain and suffering in the world? The obvious answer is no. God created Adam and
Eve and put them in a perfect world without pain, without suffering, and without loss. That is
God's desire because that is the way God made things in the beginning. Yet, we have pain,
suffering, and loss in the world. Why? Because that is the nature of our sinful system -- and God
permitted it. But are we to say that God is not in control? Again, no. In fact, God causes some of
the very things we believe He does not want.
A. "But again what is the purpose of this "SINFUL SYSTEM". Is it some cruel, sick joke being
played on the human race. If people experience varying amound [sic] of pain, suffering and
loss in this world why does God need to eternally punish. We have people who are in a literal
hell right now on Earth. God knew the results of his actions before he did them, so again
why did he do it the way he did. Perhaps you misunderstand what it is that God wants."
i. What is this except more questions without answers? This is not how one refutes a
paper. If I have misunderstood what God wants, then this person should offer
something as a correction. But, he does not...again.
9. Why would God compel people to come into His house so that it can be filled and yet send a
deluding influence upon the same people? Are not those people in 2 Thessalonians, at the time of
the Antichrist, the same people included in the highways and hedges? Does not God compel all to
enter into His house regardless of when and where they are in history?
A. "Lets examine this: does God compel all to be drawn to himself? Not if you accept many
modern church viewpoints. People in faraway lands who have never heard of God apparently
are consigned to hell without any chance of redemption. They would have you believe that
he doesn't care or is unable to save. After all the "holy mother church" hasn't sent any
representatives."
i. This is another subject altogether and needs to be dealt with elsewhere. Nevertheless,
whatever God does is holy and right. They will be judged according to Rom. 2:11-16.
Again, this isn't a refutation at all.
10. God did not cause the people who crucified Jesus to sin. But, He sure used their sin and He
predestined all of it to occur. He used the sins of Herod and Pilate along with the Gentiles to do
His divine will. In fact, God anointed them to do what they did. Why? To carry out His purpose
and His plan to bring His Son to the cross, to save sinners, and to bring glory to Himself.
A. "Again you discuss God's plan without revealing his overall divine purpose. You make it
sound as though the lay person is incapable of even beginning to understand the overall
design. Us uni's simply believe that he has already saved all us sinners. Most people
however still lack that understanding."
i. I did reveal what I believed was the overall purpose in God's plan. I stated in my
paper, "What is that plan? I believe God was arranging history to lead to the ultimate
goal of Christ's crucifixion, resurrection, and return, along with the redemption of God's
people." This objector has demonstrated yet again that he did not thoroughly read the
paper. As far as the rest of the statement from him, he is only begging the question by
assuming the very thing under discussion as a proof for his point.
11. God can and does control people's hearts and actions so that they will accomplish His purpose. He
does this sovereignty and He does it without causing people to sin. He can even make someone's
heart hard for the purpose of carrying out His plan.
A. "Once again are interests are barely conflicting, you believe that God desires to save all
men, but is either unable or unwilling to save all. We believe that God desires to save all
men and is able and willing to acheive [sic] that goal."
i. Incorrect, I fully believe God is able to achieve whatever goal He sets up. He has
devised His plan and it will be carried out. Again, nothing here that refutes the paper.
12. "And the ten horns which you saw, and the beast, these will hate the harlot and will make her
desolate and naked, and will eat her flesh and will burn her up with fire. 17 "For God has put it in
their hearts to execute His purpose by having a common purpose, and by giving their kingdom to
the beast, until the words of God should be fulfilled. -- as quoted in my paper from Rev. 17:16-
17.
A. "You assume that God who did not show mercy, will never show mercy toward those he
destroys/chastens. That he will be eternally vengeful. You assume they are eternally lost.
The scriptures about the harlot above show that though he will try her, he will not punish
forever but "until the words of God should be fulfilled." Are we not all God's people? Or are
only those that say the sinners prayer "of God". And if so then how does anyone who was
not "predestined" get saved? And since it is not "of works lest anyone should boast" we can't
even claim to be saved by saying the sinners prayer since that would be a "work"."
i. I assume no such thing. God indeed shows mercy to people He chastens. I am a good
example. This person's opinion of what the verse means is only his opinion and nothing
more. Most Christian commentators understand and accept the eternal nature of
damnation.
13. In the above verses you can see that God takes no pleasure in the death of the wicked yet he
delighted in destroying the wicked people. Is this a contradiction? No. Because God has a purpose
and a plan and He has the sovereign right to accomplish His will. On one hand, He doesn't want
people to suffer and die, yet on the other hand, He is delighted to carry out His divine plan which
necessarily includes the death of the wicked because it is in accordance with the Law which He
Himself has given us. His divine plan will be accomplished. This is further proof that God can
desire one thing and bring another to pass.
A. "And you believe that somehow these men are hindering God's will, and therefore have to be
bulldozed to make way for the "divine plan"."
i. Again, I believe no such thing and did not say that at all in my paper. In fact, I clearly
stated that all of what they did was in God's sovereign plan. It seems this person has
not read my paper very clearly and is trying to read into it what I did not say. He is
constructing straw-men to shoot down.
14. Though God hates sin, He permits it for the greater glory of Himself and His plan. In this, He
desires all to be saved, but has not ordained that it be so because, according to the Law, He must
punish sinners.
A. "The reason "God permits sin" is because it is the cornerstone of his plan, his plan for
redemption of all. God hates sin only in that it seperates [sic] himself from us."
i. No, the cornerstone of God's plan is Jesus' crucifixion, not sin. To say that God hates sin
only because it separates Him from us is incorrect. Sin is an affront to God's very
character and nature. It is breaking His law which He gave and which is a reflection of
His divine character. This person's theology is too man-centered for me. It stresses
God needing or wanting us. Mine, on the other hand, stresses the glory of God's
righteousness and the fact that sin is an offense to His holiness and righteousness.
15. God has worked all things after the counsel of His will (Eph. 1:5) and has engineered history to
bring not only the cross as the means of redemption, but also the culmination of all things for the
declaration of His own glory, righteousness, holiness, and character. Sin will be shown to be
utterly sinful and horrible. The cross demonstrates His righteousness and grace and His sovereign
will is carried out.
A. "Here we have your horrible icky "SIN" thing. To Sin is nothing less then to fall short of
God's expectations. It is no more horrible then [sic] a child falling short of it's parents
expectations. If a person commits a heinous act during his life against another is he not
usually punished by the secular world. Is it then just and righteous for God to punish him
further. Are we not all God's children? Apparently you think otherwise."
i. Again, a failure to understand what sin really is. It isn't "nothing less then [sic] falling
short of God's expectations." It is an affront to God Himself. Sin is rebellion against
God, violation of His law and character. It brings death and wrath. When this person
demonstrates an inability to rightly understand sin, he demonstrates the tendency for
further error as is demonstrated in his next response.
16. Do 1 Tim. 2:3-4 and 2 Pet. 3:9 prove that all will be saved? No, not at all. But they do show us
that God is not simplistic and that He has a divine plan that we must truly seek to fathom if we
are to rightly understand His word.
A. "Apparently only a few such as yourself are able to fathom the divine plan of God. I
personally believe that which is simple, is usually the truth. Religious/buerecratic [sic] man
complicates and adds layers upon layers of half truths upon virtually every thing that is good
and right and true."
i. Such mockery is not how a refutation is done. To say that that which is the simplest is
the truth is to oversimplify the issue. There are difficult things in scripture and being
simple about them will only lead to error.
More objections answered to “Is it God's will that all people be saved?”

The objections are listed in primary numbers and my responses are under them. I will add more as
more are stated.

1. We agree that God says that He will do *all* that He desires. We also agree that in scripture men
do things that God does not desire them to do - sin. So are there two desires of God - one
decretive and one permissive? And which type is 1 Tim. 2:4?
A. I would say that it is neither. Rather, God is stating a desire that people be saved, but not
decreeing that they are. If He did decree that all would be saved, then all would be saved.
But since God elects people (Matt. 24:24,31; Mark 13:20; Rom. 8:33), predestines them
(Rom. 8:29-30; Eph. 1:1-11), appoints them to eternal life (Acts 13:48), and grants that
they believe (Phil. 1:29), if 1 Tim. 2:4 were decretive, then He would have to predestine all,
elect all, choose all, and grant that all believe which is something not stated in scripture. On
the contrary, we have statements in scripture like:
i. "And this is the will of Him who sent Me, that of all that He has given Me I lose nothing,
but raise it up on the last day," (John 6:39). Are all given to Jesus by the Father? No.
"I ask on their behalf; I do not ask on behalf of the world, but of those whom
Thou hast given Me; for they are Thine," (John 17:9).
ii. "Enter by the narrow gate; for the gate is wide, and the way is broad that leads to
destruction, and many are those who enter by it. 14"For the gate is small, and the way
is narrow that leads to life, and few are those who find it," (Matt. 7:13-14).
iii. "For many are called, but few are chosen," (Matt. 22:14).
iv. "And He was passing through from one city and village to another, teaching, and
proceeding on His way to Jerusalem. 23And someone said to Him, "Lord, are there just
a few who are being saved?" And He said to them, 24"Strive to enter by the narrow
door; for many, I tell you, will seek to enter and will not be able. 25"Once the head of the
house gets up and shuts the door, and you begin to stand outside and knock on the
door, saying, ‘Lord, open up to us!’ then He will answer and say to you, ‘I do not know
where you are from.’ 26"Then you will begin to say, ‘We ate and drank in Your presence,
and You taught in our streets’; 27and He will say, ‘I tell you, I do not know where you
are from; depart from Me, all you evildoers,'" (Luke 13:22-27).
v. "And Isaiah cries out concerning Israel, "Though the number of the sons of Israel be as
the sand of the sea, it is the remnant that will be saved; 28for the Lord will execute
His word upon the earth, thoroughly and quickly," (Rom. 9:27).

We see no scriptures where God predestines all, elects all, grants that all believe.
A look at the word "aionion"

Universalism is the teaching that God will ultimately bring all people, in all times, and all places to a
state of reconciliation with Him. In other words, everyone who ever lived will be saved.
Consequently, universalism cannot allow the possibility of an eternal hell as a realistic biblical
teaching.
To get around the problem of the English Bibles translating Greek words into "eternal," "forever,"
and forevermore" when describing fire (Matt. 18:8) or torment (Rev. 20:10), the universalists go to
the Greek. The Greek word that is translated into eternal is "aionion." It comes from the Greek root
"aion" meaning "age." This fact combined with the various uses of Greek words derived from the root
"aion," are what the universalists use to attempt to show that "aionion" does not always mean
"eternal" but can refer to a finite period of time.
The truth is, they are right. It can be translated into a temporal sense as it is in Rom. 16:25:
"Now to Him who is able to establish you according to my gospel and the preaching of Jesus Christ,
according to the revelation of the mystery which has been kept secret for long ages (aionios 1) past."
But the reason it is translated that way is because of context, and that is extremely important.
Context determines meaning, as you will see later.
With the claim that "aionion" can be translated into something temporal and that its root means
"age," the universalist then says that any reference to "eternal fire," "eternal torment," or "eternal
punishment" is not really eternal. Instead of "eternal torment," it is "aionion torment." Instead of
"eternal punishment," it is "aionion punishment." That way, to the universalist, there is no eternal
hell, no eternal punishment, and no eternal damnation. Everyone will be saved.
This approach by the Universalists can be confusing to someone who doesn't understand Greek and
that is part of the reason that Universalism has followers. It is true that the root "aion" means age.
But just because a root means age, does not mean that every word derived from that root means a
limited duration of time. For example consider this verse that is speaking about God:

“who alone possesses immortality and dwells in unapproachable light; whom no man has seen
or can see. To Him be honor and eternal dominion! Amen,” (1 Tim. 6:16)

The context is obviously dealing with God's eternal nature. The word in Greek for "immortality" is
"athanatos." The Greek word for death is "thanatos." The "a" in front of the word is the negator,
without, non, etc. It means that God is deathless; hence, immortal. This is an eternal quality of God.
Likewise, the verse states that God has eternal dominion. The word for "eternal" is "aionios" which is
derived from the Greek root "aion" which means age. But, God is not immortal for only an "age," nor
is His dominion temporal. The word "eternal" is absolutely the best way to translate the Greek
"aionion" because God is immortal and eternal. Therefore, it would be wrong to translate the verse by
stating that God has "aionion" dominion. Rather, He has eternal dominion.

How is "aionion" used in the New Testament?

The following two sections are verses that contain the word "aionion" which is translated as
"eternal." Notice how using the word "eternal" in the first group is no problem. But, it is the second
group with which the Universalists object. Nevertheless, the same word is used in both. See for
yourself.

1. John 6:47, "Truly, truly, I say to you, he who believes has eternal (aionion) life.
2. John 20:28, "and I give eternal (aionion) life to them, and they shall never perish; and no
one shall snatch them out of My hand."
3. Acts 13:48, "And when the Gentiles heard this, they began rejoicing and glorifying the word of
the Lord; and as many as had been appointed to eternal (aionion) life believed."
4. Romans 2:7, " to those who by perseverance in doing good seek for glory and honor and
immortality, eternal (aionion) life."
5. Romans 5:21, "that, as sin reigned in death, even so grace might reign through righteousness
to eternal (aionion) life through Jesus Christ our Lord."
6. Rom. 16:26, " but now is manifested, and by the Scriptures of the prophets, according to the
commandment of the eternal (aionion) God, has been made known to all the nations,
leading to obedience of faith."
7. Gal. 6:8, "For the one who sows to his own flesh shall from the flesh reap corruption, but the
one who sows to the Spirit shall from the Spirit reap eternal (aionion) life."
8. 1 Tim. 6:16, "who alone possesses immortality and dwells in unapproachable light; whom no
man has seen or can see. To Him be honor and eternal (aionion) dominion! Amen."
9. 1 John 1:2, "and the life was manifested, and we have seen and bear witness and proclaim to
you the eternal (aionion) life, which was with the Father and was manifested to us"
10. 1 John 5:11, "And the witness is this, that God has given us eternal (aionion) life, and this
life is in His Son."

The following set of scriptures reveals the nature of eternal damnation.

1. Matt. 18:8, "And if your hand or your foot causes you to stumble, cut it off and throw it from
you; it is better for you to enter life crippled or lame, than having two hands or two feet, to
be cast into the eternal (aionion) fire.
2. Matt. 25:41, "Then He will also say to those on His left, ‘Depart from Me, accursed ones, into
the eternal (aionion) fire which has been prepared for the devil and his angels;"
3. Matt. 25:46, "And these will go away into eternal (aionion) punishment, but the righteous
into eternal (aionion) life."
4. Mark 3:29, "but whoever blasphemes against the Holy Spirit never has forgiveness, but is
guilty of an eternal (aionion) sin."
5. Mark 10:30, "but that he shall receive a hundred times as much now in the present age,
houses and brothers and sisters and mothers and children and farms, along with
persecutions; and in the age to come, eternal (aionion) life.
6. Luke 18:30, "who shall not receive many times as much at this time and in the age to come,
eternal (aionion) life."
7. 2 Thess. 1:9, "And these will pay the penalty of eternal (aionion) destruction, away from the
presence of the Lord and from the glory of His power,"
8. Jude 7, "Just as Sodom and Gomorrah and the cities around them, since they in the same way
as these indulged in gross immorality and went after strange flesh, are exhibited as an
example, in undergoing the punishment of eternal (aionion) fire."

It should be quite obvious that there is an eternal punishment and that universalism is nothing
more than a hopeful wish. The Universalists are not justified in picking and choosing the meaning of a
word based upon their interpretations of "aion" that suits them and depending on which verse is
used.
What do Greek dictionaries say about ‘aionion’?

Universalists place a great deal of weight on the word "aion" which means "age." From the Greek
root "aion" we also have the word "aionion" which is translated in most instances in most Bibles as
"eternal."
The significance of this is that Universalists maintain that there is no eternal punishment in hell
fire. Therefore, they assert that the word "aionion" is in reference to "age duration" and can have
temporal duration. With this assertion they try to substantiate their theological position that all people
everywhere will ultimately be saved.
But, what do Greek Dictionaries and Lexicons have to say about the words and phrases used in
Greek that are translated into the English "age", "world", "eternal," "forever", "forever and ever,"
etc.? Let's find out.

1. aion - , - age, world


A. "for ever, an unbroken age, perpetuity of time, eternity; the worlds, universe; period of
time, age."
i. Enhanced Strong’s Lexicon, (Oak Harbor, WA: Logos Research Systems, Inc.)
1995, [Online] Available: Logos Library System.
2. aionion, aionios - - eternal
A. "aionios," the adjective corresponding, denoting eternal. It is used of that which in
nature is endless, as, e.g., of God, (Rom. 16:26), His power, (1 Tim. 6:16), His glory,
(1 Pet. 5:10), the Holy Spirit, (Heb. 9:14), redemption, (Heb. 9:12), salvation, (5:9),
life in Christ, (John 3:16), the resurrection body, (2 Cor. 5:1), the future rule of Christ,
(2 Pet. 1:11), which is declared to be without end, (Luke 1:33), of sin that never has
forgiveness, (Mark 3:29), the judgment of God, (Heb. 6:2), and of fire, one of its
instruments, (Matt. 18:8; 25:41; Jude 7)."
i. Rom. 16:26 - " . . .according to the commandment of the eternal God. . ."
ii. 1 Tim. 6:16 - ". . . To Him be honor and eternal dominion! Amen."
iii. 1 Pet. 5:10 - " . . . who called you to His eternal glory in Christ,"
iv. Mark 3:29 - " . . . never has forgiveness, but is guilty of an eternal sin."
v. etc.
a. SOURCE: Vine, W. E., Vine’s Expository Dictionary of Old and New Testament
Words, (Grand Rapids, MI: Fleming H. Revell) 1981, Available: Logos Library
System.
3. "describes duration, either undefined but not endless, as in Rom. 16:25; 2 Tim. 1:9; Tit.
1:2; or undefined because endless as in Rom. 16:26, and the other sixty–six places in the
N.T.
A. Rom. 16:25 - " . . which has been kept secret for long ages past,"
B. Rom 16:26 - ". . . according to the commandment of the eternal God,"
C. 2 Tim. 1:9 - ". . . which was granted us in Christ Jesus from all eternity,"
D. Titus 1:2 - "the hope of eternal life, which God, who cannot lie, promised" long ages
ago"
i. SOURCE: Vine, W. E., Vine’s Expository Dictionary of Old and New Testament
Words, (Grand Rapids, MI: Fleming H. Revell) 1981, [Online] Available: Logos
Library System)
4. Eis tous aionios ton aionion - - Forever and Ever, Lit.
"into the age of the ages"
A. "unlimited duration of time, with particular focus upon the future - ‘always, forever,
forever and ever, eternally."
B. Phil. 4:20 - ". . .to our God and Father be the glory forever and ever."
C. Rev. 19:3 - " . . .Her smoke rises up forever and ever."
D. Rev. 20:20 - "And the devil who deceived them was thrown into the lake of fire and
brimstone, where the beast and the false prophet are also; and they will be tormented
day and night forever and ever."
i. SOURCE: Louw, Johannes P. and Nida, Eugene A., Greek-English Lexicon of the
New Testament based on Semantic Domains, (New York: United Bible Societies)
1988, 1989, Available: Logos Library System.

These few references and quotes should be ample evidence that eternal hell, eternal fire, is real. It
is a terrible reality and it is all the more important to preach the gospel. The universalists are wrong
and their theology only dilutes the need to come to Christ.
Forever and Ever
Is the English phrase "forever and ever" a proper translation of the Greek? Does it mean without
end? Is it ever used of something not eternal? Does it refer to eternal torment? These questions are
important because the universalist position denies the eternality of hell fire. Universalists take the
literal Greek phrase of "eis tous aionas ton aionon, -- into the age of the ages" which is commonly
translated as "forever and ever," "forevermore," and state that it refers to an age of time, a finite
period of time.
It is true that the basic root of "aion" means age. But it is not true that all words derived from that
root mean a finite duration of time. The phrase means "unlimited duration of time, with particular
focus upon the future - ‘always, forever, forever and ever, eternally."’ 111 Additionally, the phrase is
used to describe both God's eternal attributes and His eternal nature as well as eternal torment.
Following is a table containing every single usage of the Greek phrase "eis tous aionas ton aionon --
into the age of the ages." It clearly shows that it means "forever," "without end." The first two verses
deal with eternal condemnation and judgment; the next 16 deal with God's Glory and honor.

"into the age of the ages"


Translated as "forever and ever"; "forevermore"

1. "And a second time they said, "Hallelujah! Her smoke rises up forever and ever," (Rev. 19:3).
2. "And the devil who deceived them was thrown into the lake of fire and brimstone, where the
beast and the false prophet are also; and they will be tormented day and night forever and
ever," (Rev. 20:10).
3. "to whom be the glory forevermore, Amen," (Gal. 1:2)
4. "Now to our God and Father be the glory forever and ever. Amen," (Phil. 4:20)
5. "Now to the King eternal, immortal, invisible, the only God, be honor and glory forever and
ever. Amen," (1 Tim. 1:17).
6. "The Lord will deliver me from every evil deed, and will bring me safely to His heavenly kingdom;
to Him be the glory forever and ever. Amen," (2 Tim. 4:18).
7. "equip you in every good thing to do His will, working in us that which is pleasing in His sight,
through Jesus Christ, to whom be the glory forever and ever. Amen," (Heb. 13:21).
8. "Whoever speaks, let him speak, as it were, the utterances of God; whoever serves, let him do so
as by the strength which God supplies; so that in all things God may be glorified through Jesus
Christ, to whom belongs the glory and dominion forever and ever. Amen," (1 Pet. 4:11).
9. "and He has made us to be a kingdom, priests to His God and Father; to Him be the glory and the
dominion forever and ever. Amen," (Rev. 1:6).
10. "and the living One; and I was dead, and behold, I am alive forevermore, and I have the keys
of death and of Hades," (Rev. 1:18).
11. "And when the living creatures give glory and honor and thanks to Him who sits on the throne, to
Him who lives forever and ever," (Rev. 4:9).
12. "the twenty-four elders will fall down before Him who sits on the throne, and will worship Him
who lives forever and ever, and will cast their crowns before the throne," (Rev. 4:10).
13. "And every created thing which is in heaven and on the earth and under the earth and on the
sea, and all things in them, I heard saying, "To Him who sits on the throne, and to the Lamb, be
blessing and honor and glory and dominion forever and ever," (Rev. 5:13).
14. "Amen, blessing and glory and wisdom and thanksgiving and honor and power and might, be to
our God forever and ever. Amen," (Rev. 7:12).
15. "and swore by Him who lives forever and ever, who created heaven and the things in it, and the
earth and the things in it, and the sea and the things in it, that there shall be delay no longer,"
(Rev. 10:6).

Louw, Johannes P. and Nida, Eugene A., Greek-English Lexicon of the New Testament based on Semantic
111

Domains, (New York: United Bible Societies) 1988, 1989, [Online] Available: Logos Library System.
16. "And the seventh angel sounded; and there arose loud voices in heaven, saying, 'The kingdom of
the world has become the kingdom of our Lord, and of His Christ; and He will reign forever and
ever,'" (Rev. 11:15).
17. "And one of the four living creatures gave to the seven angels seven golden bowls full of the
wrath of God, who lives forever and ever," (Rev. 15:7).
18. "And there shall no longer be any night; and they shall not have need of the light of a lamp nor
the light of the sun, because the Lord God shall illumine them; and they shall reign forever and
ever," (Rev. 22:5).

Clearly, the phrase "forever and ever" is a correct translation of the Greek "eis tous aionas ton
aionon -- forever and ever" Every instance of the phrase shows eternality.
But, a word of caution: The Universalist may say that Rev. 19.3 is not eternal because it is the
description of smoke from the City of Babylon. But, the judgment that is cast upon her is only the
beginning of the eternal punishment of the wicked, indicated in the statement that the smoke from
her goes up forever and ever.
The phrase is always speaking of eternal duration.
Universalism is incorrect because not all will be saved because:

"And the devil who deceived them was thrown into the lake of fire and brimstone, where the
beast and the false prophet are also; and they will be tormented day and night forever and
ever," (Rev. 20:10).
Scriptures that say not all are saved

Universalists believe all people will be saved. They often complain against the contrary teaching
that people go to hell by posing questions such as

• "Do you really believe that God is going to lose most of mankind in hell and that only a few are
going to be saved?"
• "If most go to hell, doesn't that mean that Satan wins since God only gets a few compared to
the majority who are lost?"

Of course, these kinds of questions are the wrong ones to ask. What they are doing is using
emotionalism to sway someone's beliefs. What they should be asking are questions like these:

• "What does the Bible teach about damnation?"


• "Does the Bible tell us if most will be lost or saved?"
• "Does it tell us that all will be saved?"

The means to good biblical theology is to examine the whole of scriptures without bias so that
proper and correct doctrines can be determined. Of course, no one is without bias. But, that does not
mean that we should give up trying to be objective. We must endeavor to let God's word lead us
rather than our emotionalism and personal preferences make decisions for us, especially about
doctrine. At least, that should be the goal.
What matters is what God has revealed in His word. So, are there scriptures in the Bible that
plainly state that not all are saved? Yes, there are.

• "Enter by the narrow gate; for the gate is wide, and the way is broad that leads to destruction,
and many are those who enter by it. 14"For the gate is small, and the way is narrow that
leads to life, and few are those who find it," (Matt. 7:13-14).
• "For many are called, but few are chosen," (Matt. 22:14).
• "And He was passing through from one city and village to another, teaching, and proceeding on
His way to Jerusalem. 23And someone said to Him, "Lord, are there just a few who are
being saved?" And He said to them, 24"Strive to enter by the narrow door; for many, I tell
you, will seek to enter and will not be able. 25"Once the head of the house gets up and shuts
the door, and you begin to stand outside and knock on the door, saying, ‘Lord, open up to us!’
then He will answer and say to you, ‘I do not know where you are from.’ 26"Then you will begin
to say, ‘We ate and drank in Your presence, and You taught in our streets’; 27and He will say, ‘I
tell you, I do not know where you are from; depart from Me, all you evildoers,'" (Luke 13:22-
27).
• "And Isaiah cries out concerning Israel, "Though the number of the sons of Israel be as the
sand of the sea, it is the remnant that will be saved; 28for the Lord will execute His word
upon the earth, thoroughly and quickly," (Rom. 9:27).

These verses are plain and clear. Not all are saved; in fact, few are. Whether or not we think this
makes God a failure, or that it makes us sad, or upsets us, isn't really that important. If the Bible says
it, that settles it. What is left is to make adjustments in our understanding and feelings in order to
bring more in line with what God has stated.
After all, we do not know the mind of God. His ways are higher than our ways. I prefer to accept
what it says than feel my way through theology.
The unforgivable sin and the age to come

The Universalists teach that all people will eventually be saved through the atoning work of Christ.
Proponents of universalism must, therefore, maintain that there is no unforgivable sin. For if there
were, then their theory that all people will be saved would be proven wrong.
Jesus said there was a sin that would not be forgiven in "this age or the age to come," Matt. 12:32:

"And whoever shall speak a word against the Son of Man, it shall be forgiven him; but whoever
shall speak against the Holy Spirit, it shall not be forgiven him, either in this age, or in the age
to come."

A parallel passage is found in Luke 12:10. Jesus said,

"And everyone who will speak a word against the Son of Man, it shall be forgiven him; but he
who blasphemes against the Holy Spirit, it shall not be forgiven him."

Jesus stated, in Matt. 12:32, that there is a sin that is not forgivable either in "this age or the age
to come." In Luke 12:10, He says blasphemy against the Holy Spirit will not be forgiven at all.
The universalist qualifies their belief by stating that "the age to come" is a future age which will
terminate. Therefore, they conclude that blasphemy of the Holy Spirit will be forgiven after the end of
"the age to come." Therefore, when they say read Jesus' words in Luke 12:10, instead of them
concluding that blasphemy of the Holy Spirit won't be forgiven, they conclude it will.
The important question then becomes "What is meant by 'this age and the age to come'?" Is Jesus
dividing time into two periods? Are there only two ages or are there more than two? Does either or
both of these ages end?
I believe that the Jesus divided time into two ages and that all other ages mentioned in scripture
fall within these two over-arching categories. Furthermore, "this age" is the time period we are in now,
and "the age to come" is that future time when the Lord returns and eternity begins. Therefore, "the
age to come" is without end.

How many ages are there?

" . . . and seated Him at His right hand in the heavenly places, 21 far above all rule and
authority and power and dominion, and every name that is named, not only in this age, but
also in the one to come" (Eph. 1:20b-21).

This verse speaks about Jesus being seated at the Father's right hand and that He (Jesus) is above
all rule and authority in this age and the age to come. Jesus' dominion will never end. Therefore, the
age to come, singular, will not end either. This is why God the Father says of the Son, "Thy throne, O
God, is forever and ever," (Heb. 1:8). Also, ". . . so that in all things God may be glorified through
Jesus Christ, to whom belongs the glory and dominion forever and ever. Amen," (1 Pet. 4:11).
Let's take a look at what the Bible says about "This Age and the Age to Come."
This Age The Age to Come
• We will receive 100 times as much as • We will receive eternal life
what we lose (Mark 10:30; Luke (Mark 10:30; Luke 18:30).
18:30).
• People are given in marriage (Luke • We do not marry, (Luke
20:34). 20:35).
• The wisdom of this world is the
wisdom of this age (1 Cor. 1:20).
• The rulers of this age are coming to
nothing (1 Cor. 2:6). Note that in the Greek, the phrase
• Satan is the god of this age (2 Cor. "the age to come" is always in the
4:4). singular. It is speaking of a singular
• Jesus rescued us from the present age to come where we will have
evil age (Gal. 1:4). eternal life.
• The end of this age occurs at the
return of Jesus (1 Cor. 15:23-24).

• The tares are gathered and burned in


the fire (Matt. 13:39).

As you can see, "this age" is obviously about the present time period because in it we have
marriage, rulers, evil, etc. In the age to come, however, we receive eternal life and no marriage
occurs. The future reference of receiving eternal life does not mean that we do not posses it now. 1
John 5:13, says we do. Rather, Jesus is speaking of the completion of our redemption which includes
our bodies as well. "So also is the resurrection of the dead. It is sown a perishable body, it is raised
an imperishable body," (1 Cor. 15:42). In the age to come, we enter into eternity because it is when
we are resurrected. This happens at the return of Christ.

Eph. 2:5-7

There is one verse the New Testament that mentions ages in a future sense. The phrase is "ages
to come" and it only occurs in Eph. 2:7

"even when we were dead in our transgressions, [He] made us alive together with Christ (by
grace you have been saved), 6 and raised us up with Him, and seated us with Him in the
heavenly places, in Christ Jesus, 7 in order that in the ages to come He might show the
surpassing riches of His grace in kindness toward us in Christ Jesus."

This statement is not saying that there are future ages, plural, which are not defined in scripture,
anyway. Rather, it is a declaration that in the future state, the Christians will enjoy the "surpassing
riches of His grace" -- in the totality of the future. The phrase "ages to come" is merely an
expression.
This type of usage of "ages" to describe a very long time is also seen in Romans 16:25, "Now to
Him who is able to establish you according to my gospel and the preaching of Jesus Christ, according
to the revelation of the mystery which has been kept secret for long ages past," In Greek, "long ages
past" is "cronos aioniois," which is literally "time eternal(s)". This phrase is not saying that there are
literally eternal past "ages," but that in long times past, the mystery was hidden. Other verses with
the same usage of ages past are 1 Cor. 2:7; 10:11; Eph. 3:9; Col. 1:26; Titus 1:2; and Heb. 9:26.
We can see from the table above, that all the ages past are under the umbrella of "this age" in
which we have evil, suffering, etc.
The End of This Age

It is important to understand when "this age" ends because it will effect our understanding of the
nature of "the age to come" and further clarify that the age to come is without end.
I have compiled a chart below to make this easier to see. The left column contains the events
(resurrection, rapture, condemnation, etc.). To the right are the phrases used in the Bible to describe
the event. With them are the verse locations.

Phrase and location describing when the event occurs


The Event End of The Day of Last Last
this Age the Lord Day Trumpet
John 6:39,40,
Resurrection of 1 Cor. 15:52;
44, 54;
the dead 1 Thess. 4:16
11:24;
Harvest/Rapture
1 Thess.
(thief in the Matt. 13:39
4:16-5:2
night)
Gathering of 1 Thess.
Matt. 24:3,31
elect 5:2
Condemnation 1 Cor. 1:8 John 12:48
Wicked burned
Matt. 13:40; 49-50
with fire
Destruction
1 Cor. 5:5
and Salvation
Sun to
darkness, Acts 2:20
moon to blood
New heavens
2 Pet. 3:10
and New Earth

From the above chart you can see that all the events happen at the same time. The Resurrection
occurs on the Last day (John 6:39-40), which is also the last trumpet (1 Cor. 15:52). The resurrection
is just before the rapture (1 Thess. 4:16-5:2) which occurs on the Day of the Lord (1 Thess. 5:2) when
Jesus returns (1 Thess. 4:16-5:2).
It is with/after Jesus' return, which is simultaneous with the rapture, the harvest, etc, that we
receive our resurrected bodies and are forever with the Lord. It is then, the "age to come." This age
will not end.
It is in "the age to come" that blasphemy is not forgiven. In other words, it isn't ever forgiven.

Heb. 6:6 and Heb. 10:26

There are two other verses which also mention a unforgivable sins.

"For in the case of those who have once been enlightened and have tasted of the heavenly gift
and have been made partakers of the Holy Spirit, 5 and have tasted the good word of God and
the powers of the age to come, 6 and then have fallen away, it is impossible to renew them
again to repentance, since they again crucify to themselves the Son of God, and put Him to
open shame," (Heb. 6:4-6)
"For if we go on sinning willfully after receiving the knowledge of the truth, there no longer
remains a sacrifice for sins, 27 but a certain terrifying expectation of judgment, and the fury
of a fire which will consume the adversaries," (Heb. 10:26-27).

In the case of Heb. 6:6, repentance is an impossibility with those who have been made partakers
of the Holy Spirit and then have fallen away. If this repentance is impossible, then so is forgiveness.
Likewise, in Heb. 10:26, "if we go on sinning willfully after receiving the knowledge of the truth,
there no longer remains a sacrifice for sins." The universalists want to have the sacrifice of Jesus
cover, pay for, and remove the penalty of all people's sins who have ever lived. But, according to this
verse, there is a point when the sacrifice of Jesus is no longer available to a person.

Conclusion

Therefore, the teaching that everyone will eventually be saved cannot be true.

• This age and the to come are two over-arching categories that divide human existence.
• Blasphemy of the Holy Spirit will not be forgiven in the age to come.
• At the end of this age, "the age to come" begins and it has no end.
• Heb. 6:6 and Heb. 10:26 also show us that there are conditions of non-forgiveness.

Universalism is not true.


Does God hate anyone?
The universalists repeatedly say things like, "God loves us all so much that He will save us all"; or
"He hates the sin, but loves the sinner"; or "God is love, and will not send anyone to hell."
Universalists teach that God is so full of love, that He simply cannot send anyone to eternal hell
fire. It is against His infinite love. They want God to forgive all, even those who openly reject Him
and die cursing God.
I must admit, it is nice to think of God's love being so infinitely great that all will ultimately be
saved. Hell is a terrible place and I don't want anyone to go there. But it does not matter what I
think. It matters what the Bible says.
God is love (1 John 4:8), but God also punishes the sinner and hates all who do iniquity. God is
not one sided. He is not simply an infinitely loving God. He is also infinitely just. He must deal with
sin. He must punish the sinner.
In the truth of God's word, we find that the Lord has provided one way by which we may be saved.
That single way is through Jesus' sacrifice. For all who trust in Him, salvation will come. But to those
who turn away, God's wrath abides upon them: "He who believes in the Son has eternal life; but he
who does not obey the Son shall not see life, but the wrath of God abides on him," (John 3:36).

Does God hate anyone?

Does God hate anyone? The answer is yes.

• Psalm 5:5, "The boastful shall not stand before Thine eyes; Thou dost hate all who do
iniquity,"
• Lev. 20:23, "Moreover, you shall not follow the customs of the nation which I shall drive out
before you, for they did all these things, and therefore I have abhorred them."
• Prov. 6:16-19, "There are six things which the Lord hates, yes, seven which are an
abomination to Him: 17 Haughty eyes, a lying tongue, and hands that shed innocent blood, 18 A
heart that devises wicked plans, feet that run rapidly to evil,
19
A false witness who utters lies, and one who spreads strife among brothers."
• Hosea 9:15, "All their evil is at Gilgal; indeed, I came to hate them there! Because of the
wickedness of their deeds I will drive them out of My house! I will love them no more; All their
princes are rebels."

Are these verses hard to read? Do they make you feel uncomfortable? They should. God hates
sin. But, He does not punish sin. He punishes the sinner. Sin cannot be tied up and thrown into a
fire. It cannot be put in a box or glued to a stick. It is rebellion. It is rebellion in the heart. It is
breaking God's Law. Sin occurs inside the heart and mind of people. Therefore, God must punish the
sinner. Why? Because He is both Holy and Just and the person who sins offends God. God's Holy and
Just character will not allow Him to ignore this offense. Why?....

God's Law is Perfect

When God said, "Let there be light," it happened. When He commanded that the oceans be, they
came into existence. God's word is powerful. What He says is never futile, empty, or without power.
The Law is a reflection of God's character. It is pure and perfect. It is powerful. The Ten
Commandments reflect God's holiness and justice. These commandments are not without
punishments. A law without consequences is only an empty slogan.
To sin is to break God's Law and offend His character. To sin means to challenge His character and
authority. It means you go against His word. But God is not a liar. His word is true. He has said He
will punish the lawbreaker.
But, praise be to God, that while we were yet sinners, Jesus died for us (Rom. 5:6). There is no
way we can appease God. That is why God became one of us (John 1:1,14; Heb. 2:17), to take our
place and become sin on our behalf (2 Cor. 5:21). Therefore, people have two options:
1. Trust Jesus, God in flesh, as your savior and put your faith in the sacrifice that He made on the
cross and in nothing you do.
2. Reject the cross and let the penalty of the Law fall upon you.

Either God pays, or you do -- forever. Which will it be?

"For if we go on sinning willfully after receiving the knowledge of the truth, there no longer remains
a sacrifice for sins," (Heb. 10:26). If there is no sacrifice available for you, then God's wrath abides
upon you because He hates sin and your sin is not removed (John 3:36). Trust Jesus alone or the
wrath of God will abide upon you forever.

Conclusion

The sobering fact is that God is so holy and righteous that He hates the sinner (Psalm 5:5; Lev.
20:23; Prov. 6:16-19; Hos. 9:15). Some say that we should say that God only hates the sin but loves
the sinner. But, the above scriptures speak contrary to that. But it is also true that He is love (1 John
4:8). It is better to accept the love of God found in Jesus than to reject it and suffer His wrath.
Is Hell Eternal?
The teaching that there is an eternal hell in which hordes of mankind will suffer eternal punishment
can be a difficult doctrine to accept. We hear so much about God's infinite love and how He desires
that all men be saved (1 Tim. 2:4). However, those who develop their theologies based upon the
"gentle" side of God do so with an incomplete picture. Not only is God loving (1 John 4:8-10),
gracious (Exo. 33:19; 1 Pet. 2:3), and merciful (Exodus 34:6; Psalm 67:1; James 5:11), but He is also
holy (Isaiah 6:3; Rev. 4:8), just ( Neh. 9:32-33; 2 Thess. 1:6), and hates sin (Psalm 5:5-6; Hab.
1:13). God punishes the sinner (Jer. 50:31; Ez. 44:12; Matt. 25:46; 2 Thess. 1:9; 2 Pet. 2:9; Heb.
10:29).
The Bible teaches that there is a fiery hell, a place that Jesus warned people about.

"And if your hand or your foot causes you to stumble, cut it off and throw it from you; it is
better for you to enter life crippled or lame, than having two hands or two feet, to be cast into
the eternal fire," (Matt. 18:8).

Eternal fire is real. Jesus said it was. In fact, Jesus spoke more of hell than He did of heaven. It is
what Jesus came here to save us from.
There will be a Day of Judgment when all people will face God. Those who are not covered by the
sacrifice of Jesus on the Cross will be cast out into hell where they will undergo eternal punishment.
"And these will go away into eternal punishment, but the righteous into eternal life" (Matt. 25:46). In
this verse, the same word "eternal" is used to describe the punishment of the wicked as well as the
eternal life of the believer. The punishment is endless as is the eternal life of the believer. That is why
the gospel (1 Cor. 15:1-4) is so important, because it saves people from eternal damnation:

“Now I make known to you, brethren, the gospel which I preached to you, which also you
received, in which also you stand, 2 by which also you are saved, if you hold fast the word
which I preached to you, unless you believed in vain. 3 For I delivered to you as of first
importance what I also received, that Christ died for our sins according to the Scriptures, 4 and
that He was buried, and that He was raised on the third day according to the Scriptures.”

“Truly, truly, I say to you, he who hears My word, and believes Him who sent Me, has eternal
life, and does not come into judgment, but has passed out of death into life,” (John 5:24).

Following are a few verses that show the eternality of the hell and punishment. God uses different
phrases to describe the same thing.

• "And these will pay the penalty of eternal destruction, away from the presence of the Lord
and from the glory of His power," (2 Thess. 1:9).
• "Just as Sodom and Gomorrah and the cities around them, since they in the same way as these
indulged in gross immorality and went after strange flesh, are exhibited as an example, in
undergoing the punishment of eternal fire" (Jude 7).
• These men are those who are hidden reefs in your love feasts when they feast with you without
fear, caring for themselves; clouds without water, carried along by winds; autumn trees
without fruit, doubly dead, uprooted; 13 wild waves of the sea, casting up their own shame like
foam; wandering stars, for whom the black darkness has been reserved forever," (Jude12-13).
Is "forever and ever" without end?

The phrase "forever and ever" is used both of describing God's eternal worth and the duration of
eternal damnation. The exact same Greek phrase is used in each of the verses in the table below.

forever and ever

aionas ton aionon


"ages of the ages"
Eternal - without end Eternal Damnation
"Now to the King eternal, immortal, invisible, "And a second time they said, "Hallelujah! Her
the only God, be honor and glory forever smoke rises up forever and ever" (Rev.
and ever. Amen," (1 Tim. 1:17). 19:3).

". . . To Him who sits on the throne, and to "And the devil who deceived them was thrown
the Lamb, be blessing and honor and glory into the lake of fire and brimstone, where the
and dominion forever and ever" (Rev. beast and the false prophet are also; and they
5:13). will be tormented day and night forever and
ever," (Rev. 20:10).

The Greek phrase "aionas ton aionon," which is translated "forever and ever," occurs 18 times in
the Greek New Testament. In 17 of them, the phrase means without end, extending into infinity. In
Rev. 19:3, the phrase is used to describe the destruction of the great whore of Babylon (Rev. 17:1,4)
whose smoke ascends forever and ever. It too is eternal and it signifies the beginning of the eternal
judgment that comes upon her.
Also worth examining is Rev. 14:11: "And the smoke of their torment goes up forever and ever;
and they have no rest day and night, those who worship the beast and his image, and whoever
receives the mark of his name."
The Greek in Rev. 14:11 is only slightly different. In the table above, "forever and ever" is
translated from the Greek, "aionas ton aionon," which is literally "ages of the of ages." In Rev. 14:11,
the Greek is "aionas aionon" which is literally, "ages of ages." In the latter, the single Greek word "of
the" is missing. But it is not necessary and does not change the meaning of the text. Therefore, the
scripture teaches the smoke of their torment goes up forever, without end.

Unquenchable Fire

Some believe that the fires of hell are symbolic and/or temporal. But the following verses show
that they are not.
Matt. 3:12 says, "And His winnowing fork is in His hand, and He will thoroughly clear His threshing
floor; and He will gather His wheat into the barn, but He will burn up the chaff with unquenchable
fire." (See also Luke 3:17.)
Mark 9:43 says, "And if your hand causes you to stumble, cut it off; it is better for you to enter life
crippled, than having your two hands, to go into hell, into the unquenchable fire." The word
"unquenchable" is "asbestos" in the Greek. According to the enhanced Strong's lexicon, it means
"unquenchable, the eternal hell fire to punish the damned."
The following citations are from Greek dictionaries and Lexicons. They show that the word
"unquenchable," which is "asbestos" in the Greek, (which occurs only in Matt. 3:12, Luke 3:17, and
Mark 9:43) means unquenchable, without end.
• "unquenchable, inextinguishable" - Liddell, H. G., and Scott, Abridged Greek-English Lexicon,
(Oxford: Oxford University Press) 1992, [Online] Available: Logos Library System.
• "not quenched" - Vine, W. E., Vine’s Expository Dictionary of Old and New Testament Words,
(Grand Rapids, MI: Fleming H. Revell) 1981, [Online] Available: Logos Library System.
• "pertaining to a fire that cannot be put out" - "unquenchable." - Louw, Johannes P. and Nida,
Eugene A., Greek-English Lexicon of the New Testament based on Semantic Domains, (New
York: United Bible Societies) 1988, 1989, [Online] Available: Logos Library System
• "unquenched, unquenchable" - Enhanced Strong’s Lexicon, (Oak Harbor, WA: Logos Research
Systems, Inc.) 1995, [Online] Available: Logos Library System.
• "that cannot be put out" - Wigram-Green, The New Englishman's Greek Concordance and
Lexicon, (Peabody Mass: Hendrikson Publishers, 1982, p. 771.
• "inextinguishable" - William F. Arndt and F. Wilbur Gingrich, A Greek-English Lexicon of the
New Testament and other Early Christian Literature, 2nd ed. (Chicago, Ill: The University of
Chicago Press), 1979, p. 114.

Is hell eternal? Yes it is. Are its fires without end? Yes they are. Is it a pleasant doctrine to
discuss? Not really. But, hell is real. This is all the more reason to preach the gospel. Jesus said,

"And if your hand or your foot causes you to stumble, cut it off and throw it from you; it is
better for you to enter life crippled or lame, than having two hands or two feet, to be cast into
the eternal fire," (Matt. 18:8).
The demonic forces will not be saved

There are some universalists who believe that Satan and all of the demonic realm will be redeemed
through the atonement of Jesus. Though I have found this is a minority view among the universalists,
it is still worth addressing and correcting.
The demonic realm will not and cannot be redeemed. This simple reason is that they have no
Redeemer. Jesus is the Redeemer of humanity, not of the demonic realm.
There is a doctrine in theology called the hypostatic union. It is the doctrine of the two natures of
Jesus. Jesus is both God and man. He was God so that He could appease God the Father with the
sacrifice of infinite value. We need a sacrifice of infinite value because when we sin, we sin against an
infinite God. Therefore our offense against Him has infinite consequences.
No mere human can please and satisfy an infinitely holy and righteous God. Only God Himself can
satisfy the infinite requirements of His own holiness. Therefore, "the Word became flesh and dwelt
among us," (John 1:14). Jesus became our atoning sacrifice to turn away the wrath of God (Rom. 5:9)
by bearing our sins in His body on the cross (1 Pet. 2:24). Therefore, those who trust in Christ are
saved by grace (Eph. 2:8-9; Rom. 6:23).
Likewise, the reason he needed to be a man was so that he could atone for the sins of mankind. He
had to be made like one of us.

"For assuredly He does not give help to angels, but He gives help to the descendant of
Abraham. 17Therefore, He had to be made like His brethren in all things, that He might become
a merciful and faithful high priest in things pertaining to God, to make propitiation for the sins
of the people," (Heb. 2:16-17).

The fallen angels do not have a sacrifice on their behalf. They do not have the infinite God
becoming one of them and atoning for them. Therefore, the demonic realm will not be saved.

______________________

• John 1:14 - "And the Word became flesh, and dwelt among us, and we beheld His glory, glory
as of the only begotten from the Father, full of grace and truth."
• Rom. 5:9 - "Much more then, having now been justified by His blood, we shall be saved from
the wrath of God through Him."
• 1 Pet. 2:24 - "and He Himself bore our sins in His body on the cross, that we might die to sin
and live to righteousness; for by His wounds you were healed."
• Eph. 2:8-9 - "For by grace you have been saved through faith; and that not of yourselves, it is
the gift of God; 9not as a result of works, that no one should boast."
• Rom. 6:23 - "For the wages of sin is death, but the free gift of God is eternal life in Christ
Jesus our Lord."
Fallen angels go to the lake of fire forever

Some Universalists maintain that even the demonic horde will ultimately be redeemed and enjoy
heaven. Other Universalists deny that the demonic forces will be redeemed. You often get different
answers from different Universalists. Is it important? Yes, it is.
People are redeemed because they have a Redeemer, Jesus. Jesus is God in flesh (Col. 2:9) who
bore our sins in His body on the cross (1 Pet. 2:24). Nowhere in the Bible does it state that Jesus bore
the sins of demons. Why? Because it did not happen.
The reason this is important is because we can then see that the demonic realm does not have a
savior and cannot be redeemed from their sins. They will remain in their sins for eternity.
We see the pattern of God's incarnation, bearing of sins, dying, resurrection, and justification by
faith (Rom. 5:1). We see that God became man. We see that Jesus was both God and man (Col.
2:9). It was necessary that He be both God and man for a very simple and important reason. He had
to be God in order to offer a sufficiently holy and infinite sacrifice to counter the infinite offense against
God that creatures commit against Him. He had to be man in order to bear the sins of men. He had
to represent the ones for whom He atoned in order for them to be atoned for.
According to this pattern that we see in Scripture, for the demonic realm to be saved, God would
have become one of them, bear their sins, and, somehow, redeem them through death or some other
means that God would determine. Yet, there is nothing in scripture to substantiate that this has or will
occur. Therefore, the demonic realm has no redeemer and their sins will be retained, forever.
But some might ask, "Could God have another means by which He saves the fallen angels?" Yes.
He could, most anything is possible. But mere possibilities do not make actualities. The Bible does
not provide any redemptive plan for the demonic forces. Therefore, we can safely conclude that there
is none. Demons are not and will not be redeemed.

Reconciling the whole world

Some Universalists maintain that all the demonic forces will be redeemed because it states so in
the Bible. It doesn't. Nevertheless, following are some of the verses used by the Universalists to
claim that the demonic realm will be redeemed. I've commented following each verse.

1. 1 Cor. 15:27-28 "For He subjects all under His feet. Now whenever He may be saying that
all is subject, it is evident that it is outside of Him Who subjects all to Him. Now, whenever
all may be subjected to Him, then the Son Himself also shall be subjected to Him Who
subjects all to Him, that God may be All in all."
A. This is simply stating that all things will be subject to God. Even the damned will be
subject to God because in hell they are in a state of subjection to God's judgment upon
them. It is God's will to judge those who are unsaved. It is according to His law and
they will be subject to that righteous judgment.
2. Col. 1:20, "and through Him to reconcile all things to Himself, having made peace through
the blood of His cross; through Him, I say, whether things on earth or things in heaven."
A. This verse does not state that the demons will be saved. Reconciliation means to bring
someone or something into a right relationship with another. The lawful and proper
relationship of the unsaved before God is damnation. Those who are condemned to
damnation are rightly condemned and their relation to God is proper. They are
eternally subjected to His judgment.
B. The demonic horde is not in heaven. This is speaking of God reconciling His people to
Himself -- those who who have died and gone to be with God and those who are still
living in this world. But, it is only the Christian who is redeemed. Those who reject
Christ are not.
3. Rom. 8:19, "For the anxious longing of the creation waits eagerly for the revealing of the
sons of God. 20For the creation was subjected to futility, not of its own will, but because of
Him who subjected it, in hope 21that the creation itself also will be set free from its slavery to
corruption into the freedom of the glory of the children of God. 22For we know that the whole
creation groans and suffers the pains of childbirth together until now."
A. Does creation have a will? No it does not. Obviously we have figurative usage here.
This is dealing with creation as a whole, earth, sky, animals, stars, etc. All of creation
was affected by Adam's sin because Adam was the one who was in charge of it. Since
Adam's sin (and the sins of his progeny) are cleansed, then that which Adam
represented is also cleansed; namely, creation. Creation did not have a redeeming
sacrifice for its sin since creation did not sin. Rather, as sin entered the world through
Adam, when all sin is finally dealt with, its effect will be removed from the creation that
was given to Adam in his dominion authority.
B. The angels who fell were not under Adam's representation. They had their own
freedom to choose what they wanted to do. They willingly sinned. They willingly
rebelled. And, they have no propitiation, no atoning sacrifice to have their sins
forgiven.
C. Also note that "all creation" does not always mean every created thing. Look at Mark
16:15 Jesus said, "Go into all the world and preach the gospel to all creation." All
creation here is not inclusive of the planets, stars, nebulae, nor the angels. We are told
to preach the gospel to people. And consider Col. 1:23, ". . . the hope of the gospel
that you have heard, which was proclaimed in all creation under heaven. . . " This is
not speaking of angels, planets, etc. It is a representative usage of people all over the
place.

Why is this important?

The Lake of Fire was created for the devil and his angels. The devil and his fallen angels have no
redeeming sacrifice the way we do. The demonic forces will be cast into hell and since they are not
forgiven, their punishment is without end. Likewise, we see that people are also thrown into the Lake
of Fire. Since the angels' condemnation is without end in the Lake of Fire, so to is the punishment
delivered to those people who are also cast into that very same lake.

• Matt. 25:41, “Then He will also say to those on His left, ‘Depart from Me, accursed ones, into
the eternal fire which has been prepared for the devil and his angels.
• Rev. 20:14-15, And death and Hades were thrown into the lake of fire. This is the second
death, the lake of fire. 15And if anyone’s name was not found written in the book of life, he was
thrown into the lake of fire.
• Mark 9:43, “And if your hand causes you to stumble, cut it off; it is better for you to enter life
crippled, than having your two hands, to go into hell, into the unquenchable fire,"

The demonic realm has no redeemer. Jesus did not bear their sins in His body. He bore OUR sins in
his body (1 Pet. 2:24). Since they do NOT have a sin-bearer, they will remain in their sins. Their
punishment is eternal. Since there are people thrown into that SAME lake of fire, their punishment is
likewise eternal. Is hell eternal? Yes. Are all people saved? No.
Does eternal punishment deny God's justice?

Some of the proponents of Universalism maintain that eternal punishment cannot be true because
if God eternally damned someone, it would mean that the punishment would never be complete.
Therefore, God is not satisfied, His judgment is not realized, and justice is never accomplished.
The first problem with this objection is the idea that God's eternal judgment necessarily must have
an end. If it is eternal punishment, then it wouldn't end. "And these will go away into eternal
punishment, but the righteous into eternal life," (Matt. 25:46). Not all judgments and punishments
end. Consider a person who is executed for a capital crime. His punishment is death. In effect, it is a
judgment that is eternal. The judgment is completed by the accomplishment of a sentence:
execution. The sentence has an eternal duration which will not end and at the same time the
judgment has been accomplished. The judgment, in and of itself, is eternal by definition and this does
not mean that it is not satisfied or realized. The eternal sentence of death has been accomplished and
is still in effect. Therefore, we can see that a valid punishment with an eternal result can be a reality.
Second, it is not logically necessary that an eternal punishment upon a sinner be an insufficient or
non-accomplished judgment. It is just as logical to say that God's infinite justice is properly
accomplished with an infinite punishment. After all, an offense of infinite value would require an
infinite punishment.
Third, it would be an injustice to God's infinite righteousness and holiness to have the sinner's
punishment be terminated. Of course, I am not here speaking of discipline, where the Lord chastises a
person and welcomes him back into fellowship. I am speaking here of damnation, that pronouncement
upon a sinner who is not covered in the blood of Christ. As I said above, it follows that if God is
infinite and the sinner has offended God, then that is an infinite offense. If judgment upon the sinner
regarding his sinfulness were temporal, then it means that a sinner's suffering is sufficient to appease
an infinite God. That would be unjust since, Gal. 2:21 says, "I do not nullify the grace of God; for if
righteousness comes through the Law, then Christ died needlessly." Paul is saying that if we could be
please God by what we do (suffer), then Christ died needlessly.
Fourth, the universalists have stated that though the unrepentant sinners are truly forgiven in
Christ, they must be "punished," "purified," "corrected," for a period of time in the after life before
they are ready to be admitted into the eternal life of realized forgiveness. This is a very dangerous
teaching because it strikes at the very heart of the atonement of Christ. For a person to suffer the
judgment of God because of his sin until he is found worthy to be with God is to state that the
atonement is insufficient and must be completed through the suffering of the sinner. This is
blasphemous and must be avoided at all costs.
It should be plain to see that the universalist position is incorrect.
The Danger of Universalism

Universalism teaches that all people will eventually be saved through the atonement of Christ. It
says that all mankind, even those who have openly rejected Jesus, those who have willingly committed
horrible crimes and died without repentance, and without the covering of Christ's blood, will enjoy a
future with God. This belief is based upon the idea that God's love is so infinitely great, that His grace
in Christ is so awesome, that everyone will be saved. This simply is not true.
The danger of universalism is that it to can give someone a false sense of security about their
eternal destiny. It can remove the need of accountability. It can remove the fear of judgment. It does
not require repentance. A person who adopts universalism can easily conclude that if he is going to be
saved no matter what he does, then why be concerned about repentance or accepting Jesus as Lord
and Savior? This potential error is most dangerous. Especially because if universalism is not true, then
the false sense of security it has given to those who have not trusted in Christ, will lead them to
damnation. This is a very serious danger.
Of course, simply because it is possible that people will become lax in accepting Christ if they adopt
universalism, it does not mean this is what will happen. Nor does it mean that all Universalists think
they can go out and sin willfully. On the contrary, most Universalists are very moral. But, there is the
inherent danger in universalism that reduces the need for repentance and salvation. This is a great
risk. Eternity is a long time to be wrong and hell is a terrible place to be forever.

What does Satan want?

Satan wants the destruction of people. Satan wants people to die in their sins and go to hell. He is
utter hatred and complete evil. But, he is also extremely cunning with an intelligence that is vast.
Universalism may very well become a tool of the evil one in the last days. It weakens the need to
trust in Christ in this life.
In Universalism, Satan can work his false doctrines through its adherents. This is clearly the case
since many Universalists deny the Trinity and the deity of Christ. But in universalist theology, it really
doesn't matter. Why? Because ultimately, in the after life, people will come to a true knowledge of
God and repent and be saved. So, even if they are wrong now, they will be right later.
Satan says, "Don't worry about receiving Jesus now. You can do that later." But it is Christ who
says,

• “At the acceptable time I listened to you, And on the day of salvation I helped you” ;behold,
now is “the acceptable time,” behold, now is “the day of salvation" (2 Cor. 6:3).
• "He again fixes a certain day, “Today,” saying through David after so long a time just as has
been said before, “Today if you hear His voice, Do not harden your hearts" (Heb. 4:7).

What would be a uniting religious concept that would tend to unite different religious systems?
Universalism! Think about it. If everyone is going to be saved, then Buddhism, Islam, Catholicism,
Mormonism, Hinduism, etc. will not keep people out of hell. If all religions adopted universalism, then
each could look at the other as being a different belief (or even error) that would, nevertheless, lead a
person to redemption in the after-life.

What does Jesus save us from?

Jesus saves. But what does He save us from? Does Jesus save us from ourselves, our thoughts,
our actions, our temperament, or even our sins. No. He saves us from the wrathful judgment of God
upon us due to us because of our sinfulness. There is a natural consequence to being a sinner:
judgment. God will punish the sinner (Hosea 8:13; 9:9). The one who rejects Jesus does not have a
covering for sin, does not have forgiveness of sins, and has the wrath of God abiding upon him:
He who believes in the Son has eternal life; but he who does not obey the Son shall not see
life, but the wrath of God abides on him, (John 3:36).

Jesus saves us from that wrath. Jesus saves all those who receive Him (John 1:12; Rom. 8:1) so
that they can escape the judgment to come.

He therefore began saying to the multitudes who were going out to be baptized by him, "You
brood of vipers, who warned you to flee from the wrath to come?" (Luke 3:7).

Jesus warned us about hell (Matt. 5:22,29-30;23:33; Mark 9:45; Luke 12:5). In fact, He spoke
more of it than He did of heaven. He does not want you to go to that place of torment. That is why He
said, "And if your hand or your foot causes you to stumble, cut it off and throw it from you; it is better
for you to enter life crippled or lame, than having two hands or two feet, to be cast into the eternal
fire," (Matt. 18:8). If universalism is true, then where is the power in Jesus' warning? If universalism
is true then there is no eternal fiery hell, no dread of being cast into it, no wrath to come -- but there
is!
Hell is the real place. Jesus came to save us from it. But you must trust Christ and His atoning
sacrifice in order to escape the wrath of God.
Does universalism lead us to urgency? Does it lead us to fear the wrath to come? No. It doesn't.
It removes the urgency. It removes the fear of God.
Now, I am not saying that we must live in fear or that fear is the only motive to be saved. But,
Jesus Himself warned people about hell and the Bible tells us that "the fear of the Lord is the
beginning of wisdom," (Prov. 9:10).

Conclusion

Universalism can lead to complacency. It can easily lessen the concern for salvation and
repentance. In this, there is danger.
The Danger of Universalism Illustrated

Universalism teaches that all people will eventually be saved and that no one goes to the fiery-hell
to suffer for their sins. It means that everyone is saved, whether or not they have accepted or openly
rejected Jesus' atonement. Of course, this goes against Heb. 10:26-27 which says,

"For if we go on sinning willfully after receiving the knowledge of the truth, there no longer
remains a sacrifice for sins, 27but a certain terrifying expectation of judgment, and the fury of a
fire which will consume the adversaries" (NASB).

There is a danger in the Universalist position. It can weaken the need for someone to receive
Jesus as Savior. To prove the point I provide this "exaggerated" illustration. But, the contrived
universalist sermon below contains many quotes said by universalists to me. I've just blended them
in:

There is this horrible, hateful man who abused all sorts of people in countless ways. He stole from
them, was foul mouthed, a habitual adulterer, and he openly mocked and blasphemed God. He
repeatedly sought to destroy the Christian Church and did everything in his power to prevent the
gospel from being preached. He was a wretched and evil man.
This man died and, out of social obligation, all this relatives were at his funeral. There were lots of
impressionable children, sweet little old ladies, and many acquaintances from work who had been
appalled at the atheists horrible lifestyle yet were there nevertheless. However, some of the people
there were thinking of sin, salvation, God, judgment, and what death brings. This often happens at
funerals.
A universalist pastor was giving the message.
"We know that this man was a wretched soul who did whatever he could to oppose God and
blaspheme Christ. He habitually stole, lied, cheated, coveted, swore, drank, caroused, and injured.
We know that he lived his life in wanton sin and rebellion and that many people feared and hated him.
"But, you know what? He's going to heaven. And do you know why? Because God is too good and
loving to let even this wretched, evil soul go to an imaginary place like hell.
"I know. He hated God and rejected Jesus. He even cursed the cross and did everything he could
to oppose Christianity. But, it doesn't matter because, ultimately, you will see him again. Only, he
won't be mean and evil. He will be kind and gentle. Jesus will change him. Jesus loves him. Jesus
loves him more than we can imagine. He also loves you more than you can possibly know. There is
no need to fear Him at all.
"Some may preach that there is a hell, a place of fiery, eternal torment. But I tell you that it is a
lie. It is a tool of Tormentists who cannot accept the love of God and how often enjoy the thought of
people rotting in that awful place.
"The Bible says that Jesus is the Savior of all men. And that means even this awful, God hating,
wretch of a man that has now gone on to meet his maker.
"I preach a message of peace, of reconciliation, of joy, and of God's redeeming power.
"I preach the truth."
After the service, two of the impressionable young kids are talking.
"I liked the sermon the universalist gave. Heck, I was worried about hell. But now I don't have to
be worried about it. Now I know I'm going to heaven no matter what. I know I'm not that great a
person, but if that horrible man in the coffin is going to heaven, then I am sure I will too. After all, I'm
not as bad as he is."
His friend answers, "Yep, I don't have to give up my drugs, sleeping with my girlfriend, or stealing
CD's from the music store. Life is great!"
"Well, I don't think it means we can go sin, though."
"I know. I was just kidding. But you are right. No worries. No problem."
The two walk off enjoying the new found freedom found in the love of God, but they now don't
need to trust Jesus as their savior. They have no need to appeal to Jesus to save them, because there
isn't anything to save them from. They walk away unredeemed.
And let's not forget about the co-worker who has been thinking of going to church. He has been
committing adultery and he has been fearing the judgment of God. He thinks to himself....
"Whew! Am I glad to hear that message. That was great! I don't have to worry about anything.
Man what a relief. No hell! I'm going to heaven! Man Oh Man! this is so cool.... Guess I don't have to
go to church tomorrow and I can forget about reading what the Bible says about Jesus and sin.... no
need. Okay, maybe I should stop the adultery. But, that sure was a good message from that
universalist. I feel so much better about God's love. I feel so much better knowing there is no hell."
He walks away not having confessed his sins to Christ (1 John 1:9), not having been cleansed, not
having received Christ (John 1:12), not having been justified by faith (Rom. 5:1).
....... an eon later..... heaven.....judgment seat.... God is there.... and if the universalist is
wrong....
Two boys and a coworker are in hell. They never did trust in Jesus as their savior. They never
did confess their sinfulness and ask Jesus to forgive them. But, at least they felt good while on earth
and they didn't have to listen to those people who preached hell-fire.

Why is it that Jesus warned us to not go to the "fiery place" if it isn't real? Why did He speak more
about hell than heaven? Why did He tell us to turn from sin and be SAVED! Saved from what? Saved
from Damnation!
Universalism is a dangerous teaching. It weakens the need for a savior and that is a great risk to
take for such a dubious teaching.
If election is true what is the danger in universalism?

Universalism teaches that Jesus died for all people and that all will eventually be saved. It also
teaches that if someone rejects Christ in this life, he can accept Jesus in the next one, even if he is a
horrible person who severely blasphemed God. The danger with this is that based on this principle in
universalism, someone could adopt an attitude of complacency who would then choose to live a life of
sin and rebellion and wait until the afterlife to become a true believer -- even if there is some
"purification" involved in the afterlife. The obvious problem is that if universalism is wrong and the
person hopes to be saved in the next life, he'll face an eternity of hell instead of heaven. This is an
inherent weakness in Universalism.
CARM has many discussion boards. One of them is on universalism. I have raised this issue to
them many times and two responses have come forth from: one is to ignore the point all together and
not admit it exists. The other is to ignore the issue and counter attack. The most prevalent
counterattack against me is a challenge regarding election (I'm Reformed in theology) and ask that
since God's elect cannot be deceived, how can universalism pose a threat to the elect?
In part, the universalists deflection of the real question is understandable since they desire to
defend their position at all costs, a consistent pattern with them. But, what is amazing is their
inability to admit the possible danger in their position. To them, it is quite impossible to consider
anything in universalism to be less than perfect. This is disconcerting.
Nevertheless, I'll answer their objection, even though they refuse to address mine, in hopes of
encouraging them to actually face the issue and admit the danger.

Reformed theology teaches that God elects, from all eternity, those who will be saved and that this
election cannot fail; those who are elected to salvation will be saved and only those who wanted to be
saved are elected to be saved. Likewise, it is not possible for the elect to lose their salvation since the
cross has made it secure.
Universalism can contribute to people procrastinating regarding salvation in the here and now in
order to wait until the after-life where they have a second chance, an opportunity to be purified in a
hell-like state after which they will then be able to go to heaven and be with God forever. Of course, if
universalism is wrong, then those who had erringly put their hope in Universalism's second-chance-in-
the-afterlife-belief would be lost forever. The question then, if election is true, does it really matter
regarding universalism's truth or error?
Yes it does. First, God does not want error to be taught, regardless of election or not.
Universalism is an error.
Second, both election and warning against false teachings are taught in the Bible. Therefore, there
is no contradiction since God's word teaches both.

1. Election:
A. "And unless those days had been cut short, no life would have been saved; but for the
sake of the elect those days shall be cut short," (Matt. 24:22, cf., Mark 13:20).
B. " . . . so as to mislead, if possible, even the elect," (Matt. 24:24, cf., Mark 13:22).
C. "And He will send forth His angels with a great trumpet and they will gather together
His elect from the four winds, from one end of the sky to the other," (Matt. 24:31, cf.,
Mark 13:27).
D. "now shall not God bring about justice for His elect, who cry to Him day and night, and
will He delay long over them?" (Luke 18:7).
E. "Who will bring a charge against God’s elect? God is the one who justifies," (Rom.
8:33).
2. Warning against false teaching
A. "Beware of the false prophets, who come to you in sheep’s clothing, but inwardly are
ravenous wolves," (Matt. 7:15).
B. ". . .See to it that no one misleads you. "For many will come in My name, saying, ‘I am
the Christ,’ and will mislead many," (Matt. 24:4-5; cf, Luke 21:8).
C. See also Acts 20:29; 2 Cor. 11:13; Eph. 4:14

Therefore, for the universalist to think that the two issues are contradictory in any way is
unwarranted because the Bible teaches both concepts.
"The subject of election is God, who chooses on the basis of his sovereign will for his creation.
Associated with election are theological terms such as ‘predestination,’ ‘providence,’ and
‘covenant.’"112 The elect are God's chosen people who were set apart from the foundation of the
universe to be saved, to belong to God. So, if this is true how can I consistently warn people against
the errors of universalism? Actually, if the universalists wanted to be consistent, why would I warn
anyone about anything? Shouldn't I just sit around and do nothing because God will take care of it all
regarding the elect? The answer is simple. God ordains the means as well as the ends in election. He
uses Christians, freed from sin, to do His sovereign will according to His command to refute error and
make disciples.

• "Holding fast the faithful word which is in accordance with the teaching, that he may be able
both to exhort in sound doctrine and to refute those who contradict," (Titus 1:9).
• "Go therefore and make disciples of all the nations. . . " (Matt. 28:19).
• "For truly in this city there were gathered together against Thy holy servant Jesus, whom Thou
didst anoint, both Herod and Pontius Pilate, along with the Gentiles and the peoples of Israel, 28
to do whatever Thy hand and Thy purpose predestined to occur" (Acts 4:28).

Obviously God wants people to refute error, to make disciples, yet it is He who predestines people.
How does this work? I am not sure. But God has it under control. Furthermore, we do not know the
criteria by which God elects, but elect He does. The Bible teaches that plainly. It is not, however,
based in anything good in us for there is nothing good in us; we are sinners. That is why God shows
no partiality; that is, He does not elect based on anything in us.
Also, God says the prayers of a righteous man (Christian) can accomplish much with Him (James
5:16). How can that be if He predestines all things (Eph. 1:1-11; Rom. 8:28-29)? I don't know. But,
He predestines and He instructs us to pray because it makes a difference with Him. How does that
work? Again, I do not know and I cannot explain the mind of God. But, God predestines, commands
us to be careful about false teaching, and instructs us to go make disciples?
Is it contradictory to say that God elects and that our prayers can influence Him? Not at all, since
both are true in scripture. Furthermore, I absolutely believe that God predestines (Eph. 1:1-11; Acts
4:28) and I believe my prayers and efforts (i.e., CARM) make a difference. In fact, I am far more
evangelical than most people: I've literally got thousands of hours in evangelism experience,
preaching, teaching, witnessing, apologetics, web, radio, etc, and I have many testimonials how the
efforts have made a difference in peoples' lives. This all goes to prove one very important point: God
ordains the means as well as the end.
It isn't up to me to figure out all the nuances of God's mind and how all of it fits together. I leave
that to Him and I go forth in obedience to His declaration (predestination/election) and His command
(make disciples).

A possible explanation

Alright, so I've pleaded ignorance, in part, on this subject. Nevertheless, I offer a possible
explanation of how this can work.

1. God's total knowledge eternally encompasses all actions of all beings and all possible and
actual permutations of all events of all things.
A. This means that from before the universe existed, in the mind of God, all potential
existences and all potential combinations of all events were already known and
understood by God; hence, true omniscience.
i. This would include all things done by "free will" creatures anytime and anyplace in
the then distant future under all actual and potential circumstances.

112
Achtemeier, Paul J., Th.D., Harper’s Bible Dictionary, (San Francisco: Harper and Row, Publishers, Inc.) 1985.
2. Since all things have eternally been simultaneously known by God (even the "free will"
choices made by all people), then, when He planned the universe, it is necessarily true that
all things which were known and consulted by God were included in His plan of election as
He placed people in and where He did according to His sovereign will to "do whatever Thy
hand and Thy purpose predestined to occur" (Acts 4:28).
A. It cannot be that God remains ignorant on any subject. When He created the universe
He did so knowing everything that could and would be. Since He is all powerful, it is
natural to state that He included all possible outcomes in His sovereign plan, including
election. This election may or may not be influenced by foreknowledge concerning our
actions and desires.
B. This plan can easily include our prayers and the resulting ramifications of the teaching
of false doctrines that damn people.
C. Our prayers can, then, have an influence with Him from all eternity, yet, He has not
changed from all eternity. The only way that this can be is if He knew everything about
everything and included what He desired in the universe that He constructed and set in
motion.
3. It is perfectly logical, therefore, that in our time reference, we can make choices, influence
God, be warned about apostasy and false doctrines, and actually truly make a difference in
people's lives (for good or bad), and have all these things sovereignly included in the plan of
God by which He elects and predestines. This way, the idea of a false doctrine damning and
God's election are not mutually exclusive.
A. In other words, God knew all that what we would do (good and bad) and took it all into
account when He constructed the universe and put us in it and developed His sovereign
plan by which He has predestined what will occur. After all it says that ". . .He chose us
in Him before the foundation of the world, that we should be holy and blameless before
Him. In love He predestined us to adoption as sons through Jesus Christ to Himself,
according to the kind intention of His will," (Eph. 4:5). Is this predestination apart from
His infinite knowledge of our influence upon Him (James 5:16) from all eternity? Of
course not.
B. Therefore, what we do makes a difference in the lives of people. False doctrines are
still dangerous and are to be warned against -- the outcome, of which, are included in
God's sovereign plan of election. He still predestines, we still can influence God in our
prayers (James 5:16) and God's warning about apostasy and false doctrines are still
valid.

Objections answered

But, some universalists will object that the doctrine of election can cause the very same thing of
which universalism is accused; namely, a false sense of security in salvation. Certainly, this is a
possibility. But, the Bible says that God elects (Matt. 24:22,31), that His sheep will never perish (John
10:28), that those who say they know Jesus and do not keep His word are liars without the truth (1
John 2:4), and that the Christian is a new creature in Christ (2 Cor. 5:17): " Therefore if any man is in
Christ, he is a new creature; the old things passed away; behold, new things have come." It is this
last verse that is extremely relevant here.
Being a new creature in Christ means that the Christian is no longer a slave of sin (Rom. 6:16). He
is now free. A true Christian will not use the grace of God to sin (Rom. 6:1-3) and expect to be saved
no matter what. Why? Because a true Christian is at war with the flesh (Rom. 7) and does not remain
in sin (John 8:31; James 2:14).
But, the very fact that the universalists bring up this counter-argument is an admission of the
weakness of their own for they recognize it and try to apply it to a different situation as a means of
defense. In so doing, they do not answer the original problem; they only change the subject and
hope the problem will go away.
Also, election deals with Christians, universalism deals with Christians and unbelievers. The
Christian is regenerated and changed and will not abide in sin; he cannot because he is not a slave to
it anymore and will not use his security in Christ as a license to sin. The unbeliever is not saved and
can still hope in eternal life after death so they can sin now. And it is to this charge that the
universalist has failed to adequately respond.
Another objection raised by universalists is how do you know if you are elect or not? This question
demonstrates a lack of understanding of biblical theology. The Bible says that "These things I have
written to you who believe in the name of the Son of God, in order that you may know that you have
eternal life" (1 John 5:13). It is certainly possible, according to God's word, to know that you are elect.
On a practical level, I would say that anyone who has trusted Jesus as Savior (not a false Christ as in
the cults), will have the knowledge and testimony of the Holy Spirit living within him and he will
observe a change in his own heart. This is a demonstration of regeneration. Furthermore, only
Christians have the mind of Christ and know that Jesus is God in flesh, risen from the dead, and
ascended into heaven to which the Bible says, an unbeliever cannot accept (1 Cor. 2:14).

Note:
______________
Infralapsarianism is an issue within Reformed theology dealing with what may have happened in
God's mind regarding the logical order of His considering whom to elect into salvation before the
foundation of the world. The word means "after the fall." The position is that God first decided He
would allow sin into the world and second that he would then save people from it. By contrast, the
supralapsarian ("before the fall") position holds that God first decided that He would save some people
and then second that he would allow sin into the world.
I am somewhat bewildered by this difference within Reformed circles since all knowledge with God
is eternally known and simultaneous. It would seem that the distinctive between infra and
supralapsarianism would be moot.
Satan and universalism

Satan wants the destruction of all people. Satan wants people to die in their sins and go to hell.
He is utter hatred and complete evil. But, he is also extremely cunning with an intelligence that is vast
and it may just be that universalism will become a tool of the Devil in the last days. How? By
teaching that people can trust in Christ in the after-life, a second chance after death.
Now, am I calling Universalists satanists? Not at all. Many of them are very godly people and
serve the true God. But, whether Universalists admit it or not, the doctrine has the potential of
negating the need to come to Christ in this life because it teaches you can do it in the next -- as they
have claimed.
In Universalism, as well as most any doctrine, Satan can work various false teachings through its
adherents. No doctrine is immune to being twisted. But, many Universalists deny the Trinity and the
deity of Christ yet draw people in via universalist philosophy. This is dangerous. All the cults teach a
false god and are false because of it. But in universalist theology, it really doesn't matter. Why?
Because according to universalism, in the after-life, people will come to a true knowledge of God and
repent and be saved. So, even if they are wrong now, they will be right later. So, the need to preach
and teach the true God (Trinitarian) is greatly reduced. Again, this is dangerous.
Satan says, "Don't worry about receiving Jesus now. You can do that later." Then when people
die, they find out it is too late. But, it is God who says,

• “At the acceptable time I listened to you, And on the day of salvation I helped you” ;behold,
now is “the acceptable time,” behold, now is “the day of salvation" (2 Cor. 6:2, NASB) -
emphasis added.
• "He again fixes a certain day, “Today,” saying through David after so long a time just as has
been said before, “Today if you hear His voice, Do not harden your hearts" (Heb. 4:7, NASB)

Some Universalists will take offense at this line of reasoning. I don't blame them. But, the Bible
teaches that now is the time for salvation, not later. Some Universalists actually preach a God of love
and forgiveness and the need to come to Christ now. That's fine as far as they preach the True Christ
and Him crucified and not a false god as the Mormons, Jehovah's Witnesses, and New Age adherants
do. But what about the thousands of Universalists who deny the true God and yet teach universalism
and continue to teach a false god? If Universalism is not true, then they are hindering people from
coming to Christ. They are advocating a false god along with their universalist teachings, the very
universalist teaching that appeals to people because it teaches a God of infinite love and grace.
To deny the Trinity is to deny the true God. To deny that Jesus is God in flesh is to deny the true
Christ. This is incredibly dangerous because false gods do not save, only the True and Living God
saves! But then, in universalism, it doesn't matter what you believe now. You'll "repent" later in the
after-life and come to the true knowledge of God. Sorry. But that is not what the Bible teaches.

One world government

The Bible teaches us that in the last days there will be a one world religious system which will be
used by the devil in a great deception to ultimately war against God. Universalism could easily be
used by the devil to help him accomplish this very thing. You see, there are a great many differences
between religious systems. They cannot all be right. Because of these differences, hostilities often
arise. But, if universalism were accepted by all religious systems, then it would logically greatly
reduce hostilities both physical, intellectual, and spiritual. Sound good? It sure does, except for one
thing. It compromises the truth of the saving gospel of Jesus Christ who said He alone was the way
the truth and the life. This means that all other systems are wrong! It means that Buddhism is not
true. It means that Islam is not true. It means that only Christianity is true. There is no fellowship
with darkness. Christians need to stand on God's word of truth which includes the narrow path to God
of which not all will find it (Matt. 7:13-14), not man's word of "God will save everyone."
Universalist philosophy, if accepted by the worlds' religions, would greatly reduce the focus of
differences between them. It would greatly reduce the need for orthodoxy, for defining and preaching
the true God. But, the Bible teaches that Jesus alone is the savior and that now is the acceptable time
of salvation... not the after-life. It teaches that there is one God, not many; that Jesus is God in flesh,
not an angel or a "good man." Truth is at stake.
How many people will be deceived by universalism's comfortable message that all will be saved?
Who knows. Universalism in itself is not a deception, just an error. But, how many will take refuge in
its comforting doctrine and not come to a saving truth of Jesus? I can not say, but even one is too
many.
I cannot help but notice that the secular world teaches tolerance, be-nice philosophy, and the belief
that all will make it to heaven. Second chance redemption and a theology that teaches you'll make it
no matter what is comfortable to the world's ears. Too bad the universalists agree with the world in
this.
A Challenge to Universalists
The Bible says to "test yourselves to see if you are in the faith," (2 Cor. 13:5). This is something
that we must do. We must not carelessly assume that we know all that is true in the Bible. If
universalism is true, fine. But if it is not, then the eternal consequence of damnation is of utter
importance. Therefore, I issue this challenge to any who claim to be Universalists. Are you a
Christian?
Of course, some of you will claim that you are and it is not my place to judge you. God is the
Judge and He has revealed what His will is in the Bible about what false doctrines disqualify someone
from being a Chrsitian. Therefore, it is from God's word that I challenge you. This challenge is not
about the truth or error of universalism. It is about who Jesus is. Do you believe He is God, the
creator of the universe, worthy of all worship and honor, equal to the person of the Father? If yes,
good. If not, then you desperately need to examine yourself to see if you are in the faith because to
deny this means you are not a Christian.
Jesus said, "I said therefore to you, that you shall die in your sins; for unless you believe that I am
He, you shall die in your sins," (John 8:24, NASB). The word "He" is not in the Greek. It literally says,
"for unless you believe that I am, you shall die in your sins." Later in this same chapter in verse 58,
Jesus said, "before Abraham was, I AM." He was alluding to Exodus 3:14, where God told Moses that
His name was "I am that I am."
Likewise, in 1 John 4:2-3 it says, "This is how you can recognize the Spirit of God: Every spirit that
acknowledges that Jesus Christ has come in the flesh is from God, but every spirit that does not
acknowledge Jesus is not from God. This is the spirit of the Antichrist, which you have heard is coming
and even now is already in the world." Very few people deny that Jesus lived; that is, that Jesus came
in flesh. When John wrote this, he was not saying that you must believe that Jesus lived, but that
Jesus was God in flesh.
The time of the writing of First John is important. The Gnostic heresy was prominent. It taught that
God was too pure to have anything to do with sinful flesh. Therefore, Gnosticism taught that Jesus
could not be God in flesh. It was in this context and against this error that John was writing. Jesus is
God in flesh and to deny it is the Spirit of Antichrist.
Furthermore, the above verse needs to be cross referenced with John 1:1,14 (also written by John)
where he states that the Word was God and the Word became flesh and dwelt among us. In 1 John 4,
the apostle knew what he was writing when he spoke of Jesus being in the flesh.
The Bible states that there is only one God (Isaiah 43:10; 44:6; 44:8). It states that Jesus created
all that exists: "For by Him all things were created, both in the heavens and on earth, visible and
invisible, whether thrones or dominions or rulers or authorities—all things have been created by Him
and for Him. 17And He is before all thins, and in Him all things hold together," (Col. 1:16-17). Jesus is
the creator.
In Isaiah 44:24, it says, "Thus says the Lord, your Redeemer, and the one who formed you from
the womb, "I, the Lord, am the maker of all things, Stretching out the heavens by Myself, and
spreading out the earth all alone." This verse shows us that the Lord God created the universe -
alone. If God created the universe alone and if Jesus created all things, then Jesus is God. Jesus is
fully and completely God in flesh, second person of the Trinity. Col. 2:9, "For in Him all the fullness of
Deity dwells in bodily form."
And again, in John 5:22-23, Jesus said, "For not even the Father judges anyone, but He has given
all judgment to the Son, 23 in order that all may honor the Son, even as they honor the Father. He who
does not honor the Son does not honor the Father who sent Him."
Why is all this so important? Because it is Jesus who reveals the Father (Matt. 11:27; Luke 10:22),
who sends the Holy Spirit of truth (John 15:26), and who opens the mind to understand scripture
(Luke 24:45). If you do not have the true Jesus, you will not know the true God. You will not know
the true Holy Spirit. You will not have your mind opened to understand God's word. If you do not have
the true Christ, then you simply are not a Christian. So I challenge you, do you believe Jesus is God
in flesh, worthy of the same honor has Father, the creator of all, the risen Lord? If you cannot say yes,
then I suggest to you that you are not a Christian and need to re-examine your beliefs, regardless of
whether or not universalism is true. Furthermore, if you deny that Jesus is God in flesh and just if
universalism is false, then you would be in deep trouble come judgment day. Don't put your hope in
universalism. Put it in Jesus who is God in flesh.
What is Unitarianism?

Unitarianism is the belief that God exists in one person, not three. It is a denial of the doctrine of
the Trinity as well as the full divinity of Jesus. Therefore, it is not Christian. There are several groups
that fall under this umbrella: Jehovah's Witnesses, Christadelphianism, The Way International, etc.
Another term for this type of belief is called monarchianism.
In the context of universalism, the Unitarianism discussed here is that belief that denies the Trinity,
the deity of Christ, the personhood of the Holy Spirit, eternal punishment, and the vicarious atonement
of Jesus. Unitarian universalists use many biblical concepts and terms but with non-biblical meanings.
Unitarianism is not Christian.
There is a group known as the Unitarian Universalists Association. This denomination which was
formed in 1961 in the United States when the American Unitarian Association and the Universalist
Church of America merged. Its membership is around 175,000.
The General Convention of the Unitarian Universalists formulated the five principles of the
Universalist Faith in 1899.

• The Universal Fatherhood of God


• The spiritual authority and leadership of His Son Jesus Christ
• The trustworthiness of the Bible as containing a revelation from God
• The certainty of just retribution for sin
• The final harmony of all souls with God

Additional beliefs generally held by Unitarian Universalists are:

• Salvation is by grace through faith and not by works in any way.


• Jesus became the Son of God at His baptism.
• The Holy Spirit is not a person, does not have a will, etc.
• There now is and will be rewards and punishments according to one's actions but this does not
consist of the traditional doctrine of hell.
• Human reason and experience should be the final authority in determining spiritual truth.

This last point, "Human reason and experience should be the final authority in determining
spiritual truth," is perhaps the most revealing of the character of Unitarian Universalists. Instead of
God and his word being the final authority on truth and error, or right and wrong, Unitarian
Universalists subject God and his word to their understanding, feeling, and reason. This is exemplified
in the following quote obtain from the official Unitarian Universalist website at http://uua.org/. This
was found under the heading Unitarian Universalists say:

• "I want a religion that respects the differences between people and affirms every
person as an individual."
• "I want a church that values children, that welcomes them on their own terms—a
church they are eager to attend on Sunday morning."
• "I want a congregation that cherishes freedom and encourages open dialogue on
questions of faith, one in which it is okay to change your mind."
• "I want a religious community that affirms spiritual exploration and reason as
ways of finding truth."
• "I want a church that acts locally and thinks globally on the great issues of our
time—world peace; women's rights; racial justice; homelessness; gay, lesbian, bisexual,
and transgender rights; and protection of the environment."

Notice that each of the five statements begins with "I want..." This is not the humble attitude of
one indwelt by the Holy Spirit of God. It is not the attitude of one who wants to put God first.
It can plainly be seen that this is a religion based upon personal hopes and desires and not upon
the Bible.
I cannot help but think of the five "I will's" listed in Isaiah 14:13-14:

"But you said in your heart,


I will ascend to heaven;
I will raise my throne above the stars of God,
I will sit on the mount of assembly in the recesses of the north.
I will ascend above the heights of the clouds;
I will make myself like the Most High.’

Many commentators believe that these five "I wills" were uttered by Satan as he sought to be
exalted and equal to God. They reflect the arrogance of the evil one as his heart was filled with pride
and put his own will before God's. He had his desires before God's.
But notice what Isaiah says in the next verse:

"Nevertheless you will be thrust down to Sheol, to the recesses of the pit."

Jesus said, "Out of the abundance of the heart, the mouth speaks," (Matt. 12:34). We can see that
the Unitarian Universalists speak first from their own desires, according to their own wisdom, and not
according to the wisdom of God. What does God say about this?

"For the wisdom of this world is foolishness before God," ( 1 Cor. 3:19).
Concluding thoughts on Universalism

The teaching that God will forgive all people of all their sins is an appealing doctrine. It is
comforting to think that no one will go to hell forever-- especially ourselves. It means that we will
escape the judgment of damnation. It means we are safe even in our imperfections, our sins, our
rebellion, and our blasphemies. It means we can offend God outright, reject Him boldly, and not worry
about our salvation -- because we'll all be saved no matter what they do in this life.
On the other hand, if there are people going to hell, then it means that there is a God who holds
them eternally accountable for their actions. It means there is absolute truth concerning
condemnation. It means there exists a God who punishes sinners who reject God and separates them
from His own infinite holiness. Down deep inside this can make us uncomfortable and worried. Such
an idea of accountability might force us to examine ourselves and ask, "Am I saved?" "Am I going to
heaven?" "Have I offended God?" "Will I be punished?" "What am I really like deep, down inside?"
The answers to these questions can make us feel guilty, uncomfortable, and even worried, especially
when we compare ourselves to a Holy God.
In this world of "tolerance," diluted absolutes, and creature comforts, the idea that all people will
be forgiven fits right in. Universalism is a theology of tolerance, of ease, and comfort. It feels good.
Psychologically it can ease our conscience because if we, in the goodness of our hearts, are wishing
the forgiveness of all, doesn't it mean that we too will receive forgiveness due us because of our
merciful desire towards others? Many people think this way and somehow hope that because of their
own good will towards others, they will receive it themselves.
It is not comfortable, nor does it make us feel calm and relaxed to think that there is an infinitely
Holy God who takes sin seriously and punishes sinners. It can be terrifying to be faced with an eternity
of hell fire if you have not made yourself right with God. And such is the complaint of the universalist:
God is love and in Him there is no fear of eternal damnation.
The universalists are often guilty of pick-and-choose theology. See the papers on Matt. 25:46 and
"A look at the word aionion" as examples of how they misuse Greek meanings of words. In addition, I
have encountered many universalists who have stated that they adopted universalism because they
did not like the idea of eternal damnation. In essence, to do this this is to adopt a theology based on
feelings and this is wrong.
God punishes sinners (Matt. 25:46). Why? Because He is Holy (Isaiah 6:3; Rev. 4:8). His eyes are
too pure to look upon evil (Hab. 1:13). Is He love? Yes, He is (1 John 4:8, 16). But that isn't all He
is. He is also just (Neh. 9:32-33; 2 Thess. 1:6) and must punish sinners because sin is an offense
against Him and sin separates us from Him (Isaiah 59:2). In His love He sent the Son to die for us.
For those who reject Christ, God will be just and punish them.
Hell was not made for people. It was made for the devil and his angels who rebelled against God
(Matt. 25:41). But hell will also house those who reject God's provision for salvation and side with the
evil one (Matt. 18:8; 25:46). This is a sad reality.
Will Satan too be saved according to the Universalists? No. Will the evil people who commit the
most horrendous of crimes and who have blasphemed the name of God be allowed to escape their
judgment even after openly rejection the Lord's sacrifice? Does God simply say, "It is okay for you to
reject me, my Son, the Sacrifice, the Agony on the cross. It does not matter about your blasphemies
against Me. It does not matter that you have given yourself over to evil. I will save you after a period
of chastening in the afterlife. Enter into My rest and enjoy eternal bliss. All are saved." No, this is not
so.

"How much severer punishment do you think he will deserve who has trampled under foot the
Son of God, and has regarded as unclean the blood of the covenant by which he was sanctified,
and has insulted the Spirit of grace?" (Heb. 10:29).
The teaching of Universalism minimizes the Infinite Holiness and Infinite Justice of God which also
resides within His very essence alongside Infinite Love. It does this by daring to assert that anyone,
in the afterlife, through any form of suffering, are somehow "made ready" to be with God. That is
false! Hell is not a pleasant topic. It is an awful place. But it is real and it is powerful and it is
eternal. No one will escape the judgment of God if they forsake Christ in this world.
God gave hell its power. The power of sin is the Law (1 Cor. 15:56). To sin is to offend God and to
go against His word, His very nature. The Law is God's word. He said, "Thou shalt not...." Jesus said
that out of the abundance of the heart, the mouth speaks. God spoke the Law into existence out of
the very nature of His own heart. Therefore, to go against the Law is to go against God and to offend
Him. He is infinite. The offense is infinite.
He is just and is obligated to punish sin and, hence, the sinner.
He is love and graciously provided His Son to redeem those who would be His.

Universalism makes the latter quality of God override the other having the sinner escape eternal
judgment by going through a period of suffering in the afterlife. This is wrong. When such an
imbalance occurs, error is the result. And that is what universalism is: error. Its danger is that it
may cause the heart to be comfortable, to not worry, and to put off seeking a savior. Such a doctrine
is dangerous since it can easily encourage a casual approach to redemption.
Roman Catholicism
Introduction

The Roman Catholic Church is a massive organization encompassing 1 Billion people. But, is it biblical
in all its ways or has tradition crept into the Catholic Church and weakened the gospel?

1. Why is it necessary to examine the Roman Catholic Church? p. 96


2. What status does the Catholic church give to tradition? pp. 97-99
3. Why is the Bible alone sufficient for spiritual truth? pp. 100-101
4. What is absolution? assumption? Immaculate conception? transubstantiation? pp. 102-103
5. What are some of the title used to describe Mary by the Catholic Church? p. 104
6. Did Mary have other children? p. 105
7. Does Purgatory Deny the Sufficiency of Christ’s Sacrifice? p. 107
8. What is the Roman Catholic view on justification? pp. 114-117
9. Is the Catholic Catechism's view on the Muslim god wrong? Why? p. 119
Why is it necessary to write about Roman Catholicism?
It is necessary to write a page on Roman Catholicism because there are significant differences
between Protestant and Roman Catholic doctrines. Protestants accuse the Catholics of being
unscriptural and the Catholics state that the Protestants do not have the true faith carried through the
centuries by the Catholic Church. Whichever side you fall on, the real issue is whether or not the
Roman Catholic Church is representing true Christianity.
If you are a Roman Catholic, please understand that this site is not meant to offend you in any
way. This site is dedicated to examining the truth, all truth, and compares it to the Bible. No matter
who it is or what group is proclaiming truth, we know that the Bible is the Word of God and that no
truth from God will contradict it. I urge you to examine what is here on this site and see if what is
being taught here is true. If you do, you would be doing precisely what the Bible commands: to
examine all things (1 Thess. 5:21; 1 Pet. 3:15; 2 Tim. 2:15; Acts 17:11; Jude 3). Since the Catholic
Church holds the Bible to be the word of God, I am putting Catholic doctrine to the biblical test.
If you are a Protestant, you will probably feel more comfortable browsing these pages since they
will, essentially, confront Roman Catholic teachings. It is apparent, that the Roman Church has added
much to the scope of Christian doctrine that is not revealed in scripture. This is an issue that needs to
be addressed. It is vital.
The Protestant Church cites the Bible alone as the source of doctrinal knowledge. The Catholic
church, on the other hand, cites the Bible and Tradition. Please consider the following.

". . .the Church, to whom the transmission and interpretation of Revelation is entrusted, ‘does
not derive her certainty about all revealed truths from the holy Scriptures alone. Both Scripture
and Tradition must be accepted and honored with equal sentiments of devotion and
reverence'." (Catechism of the Catholic Church, paragraph 82.)

Apparently, it is Tradition that is the source of doctrines which are clearly not taught in the Bible
but which the Catholic Church still says are implicit within its text and elucidated through Apostolic
Tradition. Some of them are as follows: The Mass, Penance, Veneration of Mary, Purgatory,
Indulgences, the Priesthood, the Confessional, the Rosary, Venial and Mortal Sins, and statues in the
Church. The issue is whether or not these teachings of the Roman Catholic Church are credible. Do
they accurately represent Christianity? Can they be substantiated with the Bible? Do they contradict
the Bible?
This site attempts to examine the doctrines of Rome and compare them with the Bible to see if
they are supported or contradicted by God’s Word. We know that all truth in God’s Christian Church
comes from Him, through the Holy Spirit. It will not, therefore, be contradictory. Let us see what God’s
written word says and compare it to the Unwritten Word which is the Roman Catholic Church’s term for
Tradition.
Roman Catholicism, the Bible, and Tradition
One of the great differences between Protestant and Catholic doctrine is in the area of Tradition.
The Protestant church maintains that the Bible alone is intended by God to be the source of doctrinal
truth (2 Tim. 3:16). The Catholic Church, however, says, "Sacred Tradition and Sacred Scripture make
up a single sacred deposit of the Word of God . . ." (Catechism of the Catholic Church, paragraph 97.
Note, all citations in this article are from this Catechism).
The Catholic Church reasons thus:

1. "The apostles left bishops as their successors. They gave them ‘their own position of
teaching authority.'" (Paragraph 77)
2. "This living transmission, accomplished through the Holy Spirit, is called tradition..." (Par.
78)
3. "Both Scripture and Tradition must be accepted and honored with equal sentiments of
devotion and reverence." (Par. 82).

Within the Catholic scope of Tradition, many doctrines have been "revealed" to the Church over
the centuries. For example, there is the veneration of Mary, her immaculate conception and her bodily
assumption into heaven. There is also the apocrypha, transubstantiation, praying to saints, the
confessional, penance, purgatory, and more. Protestantism as a whole differs with Catholicism in these
additions.

Tradition in the Bible

The Bible speaks about tradition. Some verses speak for tradition and others speak against it. Of
course, the contexts are different and carry different meanings. For example:

For Against
2 Thess. 3:6, "Now we command you, Matt. 15:3-6, "And He answered and said
brethren, in the name of our Lord Jesus to them, And why do you yourselves
Christ, that you keep aloof from every transgress the commandment of God for
brother who leads an unruly life and not the sake of your tradition? 4For God said,
according to the tradition which you Honor your father and mother, and, He who
received from us." speaks evil of father or mother, let him be
put to death 5But you say, Whoever shall
say to his father or mother, Anything of
mine you might have been helped by has
been given to God, 6he is not to honor his
father or his mother. And thus you
invalidated the word of God for the sake of
your tradition."
2 Thess. 2:15, "So then, brethren, Mark 7:8-9, "Neglecting the
stand firm and hold to the traditions commandment of God, you hold to the
which you were taught, whether by tradition of men. 9He was also saying to
word of mouth or by letter from us." them, You nicely set aside the
commandment of God in order to keep your
tradition."
1 Cor. 11:2, "Now I praise you because Col. 2:8, "See to it that no one takes you
you remember me in everything, and captive through philosophy and empty
hold firmly to the traditions, just as I deception, according to the tradition of
delivered them to you." men, according to the elementary principles
of the world, rather than according to
Christ."
In the discussions regarding Tradition between Protestants and Catholics both sets of scriptures
are often quoted in order to establish their respective positions. The Protestants often quote Matt.
15:3-6 in opposition to Sacred Tradition. In an appeal to be biblical, many Catholic apologists cite 2
Thess. 2:15 to validate their position on Sacred Tradition. Unfortunately, this amounts to using the
Word of God against itself. Clearly, God's word is not contradictory. Rather, it is our understanding
that is in error.
The Bible is for tradition where it supports the teachings of the apostles (2 Thess. 2:15) and is
consistent with biblical revelation. Yet, it is against tradition when it "transgresses the commands of
God" (Matt. 15:3). By Jesus' own words, tradition is not to transgress or contradict the commands of
God. In other words, it should be in harmony with biblical teaching and not oppose it in any way.
Though the Catholic Church officially states that Sacred Tradition should not and does not
contradict Scripture, Protestants see much of the teaching from this Sacred Tradition as doing just
that. It isn't enough for the Catholic to say that their church is the true church, that they have the
apostolic tradition, that they hold the keys to the truth, and that they have revealed doctrines
consistent with biblical revelation. Likewise, it isn't enough for a Protestant to pass judgment upon
Catholic doctrines simply because they are Catholic and are derived via Sacred Tradition.

Are Sacred Scripture and Sacred Tradition Really Equal?

To me, it is not enough to simply say that Sacred Tradition is equal to Scripture based upon the
decree of the Catholic Magesterium. Like any spiritual teaching, I must compare it to the Bible. Jesus
own words in Matt. 15:3 lend support for myself and many non-Catholics to subject the fruit of Sacred
Tradition to the pruning of God's word. In other words, do the teachings of the Catholic church that are
derived through tradition transgress the commands of God? Of course, the Catholic will say that they
do not.
When Jesus was speaking to the Pharisees in Matt. 15:1-6, He reprimanded them for not
understanding God's word. They were appealing to the tradition of the elders, those who had passed
down oral and written tradition. Jesus, on the other hand, exposed their error by citing scripture.
Please take note of what He said in Matt. 15:1-6.

"Then some Pharisees and scribes came to Jesus from Jerusalem, saying, 2"Why do Your
disciples transgress the tradition of the elders? For they do not wash their hands when they eat
bread." 3And He answered and said to them, "And why do you yourselves transgress the
commandment of God for the sake of your tradition? 4"For God said, ‘Honor your father and
mother,' and, ‘He who speaks evil of father or mother, let him be put to death.' 5"But you say,
‘Whoever shall say to his father or mother, "Anything of mine you might have been helped by
has been given to God," 6he is not to honor his father or his mother.' And thus you invalidated
the word of God for the sake of your tradition."

Whatever might be said about this passage, at least one thing must be observed: The tradition of
the religious leaders was subject to the Word of God. Are the religious leaders of the Catholic Church
exempt from subjection to the Word of God? And likewise, is their Sacred Tradition also exempt? I
think not.
Where the Protestants would interpret Tradition in light of Scripture, it seems that the Catholic
Church does the opposite. Consider the following, "The Second Vatican Council indicates three criteria
for interpreting Scripture in accordance with the Spirit who inspired it. 1. Be especially attentive ‘to the
content and unity of the whole Scripture.'. . . 2. Read the Scripture within ‘the living Tradition of the
whole Church.' . . . 3. Be attentive to the analogy of faith." (Par. 111, 112, 13, 114).
It is number 2 that is the main concern here. What does it mean to read Scripture "within the
living Tradition of the whole Church?" If Scripture is "within the living Tradition," then Tradition
encompasses Scripture. In other words, it is the tradition of the Church that interprets Scripture. This
is in contradiction to the Word of God spoken by Jesus in Matt. 15:1-6.
Some object and say that the Pharisees didn't have apostolic authority and succession that was
ordained by the apostles as does the Catholic Church and, therefore, Matt. 15:1-6 cannot be used to
nullify Sacred Tradition.
But the issue in Matt. 15:1-6 is not succession of authority but the traditions of men being used in
opposition to the truth of the Word of God. Essentially, the Pharisees were seeing the Word of God
"within" their sacred tradition. Jesus, in contrast to this, cited the Word of God to judge their
traditions. The apostles, likewise, continuously admonished their people to check their teaching against
the Scripture (Acts 17:11), thereby substantiating the position that even what they taught was subject
to God's Word. After all, no doctrinal teaching should contradict biblical revelation and the Sacred
Word of God was and is the final authority in all things spiritual. The Catholic Church's position and
teaching is based on Sacred Tradition are no different. They must be compared to Scripture.
My desire in writing this is not to alienate Catholics nor belittle their beliefs. I believe that there
are some Catholics who love the Lord and are saved. But I would like to add that I believe it is in spite
of official Roman Catholic doctrine. Nevertheless, it is my opinion that the Catholic church has added
teachings that are not consistent with biblical revelation.
If you are a Catholic, I hope my words do not offend you. Rather, I hope and pray that you would
consider what this site has to say and compare it with the Word of God.
Is the Bible Alone Sufficient for Spiritual Truth?

According to Roman Catholicism, Sacred Tradition and the Bible together provide the foundation of
spiritual truth. From this combination the Catholic church has produced many doctrines which it says
are true and biblical. Protestantism, however, rejects Roman Catholic Sacred Tradition and holds fast
to the call "Sola Scriptura," or, "Scripture Alone." Catholics then challenge, "Is Sola Scriptura biblical?"
The Bible does not say "Do not use tradition" or "Scripture alone is sufficient." But the Bible does
not say "The Trinity is three persons in one God," either, yet it is a fundamental doctrine of
Christianity. 2 Tim. 3:16 says that scripture is inspired and profitable for correction and teaching.
Scripture states that Scripture is what is good for correction and teaching, not tradition. However, in
its comments on tradition, the Bible says both to listen to tradition as well as warning about tradition
nullifying the gospel -- which we will look at below.
In discussing the issue of the Bible alone being sufficient, several points should be made:

1. The method of the New Testament authors (and Jesus as well) was to appeal to the Scriptures as
the final rule of authority. Take, for example, the temptation of Christ in Matthew 4. The Devil
tempts Jesus, yet Jesus used the authority of scripture, not tradition, nor even His own divine
power, as the source of authority and refutation. To Jesus, the Scriptures were enough and
sufficient. If there is any place in the New Testament where the idea of extra-biblical revelation
or tradition could have been used, Jesus' temptation would have been a great place to present it.
But Jesus does no such thing. His practice was to appeal to scripture. Should we do any less
having seen His example? The New Testament writers constantly appealed to the scriptures as
their base of authority in declaring what was and was not true biblical teaching: Matt. 21:42;
John 2:22; 1 Cor. 15:3-4; 1 Peter 1:10-12; 2:2; 2 Peter 1:17-19, etc. Of course, Paul in Acts
17:11 says, "Now these were more noble-minded than those in Thessalonica, for they received
the word with great eagerness, examining the Scriptures daily, to see whether these things were
so." Paul commends those who examine God's word for the test of truth, not for the traditions of
men. Therefore, we can see that the biblical means of determining spiritual truth is by appealing
to scripture, not tradition. In fact, it is the scriptures that refute the traditions of men in many
instances.
2. Many doctrines in the Bible are not clearly stated, yet they are believed and taught by the church.
For example, there is no statement in the Bible that says there is a Trinity, or that Jesus has two
natures (God and man), or that the Holy Spirit is the third person in the Godhead. Yet, each of
the statements is considered true doctrine within Christianity, being derived from biblical
references. Therefore, it is not required of Scripture to have a statement to the effect, "The Bible
alone is to be used for all spiritual truth," in order for the statement to be true. So, for the
Catholic to require the Protestant to supply chapter and verse to prove Sola Scriptura is valid is
not necessarily consistent with biblical exegetical principles.
3. In appealing to the Bible for authentication of Inerrant Sacred Tradition, the Catholics have shown
that the Bible is superior to Sacred Tradition--for the lesser is blessed by the greater (Heb. 7:7).
You see, if the Bible said do not trust tradition, then Sacred Tradition would be instantly and
obviously invalid. If the Bible said to trust Sacred Tradition, then the Bible is authenticating it. In
either case, the Scriptures hold the place of final authority and by that position, are shown to be
superior to Sacred Tradition. If Sacred Tradition were really inerrant as it is said to be, then it
would be equal with the Bible. But, God’s word does not say that Sacred Tradition is inerrant or
inspired as it does say about itself (2 Tim. 3:16). To merely claim that Sacred Tradition is equal
and in agreement with the Bible does not make it so. Furthermore, to assert that Sacred
Tradition is equal to Scripture is to effectively leave the canon wide open to doctrinal addition.
Since the traditions of men change, to use tradition as a determiner of spiritual truth would mean
that over time new doctrines that are not in the Bible would be added and that is exactly what
has happened in Catholicism with doctrines such as purgatory, praying to Mary, indulgences,
etc. Furthermore, if they can use Sacred Tradition as a source for doctrines not explicit in the
Bible, then why would the Mormons then be wrong for having additional revelation as well?
4. If the Bible is not used to verify Sacred Tradition, then Sacred Tradition is functionally
independent to the Word of God. If it is independent of Scripture, then it has no right to exist as
an authoritative spiritual source since the Bible is what is inspired, not tradition.
5. Sacred Tradition is invalidated automatically if it contradicts the Bible, and it does. Of course, the
Catholic will say that it does not. But, Catholic teachings such as purgatory, penance,
indulgences, praying to Mary, etc., are not in the Bible. A natural reading of the Bible does not
lend itself to such beliefs and practices. Instead, the Catholic Church has used Sacred Tradition
to add to God's revealed word and then extracted out of the Bible whatever verses that might be
construed to support their doctrines of Sacred Tradition.

The Catholic apologist will state that both the Bible and Sacred tradition are equal in authority and
inspiration and to put one above another is a false comparison. But, by what authority does the
Catholic say this? Is it because it claims to be the true church, descended from the original apostles?
In response, claims do not make it true. Second, even if it were true, and I do not grant that it is,
there is no guarantee that the succession of church leaders is immune to error. We saw it creep in
with Peter in Acts. Are the Catholic Church leaders better than Peter?
To continue, is it from tradition that the Catholic Church authenticates its Sacred Tradition? If so,
then there is no check upon it. Is it from quotes of some of the church Fathers who say to follow
Tradition? If so, then the church fathers are given the place of authority comparable to scripture. Is it
from the Bible? If so, then Sacred Tradition holds a lesser position than the Bible because the Bible is
used as the authority in validating Tradition. Is it because the Catholic Church claims to be the means
by which God communicates His truth? Then, the Catholic Church has placed itself above the
Scriptures.
Finally, one of the mistakes made by the Catholics is to assume that the Bible is derived from
Sacred Tradition. This is false. The Church simply recognized the inspired writings of the Bible. They
were in and of themselves authoritative. Various "traditions" in the Church served only to recognize
what was from God. Also, to say the Bible is derived from Sacred Tradition is to make the Bible lesser
than the Tradition as is stated in Heb. 7:7 that the lesser is blessed by the greater.
Since the Bible is the final authority, we should look to it as the final authenticating and inerrant
source of all spiritual truth. If it says Sacred Tradition is valid, fine. But if it doesn’t, then I will trust
the Bible alone. Since the Bible does not approve of the Catholic Church's Sacred Tradition, then
neither should Christians.
Catholic Terminology

This list of terms used by the Catholic Church is brief and succinct. It should help those who are
learning about Catholicism and who desire to be conversant with Catholics on their terms.

1. Absolution - the act of releasing someone from their sin by God, through the means of a priest.
2. Actual grace - God's interventions, whether at the beginning of conversion or in the course of
the work of sanctification.
3. Actual sin - any sin that a person commits.
4. Annunciation - When the angel Gabriel told Mary that she was to be the mother of the Messiah.
5. Assumption - the taking of the body and soul of Mary, by God, into glory. Catholic doctrine,
apparently, does not state whether or not Mary died, but tradition holds that she died and was
immediately afterward assumed into heaven both body and soul.
6. Baptism - One of seven sacraments that takes away original sin and actual sin.
7. Bishop - the head of a diocese, successor of the apostles.
8. Blessed Sacrament - the elements of the communion supper, bread and wine, which become
the body and blood of Christ. It is offered at the altar in the church.
9. Capital sins - the seven causes of all sin: pride, covetousness, lust, anger, gluttony, envy, sloth.
10. Confession - telling sins to a priest and the Lord forgives the person through the priest.
11. Confessional - a small compartment where the priest hears the confessed sins of a sinner.
12. Confirmation - a ceremony performed by a bishop that is supposed to strengthen a person and
enable him to resist sin. It is usually done at the age of 12. The Bishop dips his right thumb in
holy oil and anoints the person on the forehead by making the sign of the cross and says, "Be
sealed with the gift of the Holy Spirit."
13. Consecration - a moment during the ceremony of the mass where God, allegedly through the
priest, changes bread and wine into the body and blood of Jesus.
14. Contrition - extreme sorrow for having sinned with a deep repentance concerning that sin.
15. Diocese - an area of many parishes presided over by a bishop.
16. Eucharist - The elements of the communion supper where the bread and wine are the body and
blood of Christ.
17. Extreme Unction - A sacrament given to a person who is ill or in danger of dying. It is intended
to strengthen the person's soul and help his love be pure so they may enter into heaven. It is
done through prayer and the anointing of oil. This is also known as Anointing of the Sick or the
Sacrament of the Sick.
18. Guardian Angel - a special angel assigned by God to each person in order to protect and guide
that person with the goal of reaching heaven.
19. Habitual grace - the permanent disposition to live and act in keeping with God's call
20. Heresy - denial of the truths found in the Catholic Church.
21. Holy Chrism - the special oil used in the sacraments of Baptism, Confirmation, and Holy Orders.

22. Holy Orders - one of the seven sacraments by which men, bishop, deacons, and priests, are
given the power and authority by a bishop to offer sacrifice and forgive sins.
23. Holy Water - Special water that has been blessed by a priest, bishop, etc. or a liturgical
ceremony. It is used to bring a blessing to a person when applied.
24. Host - the bread in the communion supper that is changed into the body of Christ.
25. Immaculate Conception - The teaching that Mary was conceived without original sin.
26. Indulgence - a means by which the Catholic Church takes away some of the punishment due the
Christian in this life and/or purgatory because of his sin.
27. Laity - the members of the Catholic church who are not in the clergy.
28. Lent - a forty-day period between Ash Wednesday and Easter Sunday. Usually it is accompanied
by some form of prayer and fasting.
29. Mass - a reenactment of the sacrifice of Christ on the cross in a ceremony performed by a priest.
This ceremony is symbolically carried out by the priest and involves Consecration where the bread
and wine are changed into the body and blood of Jesus.
30. Mortal Sin - a serious and willful transgression of God's Law. It involves full knowledge and
intent of the will to commit the sin. If left unrepentant, can damn someone to eternal hell.
31. Original Sin - the inherited sin nature of Adam that resulted from Adam's sin.
32. Parish - a subdivision of a diocese with the priest as its head.
33. Passion - The sufferings of Christ from the time of the Last Supper to His Crucifixion.
34. Penance - a means by which all sins committed after baptism are removed. The means are
assigned by a priest and usually consist of special prayers or deeds performed by the sinner.
35. Peter - the first pope.
36. Pope - Christ's representative on earth. He is the visible successor of Peter.
37. Priest - one who mediates between God and man and administers the sacraments and graces of
God. He has received the Holy Orders.
38. Purgatory - a place of temporary punishment where the Christian is cleansed from sin before
they can enter into heaven.
39. Relic - a part of the body of a saint including clothing, jewelry, etc. The relic is considered holy
due to its association with the saint.
40. Rosary - A string of beads containing five sets with ten small beads. Each set of ten is separated
by another bead. It also contains a crucifix. It is used in saying special prayers, usually to Mary
where the rosary is used to count the prayers.
41. Sacrament - an outward sign instituted by Christ to give grace.
42. Sacramentals - Special prayers, deeds, or objects used to gain spiritual benefits from God.
43. Sanctifying grace - a stable and supernatural disposition that perfects the soul itself to enable it
to live with God, to act by his love.
44. Saint - A very holy person. Usually, it is someone who has been dead for many years and has
been canonized by the Catholic Church. Saints do not have to pass through purgatory.
45. Scapular - two small cloth squares joined by a string. One cloth is positioned in the front and the
other in the back. Indulgences are attached to the them.
46. Sign of the Cross - A sacramental. It is the movement of the right hand from the forehead to
the chest and then left and right upon the shoulders.
47. Tradition - handing down through the centuries from mouth to mouth of the teachings of Jesus.
It began with the apostles and continues unbroken to the present bishopric of the Catholic
Church.
48. Transubstantiation - The teaching that the bread and wine in the communion supper become
the body and blood of the Lord Jesus at the Consecration during the Mass.
49. Venerate - to honor, admire, and regard with respect.
50. Venial Sin - A sin but not as bad as Mortal Sin. It lessens the grace of God within a person's
soul.
51. Vicar of Christ - the Pope.
Mary
Mary occupies a unique place in biblical history. She conceived Jesus by the power of the Holy
Spirit. Then she bore the Messiah. Among women, she is most blessed (Luke 1:42) and all who claim
to be Christian acknowledge her as a chosen vessel of God. While Christians admit Mary’s uniqueness,
the Catholic Church has, in its own words, "clarified her position and nature through Sacred Tradition".
Through the centuries, more and more doctrines concerning her have been ‘revealed.’ For example:

1. Mary is called the Mother of God 431


2. Prayers offered to Mary 600
3. Immaculate Conception (that she was sinless) 1854
4. Assumption of Mary 1950
5. Mary Proclaimed Mother of the Church 113 1965

An article appeared in Newsweek Magazine (8/25/97, p. 49) that examined an issue developing in
Catholicism where petitioners are requesting the Pope to exercise papal infallibility to proclaim Mary as
"Co-Redemptrix, Mediatrix of All Graces, and Advocate for the People of God." To its credit, the
Catholic church rejected this request. But it is a telling statement of the Catholic adoration of Mary --
even when that adoration exceeds biblical boundaries.
Officially, however, the Catholic Church has much to say about her. She remained a virgin after the
birth of Christ (Catechism of the Catholic Church, paragraph 510). She is "The mother of the members
of Christ" (par. 963). She was "Preserved free from all stain of original sin" (Catechism, par. 966). She
is "Queen over all things" (par. 966). By Mary’s prayers, she delivers souls from death (par. 966).
Mary, "...by her manifold intercession continues to bring us the gifts of eternal salvation.... Therefore
the Blessed Virgin is invoked in the Church under the titles of Advocate, Helper, Benefactress, and
Mediatrix" (par. 969). "The liturgical feasts dedicated to the Mother of God and Marian prayer, such as
the rosary, are an ‘epitome of the whole Gospel,’" (par. 971). Mary, "...when the course of her earthly
life was completed, was taken up body and soul into the glory of heaven..." (par. 974). "...when she
[Mary] is the subject of preaching and worship she prompts the faithful to come to her Son..." (Vatican
Council II, p. 420). "Mary has by grace been exalted above all angels and men to a place second only
to her Son" (Vatican Council II, p. 421). "This mother...is waiting and preparing your home for you"
(Handbook for Today’s Catholic, p.31).
As you can see, Mary holds an exalted position in Catholic theology. Because of her exalted position
in heaven, she is able to approach the Son with requests and petitions from her followers. She is
prayed to, adored, and sought by millions of devotees.

Mary is Most Blessed Among Women

Mary is undoubtedly blessed among women (Luke 1:42). But, is it appropriate to attribute to her
such titles as "Our Queen, Our Mother, Our Life, Our Sweetness, and Our Hope"? I cannot see how it.
Was she sinless? It would seem not since she said she needed a savior in Luke 1:47, "And my spirit
has rejoiced in God my Savior." Did she remain a virgin after the birth of Jesus? Again, it seems not
since Matt. 1:25 says that Joseph, ". . .kept her a virgin until she gave birth to a Son; and he called
His name Jesus." Does she mediate and intercede for sinners? Again, the scriptures seem to
contradict this when it states that Jesus is the only mediator between God and man ( 1 Tim. 2:5). Is
she exalted above all angels? There is no scripture stating so. Can she simultaneously hear the
prayers of countless people all over the world in different languages? Again, there is nothing in God's
word to lead us to believe this. Please understand that CARM is not attacking Mary or her wonderful
position in history. Rather, it seeks to examine her position according to biblical revelation and answer
the questions just posed. Hopefully, faithfully, and according to God's word, we can look at Scripture

113
These dates taken from Roman Catholicism, by Loraine Boettner, P & R Publishing Co. 1962, pp. 7-9
to find the answers.

Did Mary Have Other Children?


One of the more controversial teachings of the Catholic Church deals with the perpetual virginity of
Mary. This doctrine maintains that Mary remained a virgin after the birth of Jesus and that biblical
references suggesting Jesus had siblings are really references to cousins (Catechism of the Catholic
Church, paragraph 510).
As the veneration of Mary increased throughout the centuries, the vehicle of Sacred Tradition
became the means of promoting new doctrines not explicitly taught in the Bible. The virginity of Mary
is clearly taught in scripture when describing the birth of Jesus. But is the doctrine of her continued
virginity supported by the Bible? Did Mary lose her virginity after Jesus was born? Does the Bible
reveal that Mary had other children, that Jesus had brothers and sisters?
The Bible does not come out and declare that Mary remained a virgin and that she had no children.
In fact, the Bible seems to state otherwise: (All quotes are from the NASB.)

1. Matthew 1:24-25 - "And Joseph arose from his sleep, and did as the angel of the Lord
commanded him, and took as his wife, and kept her a virgin until she gave birth to a Son;
and he called His name Jesus."
2. Matthew 12:46-47 - "While He was still speaking to the multitudes, behold, His mother
and brothers were standing outside, seeking to speak to Him. And someone said to Him,
"Behold, Your mother and Your brothers are standing outside seeking to speak to You."
3. Matthew 13:55 - "Is not this the carpenter’s son? Is not His mother called Mary, and His
brothers, James and Joseph and Simon and Judas?"
4. Mark 6:2-3 - "And when the Sabbath had come, He began to teach in the synagogue; and
the many listeners were astonished, saying, "Where did this man get these things, and what
is this wisdom given to Him, and such miracles as these performed by His hands? "Is not
this the carpenter, the son of Mary, and brother of James, and Joses, and Judas, and Simon?
Are not His sisters here with us?"
5. John 2:12 - "After this He went down to Capernaum, He and His mother, and His brothers,
and His disciples; and there they stayed a few days."
6. Acts 1:14 - "These all with one mind were continually devoting themselves to prayer, along
with the women, and Mary the mother of Jesus, and with His brothers."
7. 1 Cor. 9:4-5 - "Do we not have a right to eat and drink? Do we not have a right to take
along a believing wife, even as the rest of the apostles, and the brothers of the Lord, and
Cephas?"
8. Gal. 1:19 - "But I did not see any other of the apostles except James, the Lord’s brother."

An initial reading of these biblical texts seems to clear up the issue: Jesus had brothers and sisters.
But such obvious scriptures are not without their response from Catholic Theologians. The primary
argument against these biblical texts is as follows:
In Greek, the word for brother is ‘adelphos’ and sister is ‘adelphe’. This word is used in different
contexts: of children of the same parents (Matt. 1:2; 14:3), descendants of parents (Acts 7:23, 26;
Heb. 7:5), the Jews as a whole (Acts 3:17, 22), etc. Therefore, the term brother (and sister) can and
does refer to the cousins of Jesus.
There is certainly merit in this argument; however, different contexts give different meanings to
words. It is not legitimate to say that because a word has a wide scope of meaning, that you may then
transfer any part of that range of meaning to any other text that uses the word. In other words, just
because the word ‘brother’ means ‘fellow Jews’ or ‘cousin’ in one place, does not mean it has the same
meaning in another. Therefore, each verse should be looked at in context to see what it means.
Let’s briefly analyze a couple of verses dealing with the brothers of Jesus.
1. Matthew 12:46-47, "While He was still speaking to the multitudes, behold, His mother and
brothers were standing outside, seeking to speak to Him. And someone said to Him, "Behold,
Your mother and Your brothers are standing outside seeking to speak to You."
2. Matthew 13:55 - "Is not this the carpenter’s son? Is not His mother called Mary, and His
brothers, James and Joseph and Simon and Judas?"

In both of these verses, if the brothers of Jesus are not brothers, but His cousins, then who is His
mother and who is the carpenter’s father? In other words, ‘mother’ here refers to Mary. The carpenter
in Matt. 13:55, refers to Joseph. These are literal. Yet, the Catholic theologian will then stop there
and say, "Though ‘carpenter’s son’ refers to Joseph, and ‘mother’ refers to Mary, ‘brothers’ does not
mean brothers, but "cousins." This does not seem to be a legitimate assertion. You cannot simply
switch contextual meanings in the middle of a sentence unless it is obviously required. The context is
clear. This verse is speaking of Joseph, Mary, and Jesus’ brothers. The whole context is of familial
relationship: father, mother, and brothers.

Psalm 69, A Messianic Psalm

There are many arguments pro and con concerning Jesus’ siblings. But the issue cannot be settled
without examining Psalm 69, a Messianic Psalm. Jesus quotes Psalm 69:4 in John 15:25, "But they
have done this in order that the word may be fulfilled that is written in their Law, ‘They
hated Me without a cause.’"
He also quotes Psalm 69:9 in John 2:16-17, "and to those who were selling the doves He said,
"Take these things away; stop making My Father’s house a house of merchandise." His disciples
remembered that it was written, "Zeal for Thy house will consume me."
Clearly, Psalm 69 is a Messianic Psalm since Jesus quoted it in reference to Himself two times. The
reason this is important is because of what is written between the verses that Jesus quoted.
To get the whole context, here is Psalm 69:4-9, "Those who hate me without a cause are more
than the hairs of my head; Those who would destroy me are powerful, being wrongfully my enemies,
What I did not steal, I then have to restore. 5O God, it is Thou who dost know my folly, And my
wrongs are not hidden from Thee. 6May those who wait for Thee not be ashamed through me, O Lord
God of hosts; May those who seek Thee not be dishonored through me, O God of Israel, 7Because for
Thy sake I have borne reproach; Dishonor has covered my face. 8I have become estranged from
my brothers, and an alien to my mother’s sons. 9For zeal for Thy house has consumed me, And
the reproaches of those who reproach Thee have fallen on me."
This messianic Psalm clearly shows that Jesus has brothers. As Amos 3:7 says, "Surely the Lord
God does nothing unless He reveals His secret counsel to His servants the prophets." God’s will has
been revealed plainly in the New Testament and prophetically in the Old. Psalm 69 shows us that Jesus
had brothers.
Did Mary have other children? The Bible seems to suggest yes. Catholic Tradition says no. Which
will you trust?
Of course, the Catholic will simply state that even this phrase "my mother's sons" is in reference
not to his siblings, but to cousins and other relatives. This is a necessary thing for the Catholic to say,
otherwise, the perpetual virginity of Mary is threatened and since that contradicts Roman Catholic
tradition, an interpretation that is consistent with that tradition must be adopted.
The question is, "Was Jesus estranged by His brothers?". Yes, He was. John 7:5 says "For not
even His brothers were believing in Him." Furthermore, Psalm 69:8 says bother "my brothers" and
"my mother's sons." Are these both to be understood as not referring to His siblings? Hardly. The
Catholics are fond of saying that "brothers" must mean "cousins." But, if that is the case, then when
we read "an alien to my mother's sons" we can see that the writer is adding a further distinction and
narrowing the scope of meaning. In other words, Jesus was alienated by his siblings, His very half-
brothers begotten from Mary.

It is sad to see the Roman Catholic church go to such lengths to maintain Mary's virginity,
something that is a violation of biblical law to be married and fill the earth.
Purgatory
According to the Catechism of the Catholic Church, paragraph 1030, "All who die in God’s grace and
friendship, but still imperfectly purified, are indeed assured of their eternal salvation, but after death
they undergo purification, so as to achieve the holiness necessary to enter the joy of heaven."
The Second Vatican Council, p. 63, says, "The truth has been divinely revealed that sins are
followed by punishments. God’s holiness and justice inflict them. Sins must be expiated. This may be
done on this earth through the sorrows, miseries and trials of this life and, above all, through death.
Otherwise the expiation must be made in the next life through fire and torments or purifying
punishments."
This process of purification occurs in a place designated by the Catholic Church as purgatory.
According to Catholic doctrine, purgatory is not supposed to be a place of punishment, but of
purification. The nature of this purification, according to different Catholic theologians, ranges from an
extreme awareness of loss to an intense, excruciatingly painful "purifying fire."
According to Roman Catholic Doctrine, though a person may be in a state of grace, he may not
enter heaven until he is purified from sins that were not dealt with on earth. Baptism remits sins
committed up to that point, but prayers, indulgences, penance, absolution, and the Mass are means by
which the sinner is able to expiate sins committed after baptism. If sins are not remitted, after death
he must suffer the flames of purification until he is sufficiently cleansed and pure so as to enter into
the presence of God. Additionally, intercession can be made by Catholics on behalf of those who are
presently in purgatory. This is also done through saying the Mass, certain acts of penance, saying the
Rosary, or by indulgences where the benefit is applied to the dead in purgatory.
But purgatory is not for everyone. Baptized infants who have died before the age of accountability
and Catholic saints who lived such holy lives are excused from the "purifying fires."
The length of time that someone must suffer in this state is never known, but it is considered to
be proportional to the nature and severity of the sins committed. Therefore, it could be anywhere from
a few hours to thousands of years.

Problems with the Doctrine of Purgatory

As a Christian who bases spiritual truth on the Bible alone, I see problems with the doctrine of
purgatory. For example:

1. It is not explicitly found in the Bible.


2. It implies that the righteousness of Christ does not cleanse from all sin.
3. It implies that justification is not by faith alone.
4. It implies that there is something we must do in order to be cleansed of sin.

The Catholics will disagree with my perceived problems of the doctrine of purgatory. That is to be
expected. They will cite church Fathers, the apocrypha, and various biblical references to fire and
purification. Whichever side of the argument you fall into, my goal here is to present a biblical
argument that examines the doctrine in an attempt to determine if it is biblical or not.
Of course, the Catholic will say that as a Protestant, I come to the argument with the preconceived
belief that (1) Purgatory is unbiblical, (2) that I am biased against it, and (3) that I have an agenda to
accomplish. To each of these accusations I admit guilt. None of us are perfectly unbiased and most
everyone has personal beliefs that are reflected in their actions and words. In this case, having read
and studied the Bible thoroughly, I find no place in it for the Roman Catholic doctrine of Purgatory.
Does Purgatory Deny the Sufficiency of Christ’s Sacrifice?
According to the Handbook for Today’s Catholic, page 47, "If you die in the love of God but possess
any ‘stains of sin,’ such stains are cleansed away in a purifying process called purgatory. These stains
of sin are primarily the temporal punishment due to venial or mortal sins already forgiven but for
which sufficient penance was not done during your lifetime."
The Catholic Catechism, paragraph 1030, says that purgatory is for "All who die in God's grace and
friendship, but still imperfectly purified, are indeed assured of their eternal salvation, but after death
they undergo purification, so as to achieve the holiness necessary to enter the joy of heaven."
Among the many doctrines that Catholicism claims to be derived through Sacred Tradition,
purgatory is one of the most interesting and puzzling, particularly to a Protestant. In light of the
Pauline doctrine of justification by grace through faith, how is it possible that an afterlife cleansing
through punishment is necessary for a Christian who has trusted in Jesus to cleanse him from all His
sins? Wasn't Jesus' punishment for our transgressions sufficient? Didn’t He take our place in that He
suffered our death? It would seem that the words of Christ, "It is finished" (John 19:30) do not mean
that the cleansing of our souls was completed on the cross.
Of course, Roman Catholic doctrine states that eternal life is bestowed upon the one who receives
baptism (Catechism, par. 1265 - 1266, 1992). It is the stains of the sins committed after baptism and
not removed through penance, good works, prayers, the Mass, etc., that are removed in the fires of
purgatory (Handbook for Today's Catholic, page 47).
In light of the doctrine of justification by faith ( Rom. 5:1), where Jesus bore all of our sins,
purgatory would seem to have no theologically justifiable right to exist. But the Bible alone is not
appealed to by Catholic theologians in support of Purgatory. By far, the main support for Purgatory is
found in the Catholic doctrine of Sacred Tradition. Nevertheless, what does the Bible say about
justification, punishment, and our sins?

What is justification by faith?

To ‘justify’ means ‘acquit’, ‘declare righteous’, the opposite of ‘condemn’. It means to not be guilty
of breaking the Law and to be deemed as righteous by the standard of the Law.
God gave the Law, i.e., the 10 commandments. The Law is a reflection of God’s character and it is
a perfect standard of righteousness which no one can keep. Since no one is able to keep God’s Law, no
one can be justified by the Law (Rom. 3:20). There is, therefore, none righteous (Rom. 3:10-12). This
is the problem of all people. We have all broken God’s Law and are in need of justification, of being
declared righteous in God’s sight. This can only be done through the Messiah, our sin bearer.
Jesus is the one who took our place on the cross (1 Pet. 2:24), became sin on our behalf (2 Cor.
5:21), and turned away the wrath of God from us (Rom. 5:9) by being a propitiation (1 John 2:2) that
turned away the wrath of God. He was punished in our place. Therefore, Jesus was our substitution.
The righteous work of Christ is imputed to the believer by grace (Titus 3:7) and through faith (Rom.
5:1). This justification is a legal action on the part of God ‘reckoning’ the believer as having satisfied
the Law — all of the Law.
It necessarily follows that to be justified in God’s eyes, is to be fully justified. It is not ‘part’ of the
Law that must be satisfied, but all of it. Perfection is the standard. Likewise, it is not ‘part’ of our sins
that were born by Christ, but all of them. This justification includes all of the sins of the believer (past,
present, and future) or else we could not be justified.

What does the Catholic Catechism Say?

The Catholic Catechism (paragraphs 1990-1992) says, "Justification detaches man from sin which
contradicts the love of God, and purifies his heart of sin. Justification follows upon God’s merciful
initiative of offering forgiveness. It reconciles man with God. It frees from the enslavement to sin, and
it heals"...."Justification is at the same time the acceptance of God’s righteousness through faith in
Jesus Christ..." and "...justification is conferred in Baptism, the sacrament of faith. It conforms us to
the righteousness of God, who makes us inwardly just by the power of his mercy."
Of particular interest is the reference that "justification is conferred in Baptism, the sacrament of
faith." There are many verses in the Bible that deal with baptism and ‘putting on Christ,’ ( Gal. 3:27;
Rom. 6:1-11). This paper is not intended to discuss the nature of baptism. Nevertheless, I strongly
affirm that baptism is a covenant sign for the believer who is already justified by faith and for the
children of believers who are under the covenant headship of the family. Baptism is not what justifies a
person. Rather,

• Justification is a gift by His grace through Jesus ( Rom. 3:24)


• Justification is by grace (Titus 3:7)
• Justification is by faith (Rom. 3:28; 5:1; Gal. 3:24)
• Justification is by Jesus’ blood (Rom. 5:9).
• Justification is in the name of the Lord Jesus (1 Cor. 6:11).
• Justification is not equated with baptism, but with grace, faith, and the blood of Jesus.

Jesus said, "It is finished," (John 19:30)

Jesus bore our sins in His body, paid the penalty for them, and died. He said, "It is finished." In
Greek, the phrase, "It is finished" is one word, tetelestai. In ancient Greek papyri texts that were
receipts for taxes, when a debt was paid in full, the word tetelestai, was written on the document. This
meant that the debt had been paid in full. In other words, Jesus had finished the work of atonement.
But He not only atoned (to make amends, to make right), but He also propitiated (turning away God’s
wrath). He had fully paid the debt invoked by the sinner. There was nothing more to be done... It was
finished.
Yet, the doctrine of Purgatory, in effect, is saying that we must suffer in purgatory for sins not
‘covered by baptism’ and not covered by the cross. It is to say that the work of Christ is not finished
and that there are things we must do to complete the sacrificial, cleansing work of Christ. This
amounts to earning heaven by our good works, albeit, a work of suffering. Additionally, the doctrine of
Purgatory implies that a person must atone for his own sins. It implies that the person must do more
than what the Law of God requires of him. This is called supererogation.
When Jesus said, "It is finished," all that was necessary in the atonement was concluded and all in
Christ were justified. We cannot complete or add to Christ’s work through our suffering. Purgatory is
not only unnecessary, but it contradicts God’s word.
Purgatory and 1 Cor. 3:15
The doctrine of Purgatory in the Catholic Church is explained in this statement from the Second
Vatican Council, p. 63, which says,

“The truth has been divinely revealed that sins are followed by punishments. God's holiness and
justice inflict them. Sins must be expiated. This may be done on this earth through the sorrows,
miseries and trials of this life and, above all, through death. Otherwise the expiation must be
made in the next life through fire and torments or purifying punishments.”

The Protestant church has objected to the doctrine of Purgatory by stating that this teaching denies
the sufficiency and full efficacy of Christ’s atoning sacrifice. To say that our sins are expiated by our
suffering is an insult to the cross of Christ since it says that the cross was not sufficient to cleanse us
of our sins. It says that we must suffer, that we must do something to have our sins fully cleansed.
Instead, the Protestants maintain that Jesus’ sacrifice alone is what justifies and removes from us all
guilt. We look to the cross and to the cross alone for the complete forgiveness of our sins and, though
our works will one day be judged, we have passed out of condemnation (Rom. 8:1). Our works reflect
on rewards in heaven, not to get us to heaven. Jesus bore all our sins (1 Pet. 2:24). There are no sins
left for purgatory to cleanse because it was all done by Jesus on the cross. This is why Jesus said, "It
is finished," (John 19:30). In Greek the term "it is finished" is "tetelestai." It was a term used in legal
contexts to state that a debt had been paid in full. "Papyri receipts for taxes have been recovered with
the word tetelestai written across them, meaning "paid in full." (Walvoord, John F., and Zuck, Roy B.,
The Bible Knowledge Commentary, (Wheaton, Illinois: Scripture Press Publications, Inc., 1983, 1985).
Therefore, there is no need for purgatory.
Nevertheless, because the Protestants appeal so much to the Bible, the Catholics have sought to
find the doctrine of Purgatory within its pages. One such verse is 1 Cor. 3:15.

"If any man’s work is burned up, he shall suffer loss; but he himself shall be saved, yet so as
through fire."

As with any verse in the Bible, to fully understand it, we must look at it in its biblical context.
Following is 1 Cor. 3:10-15

“According to the grace of God which was given to me, as a wise master builder I laid a
foundation, and another is building upon it. But let each man be careful how he builds upon it.
11
For no man can lay a foundation other than the one which is laid, which is Jesus Christ. 12Now
if any man builds upon the foundation with gold, silver, precious stones, wood, hay, straw,
13
each man’s work will become evident; for the day will show it, because it is to be revealed
with fire; and the fire itself will test the quality of each man’s work. 14If any man’s work which
he has built upon it remains, he shall receive a reward. 15 If any man’s work is burned up,
he shall suffer loss; but he himself shall be saved, yet so as through fire.

The context speaks of Paul having planted the Corinthian church and that another person was
building upon that work: Verse 6 says, "I planted, Apollos watered, but God was causing the growth."
Paul goes on to say that unless a person builds upon the foundation of Jesus, his work will be burned
up the in the Day of Judgment (v. 13). See also, 1 Cor. 5:5; 2 Cor. 1:14; 1 Thess. 5:2).
Paul is simply using the terms that are familiar with the people of the time. Fire was the tool used
to purify metals and to get rid of that which was unwanted, the dross. So too, on the day when our
works are examined, the fire of judgment will both purify and remove. This will not affect our
salvation, but it will affect our rewards. The theme of fire used as purification is also found in 2 Pet.
3:10-13. But this is not talking about becoming saved or staying saved.
1 Cor. 3:15 does not teach purgatory as a place we go to in order to have some of our sins
cleansed from us. It teaches that even though the person is justified by faith and cannot face
damnation, his works will, however, be judged on "that day." Those works which are good will survive
the fires of judgment the way gold, silver, and precious stones can survive fire. But false works will be
consumed the way fire consumes wood, hay, and straw. What is left has no bearing on whether or not
we are saved. It has to do with rewards in heaven.
Paul goes on to say in 1 Cor. 4:5, "Therefore do not go on passing judgment before the time, but
wait until the Lord comes who will both bring to light the things hidden in the darkness and disclose
the motives of men’s hearts; and then each man’s praise will come to him from God."
Note also, 1 Pet. 1:6-7, "In this you greatly rejoice, even though now for a little while, if necessary,
you have been distressed by various trials, 7that the proof of your faith, being more precious than gold
which is perishable, even though tested by fire, may be found to result in praise and glory and honor
at the revelation of Jesus Christ."
2 Pet. 3:10-13, "But the day of the Lord will come like a thief, in which the heavens will pass away
with a roar and the elements will be destroyed with intense heat, and the earth and its works will be
burned up. 11Since all these things are to be destroyed in this way, what sort of people ought you to
be in holy conduct and godliness, 12looking for and hastening the coming of the day of God, on account
of which the heavens will be destroyed by burning, and the elements will melt with intense heat! 13But
according to His promise we are looking for new heavens and a new earth, in which righteousness
dwells
Purgatory is a dangerous doctrine that makes the Cross of Christ insufficient by requiring the
person to undergo suffering in order to be made worthy of being with God. This is a false teaching and
is to be avoided. We are justified by faith (Rom. 5:1), not by faith and works (Rom. 3:28).
Council of Trent: Canons on Justification
Lutheranism was growing strong in the 1500's. In response to this, the Roman Catholic church
convened a council in November of 1544 in an attempt to counter the doctrines raised and supported
by the Reformers. The official opening of the council was on Dec. 13, 1545 and was closed on Dec.
14, 1563. The council delivered many statements on various subjects. These Canons have never
been denied by the Roman Catholic Church.
Following are several of the doctrinal statements made on Justification at the council of Trent.
After each Canon are scriptures that contradict that Canon. These scriptures are linked to the KJV on
CARM so you can click on them and read them in context.
Finally, you will see the word "anathema" used many times by the Council. This means that those
who disagree with the doctrines of this Council are cursed. In Gal. 1:8-9, the word "anathema" is
used. The curse must come from God. Therefore, we conclude that according to Roman Catholicism,
anyone who disagrees with the following Canons are cursed of God. The Roman Catholic Church
excommunicates those under anathema. In other words, excommunication means being outside the
Christian church. Being outside the church means you are not saved.
In spite of what Catholicism states, the Bible speaks differently. Following each Canon is a list of
appropriate scriptures countering the Catholic position.

1. CANON 9: "If any one saith, that by faith alone the impious is justified; in such wise as to mean,
that nothing else is required to co-operate in order to the obtaining the grace of Justification, and
that it is not in any way necessary, that he be prepared and disposed by the movement of his
own will; let him be anathema."
A. "Therefore by the deeds of the law there shall no flesh be justified in his sight: for by the law
is the knowledge of sin," (Rom. 3:20).
B. "Being justified freely by his grace through the redemption that is in Christ Jesus," (Rom.
3:24).
C. "Therefore we conclude that a man is justified by faith without the deeds of the law," (Rom.
3:28).
D. "For what saith the scripture? Abraham believed God, and it was counted unto him for
righteousness," (Rom. 4:3).
E. "Therefore being justified by faith, we have peace with God through our Lord Jesus Christ,"
(Rom. 5:1).
F. "For by grace are ye saved through faith; and that not of yourselves: it is the gift of God,"
(Eph. 2:8).
G. "Not by works of righteousness which we have done, but according to his mercy he saved
us, by the washing of regeneration, and renewing of the Holy Ghost," (Titus 3:5).
2. CANON 12: "If any one shall say that justifying faith is nothing else than confidence in the
divine mercy pardoning sins for Christ's sake, or that it is that confidence alone by which we are
justified ... let him be accursed"
A. "But as many as received him, to them gave he power to become the sons of God, even to
them that believe on his name," (John 1:12).
B. "Therefore we conclude that a man is justified by faith without the deeds of the law," (Rom.
3:28).
C. "For what saith the scripture? Abraham believed God, and it was counted unto him for
righteousness," (Rom. 4:3).
D. "Wherefore he is able also to save them to the uttermost that come unto God by him, seeing
he ever liveth to make intercession for them. For such an high priest became us, who is
holy, harmless, undefiled, separate from sinners, and made higher than the heavens; 27Who
needeth not daily, as those high priests, to offer up sacrifice, first for his own sins, and then
for the people’s: for this he did once, when he offered up himself," (Heb. 7:25-27).
E. For the which cause I also suffer these things: nevertheless I am not ashamed: for I know
whom I have believed, and am persuaded that he is able to keep that which I have
committed unto him against that day," (2 Tim. 1:12).
3. Canon 14: "If any one saith, that man is truly absolved from his sins and justified, because that
he assuredly believed himself absolved and justified; or, that no one is truly justified but he who
believes himself justified; and that, by this faith alone, absolution and justification are effected;
let him be anathema."
A. "For what saith the scripture? Abraham believed God, and it was counted unto him for
righteousness," (Rom. 4:3).
B. "Therefore being justified by faith, we have peace with God through our Lord Jesus Christ,"
(Rom. 5:1).
4. Canon 23: "lf any one saith, that a man once justified can sin no more, nor lose grace, and that
therefore he that falls and sins was never truly justified; or, on the other hand, that he is able,
during his whole life, to avoid all sins, even those that are venial,- except by a special privilege
from God, as the Church holds in regard of the Blessed Virgin; let him be anathema."
A. "He that believeth on the Son hath everlasting life: and he that believeth not the Son shall
not see life; but the wrath of God abideth on him," (John 3:36).
B. "And this is the will of him that sent me, that every one which seeth the Son, and believeth
on him, may have everlasting life: and I will raise him up at the last day," (John 6:40).
C. "And I give unto them eternal life; and they shall never perish, neither shall any man pluck
them out of my hand," (John 10:28).
D. "That as sin hath reigned unto death, even so might grace reign through righteousness unto
eternal life by Jesus Christ our Lord," (Rom. 5:21).
E. "They went out from us, but they were not of us; for if they had been of us, they would no
doubt have continued with us: but they went out, that they might be made manifest that
they were not all of us," (1 John 2:19).
F. "These things have I written unto you that believe on the name of the Son of God; that ye
may know that ye have eternal life, and that ye may believe on the name of the Son of
God," (1 John 5:13).
5. Canon 24: "If any one saith, that the justice received is not preserved and also increased before
God through good works; but that the said works are merely the fruits and signs of Justification
obtained, but not a cause of the increase thereof; let him be anathema."
A. "O foolish Galatians, who hath bewitched you, that ye should not obey the truth, before
whose eyes Jesus Christ hath been evidently set forth, crucified among you? 2This only
would I learn of you, Received ye the Spirit by the works of the law, or by the hearing of
faith? 3Are ye so foolish? having begun in the Spirit, are ye now made perfect by the flesh?"
(Gal. 3:1-3).
B. "Stand fast therefore in the liberty wherewith Christ hath made us free, and be not
entangled again with the yoke of bondage. 2Behold, I Paul say unto you, that if ye be
circumcised, Christ shall profit you nothing. 3For I testify again to every man that is
circumcised, that he is a debtor to do the whole law," (Gal. 5:1-3).
6. Canon 30: "If any one saith, that, after the grace of Justification has been received, to every
penitent sinner the guilt is remitted, and the debt of eternal punishment is blotted out in such
wise, that there remains not any debt of temporal punishment to be discharged either in this
world, or in the next in Purgatory, before the entrance to the kingdom of heaven can be opened
(to him); let him be anathema."
A. "Therefore being justified by faith, we have peace with God through our Lord Jesus Christ,"
(Rom. 5:1).
B. "And you, being dead in your sins and the uncircumcision of your flesh, hath he quickened
together with him, having forgiven you all trespasses; 14Blotting out the handwriting of
ordinances that was against us, which was contrary to us, and took it out of the way, nailing
it to his cross," (Col. 2:13-14).
7. Canon 33: "If any one saith, that, by the Catholic doctrine touching Justification, by this holy
Synod inset forth in this present decree, the glory of God, or the merits of our Lord Jesus Christ
are in any way derogated from, and not rather that the truth of our faith, and the glory in fine of
God and of Jesus Christ are rendered (more) illustrious; let him be anathema.
A. This council declares that if anyone disagrees with it, they are damned.

The Roman Catholic view on justification

Justification is a divine act where God declares the sinner to be innocent of his sins. It is a legal
action in that God declares the sinner righteous -- as though he has satisfied the Law of God. This
justification is based entirely on the sacrifice of Christ by His shed blood: "...having now been justified
by His blood..." (Rom. 5:9).114 Justification is a gift of grace (Rom. 3:24; Titus 3:7) that comes
through faith (Rom. 3:28; 5:1). Christians receive Jesus (John 1:12) and put their faith-filled trust in
what Jesus did on the cross (Isaiah 53:12; 1 Pet. 2:24) and in so doing are justified by God. The Bible
states that justification is not by works (Rom. 3:20, 28; 4:5; Eph. 2:8-9) because our righteous deeds
are filthy rags before God (Isaiah 64:6). Therefore, we are saved by grace alone, through faith alone,
in Christ alone.
Those who are justified are saved and salvation is a free gift (Rom. 6:23), something we cannot
earn (Eph. 2:1-10). However, Roman Catholic doctrine denies justification by faith alone and says:

• "If any one saith, that by faith alone the impious is justified; in such wise as to mean, that
nothing else is required to co-operate in order to the obtaining the grace of Justification, and
that it is not in any way necessary, that he be prepared and disposed by the movement of his
own will; let him be anathema" (Council of Trent, Canons on Justification, Canon 9).
• "If any one saith, that man is truly absolved from his sins and justified, because he assuredly
believed himself absolved and justified; or, that no one is truly justified but he who believes
himself justified; and that, by this faith alone, absolution and justification are effected; let him
be anathema." (Canon 14).

Anathema, according to Catholic theology means excommunication, "the exclusion of a sinner from
the society of the faithful." The Greek word anathema is also translated as "accursed" (Rom. 9:3; Gal.
1:8-9, NASB & KJV), "eternally condemned" (Gal. 1:8-9, NIV), and "cursed" (Rom. 9:3, NIV),. We can
see that Roman Catholic theology pronounces a curse of excommunication, of being outside the camp
of Christ if you believe that you are saved by grace through faith alone in Jesus.
Does the Roman Catholic Church specifically state that we are "saved by grace and works"? Not
that I am aware of and neither do the above Catholic Canons state such a thing. But, when the
Roman Catholic Church negates justification by faith alone, it necessarily implies that we must do
something for justification, for if it is not by faith alone, then it must be by faith and something.
At this point many Catholics appeal to James 2:24 which says, "You see that a man is justified by
works, and not by faith alone." But the context of James is speaking of dead faith as opposed to
living, saving faith. James states that if you "say" you have faith but have no works (James 2:14),
that faith cannot save you because it is a dead faith (v. 17). In other words, mere intellectual
acknowledgement of Christ is a dead faith that produces no regeneration and no change in a person's
life. This faith does not justify. Rather, it is only that real and believing faith in Christ that results in
justification. When someone is truly justified, he is truly saved and regenerate. Therefore, we see the
results of true saving faith as they are manifested in the changed life of the one justified by faith
alone. Real faith produces good works but it isn't these works that save you. Good works are the
effect of salvation, not the cause of it in any way and they certainly do not help anyone keep their
salvation. For more on this, please see "Are you justified by Faith (Romans) or works (James)?"
Protestant theology, as a whole, appeals to the Bible alone for spiritual truth and maintains that
justification is not by works in any way but is by grace through faith in Christ and His sacrifice alone.
After all, the Bible says "But if it is by grace, it is no longer on the basis of works, otherwise grace is
no longer grace," (Rom. 11:6). Furthermore, the Bible says:

The phrase "having now been" is in the perfect tense in the Greek. This signifies a past action that continues in
114

the present. In other words, Paul is saying that the Christians have been justified and still are.
• "Therefore by the deeds of the law there shall no flesh be justified in his sight: for by the law is
the knowledge of sin," (Rom. 3:20).
• "being justified as a gift by His grace through the redemption which is in Christ Jesus," (Rom.
3:24).
• "Therefore we conclude that a man is justified by faith without the deeds of the law," (Rom.
3:28).
• "For what saith the scripture? Abraham believed God, and it was counted unto him for
righteousness," (Rom. 4:3).
• "But to the one who does not work, but believes in Him who justifies the ungodly, his faith is
reckoned as righteousness," (Rom. 4:5).
• "For the promise to Abraham or to his descendants that he would be heir of the world was not
through the Law, but through the righteousness of faith," (Rom. 4:13).
• "Therefore being justified by faith, we have peace with God through our Lord Jesus Christ,"
(Rom. 5:1).
• "Much more then, having now been justified by His blood, we shall be saved from the wrath of
God through Him," (Rom. 5:9).
• "that if you confess with your mouth Jesus as Lord, and believe in your heart that God raised
Him from the dead, you shall be saved," (Rom. 10:9).
• "so that we might receive the promise of the Spirit through faith," (Gal. 3:14).
• "For by grace are ye saved through faith; and that not of yourselves: it is the gift of God,"
(Eph. 2:8).

I am bewildered when I read Catholic theology that denies justification by faith alone and requires
human effort in addition to God's grace to be saved. Of course, Catholicism denies that it is works that
save us -- and rightly so. But, it contradicts itself when it teaches that certain things must be done by
people in order to be justified and to keep that justification. Whether or not Catholicism calls these
works acts of faith or not is immaterial. The label doesn't change the substance. We are either saved
by grace through faith alone or we are not.
Of the acts to be performed by Catholics for justification, baptism is the first requirement. Please
consider these quotes:

• ". . Baptism is the first and chief sacrament of forgiveness of sins because it unites us with
Christ, who died for our sins and rose for our justification, so that 'we too might walk in
newness of life,'" (Catechism of the Catholic Church par. 977).
• "Justification has been merited for us by the Passion of Christ. It is granted us through Baptism.
It conforms us to the righteousness of God, who justifies us. It has for its goal the glory of God
and of Christ, and the gift of eternal life. It is the most excellent work of God's mercy," (CCC,
par. 2020).

I do not see the Bible saying anywhere that we are justified by baptism. Yes, there are verses that
can be interpreted that way, but if they were then they would contradict the clear teaching of Rom.
3:20, 28; 4:3; 5:1; Eph. 2:8 which says salvation by grace through faith, not grace through faith and
baptism.
However, according to Roman Catholicism even faith and baptism aren't sufficient in themselves for
you to be saved. It says that baptism is only the first sacrament of forgiveness. Good works,
according to Roman Catholicism, are also required and are rewarded with going to heaven:

"We can therefore hope in the glory of heaven promised by God to those who love him and do
his will. In every circumstance, each one of us should hope, with the grace of God, to
persevere 'to the end' and to obtain the joy of heaven, as God's eternal reward for the good
works accomplished with the grace of Christ," (CCC, par. 1821).
The above quote clearly states that heaven is the "eternal reward for the good works accomplished
with the grace of Christ." Catholic theology asserts that works are a predecessor to justification in
direct contradiction to God's word which states ". . .that a man is justified by faith without the deeds
of the law," (Rom. 3:28). What are the deeds of the Law? Anything we do in hopes of getting or
maintaining our righteousness before God.
In the CCC, par. 2010 it says,

"Moved by the Holy Spirit and by charity, we can then merit for ourselves and for others the
graces needed for our sanctification."

How does anyone merit for himself the underserved kindness of God's grace? Grace is by definition
unmerited favor. To me this is an utterly false teaching that you can earn grace from God through
works or rituals. So how does the Catholic Church get around this apparent dilemma that grace is
unmerited but it is obtained through our merits? It states that...

"Sanctifying grace is the gratuitous gift of his life that God makes to us; it is infused by the
Holy Spirit into the soul to heal it of sin and to sanctify it," (CCC, par. 2023).

This is the crux of the problem. Roman Catholic theology asserts that God's grace is granted
through baptism and infused into a person by the Holy Spirit. This then enables him or her to do good
works which then are rewarded with heaven. Basically, this is no different than the theology of the
cults which maintain that justification is by grace through faith and your works whether it be baptism,
going to "the true church," keeping certain laws, receiving the sacraments, or anything else you are
required to do. In response, I turn to God's word at Gal. 3:1-3:

"You foolish Galatians, who has bewitched you, before whose eyes Jesus Christ was publicly
portrayed as crucified? 2 This is the only thing I want to find out from you: did you receive the
Spirit by the works of the Law, or by hearing with faith? 3 Are you so foolish? Having begun by
the Spirit, are you now being perfected by the flesh?"

Does not the above scripture clearly state that receiving God's Spirit is by faith and not by what we
do? Does it not teach us that we cannot perfect our salvation by the works we do in the flesh? To
receive Jesus (John 1:12 ) means to become the temple of the Holy Spirit (1 Cor. 6:19) which means
a person is saved, justified. Is this salvation something we attained through our effort? Of course
not! Is it something we maintain through our effort? Not at all. It is given to Christians by God and
assured by God because it rests in what God has done and not in anything we have done -- that is why
salvation is by faith and not works. If it did rest in anyway in our works, then our salvation could not
be secure and we would end up trying to be good enough to get to heaven. That only leads to
bondage to the Law and the result is a lack of assurance of salvation, a constant worry that you are
not good enough, and a repeated subjection to the Church's teachings and requirements about what
you must do to be saved. The only natural effect of such a teaching would be that you can lose your
salvation over and over again and that you must perform the necessary requirements of the Catholic
Church to stay saved.

Catholic Theology teaches you maintain your justification

Because the Catholic view of justification is a cooperative effort between God and man, this
justification can be lost and regained by man's failure to maintain sufficient grace through meritorious
works. Now I must admit that within Protestant churches there are different opinions on this very
matter of eternal security. Some believe salvation can be lost while others do not. I am not here
attempting to address this issue. Rather, I seek to point out that Roman Catholicism teaches that
works are necessary for this "re-attainment" of justification. This is how...
According to Catholic theology, penance is a sacrament where a person, through a Catholic priest
(CCC, par. 987), receives forgiveness of the sins committed after baptism. The penitent person must
confess his sins to a priest. The priest pronounces absolution and imposes acts of Penance to be
performed.

"Christ instituted the sacrament of Penance for all sinful members of his Church: above all for
those who, since Baptism, have fallen into grave sin, and have thus lost their baptismal grace
and wounded ecclesial communion. It is to them that the sacrament of Penance offers a new
possibility to convert and to recover the grace of justification. The Fathers of the Church
present this sacrament as 'the second plank (of salvation) after the shipwreck which is the loss
of grace," (CCC, par. 1446).

The Council of Trent (Sess. XIV, c. i) declared regarding Penance:

"As a means of regaining grace and justice, penance was at all times necessary for those who
had defiled their souls with any mortal sin. . . .

Acts of penance vary, but some of them are prayer, saying the rosary, reading the scripture,
saying a number of "Our Father's" or "Hail Mary's" prayers, doing good works, fasting, and other such
things. Is it by doing these acts of penance that the Catholic is able to regain his justified state before
God? I am astounded to think that they are taught to believe that by their works of penance
justification is regained. In essence it is earning one's salvation. Think about it. If you do not have it
and you get it by saying prayers, fasting, and/or doing good works, then you are guilty of "works
righteousness" salvation which is condemned by the Bible. "Works Righteousness" means that a
person is trying to attain or keep his position with God based upon his works. It is a false teaching.
I confess my sins to God. He forgives me (1 John 1:9). I do not need a Catholic priest to be my
mediator of forgiveness. I need the true mediator and High Priest, Jesus. He alone is my mediator (1
Tim. 2:5). He has all authority in heaven and earth (Matt. 28:18) to forgive my sins and intercede for
me. He finished the work on the cross (John 19:30) so that I do not need to perform any work in
order to gain, maintain, or even regain my salvation. That is why the Bible teaches that we are
justified by faith (Rom. 5:1) apart from works (Rom. 3:28).
To say that we can add to the finished work of Christ on the cross is to say that what He did was
not sufficient to save us. May this never be! We are saved by grace through faith, not grace through
faith and our works. If it were, then grace would not be grace.

"But if it is by grace, it is no longer on the basis of works, otherwise grace is no longer grace,"
(Rom. 11:6).

Relationship, not Ritual

Salvation is a free gift from God given to us by His awesome Grace and is based upon the sacrifice
of Jesus on the cross. Christians receive this by faith because faith is all we have left since my works
are excluded, by God, as having anything to do with attaining salvation.
God desires fellowship with His people (1 Cor. 1:9), not rituals and works righteousness that
cannot save us.

May God receive all the glory due Him because of His grace.
Comparison Grid
I hesitate to put this grid up only because I do not want to needlessly assail the Catholic belief
system in an offensive manner. However, I cannot help but see the similarities between Catholicism
and some of the cults.
Do I think Catholicism is a cult? No. But I do think it has some very serious theological problems,
bordering on apostasy, imposed upon itself through its history of letting traditions seep into the fabric
of biblical truth.

Jehovah's
Issue Catholicism Mormonism Christianity
Witnesses
none
Specific Church The The Watchtower
(except, of course,
Leader on earth Pope Prophet Organization
Jesus)

Source of Bible and additional Bible and Watchtower


Bible and Tradition Bible
Theology Scripture Organization

Purgatory, Penance, Many gods, god from Jesus is Michael the


Sample of non-
Indulgences, praying to another world, Archangel. No blood If it isn't taught here,
biblical
Mary, goddess mother, you transfusions, no hell, I don't believe it.
teachings her assumption. can become a god. 144,000 go to heaven

Method of Faith, grace, baptism, Faith, grace, baptism, Faith, grace, baptism,
Grace through Faith
Salvation works works works

Their church Their church Their church All who are saved by
True
organization is one true organization is one organization is one grace thru faith are in
Church church true church true church the true church

Claim of Being the faithful


Apostolic Succession Apostolic Succession In Jesus alone
Authority servant

Great Wealth and Wealth, not much


Assets Great Wealth and Power Treasures in heaven
Power power

Mary has godlike


abilities: able to hear Goddess Mother who
Goddess-like and answer all prayers has populated this
Not Applicable Not Applicable
figure of all people all the world with spirits who
time, intercede with inhabit human bodies
God.

Images of God, Used in


Used in worship in
or Jesus, or temples and Not Applicable Not Applicable
churches
Mary, etc. headquarters

There are many good things in the Catholic Church, but I find it disturbing to see the similarities
between itself and the cults in regard to a final earthly authority, non biblical teachings, claims to be
the one true church, exaltation of Mary to semi-goddesshood, the use of images, and adding works to
salvation. The Catholics, of course, will complain that this chart is not fair or does not focus on the
dissimilarities. Well, they are right in that it does not focus on the dissimilarities which are many. But
that isn't the point of this comparison, is it?
The Protestant Reformation happened for a reason: to get back to biblical theology and to be rid of
extra-biblical teaching. The counter-cult movement has happened for a reason, to get back to biblical
theology and to be rid of extra-biblical teaching.
Eternity is a long time to be wrong.
Is the Catholic Catechism's view on the Muslim god wrong?

According to the Catechism of the Catholic Church, paragraph 841, Muslims together with the
Catholics, "adore the one, merciful God." This cannot be true. The Muslims and Catholics do not
adore the same God. In order to provide sufficient context for the quote from the Catechism, I have
cited 2 paragraphs before and after paragraph 841.

1. 839 "Those who have not yet received the Gospel are related to the People of God in various
ways."[325] The relationship of the Church with the Jewish People. When she delves into
her own mystery, the Church, the People of God in the New Covenant, discovers her link
with the Jewish People,[326] "the first to hear the Word of God."[327] The Jewish faith,
unlike other non-Christian religions, is already a response to God's revelation in the Old
Covenant. To the Jews "belong the sonship, the glory, the covenants, the giving of the law,
the worship, and the promises; to them belong the patriarchs, and of their race, according to
the flesh, is the Christ",[328] "for the gifts and the call of God are irrevocable."[329]
2. 840 And when one considers the future, God's People of the Old Covenant and the new
People of God tend towards similar goals: expectation of the coming (or the return) of the
Messiah. But one awaits the return of the Messiah who died and rose from the dead and is
recognized as Lord and Son of God; the other awaits the coming of a Messiah, whose
features remain hidden till the end of time; and the latter waiting is accompanied by the
drama of not knowing or of misunderstanding Christ Jesus.
3. 841 The Church's relationship with the Muslims. "The plan of salvation also includes those
who acknowledge the Creator, in the first place amongst whom are the Muslims; these
profess to hold the faith of Abraham, and together with us they adore the one, merciful God,
mankind's judge on the last day."[330]
4. 842 The Church's bond with non-Christian religions is in the first place the common origin
and end of the human race: All nations form but one community. This is so because all stem
from the one stock which God created to people the entire earth, and also because all share
a common destiny, namely God. His providence, evident goodness, and saving designs
extend to all against the day when the elect are gathered together in the holy city. . .[331]
5. 843 "The Catholic Church recognizes in other religions that search, among shadows and
images, for the God who is unknown yet near since he gives life and breath and all things
and wants all men to be saved. Thus, the Church considers all goodness and truth found in
these religions as 'a preparation for the Gospel and given by him who enlightens all men that
they may at length have life.'

If the Catholic Church is stating in paragraph 841 above, in the Catechism of the Catholic Church,
that there are those within the Muslim community who may find salvation by becoming Christians and
not remaining Muslims, then I would agree. But, it does not appear to be saying this.
What is disturbing is the statement that "together with us they [Muslims] adore the one, merciful
God, mankind's judge on the last day." So, Catholics and Muslims both "adore the one, merciful
God." Quite frankly, it would seem that the Roman Catholic church has a faulty understanding of the
God of Islam.
Islam acknowledges that it serves the God of Abraham, Isaac, and Jacob. But in so doing it is
stating that Abraham, Isaac, and Jacob, were Muslims! Furthermore, Islam denies that God is a
Trinity.

• "They do blaspheme who say: Allah is one of three in a Trinity: for there is no god except One
Allah. If they desist not from their word (of blasphemy), verily a grievous penalty will befall the
blasphemers among them," (Quran 5:73, Yusufali).
• "They surely disbelieve who say: Lo! Allah is the third of three; when there is no Allah save the
One Allah. If they desist not from so saying a painful doom will fall on those of them who
disbelieve," (Quran 5:73, Pickthal).
Since the Trinity is the true biblical doctrine of God, how can the Muslims who deny the Trinity
"adore the one, merciful God"? They cannot. Furthermore, in Christianity, Jesus is divine in nature
(John 1:1,14; Col. 2:9). Yet, the Quran states that Jesus is not divine:

• "They indeed have disbelieved who say: Lo! Allah is the Messiah, son of Mary. Say: Who then
can do aught against Allah, if He had willed to destroy the Messiah son of Mary, and his mother
and everyone on earth? Allah's is the Sovereignty of the heavens and the earth and all that is
between them. He createth what He will. And Allah is Able to do all things," (Quran 5:17,
Pickthal).
• "And the Jews say: Ezra is the son of Allah, and the Christians say: The Messiah is the son of
Allah. That is their saying with their mouths. They imitate the saying of those who disbelieved
of old. Allah (Himself) fighteth against them. How perverse are they!" (Quran 5:30, Pickthal)

Obviously, the Muslims do not adore the one true merciful God because they serve a false god.
They deny the Trinity and the deity of Christ. Quite simply, they deny the true God and are not
capable of adoring the true God as long as they hold to the false teachings of Islam. Therefore the
paragraph in the CCC115 cannot be correct.

"The Church's relationship with the Muslims. "The plan of salvation also includes those who
acknowledge the Creator, in the first place amongst whom are the Muslims; these profess to hold the
faith of Abraham, and together with us they adore the one, merciful God, mankind's judge on the last
day." (CCC 841).

CCC means Catechism of the Catholic Church. It is an official Catechism released by the Roman Catholic
115

church. "This catechism aims at presenting an organic synthesis of the essential and fundamental contents of
Catholic doctrine
Christadelphianism
Introduction
Christadelphianism is relatively new religious system. It claims, like many other non-Christian cults, to
be the authentic Christian Church with authentic Christian doctrines. However, it denies the Trinity as
well as the deity of Christ and the Holy Spirit. In addition, it states that the Devil is not a real living
entity but is, instead, our natural tendency to sin. It also says that Jesus was a created being who had
a fallen nature who himself needed to be redeemed.

There are, however, disagreements within the camp of Christadelphianism. Several Christadelphians
have pointed out to me that there are differences of opinion within their sect regarding the fallen
nature of Christ. Nevertheless, they all deny the deity of Christ and this is enough to put them outside
the camp of Christianity. A Jesus who is not divine, cannot save anyone -- especially one that needs
to be redeemed himself.

In my opinion, Christadelphianism is a dangerous cult because it brings people to eternal damnation


by teaching a false god and false gospel. It deviates from the central doctrines of the Christian faith
sufficiently to make it non-Christian.

1. What are some of the aberrant doctrines of the Christadelphians? p. 122


2. Why is Christadelphianism not Christian? p. 124
3. Is God ever seen? p. 131
4. Which Christadelphian quotes do you find most interesting? Why? p. 133
5. Why is the Christadelphian teaching of Jesus having a sin nature so bad? p. 134
6. Can the Christadelphian Jesus with a fallen nature save anyone? P. 141
7. What is the Christadelphian view of the Holy Spirit? p. 143
8. What questions might you ask a Christadelphian? p. 144
What do the Christadelphians Teach?

Though they acknowledge many truths found in the Bible, they deny many others.

1. They believe the Bible is the infallible and inerrant word of God. (The Christadelphians: What
They Believe and Preach, p. 82)
2. They teach there is only one God. (Isaiah 43-45)
3. They teach that Jesus had a sin nature (What They Believe, p. 74)
4. They teach that Jesus needed to save himself, before he could save us. (Christadelphian Answers,
p. 24)
5. They teach that Jesus will return and set up his kingdom on earth. (What They Believe , p. 268)
6. They believe that there has been an apostasy and that Christianity is a false religious system. (A
tract titled “Christendom Astray Since the Apostolic Age, Detroit Christadelphian Book Supply)
7. They believe annihilation of the wicked. (What They Believe, p. 187).
8. They believe that baptism is necessary for salvation. (What They Believe , p. 71,72, 207-210)
9. They believe that it is possible to lose one’s salvation. (What They Believe , p. 212)
10. They deny the doctrine of the Trinity. (What They Believe, p. 84-87)
11. They deny that Jesus is God in flesh. (Answers, p. 22)
12. They deny that Jesus existed prior to his incarnation. (What They Believe , p. 85,86)
13. They deny the personhood and deity of the Holy Spirit. (What They Believe , p. 115)
14. They deny the substitutionary atonement of Christ. (Answers, p. 25; What They Believe, p. 71)
15. They deny salvation by grace through faith alone. (What they Believe, p. 204)
16. They deny immortality of the soul. (What They Believe , p. 17).
17. They deny that a person exists after death. (What They Believe, p. 17)
18. They deny the existence of hell and eternal punishment. (What They Believe, p. 188-189)
19. They deny the existence of the fallen angel Lucifer as the devil. (Answers, p. 100)

Like so many other cult groups that claim to be the restored truth, they have their own
interpretations of the Bible that deviate greatly from orthodox Christianity.
Christadelphian History
Christadelphianism is a religious movement begun by Dr. John Thomas who was born in London
England on April 12, 1805. In 1832 he immigrated to the United States. On the way to New York, his
ship encountered several terrible storms that threatened shipwreck and death. Dr. Thomas promised
God that if he were delivered, he would devote his life to the study of religion. He made it to America
and kept his promise.
Upon arrival, he joined the Campbellite group also known as the Disciples. He was baptized and
began to study. He studied greatly and soon found himself at odds with the Campbellites and left.
Many from the Campbellite group followed him. This is the beginning of the Christadelphian
movement, though it wasn’t called that yet.
In 1834 Dr. Thomas started a magazine called "The Apostolic Advocate." This is where he really
began to disseminate his teachings. He was greatly interested in prophecy and devoted much effort to
understanding biblical eschatology.
In 1839 Thomas moved to Illinois and in 1842 he became editor of a magazine called "The
Investigator." Five years later, he started another magazine called "The Herald of the Future Age." By
this time he was living in Virginia.
In 1848, near the time when Christadelphianism was founded in America, he returned to England
to speak on his brand of religion and found the soil there fertile. To this day, England has the largest
number of Christadelphians. While in England he wrote the book called "Elpis Israel" which means
"Hope of Israel." It is a thorough work of his beliefs discussion creation, God’s law, sin, death,
immortality, religion, the coming Kingdom, and a host of other subjects. He then returned to America.
The Christadelphians do not believe in participating in war. So, when the Civil War broke out, they
refused to go. In order to be recognized as a religious group that did not believe in fighting, they
needed a name. Dr. Thomas gave them the name "Christadelphian" which, in Greek means "Brethren
of Christ."
In 1862, Thomas returned to England again and found that his book "Elpis Israel" had helped to
bring about congregations that followed his theology. He lectured extensively and helped to anchor
Christadelphianism in England. He returned to America again.
Thomas visited England one more time in 1869 after writing the book "Eureka."
On March 5, 1871 Dr. Thomas died in New York. He is buried in Brooklyn.

Thomas was a tireless worker who sought to study and discover God’s true meaning and doctrine
of the Bible. Unfortunately, despising the counsel and wisdom of those more learned than himself, he
sought to single-handedly "rediscover" the true gospel which, in his opinion, had been lost from the
earth. Like so many others in the 19th century, he began a religious movement that really is a
development of his personal beliefs. Therefore, the Christadelphian religion, like Mormonism,
Jehovah’s Witnesses, and Christian Science, is merely another erring religious system begun by a
single person who claimed to know more than anyone else about the Bible.
It is a non-Christian cult.
Is Christadelphianism Christian?
No, Christadelphianism is not Christian. Like all cults, Christadelphianism denies one or more of the
essential doctrines of Christianity: Jesus is God, the physical resurrection, and salvation by grace. In
this case, it is the deity of Christ and salvation by grace through faith are the problems with this
group.
In regards to Jesus, it teaches that....

• Jesus had a sinful nature The Christadelphians, What They Believe, by Harry Tennant, The
Christadelphian, England, p. 74 - this is a Christadelphian book. )
• Jesus needed salvation, (Christadelphian Answers, ed. by Frank G. Jannaway, The Herald Press,
p. 25 - another Christadelphian book).
• Jesus is not God in flesh (Answers, p. 22).
• That Jesus' atonement was not substitutionary (Answers, p. 25; What They Believe, p. 71).
• Baptism is necessary for salvation (What They Believe , p. 71,72, 207-210)

Of primary importance is what the Christadelphians say about Jesus. They deny He is divine in
nature. According to John 1:1,14, John 8:58 (with Exodus 3:14), and Col. 2:9, Jesus is God.

• "In the beginning was the Word, and the Word was with God, and the Word was God...And the
Word became flesh, and dwelt among us, and we beheld His glory, glory as of the only
begotten from the Father, full of grace and truth," (John 1:1,14).
• Jesus said to them, "Truly, truly, I say to you, before Abraham was born, I am," (John 8:58).
With, "And God said to Moses, "I AM WHO I AM"; and He said, "Thus you shall say to the sons
of Israel, ‘I AM has sent me to you,’" (Exodus 3:14).
• "For in Him all the fullness of Deity dwells in bodily form," (Col. 2:9).

Furthermore, Jesus said in John 8:24, "I said therefore to you, that you shall die in your sins; for
unless you believe that I am, you shall die in your sins." Also, John the Apostle said in 1 John 4:2-4,
"By this you know the Spirit of God: every spirit that confesses that Jesus Christ has come in the flesh
is from God; 3and every spirit that does not confess Jesus is not from God; and this is the spirit of the
antichrist, of which you have heard that it is coming, and now it is already in the world." You can see
that denying that Jesus has come in the flesh (that He is God in flesh per John 1:1,14), is the spirit of
antichrist.
Since we are justified by faith (Rom. 5:1; Eph. 2:8-9), it is crucial to crucial to have the proper
object of faith. All Satan has to do is to get someone to believe in a false Jesus and the person is lost
(Matt. 24:24). A false Jesus cannot save and only the true Jesus reveals the true God (John 14:6;
Luke 10:22; John 17:3). Since Jesus is actually God in flesh (John 1:1,14; 20:28; Col. 2:9; Phil. 2:5-
8; Heb. 1:8), it follows that those who deny His divine nature -- and ascribe a sinful one to Him as the
Christadelphians do -- cannot have the true Jesus and are, therefore, serving a false God.
Second, the Christadelphians deny the substitutionary atonement of Jesus. They say that He did
not take our place on the cross and that He did not bear our sins. This is in direct contradiction to
Scripture. 1 Pet. 2:24 says, "He Himself bore our sins in His body on the cross, that we might die to
sin and live to righteousness; for by His wounds you were healed." Instead, they teach a kind of
representation that was not effective to remove sin and say, "Christ did not die as our substitute, but
as our representative" (Answers, p. 25).
Additionally, in Answers, page 24, it says, "But it is equally true that, being 'made sin for us' ( 2
Cor. 5:21), he himself required a sin offering..." In other words, they are saying that Jesus Himself
also needed to be saved. This is absolutely unbiblical and heretical and needs to be labeled for what it
is: false doctrine. Jesus was without sin (1 Pet. 2:22), the exact representation of the nature of God
(Heb. 1:3). Since God is sinless and Holy, so is Jesus in nature and essence.
Furthermore, the Christadelphians, by having a Jesus who has a sin nature, cannot have a proper
sacrifice by which their sins are atoned for. According to the Old Testament, the sacrifice for sins had
to be without blemish (Deut. 17:1). Having a sin nature would definitely be a blemish which would
invalidate the sacrifice.
Third, the Christadelphians add a work to salvation. They say that baptism is part of the saving
process. But, baptism is not necessary for salvation. Instead, it is a representation of the inward
reality of regeneration (1 Pet. 3:21), a covenant sign of God's work upon the heart (Col. 2:11-12).
Gal. 5:1-12 speaks of the grave error of some people who thought that they needed to partake in
some part of the Law (circumcision) to be saved. Paul quickly denounced them with very strong words
(Gal. 5:12). Additionally, Rom. 5:1 says that we are justified by faith, not by faith and baptism.
Rom. 3:23 says we are saved not by the works of the Law; that is, not by anything that we do. Since
our righteous deeds are filthy rags before God (Isaiah 64:6), we must completely rely upon the grace
of God for our salvation -- which is by faith in Jesus who is God, the creator in flesh.
The Christadelphian religion is a false religion. it is definitely not Christian. This is not to say that
there are not decent people who intend to serve God honestly and truthfully. But sincerity does not
bridge the gap between God and man. Only the blood of the real Jesus does that, not a false Christ
with a sin nature who himself needed salvation.
Answering a "refutation" of "Is Christadelphianism Christian?"

It is quite a complement when the groups I tackle on CARM attempt to answer the charges of them
not being Christian. Following is one of the papers I found on Christadelphian website that attempted
to refute my paper titled "Is Christadelphianism Christian. I will respond to the "refutation" of my
paper. My comments are in italics.

Christadelphian:
The C.A.R.M. has an interesting approach to the condemnation of various cults. A religious system
is either 'Christian,' and therefore able to save despite minor errors, or 'non_Christian,' which cannot
save because of major errors. The Christadelphians, they say, are non_Christian. The Christadelphian
faith cannot save.Why not? The mind behind the C.A.R.M. has taken it upon himself to conjure up the
'Three Essential Doctrines' of Christianity. Any group which denies any one of these doctrines is
non_Christian. These three doctrines are: Jesus is God, the physical resurrection, and salvation by
grace through faith.

My Response:
No cult member considers himself to be false. But, Christianity by definition states
Christadelphianism to be false. The Three Essential Doctrines that I have on my website are not
"conjured up" by me or anyone else. These essentials are essentials proclaimed by the Bible. I have
merely brought them to peoples' attention. For this Christadelphian author to state that I have
conjured them up tells me that he did not read the article where I give ample biblical references
establishing their essential nature.

Christadelphian:
Now where did the C.A.R.M. get the idea that these are the three essential doctrines of
Christianity? Certainly not from the Bible. The Bible teaches that the gospel is 'the things concerning
the Kingdom of God and the Name of Jesus Christ' (Acts 8:12).

My Response:
Again this person fails to cite the scriptures that I have listed in that article. Therefore, I will briefly
list them here:

1. Jesus is God in flesh


A. 1 John 4:2-3, "This is how you can recognize the Spirit of God: Every spirit that
acknowledges that Jesus Christ has come in the flesh is from God, but every spirit that does
not acknowledge Jesus is not from God. This is the spirit of the Antichrist, which you have
heard is coming and even now is already in the world."
i. The above verse needs to be cross referenced with John 1:1,14 (also written by John)
where he states that the Word was God and the Word became flesh. In 1 John 4:2-3,
John is not simply stating that Jesus existed, for that would be too obvious. Rather, he is
dealing with the issue of Jesus’ incarnation.
ii. Likewise, Jesus said in John 8:24: "I said, therefore, to you, that you will die in your
sins. For if you do not believe that I am, you will die in your sins."
a. "I AM" is a phrase God used to describe Himself in Exodus 3:14.
2. Salvation by Grace
A. "For it is by grace you have been saved, through faith -- and this not from yourselves, it is
the gift of God -- not by works, so that no one can boast" (Eph. 2:8-9, NIV). "You who are
trying to be justified by law have been alienated from Christ; you have fallen away from
grace," (Gal. 5:4).
3. The Resurrection of Christ
A. "And if Christ has not been raised, our preaching is useless and so is your faith," (1 Cor.
15:14). "And if Christ has not been raised, your faith is futile; you are still in your sins," (1
Cor. 15:17).
i. To deny the physical resurrection of Jesus is to deny Jesus' work, sacrifice, and our
future resurrection.
ii. These verses clearly state that if you say that Jesus did not rise from the dead (in the
same body He died in -- John 2:19-21), then your faith is useless.

Christadelphian:
The C.A.R.M. says that Christadelphians deny the essential doctrine that Jesus is God in flesh.
Christadelphians believe that Jesus was made of the same fallen, no-good substance that composes
the rest of humanity: sinful flesh. Because of this, they follow a false Jesus, and 'they are then
damned' according to the C.A.R.M. However, never did Jesus preach that we have to believe he is God
_ it is not an essential doctrine. In fact, it is not a doctrine at all. The Old Testament teaches One God
and Father of all, not One God, a Father, a Son and a Holy Spirit. The New Testament must teach the
same thing, since God does not change. I am aware that I am arguing without Biblical backup: this is
because I do not want to repeat what is said in the Trinity section of this website. If you want to see
proof that Jesus was made of sinful flesh, go there.

My Response:
The Bible does not teach that Jesus had a sinful nature. It teaches that Jesus was sinless ( 1 Pet.
2:22). He was both God and man at the same time. Col. 2:9 says, "for in Him dwells all the fullness of
deity in the bodily form." John 1:1,14 says, "In the beginning was the Word and the Word was with
God and the Word was God... and the Word became flesh and dwelt to among us."
As far as Jesus not teaching that we needed to believe that He was God, we simply need to look at
His own words in John 8:24 where Jesus said, "unless you believe that I am, you will die in your sins."
Again, Jesus uses the title "I am" in reference to Himself. Of course, many cultists state that Jesus was
simply saying that He was the messiah. But, note that in John 8:58, Jesus said, "before Abraham was,
I am." At this, the Pharisees picked up stones to kill Him, but Jesus fled. Then in John 10:30-33, they
again picked up stones to kill Him and the reason they gave for trying to kill Him was because He
being a man made himself out to be God (John 10:33). When Jesus said "I am" He was equating to
Himself the very name of God found in Exodus 3:14 where God says to Moses, "I AM WHO I AM"; and
He said, "Thus you shall say to the sons of Israel, ‘I AM has sent me to you.’"

Christadelphian:
The C.A.R.M. also speaks of one doctrine of Christadelphians as 'absolute heresy' and 'demonic
doctrine.' This is the doctrine that Jesus Himself had to be saved. Why is this doctrine demonic? Jesus'
mother required atonement for giving birth to him (Luke 2:24) which would not have been necessary
unless she was bringing forth a fallen creature.

My Response:
The reason the Christadelphian doctrine is so bad is because it denies that Jesus Christ is God and
states that Jesus was a sinner who needed salvation. That is blasphemy.

Christadelphian:
The C.A.R.M. also says that Christadelphians deny the essential doctrine of salvation by grace
through faith. I quote, "Christadelphians add a work to salvation. They say that baptism is part of the
saving process. But, baptism is not necessary for salvation. Instead, it is a representation of the
inward reality of regeneration (1 Pet. 3:21), a covenant sign of God's work upon the heart (Col. 2:11-
12)." These are flowery words with no real meaning: inward reality? A covenant sign of God's work
upon the heart? You should note that when you are writing and put a Bible reference in brackets at the
end of a sentence, it is supposed to back up what you have said. However, the two references they
quoted have nothing to do with inward realities or the heart. What does Col. 2:11-12 say? Speaking of
Christ, "In whom ye also are circumcised with the circumcision made without hands, in putting off the
body of the sins of the flesh by the circumcision of Christ:" In the context Paul is explaining that the
traditions of men such as circumcision are obsolete (v. 8). Writing to the Colossian believers (mostly
uncircumcised Gentiles) he tells them that through the circumcision of Christ, they have put off the
sins of the flesh, and can be redeemed. What is this 'circumcision of Christ' that is required to put off
sin? Read on _ verse 12: "Buried with him in baptism, wherein also ye are risen with him through the
faith of the operation of God, who hath raised him from the dead." When a person is immersed in
water, they are figuratively buried and risen with Christ. Through this act of faith, we can obtain
resurrection from the dead, since God raised Christ from the dead and we are being 'buried with
Christ.'

My Response:
Baptism is a very important part of Christianity. All Christians should be baptized. But, baptism is
not an essential requirement of salvation. We can see this when we look at the whole of scripture
which tells us that we are justified (made righteous) by faith ( Rom. 5:1), not faith and something we
do (Rom. 4:5); that people are saved before baptism (Acts 10:44-48; that baptism is not part of the
gospel which saves (1 Cor. 15:1-4); and that Paul came to preach the gospel and not to baptize (1
Cor. 1:17). For more on this please read "Is Baptism Necessary for Salvation?"

Christadelphian:
I should also speak of how the C.A.R.M. condemns baptism as the works of the Law. Paul speaks
frequently in his epistles about how the works of the Law were not necessary to be saved anymore.
The Law had waxed old as a garment; it was a schoolmaster to teach the Jews about the coming
Messiah. This is all fine and good. Now look at the C.A.R.M.'s major stretch of reasoning. Rom. 3:20
tells us that "by the deeds of the law shall no flesh be justified in his [God's] sight." This is the Law of
Moses, which at this point had become obsolete. But the C.A.R.M. says, in other words, we are not
saved by anything we do. Where did that come from? We are not saved by the Law of Moses: we are
saved by grace through faith. And as Col. 2:12 told us, baptism is the operation of our faith! Baptism
is necessary for salvation.

My Response:
I do not want to get into a dissertation on baptismal regeneration and its error. My previous
mention of the article "Is Baptism Necessary for Salvation?" is worth reading here.

Christadelphian:
C.A.R.M.'s conclusion to this section is that Christadelphianism is a false religion. They are sincere,
but "sincerity does not bridge the gap between God and man. Only the blood of the real Jesus does
that." I agree, sincerity cannot save us. However, the C.A.R.M. has not shown us that
Christadelphianism is a false religion.

My Response:
The fact that Christadelphianism denies that Jesus is God in flesh is sufficient to make it a non-
Christian cult. Faith is only as good as who you put it in. The Christadelphians have a "savior" with a
sinful nature who himself needed to be saved. This sinful version of Jesus cannot save anyone. The
true Jesus needed no savior. The Christadelphians do not have the true Jesus and because of that their
faith is misplaced and in vain.

Dear reader, do not be deceived by the Christadelphians. Jesus is God in flesh. It is only God who
can save us from our sins. We have a perfect and holy High Priest, Jesus. He was sinless and will
forever love us and hold us close to Him. But, the Jesus of the Christadelphians, our older brother who
had a sin nature, is false. A false Christ cannot save.
Jesus said, "For false Christs and false prophets will arise and will show great signs and wonders,
so as to mislead, if possible, even the elect," (Matt. 24:24).
The Christadelphians, John 1:1, and "The Word Became Flesh"

The Bible bears witness of Jesus (John 5:39). He is our Savior, our King, and Lord. But not all who
call themselves Christian agree on who Jesus is. Some say He is God in flesh, others that He is an
angel who became a man, and still others teach He only came into existence at His birth. Such is the
position of the Christadelphians. To them, Jesus did not exist as God. To them, He was just a man who
first existed at His birth.
If you are a Christian who knows His Bible, then you will immediately recognize the error of the
Christadelphians. The Bible says that whoever denies that Jesus has come in the flesh is of the spirit of
the Antichrist (1 John 4:1-2). Of course, the Christadelphians agree that Jesus came in the flesh. But
they will not agree that He is God in flesh.
John wrote 1 John and the gospel of John. In John 1:1,14, he said, "In the beginning was the
Word, and the Word was with God, and the Word was God. . .and the Word became flesh and dwelt
among us. . ." Obviously, from the context, John is not simply saying that you must believe that Jesus
lived, you must believe that He is the Word made flesh. And since he already said that the Word was
God, Jesus, therefore, is God in flesh.
This seems simple enough. But it isn’t for the Christadelphians. In their pamphlet "Who is Jesus
Christ?", the "Word" is discussed. On page 12, in reference to John 1:1, the pamphlet says, "The
Greek term translated 'word' is logos. It signifies the outward form of inward thought or reason, or the
spoken word as illustrative of thought, wisdom and doctrine. The Bible teaching is that in the very
beginning, God’s purpose, wisdom or revelation was proclaimed through His Word. This Word was
'with God' in that it emanated from Him; it 'was God' in that it represented Him to mankind. . ."
The problem with their reasoning is not that their definition, in itself, is incorrect. For it can be said
that the Word was indeed the wisdom and emanation from God. But that is not all it is saying. It is
saying that the Word WAS God. Jesus IS the Word. He isn’t simply a manifestation of some divine
attribute or quality. Also, what about the context?
In John 1:2-3 it says, "He was with God in the beginning. Through him all things were made;
without him nothing was made that has been made" (NIV). First, the word is referred to as masculine.
Wisdom in Proverbs 8:1-2 is personified as feminine. There is a difference. Second, the Word is who
created all things (See also Colossians 1:16-17). Of course, it is naturally understood that this does
not include God Himself. But all that is made, has been made by the Word that became flesh. Third,
the Word is revered to as a person, not a quality which the Christadelphians have imposed into the
text.
In reference to Acts 1:14, "The Word became flesh and dwelt among us and we beheld His glory,
glory of the only begotten full of grace and truth," the pamphlet states on page 13, "When did the
begettal take place? When the Holy Spirit came upon Mary. By that means, the Declaration of Divine
wisdom found its substance and reality in the person of the Lord Jesus Christ." It is interesting to note
that they call wisdom ‘divine.’ They are saying that divine wisdom became flesh. Is not Jesus, then,
divine since He is the incarnation of divinity? They would disagree. But that is what they are saying,
though they don’t realize it.
If Jesus is not God in flesh, then why is He worshiped (Matt. 2:2, 11, 14:33, John 9:35-38, Heb.
1:6)? This is especially important since Jesus said that you are to worship God (the Father) only ( Matt.
4:10). Yet, Jesus receives worship and never rebukes anyone for it.
If Jesus is not God, then why is He called God by Thomas who said to Jesus in John 20:28, "My
Lord and my God." Jesus didn’t correct him for his error.
Once while in a Christadelphian church (known as a Hall; their body of believers who are
Christadelphians are called an ecclesia), a woman challenged me to find any place in the Bible where
Jesus is called God. When I showed her the verse, she was silent. No one there has answered it yet.
The verse is Hebrews 1:8. Here is the context: Heb. 1:5-8 . . .

For to which of the angels did He [God] ever say: "You are My Son, Today I have begotten
You"? And again: "I will be to Him a Father, And He shall be to Me a Son"? 6But when He again
brings the firstborn into the world, He says: "Let all the angels of God worship Him." 7And of
the angels He says: "Who makes His angels spirits And His ministers a flame of fire." 8But to
the Son He says: "Your throne, O God, is forever and ever; A scepter of righteousness is
the scepter of Your Kingdom."
Jesus is called God by God. If He is not God, then why does the Father call Him God? Is the Father
wrong? Is the writer of Hebrews wrong. Or, are the Christadelphians wrong?
While at another ecclesia, I asked some Christadelphians about Jesus being worshiped. They told
me they thought He was worthy of worship. They said they never worshiped Jesus. I asked them why
not. They didn’t have an answer.
In the Christadelphian pamphlet, "Who is Jesus Christ?" none of the verses about Jesus being
worshiped or called God were addressed. I think this is revealing. It is easy to produce clever
arguments against various proof texts of Jesus deity (Col. 1:16-17; John 8:58, etc.), as the pamphlet
does. But when it doesn’t address the most basic of verses that deal with Jesus’ being worshiped and
called God, I must conclude that they have not done all the research needed and that their conclusions
are in error. And they are in error.
Remember, faith is only as good as who it is placed in. The Christadelphian Jesus is not the Jesus
of the Bible. The Christadelphian Jesus isn't God. The Jesus of the Bible is.
Is God ever seen?
Christadelphianism is known theologically as dynamic monarchianism. It teaches there is a god,
but that god is a single person. In contrast to this, the Trinity is one God who exists in three persons:
the Father, the Son, and the Holy Spirit. To a Christadelphian, if God ever appeared in the Old
Testament period, it would have to be the Father. This is problematic for them for two reasons. First,
the Father cannot be seen according to scripture, and second, if God did appear, then it would have to
be the Father, which they cannot accept. Nevertheless, the following study answers both those issues
and leaves the Christadelphian, with a problem to solve.
In the Old testament God appeared many times to many people (Gen. 17:1; 18:1; Ex. 6:2-3;
24:9-11; 33:20; Num. 12:6-8). Yet there are verses that say that you cannot see God (Exodus 33:20;
John 1:18). It would seem that there is a contradiction, but there isn’t.
I will show you that there is no contradiction and that the theology that claims that God is one
person, is incorrect. This study is directed at the Christadelphians, but it would also be appropriate for
the Jehovah’s Witnesses and The Way International.
The Bible clearly teaches that God was seen in the O.T. This fact is even mentioned in the New
Testament. Acts 7:2 says, "The God of glory appeared to our Father Abraham. . . " Did God appear or
didn’t He? Let’s take a look at God’s word.

• Exodus. 6:2-3, "God spoke further to Moses and said to him, "I am the LORD; and I appeared
to Abraham, Isaac, and Jacob, as God Almighty, but by My name LORD I did not make myself
known to them."
• Exo. 24:9-11, "Then Moses went up with Aaron, Nadab and Abihu, and seventy of the elders of
Israel, and they saw the God of Israel; and under His feet there appeared to be a pavement of
sapphire, as clear as the sky itself. Yet He did not stretch out His hand against the nobles of the
sons of Israel; and they beheld God, and they ate and drank."
• Exodus 33:11, "Thus the LORD used to speak to Moses face to face, just as a man speaks to
his friend..."
• Num. 12:6-8, "He [God] said, "Hear now My words: If there is a prophet among you, I, the
LORD, shall make Myself known to him in a vision. I shall speak with him in a dream. Not so,
with My servant Moses, He is faithful in all My household; with him I speak mouth to mouth,
even openly, and not in dark sayings, and he beholds the form of the LORD . . . "
• Acts 7:2, "And he [Stephen] said, "Hear me, brethren and fathers! The God of glory appeared
to our father Abraham when he was in Mesopotamia, before he lived in Haran . . . "

Some will say that the appearances of God were nothing more than visions, dreams, or angels that
represented God. This is definitely the case in some instances of the O.T., but it is not the case here.
For example, is an angel God Almighty? Of course not. Yet God says in Exodus 6:2-3 that He appeared
to Abraham, Isaac, and Jacob, as God Almighty.
Are these visions or dreams? Not so with Moses. Consider Num. 12:6-8 where God says that He
does not appear to Moses in a vision or dream. Rather, Moses beholds God’s very form.
Was God seen? Yes. Consider Exodus 24:9-11 where it says that 74 people "saw the God of
Israel." It does not say a vision, a dream, a cloud, a flame, etc. It says that they saw God.
Cult groups generally teach that when God appeared in the O.T., it was a representative angel,
that it was a ‘manifestation’ of God due to that angel possessing the ‘name’ of God upon him. As God’s
direct representative, the angel was considered to be God Himself, though not really God. This is
nullified by Exodus 6:2-3 where God says that He appeared as God Almighty. This is a title only
applied to God Himself.
What then of the verses that say that God cannot be seen John 1:1 and Exodus 33:20?
John 1:18, "No one has seen God at anytime. The only begotten Son in the bosom of the Father,
He has explained Him." The context is important. John began his book with the words, "In the
beginning was the Word and the Word was with God and the Word was God." In verse 14 he says,
"And the Word became flesh and dwelt among us. . ." The Word was obviously Jesus. The Word was
with God. God, contextually, would be the Father. When it says in John 1:18 that no man has seen
God, it is referring to the Father. This is supported by Jesus’ own words in John’s gospel. Consider the
following:

"Not that anyone has seen the Father, except He who is from God; He has seen the Father," (John
6:46). Jesus declares that the Father has not been seen.
Exodus 33:20, "You cannot see My face, for no one can see Me and live." This verse is a bit more
difficult. But a look at the following verses tells us that God allowed Moses to see His backside, not His
face.
There are three other verses in the Bible worth examining here. They deal with the plurality of God
and they will help support the idea that there is more than one person in the Godhead and shed
possible light on the verses of John 1:18 and Exodus 33:20. They are as follows.
Gen. 19:24, "Then the LORD rained brimstone and fire on Sodom and Gomorrah, from the LORD
out of the heavens." The word "LORD" in the Hebrew is the word YHWH from where we get God’s
name, sometimes known as Jehovah. Look at the verse and you will see that there appears to be two
Jehovah’s; that is, two LORD’s. Why is that?
Amos 4:10-11, "I sent among you a plague after the manner of Egypt; Your young men I killed
with a sword, Along with your captive horses; I made the stench of your camps come up into your
nostrils; Yet you have not returned to Me," Says the LORD. 11"I overthrew [some] of you, As God
overthrew Sodom and Gomorrah, And you were like a firebrand plucked from the burning; Yet you
have not returned to Me," Says the LORD."
Who is the one speaking in verse 10? The LORD, YHWH. But the LORD says in verse 11, "I
overthrew some of you, as God overthrew Sodom and Gomorrah. . ." The LORD is speaking and He
speaks about God overthrowing Sodom and Gomorrah. There seems to be a plurality, a more than
one-ness occurring here.
If you are a Trinitarian, there is no problem. God Almighty was seen in the Old Testament, only, it
wasn’t the Father. It was Jesus. Jesus said in John 8:58, "Before Abraham was, I AM." He was quoting
God speaking to Moses at the burning bush in Exodus 3:14. Jesus claimed to be the I AM, YHWH. In
fact, YHWH means, I AM.
Was God seen in the Old Testament? Yes. Was it the Father? No. The Christadelphian theology is
wrong for there is more than one person in the God head.
Interesting Quotes from Christadelphianism
(Unless marked, all quotes are from the Christadelphian book, The Christadelphians: What They
Believe and Preach, by Harry Tennant, The Christadelphian, 404 Shaftmoor Lane, Birmingham B28
8SZ, England, 1986.)

• "It will surprise some readers to know that nowhere in Scripture are the words "immortal" and
"soul" brought together. Immortality is God's own inherent nature, and His alone" (page. 17).
• "The second secret of the cross is that it is the source of the forgiveness of sins. It is not a debt
settled by due payment. It is not a substitutionary offering whereby someone is paid a price so
that others might then go free" (page 71).
• "The Bible approach is much simpler and much more satisfying. Forgiveness comes to the man
who believes the Gospel, repents, and is baptized in the name of Christ" (page 71).
• "The wondrous benefits from the saving work of Jesus flow to us and are effective for us when
we come in faith, repentant, and join Jesus in his death by baptism into his name" (page 72).
• "Therefore, we conclude that it is not only that Jesus was called a sinner at his trial by his
enemies or that he was "numbered with the transgressors" when he was crucified between two
thieves, but more particularly that he shared the very nature which had made a sinner out of
every other man who had borne it" (page 74).
• "There is no hint in the Old Testament that the Son of God was already existent or in any way
active at that time" (page 85).
• "Jesus Christ, the Son of God, was first promised, and came into being only when he was born
of the virgin Mary" (page 86).
• "We ask the question: When was Jesus 'in the form of God'? Christadelphians believe that Jesus
was in the form of God by his birth through begettal by the Father, by speaking the words of
God and doing His works" (page 87).
• "Jesus worships God: God worships no one" (page 88).
• "The Spirit is not a 'separate' or 'other' person. It is God's own radiant power, ever out flowing
from Him, by which His 'everywhereness' is achieved. The Spirit is personal in that it is of God
Himself: it is not personal in the sense of being some other person within the Godhead" (page
115).
• "A believing, repentant person receives forgiveness of sins by being baptized" (pages 207-8).
• "True baptism removes past sins" (page 208).
• "Therefore the wonderful work of baptism is essential to salvation" (page 210).
• "Salvation is not a one-for-all, irreversible happening" (page 212).
• "He [Jesus] saved himself in order to save us." (Christadelphian Answers, Compiled by Frank G.
Jannaway. A reproduction of an original edition by, The Herald Press, 4011 Bolivia, Houston,
Texas, 77092, 1920. page 24)
• "And it was for that very reason -- being a member of a sinful race -- that the Lord Jesus
himself needed salvation." (Answers, p. 24)
• "The terms Satan and Devil are simply expressive of "sin in the flesh" in individual, social, and
political manifestations." (Answer, p. 100)
Did Jesus have a sin nature?

No, Jesus did not have a sin nature. However, that has not stopped the Christadelphians from
teaching He did. This is not surprising considering they deny the doctrine of the Trinity and the deity
of Jesus. As with all non-Christian cults that deny the true doctrine of God, other doctrines necessarily
become incorrect as well. In this case, their error is that Jesus has a sin nature.

• "Therefore, we conclude that it is not only that Jesus was called a sinner at his trial by his
enemies or that he was 'numbered with the transgressors' when he was crucified between two
thieves, but more particularly that he shared the very nature which had made a sinner out of
ever other man who had borne it. It is for this reason that the nature we bear is called "sinful
flesh" or more briefly, 'sin' (Rom. 7:20 and 8:4)." (The Christadelphians: What They Believe
and Preach, p. 74)
• "And it was for that very reason -- being a member of a sinful race -- that the Lord Jesus
himself needed salvation...But it is equally true that, being 'made sin for us' (2 Cor. 5:21), he
himself required a sin offering; in other words, he sacrificed himself, for himself, that he might
save us. Or, in other words, he saved himself in order to save us...That Christ needed salvation
is seen from Psalm xci.16." (Christadelphian Answers, p. 24)

One of the main verses they use to support their erroneous doctrine is Rom. 8:3-4 which says, "For
what the Law could not do, weak as it was through the flesh, God did: sending His own Son in the
likeness of sinful flesh and as an offering for sin, He condemned sin in the flesh." They teach that the
"likeness of sinful flesh" means that Jesus had a sinful nature. But it doesn't. The key to
understanding this verse is the word "likeness." If this word were omitted then the text would say
"...sending His own Son in sinful flesh..." If that is what the verse said then the Christadelphians
would have a valid argument. But the text says that Jesus came in the "likeness" of sinful flesh, not
that He came in sinful flesh. In other words, men are sinners. Jesus appeared as a man. Therefore,
Jesus appeared in the likeness of a sinner, though He was not a sinner.
Another verse they use is Heb. 2:14 which says, "Since then the children share in flesh and blood,
He Himself likewise also partook of the same, that through death He might render powerless him who
had the power of death, that is, the devil." This verse can easily be explained in the same manner as
Rom. 8:3-4 above. Jesus partook of flesh and blood. But it does not here say that He had a sin
nature.
To have a sin nature means that Jesus had a fallen, defiled, and unholy nature. I fail to see how an
unholy person can offer a holy sacrifice sufficient to please an infinitely holy God. Of course, the
Christadelphians say this is possible because, even though Jesus had a sin nature, He never committed
a sin and He kept the Law therefore satisfying God. But that still doesn't answer the objection: If
Jesus had a sinful and unholy nature, how is it possible for Him to provide a sinless and holy sacrifice
especially since Eph. 2:3 states that we are by nature children of wrath? This means that the natural
state of the fallen is judgment.
The problem with the Christadelphian position is that the Bible teaches us the sacrifice to God must
be without blemish. Deut. 17:1 says, "You shall not sacrifice to the Lord your God an ox or a sheep
which has a blemish or any defect, for that is a detestable thing to the Lord your God." (See also
Ezekiel 43:22-23, 25; 45:18, 23 for the same theme.). Of course, Jesus is not an animal, but it is clear
that the pattern for the sacrifice was that it have no defect at all. Why? Because God is holy and God
doesn't accept imperfect sacrifices! To have a sinful nature is definitely to have a defect. Contrary to
Christadelphian teaching, we can see from the Bible that Jesus has no defect, no blemish: "How much
more will the blood of Christ, who through the eternal Spirit offered Himself without blemish to God,
cleanse your conscience from dead works to serve the living God?" (Heb. 9:14). This verse states that
Jesus is without blemish. How can He be without blemish if He has a fallen and sinful nature?
For the Christadelphians to maintain that Jesus had a sinful nature is the same as saying that the
offering He made had a defect. We can see that this is a problem because the High Priests of the Old
Testament were fallen and had, themselves to be cleansed in order to offer the sacrifice to God. It
wasn't simply that they were sinners. They were fallen by nature and were unholy.
Because the Christadelphians teach that Jesus had a fallen and sinful nature, there faith is in a
defiled and imperfect sacrifice. It is, therefore, is insufficient. They are lost.

What does it mean to have a sin nature?

When we speak of the nature of something, we speak of its essence, character, and quality. The
essence of God, for example, is holiness, purity, sinlessness, etc. The essence of people, on the other
hand, is sinful. In Mark 7:21-23, Jesus discloses to us the very nature of our hearts when He said,
"“For from within, out of the heart of men, proceed the evil thoughts, fornications, thefts, murders,
adulteries, 22deeds of coveting and wickedness, as well as deceit, sensuality, envy, slander, pride and
foolishness. 23“All these evil things proceed from within and defile the man,” (NASB). This is why it
says in Eph. 2:3 that we are by nature children of the wrath; our hearts are sinful by nature which is
the source of the sins listed by Jesus. This is also why Paul said in Rom. 7:18 that nothing good dwelt
in him, that is, in his flesh. Paul knew his nature was sinful and because it was He was lost and
without hope (except for his faith in Jesus and His unblemished sacrifice).
Are we to conclude from Christadelphian thinking that Jesus' fallen, unholy, and sinful nature
produced a pure and perfect sacrifice without defect? How is that possible? How is it possible for
someone unholy to offer a holy sacrifice? How is it possible for someone that is sinful by nature, to
offer a sinless sacrifice? Just because Jesus never sinned doesn't mean that He was perfect. If He had
a sin nature, He was not perfect. He was flawed. His sacrifice would be useless.
However, to the Christadelphians, the issue is not so much Jesus' sinful and fallen nature, as it is
His ability to keep the Law. Therefore, in Christadelphianism we have a man, Jesus, with a sinful
nature being able to perfectly keep all of God's law. Contrast this with Adam who was made sinless
and yet to was not able to keep the law of God. How can Jesus have a sinful and unholy nature and
yet be sinless and holy as a perfect, unblemished sacrifice? He cannot. The Christadelphians are
wrong.

Jesus was tempted

One of the reasons the Christadelphians believe Jesus had a sinful nature is their claim that in
order for Jesus to be tempted, He had to have a sin nature. But, this does not logically follow. Adam
did not have a sinful nature and he was tempted successfully. He fell. Jesus did not have a sinful
nature. He was tempted unsuccessfully. He did not fall. So, Jesus not having a sin nature does not
mean He cannot be tempted.
Of course, the Christadelphians deny that Jesus is both God and man, even though this is what Col.
2:9 says: "For in Him all the fullness of Deity dwells in bodily form." 116 In their quest to support their
view, they sometimes quote James 1:13 which states that God cannot be tempted by evil. They ask, if
Jesus is God, then how could He be tempted with evil? This is a fair question and, to be honest, a bit
difficult to answer because the Scriptures do not explicitly explain it. Therefore, we have to work from
what we do know using reason.
If Jesus' human nature existed by itself, apart from the divine nature, it would have been a normal
human nature and capable of sin. But, Jesus' human nature is not separate from His divine nature
which is morally pure and incapable of sin. It would then seem that Jesus was able to be tempted in
His human nature but not in His divine. In the one person of Christ, there dwells two natures: God
and man (Col. 2:9). As God, Jesus could stand without the danger of sinning. As man, He could be
tempted. Exactly how these two natures relate to each other in one person is not clarified in
scripture. But, as you can see, it is possible that Jesus be divine and be tempted at the same time
because He was both God and man. To say that Jesus had to have a sin nature in order to be tempted
is incorrect. Rather, in order to be tempted, Jesus had to be human.

Christadelphians compare Col. 2:9 with Eph. 3:19 which has similar phraseology in it in an effort to deny that
116

Col. 2:9 says Jesus is deity. For a more detailed look at this, see the paper "Col. 2:9 and Eph. 3:19."
Jesus was under the Law

Another Christadelphian argument that Jesus had a sin nature is that since Jesus was under the
Law, and that a person is only under the Law if he is capable of sin, therefore Jesus had to have a sin
nature. As I've already demonstrated above, Adam did not have a sin nature and he was tempted.
But more importantly here, Adam was under the law of God even though he had a sinless nature --
though he was capable of sinning. God gave a Law to Adam when He said, "...From any tree of the
garden you may eat freely; 17but from the tree of the knowledge of good and evil you shall not eat, for
in the day that you eat from it you shall surely die," (Gen. 2:16-17). The phrase "you shall not"
should remind us of the Ten Commandments with the "you shall" and "you shall not's." Adam was
under Law and because He broke that Law, he sinned. Rom. 3:20 says, "...through the Law comes
the knowledge of sin." And, "sin is not imputed when there is no law," (Rom. 5:13).
The reason Jesus was under the Law was so that He could become a sacrifice for us and redeem
those who are under the Law (Gal. 4:4). He had to be made like His bretheren in order to satisfy the
Law requirements of being a sacrifice. He had to be a man to atone for men. He had to be God in
order to offer a sufficiently valuable atoning work.

Sin entered the world through Adam

There is debate in the theological circles concerning whether or not the sinful nature is passed
down through the father or not. The scripture is not specific about this issue, so I present this
argument as food for thought because it could shed some light on whether or not Jesus had a fallen
nature.
Even though Eve was the first person to sin, sin entered the world through Adam and not through
Eve. Rom. 5:12 says, "Therefore, just as through one man sin entered into the world..." The theory is
that Adam was the representative of mankind in the garden. When he fell, we fell because we were
"in" him. This concept of representation one person representing others is found in Heb. 7:9-10.

10
And, so to speak, through Abraham even Levi, who received tithes, paid tithes, for he was still in the
loins of his father when Melchizedek met him.

Levi was a distant descendant of Abraham. Abraham was long dead when Levi was born. But the
text says that Levi paid tithes to Melchizedek. How is this possible? It seems the answer lies in the
idea that one person represented his descendants. This would explain why Levi is said to have paid
tithes to Melchizedek because his distant father Abraham did so and because Levi was "in" the loins of
his distant father Abraham. Likewise, sin entered the world through Adam and not Eve because Adam
was the representative head of mankind. If this is so, then Jesus would not have received a sin nature
from His father Joseph since Joseph had no biological paternity in relation to Jesus. Therefore, his sin
nature would not have been passed down to Jesus. But since he had a human mother, He had human
nature. We can see He was both God and man because He is called both the son of God and the son of
man. If it is true, then we can see that Jesus had a divine nature received from God and a human
nature, but not a sinful one, from His mother Mary.
Whether or not the preceding concept is legitimate is still up for debate. But I offer it has yet
another possible reason why Jesus did not have a sinful nature.
Jesus is God in flesh

The primary biblical reason that in Jesus does not have a sinful nature is because Jesus is both God
and man in one person. Of course the Christadelphians do not accepted this since they deny the
Trinity. Nevertheless, their denial of the deity of Christ does not negate its truth. The Bible says that
Jesus is God in flesh.

• “In the beginning was the Word, and the Word was with God, and the Word was
God. 2 He was in the beginning with God.... 14And the Word became flesh, and dwelt
among us, and we beheld His glory, glory as of the only begotten from the Father, full of
grace and truth," (John 1:1-2, 14).
• "but at the proper time manifested, even His word, in the proclamation with
which I was entrusted according to the commandment of God our Savior," (Titus 1:3).
• “For in Him all the fulness of Deity dwells in bodily form," (Col. 2:9).
• “But of the Son He says, “Thy throne, O God, is forever and ever...," (Heb. 1:8).
• "looking for the blessed hope and the appearing of the glory of our great God and
Savior, Christ Jesus," (Titus 2:13).
Was Jesus' sacrifice blemished according to Christadelphian theology?

According to Christadelphian theology, Jesus had a sinful and fallen nature. 117 He therefore needed
to be redeemed just like anyone else.118 But, say the Christadelphians, Jesus never sinned, so His
sacrifice on the cross was pure and acceptable to God.
The Old Testament sacrifices were to be without blemish or defect. Of course, the blood of animals
cannot remove our sin, but these sacrifices were representative of the sacrifice of Christ. Deut. 17:1
says, "You shall not sacrifice to the Lord your God an ox or a sheep which has a blemish or any defect,
for that is a detestable thing to the Lord your God" (NASB). (See also Ezekiel 43:22-23, 25; 45:18, 23
for the same theme.).
The words "blemish" and "defect" are "mowm" and "ra", respectively. According to the Strong's
Concordance "Mowm" means "1) blemish, spot, defect 1a) of physical defect 1b) of moral stain."
Obviously, the sacrifice can have no physical defect.
The second word, "ra" means, "1) bad, evil 1a) bad, disagreeable, malignant...2) evil, distress,
misery, injury, calamity...3) evil, misery, distress, injury 3a) evil, misery, distress," etc. It is obvious
that the word here deals with the inner quality or character. Therefore, the two words emphasis the
outer and inner aspects of the sacrifice. It is to be physically pure and morally pure.
The Christadelphian will rightly respond by stating that Jesus was morally pure, and He was. He
never sinned (1 Peter 1:22). But, not having committed a sin is not the same as being pure in
nature. A baby has a sin nature, yet has committed no sin and as such is still in need of redemption
not because it has committed a sin, but because it has a fallen nature. This is why the Bible says we
are by nature children of wrath (Eph. 2:3). God clearly states that our natures are fallen and affect
our position before Him. In other words, having a sinful is indeed a defect within a person.
Nevertheless, the Christadelphians maintain that Jesus sinful nature played no part in the sacrifice.
In other words, they are, to a large degree, separating the nature of Christ and His obedience to the
Father.

1. To say that Christ had a sinful nature makes Him a blemished and imperfect sacrifice,
because
A. Jesus is the sacrifice and the sacrifice cannot be separated from the thing/person being
offered.
i. 1 Peter 1:24 says, "and He Himself bore our sins in His body on the cross, that we might
die to sin and live to righteousness; for by His wounds you were healed."
a. This verse proves that Jesus bore our sins in his body on the cross. Jesus had a
human body because He has a human nature and it was in His body that our sins
were placed.
b. If the Christadelphian states that this verse only means that Jesus bore a sinful
tendency as all humans do, then he would be contradicting the teaching of the text
which states he bore our sins in His body.
c. Furthermore, the idea of 1 Peter 1:24 is repeated in Heb. 9:28 which says, "so
Christ also, having been offered once to bear the sins of many, shall appear a
second time for salvation without reference to sin, to those who eagerly await Him."
d. The verb "bear" in both verses is "anaphero" and it means
i. "to bring or take up." (Arndt, W. (1996, c1979). A Greek-English lexicon of the
New Testament and other early Christian (Page 63). Chicago: University of
Chicago Press.)
ii. "to carry or bring up, to lead up. 1a men to a higher place. 2 to put upon the
altar, to bring to the altar, to offer. 3 to lift up one’s self, to take upon one’s
self. 3a to place on one’s self anything as a load to be carried. 3b to sustain, i.e.
their punishment." (Strong, J. (1996). Enhanced Strong's Lexicon. Ontario:
Woodside Bible Fellowship.)

117
The Christadelphians: What They Believe and Preach, p. 82.
118
Christadelphian Answers, p. 24
iii. "to be carried or borne upward—‘to carry up.’ (Louw, J. P. (1996, c1989).
Greek-English lexicon of the New Testament: Based on semantic domains (LN
15.206). New York: United Bible societies.)
e. Therefore, we can see that Jesus actually did bear our sins in His body as 1 Peter
1:24 says.
B. Heb. 7:27 says, "who does not need daily, like those high priests, to offer up sacrifices, first
for His own sins, and then for the sins of the people, because this He did once for all when
He offered up Himself."
Eph. 5:2 says "and walk in love, just as Christ also loved you, and gave Himself up for
us, an offering and a sacrifice to God as a fragrant aroma,"
i. Obviously, Jesus, the man with His human nature was offered upon the cross. Himself
is Himself. How does offering "Himself" not include His nature as a man?
C. Heb. 9:14 says, "how much more will the blood of Christ, who through the eternal Spirit
offered Himself without blemish to God, cleanse your conscience from dead works to serve
the living God?"
i. Here says that the "blood of Christ" (9:14) is what cleanses us of our sins. This proves
that the physical nature of Christ was involved as being the sacrifice.
D. Heb. 10:10-12 says, "By this will we have been sanctified through the offering of the body of
Jesus Christ once for all, 11And every priest stands daily ministering and offering time after
time the same sacrifices, which can never take away sins; 12but He, having offered one
sacrifice for sins for all time, sat down at the right hand of God."
i. It says that Jesus' body was the offering to God. Notice that the physical body of Christ
was the sacrifice. According to the Christadelphians, Jesus became a man and shared
in the sin nature. How is the sacrifice of Christ which was the offering to God which
included the physical shedding of blood, not include Jesus' human nature which,
according to the Christadelphians, was sinful?
2. If you say that Jesus' sacrifice had nothing to do with His sin nature, then...
A. Please provide biblical support for this idea of a separation between a person's nature and
his actions. If this cannot be done, then the idea cannot be claimed.
i. The Bible no where makes a distinction between a person's deeds and his nature. On
the contrary, it ties them together:
a. "Then the Lord saw that the wickedness of man was great on the earth, and that
every intent of the thoughts of his heart was only evil continually," (Gen. 6:5).
b. "Out of the wicked comes forth wickedness," (1 Sam. 24:13).
c. "The heart is more deceitful than all else and is desperately sick; Who can
understand it?" (Jer. 17:9).
d. "Out of the abundance of the heart, the mouth speaks," (Matt. 12:34).
e. "For from within, out of the heart of men, proceed the evil thoughts, fornications,
thefts, murders, adulteries, 22deeds of coveting and wickedness, as well as deceit,
sensuality, envy, slander, pride and foolishness. 23"All these evil things proceed from
within and defile the man," (Mark 7:21-23).
f. Jesus' sacrifice was related to His nature and if He had a sinful nature, then His
sacrifice was blemished.
B. You are separating what Christ is from what Christ did.
i. If you separate what Christ is from what Christ did, then anyone could have atoned for
sins provided he keep the law perfectly. All they would need would be a little help from
God with a special anointing.
C. You are saying that a concept was the offering and not Him. This is how:
i. Christadelphians claim that Jesus' obedience was offered as a sacrifice. Obedience is an
action, a concept.
ii. If obedience is NOT tied to Jesus nature, then why did Jesus need to die?
a. If it is true that only the obedience of Christ was the offering, then it is not
necessary for Jesus to have died since His nature, says the Christadelphian, had
nothing to do with sacrifice of obedience.
iii. Biblically, live things were offered in the sacrifice. It was never a concept offered up as
a sacrifice.
iv. If Jesus were the sacrifice, then Jesus, the man Himself was the sacrifice, was offered on
the cross. But, if this is so, then we have a sinful-natured man as the offering and this
is a blemished sacrifice.
3. Questions for Christadelphians regarding Jesus and His sacrifice
A. What biblical precedence and support is there for the claim that Jesus' sacrifice had nothing
to do with His "sinful" nature, as Christadelphian theology teaches?
B. Heb. 7:27 says, "who does not need daily, like those high priests, to offer up sacrifices, first
for His own sins, and then for the sins of the people, because this He did once for all when
He offered up Himself." Obviously, Jesus, the man with His human nature was offered upon
the cross as this text says. Can you please explain how this fits into your theology that
Jesus had a sin nature and that it had nothing to do with the sacrifice when the Bible says
that HE HIMSELF was the offering? In other words, since Jesus had a sin nature and he
offered HIMSELF (Heb. 7:27), how is it that his sin nature was not part of the offering?
C. In reference to Heb. 7:27 above, is "Himself" NOT including His nature? How do you
separate Jesus being Himself from His "sin nature" and still be Himself?
D. In reference to Heb. 10:10 where Jesus offered up His body as the sacrifice, how do you
exclude His physical body as being the sacrifice?
E. In reference to Heb. 10:10, since Jesus was human in nature and, as the Christadelphians
claim, had a sin nature just like all people, how do you say that his human nature was not
the sacrifice since it says his body was offered? This would mean that the Christadelphian
Jesus offered up Himself, in body, which would include his sin nature which would make the
sacrifice blemished and void.
F. What biblical precedence and support is there for the claim that Jesus' sacrifice was only an
offering of His deeds and not His person?
G. How is a person with a sin nature able to keep the Law perfectly?
H. If Jesus' sacrifice was of His obedience and had nothing to do with His sinful nature
(according to Christadelphian theology), then what does it mean when it says that Jesus
bore OUR sins in HIS body (1 Peter 1:24), that He became sin on our behalf (2 Cor. 5:21),
and that He was an offering to bear the sins of many (Heb. 9:28)?
Can the Christadelphian Jesus with a fallen nature save anyone?

According to the Christadelphians, Jesus was a fallen and unholy created thing that needed to save
Himself. But how is it that such a fallen creature can be the savior of sinners? Does it make any
sense? No. The Christadelphians are wrong.
Following is an outline that should help expose the error of the Christadelphians. Note that the
references are from Christadelphian authors.

1. Jesus had a fallen and unholy nature according to the Christadelphians.


A. God has a sinless and holy nature.
i. The Christadelphians: What They Believe and Preach, p. 74
B. Therefore, according to the Christadelphians, Jesus was not divine.
i. Christadelphian Answers, p. 22
2. Jesus never sinned according to the Christadelphians.
A. But, how is it possible for a fallen and unholy person to not sin?
i. If the Christadelphians say that Jesus, with a fallen and unholy nature, was able to keep
the Law of his own power and will, then on what basis do they justify that claim,
especially since Adam, who had a sinless nature, failed to do it?
ii. If the Christadelphians say Jesus never sinned because "God was working in Him," then
it isn't really Jesus who kept the law, was it? It would have been God working in Christ
and not Christ himself. This would mean that the person of Christ was not keeping the
Law of his own accord.
a. If it wasn't Jesus who was keeping the Law, then he had nothing righteous to offer
as a sacrifice.
b. That would mean that the sacrifice had no value and there is no redemption for the
Christadelphians.
iii. If the Christadelphians say it was the power of God working in Jesus to keep the Law
and that Jesus really did keep the Law of his own accord, then on what basis do they
claim that God doing the work in Christ is really Christ's work?
3. Jesus needed to save himself according to the Christadelphians.
Christadelphian Answers, p. 24
A. If Jesus needed to save himself, then why?
i. The Christadelphians teach that Jesus was sinless. If he was sinless, he needed no
redemption because sin causes a separate between God and man (Isaiah 59:2) and it is
sin that brings judgment (Rom. 1:18; James 2:9).
a. In addition, our very nature also brings judgment. Eph. 2:3 says we are by nature
children of wrath. This means that we are fallen and unholy by nature.
b. If Jesus was fallen and unholy by nature, then how can his sacrifice be of sufficient
value since it is by nature fallen and unholy?
i. If the Christadelphians say the sacrifice has value because Jesus never sinned,
then see #2 above.
B. If Jesus needed to save himself then from what did he need salvation?
i. If the Christadelphians say the judgment of God, then it can only be that the judgment
of God would reside upon Christ because Christ has a fallen and sinful nature -- since
the Christadelphians maintain that Jesus never sinned.
a. This would mean that the sacrifice of Christ was, by nature, also unholy and fallen
because Jesus was, by nature, unholy and fallen. The value of the sacrifice is
derived from the nature of the sacrifice.
b. If the Christadelphians say the sacrifice was not fallen and unholy because Jesus
never sinned, then see #2 above.
4. The Bible says the sacrifice must be without blemish
A. Deut. 17:1 says, "You shall not sacrifice to the Lord your God an ox or a sheep which has a
blemish or any defect, for that is a detestable thing to the Lord your God." (See also Ezekiel
43:22-23, 25; 45:18, 23 for the same theme.).
B. Having a sin nature is a blemish.
C. Jesus offered Himself on the cross. "Himself" includes His nature since that is part of
Himself. This is in contradiction to the Christadelphians who teach that Jesus only offered up
His "works" to God the Father.
i. "[Jesus] who does not need daily, like those high priests, to offer up sacrifices, first for
His own sins, and then for the sins of the people, because this He did once for all when
He offered up Himself," (Heb. 7:27).
ii. "How much more will the blood of Christ, who through the eternal Spirit offered
Himself without blemish to God, cleanse your conscience from dead works to serve the
living God?" (Heb. 9:14). (emphasis added)
iii. "just as the Son of Man did not come to be served, but to serve, and to give His life a
ransom for many," (Matt. 20:28).
iv. "looking for the blessed hope and the appearing of the glory of our great God and
Savior, Christ Jesus; 14who gave Himself for us, that He might redeem us from every
lawless deed and purify for Himself a people for His own possession, zealous for good
deeds," (Titus 2:13).
v. See also Eph. 5:2 and 1 Peter 2:24.
D. If Jesus offered up Himself, which had a fallen nature, then the sacrifice was unholy and the
Christadelphian Jesus cannot save anyone. Therefore, the Christadelphians are lost.
i. For the Christadelphians to hold their position that Jesus' nature was not offered on the
cross, then they will need to establish how when the Bible says that Jesus offered up
"Himself," that "Himself" somehow excluded Jesus' own nature.
5. The truth
A. Jesus, the man, was also divine in nature and, therefore, holy and sinless. Because he was
both God and man (John 1:1,14; 20:28; Col. 2:9; Phil. 2:5-8; Heb. 1:8), His sacrifice was,
by nature, pure and holy. Because He was also a man who kept all the Law and because He
was pure and holy by nature, the righteous sacrifice He offered was truly His to offer.
Because it was His to offer and it was pure and holy, it is sufficient to cleanse us from all our
sins.
B. The Sacrifice of Christ was substitutionary.
i. 1 Pet. 2:24, "and He Himself bore our sins in His body on the cross, that we might die to
sin and live to righteousness; for by His wounds you were healed."
ii. 2 Cor. 5:21, "He made Him who knew no sin to be sin on our behalf, that we might
become the righteousness of God in Him."
C. Jesus was never a fallen and unholy creature who Himself needed salvation. He is the
Savior, God in flesh.
i. Exodus 3:14, "And God said to Moses, "I AM WHO I AM"; and He said, "Thus you shall
say to the sons of Israel, ‘I AM has sent me to you.’"
ii. John 8:58, "Jesus said to them, 'Truly, truly, I say to you, before Abraham was born, I
am.'"
iii. John 8:24, "I said therefore to you, that you shall die in your sins; for unless you
believe that I am, you shall die in your sins."
The Christadelphian view of the Holy Spirit
The Christadelphians deny the doctrine of the Trinity. Therefore, they also deny that the Holy
Spirit is the third person in the Godhead. When we say that the Holy Spirit is a person, we do not
mean that He has a body of flesh and bones, that His first name is Holy and His last name is Spirit.
Rather, the Holy Spirit has personhood in that He is self aware, has a will, and can speak. However,
the Christadelphians teach that the Holy Spirit is a force, the invisible power and energy of the Father
by which God is everywhere present. It is a "power concentrated through an individual or angel for
the purpose of a specific miraculous event or activity." 119
They teach that the Holy Spirit is an impersonal force by which God inspired the Bible and moved
through people the same way as a wind would drive a ship, carrying it along with it. In support of their
position, they quote 2 Pet. 1:21 which says, "for no prophecy was ever made by an act of human will,
but men moved by the Holy Spirit spoke from God." They interpret the moving of the Holy Spirit as the
moving of the force. They then sometimes go to Acts 27:17 that also contains the word "moved"
when it is referring to the ship being "driven along" by the storm.
The problem with this type of thinking is that words mean what they mean in their immediate
context. You do not take the meaning of a word in one scripture, take it out of its context, and apply it
to another verse. This leads to error and is particularly dangerous when other scriptures are ignored
that contradict the interpretation that the Christadelphians are trying to support.

Who is the Holy Spirit?

The Holy Spirit is the third person in the Trinity. He is fully God. He is eternal, omniscient,
omnipresent, has a will, and can speak. He is alive. He is a person. He is not particularly visible in the
Bible because His ministry is to bear witness of Jesus (John 15:26). Please consider the following
verses as support for the personhood of the Holy Spirit.

1. The Holy Spirit is lied to


A. "But Peter said, "Ananias, why has Satan filled your heart to lie to the Holy Spirit, and to
keep back some of the price of the land?" (Acts 5:3).
2. The Holy Spirit has a will and is called "He."
A. "But one and the same Spirit works all these things, distributing to each one individually just
as He wills," (1 Cor. 12:11).
3. The Holy Spirit loves
A. "Now I urge you, brethren, by our Lord Jesus Christ and by the love of the Spirit, to strive
together with me in your prayers to God for me," (Rom. 15:30).
4. The Holy Spirit speaks
A. "And while Peter was reflecting on the vision, the Spirit said to him, "Behold, three men are
looking for you. 20"But arise, go downstairs, and accompany them without misgivings; for I
have sent them Myself," (Acts 10:19-20).
B. "And while they were ministering to the Lord and fasting, the Holy Spirit said, "Set apart for
Me Barnabas and Saul for the work to which I have called them," (Acts 13:2).
5. The Holy Spirit can be insulted
A. "How much severer punishment do you think he will deserve who has trampled under foot
the Son of God, and has regarded as unclean the blood of the covenant by which he was
sanctified, and has insulted the Spirit of grace?" (Heb. 10:29).

If the Holy Spirit is like a force, then how can it be lied to? How can a force have a will, or be able
to love, or speak, or be insulted? Clearly the Bible demonstrates that the Holy Spirit is alive and has
attributes of life, self awareness, and a will and is not a mere force.
The Christadelphians go to great lengths to deny the personhood of the Holy Spirit. But, it should

119
The Testimony: The Distinctive Beliefs of the Christadelphians, Vol. 58, No. 691, July 1988, page 254.
be obvious that the Holy Spirit is not a mere force like radar or a wind. Rather, He is the third person
in the Trinity.

Questions for Christadelphians

1. According to Christadelphian theology, Jesus had a sinful, fallen nature.


A. Deut. 17:1 says, "You shall not sacrifice to the Lord your God an ox or a sheep which has a
blemish or any defect, for that is a detestable thing to the Lord your God," (NASB, See also
Ezekiel 43:22-23, 25; 45:18, 23). Of course, Jesus is not an animal. The point is that the
sacrifice to a holy God must have no blemish or defect.
i. "defect" in Hebrew is ra. In this verse, it is translated as "evilfavourdness" in the KJV,
as "defect" in the RSV and NKJV, and as "flaw" in the NIV.
B. Question: If Jesus had a sinful, fallen nature, then isn't that a defect?
C. Question: If Jesus' sinful nature is not a defect, then what would you call it?
D. Question: If Jesus sinful nature is a defect, then doesn't that mean His sacrifice is
insufficient?
E. Question: If you state that being obedient is what makes a person "unblemished," then why
are we damned by nature (Eph. 2:3) if it is only our sinful deeds that condemn us?
2. According to Christadelphian theology, Jesus had to die in order to save himself. Yet
the Christadelphians also maintain that Jesus was without blemish or defect.
A. Question: If this is so, why would Jesus need to save Himself if He had no sin?
B. Question: If Jesus needed to save Himself, then that means He was not without defect. If
that is the case, then how can he be a pure and unblemished sacrifice?
3. Thomas said to Jesus, "My Lord and my God," (John 20:28). He was not sinning by
using God's name in vain.
A. Question: Can you, like Thomas, say to Jesus, "My Lord and my God."?
B. Question: If you do call Jesus your Lord and your God, since you believe Jesus is a creation,
isn't that idolatry?
C. Question: If you do call Jesus your Lord and your God, is Jesus the true God or not?
D. Question: If you do not call Jesus you Lord and your God, why not? It is biblical.
4. Jude 4 says, "For certain persons have crept in unnoticed, those who were long
beforehand marked out for this condemnation, ungodly persons who turn the grace of
our God into licentiousness and deny our only Master and Lord, Jesus Christ."
A. Question: Can you call Jesus your only Master and Lord?
B. Question: If you do call Jesus your only Master and Lord, then what about God the Father?
Is He not also your Lord and Master?
C. Question: If you call Jesus your "only" Lord and Master, aren't you committing idolatry?
D. Question: If you do not call Jesus your only Lord and Master, then aren't you disobeying the
truth of God's word?
5. John 1:12 says, "But as many as received Him, to them He gave the right to become
children of God, even to those who believe in His name,"
A. Question: Have you received Jesus?
6. In Matt. 11:28 Jesus says, "Come to Me, all who are weary and heavy-laden, and I will
give you rest." The rest He is referring to is rest from the law, from trying to please
God by your deeds.
A. Question: Have you gone to Jesus and rested are or you still trying to please God enough to
be saved?
B. Question: If you have gone to Jesus, how did you do this? In prayer to Jesus?
7. When we sin, we sin against God because it is His law we are breaking. He is the one
who must forgive us because we have offended Him. The one offended is the one who
forgives. Someone or something else doesn't forgive us for our sins against God, only
God can do that.
A. Question: How is it that Jesus is the one who forgives sins (Luke 5:20) if Jesus is not God,
the one who is offended?
B. Question: If you state that it is because Jesus was given authority by God to forgive sins
(Matt. 28:19), then have you gone to Jesus and asked Him to forgive you of your sins?
Remember, to do that, you must pray to Jesus. Is it right to pray to a creature?

Has God performed the greatest act of love?

This is an unusual question. Has God performed the greatest act of love? It is a question designed
to provoke deeper thought on the nature of God and what He has done in our lives. God is love (1
John 4:8). So, I ask if God been as loving as is possible? The answer would depend on whether or not
you are a Christian.
Jesus tells us what the greatest act of love is. He said in John 15:13, “Greater love has no one than
this, that one lay down his life for his friends."
To Christadelphians, Jesus is not God. He is a creation with a fallen and sinful nature. Yet it is Jesus
who sacrificed Himself for others. According to Jesus' own words He Himself has performed the
greatest act of love. His sacrifice is even more monumental and loving when we consider that He bore
hour sins in his body on cross (1 Pet. 2:24). Undoubtedly, Jesus performed the greatest act of love.
However, if you are the Christadelphian, then Jesus is not God. This would mean that Jesus, being
a created thing, has performed a greater act of love than God Himself. It would mean that in
Christadelphianism, Jesus who is a creature, has actually out performed God Himself. That would
mean that God is not one who neither has nor can perform the greatest act of love.
On the other hand, in Trinitarianism, God is the one who has performed the greatest act of love.
This is because in the doctrine of the Trinity we have Jesus, God in flesh, second person of the Trinity,
who laid His life down for His friends. In this, God has indeed performed the greatest act of love, not a
creature. And since God is love, is it not fair to expect that the infinite God who is all powerful can
fulfill the very words He inspired as truth?
When someone sins against you and you intend to forgive him, is it right to appoint someone else
to be the one who forgives that person or must you do it yourself? Is it more loving for you to appoint
another who is uninvolved in the offense against you to make satisfaction? Or, must you make
satisfaction yourself? Is it a greater love to have a creature die for offenses suffered against God, or
for God to bear the act and work of forgiveness Himself? The answer should be obvious.
In Christadelphian theology, God is less a lover of our souls than the God of Trinitarianism.
So, please Christadelphian, tell me why I should give up my Savior God ( Titus 1:3) who loved me
enough to sacrifice Himself for me, for a system of belief for a god who cannot and has not performed
the greatest act of love?
You want me to give up God as my Savior, for a creature. I want you to give up a creature for God.
Only God saves us from sins. No creature can do that. You must receive Jesus (John 1:12). You
must fellowship with Jesus (1 John 1:9). You must put your faith in the Jesus who is prayed to (Acts
7:55-60; 1 Cor. 1:2 ); called God (Heb. 1:8; John 20:28); who is the First and Last (Rev. 1:17); who
cleanses from sins (1 John 1:7); who discloses Himself to us (John 14:21); who draws all people to
Himself (John 12:32); who gives eternal life (John 10:28); who opens the mind to understand
scripture (Luke 24:45); who reveals grace and truth (John 1:17); etc.
Which Savior is true, Jesus the creator, or Jesus the creature? Which can save you from your sins,
Creator, or creature? Please come to Jesus, God in flesh, Savior.

Who are you going to put your trust in?


Christian Science
Introduction

Christian Science is an interesting religion started by Mary Baker Eddy. It teaches a non-personal
God, that sin is an illusion, that there is no devil, and that evil and good are not real. All is in
interpretation of divine mind.

Christian Science is far from Christian even though it uses the Bible. But, it distorts the biblical truths
by severely changing the meanings of biblical words. Though this cult is not very large in the United
States, it has deceived many thousands.

1. What are some of the aberrant teachings of Christian Science? p. 148


2. Why is Christian Science not Christian? p. 150
3. What are the CS meanings of Christ, devil, heaven, pastor, and salvation? pp. 151-152
4. What are some verses that show that Jesus is the Christ? pp. 153
5. Which questions would you ask a Christian Scientist? p. 156
What does Christian Science Teach?

The following doctrines are referenced out of the primary Christian Science work,
Science and Health with Key to the Scriptures, by Mary Baker Eddy.
It is supposed to be a companion to the Bible.
Science and Health together with the Bible are called the Pastor of Christian Science.

1. God is infinite...and there is no other power or source, S&H, 471:18.


2. God is Universal Principle, S&H 331:18-19
3. God cannot indwell a person, S&H 336:19-20
4. God is the only intelligence in the universe, including man S&H 330:11-12
5. God is Mind, S&H 330:20-21; 469:13
6. God is the Father-Mother, S&H 331:30; 332:4
7. The Trinity is Life, Truth, and Love, S&H 331:26
8. Belief in the traditional doctrine of the Trinity is polytheism, S&H 256:9-11
9. Christ is the spiritual idea of sonship, S&H 331:30-31
10. Jesus was not the Christ, S&H 333:3-15; 334:3
11. "Jesus Christ is not God, as Jesus himself declared..." S&H 361:12-13
12. Jesus did not reflect the fullness of God, S&H 336:20-21
13. Jesus did not die, S&H 45:32-46:3
14. The Holy Spirit is divine science, S&H 331:31
15. There is no devil, S&H 469:13-17
16. There is no sin, S&H 447:24
17. Evil and good are not real, S&H, 330:25-27; 470:9-14
18. Matter, sin, and sickness are not real, but only illusions," S&H 335:7-15;
447:27-28.
19. Life is not material or organic, "S&H, 83:21
20. The sacrifice of Jesus was not sufficient to cleanse from sin, "S&H, 25:6.
21. True healings are the result of true belief, "S&H, 194:6"

• Additionally, Christian Scientists prefer not to use doctors, medicine, or immunizations.


Christian Science Practitioners are used to help people through the false reality of illness.
• Proper prayer and training are employed to battle the "non-reality" of illness.
• They have no ordinances like the Lord's Supper or baptism.
• Church services are interspersed with Bible reading and readings from Science and Health.
• Mary Baker Eddy is highly regarded as a revelator of God's word, almost equal to Jesus.
Christian Science History
Christian Science was founded by a woman named Mary Baker Eddy. She was born Mary Ann
Morse Baker in New Hampshire in 1821. (She died in 1910.). She was the daughter of a New
Hampshire Congregationalist church member. As a child, she was frequently ill and highly emotional.
She is said to have been "domineering, quarrelsome, and extremely self centered.” 120 At age 22, she
married George Glover. He died seven months later. She then married Dr. Daniel Patterson, but that
marriage failed in divorce. In 1862, while suffering from an illness, she visited a man named Phineas
Quimby. He taught a system of healing dealing with the mind. He taught that the mind had the power
to heal the body. He exerted a significant influence on her thinking regarding spiritual matters.
In 1866, she fell and was seriously injured and she was not expected to recover. She apparently
read Matt. 9:2 ("And, behold, they brought to him a man sick of the palsy, lying on a bed: and Jesus
seeing their faith said unto the sick of the palsy; Son, be of good cheer; thy sins be forgiven thee ")
and experienced a miraculous cure. It was this experience that convinced her of the truth of Christian
Science.
She first published "Science and Health with Key to the Scriptures" in 1875, when she was 54. She
claimed it was the final revelation of God to mankind and asserted that her work was inspired of God.
The word "Key" in the title of her book is in reference to her being the woman of Revelation 12; that
she is the key to unlocking the Bible which she called a dark book. She claimed the Bible had many
mistakes and that her writings provided the "Key" spoken of in Rev. 3:7.
She married Asa Eddy in 1877.
In 1879, four years after the first publication of Science and Health, Mary Baker Eddy and some of
her students voted organized the church of Christ (Scientist) in Boston Massachusetts. Of course, like
all cults, it claimed to be the restoration of the original New Testament Church.
In 1881 she opened a metaphysical college and charged $300 for 12 healing lessons.
The Church was reorganized in 1892, and the Church Manual was first issued in 1895 which
provided the structure for church government and missions.
She died in 1910, a millionaire.

120
The Religious Bodies of America, by F. E. Meyer. Concordia Publishing House, Saint Louis, Missouri. 1961. P.
532.
Is Christian Science Christian?
Of all the biblically based cults in America today, Christian Science is one of the most interesting.
Not only does it deny the essential doctrines of Christianity, but it has completely reinterpreted the
Bible. It drastically redefines the Bible’s culture and terminology and rips thousands of scriptures out of
their historical and biblical contexts. The result is a non-Christian mixture of metaphysical and
philosophical thoughts. Christian Science is so foreign to the Bible that, if it didn’t use words like Jesus,
Trinity, Love, Grace, Sin, etc., you’d never suspect it had anything to do with the Bible at all.
Additionally, the book Science and Health with Key to the Scriptures, which is the Christian Scientist’s
mainstay of spiritual knowledge, reads with a rhythm of pseudo logical statements that has the
tendency to dull the senses when read long enough. Is Christian Science Christian? Definitely not.
Science and Health with Key to the Scriptures is the primary interpretive source of the Bible and
source guide of Christian Science. It interprets the Bible in a radically different way. It is so different,
in fact, that it absolutely rejects the substitutionary atonement of Jesus and states that it had no
efficacious value (S&H, 25:6). It denies that Jesus is God, second person of the Trinity (S&H, 361:12-
13). It says that sin is a false interpretation of Divine Mind and is nonexistent ( S&H, 335:7-15). And it
says that the Holy Spirit is Divine Science which is best represented by Christian Science (S & H,
331:31). The list can go on and, unfortunately, it does.
To the Christian Scientist, God (the Father-Mother) is a Principle known as the Divine Mind. It has
no personhood and no personality. A catch phrase used in their literature is that God is "All in All." In
other words, God is all that exists and what we perceive as matter is an interpretation of divine mind.
Since God is love, it means that sin and sickness are only errors of interpreting the Divine Mind and
have no true reality (S & H, 330:25-274; 470:9-14).
To the Christian Scientist, Jesus is a Way-shower. He is someone who epitomized the true principle
of the Christ Consciousness which indwells us all. Therefore, Jesus did not really die on the cross. He
was not God in flesh. He made no atonement in shedding His blood (S&H, 25:6).
Christian Science teaches that man does not have a sinful nature and is a reflection of Divine Mind.
To achieve "salvation," he needs only to find the true reality of understanding, as revealed in Christian
Science teachings. Unfortunately, these teachings are from Mary Baker Eddy a woman who founded
the religion in the 1870's and not from God.
The Christian Scientists consider their philosophy to be consistent with the original teachings of
Jesus. They consider truth a matter of higher understanding and learning. But the reality is that
Christian Science has only produced unbiblical and false doctrines. Eternal destruction is the only thing
that will result from its false teaching. The fires of hell will be a bitter reality for those who have been
taught that they don’t exist.
Terms and Definitions of Christian Science

1. Angels are God’s thoughts passing to man, an inspiration of goodness, purity that counters evil
and material reality.
2. Atonement is not the shedding of Christ’s blood, but "At-one-ment." "Lifting the whole man into
Christ Consciousness." The Biblical account is metaphorical, not real.
3. Baptism means the daily, ongoing purification of thought and deed. Eucharist is spiritual
communion with God, celebrated with silent prayer and Christian living. It is a "submergence in
Spirit."
4. Blasphemy of the Holy Spirit is the belief that God created disharmony in the world.
5. Body is "the form of expression of both spirit and soul" (Metaphysical Bible Dictionary, p. 628). It
is the apparent materialization of the limits of soul as influenced by a person’s conscious
development in Christian Science Principles.
6. Christ is the divine idea man. Jesus was not the Christ but a perfect representation of the Christ
consciousness that is the true and higher self of every person. Christ is the manifestation of all
that is good and true, the realization of divine principle. A Christian Scientist can say, "I am
Christ."
7. Creation is the product of Divine Mind. There is only one reality which emanates and is part of
the Divine Mind. Anything that is not in harmony with the Divine Mind is not a reality, but a lack
of understanding of the principles of divine mind brought about by people.
8. Death - "An illusion, the lie of life in matter."
9. Devil - "Evil, a lie, error." He is not an entity, not a person, has no existence. "A belief in sin,
sickness, and death."
10. Evil spirits are false beliefs
11. Flesh - "An error of physical belief; a supposition that life, substance, and intelligence are in
matter; an illusion."
12. Gods - "A belief that life, substance, and intelligence are both mental and material; a supposition
of sentient physicality.
13. God is Spirit who is a ever-present, all-knowing, all-powerful, and good. God is the Father/Mother
God. Other names for God are Divine Mind, Soul, Principle, Life, Truth, Love. To the Christian
Scientist God is the governing Principle of the universe to which a person must harmonize his
belief system.
14. Healing is accomplished by correct thinking according to Christian Science principles. A change
in belief that affects physical symptoms. (SH p. 194:6)
15. Heaven is not a literal place of eternal bliss, but a harmonious condition of understanding where
a person’s consciousness is in harmony with Divine Mind. "Harmony; the reign of Spirit;
government of divine Principle."
16. Hell is a state of mind which can include the effects of their improper understanding of Divine
Mind and Christian Science Principles. Hell is not a literal place of damnation and eternal torment.
Hell exists when a person’s thoughts are out of harmony with the reality of Divine Mind. "Mortal
belief, error; lust; hatred, sin; sickness; effects of sin."
17. Holy Spirit, the, is Divine Science. is the spirit of God and is only discernable and knowable by a
person through his spiritual awareness. It is an emanation, a presence, "a law of God in action."
18. Jesus’ stripes is simply his rejection of error, not the beating he received in the flesh (S&H,
20:15).
19. Knowledge - "Evidence obtained from the five corporeal senses; mortality; beliefs and opinions.
The opposite of spiritual Truth and understanding." (S. H. 590).
20. Material reality is really non-existent. It is only an interpretation of Divine Mind. Even though
someone might feel pain or sickness, in reality it does not exist.
21. Mortal Mind - "Nothing claiming to be something, for Mind is immortal; error creating other
errors."
22. Pastor really means the combined books of the Bible and Science and Health with Key to the
Scriptures.

23. Personhood is an aspect and reflection of Divine Mind.


24. Prayer is contemplation and internalization of divine truths. "The taking hold of God’s
willingness." It is an affirmation of God’s being in relation to man.
25. Resurrection is "Spiritualization of thought; a new and higher idea of immortality, or spiritual
existence; material belief yielding to spiritual understanding."
26. Salvation is "Life, Truth, and Love understood and demonstrated as supreme over all; sin,
sickness, and death destroyed."
27. Sickness is the false understanding given the appearance of reality by the unfaithful and
ignorant of Divine Principle and Mind.
28. Sin is not understanding and behaving according to Divine Law of God and the law of our being.
29. Soul is "man’s consciousness -- that which he has apprehended or developed out of Spirit. . .
Soul is both conscious and subconscious" (Metaphysical Bible Dictionary, p. 628).
30. Spirit is another name for God. Divine substance; Mind; divine Principle; all that is good." Christ
31. Wrath is really the working out of the law of God’s being upon a person. It is not God’s judgment
upon a sinner.
Jesus is the Christ

One of the errors of Christian Science is the belief that the Christ is the "divine-idea man." It is a
principle that dwells within each and everyone of us.
Christian Science denies that Jesus was the Christ, (Science and Health with Key to the Scriptures,
333:3-15; 334:3.)
The Bible declares in 1 John 2:22, "Who is the liar but the one who denies that Jesus is the
Christ?" It is apparent from the following verses, that Christian Science is not telling the truth about
Jesus.

1. Jesus is the Christ


A. Matt. 16:16, "And Simon Peter answered and said, "Thou art the Christ, the Son of the
living God."
B. Acts 5:42, "And every day, in the temple and from house to house, they kept right on
teaching and preaching Jesus as the Christ."
C. See also, Matt. 16:20; Luke 9:20; John 20:31; Acts 3:20; 9:22; 18:5; 18:28.
2. Jesus called Himself the Christ
A. John 4:25-26, "The woman *said to Him, "I know that Messiah is coming (He who is
called Christ); when that One comes, He will declare all things to us." 26Jesus *said to
her, "I who speak to you am He."
3. The Christ was born
A. Matt. 1:16, "and to Jacob was born Joseph the husband of Mary, by whom was born
Jesus, who is called Christ." See also Matt. 2:4; Luke 2:11.
4. Christ did works
A. Matt. 11:2, "Now when John in prison heard of the works of Christ, he sent word by his
disciples."
5. Christ is the Son of David.
A. Mark 12:35, "And Jesus answering began to say, as He taught in the temple, "How is it
that the scribes say that the Christ is the son of David?"
6. Christ suffered
A. Luke 24:26, "Was it not necessary for the Christ to suffer these things and to enter into
His glory?" See also Luke 24:46; Acts 26:23.
7. Christ rose from the dead
A. Acts 17:3, "explaining and giving evidence that the Christ had to suffer and rise again
from the dead, and saying, "This Jesus whom I am proclaiming to you is the Christ."
8. Christ was resurrected
A. Acts 2:31, "he looked ahead and spoke of the resurrection of the Christ, that He was
neither abandoned to Hades, nor did His flesh suffer decay."
9. Christ is a Son
A. Heb. 3:6, "but Christ was faithful as a Son over His house whose house we are, if we
hold fast our confidence and the boast of our hope firm until the end."
10. Christ has a body
A. Rom. 7:1, "Therefore, my brethren, you also were made to die to the Law through the
body of Christ, that you might be joined to another, to Him who was raised from the
dead, that we might bear fruit for God."
11. Christ has a body and blood
A. 1 Cor. 10:16, "Is not the cup of blessing which we bless a sharing in the blood of
Christ? Is not the bread which we break a sharing in the body of Christ?"
12. Christ became a curse for us
A. Gal. 3:13, "Christ redeemed us from the curse of the Law, having become a curse for
us—for it is written, "Cursed is everyone who hangs on a tree."
13. Christ died
A. Gal. 2:21, "I do not nullify the grace of God; for if righteousness comes through the
Law, then Christ died needlessly." See also, 1 Cor. 8:12.
14. Christ can be sinned against
A. 1 Cor. 8:12, "And thus, by sinning against the brethren and wounding their conscience
when it is weak, you sin against Christ."
15. Christ bore the sins of many
A. Heb. 9:28, "so Christ also, having been offered once to bear the sins of many, shall
appear a second time for salvation without reference to sin, to those who eagerly await
Him." See also 1 Cor. 15:23-24.
Interesting Quotes from Mary Baker Eddy
from Science and Health With Key to the Scriptures
"One sacrifice, however great, is insufficient to pay the debt of sin. The atonement requires
constant self-immolation on the sinner’s part. That God’s wrath should be vented upon His beloved
Son, is divinely unnatural. Such a theory is man-made," (S&H, p. 23:3-7).
"The material blood of Jesus was no more efficacious to cleanse from sin when it was shed upon
‘the accursed tree,’ than when it was flowing in his veins as he went daily about his Father’s business,"
(S&H, 25:6-8).
"His disciples believed Jesus to be dead while he was hidden in the sepulcher, whereas he was
alive . . ." (S&H, p. 44:28-29).
". . . his body was not changed until he himself ascended, — or, in other words, rose even higher in
the understand of Spirit, God . . .and this exaltation explained his ascension, and revealed
unmistakably a probationary and progressive state beyond the grave" (S&H, p. 46:15-17; 20-24).
"His students then received the Holy Ghost. By this is meant, that by all they had witnessed and
suffered, they were roused to an enlarged understanding of divine Science" (S&H, p. 46:30-32).
"A scientific mental method is more sanitary than the use of drugs, and such a mental method
produces permanent health," (S&H, 79:7-9).
"It is contrary to Christian Science to suppose that life is either material or organically spiritual"
(S&H, 83:21-22).
"The admission to one’s self that man is God’s own likeness sets man free to master the infinite
idea" (S&H 90:24-25).
"The theory of three person in one God (that is, a personal Trinity or Tri-unity) suggest
polytheism . . ." (S&H, p. 256:9-11).
"Father-Mother is the name for Deity, which indicates His tender relationship to His spiritual
creation," (S&H, p. 332:4-5.).
"The word Christ is not properly a synonym for Jesus, thought it is commonly so used" (S&H, p.
333:3-4).
"Mind is the I AM, or infinity. Mind never enters the finite. . .but infinite Mind can never be in
man . . .a portion of God could not enter man," (S&H, p 336:1-2,13,19-20).
". . . and recognize that Jesus Christ is not God, as Jesus himself declared, but is the Son of God,"
(S&H, p 361:11-13).
Speaking of Gen. 2:7, "Then the Lord God formed man of dust from the ground, and breathed into
his nostrils the breath of life; and man became a living being," Eddy says, "Is this addition to His
creation real or unreal? Is it the truth, or is it a lie concerning man and God? It must be a lie, for God
presently curses the ground..."(S&H, p. 524:13-27).
In describing what the Devil is, it says, "Evil; a lie; error; neither corporeality nor mind; the
opposite of Truth; a belief in sin, sickness, and death; animal magnetism or hypnotism; the lust of the
flesh, which saith: ‘I am life and intelligence in matter. There is more than one mind, for I am mind, -
a wicked mind, self-made or created by a tribal god and put into the opposite of mind, termed matter,
thence to reproduce a mortal universe, including man, not after the image and likeness of Spirit, but
after its own image," (S&H, p. 584:17-25).
"If there had never existed such a person as the Galilean Prophet, it would make no difference to
me," (The First Church of Christ Scientist and Miscellany, pp. 318, 319).
Questions to Ask Christian Scientists

1. If God is all in all, then where did evil come from?


2. If everything is an interpretation of divine mind, then why do people have different
understandings of God?
3. If sickness is an illusion, why do you have practitioners who go out to Christian Scientists in
attempts to heal them?
4. If sin is not real, then why does the Bible say that all have sinned and fallen short of the
glory of God (Rom. 3:23)? As well as, "If we say we have no sin we deceive ourselves and
the truth is not in us," (1 John 1:8).
5. In Science and Health with Key to the Scriptures, Mary Baker Eddy said, "The material blood
of Jesus was no more efficacious to cleanse from sin when it was shed upon ‘the accursed
tree,’ than when it was flowing in his veins as he went daily about his Father’s business."
Why would she contradict so plainly the teaching of Scripture that says, "but if we walk in
the light as He Himself is in the light, we have fellowship with one another, and the blood of
Jesus His Son cleanses us from all sin" (1 John 1:7).
6. Why would Mary Baker Eddy directly contradict the Jesus’ own claim of Himself? She said
that "Jesus is not God, as Jesus himself declared, but the Son of God." (S & H 361:12-13). Is
she calling Jesus a liar?
7. If "Man is incapable of sin, sickness, and death" as Eddy said in Science and Health 475:28,
then why do people die? Why does the Bible say that the wages of sin is death (Rom. 6:23)?
8. Why would Mrs. Eddy say Jesus did not die (S&H 45:32-46:3) when the Bible clearly teaches
that He died (Rom. 8:34; 1 Thess. 4:13; 1 Cor. 8:11; 1 Pet. 3:18; 1 John 1:7).
9. If our physical senses do not tell us the truth about the material world then how can we trust
them when we read the book Science and Health with Key to the Scriptures or hear its
message with our ears?
Shepherd’s Chapel

Introduction

The Shepherds’ Chapel is a new movement on TV and radio. It is spreading through the abrasive and
rough styled preacher called Arnold Murray.

Is it Christian?

1. Does Shepherd's Chapel affirm the Trinity? p. 160


2. Did we exist as souls prior to Adam's creation? p. 162
3. What does Shepherd's Chapel say about the Kenites? p. 163
4. What does Shepherd's Chapel say about the Rapture? p. 165
What is the Shepherd's Chapel?

Shepherd's Chapel is an independent Church in Gravette, Ark, which is the product of the
gentleman by the name of Arnold Murray. Arnold Murray is an avid Bible teacher with a radio and
television ministry found on over 200 TV stations. However, what Arnold Murray teaches in many
areas of theology is his own brand of Christianity. In fact, it is not Christian at all in at least one very
important essential; he denies the doctrine of the Trinity. This is a serious error for him as it is a
denial of the true and living God. Murray teaches the heresy known as modalism, that God is one
person who takes three forms: the Father who became the Son who became the Holy Spirit. Also, the
Shepherd's Chapel teaches what is called the Christian Identity movement. This is the belief that the
British and, therefore, the Americans and Canadians, are the true descendants of the ancient Israelites
— the 10 lost tribes. This later doctrine is very often used to by militia groups support racist
ideologies. It is no surprise, therefore, to find that the Shepherd's Chapel s against interracial
marriages. I am not here saying that the Shepherd's Chapel teaches racism, but their denial of
interracial marriage is unbiblical.
Mr. Murray goes through the Bible verse by verse adding his own interpretations and using the
Strong's concordance to support his position. Those who do not understand God's word, and those who
have very little biblical training are easily swayed by the pseudo logic and Greek word explanations
Murray uses to support his positions.
One comment that is very revealing is that the people who follow Murray are very often just as rude
and condescending as he is. They, like their teacher, regularly call others liars, deceivers, idiots,
losers, etc. Murray is full of quick, condescending remarks about anyone who disagrees with him.
Unfortunately, it is rubbing off on his followers.
I would like to add that obtaining specific information from Shepard's Chapel is very difficult. I have
gleaned as much as I could from their website, from reading other web sites about Shepherd's Chapel,
and from going over the newsletters from Arnold Murray. The statement of faith on their web site is
not very descriptive and I think it is this way on purpose.
I strongly recommend that Christians and non-Christians alike stay away from this pseudo-Christian
group.
Who is Arnold Murray?

Arnold Murray is the "pastor" of a church called the Shepherd's Chapel out of Gravette, Ark, with
over 200 television stations carrying his ministry. Mr. Murray presents himself a Christian scholar, calls
himself "Dr.", and says he is an ex-marine who served in the Korean war where he was wounded.
Unfortunately, proof of his doctorate degree is not available. He refuses to provide any documentation
regarding his scholastic achievements. In fact, getting any information on Mr. Murray is very difficult.
He is rather private about his personal history and has refused to provide any biographical information
about himself. What we learn of Mr. Murray is what he tells us on his television show and his website.
It is immaterial whether or not Mr. Murray has a doctorate. He does not need one to be an adequate
teacher. But since he refers to himself as "Dr.", it would be nice to know from what institution he
received his credentials.
Mr. Murray stated that when he was young he attended many different churches and that he has
incorporated some of the teachings of these different churches into his current theology. Again, these
different churches are not mentioned. I would be very curious to know which ones they were,
especially if some of them were non-Christian cults.
Mr. Murray, or Pastor Murray, as his followers call him, teaches a brand of Christianity with many
problems. Mr. Murray denies the doctrine of the Trinity, denies the existence of hell and the rapture.
When watching his television show it becomes quickly evident that he is a bit condescending to
those whom he claims are misled, something that seems to have rubbed off on his followers. He
justifies this approach by stating that he was in the Marines and learned how to "teach his troops" with
discipline. It seems his approach is lacking the humility and grace of the Holy Spirit. Even the Ontario
Consultants on Religious Tolerance, a secular religious analysis organization, admits that Murray made
a false prophecy.
Mr. Arnold Murray also frequently refers to the Christian identity movement which teaches that the
British and, therefore, the Americans and Canadians, are the true descendants of the ancient Israelites
— the 10 lost tribes. Jews in Israel are really descended from Cain who was the result of sexual
intercourse between Eve and Satan. Murray also teaches that all people had a pre-existence in spirit-
bodies before they were born here on earth.
Mr. Murray supports his off-base theological positions by interpreting verses in light of his pre-
conceived ideas, sometimes stripping biblical verses out of their context. He also selectively appeals
to the Strong's concordance, cites what Greek and Hebrew words mean, and weaves his theological
positions into the text. In reality, all he has done is justify his ideas through his unsound interpretive
techniques. This is similar to the methodology used by the Watchtower magazine of the Jehovah's
Witnesses.
The fact that Mr. Murray deviates from orthodoxy in some significant areas (i.e., trinity), that he is
cryptic about his qualifications and experience, that he is often rudely condemning those who do not
agree with him, and his non-biblically based teachings are all signs of a false teacher. I recommend
that you stay far away from this man and the Shepherd's Chapel.
What does the Shepherd's Chapel teach?

The Shepherd's Chapel is, at the very least, a controversial church with many unorthodox
teachings. The following list of teachings from the Shepherd's Chapel is derived from scouring the
Internet, chat rooms, bulletin boards, asking the followers of Arnold Murray, and looking at the
Shepherd's Chapel web site.
The Shepherd's Chapel is considered to be a non-Christian cult by many people.

1. Unorthodox
A. Denies the Trinity and teaches that the Father, Son, and Holy Spirit are really three offices
held by the one God.
B. Denies the existence of eternal Hell.
C. Denies the doctrine of the rapture.
D. Denies physical resurrection of believers. We will be raised spiritually.
E. Teaches annihilationism: non-existence after death for sinners.
F. Teaches that certain Old Testament kosher laws regarding meat should be followed.
G. Teaches Serpent Seed doctrine: Eve had literal sexual relations with the serpent.
H. The offspring of Eve's impregnation resulted in Cain, Satan's offspring.
I. The Kenites are the tares of the parable of the wheat and the tares.
J. Satan's fallen angels impregnated women around the time of Noah.
K. Interracial marriage is wrong.
L. Being born again is to enter into this body from a spiritual body of another age.
M. America and Britain are the lost tribes of Israel.
N. People were alive in a pre-existence.
O. There was an earth age prior to Adam where a race of people lived.
P. The Rapture is a false doctrine.
2. Orthodox
A. Satan is a literal being.
B. Jesus will return.
C. Jesus will set up a future millennial kingdom.
D. The Bible is the inerrant word of God.
E. The earth is millions of years old.
Is the Shepherd's Chapel Christian?

In order to ascertain whether or not something is Christian, we must first understand what makes
something Christian in the first place. Throughout history, the Christian Church has gleaned from
God's word those essential doctrines that if denied, place the person outside the camp of Christ.
Following are three of the essential doctrines declared by Scripture as being necessary. In other
words, to deny any of these is to deny the heart of Christianity and be outside the camp of Christ.

1. The Deity of Christ


A. Jesus is God in flesh (John 8:58 with Exodus 3:14). See also John 1:1,14; 8:24; 10:30-33.
i. 1 John 4:2-3: "This is how you can recognize the Spirit of God: Every spirit that
acknowledges that Jesus Christ has come in the flesh is from God, but every spirit that
does not acknowledge Jesus is not from God. This is the spirit of the Antichrist, which
you have heard is coming and even now is already in the world."
a. The above verse needs to be cross referenced with John 1:1,14 (also written by
John) where he states that the Word was God and the Word became flesh.
b. 1 John 4:2-3 is saying that if you deny that Jesus is God in flesh then you are of the
spirit of Antichrist.
2. Salvation by Grace
A. "For it is by grace you have been saved, through faith -- and this not from yourselves, it is
the gift of God -- not by works, so that no one can boast" (Eph. 2:8-9, NIV).
i. "You who are trying to be justified by law have been alienated from Christ; you have
fallen away from grace," (Gal. 5:4).
3. The Resurrection of Christ
A. "And if Christ has not been raised, our preaching is useless and so is your faith," (1 Cor.
15:14). "And if Christ has not been raised, your faith is futile; you are still in your sins," (1
Cor. 15:17).
B. To deny the physical resurrection is to deny Jesus' work, sacrifice, and our resurrection.

There are other doctrines such as the doctrine of the Trinity, the Virgin birth, etc., are derivatively
necessary. But these are the ones in the scripture that are declared, by the Bible, as being essential.
The derivative doctrines like the Trinity are orthodox and accurate teachings from the Bible.

Shepherd's Chapel denies Jesus' Resurrection

The most important doctrinal issue that the Shepherd's Chapel deviates on is the physical
resurrection of Jesus. The shepherds Chapel teaches that Jesus rose in a spirit body, not the same
physical body that he died in. Because of this, the Shepherd's Chapel is not Christian.
Note: I have received conflicting information on this issue of Jesus' resurrection in regard to the
Shepherd's Chapel. Therefore, I will revise this paper if I discover that the Shepherd's Chapel affirms
Christ's physical resurrection. However, the fact that I have discovered conflicting information on this
issue is very problematic. The Shepherd's Chapel should be very clear on the issue of the resurrection
since it is a vital Christian doctrine. Its statement of faith says that Jesus rose from the dead, but
does not tell us what it means by that statement.

Shepherd's Chapel denies the Trinity

The second problematic issue with this church is its denial of the Trinity. The Bible teaches that
there is one God in three persons: Father, the Son, and the Holy Spirit. The Shepherd's Chapel
teaches modalism, the idea that God is one person who took different forms or modes. The Christian
church long condemned modalism as contrary to sound biblical theology.
The Shepherd's Chapel is at best a very aberrant Christian church and at worst, a cult. Since it
appears to deny the physical resurrection of Christ, it would seem to be non-Christian.
Did we exist as souls prior to Adam's creation?

One of the several false doctrines of the Shepherd's Chapel is the teaching that all people had an
existence prior to being here on earth in what is called the first earth age. According to Shepherd's
Chapel, there are three earth ages:

• First: Beginning of all things up to fall of Satan. This fall caused incredible destruction and
waste. We pre-existed the earthly existence in soul bodies. Those preexistent ones who
rebelled against Lucifer and sided with God are called the elect. Gen. 1:1.
• Second: After the fall of Satan, God had to remake the world. It consists of Gen. 1:2 through
Revelation where all the world, except the elect, will accept the antichrist.
• Third: Begins with the establishment of the millennium, proceeds through its end, and extends
into eternity. During the millennium, the elect are in spirit bodies, not physical ones.

These "ages" are based, in part, on what is called the Gap Theory. The Gap Theory states that
between Gen. 1:1 and 1:2 was a huge gap of time. The theory goes that there was a great
destruction in verse one and a great rebuilding in verse two. This gap is also claimed to encompass
the geologic epochs in which the dinosaurs lived.
The Shepherd's Chapel teaches that we were created in the first earth age, a time before the fall,
where we existed in the form of soul bodies. The problem with this is that there is no biblical support
for this position. In fact, the Bible contradicts this soul-body idea.

"it is sown a natural body, it is raised a spiritual body. If there is a natural body, there is also a
spiritual body. 45So also it is written, "The first man, Adam, became a living soul." The last
Adam became a life-giving spirit. 46However, the spiritual is not first, but the natural; then the
spiritual. 47The first man is from the earth, earthy; the second man is from heaven," (1 Cor.
15:44-47).

Paul is teaching here that the first "body" we have is the natural, or the physical body. After that
comes the spiritual body which is a resurrected body. Paul is telling us about our existence, about our
bodies, and about the order of their existence. Clearly, the teaching that we had soul-bodies prior to
this life is wrong. The Shepherd's Chapel position is simply not biblical.
The serpent seed and the Kenites

Two additional distinguishing and erroneous doctrines of the Shepherd's Chapel are known as the
Serpent Seed doctrine and the Kenite doctrine. These are intimately related. The Serpent Seed
doctrine is the teaching that in the Garden of Eden, the serpent (the devil) had sexual relations with
Eve. The result was that she bore Cain. The descendents of Cain are called Kenites. Abel, however,
is the result of Adam and Eve having relations.
Arnold Murray, the pastor of the Shepherd's Chapel, is the primary advocate of these doctrines
which he adamantly teaches and which his followers have adopted as biblical truth. Mr. Murray
states,

When you look for the in-depth meaning of "men as trees, walking", you are able to see that
Christ wants us to understand there are plantings of God and plantings of the devil. The
plantings of that wicked one began in the Garden of Eden with the conception of Cain and follow
down through his progeny, the Kenites. (Newsletter #195, Jan 1995. See also, #202, August
1995).

The Kenites, according to Mr. Murray, must be exposed. "We must continue to teach who the
Kenites are," says Mr. Murray, (Newsletter #190, August 1994). He states that the Kenites survived
the flood (he denies the global flood) and are found in the lineage of Israel, not Judah, (Newsletter
#179, Sept. 1993). Eventually, the Kenites permeated the nation of Israel and are the ones who
shouted "Crucify Him," in reference to Jesus, (Newsletter #179, Sept. 1993).
In an attempt at biblical support, on his website at Answers to Critics, Mr. Murray states:

In Gen. 3:15 God is speaking to the serpent, "and I will put enmity between thee and the
woman, and between thy seed and her seed; it shall bruise thy head, and thou shall bruise his
heel."

Mr. Murray infers that when God says "thy seed" to Satan, He is referring to the Kenites, the
descendents of Cain which were literally produced through the literal "seed" of Satan.
Of course, I disagree with Mr. Murray in his analysis. We do not really know exactly what form
Satan was in the Garden, though I will submit to God's word and affirm it was a snake of some sort.
The word "serpent" is "nachash" and means serpent or snake. If we take the word literally and it
means snake, then Mr. Murray would be forced to explain how a literal snake could have sexual
intercourse with Eve. If Mr. Murray were to acknowledge the potential of a figurative usage of the
term here, then he needs to explain why the term "serpent" would be figurative and the term "seed"
would be literal. Furthermore, if the serpent were Satan in a different form, and Eve spoke to the
serpent, then did Eve have sex with a snake or with a different form of the snake; that is, did the
snake change into another more apropos form to consummate his deception? If so, wouldn't Eve have
been suspicious of a talking snake that changes form into something else with which she then agrees
to sexual intercourse? As you can see, the issue, from Mr. Murray's perspective, is wrought with
problems -- none of which he has answered.
Nevertheless, his entire position is easily refuted when we examine Gen. 4:1: "Now the man had
relations with his wife Eve, and she conceived and gave birth to Cain, and she said, "I have gotten a
manchild with the help of the Lord." We can see that the Bible clearly tells us who the Father of Cain
is: Adam. The Serpent Seed idea is proven wrong.
In addition, I believe that it is more natural to attribute the term "seed" in Gen. 3:15 as a reference
to the spiritual decedents of Satan, not his literal ones. We can see that being a true spiritual
descendent is by faith, not by biology.

• "For he is not a Jew, which is one outwardly; neither is that circumcision, which is outward in
the flesh: 29But he is a Jew, which is one inwardly; and circumcision is that of the heart, in the
spirit, and not in the letter; whose praise is not of men, but of God," (Rom. 2:28-29).
• "For all who are being led by the Spirit of God, these are sons of God," (Rom. 8:14).
• "The Spirit Himself bears witness with our spirit that we are children of God, 17and if children,
heirs also, heirs of God and fellow heirs with Christ, if indeed we suffer with Him in order that
we may also be glorified with Him," (Rom. 8:16).
• "That is, it is not the children of the flesh who are children of God, but the children of the
promise are regarded as descendants," (Rom. 9:8).

Clearly, being a descendent has a spiritual quality. Likewise, Satan's descendents are those who
identify with him in his lies. This is why Jesus said in John 8:44 to the Pharisees, "You are of your
father the devil, and you want to do the desires of your father. He was a murderer from the beginning,
and does not stand in the truth, because there is no truth in him. Whenever he speaks a lie, he speaks
from his own nature; for he is a liar, and the father of lies."
In addition, "seed" is also referred to as the word of God (Matt. 13:18-23; Luke 8:11; 1 Pet. 1:23)
and as the spiritual life in (1 John 3:9). The whole flavor of spiritual identification with God is included
in terms of being the offspring of God (Gal. 3:29; Acts 17:28) and the children of God (Rom. 8:16-
17). Consider 1 John 3:9 which says, "No one who is born of God practices sin, because His seed
abides in him; and he cannot sin, because he is born of God." Obviously, the Christian does not
contain the literal seed of God in him. Seed here, must refer to a spiritual element of indwelling, of
ideology, and of faith. The Christian does not practice sin. It is against his beliefs and confession.
Therefore, God's seed abides in the Christian, but it isn't literal seed as a descendent through
procreation, but through spiritual identification.

Us against Them

The Serpent Seed doctrine is an unscriptural and unfortunate teaching. From it is derived an "us
against them" mentality in which anyone who disagrees with Mr. Murray can easily be accused of being
a Kenite. This is obvious in some of his quotes:

• "How many today are teaching from a quarterly written by a Kenite, rather than teaching from
God's Word? (Newsletter #193, Nov. 1994).
• How can we sum this up? If you are doing God's will, don't worry about criticism from others.
"Well, Pastor Murray they say we are a cult." Who cares what they say? God is on our side.
Victory is a certainty. Does it ever seem like the enemy is winning? Anytime you get to feeling
this way, turn to Psalms 9. This Psalm tells us what we should be earnestly expecting. Keep the
meaning of "apokaradokia" in mind as we read this Psalm. (Newsletter #229 - November
1997).

Of course, if you study with him you are not being deceived, but if you are studying elsewhere,
you're studying with a Kenite or siding with the enemy. Such are the machinations of this leader who
teaches false doctrines.
Please be very wary of the Shepherd's Chapel.
Shepherd's Chapel and the rapture

The Shepherd's Chapel denies the doctrine of the rapture which is the teaching that upon the
future return of Christ those Christians who are alive will be "caught up" to meet the Lord in the air.
The word rapture is derived from the Latin "rapio" which means "to seize," "to snatch." The primary
verses used to support this doctrine are found in 1 Thess. 4:16. "For the Lord himself shall descend
from heaven with a shout, with the voice of the archangel, and with the trump of God: and the dead in
Christ shall rise first: 17Then we which are alive and remain shall be caught up together with them in
the clouds, to meet the Lord in the air: and so shall we ever be with the Lord." At the time of the
rapture all Christians will be transformed bodily. According to 1 Cor. 15:51-52, "Behold, I tell you a
mystery; we shall not all sleep, but we shall all be changed, 52in a moment, in the twinkling of an eye,
at the last trumpet; for the trumpet will sound, and the dead will be raised imperishable, and we shall
be changed."
There are three main theories to deal with when the rapture will occur in relation to the seven year
tribulation period. They are known as pre-tribulation, mid-tribulation, post-tribulation. In short, the
difference between them is in respect to when the rapture will take place. Will it occur before the
tribulation, at the midpoint of the tribulation, or at the end of the tribulation?
Mr. Murray, in an attempt to support his erring doctrine, teaches that the word "clouds" in 1
Thessalonians is not referring to clouds in the sky but a gathering of people of like mind, a collection of
witnesses who know the truth. Such interpretations are not consistent with the 1 Thess. passage.
Accepting or denying the teaching of the rapture does not affect one's salvation. It is a doctrine
that can be believed or not without any affect upon a person's position in Christ. Unfortunately, the
Shepherd's Chapel teaching is rather harsh regarding the rapture. Mr. Murray, the pastor of the
Shepherd's Chapel, states that the rapture is a false doctrine (Newsletter #193 - Nov, 1994) and that
many who believe in the rapture will be deceived into worshiping the Antichrist (Newsletter #205, Nov.
1995). At one point Mr. Murray stated that 95 percent of the churches that teach the rapture will
accept the antichrist as the savior (Newsletter #116, June 1988). Such statements are simply not
sustainable from a biblical perspective. Nevertheless, please consider some more quotes from Mr.
Murray regarding the rapture:

1. Teachers of the rapture are dangerous.


A. "The false prophets promise salvation cheaply. ... They say, 'All you have to do is believe
and then wait to be raptured out of here'. These are dangerous teachers, friends."
(Newsletter #193 - Nov. 1994).
2. Preachers have twisted the truth regarding the rapture.
A. "Christ taught in their streets, but they weren't listening. You can hear it unto this day from
the very pulpits that should be teaching truth. They have twisted the truth into teaching the
rapture, the Easter bunny and the apple Eve ate in the Garden of Eden. These false
teachings make those that accept this doctrine easy marks for Satan's bondage." (Newsletter
#230 - Dec. 1997).
3. False prophets teach the rapture
A. The false prophets utilize the rapture as a security blanket. Placing words in God's mouth is
very dangerous. Believing those words is also dangerous. Think for yourself. Believe only
what God has said in the letter He wrote to you. (Newsletter #245 - March 1999, Men Say
and God Says)
4. The Rapture doctrine is from Satan.
A. Yet, today we have false teachers leading people to Satan's doctrine of rapture from the very
pulpits from which they should be hearing the doctrine of Jesus Christ. It is very appropriate
that judgment begins at the pulpits. (Newsletter #253 - Nov. 1999).
5. The Rapture is not in God's word but is from seducing spirits.
A. The rapture is not in God's Word. God is against those seducing spirits who would teach His
children to fly to save their souls. In conclusion, let's go to the Book of Mark. The disciples
asked Jesus what it would be like when He returns at the Second Advent. Jesus gave us
seven events that must transpire before His return. (Newsletter #262 - Aug. 2000)
6. There will be no rapture
A. There will be no rapture; however, if you believe the rapture theory, Satan might use this
fact to deceive you. The false prophets promise salvation cheaply. ... They say, "All you have
to do is believe and then wait to be raptured out of here". These are dangerous teachers,
friends. (Newsletter #193 - November 1994, Prepare to Meet Your God)

As you can see, Mr. Murray adamantly denies the doctrine of the rapture and states that those who
believe in it are greatly deceived since it is a doctrine of Satan. In reality, it is Mr. Murray who is
deceived. He is perfectly free to deny the rapture doctrine. It isn't an essential teaching of the Bible.
But, his condemnation of those who believe in it goes well beyond the admonitions of Rom. 14:5 which
says, "...Let each man be fully convinced in his own mind."
An open letter to Arnold Murray of Shepherd's Chapel

Many followers of Arnold Murray have emailed me telling me that my criticisms of Dr. Murray are
unfounded and erroneous. It is not my intention to misrepresent any person or group that I write
about. If I have misrepresented Murray's teachings, then I have only my poor research to blame. I
must admit, however, that it has been very difficult obtaining precise theological positions from Arnold
Murray's information. Therefore, in my attempts to accurately represent the teachings of Arnold
Murray and the Shepherd's Chapel, I am posting this open letter to Arnold Murray.
I am politely requesting that Pastor Murray provide a response to the following list of statements.
Of course, he is not obligated to do so and I assume that he will not. Nevertheless, if he does, I will
post his response on this website, in totality, so that others might see what he really believes. I will
also change the information on my site should any of his statements correct what I have on my site.
In addition, if he were to respond to this request for doctrinal clarification, it could provide a means
by which other apologetics research groups might be corrected should they be in error in what they
have stated concerning Dr. Arnold Murray and the Shepherd's Chapel. Of course, he may certainly
expand on the answers as he sees fit if he would be so kind as to answer.
In order to verify that Dr. Murray has answered these, should he choose to, I request that the
answers be placed on his website at shepherdschapel.com in a hidden directory that would be provided
to me in an email. This way, I can assume that the answers are legitimate.
Please note that I have purposely asked some questions in both a negative and positive manner.
This is to avoid any confusion and to serve as verification of his position.
This request was posted on the Internet on 9/26/2002. Any additional questions will be appended
and dated.

Arnold Murray, as teacher of the Shepherd's Chapel, do you affirm or deny the following?

1. The Bible
A. The Bible is the only inspired word of God on earth. It is inerrant in the original documents.
B. The Bible consists only of 66 books.
C. The KJV is the only correct Bible to be used today.
D. The Pauline epistles are the only New Testament scriptures meant for the Christians today.
Comments:___________
2. Creation
A. The universe was brought into existence by God.
B. God transcends the universe, is within it as well as beyond it, but not part of it nor it part of
Him. In other words, God is separate from the universe.
C. Pantheism: The universe is identical to God and the combined laws and forces of the
universe are God. The universe is divine since God is divine.
D. Panentheism: The universe is contained within God. The universe is a part of God, but not
all of God.
E. The universe is has been affected by the fall of Adam with the introduction of sin.
F. God created the universe in six literal, 24 hour days.
G. God created the universe over long periods of time.
H. Evolution is how God brought all the animal species into existence.
Comments:___________
3. God
A. There is only one God in all existence, in all times, in all locations.
B. God is eternal. He is without beginning or end.
C. God is a trinity of persons: Father, Son, and Holy Spirit. Each person is co-eternal, co-
powerful, and co-equal with the other two. These three persons are eternally and
simultaneously co-existent
D. God is one person who manifests himself in three forms, modes, or offices and is not three
persons.
E. God is a single and absolute person.
F. There are other legitimate gods in the universe, but we serve and worship only one of them.
Comments:___________
4. Jesus
A. Jesus was born of the virgin Mary
B. Jesus while on earth was both God and man.
C. Jesus was only a man and was not divine.
D. Jesus was only divine and not a man.
E. Jesus is/was an angel who became a man.
F. Jesus became God during his earthly ministry.
G. Jesus is separate from the "Christ idea."
H. Jesus is Melchizedek of the Old Testament.
I. Jesus presently retains His two natures; He is presently both human and divine.
J. Jesus died on the cross, was buried in a tomb, and rose physically from the dead.
K. Jesus retained the wounds of His crucifixion ordeal.
L. Jesus bore our sins in His body on the cross.
M. Jesus' sacrifice was substitutionary and vicarious.
N. Jesus is presently in heaven sitting at the right hand of God the Father.
Comments:___________
5. The Holy Spirit
A. The Holy Spirit is the impersonal presence and/or force of God.
B. The Holy Spirit is a person, the third person in the Godhead, coequal with the Father and the
Son.
Comments:___________
6. Man
A. Man is by nature sinful.
B. Man is by nature morally neutral.
C. Man is by nature divine.
D. People have the potential of becoming gods.
E. Christians cannot be demon possessed.
F. Christians can be demon possessed.
G. People pre existed as spirits before coming to this world.
H. Upon death, an individual ceases to exist.
I. Upon death, an individual is still conscious.
Comments:___________
7. Satan and fallen angels
A. Satan is a fallen angel
B. Demons are disembodied spirits of the offspring of the Nephilim of Genesis who were
destroyed in the flood.
C. Demons are fallen angels.
D. Fallen angels had sexual relations with women and produced the Nephilim.
Comments:___________
8. Sin
A. Sin is rebellion against the law of God.
B. Sin is improper thoughts about the divine nature of the universe.
C. Sin is disharmony with the divine consciousness.
Comments:___________
9. Salvation
A. Forgiveness of sins before God is received completely by grace through faith in Christ and
His work on the cross and not by any works of our own doing.
B. Justification is the state of being declared righteous in God's sight and is received solely by
faith.
C. Water baptism is not necessary for salvation, but is a mandatory practice for Christians.
D. Water baptism is necessary in order to have one's sins forgiven.
E. Water baptism must be done "in Jesus' name."
F. Water baptism must be done "in the name of the Father, the Son, and the Holy Spirit."
G. Water baptism is not for today.
H. Salvation can be lost.
I. Salvation cannot be lost.
J. Salvation will result to all people who have ever lived.
Comments:___________
10. Resurrection
A. The resurrection of Christ was physical. In other words, Jesus rose from the dead in the
same body He died in.
B. Jesus was not raised physically, but spiritually. In other words, Jesus' physical body was not
raised.
C. Jesus was raised physically, but ascended into heaven spiritually, not physcially.
D. Jesus is presently a man, yet also God, in bodily form.
E. The Resurrection of the believers will be bodily. That is, they will be raised in the same
bodies they die in, though the bodies will be glorified.
F. The resurrection of the non-believers will be bodily. That is, they will be raised physically
and cast into eternal hell.
G. There is no physical resurrection of the believer. The believers are raised in spirit bodies,
not their physical bodies.
H. There is no physical resurrection of the unbelievers. They remain unconscious and/or are
utterly destroyed so as to not exist.
Comments:___________
11. Death and Judgment
A. When a Christian dies he immediately goes to be with the Lord.
B. When a Christian dies he does not exist anymore until God "remembers" him and resurrects
him.
C. When a person dies, he ceases to be conscious.
D. When an unbeliever dies, he is annihilated.
E. Unbelievers who are annihilated are raised spiritually to face judgment and then are
annihilated again.
F. Hell is an eternal place of conscious torment.
G. People are conscious after death even though separated from their physical bodies.
H. All people are annihilated after death.
Comments:___________
12. Eschatology
A. There will be a literal 7 year tribulation period before the millennium.
B. There will be a literal man known as the antichrist who opposes God.
C. We are in the millennial reign of Christ right now.
D. We are not in the millennial reign of Christ right now. Instead, Jesus will return to establish
a literal 1000 year reign on earth.
E. After the millennium, a new heavens and a new earth will be made and eternity will begin.
F. The rapture (that Christians will be caught up together in the air to meet the returning Lord
Jesus) is the future occurrence where the Christians are caught up into the heavens to be
with the Lord.
G. The rapture (that Christians will be caught up together in the air to meet the returning Lord
Jesus) is a false doctrine.
H. Matthew 24 and most, if not all, of Revelation was fulfilled around 70 A.D.
Comments:___________
13. Miscellaneous
A. The charismatic gifts have ceased with the closing of the canon.
B. The charismatic gifts have not ceased.
C. God works primarily in dispensations throughout history.
D. God works primarily in covenants throughout history.
E. Saturday is the proper day of worship.
F. We are free to worship on any day of the week.
G. The flood of Noah was worldwide.
Comments:___________
14. Particulars related to Shepherd's Chapel
A. There was an earth age prior to this age with pre-Adamic humanlike creatures.
B. Our spirits preexisted before coming to this world.
C. Being born again is to enter into this body from a spiritual body of another age.
D. Eve had sexual relations with the serpent and produced Cain.
E. The descendents of Cain are called the Kenites.
F. Adam has sexual relations with the serpent.
G. Salvation is found only in the Shepherd's Chapel's teachings.
H. Shepherd's Chapel teaching is the proper and most correct teaching of any church.
I. There are many people who are saved who are not members of Shepherd's Chapel.
J. In Matt. 13, the parable of the wheat and the tares, are the first ones taken, taken by the
antichrist?
K. Satan is the tree of knowledge in Genesis.
L. There are certain Old Testament Kosher laws that should be observed today.
M. Interracial marriage is sinful.
N. Interracial marriage is not sinful.
O. The Kenites are the tares of the parable of the wheat and the tares.
P. America and Britain are the lost tribes of Israel.
Q. Arnold Murray is one of God's true teachers on earth today.
R. The apostasy spoken of in 2 Thessalonians already occurred?
Comments:___________
International Church of Christ
Introduction
The International Church of Christ (ICC) is a break-off of the Church of Christ denomination. The
ICC is Christian in its basic theology but has some aberrant practices. The ICC was influenced by the
discipling movement on the 1950's. Its roots can be traced back to 1967 to the Crossroads Church of
Christ, in Gainesville Florida. The crossroads Church had a program on discipling which became known
as the Crossroads Movement. It is out of this Crossroads connection that the present leader of the ICC,
Kip McKeen, received his start. He and Roger Lamb were fired from the Houston, Texas, Church of
Christ. Kip McKeen then found a Church in Boston MA, was asked to come on board and began what
has come to be known as the Boston movement. The Boston Church grew by leaps and bounds due to
its heavy discipling program. Soon other churches were being planted in the United States and then in
England.
The ICC did not become known as the "International Church of Christ" until 1993 and its
headquarters became Los Angeles, California. The ICC considers Christian denominations to be sinful.
They will cite biblical passages that speak of the apostles establishing one Church per city, and claim
that there should only be one Church in each city. Of course, the one Church should be an ICC
Church. As of the year 2001, the ICC claims to have over 400 churches with a membership of
130,000 worldwide in over 150 countries.
The ICC considers itself to be "a family of Christian churches whose members are committed to
living their lives in accordance with the teachings of Jesus Christ as found in the Bible." This
commitment to biblical living includes a very strong emphasis on discipling. In fact, it is this overly
strong emphasis on discipling that has drawn much criticism from outside the church as inside from
those who were once were members.
The ICC is Orthodox affirms the Trinity, salvation by grace, Jesus' virgin birth and physical
resurrection, His deity, the personhood of the Holy Spirit, heaven and hell, and much more. But, it
deviates from orthodoxy in both its requirement of baptism as a necessary element for salvation and
its heavy requirement of discipleship. According to the ICC, baptism must be done in their church with
the person being baptized having an understand that baptism saves. Combined with this, the ICC
method of discipleship includes strong accountability to other members of the church as a necessary
element to be considered a Christian. According to the ICC, one cannot be a Christian if he is not a
true disciple, and being a disciple must precede baptism. Therefore, the International Church of Christ
tends to be very legalistic and controlling. Many of its former members attest to requirements that
they confess their sins to their disciple leaders, that they submit to the decisions of their disciple
"leaders" regarding dating, frequency of sexual relations for married couples, jobs to take, places to
move, and so on.
This discipling operation with in the ICC has drawn much criticism for its intrusive practices and has
been labeled as a form of the brainwashing and psychological and emotional manipulation. There are
numerous web sites on the Internet devoted to ex-members of the International Church of Christ who
warn people not to be involved with the movement. There are also support groups to help those who
have left to find healing and, hopefully, true Grace in Christ instead of legalism and bondage.

1. What are two aberrant teachings/practices of the ICC? p. 172


2. How is their discipling methodology misapplied? p. 175
3. What is justification? p. 177
4. Is baptism necessary for salvation? Why or why not? pp. 179-182
5. What are some textual issues concerning Mark 16:9-20? pp. 183-184
6. Why is Acts 10:44-48 so important concerning baptism? p. 190
7. Does John 3:5 teach that baptism is essential to salvation? p. 191
8. Does Acts 2:38 teach that baptism is essential to salvation? pp. 193-196
What does the International Church of Christ teach?

The international Church of Christ is Orthodox in most of its theology. It accepts and affirms the
following doctrines.

1. The Trinity is one God and three persons: Father, the Son, and the Holy Spirit.
2. The deity of Christ
3. The deity of the Holy Spirit.
4. The physical Resurrection of Christ
5. The Bible is the inspired word of God and the ultimate source of authority.
6. Baptism is for adults only.
7. There is a literal Hell.
8. Satan is a literal fallen Angel.

Aberrant Doctrines/Practices

1. Baptism, by emersion, is essential to salvation.


2. Baptism must be as a true "disciple" or it is not valid.
3. Baptism must be performed in the International Church of Christ to be valid.
4. Being a disciple is necessary to be a Christian.
5. There should only be one Church in each city or town.
6. The ICC is the true remnant of God's people as the true Church.
7. Only ICC baptized members are saved.
8. Sin lists of disciples are often kept.
9. Heavy influence into the personal lives of disciples.
Is the International Church of Christ a cult?

The answer to the question, "Is the International Church of Christ a cult?" depends on the
definition of what makes a group a cult. The definition of a cult that CARM uses is a group that may or
may not include the Bible in its set of authoritative scriptures. If it does include the Bible and it
deviates from biblical doctrines sufficient to make salvation of no effect, then it is a cult. However, it is
possible to have a truly Christian group that has cult like tendencies, such as the exercise of an
abnormal amount of control over its people. This control may be centered around theological or social
particulars. It is this issue of excessive control that to the international Church of Christ is known for.
Add to this an exclusive attitude that it is the only true Church; it is no one wonder that the ICC has
been labeled a cult by many people. Therefore, it is possible to have a Christian Church with cultic
practices and still have it be, technically, Christian and aberrant.
Additionally, the doctrines that make Christianity "Christian" are the Trinity, the deity of Christ and
the Holy Spirit, the physical resurrection of Jesus, and salvation by grace. There can be deviations
within Christianity on other issues such as the mode of baptism, frequency of communion, when the
rapture will happen, etc. Usually, it is these sorts of non-essential issues which cause denominational
splits.
The International Church of Christ (ICC), does not deny any of the essential doctrines of
Christianity. Aside from its requirements on baptism as a necessary part of salvation, it is Orthodox.
Therefore, I do not categorize the ICC as a non-Christian cult. Rather, I see the ICC as a Christian
Church with some serious problems; namely, legalistic and excessive control over its members.
I recommend that people avoid this group.
The International Church of Christ

Also known as . . .
The Boston Church of Christ, The Crossroads Movement, Multiplying Ministries,
The Discipling Movement, The International Church of Christ,
The London Church of Christ, The San Diego Church of Christ, etc.

1. Doctrines
A. Believe in the Bible as the inerrant and infallible word of God.
B. Trinitarian, believe in the resurrection of Jesus, and the sacrificial atonement.
C. The Boston Church of Christ is the only true church.
D. Baptismal Regeneration
i. Baptism is necessary for salvation
ii. Baptism in their church with a proper understanding that baptism saves.
E. Heavy discipleship
F. Unquestioned submission to authority
2. History
A. Its distant origins go back to 1967 in Gainesville Florida, under Charles Lucas.
i. Lucas started a program called the Multiplying Ministries program which was very
successful.
B. The movement we are concerned with here originated in the Crossroads Church of Christ in
Florida in 1985.
i. Kip McKean had been trained in the Discipling methodology by Mr. Lucas. From
Charleston, Ill., early in the movement. Kip later moved to Massachusetts and using the
methods he learned under Lucas. The church there began to grow.
C. In the first year, 103 people were baptized into their church. The second year 200 were
baptized; The third year 256; The fourth year 368; The fifth 457; the sixth 679, the seventh,
735; the eighth, 947; the ninth 1424; and in the tenth year, 1621 were baptized. Total of
6790 people. (as quoted in What Does The Boston Movement Teach? By Jerry Jones, Vol. 1,
p. 125).
i. "From its modest beginnings, the church has grown into 103 congregations all over the
world with total Sunday attendance of 50,000" (Time, 18 May 1992, p. 62).
D. In 1982 the Boston movement began planting their pillar churches.
i. These are churches in key cities throughout the world. The first two were established in
Chicago and London.
ii. Then in 1986, a program called "reconstruction" was undertaken. This is the process
whereby ministers in established Church of Christ churches are replaced with Boston
Church of Christ trained ministers (Ibid., pp. 126-127).
iii. This caused problems among the organization, but it helped to solidify this group.
3. Church Structure
A. Kip McKean is the director and unquestioned leader.
i. Under Kip are Elders.
ii. Under the Elders are Evangelists
iii. Then Zone Leader
iv. Then House Church Leaders (obsolete in most congregations)
v. Then Assistant Bible Talk Leaders
vi. McKean says these leaders govern by consensus but adds, `I'm the one who gives them
direction.'
vii. Al Baird, an important Boston Church of Christ elder said, "It's not a dictatorship. It's a
theocracy, with God on top," (Time, 18 May 1992, p. 62).
viii. Baird also said, "In questions of spiritual leaders abusing their authority. It is not an
option to rebel against their authority" (What Does The Boston Movement Teach? p. 7).
B. They are highly authoritarian
4. Authority and Submission (Power corrupts and absolute power corrupts absolutely)
A. In a discussion on submission, Al Baird, one of the uppermost leaders in the ICC said, "Let
us begin our discussion of submission by talking about what it is not. (1) Submission is not
agreeing. When one agrees with the decision that he is called to submit to, he does not
really have to submit in any way. By definition, submission is doing something one has been
asked to do that he would not do if he had his own way. (2) Submission is not just outward
obedience. It includes that, but also involves obedience from the heart. It is a wholehearted
giving-up of one's own desires. (3) Submission is not conditional. We submit to authority,
not because the one in authority deserves it, but because the authority comes from God;
therefore, we are in reality submitting to God."
B. Later in this same series, Baird states, "When we are under authority, we are to submit and
obey our leaders even when they are not very Christ-like. However, God has standards for
His leaders, and they will be accountable to God for ignoring those standards" (Authority and
Submission, parts III, V and VII as quoted in What Does The Boston Movement Teach?, pp.
59-63).
C. Baird has said that if the leader commands one to do something, even if it is not "Christ-
like," the member must submit! (Ibid., p. 104).
D. Many who have left the Boston Church "complain that the advice, which members are
expected to obey, may include such details as where to live, whom and when to date, what
courses to take in school, even how often to have sex with a spouse" (Time, 18 May 1992, p.
62).
E. Those who think for themselves and question the authority system are labeled.
i. "Bad heart" "Struggling" and "not really a disciple" are terms used when someone
disagrees with a leader.
ii. The group, then, instantly accepts these labels upon a person.
5. Discipleship
A. Step one: Invitation to a Bible talk.
i. Non threatening environment and topics.
a. Covers basics of Christianity, easy to understand discussions.
ii. The potential converts are befriended, with invitations to further talks.
iii. Then he/she is urged to join with a discipler to study the Bible and learn how to be more
like Jesus.
iv. From there the person is introduced into more strict and emotionally involving practices
and studies.
B. "Hooking" is when someone searches for another person's interests, hobbies, and other
personal information for the purpose of flattering them. This is done to attract them to the
group, rather than for a sincere interest.
C. "Love- bombing" is where a visitor is inundated by flattery and friendliness in order to
produce the feeling that the group will fulfill many of their needs and desires.
D. Once a member is assigned a discipler, he is led through a series of Bible studies.
i. First study is called, "First Principles." This is simple basics of the Bible.
ii. Second, "The Sins of Galatians" study.
a. To get them to repent, to confess their sins, etc.
i. The person reads Galatians 5:19-21 and then is told to make a list of their sins.
They are guided to other scriptures that have similar effects of bringing a
person to feel guilty and full of sin. More sin lists follow.
ii. Sometimes a list is kept of their sins which are sometimes brought up in various
situations. . . ultimately, to keep control over the person.
iii. This can be used to break a person down emotionally
iii. Third, "The Cross" study.
a. The disciple listens to the discipler read the crucifixion account and is asked to say,
"I am Judas -- I am Peter."
b. The crucifixion is described in more of its unpleasant details and the discipler reads
from the disciple's sin list.
c. Often, sentences are spoken such as, "You punched him in the face." "You taunted
Him." "You whipped Him."
d. The disciple is supposed to be "broken" in this process, and often is...emotionally.
6. Baptism
A. "The Boston Church of Christ teaches that when one initially receives Jesus Christ, one's
response must include faith, repentance, confession, and water baptism. It teaches that
apart from water baptism, one's sins are not forgiven" (The Issue of Water Baptism and the
Boston Church of Christ, E. Bourland, P. Owen and P. Reid, p. 1).
B. Not only must one be baptized, but one must also be baptized in the Boston Church of
Christ. If a person had been a member of some other church, then joins the Boston Church,
they must be rebaptized because their original baptism was done in a false church without a
proper understanding of baptism.
C. Therefore, salvation is gained by believing in Jesus' death on the cross and on being
baptized.
i. Also, a person must be baptized by someone in authority in the Boston Church of Christ,
i.e., International Church of Christ Movement.
7. Miscellaneous Info
A. Many people have been injured by the church's authoritarian and intrusive structure, so
much so that the Boston University, Marquette University, University of Southern California,
Northeastern University, and Vanderbilt University are some of the schools that have banned
the Boston Church of Christ (Miami Herald, 25 March 1992, p. 1A, 15A).

This group is definitely to be avoided. Along with its erring doctrine that baptism is necessary for
salvation, it is legalistic, manipulative, and uses guilt and aberrant doctrines to keep its members in
line. Though it is not a cult per se, it has many cult practices. It has destroyed many lives and left
many others wary of anything Christian.
Verses showing justification by faith.

Justification is the legal act where God declares the sinner to be innocent of his or her sins. It is
not that the sinner is now sinless, but that he is "declared" sinless. This declaration of righteousness is
being justified before God. This justification is based on the shed blood of Jesus, "...having now been
justified by His blood..." (Rom. 5:9) where Jesus was crucified, died, was buried, and rose again (1
Cor. 15:1-4). God imputes (reckons to our account) the righteousness of Christ; at the same time our
sins were imputed to Christ when he was on the cross. That is why it says in 1 Pet. 2:24, "and He
Himself bore our sins in His body on the cross, that we might die to sin and live to righteousness; for
by His wounds you were healed." Also, 2 Cor. 5:21 says, "He made Him who knew no sin to be sin on
our behalf, that we might become the righteousness of God in Him." Additionally, we are justified by
faith (Rom. 5:1) apart from works of the Law (Rom. 3:28).
To be saved means that God has delivered us (saved us) from His righteous wrathful judgment due
us because of our sins against Him. It means that we will not be judged for our sins and be sentenced
to eternal damnation. To be saved means that we are justified before God. Only Christians are
saved. Only Christians are justified. The issue at hand is whether or not this salvation, this
justification, is attained by faith or by faith and something else.
Following is a list of verses that show that salvation/justification is by faith. Bold references are
particularly pointed.

1. John 3:16, "For God so loved the world, that He gave His only begotten Son, that whoever
believes in Him should not perish, but have eternal life."
2. Rom. 3:22, "even the righteousness of God through faith in Jesus Christ for all those who believe;
for there is no distinction."
3. Rom. 3:24, "being justified as a gift by His grace through the redemption which is in Christ
Jesus;"
4. Rom. 3:26, "for the demonstration, I say, of His righteousness at the present time, that He might
be just and the justifier of the one who has faith in Jesus."
5. Rom. 3:28-30, "For we maintain that a man is justified by faith apart from works of the Law. 29Or
is God the God of Jews only? Is He not the God of Gentiles also? Yes, of Gentiles also, 30since
indeed God who will justify the circumcised by faith and the uncircumcised through faith is one."
6. Rom. 4:3, "For what does the Scripture say? "And Abraham believed God, and it was reckoned to
him as righteousness."
7. Rom. 4:5, "But to the one who does not work, but believes in Him who justifies the ungodly, his
faith is reckoned as righteousness,"
8. Rom. 4:11, "And he received the sign of circumcision, a seal of the righteousness of the faith
which he had while still uncircumcised, that he might be the father of all those who believe,
though they are uncircumcised, that righteousness might be imputed to them also,"
9. Rom. 4:16, "Therefore it is of faith that it might be according to grace, so that the promise might
be sure to all the seed, not only to those who are of the law, but also to those who are of the
faith of Abraham, who is the father of us all."
10. Rom. 5:1, "therefore having been justified by faith, we have peace with God through our Lord
Jesus Christ,"
11. Rom. 5:9, ”Much more then, having now been justified by His blood, we shall be saved from the
wrath of God through Him."
12. Rom. 9:30, "What shall we say then? That Gentiles, who did not pursue righteousness, attained
righteousness, even the righteousness which is by faith."
13. Rom. 9:33, "just as it is written, “Behold, I lay in Zion a stone of stumbling and a rock of offense,
And he who believes in Him will not be disappointed.”
14. Rom. 10:4, "For Christ is the end of the law for righteousness to everyone who believes."
15. Rom. 10:9-10, "that if you confess with your mouth Jesus as Lord, and believe in your heart that
God raised Him from the dead, you shall be saved; 10 for with the heart man believes, resulting in
righteousness, and with the mouth he confesses, resulting in salvation."
16. Gal. 2:16, "nevertheless knowing that a man is not justified by the works of the Law but through
faith in Christ Jesus, even we have believed in Christ Jesus, that we may be justified by faith in
Christ, and not by the works of the Law; since by the works of the Law shall no flesh be justified."
17. Gal.3:5-6, "Does He then, who provides you with the Spirit and works miracles among you, do it
by the works of the Law, or by hearing with faith? 6Even so Abraham believed God, and it was
reckoned to him as righteousness."
18. Gal. 3:8, "And the Scripture, foreseeing that God would justify the Gentiles by faith, preached the
gospel beforehand to Abraham, saying, "All the nations shall be blessed in you."
19. Gal. 3:14, "in order that in Christ Jesus the blessing of Abraham might come to the Gentiles, so
that we might receive the promise of the Spirit through faith."
20. Gal. 3:22, "But the Scripture has shut up all men under sin, that the promise by faith in Jesus
Christ might be given to those who believe."
21. Gal. 3:24, "Therefore the Law has become our tutor to lead us to Christ, that we may be justified
by faith."
22. Eph. 1:13, "In Him, you also, after listening to the message of truth, the gospel of your salvation
—having also believed, you were sealed in Him with the Holy Spirit of promise."
23. Eph. 2:8, "For by grace you have been saved through faith; and that not of yourselves, it is the
gift of God."
24. Phil. 3:9, "and may be found in Him, not having a righteousness of my own derived from the Law,
but that which is through faith in Christ, the righteousness which comes from God on the basis of
faith."
25. 1 Tim. 1:16, "And yet for this reason I found mercy, in order that in me as the foremost, Jesus
Christ might demonstrate His perfect patience, as an example for those who would believe in Him
for eternal life."

James 2:24, not by faith alone

The scriptures clearly teach that we are saved (justified) by faith in Christ and what He has done
on the cross. This faith alone saves us. However, we cannot stop here without addressing what
James says in James 2:24, "You see that a man is justified by works, and not by faith alone." There is
no contradiction. All you need to do is look at the context. James chapter 2 has 26 verses: Verses 1-
7 instruct us to not show favoritism. Verses 8-13 are comments on the Law. Verses 14-26 are about
the relationship between faith and works. James begins this section by using the example of someone
who says he has faith but has no works: verse 14. In other words, James is addressing the issue of a
dead faith, an empty faith that is nothing more than a verbal pronouncement that is empty of life and
action. He begins with the negative and demonstrates what an empty faith is (verses 15-17, words
without actions). Then he shows that that type of faith isn't much different from the faith of demons
(verse 19). Finally, he gives examples of living faith that is words followed by actions. He writes of
Abraham and Rahab as examples of people who demonstrated their faith by their deeds. In brief,
James is examining two kinds of faith: one that leads to godly works and one that does not. One is
true, and the other is false. One is dead, the other alive; hence, "Faith without works is dead," (James
2:20).
Also, notice that James actually quotes the same verse that Paul uses to support the teaching of
justification by faith in Rom. 4:3. James 2:23 says, "and the Scripture was fulfilled which says, ‘and
Abraham believed God, and it was reckoned to him as righteousness.'" If James was trying to teach a
contradictory doctrine of faith and works than the other New Testament writers, then he would not
have used Abraham as an example.

Conclusion

Justification is by faith. True faith results in regeneration of the sinner which, in turn, results in
good works. But it is not these works that earn our place with God nor keep it. Jesus accomplished
that on the cross. All that we need, we have in Jesus. All we need to do to be saved, to be justified, is
to truly believe in want God has done for us in Jesus on the cross. This true belief with justification
before God and regeneration in the new believer, results in good works.
Is Baptism Necessary for Salvation?
One of the most nagging questions in Christianity is whether or not baptism is necessary for
salvation. The answer is a simple, "No." But you might ask, "If the answer is no, then why are there
verses that say things like ‘. . .baptism that now saves you . . . ‘ (1 Pet. 3:21, NIV) and ‘ . . . Repent
and be baptized, every one of you, in the name of Jesus Christ for the forgiveness of your sins . . . "
(Acts 2:38, NIV)? This is an honest question and it needs a competent answer. But, before I tackle this
I need to lay a foundation of proper theology, then I'll address some of those verses that are
commonly used to support the idea that baptism is necessary for salvation.

God Works Covenantally

First, you need to understand that God works covenantally. A covenant is a pact or agreement
between two or more parties. The New Testament and Old Testaments are New and Old Covenants.
The word "testament" comes from the Latin testamentum which means covenant. So, the Bible is a
covenant document. If you don't understand covenant you cannot understand, in totality, the issue of
baptism because baptism is a covenant sign.
If you don't think that God works covenantally then look at Heb 13:20 which says, " May the God
of peace, who through the blood of the eternal covenant brought back from the dead our Lord Jesus,
that great Shepherd of the sheep" (NIV). The Eternal Covenant is the covenant between the Father
and the Son before the creation of the world, whereby the Father would give to the Son those whom
the Father had chosen. That is why Jesus says things like, " All that the Father gives me will come to
me, and whoever comes to me I will never drive away" (John 6:37, NIV). And, "And this is the will of
him who sent me, that I shall lose none of all that he has given me, but raise them up at the last day"
(John 6:39, NIV). And, "I pray for them. I am not praying for the world, but for those you have given
me, for they are yours," (John 17:9, NIV).
If you fail to understand that God works covenantally and that He uses signs as manifestations of
his covenants (rainbow, circumcision, communion, etc.) then you will not be able to understand where
baptism fits in God's covenant system.
Second, you need to know what baptism is. It is an outward representation of an inward reality.
For example, it represents the reality of the inward washing of Christ's blood upon the soul. That is
why it is used in different ways. It is said to represent the death of the person (Rom. 6:3-5), the union
of that person with Christ (Gal. 3:27), the cleansing of that person's sins (Acts 22:16), the
identification with the one "baptized into" as when the Israelites were baptized into Moses (1 Cor.
10:2), and being united in one church (1 Cor. 12:13). Also, baptism is one of the signs and seals of
the Covenant of Grace that was instituted by Jesus. It is in this sense a sacrament. A sacrament is a
visible manifestation of something spoken. It is also said to be a visible sign of an inward grace. For
example, the communion elements of bread and wine are called the sacrament of communion. When
we take communion we are partaking of the sacrament.
The Covenant of Grace is the covenant between God and Man where God promises to Man eternal
life. It is based upon the sacrifice of Jesus on the cross and the condition is faith in Jesus Christ. As the
Communion Supper replaced Passover, baptism, in like manner, replaces circumcision. "They
represent the same spiritual blessings that were symbolized by circumcision and Passover in the old
dispensation," (Berkhoff, Lewis, Systematic Theology, 1988, p. 620.).
Circumcision was the initiatory rite into the Abrahamic covenant; it did not save. A covenant is a
pact or agreement between two or more parties and that is exactly what the Abrahamic covenant was.
God said to Abraham, "I will establish my covenant as an everlasting covenant between me and you
and your descendants after you for the generations to come, to be your God and the God of your
descendants after you" (Genesis 17:7, NIV). God later instructed Abraham to circumcise not only
every adult male, but also 8 day old male infants as a sign of the covenant (Gen. 17:9-13). If the
children were not circumcised, they were not considered to be under the promissory Abrahamic
covenant. This is why Moses' wife circumcised her son and threw the foreskin at Moses' feet. (Ex.
4:24-25). She knew the importance of the covenant between God and her children. But at the same
time we must understand that circumcision did not guarantee salvation to all who received it. It was a
rite meant only for the people of God, who were born into the family of God (who were then the Jews).

An important question here is how is it possible for an infant to be entered into a covenant with
God. There could be a lot of answers given but the point remains: it was done; infants were entered
into a covenant relationship with God -- through their parents.
In the New Testament, circumcision is mentioned many times. But with respect to this topic it is
specifically mentioned in Col. 2:11-12: "In him you were also circumcised, in the putting off of the
sinful nature, not with a circumcision done by the hands of men but with the circumcision done by
Christ, having been buried with him in baptism and raised with him through your faith in the power of
God, who raised him from the dead," (NIV). In these verses, baptism and circumcision are related.
Baptism replaces the Old Testament circumcision because 1) there was a New Covenant in the
communion supper (Luke 22:20), and 2) in circumcision there was the shedding of blood but in
baptism no blood is shed. This is because the blood of Christ has been shed.
If you understand that baptism is a covenant sign, then you can see that it is a representation of
the reality of Christ circumcising our hearts (Rom. 2:29; Col. 2:11-12). It is our outward proclamation
of the inward spiritual blessing of regeneration. It comes after faith which is a gift of God (Rom. 13:3)
and the work of God (John 6:28).
Third, the Bible says that it is the gospel that saves. " By this gospel you are saved..." (1 Cor.
15:2). Also, Rom. 1:16 says, "I am not ashamed of the gospel, because it is the power of God for the
salvation of everyone who believes: first for the Jew, then for the Gentile."

What is the Gospel?

It is clearly the gospel that saves us. But what exactly is the gospel? That too is revealed to us in
the Bible. It is found in 1 Cor. 15:1-4: "Now, brothers, I want to remind you of the gospel I preached
to you, which you received and on which you have taken your stand. By this gospel you are saved, if
you hold firmly to the word I preached to you. Otherwise, you have believed in vain. For what I
received I passed on to you as of first importance: that Christ died for our sins according to the
Scriptures, that he was buried, that he was raised on the third day according to the Scriptures ." The
gospel is defined as the death, burial, and resurrection of Jesus for our sins. Baptism is not mentioned
here.
Paul said that he came to preach the gospel, not to baptize: "I am thankful that I did not baptize
any of you except Crispus and Gaius, so no one can say that you were baptized into my name. (Yes, I
also baptized the household of Stephanas; beyond that, I don't remember if I baptized anyone else.)
For Christ did not send me to baptize, but to preach the gospel..." (1 Cor. 1:14-17). If baptism is
necessary for salvation then why did Paul downplay it and even exclude it from the description of what
is required for salvation? It is because baptism isn't necessary for salvation.
Additionally, in Acts, Peter was preaching the gospel, people got saved, and then they were
baptized. Acts 10:44-46 says, "While Peter was still speaking these words, the Holy Spirit came on all
who heard the message. The circumcised believers who had come with Peter were astonished that the
gift of the Holy Spirit had been poured out even on the Gentiles. For they heard them speaking in
tongues and praising God. Then Peter said, ‘Can anyone keep these people from being baptized with
water? They have received the Holy Spirit just as we have.' So he ordered that they be baptized in the
name of Jesus Christ. Then they asked Peter to stay with them for a few days" (NIV). These people
were saved. The gift of the Holy Spirit was on the Gentiles and they were speaking in tongues. This is
significant because tongues is a gift given to believers, see 1 Cor. 14:1-5. Also, unbelievers don't
praise God. They can't because praise to the true God is a deep spiritual matter that is foreign to the
unsaved (1 Cor. 2:14). Therefore, the ones in Acts 10 who are speaking in tongues and praising God
are definitely saved and they are saved before they are baptized. This simply isn't an exception. It is a
reality.
Let's Suppose...

Another way of making this clear is to use an illustration. Let's suppose that a person, under the
conviction of the Holy Spirit (John 16:8), believed in Jesus as his savior (Rom. 10:9-10; Titus 2:13),
and has received Christ (John 1:12) as Savior. Is that person saved? Of course he is. Let's further
suppose that this person confesses his sinfulness, cries out in repentance to the Lord, and receives
Jesus as Savior and then walks across the street to get baptized at a local church. In the middle of the
road he gets hit by a car and is killed. Does he go to heaven or hell? If he goes to heaven then
baptism isn't necessary for salvation. If He goes to hell, then trusting in Jesus, by faith, isn't enough
for salvation. Doesn't that go against the Scriptures that say that salvation is a free gift (Rom. 6:23)
received by faith (Eph. 2:8-9)?
Saying that baptism is necessary for salvation is dangerous because it is saying that there is
something we must do to complete salvation. That is wrong! See Gal. 2:21; 5:4.
All right, so this sounds reasonable. But still, what about those verses that seem to say that
baptism is part of salvation? I'll address those now. But, because this subject can become quite
lengthy, in fact, sufficient for a book in itself, I'll only address a few verses and then only briefly.

Baptism Verses

John 3:5, "Jesus answered, ‘I tell you the truth, no one can enter the kingdom of God unless he is
born of water and the Spirit.'"
Some say that water here means baptism. But that is unlikely since Christian baptism hadn't yet
been instituted. If this verse did mean baptism, then the only kind that it could have been at that point
was the baptism of repentance administered by John the Baptist (Mark 1:4). If that is so, then baptism
isn't necessary for salvation because the baptism of repentance is no longer practiced.
It is my opinion that the water spoken of here means the water of the womb referring to the
natural birth process. Jesus said in verse three that Nicodemus needed to be born "again." This meant
that he had been born once--through his mother. Nicodemus responds with a statement about how he
can't enter again into his mother's womb to be born. Then Jesus says that he must be born of water
and the Spirit. Then in verse 6 He says that "flesh gives birth to flesh, but the Spirit gives birth to
spirit.." The context seems to be discussing the contrast between the natural and the spiritual birth.
Water, therefore, could easily be interpreted there to mean the natural birth process.
I would like to add that there are scholars who agree with the position and some who do not.
Some believe that the water refers to the Word of God, the Bible, and others claim it means the Holy
Spirit. You decide for yourself.

Acts 2:38, "Peter replied, ‘Repent and be baptized, every one of you, in the name of Jesus Christ for
the forgiveness of your sins. And you will receive the gift of the Holy Spirit.‘"
This verse is a tough one. It seems to say that baptism is part of salvation. But we know, from
other scriptures that it isn't, lest there be a contradiction. What is going on here is simply that
repentance and forgiveness of sins are connected. In the Greek, "repent" is in the plural and so is
"your" of "your sins." They are meant to be understood as being related to each other. It is like saying,
"All of you repent, each of you get baptized, and all of you will receive forgiveness." Repentance is a
mark of salvation because it is granted by God (2 Tim. 2:25) and is given to believers only. In this
context, only the regenerated, repentant person is to be baptized. Baptism is the manifestation of the
repentance, that gift from God, that is the sign of the circumcised heart. That is why it says, repent
and get baptized.

1 Pet. 3:21, "and this water symbolizes baptism that now saves you also -- not the removal of dirt
from the body but the pledge of a good conscience toward God. It saves you by the resurrection of
Jesus Christ."
This is the only verse that says that baptism saves. But, the NIV translation of the verse is
unfortunate. A better translation is found in the NASB which says, "and corresponding to that, baptism
now saves you." The key word in this section is the Greek “antitupon.” It means "copy," "type,"
corresponding to," "a thing resembling another," "its counterpart," etc. Baptism is a representation, a
copy, a type of something else. The question is "Of what is it a type?", or "Baptism corresponds to

what?” The answer is found in the previous verse, verse 20: "who once were disobedient, when the
patience of God kept waiting in the days of Noah, during the construction of the ark, in which a few,
that is, eight persons, were brought safely through the water. 21And corresponding to that, baptism
now saves you" (NASB).
What does baptism correspond to? Is it the flood? Or, is it the ark? What was it that saved Noah
and his family? Was it the water or the ark? Obviously, it was the Ark. Noah built and entered the ark
by faith and was saved (Heb. 11:7). The flood waters destroyed the ungodly. Peter, when referring to
the flood waters, refers to them as the means of destruction of the ungodly (2 Pet. 2:5; 3:6). It was
the Ark that saved. Noah entered the ark by faith. Baptism here, in my opinion, refers to the Ark, not
the waters. That is why the rest of the verse says, "not the removal of dirt from the body but the
pledge of a good conscience toward God" which is consistent with what Paul said in Col. 2:11-12 where
He equates baptism with being circumcised of heart.

Acts 22:16, "And now what are you waiting for? Get up, be baptized and wash your sins away, calling
on his name."
Is the washing away of sins done by baptism, the representation of the circumcised heart (Col.
2:11-12) which means you are already saved, or is it by the blood of Christ (Heb. 9:14; Rom. 5:9;
Eph. 1:7)? Obviously it is the blood of Jesus and the washing here refers to the calling on Jesus' name.

Rom. 6:4, "We were therefore buried with him through baptism into death in order that, just as Christ
was raised from the dead through the glory of the Father, we too may live a new life."
Because the believer is so closely united to Christ it is said that the symbol of baptism is our
death, burial, and resurrection. Obviously we did not die--unless, of course, it is a figurative usage.

Titus 3:5, "he saved us, not because of righteous things we had done, but because of his mercy. He
saved us through the washing of rebirth and renewal by the Holy Spirit."
The washing of rebirth can only be that washing of the blood of Christ that cleanses us. It is not
the symbol that saves, but the reality. The reality is the blood of Christ.

Gal. 3:27, "for all of you who were baptized into Christ have clothed yourselves with Christ."
This is speaking of the believer's union with Christ. It is identification with, a joining to, a
proclamation of loyalty to, etc. In 1 Cor. 10:2 the Israelites were baptized into Moses. That means
they were closely identified with him and his purpose. The same thing is meant here.

Conclusion

Baptism is not necessary for salvation. It is the initiatory sign and seal into the covenant of grace.
As circumcision referred to the cutting away of sin and to a change of heart (Deut. 10:16; 30:6; Jer.
4:4; 9:25,26; Ez. 44:7,9) baptism refers to the washing away of sin (Acts 2:38; 1 Pet. 3:21; Tit. 3:5)
and to spiritual renewal (Rom. 6:4; Col. 2:11-12). The circumcision of the heart is signified by the
circumcision of the flesh, that is, baptism (Col. 2:11-12).
One last thought: If someone maintains that baptism is necessary for salvation, is he adding a
work, his own, to the finished work of Christ? If the answer is yes, then that person would be in
terrible risk of not being saved. If the answer is no, then why is baptism maintained as being
necessary the same way as the Jews maintained that works were necessary?
Baptism and Mark 16:16

"He who believes and is baptized will be saved; but he who does not believe will be
condemned," (Mark 16:16).

This verse is frequently used by baptismal regenerationists to show that baptism is necessary for
salvation. It says he who believes and is baptized will be saved. Therefore, they conclude that
baptism is a necessary part of becoming saved. But, does this verse prove that baptism is necessary
for salvation? Not at all.
Mark 16:16 does not say that baptism is a requirement for salvation. Let me show you why. I
could easily say that he who believes and goes to church will be saved. That is true. But it is belief
that saves, not belief and going to church. Likewise, if you believe and read your Bible, you'll be
saved. But it isn't reading your Bible that saves you. Rather, belief in Christ, in His sacrifice, is what
saves. As I've stated in other papers on this subject, there are numerous verses that clearly
demonstrate that justification is by faith (Rom. 5:1; Eph. 2:8; Phil. 3:9; etc.). Belief in what God has
done, not what man can do, is what results in salvation. Baptism is simply a public demonstration of
the inner work of regeneration. This is why the rest of the verse says, "...but he who does not to
believe will be condemned." Mark 16:16 focuses on the issue of belief, not baptism.

A textual issue with Mark 16:9-20

What I will share here may not be very popular with some readers. Therefore, I need to say
upfront that I believe in the absolute inspiration and authority of the Bible. It is the word of God and
what it says is authoritative. However, the simple fact is that there are textual variations within the
biblical manuscripts. The originals are what are inspired, not the copies. We have copies of inspired
documents. These copies are not perfect, but they are very close to it.
Again, I am not saying the Bible is untrustworthy. It is 98.5% textually pure. The remaining 1.5%
of textual variation are almost entirely of insignificant spelling errors and minor word omissions or
additions that do not change the meaning of the text. However, Mark 16:9-20 is a significant textual
variant. Many scholars, Christian scholars, consider the ending of Mark to lack authenticity. Please
consider the following evidence.

3. Mark 16:9-20 doesn't appear in many of the oldest ancient manuscripts.

A. “The last twelve verses of Mark (16:9-20) are lacking in the two earliest parchment codices,
B and Aleph, in the Old Latin manuscript k,, the Sinaitic Syriac, many manuscripts of the Old
Armenian version, the Adysh and Opiza manuscripts of the Old Georgian version, and a
number of manuscripts of the Ethiopic version. Clement of Alexandria, Origen, and
Ammonius show no knowledge of the existence of these verses; other Church Fathers state
that the section is absent from Greek copies of Mark known to them (e.g. Jerome, Epist.
cxx. 3, ad hedibiam,)...The original form of the Eusebian sections makes no provision for
numbering sections after 16:8. Not a few manuscripts which contain the passage have
scholia stating that older Greek copies lack it (so, for example, MSS. 1, 20,22, &c.), and in
other witnesses the passage is marked with asterisks or obeli, the conventional sigla used by
scribes to indicate a spurious addition to a literary document." 121

121
"The Text of the New Testament," by Bruce Metzger (Professor of New Testament Language and Literature,
Princeton Theological Seminary), 2nd ed., Oxford University Press, New York, 1968, p. 226.
4. There are other endings to Mark.
A. Another ending is found in L, Psi, 099, 0112, and minuscules 274mg 579, k, Syrh and more
is as follows:
i. "But they reported briefly to Peter and those with him all that had been told. And after
this Jesus himself sent out by means of them, from east to west, the sacred and
imperishable proclamation of eternal salvation."
5. Apparent, theological error.
A. Mark 16:12 says, "And after that, He appeared in a different form to two of them, while they
were walking along on their way to the country." This verse may be problematic. Jesus rose
in the same body that he died in (John 2:19), though it was a glorified body. this is
problematic because it suggests "a different form." Jesus did not appear in a different form.
He appeared in the same body he rose in.
6. Evidence against the Mark authorship.
A. There are 17 non-marcan words used in a non-marcan sense in these verses.

This information about the ending of Mark is not intended to cast doubt upon God's word. But
the fact is that the ending is under a large cloud of doubt as to its authenticity. I would not use it
as a defense for baptismal regeneration.
Baptism and Roman 6:3-5

Romans 6:3-5 is often used as a proof text for the claim that baptism is essential for salvation. It
is a strong comparison between our baptism and Christ's death, burial, and resurrection. On the
surface, one could conclude that from these verses, that baptism is part of salvation.

"Or do you not know that all of us who have been baptized into Christ Jesus have been
baptized into His death? 4Therefore we have been buried with Him through baptism into death,
in order that as Christ was raised from the dead through the glory of the Father, so we too
might walk in newness of life. 5For if we have become united with Him in the likeness of His
death, certainly we shall be also in the likeness of His resurrection,"

Is this section of scripture teaching us that baptism is necessary for salvation? No, it is not. First,
we know from the rest of scripture that salvation is by faith, not by faith and something we do Rom.
3:28-30. Second, we can see from other scriptures that baptism follows faith. Take a look at Acts
16:30-33 where the Jailer specifically asks what he must do to be saved and where baptism fits in.

"and after he brought them out, he said, "Sirs, what must I do to be saved?" 31And they said,
"Believe in the Lord Jesus, and you shall be saved, you and your household." 32And they spoke
the word of the Lord to him together with all who were in his house. 33And he took them that
very hour of the night and washed their wounds, and immediately he was baptized, he and all
his household," (Acts 16:30-33, NASB).

If baptism were part of salvation, then Paul should have said, "Believe and be baptized and you will
be saved." But, he did not. Also, consider Acts 10:44-46.

"While Peter was still speaking these words, the Holy Spirit came on all who heard the
message. The circumcised believers who had come with Peter were astonished that the gift of
the Holy Spirit had been poured out even on the Gentiles. For they heard them speaking in
tongues and praising God. Then Peter said, ‘Can anyone keep these people from being baptized
with water? They have received the Holy Spirit just as we have.' So he ordered that they be
baptized in the name of Jesus Christ. Then they asked Peter to stay with them for a few days,"
(NIV).

These people were saved. The gift of the Holy Spirit was on the Gentiles and they were speaking in
tongues. This is significant because tongues is a gift given to believers, see 1 Cor. 14:1-5. Also,
unbelievers don't praise God. They can't because praise to the true God is a deep spiritual matter that
is foreign to the unsaved (1 Cor. 2:14). Therefore, the ones in Acts 10:44-46 who are speaking in
tongues and praising God are definitely saved and they are saved before they are baptized. This isn't
an exception. It is a reality. This proves that baptism is not necessary for salvation.
What is Romans 6:3-5 saying?

"Or do you not know that all of us who have been baptized into Christ Jesus have been
baptized into His death? 4Therefore we have been buried with Him through baptism into death,
in order that as Christ was raised from the dead through the glory of the Father, so we too
might walk in newness of life. 5For if we have become united with Him in the likeness of His
death, certainly we shall be also in the likeness of His resurrection,"

The phrase "baptized into" occurs five times in the NT in four verses as found in the KJV and the
NASB..

9. Rom. 6:3, "Or do you not know that all of us who have been baptized into Christ
Jesus have been baptized into His death?"
10. 1 Cor. 10:2, "and all were baptized into Moses in the cloud and in the sea."
11. 1 Cor. 12:13, "For by one Spirit we were all baptized into one body, whether
Jews or Greeks, whether slaves or free, and we were all made to drink of one Spirit."
12. Gal. 3:27, "For all of you who were baptized into Christ have clothed yourselves
with Christ."

To be baptized "into Christ," "into His death," "into Moses," and "into one body" is to be publicly
identified with the thing you are being baptized into. The focus is not the baptism itself, but on the
thing the baptism represents. In the case of Rom. 6:3-5, being baptized into Christ is a public
identification with Christ's death, burial, and resurrection which is said to be the gospel that saves in 1
Cor. 15:1-4. Baptism then is a public statement proclaiming that the person is trusting in the sacrifice
of Christ.
Baptism by immersion is a perfect symbol for this work of Christ with which the Christian is
identifying himself. As Christ died and was raised to a new life, so to the Christian, in Christ, is said to
have died (Rom. 6:11; Col. 3:3) and has a new life. This new life of regeneration is by faith, the
internal work. Baptism is the external work of identification with Christ. This is why the reference to
baptism in the Bible is dealing more with "our union and identification with Christ than to our water
baptism."122

• Baptism is being identified as a disciple (Matt. 28:18-9).


• Baptism may be compared to a new birth (John 3:5).
• Baptism is compared to Jesus' death and resurrection (Rom. 6:3-5).
• Baptism is compared to Israel's Exodus and passing through the Red Sea (1 Cor. 10:2).
• Baptism is compared to Noah's escaping the flood waters by entering the ark (1 Pet. 3:21).

In each of the references above, baptism is identification with something. When people were
baptized into John the Baptist's baptism of repentance, it wasn't the baptism that granted them
repentance or made repentance real. Repentance is something that happens internally and is the work
of God (2 Tim. 2:25). To participate in John's baptism was to publicly proclaim that the person being
baptized was accepting John's message or repentance. Hence, it was called a baptism of repentance.
It wasn't the baptism that brought repentance; rather, baptism was the result of repentance. The
person had to first decide to repent, and then become baptized as a proclamation of his decision.
Likewise, the Christian must first decide to repent, to receive Christ (John 1:12), to rely on the
sacrifice of Christ, by faith, and then participate in the public proclamation of identifying with Christ's
work.

It is identification with the death, burial, and resurrection of Christ that baptism represents. Jesus'
shed blood is what cleanses us from our sins (Heb. 9:22), not being washed with water. It is Christ's
death that is the payment for sin. Jesus' burial is the proof that He, in fact, died. Jesus' resurrection
is the proof of God the Father's acceptance of the sacrifice of Christ and that death is conquered.
Again, for a Christian to be baptized is to make a public proclamation that he is trusting in Christ's

122
Enhanced Strong’s Lexicon, (Oak Harbor, WA: Logos Research Systems, Inc.) 1995.
work, that he is naming himself with Christ and trusting what Christ has done. This is why it says in
Rom. 6:11, "Even so consider yourselves to be dead to sin, but alive to God in Christ Jesus," (NASB).
Why? Because "I have been crucified with Christ; and it is no longer I who live, but Christ lives in me;
and the life which I now live in the flesh I live by faith in the Son of God, who loved me, and delivered
Himself up for me," (Gal. 2:20). It is on the cross that Jesus paid for our sins, not in His baptism and
not in our baptism. It is our identification with Him, being counted "in Christ" that allows us to say we
have been crucified with Christ so that we can say we are dead to sin. We are not dead to sin by our
baptism. Rather, we are dead to sin, by faith, in what Jesus did in His sacrifice.

Conclusion

Romans 6:3-5 speaks to us of Christ's work and our public identification with it. In that ancient
world of religious plurality in Roman gods, in the strict Laws of the Jewish system, and in the gods of
different cultures, to be baptized was to make a bold statement of commitment to Christ as the risen
Lord. It was not the water that saved, but faith in Christ and His work.
Baptism and Gal. 3:27

Gal. 3:27 is often used by the baptismal regenerationists to support the idea that you must be
baptized to be saved. They maintain that baptism is the place where a person "puts on Christ," where
he is "clothed with Christ" and that it means that baptism saves. They teach that being immersed in
the baptismal water is the place and time of deliverance from sins. This is simply not true.
Gal. 3:27 cannot be understood alone. It must be examined in context.

"Therefore the Law has become our tutor to lead us to Christ, that we may be justified by faith.
25
But now that faith has come, we are no longer under a tutor. 26For you are all sons of God
through faith in Christ Jesus. 27For all of you who were baptized into Christ have clothed
yourselves with Christ. 28There is neither Jew nor Greek, there is neither slave nor free man,
there is neither male nor female; for you are all one in Christ Jesus. 29And if you belong to
Christ, then you are Abraham’s offspring, heirs according to promise," (Gal. 3:24-29).

In Roman society, children were often committed to the care of trusted slaves. This would often
happen when the child was between six or seven, and it would last until puberty. "These slaves were
severe disciplinarians and were charged with guarding the children from the evils of society and giving
them moral training. This was like the Law’s function until Christ came and people could be justified by
faith in Him." 123 The Law was a harsh master to the Jews. It was very difficult to keep. This is why
the Law points to Christ by showing us our inability to keep the Law and by showing us that we must
rely on faith instead. That is why justification is by faith (vv. 24-26), because we cannot attain
justification by Law (Rom. 3:28-30; Phil. 3:9).

"For all of you who were baptized into Christ have clothed yourselves with Christ," (Gal. 3:27).

In Roman society when a child who had been under the care of a tutor and reached a matured
enough age, he was given a special robe, or toga. It was symbolic of his full rights in the family. 124
Therefore, being "clothed with Christ" is a phrase meaning that the Christian moved out from the Law
and into the gospel of grace and can enjoy full acceptance before God the Father. It is not saying that
baptism is what saves us from our sins.

123
Walvoord, John F., and Zuck, Roy B., The Bible Knowledge Commentary, (Wheaton, Illinois: Scripture Press
Publications, Inc.) 1983, 1985, on Gal. 3:24.
124
ibid, on Gal. 3:27.
Baptism and 1 Pet. 3:21
1 Pet. 3:21 says, "and this water symbolizes baptism that now saves you also -- not the removal
of dirt from the body but the pledge of a good conscience toward God. It saves you by the resurrection
of Jesus Christ." This is the only verse that says that baptism saves. Is it teaching that we must be
baptized to be saved? No. But, but to rightly understand it, we need to look at its context.

"For Christ also died for sins once for all, the just for the unjust, in order that He might bring us
to God, having been put to death in the flesh, but made alive in the spirit; 19 in which also He
went and made proclamation to the spirits now in prison, 20 who once were disobedient, when
the patience of God kept waiting in the days of Noah, during the construction of the ark, in
which a few, that is, eight persons, were brought safely through the water. 21 And
corresponding to that, baptism now saves you—not the removal of dirt from the flesh, but an
appeal to God for a good conscience — through the resurrection of Jesus Christ, 22 who is at
the right hand of God, having gone into heaven, after angels and authorities and powers had
been subjected to Him," (1 Pet. 3:18-22, NASB).

The above translation in verse 21 from the NASB is a good translation. "And corresponding to
that, baptism now saves you." The key word in this section is the Greek antitupon. It means "copy,"
"type," "corresponding to," "a thing resembling another," "its counterpart," etc. It is what the NIV
translates as "symbolizes," the NASB as "corresponding to that," and the KJV as "like figure."
Baptism, then, is a representation, a copy, a type of something else. The question is "Of what is it a
type?", or "baptism corresponds to what?"
If we look at the context, an interesting possibility arises, though I will admit, not the favored
interpretation among scholars. What does baptism correspond to? Is it the flood? Or, is it the ark?
What was it that saved Noah and his family, the flood or the ark? Obviously, it was the Ark. Noah built
and entered the ark by faith and he was saved (Heb. 11:7). The flood waters destroyed the ungodly.
Also, Peter consistently refers to the flood waters as the means of destruction of the ungodly ( 2 Pet.
2:5; 3:6), not the salvation of Noah and his family. Rather, it was the Ark that saved, the ark that
Noah entered faith. It may very well be that baptism refers to the Ark, not the waters. That is why
the rest of the verse says, "not the removal of dirt from the body but the pledge of a good conscience
toward God" which is consistent with what Paul said in Col. 2:11-12 where He equates baptism with
being circumcised of heart.
The problem with this interpretation is that it doesn't seem to fit the "water for water typology." It
would seem more natural to equate the water of baptism with the water of the flood. Furthermore, if
we were to look at the flood waters as the thing that removed evil from the land, we could say that
"correspondingly," the waters of baptism remove removes the sin from our hearts. Though this
reading seems a bit more natural, it too has problems.
The water of baptism is not what saves us, the sacrifice of Christ does which we receive by faith.
We read numerous verses about justification by faith (Rom. 5:1), salvation by faith (Eph. 2:8), etc.,
not justification "by faith and baptism," or salvation "by faith and baptism." 125 The fact is that salvation
is received by faith. Peter, not wanting to declare that baptism itself is what saves us, quickly adds,
"not the removal of dirt from the flesh, but an appeal to God for a good conscience." Water baptism,
then, must accompany the work of the Holy Spirit in the person. Peter's explanatory comment shows
us that the act of physical baptism is not what saves, but the "baptism of appeal to God." This appeal
to God is by faith the same as Noah's faith in God led him to build the Ark, enter it, and remain in it.
It was the Ark that saved Noah, not the flood waters.
The flood was for Noah a type of baptism even as the passage through the Red Sea was a type of
baptism for the Israelites.

Mark 16:16 says, "He who believes and is baptized will be saved; but he who does not believe will be
125

condemned." Please see the article on Baptism and Mark 16:16 for an examination of this verse.
"I want you to know, brethren, that our fathers were all under the cloud, and all passed
through the sea, 2and all were baptized into Moses in the cloud and in the sea, 3and all ate the
same supernatural food 4and all drank the same supernatural drink. For they drank from the
supernatural Rock which followed them, and the Rock was Christ," (1 Cor. 10:1-4).

The "baptisms" of both Noah and the Israelites served as types of a transition; that is, they moved
people from the old world to the new, from the old covenant to the new covenant. It is not the water
that saves, but the spiritual thing associated with that water that saves. For Noah it was faith in God.
For Moses it too was faith in God.
But some may say that the work of the Holy Spirit and the act of baptism are simultaneous, that
the Holy Spirit works in and through baptism to bring regeneration. But this cannot be the case since
the Bible tells us that salvation is by faith (Rom. 5:1; Eph. 2:8). Besides, we have a clear instance in
scripture where people are saved before their baptism.

Acts 10:44-48

"While Peter was still speaking these words, the Holy Spirit fell upon all those who were
listening to the message. 45And all the circumcised believers who had come with Peter were
amazed, because the gift of the Holy Spirit had been poured out upon the Gentiles also. 46For
they were hearing them speaking with tongues and exalting God. Then Peter answered,
47
"Surely no one can refuse the water for these to be baptized who have received the Holy
Spirit just as we did, can he?" 48And he ordered them to be baptized in the name of Jesus
Christ. Then they asked him to stay on for a few days," (Acts 10:44-48).

In these verses we see that Peter had been preaching the gospel and the Holy Spirit fell upon the
listeners. In verse 45 we read that "the gift of the Holy Spirit had been poured out upon the Gentiles
also." This gift manifested itself in speaking in tongues. This is significant because tongues is a sign-
gift given to believers, see 1 Cor. 14:1-5. Also, verse 46 says they were "exalting God." Unbelievers
don't praise God. They can't because praise to the true God is a deep spiritual matter that is foreign to
the unsaved (1 Cor. 2:14). Therefore, the ones in Acts 10 who are speaking in tongues and praising
God are definitely saved because they are moving in the Holy Spirit, speaking in tongues, and
glorifying God. It is the Holy Spirit who gives charismatic spiritual gifts to the church ( 1 Cor. 12:27-
28), not to unbelievers. Now, please notice that it was after this movement of the Holy Spirit that the
believers are baptized. If baptism is necessary for salvation, then how is it that the people were
speaking in tongues and exalting God before they were baptized?
If you were to say that the Holy Spirit was simply working upon and through those not yet saved,
then remember that tongues and praise to God are for the church, not the unbelievers. The church
consists of people who are saved, not unsaved. If they were not saved until they were baptized, then
they were not in the body of Christ and would not have moved in the charismatic gifts. Therefore,
they were regenerate before they were baptized. This simply isn't an exception. It is a reality.

Conclusion

1 Pet. 3:21 is not teaching us that baptism is what saves us. Rather, it is showing us that the
water symbolizes a spiritual cleansing through the power of the Holy Spirit gained through Christ’s
victory over death. It is the person's appeal to God that saves the soul, not the washing of water upon
the body.
Baptism and John 3:5

"Truly, truly, I say to you, unless one is born again, he cannot see the kingdom of God."
4
Nicodemus *said to Him, "How can a man be born when he is old? He cannot enter a second
time into his mother's womb and be born, can he?" 5Jesus answered, "Truly, truly, I say to you,
unless one is born of water and the Spirit, he cannot enter into the kingdom of God. 6"That
which is born of the flesh is flesh, and that which is born of the Spirit is spirit. 7"Do not marvel
that I said to you, 'You must be born again.' 8"The wind blows where it wishes and you hear the
sound of it, but do not know where it comes from and where it is going; so is everyone who is
born of the Spirit," (John 3:3-8).

There are five basic interpretations to this section of scripture in reference to water.

1. The water refers to the natural birth.


2. The water refers to the Word of God.
3. The water refers to the Holy Spirit.
4. The water refers to the ministry of John the Baptist.
5. The water refers to the water of baptism as a requirement for salvation.

The first option looks to the context of Jesus' words dealing with being born "again" ( 3:3).
Nicodemus responds by mentioning the experience of being born from the womb (v. 4). Jesus then
speaks of water and the Spirit and then says, "That which is born of the flesh is flesh, and that which
is born of the Spirit is spirit" (3:6). However, this view is not the most commonly held view.
The second option holds that the water is referring to the Word of God. Eph. 5:26 says, "that He
might sanctify her, having cleansed her by the washing of water with the word." Some believe that the
washing of water is done by means of the Word of God.
The third view says that the water refers to the Holy Spirit. Perhaps Nicodemus was reminded of
Ezek. 36:25-27, "Then I will sprinkle clean water on you, and you will be clean; I will cleanse you from
all your filthiness and from all your idols. 26"Moreover, I will give you a new heart and put a new spirit
within you; and I will remove the heart of stone from your flesh and give you a heart of flesh. 27"And I
will put My Spirit within you and cause you to walk in My statutes, and you will be careful to observe
My ordinances." Certainly, Jesus' own words are applicable here when He says in John 7:37-39, "Now
on the last day, the great day of the feast, Jesus stood and cried out, saying, "If any man is thirsty, let
him come to Me and drink. 38"He who believes in Me, as the Scripture said, 'From his innermost being
shall flow rivers of living water.'" 39But this He spoke of the Spirit, whom those who believed in Him
were to receive; for the Spirit was not yet given, because Jesus was not yet glorified."
The fourth view holds that the water is in reference to the water baptism of repentance taught by
John the Baptist. Matt. 3:1-6 describes John's ministry in the desert, his teaching about repentance,
and baptizing people into that repentance. Contextually, the first chapter of John mentions John the
Baptist in verses 6-8 and 19-36. Certainly, contextually, John and his ministry is in view here. If this is
the case, then Jesus would have been speaking of the "baptism" (the initiatory ordinance) of
repentance preached by John the Baptist.
The fifth view is the one held by the International Church of Christ; namely, that the water is
referring to baptism and that it is essential to salvation.

Does John 3:5 teach that baptism is essential to salvation?

As you can see, there are different interpretations to John 3:5. But, to simply say that John 3:5
does not teach the necessity of baptism isn't enough. Some sort of proof must be offered. The proof is
found in God's word that has no contradictions. Clearly, salvation is by faith. For example, Rom. 5:1
states that we are justified (declared righteous) by faith. It does not say faith and baptism. If baptism
were part of salvation, then it would say we were justified by faith and baptism. But it does not. If
justification is by faith, then it is by faith. Baptism is not faith. It is a ceremony. Furthermore, please
consider the following verses when declare how we are saved.

1. Rom. 3:22, "even the righteousness of God through faith in Jesus Christ for all those who
believe; for there is no distinction."
2. Rom. 3:26, "for the demonstration, I say, of His righteousness at the present time, that He
might be just and the justifier of the one who has faith in Jesus."
3. Rom. 3:28, "For we maintain that a man is justified by faith apart from works of the Law."
4. Rom. 4:5, "But to the one who does not work, but believes in Him who justifies the ungodly,
his faith is reckoned as righteousness."
5. Rom. 5:1, "Therefore having been justified by faith, we have peace with God through our
Lord Jesus Christ,"
6. Gal. 3:8, "And the Scripture, foreseeing that God would justify the Gentiles by faith,
preached the gospel beforehand to Abraham."
7. Gal. 3:24, "Therefore the Law has become our tutor to lead us to Christ, that we may be
justified by faith."
8. Eph. 2:8, "For by grace you have been saved through faith; and that not of yourselves, it is
the gift of God."

Additionally, Paul tells us that the gospel is what saves us and that the gospel is the death, burial,
and resurrection of Jesus, (1 Cor. 15:1-4). Baptism is not included in the description of the gospel.
This explains why said he came to preach the gospel, not to baptize: "I am thankful that I did not
baptize any of you except Crispus and Gaius, so no one can say that you were baptized into my name.
(Yes, I also baptized the household of Stephanas; beyond that, I don't remember if I baptized anyone
else.) For Christ did not send me to baptize, but to preach the gospel..." (1 Cor. 1:14-174). If baptism
is necessary for salvation then why did Paul downplay it and even exclude it from the description of
what is required for salvation? It is because baptism isn't necessary for salvation. Therefore, John 3:5
must be interpreted in a manner consistent with the rest of scripture.
Another way of making this clear is to use an illustration. Let's suppose that a person, under the
conviction of the Holy Spirit (John 16:8), believed in Jesus as his savior (Rom. 10:9-10; Titus 2:13),
and has received Christ (John 1:12) as Savior. Is that person saved? Of course he is. Let's further
suppose that this person who confesses his sinfulness, cries out in repentance to the Lord, and
receives Jesus as Savior, then walks across the street to get baptized at a local church. In the middle
of the road he gets hit by a car and is killed. Does he go to heaven or hell? If he goes to heaven then
baptism isn't necessary for salvation. If He goes to hell, then trusting in Jesus, by faith, isn't enough
for salvation. Doesn't that go against the Scriptures that say that salvation is a free gift ( Rom. 6:23)
received by faith (Eph. 2:8-9)? Yes it does. Baptism is not necessary for salvation and John 3:5 cannot
teach that it is.
Baptism and Acts 2:38
Acts 2:38 is one of the more controversy verses in the Bible regarding baptism and whether or not
it is the requirement for salvation. On the surface it seems to support it. But upon closer examination,
we will see that it does not teach baptismal regeneration: that baptism saves.
First of all, rarely is doctrine ever made from a single verse. We need to look at all of what God's
words says about a subject in order to accurately understand what it teaches. I will briefly tackle of
this verse in the following manner.

• Examination of the verse's syntax, grammar and structure.


• Examine other verses dealing with the forgiveness of sins.
• Examine the verse in its covenant context.

Grammar and Structure of Acts 2:38

In Acts 2:38 the main verb is metanoesate (change mind), the aorist direct imperative (a
command) of metanoeo which means to repent (change mind). This refers to that initial repentance of
the sinner unto salvation. The verb translated "be baptized" is in the indirect passive imperative (a
command to receive; hence, passive voice in Greek 126) of baptizo, which does not give it the same
direct command implied in "repent." The preposition "for" in the phrase "for the remission of sins" in
Greek is "eis," unto or into, and it is in the accusative case (direct object). It can mean "for the
purpose of identifying you with the remission of sins." It is the same preposition we find in 1 Cor. 10:2
in the phrase "and were baptized unto Moses." Note that both contexts are dealing with baptism and
identification. These people were baptized or spiritually identifying themselves with the purposes and
vision of Moses. Repentance, therefore, is presented as identifying an individual with the remission of
his sins, even as baptism following repentance provides an external identification visible by others.
Repentance is something that concerns an individual and God while baptism involves others. That is
why baptistheto (let be immersed) is in the passive voice indicating that one does not baptize himself,
but is baptized by another usually in the presence of others. Repentance, however, is an act taking
place within a person's heart as the Holy Spirit moves in the sinner.
But, all this Greek stuff may be confusing. Let me break it down. All people are commanded to
repent for their sins. This is
what believers have already done by
becoming Christians. Baptism,
then, is the outward identification
with being a Christian for those
who have already repented.
Also, as the Israelites were
"baptized into Moses," (1 Cor.
10:2), so too, Christians are
baptized into Jesus. That is, they are
identifying themselves, publicly, with Christ. Likewise, in Rom. 6:1-5 where baptism is related to
death, burial, and resurrection, it is again and identification with Christ in His death, burial, and
resurrection. That is why it is said of Christians that we have died to sin ( Rom. 6:2, 11; Gal. 2:20;
Col. 2:20; Col. 3:3; 1 Pet. 2:24).
This verse is not demonstrating that baptism is essential for salvation, but that baptism is the thing
which we receive, in order to publicly identify ourselves completely and totally with Christ as a
manifestation of the inward work God has done within us.

126
Active voice is "I hit the ball." Passive voice is "The ball hit me." Middle voice is "I was hit by the ball." In
active voice, "I" performed the action. In passive voice, "I" received the action. In middle voice, "I" did
something to myself.
Other verses dealing with salvation

Justification is the work of God where the righteousness of Jesus is reckoned to the sinner so the
sinner is declared, by God, as being righteous under the Law (Rom. 4:3; 5:1,9; Gal. 2:16; 3:11). This
righteousness is not earned or retained by any effort of the saved. Justification is an instantaneous
occurrence with the result being eternal life. It is based completely and solely upon Jesus' sacrifice on
the cross (1 Pet. 2:24) and is received by faith alone (Rom. 4:5; 5:1; Eph. 2:8-9). No works are
necessary whatsoever to obtain justification. Otherwise, it is not a gift ( Rom. 6:23). Therefore, we are
justified by faith (Rom. 5:1).
Nowhere in the Bible does it state that we are justified by grace and baptism or faith and baptism
or faith and anything else. On the contrary, baptism is excluded from the gospel message. Paul said
that he came to preach the gospel, not to baptize: "I am thankful that I did not baptize any of you
except Crispus and Gaius, so no one can say that you were baptized into my name. (Yes, I also
baptized the household of Stephanas; beyond that, I don't remember if I baptized anyone else.) For
Christ did not send me to baptize, but to preach the gospel..." (1 Cor. 1:14-17).
Likewise, Paul told us exactly what the gospel that saves is. He said in 1 Cor. 15:1-4, "Now I make
known to you, brethren, the gospel which I preached to you, which also you received, in which also
you stand, 2by which also you are saved, if you hold fast the word which I preached to you, unless you
believed in vain. 3For I delivered to you as of first importance what I also received, that Christ died for
our sins according to the Scriptures, 4and that He was buried, and that He was raised on the third day
according to the Scriptures." Note that Paul state and that the gospel is what saints and he did not
include baptism in the definition of the gospel.”
So, we must ask if baptism is necessary for salvation, then why did Paul downplay it and even
exclude it from the description of what is required for salvation? It is because baptism isn't necessary
for salvation.
Further proof that baptism is not a requirement of salvation can be found in Acts 10:44-46. Peter
was preaching the gospel, people became saved, and then they were baptized. Acts 10:44-46 says,

"While Peter was still speaking these words, the Holy Spirit came on all who heard the
message. The circumcised believers who had come with Peter were astonished that the gift of
the Holy Spirit had been poured out even on the Gentiles. For they heard them speaking in
tongues and praising God. Then Peter said, ‘Can anyone keep these people from being baptized
with water? They have received the Holy Spirit just as we have.' So he ordered that they be
baptized in the name of Jesus Christ. Then they asked Peter to stay with them for a few days,"
(NIV).

These people were saved. The gift of the Holy Spirit was on the Gentiles and they were speaking in
tongues. This is significant because tongues is a gift given to believers, see 1 Cor. 14:1-5. Also,
unbelievers don't praise God. They can't because praise to the true God is a deep spiritual matter that
is foreign to the unsaved (1 Cor. 2:14). Therefore, the ones in Acts 10:44-46 who are speaking in
tongues and praising God are definitely saved and they are saved before they are baptized. This isn't
an exception. It is a reality. This proves that baptism is not necessary for salvation and that Acts 2:38
is not teaching its necessity either. But, if it isn't saying that, then why is baptism mentioned here?

Biblical Covenant Context

A covenant is a pact or agreement between two or more parties. Very often, covenants have
visible signs to represent them. The elements of bread and wine in the communion support are good
examples of this. Circumcision was both a covenant sign and and the initiatory rite into the Abrahamic
covenant (Gen. 17:10). But this covenant sign did not save anyone.
God said to Abraham, "I will establish my covenant as an everlasting covenant between me and
you and your descendants after you for the generations to come, to be your God and the God of your
descendants after you," (Gen. 17:7, NIV). God later instructed Abraham to circumcise not only every
adult male, but also eight day old male infants as a sign of the covenant ( Gen. 17:9-13). If the
children were not circumcised, they were not considered to be under the promissory Abrahamic
covenant. This is why Moses' wife circumcised her son and threw the foreskin at Moses' feet after
Moses failed to circumcise him, (Exo. 4:24-25). She knew the importance of the covenant between
God and her children. But at the same time we must understand that circumcision did not guarantee
salvation to those who received it. It was a rite meant only for the people of God, who were born into
the family of God (who were then the Jews). It was an outward sign of the covenant promise. To
reject it was to reject the covenant. But, accepting it did not guarantee salvation.

Another theological debate at risk here

There is debate within Christianity on the nature of baptism and to whom it may be administered.
I am not here trying to convince anyone of the proper objects of baptism whether it be infant baptism
or adult only baptism. I only present the following information as a proof that baptism is a covenant
sign, and not essential to salvation.
In the New Testament, circumcision is mentioned many times. But with respect to baptism it is
specifically mentioned in Col. 2:11-12: "In him you were also circumcised, in the putting off of the
sinful nature, not with a circumcision done by the hands of men but with the circumcision done by
Christ, having been buried with him in baptism and raised with him through your faith in the power of
God, who raised him from the dead," (NIV). In these verses, baptism and circumcision are related.
The extent of that relationship is still being debated. Nevertheless, Paul also says in Rom. 2:29, "But
he is a Jew who is one inwardly; and circumcision is that which is of the heart, by the Spirit, not by the
letter; and his praise is not from men, but from God." As you can see, for the Christian, circumcision
is of the heart. And because it is, we Christians are now included the Abrahamic covenant where
before, we, the Gentiles, were not. "Remember that you were at that time separate from Christ,
excluded from the commonwealth of Israel, and strangers to the covenants of promise, having no
hope and without God in the world," (Eph. 2:12, NASB).
In Gal. 3:8, Paul calls the promise of the Abrahamic covenant, the gospel. He says, "And the
Scripture, foreseeing that God would justify the Gentiles by faith, preached the gospel beforehand to
Abraham, saying, 'All the nations shall be blessed in you, 9So then those who are of faith are blessed
with Abraham, the believer.'" (Gal. 3:8-9). So, Paul calls the Abrahamic covenant, the gospel. The
sign of this Abrahamic covenant was circumcision. Here is the catch. Since the Abrahamic covenant is
still valid (we are justified by faith -- Gal. 3:8), then is there a covenant sign for us today? I think the
answer is a resounding, yes. I believe that baptism replaces the Old Testament covenant sign of
circumcision because 1) there was a New Covenant in the communion supper (Luke 22:20), and 2) in
circumcision there was the shedding of blood, but in baptism no blood is shed. The covenant sign has
changed now that the Law has been fulfilled in Christ.
If you understand that baptism is a covenant sign, then you can see that it is a representation of
the reality of Christ circumcising our hearts (Rom. 2:29; Col. 2:11-12). It is our outward proclamation
of the inward spiritual blessing of regeneration, of "heart-circumcision." It comes after faith which is a
gift of God (Rom. 13:3) and the work of God (John 6:28). Again, baptism is the covenant sign of our
covenant with God.

Acts 2:39 and "The Promise"

This would explain why Peter in verse 39 of Acts 2 says, "For the promise is for you and your
children, and for all who are far off, as many as the Lord our God shall call to Himself." What promise
is Peter speaking of when he says "the promise"? Notice that he does not say "this promise" but "the
promise." If Peter was referring to baptism as the promise he would have said "this promise."
Instead, he used a phrase "the promise." This is significant.
The phrase "the promise" occurs in 26 Bible verses in the New Testament. It is used in reference
to several different topics.

1. The Holy Spirit, (Luke 24:49; Acts 2:33; Gal. 3:14).


2. God's promise to Abraham to multiply his descendents in Egypt, physical as well as spiritual,
(Acts 7:17; Heb. 6:13, 15, 17).
3. The promise of the Messiah, (Acts 13:32; Acts 26:6-7; Rom. 4:13,14,16; Gal. 3:17,19,22;
Eph. 3:6; 2 Tim. 1:1).
4. The promise of eternal redemption (Heb. 9:15; 1 John 2:25).
5. The promise that Sarah would have a child (Rom. 4:20; Gal. 4:23).
6. The promise that through Isaac, the world would be blessed, (Rom. 9:8).
7. The promise of Jesus' return (2 Pet. 3:4).
8. The promise to kill Paul by Paul's adversaries (Acts 22:21).

But, we are most interested in its context in Acts 2 which begins with the outpouring of the Holy
Spirit (Acts 2:1-13). Peter then preaches a sermon and quotes many OT scriptures (Acts 2:14-35). In
verse 2:22, Peter specifically says, "Men of Israel, listen to these words..." Peter is speaking to the
Jews. It was to the Jews that "the promise" of the outpouring of the Spirit was given. Peter is
speaking covenant language of God as He quotes the OT. Since Peter quotes Joel 2:28-32 in Acts
2:17-18, we can easily see what Peter is talking about when speaking of "the promise" in Acts 2:39.

"And it shall be in the last days,’ God says, ‘that I will pour forth of My Spirit upon all mankind;
and your sons and your daughters shall prophesy, and your young men shall see visions, and
your old men shall dream dreams, Even upon My bondslaves, both men and women, I will in
those days pour forth of My Spirit," (Acts 2:17-18).
See also, "For I will pour out water on the thirsty land, and streams on the dry ground; I
will pour out My Spirit on your offspring, and My blessing on your descendants," (Isa. 44:3).

Peter states in Acts 2:38, "Repent, and let each of you be baptized in the name of Jesus Christ for
the forgiveness of your sins; and you shall receive the gift of the Holy Spirit." Peter is clearly speaking
of the promise of God to grant the Holy Spirit in a new and better way. But is he saying that people
become saved by baptism in water or that baptism is part of salvation? Not at all. Peter is simply
speaking covenantally about the covenant sign. Baptism! Consider this proof, from Peter, that people
are saved before baptism.

"While Peter was still speaking these words, the Holy Spirit fell upon all those who were
listening to the message. 45And all the circumcised believers who had come with Peter were
amazed, because the gift of the Holy Spirit had been poured out upon the Gentiles also. 46For
they were hearing them speaking with tongues and exalting God. Then Peter answered,
47
"Surely no one can refuse the water for these to be baptized who have received the Holy
Spirit just as we did, can he?" 48And he ordered them to be baptized in the name of Jesus
Christ. Then they asked him to stay on for a few days," (Acts 10:44-48).

Notice that Peter had been preaching the gospel and the Holy Spirit fell upon the people. In verse
45 we see that "the gift of the Holy Spirit had been poured out upon the Gentiles also." These people
were saved. The gift of the Holy Spirit was on the Gentiles and they were speaking in tongues. This is
significant because tongues is a gift given to believers, see 1 Cor. 14:1-5. Also, unbelievers don't
praise God. They can't because praise to the true God is a deep spiritual matter that is foreign to the
unsaved (1 Cor. 2:14). Therefore, the ones in Acts 10:44-48 who are speaking in tongues and praising
God are definitely saved and they are saved before they are baptized. This simply isn't an exception. It
is a reality.

Conclusion

Acts 2:38 so closely ties repentance and baptism because it is contextually covenant language and
covenant concept. It is not stating that you must be baptized in order to be saved. It is saying that
baptism is the complete and total covenantal identification with Christ in His death, burial, and
resurrection. It is not the covenant representation (baptism) of what Christ did that saves us, but the
reality of His sacrifice which we receive by faith (Rom. 5:1; Gal. 3:8). That is why we can see in Acts
10:44-48 a group of people who are saved before they are baptized.
Baptism is not what saves. It is not part of salvation. It is something someone does who is
already saved.
Cults
Introduction

Cults are everywhere. Some are mainstream and widely accepted. Others are isolationist and hide
from examination at great expense. They are growing and flourishing. Some cause great suffering
while others appear very helpful and beneficial. Which ever group it is, the ultimate end is their
destruction when the Lord returns to claim His own.

1. What is a cult? p. 2
2. What are some of the marks of a cult? p. 2
3. What makes a church or group non-Christian? p. 5
4. What is the Difference between justification and sanctification. p. 9
5. What would you say to a cultist who testifies that his faith is true? p. 14
6. Who is the True Jesus? p. 20
Cults!
1. What is a cult?
A. Generally, it is a group that is unorthodox, esoteric, and has a devotion to a person, object,
or a set of new ideas.
i. New Teaching - has a new theology and doctrine.
ii. Only True Teaching - often considers traditional religious systems to be apostate and it
alone possess the complete truth.
iii. Strong Leadership - often an individual or small but powerful leadership group holds
control of the group’s teachings and practices.
iv. Asset Acquirement - often requires tithing and/or property transfer to the religious
system.
v. Isolationist - to facilitate control over the members physically, intellectually, financially,
and emotionally.
vi. Controlling - exercises control over the members. Sometimes this is through fear,
threatening lose of salvation if you leave the group. Sometimes through indoctrination.
vii. Indoctrination - possesses methods to reinforce the cult’s beliefs and standards where
opposing views are ridiculed and often misrepresented.
viii. Apocalyptic - to give the members a future focus and philosophical purpose in avoiding
the apocalypse or being delivered through it.
ix. Experience - various practices including meditation, repetition of words and/or phrases,
and ‘spiritual’ enlightenment with God are used as confirmation of their truth.
a. Depravation - sleep and food deprivation which weakens the will of the subject.
b. This is uncommon, though practiced by more severe cults
x. Persecution - predictions of being persecuted and often combined with claiming any
opposing views demonstrated against them as a form of persecution.
B. Many have a non-verifiable belief systems
i. For example, they would teach something that cannot be verified.
a. A space ship behind Hale-Bop comet
b. Or, that God, an alien, or angel appeared to the leader and gave him a revelation
c. The members are seeded angels from another world, etc.
ii. Often, the philosophy makes sense only if you adopt the full set of values and definitions
that it teaches.
a. With this kind of belief, truth becomes unverifiable, internalized, and easily
manipulated through the philosophical systems of its inventor.
C. The Leader of a Cult
i. Often charismatic who is considered very special for varying reasons:
a. The leader has received special revelation from God.
b. The leader claims to be the incarnation of a deity, angel, or special messenger.
c. The leader claims to be appointed by God for a mission
d. The leader claims to have special abilities
ii. The leader is often above reproach and is not to be denied or contradicted.
D. Cult ethos
i. Usually seek to do good works, otherwise no one would join them.
ii. They are usually moral and possess a good standard of ethical teaching.
iii. Many times the Bible is used or additional "scriptures" are penned.
a. The Bible, when used, is always distorted with private interpretations.
iv. Many Cults recruit Jesus as one of their own and redefine him accordingly
E. Cult groups vary greatly.
i. From the ascetic to the promiscuous.
ii. From esoteric knowledge to very simple teachings.
iii. From the rich and power to the poor and weak.
2. Who is vulnerable to joining a cult?
A. Everyone is vulnerable.
i. Rich, poor, educated, non-educated, old, young, previously religious, atheistic, etc.
B. General Profile of cult member (some or all of the following)
i. Disenchanted with conventional religious establishments.
ii. Intellectually confused over religious and/or philosophical issues
iii. Sometimes disenchanted with society as a whole
iv. Has a need for encouragement and support
v. Emotionally needful
vi. Needs a sense of purpose.
vii. Financially needful
3. Recruitment techniques
A. They find a need and fill it. One of the ways they do this is called
i. "Love Bombing" - Constant positive affection in word and deed.
a. Sometimes there is a lot of physical contact like hugging, pats on the back, and
touching.
b. Cult group members will lend emotional support to someone in need.
c. Help them in various ways...whatever is needed.
i. The person then becomes indebted to the cult.
d. Compliment them, reassure them, and make them the center of attention.
B. Many Cults use the influence of the Bible and/or mention Jesus as being one of their own;
thereby adding validity to their system.
i. Scripture twisting
a. Those that use the Bible take verses out of context
b. Then mix their misinterpreted verses with their aberrant philosophy.
C. Gradualism
i. Slow altering of thinking processes and belief system through repeated teaching
a. People usually accept cult doctrines one point at a time.
b. New beliefs are reinforced by other cult members.
4. Why would someone join?
A. The cult satisfies various needs:
i. Psychological - Someone could have a weak personality, easily lead.
ii. Emotional - Someone could have recently suffered an emotional trauma
iii. Intellectual - Someone has questions that this group answers.
B. The cult gives them approval, acceptance, purpose, and a sense of belonging.
C. The cult is appealing for some reason. It could be . . .
i. Moral rigidity and purity
ii. Financial security
iii. Promises of exaltation, redemption, higher consciousness, or a host of other rewards.
5. How are they kept in the cult?
A. Dependence
i. People often want to stay because the cult meets their psychological, intellectual, and
spiritual needs.
B. Isolation
i. Outside contacts are reduced and more and more of the life of the member is built
around the cult.
ii. It then becomes very easy to control and shape the member.
C. Cognitive Reconstruction (Brainwashing):
i. Once the person is indoctrinated, their thinking processes are reconstructed to be
consistent with the cult and to be submissive to its leaders.
ii. This facilitates control by the cult leader(s).
D. Substitution
i. The Cult and cult leaders often take the place of mother, father, priest, teacher, and
healer.
ii. Often the member takes on the characteristics of a dependent child seeking to win the
approval of the leader and or group.

E. Indebtedness
i. The member becomes indebted to the group emotionally, financially, etc.
F. Guilt
i. The person is told that to leave is to betray the leader, God, the group, etc.
ii. The person is told that leave would mean to reject the love and help the group has
given.
G. Threat
i. Threat of destruction by God for turning from the truth.
ii. Sometimes physical threat is used, though not often.
iii. Threat of missing the apocalypse, or being judged on judgment day, etc.
6. How do you get them out?
A. The best thing is to try not to let them get trapped in the first place.
B. If you are a Christian, then pray.
C. But, to get a person out of a cult takes
i. Time, energy, and support.
D. Teach them the truth.
i. Give them a true replacement for their aberrant belief system
ii. Show the cult group's philosophic inconsistencies
iii. Study the group and learn its history seeking clues and information.
E. Try and get them physically away from the cult group.
F. Give them the support they need emotionally.
G. Alleviate the threat that if they leave the group they are doomed or in danger.
H. Generally, don't attack the leader of the group...that comes later.
I. Converts often feel a loyalty and respect for the founder of the group.
J. Confront them when needed.

Hopefully, this basic outline will give you information to see how Cults work and how to avoid
them. If you have someone who is lost in a cult, you need to pray and ask the Lord to remove them
and give you the insight and tools needed. It can be a long and arduous task and very often ends in
failure. This is not an easy ministry.
What makes a church or group non-Christian?
There are many non-Christian religions and cults in America: Mormonism, Jehovah's Witnesses,
Christian Science, Unity, The Way International, Unitarianism, Islam, Hinduism, etc. They all claim
special revelation and privilege and those that use the Bible invariably interpret it in disharmony with
standard biblical understanding And, groups like the Mormons and Jehovah's Witnesses object to the
label "cult" because it often gets an emotional reaction as well as is a label they want to avoid.
The dictionary defines cult as "a system of religious worship or ritual"; "devoted attachment to, or
extravagant admiration for, a person, principle, etc.", "a group of followers." This is a typical secular
definition and by it, any believer in any god is a cultist, even atheists since they have an admiration for
a principle and are a group of followers of the philosophy of atheism.
The definition I use for "non-Christian cult" or "non-Christian religion" is a group that may or may
not include the Bible in its set of authoritative scriptures. If it does include the Bible, it distorts the true
biblical doctrines that effect salvation sufficiently so as to void salvation. 127 If it doesn't use the Bible,
it is a non-Christian religion and does not participate in the benefit of divine revelation.
In Christian bookstores, there are almost always 'cult' sections which include the Mormons,
Jehovah's Witnesses, etc. So, I am not alone in describing what a non-Christian, bible based cult is.
Nevertheless, what makes something non-Christian is when it denies the essential doctrines of the
Bible.

• The Deity of Christ, which involves The Trinity


• The Resurrection, and
• Salvation by Grace

All of them add to the finished work of Jesus on the cross. Some cult groups even add to the Bible,
i.e., Mormonism which has the Book of Mormon, Doctrine and Covenants, and The Pearl of Great Price.
Also Christian Science has added Science and Health with Key to the Scriptures. The Jehovah's
Witnesses, however, have actually changed the text of the Bible to make it fit what they want it to. For
information on this see Jehovah's Witnesses and how they have changed the Bible.
Cults add their own efforts, their own works of righteousness to the finished work of salvation
accomplished by Jesus on the cross. All Cults say that Jesus' sacrifice is sufficient, but our works must
be 'mixed with' or 'added to' His in order to prove that we are saved and worthy of salvation. They say
one thing but believe another. They maintain that they must prove themselves worthy and that they
must try their best to please God and prove to Him that they are sincere, have worked hard, and are
then worthy to be with Him. In other words, they do their best and God takes care of the rest.
This is absolutely wrong. The Bible says that we are saved by grace not by works "For by grace
you have been saved through faith...not as a result of works, that no one should boast, (Eph. 2:8-9,
NASB); not by anything we do "For we maintain that a man is justified by faith apart from works of
the Law" (Rom. 3:28, NASB). Because if there was anything that we could do to merit the forgiveness
of our sins, then Jesus died needlessly "nevertheless knowing that a man is not justified by the works
of the Law but through faith in Christ Jesus, even we have believed in Christ Jesus, that we may be
justified by faith in Christ, and not by the works of the Law; since by the works of the Law shall no
flesh be justified...I do not nullify the grace of God; for if righteousness comes through the Law, then
Christ died needlessly (Gal. 2:16, 21, NASB).
People in cults will often cite James 2:26 where it says that faith without works is dead in an
attempt to demonstrate that works are part of becoming saved. While it is true that faith without
works is dead, it isn't the works that save us. James is saying that if you have real and true faith, it
will result in real and true works of Christianity. In other words, you do good works because you are
saved, not to get saved. He isn't saying that our works are what saves us, or that they, in

127
This definition of "cult" is not sufficient to cover all that needs to be discussed in cult theologies and practices
nor is it broad enough to address the topic of world religions like Hinduism, Buddhism, and Islam which are do not
fall under the definition I've employed.
The term cult can range from any group of worshippers of any God who pay no attention to the Bible, to a
small, highly paranoid, apocalyptic people who gather around a charismatic leader that uses the Bible to control
them. Nevertheless, I've chosen a definition. I'll probably modify it as I learn more.
combination with the finished work of Christ, save us. James is simply telling us that if we say we
have faith (James 2:14) but we have no works in correspondence to that faith, then that faith won't
save us because it is a dead faith. This agrees with Paul who tells us that faith is what saves us,
"Therefore having been justified by faith, we have peace with God through our Lord Jesus Christ"
(Rom. 5:1). This faith is real faith, or true saving faith, not just an empty mental acknowledgement of
God's existence which is what those who "say" they have faith but show no corresponding godliness
are guilty of. Incidentally, you should realize that faith is only as good as who you put it in. Just
having faith in something doesn't mean you're saved. That is why it is important to have the True
Jesus, because if you have great faith but it is in the wrong Jesus, then your faith is useless.
In Mormonism Jesus is the brother of the devil begotten through sexual intercourse from a God
who came from another planet. In Jehovah's Witnesses he is Michael the Archangel who became a
man. In the New Age Movement he is a man in tune with the divine consciousness. Which is true? The
only true Jesus is the one of the Bible, the one who is prayed to (1 Cor. 1:1-2 with Psalm 116:1; Acts
7:55-60); worshipped (Matt. 2:2, 11, 14:33, John 9:35-38, Heb. 1:8), and called God (John 20:28;
Col. 2:9). The Jesus of the Cults is not prayed to, worshipped, or called God. And since the Jesus of
the Bible is the only one who reveals the Father (Luke 10:22) so that you may have eternal life (John
17:3), you must have the true Jesus who alone is the way, the truth, and the life ( John 14:6).
Another common denominator among the Cults is their methods for twisting scripture. Some of
the errors they commit in interpreting Scripture are: 1) taking Scripture out of context; 2) reading into
the Scriptures information that is not there; 3) picking and choosing only the Scriptures that suit their
needs; 4) ignoring other explanations; 5) combining scriptures that don't have anything to do with
each other; 5) quoting a verse without giving its location; 6) incorrect definitions of key words; and 7)
mistranslations. These are only a few of the many ways Cults misuse Scripture.
If you want to be able to witness well to a person in a cult, you need to understand their doctrines
as well as your own. It would be a good idea to study both Christian Doctrine: the Bible, God,
Creation, Man... and Christian Doctrine: Jesus, the Holy Spirit, Salvation... as well as the The Three
Essential Doctrines of Christianity to become better equipped. Through study you will be able to
answer questions that often come up in witnessing encounters. A Christian should know his doctrine
well enough to be able to recognize not only what is true, but also what is false in a religious system
(1 Pet. 3:15; 2 Tim. 2:15).
Jesus warned us that in the last days false Christs and false prophets would arise and deceive
many (Matt. 24:24). The Lord knew that there would be a rise of the spirit of Antichrist (1 John 4:1-3)
in the last days. Its manifestation is here in the forms of Mormonism, Jehovah's Witnesses, and the
New Age Movement, among others.
Comparison Chart
This chart is a simplification of various groups and their beliefs. It is, to be realistic, an
oversimplification. But it gives the reader an idea of where the groups stand.

Scripture
Jesus
The The and
Group Name Founder God Jesus Salvation Resurrec-
Gospel Church writings
tion
of group
That Jesus rose
Those Trinity: 3
Jesus Jesus God in in the The Bible
Christianity who are Persons in by Grace
Christ saves flesh same body alone
saved one God
from sin he died in.
The Bible
John is
Thomas Faith in Created inspired.
Members One God
Christa- (1805- Christ being. A Baptism is Elpis
of their as one Yes
delphianism 1871) and man with a required Israel,
church person
founded baptism sin nature Eureka
1848 written by
founder.
Religious Bible,
A man in
Mary beliefs of Science
A tune with
Baker Jesus' Impersonal and Health
Christian collection the Divine Correct
Eddy teaching Universal No with Key to
Science of spiritual Conscious thinking
(1821- s, not the Presence the
ideas ness, not
1910) atoneme Scriptures,
the Christ.
nt. Miscellany
Created by Allah's
Faith in One God:
Muham- being. A grace and Qur'an,
Islam Allah and n/a Allah. Non No.
mad prophet. man's Hadith
works Trinitarian
Not God. works
Bible,
Created Keeping Studies in
Charles T. Jesus being. the the
Russell opened Michael command Scriptures,
Members
Jehovah's (1852- the door One the ments, presently
of their No
Witnesses 1916) for us to Person archangel being in the
church
founded earn our who their Watchtowe
1879 salvation became a Organizati r and
man on Awake
Magazines
Jesus' The Bible,
Joseph atoneme Created Book of
Church of Resurrecte
Smith nt plus being. The Mormon,
Jesus Christ Members d by grace,
(1805- the Laws Triad - 3 brother of Doctrine
of Latter day of their saved by Yes
1844) and gods. the devil and
saints church doing
founded ordinanc and of all Covenants,
(Mormons) works.
1830 es of the people Pearl of
gospel Great Price
By faith
and
Faith in Members One baptism "in
Oneness God in
N/A Christ and of their Person Jesus' Yes The Bible
Pentecostal flesh
baptism church modalism name" in
their
church
The Secret
Madme
Doctrine,
Helena
Isis
Blava- Created
Unveiled,
tsky God is a being. A
Theosophy n/a n/a * No The Key to
(1831- principle great
Theosophy
1891) teacher
and The
founded
Voice of
1875
the Silence
Charles Bible,
Adopting
Fillmore The A Created Unity
Impersonal the correct
(1854- overall collection being. A Magazine,
Unity Universal Unity No
1948) principles of spiritual man, not Metaphysi
Power thought
founded of Unity ideas the Christ cal Bible
principles
1889 Dictionary.
Bible,
Victor Created Jesus
Paul Members being. A Christ is
Way One
Weirwell Earned of their man, not By works Yes Not God,
International Person
(1917- church God in Power for
1985) flesh Abundant
Living
Justification and Sanctification: What is the Difference?
Understanding the difference between justification and sanctification can be as important as
understanding the difference between salvation and damnation. Rightly dividing between the two is of
crucial importance. When you understand the what they are, you can then draw a line in the sand and
say, "This is what saves. This is not what saves."
Justification is the work of God where the righteousness of Jesus is reckoned to the sinner so the
sinner is declared by God as being righteous under the Law (Rom. 4:3; 5:1,9; Gal. 2:16; 3:11). This
righteousness is not earned or retained by any effort of the saved. Justification is an instantaneous
occurrence with the result being eternal life. It is based completely and solely upon Jesus' sacrifice on
the cross (1 Pet. 2:24) and is received by faith alone (Eph. 2:8-9). No works are necessary whatsoever
to obtain justification. Otherwise, it is not a gift (Rom. 6:23). Therefore, we are justified by faith
(Romans 5:1).
Sanctification, on the other hand, involves the work of the person. But it is still God working in the
believer to produce more of a godly character and life in the person who has already been justified
(Phil. 2:13). Sanctification is not instantaneous because it is not the work of God alone. The justified
person is actively involved in submitting to God's will, resisting sin, seeking holiness, and working to
be more godly (Gal. 5:22-23). Significantly, sanctification has no bearing on justification. That is, even
if we don't live a perfect life, we are still justified.
Where justification is a legal declaration that is instantaneous, sanctification is a process. Where
justification comes from outside of us, from God, sanctification comes from God within us by the work
of the Holy Spirit in accordance with the Bible. In other words, we contribute to sanctification through
our efforts. In contrast, we do not contribute to our justification through our efforts.

Does this mean that those justified by grace can sin as much as they want?

Romans 6:1-2 says, "What shall we say then? Shall we continue in sin, that grace may abound?
God forbid. How shall we, that are dead to sin, live any longer in it?"
1 Thess. 4:7 says, "God has called us not for the purpose of impurity, but in sanctification."
The Scriptures teach us that we are to live holy lives and avoid sin (Col. 1:5-11). Just because we
are saved and eternally justified before God (John 10:28), that is no excuse to continue in the sin from
which we were saved. Of course, we all sin (1 John 1:8). But the war between the saved and his
sinfulness is continuous (Rom. 7:14-20) and it won't be until the return of Jesus that we will be
delivered from this body of death (Rom. 7:24). To continually seek sin and use God's grace to excuse
it later is to trample the blood of Christ underfoot (Heb. 10:29) and to reveal the person's true sinful,
unsaved nature (1 John 2:4; 2:19). (Other verses worth checking out are: Heb. 12:14; 1 Pet. 1:14-
16; and 1 Pet. 2:21-22.)

What the cults do with justification and sanctification

The cults consistently blur the meanings of the two terms and misapply the truths taught in God's
word. The result is a theology of works righteousness, of earning their salvation which only leads to
damnation. This is because by the works of the Law shall no flesh be justified (Gal. 2:16). Man cannot
contribute to his salvation (Gal. 5:1-8). Because man is sinful even his best deeds are stained and
filthy before God (Isaiah 64:6). Therefore, making a person right before God can only be God's work
(Gal. 2:20).
Typically, in cult theologies, a person is not justified (declared righteous in God's eyes) until the
final day of judgment when his works are weighed and a reward is given or he is found worthy of his
place with God. Thus, a person with this errant theology can not claim 1 John 5:13 as their own which
says, "These things have I written unto you that believe on the name of the Son of God; that you may
know that you have eternal life, and that you may believe on the name of the Son of God."
Contextually, "These things" refers to loving God, being obedient to Him, belief in Christ, and
eternal life in Jesus. Therefore, 1 John 5:13 can be considered a test. If you are believing and doing
the right things, then you will know if you have eternal life. Can a cultist know he has eternal life? No.
He cannot. But a Christian can.
Do we have the right to make these judgments?

To pronounce another religious group to be false can seem a pompous undertaking, especially in a
culture that preaches tolerance for everything from homosexuality to a mother's "right" to kill her
unborn child. Tolerance is the banner that unites much of our culture and anyone who points a judging
finger at someone or something is often ridiculed.
But Christians are told in the Bible to separate themselves from the sinful practices of man and to
expose error. "Examine all things and to hold fast to that which is true" says God's word (1 Thess.
5:21). So we do.
What does it mean to examine if we do not judge what is right and wrong? Jesus judged the
Pharisees as hypocrites. Peter judged Ananias and Saphira as liars (Acts 5:3-4). Paul judged the
Galatians as fools (Gal. 3:1).
The reason something can be said to be right or wrong is because the Bible has laid out before us
a moral and doctrinal standard that is clear. It is wrong to lie. So, we are able to say to someone who
lies, "What you are doing is wrong." That is making a judgment.
Likewise, with the cults, as Christians we are commanded to be able to give answers to everyone
(1 Pet. 3:15) and to contend for the faith that was delivered by the apostles (Jude 4). If we do not
fight for the faith, the faith will be lost. If we do not expose the errors of the cults then the cults will
move unchecked in the world and lead even more into eternal destruction.
To make a judgment means that we must recognize that there are absolutes. In a world that
worships relativism, absolutes are not welcome and the cults that espouse their demonic doctrines beg
tolerance.
CARM stands for the truth of God's word, not a compromising collection of beliefs that changes as
people's whims change. The cults are cults because they deny the true God, add works to salvation,
and corrupt a multitude of biblical truths. Their end and the end of all who follow them is damnation.
To do anything other than warn people about them would be unloving.
What is the truth?
Truth is not a feeling. Truth is not an idea. The truth is found in the Bible. The cults are wrong
because they do not have the truth. That is, they have a false understanding of God the Father, of God
the Son, of the Holy Spirit, and the work of Christ on the Cross. Because they are in error in these
things, they are in error concerning the doctrine of salvation.
Sincerity and good works do not bridge the gap of sin between God and man. Only the blood of
Jesus can cleanse a person (Heb. 9:22; John 14:6). Sincerity and good works are merely the reliance
upon what is in the individual to merit favor with God. Sincerity is a form of pride when it is appealed
to as a justification for being accepted by God: "But God, look at my heart. See how sincere I was? I
deserve to be in heaven."
No. If sincerity and good works were good enough to satisfy God, then He wouldn't have given us
the Bible to correct our sincerely wrong ideas, and He wouldn't have sent His Son to do what our good
works cannot. Truth is what God says is true.
Below is an example of someof the different teachings of Mormons and Jehovah’s Witnesses as
they define Christianity, God, Jesus, salvation, etc. Notice how not all can be true.

Only One God

1. Christianity:
True Christian doctrine is that there is only one God who has ever existed anywhere,
anyplace, anytime. There is no God formed before God; there will be no God formed after
God (Isaiah 43:10). God doesn't even know of any other Gods (Isaiah 44:8). There is only
one God in existence in the entire universe. Just one. This is called monotheism.
2. Mormonism:
The god of earth is only one of many, many gods (Bruce McConkie, Mormon Doctrine, p.
163). This is called polytheism. However, they worship only one of the many gods, the one
called Elohim. This is called monolatry.
Elohim (as the Mormons like to call the Father) used to be a man on another planet
(Mormon Doctrine, p. 321). Elohim became a God and came to this world with his goddess
wife (Articles of Faith, by James Talmage, p. 443). Mormons have the potential of becoming
gods of their own worlds (Teachings of the Prophet Joseph Smith, pages 345-347, 354).
This contradicts the teachings of scripture. Mormons are polytheists. Christianity is
monotheistic. Mormonism is very wrong.
3. J.W.:
They believe in only one God (Make Sure of All Things, p 188). They call God "Jehovah."
In this, the J.W.'s are correct. There is only one God.

The Trinity

1. Christianity:
God is a trinity of persons: The Father, the Son, and the Holy Spirit. The Father is not the
same person as the Son. The Son is not the same person as the Holy Spirit. The Holy Spirit
is not the same person as the Father. They all are eternal, divine, and omniscient.
Objections to the Trinity are that it is not logical. Logic should not rule scripture. If it is
from God, there will be things in it that are difficult to understand. Additionally, the
fingerprints of God are seen all over creation. In Romans 1:20 it says that the invisible
attributes of God are clearly seen in creation. Creation is made up of a trinity of trinities:
time, space, and matter. Time is past, present, and future. Each "part" is different, yet they
are all of the same nature: time. Space is height, width, and depth. Each "part" is different,
yet they are all of the same nature: space. Matter is solid, liquid, and gas. Each "part" is
different, yet they are all of the same nature: matter.
2. Mormonism:
The Trinity is an office held by three separate gods: a god called the father, a god called
the son, and a god called the holy spirit. They error in assuming that a "person" must be in a
fleshly form (Doctrine and Covenants, 130:22) -- something like flesh and blood.
This contradicts the orthodox view of the Trinity as well as teaching that there is more
than one God (Isaiah 43:10; 44:6,8).
3. J.W.'s:
They deny the Trinity (Let God be True, p. 100-101; Make Sure of All Things, p.386).
They say there is only one person in the Godhead: the Father.
They error in denying the truth doctrine of the Trinity.

Jesus

1. Christianity:
Jesus is the second person of the Trinity. Jesus is both God and man. He is fully God and
fully man (Col. 2:9). He was in the form of God, emptied Himself, and became a man (Phil.
2:5-8). As the God man, He is the mediator (1 Tim. 2:5). Jesus was not created (John 1:1-
3), but is the creator of all things (Col. 1:16-17).
2. Mormonism:
Jesus, the devil, and all of us are literal spirit children born in a pre-existence, the literal
offspring of God the Father and his goddess wife (Mormon Doctrine p. 516; Journal of
Discourse, Vol. 4, p. 218).
3. J.W.'s:
Jesus is Michael the Archangel who became a man, died on a stake -- not a cross -- rose
in a spirit body, and returned to heaven to be an angel again (The Watchtower, May 15,
1963, p. 307; The New World, 284).
The problem here is that Jesus (Michael) would be a created thing. This is why the J.W.
Bible adds the words "other" four times in Col. 1:16-17. The word "other" is not in the
original text of the Bible.

Salvation

1. Christianity:
Salvation, or the forgiveness of sins, is something that is given to you by God. It is a free
gift (Rom. 6:23). The sinner is made righteous in God's eyes solely by the faith that the
believer has in the sacrifice of Jesus on the cross. Justification, or being declared righteous,
is accomplished by faith (Rom. 5:1). Our works play no part in salvation. If our works could
play any part at all, then Jesus died needlessly (Gal. 2:21).
2. Mormonism:
The doctrine of the forgiveness of sins in Mormonism is that you are saved by grace after
all you can do (Article 8 of the Church of Jesus Christ of Latter-day Saints; Articles of Faith,
by James Talmage, p. 92). They add to the finished work of Christ on the cross and say that
Jesus made it possible for us to be forgiven. Our works must be mixed with the finished work
of Christ and then our forgiveness of sins is merited before God.
This error is that works play a part in our salvation, our forgiveness of sins. They do not.
In Galatians chapters 3 and 5, Paul addressed the issue and condemned the thought of
keeping even one part of the Law in order to be righteous with God. Salvation is a free gift,
paid for by the blood of Christ.
3. J.W.'s:
Forgiveness of sins is by good works and cooperation with God (Studies in the Scriptures,
Vol. 1, pp. 150, 152). They maintain that the sacrifice of Jesus (really Michael the archangel)
opened the door that Adam closed. In other words, because of Jesus' sacrifice you are able
to cooperate with God and earn salvation.
The error here is the same as that in Mormonism listed above. Works do not play a part
in our salvation. They come after we are saved, not before, and not in cooperation with
anything. To add to the work of Christ is to say that what Jesus did on the cross isn't
enough. This is an insult to God.

Conclusion

The truth is important not because it is simply true. It is important because truth is what defines
who and what we believe in. Is Jesus the brother of the devil as in Mormonism? Is he an angel who
became a man? Or, is He the creator of the universe, second person of the Trinity? Only one is right.
Faith is vital. But faith placed in something false is the same as having no faith at all. Faith is only
as good as the object in which it is placed. That is why it is important to have the true Jesus, the one
of the Bible, not the one of Mormonism, nor the one of the Jehovah's Witnesses.

Eternity is a long time to be wrong, especially about Jesus.


What about the testimonies of people in cults?
We cannot deny that people in different and contradictory religious systems have equally strong
testimonies of the truth of their beliefs. Mormons, for example, frequently appeal to a "burning in the
bosom" as a way of knowing that Mormonism is true. Likewise, Jehovah's Witnesses "know" that their
religion is correct. Christian Scientists, Christadelphians, and others each have countless members
who sincerely believe in the validity of their religions and will testify to their truth. These groups
emphasis "inner testimonies" to different degrees. But the fact remains: contradictory belief systems
have members who testify to the truth of contradictory religious systems.
Christians likewise bear their testimonies concerning the truth of Christianity. We testify to the
validity of monotheism (in opposition to Mormonism's polytheism128), of salvation by grace alone (in
opposition to Jehovah's Witnesses' teaching that obedience to Law requirements are necessary 129), of
the reality of sin (in opposition to Christian Science's non-existence of actual sin 130), and, of course,
that Jesus has saved us from our sins. Obviously, not all contradictory belief systems can be true. So,
what can we conclude about these testimonies that affirm the validity of different and contradictory
religious systems? We can conclude three things.
First, testimonies are subjective. They are not absolute. They are conveyed to us by people who
have feelings about a religious experience or belief. In essence, it is an experience and experiences
are, by nature, subjective.
Second, the fact that people bear witness to contradictory belief systems means that testimonies
are not proofs of the validity of any religion -- no matter how strong they might be. Someone who
appeals to his testimony as proof of his religion can easily be countered by someone else stating he
has an equally strong testimony of something to the contrary. Yet, both parties will still assert they
are right.
And third, if we assume that at least one person (or group) is correct, then that would mean that
all the other people who have a testimony of the truth are, in actuality, being deceived in some
way. From this we can affirm that many people are being deceived who "know for a fact" that their
religion is correct.

How would you know if you are deceived?

If it is true that people who sincerely believe in something can be deceived, how would they
discover whether or not they are, in fact, being deceived? Simple. Turn to God's word.
I believe that in order to escape deception, a person would need an authoritative and reliable
source of information outside his "testimony" by which he can judge spiritual truths and compare his
experience. He must be willing to subject his "testimony" to something other than the grilling of his
own subjective experience which he claims to be from God. Why? Because if the person used his own
testimony to validate his experience then he could not determine whether or not he is deceived since
that is appealing to that which is part of the deception to discover truth. It won't work.
Therefore, in order to discover if you are being deceived, you must appeal to God's word and
compare your "testimony" to it. If what your testimony points to is in contradiction to God's word,
then your testimony is not true.

Testing the Testimony

If someone had a testimony that a religious system was true and that system said it was okay to
lie, we could easily conclude that his testimony was incorrect since it supports something that goes
against God's revealed word. The person would be deceived. This is simple. However, applying this
principle to people isn't easy because since they believe they are not deceived, they will find a way to

128
Mormon Doctrine, by Bruce McConkie, p. 163; Articles of Faith, by James Talmage, p. 443.
129
In the Watchtower magazine of Feb. 15, 1983, p. 12, four requirements for salvation are listed. The second
one states, "Many have found the second requirement more difficult. It is to obey God’s laws, yes, to conform
one’s life to the moral requirements set out in the Bible. This includes refraining from a debauched, immoral way
of life.—1 Corinthians 6:9, 10; 1 Peter 4:3, 4." Hence, JW theology denies salvation by grace through faith alone.
130
Science and Health with Key to the Scriptures, 447:24.
adopt an interpretation consistent with their belief systems.
Some people will believe their testimonies (feelings) even if the Bible says something to the
contrary. Why? Because they will subject God's word to they own testimony. This is commonly done
by Mormons. For example, the Bible states that God does not even know of any other Gods (Isaiah
44:8). Yet, in Mormonism God has a goddess wife. 131 So, Mormons reinterpret the verse to agree
with their testimony. They will state that God knows of no other gods "of this world." They, in effect,
add words to the text. We then can become locked in the horns of an interpretive dilemma which is
sometimes difficult to overcome.
Nevertheless, among biblically based cult groups, it is almost unanimously agreed that a testimony
comes from the Holy Spirit who, according to Scripture (1 John 2:27), resides within the true believer
and bears witness of the truth (John 16:13). This testimony comes from the Holy Spirit who is
supposed to testify of Jesus (John 15:26). 132 This is what all cults claim in one form or another. Yet,
there is one factor I've encountered that is an important part of the witness of the Spirit in a true
believer.

Assurance of forgiveness of sins

"These things have I written unto you that believe on the name of the Son of God; that ye may
know that ye have eternal life, and that ye may believe on the name of the Son of God" (1 John
5:13)

Because the cults serve false gods, they also have a false gospels (Gal. 1:8-9). All these "other"
gospels are works oriented. That is, because the members of cults must cooperate with God in some
form (obeying commandments, being baptized, etc.) in order to get or maintain the forgiveness of
sins, they cannot have assurance of salvation. Yet, the Bible tells us that we can know we have
eternal life.
I know that I have eternal life. I know that all my sins are forgiven right now. I know and testify
that I am saved by the true and living God and that Jesus is my only Savior. My sins are forgiven and
I know I have eternal life.
Of all the cult members I've spoken to, none have told me that they have assurance of eternal life.
I can only conclude that they do not have the testimony that is from God. Therefore, they are wrong.
They are deceived.
It becomes necessary for all of us to examine our beleifs in the light of God's word and to change
our beliefs accordingly. Ultimately, we should come to that place where we have assurance of eternal
life -- in agreement with God's word. Do you have this assurance?

131
Articles of Faith, by James Talmage, p. 443.
132
I recommend going to the Christian Doctrine section of CARM and reading about who Jesus really is. In cults,
the definitions of who Jesus is are also contradictory.
An easy way to witness to Mormons and Jehovah's Witnesses
The following method of witnessing to Cultists is non-offensive and powerful. It focuses on Jesus,
the gospel, and uses Scripture. This is important for three reasons: first, Jesus draws all men to
Himself (John 12:32); second, the Gospel is powerful for salvation (Rom. 1:16); and third, God’s Word
accomplishes what God wants it to (Isaiah 55:11).
If someone puts his faith in the Jesus of Mormonism, Jehovah’s Witnesses, or any other cult, then
his faith is useless. The validity of faith does not rest in itself, but in its object. The greatest faith in
someone false is the same as no faith at all. That is the case with the Mormons and the Jehovah’s
Witnesses. Each group believes in a Jesus, but not in the Jesus of the Bible, and because they each
have a false Jesus (2 Cor. 11:4), they each preach a false gospel (Gal. 1:8-9). They may be sincere,
but they are sincerely wrong – dead wrong.
The official theologies of the Mormons and Jehovah’s Witnesses do not permit prayer to nor the
worship of Jesus. They also deny that He can be called their God. But the Bible permits, even
encourages, these things for the true believer. The true Jesus, the Jesus of the Bible, is prayed to,
worshiped, and called God. And, this is where we must begin.
If you can prove a Cultist wrong in a minor point of theology, he is still a Cultist. But, if you show
him that the Jesus he believes in is not the same one found in the Bible, then you have undermined
his entire theology.
In brief, you should introduce the Cultist to the real Jesus: the one of the Bible who is prayed to
(Acts 7:59; 1 Cor. 1:1-2), worshiped (Matt. 2:2,11; 14:33; 28:9; John 9:35-38; Heb. 1:6), and called
God (John 20:28; Heb. 1:8; Titus 2:13). The hope is that once the Cultist sees that he is without the
Jesus of the Bible, he will realize he doesn’t have the true God. Then, hopefully, he will accept Christ
and leave his cult. If not, at least the seeds of truth will have been planted and he will have been
exposed to the true Jesus.
The "approach" is simple.

1. Establish a common ground: the need to know the Father.


2. Establish that the only way to the Father is through Jesus: the Jesus of the Bible.
3. Show the need for having the correct Jesus, the one of historical (and Biblical) Christianity.
4. Establish that the Jesus of the Bible is prayed to, worshiped, and called God.
5. Ask the Cultist if he prays to, worships, and calls Jesus God.
6. Ask the Cultist why he is right and you are wrong if you do what the scriptures teach and he
doesn’t.
7. Present the gospel

Remember, a false Jesus cannot save. Sincerity and false messiahs do not bridge the gap of sin
between God and man, only the Jesus of the Bible does that.
Here is a sample dialogue between a Christian and a Cultist.

Christian: Would you agree with me that we both want to know the Father and do what He wants us
to do?"
Cultist: Yes.
Christian: How, then, do we get to know the Father?
Cultist: Through prayer and reading the Bible.
Christian: Well, that’s not a bad answer. But Jesus said that he was the One who revealed the Father
to us (Matt. 11:27 and Luke 10:22). So, to know the true Father we must first know the true Jesus,
right?
Cultist: Yes, that seems reasonable.
(You are not attacking his doctrine, you are appealing to his desires which, on the surface, are
identical to yours: to serve and love God. You can catch more bees with honey than with a
hammer.)

Christian: Well, let me ask you another question. Will a false Jesus reveal the true God?
Cultist: No. I suppose not.
Christian: That’s right. The real issue then, isn’t that we are going to church or are nice people. It’s
whether or not we know the true Jesus so that He can reveal to us the true God. Right?
Cultist: Right?
Christian: The question is, "How do we find the true Jesus?" The only way I can think of is if we go to
the Bible. That is where the true Jesus is, right?
Cultist: Right. But you could also pray and ask God to reveal Him to you.
Christian: I see what you mean. But how could you pray to God if the only way to get to Him is
through Jesus, and you don’t have the right Jesus? Wouldn’t prayer, then be useless?
Cultist: Not if you’re sincere.
Christian: But then you are saying that if you are sincere, you don’t need Jesus. Do you see the
problem with that? Sincerity doesn’t make access to God possible. Only Jesus does that. Remember,
Jesus said that no one comes to the Father, except by Him (John 14:6).
Cultist: But doesn’t James 1:5 say if you lack wisdom to ask of God and He will give it to you? So
couldn’t you ask God for wisdom about what is true?
Christian: James was written to those who were already believers; they already had the true Jesus
and, therefore, the true Father. Also, wisdom is the proper use of knowledge. It isn’t the gaining of
knowledge, nor is it gaining access to God. Do you see that you still have to have the true Jesus.
Because if you were to pray to God for wisdom, and you served a false Jesus, then who is going to
answer your prayers? It wouldn’t be God would it?
Cultist: I see your point.
Christian: Good. Now let me ask you a couple of questions to get things started. If you were to say,
"Father receive my spirit," who would you be praying to?
Cultist: I would be praying to the Father.
Christian: Right. If you were to say, "Jesus receive my spirit," who would you be praying to?
Cultist: I wouldn’t pray to Jesus. I would only pray to the Father. That is what He said to do in Matt.
6. He said to pray, "Our Father who art in heaven..."
Christian: Yes, that’s true. But if you believe it do you do it? Do you pray that way all the time?
Cultist: Of course I do.
Christian: No, what I mean is. Do you pray that prayer. If you believe that is what you are to pray,
then you could only pray that particular prayer. You would have to repeat it every time you prayed.
But that isn’t what Jesus intended. It was a model prayer. It is what we are to follow. May we
continue? You’ll see what I’m getting at in a moment.
Cultist: Sure. Go ahead.
Christian: Just for the sake of argument, if you were to say, "Jesus receive my spirit," who would you
be praying to?
Cultist: I would be praying to Jesus.
Christian: Right. Now, in Acts 7:55-60, Stephen, while full of the Holy Spirit, prayed to Jesus. It says,
"And they went on stoning Stephen as he called upon the Lord and said, ‘Lord Jesus, receive my
spirit.’" (See also, Acts 9:14; Rom. 10:13.) Stephen prayed to Jesus, not just through Him. If it is
acceptable for him then it should be alright for you. The Jesus of the Bible is prayed to. I pray to
Jesus. Do you? If yes, good. If not, why not?
(If you are talking to a Mormon, you may want to mention that in the Book of Mormon in 3
Nephi 19:18, Jesus is prayed to -- not just through!)

Cultist: Jesus said to pray to the Father. So, I do.


Christian: Yes, I agree. I do too. But I also pray to Jesus as Stephen did. If the church is only to pray
to the Father, then why did Stephen, under the inspiration of the Holy Spirit, address Jesus in His
prayer? Was he wrong?
Cultist: I don’t have an answer.
Christian: Also, what does it mean to call upon the name of the Lord?
Cultists: I don’t know. What does it mean?
Christian: It means to seek God, even to pray to God. For example, in Psalm 116:4 it says, "Then I
called on the name of the LORD: ‘O LORD, save me!’" In 1 Cor. 1:1-2 the church calls upon the name
of the Lord Jesus. That is, they prayed to Jesus. Now, if Stephen, full of the Holy Spirit, could pray to
Jesus, and the church in 1 Cor. 1:1-2 could too, then shouldn’t you be able to do the same thing?
Cultist: Well, I’m not sure. I’ve never really considered this before.
Christian Glad to see you’re honest. Let’s continue. Jesus was also worshiped. The verses for these
are: "And those who were in the boat worshiped Him, saying, ‘You are certainly God’s son!’" (Matt.
14:33). "And behold, Jesus met them and greeted them. And they came up and took hold of His feet
and worshiped Him" (Matt. 28:9). (See also Matt. 2:2,11; John 9:35_39; Heb. 1:6.) Do you do what
His disciples did? Do you worship Jesus?

(Mormon theology does not allow worship of Jesus. However, some Mormons do anyway. They
just don’t know that their church has, and still does, teach against doing so. If the person says
he worships Jesus, ask him how he can do that without praying to Him. If, on the other hand,
the Mormon has said he does pray to Jesus and that he does worship Jesus, then encourage
him to continue and remind him that it is Jesus who has the authority (Matt. 28:18) to forgive
sins (Luke 5:20-24; 7:48-49); He judges (John 5:22,27); He gives eternal life (John 10:28;
5:40), etc. -- See 100 Truths About Jesus. The whole point is to try to get him to ask the true
Jesus to forgive him of his sins and reveal the Father to him.)

(All Jehovah’s Witnesses say no to worshiping Jesus. They have their own Bible where they
have mistranslated the word ‘worship’, wherever it refers to Jesus, to the words ‘do obeisance’
which means to show respect or honor to someone. Because of this, using the verse about
worship will not carry much weight. In that case, you will want to substitute this...)

Christian: Do you honor Him equally with the Father as Jesus said to do in John 5:23?
Cultist:: Not equally. The Father is greater than Jesus.
Christian: The Father was greater in position. Remember, Jesus was made for a little while lower than
the angels (Heb. 2:9). It was in this humbled state that He said the Father was greater than He. He
didn’t say different or better, only greater. You must understand that Jesus was fully man as well as
fully God and as a man was in a lessor position. Still though, Scripture requires that you honor Him
equally with the Father as Jesus said. If you don‘t, then why not?
Cultist: I don’t have an answer.
Christian: Alright. There is just one more issue to address. Do you call Jesus your Lord and your God?
Cultist: No, I don’t.
Christian: After Jesus’ resurrection He showed Himself to many people. One of them was Thomas.
John 20:28 says, "Thomas answered and said to Him [Jesus], ‘My Lord and my God!’ Jesus said to
him, ‘Because you have seen Me, have you believed?’"
In addition, God calls Jesus God in Hebrews 1:8, "But of the Son He [the Father] says, "Thy throne, O
God, is forever and ever..." The Father calls Jesus God. Thomas called Him his Lord and God. Do you
call Jesus your Lord and your God?
Cultist: No. I don’t call Jesus my Lord and God.

(Jehovah’s Witnesses will say that Thomas was swearing. Ask them why Jesus didn’t rebuke
Thomas for swearing? Besides, in the Greek, Thomas literally said, "The Lord of me and the
God of me.")

(With a Mormon, you can again mention the reference in the Book of Mormon 3 Nephi 19:18
where Jesus is also called Lord and God.)

Christian: My question to you is this. If I have the wrong Jesus, and therefore I serve the wrong God,
then why do I pray to Jesus, worship Him, and call Him my Lord and God as the Scriptures teach? But,
if you have the true Jesus, why is it you don’t do those things?
Cultist: (Silence!)
Christian: It seems clear that if you want your sins forgiven then you need to go to Jesus and ask
Him to forgive you. Remember, the true Jesus, the Jesus of the Bible, is prayed to, worshiped, and
called God. That is the same Jesus I serve. Which one do you serve?
Christian: Since Jesus is the one who forgives sins, then I go to Him. You can too. All you have to do
is pray to Him and ask Him to forgive you of your sins. You already know you are a sinner. So simply
go to Him, the real Jesus, and receive the forgiveness of sins through faith in Him.

This brief approach is powerful because it brings the Cultist face-to-face with the Jesus of the Bible.
Though the Cultist won’t respond by dropping to his knees, at least you will have exposed him or her
to the real Jesus. Also, remember that the Word of God will accomplish what God wishes it to: "So
shall My word be which goes forth from My mouth; it shall not return to Me empty, without
accomplishing what I desire," (Isaiah 55:11).
The True Jesus

There is a simple way to see if someone has the true Jesus or not. By true Jesus, I mean the one
of the Bible, not the one of Mormonism who is the brother of the devil, nor the Jehovah's Witness
Jesus who is Michael the Archangel, and certainly not the one of the New Age Movement who is simply
a man in tune with the divine consciousness.

• The Jesus of the Bible is prayed to (Acts 7:55-60; Psalm 116:4 and Zech. 13:9 with 1 Cor. 1:1-
2).
• The Jesus of the Bible is worshiped (Matt. 2:2,11; 14:33; 28:9; John 9:35-38; Heb. 1:6)
• The Jesus of the Bible called God (John 10:28; Heb. 1:8).

In cult theologies, Jesus is a creation in one form or another (this is why the Jehovah's Witnesses
add the word ‘other' four times to Col. 1:16-17). Therefore, He is not to be prayed to, worshiped, or
called God.
If you are a Christian then you will be able to pray to Jesus, not just through. You will be able to
worship Jesus equally with the Father. And you will be able to call Jesus your Lord and God. A cultist
cannot do this. A cultist has a false Jesus, and, therefore, a false hope of salvation.

The following is an expansion of the above points

If you put your faith in a Jesus that is not true, then your faith is useless. The power of faith does
not rest in the act of believing, but in its object; the greatest faith in someone false is the same as no
faith at all. Sincerity and false messiahs do not bridge the chasm of sin between God and man, only
the Jesus of the Bible does that. Who then, is the true Jesus?
Jesus said that He was the only One who reveals the Father (Matt. 11:27 and Luke 10:22): "All
things have been committed to me by my Father. No one knows who the Son is except the Father, and
no one knows who the Father is except the Son and those to whom the Son chooses to reveal him,"
(NIV).
So, to know the true Father you must first know the true Jesus. The question is, how do you
recognize the true Jesus? Simple, look in the Bible.
If you were to say, "Father receive my spirit," who would you be praying to? The Father, right?
If you were to say, "Jesus receive my spirit," who would you be praying to? Jesus.
In Acts 7:59, Stephen, while full of the Holy Spirit (v. 55), prayed to Jesus:

And they went on stoning Stephen as he called upon the Lord and said, "Lord Jesus, receive my
spirit." (See also Acts 9:14; Rom. 10:13.)

Stephen prayed to Jesus, not just through Him. If it is acceptable for him then it should be alright
for you. The Jesus of the Bible is prayed to. I pray to Jesus. Do you? If yes, good. If not, why?
But you might say, "Jesus said to pray to the Father." I do. But I also pray to Jesus as Stephen
did. If the church is only to pray to the Father then why did Stephen, under the inspiration of the Holy
Spirit, address Jesus in His prayer? Was he wrong? See also 1 Cor. 1:1-2 with Psalm 116:4 where
calling upon the name of the Lord is prayer and prayer is addressed to Jesus by the Corinthian church.

Jesus was also worshipped. The verses are:

And those who were in the boat worshiped Him, saying, "You are certainly God's son! (MMatt.
14:33).
And behold, Jesus met them and greeted them. And they came up and took hold of His feet
and worshiped Him (Matt. 28:9).
See also Matt. 2:2,11; 14:33; 28:9; John 9:35-38; Heb. 1:6.
The Jesus of the Bible is prayed to and worshiped. Do you do what Jesus' disciples did? Do you
pray to and worship the true Jesus?
Since it is against Mormon and Jehovah's Witness theologies to pray to Jesus but only through if
you do worship Jesus, how can you do that without praying to Him? And, do you honor Him equally
with the Father as Jesus said to do in John 5:23? If you do not, then why not?
There is just one more issue to address. Do you call Jesus your Lord and God?
After Jesus' resurrection He showed Himself to many people. One of them was Thomas. John
20:28:

Thomas answered and said to Him [Jesus], "My Lord and my God!" The literal Greek says, "The
Lord of me and the God of me."

"My God!" is a pagan expression used today. Two points can be made from this. First, do you
agree that Thomas a devout Jew was swearing, like a pagan of today? Second, there is no biblical
account of swear words. Peter did swear in Mark 14:71 by swearing he did not know Jesus. To say
Thomas was swearing, or merely exclaiming profound surprise has no evidence.

God calls Jesus God in Heb. 1:8:

“But of the Son He [the Father] says, "Thy throne, O God, is forever and ever..."

Unfortunately, in the Jehovah's Witness Bible in Heb. 1:8 you'll see that it says, "God is your
throne, forever and ever." This, technically speaking, is a legitimate translation. The reason this is so
lies in the nature of the Greek language and the fact that the form of the word "God" and "Throne"
both end in a noun construction that is interchangeable, therefore making the NWT translation
legitimate. It is unfortunate that the Watchtower has chosen to do this. Nevertheless, if you'd like to
read more about this, then go to The Jehovah's Witnesses and Heb. 1:8 and Psalm 45:6.

Conclusion:

The Jesus of the Bible is prayed to (Acts 7:55-60; Psalm 116:4 and Zech. 13:9 with 1 Cor. 1:1-2),
worshiped (Matt. 2:2,11; 14:33; 28:9; John 9:35-38; Heb. 1:6), and called Lord and God (John
20:28; Heb. 1:8). If I have the wrong Jesus, and therefore I serve the wrong God, then why do I pray
to Jesus, worship Him, and call Him my Lord and God as the Scriptures teach? But, if you have the
true Jesus, why is it you don't do those things? Why does JW theology not agree with the scriptures?
I think the answer is simple. The Jesus of the cults is not the true Jesus. Therefore, they are
wrong.
Jehovah’s Witnesses

Introduction

The Jehovah’s Witnesses are known for going door-to-door bringing their Watchtower and Awake
magazines. They claim to be Christian, but they bring a non-historical and altered message about
God, Jesus, and biblical truth. They are not Christian.

1. What are the basic beliefs of the Jehovah's Witnesses? pp. 24-26
2. Why is the Jehovah's Witness religion not Christian? p. 28
3. Where do the Jehovah's Witnesses get their theology from? p. 29
4. How does the Watchtower organization control the JW's thinking? p. 33-34
5. Why are the JW's not the faithful and discreet slave? pp. 40-41
6. How is one saved as a Jehovah's Witness? p. 42
7. What are some of the bad translations of the JW Bible? p. 47
8. Which quotes quotes from the Watchtower strike you as interesting? Why? pp. 53-55
9. Has the Watchtower organization ever made a false prophecy? pp. 56-57
10. What do the JW's say about Jesus' resurrection that is incorrect? pp. 64-65
11. Did Jesus die on a stake or a cross? p. 66
12. What do the JW's do with John 8:58? p. 73
13. How would you respond to the JW's use of John 17:3? p. 79
14. What does it mean to call upon the name of the Lord in respect to Jesus? p. 81-86
15. What do the JW's do to Col. 1:16-17? p. 88
Jehovah's Witnesses in a Nutshell

According to Jehovah's Witness' theology, God is a single person, not a Trinity, who does not know
all things and is not everywhere. He first created Michael the Archangel through whom He created all
"other things," including the universe, the earth, Adam and Eve, etc. This creative work took God
42,000 years. At one point, The Watchtower Bible and Tract Society taught that God ruled the
universe from somewhere in the Pleiades star system. They have since modified this to say that the
"Pleiades can no longer be considered the center of the universe and it would be unwise for us to try to
fix God's throne as being at a particular spot in the universe." 133 Such changes and even
contradictions in teaching are frequent in the Watchtower organization and when a doctrine changes,
they tell their followers that the light of truth is getting brighter.
After Adam sinned, the paradise which God had created for them, was ruined. So, God instituted a
system of redemption which was revealed in the Bible and would ultimately lead to the crucifixion of
Jesus the messiah. But, in the meantime, God needed to have a visible, theocratic organization on
earth to accurately represent Him. Throughout history, this true organization had a remnant of faithful
Jehovah's Witnesses (Noah, Abraham, Moses, David, etc.) but it wasn't until the late a00's that
Charles Taze Russell formerly began what is now known as the Watchtower Bible and Tract Society
which is run out of Brooklyn, New York. This organization claims to be the only true channel of God's
truth on earth today and that it alone can properly interpret God's word since it is the angel directed,
prophet of God on earth.
When it came time for the savior to be born, Michael the Archangel became a human, in the form
of Jesus. Jesus grew and kept all the laws of God and never sinned. Finally, when Jesus died, it was
not on a cross, but on a torture stake, where he bore the sins of mankin77

d -- but this did not include Adam's sins. Jesus rose from the dead as a spirit, not physically (his body
was dissolved and taken by God) and during his visitations to people on earth, he manifested a
temporary physical body for them to see and touch. Thus began the true Christian church of
Jehovah's followers.
Throughout history there have been faithful Jehovah's witnesses who have managed to keep The
Truth in spite of the "demonic" doctrine of Trinitarianism that has permeated the Christian church in
"Christendom." Christendom is filled with pastors who are antichrists, in churches run by Satan, and
who support the earthly governments which are all of the devil. In other words, all of Christianity is
false and only the Jehovah's Witness "theocratic" organization lead by several men in Brooklyn, New
York, is true.
In the late 1800's, a young man of 18 years, by the name of Charles Taze Russell, organized a
Bible class in Pittsburgh, Pennsylvania. In 1879 he sought to popularize his ideas on doctrine so he
co-published The Herald of the Morning magazine with its founder, N. H. Barbour and by 1884 Russell
controlled the publication and renamed it The Watchtower Announcing Jehovah's Kingdom, and
founded Zion's Watch Tower Tract Society (now known as the Watch Tower Bible and Tract Society).
Russell served as the teacher and guide for the organization which taught that Jesus returned invisibly
in 1914 and is now reigning in heaven. When Jesus finally returns physical to earth, which will happen
at the time of the Battle of Armageddon, He will set up his earthly 1000 year kingdom. During this
1000 year period, people will be resurrected and have a second chance to receive eternal salvation by
following the principles of Jehovah's Organization on earth known as the Watchtower Bible and Tract
Society. After the millennium, those who reject God and His organization will be annihilated; that is,
they will cease to exist. The rest of the Jehovah's Witness who have faithfully followed God's
organization on earth will be saved from eternal annihilation and reside forever on Paradise earth.
Heaven, however, is a place for a special group of 144,000 Jehovah's Witnesses -- the only ones who
are "born again" and who alone are allowed to take communion in their annual communion service.
These are the ones who have "immortal life," all other Witnesses have "everlasting life." Those with
133
Watchtower 11/15/53, page. 703.
immortal life do not have resurrected bodies. They have "spirit bodies." Those on Paradise Earth have
everlasting life and consist of a resurrected body that must be maintained through eating, rest, etc.
When you study with the Jehovah's Witness, you agree to attend five meetings a week where you
are taught from Watchtower literature. You cannot be baptized until you have studied their material
for at least six months and have answered numerous questions before a panel of elders. Men are not
supposed have long hair or wear beards and women are to dress in modest apparel. They refuse to
vote, salute the flag, sing the "Star Spangled Banner, celebrate birthdays or Christmas, won't take
blood transfusions, and they can't join the armed forces. A schedule of door-to-door canvassing is
required where you distribute the Watchtower literature, acquire donations, and forward all monies to
the headquarters in Brooklyn, New York.
If you ever leave the Jehovah's Witness organization, you are considered an apostate and are to be
shunned.
What do the Jehovah's Witnesses Teach?

1. There is one God in one person, Make Sure of All Things, p 188.
2. There is no Trinity, Let God be True, p. 100-101; Make Sure of All Things, p.386.
3. The Holy Spirit is a force, not alive, Reasoning from the Scriptures, 1985, pp.
406-407.
4. The Holy Spirit is God's impersonal active force, The Watchtower, June 1, 1952,
p. 24.
5. Jehovah's first creation was his 'only-begotten Son'. . . was used by Jehovah in
creating all other things", Aid to Bible Understanding, pp. 390-391.
6. Jesus was Michael the archangel who became a man, The Watchtower, May 15,
1963, p. 307; The New World, 284.
7. Jesus was only a perfect man, not God in flesh, Reasoning from the Scriptures,
1985, pp. 306.
8. Jesus did not rise from the dead in his physical body, Awake! July 22, 1973, p. 4.
9. Jesus was raised "not a human creature, but a spirit." Let God be True, p. 276.
10. Jesus did not die on a cross but on a stake, Reasoning from the Scriptures, 1985,
pp. 89-90.
11. Jesus returned to earth, invisibly, in 1914, The Truth Shall Make You Free, p. 300.
12. Jesus' ransom sacrifice did not include Adam, Let God be True, p. 119.
13. Their church is the self-proclaimed prophet of God, The Watchtower, April 1,
1972, p. 197.
14. They claim to be the only channel of God's truth, The Watchtower, Feb. 15,
1981, p. 19.
15. Only their church members will be saved, The Watchtower, Feb, 15, 1979, p. 30.
16. Good works are necessary for salvation, Studies in the Scriptures, Vol. 1,
pp. 150, 152.
17. The soul ceases to exist after death, Let God be True, p. 59, 60, 67.
18. There is no hell of fire where the wicked are punished, Let God be True, p. 79, 80.
19. Only 144,000 Jehovah's Witness go to heaven, Reasoning from the Scriptures,
1985, pp. 166-167, 361; Let God be True, p. 121.
20. Only the 144,000 Jehovah's Witness are born again. Reasoning from the
Scriptures, 1985, p. 76.; Watchtower 11/15/54, p. 681.
21. Only the 144,000 may take communion,
22. Blood transfusions are a sin, Reasoning from the Scriptures, 1985, pp. 72-73.
23. The Cross is a pagan symbol and should not be used, Reasoning from the
Scriptures, 1985, pp. 90-92.
24. Salvation is by faith and what you do, Studies in the Scriptures, Vol. 1, pp.
150,152.
25. It is possible to lose your salvation, Reasoning from the Scriptures, 1985, pp.
358-359.
26. The universe is billions of years old, Your will Be Done on Earth, p. 43.
27. Each of the 6 creative days of God in Genesis 1, was 7000 years long.
Therefore, Man was created toward the end of 42,000 years of earth's
preparation, Let God be True, p. 168.
28. They also refuse to vote, salute the flag, sing the "Star Spangled Banner," or
celebrate Christmas or birthdays. They are not allowed to serve in the armed
forces.
29. Satan was entrusted with the obligation and charged with the duty of
overseeing the creation of the earth, Children, p 55
Jehovah's Witness History
The Jehovah's Witnesses was begun by Charles Taze Russell in 1872. He was born on February 16,
1852, the son of Joseph L. and Anna Eliza Russell. He had great difficulty in dealing with the doctrine
of eternal hell fire and in his studies came to deny not only eternal punishment, but also the Trinity,
and the deity of Christ and the Holy Spirit. When Russell was 18, he organized a Bible class in
Pittsburgh, Pennsylvania. In 1879 he sought to popularize his aberrant ideas on doctrine. He co-
published The Herald of the Morning magazine with its founder, N. H. Barbour and by 1884 Russell
controlled the publication and renamed it The Watchtower Announcing Jehovah's Kingdom, and
founded Zion's Watch Tower Tract Society (now known as the Watch Tower Bible and Tract Society).
The first edition of The Watchtower magazine was only 6,000 copies each month. Today the Witnesses'
publishing complex in Brooklyn, New York, churns out 100,000 books and 800,000 copies of its two
magazines--daily!
Russell claimed that the Bible could be only understood according to his interpretations. A
dangerous arrangement since he controlled what was written in the Watchtower magazine. This kind
of assertion is typical among leaders of cult religions.
After the death of Russell on Oct. 31, 1916, a Missouri lawyer named Joseph Franklin Rutherford
took over the presidency of the Watch Tower Society which was known then as the Dawn Bible
Students Association. In 1931 he changed the name of the organization to "The Jehovah's Witnesses."
After Rutherford's death, Nathan Knorr took over. After Knorr, Frederick William Franz became
president.
Today the Society is led by Mr. Henschel. The group has over 4 million members world wide. The
Watchtower Society statistics indicate that 740 house calls are required to recruit each of the nearly
200,000 new members who join every year.
The Jehovah's Witnesses have several ‘book studies' each week. The members are not required to
attend, but there is a level of expectation that gently urges converts to participate. It is during these
‘book studies' that the Jehovah's Witness is constantly exposed to counter Christian teachings. The
average Jehovah's Witness, with his constant Watchtower indoctrination, could easily pummel the
average Christian when it comes to defending his beliefs.
The Jehovah's Witnesses vehemently portray the doctrine of the Trinity as pagan in origin and that
Christendom, as a whole, has bought the lie of the devil. Along with denying the Trinity is an equally
strong denial of the deity of Christ, the deity of the Holy Spirit, the belief in hell, and eternal conscious
punishment in hell.
Is the Jehovah's Witness religion Christian?
The answer to the question is, no. It is not Christian. Like all non-Christian cults, the Jehovah's
Witness organization distorts the essential doctrines of Christianity. It denies the deity of Christ, His
physical resurrection, and salvation by grace. This alone makes it non-Christian. To support its erring
doctrines, the Watchtower organization (which is the author and teacher of all official Jehovah's
Witness theology), has even altered the Bible to make it agree with its ever changing and non-
Christian teachings.
Typical with cults that use the Bible to support its position is a host of interpretive errors:

• Taking verses out of their immediate context.


• Refusing to read verses in the entire biblical context.
• Inserting their theological presuppositions into the text.
• Altering the Biblical text to suit their needs.
• Latching onto one verse to interpret a host of others.
• Changing the meanings of words.
• Proclaiming some passages to be figurative when they contradict their doctrines.
• Adding to the Word of God.

Additionally, the Jehovah's Witness organization requires of its members regular weekly
attendance at their "Bible Study" meetings where they are repeatedly indoctrinated with anti-Christian
teachings. This is done by reading the Watchtower magazine, following along with what it says,
reading the questions it asks, and reciting the answers it gives. In other words, the Watchtower
Organization carefully trains its members to let the Organization do their thinking for them.
The Witnesses are told they will be persecuted when they go door to door teaching their doctrines.
They are further told that this is simply the enemy fighting against God's organization because they
are in "the truth." So, when someone disagrees with them, they then feel confirmed in being in “the
truth” (like all cults claim). They are strongly encouraged to have friends and acquaintances that are
only JW’s, thereby keeping outside examination to a minimum. They are told to shun those who leave
their group. That way there is no way to see why someone has left and no way to find out that they
are in error from those who have found the truth in Christ. They are conditioned to shy away from any
real biblically knowledgeable person. An example of this is frequently found on the Internet. I was
once banned from a Jehovah's Witness chat room after I not only answered their objections to the
Trinity and deity of Christ, but challenged them in return. Subsequently, my name was passed around
to all other Jehovah's Witness rooms where I was banned from them as well.
The Jehovah's Witnesses consider themselves to be Christians because they believe they are
serving the true and living God. Like many cults, they think they are the only true church on earth.
Yet, they deny the Trinity, the deity of Christ, the personhood of the Holy Spirit, Jesus' physical
resurrection, and salvation by grace through faith. In addition, the Jehovah's Witnesses are
discouraged from looking into Jehovah's Witness history or old Watchtower literature which is replete
with contradictions, altered doctrines, and false prophecies. Instead, they are indoctrinated repeatedly
against basic Christian doctrines (Trinity, deity of Christ, etc) and into the notion that they alone are
the true servants of God and that all others are either in "Christendom" or simply unbelievers.
Primarily, the Jehovah's Witness organization is a mind control organization that uses its people to
pass out literature and send in "donations" to the headquarters in Brooklyn, New York.

"Thus the Bible is an organizational book and belongs to the Christian congregation as an
organization, not to individuals, regardless of how sincerely they may believe that they can
interpret the Bible." The Watchtower, Oct. 1, 1967. p. 587.

The Watchtower organization of the Jehovah's Witnesses is a non-Christian organization that uses
its people to promulgate false doctrines, sell a multitudinous amount of literature, and expand its grip
into the lives of its members and their families.
It is a non-Christian cult.
Are Jehovah's Witness are Really Watchtowerites?
Jehovah’s Witnesses faithfully go door-to-door preaching the "Kingdom of God" that is taught them
via the Watchtower Bible and Tract Society. They are usually quite polite and sincere in their efforts of
communicating "Jehovah’s Good News." As always, they carry with them several books and
magazines, some of which are the New World Translation (their Bible which has been altered in many
places), the Awake Magazine, and, of course, the ubiquitous Watchtower Magazine.
The Jehovah’s Witnesses receive their direction from The Watchtower Bible and Tract Society
headquartered in Brooklyn, New York. This organization claims to be the channel of communication
from God to his people, that it represents Jesus on earth, and that you cannot find Scriptural guidance
outside of it as an organization. Consider the following quotes:

• "It should be expected that the Lord would have a means of communication to his people on
the earth, and he has clearly shown that the magazine called The Watchtower is used for that
purpose." 1939 Yearbook of Jehovah's Witnesses, p. 85."
• Make haste to identify the visible theocratic organization of God that represents his king, Jesus
Christ. It is essential for life. Doing so, be complete in accepting its every aspect." The
Watchtower, October 1, 1967, p. 591.
• "We all need help to understand the Bible, and we cannot find the Scriptural guidance we need
outside the ‘faithful and discreet slave' organization." The Watchtower, Feb. 15, 1981.

Obviously, the Watchtower Bible and Tract Society assumes a great deal, including being God’s
"visible theocratic organization" and the ‘faithful and discreet slave" teaching true Christian doctrine.
The only problem is that the Watchtower brand of doctrine is not biblical doctrine. It is heavily filtered
doctrine through Watchtower interpretations combined with selective questions and scripture quotes.
Of course, the Jehovah’s Witnesses will strongly disagree with this statement. They say that they
read and study their Bibles and only use the Watchtower literature as a guide to understanding God’s
word. But it is this very admission which condemns them because their doctrines are not found in the
Bible. The proof is found, believe it or not, in the Watchtower’s own writings. Consider this quote from
The Watchtower Magazine, August 15, 1981 that says:

"From time to time, there have arisen from among the ranks of Jehovah's people those, who,
like the original Satan, have adopted an independent, faultfinding attitude...They say that it is
sufficient to read the Bible exclusively, either alone or in small groups at home. But, strangely,
through such ‘Bible reading,' they have reverted right back to the apostate doctrines that
commentaries by Christendom's clergy were teaching 100 years ago..." The Watchtower,
August 15, 1981.

Did you get that? If you read the Bible by itself, you will become a Trinitarian because that is
exactly what the Watchtower is referring to here when it says "apostate doctrines." In other words, if
you read the Bible alone, you will not arrive at Watchtower doctrines. This is an amazing admission by
the Watchtower organization. It is clear, Jehovah's Witnesses do not get their teachings from the Bible,
but from the Watchtower literature.

Therefore, Jehovah's Witnesses are Really Watchtowerites

The average Kingdom Hall (Jehovah's Witness church) has "Book" studies several times a week. It
is in these meetings that the Jehovah's Witness gets new Watchtower publications, studies with the
help of the Watchtower aids, and discusses doctrines in ‘Bible studies." This is how they are
indoctrinated. Whether or not a Jehovah's Witness likes or dislikes what is being taught is not the
issue. Rather, the Jehovah's Witness is supposed to accept and believe what is taught via the
Watchtower Literature and is discouraged from independent thinking. Another quote:
"We should eat and digest and assimilate what is set before us, without shying away from parts
of the food because it may not suit the fancy of our mental taste...We should meekly go along
with the Lord's theocratic organization and wait for further clarification…" The Watchtower,
February 1, 1952, pp. 79-80.

This quote clearly shows that the Watchtower Bible and Tract Society is the Jehovah's Witness'
teacher and those who follow its teachings are Watchtowerites. That is, they are students of the
Watchtower which is why, basically, all Jehovah's Witnesses all believe the very same thing. Of
course, they will say that this is unity and not confusion as is found in Christendom’s denominations.
But precise unity in beliefs among a people is a sign not of freedom, but of control.
Within Christianity’s denominations are the core beliefs that unite Christians all over the world. We
are allowed differences of opinions on non-essential doctrines (Rom. 14:1-7). Not so with the
Jehovah's Witnesses. They are all taught the same doctrine from the same publications. They each
give identical Watchtower responses to questions and challenges, and all present the identical claims
of "The Kingdom of God," "the Errors of the Trinity," the "Coming Armageddon," etc. Essentially, if
you’ve spoken to one Jehovah's Witness, you’ve spoken to them all.
If you want to learn what the average Jehovah's Witnesses believes, you don’t read the Bible, you
read the Watchtower Magazine. This is because the Watchtower is the source of their theological
beliefs, not the Bible. If you want to quickly learn what the Watchtower teaches, spend an hour with
any Jehovah's Witness. The Jehovah's Witness is, quite plainly, a Watchtowerite.
Has Jehovah performed the greatest act of love?

According to the Watchtower Bible and Tract Society, God is not a Trinity and Jesus is not God in
flesh. Because of their position, I sometimes ask the question "Has Jehovah performed the greatest
act of love?". I get different answers but because they have not been indoctrinated by the Watchtower
Magazine, they often actually think on their own instead of repeating what the Watchtower says. Of
course, the Jehovah's Witness almost always says that the greatest act of love performed by God was
sending His Son to die for us. This is a good answer, but is it correct? I then quote them what Jesus
said in John 15:13, "Greater love has no one than this, that one lay down his life for his friends"
(NASB). The KJV says, "Greater love hath no man than this, that a man lay down his life for his
friends." The Jehovah's Witness Bible, the NWT says, "No one has love greater than this, that
someone should surrender his soul in behalf of his friends."
We can see that the greatest act of love is to sacrifice one's own life for another. Note that Jesus
said that this self sacrifice is the greatest act of love. I then again ask the Jehovah's Witness, "Has
Jehovah performed the greatest act of love?". That is when the problem arises for the Jehovah's
Witness.
I tell them that as a Trinitarian, my Lord has performed the greatest act of love. Jesus is God in
flesh (John 1:1,14). Therefore, God has laid His life down for a friend; He has performed the greatest
act of love that someone can do -- just as Jesus said. I then ask the Jehovah's Witness, "Why do you
want me to give up the greatest act of love performed by my God, for your God who cannot perform
the greatest act of love?" The conversation can take many turns at this point. But it always is good to
focus on God's love for us on the cross.
Following is an outline that helps clarify the issue as it quotes scripture.

1. God is love
A. 1 John 4:16, "And so we know and rely on the love God has for us. God is love. Whoever
lives in love lives in God, and God in him."
i. The New World Translation says, "And we ourselves have come to know and have
believed the love that God has in our case. God is love, and he that remains in love
remains in union with God and God remains in union with him."
2. God demonstrates the greatest love
A. He is infinite, holy, loving, and no one can demonstrate these qualities better than God
Himself. God has full ability to demonstrate His own nature. Can anyone "out do" God in
love?
3. Jesus accurately represents the Father
A. Heb. 1:3, "And He is the radiance of His glory and the exact representation of His nature,
and upholds all things by the word of His power. When He had made purification of sins, He
sat down at the right hand of the Majesty on high," (NASB).
i. The New World Translation says, "He is the reflection of [his] glory and the exact
representation of his very being. And he sustains all things by the word of his power;
and after he had made purification of sins he sat down on the right hand of the Majesty
in lofty places."
B. John 14:8, ". . . Anyone who has seen me has seen the Father. . ."
i. The NWT says, ". . . He that has seen me has seen the Father [also]. . ."
4. Jesus said, . . .
A. John 15:13, "Greater love has no one than this, that one lay down his life for his friends."
i. (The NWT says, "No one has love greater than this, that someone should surrender his
soul in behalf of his friends.")
5. Conclusion
A. If Jesus is not God and He laid down His life, then He is doing something greater than the
Father can do.
B. If Jesus is God and He laid down His life, then God is performing the greatest act of love.
The Watchtower Organization points to itself as the truth.

"Consider, too, the fact that Jehovah's organization alone, in all the earth, is directed by God's
holy spirit or active force (Zech. 4:6). Only this organization functions for Jehovah's purpose
and to his praise. To it alone God's Sacred Word, the Bible, is not a sealed book . . . How very
much true Christians appreciate associating with the only organization on earth that
understands the 'deep things of God"! . . . Furthermore, this organization alone is supplied with
'gifts in men,' such as evangelizers, shepherd and teachers..." (The Watchtower, July 1, 1973.

It is always a problem when a group points to itself as being the only way to God's truth.
Apparently, they aren't as is evidenced by their failed prophecies. Consider the following.

• "And the period of falling also corresponds; for the time our Lord said, 'Your house is left unto
you desolate, ' A.D. 33, to A.D. 70 was 36 1/2 years; and so from A.D. 1878 to the end of A.D.
1914 is 36 1/2 years. And with the end of A.D. 1914, what God calls Babylon, and what men
call Christendom, will have passed away, as already shown from prophecy" (Thy Kingdom
Come, 1891 ed. p. 153).
• "The date of the close of that 'battle' is definitely marked in Scripture as October, 1914. It is
already in progress, its beginning dating from October, 1874" (The Watchtower, Jan. 15, 1892,
pp. 21-23).
• "For instance, as we look back and note that the Scriptures marked 1873 as the end of six
thousand years from Adam to the beginning of the seventh thousand, and the fall of 1874 as
the beginning of the forty-year harvest of the Gospel age and day of wrath for the overthrow of
all the institutions of 'this present evil world [or order of affairs], we can see that facts have
well borne out those predictions of Scripture" (View from the Tower, Allegheny, PA., July 15,
1894. - Vol. 15, No. 14 [1675].)
• "Be not surprised, then, when in subsequent chapters we present proofs that the setting up of
the Kingdom of God is already begun, that it is pointed out in prophecy as due to begin the
exercise of power in A. D. 1878, and that the 'battle of the great day of God Almighty' (Rev.
16:14), which will end in A.D. 1914 with the complete overthrow of earth's present rulership, is
already commenced" (The Time is at Hand, 1911 ed., p. 101).
• "The Scriptural proof is that the second presence of the Lord Jesus Christ began in 1874 A.D."
(Prophecy, 1929, p. 65).

It is obvious that the Watchtower Organization was wrong. It claims to be the only channel of
God's truth yet it has missed the mark from the very beginning. Can it be trusted? No.
Does the Watchtower organization control
the Jehovah's Witnesses' thinking?
Following are quotes from the literature of the Watchtower Bible and Tract Society, better known
as the Jehovah's Witnesses organization. After reading the quotes below, it is obvious that the
Watchtower discourages individual thinking among the Jehovah's Witnesses and requires them to
submit to the teaching of the Watchtower organization.
Romans 14:5 says, "One man regards one day above another, another regards every day alike.
Let each man be fully convinced in his own mind." The Bible wants us to think and to be convinced in
our own minds about issues that are debatable (Rom. 14:1-12). However, the Watchtower does not
want its people to be convinced in their own minds. It wants them to submit to the Watchtower.

1. The Bible can only be understood via the Watchtower organization.


A. Quote: "Only this organization functions for Jehovah's purpose and to his praise. To it
alone God's Sacred Word, the Bible, is not a sealed book." The Watchtower; July 1, 1973,
pp. 402.
B. Comment: This is an amazing quote. Only the watchtower organization can properly
interpret the Bible. It is saying that the Lord Jesus, through the Holy Spirit, cannot give a
Christian a proper understanding of the Bible even though it is Jesus who opens the mind to
understand Scripture (Luke 24:45). This effectively prevents the Jehovah's Witness from
thinking and interpreting the Bible for himself apart from the watchtower organization's
guidance.
2. Individuals are not able to rightly interpret the Bible apart from the Watchtower
organization.
A. Quote: "Thus the Bible is an organizational book and belongs to the Christian congregation
as an organization, not to individuals, regardless of how sincerely they may believe that
they can interpret the Bible." The Watchtower, Oct. 1, 1967. p. 587.
B. Comment: The Watchtower restricts its followers from looking at the Bible by itself without
the guidance of the organization's input. This is not freedom to think for oneself and it is a
sure way to keep the Jehovah's Witnesses intellectually in line with Watchtower teachings.
Also, contrast the quote above with the following quote from the Watchtower: "The
Vatican belittles Bible study by claiming it is the only organization authorized and qualified
to interpret the Bible." The Watchtower, 7/1/1943, p. 201. The funny thing is that that is
exactly what the Watchtower does. It claims to be the only source of true spiritual
knowledge.
3. If you don't understand something, meekly wait for the Watchtower to tell you what
the truth is, otherwise you are foolish.
A. Quote: "We should eat and digest and assimilate what is set before us, without shying
away from parts of the food because it may not suit the fancy of our mental taste...We
should meekly go along with the Lord's theocratic organization and wait for further
clarification, rather than balk at the first mention of a thought unpalatable to us and
proceed to quibble and mouth our criticisms and opinions as though they were worth more
than the slave's provision of spiritual food. Theocratic ones will appreciate the Lord's visible
organization and not be so foolish as to put against Jehovah's channel their own human
reasoning and sentiment and personal feelings." The Watchtower, February 1, 1952, pp. 79-
80.
B. Comment: Here, the watchtower tells us that submission is to
God's "theocratic organization", the watchtower organization and that submission must be
complete and meek. Undoubtedly, this is clearly teaching that independent thought is not
welcome in the Watchtower Organization.
4. The Watchtower magazine is the means of God's communication
A. Quote: "It should be expected that the Lord would have a means of communication to his
people on the earth, and he has clearly shown that the magazine called The Watchtower is
used for that purpose." (1939 Yearbook of Jehovah's Witnesses, p. 85.)
B. Comment: This says that the Jehovah's Witness organization is the means God uses to
communicate on earth today. But, the Bible says that God speaks to us through His Son:
Heb. 1:1-2 says it is Jesus: "God, after He spoke long ago to the fathers in the prophets in
many portions and in many ways, 2in these last days has spoken to us in His Son, whom He
appointed heir of all things, through whom also He made the world."
5. Cannot understand the Bible outside of watchtower organization
A. Quote: "We all need help to understand the Bible, and we cannot find the Scriptural
guidance we need outside the ‘faithful and discreet slave' organization." (The Watchtower,
Feb. 15, 1981.)
B. Comment: The Jehovah's Witness organization has set itself up as the sole means of
understanding the Bible. In contrast to this, Jesus opens the mind to understand the
scriptures (Luke 24:45). It isn't the Watchtower organization that does this, but God.
Admittedly, we have denominational differences. But the differences are not in the
essentials and we certainly are able to understand those essentials apart from the
Watchtower.
6. Those who think apart from the Watchtower's guidance are like Satan
A. Quote: "From time to time, there have arisen from among the ranks of Jehovah's people
those, who, like the original Satan, have adopted an independent, faultfinding
attitude...They say that it is sufficient to read the Bible exclusively, either alone or in small
groups at home. But, strangely, through such ‘Bible reading,' they have reverted right back
to the apostate doctrines that commentaries by Christendom's clergy were teaching 100
years ago..." The Watchtower, August 15, 1981.
B. Comment: So, if you think independently, find fault with something the Watchtower says,
then you are like Satan. Is this freedom of thought? Is this how Christ works in the
church? Hardly.
Also, what are the apostate doctrines spoken of in the above quote? Of course, that
would be the Trinity, the deity of Christ and the Holy Spirit, the physical resurrection of
Christ, etc., all things the Watchtower organization denies. So, if you read the Bible by
itself, reading it for what it says, without the watchtower guiding you, you will adopt these
doctrines -- which the Watchtower says are not true!
7. If you love God, then you accept the Watchtower.
A. Quote: "We cannot claim to love God, yet deny his word and channel of communication."
The Watchtower, October 1, 1967, p. 591.
B. Comment: In other words, if you love God then you will be "in" the watchtower
organization. If you reject the organization, then you reject God. Amazing! So, the
Jehovah's Witness is urged to not leave the organization lest they end up rejecting God. Is
this encouraging freedom on thought? No.
8. The truth of God can be known only through the Watchtower organization.
A. Quote: All who want to understand the Bible should appreciate that the "greatly diversified
wisdom of God" can become known only through Jehovah's channel of communication, the
faithful and discreet slave. The Watchtower; 10/1/1994; p. 8.
B. Comment: Here, the Watchtower states that the only way to understand God's word is
through the Watchtower organization. In other words, your thinking must be in submission
to the teaching of the Watchtower.
These few quotes should be more than adequate to demonstrate that the Watchtower not only
maintains a control over the thinking of its adherents, but actually discourages independent thought.
This is definitely one of the signs of a cult.
Does Annihilation and resurrection make sense?

According to the Watchtower Bible and Tract Society, when you die, you cease to exist: Let God be
True, p. 59, 60, 67. On Judgment Day, only faithful Jehovah's Witnesses will be resurrected to life
eternal on Paradise Earth. The rest of all mankind will be annihilated, wiped out, made to not exist
with no eternal punishment in a fiery hell. O
There is a logical problem with this view. If a Jehovah's Witnesses believes that he ceases to exist
when he dies and that he will be resurrected at the Judgment Day, then is he really being
resurrected? In other words, if he was alive and then has ceased to exist, he is in the same state he
was before he was created. That is, he isn't. He has no existence. He is gone. The only remnant of
this person would be in the memory of God (not counting family and friends, etc.). Only God would
know if this Jehovah's Witness was good enough for Paradise Earth. If he was, then the reward would
be a new creation of someone in the exact image of the Jehovah's Witness who previously lived and
did all the works mandated by the Watchtower Organization. But, it wouldn't be the exact same
person, because that person ceased to exist and there is no continuity, no continuance of the person
since he has ceased to be.
Therefore, on Judgment day, how can he be resurrected? That is, how is he, as the same person,
resurrected when he doesn't exist anymore? Is he the exact same person or has God make an exact
copy of the person upon which to shower the blessings of Paradise Earth?
It would seem that simple logic would contradict the idea of existence, non-existence, and then
existence all being the same person. It also contradicts scripture which says,

• "We are of good courage, I say, and prefer rather to be absent from the body and to be at
home with the Lord," (2 Cor. 5:8).
• “I know a man in Christ who fourteen years ago—whether in the body I do not know, or out of
the body I do not know, God knows—such a man was caught up to the third heaven," (2 Cor.
12:2).

The Bible teaches us that we have an existence away from out bodies once we die. The Jehovah's
Witnesses are incorrect. We continue on after death.
Does the Watchtower say the Bible teaches the Trinity?

The Watchtower organization claims to be the Faithful and Discreet Slave spoken of in Matthew
24:45.

• "We all need help to understand the Bible, and we cannot find the Scriptural guidance we need
outside the ‘faithful and discreet slave' organization." (The Watchtower, Feb. 15, 1981.)
• All who want to understand the Bible should appreciate that the "greatly diversified wisdom of
God" can become known only through Jehovah's channel of communication, the faithful and
discreet slave. The Watchtower; 10/1/1994; p. 8.

This Watchtower organization, the faithful and discreet slave, claims to the means by which God
communicates His truth to us in this world. Therefore, by deduction, what the Watchtower says is
truth. Alright, let's take a look at one quote from the Watchtower.

• "From time to time, there have arisen from among the ranks of Jehovah's people those, who,
like the original Satan, have adopted an independent, faultfinding attitude...They say that it is
sufficient to read the Bible exclusively, either alone or in small groups at home. But, strangely,
through such ‘Bible reading,' they have reverted right back to the apostate doctrines that
commentaries by Christendom's clergy were teaching 100 years ago..." (The Watchtower,
August 15, 1981, p. 29).

The Watchtower says that the Trinity, the deity of Christ, and the physical resurrection of Christ are
all false doctrines. Yet, the Watchtower says that if you read the Bible by itself you will end up
believing these doctrines. In other words, if you read the Bible by itself, then you will become a
Trinitarian, believe that Jesus is God, and believe that Jesus rose from the dead physically. Why is
that? Could it be because the Bible teaches these things? So I ask the Jehovah's Witnesses, what is it
in the Bible that would lead someone to this conclusion? Again, what is in the Bible that reading it by
itself would lead you to believe in the Trinity, the deity of Christ, and His physical resurrection?
This quote from the Watchtower is proof that it is teaching contrary to the natural reading of the
Bible and that it is controlling the beliefs of those who follow it.
Questions for Jehovah’s Witnesses

1. The Watchtower organization has claimed to be the prophet of God (The Watchtower, April 1,
1972, p. 197) yet it has made numerous false prophecies. The excuse given for their false
prophecies has been to quote Proverbs 4:18 which says, "But the path of the righteous ones is
like the bright light that is getting lighter and lighter until the day is firmly established." Whether
or not the "light gets brighter" or not does not change the fact that the Watchtower made false
prophecies. The Bible says in Deut. 18:20-22, "‘However, the prophet who presumes to speak in
my name a word that I have not commanded him to speak or who speaks in the name of other
gods, that prophet must die. And in case you should say in your heart: "How shall we know the
word that Jehovah has not spoken?" When the prophet speaks in the name of Jehovah and the
word does not occur or come true, that is the word that Jehovah did not speak..."
If the NWT condemns false prophesying and states that it is proof that God is not speaking
through that prophet, then doesn’t this prove that the Watchtower Bible & Tract Society is not
speaking for God?
2. Why does the New World Translation insert the word Jehovah in the New Testament when there
are absolutely no Greek manuscripts that have it in there? Isn’t this playing with the text?
3. In the book, "Salvation" by J. F. Rutherford, 1939, p. 311, (a Watchtower Publication) it says, "At
San Diego, California, there is a small piece of land, on which, in the year 1929, there was built a
house, which is called and known as Beth-Sarim. The Hebrew words Beth Sarim mean "House of
the Princes"; and the purpose of acquiring that property and building the house was that there
might be some tangible proof that there are those on earth today who fully believe God and
Christ Jesus and in His kingdom, and who believe that the faithful men of old will soon be
resurrected by the Lord, be back on earth, and take charge of the visible affairs of earth. The title
to Beth-Sarim is vested in the Watchtower Bible & Tract Society in trust, to be used by the
president of the Society and his assistants for the present, and thereafter to be forever at the
disposal of the aforementioned princes on earth [italic added]. . . . while the unbelievers have
mocked concerning it and spoken contemptuously of it, yet it stands there as a testimony to
Jehovah’s name; and if and when the princes do return and some of them occupy the property,
such will be a confirmation of the faith and hope that induced the building of Beth-Sarim."
This place was sold in 1942 after Rutherford’s death. Therefore, it appears that the faithful
were misled since the house was to "be forever at the disposal of the aforementioned princes." Is
this really a testimony to Jehovah’s name as it said? How can it be if they sold the house?
4. The Watchtower organization states that Jesus died on a stake, not a cross. The typical
Watchtower representation of this is with Jesus on a single vertical stake, hands over his head
with a single nail in his wrists. If Jesus were crucified on a cross, then two nails would be
necessary, one in each hand. How then does the Watchtower organization handle the verse in the
Bible that states that Jesus had nails (plural) in his hands: "Consequently the other disciples
would say to him: "We have seen the Lord!" But he said to them: "unless I see in his hands the
print of the nails and stick my finger into the print of the nails and stick my hand into his side, I
will certainly not believe" (John 20:25, NWT).
Jesus had one nail in each hand. This is made clear by the use of the word ‘nails’ not ‘nail.’
Jesus must have been crucified on a cross, and not a stake as the Watchtower organization
teaches. Why is it, then, that the Watchtower teaches something that is so clearly unbiblical?
5. The Watchtower organization states that through good works and sincere effort only 144,000 elite
JW’s will go to heaven. The 144,000 are mentioned in two chapters in the Bible: Revelation 7 &
14. By looking at the verses it is obvious that the 144,000 are literal Jews of the ancient tribes
with no Gentiles among them (7:4-8). They are all males (14:4) and virgins (14:4). If the JW
states that the usage of Jewish male virgins is figurative, what gives them the right to state that
number of 144,000 is literal?
6. Where does it teach in the Bible that Jesus is Michael the archangel? Why isn't Jesus called
Michael right now since he is in heaven?
The Lord's Supper and the 144,000 Anointed Class of
Jehovah's Witnesses.

The Jehovah's Witness organization teaches that not all members of their group can take
communion. Only the 144,000 members called the "anointed class" have the right to take Communion
and they are the only ones who go to heaven (Watchtower 2/15/85, page 13). In fact, the 144,000
"anointed class" within the Jehovah's Witnesses are the only ones who are "born again."

"This "little flock" of 144,000 Kingdom heirs, then, are those ones from among mankind who
are "born again." Watchtower 11/15/54, p. 681. Watchtower CD.

The Jehovah's Witnesses celebrate the communion supper only once a year, (Watchtower CD,
Watchtower 4/1/65, p. 199.) In their annual communion service, the congregations gather in their
various kingdom halls. This is done on the 14th of Nissan, the Passover date on the Jewish calendar,
which usually falls in March or April of each year in the Gregorian calendar. During the service, the
cup is passed from Jehovah's Witness to Jehovah's Witness. None of them partake -- except the very
few who are of the 144,000.
In John 6:53-54, Jesus said, “Truly, truly, I say to you, unless you eat the flesh of the Son of Man
and drink His blood, you have no life in yourselves. 54“He who eats My flesh and drinks My blood has
eternal life, and I will raise him up on the last day," (NASB).
I am not here going to discuss the nature of the communion and compare Catholic and Protestant
views of the elements; rather, the issue is that Jesus said to the Jews that they must take of the
communion, which He later instituted in Matt. 26:26-28, and that if they did not partake, then they
had no life in them, v. 6:53.
According to the Watchtower, the "life" spoken of in 6:53 is everlasting life.

"On that occasion Jesus was not speaking to those circumcised Israelites about "everlasting life"
as human creatures on a Paradise earth under his millennial kingdom. Rather, he was speaking
about the very same opportunity that he was setting before the apostle Peter and his fellow
apostles by means of the "sayings of everlasting life." It was the opportunity to gain inherent
life with the Christ in the heavens, "life in yourselves." (John 6:53) By reigning with him in
heaven they could pass on to mankind the life-giving benefits of his sacrifice. Watchtower,
3/1/78, p. 10.

According to Jehovah's Witness theology, some Witnesses will be raised to "immortal life" and
others to "everlasting life." Immortal life is only for the 144,000 that go to be with the Lord in
heaven. They do not have resurrected bodies, but have "spirit bodies." Everlasting life is for those on
Paradise earth and consists of a resurrected body that must be maintained through eating, sleeping,
etc.
As we see in the quote above, according to the Watchtower, the life that Jesus was speaking of was
"everlasting life"; that is, life in heaven for the 144,000. Therefore, the Witnesses teach that the
communion supper is instituted only for the 144,000 Jews, not for all Witnesses.
Since Jesus bought the church with His blood (Acts 20:28), and His blood is the blood of the
covenant for the church (1 Cor. 11:25), then communion is supposed to be for all Christians, not just
144,000 elite Jehovah's Witnesses.

What does the Bible say?

The Communion supper is for the body of Christ, for the believers. The bread and the wine
represent the body and the blood of Christ that was sacrificed for the believer so the believer could
have redemption of his sins. Therefore, Communion is only for those people who have trusted in
Christ's sacrifice. Communion is not for those who are not covered by Christ's sacrifice. This is
significant since the great majority of the Witnesses do not take communion.
Paul the apostle said that the bread is a sharing in the body of Christ, that there is only one body of
Christ (the church) of which we all partake. Within the Christian Church, there is no limitation on the
who takes the Lord's supper.

• "Is not the cup of blessing which we bless a sharing in the blood of Christ? Is not the bread
which we break a sharing in the body of Christ? 17Since there is one bread, we who are
many are one body; for we all partake of the one bread. 18Look at the nation Israel; are not
those who eat the sacrifices sharers in the altar?" (1 Cor. 10:16-18, NASB).
• for"And on the first day of the week, when we were gathered together to break bread, Paul
began talking to them, intending to depart the next day, and he prolonged his message
until midnight," (Acts 20:7, NASB).
• There is no mention of limiting communion to anyone except that they examine themselves
and rightly discern that the elements represent Christ's sacrifice, not a mere meal to satisfy
hunger.
• "But let a man examine himself, and so let him eat of the bread and drink of the cup. 29For
he who eats and drinks, eats and drinks judgment to himself, if he does not judge the body
rightly," (1 Cor. 11:28-29, NASB).

No place in the Bible is communion restricted to only the 144,000, as the Witnesses teach. This is
a fabrication of the Watchtower Bible and Tract Society. The fact that the Jehovah's Witnesses do not
take communion is a demonstration that they are not in the body of Christ and do not have a covenant
relationship with Christ. The Lord's Supper is meant for the body of believers in the true and living
God who have been redeemed by the Lord Jesus Christ. The Lord's Supper is not for those who are
outside the camp of Christ. Since the great majority of the Jehovah's Witnesses do not take
communion, this is a testimony against them.
Are the Jehovah's Witnesses the faithful and discreet slave?
"Watch therefore: for ye know not what hour your Lord doth come. 43But know this, that if the
goodman of the house had known in what watch the thief would come, he would have watched,
and would not have suffered his house to be broken up. 44Therefore be ye also ready: for in
such an hour as ye think not the Son of man cometh. 45Who then is a faithful and wise [NWT =
discreet] servant, whom his lord hath made ruler over his household, to give them meat in due
season? 46Blessed is that servant, whom his lord when he cometh shall find so doing," (Matt.
24:42-46).

The Watchtower Bible and Tract Society, commonly known as the Jehovah's Witnesses, claims to
be the faithful and discreet slave spoken of in Matt. 24:42-46,

• All who want to understand the Bible should appreciate that the "greatly diversified wisdom
of God" can become known only through Jehovah's channel of communication, the faithful
and discreet slave," (The Watchtower; 10/1/94; p. 8).
• "We all need help to understand the Bible, and we cannot find the Scriptural guidance we
need outside the ‘faithful and discreet slave' organization." (The Watchtower, 2/15/81, page
19.)

As "The faithful and discreet slave" it claims that only its organization is capable of rightly
understanding God's word: "Only this organization functions for Jehovah's purpose and to his praise.
To it alone God's Sacred Word, the Bible, is not a sealed book. The Watchtower; July 1, 1973, pp. 402.
The Jehovah's Witnesses are certainly zealous for their beliefs, but they are misguided in those
beliefs. The Watchtower Organization, including all the Jehovah's Witnesses, is not the faithful and
discreet slave spoken of by Jesus in Matt. 24:42-46. The faithful and discreet slave is simply all who
are true Christians.

A faithful slave of God does not make false prophecies

Dismantling the Watchtower claim that it is the faithful and discreet slave is easy. All we need to
do is look at the track record of the Watchtower as an organization and see if it has been faithful.
Consider the following false prophecies made by the Watchtower Bible and Tract Society over the
years.

• 1899 "...the ‘battle of the great day of God Almighty' (Revelation 16:14), which will end in
A.D. 1914 with the complete overthrow of earth's present rulership, is already commenced."
(The Time Is at Hand, page 101 - 1908 edition).
• 1918 "Therefore we may confidently expect that 1925 will mark the return of Abraham,
Isaac, Jacob and the faithful prophets of old, particularly those named by the Apostle in
Hebrews 11, to the condition of human perfection." (Millions Now Living Will Never Die,
page 89.)
• 1922 "The date 1925 is even more distinctly indicated by the Scriptures than 1914." (The
Watchtower 9/1/22, page 262.)
• 1923 "Our thought is, that 1925 is definitely settled by the Scriptures. As to Noah, the
Christian now has much more upon which to base his faith than Noah had upon which to
base his faith in a coming deluge." (The Watchtower, PAGE 106 4/1/23.)
• 1931 "There was a measure of disappointment on the part of Jehovah's faithful ones on
earth concerning the years 1917, 1918, and 1925, which disappointment lasted for a
time...and they also learned to quit fixing dates." (Vindication, page 338.)
• 1941 "Receiving the gift, the marching children clasped it to them, not a toy or plaything
for idle pleasure, but the Lord's provided instrument for most effective work in the
remaining months before Armageddon." (The Watchtower, 9/15/41, page 288.)
• 1968 "True, there have been those in times past who predicted an ‘end to the world', even
announcing a specific date. Yet nothing happened. The ‘end' did not come. They were guilty
of false prophesying. Why? What was missing?...Missing from such people were God's truths
and evidence that he was using and guiding them." (Awake, 10/8/68.)

I agree with the final quote in the Awake magazine. The previous false prophecies of the
Watchtower organization demonstrate that it is lacking God's truths. If it is, then it is not the faithful
and discreet slave.

Charles Taze Russell was called the Faithful and Discreet Slave

Apparently, Charles Taze Russell the founder of the Jehovah's Witnesses was called the "faithful
and discreet slave."

“Thousands of the readers of Pastor Russell's writings believe that he filled the office of 'that
faithful and wise servant,' and that his great work was giving to the household of faith meat in
due season. His modesty and humility precluded him from openly claiming this title, but he
admitted as much in private conversation,” Watchtower 12/1/1916, page. 357.

Which is it? Is the Watchtower right or wrong about Russell? Given that it has made many errors
in the past in its prophecies which proves it does not speak for God, it seems clear that the
Watchtower Organization is clearly not the "faithful and discreet slave" mentioned by Jesus. A slave of
God would not go around changing its doctrines, making false prophecies, or attempt to control the
thinking of its people. The Watchtower Bible and Tract Society is not the faithful and discreet slave.
Salvation according to the Watchtower Organization
According to the Watchtower Organization, Feb. 15, 1983, p. 12, there are four requirements for
salvation as taught by the Watchtower magazine. Please check out the list.

1. Jesus Christ identified a first requirement when he said in prayer to his Father: "This means
everlasting life, their taking in knowledge of you, the only true God, and of the one whom you
sent forth, Jesus Christ." (John 17:3) Knowledge of God and of Jesus Christ includes knowledge of
God’s purposes regarding the earth and of Christ’s role as earth’s new King. Will you take in such
knowledge by studying the Bible?
2. Many have found the second requirement more difficult. It is to obey God’s laws, yes, to conform
one’s life to the moral requirements set out in the Bible. This includes refraining from a
debauched, immoral way of life. — 1 Corinthians 6:9, 10; 1 Peter 4:3, 4.
3. A third requirement is that we be associated with God’s channel, his organization. God has always
used an organization. For example, only those in the ark in Noah’s day survived the Flood, and
only those associated with the Christian congregation in the first century had God’s favor. (Acts
4:12) Similarly, Jehovah is using only one organization today to accomplish his will. To receive
everlasting life in the earthly Paradise we must identify that organization and serve God as part of
it.
4. The fourth requirement is connected with loyalty. God requires that prospective subjects of his
Kingdom support his government by loyally advocating his Kingdom rule to others. Jesus Christ
explained: "This good news of the kingdom will be preached in all the inhabited earth." (Matthew
24:14) Will you meet this requirement by telling others about God’s Kingdom?

According to the Bible, salvation is obtained by faith in Christ and what He did on the cross. That is
why the Bible says we are justified by faith (Rom. 5:1). Justification is the legal declaration by God
where He declares the sinner righteous. This is based on the substitutionary atonement of Jesus, on
the cross, in our place.

• 2 Cor. 5:21, "For he hath made him to be sin for us, who knew no sin; that we might be
made the righteousness of God in him."
• 1 Pet. 2:24, "Who his own self bare our sins in his own body on the tree, that we, being
dead to sins, should live unto righteousness: by whose stripes ye were healed."

Because Jesus took our place and satisfied all the law, we are not obligated to keep the law in any
way in order to be justified/saved. His righteousness, His fulfilling of the law is reckoned to us, is
given to us by faith. That is why Jesus says He gives us rest (Matt. 11:28). It is rest from trying to
earn our salvation in any way. As Christians, we are saved by grace through faith (Eph. 2:8-9)
because we can never please God by any work we do (Isa. 64:6; Gal. 2:21). Only through Jesus and
His work on the cross are we saved.
When the Galatian Christians sought to be circumcised, they were essentially trying to complete
their salvation by a work of law. To this Paul said, "And I testify again to every man who receives
circumcision, that he is under obligation to keep the whole Law. 4You have been severed from Christ,
you who are seeking to be justified by law; you have fallen from grace," (Gal. 5:3-4). Why? Because
Rom. 11:6 says, "But if it is by grace, it is no longer on the basis of works, otherwise grace is no
longer grace."
Unfortunately for the Jehovah's Witnesses, the Watchtower Organization is teaching that in order to
be saved you must become 1) knowledgeable about the Father, 2) obey God's laws, 3) be associated
with the Jehovah's Witnesses, and 4) be loyal in spreading the kingdom news of God.
Please take note that the Watchtower does not teach about coming to Christ as Jesus stated in
Matt. 11:28. It does not focus on the sacrifice of Christ on the cross and what Jesus did. It does not
mention what saves us in 1 Cor. 15:1-4; namely, the death, burial, and resurrection of Jesus.
Instead, it focuses on the Father, itself as an organization, and works. It is the works issue that is
the most problematic here. The Jehovah's Witness organization teaches works righteousness in
combination with the sacrifice of Christ. This combination of our works and Jesus' sacrifice is a
teaching that is not only a heresy, but also makes salvation void.

Works do not save us

• Rom. 3:28, "Therefore we conclude that a man is justified by faith without the deeds of the
law."
• Gal. 2:16, "nevertheless knowing that a man is not justified by the works of the Law but
through faith in Christ Jesus..."
• See also, Rom. 4:5; 5:1; Gal. 3:11; Phil. 3:9.

Again as to quote the Watchtower above, "Many have found the second requirement more difficult.
It is to obey God’s laws, yes, to conform one’s life to the moral requirements set out in the Bible." And
if this isn't enough, please consider the following quote:

"Accepting the message of salvation and devoting ourselves to God through Christ and being
baptized in water is only the beginning of our exercise of faith. It is only the beginning of our
obedience to God. It sets us on the way to everlasting life, but it does not mean our final
salvation," (This Means Everlasting Life, p. 181.")

As you can see, the Watchtower Organization teaches that salvation is dependent upon the work of
the Jehovah's Witnesses and this is a doctrine that leads to damnation.

Are you doing enough works?

If you are a Jehovah's Witnesses, then are you doing enough works to be saved? Are all your
hours going door to door really going to please God enough for Him to let you into Paradise Earth? Are
you actually obeying God's laws? If not, why not?
Do you think that sincerity will make up the difference? If so, you're in for a rude awakening.

"Not everyone who says to Me, ‘Lord, Lord,’ will enter the kingdom of heaven; but he who does
the will of My Father who is in heaven. 22Many will say to Me on that day, ‘Lord, Lord, did we
not prophesy in Your name, and in Your name cast out demons, and in Your name perform
many miracles?’ 23"And then I will declare to them, ‘I never knew you; depart from Me, you
who practice lawlessness,’ (Matt. 7:21-23).

Notice how those condemned on the day of judgment appealed to their works that they did in
Jesus' name. In other words, they combined their faith and deeds in their appeal to Jesus for
salvation. It was not by faith alone! Is that what you are doing? Are you relying on the complete
grace of God in Christ alone or are you hoping to be able prove to God you are a worthy Jehovah's
Witness by your deeds of obedience? You can never be made worthy by anything you do. All our
deeds are filthy rags before the Lord (Isa. 64:6). That is why true Christians are justified (saved) by
faith (Rom. 5:1).
True Christianity is the teaching that Jesus is God in flesh (John 1:1,14; Col. 2:9) and because He
is, His sacrifice is infinite in value. It is completely sufficient to save us from our sins, and does so. All
we need to do is believe and by faith we are justified (Rom. 5:1; Eph. 2:8-9). It is after we are saved
that we then go out and do good works and keep His commandments. We do them because we are
saved, not to get saved or to keep our salvation.
Are you good enough? Is your heart good enough? Are you sincere enough? Have you put
enough hours going door-to-door? .... do you need a rest? If so, come to Jesus. Ask Him to forgive
you of your sins.

• Matt. 28:18, "And Jesus came up and spoke to them, saying, "All authority has been given
to Me in heaven and on earth."
• John 14:14, "If you ask Me anything in My name, I will do it."
• Matt. 11:28-30, "Come to Me, all who are weary and heavy-laden, and I will give you rest.
29
"Take My yoke upon you, and learn from Me, for I am gentle and humble in heart; and
you shall find rest for your souls. 30"For My yoke is easy, and My load is light."

The New World Translation and "Proskuneo" (worship)


The chart below is a list of every occurrence of the Greek word "proskuneo" that I could find in The
Kingdom Interlinear Translation of the Greek Scriptures (1969). This work was produced by the New
World Translation Bible committee in accordance with the Watchtower organization. An interlinear is a
word-for-word translation of the Greek text. Included in the Watchtower interlinear is the text of the
New World Translation Bible placed in a column on each page so the reader can see exactly how the
New World Translation (NWT) renders each occurrence of the verb "proskuneo."
The word "proskuneo" means "to kiss the hand, bow down before, show obeisance, to worship."
Since the Jehovah's Witnesses deny that Jesus is God, they maintain that He is not to be worshipped.
So, how does the Watchtower Bible and Tract Society translate the word "proskuneo" in their Greek
New Testament in reference to Jesus and other objects of adoration? The NWT never translates the
word into "worship" when it references Jesus. It does, however, render the word as "worship" in
regards to the devil, the dragon, the beast, the image, demons, idols, and an angel. Of course, they
correctly translate it as "worship" when it deals with God.
Is the New World Translation biased against worshipping Jesus? It would surely seem so,
especially since the NASB, NIV, KJV, NKJV, etc., all translate the word as "worship" in reference to
Jesus.
What is curious is to note that the Watchtower Bible and Tract Society used to teach that
worshipping Jesus was acceptable:

"The fact that our Lord received worship is claimed by some to be an evidence that while on
earth he was God the Father disguised in a body of flesh and not really a man. Was he really
worshiped, or is the translation faulty? Yes we believe our Lord Jesus while on earth was really
worshiped, and properly so. While he was not the God, Jehovah, he was a God. The word ‘God’
signifies a ‘mighty one,’ and our Lord was indeed a mighty one. So it is stated in the first two
verses of the gospel of John. It was proper for our Lord to receive worship in view of his having
been the only begotten of the Father. . ." (The Watchtower, July 15, 1898, p. 216.)

So, it seems that the Watchtower Bible and Tract Society is inconsistent in its teaching over the
years. Its theology has changed as has its teaching. Therefore, since they needed a Bible consistent
with those changes, they produced a Bible that reflects their theological bias: The New World
Translation.
Following is a chart listing every appearance of the word "proskuneo" in the New World
Translation. Let's see if the bias of the Watchtower organization is obvious.

Verse is dragon,
referring Jesus God Demons beast, Devil Generic Idols Peter angel
to image
Matt. 2:2 obeisance
Matt. 2:8 obeisance
Matt. 2:11 obeisance
Matt. 4:9 worship
Matt. 4:10 worship
Matt. 8:2 obeisance
Matt. 9:18 obeisance
Matt. obeisance
14:33
Verse is dragon,
referring Jesus God Demons beast, Devil Generic Idols Peter angel
to image
Matt.
obeisance
15:25
Matt.
obeisance
18:26
Matt.
obeisance
20:20
Matt. 28:9 obeisance
Matt.
obeisance
28:17
Mark 5:6 obeisance
Mark
obeisance
15:19
Luke 4:7 worship
Luke 4:8 worship
Luke
obeisance
24:52
John 4:20 worship
John 4:21 worship
John 4:22 worship
John 4:23 worship
John 4:23 worship
John 4:24 worship
John 9:38 obeisance
John
worship
12:20
Acts 7:43 worship
Acts 8:27 worship
Acts 10:25 obeisance

Acts 24:11 worship

1 Cor.
worship
14:25
Heb. 1:6 obeisance
Heb.
worship
11:21
Rev. 3:9 obeisance
Rev. 4:10 worship
Verse is dragon,
referring Jesus God Demons beast, Devil Generic Idols Peter angel
to image
Rev. 5:14 worship
Rev. 7:11 worship
Rev. 9:20 worship
Rev. 11:1 worship
Rev.
worship
11:16
Rev. 13:4 worship
Rev. 13:8 worship
Rev.
worship
13:12
Rev.
worship
13:15
Rev. 14:7 worship
Rev. 14:9 worship
Rev.
worship
14:11
Rev. 15:4 worship
Rev. 16:2 worship
Rev. 19:4 worship
Rev.
worship
19:10
Rev.
worship
19:20
Rev. 20:4 worship
Rev. 22:8 worship
Rev. 22:9 worship

As you can see, the NWT is very bias in how it translates the word "proskuneo." Whenever it is in
reference to Jesus, they absolutely will not let it be translated as worship. Why? Because they
erringly deny that Jesus is God in flesh and their Bible reflects their bias. This is not how proper
theology is done.
The Watchtower Organization has changed the Bible to suit its needs.
Bad Translations of the Jehovah's Witness Bible, the New World
Translation (NWT).
1. Gen. 1:1-2 - "In [the] beginning God created the heavens and the earth.
Now the earth proved to be formless and waste and there was darkness upon the surface of [the]
watery deep; and God's active force was moving to and fro over the surface of the waters."
(New World Translation, Emphasis added)
A. The Watchtower Bible and Tract Society denies that the Holy Spirit is alive, third person of
the Trinity. Therefore, they have changed the correct translation of "...the Spirit of God was
moving over the surface of the waters," to say "...and God's active force was moving to
and fro over the surface of the waters."
2. Zech. 12:10 - In this verse God is speaking and says "And I will pour out on the house of David
and the inhabitants of Jerusalem a spirit of grace and supplication. They will look on me, the one
they have pierced, and they will mourn for him as one mourns for an only child, and grieve
bitterly for him as one grieves for a firstborn son" (Zech. 12:10, NASB).
A. The Jehovah's Witnesses change the word "me" to "the one" so that it says in their Bible,
"...they will look upon the one whom they have pierced..."
Since the Jehovah's Witnesses deny that Jesus is God in flesh, then Zech. 12:10 would
present obvious problems--so they changed it.
3. John 1:1 - They mistranslate the verse as "a god." Again it is because they deny who Jesus is
and must change the Bible to make it agree with their theology. The Jehovah's Witness version is
this: "In the beginning was the Word, and the Word was with God, and the Word was a god."
4. Col. 1:15-17 - The word "other" is inserted 4 times. It is not in the original Greek, nor is it
implied. This is a section where Jesus is described as being the creator of all things. Since the
Jehovah's Witness organization believes that Jesus is created, they have inserted the word
"other" to show that Jesus was before all "other" things, implying that He is created.
A. There are two Greek words for "other": heteros, and allos. The first means another of a
different kind, and the second means another of the same kind. Neither is used at all in this
section of scripture. The Jehovah's Witness have changed the Bible to make it fit their
aberrant theology.
5. Heb. 1:6 - In this verse they translate the Greek word for worship, proskuneo, as "obeisance."
Obeisance is a word that means to honor, show respect, even bow down before someone. Since
Jesus, to them, is created, then he cannot be worshiped. They have also done this in other verses
concerning Jesus, i.e., Matt. 2:2,11; 14:33; 28:9.
6. Heb. 1:8 - This is a verse where God the Father is calling Jesus God: "But about the Son he says,
‘Your throne, O God, will last for ever and ever, and righteousness will be the scepter of your
kingdom.'" Since the Jehovah's Witnesses don't agree with that they have changed the Bible, yet
again, to agree with their theology. They have translated the verse as "...God is your throne..."
The problem with the Jehovah's Witness translation is that this verse is a quote from Psalm 45:6
which, from the Hebrew, can only be translated as "...Your throne, O God, will last for ever and
ever; a scepter of justice will be the scepter of your kingdom." To justify their New Testament
translation they actually changed the OT verse to agree with their theology, too!

The NWT translation is not a good translation. It has changed the text to suit its own theological bias
in many places.
Jehovah's Witnesses and Mental Health

Please understand that this article is not mean as a put-down of the Jehovah's Witnesses. I am
simply displaying research information which raises serious questions about the Watchtower
Organization.
It has always been a belief of mine that cult groups put an excessive burden of legalism upon their
adherents -- this is because they do not have a proper understanding of grace due to their lack of
understanding of who God is and what He has done for us. Often these aberrant groups require
substantial commitments of time and energy from their membership in order to maintain a good
standing in the group. Since cults are typically short on grace and long on law (mixed with group
obligations, guilt-inducing teachings, and isolationism), I have always assumed that this unnecessary
difficulty would lead to emotional and mental problems.
A few years ago, I heard of an article in a mental health journal that documented the population
percentages of Jehovah's Witnesses in mental wards. It took some effort, but I found it. Following
are excerpts from that article. Judge for yourself if the Jehovah's Witness organization contributed to
the demise of some of its members.
(The following quotes are taken from the British Journal of Psychiatry: the Journal of Mental
Science. Published by authority of The Royal College of Psychiatrists, Vol. 126, Ashford, Kent, Headley
Brothers LTD, 1975. The author is John Spencer.)

"During the period of 36 months from January 1971 to December 1973 there were 7,546 inpatient
admissions to the West Australian Mental Health Service Psychiatric Hospitals. Of these 50 were
reported to be active members of the Jehovah's Witnesses movement" (p. 557).
"Of the 50 admitted 22 were diagnosed as schizophrenic, 17 as paranoid schizophrenic, 10 as
neurotic and one as alcoholic" (pp. 557, 558).

Annual rate per Jehovah's Annual rate per


Total
1,000 Witnesses 1,000
admissions
population admissions population
All diagnoses 7,546 2.54 50 4.17
Schizophrenia
1,826 .61 22 1.83
(295)
Paranoid
schizophrenia 1,154 .38 17 1.4
(195.3)
Neurosis (300) 1,182 .39 10 .76

"From the figures gathered in the Table it is clear that members of the Jehovah's Witnesses
movement are over-represented in admissions to the Mental Health Services of this State.
Furthermore, it is clear from the Table that the incidence of schizophrenia amongst them is about
three times as high as for the rest of the general population, while the figure for paranoid
schizophrenia is nearly four times that of the general population" (p. 558).
"The study does not shed light on the question of symptom or defense mechanism, but suggests
that either the Jehovah's Witnesses sect tends to attract an excess of pre-psychotic individuals who
may then break down, or else being a Jehovah's Witness is itself a stress which may precipitate a
psychosis" (p. 558).
A Biblical Response to Jehovah's Witnesses

1. Their attacks on the Deity of Jesus.


A. Why did Jesus pray to the Father? (John 17).
i. Because as a man He needed to pray to the Father.
ii. Because He was both God and man (Col. 2:9; John 8:58 with Ex. 3:14).
a. The two natures of Christ are why we have two types of scripture concerning Jesus:
those that seem to focus on His divine-side, and those that seem to focus on His
human-side. The Jehovah's Witnesses are simply ignoring, or changing, the divine-
side scriptures and concentrating on those that describe His human-side.
B. Why did He say the Father was greater than He (John 14:28)?
i. This is because His position was different than that of God, not His nature.
ii. Heb. 2:9 that Jesus is made for a little while lower than the angels; that is, when He
became a man. The Father sent the Son (1 John 4:10).
C. Why did He say, "Why call me good, only God is good?" (Luke 18:19)?
i. Jesus was confirming His own deity because what He was doing was good.
ii. Ask them, "Was Jesus good?"
D. Why did Jesus say that He could only do those things that He saw the Father do? (John
5:19).
i. This is an interesting verse and it is one that proves the divinity of Christ, not that He
wasn't God.
ii. Ask the Jehovah's Witness who can do the same things God the Father can do? Could
an angel? Could a man? Of course not. Jesus, however, says He could do whatever He
saw the Father do. "I tell you the truth, the Son can do nothing by himself; he can do
only what he sees his Father doing, because whatever the Father does the Son also
does."
E.The answer to these and other verses like them is that Jesus has two natures. Jesus was fully
man as well as fully God and as a man there will be verses that show His humanity.
2. Witnessing Approaches using the Bible.
A. John 1:1: They translate as "In the beginning was the word and the word was with God
and the word was a god."
i. Ask if Satan is a true god or a false God. The Jehovah's Witness will say a false god.
Then have them read aloud John 1:1 again in their Bible and ask them if Jesus is a true
god or a false one. If he says "true god," he's in trouble. If he says "false god," he's in
trouble.
ii. If Jesus is a god, then doesn't that mean there are two gods? They often answer, "Yes.
But Jesus is not the Almighty God, He is only the mighty god. And besides, there are
those in the Bible who are called gods but really aren't."
iii. The problem with this is that every God besides Jehovah is a false God. God says to
have no other God before Him (Exodus 20:3) because they are not by nature gods (Gal.
4:8).
B. Col. 1:15: Is used by the Jehovah's Witnesses to say that Jesus is the first created thing.
This verse says, "He [Jesus] is the image of the invisible God, the firstborn over all
creation."
i. The Jehovah's Witnesses maintain that "firstborn" means first created. This cannot be
the case because...
a. There is a Greek word for "first created" and it is not used here.
b. "First born" is proto, "first," with tikto "to bring forth, bear, produce."
c. There is no word used in the New Testament for "first created." However, if there
were, the construction would be proto, "first," with ktizo "to create." And this is not
the construction used in Col. 1:1
ii. Firstborn can certainly mean the first one born in a family. However, it can also mean
preeminence. For example:
a. In Jeremiah 31:9, the firstborn title is attributed to one of the tribes of northern
Israel. "They will come with weeping; they will pray as I bring them back. I will lead
them beside streams of water on a level path where they will not stumble, because
I am Israel's father, and Ephraim is my firstborn son."
iii. Understanding biblical culture is important when interpreting Scripture. Firstborn was a
title, not only of the first born male, but also of preeminence which is precisely what is
occurring when it is said that Jesus is the firstborn.
3. Col. 1:15-17 in the Jehovah's Witness Bible has an addition of four words. Their version reads,
"He is the image of the invisible God, the firstborn of all creation; because by means of him all
[other] things were created in the heavens and upon the earth, the things visible and the things
invisible, no matter whether they are thrones or lordships or governments or authorities. All
[other] things have been created through him and for him. Also, he is before all [other] things
and by means of him all [other] things were made to exist," (Their word "[other]" is in their Bible
with the brackets. They maintain that they know it isn't in the original Greek Scriptures but the
word is implied and should be there.)
A. Instead of refuting the bad translation, simply ask them if this means that Jesus created
everything. They will say yes. Review this and be very clear and get them to admit that it
was Jesus who created everything. Then turn to...
i. Isaiah 44:24 "This is what the LORD says -your Redeemer, who formed you in the
womb: I am the LORD, who has made all things, who alone stretched out the heavens,
who spread out the earth by myself."
ii. If Jesus created everything, then why does it say that the Lord (Jehovah in the Hebrew)
did it by Himself?
iii. The only answer is that Jehovah is not simply the name of the Father, but that it is the
name of God the Trinity. Therefore, since Jesus is God in flesh, it could be said that
Jesus created all things and that Jehovah did it alone.
B. You can also ask them to try to read the section of verses and omit the word "other". You
will find it to be an interesting experience.
4. John 8:58 in the Jehovah's Witness Bible says, "...Before Abraham came into existence, I have
been."
A. They have translated the present tense ego eimi, in the Greek, into the perfect tense, I
have been. Though this can be done rarely in the New Testament, it is not correct here
because Jesus was quoting the O.T. verse of Exodus 3:14 where God was telling Moses who
He was: "God said to Moses, ‘I AM WHO I AM. This is what you are to say to the Israelites:
"I AM has sent me to you."'" Jesus was purposely using the divine title: I AM.
B. The Jehovah's Witness won't agree. So ask him if Jesus was saying that He "had been"
before Abraham, then why does it say in the next verse that the Jews pick up stones to kill
him?
C. Additionally, about 250 years the Jews translated the Hebrew Scriptures into Greek. It is
called the Septuagint, also known as LXX. In the Septuagint Exodus 3:14 is translated in the
Greek in a present tense, i.e., I AM... The correct translation is, therefore, "Before Abraham
was, I AM."
D. If this verse should really be translated as "I have been" then why did the Jews want to kill
Jesus? The answer is simple: They knew He was claiming to be God, see the next example.
5. John 10:30-34 is a section of verses where the Pharisees say that Jesus is making Himself out
to be God (v. 33).
A. "I and the Father are one." Again the Jews picked up stones to stone him, but Jesus said to
them, "I have shown you many great miracles from the Father. For which of these do you
stone me?" "We are not stoning you for any of these," replied the Jews, "but for blasphemy,
because you, a mere man, claim to be God."
B. You can say, "See, even the Jews knew He was claiming to be God. The Jehovah's Witness
(if he's quick enough) will say something like, "Jesus wasn't God, the Jew's only thought
that Jesus was claiming to be God." Then you can say, "Oh, I see. Then let me get this
right. You agree with the Pharisees, Jesus wasn't God? Is that correct? The Jehovah's
Witness will not like it that he agrees with a Pharisee.
6. Plurality in the Godhead
A. The following group of scriptures strongly suggests a plurality within the Godhead. These
verses are translated correctly in the Jehovah's Witness Bible so you can encourage them to
use it. The NIV is not as literal in its translation in the Amos verses, so I recommend using
either the King James or the New American Standard Bible when doing your own.
i. Gen. 1:26, "Then God said, "Let Us make man in Our image, according to Our
likeness . . . "
a. They will say that angels are the ones who helped God make man. However, there
is no scriptural evidence for that. God is the only creator.
b. You can also take him to Col. 1:15-17 where it says that Jesus is the creator of all
things--including man.
ii. Gen. 19:24, "Then the LORD rained on Sodom and Gomorrah brimstone and fire from
the LORD out of heaven."
a. Is this saying there are two Lords, two Jehovah's?
iii. Amos 4:10-11, "‘I sent a plague among you after the manner of Egypt; I slew your
young men by the sword along with your captured horses, and I made the stench of
your camp rise up in your nostrils; yet you have not returned to Me,' declares the
LORD. ‘I overthrew you as God overthrew Sodom and Gomorrah . . . '"
a. Jehovah is the one talking and He says, "I overthrew you as God overthrew Sodom
and Gomorrah..." Very interesting.
iv. Isaiah 44:6, "Thus says the LORD, the King of Israel and his Redeemer, the LORD of
hosts: ‘I am the first and I am the last, and there is no God besides me . . . ‘" See also,
Isaiah 48:1
v. If you are reading these verses to a Jehovah's Witness he might say something like,
"Are you trying to show the Trinity from these verses?" You can then say, "You got the
Trinity out of these? That's very interesting."
7. John 20:25 says, "The other disciples therefore were saying to him, ‘We have seen the Lord!'
But he said to them, ‘Unless I shall see in His hands the imprint of the nails, and put my finger
into the place of the nails, and put my hand into His side, I will not believe'" (NASB).
A. The Jehovah's Witnesses deny that Jesus was crucified on a cross. They say it happened on
a torture stake where His wrists were put together over His head and a single nail was put
through both. If that is true, then why does Thomas say "Unless I shall see in His hands the
imprint of the nails..." In the Greek the word used here for "nails", helos, is in the plural.
Therefore, there was more than one nail used in the hands of the crucifixion of Christ.
8. First and Last
A. How many firsts and lasts are there? In the Bible God is called the first and last and so is
Jesus. Since God says there is no God apart from Him and Jesus and God are both
addressed by the same title, then that poses a problem for the Jehovah's Witness.
i. Isaiah 44:6, "This is what the LORD says -Israel's King and Redeemer, the LORD
Almighty: I am the first and I am the last; apart from me there is no God."
ii. Revelation 1:8, "I am the Alpha and the Omega," says the Lord God, "who is, and who
was, and who is to come, the Almighty."
iii. Revelation 1:17-18, "When I saw him, I fell at his feet as though dead. Then he placed
his right hand on me and said: "Do not be afraid. I am the First and the Last. I am the
Living One; I was dead, and behold I am alive for ever and ever! And I hold the keys of
death and Hades."
a. Obviously, Rev. 1:17-18 can only refer to Jesus.
iv. Revelation 22:12-13, "Behold, I am coming soon! My reward is with me, and I will give
to everyone according to what he has done. I am the Alpha and the Omega, the First
and the Last, the Beginning and the End."
a. Here, both the "Alpha and the Omega" and the "First and the Last" are said to be
one and the same.
b. Also, at this point go to Titus 2:13 where it says that Jesus is the one who is coming
soon, therefore, Jesus and Jehovah are the same.

9. The Holy Spirit


A. Jehovah's Witnesses teach that the Holy Spirit is an active force like radar. They deny that
He is alive, that He is a person. This is, of course, because they deny the Trinity. Yet, if the
Holy Spirit is simply a force then...
i. Why is He called God (Acts 5:3-5)?
ii. How is it that He can teach (John 14:26)?
iii. How can He be blasphemed (Matt. 12:31,32)?
iv. How can be the one who comforts (Acts 9:31)?
v. How is it possible for Him to speak (Acts 28:25)?
vi. How then can He be resisted (Acts 7:51)?
vii. How can He be grieved (Eph. 4:30)?
viii. How can He help us in our weaknesses (Rom. 8:26)?
B. If the Holy Spirit is a force, then how is it possible that the above mentioned phenomena
are attributed to Him? A force doesn't speak, teach, comfort, etc.
C. Nor can you blaspheme against a force.
10. The Resurrection of Jesus
A. The Jehovah's Witnesses deny the physical resurrection of Jesus. They say that if the
sacrifice of Jesus were real then the body had to stay in the grave. They say that He rose in
a spirit body. This body was a manifestation similar to the way angels manifested
themselves in the Old Testament.
i. The problem with their view is that the angels were not incarnated. Jesus became a
man by birth, therefore, He had a real, physical body, a permanent body. In fact, right
now, Jesus is in heaven in the form of a man. He still has two natures, God and man,
and will eternally be that.
B. For scriptural proof of Jesus being raised in the same body He died in, consider the following
verses.
i. In John 2:19-22 before the crucifixion Jesus said, "Destroy this temple and in three
days I will raise it up...He was speaking of the temple of His body." Since Jesus said He
would raise the same body He died in, then it must be true.
a. This last verse is worth focusing on. Remember, Jesus said He would be the one to
raise His body. So, it must be true.
ii. John 20:27 -(to Thomas) "reach your finger...and put it into My side..."
a. If Jesus were not raised from the dead, then why did He have a physical body.
b. They will reply that it was a temporary body materialized so the apostles would
believe that He was raised. Yet, this is not what Jesus said in John 2:19-22. He said
He would raise His very body.
iii. Luke 24:39 - "a spirit does not have flesh and bones as you see I have."
a. Jesus said that He had "flesh and bones" not "flesh and blood." This is important
because flesh and blood cannot inherit the kingdom of God (1 Cor. 15:50). The
blood of Jesus was the sacrifice for sin (Rom. 5:9). It is the blood that cleanses us
of our sin (Heb. 9:22).
b. The blood of Jesus was shed on the cross and so, most probably, Jesus doesn't have
any functioning blood in His body.
11. Similarities between the Jehovah's Witnesses and the Pharisees:
A. Both deny the Trinity and the Deity of Christ
B. Both deny the physical resurrection of Christ and salvation by grace alone.
Interesting Quotes from Watchtower Literature
It is important to understand the psychological hold the Watchtower organization has in the life of
the average Jehovah's Witness. The Watchtower organization is the guide, the teacher, and the
expounder of correct doctrine. The average Jehovah's Witness attends several meetings each week
where he is repeatedly indoctrinated to believe Watchtower doctrines. Every Jehovah's Witness thinks
very much alike and has the same standard answers because they read from the same sources and
are conditioned into the same way of thinking: the Watchtower way. So, if you've witnessed to one
Jehovah's Witness, you've heard the same arguments they all will use.
In opposition to the Watchtower, according to the Bible, Jesus is the mediator between God and
man (1 Tim. 2:5). He alone is the one who reveals truth (John 1:17), not the Watchtower
organization. As you will read in these quotes, the Watchtower organization subtly takes the place of
Jesus. Though it claims to bear witness of Him and point to Him, in reality it takes His place. This is
typical for a cult.

1. The Watchtower Organization is God's organization on earth.


A. "It should be expected that the Lord would have a means of communication to his people on
the earth, and he has clearly shown that the magazine called The Watchtower is used for
that purpose." (1939 Yearbook of Jehovah's Witnesses, p. 85.)
B. "Make haste to identify the visible theocratic organization of God that represents his king,
Jesus Christ. It is essential for life. Doing so, be complete in accepting its every aspect."
The Watchtower, October 1, 1967, p. 591.
C. "We cannot claim to love God, yet deny his word and channel of communication." The
Watchtower, October 1, 1967, p. 591.
D. No matter where we may live on earth, God's Word continues to serve as a light to our path
and a lamp to our roadway as to our conduct and beliefs. (Ps. 119:105) But Jehovah God
has also provided his visible organization, his "faithful and discreet slave," made up of spirit
anointed ones, to help Christians in all nations to understand and apply properly the Bible in
their lives. Unless we are in touch with this channel of communication that God is using, we
will not progress along the road to life, no matter how much Bible reading we do. The
Watchtower; 12/1/1981, p. 27.
2. The Watchtower Organization is the only way to understand the Bible.
A. Only this organization functions for Jehovah's purpose and to his praise. To it alone God's
Sacred Word, the Bible, is not a sealed book. The Watchtower; July 1, 1973, pp. 402.
B. "We all need help to understand the Bible, and we cannot find the Scriptural guidance we
need outside the ‘faithful and discreet slave' organization." (The Watchtower, Feb. 15,
1981.)
C. "...people cannot see the Divine Plan in studying the Bible by itself...if he then lays them
[Scripture Studies] aside and ignores them and goes to the bible alone, though he has
understood his Bible for ten years, our experience shows that within two years he goes into
darkness. On the other hand, if he had merely read the Scripture Studies with their
references, and had not read a page of the Bible, as such, he would be in the light at the
end of the two years, because he would have the light of the Scriptures." The Watchtower,
Sept. 15, 1910, p. 298.
D. "Thus the Bible is an organizational book and belongs to the Christian congregation as an
organization, not to individuals, regardless of how sincerely they may believe that they can
interpret the Bible." The Watchtower, Oct. 1, 1967. p. 587.
E. “The world is full of Bibles, which Book contains the commandments of God. Why, then,
do the people not know which way to go? Because they do not also have the teaching or law
of the mother, which is light. Jehovah God has provided his holy written Word for all
mankind and it contains all the information that is needed for men in taking a course
leading to life. But God has not arranged for that Word to speak independently or to shine
forth life-giving truths by itself. His Word says: "Light is sown for the righteous." (Ps. 97:11)
It is through his organization that God provides this light that the proverb says is the
teaching or law of the mother. If we are to walk in the light of truth we must recognize not
only Jehovah God as our Father but his organization as our mother." The Watchtower, May
1, 1957, p. 274.
i. Channel to understanding the Bible... All who want to understand the Bible should
appreciate that the "greatly diversified wisdom of God" can become known only through
Jehovah's channel of communication, the faithful and discreet slave. The Watchtower;
10/1/1994; p. 8.
3. Individual thinking discouraged by Watchtower Organization
A. "From time to time, there have arisen from among the ranks of Jehovah's people those,
who, like the original Satan, have adopted an independent, faultfinding attitude...They say
that it is sufficient to read the Bible exclusively, either alone or in small groups at home.
But, strangely, through such ‘Bible reading,' they have reverted right back to the apostate
doctrines that commentaries by Christendom's clergy were teaching 100 years ago..." The
Watchtower, August 15, 1981.
B. "We should eat and digest and assimilate what is set before us, without shying away from
parts of the food because it may not suit the fancy of our mental taste...We should meekly
go along with the Lord's theocratic organization and wait for further clarification, rather than
balk at the first mention of a thought unpalatable to us and proceed to quibble and mouth
our criticisms and opinions as though they were worth more than the slave's provision of
spiritual food. Theocratic ones will appreciate the Lord's visible organization and not be so
foolish as to put against Jehovah's channel their own human reasoning and sentiment and
personal feelings." The Watchtower, February 1, 1952, pp. 79-80.
C. "After being nourished to our present spiritual strength and maturity, do we suddenly
become smarter than our former provider and forsake the enlightening guidance of the
organization that mothers us? ‘Forsake not the law of thy mother' (Prov. 6:20-23)." The
Watchtower, February 1, 1952, p. 80.
4. Regarding Salvation
A. There are four requirements for the Jehovah's Witnesses to be able to live forever on
Paradise earth according to the Watchtower, Feb. 15, 1983, p. 12. One of them deals with
the Watchtower organization.
i. "Jesus Christ identified a first requirement when he said in prayer to his Father: "This
means everlasting life, their taking in knowledge of you, the only true God, and of the
one whom you sent forth, Jesus Christ." (John 17:3) Knowledge of God and of Jesus
Christ includes knowledge of God’s purposes regarding the earth and of Christ’s role as
earth’s new King. Will you take in such knowledge by studying the Bible?
ii. "Many have found the second requirement more difficult. It is to obey God’s laws, yes,
to conform one’s life to the moral requirements set out in the Bible. This includes
refraining from a debauched, immoral way of life. — 1 Corinthians 6:9, 10; 1 Peter 4:3,
4.
a. ["Accepting the message of salvation and devoting ourselves to God through Christ
and being baptized in water is only the beginning of our exercise of faith. It is only
the beginning of our obedience to God. It sets us on the way to everlasting life, but
it does not mean our final salvation" This Means Everlasting Life, p. 181."]
iii. "A third requirement is that we be associated with God’s channel, his organization. God
has always used an organization. For example, only those in the ark in Noah’s day
survived the Flood, and only those associated with the Christian congregation in the
first century had God’s favor. (Acts 4:12) Similarly, Jehovah is using only one
organization today to accomplish his will. To receive everlasting life in the earthly
Paradise we must identify that organization and serve God as part of it.
iv. "The fourth requirement is connected with loyalty. God requires that prospective
subjects of his Kingdom support his government by loyally advocating his Kingdom rule
to others. Jesus Christ explained: "This good news of the kingdom will be preached in
all the inhabited earth," (Matthew 24:14) Will you meet this requirement by telling
others about God’s Kingdom?
Regarding the Trinity, Jesus, Adam, and immortality of the soul

1. Regarding the Trinity


A. "Never was there a more deceptive doctrine advanced than that of the trinity. It could have
originated only in one mind, and that the mind of Satan the Devil." Reconciliation, 1928, p.
101.
B. "The doctrine, in brief, is that there are three gods in one: ‘God the Father, God the Son,
and God the Holy Ghost,; all three equal in power, substance and eternity." Let God Be
True, 1952, p. 100. (Here they misrepresent the doctrine of the Trinity.)
C. "Ask the student, ‘How many Jehovah's are there?' Let him answer. The answer is obvious
that there is only one Jehovah...If he is one Jehovah, then could he be three gods, God the
Father, God the Son and God the Holy Ghost, as the Trinitarians teach?" The Watchtower,
April 1, 1970. p. 210. (Again, the Watchtower misrepresents the doctrine of the Trinity.)
2. Regarding Jesus
A. "The fact that our Lord received worship is claimed by some to be an evidence that while on
earth he was God the Father disguised in a body of flesh and not really a man. Was he
really worshiped, or is the translation faulty? Yes we believe our Lord Jesus while on earth
was really worshiped, and properly so. While he was not the God, Jehovah, he was a God.
The word ‘God’ signifies a ‘mighty one,’ and our Lord was indeed a mighty one. So it is
stated in the first two verses of the gospel of John. It was proper for our Lord to receive
worship in view of his having been the only begotten of the Father. . ." The Watchtower,
July 15, 1898, p. 216.
B. As Jesus cleansed the temple in Jerusalem three and a half years after he was anointed with
God's spirit to be King, so three and a half years after he received kingly power in the
autumn of 1914 he came to the spiritual temple as Jehovah's Messenger and began to
cleanse it. So this occurred in the spring of 1918. That marked the beginning of the period
of judgment and inspection of his spirit-begotten followers" Let God be True, p. 202.
3. Miscellaneous
A. "This seventh 'day' on which God desisted from his work toward our planet is not to be
understood as a 24-hour day. This seventh day follows upon the preceding six days of
creation. The Scriptural evidence is to the effect that all those six preceding days were
much longer than 24 hours each. . . each of those days was 7,000 years long. Man being
created toward the close of the sixth day, he was put on earth toward the end of 42,000
years of earth's preparation" Let God Be True, p. 168.
B. "The man Adam is not included in those ransomed. Why not? Because he was a willful
sinner, was justly sentenced to death, and died deservedly, and God would not reverse his
just judgment and give Adam life. He had a perfect life, and this he deliberately forfeited"
Let God be True, p. 119.
C. "Accepting the message of salvation and devoting ourselves to God through Christ and
being baptized in water is only the beginning of our exercise of faith. It is only the
beginning of our obedience to God. It sets us on the way to everlasting life, but it does not
mean our final salvation" This Means Everlasting Life, p. 181.

"Thus it is seen that the serpent (the Devil) is the one that originated the doctrine of the inherent
immortality of human souls. This doctrine is the main one that the Devil has used down through the
ages to deceive the people and hold them in bondage. In fact, it is the foundation doctrine of false
religion" Let God be True, pp. 74-75
False Prophecies of the Jehovah's Witnesses
The Witnesses make many claims in their attempt to convert you to their faith. They profess to
have the only true Christian church, to be the only true representatives of God, to have the only
correct biblical teaching, and to be the only true announcers of Jehovah's coming kingdom.
If they are the only true church and are the only true voice of God's word, then what they say
should prove to be true, especially in prophecy. When it comes to predicting the future, the
Watchtower organization fails miserably. Following are some of the false predictions made over the
years by the Watchtower organization. If you present these to a JW, he will probably say something
like, "Those are taken out of context," or "They didn't claim to be the prophet of God," or "The light is
getting brighter and we are understanding Bible prophecy better now," etc. Make a copy of these false
prophecies, found in the appendix, and give it to them to check. They are right out of the Witnesses'
literature.
Remember Deut. 18:22, "If what a prophet proclaims in the name of the LORD does not take
place or come true, that is a message the LORD has not spoken. That prophet has spoken
presumptuously. Do not be afraid of him." If someone makes a false prophecy, and they have claimed
to be a prophet of God, then they are false prophets and are not to be listened to.
Do the Witnesses claim to be the prophet of God? Yes, they do.

In 1972 the Jehovah's Witness Watchtower claimed to be the prophet of God.

IDENTIFYING THE "PROPHET" -- "So does Jehovah have a prophet to help them, to warn them of
dangers and to declare things to come? These questions can be answered in the affirmative. Who is
this prophet?...This "prophet" was not one man, but was a body of men and women. It was the small
group of footstep followers of Jesus Christ, known at that time as International Bible Students. Today
they are known as Jehovah's Christian Witnesses...Of course, it is easy to say that this group acts as a
‘prophet' of God. It is another thing to prove it." The Watchtower, 4/1/72, p. 197. (See Deut. 18:21)

1897 "Our Lord, the appointed King, is now present, since October 1874," Studies in the Scriptures,
Vol. 4, page 621.

1899 "...the ‘battle of the great day of God Almighty' (Revelation 16:14), which will end in A.D. 1914
with the complete overthrow of earth's present rulership, is already commenced." The Time Is at
Hand, page 101 (1908 edition).

1916 "The Bible chronology herein presented shows that the six great 1000 year days beginning with
Adam are ended, and that the great 7th Day, the 1000 years of Christ's Reign, began in 1873." The
Time Is at Hand, page ii, (forward).

1918 "Therefore we may confidently expect that 1925 will mark the return of Abraham, Isaac, Jacob
and the faithful prophets of old, particularly those named by the Apostle in Hebrews 11, to the
condition of human perfection." Millions Now Living Will Never Die, page 89.

1922 "The date 1925 is even more distinctly indicated by the Scriptures than 1914." The Watchtower
9/1/22, page 262.

1923 "Our thought is, that 1925 is definitely settled by the Scriptures. As to Noah, the Christian now
has much more upon which to base his faith than Noah had upon which to base his faith in a coming
deluge." The Watchtower, page 106 4/1/23.

1925 "The year 1925 is here. With great expectation Christians have looked forward to this year.
Many have confidently expected that all members of the body of Christ will be changed to heavenly
glory during this year. This may be accomplished. It may not be. In his own due time God will
accomplish his purposes concerning his people. Christians should not be so deeply concerned about
what may transpire this year." The Watchtower, 1/1/25, page. 3.

1925 "It is to be expected that Satan will try to inject into the minds of the consecrated, the thought
that 1925 should see an end to the work." The Watchtower, Sept, 1925 page 262.

1926 "Some anticipated that the work would end in 1925, but the Lord did not state so. The difficulty
was that the friends inflated their imaginations beyond reason; and that when their imaginations burst
asunder, they were inclined to throw away everything." The Watchtower, page 232.

1931 "There was a measure of disappointment on the part of Jehovah's faithful ones on earth
concerning the years 1917, 1918, and 1925, which disappointment lasted for a time...and they also
learned to quit fixing dates." Vindication, page 338.

1941 "Receiving the gift, the marching children clasped it to them, not a toy or plaything for idle
pleasure, but the Lord's provided instrument for most effective work in the remaining months before
Armageddon." The Watchtower, 9/15/41, page 288.

1968 "True, there have been those in times past who predicted an ‘end to the world', even
announcing a specific date. Yet nothing happened. The ‘end' did not come. They were guilty of false
prophesying. Why? What was missing?.. Missing from such people were God's truths and evidence that
he was using and guiding them." Awake, 10/8/68.

1968 "Why are you looking forward to 1975?" The Watchtower, 8/15/68, page 494.

A JW might say that the organization is still learning. If that is so, then how can they trust what
they are taught now by the Watchtower? Will what they are being taught now change also?
A true prophet of God won't err in prophesying. Only a false prophet does. The Jehovah's Witness
organization, that claims to be a prophet of God, is really a false prophet. Jesus warned us by saying,
"For false Christs and false prophets will appear and perform great signs and miracles to deceive even
the elect -- if that were possible," (Matt. 24:24).
1914 A.D., 607 B.C., 586 B.C. and the Jehovah's Witnesses.

An absolutely critical date for the Jehovah's Witnesses is 1914 AD. It is the date when, according
to the Jehovah's Witnesses, the time of the Gentiles ended (Watchtower, 5/1/93, page 11) and "Jesus-
the heavenly warrior Michael-became King of God's heavenly Kingdom," (Watchtower 11/1/93, page
23). To arrive at this date, the Witnesses take the account in Daniel 4 and apply a 360 day year for
each of the seven "times" for a total of 2520 years. They add this date to 607 B.C., their date for the
fall of Jerusalem under Nebuchadnezzar, and arrive at 1914 A.D., the date when Jesus supposedly
returned invisibly in the heavens (The Truth Shall Make You Free, p. 300), the "appointed time of the
nations" ended (The Time is at Hand, page 79), and the beginning of the end of the world commenced
(Watchtower 11/15/50, page 438). Please consider the following quote.

"This marked time began in the year 1914 (A.D.). In that important year the 'appointed times
of the nations,' 2,520 years long, ran out. If we measure back that many years from
1914 we come to the ancient date of 607 B.C. That year was marked for the overthrow of
the earthly "throne of Jehovah" and for the destruction of the throne city of Jerusalem and
its sanctuary and for the total desolation of the land of the kingdom of Judah." (From the Book,
"Your Will," 1958, pp. 309-310, Watchtower CD, emphasis added).

Therefore, the date 607 BC becomes the critical date in question. Was 607 BC the date when
Jerusalem fell? No, it wasn't. No Bible scholar and no archaeological scholar holds to that date. The
correct date is 586 B.C., not 607 B.C. Therefore, the Jehovah's Witnesses are wrong about 1914 and
everything else they attach to that date based on their prophet misunderstanding. Let's verify further
that 607 B.C. is the date used by the Watchtower Bible and Tract Society before we establish the
counter evidence.

• "The true prophet Jeremiah, not the false prophets, was vindicated when Jerusalem was
razed by Babylonian soldiers in 607 B.C.E., the temple destroyed, and the populace either
killed or dragged away captive to distant Babylon. The pitiful few that were left in the land
fled into Egypt.-Jeremiah 39:6-9; 43:4-7," (Watchtower 2/1/92, page 4).
• "In 607 B.C.E., Israel was taken into captivity for 70 years," (Watchtower 4/15/92, page
10).
• "Samaria fell to the Assyrians in 740 B.C.E., and Jerusalem and its temple were destroyed
by the Babylonians in 607 B.C.E," (Watchtower, 11/1/92, page 13).
• "The Babylonians came in 607 B.C.E. and stripped Jerusalem bare. Her people and her
wealth were carried off to Babylon. The city was destroyed, the temple was burned, and the
land was left desolate.-2 Chronicles 36:17-21," (Watchtower 10/15/88, page 16).

Following are citations verifying that the correct date for the fall of Jerusalem was not 607 B.C, but
586 B.C.

• According to Encyclopedia.com, the Babylonian captivity, is defined as "the period from the
fall of Jerusalem (586 B.C.) to the reconstruction in Palestine of a new Jewish state (after
538 B.C.)."
• "You will recall that the Babylonians, under Nebuchadnezzar, after twice laying siege to
Jerusalem, finally captured it in 586 B.C.E. Nebuchadnezzar's army then pillaged the city,
destroying the Temple and sending the inhabitants off to exile in Babylonia.("Biblical
Archaeological Review, Biblical Archaeological Review).
• "...Nebuchadnezzar promptly invaded his unhappy country and besieged Jerusalem for a
year and a half. In 587 Jerusalem fell and numbers of its inhabitants were carried away
captive to Babylonia..." (Unger, Merrill, F., Unger's Bible Dictionary, Moody Press, Chicago,
1966, page 782).
• Notice that the year 587 is offered instead of 586. There is sometimes a difference of
opinion as to which year is the exact one. Nevertheless, it is obvious that 607 B.C. is not
even close.
• "586, Jerusalem destroyed and burned (Jer. 52:13f.); people taken captive (52:28-30).
(The International Standard Bible Encyclopedia, Eerdmans, Grand Rapids, Michigan, 1982,
page 1016)

It is quite clear that the Jehovah's Witness organization is wrong about the 607 B.C. date upon which
they place so much of their end times theology. If they are wrong about such a basic event and have
not changed their error to match historical fact, how can they be trusted to represent biblical truth?
They cannot.
The fact is that they can not change their date of 1914 because they have so much invested in it.
They are forced to retain their 607 BC date even though it is in obvious error. To admit they were
wrong is to undermine the whole credibility and truth of the Watchtower Bible and Tract Society. This
they cannot do because they are more dedicated to their organization than they are to the truth.
Jehovah's Witness doctrine is not from the Bible alone

Many Jehovah's Witnesses will tell you that they derive their doctrines from the Bible and only from
the Bible. In reality, they derive their doctrines from what the Watchtower says about the Bible.
Following are various quotes that demonstrate this fact.

1. If the Witness says he learns his theology by reading the Bible alone, then he
contradicts the Watchtower which says that is not possible:
A. "Let us face the fact that no matter how much Bible reading we have done, we would never
have learned the truth on our own. We would not have discovered the truth regarding
Jehovah, his purposes and attributes, the meaning and importance of his name, the
Kingdom, Jesus' ransom, the difference between God's organization and Satan's, nor why
God has permitted wickedness." (Watchtower 12/1/1990, pages 19)
B. "Thus the Bible is an organizational book and belongs to the Christian congregation as an
organization, not to individuals, regardless of how sincerely they may believe that they can
interpret the Bible." (The Watchtower, Oct. 1, 1967. p. 587.)
2. Additionally, the Watchtower says only its organization understands the Bible
A. "Only this organization functions for Jehovah's purpose and to his praise. To it alone God's
Sacred Word, the Bible, is not a sealed book." (The Watchtower; July 1, 1973, pp. 402.)
B. "All who want to understand the Bible should appreciate that the "greatly diversified wisdom
of God" can become known only through Jehovah's channel of communication, the faithful
and discreet slave." (The Watchtower; 10/1/1994; p. 8.)
3. Therefore, whatever argument the JW offers is not from their understanding of the
Bible, but from the watchtower's interpretation of it.
A. Can the Watchtower be trusted especially since it has made false prophecies?
i. "...the ‘battle of the great day of God Almighty' (Revelation 16:14), which will end in
A.D. 1914 with the complete overthrow of earth's present rulership, is already
commenced." (The Time Is at Hand, page 101 (1908 edition).
ii. "The date 1925 is even more distinctly indicated by the Scriptures than 1914." (The
Watchtower 9/1/22, page 262.)
iii. "Our thought is, that 1925 is definitely settled by the Scriptures. As to Noah, the
Christian now has much more upon which to base his faith than Noah had upon which to
base his faith in a coming deluge." (The Watchtower, page 106 4/1/23.)
iv. "Receiving the gift, the marching children clasped it to them, not a toy or plaything for
idle pleasure, but the Lord's provided instrument for most effective work in the
remaining months before Armageddon." (The Watchtower, 9/15/41, page 288.)
v. "There was a measure of disappointment on the part of Jehovah's faithful ones on earth
concerning the years 1917, 1918, and 1925, which disappointment lasted for a
time...and they also learned to quit fixing dates." (Vindication, page 338.)
vi. "Why are you looking forward to 1975?" (The Watchtower, 8/15/68, page 494.)
4. The Watchtower says if you read the Bible alone, you'll become a Trinitarian.
A. "From time to time, there have arisen from among the ranks of Jehovah's people those,
who, like the original Satan, have adopted an independent, faultfinding attitude...They say
that it is sufficient to read the Bible exclusively, either alone or in small groups at home. But,
strangely, through such ‘Bible reading,' they have reverted right back to the apostate
doctrines that commentaries by Christendom's clergy were teaching 100 years ago..." The
Watchtower, August 15, 1981.
B. What are the apostate doctrines of 100 years ago? Why, the Trinity of course.
5. Obviously, the Watchtower organization tells its members what to believe. From the last quote,
we can clearly see that the Bible teaches Trinitarianism, since that is what we would conclude if
we simply read the Bible without the Watchtower guiding our thoughts.
Contradictions in Watchtower Literature
1. What or who is the real generation of 1914?
A. only those alive in 1914 "Where are We According to God's Timetable?" The Watchtower,
May 1 (1967), p. 262.
B. anyone alive who understand the significance of 1914 "A Time to Keep Awake," The
Watchtower, November 1 (1995), p. 19.
2. Who is the Alpha and Omega of Rev. 22:12-13, Jesus or Jehovah?
A. Awake! August 22, 1978, p.28;
B. The Watchtower, October 1, 1978, p.15.
3. Resurrection of the men of Sodom and Gomorrah
A. Zion's Watch Tower Reprints, July 1879, p.7.
4. The Watchtower, June 1, 1952, p. 338;
A. August 1,1965 p.479;
B. June 1, 1988, pp.30-31.
5. The Lord in Romans 10:12ff
A. Zion's Watch Tower Reprints, December 1,1903, p.3282; The Watchtower, July 1, 1940,
p.200; May 1,1978, p.12; February 1, 1980, p.16.
6. 'Higher Powers' of Romans 13:1
A. WatchTower Reprints, September 1, 1916, p.5952, The Truth Shall Make You Free. [1943
ed.], p. 312; The Watchtower, June 15, 1964, p.20. The WT Society admitted this change in
doctrine cost many JWs their lives (The Watchtower, November 15, 1950, p.441). The tract
"Facts You Should Know About Jehovah's Witnesses" from which excerpts have been
taken is very comprehensive and includes Bibliography of sources of information. It is
available free from Cephas Ministry Inc. on request.
7. Hath life
A. How do you reconcile the JW quote in "Make sure of all things", p.332 stating: "Salvation to
life involves time and is not completed when one becomes a Christian", when in I John
5:12-13, it says, "he that has the Son, hath life". In John 3:36; I John 3:2; I Corinthians
1:18; II Corinthians 2:15; Ephesians 2:5,8; I Thessalonians 1:10; II Timothy 1:9; Titus 3:5;
I John 2:12 and Romans 8:16, eleven times it states that eternal life is the believer's present
possession, and that we are saved now, not will be later.
8. Who goes to heaven?
A. Where is the great crowd located in Revelation 19:1? Heaven. Therefore all believers go to
heaven. (JW's say only 144,000 believers go to heaven, the rest stay on earth).
B. Why does WT ignore the following verses in saying that we don't go to heaven? Colossians
1:5; Hebrews 10:34; I Peter 1:4; Revelation 19:1; Matthew 6:20; John 12:26; John 14:3;
Hebrews 3:1; II Corinthians 5:1.
9. Apostasy or not?
A. The Watchtower, 1-1-89, page 12, paragraph 12 & 13 says that by the 2nd century,
apostacy crept in, and that authentic Christian missionary work ceased, but not forever.
Because, toward the end of the 19th century, Charles T. Russell, the 1st president of the
WTS, saw the need for missionary work. Apparently, there was no one teaching correctly
from the 2nd century to the 19th. Yet, in the Wattchtower, 8-1-95, page 16, paragraph 7 &
8 it describes how the disciples formed individual congregations. Then it ask the question;
Has divine teaching in that way continued down till our day? (answer at the start of next
paragraph) Indeed, it has!

____________
Note: The contradictions were acquired from a variety of sources from my own study, books, and
the internet.
More Contradictions in Watchtower Literature.
The watchtower says it is God's only true organization: "Only this organization functions for
Jehovah's purpose and to his praise. To it alone God's Sacred Word, the Bible, is not a sealed book,"
(The Watchtower; July 1, 1973, page 402.). It claims to speak for God, yet it has produced numerous
contradictory statements. Therefore, it is not God's organization.

1. Who is the Faithful and Discreet Slave, the Watchtower Organization or Charles Taze
Russell?
A. Watchtower Organization is Faithful and Discreet Slave:
i. "Jesus foretold that among his people there would be a "faithful and discreet slave"
class who would be providing the spiritual food to God's family of devoted servants on
earth, acting as his channel of communication and overseeing the carrying out of the
Kingdom interests world wide. (Matt. 24:45-47) These anointed overseers serve as
though being guided in their activities by the right hand of Christ." (Watchtower
1/15/1969, pages 51).
B. Russell is Faithful and Discreet Slave:
i. "Thousands of the readers of Pastor Russell's writings believe that he filled the office of
'that faithful and wise servant,' and that his great work was giving to the household of
faith meat in due season. His modesty and humility precluded him from openly claiming
this title, but he admitted as much in private conversation," Watchtower 12/1/1916,
page. 357.
C. Watchtower Denies Russell ever claimed to be Faithful and Discreet Slave:
i. "From this it is clearly seen that the editor and publisher of Zion's Watch Tower
disavowed any claim to being individually, in his person, that "faithful and wise
servant." He never did claim to be such. (God's Kingdom of a thousand years has
approached, 1973, p.346).
2. Watchtower claims to be only organization to find scriptural guidance, yet condemns
the Vatican for doing the same thing.
A. Watchtower is only organization able to interpret the Bible:
i. "We all need help to understand the Bible, and we cannot find the Scriptural guidance
we need outside the ‘faithful and discreet slave' organization." (The Watchtower, Feb.
15, 1981.)
B. Vatican is only organization able to interpret the Bible:
i. "The Vatican belittles Bible study by claiming it is the only organization authorized and
qualified to interpret the Bible." (The Watchtower, 7/1/1943, p. 201.)
3. Is Jesus supposed to be worshipped or not?
A. Yes, Jesus is supposed to be worshipped:
i. "Question: The fact that our Lord received worship is claimed by some to be an
evidence that while on earth he was God the Father disguised in a body of flesh and not
really a man. Was he really worshiped, or is the translation faulty? Answer: Yes, we
believe our Lord Jesus while on earth was really worshiped, and properly so...It was
proper for our Lord to receive worship in view of his having been the only begotten of
the Father, and his agent in the creation of all things, including man." (The Watchtower,
7/15/1898, page 216.)
ii. New Heavens and a New Earth, on pages 27-28, published in 1953, "For example, to
which one of the angels did he ever say: 'You are my Son; today I have become your
Father'? And again: 'I shall be a Father to him, and he will be a Son to me'? But when
he again brings his Firstborn into the inhabited earth, he says: 'And let all God's angels
worship him.'"
B. No, Jesus is not supposed to be worshipped:
i. "He taught men, not to worship him, but to worship Jehovah his Father ... "I am
ascending to my Father and your Father, and to my God and your God." (Make Sure of
All Things p. 283)
ii. "What I learned was so different from what I had heard at the "Christian" school I had
attended. Jehovah's Witnesses do not worship Jesus. Rather, they worship Almighty
God, the One that Jesus himself worshipped." (Awake, 12/22/87, page 22.).
4. The sower in the Matt. 13 parable is Satan or Jesus?
A. The sower of the mustard seed is Satan
i. "Today it is big enough to hold them all. In the parable, the "man" that sowed the
mustard grain pictures the "wicked one," Satan the Devil. (Man's Salvation Out Of
World Distress, 1975, p. 208).
B. The sower of the mustard seed is Jesus
i. "Hence that apostate generation suffered national calamity in 70 C.E. So, now, does
anyone ask the question, How could Jesus as the Sower of the parable plant the
symbolic mustard grain and yet have it become a tree of a foreign kind, the corrupt
counterfeit called Christendom? The experience of Jehovah God with the ancient nation
of Israel gives the divine answer to such a questioner!" (Watchtower, 10/1/1975, p.
600).
5. Armageddon will be in 1914 or 1941
A. Armageddon will be in 1914
i. "...the Kingdom of God is already begun, that it is pointed out in prophecy as due to
begin the exercise of power in A.D. 18789, and that the "battle of the great day of God
Almighty (Rev. 16:14), which will end in A.D. 1914, with the complete overthrow of
earth's present rulership, is already commenced." (The Time is at Hand, 1889. -
Emphasis added).
B. Armageddon will be in 941
i. "Receiving the gift, the marching children clasped it to them, not a toy or plaything for
idle pleasure, but the Lord's provided instrument for most effective work in the
remaining months before Armageddon." (The Watchtower, 9/15/41, page 288.)
6. Armageddon will be in 1914 or 1915
A. Armageddon is in 1915
i. "...the Kingdom of God is already begun, that it is pointed out in prophecy as due to
begin the exercise of power in A.D. 18789, and that the "battle of the great day of God
Almighty (Rev. 16:14), which will end in A.D. 1915, with the complete overthrow of
earth's present rulership, is already commenced." (The Time is at Hand, 1881. -
Emphasis added - Date was changed to 1914 in a later edition).
B. Armageddon is in 1914
i. "...the Kingdom of God is already begun, that it is pointed out in prophecy as due to
begin the exercise of power in A.D. 18789, and that the "battle of the great day of God
Almighty (Rev. 16:14), which will end in A.D. 1914, with the complete overthrow of
earth's present rulership, is already commenced." (The Time is at Hand, 1889. -
Emphasis added).
The Jehovah's Witnesses and the Resurrection of Jesus
The Watchtower organization says that Jesus did not rise from the dead in the same body he died
in (You Can Live Forever on Paradise Earth, p. 143-44). Instead, it says that He rose as a spirit
creature and that the material body of Jesus was taken away by God the Father. Therefore, they deny
the physical resurrection of Christ. Is this important? Most definitely!
1 Cor. 15:14 says, "If Jesus is not raised, then our faith is in vain." In other words, if Jesus did not
rise from the dead, then Christianity is a waste of time and we are then still dead in our sins. It is
obvious that the doctrine of the resurrection of Jesus is a vital and essential element of Christianity.
But what of the Jehovah’s Witnesses? Are they accurate in their assessment of Jesus’ resurrection in
denying the bodily resurrection but affirming a "spiritual" resurrection? The answer is a definite, "No."
It is obvious from Jesus' own words in John 2:19-21 that He would raise Himself from the dead:

"Jesus answered and said to them, "Destroy this temple, and in three days I will raise it up."
20
The Jews therefore said, "It took forty-six years to build this temple, and will You raise it up in
three days?" 21But He was speaking of the temple of His body."

John 2:19-21 is a clear prophecy of Christ. Note that He said He would raise up "this temple."
John the apostle clarifies for us that "this temple" was actually Jesus physical body. Therefore, Jesus'
physical body was raised from the dead. Very simple. However, the Jehovah's Witnesses do not
believe Jesus' own words here.
In order to help you see the error of the Watchtower position and aid you in refuting their
arguments, I've compiled the following list of arguments used by the Jehovah's Witnesses to support
their position.

1. They use 1 Pet. 3:18 where it says that Christ was "put to death in the flesh, but
made alive in the spirit" as an attempt to show that Jesus was not raised physically, but as
a kind of spirit creature.
Their use of the scripture to support their position is incorrect because this verse does not say that
He was raised a spirit creature. It says that He was "made alive in the spirit." What does that mean?
Quite simply, it means that Jesus was raised in an imperishable body. This is what 1 Cor. 15:35-45
says when it refers to the body as being sown perishable, but raised imperishable; sown in dishonor
and raised in glory; sown a natural body and raised a spiritual body, etc. Jesus was the "Last Adam" a
life giving spirit. Paul is typifying the resurrection body. In this passage Paul is talking about the
resurrection of all people. All Christians will be raised in physical bodies. It is said the same of Jesus.

2. The Bible says that flesh and blood cannot inherit the kingdom of God (1 Cor.
15:44-50). Therefore, Jesus’ physical body could not be raised lest it contradict this verse.
What the Jehovah's Witnesses miss is that after His resurrection Jesus said, "Touch me and see,
for a spirit does not have flesh and bones as you see I have" (Luke 24:39). You must note that Jesus
did not say, "flesh and blood." He said, "flesh and bones." This is because Jesus’ blood was shed on
the cross. The life is in the blood and it is the blood that cleanses from sin: "For the life of the flesh is
in the blood, and I have given it to you upon the altar to make atonement for your souls; for it is the
blood that makes atonement for the soul" (Lev. 17:11). See also, Gen. 9:4; Deut. 12:23; and John
6:53-54. Jesus was pointing out that He was different. He had a body, but not a body of flesh and
blood. It was flesh and bones.
3. The Jehovah's Witnesses teach that Jesus manifested different physical forms in
order to convince the disciples that He had been raised.
This is faulty for several reasons. First, it would mean that Jesus was tricking His disciples into
believing that His body had been raised when it hadn’t. Second, it disregards the clear teaching of
Jesus Himself who said His very body would be raised. He said in John 2:29-21, "Destroy this temple,
and in three days I will raise it up." 20Then the Jews said, "It has taken forty-six years to build this
temple, and will You raise it up in three days?" 21But He was speaking of the temple of His body." Jesus
said that His body would be raised. The Jehovah’s Witnesses clearly deny Jesus’ very words. Fourth, 1
Tim. 2:5 says, "For there is one mediator between God and man, the man Christ Jesus." Jesus is said
to be a man -- in present tense language. If He was not raised physically, then how could he be a
man? He could not be.

4. Jesus manifested different bodies after the resurrection, the same way the angels
took human form in the Old Testament in order to show the disciples that He had been
raised.
Again, this contradicts what Jesus said in John 2:19-21 that He would raise Himself from the dead
physically. Also, Jesus is not an angel contrary to what the Jehovah's Witnesses believe. Jesus was
God in flesh (John 8:58; John 1:1,14; Col. 2:9; Phil. 2:5-8).

The Jehovah's Witnesses teach that Jesus did not rise from the dead in the same body He died in.
This is a dangerous doctrine that contradicts the Bible and condemns those who believe it to eternal
destruction because it is denying His physical resurrection which is the proof that He conquered death.
The Jehovah's Witnesses need to keep Jesus’ own words in mind when He said, "Destroy this temple
and in three days, I will raise it up" (John 2:19). Since He was speaking of His body says John in verse
21, then it must be true; Jesus rose from the dead in the same body He died in. Also, at His ascension
people watched Him rise to be with the Father. They saw His body ascend. That is why it can be said
that Jesus, the man, is the mediator between God and man (1 Tim. 2:5). It isn’t an angel or a spirit
creature that is the mediator. It is Jesus the man.
Did Jesus die on a stake or a cross?

One of the doctrines of the Jehovah's Witnesses that is wrong, though not an attack on an essential
doctrine of scripture, is their teaching that Jesus died on a stake instead of a cross ( Reasoning from
the Scriptures, 1985, pp. 89-90). It really doesn't matter which Jesus died on. The issue is whether
or not He shed His blood for our sins.
In support of their position, they accurately state that the Greek word used in many Bibles which is
translated into "cross" is the Greek word "stauros" which means, "an upright stake, esp. a pointed one,
a cross."134 If a stake were used, instead of a cross, then Jesus' hands would have been placed above
His head with a nail driven through His wrists. Since the wrists would most likely overlap, only one
nail is needed through both wrists. However, some Jehovah's Witnesses have maintained that Jesus'
hands may have been placed one higher than another on the stake. The reason they say this is
because of John 20:25,

"The other disciples therefore said unto him, We have seen the Lord. But he said unto them,
except I shall see in his hands the print of the nails, and put my finger into the print of the
nails, and thrust my hand into his side, I will not believe."

Notice the use of the word nails (plural) in reference to hands (plural). It makes far more sense to
say that Jesus was crucified on a cross with outstretched hands and one nail in each hand placed
above each other on a stake. That is why it says "...in his hands the print of the nails..."
Again, this is not an issue of essential doctrine, but I do believe the evidence is sufficient to
demonstrate that the Watchtower organization is incorrect in yet another matter.

134
Enhanced Strong’s Lexicon,
1 Chron. 29:20, is Jesus worshipped the same way David was?

The Jehovah's Witnesses teach that Jesus is a created thing and is not worthy of worship.
Christians counter that Jesus is worshipped in the New Testament and cite verses such as Matt.
2:2,11; 14:33; 28:9; John 9:35-38;and Heb. 1:6. The Witnesses acknowledge that at times people
bowed down before Jesus the same way they would to God the Father, but they deny that Jesus was
worshiped. In fact, in the New World Translation produce by the Jehovah's Witness, the Greek word
for worship "proskuneo" is always translated as "obeisance" whenever it refers to Christ, but is
translated as worship whenever it refers to the Father. The witnesses select which way to translate
the word "proskuneo" depending on two things: Who is being addressed and what their theology tells
them. So, the debate continues. The Witnesses deny Jesus' deity and the Christians rightfully
proclaim it. Jesus is the second person of the Trinity.
In an attempt to prove their position, Jehovah's Witnesses sometimes quote 1 Chron. 29:20 which
says in the King James, "And David said to all the congregation, now bless the LORD your God. And all
the congregation blessed the LORD God of their fathers, and bowed down their heads, and worshipped
the LORD, and the king," (KJV). By quoting the King James, the Jehovah's Witnesses is trying to
establish that since king David was worshipped along with God, it means that David was worshipped to
a lesser degree than God. Therefore, they say it follows that Jesus can also be worshiped to a lesser
degree than God in the same way that King David was and it would not require that Jesus be divine.
First of all, 1 Chron. 29:20 obviously cannot mean that David was worshiped equally with God. This
would be blasphemy. Therefore, it is best to translate the word Hebrew word "shachah" as "pay
homage," or "bow down" as is attested to by other translations.

• "Then David said to all the assembly, "Now bless the Lord your God." And all the assembly
blessed the Lord, the God of their fathers, and bowed low and did homage to the Lord and
to the king," (NASB).
• "Then David said to the whole assembly, "Praise the LORD your God." So they all praised
the LORD, the God of their fathers; they bowed low and fell prostrate before the LORD
and the king," (NIV).
• "Then David said to all the assembly, "Now bless the Lord your God." So all the assembly
blessed the Lord God of their fathers, and bowed their heads and prostrated themselves
before the Lord and the king," (NKJV).
• "Then David said to all the assembly, "Bless the Lord your God." And all the assembly
blessed the Lord, the God of their fathers, and bowed their heads and paid homage to the
Lord and to the king," (ESV).

Is it legitimate to translate the verse this way? Yes it is because it is consistent with Hebrew
dictionaries which tell us that the word "shachah" can mean worship, bow down, obeisance, reverence,
fall down, crouch, prostrate oneself, (Enhanced Strong's Lexicon, (Oak Harbor, WA: Logos Research
Systems, Inc., 1995.) See the same thing stated in The Abridged Brown-Driver-Briggs Hebrew-English
Lexicon of the Old Testament, Richard Whitaker, Editor, (Oak Harbor, WA: Logos Research Systems,
Inc., 1997). This would easily demonstrate that David was not being worshiped. Instead, homage
was being paid to him as was also being paid to God -- though, of course, there is no confusion about
who is God and who is not. But, if that weren't enough, the Jehovah's Witnesses' own New World
translation translates the Hebrew word "shachah" in 1 Chron. 29:20 as "prostrate."

"And David went on to say to all the congregation: "Bless, now, Jehovah YOUR God." And all
the congregation proceeded to bless Jehovah the God of their forefathers and bow low and
prostrate themselves to Jehovah and to the king," (NWT).

Therefore, the Jehovah's Witness own bible recognizes that King David, though greatly revered by
the people, was not worshipped. Instead, both God and David were revered by the people by
prostration, not worship.
But, let's assume the Jehovah's Witness argument for a moment. Even if we were to say that
Jesus could be with honored the same way that David was honored, in a lesser sense than God, this
does not mean that Jesus is not God. Remember, Jesus is both God and man. As a man people would
bow down to Him. As God, He was worshipped. In addition, it is said of Jesus that He knew all things
(John 21:17), that He would be with the disciples wherever they went (Matt. 28:20), etc. He is called
God by Thomas in John 20:28 and by the Father in Heb. 1:8. The divinity of Christ is well attested to
in the New Testament and only the Jehovah's Witnesses with their preconceived idea that Jesus is not
divine, will twist the scriptures to suit their own needs.
The Bible says, "In the beginning was the word and the word was with God and the word was
God...and the word became flesh and dwelt among us," (John 1:1,14). Clearly, Jesus is the word
made flesh and as it says in Col. 2:9, all the fullness of the Godhead dwells bodily in Jesus. Jesus is
not a created thing, but the creator of all things (Col. 1:16-17), just as God alone is the creator of all
(Isaiah 44:24). He is both God and man. This is called the Hypostatic Union.
The Jehovah's Witnesses are wrong and their continued attempt to dethrone Christ will continue to
fail as the truth of God's word is revealed.
Matt. 3:3, Prepare the way of the LORD

John the Baptist was the greatest of Prophets (Matt. 11:9-11) and the privileged messenger who
was sent to prepare the way of the Messiah according to the Word of God. There are four main texts
used to verify this:

1. "For this is the one referred to by Isaiah the prophet, saying, “The voice of one crying in the
wilderness, ‘Make ready the way of the Lord, make His paths straight!’” (Matt. 3:3).
2. "The beginning of the gospel of Jesus Christ, the Son of God. 2As it is written in Isaiah the
prophet, "Behold, I send My messenger before Your face, who will prepare Your way; 3The voice
of one crying in the wilderness, ‘Make ready the way of the Lord, make His paths straight,’" (Mark
1:1-3).
3. “This is the one about whom it is written, ‘Behold, I send My messenger before Your face, who
will prepare Your way before You,’ (Matt. 11:10).
4. "He [John the Baptist] said, 'I am a voice of one crying in the wilderness, 'Make straight the way
of the Lord,' as Isaiah the prophet said,'" (John 1:23).

Clearly, John the Baptist was sent by God to prepare the way for Jesus as all four quotes above
demonstrate. But, we must note that in the four citations, the Old Testament is quoted from two
sources: Isaiah and Malachi.

• Isaiah 40:3, "A voice is calling, “Clear the way for the Lord [YHWH 135] in the wilderness;
make smooth in the desert a highway for our God."

• Mal. 3:1, "Behold, I am going to send My messenger, and he will clear the way before Me.
And the Lord [adonai], whom you seek, will suddenly come to His temple; and the
messenger of the covenant, in whom you delight, behold, He is coming," says the Lord
[YHWH] of hosts."

Notice that according to the quotes above, John the Baptist was sent to prepare the way for
Jehovah. Isaiah 40:3 says that John is to clear the way for YHWH (Jehovah). In Mal. 3:1, it is God
who says that "he will clear the way before Me." Yet we see that the fulfillment of these verses is
found in the arrival of Jesus.
Clearly, the Bible prophecies that John the Baptist will prepare the way of YHWH (Jehovah), yet it is
Jesus who arrives on the scene. How can this be? The simple answer is that Jesus is divine; that is,
He is the fullness of deity in bodily form (Col. 2:9). Also, John 1:1,14 tell us "In the beginning was the
Word, and the Word was with God, and the Word was God.... 14And the Word became flesh, and dwelt
among us, and we beheld His glory, glory as of the only begotten from the Father, full of grace and
truth." The fulfillment of John the Baptist as the messenger and Jesus as YHWH make these scriptures
clear for us to understand.
Do not be deceived by the Jehovah's Witness organization that would have you deny the true
nature of Christ. Only God can save us and only God can fulfill the prophecy given in Isaiah 40:3 and
Mal. 3:1.

135
YHWH is the tetragrammaton, the four Hebrew letters used to designate the name of God. It has been
transliterated into Jehovah and Jahweh. I use "Jehovah" in this article.
John 1:1 and the Jehovah’s Witnesses
In [the] beginning the Word was, and the Word was with God,
and the Word was a god." The New World Translation

This is one of the most common verses of contention between the Jehovah's Witnesses and
Christians. Their false assumption is that Jesus is not God in flesh, but Michael the archangel who
became a man. Therefore, since they deny that Jesus is divine, they have altered the Bible in John
1:1 so that Jesus is not divine in nature. The New World Translation has added the word "a" to the
verse so it says, "...and the Word was a god." The correct translation for this verse is "In the
beginning was the Word and the Word was with God and the Word was God." This is how it is
rendered in the NASB, NIV, KJV, NKJV, ASV, RSV, etc.
The New World translation is incorrect in its translation of this verse for several reasons. First of
all, the Bible teaches a strict monotheism. To say that Jesus is "a god" is to suggest that there is
another god besides YHWH, which is contrary to scripture (Isaiah 43:10; 44:6,8, etc.). Of course, the
Jehovah's Witnesses will respond that Jesus is not the Almighty God, but a "lesser" kind of God. He is
the "mighty God" as is referenced in Isaiah 9:6, "For a child will be born to us, a son will be given to
us, and the government will rest on His shoulders, and His name will be called Wonderful Counselor,
Mighty God, Eternal Father, Prince of Peace." Therefore, they say that Jesus is the mighty god, but
not the Almighty God.
The immediate problem with this explanation is that YHWH is also called the Mighty God in
Jeremiah 21:18 and Isaiah 10:20. In all three verses, including Isaiah 9:6, the Hebrew word for
"mighty" (gibbor) is used.

• Isaiah 10:20-21, "Now it will come about in that day that the remnant of Israel, and those
of the house of Jacob who have escaped, will never again rely on the one who struck them,
but will truly rely on the LORD, the Holy One of Israel. 21A remnant will return, the remnant
of Jacob, to the mighty God."
• Jer. 21:18, "who showest lovingkindness to thousands, but repayest the iniquity of fathers
into the bosom of their children after them, O great and mighty God. the LORD of hosts is
His name."

We can see that the Jehovah's Witness explanation is not valid. Both the Son and God are
called the Mighty God.
Furthermore, how many actual gods are there in scripture? The obvious answer is that there is
only one God in existence. Though there are others who have been falsely called gods (1 Cor. 8:5-
6) or even said to be "as God" like Moses (Ex. 4:16; 7:1), there is only one real God (Gal. 4:8-9;
Isaiah 44:6,8). If Jesus is "a god" that was "with God" in the beginning, then is Jesus a true god or
a false god?
But, the Jehovah's Witnesses often claim that Jesus is a god in the sense that Moses was called
a god. But, Moses was not called a god. Rather, he would be "as God."

• "Moreover, he shall speak for you to the people; and it shall come about that he shall be as
a mouth for you, and you shall be as God to him, (Exodus 4:16).
• "Then the Lord said to Moses, 'See, I make you as God to Pharaoh, and your brother Aaron
shall be your prophet,'" (Exodus 7:1).

Why was Moses going to "as God" to Pharaoh? Because Moses was given the authority and power
to display powerful miracles that decimated much of Egypt. Was Moses really a god? Being "as God"
in regards to power given to perform miracles over Egypt is not the same thing as being called "a god"
that was in the beginning with God, (John 1:1).
John was a strict Jew, a monotheist. Does the Jehovah's Witness really think that John would be
saying that there was another God besides Jehovah, even if it were Jesus? Being raised a good Jew,
the apostle John would never believe that there was more than one God in existence. Yet, he
compared the word with God, said the word was God, and that the word became flesh (John 1:1,14).
John 1:1 in a literal translation reads thus: "In beginning was the word, and the word was with the
God, and God was the word." Notice that it says "God was the word." This is the actual word for
word translation. It is not saying that "a god was the word." That wouldn't make sense. Let me
break it down into three statements.

1. "In beginning was the word..."


(en arche en ho logos)
A. A very simple statement that the Word was in the beginning.
2. "and the word was with the God..."
(kai ho logos en pros ton theon)
A. This same Word was with God.
3. "and God was the word." -- Properly translated as "and the Word was God."
(kai theos en ho logos)
A. This same Word was God.

Regarding statement 3 above, the correct English translation is "...and the Word was God," not
"and God was the word." This is because if there is only one definite article ("ho"="the") in a clause
where two nouns are in the nominative ("subject") form ("theos" and "logos"), then the noun with the
definite article ("ho"="the") is the subject. In this case "ho logos" means that "the word" is the
subject of the clause. Therefore, "...the Word was God" is the correct translation, not "God was the
Word."136 But this does not negate the idea that John is speaking of only one God, not two, even
though the Jehovah's Witnesses maintain that Jesus is "a god," or the "mighty god" as was addressed
above.
Is there suddenly a new god in the text of John 1:1? It is the same God that is being spoken of in
part 2 as in part 3. How do the Jehovah's Witnesses maintain that the word had somehow become a
god in this context, since there is only one God mentioned? Remember, the Jehovah's Witnesses teach
that Jesus was Michael the Archangel. Therefore, is there any place in the Bible where an angel is
called "a god," besides Satan being called the god of this world in 2 Cor. 4:3-4?

John 20:28 - "Thomas answered and said to Him, 'My Lord and my God!'"

In the Greek in John 20:28 Thomas said to Jesus, "ho kurios mou, kai ho theos mou," "The Lord of
me, and the God of me." If Jesus was not God, but "a" god, then shouldn't Jesus have corrected
Thomas? Shouldn't Jesus have said, "No Thomas, I am not the God. I am a god."? But Jesus did
not. To do so would have been ludicrous. Nevertheless, the Jehovah's Witness will say that Thomas
was so stunned by Jesus' appearance that he swore. This is ridiculous because it means that Thomas,
a devout man of God, swore in front of Jesus and used the Lord's name in vain in violation of Exodus
20:7. This is hardly the case since we find no New Testament equivalent of a disciple of Christ using
God's name in vain.
In conclusion, John 1:1 is best translated without the "a" inserted into the text. "The Word was
God" is the best translation. This way, we do not run into the danger of polytheism, with Jesus being
"a god." We do not have Thomas the disciples swearing and using God's name in vain. And, we do
not have the problem of Jesus being a "mighty god" and yet not the God -- even though God Himself
is called the Mighty God (Jeremiah 21:18; Isaiah 10:20).

Chapman, B. (1994). Greek New Testament Insert. (2nd ed., revised.). Quakertown, PA: Stylus Publishing.
136

Also, Louw, J. P. (1989; Published in electronic form by Logos Research Systems, 1996). Greek-English lexicon of
the New Testament : Based on semantic domains (electronic edition of the 2nd ed.) (Page 592). New York: United
Bible societies.
John 5:30-32, “By Myself I can do nothing”

"I can do nothing on My own initiative. As I hear, I judge; and My judgment is just, because I
do not seek My own will, but the will of Him who sent Me. 31"If I alone bear witness of Myself,
My testimony is not true. 32"There is another who bears witness of Me, and I know that the
testimony which He bears of Me is true," (John 5:30-32, NASB).

Jehovah's Witnesses use these verses in their attempt to say that Jesus is not God. They reason
that if Jesus were really God in flesh, then He could do anything He wanted to do. But here we see that
Jesus says that He can do nothing on His own initiative. If this is true, then how can Jesus be God in
flesh?
The answer is that Jesus is both God and man in one person. This doctrine is called the hypostatic
union. As a man, Jesus was under the law and was obligated to keep the law (Gal. 4:4). In His
humbled state of being lower than the Angels (Heb. 2:9), Jesus was cooperating with the limitations of
being a man (Phil. 2:5-8). Therefore, He was incomplete subjection to the Father so that He might
fulfill the law and be the high priest sacrifice for our sins (Heb. 5:10).
Furthermore, Jesus did not begin His miracles until His baptism. It was at that point that the Holy
Spirit came upon Him. Therefore, Jesus was performing His miracles not by His own power, but by the
power of the Holy Spirit. This explains why in Matt. 12:22-32 when the Pharisees said that Jesus was
casting out demons by the power of the devil, Jesus said that blasphemy on the Holy Spirit of not be
forgiven. In other words, Jesus was doing His miracles by the power of the Holy Spirit and not under
His own divine power which He had laid aside the rightful use of while he walked this earth doing the
Father's will.
Therefore, these verses do not mean that Jesus is not divine. But it does mean that Jesus, as a
man, was completely and totally in submission to the will of the Father and that Jesus would only do
the will of the Father as the text clearly says.
John 8:58 and the Jehovah's Witnesses
"Jesus said to them: "Most truly I say to you, Before Abraham came into existence,
I have been," (John 8:58, New World Translation of the Jehovah's Witnesses).

The Jehovah's Witnesses deny that Jesus is God. So, when it comes to translating and interpreting
Bible verses that show the deity of Jesus, the Watchtower Bible and Tract Society will go to great
lengths to support their false presupposition. Sometimes they will even translate verses in a way that
is consistent with their belief system. In the Jehovah's Witness Bible, known as the New World
Translation (NWT), John 8:58 is a verse that they have translated in a manner deliberately consistent
with their theology. Following is the verse in context from the NASB.

"'Your father Abraham rejoiced to see My day, and he saw it and was glad.' 57The Jews
therefore said to Him, 'You are not yet fifty years old, and have you seen Abraham?' 58Jesus
said to them, 'Truly truly, I say to you, before Abraham was born, I am.' 59Therefore they
picked up stones to throw at Him; but Jesus hid Himself, and went out of the temple, " (John
8:56-59).

The issue at hand is the phrase "I am" in verse 58. The Jehovah's Witnesses have "translated" the
Greek present tense ("I am") into the English perfect tense ("I have been") which is more consistent
with their theological position that Jesus is not God in flesh. In the Greek, the words are "ego eimi."
Literally, this is "I am." "Ego eimi" is the present active indicative first person singular (I am), not the
perfect active indicative first person singular (I have been). It would seem that the natural and correct
translation into the English is "I am." But the NWT does not translate this into the present tense.
Why? I am firmly convinced it is because translating John 8:58 as "I am" would be too close to God
identifying Himself as the "I am" in Exodus 3:14. Therefore, Watchtower Bible and Tract Society has
opted for a different rendering.

"And God said to Moses, "I AM WHO I AM"; and He said, "Thus you shall say to the sons of
Israel, ‘I AM has sent me to you,’" (Exodus 3:14).

But the issue is not settled so easily. Does the Bible ever legitimately translate the present tense
'ego eimi' into the English perfect tense "I have been"? Yes it does. In John 14:8-9 it says, "Philip
said to Him, 'Lord show us the Father, and it is enough for us.' 9Jesus said to him, 'Have I been so
long with you, and yet you have not come to know Me, Philip? He who has seen Me has seen the
Father; how do you say, 'Show us the Father?'" Where Jesus says, "I have been" is in the Greek
present tense, 'ego eimi'. Literally, again, this is "I am." Here we have an example of the Greek
present tense being translated into the English perfect tense. This is the very same thing the
Watchtower organization claims is legitimate in John 8:58. Why is this translated into "I have been?"
Quite simply because if we did not do this, then the English would say, "I am with you so long...."
That is awkward in the English, so translators translate it as "Have I been so long with you...." It is
legitimate to do this in some instances where it is warranted in order to make the English more
readable and clear. But is it necessary to do this in John 8:58? I don't believe so -- unless your
underlying presupposition is that Jesus is not God in flesh.
Additionally, to make the issue even more complicated, there are some English Bible translations
that render John 8:58 other than "I AM." For example, the Living Bible (1973, a paraphrase) says,
"The absolute truth is that I was in existence before Abraham was ever born." The New Living
Translations says, "I existed." The Bible in Worldwide English translates it as "I already was." The
1960, 1973 NASB had a marginal rendering of "I have been." Because of this, the Jehovah's Witness
will claim the NWT is, therefore, legitimate since other Bibles have translations other than "I AM" in
John 8:58. But from what I have seen of these other translations, they are intended to be looser
renderings of the Greek and therefore take more liberties in translation. The NASB, for example, is
intended to be as literal as possible as does the KJV which both translate the verse as "I AM." The
1973 NASB marginal quote above is just that, marginal and is not what they rendered into the English
text. The preferred translation is "I am." Take Young's Literal Translation as another example. In
John 8:58 it states, ". . . Before Abraham's coming -- I am." In fact, other translations render it as:

1. "Verily, verily, I say unto you, Before Abraham was born, I am," (ASV).
2. "Verily, verily, I say unto you, Before Abraham was, I am," (KJV).
3. “Truly, truly, I say to you, before Abraham was born, I am," (NASB).
4. “I tell you the truth,” Jesus answered, “before Abraham was born, I am!" (NIV).
5. “Most assuredly, I say to you, before Abraham was, I AM," (NKJV).
6. “Truly, truly, I say to you, before Abraham was, I am," (RSV).
7. "Truly, truly, before Abraham was, I am," (NLT).
8. "Truly, truly, I say to you, before Abraham was, I am," (RSV)

9. "Verily, verily I say unto you, before Abraham was, I am!" (KJ21)137.

10. "Verily, verily, I say to you, Before Abraham's coming -- I am,' (YLT)138.

11. "Verily, verily, I say unto you, Before Abraham was, I am," (Darby).
12. "Antes que Abraham fuese, yo soy," (Spanish) - uses "yo soy" which is "I am."
13. "En vérité, en vérité, je vous le dis, avant qu'Abraham fût, je suis," (French) "je suis"
which is "I am."

Of course, the Jehovah's Witnesses will cite translations that have renderings other than the plain
"I am" for John 8:58 and in so doing claim legitimacy. Unfortunately, since different translations do
have different renderings, the debate will continue between the Jehovah's Witnesses and Christian
apologists until the Lord Jesus returns.
In the mean time, let's turn to page 467 of the 1969 Greek Interlinear used by the Watchtower
Bible and Tract Society:.

The Watchtower's own interlinear translates John 8:58 as "I am" even though in the NWT it
renders it as "I have been." In this, they admit that the Greek is indeed, "I am," the present tense.
They will not deny this. What they assert is that it should be translated into the English, "I have
been." Should it or could it? If it should, then Greek scholars would echo the NWT rendition in the
great majority of instances. But they do not. Essentially, the Watchtower organization is saying that
all the translations that have "I am" as the rendering are wrong, that the "proper" translation is "I
have been." In a footnote at the bottom of page 467 regarding John 8:58 in the NWT is this comment:

"I have been = ego eimi after the a'orist infinitive clause prin' Abraam genesthai and hence
properly rendered in the perfect tense. It is not the same as ho ohn', meaning "The Being" or
"The I Am") at Exodus 3:14, LXX"

The "LXX" is the Septuagint, a Greek translation of the Hebrew Old Testament. The question is
whether or not Jesus was quoting from the LXX or if He was simply translating the Hebrew. Again,
Exodus 3:14 says, "And God said to Moses, "I AM WHO I AM"; and He said, "Thus you shall say to the
sons of Israel, ‘I AM has sent me to you.’" The phrase "I AM WHO I AM" is rendered in the Greek LXX
as "Ego eimi ho on." Literally, this is "I am the being one." Most Bibles translate the Hebrew from
Exodus 3:14 as "I am" -- the present tense as did the Hebrew translators of the LXX. The LXX also

137
KJ21 = 21st Century King James Version
138
YLT = Young's Literal Translation
has it in the present tense which is what the Greek syntax states. Jesus uses the present tense in
John 8:58.
In spite of some of the translations regarding John 8:58, I do not believe the NWT's version of
John 8:58 is warranted for three reasons: First, it purports to "transmit his [God] thoughts and
declarations as accurately as possible." 139 I do not believe this is the case at all. Rather, I see the
Watchtower's bias against Jesus' divinity overtaking this verse and altering it as it has done in other
verses such as Heb. 1:8 and Col. 1:15-17. Second, the most literal translations such as the NASB, the
NIV, and the KJV do not render this verse as "I have been" but as "I AM." And, third, the context of
the verse does not support the JW position.

It isn't the English, but the Greek that upset the Pharisees

"'Your father Abraham rejoiced to see My day, and he saw it and was glad.' 57The Jews
therefore said to Him, 'You are not yet fifty years old, and have you seen Abraham?' 58Jesus
said to them, 'Truly truly, I say to you, before Abraham was born, I am.' 59Therefore they
picked up stones to throw at Him; but Jesus hid Himself, and went out of the temple, " (John
8:56-58).

The Pharisees didn't speak English. They spoke Hebrew and Greek. In the Greek text, Jesus uses
the present tense. It was this Greek present tense (I am) that upset the Pharisees so much, not the
perfect tense (I have been).
If Jesus were really saying to the Jews, "I have been," then why would the Pharisees want to kill
him (v. 59)? Since blasphemy, or calling yourself God, was punishable by death, isn't this a
confirmation that Jesus was saying "I am" and that the Jew's understood what he was saying?
Absolutely! That is why the best translation is simply, "I am." 140
I also need to mention that in Mark 14:62, where Jesus answered the High Priest who said, "Art
thou the Christ, the Son of the Blessed? 62And Jesus said, I am: and ye shall see the Son of man
sitting on the right hand of power, and coming in the clouds of heaven. 63Then the high priest rent his
clothes, and saith, What need we any further witnesses? 64Ye have heard the blasphemy. . ." Jesus
responded with "I am" which provoked the authorities and prompted them to seek death. This is
particularly revealing when we compare John 10:34 where the Pharisees want to kill Jesus because
they said He was making Himself equal to God. The phrase, "I AM" in these contexts would surely
imply that.
Undoubtedly, Jesus knew the difference in the Greek between "I am" and "I have been." Jesus did
not use the form "I have been" in the Greek, but used the form "I am." It is the Greek, not the
English that the Pharisees were upset about.

But the Pharisees were so upset

At this point, many of the Jehovah's Witnesses assert that by the time Jesus said, "Before Abraham
was I have been" (NWT), they had already been thoroughly agitated by Jesus and, basically, snapped
and tried to kill Him at this final comment. Furthermore, the Jehovah's Witnesses deny that Jesus was
claiming the divine title of Exodus 3:14 where God said to Moses, "I am that I am." But which do you
think would have upset the Pharisees more, saying "Before Abraham was, I am" or Before Abraham
was, I have been"? Obviously, the former would be more upsetting and that is exactly the phrase that
Jesus used.
If Jesus wanted to avoid any confusion with the Pharisees, why didn't He use one of the past
tenses? Certainly he must have known that saying "Before Abraham was, I am" to the Pharisees would
cause some problems. And it did. The aorist (I was), the perfect (I have been), and the pluperfect (I
had been) all deal with the past, yet Jesus chose to deliberately use the present tense "I am." He
used a past tense verb when describing Abraham ("before Abraham was..."), but a present tense verb
when describing Himself ("I am"). He deliberately brought attention to the words, "I am." The
Pharisees understood this and was indeed the last straw for them.
139
New World Translation, 1961, page 5.
140
I should note that, most probably Jesus spoke to the Pharisees in Aramaic, a Hebrew Dialect. It is possible He
spoke to them in Greek. But, since all we have is the NT Greek and no Aramaic writings of the NT, we must work
from what the Greek says.
Conclusion

The Jehovah's Witnesses have spent a great deal of time developing and crafting linguistic
arguments to favor their translation of John 8:58. Wading through their arguments dealing with Greek
tenses, verb forms, and grammar rules is beyond the scope of this paper. However, it is sufficient to
mention that the Jehovah's Witnesses have a theological bias against the deity of Christ. Their
translation of John 8:58 and their attempts to justify this translation are directly related to their
presuppositions against Christ and his deity.
The Watchtower Bible and Tract Society teaches its members to deny the deity of Christ. From this
base, any and all affirmations to Jesus' deity will be undermined in whatever way possible. John 8:58
is just another example of this bias.
John 8:58, & 10:30-33, I AM, and the Jehovah’s Witnesses
Jesus said in John 8:58, "Before Abraham was, I am." This is a very important verse to
Trinitariansbecause it is one of the places we use to show that Jesus is God. We maintain that Jesus
attributed the divine name of God ("I AM" from Exodus 3:14), to Himself. But this verse alone may
not be sufficient to prove His deity. There are a combination of other verses that contribute to the
doctrine. Nevertheless, there are many non-Christian groups that deny that Jesus is God. Therefore,
when they come to this verse, it must be dealt with. The reason is simple. If Jesus did say, "I am", it
would give strong evidence that Jesus was claiming to be God.
This paper will not attempt to analyze the Greek translation principles that have lead various Bibles
to render John 8:58 as, "I have been," or "I was in existence," etc. Suffice it to say that the best
recognized translations which have sought literal renderings of the text, have translated the verse as
"I am": NASB, NIV, KJV, RSV, etc.
The Jehovah's Witness Watchtower organization claims that the best translation of John 8:58 is
"Before Abraham was, I have been." Notice that they do not have it say, "I am." Is it legitimate for
the Watchtower organization to insist that John 8:58 is best translated as "I have been"? Let's take a
look.

Ego Eimi means "I am"

In Greek, the words recorded in John 8:58 are "'prin abraam genesthai ego eimi." Literally, this is
"Before Abraham was existing, I am." "Ego eimi" is literally, "I am." This is the present tense. To say "I
have been" is to use the perfect tense. In Greek, his would have been "aemane." But Jesus didn't use
it here. He used the present tense, "ego eimi" which is "I am."
There are places, however, in the New Testament where the Greek present tense of 'ego eimi', "I
am", can be translated into the English perfect tense, "I have been." An example of this is John 14:9
where Jesus says, "Have I been so long with you, and yet you have not come to know Me..." In this
verse, "Have I been" is originally the Greek present tense, 'ego eimi'. But here, Jesus was answering
the statement in verse 8, "Lord, show us the Father, and it is enough for us." Since in English it is
awkward to say, "I am with you so long and you still don't know me....?", it is then rendered as, "Have
I been with you so long and you have not come to know me....?" The translation of the Greek present
into the English perfect tense is perfectly justifiable here because it doesn't make sense in English. But
is it the case with John 8:58? Must it be translated as "I have been"? No. There is no linguistic
requirement to translate it as "I have been" particularly when you notice that the Jews wanted to kill
Jesus after he said, "ego eimi."

Two Views

Some say that the reason the Jews wanted to kill Jesus after He said, "Before Abraham was, I am"
is because it was the last straw in a series of difficult and insulting things Jesus had been saying to the
Jews in John chapter 8. Others say that the Jews wanted to kill Jesus for saying "Before Abraham was,
I am," because "I am" is close to God saying "I am that I am" in Exodus 3:14. In other words, we can
make the case that for Jesus to say, "Before Abraham was, I am" was equivalent to claiming God's
name for Himself. This is something the Jews would absolutely protest. Let's look at the arguments.
The first argument states that Jesus had upset the Jews so much by what He had been saying that
when he finally made his statement in verse 58, it was the last straw, the Jews snapped, and then
they tried to kill him. But, they maintain, it wasn't because Jesus was claiming the divine title. They
had just had enough.
What had Jesus been saying? Following is a list of some key statements by Jesus in chapter 8.

1. "I am the light of the world" (8:12)


2. "I am He who bears witness of Myself, and the Father who sent Me bears
witness of me" (8:18).
3. "You don't know me or my Father" (8:19).
4. "You are from below, I am from above" (8:23).
5. "Unless you believe that I am, you shall die in your sins" (8:24).
6. "The things which I heard from Him [God the Father], these I speak to the world"
(8:26).
7. "I speak these things as the Father taught me" (8:28).
8. "I always do the things that are pleasing to Him" (8:29).
9. "I speak the things which I have seen with My Father. . ." (8:38).
10. ". . . you are seeking to kill Me, a man who has told you the truth, which I heard
from God, this Abraham did not do" (8:40).
11. ". . . I proceeded forth and have come from God. . ." (8:42).
12. "Truly, truly, I say to you, if anyone keeps My word he shall never see death"
(8:51).
13. "It is my Father who glorifies Me..." (8:54).
14. "Before Abraham was, I am" (8:58).

The preceding list has many profound statements. It is perfectly understandable that the Jews
would be upset. But, it was Jesus' statement in 8:58 that triggered their murderous attempt. Was it
because Jesus said "Before Abraham was, I have been" or "Before Abraham was, I am." Which would
be the phrase most likely to be the last straw for the Jews? It is quite possible that either statement
would be sufficient. But, of course, any claim by Jesus to the divine name would be a stronger
motivation for the Jews to kill Him.
Also, notice statements 1, 5, 11, and 14. These are clear declarations by Jesus where He exalts
Himself to heavenly level. The Jews could easily see this and would protest, particularly when Jesus
said, "Before Abraham was, I am. Since He did say, in Greek, "I am", it is more likely that the Jews
wanted to kill Jesus for blasphemy. Consider Leviticus 24:16 which says, ‘Moreover, the one who
blasphemes the name of the Lord shall surely be put to death; all the congregation shall certainly
stone him" (NASB).

The Connection with John 10:30-33

Capital punishment was only for serious sins: blasphemy, adultery, etc.. From what I can see in
the Bible, saying you had a preexistence isn't blasphemy. However, claiming to be one with God is
quite different. In John 10:30-33 Jesus said, "I and the Father are one. The Jews took up stones
again to stone him. 32Jesus answered them, "I showed you many good works from the Father; for
which of them are you stoning me?" 33"The Jews answered Him, 'For a good work we do not stone You,
but for blasphemy; and because You, being a man, make Yourself out to be God" (NASB).
Between John 8:59 where the Jews picked up stones to kill Jesus and John 10:30-33 where they
again picked up stones to kill him, there is no mention of stoning whatsoever. John 10:31 is
referencing back to John 8:59 when it says "The Jews took up stones again to stone Him." Note that
they again wanted to kill Him and this time they give the reason why. They said that Jesus was
claiming to be God. Now, where would they get that idea? Could it have been where he said, "Before
Abraham was, I am"? Could it be from where Jesus said, "I and the Father are one" (10:30). Since
they wanted to kill Him both times, it would seem that Jesus had been making some very serious
claims. Or was it simply that the Pharisees misunderstood Jesus and that Jesus never did claim to be
God?
But, if Jesus was not claiming to be God in John 8:58 and 10:30, then what was it that He said
that warranted such a violent response from the Jews in both cases? What phrase from Jesus did the
Jews react to and what 'misunderstanding' did they have about what it was Jesus said that led them to
claim that he was making Himself out to be God?
In my opinion, the best explanation for the Jews wanting to kill Jesus is because Jesus was claiming
equality with God. They considered this blasphemy. The cults, like the bad religious leaders who
opposed Jesus, deny who Jesus really is as well.
John 17:3 and the Only True God
"And this is eternal life, that they may know Thee, the only true God, and Jesus Christ whom
Thou hast sent," (John 17:3, NASB).

The Jehovah's Witnesses, among others, cite John 17:3 as a proof text to deny the Trinity and
claim that Jesus Christ is not God. They reason is that if Jesus were God, then He would not have
called the Father, "the only true God." If the Father is the only true God, then it must require that
Jesus cannot be God.
First of all, it is not proper to make a theological doctrine out of one verse. Of this the Jehovah's
Witnesses are sometimes guilty. Nevertheless, they do tend to take one or two verses on a subject
and use them to interpret all the others. Instead of getting a balanced position, they arrive at an
interpretation that is in agreement with their theological position. This is called "proof-texting" and is
something the Jehovah's Witnesses do frequently.
Second, the context of Jesus' comment was that He was speaking as a man to His God.
Remember, Jesus is both God and man, second person of the Trinity, the word made flesh (John
1:1,14). Since He was both divine and man, as a man He would naturally, and properly say that His
Father was the only True God. He was not denying His own divinity, but affirming the Trueness of God
as was done in the OT: “And now, O Lord our God, deliver us from his hand that all the kingdoms of
the earth may know that Thou alone, Lord, art God,” (Isaiah 37:20). The truth is that Jesus was a
man made under the Law (Gal. 4:4) and as a man He would be subject to God. Only in this case,
Jesus was subject to the Father. That is why Jesus called the Father the only true God. But it is not a
phrase that excludes Christ for Christ Himself said "Before Abraham was, I am" (John 8:58) and did
not deny being called God by Thomas in John 20:28.
Third, John 17:3 must be examined in the light of the totality of scripture. We see that Jesus is
called God in John 1:1,14; 8:58; 20:28; Col. 2:9; Heb. 1:8. Therefore, John 17:3 cannot be
interpreted in a way that disagrees with other scriptures. Of course, some people simply state that
John 17:3 cannot allow for Jesus being God. But the simple fact is that Jesus is called God by God and
others. Therefore, the whole of scripture must be harmonized.
Fourth, this verse reflects the sonship of Jesus. The Father and the Son have a unique
relationship. Jesus is the eternal Son. The terms Father and Son denote a relationship which is why
God is called the God of the Son in 2 Cor. 11:31.
Fifth, if we are to be consistent using the Jehovah's Witness logic that the Father is the only true
God, then the following verses present a problem -- if we use their logic.

1. "For certain persons have crept in unnoticed, those who were long beforehand marked out for
this condemnation, ungodly persons who turn the grace of our God into licentiousness and deny
our only Master and Lord, Jesus Christ," (Jude 4, NASB).
A. Does this mean that the Father is not our Master and Lord? Of course not. Yet, Jesus is
called our only Master and Lord.
2. "There was the true light which, coming into the world, enlightens every man. 10He was in the
world, and the world was made through Him, and the world did not know Him," (John 1:9-10).
A. Here we see Jesus being called the true light. Does this mean that the Father is not the
true light? If not, then we have both the Son and the Father being the true light.
3. "And Jesus said to him, "Why do you call Me good? No one is good except God alone," (Mark
10:18, NASB).
A. Does this verse mean that Jesus is not good? Jesus said only God was good. Then, if we
use the Jehovah's Witness logic, Jesus is not good. Of course, that doesn't make any
sense.
4. “I, even I, am the Lord [YHWH}; And there is no savior besides Me," (Isaiah 43:11).
A. We know that Jesus is the Savior. Again, according to Witness logic, Jesus could not be the
Savior since the Bible tells us that YHWH is the only Savior.
5. "Blessed be the Lord God, the God of Israel, Who alone works wonders," (Psalm 72:18, NASB).
A. Jesus performed many miracles. But if it is the Lord [YHWH] is the one who alone performs
wonders, how then can it be that Jesus also?
6. "Thus says the Lord, your Redeemer, and the one who formed you from the womb, “I, the Lord
[YHWH], am the maker of all things, Stretching out the heavens by Myself, And spreading out the
earth all alone,'" (Isaiah 44:24, NASB).
A. According to John 1:3 and Col. 1:16-17 Jesus made all things. With JW logic would have a
problem.
B. Col. 1:16-17 says, "For by Him all things were created, both in the heavens and on earth,
visible and invisible, whether thrones or dominions or rulers or authorities—all things have
been created by Him and for Him. 17And He is before all things, and in Him all things hold
together," (NASB). Since God is alone created all things, how could Jesus have done it?
For more on this subject see the paper on Col. 1:16-17

As we can see, we cannot simply make a doctrine out of one verse. To do so is to invite error and
it only serves to use the Bible to validate preconceived ideas about doctrine.
1 Cor. 1:2, Call upon the name of the Lord Jesus

The Jehovah's Witnesses deny that Jesus is God in flesh. Therefore, they have produced a Bible,
the New World Translation, that reflects their theological bias against Christ's deity. But, their Bible
cannot completely remove all the references that show who Jesus really is. Such is the case with the
phrase, "to call upon the name of the Lord."
We can see in the Bible where the Old Testament believers "call upon the name of the Lord," as a
reference to God. Literally, in Hebrew this is "call upon the name of Jehovah." This phrase with its
variations "calling upon..." and "called upon..." etc., occurs eleven times in the Old Testament in the
King James Version, which the Jehovah's Witnesses used as their Bible for many years.
"Jehovah" is an English(ized) form of the Hebrew four letters YHWH from which we get God's name
"Jehovah" or "Yahweh." Most modern English Bibles translate the four letters "YHWH" into the word
"Lord." This is done because no one knows the true pronunciation of God's name and also because the
Jews wanted to avoid violating the sacredness of God's name if they pronounced it incorrectly.
Nevertheless, in every instance of this usage "Lord" in the Hebrew phrase, "call upon the name of the
Lord," it is the Hebrew "YHWH" and is always in reference to God.
Around 250 B.C., the Hebrew Old Testament was translated into Greek by Jewish scholars. This
translation is called the Septuagint and is known by the designation LXX. The Hebrew phrase to "call
upon the name of the Lord [YHWH]," of course, was also translated into Greek. For example, in Joel
2:32 it says, "...whosoever shall call on the name of the LORD [YHWH] shall be delivered..." This
Greek translation in the LXX is "hos an epikalesatai to onoma kuriou," literally, "whosoever shall call
upon the name of the Lord."
When we look at this phrase in the Old Testament, we see that "to call upon the name of the Lord"
was used to designate prayer to God in 1 Kings 18:24,37 and Psalm 116:4. Please consider the
following verses from the KJV.

• 1 Kings 18:24, "And call ye on the name of your gods, and I will call on the name of the
LORD [YHWH]: and the God that answereth by fire, let him be God."
• 1 Kings 18:37, "Hear me, O LORD, hear me, that this people may know that thou art the
LORD [YHWH] God, and that thou hast turned their heart back again. 38Then the fire of the
LORD fell, and consumed the burnt sacrifice, and the wood, and the stones, and the dust,
and licked up the water that was in the trench."
• Psalm 116:4, "Then called I upon the name of the LORD [YHWH] ; O LORD [YHWH], I
beseech thee, deliver my soul."

Clearly, the phrase is used of God in reference to prayer to Him. The phrase is never used in
reference to anyone else other than God. Remember that "call upon the name of the Lord [YHWH]" in
Hebrew was translated into Greek by the Jews in the LXX and they rendered "YHWH" into the Greek
"kurios," which means "lord." This phrase was then used of Jesus in

• "Unto the church of God which is at Corinth, to them that are sanctified in Christ Jesus,
called to be saints, with all that in every place call upon the name of Jesus Christ our Lord,
both theirs and ours," (1 Cor. 1:2).
• "...call upon the name of our Lord Jesus Christ..." NASB, ASV, ASV1901,
• "...call on the name of our Lord Jesus Christ..." KJV, NJKV, NIV, RSV,
• "...calling upon the name of our Lord, Jesus Christ, their Lord and ours," NWT.
o NOTE: In the Greek, it is literally, "Call upon the name of the Lord OF US, Jesus
Christ. This is why the English says, "Call upon the name of OUR Lord." Therefore,
the phrase is the same.

This is clear evidence that Jesus is called YHWH since the same phrase used to address God is also
used to address Jesus.
Below is a chart that transliterates the Greek into English phonetic sounds so you can check the
Greek for yourself in the Old Testament Greek Septuagint and the New Testament Greek.

"Call upon the name of the Lord"


Old Testament (LXX) New Testament
(Literal translation) (Literal translation)
Joel 2:32 - call on the name of the LORD
epikalesatai to onoma kuriou
call upon the name of Lord with all that in every place call upon the name of
Jesus Christ our Lord, both theirs and ours,"
1 Kings 18:24 - call on the name of the
------------
LORD
1 Cor. 1:2
epikalesomai en onomati kuriou tou theou
epikaloumenois to onoma tou kuriou
call upon in name of lord of the of god
calling upon the name of the of lord
Psalm 116:4 - called I upon the name of the hamon iasous kristou
LORD of us Jesus Christ
kai to onoma kuriou epekalesaman
and the name of lord call upon

As you can see in 1 Kings 18:24,37 and Psalm 116:4 above, the phrase is used to designate prayer
to God. This same phrase is used in reference to Jesus in 1 Cor. 1:2.
It is obvious that the Christian church is to call upon the name of the Lord, Jesus Christ. This is an
obvious declaration of prayer to Christ designating His divine nature.

To call upon the name of the Lord

Following is a list of every occurrence of the phrase "to call upon the name of the Lord," and its
various forms (call on, call upon, called on, etc.), in the KJV Bible along with the New World
Translation's rendering of the phrase in each verse cited.
It is here so you can see for yourself how it is used in both the Old and New Testaments.

1. "Call upon the name of the Lord" occurs five times in the KJV:
A. Gen. 4:26, "And to Seth, to him also there was born a son; and he called his name Enos:
then began men to call upon the name of the LORD."
i. "...calling on the name of Jehovah," (NWT).
B. Psalm 116:13, "I will take the cup of salvation, and call upon the name of the LORD."
i. "...and on the name of Jehovah I shall call," (NWT).
C. Psalm 116:17, "I will offer to thee the sacrifice of thanksgiving, and will call upon the name
of the LORD."
i. "...and on the name of Jehovah I shall call," (NWT).
D. Zeph. 3:9, "For then will I turn to the people a pure language, that they may all call upon
the name of the LORD, to serve him with one consent."
i. "...to call upon the name of Jehovah," (NWT).
E. Romans 10:13, "For whosoever shall call upon the name of the Lord shall be saved."
i. "For "everyone who calls on the name of Jehovah will be saved," (NWT).
2. "Call on the name of the Lord" occurs four times in the KJV
A. 1 Kings 18:24, "And call ye on the name of your gods, and I will call on the name of the
LORD: and the God that answereth by fire, let him be God."
i. "...call upon the name of Jehovah," (NWT).
B. 2 Kings 5:11, "But Naaman was wroth, and went away, and said, Behold, I thought, He will
surely come out to me, and stand, and call on the name of the LORD his God, and strike his
hand over the place, and recover the leper."
i. "...call upon the name of Jehovah," (NWT).
C. Joel 2:32, "And it shall come to pass, that whosoever shall call on the name of the LORD
shall be delivered: for in mount Zion and in Jerusalem shall be deliverance, as the LORD
hath said, and in the remnant whom the LORD shall call."
i. "...calls upon the name of Jehovah," (NWT).
D. Acts 2:21, "And it shall come to pass, that whosoever shall call on the name of the Lord
shall be saved."
i. "...calls upon the name of Jehovah," (NWT).
3. "Called on the name of the Lord" occurs one time in the KJV:
A. Gen. 13:4, "Unto the place of the altar, which he had made there at the first: and there
Abram called on the name of the LORD."
i. "...call there on the name of Jehovah," (NWT).
4. "Called upon the name of the Lord" occurs three times in the KJV:
A. Gen. 12:8, "And he removed from thence unto a mountain on the east of Bethel, and
pitched his tent, having Bethel on the west, and Hai on the east: and there he builded an
altar unto the LORD, and called upon the name of the LORD."
i. "...call on the name of Jehovah," (NWT).
B. Gen. 26:25, "And he builded an altar there, and called upon the name of the LORD, and
pitched his tent there: and there Isaac’s servants digged a well."
i. "...called on the name of Jehovah," (NWT).
C. Psalm 116:4, "Then called I upon the name of the LORD; O LORD, I beseech thee, deliver
my soul."
i. "But upon the name of Jehovah I proceeded to call," (NWT).
5. "Calling on the name of the Lord" occurs one time in the KJV:
A. Acts 22:16, "And now why tarriest thou? arise, and be baptized, and wash away thy sins,
calling on the name of the Lord."
i. "...calling upon his name," (NWT).

Paul, in his letter to the Corinthians said that Christians everywhere "call upon the name of the
Lord Jesus." The Jehovah's Witnesses need to do this. They need to pray to Jesus, ask Him to forgive
them of their sins, and they need to come to Him (Matt. 11:28). Jesus said, "If you ask Me anything
in My name, I will do it," (John 14:14, NASB).

Objection Answered

"The translations of the books of the OT differ in style, accuracy, and substance, indicating that
there was no single original translation into Greek....Manuscripts found at Qumran among the Dead
Sea Scrolls and other early manuscripts and quotations from the Septuagint in ancient writings all
indicate that revisions were constantly being made to the Septuagint." (Paul J. Achtemeier, Harper's
Bible Dictionary, Includes index., 1st ed., Page 925 (San Francisco: Harper & Row, 1985).
Some critics of the position of this paper regarding "calling upon the name of the Lord" say that the
Greek word "kurios", "Lord" was substituted for the tetragrammaton (YHWH - ) in the 2nd century
manuscripts of the Septuagint, well after the writing of 1 Corinthians. In other words, they say that
the orignal Septuagint, though written in Greek, had the Hebrew consonants (YHWH - ) inserted
for the divine name and that later, this was changed from YHWH to LORD (or the Greek "kurios"). This
would mean that Paul could not have been referencing the Septuagint in 1 Cor. 1:2 and the argument
in this paper is invalid.
If it is true, and I am not saying that it is, that the Septuagint did not contain the word "Lord" in
reference to "YHWH", it does not invalidate the argument because the practice of substituting "YHWH"
for another word "adonai" (Hebrew for Lord), was commonly done among the Jews and was well
known by Paul. He could easily have provided this word substitution as a natural thing to do when
writing 1 Corinthians. We see that in the existing Septuagint versions that this is exactly the case.
Second, since Paul was writing to the Corinthians who spoke Greek, to write in Greek and then insert
the Hebrew tetragrammaton (YHWH - ) into the text would have been confusing to them since they
did not read Hebrew. Remember, Corinth is about 750 miles from Israel where they spoke Hebrew.
Therefore, it would have been very natural for Paul to insert the Greek "kurios" (Lord) for the Hebrew
name of God (YHWH - ) when quoting Old Testament References. In fact, Paul did this very thing
in several places:

• Rom. 4:8, "Blessed is the man whose sin the Lord will not take into account."
• Psalm 32:2, "How blessed is the man to whom the Lord does not impute iniquity,
• Rom. 9:29, "And just as Isaiah foretold, 'Except the Lord [kurios] of Sabaoth had left to us
a posterity. We would have become as Sodom, and would have resembled Gomorrah."
• Isaiah 1:9, "Unless the Lord [YHWH] of hosts Had left us a few survivors, We would be like
Sodom, We would be like Gomorrah."
• Rom. 10:13, "for 'Whoever will call upon the name of the Lord [kurios] will be saved.'"
• Joel 2:32, "And it will come about that whoever calls on the name of the Lord [YHWH] will
be delivered,"
• Rom. 11:34, "For who has known the mind of the Lord [kurios], or who became His
counselor?"
• Isaiah 40:13, "Who has directed the Spirit of the Lord [YHWH], Or as His counselor has
informed Him?"
• Rom. 15:11, "And again, “Praise the Lord [kurios] all you Gentiles, and let all the peoples
praise Him.”
• Psalm 117:1, "Praise the Lord [YHWH], all nations; Laud Him, all peoples!"
• 1 Cor. 2:16, "For who has known the mind of the Lord [kurios], that he should instruct
Him? But we have the mind of Christ."
• Isaiah 40:13, "Who has directed the Spirit of the Lord [YHWH], Or as His counselor has
informed Him?"
• 1 Cor. 3:20, "The Lord [kurios] knows the reasonings of the wise, that they are useless.”
• Psalm 94:11, "The Lord [YHWH] knows the thoughts of man, That they are a mere breath."
• 1 Cor. 10:26, "for the earth is the Lord’s [kurios], and all it contains."
• Psalm 24:1, "The earth is the LORD’s [YHWH], and everything in it, the world, and all who
live in it."
• 2 Cor. 10:17, "But he who boasts, let him boast in the Lord [kurios]."
• Jer. 9:24, "but let him who boasts boast of this, that he understands and knows Me, that I
am the Lord [YHWH] who exercises lovingkindness, justice, and righteousness on earth; for
I delight in these things," declares the Lord [YHWH]."

At the very least, this substantiates that Paul's practice was to substitute "kurios" for the
tetragrammaton (YHWH - ) when he quoted the Old Testament. Since we see that the phrase
"call upon the name of the Lord" is used only of God in the Old Testament, it is fair to say that the
phrase applies to Jesus in 1 Cor. 1:2.
Finally, there are only two places in the entire Old Testament where the phrase "call upon the
name of" ("Lord" is omitted) is used in reference to someone other than God.

• 1 Kings 18:24-26, "Then you call on the name of your god, and I will call on the name of
the Lord, and the God who answers by fire, He is God." And all the people answered and
said, "That is a good idea. 25So Elijah said to the prophets of Baal, "Choose one ox for
yourselves and prepare it first for you are many, and call on the name of your god, but put
no fire under it." 26Then they took the ox which was given them and they prepared it and
called on the name of Baal from morning until noon saying, "O Baal, answer us." But there
was no voice and no one answered. And they leaped about the altar which they made."
• Isaiah 44:5, "This one will say, ‘I am the Lord's'; and that one will call on the name of
Jacob."
In these two cases we see that they do not affect the issue of "call upon the name of the Lord" in
any way since one is clearly about Baal and the other is about Jacob and the word "Lord" would not
suit in the translation of either. If we combine this with the knowledge that Paul translated the
Hebrew YHWH ( ) into the Greek "kurios" (Lord), we can easily see that 1 Cor. 1:2 is most
probably a reference to the Old Testament phrase, "call upon the name of YHWH."

Objection Answered

An objection has been raised by Jehovah's Witnesses who state that the word "Lord" functions as a
noun in a different way than YHWH does. They say that "Call upon the name of YHWH" is a different
sense than "Call upon the name of the Lord of us, Jesus Christ" because the word "Lord" takes a
possessive pronoun "of us". Therefore, they say, the word "Lord" does not function as a proper noun
as does "YHWH" and the phrase used of Christ is not the same as that used in the LXX. But, this is
just an attempt to strip the phrase of its biblical power so as to continue teaching taht Jesus is not
divien.
Nevertheless, the word "Lord" in the LXX phrase "Call upon the name of the Lord" is capable of
taking a possessive pronoun. This is the nature of the phrase as it appears throughout the LXX. In
other words, the fact that the LXX phrase "Call upon the name of the Lord (gk. kurios)" has the
capability of receiving a possessive pronoun, it does not change the meaning of the phrase -- whether
or not a possessive pronoun "of us" is added to it or not. We do not see "the YHWH of us." But, we do
see "the Lord of us" (our Lord). For example, in Psalm 147:5 it says, "Great is our Lord." In the LXX
it is "kurios hamon" (the Lord of us). In Neh. 10:29 of the LXX it says, "..of our Lord." The Greek is
"kuriou hamon."
The word "kurios" can take a possessive pronoun. The fact that "of us" is added to the Greek word
"kurios" does not mean that the word is no longer used in place of God's name nor does it invalidate
the power of the phrase, "Call upon the name of the Lord of us, Jesus Christ," (1 Cor. 1:2). As I have
demonstrated earlier, the LXX substitutes YHWH for kurios in the phrase "Call upon the name of
YHWH. Therefore, by the nature of the word "lord," it is possible to add a possessive pronoun (of us).
Does this then mean that the phrase looses its power?
Undoubtedly Paul knew of the phrase in the LXX. He knew that it carried the weight of proclaiming
and appealing to God -- since that is how it is used in the LXX. Should we accept the Jehovah's
Witness' notion that by adding "of us" onto the end of the phrase that the phrase then somehow looses
its majesty and Old Testament context? Hardly.
If we look at Romans 10:13 we see the very same phrase, "Call upon the name of the Lord." The
only difference is that "of us" is not there. Are we to believe that Paul who wrote Romans and 1
Corinthians used the same phrase twice but in 1 Cor. 1:2 did not mean to carry with it the Old
Testament usage by simply adding "of us" to it? Well, that is waht some Jehovha's Witnesses want us
to believe.
Unfortunately for them, the truth is that the same phrase that is used only of God in the Old
Testament is also used of Jesus in the New Testament.

_________

Note: I am indebted to a poster on my website . Mr. Frazier pointed out the following quotes
from three different Christian commentators.

• "Hereby Christians are distinguished from the profane and atheistical, that they dare not live
without prayer; and hereby they are distinguished from Jews and Pagans, that they call on the
name of Christ. He is their common head and Lord. Observe, In every place in the Christian
world there are some that call on the name of Christ. God hath a remnant in all places; and we
should have a common concern for and hold communion with all that call on Christ’s name."
o Henry, Matthew. Matthew Henry's Commentary : On the Whole Bible. electronic ed. of
the complete and unabridged edition., 1 Co 1:1. Peabody: Hendrickson, 1996, c1991.
• "with all that in every place call upon . . . Christ--The Epistle is intended for these also, as
well as for the Corinthians. The true CATHOLIC CHURCH (a term first used by IGNATIUS
[Epistle to the Smyraeans, 8]): not consisting of those who call themselves from Paul, Cephas,
or any other eminent leader ( 1Cr 1:12 ), but of all, wherever they be, who call on Jesus as
their Saviour in sincerity (compare 2Ti 2:22 ). Still a general unity of discipline and doctrine in
the several churches is implied in 1Cr 4:17 7:17 11:16 14:33, 36. The worship due to God is
here attributed to Jesus (compare Joe 2:32 Mat 4:10 Act 9:14 )." (emphasis added)
o Robert Jamieson, A. R. Fausset and David Brown, Commentary Critical and Explanatory
on the Whole Bible (1871) http://www.blueletterbible.org/tmp_dir/c/1050964815-
1211.html
• With all that call upon Associative instrumental case with sun rather than kai (and), making
a close connection with "saints" just before and so giving the Corinthian Christians a picture of
their close unity with the brotherhood everywhere through the common bond of faith. This
phrase occurs in the LXX (Genesis 12:8; Zechariah 13:9) and is applied to Christ as to Jehovah
(2 Thessalonians 1:7,9,12; Philippians 2:9,10)."
o Robertson's Word Pictures of the New Testament
http://bible1.crosswalk.com/Commentaries/RobertsonsWordPictures/
rwp.cgi?book=1co&chapter=001&verse=002&next=003&prev=001
• Also, from another commentary
• "The worship due to God is here attributed to Jesus (compare Joe 2:32 ; Mt 4:10; Ac 9:14 )."
o Jamieson, Robert, A.R. Fausset, and David and Brown. Commentary Critical and
Explanatory on the Whole Bible. electronic ed., 1 Co 1:2. Oak Harbor: Logos Research
Systems, Inc., 1997.
Col. 1:15 and the Jehovah's Witnesses
"He is the image of the invisible God, the firstborn of all creation; because by means of him all
[other] things were created in the heavens and upon the earth, the things visible and the
things invisible, no matter whether they are thrones or lordships or governments or authorities.
All [other] things have been created through him and for him. Also, he is before all [ other]
things and by means of him all [other] things were made to exists."
(Col. 1:15-17, for context. The New World Translation – Note the NWT’s addition of “other”
into the text four times. This is discussed in the next paper.)

The Jehovah's Witnesses interpret the word "firstborn" here to mean "first created" because it is
consistent with their theological presupposition that Jesus is a created thing. Of course, Jesus, the
word become flesh (John 1:1,14) is not a created thing. But that hasn't stopped the Watchtower
organization from claiming He is. Nevertheless, there is a Greek word for "first created" and it was in
use at the time of Paul's writing to the Colossians. He did not use it here. The Greek for "firstborn" is
proto with tikto which would give us "firstborn" and that is what we find here in Colossians 1:15. The
Greek for "first created" would be proto with ktizo and it is not used here.
Second, the biblical use of the word "firstborn" is most interesting. It can mean the first born child
in a family (Luke 2:7), but it can also mean "pre-eminence." In Psalm 89:20, 27 it says, " I have found
David My servant; with My holy oil I have anointed him...I also shall make him My first-born " (NASB).
As you can see, David, who was the last one born in his family was called the firstborn by God. This is
a title of preeminence.
Third, firstborn is also a title that is transferable:

• Gen. 41:51-52, "And Joseph called the name of the first-born Manasseh: For, said he,
God hath made me forget all my toil, and all my father’s house. And the name of the
second called he Ephraim: For God hath made me fruitful in the land of my affliction"
(NASB)
• Jer. 31:9, "...for I am a father to Israel, and Ephraim is My firstborn (NASB)."

Scripture best interprets scripture. Firstborn does not require a meaning of first created as the
Jehovah's Witnesses say it means here. "Firstborn" can mean the first born person in a family and it
can also be a title of preeminence which is transferable. That is obvious since Jesus is God in flesh
(John 1:1,14) and is also the first born son of Mary. In addition, He is the pre-eminent one in all
things. The Jehovah's Witnesses should consider this when they examine Col. 1:15. They should also
abandon the Watchtower which guides them in their thinking and believing.
Col. 1:16-17 and the Jehovah's Witnesses
"because by means of him all [other] things were created in the heavens and upon the earth,
the things visible and the things invisible, no matter whether they are thrones or lordships or
governments or authorities. All [other] things have been created through him and for him.
17
Also, he is before all [other] things and by means of him all [other] things were made to
exists." (The New World Translation - Emphasis added)

The Jehovah's Witness organization has altered the biblical text to suit to its theological
presupposition that Jesus is a created thing. This is why the new world translation adds the word
"other" four times in Col. 1:16-17, even though it is not in the Greek text. There exists two Greek
words for "other": allos which means another of the same kind; and heteros which means another of
a different kind. Paul could have used either word here if he wanted to show that Jesus was "another"
created thing. But he did not. There is no linguistic reason at all to insert this word here four times --
unless you are trying support the presupposition that Jesus is not God.
Below is a copy out of the Jehovah's Witness Kingdom interlinear. This book has the Greek words
and their exact English translation underneath each word. The right hand column is how the New
World translation renders the Greek into the English. I have added red squares is in order to
demonstrate the additions into the English text that are not supported in the Greek.

In the New World Translation you will notice that the word "other" is in brackets. This is an
admission that the words are not in the original text. Of course, the Watchtower Organization claims
that the insertion of the word "other" four times is necessary to clarify the text. It isn't. If anything, it
misleads the reader. Nevertheless, if you have the opportunity, ask a Jehovah's Witness to read the
text without saying the word "other." Usually, he will have difficulty. Also, ask him what he thinks the
text is saying without the word "other" added in. It will be an interesting discussion.
Basically, Jehovah's Witness theology maintains that God created Jesus and then Jesus created all
other things.141 If follows that if Jesus "was used by Jehovah in creating all other things" 1 then Jesus
was with God and used by God as the instrument of creation. Unfortunately for Jehovah's Witnesses
God says that He created the heavens and earth "all alone."

141
"Jehovah's first creation was his 'only-begotten Son'. . . was used by Jehovah in creating all other things", Aid
to Bible Understanding, pp. 390-391.
"Thus says the LORD, your Redeemer, and the one who formed you from the womb, 'I, the
LORD, am the maker of all things, stretching out the heavens by Myself, and spreading out the
earth all alone'" (Isaiah 44:24, NASB).

If the Jehovah's Witness is correct, then how is it possible for the LORD (YHWH, or Jehovah) to
stretch out the heavens by Himself, all alone (not angels with Him), and yet Jesus "the first created
thing," be the one who did it? They can't both be true to the Jehovah's Witness.
The truth is that Jehovah is the name of God. God is a trinity, and therefore Jesus can be the
creator of all things and YHWH can do it by himself since God is Father, Son, and Holy Spirit.
This is a very simple and direct demonstration in Scripture that Jesus is the Lord God in flesh. If
He is not, then the Jehovah's Witnesses have a biblical contradiction on their hands.
Heb. 1:6 and the Jehovah's Witnesses
"But when he again brings his Firstborn into the inhabited earth, he says: 'And let all God's
angels do obeisance to him," (The New World Translation, 1981.)

The problem with this verse in the JW Bible is with the word "obeisance." In the Greek, the word is
"proskuneo." It means, worship; fall down and worship, kneel, bow low, obeisance. Obeisance means,
"bending the head or body or knee as a sign of reverence or submission or shame." While it is certainly
true that people would bow down before Jesus in the Bible, the English word "obeisance" does not
carry sufficiently the act of worship due Him because of His divine nature. Since the Jehovah's
Witnesses deny that Jesus is God in flesh, they cannot have their Bible teach that Jesus was
worshipped. Therefore, every single time that the word "proskuneo" is used in reference to Christ, it is
translated as "obeisance" and never as worship. For proof of this, please see the article The New
World Translation and "Proskuneo" (worship) which has a chart of every occurrence of the Greek word
and how the New World Translation renders it.
Jesus said in Matthew 4:10 that you should worship (proskuneo) God only, yet He willing receives
worship in Matt. 2:2; 2:11; 14:33; 28:9; and John 9:35-38. Just check any Bible other than the New
World Translation and you can plainly see this.
In Acts 10:25-26 it says, "And when it came about that Peter entered, Cornelius met him, and fell
at his feet and worshiped him. But Peter raised him up, saying, 'Stand up; I too am just a man'"
(NASB). Here, Cornelius bows down before Peter. The Greek word here is "proskuneo." Peter knew
that this homage is to be given to God alone. Jesus, undoubtedly, knew the same thing. Yet, when
they bow down before Jesus, He does not rebuke them. Why? Because Jesus is worthy of worship. He
is God in flesh (John 1:1,14).
In Hebrews 1:6, God commands that all the angels worship Jesus, not just give Him obeisance.
The ASV, KJV, NASB, NIV, NKJV, and RSV all translate the verse to say "worship." Is the NWT correct
and all the others wrong? Not at all. The New World Translation is simply a slanted Bible that is used
to support Jehovah's Witness theology.

The Watchtower taught that Jesus was to be worshipped

Oddly enough, denying worship of Jesus is not what the Watchtower Organization has always
taught. On page 216 of the July 15, 1898 edition of the Watchtower magazine it says:

"Question: The fact that our Lord received worship is claimed by some to be an evidence that
while on earth he was God the Father disguised in a body of flesh and not really a man. Was he
really worshiped, or is the translation faulty?
Answer: Yes, we believe our Lord Jesus while on earth was really worshiped, and properly
so...It was proper for our Lord to receive worship in view of his having been the only begotten
of the Father, and his agent in the creation of all things, including man."

In the book, New Heavens and a New Earth, on pages 27-28, published in 1953, it says:

"For example, to which one of the angels did he ever say: 'You are my Son; today I have
become your Father'? And again: 'I shall be a Father to him, and he will be a Son to me'? But
when he again brings his Firstborn into the inhabited earth, he says: 'And let all God's angels
worship him.'"

Also, it is worth noting that the Watchtower organization changes the New World Translation as
time goes by. In the 1970 edition, Heb. 1:6 is translated as the word "worship." In the 1980 version
it is translated as 'obeisance.' Why the change? Simple, the Watchtower Organization is methodically
altering the Bible to make it agree with their theology.
The Jehovah's Witness will probably answer this issue the same way he would with the false
prophecies made by the Watchtower Magazine. He would say that "the light is getting brighter." This
means that the early Watchtowers did not have the same information and light that the present
Watchtower writers do. They made errors in the past, but now they know more and corrections need
to be made.
But if this were so, then how can the Jehovah's Witnesses trust what is being said now? If the
doctrines have changed before, what's to prevent them from changing again? How can a Jehovah's
Witness be sure that what he is believing now won't be changed later? He can't.
God doesn't change and neither does His word. But, if you are a man-made religion that is still
writing its doctrines, then you need a Bible that keeps up with those changes. The New World
Translation is made to order.
Heb. 1:8 and Psalm 45:6 and the Jehovah's Witnesses
"But with reference to the Son: 'God is your throne forever and ever, and [the] scepter of your
kingdom is the scepter of uprightness,'" (The New World Translation.)

In this particularly interesting verse, God is addressing the Son. The Greek construction of Hebrews
1:8 allows the text to be translated in two legitimate ways:

"God is your throne forever and ever....


and
"Thy Throne O God, is forever and ever..."

But because of the Watchtower presupposition that Jesus is not God, they choose the first version,
otherwise, the Father would be calling Jesus God and that goes against Jehovah's Witness theology.
Yet, most Bibles do not translate it the way the New World Translation does. They choose the other
way. Why? Two reasons.
First, Heb. 1:8 is a quote from Psalm 45:6, which says,

"Thy Throne, O God, is forever and ever; a scepter of uprightness is the scepter of Thy
Kingdom" (All Bible quotes are from the NASB).

In fact, the ASV, KJV, NIV, and NKJV all translated it as "Your throne, O God..." The RSV translates
it as "Your divine throne endures for ever and ever," "but this is a highly unlikely translation because it
requires understanding the Hebrew noun for "thrown" in construct state, something extremely unusual
whenn a noun has a pronomial suffix, as this one does...The KJV, NIV, and NASB all take the verse in
its plain, straightforward sense, as do the ancient translations..." 142
When we look at the Hebrew, we see that there is no grammatical requirement for this
translation, though it is considered to be the best translation by most translators. In and of itself, this
is not conclusive because the context of this verse in Psalm 45 is dealing with a king which would
make one wonder why he would be addressed as God. But, it is not uncommon for NT writers to take
a verse in the OT that seemingly deals with one subject and apply it to another. They knew something
we didn't. In fact, in Ezekiel 28:12-17 is a section that deals with the fall of the devil. Verse 13 says
describes how he was in the garden of Eden. Verse 14 says he was the anointed cherub, (v. 15), etc.
But the context of this section begins with an address to the king of Tyre (v. 12). Yet, right after
Ezekiel is told to write to the King of Tyre he then goes on to describe what the great majority of
theologians agree with is a description of the devil's fall. So, we need to look at the context that the
writer of Hebrews put Psalm 45:6 into. He addressed it to Jesus. Therefore, Psalm 45 is a Messianic
Psalm and must in interpreted in light of the NT, not the other way around.
Nevertheless, the context of this verse follows:

"For to which of the angels did He ever say, "Thou are My son, Today I have begotten Thee"?
And again, "I will be a Father to Him, and He shall be a Son to Me"? 6And when he again brings
the first-born into the world, He says, "And let all the angels of God worship Him." 7And of the
angels He says "Who makes His angels winds, and His ministers a flame of fire." 8But of the
Son He says, "Thy Throne, O God, is forever and ever, and the righteous scepter is the scepter
of His kingdom, 9Thou hast loved righteousness and hated lawlessness; therefore God, Thy
God, hath anointed Thee with the oil of gladness above Thy companions. 10And, "Thou, Lord, in
the beginning didst lay the foundation of the earth, And the heavens are the works of Thy
hands; 11They will perish, but though remainest...." (Heb. 1:5-11).

Grudem, Wayne, Systematic Theology, Intervarsity Press, Zondervan Publishing House, Grand Rapids,
142

Michigan, 1994, page 227.


To say "God is your throne" doesn't make sense. What does it mean to say, "But to which of the
angels did he say, God is your throne." What would that mean? Is God, Jesus' throne? God alone is
on His throne and He isn't a throne for anyone else.
Also worth noting here is verse 10: "Thou, Lord, in the beginning didst lay the foundation of the
earth, And the heavens are the works of Thy hands..." This is a quote from Psalm 102:24-25 which
says, "I say, 'O my God, do not take me away in the midst of my days, Thy years are throughout all
generations. 25Of old Thou didst found the earth; And the heavens are the work of Thy hands.'"
Clearly, God is the one being addressed in Psalm 102. It is God who laid the foundations of the earth.
Yet, in Heb. 1:10, Jesus is called 'Lord' and is said to be the one who laid the foundation of the earth.
This becomes even more interesting when we note that in Isaiah 44:24 it says, "Thus says the Lord,
your Redeemer, and the one who formed you from the womb, "I, the Lord, am the maker of all things,
Stretching out the heavens by Myself, And spreading out the earth all alone. " If God was laying the
foundations of the earth alone, that would mean that either Jesus has to be God, second person of the
trinity, who laid the foundation the same as YHWH did, or we have a contradiction in the Bible. Clearly
this section of Hebrews is proclaiming that Jesus is God. Therefore, contextually, it is best to
translate Heb. 1:8 as, "Thy Throne, O God. . ." and the Father call Jesus God.

The Watchtower organization denies that Jesus is God. Therefore, it cannot permit any verses in
the Bible to even hint that Jesus is God. That is why they choose a translation that does not best fit
the context or overall theology of the Bible.
Mormonism
Introduction

Those from the Church of Jesus Christ of Latter-day Saints say they are Christian yet they teach that
God used to be a man on another planet and he has a goddess wife.

They go door-to-door, look clean cut, and try and present a family oriented image. But, don’t let their
outward appearance fool you.

1. What are some of the stranger doctrines of Mormonism? pp. 98-99


2. Is Mormonism Christian? Why or why not? p. 102-103
3. What are some of the essential Mormon Doctrines not found in the Book of Mormon? p. 105
4. Does Mormonism Attack Other Religions? p. 114
5. Do you know the Mormon meanings to standard Christian words? pp. 116-118
6. What is wrong with praying about the Book of Mormon? p. 132
7. What are some of the major differences between Christianity and Mormonism? pp. 135-136
8. Does Mormonism teach a different Jesus than Christianity? p. 145
9. What are some particularly interesting quotes from Joseph Smith? p. 149
10. What are some particularly interesting quotes from Brigham Young? p. 150-151
11. What is the consensus of the witnesses concerning Joseph Smith's character? pp. 157-172
Are you studying with the Mormons or thinking of joining
the Mormon Church?

Are you interested in joining the Mormon Church? Are you curious about it? Maybe you are taking
or have taken the missionary lessons. Perhaps you even think that the Church of Jesus Christ of
Latter-day Saints is a good Christian church that is family oriented and filled with godly people. If so,
there are some things you need to know before you sign on the dotted line.
Mormonism is considered a non-Christian cult by all of Christianity. All you need to do is go to any
Christian bookstore and look in the cult section. You will see books on Mormonism there. Now, are
we "anti-Mormons" simply people who have a grudge against the LDS church? I can't speak for all
who oppose Mormonism, but I have no grudge against it at all. I was never a Mormon and no Mormon
injured me in the past. They are nice people. The problem isn't with them. It is with what they
teach. It simply isn't Christian.
Mormon missionaries will tell you that there was an apostasy and that Joseph Smith was the one
who restored the true gospel to this earth. This is a typical claim by every cult. Every one of them
says that the present Christian church is false and that their initial leader restored the "truth." They
have to invalidate present Christianity in order to get you to accept their brand of it.
Christianity teaches that there is only one God. Mormonism teaches that there are many gods.
Christianity teaches you cannot become a god. Mormonism says you can. In fact, Mormonism teaches
that God used to be a man on another planet who became a god and brought one of his wives with
him to this world. In case you doubt this, check out the documentation below which is taken from
Mormon writers.
Like all non-Christian cults, Mormonism uses the Bible to support its doctrines and does so
improperly. For example, when the Bible says that there are no other gods besides God (Isaiah
43:10; 44:6,8), Mormons quickly add "of this world." That means that there really are other gods out
there and that the Bible is talking only about this world and not about others. They often quote 1 Cor.
8:5-6 which mentions the existence of other gods. But when doing this, they fail to read exactly what
it says, that there are "so-called gods." In other words, they are called gods, but really are not
gods. Paul was speaking about false gods, not the true and living God. Mormons frequently
misrepresent scripture and take verses out of context to make it say what they want. Those who do
not know what the Bible really teaches will easily be fooled by this cult.

Why is this important?

This is important because eternal salvation is at stake. God warned us to not serve false gods (Exodus
20), which are really not gods by nature (Gal. 4:8). He warned us to believe in the true Christ, not the
false ones of the cults (Matt. 24:24). The god of Mormonism is false and cannot save you or anyone.
If you believe in a false god, you will be damned to eternal hell. Why, because ultimately, false
gods do not exist except in the mind of the believer. In spite of being good, in spite of attending the
Mormon church with its polished appearance, in spite of believing in Mormon doctrine about a man
from another planet, you will go to hell if you believe in Mormon doctrine. It cannot save you from the
righteous requirements that God requires. Only the true Jesus can save you from your sins. Not a
god from another planet. Only the true God who alone is God in all the universe can save you.

Do you want to trust Mormon doctrine?

Following is a list of a few doctrines of Mormonism. Do you want to put your eternal trust in a
church that teaches the following doctrines?

1. The true gospel was lost from the earth. Mormonism is its restoration, Mormon Doctrine, by
Bruce R. McConkie, p. 635. They teach there was an apostasy and the true church ceased to
exist on earth.
2. There are many gods, Mormon Doctrine, p. 163.
3. There is a mother god, Articles of Faith, by James Talmage, p. 443.
4. God used to be a man on another planet, Mormon Doctrine, p. 321. Joseph Smith, Times and
Seasons, Vol 5, pp. 613-614; Orson Pratt, Journal of Discourses, Vol 2, p. 345, Brigham Young,
Journal of Discourses, vol. 7, p. 333.
5. After you become a good Mormon, you have the potential of becoming a god, Teachings of the
Prophet Joseph Smith, pages 345-347, 354.

If you want further information, information that the Mormons won't tell you until you are very well
entrenched in their cult, then read more of this site. It is full of information about why Mormonism is
nothing more than a clever, false look-alike that cannot help you before God.
What does Mormonism teach?

The doctrines of The Church of Jesus Christ of Latter-day Saints (Mormons) are very interesting.
Most of the 'odd' ones are not initially taught to potential converts. But they should be. Instead, "they
are are revealed later as one matures and gains the ability to accept them." The LDS Church tries to
make its official doctrines appear Christian but what underlies those Christian sounding terms is far
from Christian in meaning.
Following are the teachings of its officials throughout the years. Please note that these teachings
are documented from Mormon writers, not anti-Mormon writers.
Finally, many Mormons respond that most of the the citations below are not from official Mormon
writings, as if that disproves the doctrines they teach. If they are not official, fine. But, if not, then
why have the Mormon apostles and high officials taught them, written them, and why are their books
sold in Mormon bookstores? The following is what Mormons are taught.

1. The true gospel was lost from the earth. Mormonism is its restoration, Mormon Doctrine, by
Bruce R. McConkie, p. 635. They teach there was an apostasy and the true church ceased to
exist on earth.
2. We need prophets today, the same as in the Old Testament, Mormon Doctrine, p. 606.
3. The book of Mormon is more correct than the Bible, History of the Church, 4:461.
4. If it had not been for Joseph Smith and the restoration, there would be no salvation. There
is no salvation [the context is the full gospel including exaltation to Godhood] outside the
church of Jesus Christ of Latter-day Saints, Mormon Doctrine, p. 670.
5. There are many gods, Mormon Doctrine, p. 163.
6. There is a mother god, Articles of Faith, by James Talmage, p. 443.
7. God used to be a man on another planet, Mormon Doctrine, p. 321. Joseph Smith, Times
and Seasons, Vol 5, pp. 613-614; Orson Pratt, Journal of Discourses, Vol 2, p. 345, Brigham
Young, Journal of Discourses, vol. 7, p. 333.
8. After you become a good Mormon, you have the potential of becoming a god, Teachings of
the Prophet Joseph Smith, pages 345-347, 354.
9. God the Father had a Father, Joseph Smith, History of the Church, vol. 6, p. 476; Heber C.
Kimball, Journal of Discourses, vol. 5, p. 19; Milton Hunter, First Council of the Seventy,
Gospel through the Ages, p. 104-105.
10. God resides near a star called Kolob, Pearl of Great Price, pages 34-35; Mormon Doctrine, p.
428.
11. God the Father has a body of flesh and bones, Doctrine and Covenants, 130:22.
12. God is in the form of a man, Joseph Smith, Journal of Discourses, Vol. 6, p. 3.
13. God is married to his goddess wife and has spirit children, Mormon Doctrine p. 516.
14. We were first begotten as spirit children in heaven and then born naturally on earth, Journal
of Discourse, Vol. 4, p. 218.
15. The first spirit to be born in heaven was Jesus, Mormon Doctrine, page 129.
16. The Devil was born as a spirit after Jesus "in the morning of pre-existence," Mormon
Doctrine, page 192.
17. Jesus and Satan are spirit brothers and we were all born as siblings in heaven to them both,
Mormon Doctrine, p. 163.
18. A plan of salvation was needed for the people of earth so Jesus offered a plan to the father
and Satan offered a plan to the father but Jesus' plan was accepted. In effect the Devil
wanted to be the Savior of all Mankind and to "deny men their agency and to dethrone god."
Mormon Doctrine, page 193; Journal of Discourses, vol. 6, page 8.
19. God had sexual relations with Mary to make the body of Jesus, Brigham Young, Journal of
Discourses, Vol. 4, p. 218, 1857; vol. 8, p. 115. - This one is disputed among many
Mormons and not always 'officially' taught and believed. Nevertheless, Young, the 2nd
prophet of the Mormon Church taught it.

20. Jesus' sacrifice was not able to cleanse us from all our sins, (murder and repeated adultery
are exceptions), Journal of Discourses, Vol. 3, p. 247, 1856.
21. Good works are necessary for salvation, Articles of Faith, p. 92.
22. There is no salvation without accepting Joseph Smith as a prophet of God, Doctrines of
Salvation, Vol. 1, p. 188.
23. Baptism for the dead, Doctrines of Salvation, Vol. II, p. 141. This is a practice of baptizing
each other in place of non-Mormons who are now dead. Their belief is that in the afterlife,
the "newly baptized" person will be able to enter into a higher level of Mormon heaven.
24. There are three levels of heaven: telestial, terrestrial, and celestial, Mormon Doctrine, p.
348.
25. The Holy Ghost is a male personage, A Marvelous Work and a Wonder, by Le Grand
Richards, Salt Lake City, 1956, page 118; Journal of Discources, Vol. 5, page 179

Some Mormons may disagree with a few of the points listed on this page, but all of what is stated
here is from Mormon authors in good standing of the Mormon church.
Mormonism in a Nutshell

Mormonism teaches that God used to be a man on another world and that he became a god by
following the laws and ordinances of his god on his home world. In his present god-state, he rules our
world. He has a body of flesh and bones and, according to Mormonism, he has a wife, a goddess wife.
Since they are both exalted persons, they each possess physical bodies. In their exalted states as
deities, they produce spirit children that grow and mature in the spiritual realm. The first spirit born
was Jesus. Afterwards were born the devil and all other spirit creatures. After the spirit children are
born to god and his goddess wife in heaven, they come down and entered into the bodies of human
babies that are being born on earth. During this ‘compression' into the infant state, the memories of
their pre-existence is 'veiled.' All people were, according to Mormonism, born in heaven first and then
on earth where they are to grow, learn, and return to god.
God the father, who is called Elohim, was concerned for the future salvation of the people on
earth. In the heavenly realm, the Father had a plan for the salvation of the world. Jesus endorsed the
Father's plan. Lucifer did not. Lucifer became jealous and rebelled. In his rebellion he convinced a
large portion of the spirits existing in heaven to side with him and oppose god. God being more
powerful then they, cursed these rebellious spirits to become demons.
The remaining spirits sided with God. Since they chose the better way, when it comes time for
them to live on earth, they have the privilege of being born in races and locations that are relative to
their condition and choice made in the spirit realm. 143
In the Mormon plan of salvation there needed to be a savior: Jesus. But Jesus was a spirit in
heaven. For him to be born on earth, Brigham Young the second prophet of the Mormon church said
that instead of letting any other man do it, God the Father did it with Mary. He said that the birth of
our savior was as natural as the birth of our parents. Essentially, what this means is that god the
father came down and had sexual relations with Mary, his spirit daughter, to produce the body of
Jesus. Jesus, then, was born, got married, and had children. 144 He died on the cross and paid for sins
not on the cross only, but in the garden of Gethsemane before he went to the cross.
Mormon men and women have the potential of becoming gods. President Lorenzo Snow said, "As
god once was, man is. As God is, man may become." In order to reach this exalted state a person
must first become a good Mormon, pay a full ten percent tithe to the Mormon church, follow various
laws and ordinances of the church, and be found worthy. Afterwards, he or she can enter a Mormon
temple and go through secret rituals: baptism for the dead, celestial marriage, and various oaths of
secrecy and commitment. Additionally, four secret handshakes are taught so the believing Mormon,
upon entering the third level of Mormon heaven, can shake hands with god in a certain pattern. This
celestial ritual is for the purpose of permitting entrance into that level of heaven. 145
For those who achieve this highest of heavens, exaltation to godhood awaits them. Then, he or
she, will be permitted to have his or her own planet and be the god of his own world and the Mormon
system will be expanded to other planets.

143
Page 616 of Mormon Doctrine by Bruce R. McConkie
144
Jedediah M. Grant, second Counselor to Brigham Young said so in Journal of Discourses, vol. 1, pp. 345-346.
145
What's Going on in There? An Exposing of the Secret Mormon Temple Rituals, by Bob Witte & Gordon H.
Fraser. Gordon Fraser, Publisher.
Mormonism's History

Mormonism began with Joseph Smith Jr. who was born on Dec. 23, 1805, in Vermont. He was the
fourth child of Lucy and Joseph Smith. Joseph senior was known as a money digger and sought after
buried treasure, particularly that of Captain Kidd. His mother was highly superstitious.
Joseph Smith Jr. stated that he was disturbed by all the different denominations of Christianity and
wondered which was true. In 1820, when he was 14, he went into the woods to pray concerning this
and allegedly God the Father and Jesus appeared to him and told him not to join any of the
denominational churches.
Three years later, on Sept. 21, 1823, when he was 17 years old, an angel called Moroni, who was
supposed to be the son of Mormon, the leader of the people called the Nephites who had lived in the
Americas, appeared to him and told him that he had been chosen to translate the book of Mormon
which was compiled by Moroni's father around the 4th century. The book was written on golden plates
hidden near where Joseph was then living in Palmyra, New York. Joseph Smith said that on Sept. 22,
1827 he received the plates and the angel Moroni instructed him to begin the translation process. The
translation was finally published in 1830 as the Book of Mormon. Joseph claimed that during this
translation process, John the Baptist appeared to him and ordained him to accomplish the divine work
of restoring the true church by preaching the true gospel which, allegedly, had been lost from the
earth.
The Book of Mormon is supposed to be the account of people who came from the Middle-East to
the Americas. It covers the period of about 600 B.C. to 400 A.D. It tells of the Jaredites, people from
the Tower of Babel who came to central America but perished because of their own immorality. It also
describes some Jews who fled persecution in Jerusalem and came to America led by a man called
Nephi. The Jews divided into two groups known as the Nephites and Lamanites who fought each other.
The Nephites were defeated in 428 A.D. The Lamanites continued and are known as the American
Indians. The Book of Mormon is the account of the Nephite leader, Mormon, concerning their culture,
civilization, and appearance of Jesus to the Americas.
After the publication of the Book of Mormon, Mormonism began to grow. Because their religion
was so deviant from Christianity, i.e., plurality of gods, polygamy (Joseph is said to have had 27
wives), etc., persecution soon forced them to move from New York to Ohio, then to Missouri, and
finally to Nauvoo, Illinois. After being accused of breaking some laws in Nauvoo (for destroying a
printing press that was publishing harmful information on Mormonism), Joseph and his brother Hyrum
ended up in jail. A mob later broke into the jail and killed Joseph and his brother.
After the shooting, the church divided into two groups: One led by his widow which went back to
Independence Missouri. They are known as the Reorganized Church of Jesus Christ of Latter Day
Saints. They claim to be the true Church and lay claim to the legal succession of the church presidency
which was bestowed upon Joseph's son by Joseph Smith himself. The other group was led by Brigham
Young and they went to Utah where, in 1847, they ended up in Salt Lake and founded Salt Lake City.
Brigham had 25 wives and accumulated much wealth
Is Mormonism Christian?
"Is Mormonism Christian?" is a very important question. The answer is equally important and
simple. No. Mormonism is not Christian.
If you are a Mormon, please realize that I am not trying to attack you, your character, or the
sincerity of your belief. If you are a non-Mormon looking into Mormonism or if you are a Christian who
is simply researching Mormonism, then this paper should be of help to you.
The reason Mormonism is not Christian is because it, like any other cult, denies one or more of the
essential doctrines of Christianity. Of the essential doctrines (Jesus is God in flesh, forgiveness of sins
is by grace alone, and Jesus rose from the dead physically), Mormonism distorts two of them: the
person of Jesus, and His work of salvation.
Mormonism teaches that God the Father has a body of flesh and bones (D. & C. 130:22) and that
Jesus is a creation. It teaches that he was begotten in heaven as one of God’s spirit children (See the
Book, Jesus the Christ, by James Talmage, p. 8). This is in strict contrast to the biblical teaching that
he is God in flesh (John 1:1, 14), eternal (John 1:1, 2, 15), uncreated, yet born on earth (Col. 1:15),
and the creator all (John 1:3; Col. 1;16-17). Jesus cannot be both created and not created at the
same time. Though Mormonism teaches that Jesus is god in flesh, it teaches that he is " a" god in flesh,
one of three gods that comprise the office of the Trinity (Articles of Faith, by Talmage, pp. 35-40).
These three gods are the Father, the Son, and the Holy Ghost. This is in direct contradiction of the
biblical doctrine that there is only one God (Isaiah 44:6,8; 45:5).
Because Mormonism errors in who Jesus is, salvation (the forgiveness of sins) does not occur and
the Mormon is still in his sins. Christians are saved from their sins and judgment by putting their trust
in Jesus for the forgiveness of their sins. But, faith is only as good as the object in which it is placed.
The Mormon Jesus is not the one of the Bible, even though they call him Jesus, say he died for sins,
and was born in Bethlehem. The Mormon Jesus does not exist. It is the nature of Jesus that is the
issue. Jesus must be God in flesh, (second person of the Trinity) not "a" god in flesh who is the brother
of the devil. He must be uncreated, not created. He must be the creator (Col. 1:16-17). This is who
the true Jesus really is: God, creator, uncreated, not the brother of the devil.
Mormon theology teaches that god used to be a man on another planet, that he became a god by
following the laws and ordinances of that god on that world, and that he brought one of his wives to
this world with whom he produces spirit children who then inhabit human bodies at birth. The first
spirit child to be born was Jesus. Second was Satan, and then we all followed. The Jesus of Mormonism
is definitely not the same Jesus of the Bible. Therefore, faith in the Mormon Jesus, is faith misplaced
because the Mormon Jesus doesn't exist.
Mormonism teaches that the sacrifice of Jesus on the cross itself (and receiving it by faith) is not
sufficient to bring forgiveness of sins. It teaches that the forgiveness of sins is obtained though a
cooperative effort with God; that is, we must be good and follow the laws and ordinances of the
Mormon Church in order to obtain forgiveness. Consider James Talmage, a very important Mormon
figure who said, "The sectarian dogma of justification by faith alone has exercised an influence for evil"
(Articles, p. 432), and "Hence the justice of the scriptural doctrine that salvation comes to the
individual only through obedience" (Articles, p. 81). This contradicts the biblical doctrine of the
forgiveness of sins by grace through faith (Rom. 5:1; 6:23; Eph. 2:8-9) and the doctrine that works
are not part of our salvation but a result of them (Rom. 4:5, James 2:14-18).
To further confuse the matter, Mormonism further states that salvation is twofold. It maintains
that salvation is both forgiveness of sins and universal resurrection. So when a Mormon speaks of
salvation by grace, he is usually referring to universal resurrection. But the Bible speaks of salvation as
the forgiveness of sins, not simple universal resurrection. Where Mormonism states that forgiveness of
sins is not by faith alone, the Bible does teach it. Which is correct? Obviously, it is the Bible.
Mormonism, to justify its aberrant theology, has undermined the authority and trustworthiness of
the Bible. The 8th article of faith from the Mormon Church states, "We believe the Bible to be the
word of God as far as it is translated correctly." The interesting thing is that Joseph Smith allegedly
corrected the Bible in what is called The Inspired Version, though it is not used by the LDS church.
Though they claim they trust the Bible, in reality they do not. They use Mormon presuppositions to
interpret it. For example, where the Bible says there are no other gods in the universe (Isaiah 43:10;
44:6,8), they interpret it to mean "no other gods of this world." They do not trust what it says and
they often state that the Bible is not translated correctly. This is what I have encountered numerous
times when speaking to Mormons.
Why is Mormonism a non Christian cult? Because it adds works to salvation. It denies that Jesus is
the uncreated creator. It alters the biblical teaching of the atonement. It contradicts the Christian
teaching of monotheism. It undermines the authority and reliability of the Bible.
I do not deny that Mormons are good people, that they worship "a" god, that they share common
words with Christians, that they help their people, and that they do many good things. However, Jesus
said in Matthew 7:21-23, " Not everyone who says to Me, 'Lord, Lord,' shall enter the kingdom of
heaven, but he who does the will of My Father in heaven. Many will say to Me in that day, 'Lord, Lord,
have we not prophesied in Your name, cast out demons in Your name, and done many wonders in
Your name? And then I will declare to them, 'I never knew you; depart from Me, you who practice
lawlessness!" (NKJV). Becoming a Christian does not mean belonging to a church, doing good things,
or simply believing in God. Being a Christian means that you have trusted in the true God for
salvation, in the True Jesus -- not the brother of the devil.
A Quick Look at the Book of Mormon

According to Joseph Smith, the Book of Mormon is more correct than the Bible (History of the
Church, Vol. 4, page 461) and contains the truths of Mormonism. However, if that is true, then why
does the Book of Mormon contradict Mormon theology?
This is because the theology of Joseph Smith didn't really start to go off the deep end until after
the Book of Mormon was printed. To harmonize their changing theology with their written scripture,
the Mormons gradually redefined common Christian words. That is why the definitions of Mormon
words are different than those of Christianity.

Praying About the Book of Mormon

One of the things Mormons ask potential converts to do is to read the Book of Mormon and then
pray to God and ask Him if the Book of Mormon is true. It is said that you will then receive a testimony
from the Holy Ghost that the Book of Mormon is true, that Joseph Smith is a prophet of God, and that
the Church of Jesus Christ of Latter-day Saints is God's restored church on earth today. It is difficult to
counter this testimony because it is an emotional and, I believe, spiritual phenomena. I say spiritual
but I am not saying it is of God; it is of the devil.
Mormons teach that if you are sincere and that if you ask God for wisdom as it says in James 1:5,
that God will answer you and lead you into the truth. What could be more common sense than that?
There are several reasons why this approach to determining truth is dangerous. The the most
important is as follows:
The Bible says that to determine truth you must examine the scriptures. 2 Timothy 3:16-17 says
that "All Scripture is God-breathed and is useful for teaching, rebuking, correcting and training in
righteousness, so that the man of God may be thoroughly equipped for every good work" (NIV). Acts
17:11 says, "Now the Bereans were of more noble character than the Thessalonians, for they received
the message with great eagerness and examined the Scriptures every day to see if what Paul said was
true" (NIV). If the Bereans compared what Paul the apostle said to scripture, should we do any less
with Mormonism? Of course not. You don't pray about truth you look into the Bible for it. To begin
with, the Mormons are going against scripture and trusting something unverifiable (except by their
own subjective feelings, of which the Bible says not to trust, Jer. 17:9).

The Book of Mormon Verses Mormon Doctrine

The Book of Mormon does not contain Mormonism. It is more Christian than it is Mormon. Mormon
theology is about many gods, god being a man, men and women potentially becoming gods, but the
Book of Mormon is basically Christian in its teachings.

The following information is in the Book of Mormon. Compare it to Mormon doctrine and see the
differences. It is obvious that Mormonism grew and was pasted together as it developed. It is not
internally consistent and it is self-contradictory.
The Book of Mormon Verses Mormon Doctrine

The Book of Mormon Mormon Doctrine


Mormonism teaches there are
There is only one God many gods.
Mosiah 15:1,5; Alma 11:28; 2 Nephi 31:21 Joseph Smith, Journal of Discourses,
Vol. 6, p. 5
The Trinity is three separate
The Trinity is one God gods.
Alma 11:44; Mosiah 15:5; 2 Nephi 31:21 James Talmage, Articles of Faith, p.
35. 1985.
God is unchanging God is increasing in knowledge.
Mormon 9:9,19; Moroni 8:18; Alma 41:8; 3 Joseph Smith, Journal of Discourses,
Nephi 24:6 Vol. 6, p. 120.
God has the form of a man.
God is spirit
Joseph Smith, Journal of Discourses,
Alma 18:24,28; 22:9,11
Vol. 6, p. 3.
Hell is not eternal.
Eternal hell
James Talmage, Articles of Faith, p.
Jacob 3:11; 6:10; 2 Nephi 19:16; 28:21-23.
55.
Polygamy was taught and
Polygamy condemned
practiced.
Jacob 1:15; 2:23,24,27,31;3:5; Mosiah
Brigham Young, Journal of
11:2,4; Ether 10:5,7
Discourses, Vol. 3, p. 266

12 Essential Mormon Doctrines not Found in the Book of Mormon

If the Book of Mormon is the "most correct book of any on earth" (History of the Church, vol.
4:461), then why does it not contain essential Mormon doctrines such as...

1. Church organization 7. Men may become Gods


2. Plurality of Gods 8. Three degrees of glory
3. Plurality of wives doctrine 9. Baptism for the dead
4. Word of Wisdom 10. Eternal progression
5. God is an exalted man 11. The Aaronic Priesthood
6. Celestial marriage
12. Melchizedek Priesthoo

The founder of Mormonism said the Book of Mormon was the most correct book of any book,
including the Bible (History of the Church, Vol. 4, page 461), and that a man could get closer to God
by following it than any other book. Yet, essential Mormon doctrines aren't even found in it.
This is because the Book of Mormon is nothing more than a fictional account made up by Joseph
Smith. It wasn't until after the book had been printed that the additional heretical doctrines of
Mormonism started to develop. That is why the Book of Mormon sounds so Christian -- at first.
Some of the Many Changes in the Book of Mormon
Joseph Smith said "that the Book of Mormon was the most correct of any book on earth, and the
keystone of our religion, and a man would get nearer to God by abiding by its precepts, than by any
other book" (History of the Church, Vol. 4, page 461). Allegedly it was translated by the power of God.
Nevertheless, it has some 4,000 changes in it. Some are mere spelling corrections, but others are
significant changes. Why is this so if the Book of Mormon was translated accurately by the hand of
God? Why would the Mormon Church continue to change the work even after Joseph Smith's death?

Following is a very small sample of the changes in the Book of Mormon.


Check them out for yourself.

1830 Edition of the Book of Mormon 1981 Edition of the Book of Mormon

"And he said unto me, Behold, the virgin


"And he said unto me: Behold, the virgin whom
1 Nephi which thou seest, is the mother of
thou seest is the mother of the Son of God,
11:18 [. . . . ] God, after the manner of the
after the manner of the flesh."
flesh

"And the angel said unto me, behold the "And the angel said unto me: Behold the Lamb
1 Nephi
Lamb of God, yea, even the [. . . . ] of God, yea, even the Son of the Eternal
11:21
Eternal Father!..." Father!..."

"...And I looked and beheld the Lamb of "...And I looked and beheld the Lamb of God,
1 Nephi God, that he was taken by the people; that he was taken by the people; yea, the Son
11:32 yea, [. . . . ] the Everlasting God, was of the everlasting God was judged of the
judged of the world..." world..."

"...and shall make known to all kindreds, "...and shall make known to all kindreds,
1 Nephi tongues, and people, that the Lamb of tongues, and people that the Lamb of God is
13:40 God is [. . . . ] the Eternal Father and the the Son of the Eternal Father, and the Savior
Savior of the world..." of the World..."

"...for had not the Lord been merciful, to "...for had not the Lord been merciful, to show
1 Nephi shew unto me concerning them, even as unto me concerning them, even as he had
19:20 he had prophets of old; [. . . . ] for he prophets of old, I should have perished
surely..." also."

1 Nephi "Hearken and hear this, O house of "Hearken and hear this, O house of Jacob, who
20:1 Jacob, which are called by the name of are called by the name of Israel, and are come
changed in Israel, and are come forth out of the forth out of the waters of Judah, or out of the
1964 ed. waters of Judah,[. . . . ] which swear..." waters of baptism, who swear..."

Mosiah
"...king Benjamin had a gift from God,
21:28 "...king Mosiah had a gift from God, whereby
whereby he could interpret such
changed in he could interpret such engravings;..."
engravings;..."
1964 ed.
"...yea, I know that he allotteth unto men, yea,
Alma "...yea, I know that he allotteth unto men
decreeth unto them decrees which are
29:4 [ . . . .] according to their wills..."
unalterable, according to their wills..."

"...because of the excellent qualities of the "...because of the excellent qualities of the
many plants and roots which God had many plants and roots which God had
Alma
prepared, to remove the cause of diseases prepared to removed the cause of diseases,
46:40
which was subsequent to man by the to which men were subject by the nature
nature of the climate." of the climate."

"And the land which was appointed was the


3 "And the land which was appointed was the
land of Zarahemla, and the land which was
Nephi land of Zarahemla [ . . . .] and the land
between the land of Zarahemla and the
3:23 Bountiful..."
land Bountiful."

"O ye people of these great cities which have


"O ye people of these great cities which have
3 fallen which are a descendant of Jacob; yea
fallen, who are descendants of Jacob, yea,
Nephi which are of the house of Israel; O ye people
who are of the house of Israel, [. . . . ] how
10:4 of the house of Israel, how oft have I
oft have I gathered you..."
gathered you..."

"and thus commandeth the Father that I should


"And thus commandeth the Father that I
3 say unto you at that day, When the Gentiles
should say unto you: At that day when the
Nephi shall sin against my Gospel, and shall subject
Gentiles shall sin against my gospel,[. . . . ]
16:10 the fulness of my Gospel, and shall be lifted
and shall be lifted up..."
up..."

"...for thou shalt forget the shame of thy


3 "...for thou shalt forget the shame of thy youth, youth, and shalt not remember the
Nephi [. . . . ] and shalt not remember the reproach reproach of thy youth, and shalt not
22:4 of thy widowhood any more." remember the reproach of thy widowhood
any more."

"...nevertheless, the Lord was merciful unto "Nevertheless, the Lord was merciful unto
Ether Omer, and also to his sons and to his Omer, and also to his sons, and to his
9:2 daughters, which were not, or which did not daughters [. . . . ] who did not seek his
seek his destruction." destruction."

As you can see, the Book of Mormon is a changing document. Where will the Mormon Church
change it next?

The primary research for this information was taken from the book 3913 Changes in the Book of
Mormon by Jerald and Sandra Tanner. To obtain copies of their work please write to Utah Lighthouse
Ministry P.O. Box 1884 Salt Lake City, Utah 84110.
The Book of Abraham Papyri and Joseph Smith
There are many proofs that Joseph Smith was a false prophet, but Mormons typically, will not
accept them. From the biblical evidence that contradicts Mormon theology, to the contradictions within
its own history and doctrine, proofs abound. But Mormons, completely dedicated to their religion and
their testimony, cannot and will not see the evidence. They rely not on biblical evidence, not on
historical evidence, but rather trust a 'testimony' that Mormonism is the restored church and Joseph
Smith its true prophet.
One of the tests of whether or not a belief is grounded in reality is whether or not it can be proven
to be true or false. If someone says, "I don't care what evidence you show me, I will always believe,"
then that person's faith is not rooted in reality. And since Christianity is a religion of history,
crucifixion, resurrection, an empty tomb, etc., it is a religion rooted in reality. If it could be proven
beyond doubt that Jesus did not rise from the dead, then Christianity is a false religion. Likewise, if it
could be proven that Joseph Smith was a false prophet, then Mormonism is a false religion. It just so
happens that there is such a proof.

The Book of Abraham

Joseph Smith claimed that an angel appeared to him and revealed the location of some golden
plates on which was written the account of the ancient people of the Americas. Joseph Smith later
translated those plates into what is now known as the Book of Mormon. This translation was done by
the power of God through special means. Joseph Smith, being the Lord's chosen instrument, became
the prophet of the Mormon Church, held the office of Seer. A Seer, according to the Book of Mormon in
Mosiah 8:13, can translate records that are untranslatable. Hence, Joseph Smith was able to translate
the golden plates into the Book of Mormon. But his Seer abilities did not stop there.
In July of 1835, an Irishman named Michael Chandler brought an exhibit of four Egyptian
mummies and papyri to Kirtland Ohio, then the home of the Mormons. The papyri contained Egyptian
hieroglyphics. In 1835 hieroglyphics were unreadable.
As Prophet and Seer of the Church, Joseph Smith was given permission to look at the papyri
scrolls in the exhibit and to everyone's shock, revealed that "one of the rolls contained the writings of
Abraham, another the writings of Joseph of Egypt" (History of the Church, Vol. 2: 236. July 1835). The
Church bought the exhibit for $2400. Joseph finished the translation of the Book of Abraham some
time later, but the book of Joseph was never translated. The papyri were lost soon afterwards and
thought to have been destroyed in a fire in Chicago in 1871. There was, therefore, no way to validate
Joseph's translation. If the papyri were re-discovered and translated it would either prove or disprove
the abilities of Joseph as a prophet of God. After all, he was supposed to be a prophet and have the
abilities of a Seer as the Book of Mormon and the Book of Abraham supposedly proved.
In October of 1880 The Pearl of Great Price, a collection of writings, which contained the book of
Abraham, was recognized as scripture by the Mormon Church.

The Papyri are found

To every one’s surprise, in 1966 the papyri were rediscovered in one of the vault rooms of the
New York’s metropolitan Museum of Art. The Deseret News of Salt Lake City on Nov. 27, 1967
acknowledged the rediscovery of the papyri. On the back of the papyri were "drawings of a temple and
maps of the Kirtland, Ohio area." 146 There could be no doubt that this was the original document from
which Joseph Smith translated the book of Abraham.
With the papyri rediscovered and Egyptian hieroglyphics decipherable since the late 1800's, it
would then be an easy task of translating the papyri and proving once and for all that Joseph Smith
was a prophet with the gift of "Seer" as he and the Mormon church have claimed. This would then
prove the truth of the Book of Mormon and the Book of Abraham and would vindicate Joseph Smith as
a true prophet of God.

Improvement Era, January 1968, p. 25; as cited in "..by His Own Hand Upon Papyrus" by Charles M.
146

Larson, Institute for Religious Research, Grand Rapids, MI 49505-4604, 1992


What do the Experts say?

Joseph Smith copied three drawings from the Egyptian scrolls, labeled them Facsimile No. 1, No.
2, and No. 3, and incorporated them into the Book of Abraham with explanations of what they were.
Egyptologists have viewed the drawings and found Joseph Smith's interpretation of them to be wrong.
But, the Mormons, in defense of the sacred book, maintained that the Facsimiles alone were not
sufficient to prove that Joseph Smith was erring in his translating abilities. With the rediscovery of the
papyri, not only were there the same drawings in the scrolls, but so was the text from which Joseph
Smith made his translation. It was now possible to absolutely determine the accuracy of Smith's
translating abilities.

Facsimile No. 1

Joseph Smith said that Facsimile No. 1 was of a bird as the "Angel of
the Lord" with "Abraham fastened upon an altar," "being offered up as a
sacrifice by a false priest. The pots under the altar were various gods
"Elkenah, Libnah, Mahmackrah, Korash, Pharaoh," etc.
In reality, this is "an embalming scene showing the deceased lying on a
lion-couch."147
In the original papyri, this drawing is attached to hieroglyphics (See
figure A) from which Joseph derived the beginning of the book of Abraham
which begins with the words, "In the Land of the Chaldeans, at the
residence of my father, I, Abraham, saw that it was needful for me to obtain
another place of residence"(1:1). In reality, the hieroglyphics translate as,
"Osiris shall be conveyed into the Great Pool of Khons -- and likewise Osiris
Hor, justified, born to Tikhebyt, justified -- after his arms have been placed on his heart and the
Breathing permit (which [Isis] made and has writing on its inside and outside) has been wrapped in
royal linen and placed under his left arm near his heart; the rest of the mummy-bandages should be
wrapped over it. The man for whom this book was copied will breath forever and ever as the bas of the
gods do."148
"It is the opening portion of an Egyptian Shait en Sensen, or Book of Breathings . . . a late
funerary text that grew out of the earlier and more complex Book of the Dead." "This particular scroll
was prepared (as determined by handwriting, spelling, content, etc.) sometime during the late
Ptolemaic or early Roman period (circa 50 B.C. to A.D. 50)." 149

147
Joseph Smith Among the Egyptians, by Wesley P. Walters. 1973, Reprinted by Utah Lighthouse Ministry, Box
1884, Salt Lake City, Utah 84110.

148
Dr. Klaus Baer, The Breathing Permit of Hor. A Translation of the Apparent Source of the Book of Abraham, p.
119-120 as cited in Joseph Smith Among the Egyptians, by Wesley Walters.

149
Larson, Charles M., by his own hand upon papyrus, Institute for Religious Research, Grand Rapids, Mich. 1992,
p. 62.
Figure A

Figure A is a professional reconstruction of the


original (Figure B). Note the hieroglyphics on the right
side from which Joseph Smith began his translation of
the Book of Abraham.
In actuality, it "depicts the mythical embalming and
resurrection of Osiris, Egyptian god of the underworld.
Osiris was slain by his jealous brother Set, who cut up
his body into 16 pieces and scattered them....The
jackal-headed god Anubis is shown embalming the body
of Osiris on the traditional lion-headed couch so that he
might come back to life..." 150

Figure B

Figure B (to the right) shows a reprint of the actual


papyrus used by Joseph Smith
Note the areas where the Papyrus has been lost. It is in
these that Joseph Smith "finished" the drawing resulting in
Facsimile No. 1. His restoration, according to
Egyptologists, reveals a complete lack of understanding of
Egyptian practice and theology.

Facsimile No. 2

As is explained by Joseph Smith and included in the Pearl of Great


Price, the second drawing contains different scenes which Joseph Smith
interpreted. They vary: "Kolob, signifying the first creation, nearest to
the celestial, or the residence of God." "Stands next to Kolob, called by
the Egyptians Oliblish, which is the next grand governing creation near
to the celestial or the place where God resides." "God, sitting upon his
throne, clothed with power and authority." "...this is one of the
governing planets also, and is said by the Egyptians to be the Sun, and
to borrow its light from Kolob through the medium of Kae-e-vanrash,
which is the grand Key..."
But again scholarship disagrees with Joseph’s rendition. "It is
actually a rather common funerary amulet termed a hypocephalus, so-
called because it was placed under (hypo) a mummy’s head (cephalus). Its purpose was to magically
keep the deceased warm and to protect the body from desecration by grave robbers." 151

According to Smith, this drawing shows "Abraham sitting upon Pharaoh’s throne, by the politeness
of the king, with a crown upon his head, representing the Priesthood...King Pharaoh, whose name is
given in the characters above his head...Signifies Abraham in Egypt...Olimlah, a slave belonging to the

150
Larson, p. 102.

151
Larson, p. 104
prince..."
But this is not what the Egyptologists say is the meaning of the Facsimile No. 3 is. Instead, it
shows, "the deceased being led before Osiris, god of the dead, and behind the enthroned Osiris stands
his wife Isis."152

Conclusion Facsimile No. 3

It should be quite obvious that present


scholarship has revealed that Joseph Smith did not
translate the Book of Abraham by the power of God as
he had claimed. It follows that if he did not
translate the Book of Abraham by the power of
God, then it would be very easy to conclude that
he did not translate the Book of Mormon by the
power of God either.
When Joseph first gave his translation,
hieroglyphics were undecipherable. Today they are.
He was safe in saying anything he wanted to and
there would be no way of proving him wrong. But
with the resurfacing of the same papyri he used to do his Book of Abraham translation, and the fact
that he did not in any way do it correctly, should be proof enough that Joseph Smith lied about his
abilities from God. He has been shown to be a false prophet.

_________

For some very good and extensive information on the Book of Abraham, please go to

http://www.irr.org/mit/boapage.html

152
Walters, p. 29.
7
The Mormon Church Statistics

As of 1997, the Church of Jesus Christ of Latter-day Saints, more commonly known as the
Mormons, claims membership of a little more than 10 million worldwide.

• United States . . . . . . . . . . .5 Million


• Latin America . . . . . . . . . . .3 Million
• Asian . . . . . . . . . . . . . . . . .2 Million
• South Pacific & Africa . . . . 1/2 Million

• The Mormon Church was founded on April 6, 1830.


• It was originally called the Church of Jesus Christ.
• The official name of the Church of Jesus Christ of Latter-day Saints was adopted in 1837.

• Temples are sacred structures where special Mormon rites are practiced.
• The church built its first temple in 1836 in Kirtland, Ohio.
• There are more than 50 temples world wide with another 50 either in construction or in the
early planning stages.

• The church distributes over 5 million copies of the Book of Mormon per year.
• There are approximately 800 converts to the Mormon church per day, 300,000 per year.

• There are approximately 60,000 full time missionaries world wide. Most of them are young
males in their late teens to early twenties and spend 2 years in the field.
• There are 16 missionary training centers world-wide.
• There are thousands of Mormons who are in field service in different countries doing many
charitable works ranging from doctors, to mechanics, to teachers, and more.

• Financial income is in excess of 2 million dollars per day.


• Its net worth holdings are in the billions of dollars.
Mormon Church Structure
Does Mormonism Attack Other Religions?
Mormons do not like it when their Church is labeled a cult by Christians. This bothers them and
they want desperately to be accepted as Christian by the Christian community. The Mormon church
spends a great deal of time and money on public relations with the aim of portraying a loving, family
oriented, non-condemning Christian denomination. But Christians react to this and cite the great
differences in doctrine between Mormons and Christians and continue to pronounce the Church of
Jesus Christ of Latter-day Saints as a non-Christian cult.
The battle continues and Mormons try to claim that they do not go around condemning other
religions like "anti-Mormons" do. They say they are forgiving, tolerant, good Christian people who
don't have anything against anyone. They claim they are being more Christ-like.
Their desire for a good image is understandable. But the question remains. Does the Mormon
church condemn other religious systems? The answer is definitely, "Yes." Let's look at Mormon writers
and see what they have said.

Joseph Smith said . . .

(Regarding Joseph Smith's alleged first vision where celestial personages appeared to
him.) . . .) "My object in going to inquire of the Lord was to know which of all the sects was
right, that I might know which to join. No sooner, therefore, did I get possession of myself, so
as to be able to speak, than I asked the personages who stood above me in the light, which of
all the sects was right — and which I should join. I was answered that I must join none of
them, for they were all wrong, and the personage who addressed me said that all their creeds
were an abomination in His sight: that those professors were all corrupt . . ." (Joseph Smith,
"History of the Church, Vol. 1, page 5-6.)

"What is it that inspires professors of Christianity generally with a hope of salvation? It is that
smooth, sophisticated influence of the devil, by which he deceives the whole world."
("Teachings of the Prophet Joseph Smith," Compiled by Joseph Fielding Smith, page 270.)

(In questions directed to Joseph Smith, the founder of Mormonism. . .)


First -- "Do you believe the Bible?"
If we do, we are the only people under heaven that does, for there are none of the religious
sects of the day that do."
Third — "Will everybody be damned, but Mormons?"
Yes, and a great portion of them, unless they repent, and work righteousness." (Teachings,
page 119.)

Brigham Young said. . .

"But He did send His angel to this same obscure person, Joseph Smith jun., who afterwards
became a Prophet, Seer, and Revelator, and informed him that he should not join any of the
religious sects of the day, for they were all wrong." (Brigham Young, "Journal of Discourses,"
Vol. 2, page 171. - 1855)

John Taylor said . . .

"We talk about Christianity, but it is a perfect pack of nonsense....Myself and hundreds of the
Elders around me have seen its pomp, parade, and glory; and what is it? It is a sounding brass
and a tinkling symbol; it is as corrupt as hell; and the Devil could not invent a better engine to
spread his work than the Christianity of the nineteenth century."( Journal of Discourses, Vol. 6,
page 167 - 1858)

"Where shall we look for the true order or authority of God? It cannot be found in any nation of
Christendom." (J.D.", Vol. 10, page 127. - 1863)
James Talmage said . . .

"A self-suggesting interpretation of history indicates that there has been a great departure from
the way of salvation as laid down by the Savior, a universal apostasy from the Church of
Christ". ("The Articles of Faith," Deseret Book Company, Salt Lake City, Utah. P. 182.)

Bruce McConkie said . . .

"With the loss of the gospel, the nations of the earth went into a moral eclipse called the Dark
Ages." ("Mormon Doctrine," Bookcraft, Salt Lake City, Utah, page. 44.)

Joseph Fielding Smith said . . .

"Again, following the death of his apostles, apostasy once more set in, and again the saving
principles and ordinances of the gospel were changed to suit the conveniences and notions of
the people. Doctrines were corrupted, authority lost, and a false order of religion took the place
of the gospel of Jesus Christ, just as it had been the case in former dispensations, and the
people were left in spiritual darkness." ("Doctrines of Salvation," page 266.)

The Book of Mormon says. . .

"And he said unto me: Behold there are save two churches only; the one is the church of the
Lamb of God, and the other is the church of the devil; wherefore, whoso belongeth not to the
church of the Lamb of God belongeth to that great church which is the mother of abominations;
and she is the whore of all the earth" (1 Nephi 14:10).

"And when the day cometh that the wrath of God is poured out upon the mother of harlots,
which is the great and abominable church of all the earth, whose foundation is the devil, then,
at that day, the work of the Father shall commence. . ." (1 Nephi. 14:17).

The Doctrine and Covenants says . . .

"Verily, verily, I say unto you, darkness covereth the earth, and gross darkness the minds of
the people, and all flesh has become corrupt before my face" (Doctrine and Covenants,
112:23).

When the Mormon missionaries come to the door and do their "gospel" presentation, they mention
an apostasy and the need for a prophet, their prophet, to restore the true Teachings of Jesus. Of
course, these ‘restored' teachings are completely false.
Nevertheless, the Mormon church clearly condemns other religious systems. Those Mormons who
complain about poor treatment should familiarize themselves with their teachers' words.
Mormon words don't mean the same thing

Anyone who tries to witness to a Mormon will soon find that the words they use do not always
mean the same thing to Christians. Below is a list of terms that are important for Christians to know
when discussing the truth with Mormons. It is important that you know what the Mormons mean by
the same words use by Christians.

ADAM LDS - Father of physical mankind. Adam is Bible - the first created man by whom all
also known as Michael the archangel, the of humanity descends. He was not
ancient of days, (D&C 116). Michael the archangel.
ATONEMENT LDS - The sacrifice of Christ that made Bible - The substitutionary sacrifice of
resurrection possible along with the Jesus on our behalf. He died for our sins
possibility of our earning forgiveness of (1 Peter 2:24; 1 John 2:2).
sins.
AARONIC LDS - A lesser priesthood in the LDS Bible - A priesthood that is no longer
PRIESTHOOD church. It is still used in LDS church necessary now that we have the full
practices and is held by the very young, revelation of Christ.
(D&C 107:1, 6, 10).
BAPTISM LDS - A necessary ordinance for salvation Bible - An ordinance of the Christian
in the Mormon church. By it sins are church that is not necessary for salvation
washed away. (Rom. 5:1).
BIBLE LDS - The Bible is correct only as far as it Bible - the Bible is the inspired inerrant
is correctly translated. It is basically word of God (2 Tim. 3:16).
trustworthy. It is the only one of the four
standard works (Bible, Book of Mormon,
Doctrine and Covenants, and Pearl of Great
Price) that is not considered infallible. The
KJV is the official Bible of the LDS church.
BISHOP LDS - an office in the Aaronic Priesthood of Bible - An office held by a male member
the LDS church. D&C 20:67), of the Church.
CELESTIAL LDS - The highest of the three levels of Bible - There is no such thing as a
HEAVEN heaven where faithful Mormons are exalted celestial heaven.
to Godhood.
CHURCH LDS - The LDS church with its Bible - The body of believers in the true
organizational structure, laws, and proper and living God through Jesus. It is
name. comprised of those who are redeemed
and is not limited to an earthly structure.
DAMNATION LDS - Basically, anything lesser than Bible - The state condemnation, judged
exaltation which is becoming a God. by God in eternal hell (Matt. 25:46).
DEVIL LDS - See Satan. Bible - See Satan.
ELOHIM LDS - The name of God the Father. Bible - The Hebrew word for "God." The
name of God is "YHWH," which means "I
AM," (Exodus 3:14).
ETERNAL LDS - Exaltation (exaltation to a Mormon Bible - Forgiveness of sins and life eternal
LIFE means obtaining Godhood) in the Celestial with God (John 17:3; Rom. 6:23).
Kingdom.
EXALTATION LDS - The state of becoming a god in the Bible - No such thing as becoming a God
celestial heaven. in the Bible.
FALL OF LDS - A blessing (Mosiah 3: 11-16). A Bible - The open rebellion of Adam and
MANKIND necessary step in the progression of Eve against God resulting in their
humanity to the level of Godhood. condemnation and the fall of mankind.
GOD LDS - One of countless gods in existence. Bible - The one and only God in all the
An exalted man from another world who universe, (Isaiah 44:6,8).
created the earth who's name is "Elohim."
He became a god by following the laws and
ordinances of his god on the other world.
He has a body of flesh and bones. D&C
130: 22-23.
GODHEAD LDS - An office held by three separate Bible - God Himself, not an office. Three
Gods: the Father who is a god; Jesus who persons in one God. A Trinity: The Father;
is a god; and the Holy Ghost who is a god. the Son; and the Holy Spirit.
GOSPEL LDS - The laws and ordinances of the Bible - The death, burial, and
Mormon church. resurrection of Jesus for the forgiveness
of the sins of all who would trust in Him
(1 Cor. 15:1-4).
HEAVEN LDS - Divided into three Kingdoms: Bible - The dwelling place of God (1 Kings
Celestial, Terrestrial, and Telestial. The 8:30). Christians go to heaven.
Celestial is for perfect Mormons, the
Terrestrial is for moral people and
lukewarm LDS, and the Telestial Kingdom
is for everyone else.
HELL LDS - The temporary abode in the spirit Bible - the eternal dwelling place of those
world between death and resurrection for who rejected the atoning work of Christ.
those awaiting telestial glory, (D&C 76: 84-
85, 106). Hell will come to an end.
HOLY LDS - "A spirit man. He can only be at one Bible - Third person of the Trinity. Same
GHOST place at one time... " (Mormon Doctrine by as Holy Spirit (Acts 5:3-4).
Bruce McConkie, p. 359.) The Holy Ghost is
contrasted with the Spirit of God which is
the influence of the Godhead that fills the
immensity of space which enables God to
know what is going on. It is likened to
electricity." D&C 130: 22-23.
HOLY LDS - The presence of God as distinguished Bible - An equivalent term to Holy Ghost,
SPIRIT from the Holy Ghost who is a god in the third person in the Trinity.
mormon trinity.
JEHOVAH LDS - The name of Jesus in the Old Bible - The name of God is "YHWH,"
Testament. which means "I AM," (Exodus 3:14).
JESUS LDS - Literal offspring of God the father. Bible - Jesus is God, second person of the
Spirit brother of Satan. A god in the Trinity (John 1:1,14; Col. 2:9).
Godhead. He is Jehovah of the O.T.
compared to Elohim being the Father. He
was the first spirit child to be born to the
Father and Mother gods. Ordained as the
Christ in the pre-existent Grand Council
before coming to earth.
KINGDOM LDS - Celestial heaven. The kingdom of Bible - All the believers of Christ (Matt.
OF GOD God on earth is the LDS church. 13:41-43).
MARRIAGE LDS - An eternal bonding of husband and Bible - the holy covenant between a man
wife that continues into the afterlife. These and a woman that is broken at death.
couples will continue to have children.
(D&C 132:15-20).
MELCHIZEDEK LDS - A greater priesthood in the LDS Bible - A priesthood held by Jesus alone.
PRIESTHOOD church held by elders, (D&C 107),
PRE- LDS - We existed in heaven with God our Bible - We did not exist before we came
EXISTENCE (literal) Father and mother before we to earth (1 Cor. 15:46).
became human.
SALVATION LDS - Two fold meaning: Simple bodily Bible - Forgiveness of sins with the result
resurrection of all people. Also, forgiveness of a present new life and in the future
of sins. eternal life with God (1 Cor. 15:1-4; Rom.
6:23; 10:9-10).
SATAN LDS - The opposer of God, literal son of Bible - A fallen angel who rebelled
God, brother of Jesus and all people against God.
begotten in the pre-existent spirit world.
SCRIPTURE LDS - Bible, Book of Mormon, Doctrine and Bible only
Covenants, Pearl of Great Price.
TEMPLE LDS - A present day temple used to Bible - The Old Testament building where
practice the ordinances and ceremonies of God dwelt, sacrifices were offered, and
the gospel of the LDS church on behalf of holy priestly rites were administered.
the living as well as the dead. There is no longer a need for temples.
TRINITY LDS- Three gods: a god called the Father; Bible - The one and only God in all
a god called the son; a god called the Holy existence who is comprised of three
Ghost. persons: Father, Son, and Holy Spirit.
See Trinity.

Go to http://scriptures.lds.org/bd/contents for a list of Mormon words and definitions produced by


the LDS church.
A Response From (and to) S.H.I.E.L.D.S.
Following is a copy of the SHIELDS ARTICLE responding to my posting of Difficult Questions for
Mormons To Answers, by Ira T. Ransom. Their original was located at http://www.shields-
research.org but has since been moved or removed. I have reproduced it here.
The original web page information is in signified as ORIGINAL. Their responses preceded by
SHIELDS (later it will be FARMS), then my responses to their responses are with MY RESPONSE
preceding it.

First of all, I apologize to Mr. Barker for taking far longer than necessary to answer his rebuttal.
With my busy schedule and a series of personal responsibilities that arose, I put it aside and, in time,
forgot about it. Nevertheless, here it is.
Also, I have changed some of the information on my website as a result of Mr. Barker's efforts.
For clarification, Mr. Barker’s web site is quoted here in its entirety. My comments are inserted in
blue.

SHIELDS
Introduction by Stanley D. Barker

Matthew J. Slick has created a web site to promote himself, having been rejected for ordination to the
Presbyterian ministry because of his ideas. He says:

ORIGINAL
I am Reformed in theology and believe in the continuation of the spiritual gifts which is why my
denomination at the time, the Presbyterian Church in America, has refused my ordination; they are
cessationist.

MY RESPONSE
Notice that the first thing mentioned on their site is an attack on my character. I have stated on
my web site that the Presbyterian Church in America does not believe in the continuation of the
spiritual gifts. I do. Therefore, they would not ordain me to the ministry. They did recommend that I
continue ministry and find a denomination that believed more along those lines. I still enjoy fellowship
with them, my Christian brothers. This is nothing unusual within Christianity. But, Mr. Barker, in an
attempt to discredit me and, therefore, my position against them, has begun with an attack on my
character by trying to incite within the reader a suspicion concerning my trustworthiness and Christian
character. Instead of beginning with the issues, he began with my person.

SHIELDS
Mr. Slick has placed his views of Mormonism on the Internet. He has challenged:

ORIGINAL
Now, before you go slamming me with some irate e-mail telling me I don’t know what I am talking
about, first read my material on my site, and if I am wrong, correct me by showing precisely where I
am wrong. Document the sources you want to quote to prove me wrong. If you do, I’ll change my
page.

SHIELDS
He also tells us "The web site is very well documented...." Yet all we find there are rehashes of old
anti-Mormon material. Mr. Slick has fallen into the same unethical sloppiness that can be found
amongst most critics of the LDS Church, i.e., he quotes word for word from some other anti-Mormon,
but he fails to give credit to the original source.
MY RESPONSE
My site is well documented. So what does a "rehash of old anti-Mormon material" have to do with it
being well documented? Whether or not my material is a rehash (and what is wrong with that
considering Mormon doctrine is still wrong?), its truth, or lack thereof, should be the issue, not
whether or not it is a ‘rehash.’
Mr. Barker has now accused me of plagiarism and "unethical sloppiness." Both these accusations
fall into the "attack-his-character" category. It is a shame to see that this is the method Mr. Barker
chooses to begin his apologetic.
I learn from many sources. I cannot remember the origin of everything I learn (as, I am sure, is
the case with Mr. Barker) in refuting Mormonism, nor should I be expected to. What becomes part of
my apologetic approach via learning from others does not mean that I plagiarized. Some of what I say
is original to myself. Other parts have been gained from other Christians far more knowledgeable than
myself.

SHIELDS:
(We'll give him the possibility that he simply doesn't know the origin of some of these statements and
merely chooses to repeat them.) An excellent example of this can be found in the last question
responded to by John A. Tvedtnes. This question came, word for word, from Bob Witte and can be
found on our 42 Questions section, Question 36.

MY RESPONSE
On my website (http://www.carm.org/lds/diff_questions.htm), I clearly state that the questions at
issue here are the product of another author, Ira T. Ransom in the booklet "Ask Your Bishop." I gave
full credit.

SHIELDS
Whether or not Mr. Slick will "change [his] page" as he claims, remains to be seen. We suspect that,
as with most critics who have often made this claim, nothing will happen. We hope we are proven
wrong. It would be a nice change.

MY RESPONSE
Perhaps Mr. Barker has failed to consider that the condition for changing my page is not that it be
answered, but that I am proven wrong. Note again, an issue of my character is again raised with the
hint that my integrity and honesty are at stake.
As with many cults, emotional manipulation of the reader/hearer is often used in an attempt to
influence the potential convert to adopt the new position. It seems that Mr. Barker is attempting to
recruit an emotional response from the reader of his material; namely, that if I were honest (a man of
integrity), then I will change my position once I read his material.

SHIELDS
We have not provided a link to Mr. Slick's web site since we don't want to promote sites of our critics.
Rather, we think the focus should be on the responses themselves. Some of the issues raised by Mr.
Slick will be addressed further at a later date.

In contrast, I am not afraid of Mormonism nor its attempts at defending itself. At the beginning of
this page I have provided a link to their site. I do so again here: S.H.I.E.L.D.S (On CARM, this link is
active).

Mr. Barker has copied the information from a Mormon apologetics site called FARMS. The following
information is from them. Therefore, I will now change the notation from SHIELDS to FARMS.
Response by John A. Tvetnes (FARMS)

FARMS
The Christian Apologetics and Research Ministry (C.A.R.M.), located on the Internet, has posted a list
of "Difficult Questions for Mormons to Answer," taken from a booklet entitled, Ask Your Bishop by Ira
T. Ransom. Here are the questions and some brief responses. (Actually, none of these are "difficult
questions.")

ORIGINAL
If the Book of Mormon is true, why do Indians fail to turn white when they become Mormons? (2 Nephi
30:6, prior to the 1981 revision).

FARMS
"White" need not refer to skin color, as is clear from the following passages from the biblical book of
Daniel: "And some of them of understanding shall fall, to try them, and to purge, and to make them
white, even to the time of the end: because it is yet for a time appointed (Daniel 11:35). "Many shall
be purified, and made white, and tried; but the wicked shall do wickedly: and none of the wicked shall
understand; but the wise shall understand (Daniel 12:10). In both of these passages, the meaning of
the word "white" is most obviously pure; to "make white" is to purify. When Joseph Smith first
translated the Book of Mormon, he gave the literal rendering of "white" for the passage in 2 Nephi
30:6. For the 1840 edition, it was changed to "pure," which better reflected the meaning of the word
used by Nephi. Subsequent editions, however, relied on the 1837 Book of Mormon, which still read
"white." This oversight was not rectified until the 1981 edition.

MY RESPONSE
I think you have completely missed this one, Mr. Tvedtnes. Let's look at your own Mormon
writers:

The book of Mormon says in 2 Nephi 5:21, "And he had caused the cursing to come upon them, yea,
even a sore cursing, because of their iniquity. For behold, they had hardened their hearts against him,
and they had become like unto a flint; wherefore, as they were white, and exceedingly fair and
delightsome, that they might not be enticing unto my people the Lord God did cause a skin of
blackness to come upon them."

3 Nephi 2:15 says, "And their curse was taken from them, and their skin became white like unto the
Nephites." This is obviously a reference to skin color.

Brigham Young, the second prophet of the Mormon church said, in 1859, "You may inquire of the
intelligent of the world whether they can tell why the aborigines of this country are dark, loathsome,
ignorant, and sunken into the depths of degradation ...When the Lord has a people, he makes
covenants with them and gives unto them promises: then, if they transgress his law, change his
ordinances, and break his covenants he has made with them, he will put a mark upon them, as in the
case of the Lamanites and other portions of the house of Israel; but by-and-by they will become a
white and delightsome people" (Journal of Discourses 7:336).

Brigham Young also said that those who fall away from Mormonism would, "become gray-haired,
wrinkled, and black, just like the Devil" (Journal of Discourse 5:332).
I do not know if Mr. Tvedtnes is aware of the references in the Book of Mormon and perhaps he is
also unaware of the quotes from Brigham Young, the second Prophet of his church. I do not mean this
in a derogatory manner, but if he is not knowledgeable about this basic Mormon position, how can I
trust him on other equally serious issues?

ORIGINAL
If the Book of Mormon is true, then why has the Mormon church changed it? Examples are: 1 Nephi
11:21; 19:20; 20:1 and Alma 29:4. Compare these with the original Book of Mormon. (Gerald [sic]
and Sandra Tanner have counted 3913 changes in the book of Mormon, excluding punctuation
changes.)

FARMS
The reasons for changes in the Book of Mormon are similar to the reasons why the English Bible has
experienced changes over time. Changes can be classified as (1) changes in punctuation, which was
added by the typesetter, not Joseph Smith or his scribe, (2) correction of typesetting errors, (3)
spelling errors made by either the scribe (Oliver Cowdery) or the typesetter, (4) changes to upgrade
the language to make it sound more English than Hebrew, (5) restoration of phrases or sentences left
out by the typesetter but later discovered to be in the manuscript. The addition to 1 Nephi 20:1 is an
exegetical explanation and should have been placed enclosed by parentheses. All the changes listed in
the question were made by Joseph Smith and not by "the Mormon church." As the translator of the
record, who would have been better qualified to determine how the Lord intended those passages to
be read?

MY RESPONSE
I will grant that minor variants could have crept into the B.O.M. text through typesetting errors and
that stylistic upgrades are occasionally necessary. However, the kinds of errors I am talking about are
the ones where the entire meaning of the text has been changed. I have difficulty simply accepting the
statement that they were left out of the original and latter corrected? If so, by whom and on what
basis?
Wasn't the book of Mormon translated by the gift and power of God? Didn't Smith okay the final
product at its printing? Why, then, did not God tell him there were problems?
Following is just a few of the differences that are noteworthy and don't fall under Mr. Tvetdnes
attempt to answer.

1830 Edition of the Book of Mormon 1981 Edition of the Book of Mormon
"And he said unto me, Behold, the virgin "And he said unto me: Behold, the virgin
1 Nephi
which thou seest, is the mother of [. . . . ] whom thou seest is the mother of the Son
11:18
God, after the manner of the flesh of God, after the manner of the flesh."
"...for had not the Lord been merciful, to "..."for had not the Lord been merciful, to
1 Nephi shew unto me concerning them, even as he show unto me concerning them, even as he
19:20 had prophets of old; [. . . . ] for he surely..." had prophets of old, I should have
perished also. And he surely did..."
1 Nephi "Hearken and hear this, O house of Jacob, "Hearken and hear this, O house of Jacob,
20:1 which are called by the name of Israel, and who are called by the name of Israel, and
changed are come forth out of the waters of Judah, are come forth out of the waters of Judah,
in 1964 [. . . . ] which swear..." or out of the waters of baptism, who
ed. swear..."
Mosiah "...king Benjamin had a gift from God, "...king Mosiah had a gift from God,
21:28 whereby he could interpret such whereby he could interpret such
changed engravings;..." engravings;..."
in 1964
ed.
"...yea, I know that he allotteth unto men, "...yea, I know that he allotteth unto men
Alma 29:4 yea, decreeth unto them decrees which [ . . . .] according to their wills..."
are unalterale, according to their wills..."
"And the land which was appointed was the "And the land which was appointed was the
land of Zarahemla, and the land which was land of Zarahemla [ . . . .] and the land
between the land of Zarahemla and the Bountiful..."
3 Nephi
land Bountiful."
3:23
"O ye people of these great cities which have "O ye people of these great cities which
fallen which are a descendant of Jacob; yea have fallen, who are descendants of Jacob,
3 Nephi
which are of the house of Israel; O ye yea, who are of the house of Israel, [. . . . ]
10:4
people of the house of Israel, how oft how oft have I gathered you..."
have I gathered you..."
"and thus commandeth the Father that I "And thus commandeth the Father that I
should say unto you at that day, When the should say unto you: At that day when the
3 Nephi
Gentiles shall sin against my Gospel, and Gentiles shall sin against my gospel,[. . . . ]
16:10
shall subject the fulness of my Gospel, and shall be lifted up..."
and shall be lifted up..."
"...for thou shalt forget the shame of thy "...for thou shalt forget the shame of thy
youth, [. . . . ] and shalt not remember the youth, and shalt not remember the
3 Nephi
reproach of thy widowhood any more." reproach of thy youth, and shalt not
22:4
remember the reproach of thy widowhood
any more."
"...nevertheless, the Lord was merciful unto "Nevertheless, the Lord was merciful unto
Omer, and also to his sons and to his Omer, and also to his sons, and to his
Ether 9:2
daughters, which were not, or which did daughters [. . . . ] who did not seek his
not seek his destruction." destruction."

The changes are rather blatant and have even changed the meaning of the text. The problem is
definitely still there.

ORIGINAL
How did Joseph Smith carry home the golden plates of the Book of Mormon, and how did the witnesses
lift them so easily? (They weighed about 230 lbs. Gold, with a density of 19.3 weighs 1204.7 lbs. per
cubic foot. The plates were 7" x 8" by about 6". See Articles of Faith, by Talmage, page 262, 34th ed.)

FARMS
The record was not solid gold bullion, but a set of thin metallic sheets held together by metallic rings.
So we're not dealing with a 7x8x6-inch block of gold. Besides, there's no evidence that the plates were
really made of gold. The Testimony of the Eight Witnesses declares that they had "the appearance of
gold." Joseph Smith spoke of them as "gold plates" (Joseph Smith History 1:34), but this need not
mean that they were pure gold; they may have been a gold alloy. Indeed, the only plates said to have
been "of pure gold" were the 24 plates of Ether's record, which were not part of the collection Joseph
received (Mosiah 8:9; cf. Mosiah 28:11).

MY RESPONSE
Smith said the plates were gold. Now, it is possible that they were an alloy. But how much? 51%
gold? If they were to say, 30% gold and 70% copper, would it be correct to say that the plates were
made of gold when 70% was copper? I think it is fair to say that they were at least 51% gold.
Nevertheless, they would have a substantial weight. F.A.R.M.S. has stated that the plates probably
weighed about 50 pounds.
The account of Smith running with the plates includes him being attacked and knocked down by a
robber, who hit Smith over the head with a gun, a "heavy blow". Smith then knocked him down and
ran at the top of his speed for half a mile, was attacked and knocked down again, but managed to
fight off that assailant. He then ran again and was attacked for a third time in the same manner. He
did all this carrying at the very least 50 pounds of metal under his arm traveling through a wooded
area.
Sorry, but even given the minimal weight of 50 pounds, the account is far fetched.
ORIGINAL
If Moroni devoutly practiced the Mormon Gospel, why is he an angel now rather than a God? (Doc. &
Cov. 132:17,37)

FARMS
Joseph Smith only once called Moroni an "angel," in the true sense of that word, i.e., a messenger.
And this declaration is not found in a revelation, but in a letter Joseph wrote, which means that it need
not reflect information he got from the Lord (D&C 128:20). It does not preclude Moroni from
exaltation. Moreover, we really do not know whether Moroni appeared to Joseph as a resurrected being
or a translated being, though most assume it was the former. If he has not yet been resurrected (or
changed), then he will not yet have entered into his final estate.

MY RESPONSE
I know of no Mormon who does NOT believe Moroni was an angel in the true sense. If he was not,
why don’t Mormons call him the "Messenger Moroni" and clear up the confusion?

ORIGINAL
Why do Mormons emphasize part of the Word of Wisdom and ignore the part forbidding the eating of
meat except in winter, cold or famine? (Doc. & Cov. 89:12,13).

FARMS
This is like asking why some Mormons smoke. As imperfect human beings, none of us does everything
precisely the way the Lord asks, though we should be striving to do so. There are, in fact, Latter-day
Saints who do observe the injunction about meat. We could turn this question around and ask why so
many Christians emphasize part of the plan of redemption (grace) and ignore other parts (keeping
God's commandments).

MY RESPONSE
Sorry, but Christian don’t ignore the importance of keeping God’s commandments. It is just that
perfect obedience is not a prerequisite for salvation. If smoking is against the Word of Wisdom and is
therefore sinful, why not the others? It is the Mormons who have made a big issue out of this, not us.

ORIGINAL
When Christ died, did darkness cover the land for three days or for three hours? (Luke 23:44 and 3
Nephi 8:19, 23).

FARMS
This is like asking why the snowstorm on Christmas day lasted two hours in Boston and two days in
Denver. We are, after all, dealing with different geographical regions here. The answer is that the
darkness lasted three hours in the Holy Land but three days in the lands inhabited by the Nephites,
where there appears to have been a major volcanic explosion (see John A. Tvedtnes, "Historical
Parallels to the Destruction at the Time of the Crucifixion," Journal of Book of Mormon Studies 3/1,
Spring 1994).

MY RESPONSE
I'll give this one to you, though I think Smith simply made a mistake when he was copying from
the Bible. It is not, technically, a contradiction to state the time of darkness difference is due to
geographical location.

ORIGINAL
Joseph Smith said that there are men living on the moon who dress like Quakers and live to be nearly
1000 years old. Since he was wrong about the moon, is it safe to trust him regarding the way to
Heaven? (The Young Woman's Journal, Vol. 3, pages 263-264. See reprint in Mormonism -- Shadow or
Reality? by Jerald and Sandra Tanner, page 4.)

FARMS
Just once it would be nice to see this statement in a document contemporary with Joseph Smith (who
died in 1844), rather than in something written in a journal in 1881 and published in 1892, which is
the source the Tanners cite. But even if Joseph Smith did believe this (which cannot be demonstrated),
could one blame him? After all, the press in his day had reported that British Astronomer Royal Sir
John Herschel had discovered people living on the moon. It was a newspaper hoax that was widely
believed in the 1830s. Joseph Smith could believe such a thing and still be a prophet, for prophets,
too, have a right to opinions. It was Joseph who declared "I . . . visited with a brother and sister from
Michigan, who thought that 'a prophet is always a prophet;' but I told them that a prophet was a
prophet only when he was acting as such" (History of the Church 5:265).

MY RESPONSE
I'll give you this one, too. However, Brigham Young taught there was life on the Moon as well and
also on the sun (JOD 13:271). Are you saying we can’t believe anything Smith said which was reported
after he died. Also, you quoted the History of the Church to support your position. I guess that means
I can do the same thing. Perhaps the Journal of Discourses are also fair to quote. One more thing,
how do we know that Smith's quote in History of the Church 5:265 isn't simply his opinion, just like
the men in the moon part? Who decides? Or, is it only an opinion when it is shown to be false?

ORIGINAL
Joseph Smith prepared fourteen Articles of Faith. Why has the original No. 11 been omitted?

FARMS
(Joseph Smith Begins His Work, Vol. 2, three pages after page 160, among the photos.) The Articles
of Faith were not received by revelation, but were merely a summation of the beliefs of the Church.
There are only 13 of them in the letter Joseph Smith wrote to John Wentworth in 1842. The fourteen
published by Wood derive from a later source, a pamphlet published in England in April, 1849, by
James H. Flanigan. It is therefore incorrect to associate Joseph Smith's name with that list.

MY RESPONSE
Okay, I'll give you that one, too.

ORIGINAL
Why did the Nauvoo House not stand forever and ever? (Doc. & Cov. 124:56-60).

FARMS
The original Nauvoo House is still standing and can be seen by visitors to that city. It is the Mansion
House, Joseph Smith's residence, that had to be reconstructed.

MY RESPONSE
The problem is it was prophesied to be "for the boarding of strangers" "from generation to
generation" not to be gawked at by tourists. The fact is Joseph Fielding Smith admitted this building
was never finished. See Doctrines of Salvation 3:218.

ORIGINAL
How can a man who is not a descendant of Aaron hold the Aaronic Priesthood? (Numbers 16:40; Heb.
7:13,14).
FARMS
Through the prophet Isaiah, the Lord promised that, in the last days, he would gather Israel and would
"take of them for priests and for Levites" (Isaiah 66:21). If he intended to authorize only descendants
of Aaron to hold that priesthood, why would he have to designate priests and Levites?

MY RESPONSE
Sorry, I am not sure I understand your point. But I'll take a shot at it.
The Isaiah citation is eschatological; that is, it is dealing with the future of Jerusalem, probably in
reference to the millennial reign. We could argue a long time on typologies and meanings and it would
accomplish little. But, for the sake of keeping it simple, I've removed the original question from the
list.

ORIGINAL
Since Mormonism teaches that only God the Father had a physical body at the time Adam was created,
why did God say, "Let us make man in OUR image"? Why didn't He say, "Let us make man in MY
image?" (Gen. 1:26).

FARMS
According to Ether 3:15-16, Christ, as a spirit, appeared as he would in mortality, and it was after his
spirit that man was patterned (see also Mosiah 7:27). Consequently, the shape of our bodies is the
same as both the Father as a physical being and the Son while yet in his spiritual state. So there is no
contradiction here.

MY RESPONSE
Instead of arguing technicalities, I'll give you this one, too. However, I had always thought that the
Mormon's position was that it was the flesh and bones that made us in the image of God.

ORIGINAL
If Jesus was conceived as a result of a physical union between God and Mary, how was Jesus born of a
virgin? (Journal of Discourses, Vol. 1, page 50).

FARMS
Mary was a virgin because she had known no man (Luke 1:34), not because she bore the Son of God.

MY RESPONSE
But that isn't answering the question. You simply say she was a virgin. Brigham Young said they
had relations, that God didn't let any other man 'do it' with Mary, that Jesus' birth, etc, was the result
of natural action…. Now, since the god of Mormonism is an immortal MAN, and, I assume, with
genitalia, and Young said it was a natural conception, then how did she remain a virgin? THAT is the
question.

ORIGINAL
How did Nephi with a few men on a new continent build a temple like Solomon's while Solomon needed
163,300 workmen and seven years to build his temple? (1 Kings 5:13-18 and 2 Nephi 5:15-17).
FARMS
Nephi probably did it the same way the small Israelite garrison at Arad constructed a temple patterned
after Solomon's in the ninth century B.C. Like the Arad temple, Nephi's structure could have been
rather small. Half a dozen people could have completed the Arad temple working part-time for less
than a year. [See also 42 Questions, Question # 2]

MY RESPONSE
Then I guess it wasn't like Solomon's temple, was it?

ORIGINAL
Why was Joseph Smith still preaching against polygamy in October 1843 after he got his revelation in
July 1843 commanding the practice of polygamy? (Doc. & Cov. 132; and History of the Church Vol. 6,
page 46, or Teachings of the Prophet, page 324).

FARMS
Actually, Joseph Smith had received the revelation more than a decade before it was written in 1843.
He always maintained that, unless commanded to do so by the Lord, a man should have only one wife.
Similarly, the Lord commanded Lehi's family to have but one wife, but reserved the right to command
otherwise should he wish to do so (Jacob 2:24-30). Joseph Smith's declarations against plural
marriage were aimed at those who claimed to have the authority to perform them within the Church.
He recorded, "Gave instructions to try those persons who were preaching, teaching, or practicing the
doctrine of plurality of wives: for, according to the law, I hold the keys of this power in the last days;
for there is never but one on earth at a time on whom the power and its keys are conferred; and I
have constantly said no man shall have but one wife at a time, unless the Lord directs otherwise"
(History of the Church 6:46).

MY RESPONSE
But I am still confused. If Smith received the revelation, then why did he go against it? I am
sorry, but your explanation seems to reveal that a double standard is okay.

ORIGINAL
God rejected the fig leaf aprons which Adam and Eve made (Gen. 3:21). Why do Mormons
memorialize the fall by using fig leaf aprons in the secret temple ceremonies?

FARMS
The aprons are mentioned only in Genesis 3:7. Nowhere does the text tell us that God "rejected"
them, only that, in place of the temporary fig leaf aprons (fig leaves dry up and blow away), God
provided more permanent skin clothing (Genesis 3:21). The Latter-day Saints recognize the symbolic
nature of the fall, represented by the fig-leaf apron and of God giving mankind a probationary time in
which to repent, as represented by the "coats of skins." [For additional discussion see the response to
Question 36 (42 Questions).]

MY RESPONSE
I am sorry, but you fail to understand the significance of the biblical account. Adam and Eve
covered themselves with their own works and God rejected it by replacing their works with His: animal
skins. This is significant because it involved the shedding of blood (in order to get the skins, typifying
the atoning work of Christ.). Theologically, the fig leaf aprons symbolize their own efforts to be
covered before God. This is not sufficient and is rejected by God. Additionally, from what I understand
of the temple ceremony, the aprons are the same as the one worn by Lucifer. After Lucifer is asked by
Adam, "What is that apron you are wearing?" Lucifer replies that it is the symbol of his power and
priesthoods. Immediately afterwards, the people going through the temple ceremony are asked to put
their aprons on.
Please let me know if I am incorrect about this.
Jehovah is Elohim
In Mormon theology God is not a Trinity but a triad. The doctrine of Trinity is the teaching that
there is a single God who exists in three persons: Father, son, and Holy Spirit. There are not three
gods, only one. The Trinity doctrine states there is only one God in all the universe, in all dimensions,
in all time, and without end. God is eternal and there was never a time when he was not God. He has
always been completely and fully God.
A Triad is the teaching that there are three separate gods in one "office" or "position." Mormons
often refer to God this way. When they say God, it can mean the Father, whom they call Heavenly
Father, or the Godhead rich, to them, is three separate gods in one office. In other words, the Mormon
defines the Trinity as a god called the Father (also known as Elohim and Heavenly Father), along with
the Son (Jesus), and the Holy Ghost. Mormonism teaches a Triad and not a Trinity.
The following quote substantiates the Mormon position:

"This reference, then, to the three as one God, must be interpreted to mean that they
constitute one Godhead or Supreme Council, composed of three separate personages, the
Father, the Son, and the Holy Ghost... it is very strange that Christian people can be confused
and believe that the Father, Son and Holy Ghost are one substance or entity." 153

In addition to teaching that God, the Godhead, is really three personages or gods, Mormonism also
teaches that there are many many gods in existence.
Mormon theology has erringly named God the Father as "Elohim."

" . . . God the Eternal Father, whom we designate the by the exalted name-title "Elohim," is the
literal Parent of our Lord and savior Jesus Christ, and of the spirits of the human race.” 154

Mormon theology also declares that in the old Testament, Jesus is known as "Jehovah."

"With this meaning, as the context shows in every case, Jehovah who is Jesus Christ the son of
Elohim, is called "the Father," and even "the very Eternal Father of heaven and earth..." 155

The belief that God the Father is called Elohim and Jesus is called Jehovah does not agree with
what the Bible says. In actuality, in Hebrew the word for "God" is the word "elohim." Likewise, the
word for the name of God (elohim) is "Jehovah." In the Bible, when the word "Jehovah" appears in the
Hebrew text, it is rendered as LORD (all caps) in the English text. Also, the Hebrew word "elohim" is
translated as "God."
Please consider the following verses:

"Ye are my witnesses, saith the LORD, and my servant whom I have chosen: that ye may know
and believe me, and understand that I am he: before me there was no God formed, neither
shall there be after me" (Isaiah 43:10-11).

This verse is important because if you read what it is saying, it states that Jehovah (LORD also
known to the Mormons as Jesus), is stating that there will be no God (elohim) formed after him. But
this is a problem for the Mormons since it could not be saying that Jehovah is the only elohim. In other
words, this verse is stating that the LORD (Jehovah), is elohim.
Let's look at two more verses.
153
Selections from Answer to Gospel Questions, published by the First Presidency of the Church of Jesus Christ of
Latter-day Saints, 1972, p. 10.

154
Articles of Faith, by James Talmage, Desert Book Company, Salt Lake City, Utah. 1985, p. 421.
155
Ibid. p. 421
"Thus saith the LORD the King of Israel, and his redeemer the LORD of hosts; I am the first,
and I am the last; and beside me there is no God," (Isaiah 44:6).

In this verse, LORD is Jehovah in the Hebrew. Jehovah is saying there is no God (elohim) besides
him.

"Fear ye not, neither be afraid: have not I told thee from that time, and have declared it? ye
are even my witnesses. Is there a God beside me? yea, there is no God; I know not any,"
(Isaiah 44:8).

The context of this verse is that Jehovah (LORD) is a speaking. He states here that there is no God
(elohim) besides him. He is stating that he does not even know of any other elohim (God) besides
himself.
My point is that the name of God (elohim) is Jehovah (LORD) and that the LORD is stating that he
alone is God. In other words, Jehovah is stating that he alone is elohim. Therefore, the Mormon idea
that God the Father is called "elohim" and that the son is called "Jehovah" is erroneous.
In actuality, the name of God is Jehovah and the Mormons are incorrect.
Remember, in Hebrew text LORD equals Jehovah. God equals elohim.

• "Unto thee it was shewed, that thou mightest know that the LORD he is God; there is none else
beside him," (Deut. 4:35).
• "That all the people of the earth may know that the LORD is God, and that there is none else,"
(1 Kings 8:60).
• "Know ye that the LORD he is God: it is he that hath made us, and not we ourselves;" (Psalm
100:2).
• "And I will bring the third part through the fire, and will refine them as silver is refined, and will
try them as gold is tried: they shall call on my name, and I will hear them: I will say, It is my
people: and they shall say, The LORD is my God," (Zech. 13:9).
A Biblical Response to Mormons
Witnessing to a Mormon is like trying to climb Jell-O: it's hard to get a foothold. But, if you know
what Mormonism teaches then you are already well on your way. Following are basic approaches that
should aid you in witnessing to a Mormon. Though none of these approaches are fool proof they will
provide you with the basic framework you need to be able to witness to a Mormon. It will be up to you
to use what you have learned, develop more skill in witnessing, and perfect your method as you go.
Remember, the best way to learn to witness is to witness.
There are two important things to know before you begin evangelizing Mormons. First, you need to
understand their definitions to the same biblical words that you use: Trinity, Jesus, Salvation, Heaven,
etc. Second, you must be able to show them that they believe in a wrong Jesus. This is important
because only the true Jesus gives eternal life (John 10:28), reveals the Father (Matt. 11:27), and
sends the Holy Spirit (John 15:26).
To witness means you must teach. To teach means you must understand. To understand means
you must know not only what you believe, but also what they believe.

1. Terminology
A. When a Mormon says he believes in the Trinity he does not mean the historical orthodox
Trinity of one God who exists in three persons. To a Mormon, the Trinity is an office held by
three separate gods: the Father, the Son, and the Holy Ghost.
i. Remember, the correct doctrine of the Trinity is that there is only one God who has
existed for eternity. This one God exists in three persons: the Father, the Son, and the
Holy Spirit. They are not three separate gods, but only one God.
B. When a Mormon says he believes in God he does not mean in the one true God, the creator
of all things, the One who has always existed from all time. He means he believes in a god
who used to be a man on another planet, who followed the laws and ordinances of that god
on that planet and became exalted to godhood. And, to top it all off, he has a wife who is a
goddess.
C. If you are in a witnessing situation with a Mormon you might be using the same words, but
you won't be speaking the same language.
2. They Have a Testimony
A. Mormons will bear their testimony to you and tell you that they know the Mormon church is
true and that Joseph Smith was a true prophet of God.
B. There are two basic approaches you can take.
i. Ask them where their testimony is.
a. "In my heart." They'll say.
b. You say, "Did you know that the Bible says not to trust your heart because it is
deceitful?"
c. "The heart is deceitful above all things and beyond cure. Who can understand it?"
(Jer. 17:9).
ii. Second, you can ask them how they get their testimony. They will say by the Holy Spirit.
a. Ask, "Who bears witness of the truth?"
b. They will say that the Holy Spirit does.
i. Correct him gently by showing him that the Holy Spirit Bears witness of Jesus
(John 15:26) and that Jesus sends the Holy Spirit (John 15:26).
ii. Once you've shown them that the Holy Spirit is sent from Jesus ask them if a
false Jesus will send the true Spirit of God. The answer, of course, is no.
iii. The point is that only the Jesus of the Bible will send the Holy Spirit. If they don't have
the right Jesus they can't have the true Holy Spirit, and their testimony is invalid.
3. Praying about the Book of Mormon
A. Mormons believe that if you read the Book of Mormon and then pray and ask God whether or
not it is true, you will receive a testimony from the Holy Spirit verifying its truth. If it is true,
then Joseph Smith is true and so is Mormonism. Many Mormons claim to have this
testimony.
i. First of all, God never says to pray about truth. He says to search the Scriptures to find
truth (Acts 17:11; 2 Tim. 3:16).
a. So, what the Mormon is doing is unbiblical.
ii. Second, it doesn't matter what you feel. If what you feel contradicts the Bible, then what
you feel is wrong.
iii. Third, ask them if they ever had to pray about the Bible to see if it is true. Of course
they haven't. So why are they supposed to pray about the B.O M.?
a. Their answer will be that the B.O.M. says to pray about it.
b. Still, the Bible says to study God's word for truth, not pray about it.
B. A common verse that Mormons use to support their belief that you can pray about the B. O.
M. is found in James 1:5: "If any of you lacks wisdom, he should ask God, who gives
generously to all without finding fault, and it will be given to him." They say that because
since they believe they're sincere, God will answer them.
i. First of all, the problem with sincerity is that it becomes works righteousness because
the person is saying "Because of my sincerity, God will listen to me." In other words,
because of what's in the person God will look favorably upon him. God does not look into
a person and find something good because there is no good in anyone (Rom. 3:10-12;
Eph. 2:3).
ii. Second, this verse is about wisdom, not about praying to see if the Book of Mormon is
true.
iii. In James 1:1 it says, "James, a servant of God and of the Lord Jesus Christ, To the
twelve tribes scattered among the nations: Greetings." So, the book of James was
written to those who were believers and already had the truth. That is why James calls
them "brothers" in verse 2.
C. Third, wisdom is the proper use of knowledge, not the acquisition of knowledge. You acquire
true spiritual knowledge from the Bible, not your heart. You don't pray about the B.O.M., you
pray about the truth you've learned from the Bible and ask God to teach you more, and how
to apply properly what He's already shown you.
4. What is the Gospel?
A. The following approach is direct and hard hitting. Sometimes it is necessary to be blunt in
order to get their attention. Ask a Mormon "What is the gospel?".
i. He will say something like, "The gospel is the laws and the ordinances of the Mormon
church."
ii. Ask again what it is and listen closely for any hint of the free forgiveness of sins through
the sacrifice of Jesus on the cross. You usually hear an answer dealing with works,
obedience, doing something, etc.
iii. After the person has answered, explain that according to the Bible, the gospel is what
saves us, what cleanses us of our sins, and enables us to stand in the presence of God
the Father. Explain that Bible specifically defines the gospel and that the gospel is what
makes you a Christian and then ask again, "What is the gospel?"
iv. After you've heard a works-righteousness-type answer, turn in your Bible to 1 Cor 15:1-
4 and read: "Now, brothers, I want to remind you of the gospel I preached to you,
which you received and on which you have taken your stand. By this gospel you are
saved, if you hold firmly to the word I preached to you. Otherwise, you have believed in
vain. For what I received I passed on to you as of first importance: that Christ died for
our sins according to the Scriptures, that he was buried, that he was raised on the third
day according to the Scriptures."
a. Explain that the gospel is the death, burial, and resurrection of Jesus...for sins!
v. Then turn to 2 Cor. 4:3-4 and read again. "And even if our gospel is veiled, it is veiled to
those who are perishing. The god of this age has blinded the minds of unbelievers, so
that they cannot see the light of the gospel of the glory of Christ, who is the image of
God."
a. Say something like, "You clearly did not understand the gospel message of Jesus the
Savior and the Bible clearly shows you why. It is because your mind has been
blinded."
5. The Apostasy
A. Mormonism maintains that the true gospel message was lost from the earth shortly after the
apostles died.
i. The Mormon Apostle Orson Prat said, "Jesus...established his kingdom on earth...the
kingdoms of this world made war against the kingdom of God, established eighteen
centuries ago, and they prevailed against it, and the kingdom ceased to exist." (Journal
of Discourses. Vol. 13, page 125).
ii. But Jesus said, "And I say also unto thee, That thou art Peter, and upon this rock I will
build my church; and the gates of hell shall not prevail against it" (Matt. 16:18, KJV).
iii. As you can see, Mormonism contradicts what Jesus said. That is why they must say that
the Bible is not trustworthy. That is, it isn't trustworthy wherever it disagrees with
Mormonism.
6. Authority and The Mormon Priesthoods
A. Since Mormonism claims to be the restoration of the gospel, it also claims to have the
authority to perform priestly duties and, therefore, properly represent God here on earth.
B. All offices of the Mormon church grow out of the priesthoods.
i. Melchizedek - This is the greater priesthood. It consists of several offices:
a. Elder, seventy, high priest, patriarch or evangelist, and apostle.
b. Aaronic - a part of the greater Melchizedek priesthood.
ii. Aaronic priesthood - This is the lesser priesthood
a. Is synonymous with the Levitical Priesthood (D.&C. 107:1,6,10)
b. Performs the administration of the ordinances (D.&C. 107:13-14)
c. Deacon, teacher, priest.
C. Quite simply, the Bible contradicts what Mormons believe concerning the priesthood.
i. Jesus is the only high priest after the order of Melchizedek (Heb. 3:1; 5:6,10; 6:20;
7:11,15,17,21,24,26; 8:1; 9:11).
a. "Where Jesus, who went before us, has entered on our behalf. He has become a
high priest forever, in the order of Melchizedek" (Hebrews 6:20).
b. "And what we have said is even more clear if another priest like Melchizedek
appears, one who has become a priest not on the basis of a regulation as to his
ancestry but on the basis of the power of an indestructible life" (Heb. 7:15-16).
ii. The Melchizedek Priesthood is unchangeable and non-transferable
a. "but because Jesus lives forever, he has a permanent priesthood" (Heb. 7:24).
7. Many Gods
A. One of the truly dividing lines between Christianity and Mormonism is their doctrine of the
plurality of Gods.
i. Mormonism teaches that there are many many gods. (Mormon Doctrine by Bruce
McConkie, page 163; Teachings pages 348-349).
ii. In there desire for legitimacy they will even quote 1 Cor. 8:5 to say that the Bible also
teaches many gods. 1 Cor. 8:5 says, "For even if there are so-called gods, whether in
heaven or on earth (as indeed there are many ‘gods' and many ‘lords')."
a. They will say, "see even the Bible says there are many gods."
b. You can say, "It says there are many that are called gods. It doesn't say they really
are gods. It is saying that they only called gods. The Scriptures recognize that there
are false gods (Gal. 4:8).
c. Besides, the Bible flatly denies the existence of any other gods.
i. "You are my witnesses," declares the LORD, "...Before me no god was formed,
nor will there be one after me" (Isaiah 43:10).
ii. "This is what the LORD says -- ...I am the first and I am the last; apart from
me there is no God...Is there any God besides me? No, there is no other Rock; I
know not one" (Isaiah 44:6,8).
iii. "I am the LORD, and there is no other; apart from me there is no God" (Isaiah
45:5).
8. Errors in the Book of Mormon
A. Saved by grace after all you can do? (2 Nephi 25:23)
B. How could Moroni "read" Heb. 13:8 and James 1:17 when the N.T. never reached America?
(Mormon 9:9).
C. Helaman 12:25-26, written 6 B.C. says, "we read," quoting 2 Thess. 1:9 and John 5:29, 90
years too early since 2 Thessalonians and John hadn’t been written yet.
D. Jesus, a son of God (Alma 36:17).
E. Mosiah 21:28 says King Mosiah had a gift from God, but original B. of M. manuscript reads
"King Benjamin".
F. Jesus was born in Bethlehem (Micah 5:1-2; Matt. 2:1). In the Book of Mormon (Alma
7:9,10) it says it was Jerusalem.
A Comparison between Christian Doctrine and Mormon Doctrine
"Take up the Bible, compare the religion of the Latter-day Saints with it and see if it will stand
the test," (Brigham Young, May 18, 1873, Journal of Discourses, Vol. 16, page 46.)

Following is a comparison between Christian doctrine and Mormon doctrine. It will become very
obvious that Mormonism does not agree with the Bible. In fact, Mormonism has simply used the same
words found in Christianity and redefined them. But with a proper understanding of what Mormonism
really teaches, you will be able to see past those definitions into the real differences between
Christianity and Mormonism.
The difference is the difference between eternal life and damnation.

Topic Christian Mormon

There is only one God (Isaiah "And they (the Gods) said: Let there be light: and
43:11; 44:6,8; 45:5) there was light (Book of Abraham 4:3)

"God himself was once as we are now, and is an


exalted man, and sits enthroned in yonder
God has always been God (Psalm heavens!!! . . . We have imagined that God was
90:2; Isaiah 57:15) God from all eternity. I will refute that idea and
take away the veil, so that you may see"
(Teachings of the Prophet Joseph Smith, p. 345
GOD
"The Father has a body of flesh and bones as
tangible as man's" (Doctrine and Covenants
130:22. Compare with Alma 18:26-27; 22:9-10)
God is a spirit without flesh and "Therefore we know that both the Father and
bones (John 4:24; Luke 24:39) the Son are in form and stature perfect men; each
of them possesses a tangible body . . . of flesh and
bones." (Articles of Faith, by James Talmage, p.
38).

The Trinity is the doctrine that The trinity is three separate Gods: The Father, the
there is only one God in all the Son, and the Holy Ghost. "That these three are
universe and that He exists in separate individuals, physically distinct from each
TRINITY
three, eternal, simultaneous other, is demonstrated by the accepted records of
person: The Father, the Son, and divine dealings with man." (Articles of Faith, by
the Holy Spirit. James Talmage, p. 35.)

"The birth of the Saviour was as natural as are the


births of our children; it was the result of natural
action. He partook of flesh and blood - was
begotten of his Father, as we were of our fathers."
Jesus was born of the virgin Mary
(Journal of Discourses, Vol. 8: p. 115)
(Isaiah 7:14; Matt. 1:23)
"Christ was begotten by an Immortal Father in
the same way that mortal men are begotten by
mortal fathers" (Mormon Doctrine," by Bruce
JESUS
McConkie, p. 547)

Jesus is the eternal Son. He is


second person of the Trinity. He
has two natures. He is God in Jesus is the literal spirit-brother of Lucifer, a
flesh and man (John 1:1, 14; Col. creation. (Gospel Through the Ages, p. 15)
2;9) and the creator of all things
(Col. 1:15-17)
Mormonism distinguishes between the Holy Spirit
(God's presence via an essence) and the Holy
Ghost (the third god in the Mormon doctrine of the
THE The Holy Spirit is the third person
trinity).
HOLY of the Trinity. He is not a force.
"He [the Holy Ghost] is a being endowed with
SPIRIT He is a person. (Acts 5:3-4; 13:2)
the attributes and powers of Deity, and not a mere
force, or essence (Articles of Faith, by James
Talmage, p. 144)

Salvation has a double meaning in Mormonism:


universal resurrection and . . .
Salvation is the forgiveness of sin
"The first effect [of the atonement] is to secure
and deliverance of the sinner
to all mankind alike, exemption from the penalty of
from damnation. It is a free gift
the fall, thus providing a plan of General Salvation.
received by God's grace (Eph.
The second effect is to open a way for Individual
2:8; Rom. 6:23) and cannot be
Salvation whereby mankind may secure remission
earned (Rom. 11:6).
of personal sins (Articles of Faith, by James
SALVATION
Talmage, p. 78-79.

"As these sins are the result of individual acts it is


just that forgiveness for them should be
Salvation (forgiveness of sins) is
conditioned on individual compliance with
not by works (Eph. 2:8; Rom.
prescribed requirements -- 'obedience to the laws
4:5; Gal. 2:21)
and ordinances of the Gospel.'" (Articles of Faith p.
79)

The inspired inerrant word of God "We believe the Bible to be the word of God as far
BIBLE (2 Tim. 3:16). It is authoritative as it is translated correctly. . ." 8th Article of Faith
in all subjects it addresses. of the Mormon Church.

This is only a sample of many of the differences between Christianity and Mormonism. As you can
see, they are quite different doctrines. God cannot be uncreated and created at the same time. There
cannot be only one God and many gods at the same time. The Trinity cannot be one God in three
persons and three gods in an office known as the Trinity, etc. These teachings are mutually exclusive.
This is important because faith is only as good as the object in which it is placed. Is the Mormon
god the real one? Or, is the God of historic and biblical Christianity the real one?
Mormonism is obviously not the biblical version of Christianity. It is not Christian and Mormons
serve a different god than do the Christians -- a god that does not exist. Paul talks about this in Gal.
4:8, "when you did not know God, you were slaves to those which by nature are no gods." Only the
God of the Bible exists. There are no others. Mormonism puts its faith in a non-existent god.
What is Baptism for the Dead mentioned in 1 Cor. 15:29?
"Otherwise, what will those do who are baptized for the dead? If the dead are not raised at all,
why then are they baptized for them?" (1 Cor. 15:29).

Numerous explanations have been offered for this verse ranging from the inane to the
sophisticated. Mormonism, in particular, has claimed that this verse supports their view of baptism for
the dead. In their practice, individuals go to their local Mormon temple, dress appropriately for a
baptism, representatively adopt the name of a person who has died, and then the Mormon is baptized
in water for that deceased person. This way, the dead person has fulfilled the requirements of
salvation in the afterworld and can enjoy further spiritual benefits in the spiritual realm. But, the
Mormons are incorrect. They have usurped this verse and taken it out of context. So, let's examine 1
Cor. 15 briefly so we can see what Paul is talking about when he mentions baptism for the dead.
In Verses 1-19, the fact of Christ's resurrection is detailed by Paul. Beginning in verse 20 and going
through verse 23, Paul speaks about the order of the resurrection. Christ is the first one raised -- in a
glorified body -- and then who are His at His return. Next, verses 24 - 29 mention Christ's reign and
the abolition of death. This is when this controversial verse occurs: "Otherwise, what will those do who
are baptized for the dead? If the dead are not raised at all, why then are they baptized for them?"
Just north of Corinth was a city named Eleusis. This was the location of a pagan religion where
baptism in the sea was practiced to guarantee a good afterlife. This religion was mention by Homer in
Hymn to Demeter 478-79.156 The Corinthians were known to be heavily influenced by other customs.
After all, they were in a large economic area where a great many different people frequented. It is
probable that the Corinthians were being influenced by the religious practices found at Eleusis where
baptism for the dead was practiced.
Paul used this example from the pagans in 1 Cor. 15:29, when he said, "...if the dead are not
raised, then why are they baptized for the dead?" Paul did not say we. This is significant because the
Christian church was not practicing baptism for the dead, but the pagans were.
Paul's point was simple. The resurrection is a reality. It is going to happen when Jesus returns.
Even the pagans believe in the resurrection, otherwise, why would they baptize for the dead?
However, some are not convinced by this argument and state that the word "they" is not in the
Greek and, therefore, Paul is not speaking about the pagans.. Let's take a look.
Literally, the verse is translated as "Since what will do the being immersed on behalf of the dead if
wholly dead not are raised why also are they immersed on behalf of them."
The issue here is the word, "baptizontai" -- "they are baptized." It is the present, passive,
indicative, 3rd person, plural. In other words, it is THEY ARE BEING BAPTIZED or, THEY ARE
BAPTIZED.

I - first person singular we -- first person plural


you (singular) -- second person singular you (plural) -- second person plural
he/she/it -- third person singular they -- third person plural

It is the latter form, the third person plural (they) which the verb "baptizo" is in. Therefore, the
best translation is "THEY are baptized." The KJV renders it as, "Else what shall they do which are
baptized for the dead, if the dead rise not at all? why are they then baptized for the dead?" The NKJV,
"Otherwise, what will they do who are baptized for the dead, if the dead do not rise at all? Why then
are they baptized for the dead?" The NASB, "Otherwise, what will those do who are baptized for the
dead? If the dead are not raised at all, why then are they baptized for them?"

The True Jesus


156
Bible Knowledge Commentary on 1 Cor. 15:29. Dallas Seminary Faculty.
There is a simple way to see if someone has the true Jesus or not. By true Jesus, I mean the one
of the Bible, not the one of Mormonism who is the brother of the devil, nor the Jehovah's Witness
Jesus who is Michael the Archangel, and certainly not the one of the New Age Movement who is simply
a man in tune with the divine consciousness.

• The Jesus of the Bible is prayed to (Acts 7:55-60; Psalm 116:4 and Zech. 13:9 with 1 Cor. 1:1-
2).
• The Jesus of the Bible is worshiped (Matt. 2:2,11; 14:33; 28:9; John 9:35-38; Heb. 1:6)
• The Jesus of the Bible called God (John 10:28; Heb. 1:8).

In cult theologies, Jesus is a creation in one form or another (this is why the Jehovah's Witnesses
add the word ‘other' four times to Col. 1:16-17). Therefore, He is not to be prayed to, worshiped, or
called God.
If you are a Christian then you will be able to pray to Jesus, not just through. You will be able to
worship Jesus equally with the Father. And you will be able to call Jesus your Lord and God. A cultist
cannot do this. A cultist has a false Jesus, and, therefore, a false hope of salvation.

The following is an expansion of the above points

If you put your faith in a Jesus that is not true, then your faith is useless. The power of faith does
not rest in the act of believing, but in its object; the greatest faith in someone false is the same as no
faith at all. Sincerity and false messiahs do not bridge the chasm of sin between God and man, only
the Jesus of the Bible does that. Who then, is the true Jesus?
Jesus said that He was the only One who reveals the Father (Matt. 11:27 and Luke 10:22): "All
things have been committed to me by my Father. No one knows who the Son is except the Father, and
no one knows who the Father is except the Son and those to whom the Son chooses to reveal him"
(NIV).
So, to know the true Father you must first know the true Jesus. The question is, how do you
recognize the true Jesus? Simple, look in the Bible.
If you were to say, "Father receive my spirit," who would you be praying to? The Father, right?
If you were to say, "Jesus receive my spirit," who would you be praying to? Jesus.
In Acts 7:59, Stephen, while full of the Holy Spirit (v. 55), prayed to Jesus:

“And they went on stoning Stephen as he called upon the Lord and said, "Lord Jesus, receive
my spirit." (See also Acts 9:14; Rom. 10:13.)

Stephen prayed to Jesus, not just through Him. If it is acceptable for him then it should be alright
for you. The Jesus of the Bible is prayed to. I pray to Jesus. Do you? If yes, good. If not, why?
But you might say, "Jesus said to pray to the Father." I do. But I also pray to Jesus as Stephen
did. If the church is only to pray to the Father then why did Stephen, under the inspiration of the Holy
Spirit, address Jesus in His prayer? Was he wrong? See also 1 Cor. 1:1-2 with Psalm 116:4 where
calling upon the name of the Lord is prayer and prayer is addressed to Jesus by the Corinthian church.

Jesus was also worshipped. The verses are:

And those who were in the boat worshiped Him, saying, "You are certainly God's son! (Matt.
14:33).
And behold, Jesus met them and greeted them. And they came up and took hold of His feet
and worshiped Him (Matt. 28:9).
See also Matt. 2:2,11; 14:33; 28:9; John 9:35-38; Heb. 1:6.

The Jesus of the Bible is prayed to and worshiped. Do you do what Jesus' disciples did? Do you
pray to and worship the true Jesus?
Since it is against Mormon and Jehovah's Witness theologies to pray to Jesus but only through if
you do worship Jesus, how can you do that without praying to Him? And, do you honor Him equally
with the Father as Jesus said to do in John 5:23? If you do not, then why not?
There is just one more issue to address. Do you call Jesus your Lord and God?
After Jesus' resurrection He showed Himself to many people. One of them was Thomas. John
20:28:

Thomas answered and said to Him [Jesus], "My Lord and my God!" The literal Greek says, "The
Lord of me and the God of me."

"My God!" is a pagan expression used today. 157 Two points can be made from this. First, do you
agree that Thomas a devout Jew was swearing, like a pagan of today? Second, there is no biblical
account of swear words. Peter did swear in Mark 14:71 by swearing he did not know Jesus. To say
Thomas was swearing, or merely exclaiming profound surprise has no evidence.

God calls Jesus God in Heb. 1:8:

But of the Son He [the Father] says, "Thy throne, O God, is forever and ever..."

Unfortunately, in the Jehovah's Witness Bible in Heb. 1:8 you'll see that it says, "God is your
throne, forever and ever." This, technically speaking, is a legitimate translation. The reason this is so
lies in the nature of the Greek language and the fact that the form of the word "God" and "Throne"
both end in a noun construction that is interchangeable, therefore making the NWT translation
legitimate. It is unfortunate that the Watchtower has chosen to do this.

Conclusion:

The Jesus of the Bible is prayed to (Acts 7:55-60; Psalm 116:4 and Zech. 13:9 with 1 Cor. 1:1-2),
worshiped (Matt. 2:2,11; 14:33; 28:9; John 9:35-38; Heb. 1:6), and called Lord and God (John
20:28; Heb. 1:8). If I have the wrong Jesus, and therefore I serve the wrong God, then why do I pray
to Jesus, worship Him, and call Him my Lord and God as the Scriptures teach? But, if you have the
true Jesus, why is it you don't do those things? Why does JW theology not agree with the scriptures?
I think the answer is simple. The Jesus of the cults is not the true Jesus. Therefore, they are
wrong.

It pains me to even quote the ungodly’s misuse of the Lord’s name. But for the purpose of illustrating their sin
157

and making the point of truth concerning Thomas’ statement, I have quoted it here.
Mormon Objections Answered
This web site has been active since October 1995 and many Mormons have e-mailed me with
complaints and comments. Following are the actual quotes of some of them:

"Mormons are too Christians!"; "You need to read the book of Mormon and pray sincerely";
"Stop wasting your time fighting the true church"; "You don't know what you are talking
about"; "You misrepresent the Mormon Church"; "You do not document enough of your
statements from Mormon canonical writings."

You can either continue reading through the answers or click on the appropriate objection to get my
response.

"The Mormons are too Christians"

Mormons think they are Christians. They use Christian words. They say they believe in Jesus and
the Bible. They say they pray to God through Jesus. And they try to live good lives. The truth is that
Mormons are not Christians. They use Christian words but the words have non-Christian definitions.
They believe in a Jesus, but the Jesus they believe in is not the Jesus of the Bible. Compare the
following:

The Jesus of Mormonism The Jesus of the Bible

Jesus is created. Jesus is uncreated


Mormon Doctrine by (John 1:1-3; Col. 1:16-17)
Bruce McConkie, pp. 192, 589

Jesus is the brother of the devil. Jesus is not the brother of the devil
Mormon Doctrine by The devil is a fallen created angel.
Bruce McConkie, pp. 192, 589. Jesus created all things Col. 1:16-17.
Therefore, Jesus is not the brother of the devil.

Jesus is one of three gods. Jesus is second person of the Trinity


Mormon Doctrine by
Bruce McConkie, p. 319.

Jesus is not prayed to Jesus is prayed to


(Acts 7:60; 1 Cor. 1:1-2 with Zech. 13:9)

Jesus did not pay for all sins Jesus did pay for all sins
(Doctrines of Salvation by (1 John 2:2; 1 Pet. 2:24)
Joeseph F. Smith, Vol. 1, p.
135)

The Jesus of Mormonism is not the Jesus of the Bible. They cannot be both created and non
created, prayed to and not prayed to, the brother of the devil and not the brother of the devil. They
are different. Because the Jesus of Mormonism is false, the faith of Mormons is useless. Faith is only
as good as the object in which it is placed. The Jesus of Mormonism should be called something else,
like Barana, Joe, Sasquatch, or something. Then there wouldn't be any problem at all identifying it as
something other than biblical.

"You need to read the Book of Mormon and pray sincerely."

I get this objection quite often. Mormons believe that if you read the Book of Mormon and sincerely
pray about it that God will hear your prayer, answer you, and give you a testimony that the Book of
Mormon is true. They often quote James 1:5, "If any of you lack wisdom, let him ask of God..." to
support their belief. There are several problems with their thinking.

1. James 1:5 was written to those who already were Christians, not to those seeking to
become Christians or seeking to find truth in a book. As is often the case, Mormons take
verses out of context. The context of James is clear when James 1:1 is read: "James, a
bond-servant of God and of the Lord Jesus Christ, to the twelve tribes who are dispersed
abroad, greetings."
This is obviously written to those who are Jewish Christians scattered abroad. They
already knew the truth of the Word of God. James instructs them to ask for wisdom from
God.
Wisdom is the proper use of knowledge and truth. We need wisdom to know what to do
in certain situations. If you receive counsel that is contrary to the Word of God, then it is
obviously false. All counsel must be in agreement with the Bible. Nowhere in God's Word
does it instruct us to pray about a book to see if it true. This leads us to the next point.
2. You don't pray about truth, you go to the Bible and you look there to find it. The Bible is
God's Word. It is inerrant and infallible. It is the judge of all things that it addresses and is
correct in all areas of which it speaks. The Mormon should put the Bible above his own
feelings.
3. Sincerity is no guarantee that God will answer your prayer. You could be sincerely wrong,
sincerely praying but praying falsely, sincerely praying to a false god, etc.
Just because someone is 'sincere' does not mean that God will answer them. If someone
looks to their own sincerity as a means for deserving or expecting something from God, then
they are basing their hope on what is inside of them (sincerity) instead of what Jesus did on
the Cross.
4. If Mormonism is wrong and you pray to the god of Mormonism then the testimony you
receive will be false. Just because the Mormons use Christian words does not mean they are
Christian. The god of Mormonism is not the God of the Bible. Therefore, when a Mormon
prays to the god of Mormonism he is ultimately praying to the devil and will receive a false
testimony, no matter how good it "feels."
5. Mormons are trusting their feelings. The Mormons trust what they feel in their hearts instead
of what the Bible says. Many Mormons have told me that there is nothing that I could show
them that would make them change their minds because they have a testimony to the truth.
This is a serious problem because God wants us to examine the evidence. That is why there
is an empty tomb; that is why Jesus said, "Touch me and see. A spirit does not have flesh
and bones as you see Me have" (Luke 24:39). That is why the apostles quoted the life of
Jesus and what He did so much. To ignore evidence is to open yourself up for deception.
By trusting their testimony, their feelings, the Mormons put what they feel above God's
word. Mormon's have a testimony. They say that Mormonism is true, that Joseph Smith was
a true prophet, and that the Book of Mormon is another Testament to Jesus. But,
Mormonism contradicts the Bible. Mormonism says there are many gods (even though they
worship only one of them); the Bible says there is only one God (Isaiah 43:11; 44:6,8). This
is only one of the areas where Mormonism contradicts the Bible, yet Mormons choose their
testimony above God's word.
6. Why do you never hear a Mormon say, "Pray about the Bible to see if it is true."? This has
always puzzled me. God's Word does not need verification. It simply is true. But the Book of
Mormon, on the other hand, must be prayed about (so they say).
"Stop wasting your time fighting the true church"

I'm not wasting my time fighting the true church. I am fighting a cult. Mormonism teaches that
God came from another world, that he used to be a man, that we are all born in heaven from god and
his goddess wife, that there is a secret temple ceremony where you learn secret handshakes and hugs
in order to get into the highest level of heaven, that Jesus blood is not sufficient to cleanse you of all
your sins, etc. There is no way that that is the truth. It is completely false.
Also, Mormonism says that all other churches are false, that there was a great apostasy, and that
Mormonism is the restoration of the true gospel. So, in reality, I am simply defending my faith, which
is what I am told to do in 1 Pet. 3:15.

"You don't know what you are talking about."

Mormons often say that I don't know what I am talking about. They've never disproved anything
I've said, but they are quick to say I am wrong.
I've studied Mormonism for 14 years and have a very good understanding of it. I've spoken to
hundreds of Mormon missionaries who've confirmed Mormon theology to me. I've got the Mormon
books that document their doctrines that I've cited my pages. The truth is that my quotes of
Mormonism are accurate...which is why I fight against Mormonism.

"You misrepresent the Mormon Church."

Again, if I am wrong, show me where and how. I document everything in Mormonism and show
why it is wrong. I don't deny that the Mormons are good people and that they do good things and try
to honor God. It isn't their moral behavior I have a problem with, it is their view of God and salvation
that is wrong. Mormonism misrepresents Christianity, the Cross, Salvation by Grace, the Deity of
Christ, the Atonement, and more. It is a vast misrepresentation of Christianity. Mormonism is a non-
Christian religion.

“You do not document enough of your statements from Mormon canonical writings.”

This is a common complaint made by several LDS critics of CARM. It is an attempt to harm my
credibility and the accuracy of the documentation. Many of them demand that I produced
documentation from their four standard works. But, this is not always possible with many of their
teachings. There are many Mormon doctrines taught that are not explicitly stated in the four standard
works (The Bible, The Book of Mormon, The Doctrine and Covenants, and The Pearl of Great Price).
My critics know this. Yet, they want me to comply with their standards of documentation and if I do
not, they often complain that I have failed to provide citations from official LDS sources.
For example, it does not state in official Mormon Scripture that there is a goddess wife of God the
Father in heaven. Yet, this is taught in non "Official" writings: Articles of Faith, by James Talmage, p.
443, is a good example.
My question to my critics in this regard is, "If it is not official Mormon doctrine, then why do so
many Mormons believe it and why do your Mormon authorities teach it?"
Was the LDS Jesus born of the Virgin Mary?
The Bible teaches that Jesus was born of the virgin Mary. This means that Mary had no sexual
relations with any man prior to the birth of Jesus.
Likewise, the Mormon Church also maintains that Jesus was born of the Virgin Mary. However,
there is quite an interesting array of Mormon authorities who have said some very interesting things
about the relationship between God and Mary in regards to Jesus’ birth. Let’s take a look at some of
them and see what we find.
Brigham Young, second prophet and president of the LDS church said,

"The birth of the Saviour was as natural as are the births of our children; it was the result of
natural action. He partook of flesh and blood—was begotten of his Father, as we were of our
fathers." (Journal of Discourses, v. 8, p. 115).

Brigham Young also said, "Now, remember from this time forth, and for ever, that Jesus Christ
was not begotten by the Holy Ghost." (Journal of Discourses, Vol. 1, page 51).

Brigham Young said, "When the time came that His first-born, the Saviour, should come into
the world and take a tabernacle, the Father came Himself and favoured that spirit with a
tabernacle instead of letting any other man do it. The Saviour was begotten by the Father of
His spirit, by the same Being who is the Father of our spirits." (Journal of Discourses, Vol. 4,
page 218, 1857.)

Joseph Fielding Smith, stated:

"The birth of the Savior was a natural occurrence unattended with any degree of mysticism,
and the Father God was the literal parent of Jesus in the flesh as well as in the spirit."
(Religious Truths Defined, p. 44) as cited in the book, Mormonism: Shadow or Reality, by
Gerald and Sandra Tanner, Utah Lighthouse Ministry, P.O. Box 1854, Sal Lake City, Utah
84110, Bookstore at 1350 South West Temple. 1982, page 260).

"They tell us the Book of Mormon states that Jesus was begotten of the Holy Ghost. I challenge
that statement. The Book of Mormon teaches no such thing! Neither does the Bible." (Doctrines
of Salvation, Vol. 1, page 19)

Bruce McConkie, who was a member of the First Council of the Seventy stated,

"Christ was begotten by an Immortal Father in the same way that mortal men are begotten by
mortal fathers," (Mormon Doctrine, 1966, page 547.)

"And Christ was born into the world as the literal Son of this Holy Being; he was born in the
same personal, real, and literal sense that any mortal son is born to a mortal father. There is
nothing figurative about his paternity; he was begotten, conceived and born in the normal and
natural course of events,...Christ is the Son of Man, meaning that his Father (the Eternal God!)
is a Holy Man." (Mormon Doctrine, by Bruce McConkie, page 742.)

Heber C. Kimball who was a member of the first presidency said,

"In relation to the way in which I look upon the works of God and his creatures, I will say that I
was naturally begotten; so was my father, and also my saviour Jesus Christ. According to the
Scriptures, he is the first begotten of his father in the flesh, and there was nothing unnatural
about it." (Journal of Discourses, v. 8, p. 211)
"The man Joseph, the husband of Mary, did not, that we know of, have more than one wife, but
Mary the wife of Joseph had another husband" (Deseret News, October 10, 1866) as cited in
the book, Mormonism: Shadow or Reality, by Gerald and Sandra Tanner, Utah Lighthouse
Ministry, P.O. Box 1854, Sal Lake City, Utah 84110, Bookstore at 1350 South West Temple.
1982, page 261.

What conclusions can we draw from the words of the leaders of the Mormon church regarding
Jesus’ birth?

1. It was the result of natural action, (Brigham Young, Journal of Discourses, v. 8, p. 115).
2. Jesus was not begotten by the Holy Ghost." (Journal of Discourses, Vol. 1, page 51); (Doctrines
of Salvation, Vol. 1, page 19).
3. "The Father came Himself and favoured that spirit with a tabernacle instead of letting any other
man do it" (Journal of Discourses, Vol. 4, page 218, 1857.)
4. The birth was the result of natural action, (Journal of Discourses, Vol. 8, p. 115).
5. "The Father God was the literal parent of Jesus in the flesh as well as in the spirit." (Religious
Truths Defined, p. 44)
6. "Christ was begotten by an Immortal Father in the same way that mortal men are begotten by
mortal fathers," (Mormon Doctrine, 1966, page 547.)
7. "There is nothing figurative about his [Jesus’] paternity; he was begotten, conceived and born
in the normal and natural course of events" (Mormon Doctrine, by Bruce McConkie, page 742.)

Remember, according to Mormon teaching the Holy Ghost is a male personage, a man. (A
Marvelous Work and a Wonder, by Le Grand Richards, Salt Lake City, 1956, page 118; Heber C.
Kimball, in Journal of Discourses, Vol. 5, page 179.)
The Father, who is God, is also in the form of a man (Joseph Smith, Journal of Discourses, Vol. 6,
p. 3; and Doctrine and Covenants, 130:22.
Mary, of course, was a woman.
This is even more interesting when we realize that the Mormon church officially proclaims that
Jesus was born of a virgin. For example, Bruce McConkie stated "Modernistic teachings denying the
virgin birth are utterly and completely apostate and false." (Bruce R. McConkie, Mormon Doctrine,
page 822.) That is fine. Let them proclaim it. But quite honestly, I fail to see how the Mormon people
can assert that Mary remained a virgin in light of this evidence from their prophets and apostles. I see
them saying two different things and backpedaling trying to sound Christian.
Of course, this is my opinion and the Mormons who read this will accuse me of sensationalism.
But, I am simply pointing out what many of their official church leaders have said. In the changing
teachings of Mormonism, you never know what you'll find next.

__________________

Bibliography:

• Doctrines of Salvation, Vol. 1, by Joseph Fielding Smith, Bookcraft, Salt Lake City, Utah, 1954.
• Mormon Doctrine, by Bruce R. McConkie, 2 nd Edition, Bookcraft, Salt Lake City, Utah, 1979
• A Marvelous Work and a Wonder, by LeGrand Richards, Deseret Book Company, Salt Lake City,
Utah, 1950
• Mormonism: Shadow or Reality, by Gerald and Sandra Tanner,Utah Lighthouse Ministry, P.O.
Box 1854, Sal Lake City, Utah 84110, Bookstore at 1350 South West Temple. 1982,
• Where Does it Say That? By Bob Witte, (No publisher or location).
Hinckley says Mormons Believe in a Different Jesus

"In bearing testimony of Jesus Christ, President Hinckley spoke of those outside the Church
who say Latter-day Saints 'do not believe in the traditional Christ.' 'No, I don't. The traditional
Christ of whom they speak is not the Christ of whom I speak. For the Christ of whom I speak
has been revealed in this the Dispensation of the Fullness of Times. He together with His
Father, appeared to the boy Joseph Smith in the year 1820, and when Joseph left the grove
that day, he knew more of the nature of God than all the learned ministers of the gospel of the
ages.'" (LDS Church News Week ending June 20, 1998, p.7 )

Christians have long maintained that Mormonism teaches a Jesus that is different from what the
Bible reveals. Of course, the Mormons say that they believe in the same Jesus that originally walked
on the earth and is revealed in the Bible. Though they may make this claim, it is up to them to prove
it. Especially in light of what Mormonism says about God and Jesus.
In Mormonism, Jesus is a creation, the product of relations between god and his goddess wife who
used to be people from another world (Mormon Doctrine by Bruce McConkie, pp. 192, 321, 516,
589). Jesus is the literal spirit brother of the devil and of you and I (McConkie, pp, 192, 589). Also,
in Mormon theology, God has a body of flesh and bones (Doctrine & Covenants, 130:22.) as does his
wife and together they produce spirit offspring in heaven who inhabit human bodies on earth.
Very few, if any, of the 'different' Mormon doctrines are found in their Standard Works: the Bible,
The Book of Mormon, Doctrine & Covenants, and The Pearl of Great Price. Rather, they are taught by
Mormons of high standing: prophets, apostles, members of the 70 Quorum. McConkie, for example,
was a member of the 70's Quorum, a very high ranking Mormon and wrote the book, Mormon Doctrine
where much of the documentation for this is taken from.
President Hinkley, the prophet and revelator of the Mormon Church, has publicly declared that the
Jesus of Mormonism and traditional Christianity are not the same. Let's take a look at the difference
between the Mormon Jesus and the one of Traditional Christianity.

The Mormon Jesus The Christian Jesus


1. The literal son of god and his 1. Not the literal son of god and his goddess
goddess wife begotten in the pre- wife.
existence. 2. Not the brother of all spirits born in
2. The brother of all spirits born in heaven in a premortal existence.
heaven in the premortal existence. 3. Not one of 3 gods in the godhead.
3. One of 3 gods in the godhead. 4. The Trinity is 3 persons in one God.
4. The Trinity is three separate gods. 5. Was always spirit from eternity.
5. First one to receive a spirit body.
6. Atoned for sin on the cross alone.
6. Atoned for sin on the cross and in
the garden of Gethsemane.
This brief comparison should help you see the difference between the two. Jesus cannot
be both literal son of god and his goddess wife and not the literal begotten son. He cannot
be both one of 3 gods, and not one of 3 gods. They are different. In fact, it would be a lot
easier if the Mormons called him by a different name. For example, Bolok would be good.
Bolok is one of three gods, but Jesus is not. etc. That way, it would be a lot easier to tell
them apart.
Mormonism definitely does not teach the same Jesus as Christianity.

The Mormon Plan of Eternal Progression


Mormonism and the Negro
According to Mormon history and authoritative Mormon teachers, the Negro, as they were referred
to, are a cursed race because they were not faithful to God in their first estate (the pre-existent life
with God). Hence, when they are born they are born in black skinned families.
The early Mormon church was highly prejudicial against black people. And though there has been a
change in attitude to blacks as of June 9, 1978 when they were finally allowed into the Mormon
priesthood, it cannot be denied that Mormonism was, up until very recently, a segregated church.
Please consider the following small sample of official Mormon writings as proof of their prejudice,
their inconsistency, their doctrinal waffling, their yielding to political pressure, and their failure to truly
represent Christ on earth.

1. "At the time the devil was cast out of heaven, there were some spirits that did not know who had
authority, whether God or the devil. They consequently did not take a very active part on either
side, but rather thought the devil had been abused, and considered he had rather the best claim
to the government. These spirits were not considered bad enough to be cast down to hell, and
never have bodies; neither were they considered worthy of an honorable body on this earth." A
speech by Elder Orson Hyde, delivered before the High Priests' Quorum,, in Nauvoo, April 27th,
1845, printed in Liverpool, page 30.
2. "The very fact that God would allow those spirits who were less worthy in the spirit world to
partake of a mortal body at all is further evidence of his mercy." Mormonism and the Negro, part
I, pages 48-50.
3. "It is the Mormon belief that in our pre-mortal state there were a large number of individuals
who, due to some act or behavior of their own in the pre-existence, forfeited the right to hold the
Priesthood during their mortal lives...the Negro is thus denied the Priesthood because of his own
behavior in the pre-existence." The Church and the Negro, pages 42-43.
4. "As a result of his rebellion, Cain was cursed and told that "the earth" would not thereafter yield
him its abundance as previously. In addition he became the first mortal to be cursed as a son of
perdition...The Lord placed on Cain a mark of a dark skin, and he became the ancestor of the
black race. (Moses 5; Gen. 4; Teachings, p. 169)." Mormon Doctrine by Bruce McConkie, page
109.
5. "Shall I tell you the law of God in regard to the African race? If the white man who belongs to the
chosen seed mixes his blood with the seed of Cain, the penalty, under the law of God, is death on
the spot. This will always be so." Brigham Young, Journal of Discourses, Vol. 10, page 110.
The reader should be reminded that Brigham Young, the second prophet of the Mormon
church said that whatever he preached was as good as scripture -- J. D. Vol. 13, page 95, 264.
6. "...the church went to court on several occasions to block Negroes from moving into the San
Francisco neighborhood in which the headquarters was located." The Christian Century, Sept. 29,
1965, page 1184.
7. On Oct. 28, 1865, the Mormon Church paper Millennial Star printed an article which stated that
"Mormonism is that kind of religion the entire divinity of which is invalidated, and its truth utterly
rejected, the moment that any one of its leading principles I acknowledged to be false..."
8. The Mormon write John J. Stewart stated: "If we as members of the Church are going to pick and
choose among the Prophet's teachings, and say ‘this one is of God, we can accept it, but this one
is of man, we will reject that,' then we are undermining the whole structure of our faith..."
Mormonism and the Negro, page 19.

9. "The prophets have declared that there are at least two major stipulations that have to be met
before the Negroes will be allowed to possess the Priesthood. The first requirement relates to
time. The Negroes will not be allowed to hold the Priesthood during mortality, in fact, not until
after the resurrection of all of Adam's children. The other stipulation requires that Abel's seed
receive the first opportunity of having the priesthood....the last of Adam's children will not be
resurrected until the end of the millennium. Therefore, the Negroes will not receive the Priesthood
until after that time... this will not happen until after the thousand years of Christ's reign on
earth." The Church and the Negro, 1967, pages 45-48.
10. Albert B. Fritz, NAACP branch president, said at a civil rights meeting Friday night that his
organization promised not to picket the 133rd Semi-Annual General Conference of the Church on
Temple Square. He added, however, that the NAACP will picket Temple Square, next Saturday if
the Church does not present an ‘acceptable' statement on civil rights before that day." Deseret
News, Oct. 5, 1963.
11. "Some 3,000 students, led by the BSU, paraded peacefully on the school's campus in Seattle
Monday over the issue of alleged racism at BYU." Deseret News, March 10, 1970.
12. "The demonstration was the latest in a series of protests against The Church of Jesus Christ of
Latter-day Saints (Mormon) because the church bars Negroes from its priesthood." Deseret News,
Jan. 10, 1970.
13. In June of 1978 the Mormon Church officiated a ‘revelation' stating that blacks could then hold
the priesthood.

The Bible says that God is not a respecter of persons, but the Mormon church is: First in its
prejudice of the blacks for being black, and second in its succumbing to political and social pressure to
change its doctrinal views. Is such a church really from God?

I am greatly indebted to the book Mormonism Shadow or Reality by Jerald and Sandra Tanner for
the sources cited above.
Interesting Quotes from Joseph Smith the Founder of Mormonism
1. Joseph Smith boasted that he did more than Jesus to keep a church together.
"God is in the still small voice. In all these affidavits, indictments, it is all of the devil--all
corruption. Come on! ye prosecutors! ye false swearers! All hell, boil over! Ye burning mountains,
roll down your lava! for I will come out on the top at last. I have more to boast of than ever any
man had. I am the only man that has ever been able to keep a whole church together since the
days of Adam. A large majority of the whole have stood by me. Neither Paul, John, Peter, nor
Jesus ever did it. I boast that no man ever did such a work as I. The followers of Jesus ran away
from Him; but the Latter-day Saints never ran away from me yet . . . " (History of the Church,
Vol. 6, p. 408-409).
2. Joseph Smith said the Book of Mormon was more correct than the Bible.
"I told the brethren that the Book of Mormon was the most correct of any book on earth, and
the keystone of our religion, and a man would get nearer to God by abiding by its precepts, than
by any other book." (History of the Church, Vol. 4, page 461)
3. Joseph Smith made a false prophecy (one of several).
". . .I prophesy in the name of the Lord God of Israel, unless the United States redress the
wrongs committed upon the Saints in the state of Missouri and punish the crimes committed by
her officers that in a few years the government will be utterly overthrown and wasted, and there
will not be so much as a potsherd left . . . " (History of the Church, Vol. 5, p. 394
4. Joseph Smith said mothers have babies in eternity and some are on thrones.
"A question may be asked, ‘Will mothers have their children in eternity?' Yes! Yes! Mothers,
you shall have your children." (Journal of Discourses, Vol. 6, page 10). "Eternity is full of thrones,
upon which dwell thousands of children reigning on thrones of glory, with not one cubit added to
their stature." (Journal of Discourses, Vol. 6, p. 10).
5. Joseph Smith said there are many Gods.
"Hence, the doctrine of a plurality of Gods is as prominent in the Bible as any other doctrine.
It is all over the face of the Bible . . . Paul says there are Gods many and Lords many . . . but to
us there is but one God--that is pertaining to us; and he is in all and through all" (History of the
Church, Vol. 6, page 474). "In the beginning, the head of the Gods called a council of the Gods;
and they came together and concocted a plan to create the world and people it." (JD, Vol. 6, p.
5).
6. Joseph Smith said the Trinity is three gods.
"I have always declared God to be a distinct personage, Jesus Christ a separate and distinct
personage from God the Father, and the Holy Ghost was a distinct personage and a Spirit: and
these three constitute three distinct personages and three Gods." (Teachings of Prophet Joseph
Smith p. 370).
7. Joseph Smith said God was once a man.
"God himself was once as we are now, and is an exalted Man, and sits enthroned in yonder
heavens...I say, if you were to see him to-day, you would see him like a man in form -- like
yourselves, in all the person, image, and very form as a man....it is necessary that we should
understand the character and being of God, and how he came to be so; for I am going to tell you
how God came to be God. We have imagined and supposed that God was God from all eternity, I
will refute that idea, and will take away and do away the veil, so that you may see....and that he
was once a man like us; yea, that God himself the Father of us all, dwelt on an earth the same as
Jesus Christ himself did." (Journal of Discourses, Vol. 6, p. 3).
8. Joseph Smith said that there are men living on the moon who dress like Quakers and live to
be nearly 1000 years old. Since he was wrong about the moon, is it safe to trust him regarding
the way to Heaven? (The Young Woman's Journal, Vol. 3, pages 263-264. See reprint in
Mormonism -- Shadow or Reality? by Jerald and Sandra Tanner, page 4.)
9. Joseph Smith said our greatest responsibility is to seek after our dead.
"The greatest responsibility in this world that God has laid upon us is to seek after our dead"
(Journal of Discourses, Vol. 6, page 7).
Interesting Quotes from Brigham Young the Second Prophet of the
Mormon Church
1. Brigham Young said your own blood must atone for some sins.
"There is not a man or woman, who violates the covenants made with their God, that will not
be required to pay the debt. The blood of Christ will never wipe that out, your own blood must
atone for it . . . " (Journal of Discourses, Vol. 3, page 247; see also, Vol. 4, pp. 53-54, 219-220.)
2. Brigham Young said you must confess Joseph Smith as a prophet of God in order to be
saved.
"...and he that confesseth not that Jesus has come in the flesh and sent Joseph Smith with
the fullness of the Gospel to this generation, is not of God, but is Antichrist." (Journal of
Discourses, Vol. 9, p. 312).
3. Brigham Young said his discourses are as good as Scripture.
"I say now, when they [his discourses] are copied and approved by me they are as good
Scripture as is couched in this Bible . . . " (Journal of Discourses, Vol. 13, p. 264; see also page
95.)
4. Brigham Young said he had never given any counsel that was wrong.
"I am here to answer. I shall be on hand to answer when I am called upon, for all the counsel
and for all the instruction that I have given to this people. If there is an Elder here, or any
member of this Church, called the Church of Jesus Christ of Latter-day Saints, who can bring up
the first idea, the first sentence that I have delivered to the people as counsel that is wrong, I
really wish they would do it; but they cannot do it, for the simple reason that I have never given
counsel that is wrong; this is the reason." (Journal of Discourses, Vol. 16, page 161.)
5. Brigham Young compared his sermons with scripture.
"I know just as well what to teach this people and just what to say to them and what to do in
order to bring them into the celestial kingdom...I have never yet preached a sermon and sent it
out to the children of men, that they may not call Scripture. Let me have the privilege of
correcting a sermon, and it is as good Scripture as they deserve. The people have the oracles of
God continually." (Journal of Discourses, Vol. 13, p. 95.)
6. Brigham Young said you are damned if you deny polygamy.
"Now if any of you will deny the plurality of wives, and continue to do so, I promise that you
will be damned." (Journal of Discourses, Vol. 3, p. 266). Also, "The only men who become Gods,
even the Sons of God, are those who enter into polygamy." (Journal of Discourses, Vol. 11, page
269).
7. Brigham Young said you can't get to the highest heaven without Joseph Smith's
consent. "...no man or woman in this dispensation will ever enter into the celestial kingdom
of God without the consent of Joseph Smith." (Journal of Discourses, Vol. 7, p. 289).
8. Brigham Young said God was progressing in knowledge.
"God himself is increasing and progressing in knowledge, power, and dominion, and will do
so, worlds without end." (Journal of Discourses, Vol. 6, p. 120).
9. Brigham Young boasted.
"What man or woman on earth, what spirit in the spirit-world can say truthfully that I ever
gave a wrong word of counsel, or a word of advice that could not be sanctioned by the heavens?
The success which has attended me in my presidency is owing to the blessings and mercy of the
Almighty . . . " (Journal of Discourses, Vol. 12, p. 127).
10. Brigham Young said that we are obligated to keep all the laws and ordinances of God.
"Some of you may ask, ‘Is there a single ordinance to be dispensed with? Is there one of the
commandments that God has enjoined upon the people, that he will excuse them from obeying?'
Not one, no matter how trifling or small in our own estimation. No matter if we esteem them non-
essential, or least or last of all the commandments of the house of God, we are under obligation
to observe them." (Journal of Discourses, Vol. 8, p. 339).
11. Brigham Young said Jesus' birth was as natural as ours.
"The birth of the Savior was as natural as the births of our children; it was the result of
natural action. He partook of flesh and blood--was begotten of his Father, as we were of our
fathers." (Journal of Discourses, Vol. 8, p. 115).

12. Brigham Young said that God the Father and Mary 'do it.'
"When the time came that His first-born, the Saviour, should come into the world and take a
tabernacle, the Father came Himself and favoured that spirit with a tabernacle instead of letting
any other man do it." (Journal of Discourses, Vol. 4, Page 218.) "The birth of the Savior was as
natural as are the births of our children; it was the result of natural action. He partook of flesh
and blood -- was begotten of his Father, as we were of our fathers." (Journal of Discourses, Vol.
8, page 115). Note: the late Bruce McConkie who was a member of the First Council of the
Seventy stated "There is nothing figurative about his paternity; he was begotten, conceived and
born in the normal and natural course of events..." (Mormon Doctrine, by Bruce McConkie, page
742.)
13. Brigham Young said that Jesus was not begotten by the Holy Spirit
"I have given you a few leading items upon this subject, but a great deal more remains to be
told. Now, remember from this time forth, and for ever, that Jesus Christ was not begotten by
the Holy Ghost." (Journal of Discourses, Vol. 1, page 51).
14. Brigham Young taught that Adam was God.
"Now hear it, O inhabitants of the earth, Jew and Gentile, Saint and sinner! When our father
Adam came into the garden of Eden, he came into it with a celestial body, and brought Eve, one
of his wives, with him. He helped to make and organize this world. He is Michael, the Archangel,
the Ancient of Days! about whom holy men have written and spoken -- He is our Father, and our
God, and the only God with whom we have to do. Every man upon the earth, professing
Christians or non professing, must hear it, and will know it sooner or later." (Journal of
Discourses, Vol. 1, page 50).
15. Brigham Young made a false prophecy?
"In the days of Joseph [Smith] it was considered a great privilege to be permitted to speak to
a member of Congress, but twenty-six years will not pass away before the Elders of this Church
will be as much thought of as the kings on their thrones." (Journal of Discourses, Vol. 4, page
40.)
16. Brigham Young comments about blacks
"You see some classes of the human family that are black, uncouth, uncomely, disagreeable
and low in their habits, wild, and seemingly deprived of nearly all the blessings of the intelligence
that is generally bestowed upon mankind....Cain slew his brother. Can might have been killed,
and that would have put a termination to that line of human beings. This was not to be, and the
Lord put a mark upon him, which is the flat nose and black skin." (Journal of Discourses, Vol. 7,
page 290).
"In our first settlement in Missouri, it was said by our enemies that we intended to tamper
with the slaves, not that we had any idea of the kind, for such a thing never entered our minds.
We knew that the children of Ham were to be the "servant of servants," and no power under
heaven could hinder it, so long as the Lord would permit them to welter under the curse and
those were known to be our religious views concerning them." (Journal of Discourses, Volume 2,
page 172.)
"Shall I tell you the law of God in regard to the African race? If the white man who belongs to
the chosen seed mixes his blood with the seed of Cain, the penalty, under the law of God, is
death on the spot. This will always be so." (Journal of Discourses, Volume 10, page 110.)
Interesting Quotes from the book Articles of Faith, by James Talmage
On the jacket cover of James Talmage's book it says, "For clarity, brevity, and forthrightness, there
is no finer summary statement of the basic beliefs of Latter-day Saints than the Articles of Faith, which
were written by the Prophet Joseph Smith....For more than eighty years this book has been a standard
text for gospel students and teachers alike. The publication of the work preceded Elder Talmage's call
to the apostleship" (Deseret Book Company, Salt Lake City, Utah.1984.

The Publisher's Preface in the book says, "Articles of Faith is considered one of the classics in
Latter-day Saint literature. It is the outgrowth of a series of lectures in theology give by Dr. James E.
Talmage, commencing in October of 1893. At that time Dr. Talmage was serving as the president of
the LDS College in Salt Lake City. The First Presidency of the Church invited Dr. Talmage to prepare a
text for use in Church schools and religion classes...."On December 7, 1911, he was called as a
member of the Quorum of the Twelve Apostles, where he served faithfully until his death on July 27,
1933."

1. "Therefore we know that both the Father and the Son are in form and stature perfect men;
each of them possesses a tangible body, infinitely pure and perfect and attended by
transcendent glory, nevertheless a body of flesh and bones" (page 38).
2. "Jesus Christ is the Son of Elohim both as spiritual and bodily offspring; that is to say,
Elohim is literally the Father of the spirit of Jesus Christ and also of the body in which Jesus
Christ performed His mission in the flesh..." (The Articles of Faith, James Talmage, pp. 466-
467).
3. "Jesus Christ is the Son of Elohim both as spiritual and bodily offspring; that is to say,
Elohim is literally the Father of the spirit of Jesus Christ and also of the body in which Jesus
Christ performed his mission in the flesh" (page 421).
4. "The twofold effect of the atonement is implied in the article of our faith now under
consideration. The first effect is to secure to all mankind alike, exemption from the penalty
of the fall, thus providing a plan of general Salvation. The second effect is to open a way for
Individual Salvation whereby mankind may secure remission of personal sins. As these sins
are the result of individual acts it is just that forgiveness for them should be conditioned on
individual compliance with prescribed requirements--’obedience to the laws and ordinances
of the Gospel’" (pages 78-79).
5. "Hence the justice of the scriptural doctrine that salvation comes to the individual only
through obedience" (page 81).
6. "There are some who have striven to obey all the divine commandments, who have accepted
the testimony of Christ, obeyed ‘the laws and ordinances of the Gospel,’ and received the
Holy Spirit; these are they who have overcome evil by godly works and who are therefore
entitled to the highest glory" (page 83).
7. "The sectarian dogma of justification by faith alone has exercised an influence for evil. The
idea upon which this pernicious doctrine was founded was at first associated with that of an
absolute predestination, by which man was foredoomed to destruction, or to an undeserved
salvation" (page 432.)
8. "...the spirits of mankind passed through a stage of existence prior to their earthly
probation. This antemortal period is oftentimes spoken of as the stage of primeval childhood
or first estate (page 174).
9. "The Church of Jesus Christ of Latter-day Saints proclaims against the incomprehensible
God, devoid of ‘body, parts, or passions,’ as a thing impossible of existence . . . " (page 44).
10. "The opportunity of winning the victor’s reward by overcoming evil was explained to our
parents, and they rejoiced. Adam said: ‘Blessed be the name of God, for because of my
transgression my eyes are opened, and in this life I shall have joy, and again in the flesh I
shall see God.’ Eve was glad and declared: ‘Were it not for our transgression we never
should have had seed, and never should have known good and evil, and the joy of our
redemption, and the eternal life which God giveth unto all the obedient" (page 62).
11. "The redemption of the dead will be effected in accordance with the law of God, which is
written in justice and framed in mercy. It is alike impossible for any spirit, in the flesh or
disembodied, to obtain promise of eternal glory except on condition of obedience to the laws
and ordinances of the Gospel. And, as baptism is essential to the salvation of the living, it is
likewise indispensable to the dead" (page 134-135).
12. "Temples or other sacred places are required for the administration of the ordinances
pertaining to the salvation of the dead, and in certain ordinances for the living" (page 138).
13. "Salvation is attainable only through compliance with the laws and ordinances of the Gospel"
(page 422).
14. "Those who have been born unto God through obedience to the Gospel may by valiant
devotion to righteousness obtain exaltation and even reach the status of godhood" (page.
424).
15. "The preexistent condition is not characteristic of human souls alone; all things of earth have
a spiritual being of which the temporal structure forms but the counterpart" (page 442).
16. "Man in his mortal state is the union of a preexistent spirit with a body composed of earthly
elements. This union of spirit and body marks progress from the unembodied to the
embodied condition..."(page 428),
17. "Jesus Christ is not the Father of the spirits who have taken or yet shall take bodies upon
this earth, for He is one of them....Only such exalted souls have reached maturity in the
appointed course of eternal life; and the spirits born to them in the eternal worlds will pass
in due sequence through the several stages or estates by which the glorified parents have
attained exaltation" (page 426).
Interesting Quotes from Various Mormon Authorities
Bruce McConkie states that the conception and birth of Jesus was completely natural.
"And Christ was born into the world as the literal Son of this Holy Being; he was born in the same
personal, real, and literal sense that any mortal son is born to a mortal father. There is nothing
figurative about his paternity; he was begotten, conceived and born in the normal and natural course
of events,...Christ is the Son of Man, meaning that his Father (the Eternal God!) is a Holy Man."
(Mormon Doctrine, by Bruce McConkie, page 742.) - Note. Bruce McConkie was a member of the first
council of the Seventy, a very high and important position.
"As far as this life is concerned, [Jesus] was born of Mary and of Elohim; he came here as an
offspring of that Holy Man who is literally our Father in heaven. He was born in mortality in the literal
and full sense as the Son of God. He is the Son of his Father in the same sense that all mortals are the
sons and daughters of their fathers" (Bruce McConkie, Mortal Messiah 1:330).
"The Father had a Son, a natural Son, his own literal Seed, the Offspring of his body" (Bruce
McConkie, The Promised Messiah, pg.355).
"There is no need to spiritualize away the plain meaning of the scriptures. There is nothing
figurative or hidden or beyond comprehension in our Lord's coming into mortality. He is the Son of God
in the same sense and way that we are the sons of mortal fathers. It is just that simple" (The
Promised Messiah, pg.468).

Joseph F. Smith says God the Father is the father of Jesus' body.
"Now, we are told in scriptures that Jesus Christ is the only begotten Son of God in the flesh. Well,
now for the benefit of the older ones, how are children begotten? I answer just as Jesus Christ was
begotten of his father ...Jesus is the only person who had our Heavenly Father as the father of his
body" (Family Home Evening Manual, 1972, Joseph F. Smith, pp. 125, 126).
"Christ Not Begotten of Holy Ghost ...Christ was begotten of God. He was not born without the aid
of Man, and that Man was God!" (Doctrines of Salvation, Joseph Fielding Smith, 1954, 1:18).

First Presidency says God is the father of the body of Christ.


"Elohim is literally the Father of the spirit of Jesus Christ and also of the body in which Jesus Christ
performed His mission in the flesh ..." (First Presidency and Council of the Twelve, 1916, God the
Father, compiled by Gordon Allred, pg.150).

Articles of Faith says God is the father of the body of Jesus.


"Jesus Christ is the Son of Elohim both as spiritual and bodily offspring; that is to say, Elohim is
literally the Father of the spirit of Jesus Christ and also of the body in which Jesus Christ performed His
mission in the flesh..." (The Articles of Faith, James Talmage, pp. 466-467).

Bruce McConkie states that God became a God by being saved by obedience to laws
“The Father is a glorified, perfected, resurrected, exalted man who worked out his salvation by
obedience to the same laws he has given to us so that we may do the same.” (McConkie, Bruce R. A
New Witness for the Articles of Faith. Salt Lake City: Deseret Book Company, 1985. p. 64)
“Man and God are of the same race, and it is within the power of righteous man to become like his
Father, that is to become a holy Man, a Man of Holiness.” (Mormon Doctrine, pp. 465-466)
“This Holy Man, the Father of us all, who reigns supreme and is a saved being , ordained and
established a plan of salvation so that his Firstborn and all his spirit children might advance and
progress, become like him, have all power, know all things, live in the family unit, having eternal
increase of their own – or in other words, that they might gain for themselves immortality and eternal
life.” (A New Witness, p. 704)

Heber C. Kimball, who was a member of the First Presidency, stated that Brigham Young
would become President.
"The Church and kingdom to which we belong will become the kingdom of our God and his Christ,
and brother Brigham Young will become President of the United States." (Journal of Discourses, Vol. 5,
page 219).
Heber C. Kimball said the earth conceives and is alive.
"Does the earth conceive? It does and it brings froth. If it did not, why do you go and put your
wheat into the ground? Does it not conceive it?.... Where did the earth come from? From its parent
earths. Well, some of you may call that foolish philosophy. But if it is, I will throw out foolish things,
that you may gather up wise things. The earth is alive. If it was not, it could not produce." (Journal
of Discourses, Vol. 6, page 36).

Orson Pratt (a Mormon apostle in the 1800's) makes a false prophecy.


"God promised in the year 1832 that we should, before the generation then living had passed
away, return and build up the City of Zion in Jackson County....We believe in these promises as much
as we believe in any promise ever uttered by the mouth of Jehovah. The Latter-day Saints just as
much expect to receive a fulfillment of that promise during the generation that was in existence in
1832 as they expect that the sun will rise and set to-morrow. Why? Because God cannot lie. He will
fulfill all His promises. He has spoken, it must come to pass." (Journal of Discourses, Vol. 13, page
362).
He also said, "The fleshly body of Jesus required a Mother as well as a Father. Therefore, the
Father and Mother of Jesus, according to the flesh, must have been associated together in the capacity
of Husband and Wife; hence the Virgin Mary must have been, for the time being, the lawful wife of
God the Father. (Orson Pratt, The Seer, page 158, 1853.) -- Note: the Seer is often regarded by
Mormons as untrustworthy though it was published by Pratt who was a Mormon apostle. Its purpose
was to "Illucidate the doctrines of the Church of Jesus Christ of Latter Day Saints," The Seer, Vol. 1,
Jan. 1853, No. 1, Nevertheless, Brigham Young said in Journal of Discourses, Vol. 11, page 268, "The
man Joseph, the husband of Mary, did not, that we know of, have more than one wife, but Mary the
wife of Joseph had another husband."

John Taylor, the third President of the Mormon Church, said Christianity is of the Devil.
"Myself and hundreds of the Elders around me have seen its [Christianity] pomp, parade, and
glory; and what is it? It is a sounding brass and a tinkling symbol; it is as corrupt as hell; and the
Devil could not invent a better engine to spread his work than the Christianity of the nineteenth
century." (Journal of Discourses, Vol. 6, page 167).

History of the Church states that Joseph Smith fired a gun into the mob that killed him.
"Joseph reached round the door casing, and discharged his six shooter into the passage, some
barrels missing fire....When Hyrum fell, Joseph exclaimed, "Oh dear, brother Hyrum!" and opening the
door a few inches he discharged his six shooter in the stairway (as stated before), two or three barrels
of which missed fire." (History of the Church, Vol. 6, page 618.)

Wilford Woodruff, the fourth President, said God was increasing in knowledge
"If there were a point where a man in his progression could not proceed any further, the very idea
would throw a gloom over every intelligent and reflecting mind. God himself is increasing in
knowledge, power and dominion, and will do so, worlds without end. It is just so with us." (Journal of
Discourses, Vol. 6, page 120).

James Talmage, a Mormon Apostle, said Psalm 82:6 is not about becoming gods.
"In Psalm 82:6, judges invested by divine appointment are called 'gods.' To this scripture the
Savior referred in His reply to the Jews in Solomon's Porch. Judges so authorized officiated as the
representatives of God and are honored by the exalted title 'gods.' Compare the similar appellation
applied to Moses (Exo. 4:16; 7:1). Jesus Christ possessed divine authorization, not through the word
of God transmitted to Him by man, but as an inherent attribute. The inconsistency of calling human
judges 'gods,' and of ascribing blasphemy to the Christ who called Himself the Son of God, would have
been apparent to the Jews but for their sin-darkened minds." (James Talmage, Jesus the Christ, page
501). -- Mormons often quote Psalm 82:6 which Jesus quoted in John 10:30-34 to show that we can
become gods. Rather than them believing the truth from a Christian, perhaps they will believe it from
their own apostle.
Various comments by authorities on the negro race.
Joseph Fielding Smith: "That negro race, for instance, have been placed under restrictions because
of their attitude in the world of spirits, few will doubt. It cannot be looked upon as just that they
should be deprived of the power of the Priesthood without it being a punishment for some act, or acts,
performed before they were born." (The Way to Perfection, page 43.)
"Ham, through Egyptus, continued the curse which was placed upon the seed of Cain. Because of
that curse this dark race was separated and isolated from all the rest of Adam's posterity before the
flood, and since that time the same condition has continued, and they have been 'despised among all
people.' This doctrine did not originate with President Brigham Young but was taught by the Prophet
Joseph Smith .... we all know it is due to his teachings that the negro today is barred from the
Priesthood." (The Way to Perfection, pages 110-111.)
John Taylor: " . . . after the flood we are told that the curse that had been pronounced upon Cain
was continued through Ham's wife, as he had married a wife of that seed. And why did it pass through
the flood? Because it was necessary that the devil should have a representation a upon a the earth as
well as God;.... " (Journal of Discourses, Volume 22, page 304.)
"When he (Satan) destroyed the inhabitants of the antediluvian worlds, he suffered a descendant of
Cain to come through the flood in order that he might be properly represented upon the earth."
(Journal of Discourses, Volume 23, page 336.)
"As a result of his rebellion, Cain was cursed with a dark skin; he became the father of the
Negroes, and those spirits who are not worthy to receive the priesthood are born through his lineage."
(Mormon Doctrine, 1958, page 102.)
Mormonism Unvailed,158 eyewitness testimonies against Joseph Smith
Mormonism Unvailed is a book written in 1834 by E. D. Howe. It was printed and published by the
author. The copyright on it is long expired so I am free to reproduce the information here.
Mormonism Unvailed can be considered the first 'anti-Mormon' work. It is an examination of the
teachings of Mormonism existing at that time and claims to contain eye-witness accounts and sworn
affidavits by those who knew the family of Joseph Smith. If this is true, then we can actually read what
people who knew Smith and his family had to say about their character, manners, disposition, etc.
These written accounts, gathered by a Mr. Philastus Hurlbut and included in the book by Howe, were
taken under oath and before judges and justices of the peace, are legal documents, and worthy of
examination regarding Joseph Smith's character.
Howe said,

We have not only testimony impeaching the moral characters of the Smith family, but we show by the
witnesses, that they told contradictory stories, from time to time, in relation to their finding the plates,
and other circumstances attending it, which go clearly to show that none of them had the fear of God
before their eyes, but were moved and instigated by the devil. (Mormonism Unveiled, p. 232.).

Of course, the Church of Jesus Christ of Latter-day Saints (the Mormons) repudiates the claims of
Howe and the testimonies of the witnesses by stating they are not trustworthy since they had an
agenda hostile to the Smith family and are, therefore, prejudiced. This sword cuts both ways since
that would then invalidate anything from Mormon sources that support the idea that Smith's character
was admirable since they too would be authored by those with an agenda.
Also, it is alleged that statements from Mormon scholars and, they claim non-Mormon scholars,
validate the proposition that Howe himself wrote the affidavits against Smith. The proofs offered from
these scholars are, from what I have seen, opinions dealing with alleged similarities in style of the
testimonies. But, I see no evidence of this as I read the testimonies. Also, note that many of the
testimonies are purported to have been made under oath, before judges, and pastors. It would be up
to the LDS church to prove that these are all lies. They haven't yet, so far as I am aware.
The Mormons must attempt to repudiate the eye-witness accounts against Joseph Smith.
Because, if Smith's character is shown to be untrustworthy, it would cast a huge shadow of doubt over
Mormonism as a whole. The LDS church does not want this.
But, these testimonies are not needed to cast a doubt upon Mormonism's validity. Doubt is
automatically cast when the doctrines of Mormonism come to light and are compared to the Bible. For
example: God used to be a man on another world; he has a goddess wife; they both have bodies;
they produce spirit children; Jesus, Satan, you and I are all brothers and sisters from the spirit realm;
a good temple-worthy Mormon has the hope of becoming a god of his own world with his own wife
with whom he can populate that planet, etc. Such doctrines defy common sense but are gradually
swallowed, bit by bit, by those who do not know God's word, nor possess true spiritual discernment,
and are slowly led astray by the teachers of Mormonism.
I leave these testimonies for your examination.

158
The title of the original book had the spelling of “Mormonism Unvailed” and not “..Unveiled.”
Mormonism Unveiled Defended
The book Mormonism Unvailed (sic) 159 by Eber D. Howe, printed in 1834, contains sworn
testimonies from neighbors and acquaintances of Joseph Smith concerning the dubious character of
him and his family. The testimonies were gathered by a Mr. Philastus Hurlbut and were included in E.
D. Howe's Book. Many of the witnesses deal specifically with Joseph Smith, the founder of Mormonism,
and show him being involved in money digging, divination, and lacking moral integrity.
If these testimonies were true about Joseph Smith, then it would be difficult for Mormons to
continue to proclaim him as a true and virtuous prophet of God. For this reason, many Mormons have
attempted to discredit the accounts contained in Mormonism Unveiled as well as other equally
damaging testimonies by other witnesses concerning Joseph Smith's character. This is to be expected.
Some Mormons have felt that they have an ally in Fawn McKay Brodie. Though Mrs. Brodie's
biography of the Mormon prophet is anything but faith-promoting, she also was suspect of the Hurlbut
testimonies. She says this on page 143 of her book No Man Knows My History: 160

The past that Joseph had hoped to bury in New York now returned to plague him. He had made a
vindictive enemy of Philastus Hurlbut, a handsome, ambitious convert whom he had excommunicated
in Jun 1833 for "unchristian conduct with the ladies." In vengeful mood, Hurlbut began an
investigation of the beginnings of the Mormon Church.

First, I suspect the past that Joseph Smith was trying to bury in New York was his money digging
ventures, being a drunkard, and various schemes to acquire wealth. For these he had become well
known. Nevertheless, if what Mrs. Brodie says is true, that Mr. Hurlbut was excommunicated from the
LDS church, it does not necessarily discredit the information presented by him. After all, the affidavits
are sworn testimonies by eyewitnesses. Mr. Hurlbut is merely the messenger.
Second, to label Mr. Hurlbut as one who is excommunicated is, in my opinion, a type of character
assassination. I have seen this approach used by Mormons many times. In twenty years of dealing
with them, I have spoken with many ex-Mormons who had discovered that Mormonism was false, left
the church, and then spoke out against it. Many have told me how they later discovered that they had
been ex-communicated, in absentia, and were now the victims of vicious rumors that involved
accusations of immorality -- charges they flatly denied. That way, any testimony against the Mormon
Church could be easily dismissed by its members. I am not denying that there are people who truly
deserve excommunication. It is just that I've seen the same accusation falsely raised against ex-
Mormons too many times. Therefore, I am not surprised by the charge against Hurlbut considering the
defensive agenda that Mormons often take.
Third, I would like to add that through word of mouth from Mormon missionaries, from emails, and
on my web site, I too have been accused by Mormons of various things - things I have not done. I can
remember speaking to Mormon missionaries, not letting them know who I was, and listening to them
talk about this guy named "Matt Slick" and saying things about me that were not true. This has
happened more than once. In the case of my web site, I have been accused of lying and great efforts
have been taken by Mormons to attempt to prove this. Why? To discredit the messenger. 161
Again, the Mormons have a strong need to discredit the critics of Mormonism. Character
assassination is the most common and easiest way to accomplish it.

159
"Sic" means the error is in the original quote. In this case, the word "unvailed" is incorrect. It should be spelled
"unveiled." Throughout this paper, I use "unvailed" in reference to the E.D. Howe's book since that is the original
usage.
160
Fawn M. Brodie, No Man Knows My History. The life of Joseph Smith, 2nd ed. New York, Alfred A. Knopf, 1985,
p. 143. It is obvious when reading the context of this quote, that Mrs. Brodie favors Joseph Smith.
161
I would like to add that I have, on occasion, on my web site, attempted to vindicate myself of the false charges
of lying. However, the Mormons on the boards were incessant, accusatory, and quite rude in their attempts to
discredit me. I then decided to not answer them any more.
Richard L. Anderson's Article, Joseph Smith's New York Reputation Reappraised

Richard L. Anderson, a religion professor at Brigham Young University, authored one article in
particular that was written against the Hurlbut Affidavits. It can be found at: Richard L. Anderson ,
Joseph Smith's New York Reputation Reappraised, BYU Studies, Vol. 10, Spring 1970, pages 283-314.
Many Mormons have used his article as a springboard from which to pose rebuttals against the Hurlbut
Affidavits. However, a Mr. Roger Anderson, not to be confused with Richard L. Anderson mentioned
above, wrote a book "Joseph Smith's New York Reputation Reexamined," published in Salt Lake City by
Signature Books in 1990.162
He has taken Richard L. Anderson's article and found many glaring weakness in it. I will use his
comments as a springboard of my own to counter the general claims of those attempting to discredit
Mr. Hurlbut's affidavits. The section by Roger Anderson is more than 30 pages and cannot be
reproduced here.
Nevertheless, Roger Anderson says of Richard L. Anderson's article:

Superior as it is to Nibley's analysis in method and scholarly apparatus, Anderson's article still falls
short on several counts. Its errors may be summarized under three main headings: misrepresentation
of the contents and circumstances surrounding the compilation of the affidavits; failure to consider
alternative interpretations for the evidence; and invalid conclusions based on faulty premises. In
Anderson's analysis these errors recur regularly and sometimes flagrantly. (pages 27-28).

He also says:

Anderson's first charge of substance is that Hurlbut either composed or heavily edited the
depositions he collected. Anderson finds evidence of this contention in the similar structuring of the
affidavits and the use of certain recurring words: "acquainted with," "entitle," "digging for money,"
"addicted to," "lazy," "liar," "intemperate," "pretended," "visionary," "general employment," etc. What
Anderson did not mention is that other statements about Joseph Smith dating from the early 1830s,
statements which Hurlbut did not collect and which are not dependent on him, display many of the
same characteristics. In the Pennsylvania statements made during the same period certain words
recur: "acquainted with," "pretended," "liar," "digging for," "money-diggers." In an 1833 letter written
by Jesse Townsend, minister of Palmyra's Presbyterian church, the following words appear:
"intemperate," "pretended," "digging for money," and "visionary." This letter is similar in structure with
Hurlbut's general Palmyra statement and also with the statement of Parley Chase. 163 The structure and
wording of all of these statements seem to reflect more about the period, geographic location, and
level of education than an undisclosed common authorship. (page 28).

Here we have evidence that similar language used to describe Joseph Smith was used by another
source not even connected with Hurlbut. It seems then, that Mr. Anderson's charge isn't a very good
one.
It is worth noting, that Fawn Brodie makes an identical comment against the Hurlbut Affidavits in
her book, No Man Knows My History:

It can clearly be seen that the affidavits were written by Hurlbut, since the style is the same
throughout. It may be noted also that although five out of the eight had heard Spaulding's story only
once, there was a surprising uniformity in the details they remembered after twenty-two years. Six
recalled the names Nephi, Lamanite, etc.; six held that the manuscript described the Indians as
descendants of the lost ten tribes; four mentioned that the great wars caused the erection of the

162
Signature books, Inc., 564 West 400 North Street, Salt Lake City UT 84116-3411. Note that this is a Mormon
publishing company.
There is also a review of this book in "Dialogue, A Journal of Mormon Thought," (Vol.24, No.2, p.146) by Roger
D. Launius.
163
Townsend's Letter dated 24 Dec. 1833, Originally appeared in Pomeroy Tucker, The Origin, Rise and Progress of
Mormonism. (New York: D. Appleton and Co. 1867), 288-291. As referenced in Roger Anderson's Article in his
footnote #4.
Parley Chase was one of the testifiers against the character of Joseph Smith in the book Mormonism Unvailed.
Indian mounds; and four noted the ancient scriptural style. The very tightness with which Hurlbut here
was implementing his theory rouses an immediate suspicion that he did a little judicious prompting. 164

This comment has already been addressed above. But, it is worth noting that Mr. Roger Anderson
states that the same questions given to different people who knew Joseph Smith can easily result in
common answers. For example, "How long were you acquainted with the Smith family? What was the
general reputation of the Smiths? Was it such as to entitle them to respectability among their
neighbors, or were they addicted to indolence, intemperance, or lying?" 165
There is nothing wrong with asking the same question to different people. It could easily explain the
similarities in answers. This is something that neither Mrs. Brodie nor Richard L. Anderson addressed.
Therefore, the negative opinions of Anderson and Brodie on this issue carry far less weight and amount
to nothing more than what they are: opinions.
What we do have along the lines of hard evidence are written affidavits sworn to and signed by
people who claimed to know Joseph Smith. The documents were attested to be true by the witnesses
and signed off as being true affidavits. In fact, these affidavits were signed in front of Thomas P.
Baldwin a Judge of Wayne County Court in New York; Fred'K. Smith, Justice of the Peace of Wayne
County; Jonathan Lapham, Justice of the peace; Charles Dimon, Justice of the Peace; and, some of the
signers of the testimonies even had people who signed off as character references for them!
(Mormonism Unveiled, page 248).
In Howe's book on pages 261 - 262, are two affidavits dated 12/4/1833 from Palmyra New York
and 11/3/1833 from Manchester, New York. The first states that the Smith family was "destitute of
moral character" among other things. It is then signed by 51 people. The second of the accounts is
much shorter and is signed by 11 people. I reproduce it here:

We, the undersigned, being personally acquainted with the family of Joseph Smith, Sen. with whom
the celebrated Gold Bible, so called, originated, state: that they were not only a lazy indolent set of
men, but also intemperate; and their word was not to be depended upon; and that we are truly glad to
dispense with their society. Parton Butts, Warden A. Reed, Hiram Smith, Alfred Stafford, James Gee,
Abel Chase, A.H. Wentworth, Hoses C. Smith, Joseph Fish, Horace N. Barnes, Silvester Worden.
(Mormonism Unveiled, page 262).

Are we to say that all these people, 66 in total, who signed these documents, were lying or had been
coerced or unduly influenced by Mr. Hurlbut? Do the Mormons actually think that Mr. Hurlbut was of
sufficient deceit and mesmerizing influence that he could get so many people to sign documents that
were not true?
The Mormons often state that the documents were "doctored" by Mr. Hurlbut and are not
trustworthy. If this is so, then where is the proof? I have never read any documentation where the
signers of the affidavits complained that Mr. Hurlbut had misrepresented what they said. There was
ample opportunity for them to complain if things were not accurate. With so many witnesses, surely
some evidence of rebuttal would have surfaced. But none has.
Mormons sometimes state that the affidavits are not trustworthy because Mr. Hurlbut had a
strongly biased agenda against Joseph Smith and that any "anti-Mormon" information produced by him
is automatically suspect. Well, the sword cuts both ways. If Hurlbut cannot be trusted because he had
an agenda to disprove Joseph Smith, then neither can the Mormons who produce positive testimonies
concerning Joseph Smith's character be trusted, since they too had an agenda; namely, to prove the
opposite.

164
Fawn M. Brodie, No man knows my history. The life of Joseph Smith, 2nd ed. New York, Alfred A. Knopf, 1985,
pp. 446-447.
165
Roger Anderson, page 29.
Mr. Howe checked from himself.

Mr. Howe was weary about including the affidavits in his book, Mormonism Unvailed because he
knew that Hurlbut was having legal difficulties with Joseph Smith. Mr. Howe was naturally cautious
about the trustworthiness of the documents. But, it turns out that Mr. Howe actually conducted some
spot checks by interviewing some of the witnesses against Joseph Smith. Consider this comment
written by Mr. Roger Anderson on page 30 of his book:

When Hurlbut submitted his collected statements to newspaper editor Eber D. Howe for publication,
Hurlbut was embroiled in legal difficulties with Joseph Smith which made Howe suspect Hurlbut's
motives. The Mormons were also denouncing Hurlbut's statements as fabrications, a charge which
Howe had no way of controverting without independently verifying Hurlbut's statements. Accordingly
Howe decided upon a "spot check" of Hurlbut's affidavits, hoping thereby to determine their
authenticity without having to reinterview every witness. He first wrote to Isaac Hale and received in
reply a long notarized statement and an affidavit from Hale's son Alva testifying that the notarized
statement was "correct and true." 166 Howe then traveled to Conneaut, Ohio, to see if the statements
Hurlbut had collected there accusing Smith of plagiarism in writing the Book of Mormon were
authentic. While there he "saw most of the witnesses and was satisfied they were not mistaken in their
statements."167 Apparently this was enough to satisfy Howe of the integrity of Hurlbut's reports. He
promptly published them as part of his book, Mormonism Unveiled.

Why is it that the Mormons don't include this information in their rebuttals to the Hurlbut
affidavits? Is it because they don't do sufficient research or is it because they don't like the evidence
and exclude it? This kind of evidence strongly supports the reliability and accuracy of the eyewitness
accounts against the character of Joseph Smith.

Are there other sources that say the same thing? Yes there are.

Hurlbut's affidavits are not the only place where accusations like this have been made. In the
booklet Book of Mormon Authorship: A closer Look, Vernal Holley states that Joseph Smith in 1826
was arrested and stood trial for "glass Looking," a misdemeanor. Smith was allowed "leg bail." Leg bail
meant that he had to leave the area. 168
Holley also quotes A. W. Benton in a letter to the editor of the Evangelical Magazine and Gospel
Advocate, April 9, 1831 which states: "In this town (South Bainbridge, N.Y.), a wealthy farmer, named
Josiah Stowell, together with others, spent large sums of money in digging for hidden money, which
this Smith pretended he could see, and told them where to dig; but they never found their
treasure."169
Nevertheless, below is a chart where I have gone through the affidavits in Howe's Book, Mormonism
Unvailed, and mapped the page numbers where the accusations occur against Joseph Smith. On the
left side of the chart are the names of the people. You will notice that many of the witnesses say the
same thing. You will also notice that I have included other writings by other authors, some are
Mormon, with page numbers under the appropriate category demonstrating that some of the claims of
the eyewitness accounts contained in Howe's book are not solitary.

166
Susquehanna Register, 1 May 1834. As noted in footnote # 8 of Roger Anderson's article on page 30
167
Statement of E. D. Howe, 8 April, 1885, Painesville, Lake County, OH. Original in the Arthur Deming File,
Mormon Collection, Chicago Historical Society. As noted in Roger Anderson's Article, footnote # 9 on page 30.
168
Letter of Judge Joel K. Noble, Turner Collection, Illinois State Historical Library as cited in the booklet on page
48, footnote # 40.
169
As cited in the booklet by Vernal Holley, Book of Mormon Authorship: A Closer Look. Ogden, UT: Zenos
Publications, 1983, page 48, footnote # 41.
Money Buried Stone Gold
Liar Drunkard
Digging Treasure (in Hat) Plates
on
on page(s) page(s) on page(s) on page(s) on page(s) on page(s)
Peter
232
Ingersol
William
237, 239 238, 239 237, 238 239
Stafford
Willard
240 241, 247 242, 243, 245
Chase
Parley
248 248
Chase
David
249 249 249
Stafford
Barton
250
Stafford
Henry
251 251 251 251, 252
Harris
Abigal
253 253
Harris
Lucy
255
Harris
Roswell
257 257
Nichols
Joshua
258
Stafford
Joseph
260 260 259 259 260
Capron
51 signers 261 261
11 signers 262
Isaac
263, 265 263 263, 265 264
Hale
Nethaniel C.
266, 267
Lewis
Joseph
Verse 56
Smith's Story
(it is verse. 34
in Perl of
mentioned)
Great Price
Money Buried Stone Gold
Liar Drunkard
Digging Treasure (in Hat) Plates

on
on page(s) page(s) on page(s) on page(s) on page(s) on page(s)
David
Whitmer, An
12,30,31,
Address to All
37
Believers in
Christ
A Voice of
Warning Or
an
Introduction
to the Faith
and Doctrine 71, 75, 81
Of the Church
of Jesus
Christ of
Latter Day
Saints

Volume 5, page 103


Journal of
and numerous other
Discourses
references therein.

What we can see in this chart is that the eyewitnesses are saying the same things. This is exactly
what you'd expect of real witnesses. Furthermore, other, independent resources also mention items
covered in the Hurlbut affidavits: Gold Plates, putting a stone in a hat to see things in it, and money
digging. The eyewitnesses had personal experience with Joseph Smith so their subjects covered a
wider range whereas the Mormon authors had every opportunity to omit any and all unfavorable
accounts that they knew about.
The point is that the affidavits contain many of the same accounts that are documented in LDS
church writings. Are the eyewitnesses then wrong in some areas but right in others? Or, are their
sworn affidavits before Judges and Justices of the peace indeed accurate?

What is the real issue here?

It seems the real issue behind the Mormon complaints about the affidavits is not concerning their
reliability and accuracy. Remember, these are legal documents. Rather, the Mormon is concerned
with their content. I do not believe the Mormons want to hear about their prophet's sins and shameful
character. They claim a testimony from God that Mormonism is true and, in their eyes, there is no way
that the accusations against Joseph Smith could be valid. Therefore, they automatically assume the
affidavits must be wrong and react and believe accordingly.
But, I believe they are sufficiently accurate to accurately reflect the true nature of Joseph Smith: a
false prophet.
However, it is not the eyewitness accounts that convince me of Joseph Smith's error. It is the word
of God that contradicts his teachings and condemns his doctrines. Joseph Smith isn't wrong because
the eyewitnesses said he is. He is wrong because God's word says he is.
Mormonism Unveiled: Testimonies of Barton Stafford and Henry Harris

THE TESTIMONY OF BARTON STAFFORD

Manchester, Ontario Co., N. Y. Nov. 3d, 1833

Being called upon to give a statement or the character of the family of Joseph Smith, Sen. as far as
I know, I can state that I became acquainted with them in 1820, and knew them until 1831, when
they left this neighborhood.-- Smith, Sen. was a noted drunkard and most of his family followed his
example, and Joseph, Jr. especially, who was very much addicted to intemperance. In short, not one
of the family had the least claims to respectability. Even since he professed to be inspired of the Lord
to translate the Book of Mormon he one day while at work in my father's field, got quite drunk on a
composition of cider, molasses and water. Finding his legs to refuse their office he leaned upon the
fence and hung for sometime; at length recovering again, he fell to scuffling with one of the workmen,
who tore his shirt nearly off from him. His wife who was at our house on a visit, appeared very much
grieved at his conduct, and to protect his back from the rays of the sun, and conceal his nakedness,
threw her shawl over his shoulders and in that plight escorted the Prophet home. As an evidence of his
piety and devotion, when intoxicated, he frequently made his religion the topic of conversation! !

BARTON STAFFORD. State of New York, Wayne County, ss :

I certify that on the 9th day of December 1833, he personally appeared before me, the above
named Barton Stafford, to me known, and solemnly affirmed according to law, to the truth of the
above statement and subscribed the same.

THOS. P. BALDWIN, a Judge of Wayne County Court.

_______________________

I, Henry Harris, do state that I became acquainted with, the family of Joseph Smith, Sen. about the
year 1820, in the town of Manchester, N. York. They were a family that labored vary little--the chief
they did, was to dig for money. Joseph Smith, Jr. the pretended Prophet, used to pretend to tell
fortunes; he had a stone which he used to put in his hat, by means of which he professed to tell
people's fortunes.
Joseph Smith, Jr. Martin Harris and others, used to meet together in private, a while before the
gold plates were found, and were familiarly known by the name of the "Gold Bible Company." They
were regarded by the community in which they lived, as a lying and indolent set of men and no
confidence could be placed in them.
The character of Joseph Smith, Jr. for truth and veracity was such, that I would not believe him
under oath. I was once on a jury before a Justice's Court and the Jury could not, and did not, believe
his testimony to be true. After he pretended to have found the gold plates, I had a conversation with
him, and asked him where he found them and how he come to know where they were. He said he had
a revelation from God that told him they were hid in a certain hill and he looked in his stone and saw
them in the place of deposit; that an angel appeared, and told him he could not get the plates until he
was married, and that when he saw the woman that was to he his wife, he should know her; and she
would know him. He then went to Pennsylvania, got his wife, and they both went together and got the
gold plates -He said it was revealed to him, that no one must see the plates but himself and wife.
I then asked him what letters were engraved on them, he said italic letters written in an unknown
language, and that he had copied some of the words and sent them to Dr. Mitchell and Professor
Anthon of New York. By looking on the plates he said he could not understand the words, but it was
made known to him that he was the person that must translate them, and on looking through the
stone was enabled in translate.
After the Book was published, I frequently bantered him for a copy. He asked fourteen shillings a
piece for them; I told him I would not give so much; he told me had had a revelation that they must
be sold at that price.
Sometime afterwards I talked with Martin Harris about buying one or the Books and he told me
they had had a new revelation, that they might be sold at ten shillings a piece.

State of Ohio, Cuyahoga County, ss :

Personally appeared before me, Henry Harris, and made oath in due form of law, that the foregoing
statements subscribed by him are true.

JONATHAN LAPHAM, Justice of the peace.


Mormonism Unveiled: Testimonies of Abigail Harris and Lucy Harris
(Martin Harris' wife)
Palmyra, Wayne Co. N. Y. 11th mo. 28 th, 1833.

In the early part of the winter in 1828, I made a visit to Martin Harris and was joined in company
by Jos. Smith, sen. and his wife. The Gold Bible business, so called, was the topic of conversation, to
which I paid particular attention that I might learn the truth of the whole matter.--They told me that
the report that Joseph, jun. had found golden plates, was true, and that he was in Harmony, Pa.
translating them--that such plates were in existence, and that Joseph, jun. was to obtain them, was
revealed to him by the spirit of one or the Saints that was on this continent, previous to its being
discovered by Columbus. Old Mrs. Smith observed that she thought he must be a Quaker, as he was
dressed very plain. They said that the plates he then had in possession were but an introduction to the
Gold Bible--that all or them upon which the bible was written, were so heavy that it would take four
stout men to load them into a cart--that Joseph had also discovered by looking through his stone, the
vessel in which the gold was melted from which the plates were made, and also the machine with
which they were rolled; he also discovered in the bottom of the vessel three balls of gold, each as
large as his fist. The old lady said also, that after the book was translated, the plates were to be
publicly exhibited--admittance 0-5 cents. She calculated it would bring in annually an enormous sum
of money-that money would then be very plenty, and the book would also sell for a great price, as it
was something entirely new-that they bad been commanded to obtain all the money they could borrow
for present necessity, and to repay with gold. The remainder was to be kept in store for the benefit of
their family and children. This and the like conversation detained me until about 11 o'clock. Early the
next morning, the mystery of the Spirit being like myself (one of the order called Friends) was reveal
by the following circumstance: The old lady took me into another room, and after closing the door, she
said, "have you four or five dollars in money that you can lend until our business is brought to a close?
the spirit has said you shall receive four fold." I told her that when I gave, I did it not expecting to
receive again--as for money I had none to lend. I then asked her what her particular want of money
was; to which she replied "Joseph wants to take the stage and come home from Pennsylvania to see
what we are all about." To which I replied, he might look in his stone and save his time and money.
The old lady seemed confused, and left the room, and thus ended the visit.
In the second month following, Martin Harris and his wife were (at my house. In conversation
about Mormonites she observed, that she wished her husband would quit them as she believed it was
all false and a delusion. To which I heard Mr. Harris reply : "What if it is a lie ; if you will let me alone I
will make money out of it ! I was both an eye and an ear witness of what has been stated above,
which is now fresh in my memory, and I give it to the world for the good of mankind. I speak the truth
and lie not, God bearing me is witness. ABIGAIL HARRIS

___________________________

Lucy Harris, wife of Martin Harris

Palmyra, Nov. 29, 1833

Being called upon to give a statement to the world of what I know respecting the (Gold Bible
-speculation, and also or the conduct of Martin Harris, my husband, who is a leading character among
the Mormons, I do it free from prejudice realizing that I must give an account at the bar of God for
what I say. Martin Harris was once industrious attentive to his domestic concerns, and thought to be
worth about ten thousand dollars. He is naturally quick in his temper and in his mad-fits frequently
abuses all who may dare to oppose him in his wishes. However strange it may seem, I have been a
great sufferer by his unreasonable conduct. At different times while I lived with him, he has whipped,
kicked, and turned me out of the house. About a year previous to the report being raised that Smith
had found gold plates, he became very intimate with the Smith family, and said he believed Joseph
could see in his stone any thing he wished. After this he apparently became very sanguine in his belief,
and frequently said he would have no one in his house that did not believe in Mormonism, and because
I would not give credit to the report he made about the gold plates, he became more austere towards
me. In one of his fits of rage he struck me with the but end or a whip, which I think had been used for
driving oxen, and was about the size or my thumb, and three or four feet long. He beat me on the
head four or five times, and the next day turned me out of doors twice, and beat me in a shameful
manner. The next day I went to the town or Marion, and while there my flesh was black and blue in
many places. His main complaint against me was, that I was always trying to hinder his making
money.
When he found out that I was going to Mr. Putnam's, in Marion, he said he was going too, that they
had sent for him to pay them a visit. On arriving at Mr. Putnam's, I asked them if they had sent for Mr.
Harris; they replied, they knew nothing about it; he, however, came in the evening. Mrs. Putnam told
him never to strike or abuse me any more; he then denied ever striking me; she was however
convinced that he lied, as the marks of his beating me were plain to be seen, and remained more than
two weeks. Whether the Mormon religion be true or false, I leave the world to judge, for its effects
upon Martin Harris have been to make him more cross, turbulent and abusive to me. His whole object
was to make money by it. I will have one circumstance in proof of it. One day, while at Peter Harris
house, I told him he had better leave the company of the Smiths, as their religion was false; to which
he replied, if you would let me alone, I could make money by it.
It is in vain for the Mormons to deny these facts; for they are all well known to most of his former
neighbors. The man has now become rather an object of pity; he has spent most of his property, and
lost the confidence of his former friends. If he had labored as hard on his farm as he has to make
Mormons, he might now be one of the wealthiest farmers in the country. He now spends his time in
traveling through the country spreading the delusion of Mormonism, and has no regard whatever for
his family.
With regard to Mr. Harris being intimate with Mrs.. Haggard, as has been reported, it is but justice
to myself to state what facts have come within my own observation, to show whether I had any
grounds for jealousy or not. Mr. Harris was very intimate with this family, for some time previous to
their going to Ohio. They lived a while in a house which he had built for their accommodation, and here
he spent the most of his leisure hours ; and made her presents of articles from the store and house.
He carried these presents in a private manner, and frequently when he went there, he would pretend
to be going to some of the neighbors, on an errand, or to be going into the fields. After getting out of
sight of the house, he would steer a straight course for Haggard's house, especially if Haggard was
from home. At times when Haggard was from home, he would go there in the manner above
described, and stay till twelve or one o'clock at night, and sometimes until day light.
If his intentions were evil, the Lord will judge him accordingly but if good, he did not mean to let
his left hand, know what his right hand did. The above statement or facts, I affirm to be true.

LUCY HARRIS.
Mormonism Unveiled, Testimony of Artemas Cunningham
Artemas Cunningham, of Perry, Genuga county, states as follow:

"In the month of October, 1811, I went from the township of Madison to Conneaut, for the purpose
of securing a debt due me from Solomon Spalding. I tarried with him nearly two days, for the purpose
of accomplishing my object, which I was finally unable to do. I found him destitute of the means of
paying his debts. His only hope of ever paying his debts, appeared to bear upon the sale of a book,
which he had been writing. He endeavored to convince me from the nature and character of the work,
that it would meet with a ready sale. Before showing me his manuscripts, he went into a verbal
relation of its outlines, saying that it was a fabulous or romantic history of the first settlement of this
country, and as it purported to have been a record found buried in the earth, or in a cave, he had
adopted the ancient or scripture style of writing. He then presented his manuscripts, when we sat
down and spent a good share of the night, in reading them, and conversing upon them. I well
remember the name of Nephi, which appeared to be the principal hero of the story. The frequent
repetition of the phrase, "I Nephi," I recollect as distinctly as though it was but yesterday, although the
general features of the story have passed from my memory, through the lapse of 22 years. He
attempted to account for the numerous antiquities which are found upon this continent, and remarked
that, after this generation had passed away, his account of the first inhabitants of America would be
considered as authentic as any other history. The Mormon Bible I have partially examined, and am
fully of the opinion that Solomon Spalding had written its outlines before he left Conneaut."
Statements of the same import, might be multiplied to an indefinite length; but we deem it
unnecessary. We are here willing to rest the question, in the hands of any intelligent jury, with a
certainty that their verdict would be, that Solomon Spalding first wrote the leading incidents of the
Book of Mormon, instead of its being found by the Smith family, while digging for gold, and its
contents afterwards made known by the Supreme Being.
But our inquiries did not terminate here. Our next object was to ascertain, if possible, the
disposition Spalding made of his manuscripts. For this purpose, a messenger was dispatched to look
up the widow of Spalding, who was found residing in Massachusetts. From her we learned that
Spalding resided in Pittsburgh, about two years, when he removed to the township of Amity,
Washington Co. Pa. Where he lived about two years, and died in 1816. His widow then removed to
Onondaga county, N.Y., married again, and lived in Otsego county, and subsequently removed to
Massachusetts. She states that Spalding had a great variety of manuscripts, and recollects that one
was entitled the "Manuscript Found," but of its contents she has now no distinct knowledge. While they
lived in Pittsburgh, she thinks it was once taken to the printing office of Patterson & Lambdin; but
whether it was ever brought back to the house again, she is quite uncertain: if it was, however, it was
then with his other writings, in a trunk which she had left in Otsego county, N.Y. This is all the
information that could be obtained from her, except that Mr. Spalding, while living, entertained a
strong antipathy to the Masonic Institution, which may account for its being so frequently mentioned in
the Book of Mormon. The fact also, that Spalding, in the latter part of his life, inclined to infidelity, is
established by a letter in his hand-writing, now in our possession.
The trunk referred to by the widow, was subsequently examined, and found to contain only a single
M.S. book, in Spalding's hand-writing, containing about one quire of paper. This is a romance,
purporting to have been translated from the Latin, found on 24 rolls of parchment in a cave, on the
banks of the Conneaut Creek, but written in modern style, and giving a fabulous account of a ship's
being driven upon the American coast, while proceeding from Rome to Britain, a short time previous to
the Christian era, this country then being inhabited by the Indians. This old M. S. has been shown to
several of the foregoing witnesses, who recognize it as Spalding's, he having told them that he had
altered his first plan of writing, by going farther back with dates, and writing in the old scripture style,
in order that I might appear more ancient. They say that it bears no resemblance to the "Manuscript
Found."
Here, then, our esquires after facts partially cease, on this subject. We have fully shown that the
Book of Mormon is the joint production of Solomon Spalding and some other designing knave, or if it is
what it purports to be, the Lord God has graciously condescended, in revealing to Smith his will,
through spectacles, to place before him nd appropriate to his own use, the writings and names of men
which had been invented by a person long before in the grave. Having established the fact, therefore,
that most of the names and leading incidents contained in the Mormon bible, originated with Solomon
Spalding, it is not very material, as we conceive, to show the way and manner by which they fell into
the hands of the Smith family. To do this, however, we have made some inquiries.
It was inferred at once that some light might be shed upon this subject, and the mystery revealed,
by applying to Patterson & Lambdin, in Pittsburgh. But here again death had interposed a barrier. That
establishment was dissolved and broken up many years since, and Lambdin died about eight years
ago. Mr. Patterson says he has no recollection of many such manuscript being brought there for
publication, neither would he have been likely to have seen it, as the business of printing was
conducted wholly by Lambdin at that time. He says, however, that many M. S. books and pamphlets
were brought to the office about that time, which remained upon their shelves for years, without being
printed or even examined. Now, as Spalding's book can no where be found, or any thing heard of it
after being carried to this establishment, there is the strongest presumption that it remained there in
seclusion, till about the year 1823 or 24, at which time Sidney Rigdon located himself in that city. We
have been credibly informed that he was on terms of intimacy with Lambdin, being seen frequently in
his shop. Rigdon resided in Pittsburgh about three years, and during the whole of that time, as he has
since frequently asserted, abandoned preaching and all other employment, for the purpose of studying
the bible. He resided in this vicinity about four years previous to the appearance of the book, during
which time he made several long visits to Pittsburgh, and perhaps to the Susquehannah, where Smith
was then digging for money, or pretending to be translating plates. It may be observed also, that
about the time Rigdon left Pittsburgh, the Smith family began to tell about finding a book that would
contain a history of the first inhabitants of America, and that two years elapsed before they finally got
possession of it.
We are, then, irresistibly led to this conclusion: --that Lambdin, after having failed in business, had
recourse to the old manuscripts then in his possession, in order to raise the wind, by a book
speculation, and place the "Manuscript Found," of Spalding, into the hands of Ridgon, to be
embellished, altered, and added to, as he might think expedient; and three years' study of the bible
we should deem little time enough to garble it, as it is transferred to the Mormon book. The former
dying, left the latter the sole proprietor, who was obliged to resort to his wits, and in a miraculous way
to bring it before the world; for in no other manner could such a book be published without great
sacrifice. And where could a more suitable character be found that Jo Smith, who's necromantic fame
and arts of deception, had already extended to a considerable distance? That Lambdin was a person
every way qualified and fitted for such an enterprise, we have the testimony of his partner in business,
and others of his acquaintance. Add to all these circumstances, the facts, that Rigdon had prepared
the minds in a great measure, of nearly a hundred of those who had attended his ministration to be in
readiness to embrace the first mysterious ism that should be presented -- the appearance of Cowdery
at his residence as soon as the Book was printed -- his immediately repairing to the residence of
Smith, 300 miles distant, where he was forthwith appointed an elder, high priest, and a scribe to the
prophet--the pretended vision that his residence in Ohio was the "promised land,"--the immediate
removal of the whole Smith family thither, where they were soon raised from a state of poverty to
comparative affluence. We therefore, must hold out Sidney Rigdon to the world as being the original
"author and proprietor" of the whole Mormon conspiracy, until further light is elicited upon the lost
wrings of Solomon Spalding.
Mormonism Unveiled, Testimonies of Nahum Howard and Oliver Smith

Nahum Howard
Conneaut, August, 1833
I first became acquainted with Solomon Spalding, in Dec, 1810. After that time I frequently saw
him at his house, and also at my house. I once in conversation with him expressed a surprise at not
having any account of the inhabitance once in this country, who erected the old forts, mounds, &c. He
then told me that he was writing a history of that race of people; and afterwards frequently showed
me his writings, which I read. I have lately read the Book of Mormon, and believe it to be the same as
Spalding wrote, except the religious part. He told me that he intended to get his writings published in
Pittsburgh, and he thought that in one century from that time, it would believed as much as any other
history.

----------------------------------------

Oliver Smith
Conneaut, August, 1833
When Solomon Spalding first came to this place, he purchased a tract of land, surveyed it out and
commenced selling it. While engaged in this business, he boarded at my house, in all nearly six
months. All his leisure hours were occupied in writing a historical novel, founded upon the first settlers
of this country. He said he intended to trace their journey from Jerusalem, by land and sea, till their
arrival in America, give an account of their arts, sciences, civilization, wars and contentions. In this
way, he would give a satisfactory account of all the old mounds, so common to this country. During
the time he was at my house, I read and heard read one hundred pages or more. Nephi and Lehi were
by him represented as leading characters, when they first started for America. Their main object was
to escape the judgments which they supposed were coming upon the old world. But no religious
matter was introduced, as I now recollect. Just before he left this place, Spalding sent for me to call on
him, which I did. -- He then said, that although he was in my debt, he intended to leave the country,
and hoped I would not prevent him, for, says he, you know I have been writing the history of the first
settlement of America, and I intend to go to Pittsburgh, and there live a retired life, till I have
completed the work, which will enable me to return and pay off all my debts--the book, you know will
sell, as every one is anxious to learn something upon that subject. This was the last I heard of
Spalding or his book, until the Book of Mormon came into the neighborhood. When I heard the
historical part of it related, I at once said it was the writings of old Solomon Spalding. Soon after, I
obtained the book, and on reading it, found much of it the same as Spalding had written, more than
twenty years before.
Mormonism Unveiled, Testimony of John N. Miller

John N. Miller
Springfield, Pa. September, 1833.

In the year 1811, I was in the employ of Henry Lake and Solomon Spalding, at Conneaut, engaged
in rebuilding a forge. While there, I boarded and lodged in the family of said Spalding, for several
months. I was soon introduced to the manuscript of Spalding, and perused them as often as I had
leisure. He had written two or three books or pamphlets on different subjects; but that which more
particularly drew my attention, was one which he called the "Manuscript Found." From this he would
frequently read some humorous passages to the company present. It purported to be the history of
the first settlement of America, before discovered by Columbus. He brought them off from Jerusalem,
under their leaders; detailing their travels by land and water, their manners, customs, laws, wars, &c.
He said that he designed it as a historical novel, and that in after years it would be believed by many
people as much as the history of England. He soon after failed in business, and told me he should
retire from the din of his creditors, finish his book and have it published, which would enable him to
pay his debts and support his family. He soon after removed to Pittsburgh, as I understand.
I have recently examined the Book of Mormon, and find in it the writings of Solomon Spalding,
from beginning to end, but mixed up with scripture and other religious matter, which I did not meet
with in the "Manuscript Found." Many of the passages in the Mormon Book are verbatim from
Spalding, and others in part. The names of Nephi, Lehi, Moroni, and in fact all the principal names, are
bro't fresh to my recollection, by the Gold Bible. When Spalding divested his history of its fabulous
names, by a verbal explanation, he landed his people near the Straits of Darien, which I am very
confident he called Zarahemla, they were marched about that country for a length of time, in which
wars and great blood shed ensured, he brought them across North America in a north east direction.
Mormonism Unveiled, Testimonies of Roswell Nichols & Joshua Stafford
Manchester, Ontario County, Dec. 1st, 1833

I, Roswell Nichols, first became acquainted with the family of Joseph Smith, Sen. nearly five years
ago, and I lived a neighbor to the said family about two years. My acquaintance with the family has
enabled me to know something of its character for good citizenship, probity and veracity -- for breach
of contracts, for the nonpayment of debts and borrowed money, and for duplicity with their neighbors,
the family was notorious. Once, since the Gold Bible speculation commenced, the old man was sued;
and while the sheriff was at his house, he lied to him and was detected in the the falsehood. Before he
left the house, he confessed that it was sometimes necessary for him to tell an honest lie, in order to
live. At another time, he told me that he had received an express command for me to repent and
believe as he did, or I must be damned. I refused to comply, and at the same time told him of the
various impositions of his family. He then stated that their digging was not for money but it was for
the obtaining the Gold Bible. Thus contradicting what he had told me before: for he had often said,
that the hills in our neighborhood were nearly all erected by human hands -- that they were all full of
gold and silver. And one time, when we were talking on the subject, he pointed to a small hill on my
farm, and said, "in that hill there is a stone which is full of gold and silver. I know it to be so, for I
have been to the hole, and God said unto me, do not it now, but at a future day you shall go find the
book opened, and then you shall have the treasures." He said that gold and silver was once as plenty
as the stones in the field are now-- that the ancients, half of them melted the ore and made the gold
and silver, while the other half a read it deeper in the earth, which accounted for disease hills. Upon
my inquiring who furnished the food for the whole, he flew into a passion, and called me case in their,
and said he, "you must be eternally damned."
I mentioned these facts, not because of their intrinsic importance, but simply to show the weak
mindedness and low character of the man. ROSWELL NICHOLS

______________________

Manchester, Ontario County, Nov. 15th, 1833.

I, Joshua Stafford, became acquainted with the family of Joseph Smith, Sen. about the year 1819
or 20. They then were laboring people, in low circumstances. A short time after this, they commenced
digging for hidden treasures, and soon after they became indolent, and told marvelous stories about
ghosts, hob-goblins, caverns, and various other mysterious matters. Joseph once showed me a piece
of wood which he said he took from a box of money, and the reason he gave up for not obtaining the
box, was, that it moved. At another time, he, (Joseph, Jr.) at a husking, called on me to become a
security for a horse, and said he would reward me handsomely, for he had found a box of watches,
and they were as large as his fist, and he put one of them to his ear, and he could hear it to "tick forty
rods." Since he could not dispose of them profitably at Canandaigua or Palmyra, he wished to go east
with them. He said if he did not return with the horse, I might take his life. I replied, that he knew I
would not do that. Well, said he, I did not suppose you would, yet I would be willing that you should.
He was nearly intoxicated at the time of the above conversation. JOSHUA STAFFORD
Atheism
Introduction

Atheism denies God’s existence. It is a growing movement in the world. Atheists repeatedly attack
Christianity. But, is it an intellectually viable system of thought? Does it have merit? Can it be
opposed successfully?

Check out this section and see.

1. What are some of the definitions of atheism? p. 2


2. What is deism? p. 6
3. What are some mistakes Christians make when dialoguing with atheists? p. 9
4. Why does do atheists need to attack Christianity? p. 12
5. Is “I lack belief in a god” a truly defensible position? p. 29
6. What are some of the problems with saying there is no God? p. 50
7. What can you say to “There is no sufficient evidence for God's existence.” p. 51
8. Can God make a rock so big He can't pick it up? p. 58
9. What is the problem with saying there is no proof that God exists? p. 61
10. How do you respond to, "If everything needs a creator, then who or what created God?" p. 70
11. Why believe in Christianity over all other religions? p. 73
12. How can entropy and causality be used as a proof for God's existence? p. 76
13. Can an atheist account for the laws of logic? pp. 79-80
Atheism
The word ‘atheism’ comes from the negative ‘a’ which means ‘no’ and ‘theos’ which means ‘god.’
Hence, atheism in the basest terms means ‘no god.’ Basically, atheism is the lack of belief in a god
and/or the belief that there is no god. By contrast, theism is the belief that there is a God and that He
is knowable. I need to mention that most atheists do not consider themselves anti-theists. Most
consider themselves as non-theists.
I've encountered many atheists who claim that atheism is not a belief system while others say it
is. Since there is no official atheist organization, nailing down which definition of atheism to use can
be difficult. Following are some definitions offered by atheists.

• "An atheist is someone who believes and/or knows there is no god."


• "An atheist lacks belief in a god."
• "An atheist exercises no faith in the concept of god at all."
• "An atheist is someone who is free from religious oppression and bigotry."
• "An atheist is someone who is a free-thinker, free from religion and its ideas."

Which ever definition you go by, atheism denies God.


There are two main categories of atheists: strong and weak, with variations in between. A strong
atheist actively believes and states that no God exists. They expressly denounce the Christian God
along with any other god. Strong atheists are usually more aggressive in their conversations with
theists and try to shoot holes in theistic beliefs. They like to use logic and anti-biblical evidences to
denounce God's existence.
Agnostic Atheists, as I call them, are those who deny God's existence based on an examination of
evidence. Agnosticism means 'not knowing,' or 'no knowledge.' I call them agnostic because they
state they have looked at the evidence and have concluded that there is no God. But, the interesting
thing with them is that they say they are open further evidence for God's existence.
Weak atheists simply exercise no faith in God. The weak atheist might be better explained as a
person who lacks belief in God the way a person might lack belief that there is a green lizard in a
rocking chair on the moon; the subject simply isn't an issue and they don't believe or not believe it.
Finally, there is a group of atheists that I call militant atheists. They are, fortunately, few in
number. They are usually highly insulting and profoundly terse in their comments to theists,
particularly Christians. I’ve encountered a few of them and they are vile, rude, and highly
condescending. Their language is full of insults, profanity, and blasphemies. Basically, no meaningful
conversation can be had with them at all.

Two Main Types of Arguments from Atheists

Atheist positions seem to fall into two main categories. The first is the lack of evidence category
where the atheist asserts that the supporting evidence isn't good enough for him to affirm God's
existence. The second is the category where they believe that the idea of God existing is illogical and
contrary to the evidence at hand. To simplify, one says there isn't enough evidence to decide and the
other says there is evidence contrary to God's existence. For those atheists who simply lack belief and
exercise no energy in the discussion, neither category applies because are not involved in the debate.
A typical argument posed by an atheist to show why God does not exist is as follows: God is
supposed to be all good and all powerful. Evil and suffering exist in the world. If God is all good he
would not want evil and suffering to exist. If He is all powerful then He is able to remove all evil and
suffering. Since evil and suffering exist, God is either not all good (which means he is not perfect and
not God), or he is not all powerful (and limited in abilities and scope). Since either case shows God is
not all good and powerful, then He does not exist.
Some Basic Tenets of Atheism

Presuppositions are important to us all. We look at the world through them. The atheist has a set
of presuppositions, too. Though there is no definitive atheist organization that defines the absolutes of
atheism, there are basic principles that atheists, as a whole, tend to adopt. They are listed below.
Please note however, that not all atheists assert all of these tenets. The only absolute common one
they hold to is that they do not believe in a God or gods.

1. There is no God or devil. 5. Generally, the universe is materialistic and


2. There is no supernatural realm. measurable.
3. Miracles cannot occur. 6. Man is material.
7. Generally, evolution is considered a
4. There is no such thing as sin as a violation scientific fact.
of God's will.
8. Ethics and morals are relative

For the Christian, atheism clashes with many aspects of our faith. Some atheists openly attack
Christianity citing apparent contradictions in the Bible, perceived philosophical difficulties related to
God, and what they consider as logical evidences against God's existence. But the atheists' criticisms
are not without answers. Hopefully, this information will help answer some of their claims and give
reasons for believing in God.
Terms and Definitions

1. A priori - Knowledge, judgments, and principles which are true without verification or testing. It
is universally true.
2. Agnosticism - The belief that the existence of God is not knowable. The word is derived from the
negative ‘a’ combined with the Greek word ‘gnosis’ which means ‘knowledge.’ Hence, agnosticism
is the belief that God cannot be known.
3. Argumentum ad hominem - An irrelevant attack upon a person to deflect the argument from
the facts and reasons.
4. Argumentum ad populum - An argument where appeal is made to emotions: loyalties,
patriotism, prejudices, etc.
5. Argumentum ad verecundiam - An argument using respect for great men, customs,
institutions, and authority in an attempt to strengthen one's argument and provide an illusion of
proof.
6. Atheism - The lack of belief in a god and/or the belief that there is no god. The position held by
a person or persons that ‘lack belief’ in god(s) and/or deny that god(s) exist.
7. Autonomy - Freedom from all external constraints. Independence consisting of self-
determination.
8. Causality - The relationship between cause and effect. The principle that all events have
sufficient causes.
9. Chance - Being undetermined. Events without apparent cause. An accidental happening.
10. Choice - Action based on one's volition, will, desire.
11. Christian - A person who believes in biblical person of Jesus who claimed to be God in flesh,
died, and rose again from the grave and who lives according to the principles of Christ’s teaching.
12. Cosmological argument - An attempt to prove that God exists by appealing to the principle that
all things have causes. There cannot be an infinite regress of causes, therefore, there must be an
uncaused cause: God.
13. Cosmology - Study of the origin and structure of the universe.
14. Deduction - A system of logic, inference and conclusion drawn from examination of facts.
Conclusions drawn from the general down to the specific.
15. Deism - The belief that there is a God but that God is not involved in the world. Deism denies
any revelatory work of God in the world whether it be by miracles or by scripture.
16. Deontology - The study of moral obligation.
17. Determinism - The teaching that every event in the universe is caused and controlled by natural
law.
18. Dialectic - The practice of examining ideas and beliefs using reason and logic. It is often
accomplished by question and answer.
19. Didactic - The branch of education dealing with teaching.
20. Dogma - A generally held set of formulated beliefs.
21. Empiricism - The proposition that the only source of true knowledge is experience. Search for
knowledge through experiment and observation. Denial that knowledge can be obtained a priori.
22. Epistemology - The branch of philosophy that deals with knowing and the methods of obtaining
knowledge.
23. Ethics - Study of right and wrong and wrong, good and bad, moral judgment, etc.
24. Evolution - Change from simple to complex. That system of study authored by Charles Darwin
that seeks to explain the development of life.
25. Fact - An indisputable truth.
26. Faith - Acceptance of ideals, beliefs, etc., which are not necessarily demonstrable through
experimentation or reason.
27. Free will - Freedom of self determination and action independent of external causes.
28. Freethinker - A person who forms his opinions about religion and God without regard to
revelation, scripture, tradition, or experience.
29. God - Deity, infinite being of power, influence, knowledge, and immortality.
30. Hedonism - That pleasure is the principle good and proper goal of all action. Self indulgence.
31. Humanism - The system of philosophy based upon human reason, actions, and motives without
concern of deity or supernatural phenomena.
32. In facto - Something that exists and is complete.
33. In fieri - Beginning to be, but not yet complete.
34. Induction - A system of logic where specific facts are used to draw a general conclusion.
35. Infidel - A person who does not believe in any particular religious system.
36. Karma - In Hinduism, the total compilation of all a person's past lives and actions that result in
the present condition of that person.
37. Metaphysics - The study of the nature and being of reality and its origin and structure.
38. Monism - The view that there is only one basic and fundamental reality, that all existence is this
one reality.
39. Monolatry - The belief that there are many gods but only one of them is served and worshipped.
40. Monotheism - The belief that there is only one God in the universe.
41. Morals - Ethics, the codes, values, principles, and customs of a person or society.
42. Myth - Something not true, fiction, or falsehood. A truth disguised and distorted.
43. Ontological argument - An attempt at proving the existence of God by stating that God exists
because our conception of Him exists and nothing greater than God can be conceived of.
44. Ontology - The study of the nature of being, reality, and substance.
45. Panentheism - The belief that God is in the universe. It differs with pantheism which states that
God is the universe and all that it comprises.
46. Pantheism - The belief that God is the universe and all that comprises it: laws, motion, matter,
energy, consciousness, life, etc. It denies that God is a person and is self aware.
47. Philosophy - The study of seeking knowledge and wisdom in understanding the nature of the
universe, man, ethics, art, love, purpose, etc.
48. Polytheism - The belief that there are many gods in existence in the universe.
49. Pragmatism - A method in philosophy where value is determined by practical results.
50. Rationalism - A branch of philosophy where truth is determined by reason.
51. Relativism - The view that truth is relative and not absolute. It varies from people to people,
time to time.
52. Religion - Generally a belief in a deity and practice of worship, action, and/or thought related to
that deity. Loosely, any specific system of code of ethics, values, and belief.
53. Teleological argument - An attempted proof of God's existence based upon the premise that
the universe is designed and therefore needs a designer: God.
54. Teleology - The study of final causes, results. Having a definite purpose, goal, or design.
55. Theism - The belief that there is a God and that He is knowable and involved in the world.
56. Theodicy - The study of the problem of evil in the world in relation to the proposition that there
is an all powerful good God.
57. Theology - The study of things pertaining to God and/or the relation of God to the world.
58. Transcendent - That which is beyond our senses and experience. Existing apart from matter.
59. Trinity - The Christian doctrine that there is only one God in existence and that He consists of
three separate and ontologically divine persons.
60. Yin and Yang - A dualistic philosophy of passive and active, good and bad, light and dark,
positive and negative, male and female, etc., and that they are in opposition, each is part of the
whole and works together.

____________
Resources:

• Baker's Dictionary of Theology, Edited by Everett F. Harrison, Baker Book House, Grand Rapids,
Michigan, 1960.
• Christian Apologetics, by Norman Geisler, Baker Book House, Grand Rapids, Michigan, 1976
• Dictionary of Philosophy, edited by Dagobert D. Runes, Ph.D., Philosophical Library, New York,
1942.
• Webster's New World Dictionary, edited by David B. Guralnik, Simon and Schuster, 1986
Can Atheists be ethical?

The answer to this question is a definite, "Yes." Atheists are people who, whether they like it or
not, have the law of God written on their hearts (Rom. 2:15). They are subject to the same laws of our
country (and other countries). They have a sense of right and wrong. They must work with people and
being unethical in society would not serve them very well. It is practical and logical for an atheist to be
ethical and work within the norms of social behavior. Atheists, generally, are honest, hardworking
people.
Nevertheless, some Christians raise the question, "What is to prevent an atheist from murdering
and stealing? After all, they have no fear of God and no absolute moral code." The answer is simple:
Atheists are capable of governing the own moral behavior and getting along in society the same as
anyone else.
At the risk of labeling the atheist as self-centered, it does not serve the best interests of an atheist
to murder and steal. It would not take long before he was imprisoned and/or killed for his actions.
Basically, society will only put up with so much if it is to function smoothly. So, if an atheist wants to
get along and have a nice life, murdering and stealing won't accomplish it. It makes sense for him to
be honest, work hard, pay his bills, and get along with others. Basically, he has to adopt a set of ethics
common to society in order to do that. Belief in God is not a requirement for ethical behavior or an
enjoyable life.

On the other hand

Atheists' morals are not absolute. They do not have a set of moral laws from an absolute God by
which right and wrong are judged. But, they do have a legal system with a codified set of moral laws.
This would be the closest thing to moral absolutes for atheists. However, since the legal system
changes (slavery was legal 200 years ago but is not now), the morals in a society can still change. At
best, these codified morals are "temporary absolutes." This can be a problem as the norms of society
shift and the ethics shift with them. In one century abortion is wrong. In another, it is right. Well, is it
or isn't it right? If there is a God, killing the unborn is wrong. If there is no God, then who cares? If it
serves the best interest of society and the individual, then kill. This can be likened to something I call,
"experimental ethics." In other words, whatever works best is right. Society experiments with ethical
behavior to determine which set of rules works best for it. Unfortunately, however, social
experimentation is often harmful.
There are potential dangers in this kind of ethical system. If a totalitarian political system is
instituted and a mandate is issued to kill all dissenters, or Christians, or mentally ill, what is to prevent
the atheist from joining forces with the majority system and support the killings? It serves his self-
interests, so why not?
But, to be fair, just because someone has an absolute ethical system based upon the Bible is no
guarantee that he will not also join forces for the killings. But the issue is the base and ramifications of
that base. Beliefs affect behavior. That is why belief systems are so important and absolutes are so
necessary. A boat adrift without an anchor soon crashes into the rocks.
The Bible teaches love, patience, and seeking the welfare of others even when it might harm the
Christian; in this the Ten Commandments are a summary. In contrast, the atheists' presuppositions
must be evolutionary. Since evolution teaches that life is the product of purely natural and utilitarian
properties of our world, survival of the fittest, natural selection, and equating humans to animals as a
species are the ontological basis for our existence and living. With this the value of man is lowered. In
contrast, it is a very high calling to treat people properly who also are made in the image of God.
Basically, I do not see how the atheist could claim any moral absolutes at all. To an atheist, ethics
must be variable and evolving. This could be good or bad. But, given human nature being what it is,
I'll opt for the moral absolutes -- based on God's word.
Mistake Christians make when dialoguing with Atheists

Most Christians mean well when they defend their faith. But, too often, many make fundamental
errors when dialoguing with atheists. We need to make as few errors as possible, not simply to win an
argument, but to help the atheist come to a saving relationship with the Lord Jesus.
Following are common mistakes made by theists when dialoguing with atheists.

Asking an atheist to prove there is no God


Sometimes Christians will attempt to dismantle an atheist argument by demanding that the atheist
prove there is no God. Well, to be fair, an atheist can no more prove there is no God than he can
prove that there isn't an ice-cream factory on Jupiter. The problem does not lie with the atheist, but
with the theist who demands such an impossible and illogical request. Generally, you do not try to
prove a negative. If I asked you to prove there wasn't an ice-cream factory on Jupiter, could you do
it? Of course not. It isn't a fair request. In fact, it isn't even a good request. Let's think about this
idea of proving there is no God.
First of all, how could an atheist prove there is no God? Can he know all things to know there is no
God? Well, of course not. If he knew all things, he'd be God. Can he answer every bit of evidence
raised in support of theism? Again no. He is not omniscient. There is simply too much information in
the world for one person to know.
Again, in argumentation you don't try to prove a negative. It’s the same thing as making
something up and then asking a person to prove it doesn't exist -- like an ice-cream factory on Jupiter.

Labeling Atheists
Some Christians have labeled atheists as evil, stupid, devil-worshippers, or morally void. Though
there may be some atheists who fit these categories (as would many in the general population),
atheists are not evil, stupid, devil-worshipping, degenerates with no morals. Many of them are fine
citizens, honest, caring, loving, and patient. For a Christian, or anyone, to make a blanket statement
about atheists in a derogatory manner is wrong. It is the same thing atheists sometimes do when they
accuse Christians of being irrational, psychotic, or stupid. Such accusations have no place on either
side of the argument of truth.
Generally, atheists are not stupid. Many of them have thought through their position over a long
period of time. Some were raised in religious homes, have seen what religion has to offer, and have
rejected it. Of course, I think that atheists have drawn incorrect conclusions about God, but it doesn't
mean they are dumb. Some atheists have presented very cogent arguments against the existence of
God -- which need to be addressed -- and rest their eternity on their arguments.
So, just because someone believes in God and encounters someone who doesn't, that does not
mean that either side is stupid. Labeling and name-calling have no place in the discussion.

Ignoring Atheists' Questions


If you were standing on a railroad track and a train was heading your way, closing your eyes and
ignoring the locomotive will not make it go away. If an atheist asks a question and you ignore it
repeatedly, it would be fair for him to conclude you were incapable of answering the objection. Of
course, this does not mean you have to always answer everything because dialogue flows both ways.
But, it is important that you face issues. If you don't have an answer, admit it. That's okay. It
doesn't mean you are wrong. It means you don't have an answer. Go study and get an answer and
get back to him.
Stating that Atheism is a religion
Atheists will repeatedly tell you that they are not in a religion. A religion almost always is defined to
include belief in a deity of some sort. Atheism is non-belief in a deity. It isn't necessarily a "belief that
there is no God," but is "not believing either way."
To label an atheist as a religious person is to put up a roadblock to any effective communication. It
would be like someone saying to a Christian, "You believe in a mean, tyrannical being who likes to
torture people." The Christian would simply role his eyes and think that the person doesn't know what
he's talking about. So, how much effective conversation could there be in either instance? Not much.

Stating unsupportable facts


No one has all documentation for everything they say. It is not reasonable to require proof from an
atheist on everything said. Nevertheless, if you are going to state a fact or two, it is good to have the
documentation at the tip of your tongue -- at least occasionally. It adds an air of credibility to your
argument. Of course, you don't have to document everything, but if you have some illustrious fact to
use, try and have it documented.

Never admitting when you are wrong


Pride is a harmful thing. It caused the fall. It ruins marriages. It leads to anger and self-
righteousness. It has no place in the Christian's life. Never admitting you are wrong is being prideful.
If an atheist, or anyone, proves you wrong in something, be kind and courteous. Admit you made a
mistake and go on. Everyone makes mistakes, even atheists. There is nothing wrong with admitting
an error. It no more proves you are wrong about Christianity than being wrong about the color of a
boat means boats don't exist. But, if you never admit when you are wrong, you will not be able to
convince anyone in a discussion of your position. You will simply loose the respect of the one with
whom you are debating.
Mistakes Atheists make when dialoguing with Christians.

Following are examples of mistakes I've seen atheists make when dialoguing with Christians. They
are important to know for both sides. First of all, the atheist must realize that making these mistakes
lessens his credibility with a Christian and does not help his cause. Second, the Christian should know
these errors so he can identify them during a conversation and, hopefully, not commit them himself.

Atheists often challenge the theist to prove God’s existence only within the confines of
science.
Science has served humanity well. Through it we have discovered countless natural laws of
universe and use that knowledge to make our lives easier in every area of our existence. But to limit a
theist's proofs to the confines of what the atheist determines is one sided. To a Christian, there are
experiences that science and logic cannot explain and these experiences are real. The atheist needs to
recognize that we have experiences that are life changing. No mere psychological set of theories
explains the changes in our lives. So please, don't mock them. Can science nail down all that exists in
mind, body, and soul? No. Can it quantify the beauty of a sunset, the cooing of a baby, or the love of
a man and a woman? Science and logic have served us well, but they are not the ultimate truth to all
things.
Of course, that does not mean we ignore science. In fact, we use it in our proofs for God. But to
limit the playing field to your set of rules is an unfair way to start. It is mostly an attempt to initiate
control and keep command of the conversation by setting the ground rules according to your criteria.

Name-calling and insults.


Of course, this is obvious. I heard it said once that the man who strikes first admits his arguments
have run out. In some of the discussions I've had with atheists, when I've made a valid point in logic,
I have been insulted. To call someone a name is to attack the person and not the issue and it closes
the door to true discussion.

Condescension
This is the most common of all mistakes I've encountered with atheists. I've been told by atheists
that I'm an idiot for believing God, that if I were truly intelligent I'd abandon my anachronistic
thinking, etc. I've yet to meet a single humble atheist.

Straw-man argumentation
Sometimes atheists will construct an argument against Christianity that does not reflect a true
Christian position. For example, one atheist stated that the Trinity was illogical because three gods
could not be one God. I had to correct him and show him that the Trinity is the doctrine that there is
only one God in three persons, not three gods.
Other straw-man arguments deal with person's who claim to be Christians and act in an unchristian
manner. A typical example is the white supremacists often claim to be true Christians and when they
do something which is against the Bible, their bad example is used to label all Christians.
Is Atheism viable?

Atheism is, essentially, a negative position. It is not believing in a god, or actively believing there
is no God or choosing to not exercise any belief or non-belief concerning God, etc. Which ever flavor is
given to atheism, it is a negative position.
In discussions with atheists, I don't hear any evidence for the validity of atheism. There are no
"proofs" that God does not exist in atheist circles; at least, none that I have heard -- especially since
you can't prove a negative regarding God's existence. Of course, that isn’t to say that atheists haven’t
attempted to offer some proofs that God does not exist. But their attempted proofs are invariably
insufficient. After all, how do you prove there is no God in the universe? How do you prove that in all
places and all times, there is no God? Besides, if there were a proof of God’s non-existence, then
atheists would be continually using it. But we don’t hear of any such commonly held proof supporting
atheism or denying God’s existence. The atheist position is very difficult, if not impossible, to prove
since it is an attempt to prove a negative. Therefore, since there are no proofs for atheism’s truth and
there are no proofs that there is no God, the atheist must hold his position by faith.
Faith, however, is not something atheists like to claim as the basis of adhering to atheism.
Therefore, atheists must go on the attack and negate any evidences presented for God’s existence in
order to give intellectual credence to their position. If they can create an evidential vacuum in which
no theistic argument can survive, their position can be seen as more intellectually viable. It is in the
negation of theistic proofs and evidences that atheism brings its self-justification to self-proclaimed
life.
There is, however, only one way that atheism is intellectually defensible and that is in the abstract
realm of simple possibility. In other words, it is possible that there is no God. But, stating that
something is possible doesn't mean that it is a reality or that it is wise to adopt the position. If I said
it is possible that there is an ice cream factory on Jupiter, does that make it intellectually defensible or
a position worth adopting merely because it is merely a possibility? Not at all. So, simply claiming a
possibility based on nothing more than it being a logical option is not sufficient grounds for atheists to
claim viability. They must come up with more than "It is possible," otherwise, there really must be an
ice cream factory on Jupiter and the atheist should step up on the band wagon and start defending the
position that Jupiterian ice cream exists.
But there is another problem for atheists. Refuting evidences for God’s existence does not prove
atheism true anymore than refuting an eyewitness testimony of a marriage denies the reality of the
marriage. Since atheism cannot be proven and since disproving evidences for God does not prove
there is no God, atheists have a position that is intellectually indefensible. At best, atheists can only
say that there are no convincing evidences for God so far presented. They cannot say there are no
evidences for God because the atheist cannot know all evidences that possibly exist in the world. At
best, the atheist can only say that the evidence so far presented has been insufficient. Therefore,
since there could be evidences presented in the future, the atheist must acknowledge that there may
indeed be a proof that has so far been undiscovered and that the existence of God is possible. This
would make the atheist an agnostic since at best the atheist can only be skeptical of God’s existence.
This is why atheists need to attack Christianity. It is because Christianity makes very high claims
concerning God’s existence which challenges their atheism and pokes holes in their vacuum. They like
the vacuum. They like having the universe with only one god in it: themselves.
Response to criticism of "Is Atheism Viable"

Following is my response to the first portion of an atheist's critique of two of my papers dealing
with atheism. His original criticism was one page, but I have broken it up into two pages relating to
each paper he addressed.
The article was posted on infidelguy.com, an atheistic website, and that is the only reason I am
responding to the paper which, in my opinion, does not present its case very well. Nevertheless, I
have copied the entire article with the author's permission and reproduced the the two halves, one
here, here so that it can be more easily address. His original comments are in black, and my
comments are in italics. I have left his typo's and grammar errors intact.

"Is atheism viable?"

This is my refutation of Matt Slick's other article, "Is atheism viable?” I'll show why he is wrong
again, point by point. Follow along in his article.
Atheism is a negative position. This is true. It doesn't sound so good, and this is what he's looking
for, to make atheists look bad, but it is technically true.

Mr. Lonovy is trying to play the mind reader. He does not know if I am trying to make atheists
look bad or not. In fact, why would I want to do that? Making an atheist look bad isn't how truth is
established. Rather, I attempt to tackle the issues and not the individuals -- unlike Mr. Lonovy who
has stated in his original paper that I am an idiot.

Matt says he doesn't hear any evidence for atheism when he has discussions with atheists. I'm
sorry that there are so many stupid atheists, then. There is proof against God's existance, even if Matt
doesn't discuss it with atheists who know this. Is it enough to completely rule Him out? No. It is
enough to say what He does and does not do. As I stated earlier, the only scientific place left for Him
is the Big Bang. People used to believe He did many different things, until science proved that they
could function on their own without Him. Is that not a good point against Him? It is hard to prove He
doesn't exist, because it is proving a negative, but we can prove the negative that His Christian
interpretation is false by proving positives - That things like evolution, abiogenesis, and an atheistic
[without God] Big Bang can and have actually occured. These things are strongly rooted in truthful
science. There is our best weapon.

Mr. Lonovy not only insults me, but now he insults atheists.
For him to say there is proof that God does not exist is really quite a statement. How do you
prove a negative? How do you prove that God, the creator of the universe, who exists outside of time
and space, does not exist? That is a tall order and I would truly love to see the proof. If it is indeed
proof, I will abandon my Christianity. After all, proof is proof. I have asked atheists for proofs and
have not yet seen one offered that has stood the test of cross examination.
Mr. Lonovy fails to understand that even though science has answered many issues about life,
medicine, mechanics, the universe, etc., it does not invalidate God's existence nor is it in any way a
proof or evidence that God does not exist. The only thing science does is explain things using
naturalistic principles. But, since Christians define God as being outside of time and space (yet able to
interact within it), explaining things naturalistically does not effect the proposed existence of God or
not since He is not limited to a naturalistic system. After all, the Bible states that God created the
naturalistic principles working in the universe. Since these principles exist, how is it that it means that
God does not exist? It doesn't. Therefore, Mr. Lonovy is again failing to make his point.
As for the rest of the paragraph, Mr. Lonovy again begs the question regarding evolution,
abiogenesis, et. al. He assumes that all of it occurs due to naturalistic principles in the universe
though he has not offered any evidence for this. The topics he introduces are too deep and varied to
address here (as they have been addressed elsewhere on my site), but the principle of his
presuppositions clouding his objectivity is, to me, very obvious.
Atheists don't hold their position by faith, as a Christian does. Faith is defined as being belief in
something for which there is no proof. There is proof that things other than what the Bible says have
occured. We prove the negative by proving a positive that will contradict it, and therefore render it
false. But even with this, how dare a Christian say someone else can't hold their position by faith? With
this logic, we should all be agnostics! Attacking yourself in the process of attacking someone else
doesn't help you. It leaves a level playing field. Why do it?

I appreciate that Mr. Lonovy attempts to define faith. But I do not accept his definition as being
sufficient, though there is some merit in it. I would agree that if something had proof, then there
would be no faith. But that isn't all there is to it. Faith can rest on evidence. That is, a person can
decide to have faith based upon evidence. I am sure Mr. Lonovy lives this kind of faith regularly. Let
me illustrate. I assume Mr. Lonovy drives a car. Can Mr. Lonovy offer proof before he takes his next
drive, that the next time he drives to the store that he will make it there alive? No, he cannot. But,
past evidence of him being able to drive, people abiding by driving laws, and previous successes of
him getting to the store and back safely, are all evidences by which Mr. Lonovy decides to have the
faith that he will be able to get to the store alive...even though there is no proof that he will. He is
acting faithfully to the evidence. So, Mr. Lonovy's definition, though true in part, is insufficient and it
does not reflect the biblical representation of faith which rests on evidence, i.e., the resurrection,
Jesus' miracles, etc.
My proposition that atheists hold their position by faith is based upon the idea that there is no
proof for atheism; there is no evidence that God does not exist; and that atheism only succeeds if it
can refute all theistic proofs and evidences -- which they can only hope they can do. Therefore, I
conclude that there is a large measure of faith that the atheists use to hold to their atheism since
there is no proof.

Again, atheists do attack Christian claims. It is the duty of a skeptic to do so. It is not only this
'evidential vacuum' that atheism gains it's justification, but it is one of the ways. Proof for things which
contradict the Bible is much more effective, though.

I have not yet seen any "proof" offered by an atheist that contradicts the Bible. There may be
something out there that does, but I have not yet seen it. So, I really cannot comment beyond that.

Atheism can only be defended by it's status as a possibility? Wrong. Science and logic have clearly
shown that it is much more than a simple possibility. But what right does a Christian have to say
someone else can't believe something because it's a possibility? That's exactly what they do.

Again, Mr. Lonovy begs the question. Science has not shown that there is no God nor is there any
logical proof (that I am aware of) that there is no God. Since atheism is the position of "no God"
either in belief or "lack of belief," and since there is no proof that God does not exist, then faith must
make up the difference.
Either atheism is absolutely true or it is possibly true. Since it cannot be proven that atheism is
absolutely true (i.e., prove that there is no God in all space and time, etc.), then all that is left is that
it is a possibility that it is true -- or, dare I say, that it simply is not true.
Furthermore, Mr. Lonovy seems to believe that because science can explain things that it means
there is no God. But, this is not logical as I have demonstrated above.
Finally, people can believe what they want to believe. I simply question the evidential and logical
validity of the atheistic belief system.

The ice cream factory on Jupiter really shows Matt's stupidity. Why would anyone believe that there
actually is an ice cream factory on Jupiter when there is no evidence for it and many other things, like
the planet Jupiter itself, contradict such a possibility? I don't know, but this is exactly what Christians
do with God.

I suggest that the reader actually read the article in question and read the context of my
statement about the ice cream factory on Jupiter. It was merely an illustration. Nevertheless, I will
quote the relevant material from that paper
"...stating that something is possible doesn't mean that it is a reality or that it is wise to adopt the
position. If I said it is possible that there is an ice cream factory on Jupiter, does that make it
intellectually defensible or a position worth adopting merely because it is a possibility? Not at all. So,
simply claiming a possibility based on nothing more than it being a logical option is not sufficient
grounds for atheists to claim viability."

Again, attacking me personally is not the best way to establish a point. Whether or not I am stupid
is, I am sure, a debatable issue among the atheist community but it should be best left aside when
addressing issues of truth.

Refuting 'evidence' for God's existance doesn't prove atheism true. This is correct. What about
this: Does proving atheism false prove Christianity true? No, it doesn't. It works both ways. But I'm
quite sure I've shown that we have many other ways to prove that the Christian god doesn't exist, so
his point is null.

Finally, Mr. Lonovy is logical. However, he has failed to show proof that the Christian God does not
exist.

An atheist can't say he knows everything in the universe. Can a Christian? No. Then, how can he
say God does exist? Again, Matt's logic attacks his own religion, too. How stupid. An atheist can know
many, many things, though. Enough to come to a logical conclusion that God, at least the Christian
interpretation of such a being, does not exist.

As a Christian my belief in God rests on evidence, experience, and decision. I see the biblical
evidence, experience the work of God in my life, and I have chosen to continue in belief based upon
these factors. What I lack in absolute proof, I complete in faith.
I certainly agree that an atheist can conclude that God does not exist. But, it does not mean that
his conclusion is correct. I can conclude that screaming blue ants are spying on me, but that doesn't
mean I am right.

An atheist can say that there is more than just no available proof for God's existance. There is
proof that contradicts the Christian notion of God. Science has shown us how many things in this
universe occured, esspecially the appearance and evolution of life. The truth of these things is for a
different discussion, though.

Again, I have not seen this proof Mr. Lonovy keeps mentioning. If there is such a proof, why is it
that the atheists are not unanimously using it?
I do not believe in macro evolution, but even if it were true, it is not proof that God does not exist.

An atheist must acknowledge that there may be proof for God in the future if they are truely
intelligent. Atheism isn't a dogmatic religion like Christianity, atheists are allowed to do this. This
doesn't make the atheist an agnostic. It makes them open minded. You can accept that such a being
may exist without actually believing it does exist. Why don't Christians accept that there may be no
God? If Matt wants atheists to be agnostic because they accept that God might exist, then why isn't he
an agnostic? It works both ways. That is by far the stupidest thing he's said in his papers.

As a Christian, I can accept the possibility that there might not be a God. However, I most
definitely believe and affirm that the God of the Bible exists and is the only true God. This does not
make me agnostic; that is, it does not mean that I don't know if God exists or not. On the other hand,
the atheist states, basically, that there is no God. But if this same person states that God may exist,
then doesn't that mean that he isn't sure, that he doesn't know if God does or does not exist? That is
not the same position I hold at all.

That is not why atheists attack Christianity. They don't even need to. I've shown this already.
Christianity makes irrational claims about God that make me think the Christians are high. [LOL] Their
claims no more "poke holes in my vacuum" than someone claiming that there is an ice cream factory
on Jupiter. I've shown why already.

I am sure that there are Christians who make irrational claims about God. I am also sure there
are Christians who make rational claims about God.

I do not like my "vacuum". I'd much rather there be a loving god who would take us all to Heaven
when we die. I'd extremely afraid of dying. I'd give up my atheism for immortality in Heaven any day!

I am glad to see this honesty. But, God allows us to have what we want. If you want your sin
and independence from God, He will let you have it and He will not reveal Himself to you. The
evidence is there in the Bible.

We like having ourselves as gods? We're not all that egotistical. How dare you say this, Matt? Not
only is Matt stupid, he's an as***le to atheists, too. All too much like a typical Christian. I've shown
why I believe him to be this way. Thank you for reading. Again, anyone who sees my points, please
urge Matt to take down his fallacious articles. We don't need him lying to people about what we
believe. [I substituted three asterisks in the cuss word with which Mr. Lonovy referred to me.]

Mr. Lonovy is referring to my closing statement quoted here...

"This is why atheists need to attack Christianity. It is because Christianity makes very high claims
concerning God’s existence which challenges their atheism and pokes holes in their vacuum. They like
the vacuum. They like having the universe with only one god in it: themselves."

Of course, the context is that God is the true sovereign and that atheists want that for themselves.
In this, they take the place of God and set themselves up as master of their own lives, future, etc.
Again, Mr. Lonovy uses a personal attack in his paper. This is definitely a poor way to address an
opponent.

Conclusion: I am always open to intelligent dialogue with atheists and have even changed parts of
my site in response to some well intentioned and well delivered correspondence from atheists.
However, Mr. Lonovy demonstrates a lack of tact and logical acumen. He has not established his case
nor has he 'refuted' my paper.
Another response to criticism of "Is atheism viable?"

Some atheists take their atheism seriously and others do not. Either way, I am pleased to see
atheists attempt to refute what I've written on CARM. The following article is just such a case.
I was contacted by a Mr. Dawson Bethrick who told me he had written a response to my paper "Is
Atheism Viable?" After glancing at his piece I noticed it was a bit condescending. I mentioned this to
him in a return email and Mr. Bethrick proceeded to challenge me on proving his condescension. He
further stated I "obviously could not refute" what he had to say. Anyway, I have reproduced most of
his comments in order to address important information. His original comments are in black, and my
comments are in italics. I have left his typo's and grammar errors intact.
______________________

Slick's Fuss: A Review of CARM's "Is Atheism Viable?"


by Dawson Bethrick

The Christian Apologetics & Research Ministry, or "CARM.org," has an entire section of their website
devoted to responding to atheism. One of the site's articles is a short essay called Is Atheism Viable?
by author Matt Slick. This essay gives the author's reasons for why he thinks atheism is wrong and
indefensible. As is the case with many attempts to make mysticism seem rational, Slick's faulty
conceptions leave a lot to be desired.

Mr. Bethrick implies that mysticism cannot not rational. Mysticism is a belief in realities or
existences outside our perceptual and/or one's intellectual apprehension. This would include the idea
that God exists. But, is it irrational to believe that there are things in existence beyond our
apprehension? Of course not. Furthermore, he has not demonstrated why belief in God is not
rational. He just states it is not rational. In so doing he commits the fallacy of begging the question;
that is, he assumes the thing true he is trying to prove. He assumes atheism is true and labels theism
as irrational mysticism. This is neither a competent nor logical assertion on his part.

The title of Slick's essay asks the question "Is Atheism Viable?" What does it mean to be viable?
Webster's Dictionary defines 'viable' as "capable of working, functioning, or developing adequately";
"capable of existence and development as an independent unit"; "having a reasonable chance of
succeeding." Is atheism any one of these things?

Please notice that Mr. Bethrick does not even afford me the respect of calling me Mr. Slick.
Instead, it is just "Slick." This is a personal preference, but when I address someone I criticize I try
and show him the respect of calling him "Mr." as in Mr. Bethrick. But Mr. Bethrick mentions "Slick" 84
times by itself in his response, not once with Mr. in front of it. Nevertheless, the issue is not whether
or not someone can believe in atheism. The issue is whether or not it is a defensible and logical
position to hold.

Since atheism is essentially a negation or negative condition, it is up to considerable debate


whether atheism can be said to be "viable." One does not typically think of a negation as being
"capable of working" or of developing "as an independent unit." But the essence of the Slick's question
is clear: Is atheism the proper alternative to god-belief? As an atheist myself, I would answer with an
emphatic yes to this question. This is, of course, because I think god-belief is irrational.170

170
I agree fully with CJ Holmes' arguments as presented in his essay Why God-Belief Is Irrational.
At least Mr. Bethrick is stating an opinion when he says that he "thinks" belief in God is irrational.
If had stated it was irrational without logical support, he would be offering nothing but opinion in the
place of fact. This is, to be sure, what many atheists accuse Christians of doing in believing in God.

Slick states that, "In discussions with atheists, I don't hear any evidence for the validity of
atheism." But what would Slick consider to be "evidence for the validity of atheism"? As he
acknowledges in the opening of his essay, he is essentially asking for evidence for the validity of a
negation. But a negation is necessarily valid, epistemologically, in the absence of evidence or
convincing argument for the positive. To illustrate, consider the example of the Greek god Zeus. Let us
call "Zeusism" the belief that Zeus exists and that he is the supreme being. Let the term "aZeusism"
mean the absence of such a belief. Clearly, the term "aZeusism" is a negation, just as the term
'atheism' is: it is the absence of a particular kind of belief. Does Slick hold to Zeusism, or to aZeusism?
I would wager that he is an aZeusist, i.e., one who has no Zeus-belief. But what would one consider to
be evidence for the validity of aZeusism? If Slick is an aZeusist, he would have to present such
evidence if he wants to be consistent with his expectation that atheists should present "evidence for
the validity of atheism." I don't suppose we should hold our breath.

Mr. Bethrick's defense here is basically worthless. He states that the "absence of evidence or
convincing argument for the positive" is what makes atheism viable. But this is nothing more than a
statement centered around subjectivity; namely, his subjective atheistic presupposition. Mr.
Bethrick's atheistic presupposition does not allow him to view theistic arguments with any serious
acumen because he has already stated he believes that theism is irrational. Therefore, by default, any
argument proposed for the existence of God must be, according to his presupposition, irrational and
invalid from the beginning. Mr. Bethrick has effectively cut off any true and convincing dialogue on
the existence of God and forced all logic and evidence to fit into his subjective mental box or else it is
irrational. This is not the method of serious intellectual inquiry. For more information on this, please
read I don't see any convincing evidence for the existence of God.

The expectation that non-believers present "evidence for the validity of atheism" is symptomatic of
the intention to evade the onus of proving one's existentially positive claims. If Slick claims that a god
exists and he expects others to accept this claim as truth, then he would have to support this claim.
Clearly the default is not belief, but non-belief. Pining as Slick does that he doesn't "hear any evidence
for the validity of atheism" simply misses the point.

Mr. Bethrick misses the point. Elsewhere on CARM I present evidences for God's existence. Some
of it is on the atheism section which, if he had read more thoroughly, would negate his statement here
about proving God's existence. But, I never maintain that I can prove God exists. Instead, I have
offered various evidences for God's existence. But since Mr. Bethrick's presupposition is that God does
not exist, any evidence I offer will be, by default, insufficient and my argumentation must also be
suspect and irrational.
Mr. Bethrick again assumes too much. He says, "Clearly the default is not belief, but non-belief."
This is again nothing more than guesswork. What "default" is he speaking about? If he means that
babies are born without belief in God, that is nothing more than a guess. How does he know what is
or is not in the mind of a baby? If we define belief as a cognitive assent, then babies don't believe in
God since they do not (we assume) cognitively assent that God exists. But if we define belief as the
presupposition that God exists due to some innate quality in a person, then the baby does believe in
God. It depends on definition and since Mr. Bethrick has not been specific here, we cannot be sure
what he means. Also, we cannot know the mind of infants, so we cannot authoritatively state which is
the case.
Finally, I am not "pining." His use of the word means that I am nostalgic, or have a lingering
desire. But, desire for what? God? I do not "pine" for God's existence. There is no nostalgia involved
in this. I simply believe that God exists and that the Christian revelation of God is the only correct
one. I would prefer that Mr. Bethrick stick to the issues instead of trying to play the mind-reader and
disclose to the world what he thinks are my motives and emotions concerning God. To further commit
such errors of argumentation is a demonstration of a clouded judgment on his behalf.
However, Slick does give some indication of what he would consider to be "evidence for the validity
of atheism" when he states, "There are no 'proofs' that God does not exist." Of course, this is the
expectation that one prove a negative, an onus which non-believers do not bear. To illustrate, how
does Slick prove his claim that "There are no 'proofs' that God does not exist"? This is a negative
claim, but where's the proof? Does Slick special plead his case, assuming that he has no onus to prove
that "There are no 'proofs' that God does not exist" while those who do not believe the claim that there
is a god must "prove" that god does not exist? How does Slick know that "There are no 'proofs' that
God does not exist," and how does he show this claim to be true?

As I said earlier, my evidence for God is elsewhere on CARM and a section is devoted to it on the
atheism section -- which Mr. Bethrick, apparently, has not chosen to examine or mention. Here is the
paragraph that Mr. Bethrick is referring to from my paper:

"In discussions with atheists, I don't hear any evidence for the validity of atheism. There are no
"proofs" that God does not exist. Of course, that isn’t to say that atheists haven’t attempted to offer
some. But their attempted proofs are invariably insufficient. After all, how do you prove there is no
God in the universe? Besides, if there were a proof of God’s non-existence, then atheists would be
continually using it. But we don’t hear of any such commonly held proof supporting atheism or
denying God’s existence. The atheist position is very difficult, if not impossible, to prove since it is an
attempt to prove a negative. Therefore, since there are no proofs for atheism’s truth and there are no
proofs that there is no God, the atheist must hold his position by faith."
I make one clarifying comment here. To say "there are no proofs of God's existence" is not
completely logical since I cannot know all proofs that might exist. I could really only state that so far I
have not seen any sufficient proof for God's non-existence. But, in the paragraph above, the context
is dealing with the conversations I've had with atheists where I "don't hear any evidence for the
validity of atheism." It is that context that I have said there are no proofs for God's non-existence.
Furthermore, how do you prove that in all places and in all times and in all dimensions, God doesn't
exist? In order to do that, you'd have to be God to know all things to know there isn't a God, which is
not logical. Nevertheless, I have since modified the paragraph in the original paper to make the point
more clear.

The irony of Slick's predicament, however, does not stop here. For even Slick, after announcing
that "attempted proofs [that God does not exist] are invariably insufficient," asks, "how do you prove
there is no God in the universe?" (I thought Christians believed that God exists "beyond" the
universe…) Does Slick think that "a proof of God's non-existence" is necessary for atheism to be the
proper response to theism? If so, why does he think this? Apparently, Slick thinks that god-belief -
indeed, his god-belief - is true until proven false. It seems that he thinks a proof for God's existence is
not necessary. One does not need a reason to believe; rather, one needs a reason not to believe. How
consistently would Slick apply such a reversal of rational principle?

Mr. Bethrick misses the point and offers a distraction instead of addressing the issue. I asked "how
do you prove there is no God in the universe?" Instead of addressing that question, he offers a non-
sequitor by stating (I thought Christians believed that God exists "beyond" the universe…). It seems
Mr. Bethrick needs to adjust his thinking since we Christians believe that God is also in the universe as
well as outside of it. He is, after all, omnipresent. Mr. Bethrick then goes on to raise issues which are
constructed upon his erring premise. He asks several questions about what I might be thinking and
then tries to address the straw man answers he's constructed.. Again, Mr. Bethrick should leave the
mind-reading and guess-work out of this discussion, adopt a more logical approach, and stick to the
issue at hand.

Slick continues: "Besides, if there were a proof of God's non-existence, then atheists would be
continually using it." This would only be true if the atheists in question a) knew about the supposed
proof, and, perhaps, b) considered the proof to be consistent with their own worldview outlook.
However, as indicated above, a proof of the non-existence of something is certainly not warranted
simply because someone makes the claim that the something in question exists. One does not inherit
an obligation simply because another presents a claim.
It is the atheists who claim atheism is valid. Why is the atheist not able to prove his position or
offer evidence for its validity? He can't. That is why Mr. Bethrick is trying to shift the onus of proof
onto me by trying to get me to prove God exists. I may not be able to prove God exists, but I do have
evidence (as given on CARM). It is up to Mr. Bethrick if he wants to examine it or not. But given his
atheistic presupposition, I am sure that all such evidences would be insufficient.
The problem for Mr. Bethrick is, as I have stated before, that atheism can only survive in a theistic
vacuum. It only exists in the minds of atheists who claim a position that is, as far as I can tell,
logically unprovable. I do not see how anyone could prove there is no God in all the universe since we
cannot know all things about all places in all times about the universe in order to determine that there
is no God. If there is some other way of proof, i.e., logic, then let's see it. But until then, I am
"atheistic" about atheistic proofs for God's non-existence and will stick to the evidence supporting the
reality of God.
Again, Mr. Bethrick is not addressing the real issue. He is attempting to shift the topic to theistic
proofs. This is only a demonstration of the validity of my premise that atheism exists in a theistic
vacuum which it must construct by presupposing God does not exist and then negating all proposed
theistic proofs. But, since there are no known atheistic proofs, atheism is not a viable option. It is
only a belief system; that is, a belief system which states there is no God -- or lacks belief in God.

Slick himself acknowledges the problematic nature of his expectations when he states, "The atheist
position is very difficult, if not impossible, to prove since it is an attempt to prove a negative." If it is
the case that it "is very difficult, if not impossible, to prove… a negative," then what exactly is Slick's
fuss? And, furthermore, if perchance an atheist were to present a proof that god-belief is irrational,
would Slick accept it and abandon his god-belief? Again, I am not supposing that we should hold our
breath. Reason as such does not seem to be his epistemological absolute. Instead, a commitment to a
primitive worldview, complete with invisible magic beings, is what he considers to be non-negotiable.
Reason is dispensable when it gets in the way: he has no onus of proving his positive claims, but those
who do not accept his claims bear an onus of disproving those claims. Where is this man's confidence?

First of all, Mr. Bethrick has admitted his difficult position of substantiating atheism by saying,
"Slick himself acknowledges the problematic nature of his expectations when he states, 'The atheist
position is very difficult, if not impossible, to prove since it is an attempt to prove a negative.'" Exactly
correct. Mr. Bethrick agrees with me that it is problematic for atheists to prove their position which
only supports my premise in the original paper. Thank you, Mr. Bethrick.
Also, it is not me who is making a fuss. It is Mr. Bethrick. My paper was on the viability of
atheism. But it seems that Mr. Bethrick is addressing something I haven't' written. Note how he
quotes me saying atheism is "very difficult, if not impossible to prove" and yet ignores the implications
of that statement by saying I am making a fuss. In other words, he offers no rebuttal. He says I am
making a "fuss," something children are known to do, instead of addressing the issue.
Furthermore, I suggest that Mr. Bethrick stop holding his breath. If I see a proof that there is no
God, I'll become an atheist. After all, proof is proof. It would not be rational for me to believe in God
in light of "proof." Furthermore, I would abandon Christianity if it can be reasonably demonstrated
that Jesus did not rise from the dead. I say these things because I have long ago come to grips with
the acceptance of facts and evidence. I am not a brainwashed, non-thinking believer who holds on to
God so desperately that nothing, not even the facts, can shake it. On the contrary. I have no fear of
the facts and welcome them. I welcome any atheist proofs and counter evidence. I welcome any
proofs that there is no God. I would love to see them -- or it.
Mr. Bethrick is the one, in my opinion, who is dispensing with reason. The paper "Is atheism
viable" is not about proofs for God's existence. It is about whether or not atheism is logically viable. I
suggest that he reread the paper and then address what it actually says instead of what it does not
say.

Slick concludes his point with the following statement: "Therefore, since there are no proofs for
atheism’s truth and there are no proofs that there is no God, the atheist must hold his position by
faith." I am compelled to ask: how does he show that "there are no proofs for atheism's truth"? Even
Slick himself makes the point later in his essay that atheists "cannot say there are no evidences for
God because the atheist cannot know all evidences that possibly exist in the world." Where does Slick
present an argument which soundly concludes, "therefore, there are no proofs for atheism's truth"?
Indeed, he nowhere presents such an argument so far as I can determine. Perhaps he knows of one,
but insists on holding back?

Just as Mr. Bethrick asserts that I must provide proof for the validity of the position I hold, I ask
where is the proof of the validity of the atheistic position that he holds? Is this a double standard?
Atheism, as I have said before, lives in a theistic vacuum. It exists only by attempting to disprove
theistic evidences and/or offering attempted reasons why no God can exist. But, I have already stated
above that the atheist has no proof that God does not exist because it is impossible to prove that in all
the universe in all places and in all times God does not exist; at least, I don't see how it is possible. If
Mr. Bethrick would like to show me how that is possible, I'm open to that discourse. Nevertheless,
since it seems that the atheistic position is not provable, how is atheism intellectually viable based
upon logic, proofs, or evidence? I have ask this question and Mr. Bethrick has not answered it.
However, Atheism is viable in one sense: it is simply a possibility. But, being a possibility does
not mean it is a reality.

But Slick's immediate agenda comes quickly to light when he insinuates that "the atheist must hold
his position by faith." Apparently, holding a position "by faith" is philosophically suspicious in Slick's
view. I would agree. But even before we attempt to examine Slick's dependence on faith, we must
ask: does Slick present a sound case to establish the supposition that "the atheist must hold his
position by faith"? I thought atheism was a negation, yet here Slick is treating atheism as if it were a
positive belief. If atheism is the absence of god-belief, then atheism is properly classed as a negation,
not as a positive belief. Atheism "means not believing in God - which leaves wide open what you do
believe in."171 This is a point which Slick and other theists need to integrate into their thinking.

I did not say that holding a position by faith was philosophically suspicious. This statement by Mr.
Bethrick demonstrates that he is trying to add into what I am saying in order to bolster his position.
Since the atheist cannot prove that God does not exist and since at best all he can do is offer negation
so theistic proofs, and since he has not negated all theistic proofs, he holds his position that there is
no God, at least to some extent, by faith. That is, he believes that the future will not provide proof
contrary to what he already believes. This is an act of faith, not fact.
And yes, I do treat atheism as a positive believe system. I have stated this already on my web site
and in public in debates with atheists. If you read my paper dealing with the atheistic position of
lacking belief in God, you can see this. Also, different atheists have different definitions of what
atheism is. So far, Mr. Bethrick has not stated which particular position he holds. Atheists consistently
and in all practicality assert that God does not exist. If they say that they merely have no intellectual
position or commitment concerning belief in God, then I ask why they go through such a long and
arduous effort to denounce and disprove theistic proofs as well as refute papers attacking atheism --
as he has done here. In other words, he is behaving as though he believes there is no God by trying to
refute my paper dealing with the viability of atheism. He is therefore confirming my assertion which he
complains about; namely, that atheism is a positive belief. He is behaving consistent with his belief
and his behavior is to substantiate atheism. Therefore, it is a positive belief system.
Mr. Bethrick has not read my information on this subject which explains why he has not
incorporated my comments about this subject into this paper. I suspect that Mr. Bethrick may have
only read one paper and then decided to tackle it without reading the rest of what I have a written on
atheism. Again, this is not the best way to do things.

In contrast to what Slick apparently desires, he does not establish that atheists must assume their
positions as a matter of faith. He seems to be assuming that one must either hold a belief by proof, or
by faith. While there are good reasons not to accept this dichotomy, 172 Slick overlooks the fact that
171
Leonard Peikoff, "Religion Versus America," The Objectivist Forum, June 1986, p. 14.

172
In response to Christian apologist John Robbins' essentially identical criticism of the Objectivist axioms (insinuating,
as Slick does here) that a thought must be either a product of proof or accepted on faith if it is to be believed as truth,
Bryan Register points out the fact that Robbins' criticismassumes that there are only two kinds of claims: those one
proves and those which one takes on faith. In fact, as the Objectivist literature makes clear, there is a third type of
claim: one which is valid because it formulates a fact that is directly perceived. Such are the most fundamental
atheism is not a belief, but the absence of a belief. Does one need faith not to believe that Zeus
exists? It would be preposterous to suggest this, yet this is precisely what Slick is suggesting when he
wants to conclude that "the atheist must hold his position by faith," even when Slick himself has
recognized that atheism is a negation.

I do not agree with his assertion that atheism is simply on "absence of belief." As I have stated in
other papers on atheism, when atheists behave in such a manner as to demonstrate their belief that
there is no God, then I am forced to conclude that the particular belief system they adhere to is the
one that says "I believe there is no God." I would assume that if Mr. Bethrick were asked directly if he
believes that a God exists or not, he would conclude that one does not exist. After all, this is how he is
behaving in his attempts to verify atheism. Of course, I could be wrong.
Mr. Bethrick has apparently missed this point that I have raised before on the web site concerning
this issue. It is regrettable that he has not included this information in these comments.

Slick states that faith "is not something atheists like to claim as the basis of adhering to atheism,"
thus making the whole matter sound like it's an issue of likes or dislikes, or whimsical preference.
Apparently Slick resents the prospect that non-theists are justified in not accepting his god-belief
claims, so he attempts to derogate non-belief as such. This is a sign that such a course is all Slick has:
make atheism as such appear to be ridiculous. But to do so, he must characterize it in such a way to
make it appear ridiculous. If Slick's god-belief had a genuine rational basis, he would not need to do
this. All he would need to do is present his proofs for the existence of his god, and leave it at that.
Those who do not accept his proofs certainly do not need Slick's approval not to accept them.

For the most part, atheists' preferences are not "whimsical." However, I have yet to meet an
atheist who actually likes the idea of the existence of god. My experience with atheists has lead me to
conclude that actually dislike theistic proofs. This makes sense since theist proofs would challenge
their presuppositions and people don't like their presuppositions challenged. This has been consistent
with every atheist that I have encountered. Mr. Bethrick seems no different.
Furthermore, Mr. Bethrick again tries to sidestep the real issue about the lack of intellectual
viability in atheism as a verifiable position/system by trying to attack the rational basis for belief in
God. This is typical of atheists and is a verification of my earlier comment that atheism must exist in a
theistic vacuum which atheists must try and maintain. So far in his response, he has offered nothing at
all for they validity of atheism. The only thing he has tried to do, is attempt to demonstrate that my
thinking is incorrect. Perhaps it is. But in so doing, he has taken his eyes off the subject and tried to
misdirect to another topic. Again, where it is his validity for atheism? He has offered none.

What we have here, and with so many apologists (particularly on the internet) is a kind of sobbing
complaint: "they don't believe my god-belief claims, those horrible atheists!!" To say that this non-
acceptance of god-belief claims is itself an expression of faith, is to miss the point and set up a straw
man. In addition to these, such subterfuge on Slick's part simply closes him off to an honest
examination of reasons why god-belief is irrational (indeed, look what defending god-belief drives Slick
to do!). Thus, he vilifies atheism to spite himself.

Now we see a caricature introduced by Mr. Bethrick in his condescending rebuttal. This caricature
of a sobbing and whining apologist is a misrepresentation of the facts. Mr. Bethrick fails to understand
the point that since atheism has no proof that God does not exist, and since it can only exist in an
evidential and theistic vacuum, and since it has not refuted all evidences, it must therefore be held, at
least to some degree, by faith. This is perfectly logical. Of course, Mr. Bethrick does not afford the
same consideration to Christians as he does to himself or up other atheists. If a Christian were to
present evidences i.e., the biblical accounts of Christ and his resurrection, etc., these cannot be
accepted due to the atheists non-God presupposition and therefore the atheist judges the Christian as
irrational -- even though the Christian can offer evidence. But the atheist, at least Mr. Bethrick, who
has no evidence or logical proofs for his position of atheism, is considered rational. Again, I cannot

perceptual judgments and such are the axioms. (Has Objectivism Been Refuted?)Thus Slick's own endorsement of
the "proof or faith" dichotomy, like Robbins, is simply a ploy in the attempt to discredit non-believers.
help but notice that he does not provide any evidence for the truth of atheism. All he is doing is trying
to unravel a paper written exposing the fact that atheism has no proof.

Slick then makes the charge that "atheists must go on the attack and negate any evidences
presented for God's existence in order to give intellectual credence to their position." Exactly who is
"on the attack"? Atheists in western society are not a new thing per se, but their freedom of
expression is relatively new, thanks to secular rights-affirming philosophy and documents like the US
Constitution. Slick ignores the fact that, historically, atheists in western cultures have for the most part
found themselves in a predominantly theistic society where god-belief is the norm and atheism has
been vilified and discouraged, even at the level of the state, sometimes to the point of intolerable
persecution. If anyone has been "on the attack," history shows that it has not been the atheist, but
those theists who believe they are charged with the mission of converting the world. To say that
"atheists must go on the attack" is symptomatic of Slick's myopia on this larger context of the matter.
Apparently he considers it to be an affront to him personally when people do not accept his god-belief
claims.

Mr. Bethrick again commits the logical fallacy of the red herring. In other words, instead of
addressing the issue at hand he introduces something off-topic. He quotes where I said that atheists
must go on the attack and negate any evidence presented for God's existence in order to give
intellectual credence to their position. This is true as is demonstrated by Mr. Bethrick himself. But,
instead of addressing the issue of the atheists attack, he then mentions the US Constitution, history,
persecution and other emotionally laden concepts instead of dealing with the issue. He concludes this
paragraph with yet another emotionally heavy, ad hominim sentence when he says "apparently he
considers it to be an affront to him personally when people do not accept his god-belief claims." The
truth is that contrary to Mr. Bethrick's faulty guesswork. I do not consider it a personal affront when
someone does not believe in God. I have friends who are atheists and we get along fine. I am not
offended by their atheism and I do not hound them about their position. The problem with Mr.
Bethrick is his lack of concentration on the issues and his introduction of emotional issues not related
to the subject.
Personally, and this is my opinion, I have seen this with atheists before and I believe that it is
because their position is weak and they cannot substantiate it with logic and evidence. The only thing
they have left to do is exactly what I have said before and that is to attack theistic proofs and
evidences. In this case Mr. Bethrick is attacking my paper which attacks atheism. This is nothing new.

And, precisely what does Slick take to be "evidences presented for God's existence"? Are those
"evidences" the same as those which theists defending a different god present in defense of their god-
beliefs? And are atheists necessarily acting "on faith" if they find the "evidences" proposed to support
the claim that a god exists insufficient to the task? If one were to claim that the moon is made of
green cheese and presented "evidences" for this claim, would Slick be acting "on faith" if he found
those "evidences" insufficient to establish this claim and thus did not accept it as truth? What exactly
does Slick consider 'faith' to mean? And does he not recognize that the Bible, which nowhere
advocates rationality, claims all its "truths" as a matter of faith? 173

If Mr. Bethrick had simply bothered read a little more of my web site in the atheism section under
proofs for God's existence, he would have his question answered. Then, being more educated on the
topic at hand, he could have addressed that issue instead of asking questions which are already
answered in the atheism section on CARM.
In addition Mr. Bethrick is slowly sliding into more and more irrationality in his arguments. The

173
CJ Holmes has prepared two lengthy discussions about the nature of 'faith' as it is both defined and used in the
New Testament. In Some Comments About 'Faith' Part I, he discusses the various ways one can interpret Hebrews
11:1, which is supposed to serve as a definition of 'faith'. In Some Comments About 'Faith' Part II, he responds to
a Christian apologist who attempted to defend the idea that faith is compatible with reason, and in so doing he
points out the exceedingly problematic fact that the definition for 'faith' provided in Hebrews 11:1 is insufficient
given the many examples of faith in the gospels which portray faith as a means of conforming nature to one's will.
It is doubtful that someone who is so confessionally motivated as Matt Slick of CARM.org would accept any of
these points, even though they must be contended with if one wants to defend the idea that biblical faith is in any
way rational.
moon-made-of-green-cheese comment does not logically follow. Furthermore, to say that the Bible
nowhere advocates rationality is a statement of his ignorance. The Bible says in Isaiah 1:18 “Come
now, and let us reason together,” Says the Lord..." reason deals with logic. God wants us to be logical.
Mr. Bethrick is proven wrong on this point and has demonstrated that he does not know the Bible very
well - yet he condemns it.

Slick writes, "If they [i.e., atheists] can create an evidential vacuum in which no theistic argument
can survive, their position can be seen as more intellectually viable." Why would one have to "create
an evidential vacuum" in order to show why theism is irrational? One merely needs to show why the
notion of a universe-creating, reality-ruling god is a terminally invalid idea to show that belief in such a
being is wholly contrary to reason. Theistic arguments quite literally invalidate themselves before they
even attempt to get off the ground.

The logical reason why someone would need to create an "evidential vacuum" in order to show why
theism is irrational is because if there is evidence that there is a god, that would disprove atheism.
Therefore, atheists don't want theistic evidence to stand lest their position fail and atheism is be
proven wrong. This is a simple matter of logic and Mr. Bethrick, apparently, missed it.
Also note that Mr. Bethrick, yet again, offers an unsubstantiated claim. He says, "Theistic
arguments quite literally invalidate themselves before they even attempt to get off the ground." I
have seen no demonstration of this by Mr. Bethrick. He simply makes the assertion without logic or
evidence and then goes on as though his self-assumed point is true. This is not how debate nor logic
works.

Slick thinks that there is "only one way that atheism is intellectually defensible and that is in the
abstract realm of simple possibility." By this, he is referring to the supposition that "it is possible that
there is no God." This, however, Slick holds, is not sufficient to disprove theistic claims since "stating
that something is possible doesn't mean that it is a reality or that it is wise to adopt the position." Slick
gives an example to demonstrate his point. He reasons:

If I said it is possible that there is an ice cream factory on Jupiter, does that make it
intellectually defensible or a position worth adopting merely because it is merely a possibility?
Not at all. So, simply claiming a possibility based on nothing more than it being a logical option
is not sufficient grounds for atheists to claim viability. They must come up with more than "It is
possible," otherwise, there really must be an ice cream factory on Jupiter and the atheist should
step up on the band wagon and start defending the position that Jupiterian ice cream exists.

Ironically, the very point which Slick is making here is one which works against his own
commitment to theism in two fundamental ways. For one, this same objection can serve adequately to
parody Slick's god-belief in order to show just how unstable it is. For, simply by saying that it is
possible that there is a god which created the universe, does that make it intellectually defensible or a
position worth adopting merely because it is suggested as a possibility? Slick should agree with himself
here: "Not at all." And he provides the reason why: "simply claiming a possibility based on nothing
more that it being a logical option is not sufficient grounds for [theists] to claim viability." Indeed, it's
even worse if the proposed possibility in question does not have the advantage of being "a logical
option," but turns out to be an idea completely antithetical to the very foundations of logic.

What Mr. Bethrick fails to understand is that we Christians, myself in particular, do not simply offer
an evidential-less and non-rational reason for God's existence. I have never stated that believing in
God is sufficient simply because believing in God is sufficient. I have no problem with producing logical
proofs for God's existence nor do I have any problem defending the reliability of the biblical accounts
of God's miraculous works in the person of Jesus Christ. Simply go to the atheism section for proofs
and the Bible section on CARM for biblical evidences. So Mr. Bethrick fails to take this into account --
again.

And notice something else: Slick is obviously assuming the primacy of existence principle in his
reasoning here: he is assuming that reality does not conform to one's desires or hypotheses. And he's
right - reality does not conform to consciousness. However, it is this very principle, ironically, which
tells us why god-belief is irrational, since it is this principle which god-belief essentially contradicts. 174
Rather than bolstering his overall case, he simply points to the very principle which one must both
assume and deny whenever he makes the claim that a god exists, since the notion of a god is squarely
planted on the contradiction of the primacy of existence principle, which is the primacy of
consciousness.

Back the truck up. I have made no assumption that reality does or does not not conform to my
desires or hypotheses. His insertion of an irrelevant and unsubstantiated comment further invalidates
the ability of Mr. Bethrick to validate his position of atheism. Seriously, what relevance is this to the
issue of the viability of atheism? It would be far better for him to actually tackle the issue at hand
instead of inserting into the argument things which I have not stated nor assumed. Is he at a loss for
logical response and so needs to introduce irrelevant material?

Moreover, Slick's entire analogy is wholly misapplied in the context of the debate on the existence
of a god. He wants to characterize "the atheistic position" with one asserting the existence of an ice
cream factory on Jupiter. However, it's more likely to be the other way around: the theist is the proper
analogue of the hypothetical somebody claiming the possibility that an ice cream factory exists on
Jupiter, and the atheist is the who points out that merely asserting the existence of a god is not
sufficient to show that assertion to be true. The theist, like the one making a claim about an ice cream
factory on Jupiter, is making an existentially positive claim, for he is the one saying that somewhere a
particular something exists. The atheist is making no such claim; he simply does not accept the
existentially positive claim which the theist asserts. Ironically, Slick not only inadvertently makes the
atheist's case all the more simple to grasp, he also weaves the rope to hang himself in the process.

The truth is that it is Mr. Bethrick's missed application of the information that is demonstrated here
in his paper. Atheism is a claim. Atheism is not a "non position." A non position has no existence. The
atheist has a position called atheism which he can define and try to defend. How do you defend a non
position? Mr. Bethrick is doing nothing more than dodging the real issue and attempting to introduce
concepts not dealt with nor addressed in the original paper. Additionally, he has repeatedly based
comments upon what he thinks I believe or know or assume. At least, he has admitted some
subjectivity in a few of his comments in this regard. But it is unfortunate that he cannot retain such
subjectivity in regards to his unprovable atheistic position.

But the point which Slick should be considering is not whether something is possible simply at the
suggesting thereof, but whether or not he can prove the negative. Does Slick believe that "it is
possible that there is an ice cream factory on Jupiter"? If he does, how would he substantiate this
belief? If he doesn't believe this supposed possibility, how does he rule it out? If Slick accepts the
proposed possibility that a god exists, why wouldn't he accept the proposed possibility that an ice
cream factory exists on Jupiter? He does not argue in an attempt to prove that there is no such ice
cream factory on Jupiter; rather, his whole concern is to point to reasons why one can justifiably
dismiss such claims.

I am actually amused (no disrespect meant) at the attempts to try and turned the ice cream
factory on Jupiter illustration around against me. Mr. Bethrick needs to read the context again.
Atheism, like an ice cream factory on Jupiter, is an intellectually possible position if we were to assert
that basically anything is possible. But being possible does not mean that it is probable, let alone an
actuality. That is the point of the ice cream factory on Jupiter. Atheism has no proof for its position. It
has no evidence to substantiate itself. At best all it has is attempts to invalidate theistic proofs and
evidences. That is it. That is all there is for atheism. Aside from the mere intellectual "possibility" that
there might be no god and existence, atheism doesn't have much going for it any more than there is
the intellectual "possibility" of an ice cream factory on Jupiter. And, as I said before, trying to claim
"non belief" or "lack of belief" concerning God has its problems as well. For this, see my paper
responding to the atheist position of "I lack belief in God."

For details on why this is the case, see Anton Thorn's How the Theist Checkmates Himself and How the Claim
174

"God Exists" Contradicts Itself.


Slick points out that "there is another problem for atheists." That problem, he holds, consists of the
following: "Refuting evidences for God’s existence does not prove atheism true anymore than refuting
an eyewitness testimony of a marriage denies the reality of the marriage." How are the two situations
which Slick has in mind here at all analogous? In the case of a disputed marriage, what is being
debated is a contract between two people. This dispute can be settled simply by asking the couple
involved. But is there any dispute that either party of the marriage in question exist?

But even more importantly, Slick simply points to the evasive nature of his god-belief commitment.
Where earlier he expected proofs of god's non-existence from atheists, he now admits that he would
not allow himself to accept any such proofs as conclusive. In other words, Slick is simply announcing
that he intends to believe that there is a god regardless of the rational merits (supposing there are
any) of his particular god-belief. As Richard Robinson points out, the essence of faith is "the
determination to believe that there is a god no matter what the evidence may be." 175 In other words,
no matter what criticisms are brought forward against his god-belief, and no matter how irrational his
god-belief turns out to be, Slick is determined to believe anyway. So the ultimate question amounts
to: what relevance does argument have to Slick's god-belief in the first place?

It is Mr. Bethrick who is being evasive about his atheist belief commitment. He has not
demonstrated the intellectual viability of atheism at all -- and that is what the paper was about. Mr.
Bethrick has not invalidated the original paper nor validated atheism. Furthermore, he makes even
more inaccurate statements when he says "Slick is simply announcing that he intends to believe that
there is a god regardless of the rational merits (supposing there are any) of his particular god belief."
What is that? I did not announce anywhere in the original paper that I intended to believe in God
regardless of rational merits. Again, this is another fabrication on the part of Mr. Bethrick in his
attempts to weaken the paper. It is a faulty method of defense and a faulty method of attack. But
the fact that he has introduced innuendo without substantiation only weakens his arguments.
He quotes a Mr. Robinson who says that faith is "the determination to believe that there is a god
no matter what the evidence may be." I do not know who Mr. Robinson is, but I do not agree with
what his definition of faith is and I know no Christian who would adopt such a fallacious definition. I do
not believe contrary to evidence nor rationality. But, this is not something that Mr. Bethrick seems to
care to admit. Instead, Mr. Bethrick has only quoted a definition of faith that agrees with his
premises. Since the definition is faulty it is a straw man argument that he is trying to establish.
Mr. Bethrick then goes on to say then I am determined to believe no matter what the criticisms
and evidences are. Since I have already stated contrary to that and since I have already commented
upon the lack of, shall we say, mind reading ability on the part of Mr. Bethrick, I will leave it to the
reader to discern whether or not he is being rational in his argumentation or if he is merely inserting
his own prejudices and emotions into the subject in an attempt to dismantle my paper.

Slick reasons, "Since atheism cannot be proven and since disproving evidences for God does not
prove there is no God, atheists have a position that is intellectually indefensible." But where does Slick
prove that "atheism cannot be proven"? How does Slick establish this claim? All he does is assert it in
one form or another. But assertion does not equal proof. Besides, Slick is again missing the point by
treating atheism as if it were a positive claim which needs to be proven. As has already been shown,
atheism is the absence of a belief, and it is fully justified if the context of one's knowledge does not
support the claim that there is a god.

Again, I have offered the proof that atheism cannot be proven by stating that in order to prove
there is no god one must know all things in all places in all times to be able to determine that there is
no god anywhere. I cannot see how offering an intellectual proof that God does not exist is possible. If
Mr. Bethrick would care to offer some intellectual proof that God does not exist, I would be more than
happy to examine it. But since neither he nor any other atheists have provided such proof, as far as I
have seen, and since he cannot know all things in all places in the universe, I assume that no proof for
God's non existence exists. Perhaps I have assumed too much, but since the subject is atheism and its

175
Religion and Reason," An Atheist's Values, pp. 113-123, quoted in Holmes, Some Comments About Faith Part I.
viability, and since Mr. Bethrick has chosen to defend its viability by attacking the paper, I await his
proof that there is no god.
I have tackled the "absence of belief" issue on CARM already under the paper titled "I lack belief in
God" and attempted to demonstrate in the paper that any atheist who openly states that he lacks
belief in God and in so doing negates proofs and/or evidences for God's existence is in reality believing
that there is no God since his actions reflect his belief system. If someone has a lack of belief in
something, then his actions would be consistent with that. I lack belief in the existence of screaming
blue ants from Venus. And because I lack belief in them, I do not try and defend the position that I
lack belief in them nor do I go around announcing to people that I lack belief in screaming blue ants
from Venus. Unlike Mr. Bethrick, I am not try to prove or substantiate a negative position or "non
position" about something. In other words, his actions speak louder than his words as he claims that
he lacks belief in God yet behaves as though he believes there is no god.

Slick then admits that "atheists can only say that there are no convincing evidences for God so far
presented." But if that's the case, then what precisely is Slick's fuss? If a particular non-believer holds
that the "evidences for God" are not convincing, does Slick think that the non-believer should believe
anyway? If a person is honest to himself and finds, after reviewing arguments and "evidences"
proposed on behalf of proving one's god-belief, that those arguments and "evidences" are insufficient
to convince him, he will acknowledge that believing the claim that there is a god would be irrational.

I have no fuss. Mr. Beth Frank does not challenged my comment the atheists can only say there
are no convincing evidences for God. That is all atheists can really say in spite of what the evidence of
the logic might be. Remember, the atheists presupposition will not allow him or her to seriously
entertain the possibility of does existence lest he undermine his own belief system; namely, that there
is no god. Again, let me say that I am far from convinced that the "lack of belief "position of atheists is
anything more than an attempt to get around the weakness of their position. Logically, if they said
there was no god, and they would be at a great loss to prove their position. If they believe that there
is no god, this opens them up to further cross-examination. So, atheists tend to say "I lack belief" as if
to say that they have no position at all about God. But, as I have stated elsewhere on Carm, in the I
lack belief paper, if the atheist lacks belief and why is he so adamant about attacking theistic proofs?
His behavior is that he believes there is no god and works to substantiate the belief.

When Slick points out that atheists "cannot say there are no evidences for God because the atheist
cannot know all evidences that possibly exist in the world" [sic], he overlooks the fact that an atheist
can justifiably say that he does not know of any legitimate evidences for god. And, given that the
same "evidences" are offered in support of claims that different gods exists, it is hard to see how any
"evidences" can be considered legitimate. Again, knowledge and belief are hierarchically and
contextually dependent upon prior knowledge and beliefs, reducing ultimately to one's starting points
(assuming one is systematic about the content of his mind). If one is not aware of any "evidences for
God," he is certainly justified in not accepting the claim that there is a god. If one has been presented
with "evidences for God" and, after examining those evidences, concludes that they are insufficient, he
is justified in not accepting the claim that there is a god. Furthermore, if one can present good reasons
for why belief in a god is irrational, then by all means, he is fully justified in rejecting the claim that a
god exists. Slick does not seem to be aware of these points.

Again Mr. Bethrick makes another mistake. I stated in the paper "Is atheism viable?", "At best,
atheists can only say that there are no convincing evidences for God so far presented." This means,
logically, that the atheist does not yet know of any legitimate evidence for God. That is why I said "so
far presented."
I am fully a ware of the points that Mr. Bethrick has raised in the preceding paragraph and I have
attempted to deal with them in my writing against atheism in other papers by stating that there is a
degree of agnosticism among atheists. What I mean that is that since the atheist cannot know all
evidences, or anti-god evidences, or anti-god proofs, etc., it is logically necessary that there may
indeed be such evidence and/or proofs out there not yet known. Since he cannot know that there is no
god, he can only believe, based upon a "lack of evidences" that there is no god. This is not proof. This
is a measure of faith in knowing you don't know enough; hence, the agnosticism, or the not knowing
for sure.
My position is that the atheist makes a choice to believe that there is no god and/or makes a
choice to hold the position of "lack of belief." I further maintain that logic requires agnosticism rather
than atheism.

Slick then wants to conclude that "since there could be evidences presented in the future, the
atheist must acknowledge that there may indeed be a proof that has so far been undiscovered and
that the existence of God is possible." That would be the case if the what is claimed were shown to be
a legitimate possibility. However, if one recognizes that god-belief is inherently contradictory to reality
and thus irrational, then it would be wrong to think that "there could be evidences presented in the
future" for such belief. Thus I cannot accept Slick's contention that an atheist is really only an agnostic
"since at best the atheist can only be skeptical of God's existence." Clearly this is not the case for
those who embrace reason consistently. One need not "prove that there is no god" nor does he have
to worry about hypothetical "evidences presented in the future." If he does not believe, he is an
atheist.

Mr. Bethrick continues to commit logic fallacies. Here he begs the question. He says "if one
recognizes that god belief is inherently contradictory to reality and thus irrational" as though this is the
truth. He has not established its defensibility nor rationality. He simply stated an "if then" premise that
is unsubstantiated in order to sound more rational. But it is not logically consistent to base the
conclusion upon a premise that is unsubstantiated, which is what he has done here and other places.

Slick closes his little piece with the following statement:

This is why atheists need to attack Christianity. It is because Christianity makes very high
claims concerning God’s existence which challenges their atheism and pokes holes in their
vacuum. They like the vacuum. They like having the universe with only one god in it:
themselves.

While it is the case that "Christianity makes very high claims concerning God's existence," it is not
the case that these claims pose a challenge to atheists. Christianity is long on claims, but short on
proofs, and even shorter on rationality. Slick seems to think that the recognition of these facts
constitutes a "vacuum" and reflects a desire to see oneself as a god. However, it is hard to rule out,
judging by Slick's tone of resentment here, that his comments are not motivated by disingenuous
intentions. For not believing in the Christian god does not mean that one necessarily considers himself
to be the Christian god, any more than not believing that Zeus exists means that one necessarily
considers himself to be Zeus. If Slick could achieve any consistency in applying the principles which he
himself wants to throw around in his rant against atheism, he might begin to see some of the holes in
his own faith commitments.

If the claims of Christianity posed no challenge to atheism, then why are atheists constantly
attacking the Bible and Christian theistic proofs? Take this very paper that I am answering. Mr.
Bethrick is going to great lengths to "refute" a challenge that he says is no challenge. If it were no
challenge, then why is he tackling it?
Again, how does Mr. Bethrick justify his mind-reading abilities by stating my "tone" and my
"resentment" and my "disingenuous intentions." I say mind-reading because what else could it be?
My emotions are not stated in the paper. He must either guess or manufacture information to bolster
his position. Either way, he is failing to stick to the issue at hand as well as failing to present a logical
defense of the viability of atheism.
Perhaps it is Mr. Bethrick who needs to examine what true rationality is. I do not know what he
thinks, since I cannot claim the ability to read minds. However by his claiming that belief in God is
irrational, he must by necessity also condemn to the ranks of irrationality the likes of Einstein, Galileo,
Isaac Newton, Pascal, etc. who all believed in God. Who is Mr. Bethrick to state what is and is not
rational in light of the intellectual giants of history who have believed contrary to what he claims is
true rationality not to mention the fact that he has been irrational in his paper by playing the mind-
reader?
In conclusion, the existence of God is not established nor verified by whether or not intellectual
giants of history have or have not believed in God. To make the assertion is a logical fallacy. I simply
stated that those who are rational and brilliant have indeed believed in God. I am far from brilliant, but
I do not consider myself to be irrational. My belief in God is based both on experience, logic, and
evidence. My presupposition allows me to examine evidence even against God's existence. As I said
earlier, if it can be reasonably demonstrated that Jesus did not rise from the dead I would give up
Christianity. This is not irrational faith. On the contrary, it is faith based upon evidence and rationality
as well as biblical revelation.
And finally, Mr. Bethrick has not supported atheism in any way.
"I lack belief in a god"

The statement “I lack belief in a god” is becoming a common position of atheists. In discussions
with them, they tell me they lack belief in God the way they lack belief in invisible, pink unicorns. In
other words, they have no position, take no intellectual action, have no “belief or unbelief” on the
matter concerning God. To them it is a non-issue. Though this may sound sensible to some, the
problem is that once you are introduced to an idea you cannot stay neutral about it. You invariably
make a judgment about an idea once it has been introduced to you. You can brush it off as ridiculous,
ponder its possibility, accept it, reject it, or do something in between. But, you cannot return to a
“lack of belief” position if “lack of belief” is defined as a non-intellectual commitment or non-action
concerning it. Though I admit that an atheist can claim he lacks belief even after being exposed to an
idea and contemplating its rationality, I still assert that a position of some sort is required.
Let’s pick a baby that has no awareness of the concept of invisible, pink unicorns. Later in life,
when the baby is mature and is introduced to the concept, he either accepts the existence of invisible
pink unicorns, rejects them as a ridiculous notion, chuckles about it and dismisses it, becomes unsure
about them, holds off judgment until later, etc. Either way, he develops a position on the concept of
invisible pink unicorns. He has to do something with the concept once he’s been exposed to it. He
doesn’t continue in a lack-of-belief or a lack-of-awareness state of mind because the fact is, some sort
of intellectual action occurs in regard to it. He cannot become unaffected by the concept. He has been
made aware of it and he, by default, does something with it.
Nevertheless, some might say that to hold off judgment until later is to be "atheistic" concerning
pink unicorns and therefore support the atheist position of "lack of belief." But, as I said earlier, after
being exposed to a concept a decision is made about that concept even if it is to withhold judgment.
In other words, an assessment has been made and a position taken. This is not the same as going
back to a state of unawareness. To suspend belief on a subject is to hold off judgment until more
information is acquired. This is agnosticism, not atheism. It is an admission that not all information is
acquired thus logically requiring the possibility of the existence of the thing being considered. This is
something atheists do not do by definition, but agnostics do. Agnosticism is the position, in part, that
"suspension of belief" is maintained until further information is acquired.
If I said that there was an ice cream factory on Jupiter, what would you think? Would you
entertain the idea as a serious possibility? Would you quickly dismiss it as an outlandish absurdity?
Would you request evidence for it? Or, did you suddenly have a desire to go to Jupiter for some
Jupiterian Swirl? Of course, an ice-cream factory on Jupiter is ridiculous and we automatically know
this so we naturally make a judgment on it. Thus, we cannot remain in a state of “lack of belief”
concerning the concept once we’ve been introduced to it. We assign it to the “that is ridiculous”
category.
This is why the lack of belief defense of atheists is not logical. It ignores the reality that people
categorize concepts anywhere in the range of total acceptance to total rejection. It is our nature and it
is the nature of the human mind.

Is my cat an atheist?

Animals lack belief in God. Are they atheistic? Should we include atheists and infants and plants
and rocks and water and air in the category of atheism since they too lack belief in God? Of course
not. It would be ridiculous to include animals in the category of atheists.
I had a cat named “Punchface.” (It’s a long story.) Punchface was a beautify cat with long white
hair and powder blue eyes. He was very smart, even brilliant. He could play tag, fetch, play hide and
seek, catch mice with Olympian skill, and enjoy an evening of watching Star Trek with me. I would
defend completely the fact that he had quite a personality. As brilliant as my cat was, he lacked belief
in God. I could have sat him down, looked him in the eye and said, “Punch, there is something I have
to talk to you about. It’s God. You see, God is the being that created the universe and everything in
it, including you and me.” Of course, after I would say this, Punch would probably be chasing a piece
of air-born lint. He had no concept whatsoever of God. Does that mean my dear cat Punchface was
an atheist? Of course not. He was only a cat, even if he is a brilliant one.
Nevertheless, the atheist will assert that the position of "lack of belief" relates only to sentient
beings. This would be a necessary position given that cats cannot be atheistic; that is, they can't
make a choice to accept or deny God's existence. Therefore, the atheist should amend his statement
and say something like "As a person, I lack belief," or "I have decided to lack belief in God," or
"Lacking belief in God is a position for sentient beings only." This would negate my cat as being
included since to describe an atheistic position as simply "lacking belief" is too broad.

So what is this position of “lack of belief” really about?

In my opinion, “lack of belief” is really an attempt by atheists to avoid facing and defending the
problems in their atheistic position. You see, if they say they have no position, by saying they lack
belief, then their position is not open to attack and examination and they can quietly remain atheists.
The problem for atheists, however, is that atheism is coming under more serious attack by
Christians and others who recognize its problems and are exposing them. Without a doubt, there are
far more people in the world who believe in God (or a god) than don’t and more and more Christians
are tackling atheism as an untenable position. The majority belief doesn't make it right, but the
increase of examination of atheism has made it more difficult for atheists to defend their position. This
also explains why atheists seem to becoming more aggressive in their attacks on theism in its different
forms. There is an intellectual battle being waged and both defensive and offensive measures are
being taken on both sides. In the end, the truth will be known.
Response to criticism of "I lack belief in God"

Following is my response to the first portion of an atheist's critique of two of my papers dealing
with atheism. His original criticism was one page, but I have broken it up into two pages relating to
each paper he addressed.
The article was posted on infidelguy.com, an atheistic website, and that is the only reason I am
responding to the paper which, in my opinion, does not present its case very well. Nevertheless, I
have copied the entire article with the author's permission and reproduced the the two halves, one
here, here so that it can be more easily address. His original comments are in black, and my
comments are in italics. I have left his typo's and grammar errors intact.
_________________

Donny Kay Lonovy's


Refutations of Matt Slick's Articles on Atheism

This essay is a refutation of Matt Slick of CARM.org's "I lack belief in God" and "Is atheism viable?"
articles. As an atheist, I find his position to be idiotic and even downright offending. I'm going to show
you exactly why he is an idiot when it comes to atheism, point by point. Read his papers along with
mine to understand what I'm saying, as the paragraphs skip from point to point and wouldn't seem as
coherent otherwise.

Unfortunately, this individual begins his paper with an ad hominem attack. Name-calling is
generally considered a poor way to make a point. He should address the issues and not insult the
person.

"I lack belief in God"

This is my refutation of Matt's "I lack belief in God"article. It is fallacious and misinformative to the
core.
Let us first get our definition of "atheism" straight. Atheism is a lack of belief in any deity, which
means an atheist lacks belief in God. Matt got this point right, but he doesn't seem to understand that
lacking belief is not holding off on having a position. Once you are exposed to the concept, you do
have to make a choice, which he also got right. To say that you lack belief in God is to say that you do
not believe He exists. It's the same thing. You have taken the position that you believe God does not
exist if you choose not to believe he exists. It's that simple. You have a position if you are an atheist
after being exposed to theism.
Matt set up a straw man and tore it down by confusing the concept. You can disregard all of the other
points he makes in that section because it was based on that straw man.

Quite honestly, I am not sure what his point was. He said I got several points correct. But, to say,
"lacking belief is not holding off on having a position" is a confusing statement. "Not holding off" is
essentially a double negative. Is he saying he then does have a position because of his lack of belief?
Or is he saying he does not have a position? He says I set up a straw man but I cannot determine
what that alleged straw man really is.

"Is my cat an atheist?"

Your cat is an atheist by definition. All things which cannot understand the concept of a deity are
atheists. The false premise here is that they have never been mentally exposed to the concept of
theism. They can't understand it. They have made no choice to remain atheists. The difference is that
mentally mature humans who are exposed to theism are capable of making the choice to remain an
atheist. A cat can't make this choice...as far as we know.
Matt Slick tried to use this, another straw man, to attack atheists. He is wrong again.
To say that "All things which cannot understand the concept of a deity are atheists" is not logically
necessary. It all depends on which definition of atheist you want to go by and what level of
understanding is to be had regarding deity. According to the Oxford English Dictionary, 2nd edition,
Atheism is "disbelief in, or denial of, the existence of a god." There can be a wide range of meaning
held by this definition. As far as deity goes, there are a host of different definitions in the world on
what constitutes deity. Therefore, Mr. Lonovy cannot rightfully make his claim since he has not
adequately defined nor established his position.
If a cat can't make the choice to accept God or not, then perhaps this atheist might want to refine
his definition of atheism since, rocks cannot understand the concept of a god either. Are they atheistic
as well? Are we to include water, rainbows, and clouds as members of the atheist community as well?
Obviously this "lack of belief" position is inadequate to sufficiently define atheism.

"So what is this 'lack of belief' really about?"

Lacking belief is not some scapegoat technique to get out of defending the atheistic position. It's
what an atheist believes by definition. Atheists do have a position and defend it very well.

My assertion about "lack of belief" is found in my original paper. I will reproduce the first paragraph
from my paper "I lack belief in God".

The statement “I lack belief in a god” is becoming a common position of atheists. In discussions with
them, they tell me they lack belief in God the way they lack belief in invisible, pink unicorns. In other
words, they have no position, take no intellectual action, have no “belief or unbelief” on the matter
concerning God. To them it is a non-issue. Though this may sound sensible to some, the problem is
that once you are introduced to an idea you cannot stay neutral about it. You invariably make a
judgment about an idea once it has been introduced to you. You can brush it off as ridiculous, ponder
its possibility, accept it, reject it, or do something in between. But, you cannot return to a “lack of
belief” position if “lack of belief” is defined as a non-intellectual commitment or non-action
concerning it. Though I admit that an atheist can claim he lacks belief even after being exposed to
an idea and contemplating its rationality, I still assert that a position of some sort is required.
[emphasis added to my quote]

Whether or not atheists defend their "lack of belief" very well or not is a debatable issue. But, this
really shouldn't be a battle of opinions. Rather, we should be using logic and evidence to persuade.

Matt Slick is trying to trick the readers of his article into believing that atheists really are liars who
have no true basis for what they believe. He claims that Christians who see it's problems are attacking
it. From what I've seen, atheism is coming under attack by Christians who don't understand it or
science. They aren't recognizing it's problems. They're making false claims about it, then attacking
their own misconceptions. Yet they still continue to do this when they've been proven wrong. Now, tell
me why I believe most Christian debaters are liars.
There are more people who believe in God in this world than there are atheists. Sadly, this is true.
Does that make them right? No. Using the number of people who believe in God to your advantage is
pathetic. If you need to use that, then you really are out of good arguments.
Atheists do defend atheism by attacking Christianity and any other theistic religion that trys to convert
them. Does this mean that they can't defend their own position? No. Atheists use science and logic to
prove their point. There is no place left for God to have done anything in the universe that wouldn't
have occured anyways, except for at the Big Bang, which we don't fully understand yet. There's some
science for you. God can't be all of the "omnis" at one time, as they contradict each other. Actually,
being omnipotent is a logical fallacy all by itself. There's a logical point. Matt Slick is, once again,
wrong.

First of all, Mr. Lonovy has erringly stated that I am trying to trick people. He is either trying to
read my mind or extrapolate my alleged deceptive intent. Of course, I am not trying to trick anyone
nor am I trying to convince anyone that atheists are liars. I do, however, maintain that atheism is, to
a large extent, a matter of faith and not evidence with atheists -- but that is another subject. Mr.
Lonovy states that I am making false claims about atheism, but he does not mention what they are.
Mr. Lonovy continues and makes several unsubstantiated claims. For example, he says, "Atheists
use science and logic to prove their point. There is no place left for God to have done anything in the
universe that wouldn't have occurred anyways..." Apparently, Mr. Lonovy is unaware of the logical
flaw of "begging the question." That is, he assumes the thing to be true that he is trying to prove. He
assumes there is no God and then says that all things that exist in the universe could have occurred
without God. He assumes that naturalism is true yet provides no evidence for it nor does he give a
logical reason why there is no God. He just makes assertions and continues on them.

I find this part of his paper to be very offensive. I hope any informed atheist would too. I've shown
that he is wrong, and I hope that anyone who reads this will urge him to remove his fallacious article.

Since Mr. Lonovy has not established logically anything wrong with what I have said and since he
has only given unsubstantiated opinions, I will assume that he simply has a chip on his shoulder and is
venting his frustration and that is why he is offended.
Another response to criticism of “I lack belief in a god."

Following is another response to my paper dealing with "I lack belief in God." I have reproduced
his paper here and responded to it. His original comments are in black, and my comments are in
italics.

----------------

Matt's first point is that once we are exposed to a claim it is more or less impossible to stay
completely neutral to it, and that therefore the claim by atheists that they merely lack belief isn't
credible.
This is fine as far as it goes, but I believe it is an oversimplification of the true atheist position
which includes a default belief that X is not true, where X is unparsimonious. That is, the atheist
rejects claims about God in much the same way that we reject claims about an ice cream factory on
Jupiter, as being extraordinary claims that are not supported by evidence.

The atheist has admitted that he cannot remain neutral on a position once he has been exposed to
it. But, how is it an oversimplification? It is either true or it is not (Is that too, an
oversimplification?). Since there are many who claim to be "true atheists" yet contradict each other
on exactly what atheism is, and since he has not defined what true atheism is, it isn't possible to fully
delve into his position. Nevertheless, basing atheism on a lack of evidence for God's existence has
been dealt with in my paper "I don't' see any convincing evidence for the existence of God."

I can truthfully say that I believe God does not exist, but it is more accurate for me to say I have
nowhere near sufficient positive reason to believe in God, and that, lacking belief in God, I hold to the
default position that God doesn't exist.

If he believes that God does not exist, then his atheism is a belief system since he is holding a
belief that classifies him in a position. He said he holds to the default position that God doesn't exist.
I'm not sure if he means that he holds that position because of lack of evidence or that atheism is the
default position. If the former, the like above dealing with evidence is worth reading. If the latter,
then he is simply assuming something that cannot be proven; namely, that atheism is a default
position. No one knows what is automatic within the minds of people at birth. If atheists assume that
non-cognition (non intellectual contemplation of God) is atheism, then infants, by definition, would be
atheists. But that would also make cats, rocks, and chairs into atheists. Therefore, the latter
definition of atheism would be insufficient.

Is this sophistry? No. Rather, it reflects my thought process and my assessment of the burden of
proof. Positive belief in something extraordinary must be established before I will accept it; lacking
evidence or reason for such belief, I reject it.

It is not as simply as providing sufficient evidence. A person's presuppositions strongly effect what
evidence is accepted and how it is interpreted. If a person has an atheist presupposition, then is it
possible he can be objective in his examination of theistic evidence? That would depend on the
person, but convincing such a person would be a difficult task at best.
Various evidences have been offered to atheists. Some have been convinced and converted to
theism. Others have not. Therefore, it is not the evidence that is the issue, it is the atheist. In other
words, different people see things differently. What is convincing to one person is not convincing to
another. This is why presuppositions are so important. To someone who "will not see", no evidence
will be sufficient.
Finally, this person has not even offered any criteria by which evidence might be assessed. What
would be sufficient evidence and why? If this person cannot describe logically what that would be,
then he hasn't thought through his atheism sufficiently and really hasn't any right to make the claims
that the evidences offered aren't good enough.
Describing my position as a lack of belief is sometimes helpful in explaining my position to theists.
When someone demands that I prove God doesn't exist (or at least provide evidence that God doesn't
exist), it helps to show that my position is not based solely on evidence for and evidence against a
particular God concept but more fundamentally on the extraordinary nature of any claims about God
and my default assumption about any such claim.
Finally, distinguishing between my lack of belief in gods in general and my specific disbelief in
particular God concepts can also be helpful in conversation. I lack belief in an omnipotent God because
that is an extraordinary claim I do not believe is supported by the evidence. I disbelieve in any concept
of a non-deceiving God who created the Earth 10,000 or fewer years ago because I believe the
observable data strongly contradicts this age.

It seems that this atheist has missed the point of my paper on "I lack belief". I stated that once a
person is exposed to a concept, he categorizes that subject. He does something with it. Furthermore,
I stated that if "I lack belief in God" means that the person has no mental assertions either positive or
negative concerning God, then that is an illogical position because people do things intellectually with
information and concepts. Even this atheist at the beginning of his response stated he cannot remain
neutral on a position once he has been exposed to it. Yet he wants to hold a "lack belief" system.
This is inconsistent.
Also, this atheist fails to understand that not all Christians, myself included, believe in a young
earth. I believe the earth is old. What he has done is narrowed the field to far and not allowed for
other positions on an issue that he has stated negates belief in God(s). This is illogical to do and
unfortunate.

Unfortunately, Matt then shifts from attacking the position to ad hominem, attacking the
motivations of atheists.
Matt: “Lack of belief” is really an attempt by atheists to avoid facing and defending the problems in
their atheistic position. You see, if they say they have no position, by saying they lack belief, then their
position is not open to attack and examination and they can quietly remain atheists.
The irony of this is that the great majority of atheists on these boards who describe their atheism
as a lack of belief in gods are quite willing to explain their views in depth, including exactly what they
mean by lack of belief (as I've described here), as well as positive beliefs they do hold. Matt knows
this, so for him to claim otherwise is both an ad hominem and a straw man.

My concern is to not produce ad hominem attacks. They are weak. However, the context of my
comment he quoted above was at the end of the paper where I draw a conclusion after attempting to
demonstrate that the "I lack belief" position is itself weak. Having done that, I draw the conclusion
that some atheists are indeed trying to avoid the weaknesses of atheism in general. Nevertheless, I
have modified the comment on that paper by adding "In my opinion."
Stating what I know and do not know is risky business. My experience on the CARM discussion
boards with the atheists has been that only a few are willing to discuss their "lack of belief" position in
depth for very long. But, this is my opinion.

Matt: The problem for atheists, however, is that atheism is coming under more serious attack by
Christians and others who recognize its problems.
How is this a problem for me or for any other atheist? Matt must presuppose that atheists do not
want to find truth, since if they did any challenge to their position could only be a source of knowledge,
not a problem. More ad hominem?

This atheists asks how is it a problem for atheists that atheism is coming under more serious
attack from Christians? Well, it is a problem in that many of their arguments have been answered,
some of which I've demonstrated on the atheism section here on CARM.
I admit that this is a subjective statement I am going to make here, but my experience with
atheists over the past twenty years is that they have generally moved away from saying "There is no
God" to "I lack belief in God." I'm not the only one who has noticed this shift. Why? Again, my
opinion is that atheists are discovering some weaknesses in their position and are adapting their
atheism (even redefining it in some instances) so as to make it less vulnerable to attack. Now
whether or not this critic wishes to acknowledge this matters little since I am offering my opinion
based on my observations.

Matt: Without a doubt, there are far more people in the world who believe in God (or a god) than
don’t and more and more Christians are tackling atheism as an untenable position.
Irrelevant. If popular vote decided what is true, astrology would work. More interestingly, the sun
would once have revolved around the earth, with the relationship switching sometime in the past few
thousand years.

It is not irrelevant. If it were irrelevant, then why is this atheist responding here? I am one of
those included in "more and more Christians" who are attacking atheism. This atheist has just proved
the relevancy of my comment that atheism is being tackled by Christians.
Furthermore, I did not commit the fallacy of ad populum, that the majority belief is correct. I made
no such statement. I only said that more and more Christians are tackling atheism as an untenable
position. Atheism's validity is not determined by "more and more." It is determined by logic and
evidence.

Matt: After all, how does an atheist defend atheism? He can’t. He has to attack theism in its
different forms. This is why atheists attack Christianity, the Bible, and other religious systems and try
and invalidate them. That really is all they have to go on.
More ad hominem...and simply false. It would be more accurate to say that the atheist doesn't
have to defend atheism rather than that he can't, since atheism is a perfectly reasonable default view.
In any case, Matt would do better to challenge (real) atheist logic or arguments than to mind-read and
tell the rest of us why atheists debate with theists.

It may be that I committed the ad hominem fallacy here. Instead of defending the statement and
after rereading my final comments on that paper, I have modified it to the following.

The problem for atheists, however, is that atheism is coming under more serious attack by
Christians and others who recognize its problems and are exposing them. Without a doubt, there are
far more people in the world who believe in God (or a god) than don’t and more and more Christians
are tackling atheism as an untenable position. The majority belief doesn't make it right, but the
increase of examination of atheism has made it more difficult for atheists to defend their position.
This also explains why atheists seem to becoming more aggressive in their attacks on theism in its
different forms. There is an intellectual battle being waged and both defensive and offensive
measures are being taken on both sides. In the end, the truth will be known.

I thank this atheist for trying to address the paper and for helping me improve it through my
modification. However, I do not believe that he sufficiently addressed all the issues raised in it.
Additional response to criticism of "lack of belief"
and "Is atheism viable"

Mr. Lonovy is a persistent atheist. I suspect that Mr. Lonovy is zealous for his faith of atheism and
will attempt to defend it accordingly. But, since this can go on forever and since I have other things to
do on CARM, I'll probably stop responding after this paper.
My response is in italics. I have again received permission from him to quote him in his entirety for
this work. Also, I have left his typo's and grammar errors intact.

-------------------

I refuted everything I felt needed it in my first essay, but Matt has either misunderstood the whole
essay or dodged every point I made. He acts like he doesn't understand what I was saying, but I know
Matt has a brain. He couldn't misunderstand everything in my essay...I'll leave it to the reader to
decide why he responded the way he did. Was it really THAT hard to understand?
Of course, Mr. Lonovy mistakenly thinks that he had refuted everything in his paper. He did not
and he made several errors in judgment as well as in logic which I pointed out. Nevertheless, Mr.
Lonovy continues to defend his position.
I'd also like to inform the reader that I specifically asked Mr. Slick to inform me if he replied to my
essay, so that I could defend myself. Here is the exact, quoted request for that: Matt: If i decide to do
so, may I have your permission to copy your paper so that I may reproduce it on carm with my
answers to your criticism? I would like to answer it and it is much easier to do if I can reproduce your
paper. Myself: Sure. Just don't change my words or take anything out of context. I'd like to be
informed if you do that, too, so I can defend myself. Thanks.
I never received any notification about his new papers. I found out about them by checking his
site. Thanks alot, Matt. I informed him of my original essay right after it was put up on
InfidelGuy.com. He obviously thought he wouldn't have to deal with anything else from me if I didn't
know.
To be honest, I had completely forgotten about informing Mr. Lonovy about my refutations of his
paper. I have been involved in many debates lately and it slipped my mind. I hope that Mr. Lonovy
would forgive me my oversight.
I'm now going to restate my two main points in an easier to understand manner so that Matt may
understand me, and then deal with everything he said in his new articles. Please forgive the length of
this essay. I'm going to be thorough this time.
One of the issues that Mr. Lonovy needs to work on is his condescension. If he wants to be taken
seriously as an atheist, then he needs to learn how to address the issue and not the individual. His
attacks on my person is known as ad hominim attacks. Unfortunately, far too many atheists adopt this
tactic and it is not productive.
What is atheism?
First, let's define atheism. Atheism is a "lack of belief in a deity or deities". The word itself literally
means 'a': basically 'no', and 'theism': basically 'god belief'. Without, or "lack of", belief in gods. This is
the generally accepted definition among atheists as far as I've seen, and the definition Matt and myself
were talking about. Matt states in "I lack belief in God" that this is becoming a common position
among atheists. No, it isn't. It's the definition of atheism, and Matt was trying to say that "lack of
belief" isn't a position, yet he calls it a position. That doesn't make sense.
Mr. Lonovy defines atheism a little different than the dictionaries I have read. Though these
dictionaries include the "lack of belief" idea, they also add that it is a denial of God, or a doctrine that
there is no god or gods. Therefore, atheism is not always defined as "lack of belief" in God, but also as
the belief that there is no God since there are those who call themselves atheists who believe there is
no God(s).
I said on my section on Atheism under definitions and terms, that atheism is "the lack of belief in
a god and/or the belief that there is no god." Following are some definitions offered by dictionaries.
"Disbelief in or denial of the existence of God or gods b) The doctrine that there is no God or
gods." (American Heritage Dictionary of the English Language, fourth addition, 2000.)
"Disbelief in, or denial of, the existence of a god." (Oxford English Dictionary, Second Edition)
It seems that my definition is more consistent with the dictionaries than is Mr. Lonovy's since I
include his definition as well as the other. Therefore, should I restrict my definition to "lack of belief"
or should I also include in it "the belief there is no god(s)"? Both are offered in dictionaries and both
are held by atheists. Therefore, I will use the broader definition.
Furthermore, I suspect, that Mr. Lonovy actually believes that no God exists. But, that is just my
opinion.
A point in my original essay was to show that "lacking belief" can and is a position with the right
qualifiers, which most every human who calls themself an atheist fit. These three qualifiers are:
1) The atheist must be able to understand the concept of a deity. You can't make a decision or
have an intellectual position on something you can't understand. Matt has a whole section called "Is
my cat an atheist?" about how atheists are no different in position than animals or non-animate
objects, his cat in particular, in that they have no position concerning their lack of belief. I think he
would agree with me about this qualifier, because we are obviously talking about human atheists who
can understand this.
I would like to add that the definition Mr. Lonovy offers above "Atheism is a 'lack of belief in a
deity or deities'," logically includes my cat since he also lacks belief in god(s). Rocks also lack belief in
god(s), etc. But, I applaud Mr. Lonovy for clarifying the definition to include the understanding of a
concept of deity. So, if we were to modify the definition that Mr. Lonovy originally gave according to
this additional information, we might say, "Atheism is the position held by a person or persons that
'lack belief' in god(s)."
2) The atheist must then actually understand the concept of the deity in question. You can't have a
position on something if you don't even know what it is. Matt's part about cats, rocks, etc. has no
bearing on if a human atheist has a position or not. We know what a "god" is, even though it is true
that there is no real definition of what one is among theists, esspecially one that isn't full of
contradiction, but we still have a general concept of it, so we can come to a decision and have a
position.
Actually, my original point was precisely relevant and demonstrated the insufficiency of Mr.
Lonovy's original position on what atheism was. He has demonstrated its insufficiency by clarifying
what he means by atheism. However, there is a problem. Mr. Lonovy claims to be an atheist. Others
make that claim too. But, some atheists believe that there is no god(s). Therefore, the definition of
atheism must be sufficient to include those atheists as well. I offer the following definition to cover all
the bases: "Atheism is the position held by a person or persons that 'lack belief' in god(s) and/or deny
that god(s) exist." I believe my definition is far more sufficient than is Mr. Lonovy's.
3) The atheist must make the CHOICE to be an atheist. Once someone understands what a "god"
is, then he must come to a decision about it: to believe in it or not. Matt did understand that when you
have been exposed to such a concept, you have to make a mental decision about it, and THAT is
where the fact that you have taken a position comes in. Now, let me explain what I mean when I
say it is a position. If you "lack belief" in something, then you don't "believe in" it. To "believe in" a
deity, such as the Christian God, who I will use as my example here, is to believe it exists. If you lack
this belief in God, then you don't believe He exists, right? Now, if you don't believe He exists, then it is
only logical that you believe He doesn't exist. He must either exist or not, so there's no middle ground
there. Let me make an example...
theism=belief=existance
atheism=no belief=no existance
Isn't that clear enough? When an atheist CHOOSES to be an atheist, knowing what he's choosing
not to believe in, it is the position that he believes it doesn't exist. Please remember the quealifiers for
this. It isn't a way out of having a position or an excuse not to defend yourself. It's just being a human
atheist.
Now, Matt claims he didn't understand this point in my original essay. It's all too clear this time. I
apologize if I wasn't thorough enough the first time. I was relying on the reader to put it together and
do the logical math themselves so I wouldn't have to write a very long essay.
I agree that the atheist must make a choice to be an atheist. Logically, this is a position, the
position of atheism. Can you have a position of "lack"? The point I was trying to make in my original
paper was that "lacking belief" in something, if it means to have no position on something, is only
possible for people (or cats) that are unaware of a concept. Once you are made aware of the concept,
then you adopt a position whether it be rejection, acceptance, or waiting until more data comes in.
Either way, you categorize the concept and you have a position.
If someone takes "lack of belief" in God to be a neutral, no committed position, then why do so
many atheists who "lack belief" behave as though they believe there is no God? That is the question I
have been asking. It would seem that a non committed position would be held by those who are
objective. But that does not appear the case with atheists whose position is "lack of belief in God"
because they sure are actively trying to demonstrate that there is no God.

Atheistic proof against the Christian God


What? I can prove a negative? Yes, actually. I can do this by proving a positive that will contradict
the positive claim of Christianity, and therefore render it false...as I said before, in my first essay.
As we all know, the Bible is the only record of the Christian God's existance. Now, what if you
prove something extremely important in the Bible to be false? That would mean that the Bible is not
reliable as divinely inspired truth. That would mean there is no reason to trust it and base your
"eternal fate" on it.
I do not believe that the Bible is the only record of the Christian God's existence. I believe that the
creation of the universe, its order, logical absolutes, etc., are also evidences for the God of
Christianity. Nevertheless, let's see what Mr. Lonovy's proof is.
It would also mean that the Christian God doesn't exist. Why? Because God can only be defined as
being the "God of the Bible". What the Bible says He is, is what He is. What the Bible says He did, He
did. Or the "God of the Bible", the Christian God, doesn't exist, even if some other god exists. But I'm
only dealing with Christianity right now.
This is not logically necessary. You have not declared what the "extremely necessary" thing is. If
the truth of the Bible were based on this "extremely necessary" thing, then perhaps you might be
correct. but that is yet to be established.
So, what is my proof against the Christian God? There are so many logical and philosophical
problems and contradictions in the very concept of this God that I refer you to the Infidel Guy's
Questions About God page. The other evidence is scientific. Let's see about that evidence...
Hold on, the proof that the Christian God is not true is that it is found on an anti-Christianity
website? I would hope that Mr. Lonovy is able to provide this proof here instead of pointing to
someone else's work.
Also, though there are indeed Bible difficulties, they are not without answers. It seems Mr. Lonovy
is only studying one side of the issue.
There is a section of the Bible, perhaps the most important part of the Bible, actually, that has
been proven false: The Genesis creation account. Anyone who has really researched it knows that the
Earth is not 6,000-12,000 years old. Evolution is as close to being a proven fact as it could ever be,
and the evidence keeps on rolling in. There is evidence that abiogenesis could have occured, which
means that it most likely did occur, and there's no reason to invoke the supernatural when it isn't
needed. We're not even in an important part of the universe. We're a crappy little planet with a yellow
star. Not the center of the universe or God's attention. Genesis doesn't fit with the scientific
explanation of how the Earth was formed. The Big Bang is also as close to being a proven fact as it
could ever be. We just don't know how it happened yet. We only know that it did happen.
Mr. Lonovy has demonstrated his lack of understanding concerning what is most important in the
Bible. It is not the alleged age of the earth. It is, to be sure, the death, burial, and resurrection of
Jesus (God in flesh, John 1:1,14; Col. 2:9) from the dead (1 Cor. 15:1-4) that is most important since
by it, we are saved from damnation.
Furthermore, the Bible does not teach that the earth is 6,000-12,000 years old. There are
Christians who hold that position and there are other Christians who do not. Quite simply, the Bible
does not tell us for sure how old the earth is. I hold to a very old earth position as do many Christians
and we interpret the "days" of Genesis to be periods; particularly since the days mentioned were
before the creation of the sun and the sun and earth rotation together determine day length. Of
course, there is debate on this within Christianity, but the point is that the Bible is not explicitly clear
on this issue here. Therefore, Mr. Lonovy is making an assumption he has not qualified. Second, to
say that evolution is a fact also assumes something to be true that has not been qualified. Micro
evolution is a fact (minor variations in concentrations of extant genetic material in a gene pool), but
macro (new speciation through mutation) is not. Macro evolution has not been observed. It has only
been inferred by analogy from micro evolution...and that does not make evolution a fact. But, Mr.
Lonovy simply believes, by faith, what some scientists say concerning evolution. Furthermore,
abiogenesis is wrought with problems, i.e., mathematical permutation problems when dealing with the
spontaneous generation of life. The complexity of DNA is so great, that random formation of it is
exceedingly small. Please consider these following quotes.
• "The probability of life having originated through random choice at any one of the 10 46
occasions is then about 10 -255. The smallness of this number means that it is virtually
impossible that life has originated by a random association of molecules. The proposition
that a living structure could have arisen in a single event through random association of
molecules must be rejected." [Quastler, Henry. The Emergence of Biological Organization,
New Haven and London, Yale University Press, 1964, p. 7.]
Note, there are approximately 1080 electrons in the known universe. This should help
give an idea of the insurmountable odds against abiogenesis.
• "The more statistically improbable a thing is, the less we can believe that it just happened
by blind chance. Superficially the obvious alternative to chance is an intelligent Designer."
[R. Dawkins, "The Necessity of Darwinism". New Scientist, Vol. 94, April 15, 1982, p. 130.]
If the Big Bang is true, then how did it happen? Would Mr. Lonovy say that out of nothing,
something came, that when nothing existed, somehow something came into existence? Or would he
assert that the Big Bang has no explanation "yet" but that, by faith, some scientist in the future will
prove how it happened...even though it violates the laws of known physics? Actually, I believe in
the Big Bang. I believe that God caused it.
The Big Bang is the only place left for a god to have done anything important, and the intellegent
"Christians" have realized this and taken to saying that's how "God" made the universe, and then
directed the formation of life through evolution. But they have made up their own religion. They must
contort the Bible to fit reality. It's common knowledge that the Genesis story was to be taken
completely literal until science showed that it was actually false in the literal state. It was even talked
about later in the Bible in a very literal way that God made Adam and Eve, rested on the Sabbath, and
so forth. Now people have gone crazy about interpreting it right so that they can hold onto their
religion.
So, according to Mr. Lonovy, if a "Christian" does not accept the Big Bang and evolution, then he
isn't intelligent? Also, how do the Christians "contort the Bible to fit reality"? Mr. Lonovy may not like
the fact that some Christians believe in a literal 6 day 24 hour day creation scenario, but their belief or
"lack of belief" in in this does not bear on the validity of the Bible or not. Furthermore, if there is a
God, why can't He create the earth in 6 literal 34 hour days? But, we digress.
For clarification, I absolutely believe that God made Adam and Eve, that they were in a garden,
and that they sinned.
To the "Christians" who do this reinterpretation the Bible: Just because some parts of the Bible are
to be taken metaphorically, that doesn't mean the rest is. Why can't people accept that it's just a false
myth? I don't get it. If God wanted it to be a scientifically accurate book, He would have made it that
way. There was never anything more than moral commandments and small philosophical points to
make the Bible even worth having unless you believed the stuff in it to be true. It didn't have any new,
divine scientific knowledge in it. It's really crazy when people say the Christian God used evolution
when there's no way you can make Genesis mean that even if you take it metaphorically. This is just
my opinion, but I believe a religion recently made up [or "reinterpreted"] by humans is no religion to
trust.
Mr. Lonovy fails to understand that the Bible is not intended to be a book on science. Also, he
simply offers several opinions as if his opinions matter in establishing the validity of atheism.
Now, I'm sure there are alot of people who're going to argue that none of these scientific things
ever even happened and that they have some "proof" that they didn't. If you disagree with me, I'd like
to ask you to do something: Please, go research all of these subjects. Seriously read up on them.
Don't rely on your religious propaganda. Always be skeptical of things, nomatter what side it's from.
The evidence for each of these things, especially evolution - which alone would be enough to falsify
Genesis, is overwhelming. Look at it with an open mind, but question everything. The facts will show
you what is true and what is not. Anyways, why should you be afraid to accept science? If God told the
truth, then science will only reveal that fact. If God didn't, then either you shouldn't be worshipping
Him, or He doesn't exist.
If Mr. Lonovy wants people to always be skeptical, then is he being skeptical about his atheism?
Furthermore, I do not accept the notion that the evidence for evolution is overwhelming. Mr. Lonovy
essentially begs the question by assuming evolution to be true when it is still only a theory. If he were
to examine the fossil record, he would see that there are many problems related to missing
transitional forms. Of course, this is not a debate on the validity of evolution. There have been many
books written on both sides of the discussion. But, I wonder how many anti-evolution books Mr.
Lonovy has read, if any. If he wants people to be skeptical, then perhaps he could list for us the titles
of the books that "disprove" evolution. If he has read none, then he is not following his own skepticism
and he would not be consistent with his own advice.
Now, about other gods. When it comes down to it, we can't absolutely prove no god exists. That
doesn't really mean much. A very good quote on this is: "God is just a statistic." It's a 50/50% chance
when you first look at it - god, or no god. Then, you must look at the evidence so far to see which one
you should choose. We have no evidence that a god has ever actually done anything. I, personally,
haven't seen any reason to believe a god has ever made contact with a human. Scientifically, we have
no confirmation at all that any god has ever been seen. There is a startling lack of evidence for any
kind of god. All we have is the fact that people have made up false gods. Even Christians know this.
Why is it so hard to think that it's all just something humans came up with?
To say that we have no evidence that "a god has ever actually done anything" is an
unsubstantiated claim. It is also illogical. To say that there is "no evidence at all" means that all
evidence has been examined. Of course, this cannot be the case; therefore, Mr. Lonovy cannot
logically make his assertion. Also, a person's presuppositions will greatly influence how evidence is
interpreted. If I were to present biblical evidence of the miraculous, of God's intervention in the world,
Mr. Lonovy, because of his atheistic presupposition, would not be allowed to seriously entertain the
information. He would be required to negate it. He could not be objective in his examination of it.
Does this lend itself towards objectivity? I think not.
Those are my biggest points. The rest will be addressed below, where I will be answering each of
Matt's new points. Every space between a group of paragraphs means to skip to the next blue section
on his paper. Read along in his articles so you'll know what I'm talking about, and I won't be able to
take anything out of context. I'm going to answer his "I lack belief in God [response]", then go to "Is
atheism viable? [response]".
If these are Mr. Lonovy's biggest points, then Christianity is quite safe.
I'll begin with his opening paragraphs, which aren't entirely in blue, and then every new set of
paragraphs will deal with his blue replies:

I lack belief in God


Matt begins both of his replies by saying that he only wrote them because I had my essay on
InfidelGuy.com. I'm sure he wouldn't have even cared to read it if it wasn't there, as he did with some
of my emails to him. And he doesn't fail to mention that I had typos and grammar errors. Isn't that
kind?
I receive a great many e-mails everyday. I cannot answer all of them. Considering that Mr. Lonovy
was rather insulting and condescending in his communication with me, I am sure that I simply
disregarded what appeared to be yet another obstreperous atheist. Since I had debated, if you want to
collect a debate, the infidel guy, I thought it might be interesting to respond to Mr. Lonovy's attempt
to refute my papers, since the infidel guy thinks they are worthy of being posted. Fortunately for me,
I have found his reasoning very easy to refute.
Yes, I did start off with an ad hominum attack. I meant to. Matt offended me and insulted my
intellegence in his original papers. I am an atheist, and he was talking about atheists. I'm just cutting
to the chase. He fluffed up the fact that he's basically saying atheists are stupid, egotistical, and
denying God so they can control their lives and continue to sin. I'll just tell it how I see it. He was
being an ass. I'm glad to see that he has now taken off some of those insulting parts.
As you can see, by his own admission, he was offensive to begin with. This would naturally result
in me disregarding his e-mails. Of course, I never intended to offended or insult Mr. Lonovy. I have
tried to be respectful in spite of his numerous personal attacks. But, Mr. Lonovy misrepresents me. I
do not believe atheists are stupid. However, many of them have been rude, insulting, and
condescending. Mr. Lonovy is no exception in his rudeness and use of insults.
I probably shouldn't have said what I did, but I'm entitled to my opinion.
Why is it that you should not have said what you did? Are you saying there is a standard of
righteousness that you must adhere to? If so, where is it? On what is it based? If you are just simply
expressing remorse for attacking my character why should that matter? After all, as an atheist, you
answer to no one and have no absolute set of morals to follow.
Other than simply complaining about my ad hominum attacks, Matt uses this as a distraction
against my actual points. He dares to say I should deal with the issues instead of attacking him, but he
obviously only does this so he won't have to take on the issues. He should've ignored the personal
attacks if he doesn't like that kind of arguing and got on with the real issues. He did not deal with any
of the important points I made, if any at all.
No Mr. Lonovy, this is not a distraction. It is a simple fact that you began to your argumentation
with an attack on my person. This is something that you should not do if you want to have a decent
conversation. I have repeatedly stated that you need to stick with the issues. If you feel you have
done something wrong, then you need to apologize. As far as "the issues" goes, I tackled them. You
can say I did not, but I did. The papers are there for others to read.
I'm entitled to my opinion of him and I'm going to state that opinion because it's MY essay. I know
alot of people would disagree with my use of personal attacks, but there are still alot fine points, so
just look over it if you don't like it. The ad hominum attacks are irrelevant to the actual issues.
If they are irrelevant to the actual issues then why are you committing them? Is this an example
of your logic or is it an example of your emotionalism getting the best of you? This is a relevant point
to me only in that it verifies my observation about atheists. The majority of my encounters with
atheists on the Internet have been met with rudeness, insults and condescension from them. If
atheism is so logical and true, then why do so many hurl insults instead of stick to the issues? This is
observation after having encountered many many atheists. When Mr. Lonovy began with his personal
attacks, I noticed the unfortunate but typical modus operandi yet again and most probably dismissed
it outright.
In my opinion, Matt now plays dumb so he won't have to take on the real issue. But he doesn't fail
to mention that I acknowledged he did get some things right, but he won't do the same for me. I
stated very clearly that "You have a position..." Matt doesn't see that I'm saying this? When it's right
there in plain English? "Holding off" is not negative, either. "I am not holding off" is a fine sentence. I
don't know where he got that from. The statement is quite clear.
I would hope that you will be more specific and your complaints. You seem to be addressing
generalities.
The straw man I was talking about is also quite clear when you understand my point, which I
restated at the beginning of this essay. He was making "lack of belief" out to be something it's not.
That's the straw man.
How is it not logically necessary? The definition of atheism is "lack of belief...", as I showed earlier.
Matt's own paper is named after that. It's obvious what definition we're dealing with, and all things
which cannot understand the concept of a deity fit into the definition. Matt's new definition of atheism
doesn't even change anything. The wording is just different. Look up the meanings of the words.
I made no new definition of atheism. Mr. Lonovy had simply stated that atheism was a lack of
belief in God. Fine, then my cat qualifies as an atheist since he also has a lack of belief in God. We
have already addressed this issue above, yet Mr. Lonovy continues to bring it up. We can see that Mr.
Lonovy has already modified his definition of atheism to restrict it to those who are able to have on
awareness of the concept of God. This is on admission by him that my point about who (or what) can
qualify as atheistic was valid. If it was not true, then why it he modify his definition?
I can't redefine the word atheism. I'm not the authority of what "atheism" is. I did explain the
difference between a human atheist and a non-human animal/non-living atheist, though. Matt
completely ignores this, and gives examples of very atheistic natural things to make atheism look
stupid. Lack of belief is quite sufficient if you know that we're talking about humans. Please refer to
the qualifiers I stated earlier.
So, apparently I am correct in my original statement about nonliving and nonhuman atheists, my
cat included.
I don't know where Matt got the crazy idea that "lack of belief" is a "non-intellectual commitment
or non-action". Lack means to be without. Belief, which in this context would be "belief in", means to
think something exists. We do not think that God exists. It's that simple. He makes up a straw man
definition and then attacks it. It's rediculous.
Then perhaps Mr. Lonovy should we read my article "I lack belief in God." It is explained there.
All Matt has done is use evasiveness to dodge all of the real issues. Either that or he truely didn't
get anything I said. I believe he has more intellegence than that. You can decide for yourself if he is
trying to trick people. I stated my opinion already.
I showed exactly what false claims Matt has made. He ignored everything.
I did not "beg the question". Matt is obviously unaware of the usual way to debunk a paranormal
claim. If there is a scientific/natural way that something could happen, then it did happen that way.
The only way to prove something supernatural actually occured is to have absolutely no natural
alternative. You can still go on saying that it did happen supernaturally, but in almost any case but this
one, people will know you're wrong. I showed earlier in this essay why I said it the way I did,
anyways.
There are so many "goofs" in Mr. Lonovy's comments that I am not sure if I should take the time
filling up the space to address each one. For example, he says I am "obviously unaware of the usual
way to debunk a paranormal claim." Considering that he has never asked me about this, he would not
know. Furthermore, since I made no comments about addressing or debunking alleged paranormal
phenomena, he again jumps to conclusions which have no basis. Just because I'm a Christian does not
mean that I do not consider the natural explanations for events before the spiritual. As a matter of
fact, that is exactly what I do. It is only after natural explanations fail to explain a phenomena, that I
would then look to the supernatural. Of course, my Christian presupposition allows me to do this; that
is, to entertain both the natural and the supernatural. However, Mr. Lonovy's presupposition does not
allow him the breadth of intellectual openness that I enjoy as a Christian.
I established my points very clearly, but Matt completely ignored every single point.
I was originally writing in hopes of showing him what was wrong with his papers, so that he would
remove them or drastically change them. I don't want anyone else to be influenced by the misleading
information in them. He could at least make a decent counter to my essay to prove he's right.
Does anyone think that Mr. Lonovy's statement that I "completely ignored every single point" is a
legitimate statement? I read through sentence by sentence Mr. Lonovy's paper and responded.
Certainly, "every single point" was not ignored.
So far, Mr. Lonovy has been less than convincing in his argumentation. I wonder what he or other
atheists would say if I said, "he could at least make a decent counter to my essay." After all, I do not
believe that he has done so. But, that is my opinion.
Matt has shown me that misleading people is exactly what he wants to do. Again, only my opinion.
I am actually surprised that Mr. Lonovy believes in the supernatural as is evidenced in his previous
statement. You see, he apparently is able to read minds. So far there is no naturalistic explanation for
this phenomena that I'm aware of. Mr. Lonovy is now trying to tell us all what my desires are. So, let
me simply put this to rest. Since I know myself better than Mr. Lonovy does, I can tell all the readers
that it is not to my desire to mislead anyone.
I had no chip on my shoulder. I wrote the essay because I felt he was wrong, and I still feel that
way. Matt could have shown me that he was right, but he did not counter anything I said. He dodged it
all.
I do have a chip on my shoulder now. Matt has angered me with his evasiveness. I'm not going to
let him get away with brushing off every point I made, if he did it intentionally or not. That's the point
of this essay.
Well, Mr. Lonovy is certainly entitled to his emotional reactions.
I was offended because Matt was lying [if intentional or not] about atheists, as I see it, and
insulting them, which was very clear. I'm an atheist, you know.
Need I comment about this statement?
Now we will go to his second article:

Is atheism viable?
I think I can judge people quite well. I admit that the personal attacks are only my opinion, of
course, and I'm refraining from name calling now, but it's obvious that Matt is trying to play dumb to
dodge what I said or he really doesn't have the intellegence to put it together. I believe he's smarter
than that.
I cannot state whether or not Mr. Lonovy is a good judge of people. But I can tell confidently state
that his assessments of me, my intentions, my desires, etc. have been less than accurate. Consider
his comments that I am "trying to play dumb." I admit that I may miss a point or two of what Mr.
Lonovy intends, but I'm certainly not trying to play dumb. On the other had, perhaps he is correct in
that I lack the intelligence for a meaningful dialogue with him.
Making atheists look bad is exactly what he attempts to do. I was a little harsh on the "negative
position" part, though. He probably didn't say that to make atheists look bad, I admit, but he
attempted to make atheists look bad in enough other ways for me to say this. .....???
Again, he complains about me calling him an idiot. I'll attack him all I want. I make enough real
points. I'm allowed to state my opinion of him.
If calling me names and insulting my character makes Mr. Lonovy feel better, I would not want to
rob him of emotional satisfaction. However, I would recommend that in a debate ad hominim attacks
do not help in establishing a position.
If an atheist is stupid, I'm damn well gonna say it. I'm not siding with all atheists. I'm quite sure
there are some very stupid atheists. I'm quite sure there are atheists who fit exactly what Matt
portrayed them as. I'm not going to defend someone just because they're an atheist. I have seen
many very intelligent atheists, though, and I believe the atheistic community is mostly made up of
these kind of people. These are the atheists I would side with.
Is Mr. Lonovy telling us that he is able to differentiate between the intelligent and stupid atheists?
I dealt with all of this earlier. I'm referring to the Christian God of the Bible. I admit that I erringly
stated just "God" in my first essay without making it known that I meant ONLY the Christian god. He
CAN be shown to not exist by proving the Bible false.
Whether or not you have or can prove that the Bible is false is very debatable. From what I have
seen of your argumentation so far, you have not shown the slightest ability to prove the Bible and
incorrect.
Again, I did not beg the question. Refer to what I just said.
The topics are too deep and varied to address here, I agree. I may just write refutations of other
portions of Matt's site about these things later if I disagree with him.
Matt can attempt to define faith in whatever way he wants, but I got my definition from a
dictionary. He is making up the definition he wants for faith. Why don't we look at the definitions...
If Mr. Lonovy is allowed to use a dictionary to define faith, then certainly I am also able to use a
dictionary to define atheism...as I have above. Therefore, I thank Mr. Lonovy for vindicating my
definition of atheism.
Faith: Belief in something for which there is no proof.
Proof: The cogency of evidence that compels acceptance by the mind of a truth or a fact.
Cogency: The quality or state of being cogent.
Cogent: Appealing forcibly to the mind or reason: CONVINCING
If you are so convinced by evidence that you are willing to bet your life or "eternal safety" on it,
then that evidence is defined as proof in my dictionary.
T he Bible is evidence, whether you like it or not. You may not except it as evidence, but it is
evidence. The issue is what you do with that evidence. There are old Testament prophecies fulfilled
in the New Testament. There are the eyewitness accounts of the miraculous and of Jesus'
Resurrection. Of course, I suspect that you are not capable of an unbiased the examination of the
evidence since your atheistic presupposition will not allow you to consider the supernatural.
Therefore, you must find a way to disregard the evidence.
I am NOT taking my disbelief in the Christian God on faith. I showed a lot of my reasons for this
earlier in this essay. I am also not taking my disbelief in other mythical deities by faith, but I'm not
digging up things about every other god for this essay. It should be obvious that they aren't real,
anyways.
I cannot say that I know there is absolutely no deity anywhere in the universe, but there are
reasons not to believe in them. I showed some of these earlier. Even if they exist, they have no effect
on our lives, anyways. So, what would the purpose of believing in one even be? There is no proof for a
deity, yet the amazingly complete lack of evidence and need for them is certainly evidence against
them. Anyone who believes in them is actually in a less logically defensible position.
How would Mr. Lonovy be able to ascertain whether a deity was affecting his life not? He simply
says there is no evidence that this occurs. I have evidence to the contrary. My evidence is my
encounter with God, the profound change in my life, that "awareness" of his presence, and much
more. These are subjective but they are real. If I were of the mind to do so, I could pursue the issue
of logical absolutes which exist but no atheist, that I have encountered so far, can logically explain. I
have used this fact of logic and its existence to demonstrate the existence of an absolute mind...but I
digress.

Ha. Very funny. Do some research, Matt. You'd be surprised how much of this proof there is. And
don't try to weasel out of answering that this time with saying I didn't tell you what the proof is. I said
it very clearly earlier.
Actually Mr. Lonovy, I have done quite a bit of research and it has verified my belief in God.
Science has not shown that there is absolutely no god [not capitalized]. Science has shown that
the Christian God of the Bible doesn't exist, and science has shown a complete lack of evidence for any
other god. That's enough for me.
Could you please document for me precisely where science has shown that the Christian God does
not exist? I would really like to have the documentation. As far as science showing that there is a
complete lack of evidence for any other god, of course I would agree with you, since I believe that
there is only one true God, the God of the Bible. Therefore all other god's would not exist.
There are those who would say that there aren't even any absolute truths, but let's just say that
there are. As far as I know, we can't prove there are no gods, so yes, atheism is a possibility, but I
would definitely argue that atheism is much more than just a simple possibility. I've shown reasons for
this earlier.
To say that there are no absolute truths would be an absolute truth, which would be self-
contradictory and not true. To be honest, Mr. Lonovy has not shown much of anything as far as facts
or evidence goes for his position.
If science explains something, that means God didn't do it. If God didn't do it like the Bible says,
then the God of the Bible doesn't exist.
This is not a logical statement. The aurora borealis, for example, can be easily explained in
science. But who is to say that God did not arrange it so that the aurora borealis would exist for His
good pleasure?
People can believe what they want to believe as long as it doesn't hurt anyone else, but Matt is
spreading things which aren't true about another religion, or lack of religion, actually, with his articles.
I feel this is wrong.
I assume Mr. Lonovy is alluding to my articles on Islam, Mormonism, Jehovah's Witnesses, etc.
He is stating that I am spreading things which are not true about other religions. Okay, what is it
about these other religions that I'm stating that is not true? I am always open to being corrected.
Perhaps Mr. Lonovy can explain to me, with documentation, where I am incorrect.
I encouraged the readers of my essay to read Matt's paper along with mine. I did not take
anything out of context. I was simply showing that the ice cream factory on Jupiter was a bad
illustration for Matt's religion. It's a wonderful illustration AGAINST his religion, though. That is why I
said what I did.
Actually, the ice cream factory on Jupiter was used as an illustration against atheism, not religion.
In it, I simply stated that atheism is viable only as a "possibility." That is, it is a logical possibility. But
being a logical possibility does not necessitate that it is an actuality anymore then the possibility of an
ice cream factory on Jupiter means that there is one. It is certainly possible, but not probable.
I can attack Matt personally all I want, as I have already said. I didn't attack him to establish any
point. I did it because it is my opinion. The points stand on their own without the insults.
Is Mr. Lonovy attempting to expunge his ad hominim attacks do this repeat it clarification of why
he has been personally insulting? I suggest that a better approach that he should take would be to
admit that he was in error and that he would not do it again. That would be fine and then we could be
done with it. Perhaps Mr. Lonovy is feeling a tinge of guilt. I do not know.
I believe I was quite logical all through that essay, and I've explained this proof already.
There is no evidence in the Bible. Nothing outside of it confirms any of the important things in it.
The prophecies are the main claim of Christians, but it fulfills its own prophecies. The people, who
wrote the later fulfillments of prophecy, even hundreds of years, read those same prophecies. It's not
that hard to see how they were fulfilled.
Really? There is no evidence in the Bible? You mean the eyewitness accounts of the miraculous,
that they were written down, that they were accurately transmitted to us, that those who wrote what
they saw, paid with their lives for what they said, etc., really is no evidence that all? As far as external
confirmations, we certainly can go to the Middle East and find all sorts of cities and archeological digs
is verifying biblical accounts. The Old and New Testament documents are ancient documents and, by
default, are evidence. What you want to do with the evidence is up to you -- and your presuppositions.

I suggest that Mr. Lonovy read some of the works dealing with the prophetic nature of the Bible
accounts. He is not being lucid in his statements concerning them. Perhaps more information would
help him in this area.
Matt again uses a very bad example. Atheists would be the ones saying his screaming blue ants
don't exist. Theists make that kind of positive claim. They're the ones who fit right in with people who
see screaming blue ants.
Actually, I really liked the screaming blue ants comment.
Do some research, Matt.
I have done a great deal of research. Just take a look at www.carm.org.
Macro-evolution is obviously real and easy to understand if you do the research, and it would
falsify the Christian God, as I have shown.
Two things. I understand the theory of macro evolution. I have done research on it. However, I'm
not convinced that it is true. Second, even if macro evolution were true it would not falsify the
Christian God. It would be logically possible that if God exists, and He created the universe, that He
would also have put in it the means by which life could have developed. If macro evolution were true,
and I am not admitting that it is, it would not necessitate that God does not exist because it could
mean that God used it.
Most Christians I have seen refuse to accept any possibility that there is no God.
I have certainly said before that it is possible that there is no God. However, I believe there is a
God and that he has saved me from the coming judgment through Jesus Christ who died, was buried,
and rose from the dead (1 Cor. 15:1-4).
Matt can claim to "know" God exists all he wants...That's all I say.....
Ok, a theist believes that God does exist, and an atheist doesn't, right? Now, I would recommend
that both accept the possibility that they are wrong. If they do this, that does not change their position
at all. Matt tried to make it seem as if an atheist says God may exist, then he's not sure, but if a
Christian says He may not exist, then he's still sure. What the Hell? That's a double standard.
Mr. Lonovy's comment is not that clear. Perhaps it is my lack of intelligence getting in the way,
yet again. But, I have been known to miss simple concepts.
I don't think God is even a reasonable concept. There are some Christians who are reasonable,
though. I'll give him that.
Matt now decides to be an a***ole for no reason. He obviously doesn't know me.
Amazing...
God has obviously not allowed me to have what I've asked of Him. I don't want any sin or
independance from God. I don't know where Mr. Slick got that bull*** from. I've asked God to reveal
Himself to me so **** many times that it's not funny. I'm just trying to find the truth and spread it to
others.
He has already revealed Himself in the person of Jesus. If Mr. Lonovy or others have rejected
Jesus, what should God to do now?
Once again, there is no evidence in the Bible. You can write whatever you want in a book, but that
doesn't make it true, esspecially if nothing else confirms what it says.
That is not logically necessary, but I understand your frustration.
How does this context change anything? This is the kind of **** that offended me, people. How
many times must I say this? I DON'T WANT TO BE GOD! I WOULD LIKE FOR GOD TO SHOW HIMSELF
TO ME! I WOULD LIKE TO GO TO HEAVEN!
In that case, why don't you look to Jesus. If you'd like help, we can chat on the phone.
Now Matt DARES to say I used a personal attack, when RIGHT ABOVE THAT, HE used a personal
attack on ALL ATHEISTS. I believe my attacks were VERY fair.
I don't see how I used a personal attack on all atheists. I have, however, stated that many
atheists are quite crass and condescending in their opposition to Christianity. That is a fact. How is it
an attack on Mr. Lonovy? Whether or not he fits into the category or not is up to the reader to decide
at this point.

Matt has ignored my attempts at friendly dialogue before. I admit that I said some things I
shouldn't have in an email dialogue with him before, and this may be his reason. I was going through
some bad things at the time, and I explained this, heartfeltly apologized to him, and attempted to
continue with a friendly dialogue. I thought Christians were supposed to forgive and forget? Well, Matt
ignored me. He would've ignored me again if Infidel Guy hadn't posted my essay on his site. I thank
IG very much for this.
You do not have a friendly dialogue by beginning it with an insult. Also, I get a great many emails
every day. I cannot answer them all.
My tact is not in question here. I did refute him, and he ignored my points. I'm trying to make
sure he won't get away with that.
Sorry, Mr. Lonovy, you did not refute anything.
Thank you for reading.
Donny Kay Lonovy

This paper resembles more of a chat room dialogue than a 'debate' of some sort. I am sure that
Mr. Lonovy will continue on with his responses. But, I doubt that I will continue with this since it is
unproductive and is a bit rambling. If Mr. Lonovy wishes to question me about how he might
encounter God, I'd be happy to help him as I would be happy to help anyone who also seeks to have
an encounter with Him.
Matt Slick
Lack-of-belief analysis outline

1. What does "lack belief in God" mean?


A. "Lack" means, deficiency or absence. "Belief" means, acceptance and conviction that
something is true or valid.
i. Therefore, lack of belief would mean, basically, an absence of belief that something is
true. But even this is debatable on what "absence of belief" can mean. Someone can
say, "I have absence of belief in screaming blue ants" but it is a meaningless statement.
B. If "Lack of belief" is complete ignorance about something, then it is a state of non-awareness
about it.
i. It is not a purposeful chosen neutrality about something since this is an intellectual
categorization which implies awareness of a concept or thing -- even if the category is
called neutrality.
ii. We lack belief in concepts we are not aware of and we categorize/assess concepts we
are aware of.
C. If "lack of belief" means that a person chooses to not make an intellectual commitment to a
position, but to remain intellectually neutral regarding belief or disbelief, that would be more
logical.
i. However, complete neutrality about a concept is impossible since all concepts have an
effect upon the hearer and illicit a response.
ii. Once you have been exposed to a concept, you categorize it as
a. True, False, Ridiculous, Unsure, etc., but you do not return to a complete mental
neutrality or state of ignorance.
iii. We do not "lack belief" in invisible pink unicorns. That is, we do not hold a mentally
neutral position of the concept. We make a decision to categorize them as
iv. True, False, Ridiculous, Unsure, etc. based upon our scope of knowledge and experience.
v. To the extent that this categorization occurs, belief or disbelief is associated with it.
a. If True, then positive belief is applied.
b. If False, then disbelief (the positive belief that it is false) is applied.
c. If Ridiculous, then disbelief (the positive belief that it is false) is applied.
d. If Unsure, then belief and disbelief are pending with either as the outcome.
i. This is because we realize that belief in the concept (acceptance) is possible as
also is disbelief (rejection) depending on further information.
ii. Being unsure about something is as close to "lack of belief" as one can logically
get but even this is a categorization with pending commitment to belief or
disbelief.
2. Actions reflect belief
A. We act based upon what we do believe, not upon what we do not believe. In other words, I
do something because I believe something whether it be that my house is on fire or that
there is food in my refrigerator.
i. However, to say that I believe there is no food in my refrigerator and therefore, I do not
get up to go get food is actually the active belief that there is no food there and the
resulting decision (action) to remain where I am is the result.
B. I lack belief in concepts I am unaware of. Therefore, I do not and cannot act based upon
them since I am unaware of them.
C. I can only act or not act based upon concepts I am aware of.
i. If I believe there are invisible pink unicorns, I would act accordingly and either defend
their existence or behave in a manner consistent with the belief that they exist.
ii. If I believe there are no such things as invisible pink unicorns, I may or may not defend
my position depending on the circumstances. But, I do not promote their non existence
since it is not necessary to do so anymore than it is necessary to promote the assertion
that there is no ice cream factory on Jupiter.
iii. If I believe that the existence of Invisible Pink Unicorns is ridiculous, I may or may not
assert that it is ridiculous, but I have categorized them and believe they do not exist.
iv. If I am unsure about the existence of Invisible Pink Unicorns, I would wait for further
information before making my decision. In this, I would be agnostic about their
existence.
D. If an atheist says he (or she) lacks belief in God, yet actively seeks to undermine theistic
proofs and promote atheistic principles, then we must conclude that his actions are
consistent with his beliefs; namely, that he actively believes that God does not exist.
i. Furthermore, if the atheist is actively promoting the non-existence of God yet says he
lacks belief in God, then his words and actions are inconsistent.
E. Atheists who say they lack belief in God, or have disbelief in God, yet actively attack theistic
proofs and seek to promote atheism, are acting according to their beliefs, not their "lack of
belief." It is more consistent to say the atheist who supports and promotes the idea that
there is no God, believes that there is no God, not that he lacks belief in God. Otherwise, he
is behaving either without a reason, which is not logical, or with a reason; namely, the belief
that God does not exist.
3. To say that you believe there is no God has problems
A. To say "I believe there is no God" is a conscious choice. Then, on what would the atheist be
basing his belief that there is no God: evidence, lack of evidence, logic, faith, or a
combination of all?
i. If evidence, then what positive evidence is there that disproves God's existence?
a. There can be no such evidence since evidence is physical in nature (evidence is an
effect and/or result of something in reality). How could evidence disprove God's
existence who is, by definition, the creator of reality and separate from it?
(I am defending the Christian God as revealed in the Bible).
b. Testimony is admissible in court as evidence, but no one can rightly testify that God
does not exist.
ii. If lack of evidence, then it means he has not yet seen all evidence and there might be
sufficient evidence to demonstrate God's existence. This would mean that God may
indeed exist and the person really is an agnostic concerning God and his atheist position
is inconsistent with his statement.
iii. If logic then what logical proof do you have that negates God's existence?
a. At best, logic can only disprove theistic proofs. Disproving theistic proofs does not
mean there is no God. It only means that the proofs thus presented are insufficient.
b. Logic can only disprove theistic proofs that are presented and negating such proofs
is not a refutation of all possible proofs since no one can know or present all
possible proofs of God's existence. Therefore, negation of proofs does not disprove
God's existence.
c. If there were a logical argument that proved that God did not exist, it either has not
yet been made known. If it were known then it would be in use by atheists. But
since no proof of God's non-existence has been successfully defended by atheists,
we can conclude that thus far, that there are no logical proofs for God's non-
existence.
iv. If faith alone, then the position is not held by logic or evidence and is an arbitrary
position.
v. If by a combination of evidence, logic, and/or faith, then according to the above
analysis, neither is sufficient to validate atheism. A combination of insufficient means
does not validate atheism.
B. For someone to believe there is no God is to hold that belief by faith since there is no
evidence that positively supports atheism and there are no logical proofs that God does not
exist. It is, after all, virtually impossible to prove a negative.
Answers to positions held by atheists

1. There is no God
A. This is not a logical position to hold since to know there is no God means the person would
have to know all things to know there is no God. Since he cannot know all things (if he did
he would be God), then he cannot logically say there is no God.
2. I believe there is no God
A. To say "I believe there is no God" is a conscious choice. Then, on what do you base your
choice: evidence, logic, faith, or a combination of the three?
i. If evidence, then what positive evidence is there that disproves God's existence?
a. There can be no such evidence since evidence is physical in nature (evidence is an
effect and/or result of something in reality). How could evidence disprove God's
existence who is, by definition, the creator of reality and separate from it?
(I am defending the Christian God as revealed in the Bible).
b. Testimony is admissible in court as evidence, but no one can rightly testify that God
does not exist.
ii. If logic then what logical proof do you have that negates God's existence?
a. At best, logic can only disprove theistic proofs. Disproving theistic proofs does not
mean there is no God. It only means that the proofs thus presented are insufficient.
b. Logic can only disprove theistic proofs that are presented and negating such proofs
is not a refutation of all possible proofs since no one can know or present all
possible proofs of God's existence. Therefore, negation of proofs does not disprove
God's existence.
c. If there were a logical argument that proved that God did not exist, it either has not
yet been made known. If it were known then it would be in use by atheists. But
since no proof of God's non-existence has been successfully defended by atheists,
we can conclude that thus far, that there are no logical proofs for God's non-
existence.
iii. If faith alone, then the position is not held by logic or evidence and is an arbitrary
position.
iv. If by a combination of evidence, logic, and/or faith, then according to the above
analysis, neither is sufficient to validate atheism. A combination of insufficient means
does not validate atheism.
B. For someone to believe there is no God is to hold that belief by faith since there is no
evidence that positively supports atheism and there are no logical proofs that God does not
exist. It is, after all, virtually impossible to prove a negative.
3. There is no evidence for God
A. This is not a logical position to hold since to know there is no evidence for God's existence
necessitates that the person knows all possible evidences for God's existence. Since he
cannot do this (if he did he would be God), then he cannot logically say there is no evidence
for God.
4. I have not seen sufficient evidence for God's existence.
A. To say you haven't seen sufficient evidence for God's existence is a more intellectually
honest position, but it is really a form of agnosticism which maintains that God is not known
or knowable while admitting that the possibility of God's existence.
B. If a person has not seen sufficient evidence for God, then it means he has not yet seen all
evidence and there might be sufficient evidence. This would mean that God may indeed
exist and the person really is an agnostic concerning God and his atheist position is
inconsistent with his statement.
5. I lack belief in God.
A. To lack belief in God appears to be a defensive position since the assertive atheist positions
are wrought with logical problems (shown above). If the atheist says he "lacks belief" in
God, then it appears its goal is to maintain a position that is unattackable since then he has
no position to attack.

The problem is that "lacking belief" in God is an intellectual position made by a choice to
"lack belief." Therefore, it is a position since it is the result of a choice. Any position held,
must have reasons or it is not a position. It would be nothing. The atheist who asserts that
he lacks belief is asserting a position of lack of belief.
B. My cat lacks belief in God as does my computer. Are they also atheists? Therefore, simply
lacking belief is not a sufficient statement since it can include animals and inanimate objects.
C. If you say that "lacking belief" refers only to yourself as a human being, then see point A.
6. I don't believe in God.
A. Is this a choice you have made? If so, why? What made you not believe in God?
B. Is there an intelligent reason that you do not believe in God? Can you please tell me what it
is?
7. Naturalism is true; therefore, there is no need for God.
A. Naturalism is the belief that all phenomena can be explained in terms of natural causes and
laws. If all things were explainable through natural laws, it does mean God does not exist
since God is, by definition, outside of natural laws since He is the creator of them.
B. Some might say that if all things can be explained via natural laws, then it means there is no
evidence for God.
i. But, can all things be explained via naturalism? No, because naturalism has not
explained all phenomena known today, nor can we assert that all things in the future will
be explained via naturalism because we do not know all phenomena that can and will
occur. Therefore, it is not a fact that naturalism can explain all things. Therefore, God
is not negated via naturalism.
Comments from atheists

Sometimes, debating atheists on the internet is like trying to heard cats. It just doesn't work very
well. There are several options for Internet debates with atheists: AOL instant messenger, Yahoo chat
rooms, my website, email, etc. But, my favorite is Paltalk, a voice enabled chat system. It works very
well and allows instant communication via a microphone and speakers. It is a great place to debate,
to learn, and to teach.
Periodically, I jump into various atheist rooms on Paltalk, or AOL, Yahoo, or some atheist based
discussion board, read, listen, and sometimes comment. The standard reception I get from atheists is
less than cordial. Now, I realize that some "Christians" misrepresent Christ and show no politeness to
the atheists and some of them want to retaliate. But, when I quietly "lurk" in atheist lead rooms and
Christians come in, it isn't the Christians who are overwhelmingly rude. It is the atheists. I mean, the
majority of the atheists are rude, condescending, insulting, and foul. I do not mean to offend any
atheists, but this is my observation.
Okay, so why am I bringing this up? Because it is so prevalent among atheists that I cannot help
but wonder why so many are so belligerent, so full of anger and condemnation. When I have asked
this, and after the insults are finished, some of them have told me that they hate that the Christians
try and tell them that they need to believe in Jesus or they will go to hell. Okay, so I can see how
that might upset them. But, that's too bad. The truth is that all people need Jesus and the Christians
are simply telling them the truth. Do they need to be insulted, cussed at, and called names? No, they
don't.
I have always tried to be civil to atheists. I do not cuss at them, call them names, and I try not to
personally attack individuals, though, I admit, I've gotten upset with a few of them. Nevertheless,
following are some samples of actual comments I've received from atheists. Of course, this does not
represent all atheists, but it does represent a significant portion -- at least in my experience. So,
following are just a fraction of the insults I've personally received.

• You brain dead idiot. Take your brain dead hypocritical sorry self and leave.
• You lying sack of crap.
• Matt you fool
• You play mind games.
• I copied his [my text chat] comments to show how inane it was.
• "slick" - how appropriate a nick is that? Just like all the slick preachers.
• You ignorant peace of Christian fecal matter.
• Go to **** you *******.
• We will let you think when you get a brain Matt.
• Matt...when was the last time you actually thought?
• You sound like you are in second or third grade.
• Why does your freaking god judge the creation it made? Matt is a retard.
• You are repulsive as a human being and you need to stay away from children.
• Matt, insults where insults are due is just plain speaking truth.
• You just want to fly planes into buildings.

Amazing comments. In fact, some of the insults are so poorly stated that I sometimes suggest
that they go to my Pick Your Insult page on CARM they can at least insult me properly. Anyway, if the
atheists are so sure of themselves and so rational as they claim, why do so many of them resort to
insults and foul language instead of addressing the issues? Can't they rationally defend their position
of atheism? And, what is it about atheism that leads them to such behavior?
I received an e-mail from an atheist recently. It represents about 50%-60% of the emails I get
from atheists. The rest of them are less insulting and a minority of them are actually thoughtfully
written. I can count on two hands the emails from atheists that have been polite and intelligently
written. Anyway, this atheist wrote me complaining about the atheism section on CARM. His e-mail
was quite lengthy and I responded to him that I do not read long e-mails because I get so many every
day. This is clearly stated on the email page of CARM where he had to go in order to email me:
"Please keep your emails short and to the point." Therefore, I was not asking him anything out of the
ordinary. He then wrote me the following:

"I took the time to write in it hope of bringing a little truth to you, you could take the time to read it.
You're an apologist, aren't you? Don't you think it'd be smart to at least see my arguments, so you
can have a counter-argument ready if you ever encounter them again? No, you don't care. You either
don't have the attention span of a chimpanzee or you don't want to read it because it's the truth,
something you don't care to hear. I'm right and you know it. Don't make pathetic excuses. Your
page is full of lies and you don't want to change it. Just like on all the other Christian websites, you
have to lie to keep any argument going. You guys never welcome change when it doesn't fit your
agenda. I'll have my own page soon and expose your pathetic lies."

Why would it be necessary for this atheist and other atheists to be so abusive? I ask them this and
they accuse me of whining and then they often strongly demand that I prove there is a god. Most
interesting.
Again, what is it in atheism that lends itself to such behavior?
Concerning atheist attacks on Theism
Some might think that atheists would be content with simply not believing in God and leave the
theists to themselves. After all, if God doesn't exist then what's the big deal? Why not let the theists
believe in God the way a child believes in the tooth fairy? To the atheist neither exists. So why bother?
Even though many atheists don't care if people believe in God or not, others feel obligated to fight
what is often labeled as "oppressive religious bigotry." To this end, many of them are active in
politics, social groups, the Internet, and various lawsuits with the intent of changing society to a more
atheistic nature. They often consider Christians as a threat to freedom, common sense, and a good
life. Consider this quote I found on an atheist web site at atheists.org.

"We are constantly being overrun by people trying to get their ticket to Heaven at our expense, and if
we don’t stand up and be counted we will lose the very freedom we hold most dear; freedom of
thought..."

This kind of statement is quite common in atheist circles. Many atheists I've spoken to tell me that
I cannot think logically, that I am deluded, and that I believe in myths. They tell me that I am bound
by foolish antiquated beliefs and need to abandon my religious bigotry and become a 'free thinker' like
them.
Additionally, after reading much atheist material and debating with them over the internet, I've
discovered that they often use mockery of God, religious leaders, and the Bible as weapons. This isn't
the case with all atheists, as I have had very good conversations with some of them, but ridiculing
attitudes are surprisingly prevalent and strong. Character assassination, half-truths, and out-of-
context Bible quotes are typical tools used by many of them in attempts to make Christianity look bad.
Now, I am not trying to dismantle the atheist position with a generic character assassination. I am
only making on observation. In the majority of my dealings with atheists, I have encountered great
arrogance, rudeness, and condescension. Atheists have told me that religion is only a giant con-game
designed to get peoples' money, that clergymen are in business for themselves, and that I was
mentally ill for believing in God. Following are other comments from atheists:

• "I do not want to be bound to archaic mythologies. This is the 20th century."
• "Christianity is an oppressive system used to control and manipulate people."
• "Logic demands that religion be proven wrong."
• "Christians should all be in mental wards."
• "We are free thinkers and not bound by outdated and oppressive myths."
• "Christians are sycophantic sheep."

Atheists often imply that reason is best used by them and not by Christians who, many say, need
psychological help for believing in God. This condescending attitude is a fountain for derogatory
comments. I have been called stupid, absurd, illogical, and a slave to my religion. I get the impression
from atheists that they are so convinced they have the truth that no other options are available to
them and that if you don't agree with them, you're not smart. Of course, they will deny this and say I
am being ridiculous, but this is what I have observed -- right or wrong.
Consider some of the terms the atheists use to describe themselves: "Free thinkers," "Free from
religion," "Rational," etc. They use these self-descriptive terms in juxtaposition to statements of
Christians as religious bigots, losers, and brainwashers. On the atheist.org website I read,

"Critical thinking, objectivity, scientific methodology, and peer review are all hallmarks of Atheism.
Submission, fear, credulity, and insupportable claims are the hallmarks of religious belief. "

When I read statements like this I can't help but wonder which religion they are referring to. It
can't be Christianity because the Bible teaches us to love God and love our fellow man. It teaches that
the fear of the Lord is wisdom, that truthfulness is a way of life, and that eyewitness accounts of the
miraculous is one of the evidences for its validity. Of course, the atheist would argue with all of this
because he must. But still, if an atheist wants to attack religion in general and Christianity specifically,
it should, at least, do so objectively. But this doesn't seem to be the hallmark of the atheistic
movement; at least not from what I've seen so far.
Consider the following statements from Atheists.

• "Godism is consistent with crime, cruelty, envy, hatred, malice, and


uncharitableness."
• "As long as religious purposes are served, ethics, inquiry and reason are
abandoned."

Are these the statements of tolerance, impartiality, truth, and sound judgment? Not at all. It
seems to me that if the atheists who expressed the above quotes were in power, with their views of
religion being cruel, evil, and unreasonable, would they then either jail the 'offenders' or legislate
complete and total annihilation of all things religious? Who knows, but it is something to ponder.
Does atheism really teach freedom? No. It teaches bondage for its adherents and for those who
disagree with it.
I don't' see any convincing evidence for God

Following are some of the approaches I use when dealing with atheists in conversations that deal
with alleged lack of evidence for God's existence. Now, no argument is fool-proof and no single
argument answers all the objections. Nevertheless, it is important to have thought out some of the
implications of the statements and bring them up during conversations. Of course, conversations rarely
follow a logical format. They usually take tangents and detours. That is normal and good. But we
need to be prepared as much as possible.

I don't see any convincing evidence for the existence of God,


.A That does not mean there is no God.
.i Since you cannot know all evidence, it is possible that evidence exists that proves God's
existence, or at least supports his existence.
.a Therefore, it is possible that God exists.
.i If it is possible, then faith has its place.
.ii If it is possible that God exists, then you should be an agnostic (an agnostic
holds that God may exist but no proof can be had for His existence.)
.B It is possible that there is no evidence at all for God.
.i But this cannot be stated absolutely, since all evidence would need to be known to show
there is no evidence.
.a Therefore, since all evidence cannot be known by any one person, it is possible that
evidence exists that supports theism.
.C Then what kind of evidence would be acceptable?
.i If you have not decided what evidence would be sufficient and reasonable, then you
cannot state that there is no evidence for God.
.ii If you have decided what evidence is sufficient, what is it?
.a Does Christianity fit within those criteria?
.i If not, why not?
.D Is it possible that your criterion for evidence is not reasonable?
.i Does your criteria put a requirement upon God (if He exists) that is not realistic? For
example
.a Do you want Him to appear before you in blazing glory?
.i Even if that did happen, would you believe he existed or would you consider it a
hallucination of some sort or a trick played on you?
.ii How would you know?
.ii Do your criteria put a requirement on logic that is not realistic?
.a Do you want him to make square circles, or some other self-contradictory
phenomena or make a rock so big He cannot pick it up?
.b If God exists, He has created the laws of logic. He, then, cannot violate those laws.
.E Are you objectively examining evidence that is presented?
.i Granted, objectivity is difficult for all people, but are you being as objective as you can?
.ii But, do you have a presupposition that God does not exist or that the miraculous cannot
occur?
.a If so, then you cannot objectively examine the evidence.
.i Therefore, the presuppositions you hold regarding the miraculous may prevent
you from recognizing evidence for God's existence.
.a If so, then God becomes unknowable to you and you have forced yourself
into an atheistic/agnostic position.
.b Do you define the miraculous out of existence?
.i If so, on what basis do you do this?
.iii If you assume that science can explain all phenomena then there can be no miraculous
evidence ever submitted as proof.
.a If you made that assumption, it is, after all, only an assumption.
Can God make a rock so big He can't pick it up?

This question is representative of the type of paradoxes atheists use in attempts to prove that God
cannot exist. It works like this. God is supposed to be omnipotent. If He is omnipotent, then He can
create a rock so big that He can't pick it up. If He cannot make a rock like this, then He is not
omnipotent. If He can make a rock so big He can't pick it up, then He isn't omnipotent either. Either
way demonstrates that God cannot do something. Therefore God is not omnipotent. Therefore God
does not exist.
Is this logical? A little. However, the problem is that this bit of logic omits some crucial
information, therefore, it's conclusion is inaccurate.
What the above "paradox" lacks is vital information concerning God's nature. His omnipotence is
not something independent of His nature. It is part of His nature. God has a nature and His attributes
operate within that nature, as does anything and everything else.
For example, I have human nature. I can run. But, I cannot outrun a lion. My nature simply does
not permit it. My ability to run is connected to my nature and I cannot violate it. So too with God.
His omnipotence is connected to His nature since being omnipotent is part of what He is.
Omnipotence, then, must be consistent with what He is and not with what He is not since His
omnipotence is not an entity to itself. Therefore, God can only do those things that are consistent with
His nature. He cannot lie because it is against His nature to do so. Not being able to lie does not mean
He is not God or that He is not all powerful. Also, He cannot cease to be God. Since He is in all places
at all times, if He stopped existing then He wouldn't be in all places at all time. Therefore, He cannot
cease to exist without violating His own nature.
The point is that God cannot do something that is a violation of His own existence and nature.
Therefore, He cannot make a rock so big he can't pick up, or make something bigger than Himself,
etc. But, not being able to do this does not mean He is not God nor that He is not omnipotent.
Omnipotence is not the ability to do anything conceivable, but the ability to do anything consistent
with His nature and consistent with His desire within the realm of His unlimited and universal power
which we do not possess. This does not mean He can violate His own nature. If He did something
inconsistent with His nature, then He would be self contradictory. If God were self contradictory, He
would not be true. Likewise, if He did something that violated his nature, like make a rock so big He
can't pick it up, He would also not be true since that would be a self contradiction. Since truth is not
self contradictory, as neither is God, if He were not true, then He would not be God. But God is true
and not self contradictory, therefore, God cannot do something that violates His own nature.
Another way to look at it is realize that in order for God to make something so big He couldn't pick
it up, He would have to make a rock bigger than Himself. Since He is infinite in size, He would have to
make something that would be bigger than Himself. Since it is His nature to be the biggest thing in
existence because He created all things, He cannot violate His own nature by making a rock that is
larger than He.
Also, since a rock, by definition, is not infinitely big, then it isn't logically possible to make a rock,
something that is finite in size, be infinite in size (no longer a rock) since only God is infinite in size. At
dictionary.com, a rock is defined as a "Relatively hard, naturally formed mineral or petrified matter;
stone. a) A relatively small piece or fragment of such material. b) A relatively large body of such
material, as a cliff or peak. c) A naturally formed aggregate of mineral matter constituting a significant
part of the earth's crust." A rock, by definition is not infinitely large. So, to say that the rock must be
so big that God cannot pick it up is to say that the rock is no longer a rock.
What the critics are asking is that God become self contradictory as a proof He doesn't exist. Their
assertion is illogical from the start. So what they are doing is trying to get God to be illogical. They
want to use illogic to prove God doesn't exist instead of logic. It doesn't work and the "paradox" is
self-refuting and invalid.
God cannot exist because His attributes would require limits

I received an email from a Christian who was taking a philosophy class. He stated that the
following argument as a proof against God's existence was presented in the book, Atheism: The Case
Against God, written by George H. Smith (page 41). I have reproduced the argument given to me in
his email and will attempt to answer it.

If a supernatural being is to be exempt from natural law, it cannot possess specific, determinate
characteristics. These attributes would impose limits and these limits would restrict the capacities of
this supernatural being....A supernatural being, if it is to differ in kind from natural existence, must
exist without a limited nature--which amounts to existing without any nature at all.

If I understand the argument correctly, the argument is that God can't exist because an existent
being would have specific attributes which are naturally limits. These limits would mean it is subject to
natural laws and therefore, not supernatural. Therefore, God cannot exist.
In order to answer this objection, I will break down his statements into an outline with answers
included in the outline.

1. If a supernatural being is to be exempt from natural law, it cannot possess specific,


determinate characteristics.
A. "Determinate" means "precisely limited or defined." A "characteristic" is a distinguishing
feature or prominent aspect of something. An "attribute" is a quality in something, usually
that which identifies a characteristic. A characteristic of God is that He works patiently with
people. An attribute of God is that He is all knowing.
B. I assume that Mr. Smith means that to be "exempt" from natural law means that God is not
restricted to them or by them. That would make sense since, if God exists and if He created
the universe, then by nature, all laws thus created by Him are a reflection of His creative will
and nature and therefore subject to Him.
C. However, I see no logical reason to state that God cannot have limited or defined
characteristics. A supernatural being must, by logical necessity, operate in concert with His
nature. In other words, God could not violate His own nature. This non-self-violation truth
would be a characteristic: the inability of self-contradiction. By default, this is a limitation
upon God that does not negate His existence. Since God is eternal by nature, He could not
destroy Himself and thus violate His attribute of eternality. In fact, such thinking would
demonstrate the logic of His existence by affirming the lack of ability of self-contradiction.
As we can see around us, all things that exist have a nature against which they cannot act in
a contrary manner. A tree cannot be a galaxy; their natures are different. A cat cannot be a
jumbo jet; their natures are different. The fact that a cat cannot be a jet does not mean
that neither the cat nor the jet exist. The same with God. God is limited to His own nature
because He cannot do things which are against His nature. For example, God cannot lie,
stop being God and then become God again, etc. Therefore, the claim that attributes which
necessitate limits disproves God's existence is illogical.
D. Additionally, natural Laws are creations of God and are reflections of His creative nature.
But, they are not beyond Him nor greater than Him since, by definition, God is greater than
all things.
2. These attributes would impose limits and these limits would restrict the capacities of
this supernatural being.
A. As stated above, a restriction of capacity does not negate the possibility of God's existence.
Again, God does not have the capacity to lie, steal, cheat, not be God, or violate His own
nature and this limits do not prove He cannot exist.
3. A supernatural being, if it is to differ in kind from natural existence, must exist without
a limited nature -- which amounts to existing without any nature at all.
A. This is not a logical statement. The "must" in the statement is unqualified. Why "must" a
supernatural being exist without limits? Limited how? In scope, size, duration, extent,
nature, etc.? It is unspecified and, therefore, difficult to address in detail.
B. Furthermore, above I demonstrated that God is limited to the characteristics of His own
nature in that He cannot violate His own nature. His attributes (omniscience, omnipresence,
etc.), which are reflective of His nature, cannot be self-contradictory.
C. Since God is not self-contradictory, He would act in concert with the natural laws which have
been "designed into" the universe. However, since these natural laws are created by Him, He
is in full control of them all and can act in a way that to us is Miraculous.
D. Furthermore, since God would be infinite, any natural laws reflected in a universe would be,
by definition, limited since the universe, a created thing, cannot exceed the scope, nature,
and power of God since it is a created thing. By necessity, then, when God created the
limited universe, the natural laws in it, cannot control or restrict Him since they are finite
and He is infinite. At best, the natural laws are only a reflection of His infinitude and internal
consistency and are subject to His control.

Also, I noticed the switch in terms from characteristic to attribute. Mr. Smith said, "If a
supernatural being is to be exempt from natural law, it cannot possess specific, determinate
characteristics. These attributes would impose limits..." The terms are not defined (at least not in the
email I received) and are not identical words. Yet, Mr. Smith uses them synonymously which, I
believe, weakens his argument further by demonstrating a lack of consistency.
There is no proof that God exists

Sometimes atheists assert that there is no proof that God exists. The only problem is that an
atheist cannot logically make that claim.
In order to state that there is no proof for God's existence, the atheist would have to know all
alleged proofs that exist in order to then state that there is no proof for God's existence. But, since he
cannot know all things, he cannot logically state there is no proof for God's existence.
At best, an atheist can only state that of all the alleged proofs he has seen thus far, none have
worked. He could even say that he believes there are no proofs for God's existence. But then, this
means that there is the possibility that there is a proof or proofs out there and that he simply has not
yet encountered one.
Nevertheless, if there was a proof that truly did prove God's existence, would the atheist be able to
accept it given that his presuppositions are in opposition to the existence of God? In other words,
given that the atheist has a presuppositional base that there is no God, in order for him to accept a
proof for God's existence; he would have to change his presuppositional base. This is not easy to do
and would involve a major paradigm shift in the belief structure of the atheist. Therefore, an atheist is
presuppositionally hostile to any proofs for God's existence and is less likely to be objective about such
attempted proofs.
If God is unchanging, why does the world change if it reveals God?

If God is unchanging, why does the world change if it reveals God? The answer to the question is
found in understanding the difference between the nature of God and the nature of the world. God's
nature does not change. He is always the same, always consistent, always true, etc. The world, on
the other hand, is separate from God. It is His creation and it does not share in His nature. Since it is
different and separate from God, it is not logically necessary that the world be unchanging.
Nevertheless, the Bible states that God is known in creation to some degree: Romans 1:19-20
says, "because that which is known about God is evident within them; for God made it evident to
them. 20For since the creation of the world His invisible attributes, His eternal power and divine nature,
have been clearly seen, being understood through what has been made, so that they are without
excuse." When the Bible says that God's attributes can be known, it is dealing with the quality and
characteristics of God. No created thing can properly represent God. All that is created bears, to
some degree, the fingerprints of God. But each created thing does not sufficiently represent the
creator. Therefore, the creation, the world, does not accurately reflect the nature of God which is
unchangeable.
The attributes of God can, to a degree, be seen in creation. For example, we can see from space
(height, width, and depth), time (past, present, and future), and matter (solid, liquid, gas), that God is
triune since each of the three properties of the universe is comprised of three parts. We can deduce
that God has an aspect to His nature that is three in one since we see it reflected in nature.
We can also see that God is a God of order since the world has order. We can see that God is wise
since that order provides sustenance for the created order. We can see the incredible knowledge of
God in the creation of the variety and complexity of the different animals. We can see that the world
did not come into existence by itself and we can look at the beauty of a sunset and realize that God
has a quality of beauty to Himself. It is these kinds of things that help us to see the attributes of God
in creation.
There will be some who will argue that the world is full of killing, suffering, earthquakes, famine,
etc. If the world demonstrates attributes of God, then is it not fair to say that God likes suffering since
it is in the world? But, God did not create the world this way. He created it good. There was no
suffering in the world when God created it. But, with the fall, with the entrance of sin in the world,
suffering was also introduced and the creation moans to be delivered from its present bondage.

Romans 8:20-22, "For the creation was subjected to futility, not of its own will, but because of
Him who subjected it, in hope 21that the creation itself also will be set free from its slavery to
corruption into the freedom of the glory of the children of God. 22For we know that the whole
creation groans and suffers the pains of childbirth together until now."
Only atheism offers a predictable universe

The following objection was presented on the Atheism discussion board on CARM as an
"evidence" for atheism. It is as follows:

"To my mind, the best evidence for atheism is the predictability of the universe. Atheism (or
perhaps I should say naturalism) posits that there exists nothing capable of circumventing the laws by
which the universe runs. Theism, on the other hand, says that there is an omnipotent being who, by
definition as omnipotent, could cause the universe to run in any manner he/she/it chooses. Any "laws"
we might think we observe are merely the coincidental result of God's choice to make things happen
that way when we're looking. Atheism thus makes a specific prediction that theism does not. It says
that everything within the universe must always follow natural law, since there is no being who could
make it otherwise. Theism has no equivalent prediction.

I will break the argument down into its parts and deal with it accordingly. The argument is
reproduced with comments in an outline form.

1. Premise: "Atheism (or perhaps I should say naturalism) posits that there exists nothing capable
of circumventing the laws by which the universe runs."
A. Response: Naturalism is a logical conclusion for atheists. It maintains that all things in the
universe are the products of natural laws, behave according to natural laws, and that these
laws cannot be violated.
2. Premise: "Theism, on the other hand, says that there is an omnipotent being who, by definition
as omnipotent, could cause the universe to run in any manner he/she/it chooses."
A. Response: This is a subjective statement with an erring premise. There is no ultimate
definition of the actions of God as defined in theism in general. But, the Christian God is
absolute and knows all the laws of the universe, since He incorporated them into the
universe as He created it. This would mean that He knows all laws in the universe and can
perform actions which would appear to violate other laws -- only they do not. The Laws in
the universe are a reflection of the absoluteness of the nature of God.
3. Premise: "Any 'laws' we might think we observe are merely the coincidental result of God's
choice to make things happen that way when we're looking."
A. Response: The laws would not be arbitrary (coincidental). If God exists and He has a
nature, then what He created (laws and all) would be made in a way that is consistent with
His nature. He would not created in a manner insconsistent with Himself because this would
be self-contradictory "The Christian God is eternal and unchangeable (as the Christian
believes according to the Bible). Therefore, the laws in the universe would be consistent and
absolute and also result in predictability. This is why the existence of physical laws in the
universe is just as easily and logically explained by the Christian as the atheist.
4. Premise: "Atheism thus makes a specific prediction that theism does not."
A. Response: On the contrary as demonstrated above. The Christian has every right to claim
the predictability of the universe based upon the absoluteness of God's nature. Instead of
randomness that atheism would suggest since the universe and life are the product of
chance, Christian theism supports absoluteness and consistency based upon the
absoluteness of God's nature.
5. Premise: "It says that everything within the universe must always follow natural law, since there
is no being who could make it otherwise."
A. Response: And what natural laws must God follow? If he creates the universe as a
reflection of His natural absoluteness, then it is logical that the attributes of absoluteness in
physics, etc., also reflect His nature. If miracles occur at the hand of God, then they occur in
a system of laws consistent with His nature. What we observe as supernatural is in reality
natural to God and consistent with His abilities and attributes.
The extent of natural law can and does exist beyond the scope of human understanding.
Take quantum physics as an example; there are things we just do not understand.
Furthermore, if God exists and He created the universe with all that is in it, then why not
admit that there will be laws that may never be fully understood by people? There is no
logical reason that requires that if God exists that His abilities and knowledge of the laws of
the universe (which created) cannot and do not extend beyond the scope of human grasp.
This would mean that the "supernatural" is simply natural to God and miraculous to us.
6. Premise: "Theism has no equivalent prediction."
A. Response: Yes it does. Christian theism states that since God is absolute and created the
universe, it will demonstrate the absolute nature of laws. It further states that the
supernatural, the miraculous, are consistent with God's nature and since God is beyond us,
God's behavior will also often appear beyond us.

This objection is not a proof for God's non-existence and it does not offer a theory of natural law
predictability that Christian theism cannot. Therefore, it is not a proof for atheism.
All of reality and God's existence

The following logical argument against God's existence was offered on the CARM atheism
discussion board. I have reproduced it and shown where the argument is invalid.

1. "Reality" is denotative of all of that which exists.


2. Entity X is postulated to exist outside of reality.
3. Statement 2 reduces to: entity X is postulated to exist outside of all that which exists.
4. Statement 3 is a logical contradiction (semantically) and hence cannot meaningfully
correspond to anything.
5. Therefore, any entity attributed with such an "existence" cannot logically exist (since it
is defined not to exist by placing it outside of reality).

I will break the argument down into its parts and deal with it accordingly. The argument is
reproduced with comments in an outline form.

1. Premise: "Reality" is denotative of all that exists."


A. Response: The atheist is simply stating a definition for what reality is.
2. Premise: "Entity X is postulated to exist outside of reality."
A. Response: This is an illogical statement since it contradicts Premise 1. If reality is all that
exists, then by definition, if God exists, God is part of that reality. He can then not simply
manufacture another premise in contradiction to the first and continue as if there is no
contradiction. Furthermore, the attempt to do so also commits the fallacy of equivocation
where the meaning of "Reality" is altered during the discussion. In Premise one reality is all
that exists. In Premise two, reality is not all that exists.
If a logical argument is then to follow, it cannot contradict the premises which go before
it lest the argument be disproved.
In this case, Premise 2 contradicts premise one. Therefore, premise 2 is illogical and the
premises or conclusion based upon a self contradictory set of premises is most probably
incorrect.
3. Premise: "Statement 2 reduces to: entity X is postulated to exist outside of all that which
exists."
A. Response: Again, postulating that entity X exists outside of all that exists is self
contradictory.
4. Premise: Statement 3 is a logical contradiction (semantically) and hence cannot meaningfully
correspond to anything.
A. Response: Correct. If it is a logical contradiction, then it is illogical. The illogic is in the
statement of the argument in premise two which contradicts premise one. Merely stating
that premise two is as it is, does not mean that it is, especially when it contradicts the
previous premise.
5. Premise: Therefore, any entity attributed with such an "existence" cannot logically exist (since it
is defined not to exist by placing it outside of reality).
A. Response: This is correct. And since God, by definition, would be part of reality since
reality is all that exists, then this argument is invalid and has not disproved the existence of
God.

The problem with the argument above is that it is self contradictory. Therefore, the proof that
there is no God is invalid.
If God exists, then...

Following are four attempted proofs for why God does not exist taken from the CARM atheism
discussion board. I have reproduced to them and addressed each one.

First

If God exists He would make himself clearly known to me.


He has not made Himself clearly known to me.
Therefore God does not exist.

1. Premise: If God exists He would make himself clearly known to me.


A. Response: It is not logically necessary that if God exists He must make Himself known to
any individual. There is no "must" or "ought" or "would" to it as required by an atheist. If
God exists He can do what He wants whether it is to remain undetected or detected to
anyone.
2. Premise: He has not made Himself clearly known to me.
A. Response: This may or may not be the case. It may be that such a person has indeed
encountered a revelation of God. But, if the person has a presuppositional base that
excludes the existence of God, then such evidence of God would be discarded and missed.
3. Premise: Therefore God does not exist.
A. Response: Since premise one and two are not logically necessary and true, the conclusion
does not necessarily follow from the premises.

Second

If God exists, then literalistic fundamental Christianity would integrate seamlessly with all natural
science.
Literalistic, fundamental Christianity does not integrate seamlessly with all natural science.
Therefore God does not exist.

1. Premise: If God exists, then literalistic fundamental Christianity would integrate seamlessly with
all natural science.
A. Response: This is a subjective statement and subjective statements do not make logical
proofs. Nevertheless, there is nothing in the Bible that contradicts scientific fact. Evolution
of species from inorganic material is not a scientific fact. It is a theory. Science agrees with
the Bible.
2. Premise: Literalistic, fundamental Christianity does not integrate seamlessly with all natural
science.
A. Response: This is another subjective statement since it is an over generalized statement.
Furthermore it assumes that the Bible and Science contradict in most, if not all areas. This
is not true.
3. Premise: Therefore God does not exist.
A. Response: Since premise one and two are subjective statements, there is no logical reason
to require the conclusion and it is still possible that God exists.
Third

If God exists there would be convincing proofs of His existence.


There are no convincing proofs of His existence.
Therefore God does not exist.

1. Premise: If God exists there would be convincing proofs of His existence.


A. Response: This is not a logical necessity. To say that there would be convincing proofs of
His existence is an unfounded statement. If God exists, He may or may not choose to
provide sufficient proof for His existence.
The Bible teaches us that God works through people and history and that there is
evidence for His existence. But, people dispose of the evidence due to the hardness of their
hearts, i.e., their presuppositions that negate God's existence.
It may also be that there are convincing proofs of His existence but people choose to
ignore them or explain them away or are not aware of them.
Finally, the premise admits the possibility of proofs of God's existence. But since all
proofs cannot be known by any one person, it is possible that there are proofs that exist that
are not known.
2. Premise: There are no convincing proofs of His existence.
A. Response: This is an opinion since the word "convincing" is included. For some, there is
convincing proof of God's existence. That is why this is a subjective statement, an opinion.
Because it is subjective it is not a proof.
3. Premise: Therefore God does not exist.
A. Response: Therefore, because of the subjective nature of the alleged proof, this attempt
does not disprove God's existence.

Fourth

If God exists in the real world there would be a coherent definition of Him.
There is no coherent definition of God.
Therefore God does not exist outside the human mind.

1. Premise: If God exists in the real world there would be a coherent definition of Him.
A. Response: "coherent definition" is not defined. Therefore, it is not possible to sufficient
respond to the statement since there is not enough information with which to draw logical
conclusions.
Many things exist in the world which are not defined because they have not yet been
discovered. They do not suddenly exist because they are then discovered and defined.
Therefore, existence is independent of definition and whether or not God is defined properly
does not mean He does or does not exist.
2. Premise: There is no coherent definition of God.
A. Response: Depending on what is meant by "coherent definition of God" this statement may
or may not be true. But since no meaning is given to the statement, logically necessary
conclusions cannot be drawn.
Furthermore, to say there is no "coherent definition of God" may be presumptuous since
the person cannot know all things to know whether or not there is a coherent definition of
God. In other words, he cannot say there is no "coherent definition of God" because he does
not know all definitions of God.
3. Premise: Therefore God does not exist outside the human mind.
A. Response: The conclusion does not follow from the premise since a thing defined does not
make it real or not real and also because all definitions of God are not known by the critic so
as to make a logical assertion.
Any entity that is not the source of all power within reality is not God

The following logical offering as an attempt to demonstrate that there is no God is taken from the
CARM atheism discussion board. I have reproduced it here. Following is an analysis.

1. If God exists God must be omnipotent. (If not he is not God)


2. If God is omnipotent then:
A. God is omnipotent because he willed that he would be omnipotent or
B. he is omnipotent without having willed himself omnipotent.
3. A cannot be true because God could not will himself omnipotent prior to being omnipotent.
4. If A cannot be true then A is not true.
5. If A is not true then God is not the source of his own omnipotence.
6. Any entity that is not the source of all potency within reality is not God.
7. Therefore no entity can fulfill the requirements for being a God.
8. If no entity can fulfill the requirements for being a God, then there is no God.

1. Premise: If God exists God must be omnipotent. (If not he is not God)
A. Response: This is reasonable. However, omnipotence does not mean that God can do
anything. Omnipotence means that God can do anything that is consistent with His nature.
God could not violate His own essence and nature. Some would then say that God is not
omnipotent. So be it. But, by what logic can anyone claim that a creature or object can
behave in a manner contrary to its own nature? If this can be established, then
omnipotence can be defined as being able to do anything including violating one's own
nature (behaving in a manner contrary to the nature and abilities and restrictions of its own
nature). If this cannot be logically established, then omnipotence must be the ability to do
anything that is consistent with one's nature, attributes, and essence.
We see this reflected in the Bible when it says, for example, that God cannot lie (Titus
1:2). This is a logical necessity since to lie would be a self contradiction and if God is true,
then He cannot be self contradictory.
2. Premise: If God is omnipotent then:
A. Premise: God is omnipotent because He willed that he would be omnipotent or
i. Response: Omnipotence would be an attribute of God. An attribute of God is a natural
part of God's being. Therefore, God would not will omnipotence in that He was not
omnipotent and then decided He would be omnipotent. This would be a change in His
nature and violates the biblical doctrine of God's immutability -- that He changes not
(Psalm 90:2; Heb. 13:8).
B. Premise: he is omnipotent without having willed himself omnipotent.
i. Response: This would be the more logical statement.
3. Premise: A cannot be true because God could not will himself omnipotent prior to being
omnipotent.
A. Response: Correct.
4. Premise: If A cannot be true then A is not true.
A. Response: This is logically correct since if A cannot be true then it is not true.
5. Premise: If A is not true then God is not the source of his own omnipotence.
A. Response: The problem here is that Premise 2A does not apply to the conclusion of
Premise 5. The negation of 2A is simply that God did not will to become omnipotent
because He already was omnipotent since omnipotence is part of His nature. To will
something means to make a decision and act upon it. In this case it would imply that God
willed to change His nature (by becoming omnipotent when He wasn't) which would pose the
logical problem of God becoming something He was not in violation of His previous nature.
How does something not omnipotent become omnipotent? Where would it get its "all power"
from, something greater than itself? If so, this would mean that God was not God since
there is an omnipotent source greater than God. So, Premise 5 cannot be true.
Second, since God is, by definition, already omnipotent, then He is the source of His own
omnipotence and this further contradicts Premise 5. But, He is not the source in that He
became something He was not or originated something in Himself that was previously non-
existent. This would mean that God changed and would violate the definition of God (at
least the Christian one). He is the source in that it exists within His own nature. As God is,
His omnipotence is.
Third, since 2A is not correct, and 2B is correct, the argument should have addressed
this possibility in contrast to 2A since it raised it. It did, after all, offer two possibilities and
did not negate the second. Instead, it ignored 2B as a logical alternative, thereby
demonstrating that it the argument is insufficient.
6. Premise: Any entity that is not the source of all potency within reality is not God.
A. Response: Since Premise 5 has been shown to be faulty, Premise 6 is not a logical
necessity.
7. Premise: Therefore no entity can fulfill the requirements for being a God.
A. Response: Since Premise 6 has been shown to be faulty, Premise 7 is not a logical
necessity.
8. Premise: If no entity can fulfill the requirements for being a God, then there is no God.
A. Response: Since Premise 7 has been shown to be faulty, Premise 8 is not a logical
necessity.

Logic is good for finding errors in thinking. The error in this argument rests in Premise 5.
Therefore, the concluding argument is not logically necessary and this does not suffice as a proof that
God does not exist.
If everything needs a creator, then who or what created God ?

This question is logically problematic. If everything needs a creator, than no matter what exists, it
must have been created. Furthermore, to be created means that someone or something had to create
it. But then, who created the creator and so on? Logically, this would mean there would be an infinite
regression of creators and we would never be able to find the first, uncaused cause since, by definition
(the questions says that "everything needs a creator") there wouldn't be any uncaused cause. This
would mean that the sequence of creations is eternal. But, if it exists that there is an eternal
regression of creators, then who created the infinite regression of creators? Remember, the question
presupposes that all things need a creator -- even the eternal sequence of creators -- which becomes
logically absurd. Furthermore, if there is an eternal regression of creators that are eternal, then the
question is not answered. In fact, it cannot be answered since it weakness is that "all things need a
creator." Of course, this only begs the question in that how did the process begin? Therefore, the
question only raises the same problem it asks and it is a question that, by its own design, cannot be
answered. Therefore, it is invalid.
The question is better phrased as a statement: "Everything that has come into existence, was
brought into existence by something else." This is a more logical statement and is not wrought with
the difficulties of the initial question. In the revised statement "Everything that has come into
existence," implies that the thing that "has come into existence" did not already exist. If it did not
already exist but then came into existence, then something had to bring it into existence because
something that does not exist cannot bring itself into existence (a logical absolute). This pushes the
regression of creators back to what we would call the theoretical "uncaused cause" since there cannot
be an infinite regression of creators as discussed above and since in infinite number of creators would
mean there was an infinite number of creations and created things including things that cannot be
destroyed since they would constitute things that exist. If that is so, then the universe would have
had an infinite number of created things in it and it would be full. But it is not full. Therefore, there
has not been an infinite regression of creations.
By definition, the Christian God never came into existence; that is, He is the uncaused cause
(Psalm 90:2). He was always in existence and He is the one who created space, time, and matter.
This means that the Christian God is the uncaused cause and is the ultimate creator. This eliminates
the infinite regression problem.
But some may ask, "But who created God?" But the answer is that by definition He is not created;
He is eternal. He is the One who brought time, space, and matter into existence. Since the concept of
causality deals with space, time, and matter, and since God is one who brought space, time, and
matter into existence, the concept of causality does not apply to God since it is something related to
the reality of space, time, and matter. Since God is before space, time, and matter, the issue of
causality does not apply to Him.
How can something that cannot be described be said to exist?

Whether or not something exists does not depend on whether or not it can be described. If
something exists, it exists independently of someone's ability to describe it. That is the nature of
existence; that is, things exist whether or not someone knows it exists, let alone accurately describe
its existence. Furthermore, something that exists is not limited in existence by the inability of
someone to accurately describe its existence. For example, there are galaxies and phenomenon within
galaxies that have not yet been discovered. But this does not mean that they do not exist since we
cannot describe knowledgeably, sufficiently, or accurately.
We know that things exist which are difficult to adequately describe. A rainbow is another good
example. A verbal description of a rainbow cannot compare to actually seeing one. By default, the
description is inadequate as compared to the reality of eyewitness encounter. But, that does not mean
that rainbows do not exist because we cannot adequately describe them with words.
As far as God is concerned. God can be described, though not perfectly. He can be sufficiently
described so that we can gain an understanding of His nature, greatness, and differences from
ourselves. Though we will always fall short when trying to describe an infinite being, we can still say
enough about Him to convey who and what He is so that the concept of God can be communicated.
We can say that He is infinite, gracious, loving, all knowing, all-powerful, all present, holy, pure,
righteous, that he is not flesh and bones, the only uncaused and infinitely eternal being in existence,
etc. Though we may not be able to perfectly describe Him we can describe Him by listing His
attributes and qualities. Attributes and qualities reflect the nature of the thing described. Therefore, we
can describe God and our insufficiency to adequately describe Him in certain areas does not diminish
His existence nor mean that He does not exist. It simply means that He is great.
Why do you believe in Jesus but not Santa Claus?

Sometimes atheists will ask why Christians believe in Jesus and not Santa Claus. Some atheists
say that you cannot prove Jesus existed anymore than you can prove Santa Claus exists. Of course,
this is not accurate for several reasons.
First of all, Jesus is presented as a historical figure by reputable people in both secular and sacred
historical writings. Santa Claus is simply presented as a fictional character.
Second, Jesus is presented as a real person who claimed to be divine and who performed miracles.
These accounts are attested to by reputable witnesses and have been transmitted to us reliably; the
New Testament documents are 99.5% textually pure. Santa Claus is intentionally and knowing
presented as a fictional character who lives at the North Pole.
Third, the intention of the gospel writers was to convey the physical reality of Jesus to responsible
adults where the accounts of Santa are intended to entertain the wild imaginations of children. This is
why the vast majority of healthy, mentally competent adults do not believe in a real person known as
Santa who can travel through air being pulled by several flying reindeer, who can carry in his sled
enough presents for all the good children in the entire world, and who can descend and ascend
through chimneys even though he is quite overweight. 176
Fourth, the writings concerning Jesus exhibit a historical, cultural, religious, and political context
with verifiable names, events, and places being an integral part of the record of that context and
reality. Santa Claus stories do not contain any such integral contextualization except to state that
there is a north pole and that there are cities and countries where Santa visits at night.
Fifth, the facts are that parents are the ones who buy, wrap, and deliver presents to children and
we know of no documented occurrences where Santa Claus has been caught breaking and entering,
tripping home alarm systems, caught on film, vanishing up a chimney, and riding a sleigh through the
air pulled by flying reindeer. This latter point is worth commenting on since we additionally have no
evidence at all that reindeer can fly which further adds to the irrationality of the Santa Claus story.
Additionally, if a large sleigh (sufficient to carry millions of toys) approached the Washing D.C. area
(surely there are at least some good children there), we would expect to hear of military fighter jets
being scrambled to intercept the intruder. No records of this have yet surfaced.
Sixth, given that the gospel accounts were written by individuals who knew Jesus personally (or
were under the guidance of those who knew Him), that the gospels are historically accurate, superbly
transmitted to us through the copying method, we can then assume at the very least, that Jesus was
an actual historical person. But, we have no hard evidence to establish the validity of Santa Claus.
We have found no reindeer tracts on the roofs of snow covered homes strewn about millions of homes
on Christmas Eve. There are no video accounts of Santa roaming throughout peoples' homes. We
know of no flying reindeer, and no one has yet established how Santa can live at the North Pole for
hundreds of years without being detected -- particularly in this technologically advanced culture. Add
to that the lack of Santa Disciples going about the world, risking their lives, being ridiculed by religious
and political adversaries, writing inspirational text, performing miracles, etc., and you really don't have
much evidence at all that Santa exists except in the mind of children.
Finally, it really comes down to whether or not either one can be reasonably supported to exist.
Very few people deny the historic reality of Jesus and though millions of children affirm the existence
of Santa, we know well that the minds of children are not capable of differentiating between fantasy
and reality -- particularly when the parents they are trusting tell them that Santa is real.
For an atheist to reject Jesus' existence based on arguments found against Santa Claus
demonstrates the inability for the atheist to distinguish between historical verifiable documents and
known constructed children's' stories. Jesus was an actual historical figure. Santa, of course, is not.

176
Granted there are probably some adults why may believe in Santa Claus as a reality, but I suspect they would
be mental patients or some other such people not fully in touch with reality.
Why believe in Christianity over all other religions?

Critics often ask why Christianity is any better than any other religion in the world. After all, of
all the religions that exist how can it be that only Christianity is true? If God exists, why can't God use
different religions? Don't all paths lead to God? These kids of questions are asked all the time of
Christians and unfortunately the answers aren't always very good ones particularly when dealing with
people who have a relativistic truth base and don't believe in absolutes.
Therefore, in an attempt to demonstrate why Christianity is true and all other religious systems are
false, I've prepared the follow list of reasons for Christianity's superiority.

There are such things as absolute truths

If truth is relative, then the statement that truth is relative is an absolute truth and would be self
defeating statement by proving that truth is not relative. But, if truth is absolute, then the statement
"truth is absolute" is true and not self defeating. It is true that truth exists. It is true that truth will
not contradict itself as we have just seen. In fact, it is absolutely true that you are reading this paper.
If we can see that there is such a thing as truth in the world, then we could also see that there can
be spiritual truth as well. It is not absurd to believe in spiritual absolutes anymore than physical or
logical absolutes. Even the statement that all religions lead to God is a statement held to be a spiritual
absolute by many people. This simply demonstrates that people do believe in spiritual truth. Why?
Because truth exists. However, not all that is believed to be true actually is true. Therefore, all belief
systems cannot be true since they often contradict each other in profound ways.

Religions contradict each other; therefore, they cannot all be true.

Mormonism teaches that there are many gods in existence and that you can become a god.
Christianity teaches that there is only one God and you cannot become a god. Islam teaches that
Jesus is not God in flesh where Christianity does. Jesus cannot be both God and not God at the same
time. Some religions teach that we reincarnate while others do not. Some teach there is a hell and
others do not. They cannot all be true. If they cannot all be true, it cannot be true that all religions
lead to God. Furthermore, it means that some religions are, at the very least, false in their claims to
reveal the true God (or gods). Remember, truth does not contradict itself. If God exists, He will not
institute mutually exclusive and contradictory belief systems in an attempt to get people to believe in
Him. God is not the author of confusion (1 Cor. 14:33). Therefore, it is reasonable to believe that
there can be an absolute spiritual truth and that not all systems can be true irregardless of whether or
not they claim to be true. There must be more than a mere claim.

Fulfilled Prophecy concerning Jesus

Though there are other religions that have prophecies in them, none are 100% accurate as is the
Bible and none of them point to someone like Jesus who made incredible claims and performed
incredible deeds. The Old Testament was written hundreds of years before Jesus was born. Yet, the
Old Testament prophesied many things about Jesus. This is undoubtedly evidence of divine influence
upon the Bible.
Please consider some of the many prophecies of Jesus in the following chart.
Old Testament New Testament
Prophecy
Prophecy Fulfillment
Born of a virgin Isaiah 7:14 Matt. 1:18,25
Born at Bethlehem Micah 5:2 Matt. 2:1
He would be preceded by a Messenger Isaiah 40:3 Matt. 3:1-2
Rejected by His own people Isaiah 53:3 John 7:5; 7:48
Betrayed by a close friend Isaiah 41:9 John 13:26-30
His side pierced Zech. 12:10 John 19:34
Psalm 22:1, Luke 23:33;
Crucifixion
Psalm 22:11-18 John 19:23-24
Resurrection of Christ Psalm 16:10 Acts 13:34-37

Fulfillment of prophecy can have different explanations. Some state that the NT was written and
altered to make it look like Jesus fulfilled OT prophecy (but there is no evidence of that). Others state
that the prophecies are so vague that they don't count (but many of the prophecies are not vague at
all). Of course, it is possible that God inspired the writers and Jesus, who is God in flesh, fulfilled
these prophecies as a further demonstration of the validity of Christianity.

The Claims and Deeds of Christ

Christianity claims to be authored by God. Of course, merely making such a claim does not make it
true. Anyone can make claims. but, backing up those claims is entirely different. Jesus used the
Divine Name for Himself (John 8:58), the same Divine Name used by God when Moses asked God
what His name was in (Exodus 3:14). Jesus said that He could do whatever He saw God the Father do
(John 5:19), and He claimed to be one with the God the Father (John 10:30; 10:38). Likewise, the
disciples also called Him God (John 1:1,14; John 10:27; Col. 2:9). By default, if Jesus is God in flesh,
then whatever He said and did would be true. Since Jesus said that He alone was the way, the truth,
and the life and that no one can find God without Him (John 14:6), this all becomes incredibly
important.
Again, making a claim is one thing. Backing it up is another. Did Jesus also back up His words
with His deeds? Yes, He did.

• Jesus changed water into wine (John 2:6-10).


• Jesus cast out demons (Matt. 8:28-32; 15:22-28).
• Jesus healed lepers (Matt. 8:3; Luke 17:14).
• Jesus healed diseases (Matt. 4:23,24; Luke 6:17-19)
• Jesus healed the paralytic (Mark 2:3-12).
• Jesus raised the dead (Matt. 9:25; John 11:43-44).
• Jesus restored sight to the blind (Matt. 9:27-30; John 9:1-7).
• Jesus restored cured deafness (Mark 7:32-35).
• Jesus fed the multitude (Matt. 14:15-21; Matt. 15:32-38).
• Jesus walked on water (Matt. 8:26-27).
• Jesus calmed a storm with a command (Matt. 8:22-27; Mark 4:39).
• Jesus rose from the dead (Luke 24:39; John 20:27).
• Jesus appeared to disciples after resurrection (John 20:19).

The eyewitnesses recorded the miracles of Jesus and the gospels have been reliably transmitted to
us. Therefore, we can believe what Jesus said about Himself because Jesus performed many
convincing miracles in front of people who testified and wrote about what they saw Him do.
Christ's resurrection

Within Christianity, the resurrection is vitally important. Without the resurrection, our faith is
useless (1 Cor. 15:14). It was the resurrection that changed the lives of the disciples. After Jesus was
crucified, the disciples ran and hid. But when they saw the risen Lord, they knew that all that Jesus
had said and done proved that He was indeed God in flesh, the Savior.
No other religious leader has died in full view of trained executioners, had a guarded tomb, and
then risen three days later to appear to many many people. This resurrection is proof of who Jesus is
and that He did accomplish what He set out to do: provide redemption for mankind.
Buddha did not rise from the dead. Muhammad did not rise from the dead. Confucius did not rise
from the dead. Krishna did not rise from the dead, etc. Only Jesus has physically risen from the dead,
walked on water, claimed to be God, and raised others from the dead. He has conquered death. Why
trust anyone else? Why trust anyone who can be held by physical death when we have a Messiah who
is greater than death itself?

Conclusion

Why should anyone trust in Christianity over Islam, Buddhism, Mormonism, or anything else? It is
because there are absolute truths, because only in Christianity is there accurate fulfilled prophecies of
a coming Messiah. Only in Christianity do we have the extremely accurate transmission of the
eyewitness documents (gospels) so we can trust what was originally written. Only in Christianity do
we have the person of Christ who claimed to be God, performed many miracles to prove His claim of
divinity, who died and rose from the dead, and who said that He alone was the way the truth and the
life. All this adds to the legitimacy and credibility of Christianity above all other religions -- all based
on the person of Jesus. If follows that if it is all true about what Jesus said and did, then all other
religions are false because Jesus said that He alone was the way, the truth, and the life and that no
one comes to the Father except through Him (John 14:6). It could not be that Jesus is the only way
and truth and other religions also be the truth.
Either Jesus is true and all other religions are false or other religions are true and Jesus is false.
There are no other options. I choose to follow the risen Lord.
Entropy and Causality used as a proof for God's existence

Definition: The second law of thermodynamics states that the amount of energy in a system that is
available to do work is decreasing. Entropy increases as available energy decreases. In other words,
the purely natural tendency of things is to move toward chaos, not order, and available energy
necessary for work is lost in this process. Eventually, the universe will run down and all life and motion
will cease. This is the natural tendency of all things. Batteries run down, machines break, buildings
crumble, roads decay, living things die, etc. Left to the natural state, all things would eventually cease
to function.

1. The universe is not infinitely old because it has not “run down.”
A. If the universe were infinitely old, it would have reached a where all usable energy is gone.
B. But, we are not in this state; therefore, the universe is not infinitely old and must have had a
beginning.
2. Because the universe has had a beginning it is not infinite in size.
A. It would require an infinite amount of time to become infinite in size. Since the universe had
a beginning, it has not had an infinite amount of time to expand, therefore it is finite in size.
3. All events have causes.
A. There cannot be an infinite regress of events because that would mean the universe were
infinitely old.
i. If it were infinitely old, the universe would be in a state of unusable energy, which it is
not.
ii. If it were infinitely old, the universe would be infinitely large, which it is not.
4. Since the universe is finite and had a beginning and there cannot be an infinite number
of regressions of causes to bring it into existence, there must be a single uncaused
cause of the universe.
A. A single uncaused cause of the universe must be greater in size and duration than the
universe it has brought into existence.
i. Otherwise, we have the uncaused cause bringing into existence something greater than
or equal to itself.
B. Any cause that is natural to the universe is part of the universe.
i. An event that is part of the universe cannot cause itself to exit.
ii. Therefore, there must be an uncaused cause outside the universe.
C. An uncaused cause cannot be a natural part of the universe which is finite.
i. An uncaused cause would be infinite in both space and time since it is greater than
which it has caused to exist.
5. This uncaused cause is supernatural.
A. By supernatural I mean it is completely 'other' than the universe is not natural to it.
i. This would make the uncaused cause supernatural.
ii. This uncaused cause is God.

The Bible teaches that God is infinite in time and scope and is wholly other than the universe of which
He created.
God is defined as being infinite in size, duration, and power.
Therefore, the God of the Bible is the uncaused cause of the universe.
Atheism, Evolution, and Purpose
This outline attempts to show that the evolutionary theory, based on naturalistic principles, leads to
purposelessness.
Purpose is indicative of a purpose giver, a designer. I propose that God gives us purpose. Also,
with this outline, I am trying to show that the best the naturalist position can offer is an illusion of
purpose.

Premise - from an Atheistic Perspective.

1. The universe exists.


2. The universe has principles and laws inherent in its properties of matter, energy flow, chemical
reaction, etc.
A. Any derivative principles based upon the laws must be consistent with the inherent laws.
3. These inherent, natural laws cannot be violated.
A. Any apparent violation of these laws is only a display of our lack of understanding of all the
laws and is consistent with more complex inherent laws.
4. Life is the product of these inherent natural laws of the universe.
A. That is, due to the properties of matter and energy, life necessarily arose since we exist.
5. Life can only develop in harmony with the natural laws in the universe.
6. Life is limited to and governed by these inherent principles since life is a product of the inherent
laws and cannot violate them.
7. Therefore,
A. Human existence, thoughts, feelings, etc., are merely the end result of the inherent
universal laws and principles of matter, energy flow, chemical reaction, etc. that has resulted
in life.

Question: From an atheistic point of view, what purpose does Mankind have for
existence?

8. Since the laws of the universe are immutable and cannot be violated, any reason given by an
atheist for claiming purpose in existence can be properly attributed to be the result of chemical
reactions in his/her brain leading him to say he has purpose.
A. The atheist, therefore, is nothing more than the product of your environment and naturals
laws.
i. He is guided and lead by these Laws and react, plot, hope, and will only in agreement
with these Laws.
ii. Any purpose thus offered is still nothing more than the product of natural Laws of
matter, chemistry, and energy flow. In other words, the atheist is nothing more than the
result of naturals laws inherent in the natural universe.
B. If the atheist admits that his mind is the derivative product of these natural Laws, but that
his mind and will have "risen above" these laws and that he is now able to escape the
limitations of the natural laws and give himself purpose...
i. Then it can still be asserted that his reasoning is nothing more than the result of
chemical reactions in his brain causing him to say and believe this.
ii. Then he has violated principle 5 above which is.
a. Life can only develop in harmony with the natural laws in the universe.
C. If the atheist states that the natural laws are not exhaustively known and that they can
produce truly "free-will" creatures,
i. Then he is making his point based upon what we do not know about the natural laws
and stating that since we do not know what they can do, therefore, I am free to not be
bound by the natural laws.
ii. Then it is, essentially, an argument from silence.
D. Therefore, from the atheist perspective, he is not independent, autonomous, nor does he
possess a free will.
Conclusion

9. Therefore, the Concept of "Having a Purpose" becomes meaningless because


A. The atheist has no purpose beyond the programming inherent in himself.
i. Therefore, he has no independence and no free will.
B. If he claims he is thinking in harmony with the limitations imposed by Natural Laws and that
the sum of his evolution is greater than those individual Natural Laws, then he has again
violated principle 5 above.
i. Principle 5: Life can only develop in harmony with the inherent laws in the universe.
a. If life only develops in harmony with the laws then it is restricted to those laws and
cannot exceed them.
C. Also, it can still be said that the atheist claim of independence is nothing more than the
chemical reactions in his brain.
10. If the atheist says he has purpose not derived from or that is beyond the mere derivation of life
from the original, inherent natural Laws, then....
A. This implies the existence of the supernatural.
i. If the supernatural exists, then it is certainly possible that God exists.
B. The atheist is denying the principles from which evolution is derived.
i. This would mean that evolution is not true, and/or
11. If the atheist acknowledges that his mind, will, hopes, desires, etc., are nothing more than the
product of the natural universe, then...
A. He has no self-determined purpose.
B. He has no will other than that which is governed by the natural laws and programmed within
him.
C. He serves nothing more than natural laws.
12. Therefore, the atheist has no freely chosen, self intended purpose for existence.
13. If there is a God, then I have purpose, since I have a will and my purpose is given to me by God.
A. Since I claim to have a purpose, not derived from natural Laws, it follows that I claim there
is a God.
i. Since, to claim purpose outside the natural is to conclude that purpose is derived from
something beyond the natural.
14. Since I determine I have a purpose and I deny the limitations of the boundaries set by Natural
Laws, it is reasonable to assume that I believe in God and that there is a God.
A. Otherwise, we are merely bags of chemicals reacting to stimuli. I believe man is more than
that.
The Christian Worldview, the Atheist Worldview, and Logic.
Can the atheistic worldview present a logical reason why its worldview can account for the abstract
laws of logic? I think not. But, the Christian world view can. The Christian worldview states that God
is the author of truth, logic, physical laws, etc. Atheism maintains that physical laws are properties of
matter, and that truth and logic are relative conventions (agreed upon principles). Is this logically
defensible?
I present this outline in hopes of clarifying the issue and presenting, what I consider, an
insurmountable problem of the atheistic worldview. I hesitate to state that this is a proof that God
exists, but I think that it is evidence of the Absolute Nature of God.
This argument is adapted from the Transcendental Argument championed by Greg Bahnsen.

1. How does a Christian account for the laws of logic?


A. The Christian worldview states that God is absolute and the standard of truth.
B. Therefore, the absolute laws of logic exist because they reflect the nature of an absolute
God.
i. God did not create the laws of logic. They were not brought into existence since they
reflect God's thinking. Since God is eternal, the laws of logic are too.
C. Man, being made in God’s image, is capable of discovering these laws of logic. He does not
invent them.
D. Therefore, the Christian can account for the existence of the Laws of logic by acknowledging
they originate from God and that Man is only discovering them.
2. How does the atheist account for the laws of logic?
A. If the Atheist states that the laws of logic are conventions (mutually agreed upon
conclusions), then the laws of logic are not absolute because they are subject to "vote."
B. The laws of logic are not dependent upon different peoples minds since people are different.
Therefore, they cannot be based on human thinking since human thinking is often
contradictory.
C. If the atheist states that the laws of logic are derived through observing natural principles
found in nature, then he is confusing the mind with the universe.
i. We discover laws of physics by observing and analyzing the behavior of things around
us. The laws of logic are not the result of observable behavior of object or actions.
a. For example, we do not see in nature that something is both itself and not itself at
the same time.
i. Why? Because we can only observe a phenomena that exists, not one that does
not exist. If something is not itself, then it doesn't exist. How then can the
property of that non-existent thing be observed? It cannot.
ii. Therefore, we are not discovering a law of logic by observation, but by thought.
b. Or, where do we observe in nature that something cannot bring itself into existence
if it does not already exist?
i. You cannot make an observation about how something does not occur if it does
not exist. You would be, in essence, observing nothing at all and how can any
laws of logic be applied to or derived from observing nothing at all?
ii. The laws of logic are conceptual realities. They only exist in the mind and they do not
describe physical behavior of things since behavior is action and laws of logic are not
descriptions of action, but of truth.
a. In other words, laws of logic are not actions. They are statements about conceptual
patterns of thought. Though one could say that a law of physics (i.e., the angle of
reflection is equal to the angle of incidence) is a statement which is conceptual, it is
a statement that describes actual physical and observable behavior. But, logical
absolutes are not observable and do not describe behavior or actions of things since
they reside completely in the mind.
b. We do not observe the laws of logic occurring in matter. You don't watch an object
NOT bring itself into existence if it doesn't exist. Therefore, no law of logic can be
observed by watching nothing.
D. If the atheist appeals to the scientific method to explain the laws of logic then he is using
circular argumentation because the scientific method is dependent upon logic; that is,
reasoned thought applied to observations.
E. If logic is not absolute, then no logical arguments for or against the existence of God can be
raised and the atheist has nothing to work with.
F. If logic is not absolute, then logic cannot be used to prove or disprove anything.
3. Atheists will use logic to try and disprove God’s existence, but in so doing they are
assuming absolute laws of logic and borrowing from the Christian worldview.
A. The Christian worldview maintains that the laws of logic are absolute because they come
from God who is Himself absolute.
B. But the atheist worldview does not have an absolute God.
i. So, we ask, "How can absolute, conceptual, abstract laws be derived from a universe of
matter, energy and motion?"
ii. In other words, "How can an atheist with a naturalistic presupposition account for the
existence of logical absolutes when logical absolutes are conceptual by nature and not
physical, energy, or motion?"

Conclusion

The Christian theistic worldview can account for the laws of logic by stating that they come from God.
.A God is transcendent; that is, He is beyond the material universe being its creator.
.B God has originated the laws of logic because they are a reflection of His nature.
.C Therefore, the laws of logic are absolute.
.D The are absolutes because there is an absolute God.
The atheistic worldview cannot account for the laws of logic/absolutes, and must borrow from the
Christian worldview in order to rationally argue.
An answer to a refutation of the Transcendental Argument
The Transcendental Argument is the argument that attempts to prove God's existence by arguing
that logic, morals, and science presuppose the Christian world view; that it must be that God's
absolute nature is the source of logic, morals, and the laws of science.
Some argue that the transcendental argument is insufficient and flawed. One such argument
comes from Michael Martin at Infidels.org a website with has obvious anti-Christian overtones.
Following are excerpts from Mr. Martin's paper on that site titled "The Transcendental Argument for
the Nonexistence of God." Mr. Martin's paper is worth a read, but, in my opinion, it is flawed. He
said,

"Consider logic. Logic presupposes that its principles are necessarily true. However, according
to the brand of Christianity assumed by TAG [transcendental argument for God], God created
everything, including logic; or at least everything, including logic, is dependent on God. But if
something is created by or is dependent on God, it is not necessary -- it is contingent on
God..."

If God exists, then God has attributes such as thoughts, character, essence, nature, etc. His
attributes would be perfect since He would be, by default, the standard of perfection, the omniscient,
omnipotent creator.. His thoughts would necessarily be consistent within Himself: perfect. God is by
nature is non-self-contradictory since a self-contradictory thing cannot exist. Logic, then, would not be
a created thing, but an attribute of God's perfect existence and nature as it relates to His thought
processes. Since God exists eternally in all places, logic cannot have been created, but is, so to speak,
eternal as well. In other words, the existence of logic based upon God's existence is indeed a
necessary thing since it would be a "part" of God's own natural thought processes. It would then be
something innate, natural, and ontologically necessary because God exists and not contingent on
anything but God. Therefore, logic would not be a creation of God, but a concomitant necessary truth
in existence because God exists.

"And if principles of logic are contingent on God, they are not logically necessary. Moreover, if
principles of logic are contingent on God, God could change them. Thus, God could make the
law of non-contradiction false; in other words, God could arrange matters so that a proposition
and its negation were true at the same time. But this is absurd. How could God arrange matters
so that New Zealand is south of China and that New Zealand is not south of it? So, one must
conclude that logic is not dependent on God, and, insofar as the Christian world view assumes
that logic so dependent, it is false.

As I stated above, logic is a necessary, uncreated, and non-contingent necessity given God's
absolute, perfect, and eternal existence. Logic would not be changeable because God is not
changeable. This explains why logical truths are always true. They are absolute and transcendent by
nature because God is absolute and transcendent by nature. Logic is non-self-contradictory, because
God is non-self-contradictory.

"Consider science. It presupposes the uniformity of nature: that natural laws govern the world
and that there are no violations of such laws. However, Christianity presupposes that there are
miracles in which natural laws are violated. Since to make sense of science one must assume
that there are no miracles, one must further assume that Christianity is false. To put this in a
different way: Miracles by definition are violations of laws of nature that can only be explained
by God's intervention. Yet science assumes that insofar as an event as an explanation at all, it
has a scientific explanation -- one that does not presuppose God. Thus, doing, science assumes
that the Christian world view is false."

We do not know all the laws of the universe. God does, since He brought the universe into
existence. What we would consider supernatural may simply be the working of natural laws on a level
that only God understands. Mr. Martin's assertion is not necessitate a violation of natural laws; rather,
a violation of the extent of natural laws as they are known now.

"Consider morality. The type of Christian morality assumed by TAG [transcendental argument
for God] is some version of the Divine Command Theory, the view that moral obligation is
dependent on the will of God. But such a view is incompatible with objective morality. On the
one hand, on this view what is moral is a function of the arbitrary will of God; for instance, if
God wills that cruelty for its own sake is good, then it is. On the other hand, determining the
will of God is impossible since there are different alleged sources of this will (The Bible, the
Koran, The Book of Mormon, etc) and different interpretations of what these sources say;
moreover; there is no rational way to reconcile these differences. Thus, the existence of an
objective morality presupposes the falsehood of the Christian world view assumed by TAG."

Mr. Martin uses the phrase "objective morality." That phrase in itself is worthy of a long
discussion. Nevertheless, I see no reason to state that moral obligation being dependent upon the will
of God is incompatible with objective morality. Furthermore, God's will is not arbitrary. It is
consistent with His own nature and purpose which is absolute by definition of an omniscient God. In
other words, and God of omniscience eternally knows all things and his eternal known would always
have been known. Therefore, it could not be arbitrary. Furthermore, to state that God and His will are
arbitrary is to state that God can be self-contradictory which is illogical. Mr. Martin has failed to see
this.
Mr. Martin further fails to see the philosophical difference between the Islam and Mormonism. In
Islam, Allah is capricious. In Mormonism, God is changeable and not absolute. The God of Christianity
is eternally immutable. There is a large difference between the Christian God and the ones of the
others he mentioned.

"There are, of course, ways to avoid the conclusions of TANG [Transendental Argument for the
Non-Existence of God]. One way is to reject logic, science and objective morality. Another is to
maintain belief in God but argue that logic, science and morality are not dependent on God's
existence. However, the first way is self-defeating since Christian apologists use logic to defend
their position and the second way presumes that TAG is invalid since it assumes that logic,
science, and morality do not assume God's existence. Finally, one can object to particular
aspects of TANG, for example, the claim that there is no rational way to reconcile different
interpretations of the Bible. However, this tack would involve a detailed defense of TAG --
something that has yet to be provided."

Mr. Martin is stating that it is the theist who would need to reject logic. But since Mr. Martins TANG
has logical weaknesses, it would seem that it is he who would need to reject logic in order to retain his
atheist position.
An answer to another response to the Transcendental Argument

The following paper is my rebuttal to an attempted rebuttal by an atheist of my paper called The
Christian Worldview, the Atheist Worldview, and Logic located at
http://www.carm.org/atheism/logic.htm. The parts of the original paper that the atheist has quoted
are in small caps. The atheist comments are in black. My response is in blue. The atheist goes by the
nick name of "ohwow" on the CARM discussion boards where his original post was made on Saturday,
January 25th, 2003.

----------------

Original paper: "CAN THE ATHEISTIC WORLDVIEW PRESENT A LOGICAL REASON WHY ITS WORLDVIEW CAN ACCOUNT FOR THE
ABSTRACT LAWS OF LOGIC?"
Atheist comment: First, you are asking us to provide a reason based on logic, so you are asking
us to beg the question that logical absolutes exist.
Response to comment: No, this is not begging the question. Begging the question means you
assume the thing to be true that you are attempting to prove. I am not asking for a proof that logical
absolutes exist and I am not trying to prove their existence. I am asking atheists to provide an
explanation for their existence -- because they already exist. This atheist, by default, agrees to this
since he is trying to use logic to refute my paper. Therefore, he assumes the existence of logical
absolutes.
If there were no absolutes in logic, then logic would be relative. If logic were relative then no
argument can neither be won or lost, or proven or disproved since it would all be relative anyway. But,
in order to prove or disapprove a premise, there must be logical absolutes by which error can be
discovered.
My statement does not qualify as a logical fallacy since there is nothing that logically prevents
someone to use logic in an attempt to discuss logic or attempt to account for the existence of logic.
This atheist is, therefore, in error in his premise that I have committed the logical fallacy of "begging
the question." Furthermore, if his premise is in error and he builds upon this error, his conclusions will
most probably also be in error.

For example: Can you give a rational reason why rationality exists? If your answer is itself
rational , (based on rationality) your entire answer is then based on the prior assumption that
rationality does exist, which equals "question begging".
Again, this is not begging the question. The problem is not with using logic to account for the
existence of logic since using that which exists does not invalidate the discussion of the thing that
exists. There is no 'begging the question' fallacy in here. He has misapplied what the fallacy is.

Second, if "account for logical absolutes", means "tell us WHY they exist", well, If you really are
asking "WHY do logical absolutes exist", your question itself assumes logical absolutes "came from"
some source. You need to be careful when asking "why x exists", because that kind of question
assumes x must derive from something outside itself. We would all presume that YOU originate from a
source outside yourself, therefore we sense no problem within the question "how do we account for
Matt Slick?" Therefore, if you ask similarly "how do we account for logical absolutes?" you are
presuming that they do indeed come from a source outside themselves, so your question itself is
loaded in a way that atheists cannot answer it directly, just like "have you stopped beating your wife"
can't be answered directly by someone who never beat his wife. Therefore rephrase your question to
atheists so it isn't loaded.
Note, I did not say "tell us WHY they exist." I asked how an atheist accounts for their existence.
There is a difference, even if it is subtle. Atheists and Christians both know that logical absolutes exists
since we both use them. I am simply asking atheists to give a rational reason for their existence in the
context of their atheism.
Furthermore, the question is not asked in such a way that atheists cannot answer it. But, if this
atheist admits that it is something he cannot answer, then he helps to validate the premise of the
paper that atheists cannot account for them where the Christian can.

"THE CHRISTIAN WORLDVIEW STATES THAT GOD IS THE AUTHOR OF TRUTH, LOGIC, PHYSICAL LAWS, ETC."
You never demonstrate that god DID create logic, you just "state" that god authored such.
First of all, I did not ask HOW God would create such a thing as logic. Second, the paper is not
intended to be a demonstration that God created logic. Third, the paper asks the question of the
atheist in the context of the atheist worldview, how the atheist could account for the existence of
logical absolutes. Further, I demonstrate that within the Christian worldview we are able to account for
the existence of logical absolutes. Fourth, the fact that this atheist is having difficulty answering to
charge of the paper is a demonstration that his worldview is not capable of answering the question at
hand which helps to validate the premise of the paper. Fifth, I do not hold the position that God
"created" logic. I believe I can logically defend the premise (later in this paper) that logical absolutes
are a reflection of the mind of God. He did not invent them. In other words, there was no time when
they did not exist and then God decided to bring them into existence. It is God who is eternal by
nature, who is the author of "absoluteness" by the very nature of His eternal and absolute existence.
Therefore, within this perspective God would not have created logic. It would have already existed
because it is a reflection of His eternal mind.

"ATHEISM MAINTAINS THAT PHYSICAL LAWS ARE PROPERTIES OF MATTER, AND THAT TRUTH AND LOGIC ARE RELATIVE CONVENTIONS
(AGREED UPON PRINCIPLES). IS THIS LOGICALLY DEFENSIBLE?"
If matter and energy are NOT essential to logic, you should be able to prove that logic can exist
apart from them. Suppose all matter and energy in the universe disappear, where then would 'logic' be
found? In the spot where the Big Dipper was? Clearly logic cannot make sense unless matter and
energy exist first, which allow us to provide examples to prove logical laws. What logical law does NOT
apply to realities of matter and energy, and can be demonstrated as such without appealing to things
constituted of matter or energy?
One of the points of the original paper is that the nature of logical absolutes is not physical, but
conceptual. This is an ontological [deals with the nature of something] difference between the nature
of the physical universe and conceptual realities. What I perceive to exist in my mind does not mean
that it exists in the physical universe. If I perceive that I am actually larger in mass than the sun, my
perception has no bearing upon the reality or lack of reality of it.
This atheist simply states that matter and energy are essential to logic yet he does not
demonstrate how they are. When he asks "Suppose all matter and energy in the universe disappear,
where then would logic be found?", he fails to discuss the nature or essence of logic and absolutes
which is so necessary in this discussion. He mentions the "physical" universe, but does not mention
the "conceptual" nature of logic. He ignores the latter and attempts to contrast it with the former
without developing a sufficient discussion of the nature of each or how they relate. This is important
because the nature of something reveals its characteristics. If the nature of something is that it is not
physical and energy, then for him to assume that it is dependent upon the physical universe would not
necessarily be logical.
Additionally, he asks questions without providing answers. When I asked the atheist to
demonstrate or give a logical reason for the existence of "logical absolutes", I did not stop there. I
went on to provide a solution that could account for their existence. This solution is easily explainable
in a Christian presuppositional system, but not so easily explainable in an atheistic one.

Another problem, if you are asking atheists to account for "why" logic exists, you should not
trumpet too loudly your victory if the atheist says "I don't know". There will be one atheist in the
crowd who says "can you tell us WHY god exists?", and then they will put the trumpet to their lips,
ready to blast as soon as you say "I can't answer that."
The point of the paper is not to trumpet a victory. The purpose of the paper is to demonstrate that
the atheistic system cannot account for logical absolutes where a Christian one can. Of course, the
Christian assumption is that God is the "source" of logical absolutes.
"HOW DOES A CHRISTIAN ACCOUNT FOR THE LAWS OF LOGIC? THE CHRISTIAN WORLDVIEW STATES THAT GOD IS ABSOLUTE AND THE
STANDARD OF TRUTH. THEREFORE, THE ABSOLUTE LAWS OF LOGIC EXIST BECAUSE THEY REFLECT THE NATURE OF AN ABSOLUTE GOD. GOD
DID NOT CREATE THE LAWS OF LOGIC. THEY WERE NOT BROUGHT INTO EXISTENCE SINCE THEY REFLECT GOD'S THINKING. SINCE GOD IS
ETERNAL, THE LAWS OF LOGIC ARE TOO."
That's all nice, but the conclusions of your last two sentences merely derive from your gratuitous
assumptions immediately above them. So this is no argument at all.
On the contrary. I have a reason to be able to account for the existence of logical absolutes where
atheists do not. And because they do not, they are forced (at least this seems to be) to try and
undermine my logic. This is a valid tactic, but he does not accomplish it.
He mentions "gratuitous assumptions." Yes, I do assume God exists - but not without logical
reason (discussed elsewhere on carm). Given that I can make assumptions, even as he does in certain
areas of his life, what is to prevent me from drawing logical conclusions based upon those
assumptions? Again, the atheist has not demonstrated that my conclusions are in error. At best, all he
can do is attack my presupposition of God's existence. Of course I do not introduce the existence of
God until later in the paper when, in my opinion, I have demonstrated that atheism cannot account for
the existence of logical absolutes. It is then that I introduce the notion of God's existence to account
for the existence of logical absolutes. Whether or not the atheist like this introduction bears no weight
in the argument. The point is that as a Christian I have a logical means of accounting for their
existence where, it would seem, the atheist does not.

"THEREFORE, THE CHRISTIAN CAN ACCOUNT FOR THE EXISTENCE OF THE LAWS OF LOGIC BY ACKNOWLEDGING THEY ORIGINATE FROM
GOD AND THAT MAN IS ONLY DISCOVERING THEM."
So? I can account for black holes in space by "acknowledging" that the tooth-fairy created them,
but "acknowledging" an unsupported assertion doesn't support the assertion.
I do not nor have I ever defended the existence of the tooth fairy. What is interesting to me is that
atheists will often throw in known fairy tales as an attempt to logically disprove God's existence. But
this is an ill logical thing for them to do since a known fairy tale is a false analogy to the question of
whether or not God exists. Merely claiming that God is a fairy tale is not make it so anymore than
claiming that God exists makes it so. Instead, evidence and rationality should be examined. But then,
we are back in full circle as we discover that our presuppositions will govern how evidence and
rationality are examined. Given that the atheist presupposition is that there is no God (generally
speaking), in order for him to be consistent with his presuppositions, he must interpret all evidence
and rationality in a manner consistent with his atheism. I assert that this is a biased and restricting
process that hamstrings the atheist's intellectual ability to properly weigh the evidence since the
option of God as an explanation is automatically rejected.
Whether or not this atheist considers the existence of logical absolutes, his inability to account for
their existence, and the Christian's ability to account for them as a valid argument would depend upon
two things: the validity of the argument and the presuppositions involved. By default, the atheist
could not logically allow me the rational system I proposed to account for logical absolutes. If he did,
this would mean that atheism is weakened. But since he must defend his position, he must find
anyway he can to invalidate my argument -- which is why he erringly tried at the beginning to accuse
me of committing the logical fallacy of "begging the question."

"HOW DOES THE ATHEIST ACCOUNT FOR THE LAWS OF LOGIC?"


If that means "can the atheist account for WHY logical absolutes exist", then that is the same as
saying "where did the laws of logic come from", clearly begging the question that they must have
come from somewhere, when it is not yet proven they had to.
This atheist continues to misapply the logical fallacy of "begging the question" which I explained
above. Nevertheless, logical absolutes exist. When he says it has not been proven that they had to
come from somewhere, then please offer something else. Perhaps he would like to assert that they
never came from anywhere, or that they are eternal by nature, or that they brought themselves into
existence? But each has its problem.
If these conceptual realities are eternal by nature, then I can make a case that these eternal
conceptual realities are from God who is, by definition, eternal. Remember, the nature of logical
absolutes is conceptual; that is, they exist in the mind. They cannot be measured, weighed, frozen,
etc. If they exist in the mind and if they are always true everywhere (transcending space and time), I
assert that it is a logical requirement that there is an eternal mind is "housing" them. If on the other
hand, the atheist assumes that they brought themselves into existence, then he violates the logical
absolute that "something that does not exists cannot bring itself into existence.” If he did this, he
would be illogical.

Same thing if an atheist says "where did god come from?" The question itself assumes that god
originated from something outside himself, before having proved she needed to "begin to exist".
There is nothing wrong with asking the question "Where did god come from?" We Christians simply
state did God did not "come from" anywhere; He had no beginning because He is the creator of space
and time and our question about God's beginning deals with time. Therefore, since God is the creator
of time, he cannot be measured by it since it does not encompass Him.
We Christians reflect with the Bible says God always existing. Atheists do not like this explanation
for obvious reasons.

The truth is that I myself believe that matter and energy are essential to logic, and logic has thus
existed as long as they have.
I am sure that it is truth that this atheist believes what he does. It is a nice faith based
explanation for logical absolutes. But, he has offered nothing of substance to account for the existence
of logical absolutes. He has not provided a reason why logical absolutes are dependent upon, or
products of, matter and energy. Remember, the nature of logical absolutes is that they are conceptual.
They "exist" in the mind since the use of logic is a mental process. Logical processes of thought, which
is what logic is - a process - occur in rational minds, not in inanimate objects. The process of logical
reasoning does not occur in a rock or in heat. A rock and heat simply exist without the capacity of
rational thought. Rationality, logic, etc. are by default, processes that involve and necessitate
cognition. Physical and energy realities are not and have no conceptual processes. They are simply
reflections of the nature of materiality and energy. Therefore, they are not the source of logical
absolutes nor can they account for the existence of these logical absolutes.

"WE DO NOT OBSERVE THE LAWS OF LOGIC OCCURRING IN MATTER.


YOU DON'T WATCH AN OBJECT NOT BRING ITSELF INTO
EXISTENCE IF IT DOESN'T EXIST.
THEREFORE, NO LAW OF LOGIC CAN BE OBSERVED BY WATCHING NOTHING."
That which is not logical, must be illogical, since there is no happy medium between logical and
illogical, as such, if you don't see logic in matter, then you think matter is illogical by default.
To say "that which is not logical, must be illogical" may be an ill logical statement. For example, is
the existence of matter itself "not logical"? If it is not logical, how so? If it is illogical, how? Matter
simply exists. Stating that it exists is a logical process (which occurs in the mind) since we are
identifying something and making a statement about it. So, I propose that your statement is not
logically necessary.

Baking soda and vinegar are matter, and their combination creates instant carbon dioxide. Does
this reaction within matter take place for ILLOGICAL reasons? If not, then we do indeed see logic in
matter. Also, to answer your own illustration more directly: Correct, we do not see non-existent
objects refusing to bring themselves into existence. However, this is for a very logical reason; objects
cannot do anything anyway, if at first they don't exist. So if at first you don't object, exist exist again.
The reaction between baking soda and vinegar is not an issue of logic or ill logic. It is simply the
natural reaction to the physical properties of each item. What he has done is observed a physical
reaction and then asked if it occurs for illogical or ill logical reason. But what logical order to logical
reason what you apply to the reality of physical reactions? They are different things. The physical
reaction does not violate the law of non-contradiction since it does not even apply. Nor does it apply to
the law of identity, or excluded middle. It is simply something that happens and we can observe the
reaction. It is not logical or ill logical. It is simply reflective of the nature of the reaction of the
substances involved.
He has apparently tried to wed a physical reality with the conceptual one as if they are the same.
But, observing what happens does not mean that the observer is making it happen or preventing it
happen by the application of logic. Furthermore, whether or not the observer agrees or disagrees with
the reality of the reaction and attempts to apply a logical reason why the reaction should not occur,
does not affect the reality of the reaction since it doesn't effect the reaction. Therefore, the reaction is
independent of the issue of logic. It is simply the reaction.
"IF THE ATHEIST APPEALS TO THE SCIENTIFIC METHOD TO EXPLAIN THE LAWS OF LOGIC THEN HE IS USING CIRCULAR ARGUMENTATION
BECAUSE THE SCIENTIFIC METHOD IS DEPENDENT UPON LOGIC; THAT IS, REASONED THOUGHT APPLIED TO OBSERVATIONS."
Don't be too hard on logical absolutes that exist without deriving from another source, because you
believe in one great big thing that exists, but doesn't derive from anything else. And if we were to
imply that this god came from some source by asking "WHY does god exist", you would immediately
notice how loaded the question is.
I AM NOT SURE WHAT HE MEANS BY "DON'T BE TOO HARD ON LOGICAL ABSOLUTES THAT EXIST..."? I AM NOT "BEING" HARD ON
THEM. LOGICAL ABSOLUTES EXIST. THE ATHEIST'S INITIAL SENTENCE HERE IS NOT VERY CLEAR.
OFTENTIMES, QUESTIONS ARE "LOADED." BUT THAT DOES NOT MEAN THEY CANNOT BE ASKED OR ANSWERED.

Atheist: "I cannot account for WHY logic exists."


Christian: "then something is wrong with atheism, because logic clearly does exist."
Christian: "I cannot account for WHY God exists."
Atheist: Then something is wrong with theism, because god clearly does exist."
You see then that inability to explain WHY something exists, doesn't mean the worldview
containing that disability is necessarily wrong.
I find it necessary again to correct this atheist. I did not ask the atheist to explain "why" logic
exists. I asked that the atheist to give a valid reason to account for their existence given the
presupposition of materialism and no God. Given that logical absolutes are by nature conceptual, how
does the atheist account for their existence in the universe of nothing but mater and energy? That is
question he still has not answered.
Also, I noticed that this atheist added words I did not say into the conversation. I did not say that
something is wrong with atheism (though I believe it to be true). I said atheism is not able to account
for the existence of logical absolutes. This does not necessitate that atheism is a false system as a
whole -- unless it can be "proven" that the only way to account for logical absolutes is the existence of
God. Nevertheless, atheism's weakness here is demonstrated. If the atheist were to assume that
since he cannot account for the existence of logical absolutes and it makes no difference whether or
not atheism is true, then is it also not logically consistent that in the your dialogue if a theist cannot
account for the "why" of God's existence, that it also does not invalidate theism? Of course not. But
remember, we are not specifically arguing God's existence. We are debating the ability of the atheist
to account for the existence of logical absolutes.
The issue is not properly reflected in this atheist’s hypothetical dialogue. I have simply proposed
that Christian theism can account for the existence of logical absolutes were atheism cannot. Since the
accounting for them includes the existence of God, then theism is that much more validated -- and
atheism that much more invalidated.

"IF LOGIC ITSELF IS USED TO VALIDATE LOGIC, THEN CIRCULAR REASONING AGAIN IS USED AND THE ATHEIST HAS STILL FAILED TO
ACCOUNT FOR THEIR EXISTENCE."
By those standards, you cannot make your own point! here's why:
If you use god to validate logic, you have begged the question, because his existence wasn't
proved first. If you prove his existence first, and then use god to validate logic, let's go back to that
proof for god for just a second...was it logical? If it was, logic was assumed existing already right
there, to ground the god-proofs. So you couldn't take the next step and use god to justify logic,
because in your concern to do things orderly, and prove A)god and then B)prove god justified logic,
you have already presupposed logic to be existing at A). You can certainly avoid this inconsistency of
yours by making sure that your first step in this argument (proving god exists) is NOT based on logic.
But then, nobody around here likes proofs that aren't logical.
Since logic is the reason god proofs work (at least as far as you believe), logic cannot wait to be
proved after god is proved, because logic is what was already there to help you first prove god. Hence,
logic must be presumed to exist before and thus independent of god. You would wish you could have it
both ways: "god proves logic, and logic proves god", but this is circular, and wherever you break the
circle, logic will win out over god as that which must be assumed first!
First of all, after reading his quote of my original statement, I immediately recognized the problem
with it and I have removed it from the original paper. The reason is because the statement is not
necessarily logical. It did not take the explanation of the atheist to demonstrate that and I should have
caught it the first time. Nevertheless, since it is my desire to improve the quality of my papers, I've
removed that "point."
Second, using logic to validate logic is indeed a circular reasoning process, but this does not
invalidate logic. The point of the statement was the atheist has still failed to account for the existence
of logical absolutes.
Third, the original statement above is not really a very good statement since it may be that an
atheist has indeed used logic to account for its own existence. The question would then be to examine
the logic of that claim to see whether or not it is valid. Therefore, my initial statement failed to take
this into account and is not a will reasoned statement. Again, after reflecting on it, I have removed it
from the original paper.

"IF LOGIC IS NOT ABSOLUTE, THEN NO LOGICAL ARGUMENTS FOR OR AGAINST THE EXISTENCE OF GOD CAN BE RAISED AND THE
ATHEIST HAS NOTHING TO WORK WITH."
Logic IS absolute, but I've shown it only comes from god if you use unsubstantiated
'acknowledgements' such as Matt Slick does in this article.
I do not follow the statement and cannot respond since it seems makes no sense to me.

"ATHEISTS GOD’S EXISTENCE, BUT IN SO DOING THEY ARE ASSUMING ABSOLUTE LAWS OF
WILL USE LOGIC TO TRY AND DISPROVE
LOGIC AND BORROWING FROM THE CHRISTIAN WORLDVIEW."
Everything you asserted prior to this was either a failed argument to prove logic comes from god or
else the bare unsupported "acknowledgement" that logic comes from god. You simply asserted with
great confidence that Christianity can account for logic. Assertions don't qualify as evidence. When
atheists use logic, they use nothing that you have proven must come from god.
First of all, my argument has not failed. The atheist has not refuted it. He did, however, incorrectly
begin his analysis with a misapplication of the logical fallacy known as "begging the question." He also
misapplied several other points which I have already pointed out. Second, I did not simply assert that
Christianity can account for logic without first trying to demonstrate that atheism can't. I tried to
establish that the nature of the logical absolutes is they are transcendent, eternal, and not dependent
a material or energy. Also, since I stated that they are conceptual by nature and that they are
absolute, that it is logical to assume that they are "housed" by an absolute mind since logical thought
is a mental process. This is not a leap of logic (or of faith) and he should reflect this analysis in his
criticism here instead of taking my comments out of logical context. Third, if assertions do not qualify
as evidence, then his earlier statement above "The truth is that I myself believe that matter and
energy are essential to logic, and logic has thus existed as long as they have," is no evidence
whatsoever for the validity of his position. I would hope that he would stay away from offering his
belief system has a justification for why his atheism.
Finally, I have not randomly offered my belief system as an explanation for the existence of logical
absolutes. I introduced the concept of God only after laying out several other logical statements and
then attempting to make a logical conclusion.

"THE CHRISTIAN WORLDVIEW MAINTAINS THAT THE LAWS OF LOGIC ARE ABSOLUTE BECAUSE THEY COME FROM GOD WHO IS HIMSELF
ABSOLUTE.“
Yup, asserting again without evidence.
Is not the logical development in my original paper that logical absolutes are conceptual by nature,
not dependent upon consensus of humanity, not based in material or energy, but are in reality the
product of a mind, at least an attempt at a rational proposition for their existence?
This is not a mere assertion without evidence. Remember, the point of the paper is that logical
absolutes are conceptual by nature and their characteristics should reflect their nature. Since the
characteristic of logical processes is that they are conceptual by nature, it is perfectly logical to
assume that there is a mind involved with their existence. This is not simply an empty assertion. It is a
logical conclusion. Therefore, since the logical absolutes are absolute, are always true every where, I
am concluding that there is a mind at work that reflects the nature of the absolutes of logic. This is an
attempt to offer the evidence of logic thereby substantiation my assertion.
“BUT THE ATHEIST WORLDVIEW DOES NOT HAVE AN ABSOLUTE GOD.
SO, WE ASK, "HOW CAN ABSOLUTE, CONCEPTUAL, ABSTRACT LAWS BE DERIVED FROM A UNIVERSE OF MATTER, ENERGY AND MOTION?"
IN OTHER WORDS, "HOW CAN AN ATHEIST WITH A NATURALISTIC PRESUPPOSITION ACCOUNT FOR THE EXISTENCE OF LOGICAL ABSOLUTES
WHEN LOGICAL ABSOLUTES ARE CONCEPTUAL BY NATURE AND NOT PHYSICAL, ENERGY, OR MOTION?"
That’s a most confused set of phrases. Just because logic works on paper as a concept doesn't
mean it is independent of matter, energy or motion. 2+2=4 is mathematical concept as well as a
logical concept. Being a concept by nature, does that somehow argue that this equation can be proven
true independent of the matter and energy used to demonstrate it? No. If you didn’t have matter and
energy to work with, you could never prove that logical absolutes exist. You may say “god is still there,
even if matter and energy are gone and he justifies the existence of logical absolutes.” I say, 'but if
you get to god before logic, then you didn’t get to god USING logic, unless you agree you were
thereby using false logic.
At this point the atheist opens himself up for an ad hominem attack. I could comment about the
set of statements not being confusing to others and then build upon it, but I won't. Nevertheless, he
tries to state that the concept 2+2=4 cannot be solved without the existence of matter and energy.
This may or may not be correct since we do not know if all cognitive beings in the universe are
comprised of matter and energy. Hypothetically, there could be some life forms comprised only of
energy with a mind (i.e., God?). If so, the statement 2+2=4 would still be true. But what he does is
presuppose the existence of a mind that works in space and time in order to process the statement
2+2=4. By so doing, he inadvertently requires the existence of a mind to work out conceptual
formulas thereby validating the earlier premise that logical absolutes are processes of a mind.
And yes, I can indeed say that God is before "matter and energy." But what does that mean since
we could say God is immaterial. But if He exists shouldn't He then have some form of energy about
him? At this point, we get into a philosophical debate which is irrelevant to the conversation at hand.
Nevertheless, the statement 'but if you get to god before logic, then you didn't get to god USING
logic, unless you agree you were thereby using false logic," makes no sense to me -- at which point I
am probably opening up myself to an ad hominem attack by the atheist stating that others may
understand it with no problem. Perhaps also will not try to build a case on that.

"THE CHRISTIAN THEISTIC WORLDVIEW CAN ACCOUNT FOR THE LAWS OF LOGIC BY STATING THAT THEY COME FROM GOD.”
Doesn’t mean that’s where they came from, it means you’ve suggested an idea, that’s it.
Yes, I have suggested a way to account for the laws of logic. In my opinion, my analysis is for
more logical than yours.

“GOD IS TRANSCENDENT; THAT IS, HE IS BEYOND THE MATERIAL UNIVERSE, BEING ITS CREATOR.“
Another assumption that you worked with but never proved in this entire article of yours.
Yes, I admit that this is more or less an assumption. But, the point of my conclusion was that the
insertion of the Christian concept of God does account for the existence of logical absolutes in such a
way that is consistent with Christian theism which teaches a transcendent and absolute Mind.

“GOD HAS ORIGINATED THE LAWS OF LOGIC BECAUSE THEY ARE A REFLECTION OF HIS NATURE.“
Another assumption you used but never proved.
Again, I do admit slipping into a biblical presupposition at this point. I therefore repeat my above
statement that the point of my conclusion was that the insertion of the Christian concept of God does
account for the existence of logical absolutes in such a way that is consistent with Christian theism
which teaches a transcendent and absolute Mind.

“THE ATHEISTIC WORLDVIEW CANNOT ACCOUNT FOR THE LAWS OF LOGIC/ABSOLUTES, AND MUST BORROW FROM THE CHRISTIAN
WORLDVIEW IN ORDER TO RATIONALLY ARGUE."
But if the Christian world view (that god exists) is true, “true” means “logical”. And if you have
already proven god logically/truly exists, you did it only because your proofs presumed logic
themselves. They cannot presume it in their case for god, unless it exists apart from and before god.
This atheist may have inadvertently substantiated my case. When he said "...if you have already
proven god logically/truly exists, you did it only because your proofs presumed logic themselves," it

seems as though he is saying I accomplished what it was setting up to accomplish. We both presume
that logic exists. But, so far, the atheist cannot account for the existence of logical absolutes.
We have common ground in that we both believe in and use logical absolutes. But as I said before,
what exists exists as a reflection of its own nature and its characteristics reflect that nature. The
characteristics of logical absolutes are that they are conceptual by nature and and they are logical
process which must, by default, occur in a mind. If it is true that characteristics reflect nature, then
we could assume that the nature of the mind that is housing these logical absolutes must also be
absolute. Since the logical absolutes are always true everywhere in the universe (transcendence), then
the mind that houses them must also be transcendent and absolute. It is the Christian scriptures that
teach that there is a transcendent, absolute God with an absolute mind -- not atheism. Therefore,
atheism is still not able to account for their existence where Christianity can.
Relativism
Introduction

Relativism makes so many illogical and self contradictory attitudes possible. It is inconsistent, self
refuting, and sadly, held by more and more people who deny the existence of absolutes.

You’ll find that it doesn’t take much to refute it. In fact, learning how to refute relativism is a good
starting point in learning how to apply logic.

1. What is relativism? p. 92
2. What is ethical relativism and why is it problematic? p. 94
3. Is all truth relative? Why or why not? p. 97
4. What is truth? p. 101
What is relativism?
Relativism is the philosophical position that all points of view are equally valid and that all truth is
relative to the individual. This means that all moral positions, all religious systems, all art forms, all
political movements, etc., are truths that are relative to the individual. Under the umbrella of
relativism whole groups of perspectives are categorized. In obvious terms, some are:

• cognitive relativism (truth) - Cognitive relativism affirms that all truth is relative. This would
mean that no system of truth is more valid than another one and that there is no objective
standard of truth. It would, naturally, deny that there is a God of absolute truth.
• moral/ethical relativism - all morals are relative to the social group within which they are
constructed.
• situational relativism - that ethics (right and wrong) are dependent upon the situation.

Unfortunately, the philosophy of relativism is pervasive in our culture today. With the rejection of
God, and Christianity in particular, absolute truth is being abandoned. Our pluralistic society wants to
avoid the idea that there really is a right and wrong. This is evidenced in our deteriorating judicial
system that has more and more trouble punishing criminals, in our entertainment media which
continues to push the envelope of morality and decency, in our schools which teach evolution and
"social tolerance", etc. In addition, the plague of moral relativism is encouraging everyone to accept
homosexuality, pornography on TV, fornication, and a host of other "sins" that were once considered
wrong, but are now being accepted and even promoted in society. It is becoming so pervasive that if
you speak out against moral relativism and its "anything goes" philosophy, you're labeled as an
intolerant bigot. Of course, this is incredibly hypocritical of those who profess that all points of view
are true, yet reject those who profess absolutes in morality. It seems that what is really meant by
the moral relativists is that all points of view are true except for the views that teach moral absolutes,
or an absolute God, or absolute right and wrong.
Some typical expressions that reveal an underlying presupposition of relativism are comments such
as "That is your truth, not mine," "It is true for you, but not for me," and "There are no absolute
truths." Of course, these statements are illogical, which I demonstrate in the paper "Refuting
relativism." Relativism is invading our society, our economy, our schools, and our homes. Society
cannot flourish nor survive in an environment where everyone does what is right in his own eyes,
where the situation determines actions and if the situation changes, lying or cheating is acceptable --
as long as you're not caught. Without a common foundation of truth and absolutes, our culture will
become weak and fragmented.
I must admit, however, that there is validity to some aspects of relativism. For example, what one
society considers right (driving on the left side of the road) another considers wrong. These are
customs to which a "right and wrong" are attached, but they are purely relativistic and not universal
because they are culturally based. Child rearing principles vary in different societies as do burial
practices and wedding ceremonies. These "right and wrong ways" are not cosmically set in stone nor
are they derived from some absolute rule of conduct by some unknown god. They are relative and
rightly so. But, their relativism is properly asserted as such. It doesn't matter what side of the road
we drive in as long as we all do it the same way.
Likewise, there are experiences that are valid only for individuals. I might be irritated by a certain
sound, where another person will not. In this sense, what is true for me is not necessarily true for
someone else. It is not an absolute truth that the identical sound causes irritation to all people. This is
one way of showing that certain aspects of relativism are true. But, is it valid to say that because
there is a type of personal relativism that we can then apply that principle to all areas of experience
and knowledge and say that they too are also relative? No, it is not a valid assumption. First of all, to
do so would be an absolute assessment which contradicts relativism.
Furthermore, if all the things are relative, then there cannot be anything that is absolutely true
between individuals. In other words, if all people deny absolute truth and establish relative truth only
from their experiences, then everything is relative to the individual. How then can there be a common
ground from which to judge right and wrong or truth? It would seem that there cannot be.
Of course, the issue that is important here is whether or not there are absolute truths. Also, can
there be different kinds of absolute truths if indeed there are absolute truths? We might ask if it is
always wrong to lie? Or, does 1 + 1 always equal 2? Is it always true that something cannot be both
in existence and not in existence at the same time? Is it always true that something cannot bring itself
into existence if it first does not exist? If any of these questions can be answered in the affirmative
then relativism is refuted -- at least to some degree.
More questions arise. If all moral views are equally valid, then do we have the right to punish
anyone? Can we ever say that something is wrong? In order to say that something is wrong we must
first have a standard by which we weigh right and wrong in order to make a judgment. If that
standard of right and wrong is based on relativism, then it is not a standard at all. In relativism,
standards of right and wrong are derived from social norms. Since society changes, the norms would
change and so would right and wrong. If right and wrong change, then how can anyone be rightly
judged for something he did wrong if that wrong might become right in the future?
Finally, is it fair to apply logical analysis to relativistic principles? Many relativists say no. But, I do
not see why not. If a relativist were to convince me that logic isn’t necessary in examining relativism,
he’d have to convince me using logic, which would be self defeating. If a relativist uses relativism --
the subjective view of his own opinions -- to validate his position, he is using circular reasoning;
namely, he is using relativism to establish relativism. So, either way, he has lost the argument.
To conclude, if relativism is true and all points of view are true, then is my view that relativism is
false, true?
Does truth contradict itself? No it doesn't.
Ethical relativism
Ethical relativism is the position that there are no moral absolutes, no moral right and wrongs.
Instead, right and wrong are based on social norms. Some have heard of the term situational ethics
which is a category of ethical relativism. At any rate, ethical relativism would mean that our morals
have evolved, that they have changed over time, and that they are not absolute.
One advantage of ethical relativism is that it allows for a wide variety of cultures and practices. It
also allows people to adapt ethically as the culture, knowledge, and technology change in society. This
is good and a valid form of relativism.
The disadvantage of ethical relativism is that truth, right and wrong, and justice are all relative.
Just because the group of people thinks that something is right does not make so. Slavery is a good
example of this. Two hundred years ago in America, slavery was the norm and morally acceptable.
Now it is not.
Relativism also does not allow for the existence of an absolute set of ethics. Logically, if there are
not absolute ethics, then there can be no Divine Absolute Ethics Giver. Requiring an absolute set
ethics implies an Absolute Ethics Giver which can easily be extrapolated as being God. This would be
opposed to ethical relativism. Therefore, ethical relativism would not support the idea of an absolute
God and it would exclude religious systems based upon absolute morals; that is, it would be absolute
in its condemnation of absolute ethics. In this, relativism would be inconsistent since it would deny
beliefs of absolute values.
Furthermore, if ethics have changed overtime there is the problem of self contradiction within the
relativistic perspective. 200 years ago slavery was socially acceptable and correct. Now it is not. There
has been a change in social ethics in America regarding this issue. The problem is that if slavery
becomes acceptable again in the next 200 years, who is to say if it is right or wrong? We would have a
contradictory set of right and wrong regarding the same issue. To this I ask the question, does truth
contradict itself?
Within ethical relativism, right and wrong are not absolute and must be determined in society by a
combination of observation, logic, social preferences and patterns, experience, emotions, and "rules"
that seem to bring the most benefit. Of course, it goes without saying that a society involved in
constant moral conflict would not be able to survive for very long. Morality is the glue that holds a
society together. There must be a consensus of right and wrong for a society to function well. Ethical
relativism undermines that glue.
It seems to be universal among cultures that it is wrong to murder, to steal, and to lie. We see that
when individuals practice these counterproductive ethics, they are soon in prison and/or punished.
Since ethics are conceptual in nature and there are some ethics that seem to transcend all cultures (be
true for all societies) I conclude that there is a transcendent God who has authored these ethics -- but
that is another discussion.
I do not believe that the best ethical patterns discovered by which societies operate (honesty,
fidelity, truth, no theft, no murder, etc.) are the product of our biological makeup or trial and error. As
a Christian, I see them as a reflection of God’s very character. They are a discovery of the rules God
has established by which people best interact with people because He knows how He has designed
them. The 10 commandments are a perfect example of moral absolutes and have yet to be improved
upon. They are transcendent; that is, they transcend social norms and are always true.
I was once challenged to prove that there were moral absolutes. I took up the challenge with the
following argument. I asked the gentleman whether or not there were logical absolutes. For example, I
asked if it was a logical absolute that something could exist and also not exist at the same time. He
said, no that it was not possible. Another example is that something cannot bring itself into existence.
To this he agreed that there were indeed logical absolutes. I then asked him to explain how logical
absolutes can exist if there is no God. I questioned him further by asking him to tell me how in a
purely physical universe logical absolutes, which are by nature conceptual, can exist. I said, they
cannot be measured, put in the test tube, weighed, nor captured; yet, they exist. So, I asked him to
please tell me how these conceptual absolute truths can exist in a purely physical universe...without a
God. He could not answer me. I then went on to say that these conceptual absolutes logically must
exist in the mind of an absolute God because they cannot merely reside in the properties of matter in a
purely naturalistic universe. And since the logical absolutes are true everywhere all the time and they
are conceptual, it would seem logical that they exist within a transcendent, omnipresent, being. If
there is an absolute God with an absolute mind then he is the standard of all things – as well as
morals. Therefore, there would be moral absolutes. To this argument the gentleman chuckled, said he
had never heard it before, and conceded that it may be possible for moral absolutes to exist.
Of course, as a Christian, as one who believes in the authority and inspiration of the Bible, I
consider moral absolutes to be very real because they come from God and not because they somehow
reside in a naturalistic universe.
Ethics are important in society, in the home, and in all interactions. Would you believe me if I
started lying to you in this paper? No. You expect me to be fair, honest, logical, and forthright. Can I
be that if I believe all ethics are relative? Heck, if I did, I could try and deceive you into getting me to
believe what I want you to.
Cognitive relativism
Cognitive relativism affirms that all truth is relative. This would mean that no system of
determining truth is more valid than another system and that there is no objective standard of truth to
be found or claimed. It would, naturally, deny that there would be a God of absolute truth. It would
also deny the belief that rational thought can discover and verify truth. But, cognitive relativism does
not deny that there are differences in perspective in different cultures. In fact, it affirms them. Rather,
the issue with cognitive relativism is that there is no epistemological (method of knowing something)
system that is inherently superior over another. Of course, this seems to be self refuting since it claims
that its own principal of relative truth is absolutely true and uses it to determine that cognitive
relativism is true.
Many believe that this relativism is self-contradictory. But, why has relativism gained a foot-hold in
modern society? I think there are several factors contributing to its acceptance.
First, the success of science has increasingly promoted the idea that true answers are found within
science. Many people believe whatever "scientists" tell them is factual. When science cannot answer
something it simply states that the truth will become known later. With this, people have faith in
science and the only absolute is that what we know now, may not be true later. Thereby, it can
undermine absolute truth.
Second, with the broad acceptance of the evolutionary theory, God is pushed more and more out of
the picture. Without God as a determiner of what is true and not true, we are left to do and believe
"what is right in our own eyes."
Third, we are encountering more and more diverse cultures in the world. This tends to make us
more comfortable with the idea that there is more than one way to do something, more than one way
for a culture to operate, more than one way for something to be true or right. This isn't necessarily
wrong, but it does contribute to a denial of absolutes.
Fourth, increasingly, the content of film, academia, and literature is moving away from the notion
of the absolute and towards relativism. These media help shape our culture.
Fifth, an increase in relativistic philosophies particularly those found in the New Age movement
which teaches that there is no absolute truth and that each person can create his own reality. Though
this movement is part of the relativistic "problem" it is well permeated into society.
Sixth, passed philosophers such as Wittgenstein, Khuh, Kant, Marx, and Neitsche, have influenced
the thinking of many with their relativistic principles and attacks on absolute truths.
The problem I see with cognitive relativism is that it denies the possibility of absolute truth.
Furthermore, I believe cognitive relativism is easily refutable with the following example of a logical
absolute:

Something cannot bring itself into existence.

My proposed logical absolute is indeed logical and always true. Let’s look at this. For something to
bring itself into existence it must first exist. If it first existed then it cannot bring itself into existence
because it already is existing. Likewise, if something does not exist then it is not possible for it to
bring itself into existence since it isn't there to do anything.
This is an absolute truth and it is knowable. Since it is absolutely true, cognitive relativism, which
states that all truth is relative, is false.
Refuting relativism
Relativism is the philosophical position that all points of view are equally valid and that all truth is
relative to the individual. But, if we look further, we see that this proposition is not logical. In fact, it
is self refuting.

1. All truth is relative


A. If all truth is relative, then the statement "All truth is relative" would be absolutely true. If it
is absolutely true, then not all things are relative and the statement that "All truth is
relative" is false.
2. There are no absolute truths
A. The statement "There are no absolute truths" is an absolute statement which is supposed to
be true. Therefore it is an absolute truth and "There are no absolute truths" is false.
B. If there are no absolute truths, then you cannot believe anything absolutely at all, including
that there are no absolute truths. Therefore, nothing could be really true for you - including
relativism.
3. What is true for you is not true for me
A. If what is true for me is that relativism is false, then is it true that relativism is false?
i. If you say no, then what is true for me is not true and relativism is false.
ii. If you say yes, then relativism is false.
B. If you say that it is true only for me that relativism is false, then
i. I believe something other than relativism; namely, that relativism is false. If that is true,
then how can relativism be true?
ii. Am I believing a premise that is true or false or neither?
a. If it is true for me that relativism is false, then relativism (within me) holds the
position that relativism is false. This is self-contradictory.
b. If it is false for me that relativism is false, then relativism isn't true because what is
true for me is not said to be true for me.
c. If you say it is neither true nor false, then relativism isn't true since it states that all
views are equally valid and by not being, at least true, relativism is shown to be
wrong.
C. If I believe that relativism is false, and if it is true only for me that it is false, then you must
admit that it is absolutely true that I am believing that relativism false.
i. If you admit that it is absolutely true that I believe relativism is false, then relativism is
defeated since you admit there is something absolutely true.
D. If I believe in something other than relativism that is true, then there is something other
than relativism that is true - even if it is only for me.
i. If there is something other than relativism that is true, then relativism is false.
4. No one can know anything for sure
A. If that is true, then we can know that we cannot know anything for sure which is self
defeating.
5. That is your reality, not mine
A. Is my reality really real?
B. If my reality is different than yours, how can my reality contradict your reality? If yours and
mine are equally real, how can two opposite realities that exclude each other really exist at
the same time?
6. We all perceive what we want
A. How do you know that statement is true?
B. If we all perceive what we want, then what are you wanting to perceive?
i. If you say you want to perceive truth, how do you know if you are not deceived?
ii. Simply desiring truth is no proof you have it.
7. You may not use logic to refute relativism
A. Why not?
B. Can you give me a logical reason why logic cannot be used?
C. If you use relativism to refute logic, then on what basis is relativism (that nothing is
absolutely true) able to refute logic which is based upon truth.
D. If you use relativism to refute logic, then relativism has lost its relative status since it is used
to absolutely refute the truth of something else.
8. We are only perceiving different aspects of the same reality.
A. If our perceptions are contradictory, can either perception be trusted?
B. Is truth self contradictory?
i. If it were, then it wouldn't be true because it would be self refuting. If something is self
refuting, then it isn't true.
C. If that is true that we perceive different aspects of the same reality, then do I believe
something that is false since I believe that your reality is not true? How then could they be
the same reality?
D. If you are saying that it is merely my perception that is not true, then relativism is refuted.
i. If I believe something that is false, then relativism is not true since it holds that all
views are equally valid.
E. If my reality is that your reality is false, then both cannot be true. If both are not true, then
one of us (or both) is in error.
i. If one or both of us is in error, then relativism is not true.
9. Relativism itself is excluded from the critique that it is absolute and self-refuting.
A. On what basis do you simply exclude relativism from the critique of logic?
i. Is this an arbitrary act? If so, does it justify your position?
ii. If it is not arbitrary, what criteria did you use to exclude it?
B. To exclude itself from the start is an admission of the logical problems inherent in its system
of thought.
What if relativism were true? An illustration.
Relativism is the position that all points of view are as valid and as any other points of view and that
the individual is the measure of what is true for that person. I see a big problem with this. Following is
an illustration to demonstrate it.
The setting: A thief is casing a jewelry store so he can rob it. He has entered it to check out any
visible alarm settings, locks, layout, etc. In the process, he has unexpectedly gotten involved in a
discussion with the owner of the jewelry store whose hobby is the study of philosophy and believes
that truth and morals are relative.

"So," says the owner, "everything is relative. That is why I believe that all morals are not absolute
and that right and wrong is up to the individual to determine within the confines of society. But there
is no absolute right and wrong."
"That is a very interesting perspective," says the thief. "I was brought up believing that there was a
God and that there was right and wrong. But I abandoned all of that and I agree with you that there is
no absolute right and wrong and that we are free to do what we want."
The thief leaves the store and returns that evening and breaks in. He has disabled all the alarms
and locks and is in the process of robbing the store. That is when the owner of the store enters
through a side door. The thief pulls out a gun. The owner cannot see the man's face because he is
wearing a ski mask.
"Don't shoot me," says the owner. "Please take whatever you want and leave me alone."
"That is exactly what I plan to do," says the thief.
"Wait a minute. I know you. You are the man that was in the store earlier today. I recognize your
voice."
"That is very unfortunate for you," says the thief. "Because now you also know what I look like.
And since I do not want to go to jail I am forced to kill you."
"You cannot do that," says the owner.
"Why not?"
"Because it is not right," pleads the desperate man.
"But did you not tell me today that there is no right and wrong?"
"Yes, but I have a family, children, that need me, and a wife."
"So? I am sure that you are insured and that they will get a lot of money. But since there is no
right and wrong it makes no difference whether or not I kill you. And since if I let you live you will turn
me in and I will go to prison. Sorry , but that will not do."
"But it is a crime against society to kill me. It is wrong because society says so."
"As you can see, I don't recognize society's claim to impose morals on me. It's all relative.
Remember?"
"Please to not shoot me. I beg you. I promise not to tell anyone what you look like. I swear it!"
"I do not believe you and I cannot take that chance."
"But it is true!" I swear I'll tell no one."
"Sorry, but it cannot be true because there is no absolute truth, no right and wrong, no error,
remember? If I let you live and then I left, you will break your so-called promise because it is all
relative. There is no way I could trust you. Our conversation this morning convinced me that you
believe everything is relative. Because of that, I cannot believe you will keep your word. I cannot trust
you.
"But it is wrong to kill me. It isn't right!"
"It is neither right or wrong for me to kill you. Since truth is relative to the individual, if I kill you,
that is my truth. And, it is obviously true that if I let you live I will go to prison. Sorry, but you have
killed yourself."
"No. Please do not shoot me. I beg you."
"Begging makes no difference."
.... Bang....
If relativism is true, then was it wrong to pull the trigger? Perhaps someone might say that it is
wrong to take another life needlessly. But why is that wrong, if there is no standard of right or wrong?
Others have said that it is a crime against society. But, so what? If what is true for you is simply true,
then what is wrong with killing someone to protect yourself after you have robbed him? If is true for
you that to protect yourself you must kill, then who cares what society says? Why is anyone obligated
to conform to social norms? Doing so is a personal decision.
Though not all relativists will behave in an unethical manner, I see relativism as a contributor to
overall anarchy. Why? Because it is a justification to do whatever you want.
What is truth?

"What is truth?" is a very simple question. Of course, answering it isn't so simple. We can offer
definitions like "Truth is that which conforms to reality, fact, or actuality." But this basic definition is
not complete because its definition is open to interpretation and a wide variety of applications. What is
reality? What is fact? What is actuality? How does perception affect truth? We could offer answers
for each of these questions, but then we could again ask similar questions of those answers. I am
reminded of the paradox of throwing a ball against a wall. It must get half way there, and then half
way of the remaining distance, and then half of that distance, and so on. But, an infinite number of
halves in this scenario never constitutes a whole. Therefore, it would seem that the ball would never
reach the wall if we applied the conceptual truths of halves.
The ball-against-the-wall scenario simply illustrates that defining and redefining things as we try to
approach a goal actually prevents us from getting to that goal. This is what philosophy does
sometimes as it seeks to examine truth. It sometimes clouds issues so much, that nothing can be
known for sure.
But, even though it is true that an infinite number of halves (1/2 of "a" + 1/2 of the remainder +
1/2 of the remainder of that, etc.) does not equal a whole, we can "prove" that it does by simply
throwing a ball at a wall and watch it bounce off. Actually, the "1/2" equation above does not equal a
whole -- mathematically. The problem is not in the truth, but in its application as is often the case
with philosophical verbal gymnastics.

"See to it that no one takes you captive through philosophy and empty deception, according to the
tradition of men, according to the elementary principles of the world, rather than according to Christ,"
(Col. 2:8).

In order for truth to be defined properly, it would have to be a factually and logically correct
statement. In other words, it would have to be true. But, perhaps we could look further look at truth
be determining what it is not. Truth is not error. Truth is not self-contradictory. Truth is not
deception. Of course, it could be true that someone is being deceptive, but the deception itself isn't
truth.
In relativism, all points of view are equally valid and that all truth is relative to the individual. If
this were true, then it would seem that this is the only truth relativism would have to offer. But, the
problem is that in reality, relativism isn't true for the following basic reason. If what is true for me is
that relativism is false, then is it true that relativism is false? 1) If you say no, then what is true for
me is not true and relativism is false. 2) If you say yes, then relativism is false. Relativism seems to
defy the very nature of truth; namely, that it is not self contradictory.

Again, what is truth?

If there is such a thing as truth, then we should be able to find it. If truth cannot be known, then it
probably doesn't exist. But, it does exist. For example, we know that it is true that you are reading
this.
Is there such a thing as something that is always true all the time? Yes, there is. For example,
"Something cannot bring itself into existence." This is an absolutely true statement. In order for
something to bring itself into existence, it would have to exist in order to be able to perform an
action. But if it already existed, then it isn't possible to bring itself into existence since it already
exists. Likewise, it if does not exist then it has no ability to perform any creative action since it doesn't
exist in the first place. Therefore, "Something cannot bring itself into existence" is an absolute truth.
The preceding example is a truth found in logic. But, there are truths that are not logical by
nature. It is true that I love my wife. This isn't logically provable via theorems and formulas and logic
paradigms, but it is, nevertheless, true. Therefore, we can say that truth conforms and affirms reality
and/or logic.
Is this what relativism does? Does relativism confirm to reality and logic? To be honest, it does to
some degree. Relativistically speaking, there is no absolute right or wrong regarding which side of
your head you should part your hair, if you part it at all. To this we must concede relative "truths"
that are different for different people. But, these are relativistic by nature. Examples of relativistic
truths are, 1) people drive on the right side of the street in America and the left in England; 2) I prefer
to watch science fiction over musicals; 3) snow is better than rain, etc. These things are relative to
culture, individuals, preferences, etc., and rightfully so.
If we are to ever hope to determine if there is such a thing as truth apart from cultural and
personal preferences, we must acknowledge that we are then aiming to discover something greater
than ourselves, something that transcends culture and individual inclinations. To do this is to look
beyond ourselves and outside of ourselves. In essence, it means that we are looking for God. God
would be truth, the absolute and true essence of being and reality who is the author of all truth. If
you are interested in truth beyond yourself, then you must look to God.

"I am the truth"

For the Christian, the ultimate expression of truth is found in the Bible, in Jesus who said, "I am
the way, the truth, and the life..." (John 14:6). Of course, most philosophers and skeptics will dismiss
His claim, but for the Christian, He is the mainstay of hope, security, and guidance. Jesus, who walked
on water, claimed to be divine, and rose from the dead, said that He was the truth and the originator
of truth. If Jesus is wrong, then we should ignore Him. But, if He is right, then it is true that we
should listen to Him.
The eyewitnesses wrote what they saw. They were with Him. They watched Him perform many
miracles, heal the sick, calm a storm with a command, and even rise from the dead. Either you
believe or dismiss these claims. If you dismiss them, that is your prerogative. But, if you accept
them, then you are faced with decisions to make about Jesus. What will you believe about Him? What
will you decide about Him? Is He true? Is what He said true?
Truth conforms to reality. Jesus performed many miracles and rose from the dead.
Islam
Introduction

Islam is a world religion. Is it a religion of peace? What does it teach?

Christians need to study Islam. It isn’t going away. Also, it denies all the essential doctrines of
Christianity – and it is growing with the goal of taking over the world.

1. What does the word "Qur’an" mean? p. 106


2. What century was Muhammad born? p. 108
3. What is the Hegira? p. 109
4. What is the Qur'an? p. 111
5. What are some of the doctrines of Islam? p. 118
6. What are the five pillars of Islam? p. 119
7. Does Islam teach salvation by works? p. 129
8. Who has performed the greatest act of love, Yahweh or Allah? p. 137
9. What does "Jihad" mean? p. 139
10. Is the Trinity illogical? Why or why not? pp. 143-144
11. Why is it necessary for God to die for our sins? p. 150
12. What are some of the contradictions in the Qur'an? p. 152
13. What is the Hadith? p. 157
14. What are some of the more interesting quotes from the Hadith? pp. 158-163
15. What are some of the more interesting quotes from the Hadith about Jesus? pp. 166-167
16. What are some of the more interesting quotes from the Hadith about Jihad? pp. 168-169
17. What are some of the more interesting quotes from the Hadith about Satan pp. 172-174
The Gospel for Muslims

The Gospel of Jesus is Good News. That is what "gospel" means in Greek. It is the good news
because Jesus has removed the requirements of keeping the Law in order to obtain salvation and that
through Jesus, we can obtain eternal life. Jesus made it possible for people to receive the free and
complete gift of salvation by faith.
Our father Abraham believed God and it was counted to him as righteousness (Gen. 15:6). It was
his faith in God that made Abraham righteous before God, not keeping the law, not keeping the
commandments.
The Law of God is perfect because God is perfect. The Law is a reflection of the character of God.
It is wrong to lie because lying is against God's character. It is wrong to steal because stealing is
against God's character. That is why the Law tells us what is wrong. God is not arbitrary and neither
is His Law. Though the Law is good and perfect, no man can keep it perfectly. No person can keep
the Law.
Jesus taught us that to even look on a woman with lust in your heart is to commit adultery with her
(Matt. 5:27-28). You see, the Law of God is not simply to govern actions. It is for our hearts and
attitudes. Purity of heart is what God wants from us. He wants purity of heart down to the deepest
part of our being. Why? Because God's heart is the Purest and Most Holy of all. And since the Law
was spoken by Him and came from Him, the Law is Perfect and Holy. That is the level of perfection
you must have when trying to keep the Law.
However, we are not able to keep the Law. We sin. We fail. The Law says do not lie, but shows
us where we lie -- in our very hearts. It says do not commit adultery, but shows us where we commit
adultery -- in our hearts. The Law of God is perfect. We are not. God is perfect and Holy. We are
not. We are not able to keep the Law of God because we are finite, limited, and affected by sin. How
can anyone ever hope to please God through keeping the Law? How can anyone ever hope to please
God and attain heaven by doing good deeds? It is not ourselves that we must please, but a Holy and
Pure God.

The Good News

The Good News is that you do not have to try and keep the Law of God to please Him. You do not
have to try and raise yourself to the level of God's Perfection by trying to keep His Holy and Perfect
Law. You cannot do that. If you thought you could, then your heart is full of pride. What you can not
do, Jesus did do. He kept all the Law perfectly (1 Pet. 2:22).
Jesus said that "Greater love has no man than this, that he lay his life down for his friend," (John
15:13). Jesus laid his life down for his friends. Jesus performed the greatest act of love in the
universe. He died for our sins. He paid the penalty of breaking the Law, which is death. If this were
not so, there would be no damnation. Jesus took our sins and died on the cross in our place (1 Pet.
2:24). This great act of love is unsurpassed in all the world. It means that you can, like Abraham
(Gen. 15:6), be righteous by faith. All you need is faith in Jesus.
Are you tired of trying to keep all the Laws in Islam as you strive to do more good deeds than bad
deeds in the hope that on the Day of Judgment your good deeds will outweigh your bad? Because you
earn in large part your salvation, you cannot know whether or not you will be saved. If you are tired
of trying to be perfect, of trying to obtain Paradise through your works, then you need Jesus. Jesus
said, "If any of you are heavy laden, come to me, and I will give you rest," (Matt. 11:28).
Jesus forgave sins in Luke 5:20 and 7:48-49. He walked on water (Matt. 14:25). He rose from the
dead (Matt. 28:6). Have you done more than He in keeping the Law or performing miracles? Has
even the Prophet Muhammad done more than Jesus?
The Good News is that you, like Abraham, can be made righteous by faith. Do you want the
righteousness that is by faith? Or, do you want to earn Paradise through your deeds? Can you earn
it? Have you done it so far? Have you been doing enough?
Jesus said:

• that He gives eternal life, (John 10:28-30).


• that He received all authority in heaven and earth, (Matt. 28:18).
• that He was the Way the Truth and The Life, (John 14:6).
• that He will resurrect people on the Last Day, (John 6:40, 44, 54).
• that the Holy Spirit bears witness of Him, (John 15:26).

Do you want to try and please God through keeping the Law of Allah or by the grace of Christ? Is
the greatest act of love to ask you to earn heaven through good deeds or is it to be a sacrifice in order
to give to you what you cannot attain yourself? In which is the greatest act of love?
If you want the eternal life that Jesus can give you, then trust what He did on the cross and do not
rely on your own works to please God. Trust Jesus by faith. It is not Muhammad who forgives sins.
Jesus does that (Luke 5:20; 7:48-49). Jesus said, "Believe in God. Believe also in me," (John 14:1).
Like Abraham, be righteous in God's eyes . . by faith.
What is Islam?
Islam (1.2 billion adherents) is one of the major world religions that, along with Christianity (1.9
billion adherents) and Judaism (14 million adherents), teaches monotheism which is the doctrine that
there is only one God in all existence. Like Christianity and Judaism, Islam traces its roots back to the
patriarch Abraham (Gen. 12). The word "Islam" means "surrender" or "submission" 177 and it comes
from the root word "salam" which means "peace." A Muslim (or Moslem - which means one who
surrenders to God) is an adherent of Islam, a religion with precise theological doctrines about God,
judgment, heaven, hell, angels, prophets, salvation, etc. The Arabic word for god is "allah" which has
become a kind of name of God in Islam. Islam teaches that Allah is the one and only deity in all
existence (Qur'an 5:73; 112:1-4). He is supreme, all knowing (40:20), ever-present, different from
all of creation (3:191), and in complete control of all things. According to Islam, Allah created the
universe in six days (2:29; 25:61-62) and all that is in it continues to exist by his permission and will.
Allah is non-Trinitarian (5:73), absolute, and eternal.
The Koran (or Qur’an, which means "the reading" in Arabic) is the sacred book of Islam and is
broken up into 114 chapters called Suras which cover the subjects of ethics, history, law, and
theology. It is highly revered by Muslims as the direct, literal word of God. The Qur'an (also spelled
Quran and Koran) was delivered by the angel Gabriel (also known as the Holy Spirit) to Muhammad
over a 23 year period after Muhammad's initial encounter with Gabriel in a cave when he was 40 years
old. Muslims consider Muhammad (full name of Muhammad Ibn Abdullah) to be the final prophet of
God to the world. Muhammad was born in 570? AD in Mecca and died in 632 AD.
Second only to the Islamic belief in the unity/oneness of God is the supremacy of Muhammad as
Allah's prophet. But, Islam acknowledges that several prophets preceded Muhammad. The major
ones are Noah, Abraham, Moses, David, and Jesus. These prophets gave revelations from God which
were written as scriptures; mainly, the Old and New Testaments. These predecessors to Muhammad
are considered great prophets who spoke for God to specific people and whose message was meant for
that time. Jesus, then, was simply one of many prophets according to Islam. Therefore, they deny
the Christian doctrine of the deity of Jesus, the need for His atoning sacrifice (4:157-158), the Trinity
(5:73), and much more. According to Islam, no sacrifice is needed to be forgiven, only faith in Allah,
sincere repentance, and obedience to Islamic law (3:135; 7:8-9; 21:47; 49:14; 66:8-9). In fact, in
Islam, the greatest of sins, called shirk, is to attribute "partners" to God. In other words, to say that
God is a Trinity of persons is an unforgivable sin to a Muslim.
In addition to the Qur'an is the Hadith. It is another source of authority in Islam, though it is
secondary to the Qur'an. The Hadith are the sayings and deeds of Muhammad as recorded by his
companions. They are oral traditions and are considered authoritative and instructive as
commentaries and applications of Qur'anic principles. The Hadith has additional principles not found in
the Qur'an. The Hadith are the inspired truths of God transmitted to us in the style and words of
Muhammad where the Koran is the exact words of God which is supposed to be protected from
corruption by Allah.
In Islam, all Muslims are united by the common faith irrespective of class, location, race, or
gender. Therefore, they have a special bond of unity and equality. The primary "truth" of Islam is
found in the first pillar of Islam known as the shahda: "There is no true God except Allah and
Muhammad is the Messenger of Allah."
Islamic theology also teaches that angels were created from light, that jinn are another race of
beings, created from fire, who are invisible yet all around us, that there is an eternal judgment to
Paradise for the good and hell for the bad, that Jesus was never crucified, that drinking alcohol is
forbidden as is gambling, etc.
Within the first two centuries after its inception in Arabia, Islam spread very quickly, often aided by
sword (jihad178), into North Africa, up through Europe to Spain, and east to India. Presently, about 1
billion people are Muslim world wide with adherents on every continent and nation. It is the world's
fastest growing religion and second in size only to Christianity.
Like most ancient religions, there are sects. Islam is no different. The major sects in Islam are the
Sunnites and the Shiites. The Sunnites are the largest group and comprise about 90% of all Muslims.
177
The One (True) God; To Him do we submit" (Qur'an 2:133).
178
Jihad means "striving." Fighting against one's own sinful self. Also, a physical fight for the truth of Islam, not
allowing anyone to steal the ability to worship. It also means "holy war."
The Shiites, though smaller in number, are significant in Islamic history and presently occupy the lands
of Iran, Iraq, Lebanon, Syria, Saudi-Arabia, Yemen, and Persian Gulf states.
The most important place of worship for the Muslim is the Mosque which is always pointed towards
Mecca, the birthplace of Muhammad which is located in Saudi Arabia. All Muslims must face Mecca
during their times of prayer because in Mecca is the Ka'aba, a cube structure allegedly built by
Abraham which contains a sacred stone. When a Muslim is in Mecca, he or she faces the Ka'aba.
Many Muslims hope for shari'ah the complete rule of Islamic law in the world. To this end, Muslims
are seeking more converts, attacking other religious systems both by the sword and by word, moving
into every nation, and seeking political power wherever they can achieve it. Islam is a growing reality.
Muhammad

Few people in all of history ever have an influence so far reaching that the course of nations are
changed. Muhammad is just such a person. For most of the world, Muhammad was an Arab who lived
in the Middle East in the 7th century and is the founder of Islam. For Muslims, Muhammad is the final
prophet of Allah who supercedes all other prophets and who alone delivered the final and perfect word
of God. Whichever your position, Muhammad is an important figure in human history.
Muhammad (full name is Muhammad Ibne Abdullah) was born in 570(?) in Mecca which is now
located in Saudi Arabia. Mecca was then the cultural and religious center of Arabia. The area had no
central government and was full semi-warring tribes with numerous, competing, polytheistic religions.
At the heart of Mecca was the Ka'aba (cube in Arabic), a shrine about 60 feet, by 60 feet, by 60 feet,
containing hundreds of idols, and known as the House of Allah. Allah was recognized as the supreme
deity, but was worshipped along with other deities. The Ka'aba was believed to have been built by
Abraham and his son Ishmael on the same spot as the first shrine to God built by Adam. On the
eastern corner of the Ka'aba is the Black stone called in Arabic, Hajar al Aswad. The Black Stone is
probably a meteorite.

Muhammad is born

Muhammad was born to his mother Amina, into the Quraish, the then ruling tribe of Mecca. Up to
the age of eight, he was raised by his grandfather Abdul Muttalib because Abdallah, his father, died in
Yathrib a few weeks before Muhammad was born. Amina, his mother, died when he was six. After the
death of his grandfather, his uncle Abu Talib then assumed responsibility for raising Muhammad. Abu
Talib was a businessman involved in trade so it is likely that Muhammad went with him on business
trips and encountered both Jews, 280 miles to the north in Madina, and Christians also to the north
and to the south in Nejran. History tells us that when he was 12 he accompanied his uncle on a
trading caravan to Syria. His encounters with Jews and Christians seem to be reflected in the Qur'an
in passages that refer to "The People of the Book" (3:64, 71, 187; 5:59). The term "People of the
Book" is a reference to Jews and Christians who had received God's word through the prophets before
Muhammad.
At 25 years old, Muhammad was hired to manage the business of a wealthy widow named Khadija
who was 15 years older than he. He went to Syria and traded there successfully. Apparently this
impressed Khadija. She ended up proposing to Muhammad later and in 595 they were married. They
had two sons, who died in infancy, and four daughters: Zaynab, Ruqaiyah, Fatima and Umm
Kulthum. Muhammad and Khakija were married for 25 years until Khadija died at the age of 65 during
the month of Ramadan, well after the start of Islam.
Around 35 Muhammad assumed the habit of going outside of Mecca to Mt. Hira for meditation and
contemplation. There was a cave there and he often went there for solitude. It was during one of
these times of meditation that Muhammad said an angelic being appeared to him, calling him. This
disturbed Muhammad (Qur'an 81:19-29) and he told his wife Khadija that he thought he had been
visited by an evil Jinn. Jinn are supposed to be living beings like people, but not angels, who were
created from fire and are invisible, yet dwell on the earth. A short time later, in the year 610 (believed
to be the 26th of Ramadan), while in a cave on Mt. Hirah, Muhammad said that the angel Gabriel
appeared to him and commanded him to recite (96:1-19). This recitation became the Qur'an. In
these encounters with the angel Gabriel, sometimes he would see the angel, other times he would only
hear him, and at others he only heard the sound of a bell through which the words of the angel came.
Muhammad could neither read nor write so he was instructed to memorize the words given to him
by Gabriel. This complete recitation which Muhammad received over a 23 year period, ending in 632,
the year of his death, is known as the Qur'an. Initially, Muhammad doubted that he was being called
by Allah to be a prophet. Others, including his wife and a cousin, counseled him by saying that Allah
would only be truthful to him and would not allow him to be deceived. Muhammad became convinced
and even wrote in the Qur'an, "Say: Whoever is an enemy to Gabriel-for he brings down the
(revelation) to thy heart by Allah's will, a confirmation of what went before, and guidance and glad
tidings for those who believe," (2:97).

Islam takes root


It became the mission of Muhammad to proclaim the truth of Islam, given to him by Allah, through
the angel Gabriel. Muhammad called the people of his area to repent from their idol worship, to do
good, and to serve the one and true God, Allah. He taught that man is God's slave and it is his duty to
submit to God and obey him. He said that the Day of Judgment was coming and that a man's works
will be weighed on that day. Those whose good deeds out weigh their bad may, by Allah's grace, be
saved and enter Paradise which is full of sensual pleasures. The unsaved go to hell. His first
converts were his wife, Khadija, his cousin Ali, and his adopted son Zaid ibn Haritha. Soon afterwards,
Abu Bakr also believed. In his first three years of proclaiming Islam, he had 40 converts.
Though his continued preaching brought only a few converts, it did bring much opposition. The
ruling tribe, the Quraish, tried to get Muhammad to stop his preaching by appealing to his uncle, Abu
Talib. But, Muhammad adamantly refused to stop proclaiming the message he had received. Because
Abu Talib was very influential in the Quraish, Muhammad's life was protected and he was able to
continue his preaching which angered many people. The Quraish began to persecute the Muslims by
beating them and boycotting their businesses. During public prayers, Muhammad was accosted and
mocked. His followers were likewise treated poorly. But, Muhammad remained steadfast.
Because of the persecution, the Muslims moved to Abyssinia, Ethiopia today, and was protected by
the Christian ruler there. After a time, he returned to Madina and continued his preaching. More
converts joined his ranks and more idolaters sought to defeat him. This is because the message of
Islam was socio-political. Islam covers belief, society, behavior, ethics, etc. This monotheistic belief
system threatened the lucrative business that grew around the pilgrimages to the Ka'aba that so many
Arabs enjoyed. The ruling tribe, the Quraish, soon found that within their reign a small band of
believers, a small "country" unto themselves, was rising up. The ruling party became more and more
concerned and threatened by the Muslims and their pressure increased against Muhammad.
In the year 620 Muhammad lost his beloved uncle Abu Talib (who never became a Muslim) and his
wife Khadija. "After a few months Muhammad sought comfort by marrying the widow of one of the
believers named Sawdah. He also later married Ayisha, the seven-year old daughter of his friend Abu
Bakr, who he took into his home three years later." 179 According to Muslim historians, Muhammad has
12 wives when he died.

Hegira

622 is a significant date for Muslims. It is known as the year of the Hijra, or Migration from Mecca
to Yathrib (which later became Medina) where they established their first real Islamic community. The
Muslim calendar begins its history from July 16, 622 the first day of the lunar year in which the Hegira
took place.
In Medina, he preached about Allah and monotheism and urged all people to return to the true
faith of Abraham. At that time in Medina, he would pray facing Jerusalem, as did the Jews who were
very populace in that city. He preached about repentance, one God, and forgiveness of sins. His first
sermon in Medina was on a Friday. Therefore, Islamic congregational worship occurs on Fridays.
Gradually, however, the Jews began to disapprove of him and his movement. He confronted them
and told them they had misread the Scriptures. This estranged the Jews in the region and finally one
day while praying, he suddenly changed direction and faced Mecca. He said the Ka'aba, in Mecca, was
the true place of worship since it was built by Abraham. To this day, all Muslims are to face Mecca
when praying.

After two years in Medina, the Muslims were not fairing too well financially and that, combined with
mild persecution, prompted a revelation to come to Muhammad permitting him to raid passing
caravans. This he did and the Muslim financial problems were solved. Soon afterward, there was then
179
Miller, William M., A Christian's Response to Islam, Presbyterian and Reformed Publishing, Phillipsburg, New
Jersey, 1976. page 23.
a significant battle at Badr where Muhammad, with 350 men, defeated an army of 1,000 men. This
boost gave confidence to the Muslims, encouraged more converts, and made the Quraish even more
uneasy.
In the fifth year of the Hegira, the Quraish tried to destroy the Muslims but failed. By now the
Muslims were too strong so the Quraish never again tried to defeat them. Muhammad then set his
sights on Mecca. At one point in 628, Muhammad took 10,000 men and entered Mecca unchallenged.
The leader of the Quraish converted to Islam.
From there, Muhammad's movement gained further momentum. In 631 two tribes joined
Muhammad. They were the Hijaz and Najd. More and more important people began to convert to
Islam including Umar and Hamza, two powerful people in the region. From this time on, many battles
ensured. In 625 there was the Battle of Uhud. In 627, the Battle of the Trench. In 628 Muhammad
signs a treaty with Quraish. There is the Battle of Hunsin. In 630 Muhammad had conquered Mecca
and he destroys all the idols in Mecca.
In 632, Muhammad delivers his last sermon, later falls ill, and dies in the presence of his favorite
wife, Aisha, and her father, Abu-Bakr. He was buried in Medina in his own house. His father in law,
Abu-Bakr, becomes Caliph, the religious leader of Islam.
The Qur'an
The Qur'an (Koran, Quran) is the Holy Book of Islam and the religions most sacred writing.
Muslims consider it the actual word of Allah and not the word of Muhammad to whom it was given.
Muslim tradition states that the angel Gabriel visited Muhammad and gave him the words directly from
Allah. These words were Allah's words of wisdom, truth, and commandments to His creation. The
Qur'an (which means recitation) was revealed in the Arabic dialect used by the Quraish tribe of Mecca
of that time. This dialect became the formal Arabic of the Islamic nations due to the distribution of
Qur'anic scriptures throughout the Islamic empire. In the Arabic the Qur'an is poetic in style with
rhymes, meter, and shifts in line lengths. Those who speak the language say it is a beautiful work.
The Qur'an deals mainly with what and how Allah wants mankind to believe and do in Man's moral
struggle. Its primary theme is that of complete submission to the will of Allah. However, it also, it
teaches . . .

• there is only one sovereign God (3:191; 5:73; 112:1-4).


• there will be an end of the world and judgment day (3:30; 35:33-37).
• those who are not Muslims will go to hell (3:13; 19:49).
• that those whose good deeds exceed their bad will obtain paradise (3:135; 7:8-
9; 21:47).
• social and ethical behavior for Islamic society.

In the year 610 (believed to be the 26th of Ramadan), while in a cave on Mt. Hirah, which is now
called Mount Jabal Nur, Muhammad said that the angel Gabriel appeared to him and commanded him
to recite (96:1-19). From that point on, Muhammad received revelations up to the time of his death,
23 years later in 632. In these encounters with the angel Gabriel, sometimes Muhammad would see
the angel, other times he would only hear him, and at others he only heard the sound of a bell through
which the words of the angel came.
Since Muhammad could not read or write, his companions wrote down what he said. These
recitations were copied onto a variety of materials: papyrus, flat stones, palm leaves, shoulder blades
and ribs of animals, pieces of leather and wooden boards. 180 Additionally, these sayings were also
being memorized by Mohammad's followers. In fact, to this day, great emphasis is placed upon
memorizing the entire Qur'an and there are many thousands of Muslims who have committed it to
memory. The work is roughly the same size as the New Testament.
Apparently, there was no attempt made to collect all of the sayings given by Muhammad during his
lifetime. After all, Mohammad was continuing to give ' recitations' on a somewhat regular basis. But,
after he died in 632, Abu-Bakr, Muhammad's father in law, became the caliph (religious leader of the
Muslims) and there was a small effort to collect the fragments of Qur'anic sayings into a common
place. But, it wasn't until the fourth leader of Islam, Caliph Uthman, that the whole Qur'an was finally
assembled, approved, and disseminated throughout the Muslim world.
The Koran also contains many biblical figures (Abraham, David, Moses, and Jesus) as well as non-
biblical figures. However, some of the accounts of biblical characters are different than the Bible.
The Koran is divided into 114 chapters, called Surahs. The word surah means "row". Today the
Koran is arranged with the longer surahs first and the shorter ones after. The whole thing is divided
into 30 approximately equal lengths. "Islamic law prohibits the touching of the physical Arabic Qur'an
(and formal, but not casual, recitation) unless the person he is in a state of purity which corresponds
to the greater of Ablution... it is a priest group Chin that every Moslem must commit at least 12 vs. or
lines of the Qur'an to memory." 181
The revelations are identified has having been revealed either in Mecca or Medina. Generally, those
revealed in Mecca are the earlier ones and are more poetic and deal with apocalyptic themes. The
Medina revelations deal more with the law of Allah. Many have noted that the arrangement of the
Qur'an is not chronological or thematic. The subjects tend to be disjointed and shifting. This is due in

180
Watt, W. Montgomery, Islamic Surveys: Bell's Introduction to the Qur'an, Aldine Publishing Company, Chicago,
1970, page 40.
181
(Glasse, Cyril, The Concise Encyclopedia of Islam, Harper & Row, Publishers, Inc. San Francisco, 1989. p. 220)
part to the directions of Mohammad to put certain savings in different places in the Surahs. Muslims
are aware of this and considered as to be the divine order in the Koran.

Chronology of early Islam


Notice how closely Islam's inception is associated with war. From 623 to 777, a span of 154 years,
there are 83 military conflicts involving the Muslims...and that is just what I have recorded here. Is
Islam a religion of peace? Muslims tell me it is. But....

• 570 - Birth of Muhammad in Mecca into the tribe of Quraish.


• 577 - Muhammad's mother dies
• 580 - Death of Abdul Muttalib, Muhammad's grandfather.
• 583 - First journey to Syria with a trading Caravan
• 595 - Muhammad marries Khadijah a rich widow several years older than him.
• 595 - Second journey to Syra
• 598 - His son, Qasim, is born
• 600 - His daughter, Zainab, is born
• 603 - His daughter, Um-e-Kalthum, is born
• 604 - His daughter, Ruqayya, is born
• 605 - Placement of Black Stone in Ka'aba.
• 605 - His daughter, Fatima, is born
• 610 - Mohammed, in a cave on Mt. Hira, hears the angel Gabriel tell him that Allah is
the only true God.
• 613 - Muhammad's first public preaching of Islam at Mt. Hira. Gets few converts.
• 615 - Muslims persecuted by the Quraish.
• 619 - Marries Sau'da and Aisha
• 620 - Institution of five daily prayers
• 622 - Muhammad emigrates from Mecca to Medina, which was then called Yathrib,
gets more converts.
• 623 - Battle of Waddan
• 623 - Battle of Safwan
• 623 - Battle of Dul-'Ashir
• 624 - Muhammad and converts begin raids on caravans to fund the movement.
• 624 - Zakat becomes mandatory
• 624 - Battle of Badr
• 624 - Battle of Bani Salim
• 624 - Battle of Eid-ul-Fitr and Zakat-ul-Fitr
• 624 - Battle of Bani Qainuqa'
• 624 - Battle of Sawiq
• 624 - Battle of Ghatfan
• 624 - Battle of Bahran
• 625 - Battle of Uhud. 70 Muslims are killed.
• 625 - Battle of Humra-ul-Asad
• 625 - Battle of Banu Nudair
• 625 - Battle of Dhatur-Riqa
• 626 - Battle of Badru-Ukhra
• 626 - Battle of Dumatul-Jandal
• 626 - Battle of Banu Mustalaq Nikah
• 627 - Battle of the Trench
• 627 - Battle of Ahzab
• 627 - Battle of Bani Quraiza
• 627 - Battle of Bani Lahyan
• 627 - Battle of Ghaiba
• 627 - Battle of Khaibar
• 628 - Muhammad signs treaty with Quraish.
• 630 - Muhammad conquers Mecca.

• 630 - Battle of Hunsin.


• 630 - Battle of Tabuk
• 632 - Muhammad dies.
• 632 - Abu-Bakr, Muhammad's father-in-law, along with Umar, begins a military move
to enforce Islam in Arabia.
• 633 - Battle at Oman
• 633 - Battle at Hadramaut.
• 633 - Battle of Kazima
• 633 - Battle of Walaja
• 633 - Battle of Ulleis
• 633 - Battle of Anbar
• 634 - Battle of Basra,
• 634 - Battle of Damascus
• 634 - Battle of Ajnadin.
• 634 - Death of Hadrat Abu Bakr. Hadrat Umar Farooq becomes the Caliph.
• 634 - Battle of Namaraq
• 634 - Battle of Saqatia.
• 635 - Battle of Bridge.
• 635 - Battle of Buwaib.
• 635 - Conquest of Damascus.
• 635 - Battle of Fahl.
• 636 - Battle of Yermuk.
• 636 - Battle of Qadsiyia.
• 636 - Conquest of Madain.
• 637 - Battle of Jalula.
• 638 - Battle of Yarmouk.
• 638 - The Muslims defeat the Romans and enter Jerusalem.
• 638 - Conquest of Jazirah.
• 639 - Conquest of Khuizistan and movement into Egypt.
• 641 - Battle of Nihawand
• 642 - Battle of Rayy in Persia
• 643 - Conquest of Azarbaijan
• 644 - Conquest of Fars
• 644 - Conquest of Kharan.
• 644 - Umar is murdered. Othman becomes the Caliph.
• 647 - Conquest of the island of Cypress
• 644 - Uman dies and is succeeded by Caliph Uthman.
• 648 - Campaign against the Byzantines.
• 651 - Naval battle against the Byzantines.
• 654 - Islam spreads into North Africa
• 656 - Uthman is murdered. Ali becomes Caliph.
• 658 - Battle of Nahrawan.
• 659 - Conquest of Egypt
• 661 - Ali is murdered.
• 662 - Egypt falls to Islam rule.
• 666 - Sicily is attacked by Muslims
• 677 - Siege of Constantinople
• 687 - Battle of Kufa
• 691 - Battle of Deir ul Jaliq
• 700 - Sufism takes root as a sect of Islam
• 700 - Military campaigns in North Africa
• 702 - Battle of Deir ul Jamira
• 711 - Muslims invade Gibraltar
• 711 - Conquest of Spain
• 713 - Conquest of Multan

• 716 - Invasion of Constantinople


• 732 - Battle of Tours in France.
• 740 - Battle of the Nobles.
• 741 - Battle of Bagdoura in North Africa
• 744 - Battle of Ain al Jurr.
• 746 - Battle of Rupar Thutha
• 748 - Battle of Rayy.
• 749 - Battle of lsfahan
• 749 - Battle of Nihawand
• 750 - Battle of Zab
• 772 - Battle of Janbi in North Africa
• 777 - Battle of Saragossa in Spain

References:

• Miller, William M., A Christian's Response to Islam, Presbyterian and Reformed Publishing,
Phillipsburg, New Jersey, 1976.
• Geisler, Norman, Baker Encyclopedia of Christian Apologetics, Grand Rapids, Michigan,
Baker Books, 1999.
• Glasse, Cyril, The Concise Encyclopedia of Islam, Harper & Row, Publishers, Inc. San
Francisco, 1989.
• Morey, Robert, The Islamic Invasion, Harvest House Publishers, Eugene Oregon, 1992.
• Early Islam Chronology
• Islam Chronology
Divisions within Islam

Because of Islam's great growth geographically in the first two centuries of its inception, there
needed to be a larger set of Islamic laws capable of handling the different needs of Muslims throughout
the Empire. The Qur'an and the Hadith were not detailed enough to provide all the answers.
Therefore, in the 8th century A.D., there arose a school of legal experts who interpreted and applied
Islamic principles to different situations throughout the Empire. However, different scholars disagreed
with these experts in various areas. This led to a variety of legal schools of thought within Islam.
These different schools became different sects within Islam. The largest of the sects is the Sunni
which comprises about 90% of all Muslims. The next two largest are the Shi'i and Sufi. After these,
there are numerous splinter groups which are often named after the individual scholars who began
them: Hanifa, after Abu Hanifa; Maliki, after Malik ibn Anas; Shafi'i, after Muhammad ibn Idris al-
Shafi'i; Zaydi, after Zayd ibn Ali; the Nusayri, Ismaili, Murji'ah, etc.

Sunni Muslims

Sunni Muslims These are followers of the Hanifa, Shafi, Hanibal and Malik schools. They constitute
a 90% majority of the believers, and are considered to be main stream traditionalists. Because they
are comfortable pursuing their faith within secular societies, they have been able to adapt to a variety
of national cultures, while following their three sources of law: the Qur'an, Hadith and consensus of
Muslims.
The Sunni emphasize the power and sovereignty of Allah and his right to do whatever he wants
with his creation. A strict predeterminism is taught. Its rulership is through the Caliphate, the office
of Muslim ruler who is considered the successor to Muhammad. This successor is not through
hereditary lineage.

Shi'ite Muslims

The Shi'ites (also known as the Ja'firi school) split with the Sunni over the issue of the successor to
Muhammad. This split occured after the assassination of the fourth caliph in 661. Shi'ites believe that
the successor to Muhammad should have been Ali, his son in law, and that subsequent successors
should have been through his lineage through his wife Fatima.
Shi'ism is broken into three main sects: the Twelve-Imam; Persia, Iraq, Afghanistan, Lebanon,
Pakistan, and Syria), the Zaydis (Yemen), and the Ismailis (India, Iran, Syria, and East Africa). Each
group, of course, has differences of doctrine.
"Shi'ite theology includes a doctrine known as the five supports: these are Divine Unity (tawhid),
prophecy (nubuwwah), resurrection of the soul and body at the Judgment (ma'ad), the Imamate 182
(imamah), and justice ('adl). The first three are found in Sunni Islam, albeit with some differences of
emphasis; the Imamate, however , is the essence of Shi'ism, and the last, justice, is an inheritance
from the Mu'tazilites, or rationalists, whose system is in many ways perpetuated in Shi'ite
theology..."1 The Imamate, fom the word "Imam", in the Shi'ite traditions is the political and religious
leader of the Shi'ite sect. This person possesses great power and influence. According to Shi'ite
doctrine, the Imam must be a biological successor of Ali. The Imam is also sinless and infallible on all
matters of Islamic doctrine and will intercede for Muslims in the afterlife. The Shi'i and the Sunni differ
in some interpretations of the Qur'an and Hadith and even have a different canon of Hadith and the
Sunni.

Glasse, Cyril, The Concise Encyclopedia of Islam, Harper & Row, Publishers, Inc. San Francisco, 1989, page
182

368.
Sufi Muslims

The Sufi are a mystical tradition where the followers seek inner mystical knowledge of God. This
sect "officially" developed around the 10th century and has since fragmented into different orders:
Ahmadiyya, Qadariyya, Tijaniyya, etc. Of course, the Sufi believe their roots can be traced back to the
inception of Islam in the early 7th century.
The Sufi mystic must follow a path of deprivation and meditation. There are various forms of
abstinence and poverty. Worldly things are renounced and a complete trust in God's will is taught.
The goal is to attain to a higher knowledge and experience of Allah. The mystical focus meant that the
Qur'an could be interpreted in different ways and so Sufism taught that the Qur'an had mystical
meanings hidden within its pages. Out of this mysticism a type of pantheism developed among some
Sufi believers. Pantheism is the teaching that God and the universe are one. Of course, the orthodox
Muslims, called the Sunni, reject this idea since they claim that Allah is the creator of the universe and
distinct from it.
In part, Sufism arose as a reaction to the growing Islamic materialism that had developed in the
Empire at that time. Islam had achieved great power and geographical scope and with it, the material
gain was great.

Conclusion

As you can see, Islam is not the united religious system it claims to be. There are divisions among
its ranks and even those divisions have divisions. But what is interesting is that the Qur'an tells the
Muslims to have no such divisions.

"The same religion has He established for you as that which He enjoined on Noah - the [sic] which we
have sent by inspiration to thee - and that which we enjoined on Abraham, Moses, and Jesus: namely,
that ye should remain steadfast in religion and make no divisions therein: to those who worship other
things than Allah, hard is the (way) to which thou callest them..." (42:13)

If this is the case, then the Muslim must admit that the divisions within Islam are sinful. But, such
is the nature of humanity, to divide and set ourselves against one another.
What are the doctrines of Islam?
Note: All references are to the Koran where applicable.

God
.A There is only one God (5:73; 112:1-4).
.B God is called Allah by Muslims (5:73)
.C Allah sees all things (40:20), is present everywhere (2:115; 7:7).
.D Allah is the sole creator and sustainer of the universe (3:191).
.E Allah is not a Trinity, but is one (5:73).
.F Allah is all-knowing (2:268; 10:61) and all powerful (6:61-62).
.G Allah created the heaven and earth (2:29; 6:1, 73; 25:61-62; 36:81; 46:33).
Salvation and Judgment
.A Allah will judge all people on the Day of Judgment (3:30; 35:33-37; 99:6-8).
.B If your good deeds exceed your bad deeds, and you believe in Allah, and sincerely repent of
sins, you may go to heaven (3:135; 7:8-9; 21:47; 49:14; 66:8-9).
.C There is an eternal hell for those who are not Muslims, not practicing and of the truth faith
(3:77).
.D Hell is a place of unlimited capacity (50:30) eternal torment (2:39; 14:17; 25:65; 39:26),
fire (9:63; 11:16; 25:11-12; 104:6-7), with boiling water (38:55-58; 55:43-44), where skin
is burned and renewed (4:56), for unbelievers (3:13; 19:49) and Jinn (11:119), with faces
covered with fire (14:49-50).
.E There is a tree in hell, named Tree of Zaqqum, from which bad fruit is given and the damned
are forced to eat (37:62-67; 44:43-48; 56:52-55).
.F Heaven (Paradise), a Garden (79:41) of bliss and fruit (69:21-24), has rivers (3:198), with
maidens pure and holy (4:57), and carpets and cushions, (88:8-16).
.G There will be a physical resurrection of all people (19:93-95) on the day of judgment (3:77;
15:25; 16:38; 42:29).
.H Judgment is based on a person's sincere repentance (66:8-9) and righteous deeds (5:9;
24:26; 45:21-22; 64:7).
Other
.A There is an afterlife (2:154;75:12).
.B There are such things as angels, created by Allah, that are created from light. Angels are
obedient slaves incapable of refusing to do Allah’s will. The angel Gabriel brought the
revelation of the Koran to Muhammad (2:97).
.C The Holy Spirit is the angel Gabriel (2:97; 16:102).
.D Jinn are unseen beings, created (51:56) from fire (15:27; 55:15), but are not angels. They
have communities. There are good and bad Jinn.
.E The Devil, called Iblis, (2:34) is a bad Jinn.
.F Jesus was a great prophet but not the son of God (9:30), is not divine (5:17, 75), was not
crucified (4:157).
.G Muhammad is Allah’s greatest and last prophet and his message supercedes all other past
prophets including Jesus.
.H The Koran is Allah’s word. He literally spoke it to Gabriel who gave it to Muhammad.
.I There are other holy writings but they are superceded by the Koran.
.i Torah - the First Five books of Moses
.ii Injeel - the message that Jesus gave, written down, but no longer exists. The writings
have been altered by scholars. Whatever agrees with the Koran is true.
.iii Zaboor - the Psalms
.J Pre-ordainment (Qadar) is the teaching that all things, good and bad, are preordained to
occur.
.K Fasting is to be observed during the month of Ramadan (2:185).
.L Drinking alcohol is forbidden (2:219; 4;43; 5:93-94; 16:67)
.M Gambling is forbidden (2:219; 5:90-94).
.N Man is made from the dust of the earth (23:12).
.O There is no last minute repentance (4:18).
The Five Pillars of Islam
The Five Pillars of Islam are core beliefs that shape Muslim thought, deed, and society. A Muslim
who fulfills the Five Pillars of Islam, remains in the faith of Islam, and sincerely repents of his sins, will
make it to Jannah (paradise). If he performs the Five Pillars but does not remain in the faith, he will
not be saved.

1. Shahada
A. The Shahada is the Islamic proclamation that "There is no true God except Allah and
Muhammad is the Messenger of Allah."
B. This is the confession that Allah is the one and only true God, that Allah alone is worthy of
worship, that Allah alone is the sovereign lord who does what he wills with whoever he wills.
It means that all his rules and laws found in the Koran are to be followed. It means that the
Christian doctrine of God as a Trinity is false as are all other belief systems including
pantheism.
C. Muhammad is the true and greatest prophet of Allah and recognition of Muhammad as the
Prophet of God is required. It was through Muhammad that Allah conveyed the last and final
revelation.
2. Prayer (Salat)
A. Prayer involves confession of sins which begins with the purification of the body and ends
with the purification of the soul. Prayer is performed five times a day. The first prayer is at
dawn and the last at sunset.
3. Fasting (Saum)
A. The month of Ramadan is the month of fasting in Islam. It is an act of worship where the
faithful follower denies his own needs and seeks Allah. Usually, this fasting entails no
drinking, eating during, or sexual relations during the daylight hours for the entire month of
Ramadan.
4. Alms-giving or charity (Zakat)
A. Charity given to the poor. It benefits the poor and it helps the giver by moving him towards
more holiness and submission to Allah. Alms-giving is considered a form of worship to God.
5. Pilgrimage (Hajj)
A. This is the pilgrimage to Mecca. All Muslims, if they are able, are to make a pilgrimage to
Mecca. It involves financial sacrifice and is an act of worship. Muslims must make the
pilgrimage the first half of the last month of the lunar year
True faith in Islam
The pillars of the faith of Islam can be compared to the concept of a Statement of Faith, or Articles
of faith. These are Islamic concepts of essentials of the faith.

1. Allah
A. Allah is the supreme being of all. He is uncreated, the creator of all, without beginning or
end. He is completely sufficient to himself and needs no other. He does not have offspring
nor a spouse. He knows all things, is everywhere, and is all powerful. He hears all prayers.
Everything that occurs, does so by his permission.
2. His Angels
A. Angels reside in the unseen world and carry out the commands of Allah. They cannot sin.
Muhammad stated that it was the angel Gabriel that brought the message of the Koran to
him.
3. His Messengers
A. People who have been sent from Allah to a particular group of people for the purpose of
giving to them the message revealed by Allah. Some of them are Noah, Abraham, Moses,
David, Jesus, and, of course, Muhammad. Islam teaches that all messengers previous to
Muhammad were sent to limited people groups where Muhammad was sent to all people.
4. His Books
A. Islam recognizes many sacred scriptures that have been given by Allah throughout history.
However, Muslims claim that only the Qur’an is trustworthy and that the other scriptures
have been compromised because we do not posses their original manuscripts. They assert
that the accounts of the Bible were written down hundreds of years later and cannot be
considered inerrant, and they were written in ancient languages which have been lost.
Therefore, exact translations are not possible. Nevertheless, the scriptures recognized in
Islam are:
i. The Koran - The Koran (Qur'an) is the inspired word of Allah given to people through the
Prophet Muhammad and it supercedes all other scriptures before it including. It alone is
inerrant and trustworthy as a revelation for today. It is unchanged from the beginning.
ii. The Torah -the first five books of Moses.
iii. The Injil - the gospel message of Jesus in the New Testament
iv. The Psalms - the sacred writings given to David.
5. The Last Day
A. There is a future day in which this world and its governments and systems will come to end
and all people will face judgment based upon their deeds. Muslims go to paradise and non-
Muslims go to hell.
6. Divine Preordainments good or bad
A. In Islam, Allah is completely control of all things and ordains all things that occur.
Islamic Terms
1. Adhan - The call to prayer.
2. Ahmad - Another name for Muhammad.
3. Allah - The Arabic word for "god." It is often used as a name for God in Islam.
4. Badr - The place of the first significant battle between and the pagans of the Quraish. It is
located in Saudi Arabia.
5. Caliph - A Muslim ruler.
6. Dajjal - Antichrist.
7. Dawah - The proliferation of Islamic teachings through word and deed.
8. Din - Obedience to the revelation of Allah's Qur'an (Koran). It involves total submission.
9. Fatwa - Legal verdict given based on the Qur'an (Koran) and the Sunnah which are the recorded
sayings and deeds of Muhammad.
10. Fiqh - Religious law.
11. Hadith - The sayings and deeds of the prophet Muhammad recorded by his followers.
Considered authoritative and perfect. A saying is called a Sunnah.
12. Hajar - The Black Stone set into the corner of the Ka'aba in Mecca. Tradition states it fell from
heaven.
13. Hajj - The pilgrimage to Mecca which takes place in the last month of the Islamic calendar. One
of the five pillars of Islam.
14. Hawijah - The sixth level of hell which is the place for Christians.
15. Hegirah - Muhammad's immigration to Medina. It begins the Muslim calendar.
16. Hijrah - Moving from a land where a Muslim cannot practice his faith to a land where he can.
17. Ibadah - All the words and deeds with which Allah is pleased. These deeds could be prayer and
charity.
18. Iblis - Satan, a fallen Jinn.
19. Imam - The political head of an Islamic state.
20. Injil - The inspired sayings of Jesus. The message of Jesus.
21. Islam - Submission, the religion of all the prophets of Allah culminating in Muhammad.
22. Jannah - The heavenly garden, Paradise. The place of the faithful in the afterlife.
23. Jihad - Striving. Fighting against one's own sinful self. Also, a physical fight for the truth of
Islam, not allowing anyone to steal the ability to worship. It also can mean "holy war."
24. Jinn - Supernatural, invisible beings race of beings, below angels. They were made from fire and
are capable of looking like humans or animals. Some may dwell in rocks, trees, etc, and may
possess black dogs, and black cats. There are good and bad Jinn and all will be judged on
Judgment Day.
25. Ka'aba - A cube shaped building in Mecca containing a stone laid there by Abraham and
Ishmael. All Muslims face this cube when praying.
26. Koran - Also spelled Qur'an. The holy book of Islam given to Muhammad by Allah through the
Archangel Gabriel. Koran literally means "the recital." It is the final revelation of Allah given to
the prophet Muhammad. It has 114 surahs, or chapters.
27. Kufr - Disbelief
28. Khutbah - A sermon given in a Mosque, usually on Friday.
29. Maksiat - Sinful act
30. Masjid - A center for Muslim activity. It is like a local mosque.
31. Mecca - The Holy City of Islam. It is the birthplace of Muhammad.
32. Medina - The city, then called Yathrib, that Muhammad fled to after announcing Islam.
33. Mosque - A Muslim house of worship.
34. Muhajir - Immigrant, one who leaves his home town to join a Muslim community.
35. Muhammad - the final messenger/prophet of God whose message abrogated all previous
revelations. He received the Koran through the angel Gabriel over a 23 year period.
36. Muhammad ibn Abd Allah - the full name of Muhammad.
37. Muslim - Someone who holds to the religion of Islam.
38. Nas - The multitude of people who are not dedicated to Allah and sway to and fro to various
teachings.
39. Nasara - A word used in the Koran to designate those who are Christians.
40. P.B.U.H. - A shortened designation for "Peace be upon him" which is placed in writing or said
after the word "Muhammad" is used.
41. Paradise - Another word for heaven. A garden (79:41) of bliss and fruit (69:21-24), has rivers
(3:198), with maidens pure and holy (4:57), and carpets and cushions, (88:8-16). It is the hope
of all Muslims.
42. Qadar - Preordainment is the teaching that all things, good and bad, are preordained to occur.
43. Qatl - Murder
44. Qibla - The direction which Muslims turn for daily prayers, towards Mecca.
45. Quraish - An ancient Arab tribe to which Muhammad once belonged.
46. Rakat (rak'ah) - One complete cycle of sacred words and gestures during the ritual prayer.
47. Ramadhan - The ninth month of the Islamic calendar which is the month of the fast.
48. Salat - Prayers
49. Sawm - Fasting
50. Shi'ites - A sect of Islam that teaches that leaders should be political rulers.
51. Shirk - Associating another god with Allah. Associating anything with Allah that is not true and
revealed in the Koran.
52. Sirq - Theft
53. Sufi - A sect of Islam. It is very mystical and teaches strong self denial with the hope of union
with God.
54. Sunnah - The life, practices, and sayings of Muhammad recorded as examples of perfect conduct
in society, religion, action, etc. They contain the Hadith.
55. Sunnis - One of the sects of Islam
56. Surah - A chapter of the Koran.
57. Taghut - Everything that is worshipped or followed other than Allah.
58. Taiyib - Pure, clean, wholesome.
59. Taqwah - Proper fear and veneration of Allah. A divine spark that enables the person to
understand God.
60. Tauhid - Monotheism in Islam is the teaching that there is only one God who alone is worthy of
worship.
61. Tauhid-ar-Rububiyah - Declaring that God is one, the sovereign who performs all his will.
62. Tauhid-al-Uluhiyah - Declaring that God is the only one worthy of worship.
63. Ummah - A religious community, usually referring to an Islamic one.
64. Umrah - A Minor form of pilgrimage to Mecca.
65. Wa Alaikum Assalam - The Arabic way of saying "peace be upon him."
66. Zaboor - The Psalms
67. Zakat - The third pillar of Islam. Alms giving, charity that is given to the poor.
68. Zinah - Fornication and adultery.

These words were collected from numerous websites and books on Islam listed in the Bibliography
and are words used in English writings by Muslims.
Methods Muslims use to attack Christianity

When dealing with Muslims it is wise to understand some of the approaches used by them to
discredit Christianity. Much could be written on each of the following subjects with numerous
examples. But, instead, I will simply expound on the areas common among Muslim approaches and
point out what to watch for.

No. 1 Attack the validity of the Bible

This is expected. If the Muslim can undermine the strength and integrity of God's word, then it
would be much easier for him to win arguments, confound the Christian, and make converts of those
who don't know the truth and power of the Bible. This is what the devil did in the Garden of Eden.
Satan said, "You truly will not die," (Gen. 3:4). I am not calling Muslims satanic. I am simply pointing
out that that is how deception begins, by bringing doubt upon God's word, and that this is exactly
what Muslims do. They try and get people to doubt the Bible and then tell you how great Islam is.
Various methods are used here to accomplish this:

Stating that the Bible has numerous contradictions

.A Of course, I cannot go through all the alleged Bible contradictions here. But my observation
has been that the majority of "biblical contradictions" raised by Muslims are nothing more
than examples of their lack of understanding of biblical theology and context. Always read
the context of verses. Don't let a Muslim simply state that there are contradictions and
leave it at that. Ask him to give you one. If you cannot answer it, do research and get back
to him. Sure, there are some tough areas of scripture, but there are no contradictions in
God's word.183

Criticizing the lack of original manuscripts


.A The point here is that because we do not have the original manuscripts of the Bible, we
cannot really know what the originals said and, therefore, the Bible could have been
corrupted. They then compare the Bible to the Koran and state that the Koran is the
guaranteed, preserved, direct word of Allah given by the angel Gabriel to Muhammad. Of
course, what they fail to mention is
.i The Bible documents are well attested as being reliable and accurate. (See the book,
Evidence that Demands a Verdict by Josh McDowell.)
.ii Regarding the Koran, Muhammad couldn't read or write, so he recited the Koran to
people who then wrote it down. There is no evidence at all that the Koran was written
down in its entirety in Muhammad's lifetime and compiled as a unit. So how could he
have verified its truth?
.iii Shortly after Muhammad's death, the Muslim Uthman ordered all sets of the Koran
manuscripts to be destroyed except the codex of Zaid. Why? Is it because Zaid's copy
was better? If so, how do we know? Did differences in the copies arise so quickly that
discrepancies were evident and Uthman recognized the need for a standardized copy lest
Islam suffer division? It raises doubt on the Koran's supposed incorruptibility.
.iv Muslims claim that Allah said the Koran would be preserved. But, the mere claim is not
enough. It is using the Koran to substantiate the Koran which is circular reasoning.

183
There is an exception due to a textual copyist error. In 2 Chr. 36:9, it states that "Jehoiachin was eight years
old when he became king," and 2 Kings 24:8, states that "Jehoiachin was eighteen years old when he became
king..." This is not an error in the original manuscripts. Most likely, one of the small horizontal strokes used in
recording numbers during the time of Chronicles writing was either smudged out or faded. (Encyclopedia of Bible
Difficulties, by Gleason Archer, Zondervan Publishing, Grand Rapids, MI., 1982, page 215.)
Claiming that the Bible is false because it contradicts the Koran.
.A This is simply begging the question. That means that one assumes the validity of the thing
that he is trying to prove. The Muslim assumes the validity of the Koran and because it
contradicts the Bible, therefore the Bible is wrong. Well, the Christian can just as easily
state that the Koran is wrong because it contradicts the Bible. But the Muslims would not
accept that. Therefore, why should we accept their argument?

No. 2 Attempt to set Paul against Jesus

Muslims often make the claim that Paul never met Jesus and was not a disciple or apostle of Jesus.
Of course, this is not true. Paul encountered Jesus on the rode to Damascus in Acts 9, after Jesus'
resurrection. Jesus spoke to him and commissioned him. So, Paul met Jesus. Also, Peter, who was a
disciple of Jesus, authenticated Paul's writings by calling them scripture in 2 Pet. 3:15-16. If they are
inspired, then they cannot contradict Jesus' words.
In addition, many Muslims claim that Jesus never claimed to be God and that Paul is the one who
wrote that Jesus was God. First of all, if they admit that Paul wrote that Jesus was God, then remind
them of 2 Pet. 3:15-16 where Peter calls Paul's writings Scripture. Nevertheless, they sometimes
assert that Paul hijacked Christianity and took it over and made Jesus into something He was not.
This claim is false.
Perhaps the primary area where Muslims think Paul and Jesus contradict is in the area of who Jesus
is. Paul states that Jesus is God in flesh: Col. 2:9 says, "For in Him the fullness of deity dwells in
bodily form." Muslims assert that no where in the Gospels did Jesus claim to be God. Therefore, they
claim, Paul's words are not true and the Bible is not trustworthy.
This attack by Muslims is an attack based out of opinion. Jesus did claim to be God In John 8:56-
59, it says, "Your father Abraham rejoiced to see My day, and he saw it and was glad." 57The Jews
therefore said to Him, "You are not yet fifty years old, and have You seen Abraham?" 58Jesus said to
them, "Truly, truly, I say to you, before Abraham was born, I am." 59Therefore they picked up stones
to throw at Him; but Jesus hid Himself, and went out of the temple." 184 Why did the Pharisees want to
kill Jesus? They explain their reason in John 10:33 when they say, "For a good work we stone thee
not; but for blasphemy; and because that thou, being a man, makest thyself God," (KJV). Whether or
not the Muslim will accept this, let alone agree that this is correct, matters little because his
presupposition will not allow him to accept, now matter what. Nevertheless, the text clearly states
that the Pharisees understood that Jesus was claiming to be God. Also, consider John 5:18 where the
Apostle John says, "Therefore the Jews sought the more to kill him, because he not only had broken
the sabbath, but said also that God was his Father, making himself equal with God," (KJV). In this
verse Jesus healed on the Sabbath and the Pharisees thought He was breaking the Sabbath law. John
the Apostle also states that when Jesus claimed that God was His Father, that it was "making himself
equal with God." The Muslim will always find a way to argue out of these texts. But, two facts
remain. First, Jesus claimed to be God. Second, the Pharisees denied that Jesus was God and the
Muslims agree with them.
There are other areas that the Muslims will say are where Jesus and Paul do not agree, but when
they bring it up, always ask for an example. Each time I've done this, I've discovered that the Muslim
did not have a sufficient understanding of what the text is saying. Remember, always read the
context.

No. 3 Misrepresentation of Christian doctrine

Sadly, this is a very common error of the Muslims. The single greatest instance of this is in the
doctrine of the Trinity. Muslims so often attack a false understanding of the Trinity by stating that it is
three gods. That is not the correct Christian definition of the Trinity doctrine. Christianity does not
teach there are three gods. It never has and it never will. The doctrine of the Trinity is that there is
only one God who exists in three persons: Father, Son, and Holy Spirit. Trinitarianism is

monotheistic. If a Muslim continues to proclaim that the Trinity is three gods, then I simply stop
Note: in Exodus 3:14 God says, "And God said unto Moses, I AM THAT I AM: and he said, Thus shalt thou say
184

unto the children of Israel, I AM hath sent me unto you," (KJV).


discussing the issue with him because he is not willing to accept what the definition is and it isn't
possible to have a meaningful dialogue.
Another Christian doctrine they fail to understand is the Hypostatic Union. This is the teaching that
Jesus is one person with two natures. He is both God and man as is declared in Col. 2:9, "For in Him
[Jesus] dwells the fullness of the Godhead bodily." Because Jesus was also a man, we have verses
such as John 14:28 where Jesus says "the Father is greater than I." Muslims will say that if Jesus is
God, how could He be greater than Himself? Of course, they fail to understand the Trinity (three
persons) and they fail to understand that Jesus, as a man (Phil. 2:5-8), cooperated with the limitations
of being a man and was in a lower position than the Father (Heb. 2:9) for a while.
Sometimes Muslims refuse to accept Christian explanations for things because it doesn't fit their
agenda nor their preconceived ideas of what they think Christianity is. Oddly enough, Christians often
contribute to this problem by offering inadequate and sometimes erring explanations of Christian
doctrine. Thus, many Muslims are led into error regarding what Christianity really teaches. Christians
need to know their doctrine, and Muslims need to understand the proper explanations for those
doctrines.

No. 4 Misinterpreting various Scripture passages

A very good example of misrepresentation of biblical passages can be found in a dialogue I had
with a Muslim regarding John 1:1, 14. These verses say, "In the beginning was the word and the
word was with God and the word was God." Verse 14 says "and the Word became flesh and dwelt
among us..." The Muslim I was speaking to reasoned thus:

If the word is God then we can insert the word "god" into the verse each time it says "word."
Therefore, we would have it say 'In the beginning was the God and the God was with God and God was
God.' As you can see, this makes no sense. Therefore, when you go to verse 14 where it says the
word became flesh, it cannot mean that God became flesh, because John 1:1 makes no sense."

As you can see, this kind of logic is very bad. First of all, John 1:1 does not say what he said it
does. It does not speak in contradictory terms the way he tried to make it sound. Instead, it uses
both the word "word" and the word "god" in the sentence. I told him that he needed to go with what
the text does say and not with what it does not say. In other words, he reconstructed it in such a
way as to purposely not make sense and then he attacked that.
Another verse, or verses, that they misinterpret is when Jesus says that He is the Son of God. To
the Muslim, this means that God literally had a wife and produced a literal son. Of course, this is not
what is meant by the biblical account. Sonship is in relation to the Fatherhood of God in a spiritual and
metaphoric sense. After all, Jesus says that God is spirit (John 4;24). Of course, God would not have
a body of flesh and bones with which to procreate children. What the Muslims are guilty of doing is
imposing Islamic understandings upon biblical texts and then complaining about the biblical texts in
light of how they interpret them. This is not how one should go about "refuting the opposition."
Rather, the Muslim should seek to understand the biblical/cultural context and deal with the issue from
that perspective not a fabricated one as this example shows.

No. 5 Failing to differentiate between Protestant and Catholic doctrines

Sadly, Christianity is not in complete unity in all things which is why we have denominations. I am
dismayed at the fragmentation within Christianity and think it is a poor witness to the world. But, the
fact is that differences of opinions among Christians do occur. In fact, we are allowed to have
differences of opinion according to God's word found in Rom. 14: 1-13. True Christians are all united
in the essentials of the faith and are often divided on the non-essentials. This does not mean we are
not all Christians, but that we have differences of opinions on some things.
However, in the 1500s, there was a major split in the Catholic Church. It was called the
Reformation. As a Protestant, I believe the Reformation was necessary because the Catholic Church
had adopted some dangerous and erroneous doctrines that extend far beyond mere differences of
opinion; namely, prayer to Mary, purgatory, indulgences, etc.
Muslims often fail to differentiate between Catholicism and Protestantism in their arguments. I've
heard and read where Muslims attack, for example, the Catholic teaching of the Eucharist where the
bread and wine of the Lord's Supper are claimed by the Catholics to actually become the body and
blood of Christ during the communion service. Of course, we Protestants strongly disagree with this.
But this disagreement is rarely, if ever, brought up when attacking Christian doctrine. So, when
Muslims say "Christians believe ...", be careful that they do not make too broad a generalization as
they continue their attack.

No. 6 Claim their logic is correct and Christian logic is incorrect

Many Muslims have told me that what I am saying about God, Salvation, the Bible, etc., isn't
logical. Now, perhaps some things I say are not logical. But, I've not heard any convincing arguments
yet demonstrating what is and is not logical regarding Christian doctrine. Usually, the Muslim will
simply say that Jesus having two natures is not logical or that the Trinity being three persons doesn't
make sense. But saying so doesn't make it so. There is nothing illogical about a part of God being
able to become a person and add human nature to Himself. It may not be the easiest thing in the
world to understand, but it is not illogical. Neither does saying that the Trinity is three persons is
illogical make it so. Surely anyone would agree that when we encounter God and His self revelation
there will be things that are difficult to understand. The Trinity certainly falls under that category.
But, the Trinity doctrine is not against logic. It would be illogical to say that one God is three gods, or
that one person is three persons. But that is not what Christianity teaches.
I've found that when dialoguing with Muslims and when reading their material against Christianity,
that their claim to know real logic is really an extension of their Muslim thought and not a mastery of
logic at all.

No. 7 Switching topics when challenged

Sometimes when discussing subjects that Muslims find difficult to answer, they will quickly change
the subject. Very often this change involves attacking the Bible. Other times they will testify that they
know Islam is true or they will simply say that you do not known what you are talking about. But when
they change the subject you must be patient. Lovingly bring them back to the subject at hand. I have
had to do this many times when discussing Islam with Muslims.
This is a small but very important point. Too many Christians fall into the trap of allowing
themselves to be diverted from the subject at hand. Do not let a Muslim simply ignore a question and
start a new subject when it gets tough. Likewise, Christians should not simply change the subject
when it gets difficult for them either. Instead, if you do not know the answer to a question, simply
admit it. Go to some research and then get back with them.
Always remember to be gracious. You will not win the Muslim to the Lord with cruelty and
rudeness. And remember that we are in the spiritual battle. Love and truth in the name of Jesus is
more powerful than any perfect answer.

When dialoguing with Muslims, please remember to be respectful and patient. But, check
everything they say and listen to them. They do not have the market cornered on truth, even though
they think they do.
Comparison grid between Christianity and Islamic doctrine

Term Christianity Islam


Christians will be with the Lord in There is an afterlife (75:12) experienced as
heaven (Phil. 1:21-24), in our either an ideal life of Paradise (29:64), for
Afterlife resurrected bodies (1 Cor. 15:50-58). faithful Muslims or Hell for those who are
Non-Christians will be cast into hell not.
forever (Matt. 25:46).
Created beings, non-human, some of Created beings without free will that serve
which, fell into sin and became evil. God. Angels were created from light.
Angels
They are very powerful. The unfallen
angels carry out the will of God.
The sacrifice of Christ on the cross (1 There is no atonement work in Islam other
Pet. 2:24) whereby His blood becomes than a sincere confession of sin and
the sacrifice that turns away the wrath repentance by the sinner.
Atonement of God (1 John 2:2) from the sinner
when the sinner receives (John 1:12),
by faith (Rom. 5:1), the work of Christ
on the cross.
The inspired and inerrant word of God Respected word of the prophets but the
in the original manuscripts (2 Tim. Bible has been corrupted through the
Bible
3:16). centuries and is only correct in so far as it
agrees with the Koran.
The place where Jesus atoned for the Jesus did not die on the cross. Instead, God
sins of the world. It is only through allowed Judas to look like Jesus and he was
Crucifixion
this sacrifice that anyone can be saved crucified instead.
from the wrath of God (1 Pet. 2:24).
A fallen Angel who opposes God in all Iblis, a fallen jinn. Jinn are not angels nor
ways. He also seeks to destroy men, but created beings with free wills.
Devil
humanity (Isaiah 14:12-15; Ezek. Jinn were created from fire, (2:268; 114:1-
28:13-15). 6).
God is a trinity of persons: Father, God is known as Allah. Allah is one person,
Son, and Holy Spirit. The Trinity is not a strict unity. There is no other God in
three gods in one god, nor is it one existence. He is the creator of the universe
God
person who took three forms. (3:191), sovereign over all (6:61-62).
Trinitarianism is strictly monotheistic.
There is no other God in existence.
The place where God dwells. Heaven is Paradise to Muslims, a place of
the eventual home of the Christians unimaginable bliss (32:17), a garden with
Heaven who are saved by God's grace. It is trees and food (13:35;15:45-48) where the
(Paradise) heaven because it is where God is and desires of faithful Muslims are met, (3:133;
Christians will enjoy eternal Fellowship 9:38; 13:35; 39:34; 43:71; 53:13-15).
with Him.
A place of torment in fire out of the Hell is a place of eternal punishment and
presence of God. There is no escape torment (14:17; 25:65; 39:26), in fire
from Hell (Matt. 25:46). (104:6-7) for those who are not Muslims
Hell
(3:131) as well as those who were and
whose works and faith were not sufficient
(14:17; 25:65; 104:6-7).
Third person of the Trinity. The Holy The arch-angel Gabriel who delivered the
Holy Spirit
Spirit is fully God in nature. words of the Koran to Muhammad.
Second person of the Trinity. He is the A very great prophet, second only to
word who became flesh (John 1:1, Muhammad. Jesus is not the son of God
Jesus
14). He is both God and man (Col. (9:30) and certainly is not divine (5:17,
2:9). 75)) and he was not crucified (4:157).
Occurs on the day of resurrection (John Occurs on the day of resurrection where
12;48) where God will judge all God will judge all people. Muslims go to
Judgment
people. Christians go to heaven. All paradise (3:142, 183-185, 198). All others
Day
others to hell (Matt. 25:46). to hell (3:196-197). Judgment is based on
a person's deeds (5:9; 42:26; 8:29).
The work of Muhammad. It is not The final revelation of God to all of mankind
inspired, nor is it scripture. There is no given through the archangel Gabriel to
Koran, The verification for its accurate Muhammad over a 23 year period. It is
transmission from the originals. without error and guarded from error by
Allah.
Made in the image of God (Gen. 1:26). Not made in the image of God (42:11).
This does not mean that God has a Man is made out of the dust of the earth
Man
body, but that man is made like God in (23:12) and Allah breathed life into man
abilities (reason, faith, love, etc.). (32:9; 15:29).
A non-inspired man born in 570 in The last and greatest of all prophets of Allah
Muhammad Mecca who started the Islamic religion. whose Qur'an is the greatest of all inspired
books.
This is a term used to describe the There is no original sin. All people are
effect of Adam's sin on his descendants sinless until they rebel against God. They
(Rom. 5:12-23). Specifically, it is our do not have sinful natures.
inheritance of a sinful nature from
Original Sin
Adam. The sinful nature originated with
Adam and is passed down from parent
to child. We are by nature children of
wrath (Eph. 2:3).
Bodily resurrection of all people, non- Bodily resurrection, some to heaven, some
Christians to damnation and Christians to hell (3:77; 15:25;75:36-40; 22:6).
Resurrection
are resurrected to eternal life (1 Cor.
15:50-58).
A free gift of God (Eph. 2:8-9) to the Forgiveness of sins is obtained by Allah's
person who trusts in Christ and His grace without a mediator. The Muslim must
sacrifice on the cross. He is our believe Allah exists, believe in the
Salvation mediator (1 Tim. 2:5). No works are fundamental doctrines of Islam, believe that
sufficient in any way to merit salvation Muhammad is his prophet, and follow the
since our works are all unacceptable to commands of Allah given in the Koran.
God (Isaiah 64:6).
A term used to designate that Jesus is A literal son of God. Therefore, Jesus
Son of God divine though he is not the literal son cannot be the son of Allah.
of God in a physical sense (John 5:18).
"In the beginning was the word and the Allah's command of existence which
word was with God and the word was resulted in Jesus being formed in the womb
Word, The
God...and the word became flesh and of Mary.
dwelt among us..." (John 1:1, 14).

Does Islam teach salvation by works?


Generally speaking, there are only two methods of salvation in all the religions of the world:
grace and works. Christianity is a religion of salvation by grace alone: "For by grace through faith
you have been saved, not of works..." (Eph. 2:8-9). All other systems rely totally or in part on the
works of the believer to merit salvation. Mormons, for example, say that you are saved by grace
through faith after all you can do. In Roman Catholicism, God's grace is infused into a believer that
enables him to do good works by which he is judged for salvation. In Islam, forgiveness is based on a
combination of Allah's grace and the Muslim's works. On the Day of Judgment, if a Muslim's good
works outweigh his bad ones, and if Allah so wills it, he may be forgiven of all his sins and then enter
into Paradise. Therefore, Islam is a religion of salvation by works because it combines man's works
with Allah's grace. Consider the following verses from the Qur'an. (All quotes from the Qur'an are
from this version unless otherwise specified.)

1. "To those who believe and do deeds of righteousness hath Allah promised forgiveness
and a great reward" (Surah 5:9).
2. "And He answers those who believe and do good deeds, and gives them more out of His
grace; and (as for) the unbelievers, they shall have a severe punishment," (42:26, online,
trans. by M.H. Shakir).
3. "O you who believe! If you are careful of (your duty to) Allah, He will grant you a
distinction and do away with your evils and forgive you; and Allah is the Lord of mighty
grace," (8:29, online, trans. by M.H. Shakir).

Of course, the Muslims will tell us the Qur'an teaches that Allah is gracious to them and that they
do not earn their forgiveness. I acknowledge this. The Qur'an says, ". . . Allah is the Lord of grace
unbounded," (8:29), and also, ". . . But Allah will choose for his special mercy whom he will - for Allah
is lord of grace abounding," (2:105). But, as you can see from the quotes 1, 2, and 3 above, Allah's
forgiveness is tied to the Muslim's works. Therefore, I ask the question, how is it really grace if it is by
also by works? Isn't grace the unmerited favor from God? It would seem the Islamic system of
salvation is more a reward than grace.
For further confirmation that Allah's grace is dependent upon the deeds of faithful Muslims, here
are more quotes from the Qur'an:

4. "O you who believe! be careful of (your duty to) Allah and speak the right word, He will
put your deeds into a right state for you, and forgive you your faults; and whoever obeys
Allah and His Apostle, he indeed achieves a mighty success," (33:70-71, online, trans. by
M.H. Shakir).
5. ". . . But if ye obey Allah and his messenger, he will not belittle aught of your deeds:
for Allah is Oft-Forgiving, Most Mmerciful," (49:14).
 "If you obey GOD and His messenger, He will not put any of your works to waste. GOD
is Forgiver, Most Merciful." (49:14).

Notice how the Qur'an teaches forgiveness based upon Allah's grace and man's works. Can any
Muslim be assured of his salvation before his God? No.185 Numerous Muslims have told me that they
do not know if they will make it to heaven because they do not know if their good deeds outweigh their
bad ones. Unlike Christianity where we have assurance of salvation (1 John 5:13), there is no
assurance in Islam because it rests in part on the obedience and good works of Muslims. Unlike
Christianity where salvation is an unearned, free gift from God (Rom. 4:3; Eph. 2:89), the Muslim can
at best only hope he has performed enough good works to outweigh his bad ones and that Allah so
wills to forgive him.

Another requirement for forgiveness for the Muslim is sincerity when repenting of sins.

185
There are many Muslims who believe that if a Muslim dies in battle defending Allah, that he is guaranteed to go
to heaven.
6. "O ye who believe! Turn unto Allah in sincere repentance! It may be that your Lord will
remit from you your evil deeds and bring you into Gardens underneath which rivers flow, on
the day when Allah will not abase the Prophet and those who believe with him. Their light
will run before them and on their right hands; they will say: Our Lord! Perfect our light for
us, and forgive us! Lo! Thou art Able to do all things," (66:8-9).
7. "God accepts the repentance of those who have sinned in ignorance and who realizing
the ugliness of their deed swiftly turn toward Him in repentance" (3:16).

I am not saying that we should not be sincere when repenting of our sins. But, the problem with
sincerity is that it can easily become a form of pride. After all, if a person says he is truly sincere
enough to be forgiven of his sins, then isn't he appealing to something within himself, a finite sinner,
as part of the basis of receiving forgiveness from a holy and infinite God? Is that not presumptive and
prideful to do? Furthermore, the issue of sincerity is a subjective thing. How do you know you are
being sincere enough? Is it because the Muslims simply believes he is? It seems to me that at best,
the Muslim can only hope he is sincere enough. But how can he really know for sure. Instead, the
Bible says that the heart is desperately wicked and deceitful and cannot be trusted ( Jer. 17:9).
In Christianity, we appeal to the work of Christ on the cross (1 Pet. 2:24) completely and totally
and in nothing in ourselves as a basis for forgiveness, because no good thing dwells within us ( Rom.
7:18); that is, apart from Christ. We sincerely believe in Christ, but we never claim that forgiveness is
in anyway merited or gained because of our sincerity or our works. Rather, our forgiveness is based
on faith and trust in God in what He has done for us in Christ. Salvation in Christianity is God
centered. In Islam, forgiveness of sins is man-centered in that it is dependent upon man's sincerity
and man's works in combination with Allah's forgiveness.
Both Christianity and Islam teach that we must have faith in God. But in Christianity, this faith in
God is enough to save us (Rom. 5:1; Eph. 2:89). In Islam, faith in God it is not. In Islam, the
Muslim's works will be weighed on the Day of Judgment and it will then be decided who is saved and
who is not -- based upon whether the person was a Muslim, whether or not they were sincere in
repentance, and whether or not they performed enough good works to out weigh the bad ones.
Please consider the following verses from the Qur'an about how a Muslim's deeds are weighed in
the balance on Judgment Day to see if he might be saved or not.

8. "Then those whose balance (of good deeds ) is heavy, they will be successful. But those
whose balance is light, will be those who have lost their souls; in hell will they abide,"
(23:102-103).
9. "And We set a just balance for the Day of Resurrection so that no soul is wronged in aught.
Though it be of the weight of a grain of mustard seed, We bring it. And We suffice for
reckoners," (21:47).
10. "They are those who deny the Signs of their Lord and the fact of their having to meet Him
(in the Hereafter): vain will be their works, nor shall We, on the Day of Judgment, give
them any weight," online Qur'an, 18:105

Is the Islamic system of salvation really enough to save Muslims? They will say that it is. But, as a
Christian, I cannot see how anyone in Islam can have security and honest expectation of obtaining
Paradise. How can anyone who must be completely sincere in repentance and be required to perform
more good works than bad, ever hope to make it to heaven?
The problem with being saved by God's grace and human works is that human works are never
sufficient to please God. God is infinite and holy. How can we finite sinners ever hope to please God
by our deeds?

Muslims need the gospel


Instead of relying in any way on our own works, the gospel of Jesus teaches us we do not have to
do that. The gospel of Jesus is that He died for our sins and rose again from the dead (1 Cor. 15:1-
4). He fulfilled all the Law so we don't have to (Rom. 8:3-4). He took our place and received the
punishment due our sins (2 Cor. 5:21). Because we are sinners and because we cannot please an
infinitely Holy God on our own, because we can never fulfill the Law of God perfectly, and because
God's eyes are too pure to look upon evil (Hab. 1:13), salvation must be by total grace (Eph. 2:8).
Salvation must be the work of God, not of man (Gal. 2:21).
1 John 5:13 says, "These things were written so you may know you have eternal life..." Can the
Muslim say he knows he has eternal life? He cannot. I know I do as a Christian precisely because it is
not of my works. So, why would a Muslim want me to give up my assurance and free gift of salvation
found in Jesus for the Islamic system of works that, at best, only provides the possibility of salvation if
I have been sincere enough and if I have done enough good works?
Questions for Muslims

Dear Muslim, I do not post these questions as a "proof" that Islam is false. I do not believe that is
possible with a simple list of questions. Nevertheless, they are here to encourage discussion that the
truth may be known.

1. The Qur'an says "To those who believe and do deeds of righteousness hath Allah promised
forgiveness and a great reward" (Surah 5:9).
A. Question: Are you doing enough good deeds to receive salvation on the Day of Judgment?
B. Question: Are you doing all you can or are you relaxing in your dedication to Allah?
2. The Qur'an says, "O ye who believe! Turn unto Allah in sincere repentance! It may be that your
Lord will remit from you your evil deeds and bring you into Gardens underneath which rivers flow,
on the day when Allah will not abase the Prophet and those who believe with him. Their light will
run before them and on their right hands; they will say: Our Lord! Perfect our light for us, and
forgive us! Lo! Thou art Able to do all things," (66:8-9). Notice how it says if you are sincere you
may receive forgiveness.
A. Question: How do you know you are sincere enough to be forgiven of Allah?
B. Question: Does it give you peace to know that even if you are very sincere that at best, you
may receive forgiveness?
C. Question: If you say that you know you are sincere enough in your repentance before Allah,
how do you know you are not deceiving yourself?
D. Question: Is your heart really good enough to muster enough sincerity before a Holy and
Righteous God?
E. Question: If you say yes, I honestly and humbly ask you, "Are you being prideful?"
F. Question: If you say you are not being prideful, then are you boasting in your sincerity?
3. In Christianity, Jesus is God in flesh who paid for our sins on the cross (1 Pet. 2:24). Because of
that, we Christians are secure in Him and do not have to worry about doing enough good works
to please God since we are saved by grace through faith in Him, (Eph. 2:8-9).
A. Question: Why should we Christians give up our guarantee of salvation in Jesus for the
requirements of your Qur'anic law when you yourselves don't even know if you have done
enough good deeds to be saved on the Day of Judgment?
4. The Bible says that God is love (1 John 4:16) and that He loves all people (Matt. 5:43-48; John
3:16). The Qur'an never says that "God is love." In fact, the Qur'an says that Allah does not love
unbelievers (2:98; 3:32).
A. Question: If Allah does not love unbelievers, can you say that Allah is love, especially if the
Qur'an does not say it?
B. Question: If you say yes, that Allah is love, then why does he only love the Muslims and not
all people?
C. Question: If you say Allah is love, is he more loving that the God of the Bible who loves all
people?
5. In the Bible, Jesus said in John 15:13, "Greater love hath no man than this, that a man lay down
his life for his friends." In Christianity, the greatest act of love is performed by God Himself --
since Jesus is God in flesh (John 1:1, 14; Col. 2:9). Jesus is the one who fulfilled His own words
on this. He laid His life down for us.
A. Question: What is the greatest act of love performed by Allah?
B. Question: If what Jesus said is true, then hasn't someone besides Allah has performed the
greatest act of love?
C. Question: Why do you, as a Muslim, want me to give up such a great love performed by
God Himself (from a Christian perspective) for your belief in Allah who only loves people if
they are Muslims?
6. Islam teaches that the Holy Spirit is Gabriel. In the Bible, the Holy Spirit lives in the Christians.
A. Question: If the angel Gabriel is the Holy Spirit, how can he dwell in us? (Note: According
to the Nestle Aland Greek New Testament Textual Apparatus, there are no textual variations
any of the following biblical references. They are recorded and transmitted to us
accurately.)
i. "Guard, through the Holy Spirit who dwells in us, the treasure which has been entrusted
to you," (2 Tim. 1:14, NASB).
ii. "Do you not know that you are a temple of God, and that the Spirit of God dwells in
you?" (1 Cor. 3:16, NASB).
More questions for Muslims
Dear Muslim, I do not post these questions as a "proof" that Islam is false. I do not believe that is
possible with a simple list of questions. Nevertheless, they are here to encourage discussion that the
truth may be known.

1. The Qur'an states that you shall marry only up to four women: "If ye fear that ye shall not be
able to deal justly with orphans, marry women of your choice, two, or threee, or four; but if ye
fear that ye shall not be able to deal justly (with them), then only one, or that which your right
hands possess..." (4:3).
A. If the Qur'an is eternal, having been written on the table in heaven, then the four wife limit
was an eternal decree.
B. Question: Why did Muhammad have 12 wives if the Qur'an says to have only four? Khadija,
sawda, Aesah (8 years old), Omm Salama, Hafsa, Zaynab (originally the wife of
Muhammad's adopted son), Jowayriya, Omm Habiba, Safiya, Maymuna, Fatima, Hend, Asma
(of Saba), Zaynab, Habla, Asma?
2. The Qur'an says that Allah created the heavens and earth in six days. "Your Guardian Lord is
Allah, who created the heavens and the earth in six days..." (7:54) See also, 10:3.
A. 41:9 - "Say: Is it that ye deny him who created the earth in two days..."
i. Question: Which should we believe, the Bible or the Qur'an? The Bible says God
created the earth in one day (Gen. 1,2).
B. 41:10 - "He set on the (earth), mountains standing firm, high above it, and bestowed
blessings on the earth, and measured therein its sustenance in four days..."
i. Question: Does 41:10 included the 2 days of 41:9 above? If so, why does 41:9 say
God created the earth in two days, but 41:10 says that God measured its sustenance in
four days? They are different occurrences.
C. 41:11 - "Then he turned to the sky, and it had been (as) smoke: he said to it and to the
earth: 'Come ye together, willingly or unwillingly..."
D. 41:12 - "So he completed them as seven firmaments in two days, and he assigned to
heaven its duty and command..."
E. Question: the two days of 41:9 plus the four days of 41:10 and the 2 days of 41:12 equals
eight days of creation, not six. Why is that?
3. It is a commonly held belief among Muslims that Muhammad was sinless.
A. If Muhammad was sinless, then why does the Qur'an state: "Patiently, then, persevere: for
the promise of Allah is true: and ask forgiveness for thy fault, and celebrate the praises of
they Lord in the evening and the morning." (40:55).
i. Pickthall translates 40:55 thus: "Then have patience (O Muhammad). Lo! the promise
of Allah is true. And ask forgiveness of thy sin, and hymn the praise of thy Lord at fall of
night and in the early hours."
B. The Hadith says, "I heard Allah's Apostle saying." By Allah! I ask for forgiveness from Allah
and turn to Him in repentance more than seventy times a day." Volume 8, Book 75, Number
319, Narrated Abu Huraira:
C. Question: If it is the common belief that Muhammad was sinless, was do the Qur'an and
Hadith contradict that notion? Which is correct?
4. The Qur'an says to have no divisions within Islam. "The same religion has He established for you
as that which He enjoined on Noah - the [sic] which we have sent by inspiration to thee - and
that which we enjoined on Abraham, Moses, and Jesus: namely, that ye should remain steadfast
in religion and make no divisions therein: to those who worship other things than Allah, hard is
the (way) to which thou callest them..." (42:13).
A. Question: If Islam is the truth, which of the divisions within it is the "most" true?
B. Question: Is Islam in a state of sin since there are divisions within its ranks?
5. The Qur'an says, "And dispute ye not with the People of the Book [Jews and Christians and the
Bible], except in the best way, unless it be with those of them who do wrong but say, "We believe
in the revelation which has come down to us and in that which came down to you," (29:46).
A. Question: Isn't the Qur'an here saying that the Muslim is to believe what the Bible says?
Differences between the Bible and the Qur'an

Bible Qur'an
Monotheistic, Trinitarian, (Isaiah 43:10; 44:6-8; Monotheistic (5:73; 112:1-4), denies the Trinity
Matt. 28:19; 2 Cor. 13:14) (5:73).
Jesus is God in flesh (Col. 2:9) Jesus is not God, (5:17, 75)
Jesus was crucified (1 Pet. 2:24). Jesus was not Crucified, (4:157).
Jesus rose from the dead (John 2:19-20). Jesus did not rise from the dead.
Jesus was the Son of God (Mark 1:1). Jesus was not the Son of God ( 9:30)

Holy Spirit, 3rd person in the Godhead. He will


The Holy Spirit is the angel Gabriel (2:97; 16:102).
bear witness of Jesus (John 14:26; 15:26).

Salvation by sincerity and works (3:135; 7:8-9;


Salvation by grace through faith (Eph. 2:89).
21:47; 49:14; 66:8-9).
The Devil, Satan, is not a fallen angel, but a fallen
The Devil is a fallen angel (Isaiah 14:12-15).
Jinn (2:34; 7:12; 15:27; 55:15).
Man is fallen, a sinner (Rom. 3:23). Man is basically good.

Disciples were Christians (Acts 11:26). Disciples declare themselves Muslims, (5:111).

Worship on Sabbath (Exodus 20) then later on


Worship on Friday (62:9).
Sunday (Rom. 14:5-6; Acts 20:7; 1 Cor. 16:1-2).
Miracles, numerous are recorded. No Miracles recorded.
Makes numerous prophecies Makes no prophecies.
The Qu'ran says the Bible is not corrupt

The Muslims repeatedly claim that the Bible has been corrupted and that the Qu'ran is the only
trustworthy scripture in existence. This is why Muslims often attack the Bible. But this cannot be
according to the Quran. The Quran186 says that the books of Moses, the Psalms, and the gospel were
all given by God.

• TORAH - "We gave Moses the Book and followed him up with a succession of messengers,"
(Sura 2:87).
• PSALMS - "We have sent thee inspiration, as We sent it to Noah and the Messengers after him:
we sent inspiration to Abraham, Isma'il, Isaac, Jacob and the Tribes, to Jesus, Job, Jonah,
Aaron, and Solomon, and to David We gave the Psalms," (4:163).
• GOSPEL - "It is He Who sent down to thee (step by step), in truth, the Book, confirming what
went before it; and He sent down the Law (of Moses) and the Gospel (of Jesus) before this, as a
guide to mankind, and He sent down the criterion (of judgment between right and wrong),"
(3:3).
Also, "And in their footsteps We sent Jesus the son of Mary, confirming the Law that had
come before him: We sent him the Gospel: therein was guidance and light, and confirmation of
the Law that had come before him: a guidance and an admonition to those who fear Allah,"
(5:46).

We see that the Qu'ran states that the Torah, the Psalms, and the Gospel were all given by God.
With this we Christians heartily agree. But, the Muslims claim that the Bible is corrupted and full of
contradictions. If that is so, then it would seem they do not believe the Qu'ran since the Qu'ran says
that the Word of God cannot be altered:

• "Rejected were the messengers before thee: with patience and constancy they bore their
rejection and their wrongs, until Our aid did reach them: there is none that can alter the words
(and decrees) of Allah. Already hast thou received some account of those messengers," (6:34).
• "The word of thy Lord doth find its fulfillment in truth and in justice: None can change His
words: for He is the one who heareth and knoweth all," (6:115).
• "For them are glad tidings, in the life of the present and in the Hereafter; no change can there
be in the words of Allah. This is indeed the supreme felicity," (10:64).

When Muhammad (570 - 632) was alive, he claimed to receive the revelation of the Qu'ran from
Allah. This means that at that time, the Bible which was in existence could not have been corrupted
because the Qu'ran states that God's word cannot be corrupted. The question I have for the Muslims
is "When and where was the Bible corrupted, since the Qu'ran says that the Torah, the Psalms, and the
Gospel are from Allah and Allah's words cannot be changed?"

186
The Quranic quotes used in this paper can be found online at http://www.usc.edu/dept/MSA/quran/.
Who has performed the greatest act of love? Yahweh or Allah
In Islam, what is the greatest act of love Allah has ever accomplished? I asked this question of
several Muslims and I got similar answers: he forgave us of our sins; he gave us families and
provisions; he showed us mercy; he gave us the Qur'an. The answers didn't vary much beyond these
responses. I found them lacking.
Most Muslims believe that the Bible is not trustworthy, that it has been corrupted, that the Injeel
(gospel) of Jesus has been lost, and the Qur'an restores God's truth to mankind. But, that is another
subject to be debated.
In John 15:13 Jesus said, “Greater love has no man than this, that he lay his life down for his
friend.” I have the Nestle Aland Greek New Testament with the textual apparatus is included in it.
The textual apparatus is the complete listing (per verse) of any textual variants that occur in any of
the ancient New Testament manuscripts. Therefore, it is a very easy thing to go to John 15:13 and
look at the textual evidence to see if there are any manuscripts at all, anywhere that have any
variation on that verse. There are none. In other words, there is not a single manuscript of the more
than 25,000 manuscripts of the NT that have any derivation on that verse. Every single one of them
says the exact same thing. I will, therefore, conclude that it is an authentic and reliable saying of
Jesus.
Again, Jesus said, “Greater love has no man than this, that he lay his life down for his friend.”
According to Jesus, the greatest act of love is to sacrifice one’s life for another. This means that giving
one’s life for another is a greater act of love than providing food for him, giving him a family, being
nice to him, or honest, or helpful, or whatever. Self-sacrifice, to the point of death, is the very
greatest act of love.
Has Allah performed the greatest act of love? The answer is no. Allah has not sacrificed himself at
all. Allah has not died for another. Allah has not loved us to the point of death.
In Christianity, Jesus, who is God in flesh (John 1:1, 14), laid His life down for us. Jesus performed
the greatest act of love.

If Islam is true...

If Islam is true and Allah is the true God, then Jesus, a creation (according to Islam) has performed
a greater act of love than Allah (according to the Bible). A mere man has outdone Allah in love and
sacrifice. But, of course, Islam denies that Jesus ever died. They then say that Jesus has not done
the greatest act of love. Their denial does not change the fact that Jesus died on the cross as is amply
attested to by the eyewitnesses who wrote the gospel. Besides, whether or not the Muslim believes
Jesus died on the cross does not change the fact that Jesus told us what the greatest act of love was --
and Allah has not done it. Yet, according to Christianity He has. Since Muslims want Christians to
adopt Islam, they are asking us Christian to give up our Lord who has performed the greatest act of
love on our behalf. Why would we want to do that?
If Christianity is true, then God has performed the greatest act of love. If Islam is true, then God
hasn't. Which "god," then is more loving, the one who speaks of love, or the one who acts out love?
I have found no where in the Qur'an where it says that Allah is love. The Qur'an says that Allah
loves people, but it never says that Allah is love. By contrast, the Bible clearly tells us that God is
love. "And we have known and believed the love that God hath to us. God is love; and he that
dwelleth in love dwelleth in God, and God in him," (1 John 4:16).
Who does God love?

Does God love all? In Islam, the answer is no. In Christianity, the answer is yes. Consider the
following verses from the Qur'an.

• "Whoever is an enemy to Allah and His angels and messengers, to Gabriel and Michael,- Lo!
Allah is an enemy to those who reject Faith," (2:98, Trans. Yusuf Ali)
• "Say: Obey Allah and the Messenger; but if they turn back, then surely Allah does not love the
unbelievers," 3:32, Trans. Shakir).

Consider the following verses from the Bible

• "For God so loved the world, that He gave His only begotten Son, that whoever believes in Him
should not perish, but have eternal life," (John 3:16).
• "You have heard that it was said, ‘You shall love your neighbor, and hate your enemy.’ 44 "But I
say to you, love your enemies, and pray for those who persecute you 45 in order that you may
be sons of your Father who is in heaven; for He causes His sun to rise on the evil and the good,
and sends rain on the righteous and the unrighteous. 46 "For if you love those who love you,
what reward have you? Do not even the tax-gatherers do the same? 47 "And if you greet your
brothers only, what do you do more than others? Do not even the Gentiles do the same? 48
"Therefore you are to be perfect, as your heavenly Father is perfect. (Matt. 5:43-48).

We can easily see the huge difference between the God of Islam and the God of the Bible. In
Islam, God does not love all people. In the Bible, God does love all people. In Islam, Allah did not die
for the sins of those who were not his. In the Bible, God did do that. In Islam, Allah has not
performed the greatest act of love. In the Bible, God did exactly that.
My question to the Muslims is, "What makes you think that I want to give up my Lord who loves
me so much that He would die for me, and did, for a god who has not and cannot perform the greatest
act of love?
Jihad: holy struggle or holy war?
Among most Westerners, the term "Jihad" ("struggle" in Arabic) often brings up images of Muslim
terrorists killing people who disagree with them. Jihad is an emotionally charged word that is heralded
by the Western news media in descriptions of Middle East activities. People need not wait long to hear
the term used during nightly news and see the affects of present day Islamic struggles in vivid pictures
of destruction beamed to our televisions. But is this a fair assessment of the Muslim community as a
whole?
Jihad has been interpreted by Muslims in different ways. The Muslim sect of the Kharijites has
elevated Jihad to one of the Five Pillars of Islam -- making it Six Pillars. This kind of belief is seen in
the extremist Muslim groups we call terrorists. They use the concept of Jihad as a justification for
killing anyone who isn't a Muslim. However, most Muslims disagree with this extremist position of
some Muslims and advocate peace. These Muslims view Jihad as a spiritual struggle against evil in a
metaphorical sense.187
For the most part, there is the Greater and Lesser Jihad. The Greater Jihad is the internal spiritual
struggle of the Muslim toward submission to Allah. The Lesser Jihad is Holy War against non-Muslims
based on principle of belief. It is this latter that has caused the most concern among Westerners. Is
that concern warranted? Many think so.

Islamic scholar Jamal Badawi, chairman of the Islamic Information Foundation in Halifax, insists
that a jihad is `permitted only in self-defense or against tyranny and oppression--not as a tool to
promote Islam.'' But, experts added, the ancient Islamic empires were built as much by force as by
persuasion. Islam's founder, Mohammed, frequently used force, or the threat of it, to unify the
nomadic tribes of the Arabian peninsula. The caliphs, who succeeded Mohammed as leaders of the
Arab world, successfully took up arms against the Christian Byzantine Empire in Egypt and the Holy
Land. By the end of the ninth century, Arabian armies had extended Islamic power from Spain to the
borders of India.188

Anyone who has studied Islamic history must surely notice how frequently the Muslims were
involved in battle after battle. Within 200 years after its inception, Islam had spread through a huge
geographical area and many converts were made by the sword.

What does the Qur'an say about Jihad?

The Qur'an is the single most important authority in all of Islam. It is the scripture given from
Allah through the angel Gabriel. Does the Qur'an teach Jihad? Absolutely yes. As you will see in the
following quotes from the Qur'an, Holy War is definitely taught and encouraged.

1. "Truly Allah loves those who fight in His Cause in battle array, as if they were a solid cemented
structure," (Surah 61:4).
2. "Fight in the cause of Allah those who fight you but do not transgress limits...191And slay them
wherever ye catch them. and turn them out from where they have turned you out; for
persecution is worse than slaughter; But fight them not at the sacred Mosque unless they (first)
fight you there; But if they fight you, slay them. Such is the reward of those who reject faith.
192
But if they cease, Allah is oft-forgiving, Most Merciful. 193And fight them on until there is no
more persecution. And the religion becomes Allah's. But if they cease, Let there be no hostility
except to those who practice oppression" (The Qur'an, Surah 2:190-193).
3. "O ye who believe! what is the matter with you, that, when ye are asked to go forth in the
cause of Allah, ye cling heavily to the earth? Do ye prefer the life of this world to the Hereafter?
But little is the comfort of this life, as compared with the Hereafter. Unless ye go forth, He will
punish you with a grievous penalty, and put others in your place; but Him ye would not harm in
the least. For Allah hath power over all things. Unless ye go forth, He will punish you with a
187
Sukhvinder Stubbs, The New Stateman, LTD, "The hooded hordes of prejudice: to typecast all Muslims as
fanatical militants is unfair and offensive," Feb 28, 1997
188
James Deacon and Diane Brady, "The will to fight--and die," Maclean's, 2/11/91, Vol. 104 Issue 6, p 39.
grievous penalty, and put others in your place; but Him ye would not harm in the least. For
Allah hath power over all things," (Surah 9:38-39).
4. See also Surah 4:74-76; 61:10-12.

What does the Hadith say about Jihad?

The Hadith are the recorded sayings and deeds of the Prophet Muhammad. It is second in authority
only to the Qur'an and is often used to clarify things not specified in the Qur'an. What did Muhammad
say about Jihad as recorded in the Hadith?

5. "The Prophet said, "The person who participates in (Holy battles) in Allah's cause and nothing
compels him to do so except belief in Allah and His Apostles, will be recompensed by Allah
either with a reward, or booty (if he survives) or will be admitted to Paradise (if he is killed in
the battle as a martyr). Had I not found it difficult for my followers, then I would not remain
behind any sariya going for Jihad and I would have loved to be martyred in Allah's cause and
then made alive, and then martyred and then made alive, and then again martyred in His
cause."Volume 1, Book 2, Number 35, Narrated Abu Huraira
6. "Allah's Apostle said, "A pious slave gets a double reward." Abu Huraira added: By Him in
Whose Hands my soul is but for Jihad (i.e. holy battles), Hajj, and my duty to serve my mother,
I would have loved to die as a slave. Volume 3, Book 46, Number 724: Narrated Abu Huraira
7. "Allah's Apostle said, "Allah guarantees (the person who carries out Jihad in His Cause and
nothing compelled him to go out but Jihad in His Cause and the belief in His Word) that He will
either admit him into Paradise (Martyrdom) or return him with reward or booty he has earned
to his residence from where he went out." Volume 9, Book 93, Number 555: Narrated Abu
Huraira.

Obviously Muhammad taught that Holy War was an acceptable and good thing to do. To clarify, he
even stated that if a Muslim were to die in battle, fighting for the cause of Allah, that he would be
guaranteed to go to Paradise.

Why is this important?

Why is understanding the Islamic position of Jihad important? Simple. People act according to their
beliefs. If a large group of people believes that war against "unbelievers" is a holy thing, that it is a
thing sanctioned from God, then those who are not Muslims should be concerned. Of course, at this
point, most Muslims might accuse me of being sensationalistic and pointing to only a few extremists
and out-of-context verses to make Islam look bad. First, let me say that by far the majority of
Muslims I have encountered here in the United States have been polite and peace loving. Second, in
other parts of the world, Jihad is taken to extremes not simply by terrorists, but by Islamic led
governments.

In Egypt, a Muslim country, Christians have been persecuted heavily for their faith and only recently
are things beginning to change. 189

"Roman Catholic Bishop John Joseph of Pakistan shot himself to death on May 6 to highlight the case
of Ayub Massih, a Christian sentenced to death for supposedly making blasphemous remarks against
the Prophet Muhammad and thus against Islam. In a letter sent to a local newspaper just before his
death, the bishop stated that he hoped his suicide would galvanize his fellow bishops and others to
work for the repeal of sections 295 B and Cot the Pakistan Penal Code (PPC), which make any
blasphemy against Islam a serious crime and blasphemy against Muhammad punishable by death." 190

"Farag Foda, an Egyptian intellectual who expressed scorn for the Islamist program, was shot and
murdered. And Naguib Mahfouz, the elderly and much-celebrated Nobel Prize laureate for literature,
189
Saad Michael Saad, The Christian Century, "A Christian appeal to Islam. Treatment of Copts in Egypt," by Feb
23, 2000.
190
Cris E. Toffolo, The Christian Century, "Christians in Pakistan confront charges of blasphemy," July 29, 1998.
was seriously injured in Cairo when an assailant knifed him in the neck, presumably in revenge for an
allegorical novel written decades earlier." 191

"Hundreds of thousands of Muslims assembled in Jakarta and declared a holy war against Indonesian
Christians shortly after dawn Friday to avenge the deaths of Muslims in religious clashes in the Maluku
Islands (the Spice Islands). Assembled in central Jakarta, the Muslims shouted "Jihad (Holy War)!
Jihad!" Most of those gathered wore white robes and white bandannas marked with quotes from the
Koran about the "Holy War." It is time for us to do a jihad against Christians," said Husen al-Habsyi, a
former political prisoner, who was jailed for masterminding an explosion in the Borobudur Buddhist
Temple in the early 1980s.192

This Christmas season, Pastor Rod Parsley is taking a lead in an effort to help free the tens of
thousands of Sudanese women and children held in captivity in the Sudan, many of which are
Christians. This great effort will help stem the horrific tide of genocide and enslavement of Christians in
the African nation. Bridge of Hope, the missions outreach of Breakthrough is located in Columbus,
Ohio...The Government of Sudan, a fundamentalist regime that represents only 10% of the population
in Sudan, has declared a holy war (jihad) against Christians and animists in their own country. Since
1985, this reign of terror is responsible for the murder of over 2 million Sudanese ... and over 4 million
have been displaced. The Government of Sudan will not stop short of total annihilation of all Christians
and all others that do not believe in this totalitarian regime...During Government sponsored raids in
peaceful villages, men are killed; village elders are hacked with machetes and left for dead; the village
is burned and devastated; and women and children are captured as slaves. Slaves are subjected by
their masters to systematic physical and psychological torture, including gang rape, beatings, death
threats, genital mutilation and forcible conversion to Islam. 193

Anyone can make any group look bad through selective quotes. Each religious group has elements
of its history it wish it could ignore. The Muslims could cite the Crusades or the Inquisition as
examples of "Christian behavior." In response, the Crusades, right or wrong, were a retaliation
against the Islamic Jihad that was sweeping through Europe. The Inquisition, on the other hand, is a
perfect example of what happens when a religious group (the Roman Catholic Church) gets in power
and tries to root out heretics and blasphemers. Islam is no different.
The Islamic run country of Pakistan (No. 9 above) has anti blasphemy laws where the punishment
for speaking blasphemy against Muhammad and the Qur'an is death. Islamic run Sudan has already
killed, and still is killing, millions of people, mainly Christians, in its own country in addition to making
many of them slaves. (No. 12 above). It is these kinds of facts that cannot be ignored and should not
be ignored. Muslim and Christian alike should be very concerned.
I do not know if other Muslim countries are condemning the actions of these Islamic nations that so
easily violate human rights. I do not know if Muslims outside of those countries are even aware of the
problems going on within their theologically diverse ranks of other nations. But, when a Holy Book like
the Qur'an advocates Holy War, when the very sayings and deeds of their beloved Prophet Muhammad
advocate Jihad, and when we see some Islamic nations killing non Muslims -- because they are not
Muslims, how can we not be worried about what they would do if they had control of the world....as is
their goal.
Here in the States, Muslims enjoy freedom of religion and expression. Such freedoms for
Christians are basically non existent in many Islamic nations. Is that right?

191
Daniel Pipes, The American Jewish Community, "How Dare You Defame Islam,". Nov, 1999.
192
Asian Political News, Jan 10, 2000
193
(PR Newswire, "Breakthrough Bridge of Hope Missions to Partner With Christian Solidarity International to Free
Slaves in Sudan." Issue: Nov 30, 2000.
Is the Trinity possible?
Muslims deny the Trinity doctrine of one God in three persons. They are so strict in their
monotheism that no plurality within God can possibly exist. Many of them erringly assert that the
Trinity is really a deceptive doctrine of three gods. Of course, it is not, but that does not stop them
from claiming it is not rational.
Muslims often appeal to logic as a verification for their theological systems. They claim that Islam
is true and Christianity false, that the Trinity simply doesn't make sense. In spite of their claims, the
doctrine of the Trinity is not illogical at all. In fact, most people believe in trinities without even
knowing it.
As a painter reveals part of himself, his style, what he is, etc., in his painting, so too, God has
revealed part of Himself, His style, and what He is in His creation. Creation reflects, to a large degree,
the qualities of God Himself. Let's take a look.

1. The universe is ordered; therefore, we know that God is a God of order.


2. The universe operates on laws; therefore, God is a God of law.
3. The universe has a beginning; therefore, God is the creator.
4. In mathematics, there is an infinity of numbers. In the universe there is an infinity of distance;
Therefore, God the creator of the universe, is infinite.
5. Absolute truths exist in creation, (i.e., something can not be both itself and not itself at the same
time); therefore, God is absolute truth.
6. The universe is comprised of three primary aspects: Space, Time, and Matter.
A. Space is comprised of height, width, and depth - a trinity - but each aspect is by nature
space.
B. Time is comprised of past, present, and future - a trinity - but each aspect is by nature time.
C. Matter is comprised of solid, liquid, and gas - a trinity - but each aspect is by nature matter.
7. Therefore, we can conclude from looking at the universe, and God as its creator, that it is possible
for God to have a trinitarian aspect to His nature since we see it in nature.
8. If it is fair to say that God may indeed be trinitarian in some aspect of His nature,
A. then God could easily be trinitarian in nature and still be the one and only God.
B. God could easily be a plurality and all aspects of this plurality, being of God, would be divine
by nature.
9. Since God is self-aware, has a will, can speak, etc., then it follows that the plural aspects of God
could share, in some way, those same qualities.
A. If this is possible, then why cannot part of God, since God is a plurality, become a man and
add human nature to itself?

We can see that there are trinities in nature. So, why can't God be a trinity as well and creation
simply be a reflection, in part, of His greatness? It is certainly possible. If the Trinity were illogical,
then the possibility would not exist. There is no logical reason why the trinity can not be a reality. It
is up to the Muslim to demonstrate a logical contradiction regarding the doctrine of one God in three
persons. Simply stating it isn't logical or that it isn't possible proves nothing at all.
The Bible has declared that God is indeed a Trinity and that Jesus is both God and man (John
1:1, 14; Col. 2:9; etc.). Christians have a living redeemer. Christians have God as their savior.
The Trinity makes no sense. It isn't logical.

Muslims often state that the Trinity doctrine lacks both common sense and logic. Additionally, they
sometimes accuse the Christians of being polytheists by saying that the Trinity teaches three gods.
Other times they state that it is illogical for three gods to be one god and for that reason, the Trinity
can't be true. Their objections need to be addressed. But, before we do, we need to understand what
the Trinity is and what it is not.
The doctrine of the Trinity1 is that there is one God who exists in three persons: Father, Son, and
Holy Spirit. Each person is not the same as the other person; that is, the Father is not the same
person as the Son who is not the same person as the Holy Spirit. Each is fully God in nature. Each
person is not a god in itself. Instead, the totality of all three persons comprises the one God. There
are not three gods, but one. We believe there are no partners with God because we believe there is
only one God in all existence.
At first, some may look at this teaching and be confused by it. How can God be three persons in
one God? This is a good question because it is a bit difficult to grasp. But, that is what we would
expect isn't it, when we encounter God? Would we not expect to find some things about God's Infinite
nature a bit beyond our comprehension? This is not unreasonable. However, we must not make the
mistake of saying something as ridiculous as, "It doesn't make sense. Therefore it is true."
Following is a small chart that helps clarify how the doctrine of the Trinity is arrived at through the
Bible.

TRINITY

FATHER SON HOLY SPIRIT


Called God Phil. 1:2 John 1:1,14; Col. 2:9 Acts 5:3-4
Creator Isaiah 64:8; 44:24 John 1:3; Col. 1:15-17 Job 33:4; 26:13
Indwells 2 Cor. 6:16 Col. 1:27 John 14:7
Everywhere 1 Kings 8:27 Matt. 28:20 Psalm 139:7-10
All knowing 1 John 3:20 John 16:30; 21:17 1 Cor. 2:10-11

The above chart is representative and not complete but it shows that there is scriptural support for
the doctrine. Following is another way of illustrating the Trinity.

The Trinities in Nature

People already believe in trinities. They just don't know they do. Here is how. Basically, the
universe consists of three elements: Time, Space, and Matter. Each of these is comprised of three
'components.'

Time Past Present Future


Space Height Width Depth
Matter Solid Liquid Gas
TIME SPACE MATTER

As the Trinitarian doctrine maintains, each of the persons of the Godhead is distinct, yet they are
all each, by nature, God. The same idea can be presented in the above examples. With time, for
example, the past is not the same as the present, which is not the same as the future. Each is
simultaneous (according to some time theorists). Yet, they are not three 'times,' but one. That is, they
all share the same nature: time.
With space, height is distinct from width, which is not the same as depth, which is is not the same
as height. Yet, they are not three 'spaces,' but one. That is, they all share the same nature: space.
With matter, solid is not the same as liquid, which is not the same as gas, which is not the same
as solid. Yet, they are not three 'matters,' but one. That is, they all share the same nature: matter.
Note that there are three sets of threes. In other words, there is a trinity of trinities. If we were to
look at the universe and notice these qualities within it, is it really so difficult to imagine that God can
be a Trinity of persons? Furthermore, is it fair to say that this "trinity of trinities" are the fingerprints
of God upon His creation? I think so.

"For the invisible things of him from the creation of the world are clearly seen, being understood by
the things that are made, even his eternal power and Godhead; so that they are without excuse,"
(Rom. 1:20).

Of course, there are always people who will say that this is ridiculous. Perhaps it is. But if it is it is
up to them to demonstrate why it cannot be true. If we see the analogy of the Trinity within the
nature, why is it so difficult to believe that God could be the Trinity as well?

Demonstrate that it is illogical

One of the questions I ask the anti-Trinitarians is "Can you please show me how the Trinity is
illogical?". Usually, they respond with something like, "It just doesn't make sense," or "It simply can't
be." But making such statements doesn't prove or disprove anything. The question is, "How is it
illogical?" I have yet to hear a logical explanation.
It isn't against logic for God to be three persons. It may be difficult to understand, and some may
not like it, but it isn't illogical. For it to be illogical, there must be some rule of logic that is violated
that makes it impossible for God to exist as a Trinity. For example, to say that one god is really three
gods is illogical because the quantity of one is not the same quantity as three and since they (one and
three) are mutually exclusive as to quantity in this situation, to say one god is three gods is illogical.
But that isn't what the Trinity is anyway, so this can't be used to demonstrate that the Trinity is
illogical. Furthermore, there is no logical reason why God cannot be three persons.
Trinitarianism is monotheistic. That means that it is a doctrine that teaches the existence of a
single being who is God and that there is only one God in all existence. This is sometimes ignored or
not known when people (Muslims) criticize the Trinity by asserting that the Trinity teaches three gods.
But, it does not. As is stated above, the Trinity is a monotheistic theological position.
When Muslims state that the Trinity isn't logical, it is up to them to demonstrate how. If they are
not able to do it, then they should stop making the claim.
Jesus cannot be God's son

To some Muslims, the term “Son of God” brings up images of a sort of divine being with a goddess
wife who together have somehow produced a child. When Christians use the term in reference to
Jesus, they immediately assume that the Christians are committing blasphemy by stating that God has
participated in some sort of sexual union with another god – a goddess wife.

They say: "the most gracious has betaken a son!" Indeed ye have put forth a thing most monstrous!
At it in the skies are about to burst, the earth to split asunder, and the mountains to fall down in utter
ruin, that they attributed a son to the Most Gracious, for it is not consonant with the majesty of the
Most Gracious that he should beget a son. (The Qur'an, 5:88-92).

This is naturally a ridiculous scenario and is a false assumption. No where in the Bible does it say
that God had relations with anyone to produce a literal son, nor has Christianity taught that God
produced a son through any physical act whatsoever. Such a thing is heretical. Nevertheless, the Bible
in numerous places calls Jesus the Son of God. But, it does not mean that Jesus is the literal offspring
of God.
The Muslims need to as ask what does that term mean, in its historic and biblical context. Instead
of imposing upon the biblical term a meaning that is foreign to it, the Muslim should learn what the
Bible means by the term and think of it in the context as revealed in the Scriptures where it is used.
To not do that would be the same as me taking a term out of the Qur'an, remove it from its Qur'anic
context, and applying another meaning to it and then saying what the Qur'an teaches is false. The
term “Son of God” is used in different senses in the Bible. But, never does it mean that God has a wife
and produces offspring.

Old Testament usage of the term Son of God

The term “son of God” is used in two main ways in the Old Testament. Neither way denotes any
physical relation to God. Rather, the references deal with those who under divine obedience to the call
of God. It is used of Israel as a nation through the Exodus. Hosea 11:1 says, “When Israel was a
youth I loved him, and out of Egypt I called My son.” 1 It is also used in reference to angels. Job 1:6
says, "Now there was a day when the sons of God came to present themselves before the Lord, and
Satan also came among them.” Also, in Job 38:7 it says, "When the morning stars sang together, and
all the sons of God shouted for joy?" These are in reference to angels who are created beings and in
now way implies literal dependency from God.

New Testament usage of the term Son of God

The Term "Son of God" occurs 47 times in the King James New Testament. In reference to Jesus,
it is a title as the heavenly, eternal Son who is equal to God the Father ( John 5:18-24). It is Jesus who
fully reveals the Father (Matt. 11:27). He is the exact representation of the Father (Heb. 1:1-3), He
possesses all authority in heaven and earth (Matt. 28:18), and Jesus had glory with the Father before
the world was made (John 17:5).
The Muslim is taught from the Qur'an and therefore cannot accept the fact that Jesus is divine. To
the Muslim, that is shirk, blasphemy of the worst kind. But believing it doesn't make it so. To the
Christian, and according to the Bible, Jesus is the one who alone saves us from our sins. We cannot
earn our way to heaven, perform enough good works to please God, or ever be "sincere enough" in
repentance to somehow obtain forgiveness from God. Instead, Christianity is a faith of God's great
love and sacrifice for His creation. Jesus, the Son of God, is the divine one who fulfilled prophecies,
walked on water, healed the sick, and rose from the dead. Only the Son of God can do these things.
If Jesus is God, then who did He pray to?

This is a very common question among critiques of Jesus' deity, Muslims included. The answer is
found in understanding the Trinity and the incarnation of Jesus.
The Trinity is the doctrine that there is only one God in all existence. This one God exists as three
persons: The Father, The Son, and the Holy Spirit. They are not three gods, but one God. Each is a
separate person, yet each of them is, in essence, divine in nature.
A close analogy of the Trinity can be found by looking at the concept of time. Time is past, present,
and future. There are three "aspects" or "parts" of time. This does not mean that there are three
"times," but only one. Each is separate, in a sense, yet each shares the same nature, or essence. In a
similar way, the Trinity is three separate persons who share the same nature.

The Incarnation

The doctrine of the incarnation in Christian teaching is that Jesus, who is the second person of the
Trinity, added to himself human nature and became a man.
The Bible says that Jesus is God in flesh, "In the beginning was the Word, and the Word was with
God, and the Word was God.....and the word became flesh and dwelt among us" (John 1:1, 14); and,
"For in Him all the fullness of Deity dwells in bodily form" (Col. 2:9). Jesus, therefore, has two natures.
He is both God and man.
Jesus is completely human, but He also has a divine nature.

GOD MAN

He is worshiped (Matt. 2:2,11; 14:33; 28:9) He worshiped the Father (John 17)
He is prayed to (Acts 7:59; 1 Cor. 1:2) He prayed to the Father (John 17:1)
He was called God (John 20:28; Heb. 1:8) He was called man (Mark 15:39; John 19:5).
He was called Son of God (Mark 1:1) He was called Son of Man (John 19:35-37)
He is sinless (1 Pet. 2:22; Heb. 4:15) He was tempted (Matt. 4:1)
He knew all things (John 21:17) He grew in wisdom (Luke 2:52)
He gives eternal life (John 20:28) He died (Rom. 5:8)
The fullness of deity dwells in Him (Col. 2:9) He has a body of flesh and bones (Luke
24:39)

As a man, Jesus needed to pray. When He was praying he was not praying to Himself, but to God
the Father.
God cannot be tempted. Jesus was tempted.
Therefore, Jesus cannot be God.

In their attempt to deny the deity of Jesus, the Muslims sometimes raise the objection that Jesus
was tempted and God cannot be tempted. Therefore Jesus cannot be God.
James 1:13 says, "Let no one say when he is tempted, "I am being tempted by God"; for God
cannot be tempted by evil, and He Himself does not tempt anyone." It also says in Heb. 4:15, "For we
do not have a high priest who cannot sympathize with our weaknesses, but One who has been
tempted in all things as we are, yet without sin."
Answering this objection is a bit more difficult than answering the other objections to Christ's deity
because it deals with an the area of scripture that is not explicitly clear: the relation between the
divine and human natures of Jesus. We see that Jesus has two natures as is taught in the chart
below, but how they related is not clarified.

Jesus as one person


GOD MAN
He is worshiped (Matt. 2:2,11; 14:33; 28:9) He worshiped the Father (John 17)
He is prayed to (Acts 7:59; 1 Cor. 1:1-2) He prayed to the Father (John 17:1)
He was called God (John 20:28; Heb. 1:8) He was called man (Mark 15:39; John 19:5).
He was called Son of God (Mark 1:1) He was called Son of Man (John 9:35-37)
He is sinless (1 Pet. 2:22; Heb. 4:15) He was tempted (Matt. 4:1)
He knew all things (John 21:17) He grew in wisdom (Luke 2:52)
He gives eternal life (John 20:28) He died (Rom. 5:8)
The fullness of deity dwells in Him (Col. 2:9) He has a body of flesh and bones (Luke 24:39)

We see from scripture that Jesus' human nature never existed apart from the union of his divine
nature. We also see in scripture that God cannot sin and that in Christ dwelt the fullness of the
Godhead in bodily form (John 1:1,14; Col. 2:9). Therefore, since we acknowledge that Jesus was
divine, we could easily conclude that it was not possible for Jesus to have sinned. On the other hand,
Jesus was truly man. Therefore, it is fair to say that Jesus could have been truly tempted. But, the
question persists: if it was not possible for Jesus to have sinned then how could He be truly
tempted? I do not know if I have a sufficient answer to this. But I will offer one anyway.
In all that Jesus did, he did by looking to the Father. Jesus said, "Truly, truly, I say to you, the Son
can do nothing of Himself, unless it is something He sees the Father doing; for whatever the Father
does, these things the Son also does in like manner," (John 5:19). Also, Jesus said, "I can do nothing
on My own initiative. As I hear, I judge; and My judgment is just, because I do not seek My own will,
but the will of Him who sent Me" (John 5:30).
In Matt. 12:2232, Jesus was casting out demons. The Pharisees accused Jesus of doing this by the
power of the devil. Jesus replied to them that blasphemy against the Holy Spirit would not be forgiven.
Why did he say this? I believe that it is because Jesus did none of his miracles out of his own divine
nature but did them as a man working through and by the Holy Spirit who indwelt Him. Therefore,
Jesus was casting out demons by the power of the Holy Spirit. We see that Jesus' miracles began after
his baptism and that is when the Holy Spirit descended upon him.
Jesus came as a man in order to fulfill the law of God and to be the sacrifice for sin. He did this as
a man. When He resisted the temptations of the devil, He quoted scripture -- as a man. He did not at
that time rely on His divine nature when going about His earthly ministry in Israel. As a man, He was
tempted and as a man He resistant temptation by relying on God's word. He cast out demons by the
Holy Spirit and not by His own divine nature. Therefore, Jesus was tempted in His human nature, not
in His divine. He did not rely on His divine "side" to help Him out. Instead, He completely relied on
the Father, the Holy Spirit, and God's word to successfully resist the temptations that came to Him.
God is infinite. Matter is finite. God could not become a man.
Muslims deny that Jesus could be God in flesh. They affirm that He was a great prophet, but they
clearly deny His divinity. One of the reasons is Muslims are taught that God is infinite and that He
could not become a finite man. In other words, the infinite God cannot become finite man. They say
it doesn't make sense. One Muslim asked me how God who is dependent on nothing, can then
become dependent as a man. He said that by definition God is not dependent upon anyone, therefore
to become dependent is impossible. Another said that if God became a man, he could not then
become a god again because a man cannot become a god.
These questions reveal how Muslims think. They have such a strict idea about God, that they
cannot admit the possibility of Him (or part of Him) becoming a man. Any idea of an incarnation
becomes ludicrous to them. They claim that it isn't logical.
Muslims are also taught that the Bible is corrupted and that only the Qur'an is perfect. So, to
quote from the Bible makes little impact on them. Many Muslims have required logical proofs for the
theory of the incarnation instead of biblical references. I attempt to oblige them here.
The following outline is designed to answer the objections raised by Muslims. In "Premise one," the
objections are in bold. The answers to them follow.

Premise one: According to Islam, God can do anything.


If this is so, then it necessarily follows that if God can do anything, then he can become a man
since that possibility falls under the scope of "God can do anything."

1. This would mean that God stopped being God.


A. Since God can do anything, according to the premise above, then God could do this without
stopping being God. See part "b." in next objection.
B. If God, in some way, became a man, it does not necessitate that He stop being divine. He
could simple add to Himself human nature.
2. This would mean that the infinite God became finite.
A. Not if a "part" of God entered into a human form. The totality of God could still exist, yet a
localized "part" could take the form of a man.
B. Is not the Qur'an the word of Allah? Is not His word a reflection of His character since it
proceeds from Him? Is not the infinite word of Allah made to become knowable, readable in
a physical form for us to understand? Since this is so, why cannot the Word of God become
flesh -- as the Bible says? Why cannot a representation of God (His word) take a physical
form (Qur'an) or even a human form (Jesus) -- since God can do anything?
3. This would mean that the independent became dependent.
A. It would not necessitate that the totality of God became dependent, per point "b." above: a
part of God could become man.
B. God can choose to become in part, as a man. He can make that choice, can he not?
4. This would mean that the eternal became temporal.
A. Again, by premise one, God could do it since He can do all things.
B. If God, in some way, became a man by adding human nature to Himself, it would not
necessitate that God stop being eternal since His divine nature would be, by nature, eternal
as it is retained within the human form.
5. If God became man, then he could not become God again.
A. If only a "part" of God became man, then God would never have ceased being God and the
objection is moot.
B. If God can do all things, then a part of Him can become a man and retain His divine nature
and never have stopped being God at all.
6. Why would God need to become a man? Showing He has a need means he is
dependent.
A. It is not a need. It is a choice. God is not compelled to do anything -- except be Himself.
If He chose to become a man, it would be by His desire, not by His need.
B. If God can do anything, then He can choose to share in the dependency of a human and not
deny his own nature of being God.
Premise two: God cannot do anything, because He cannot do anything that conflicts with
His nature. Becoming a man conflicts with His nature.

1. To say God's nature does not permit Him, in some way, to become a man requires that the
Muslim establish those aspects of God's nature that negate the possibility of an incarnation,
otherwise it is only the Muslim's opinion.
A. God's nature has to do with essential character and essence of His being like holiness, love,
compassion, goodness, patience, etc.
i. There is nothing in holiness, love, compassion, goodness, patience, etc., that would
mean God could not become a man.
B. God's attributes are inherent characteristics like eternality, infinity, invisibility, omniscience,
omnipotence, omnipresence, speech, creativity, etc.
i. None of the above attributes negate the possibility of a part of God becoming man.
a. The essential nature of something is not changed if a part of it adds humanity.

There is no logical reason to declare the impossibility of God being Trinitarian or that He, in some
way, could become a man.
The Bible has declared that God is indeed a Trinity and that Jesus is both God and man (John
1:1, 14; Col. 2:9; etc.).
Why is it necessary for God to die for our sins?
Muslims often ask why it is necessary for God to die for man's sins. Why can't we just confess our
sins and have God forgive us? Isn't that enough?
Following is an attempt to logically demonstrate the necessity of God atoning for our sins.

1. God is infinite.
A. There is no limit to Him. He is endless.
2. God is holy.
A. Holiness is purity. God is incapable of doing anything wrong. Part of the quality of holiness
is the inability to do wrong.
3. God is just.
A. He always does what is right.
B. God cannot violate His own righteousness declarations, because that would mean God is
contradicting Himself.
C. This justice is according to His nature since it is He is who tells us what is right and wrong.
4. Therefore, God is infinitely holy and infinitely just
A. Neither His holiness nor justice can be denied since they are part of His character and God
cannot be denied.
5. We are not infinite and not holy
A. We are not infinite because we are creations.
B. We are not holy because we have sinned.
6. Sin is doing anything against God's Law.
A. It is God who declares what is right and wrong. He has revealed this to us in the scriptures.
7. The Law is a reflection of God's character.
A. God speaks out of what is in His mind and heart. If He says do not lie, it is because it is
against God's nature to lie.
B. God is not speaking without reason or purpose. If He had no reason or purpose, this would
mean that God is not trustworthy.
C. God is trustworthy; therefore God's Law is the standard of perfection, justice, and holiness.
8. God's Law carries a penalty upon the sinner which is damnation.
A. Damnation is the act of God where He passes righteous judgment upon a person because of
the person's sin against Him.
B. If breaking God's law did not carry a penalty, then there would be no damnation. But since
there is damnation, we can conclude that breaking God's law carries a penalty.
C. If He did not damn based upon righteousness, then God is doing wrong. Since God cannot
do anything wrong, then damnation is righteous.
9. God is affected by what we do.
A. Proof of this is found in our prayers. Since God answers our prayers, our prayers have an
affect upon God because God is moved to answer.
B. If our prayers have no affect upon God, then prayer is useless since it accomplishes nothing.
10. Breaking God's Law, sinning, has a negative effect upon our relationship with God.
A. God is not injured in a physical way by our sins since God is spirit, perfect, and complete.
B. But, since damnation exists (because of the justice of God), we can conclude that sin has a
negative affect upon the relationship between the sinner and God. If this were not so, there
would be no damnation.
11. Since God is infinite, our offense against Him has an infinite effect.
A. It is the infinite God we have offended, therefore, the sin results in an infinite offense against
God.
12. A finite person cannot remove an infinite offense against an infinite God.
A. A finite work cannot remove an infinite offense because the effort of a finite person will
always fall short of meeting the justice of an infinite God.
13. God cannot arbitrarily forgive the sinner without satisfying His infinite justice.
A. If damnation is righteously given because of justice, so too, forgiveness must be in
accordance with justice because both are dealing with sin.
B. To simply dismiss sin in order to forgive is to deny justice.
C. If forgiveness is not consistent with God's justice, then God is arbitrary, inconsistent, and
unjust.
D. Therefore, the act of forgiveness also requires an act of justice.
14. Since it is just that the sinner die and be damned, this justice cannot be ignored.
A. If it were ignored, then God is not being consistent in His justice.
B. If it were ignored, then God has no right to damn anyone.
C. No one is damned who is alive. Only the dead are damned.
15. Since man cannot earn forgiveness from God through his finite works, it must be God
who makes forgiveness possible.
A. This is so, because there is no one left to make things right, other than God.
16. Since it is not just to ignore the penalty for sin, and since man cannot satisfy God, there
is none left but God to pay for the just penalty of sin.
A. The sin cannot be ignored because the act of forgiveness also requires and act of justice.
B. This justice cannot be ignored because God would then be inconsistent.
17. It is just that the sinner dies and suffer judgment.
A. Death is a punishment of God and damnation follows death.
18. Since it is just that sin must be dealt with, God must meet that requirement of justice.
A. This is so, because a finite person cannot please and infinite God's just requirements of
holiness and purity.
B. God must then do what is just in forgiveness of sins or justice cannot be met.
19. God must then take the place of the sinner and suffer the consequence of the lawful
judgment of death upon the sinner.
20. With justice met, forgiveness can then be rightly given.
21. This forgiveness, which cannot be earned by man's effort, can only be received from
God by man's faith - because there is nothing else he can do.
Contradictions in the Qur'an

The Qur'an states that it is a perfect book preserved on tablets in heaven (Surah 85:21-22). If the
Qur'an is a perfect book from Allah, then there shouldn't be any contradictions in it. Of course, the
Muslims will deny any contradictions exist in the Qur'an, but they do. Some of the contradictions
below could be debated, but some of them are clearly contradictions.
A contradiction occurs when one statement on a subject excludes the possibility of another. The
first one here is a good example. In Surah 19:67, it states that man was created out of nothing. In
15:26, man is created from clay. Since clay is something, we have a contradiction since "nothing"
excludes the possibility of "clay." Both cannot be true.
All quotes from the Qur'an, unless otherwise specified, are from Yusuf Ali and can be found at the
Qur'an online.

1. What was man created from, blood, clay, dust, or nothing?


A. "Created man, out of a (mere) clot of congealed blood," (96:2).
B. "We created man from sounding clay, from mud molded into shape, (15:26).
C. "The similitude of Jesus before Allah is as that of Adam; He created him from dust, then said
to him: "Be". And he was," (3:59).
D. "But does not man call to mind that We created him before out of nothing?" (19:67, Yusuf
Ali). Also, 52:35).
E. "He has created man from a sperm-drop; and behold this same (man) becomes an open
disputer! (16:4).
2. Is there or is there not compulsion in religion according to the Qur'an?
A. "Let there be no compulsion in religion: Truth stands out clear from Error: whoever rejects
evil and believes in Allah hath grasped the most trustworthy hand-hold, that never breaks.
And Allah heareth and knoweth all things," (2:256).
B. "And an announcement from Allah and His Messenger, to the people (assembled) on the day
of the Great Pilgrimage,- that Allah and His Messenger dissolve (treaty) obligations with the
Pagans. If then, ye repent, it were best for you; but if ye turn away, know ye that ye cannot
frustrate Allah. And proclaim a grievous penalty to those who reject Faith," (9:3).
C. "But when the forbidden months are past, then fight and slay the Pagans wherever ye find
them, and seize them, beleaguer them, and lie in wait for them in every stratagem (of war);
but if they repent, and establish regular prayers and practice regular charity, then open the
way for them: for Allah is Oft-forgiving, Most Merciful," (9:5).
D. Fight those who believe not in Allah nor the Last Day, nor hold that forbidden which hath
been forbidden by Allah and His Messenger, nor acknowledge the religion of Truth, (even if
they are) of the People of the Book, until they pay the Jizya with willing submission, and feel
themselves subdued," (9:29).
3. The first Muslim was Muhammad? Abraham? Jacob? Moses?
A. "And I [Muhammad] am commanded to be the first of those who bow to Allah in Islam,"
(39:12).
B. "When Moses came to the place appointed by Us, and his Lord addressed him, He said: "O
my Lord! show (Thyself) to me, that I may look upon thee." Allah said: "By no means canst
thou see Me (direct); But look upon the mount; if it abide in its place, then shalt thou see
Me." When his Lord manifested His glory on the Mount, He made it as dust. And Moses fell
down in a swoon. When he recovered his senses he said: "Glory be to Thee! to Thee I turn in
repentance, and I am the first to believe." (7:143).
C. "And this was the legacy that Abraham left to his sons, and so did Jacob; "Oh my sons! Allah
hath chosen the Faith for you; then die not except in the Faith of Islam," (2:132).
4. Does Allah forgive or not forgive those who worship false gods?
A. Allah forgiveth not that partners should be set up with Him; but He forgiveth anything else,
to whom He pleaseth; to set up partners with Allah is to devise a sin Most heinous indeed,"
(4:48). Also 4:116
B. The people of the Book ask thee to cause a book to descend to them from heaven: Indeed
they asked Moses for an even greater (miracle), for they said: "Show us Allah in public," but
they were dazed for their presumption, with thunder and lightning. Yet they worshipped the
calf even after clear signs had come to them; even so we forgave them; and gave Moses
manifest proofs of authority," (4:153).
5. Are Allah's decrees changed or not?
A. "Rejected were the messengers before thee: with patience and constancy they bore their
rejection and their wrongs, until Our aid did reach them: there is none that can alter the
words (and decrees) of Allah. Already hast thou received some account of those
messengers," (6:34).
B. "The word of thy Lord doth find its fulfillment in truth and in justice: None can change His
words: for He is the one who heareth and knoweth all, (6:115).
C. None of Our revelations do We abrogate or cause to be forgotten, but We substitute
something better or similar: Knowest thou not that Allah Hath power over all things?"
(2:106).
D. When We substitute one revelation for another,- and Allah knows best what He reveals (in
stages),- they say, "Thou art but a forger": but most of them understand not," (16:101).
6. Was Pharaoh killed or not killed by drowning?
A. "We took the Children of Israel across the sea: Pharaoh and his hosts followed them in
insolence and spite. At length, when overwhelmed with the flood, he said: "I believe that
there is no god except Him Whom the Children of Israel believe in: I am of those who submit
(to Allah in Islam). (It was said to him): "Ah now!- But a little while before, wast thou in
rebellion!- and thou didst mischief (and violence)! This day shall We save thee in the body,
that thou mayest be a sign to those who come after thee! but verily, many among mankind
are heedless of Our Signs!" (10:90-92).
B. Moses said, "Thou knowest well that these things have been sent down by none but the Lord
of the heavens and the earth as eye-opening evidence: and I consider thee indeed, O
Pharaoh, to be one doomed to destruction!" So he resolved to remove them from the face of
the earth: but We did drown him and all who were with him," (17:102-103).
7. Is wine consumption good or bad?
A. O ye who believe! Intoxicants and gambling, (dedication of) stones, and (divination by)
arrows, are an abomination,- of Satan's handwork: eschew such (abomination), that ye may
prosper," (5:90).
B. (Here is) a Parable of the Garden which the righteous are promised: in it are rivers of water
incorruptible; rivers of milk of which the taste never changes; rivers of wine, a joy to those
who drink; and rivers of honey pure and clear. In it there are for them all kinds of fruits; and
Grace from their Lord. (Can those in such Bliss) be compared to such as shall dwell for ever
in the Fire, and be given, to drink, boiling water, so that it cuts up their bowels (to pieces)?"
(47:15).
C. Truly the Righteous will be in Bliss: On Thrones (of Dignity) will they command a sight (of all
things): Thou wilt recognize in their faces the beaming brightness of Bliss. Their thirst will be
slaked with Pure Wine sealed," (83:22-25).
Interesting quotes from the Qur'an

The Qur'an is the sacred book of Islam. It is supposed to be a perfect book, inspired, and
flawless. Would you expect the following quotes from an inspired and flawless book?
All quotes from the Qur'an, unless otherwise specified, are from Yusuf Ali and can be found at the
Qur'an online.

1. It is not good to enter a house from the back


A. "They ask thee concerning the New Moons. Say: They are but signs to mark fixed periods of
time in (the affairs of) men, and for Pilgrimage. It is no virtue if ye enter your houses from
the back: It is virtue if ye fear Allah. Enter houses through the proper doors: And fear Allah:
That ye may prosper," (2:189).
2. Cities (Sodom and Gomorrah) are turned upside down - literally!
A. "(The Messengers) said: "O Lut! We are Messengers from thy Lord! By no means shall they
reach thee! now travel with thy family while yet a part of the night remains, and let not any
of you look back: but thy wife (will remain behind): To her will happen what happens to the
people. Morning is their time appointed: Is not the morning nigh?...When Our Decree issued,
We turned (the cities) upside down, and rained down on them brimstones hard as baked
clay, spread, layer on layer," (11:81-82)
B. "And We turned (the cities) upside down, and rained down on them brimstones hard as
baked clay," (15:74).
3. A boy and his dog sleep for 309 years in a cave.
A. "Such (being their state), we raised them up (from sleep), that they might question each
other. Said one of them, "How long have ye stayed (here)?" They said, "We have stayed
(perhaps) a day, or part of a day." (At length) they (all) said, "Allah (alone) knows best how
long ye have stayed here...So they stayed in their Cave three hundred years, and (some)
add nine (more)," (18:19,25).
4. The sun set in a pool of murky water
A. "Until, when he reached the setting of the sun, he found it set in a spring of murky water:
Near it he found a People: We said: "O Zul-qarnain! (thou hast authority,) either to punish
them, or to treat them with kindness." (18:86, Yusuf Ali, translation).
5. Jesus spoke while in the cradle
A. “But she pointed to the babe. They said: "How can we talk to one who is a child in the
cradle?" 30He said: "I am indeed a servant of Allah: He hath given me revelation and made
me a prophet," (19:29-30).
6. King Solomon learned the speech of birds
A. "And Solomon was David's heir. He said: "O ye people! We have been taught the speech of
birds, and on us has been bestowed (a little) of all things: this is indeed Grace manifest
(from Allah.)" (27:16).
B. "And Solomon was David's heir. And he said: O mankind! Lo! we have been taught the
language of birds, and have been given (abundance) of all things. This surely is evident
favour," (27:16, Pickthall, trans.).
7. Ants can speak
A. "At length, when they came to a (lowly) valley of ants, one of the ants said: "O ye ants, get
into your habitations, lest Solomon and his hosts crush you (under foot) without knowing it,"
(27:18).
B. "Till, when they reached the Valley of the Ants, an ant exclaimed: O ants! Enter your
dwellings lest Solomon and his armies crush you, unperceiving," (27:18, Pickthal, trans.).
8. Allah made seven heavens and seven earths
A. "Allah is He Who created seven Firmaments and of the earth a similar number. Through the
midst of them (all) descends His Command: that ye may know that Allah has power over all
things, and that Allah comprehends, all things in (His) Knowledge," (65:12)
9. Shooting stars are for driving away evil spirits
A. And we have, (from of old), adorned the lowest heaven with Lamps, and We have made
such (Lamps) (as) missiles to drive away the Evil Ones, and have prepared for them the
Penalty of the Blazing Fire, (67:5).
10. The soul exits through the collar-bone when leaving the body.
A. "Yea, when (the soul) reaches to the collar-bone (in its exit), 27And there will be a cry, "Who
is a magician (to restore him)?" 28And he will conclude that it was (the Time) of Parting,:
(75:26-28).
Interesting Quotes about women from the Qur'an
It can be said that a religion is judged by what it says about its women. The Qur'an says much of
them.
All quotes from the Qur'an are from Yusuf Ali and can be found at
http://www.usc.edu/dept/MSA/quran/

1. Men have more rights regarding divorce than do women


A. Divorced women shall wait concerning themselves for three monthly periods. Nor is it
lawful for them to hide what Allah Hath created in their wombs, if they have faith in
Allah and the Last Day. And their husbands have the better right to take them back in
that period, if they wish for reconciliation. And women shall have rights similar to the
rights against them, according to what is equitable; but men have a degree (of
advantage) over them. And Allah is Exalted in Power, Wise," (2:28)
2. Muslim men may marry up to four women, but no such provision is made for Muslim
women.
A. "If ye fear that ye shall not be able to deal justly with the orphans, Marry women of your
choice, Two or three or four; but if ye fear that ye shall not be able to deal justly (with
them), then only one, or (a captive) that your right hands possess, that will be more
suitable, to prevent you from doing injustice," (4:3).
3. A man's inheritance should be a portion of two females
A. Allah (thus) directs you as regards your Children's (Inheritance): to the male, a portion
equal to that of two females: if only daughters, two or more, their share is two-thirds of
the inheritance; if only one, her share is a half," (4:11).
4. It is okay to beat wives
A. "Men are the protectors and maintainers of women, because Allah has given the one
more (strength) than the other, and because they support them from their means.
Therefore the righteous women are devoutly obedient, and guard in (the husband's)
absence what Allah would have them guard. As to those women on whose part ye fear
disloyalty and ill-conduct, admonish them (first), (Next), refuse to share their beds,
(And last) beat them (lightly); but if they return to obedience, seek not against them
Means (of annoyance): For Allah is Most High, great (above you all)," (4:34).
5. In Paradise, voluptuous women await men for sensual gratification
A. "In them will be (Maidens), chaste, restraining their glances, whom no man or Jinn
before them has touched," (55:56).
B. "We have created (their Companions) of special creation. And made them virgin - pure
(and undefiled), - Beloved (by nature), equal in age,- For the Companions of the Right
Hand," (56:35-38)
C. "Verily for the Righteous there will be a fulfillment of (the heart's) desires; Gardens
enclosed, and grapevines, And voluptuous women of equal age," (78:31-33).
The Hadith

The Hadith, in Islam, is second in authority only to the Qur'an. The Hadith is a record of the
prophet Mohammed's life, actions, and deeds. A saying in the Hadith is called a sunnah. These
sunnah were transmitted by word of mouth down through the centuries having been memorized first
by Muhammad's companions and then later by subsequent Muslims. Therefore, the Hadith is the
written record of the oral traditions passed down from the Muslim to Muslim of what Mohammed was
supposed to have said and done.
The Qur'an is considered to be the absolute and infallible word of Allah. The Hadith, however,
though the words of the inspired prophet Muhammad, are not necessarily infallible. The oldest
collection to date dates from the 9th century.
The Hadith fall into two categories, "Hadith qudsi (sacred Hadith) in which God Himself is speaking
in, as it were, a complementary revelation through the Prophet, and hadith sharif (noble Hadith), the
Prophet's own acts and utterances." 194
The most famous and universally accepted among the six collections of Hadith are those of Sahih
Al-Bukhari (d. 870) -- which are used in the Interesting Quotes from the Hadith section -- and Abu al-
Hajjaj (d. 875), which is usually called "Muslim." The Hadith of Budhari can be found at
http://www.usc.edu/dept/MSA/fundamentals/hadithsunnah/bukhari/ and are the ones quoted on
CARM.
The Hadith are important because they elucidate many areas not covered by or were not very clear
in the Qur'an. The Hadith is appealed to in legal decisions and consulted in debate among many
Muslims.
There is debate among western scholars as to the historicity and accuracy of the Hadith. Some
believe additions were made to it to serve political and theological purposes of various Mulsim leaders.
Many Muslims will disagree and some even claim all the Hadith are inspired and accurate. This is quite
debatable.

Glasse, Cyril, The Concise Encyclopedia of Islam, Harper & Row, Publishers, Inc. San Francisco, 1989, page
194

141.
Interesting Quotes from the Hadith

1. Angels stop asking Allah to forgive people when they pass wind.
A. Allah's Apostle said, "The angels keep on asking Allah's forgiveness for anyone of you, as
long as he is at his Mu,salla (praying place) and he does not pass wind (Hadath). They say,
'O Allah! Forgive him, O Allah! be Merciful to him." - Volume 1, Book 8, Number 436:
Narrated Abu Huraira:
B. When he enters the mosque he is considered in prayer as long as he is waiting for the prayer
and the angels keep on asking for Allah's forgiveness for him and they keep on saying: 'O
Allah! Be Merciful to him, O Allah! Forgive him, as long as he keeps on sitting at his praying
place and does not pass wind. (See Hadith No. 620). - Volume 1, Book 8, Number 466:
Narrated Abu Huraira:
2. You should not pray to Allah saying "If you wish."
A. Allah's Apostle said, "None of you should say: 'O Allah, forgive me if You wish; O Allah, be
merciful to me if You wish,' but he should always appeal to Allah with determination, for
nobody can force Allah to do something against His Will." Volume 8, Book 75, Number 351:
Narrated Abu Huraira
3. What Jesus looks like is described
A. The Prophet said, "On the night of my Ascent to the Heaven, I saw Moses who was a tall
brown curly-haired man as if he was one of the men of Shan'awa tribe, and I saw Jesus, a
man of medium height and moderate complexion inclined to the red and white colors and of
lank hair. I also saw Malik, the gate-keeper of the (Hell) Fire and Ad-Dajjal amongst the
signs which Allah showed me." (The Prophet then recited the Holy Verse): "So be not you in
doubt of meeting him' when you met Moses during the night of Mi'raj over the heavens"
(32.23) Volume 4, Book 54, Number 462: Narrated Ibn Abbas:
4. Satan touches people when they are born.
A. The Prophet said, "When any human being is born. Satan touches him at both sides of the
body with his two fingers, except Jesus, the son of Mary, whom Satan tried to touch but
failed, for he touched the placenta-cover instead." Volume 4, Book 54, Number 506:
Narrated Abu Huraira:
5. What Moses looks like is described
A. The Prophet said, "One should not say that I am better than Jonah (i.e. Yunus) bin Matta."
So, he mentioned his father Matta. The Prophet mentioned the night of his Ascension and
said, "The prophet Moses was brown, a tall person as if from the people of the tribe of
Shanu'a. Jesus was a curly-haired man of moderate height." He also mentioned Malik, the
gate-keeper of the (Hell) Fire, and Ad-Dajjal. Volume 4, Book 55, Number 608: Narrated
Ibn 'Abbas:
6. Jesus spoke while still in the cradle.
A. The Prophet said, "None spoke in cradle but three: (The first was) Jesus, (the second was),
there a man from Bani Israel called Juraij. . . " Volume 4, Book 55, Number 645: Narrated
Abu Huraira
7. Upon dying, two angels come and ask you about Muhammad
A. The Prophet said, "When a human being is laid in his grave and his companions return and
he even hears their foot steps, two angels come to him and make him sit and ask him: What
did you use to say about this man, Muhammad? He will say: I testify that he is Allah's slave
and His Apostle. Then it will be said to him, 'Look at your place in the Hell-Fire. Allah has
given you a place in Paradise instead of it.' " The Prophet added, "The dead person will see
both his places. But a non-believer or a hypocrite will say to the angels, 'I do not know, but I
used to say what the people used to say! It will be said to him, 'Neither did you know nor did
you take the guidance (by reciting the Quran).' Then he will be hit with an iron hammer
between his two ears, and he will cry and that cry will be heard by whatever approaches him
except human beings and jinns." Volume 2, Book 23, Number 422: Narrated Anas.
8. A man slept through prayer time and the devil peed in his ear.
A. It was mentioned before the Prophet that there was a man who slept the night till morning
(after sunrise). The Prophet said, "He is a man in whose ears (or ear) Satan had urinated."
Volume 4, Book 54, Number 492: Narrated 'Abdullah:
B. A person was mentioned before the Prophet (p.b.u.h) and he was told that he had kept on
sleeping till morning and had not got up for the prayer. The Prophet said, "Satan urinated in
his ears." Volume 2, Book 21, Number 245: Narrated 'Abdullah.
9. Satan puts three knots in the back of the head of people who sleep.
A. Allah's Apostle said, "Satan puts three knots at the back of the head of any of you if he is
asleep. On every knot he reads and exhales the following words, 'The night is long, so stay
asleep.' When one wakes up and remembers Allah, one knot is undone; and when one
performs ablution, the second knot is undone, and when one prays the third knot is undone
and one gets up energetic with a good heart in the morning; otherwise one gets up lazy and
with a mischievous heart." Volume 2, Book 21, Number 243: Narrated Abu Huraira
10. Magic worked on Muhammad
A. Magic was worked on the Prophet so that he began to fancy that he was doing a thing which
he was not actually doing. One day he invoked (Allah) for a long period and then said, "I feel
that Allah has inspired me as how to cure myself. Two persons came to me (in my dream)
and sat, one by my head and the other by my feet. One of them asked the other, "What is
the ailment of this man?" The other replied, 'He has been bewitched" The first asked, 'Who
has bewitched him?' The other replied, 'Lubaid bin Al-A'sam.' The first one asked, 'What
material has he used?' The other replied, 'A comb, the hair gathered on it, and the outer skin
of the pollen of the male date-palm.' The first asked, 'Where is that?' The other replied, 'It is
in the well of Dharwan.' " So, the Prophet went out towards the well and then returned and
said to me on his return, "Its date-palms (the date-palms near the well) are like the heads of
the devils." I asked, "Did you take out those things with which the magic was worked?" He
said, "No, for I have been cured by Allah and I am afraid that this action may spread evil
amongst the people." Later on the well was filled up with earth. Volume 4, Book 54,
Number 490: Narrated 'Aisha:
11. Magic doesn't affect you if you eat seven Ajwa dates every morning.
A. Allah's Apostle said, "He who eats seven 'Ajwa dates every morning, will not be affected by
poison or magic on the day he eats them." Volume 7, Book 65, Number 356: Narrated
Sad: Also, Volume 7, Book 71, Number 663: Narrated Saud:
12. Muhammad ordered a man who drank wine to be beaten.
A. Abu Huraira said, "A man who drank wine was brought to the Prophet. The Prophet said,
'Beat him!" Abu Huraira added, "So some of us beat him with our hands, and some with their
shoes, and some with their garments (by twisting it) like a lash, and then when we finished,
someone said to him, 'May Allah disgrace you!' On that the Prophet said, 'Do not say so, for
you are helping Satan to overpower him.' " Volume 8, Book 81, Number 768: Narrated Abu
Salama, Also, Volume 8, Book 81, Number 772: Narrated Abu Huraira:
13. You should not point towards a Muslim brother lest Satan tempt you to hit him.
A. The Prophet said, "None of you should point out towards his Muslim brother with a weapon,
for he does not know, Satan may tempt him to hit him and thus he would fall into a pit of
fire (Hell)" Volume 9, Book 88, Number 193, Narrated Abu Huraira:
14. Muhammad Spit into a dry well which filled and watered 1400 men.
A. We were one-thousand-and-four-hundred persons on the day of Al-Hudaibiya (Treaty), and
(at) Al-Hudaibiya (there) was a well. We drew out its water not leaving even a single drop.
The Prophet sat at the edge of the well and asked for some water with which he rinsed his
mouth and then he threw it out into the well. We stayed for a short while and then drew
water from the well and quenched our thirst, and even our riding animals drank water to
their satisfaction. Narrated Al-Bara: Volume 4, Book 56, Number 777.
The preceding Hadith are translated by Sahih Bukhari and are quoted from and online source
found at http://www.usc.edu/dept/MSA/fundamentals/hadithsunnah/bukhari/ .
Interesting Quotes from the Hadith, Part two.

1. Allah made Adam 60 cubits high (generally a cubit was 18 inches. Therefore, Adam
was 90 feet tall)
A. The Prophet said, "Allah created Adam, making him 60 cubits tall. When He created him, He
said to him, "Go and greet that group of angels, and listen to their reply, for it will be your
greeting (salutation) and the greeting (salutations of your offspring." So, Adam said (to the
angels), As-Salamu Alaikum (i.e. Peace be upon you). The angels said, "As-salamu Alaika wa
Rahmatu-l-lahi" (i.e. Peace and Allah's Mercy be upon you). Thus the angels added to
Adam's salutation the expression, 'Wa Rahmatu-l-lahi,' Any person who will enter Paradise
will resemble Adam (in appearance and figure). People have been decreasing in stature since
Adam's creation." Volume 4, Book 55, Number 543, Narrated Abu Huraira.
2. Water flowed out from under the fingernails of Muhammad
A. We used to consider miracles as Allah's Blessings, but you people consider them to be a
warning. Once we were with Allah's Apostle on a journey, and we ran short of water. He
said, "Bring the water remaining with you." The people brought a utensil containing a little
water. He placed his hand in it and said, "Come to the blessed water, and the Blessing is
from Allah." I saw the water flowing from among the fingers of Allah's Apostle , and no
doubt, we heard the meal glorifying Allah, when it was being eaten (by him). Volume 4,
Book 56, Number 779, Narrated 'Abdullah:
3. A man was blown by a wind to a distant mountain
A. We took part in the holy battle of Tabuk in the company of the Prophet and when we arrived
at the Wadi-al-Qura, there was a woman in her garden. The Prophet asked his companions
to estimate the amount of the fruits in the garden, and Allah's Apostle estimated it at ten
Awsuq (One Wasaq = 60 Sa's) and 1 Sa'= 3 kg. approximately). The Prophet said to that
lady, "Check what your garden will yield." When we reached Tabuk, the Prophet said, "There
will be a strong wind to-night and so no one should stand and whoever has a camel, should
fasten it." So we fastened our camels. A strong wind blew at night and a man stood up and
he was blown away to a mountain called Taiy, The King of Aila sent a white mule and a sheet
for wearing to the Prophet as a present, and wrote to the Prophet that his people would stay
in their place (and will pay Jizya taxation.) Volume 2, Book 24, Number 559, Narrated Abu
Humaid As-Sa'idi
4. Muhammad is ordered by Allah to fight all people until they worship Allah
A. Allah's Apostle said: "I have been ordered (by Allah) to fight against the people until they
testify that none has the right to be worshipped but Allah and that Muhammad is Allah's
Apostle, and offer the prayers perfectly and give the obligatory charity, so if they perform a
that, then they save their lives an property from me except for Islamic laws and then their
reckoning (accounts) will be done by Allah." Volume 1, Book 2, Number 24, Narrated Ibn
'Umar.
5. The Qur'an was revealed in the dialect of the Quraish
A. "(The Caliph 'Uthman ordered Zaid bin Thabit, Said bin Al-As, 'Abdullah bin Az-Zubair and
'Abdur-Rahman bin Al-Harith bin Hisham to write the Quran in the form of a book (Mushafs)
and said to them. "In case you disagree with Zaid bin Thabit (Al-Ansari) regarding any
dialectic Arabic utterance of the Quran, then write it in the dialect of Quraish, for the Quran
was revealed in this dialect." So they did it." Volume 6, Book 61, Number 507, Narrated
Anas bin Malik.
6. The Qur'an was not collected together by Muhammad
A. Abu Bakr As-Siddiq sent for me when the people! of Yamama had been killed (i.e., a number
of the Prophet's Companions who fought against Musailama). (I went to him) and found
'Umar bin Al-Khattab sitting with him. Abu Bakr then said (to me), "Umar has come to me
and said: "Casualties were heavy among the Qurra' of the! Qur'an (i.e. those who knew the
Quran by heart) on the day of the Battle of Yalmama, and I am afraid that more heavy
casualties may take place among the Qurra' on other battlefields, whereby a large part of
the Qur'an may be lost. Therefore I suggest, you (Abu Bakr) order that the Qur'an be
collected." I said to 'Umar, "How can you do something which Allah's Apostle did not do?"
'Umar said, "By Allah, that is a good project. "Umar kept on urging me to accept his proposal
till Allah opened my chest for it and I began to realize the good in the idea which 'Umar had
realized." Then Abu Bakr said (to me). 'You are a wise young man and we do not have any
suspicion about you, and you used to write the Divine Inspiration for Allah's Apostle. So you
should search for (the fragmentary scripts of) the Qur'an and collect it in one book)." By
Allah If they had ordered me to shift one of the mountains, it would not have been heavier
for me than this ordering me to collect the Qur'an. Then I said to Abu Bakr, "How will you do
something which Allah's Apostle did not do?" Abu Bakr replied, "By Allah, it is a good
project." Abu Bakr kept on urging me to accept his idea until Allah opened my chest for what
He had opened the chests of Abu Bakr and 'Umar. So I started looking for the Qur'an and
collecting it from (what was written on) palmed stalks, thin white stones and also from the
men who knew it by heart, till I found the last Verse of Surat At-Tauba (Repentance) with
Abi Khuzaima Al-Ansari, and I did not find it with anybody other than him. The Verse is:
'Verily there has come unto you an Apostle (Muhammad) from amongst yourselves. It
grieves him that you should receive any injury or difficulty..(till the end of Surat-Baraa' (At-
Tauba) (9.128-129) Then the complete manuscripts (copy) of the Qur'an remained with Abu
Bakr till he died, then with 'Umar till the end of his life, and then with Hafsa, the daughter of
'Umar." Volume 6, Book 61, Number 509, Narrated Zaid bin Thabit.
7. The Qur'an was compiled because of differences among various manuscripts
A. Hudhaifa bin Al-Yaman came to Uthman at the time when the people of Sham and the
people of Iraq were Waging war to conquer Arminya and Adharbijan. Hudhaifa was afraid of
their (the people of Sham and Iraq) differences in the recitation of the Qur'an, so he said to
'Uthman, "O chief of the Believers! Save this nation before they differ about the Book
(Quran) as Jews and the Christians did before." So 'Uthman sent a message to Hafsa saying,
"Send us the manuscripts of the Qur'an so that we may compile the Qur'anic materials in
perfect copies and return the manuscripts to you." Hafsa sent it to 'Uthman. 'Uthman then
ordered Zaid bin Thabit, 'Abdullah bin AzZubair, Said bin Al-As and 'AbdurRahman bin Harith
bin Hisham to rewrite the manuscripts in perfect copies. 'Uthman said to the three Quraishi
men, "In case you disagree with Zaid bin Thabit on any point in the Qur'an, then write it in
the dialect of Quraish, the Qur'an was revealed in their tongue." They did so, and when they
had written many copies, 'Uthman returned the original manuscripts to Hafsa. 'Uthman sent
to every Muslim province one copy of what they had copied, and ordered that all the other
Qur'anic materials, whether written in fragmentary manuscripts or whole copies, be burnt.
Said bin Thabit added, "A Verse from Surat Ahzab was missed by me when we copied the
Qur'an and I used to hear Allah's Apostle reciting it. So we searched for it and found it with
Khuzaima bin Thabit Al-Ansari. (That Verse was): 'Among the Believers are men who have
been true in their covenant with Allah.' (33.23) Volume 6, Book 61, Number 510, Narrated
Anas bin Malik.
8. A Qur'anic verse is cancelled
A. The Prophet sent seventy men from the tribe of Bani Salim to the tribe of Bani Amir. When
they reached there, my maternal uncle said to them, "I will go ahead of you, and if they
allow me to convey the message of Allah's Apostle (it will be all right); otherwise you will
remain close to me." So he went ahead of them and the pagans granted him security But
while he was reporting the message of the Prophet , they beckoned to one of their men who
stabbed him to death. My maternal uncle said, "Allah is Greater! By the Lord of the Kaba, I
am successful." After that they attached the rest of the party and killed them all except a
lame man who went up to the top of the mountain. (Hammam, a sub-narrator said, "I think
another man was saved along with him)." Gabriel informed the Prophet that they (i.e the
martyrs) met their Lord, and He was pleased with them and made them pleased. We used to
recite, "Inform our people that we have met our Lord, He is pleased with us and He has
made us pleased " Later on this Quranic Verse was cancelled. The Prophet invoked Allah for
forty days to curse the murderers from the tribe of Ral, Dhakwan, Bani Lihyan and Bam
Usaiya who disobeyed Allah and his Apostle." Volume 4, Book 52, Number 57, Narrated
Anas. See also Volume 4, Book 52, See also, 69, and 299.
9. Various Qur'an manuscripts had missing verses
A. Zaid bin Thabit said, "When the Quran was compiled from various written manuscripts, one
of the Verses of Surat Al-Ahzab was missing which I used to hear Allah's Apostle reciting. I
could not find it except with Khuzaima bin Thabjt Al-Ansari, whose witness Allah's Apostle
regarded as equal to the witness of two men. And the Verse was:-- "Among the believers are
men who have been true to what they covenanted with Allah." (33.23) Volume 4, Book 52,
Number 62, Narrated Kharija bin Zaid.
10. Muhammad was once bewitched
A. "Once the Prophet was bewitched so that he began to imagine that he had done a thing
which in fact he had not done." Volume 4, Book 53, Number 400, Narrated Aisha.
11. Muhammad said that one wing of a house fly has a disease and the other wing has the
cure
A. The Prophet said "If a house fly falls in the drink of anyone of you, he should dip it (in the
drink), for one of its wings has a disease and the other has the cure for the disease." Volume
4, Book 54, Number 537, Narrated Abu Huraira.
12. Allah forgave a prostitute of her sins because she gave water to a dying dog
A. Allah's Apostle said, "A prostitute was forgiven by Allah, because, passing by a panting dog
near a well and seeing that the dog was about to die of thirst, she took off her shoe, and
tying it with her head-cover she drew out some water for it. So, Allah forgave her because of
that." Volume 4, Book 54, Number 538, Narrated Abu Huraira.
13. Angels don't enter houses with dogs in them
A. The Prophet said, "Angels do not enter a house witch has either a dog or a picture in it."
Volume 4, Book 54, Number 539, Narrated Abu Talha.
14. Jinn eat dung and animal bones
A. That once he was in the, company of the Prophet carrying a water pot for his ablution and
for cleaning his private parts. While he was following him carrying it(i.e. the pot), the
Prophet said, "Who is this?" He said, "I am Abu Huraira." The Prophet said, "Bring me stones
in order to clean my private parts, and do not bring any bones or animal dung." Abu Huraira
went on narrating: So I brought some stones, carrying them in the corner of my robe till I
put them by his side and went away. When he finished, I walked with him and asked, "What
about the bone and the animal dung?" He said, "They are of the food of Jinns. The delegate
of Jinns of (the city of) Nasibin came to me--and how nice those Jinns were--and asked me
for the remains of the human food. I invoked Allah for them that they would never pass by a
bone or animal dung but find food on them." Volume 5, Book 58, Number 200, Narrated
Abu Huraira.
15. Women and sons can be mortgaged
A. ". . . Now we want you to lend us a camel load or two of food." (Some difference between
narrators about a camel load or two.) Kab said, "Yes, (I will lend you), but you should
mortgage something to me." Muhammad bin Mas-lama and his companion said, "What do
you want?" Ka'b replied, "Mortgage your women to me." They said, "How can we mortgage
our women to you and you are the most handsome of the 'Arabs?" Ka'b said, "Then
mortgage your sons to me." They said, "How can we mortgage our sons to you? Later they
would be abused by the people's saying that so-and-so has been mortgaged for a camel load
of food. That would cause us great disgrace, but we will mortgage our arms to you." Volume
5, Book 59, Number 369, Narrated Jabir bin 'Abdullah.
16. The Angel Gabriel had 600 wings.
A. I asked Sir about the Statement of Allah: 'And was at a distance of but two bow-lengths or
(even) nearer. So did Allah convey the Inspiration to His slave (Gabriel) and then he
(Gabriel) conveyed that to Muhammad.' (53.10) He said, "Abdullah (bin Mas'ud) informed us
that Muhammad had seen Gabriel with six hundred wings." Volume 6, Book 60, Number
380, Narrated assailant.
17. The moon was cut into two pieces
A. During the lifetime of Allah's Apostle the moon was split into two parts; one part remained
over the mountain, and the other part went beyond the mountain. On that, Allah's Apostle
said, "Witness this miracle." 388The moon was cleft asunder while we were in the company of
the Prophet, and it became two parts. The Prophet said, Witness, witness (this miracle)."
Volume 6, Book 60, Number 387-388, Narrated Ibn Masud and Abdullah. See also 389-391.
18. A fever is from the heat of hell fire
A. I used to sit with Ibn 'Abbas in Mecca. Once I had a fever and he said (to me), "Cool your
fever with Zam-zam water, for Allah's Apostle said: 'It, (the Fever) is from the heat of the
(Hell) Fire; so, cool it with water (or Zam-zam water)." Volume 4, Book 54, Number 483,
Narrated Abu Jamra Ad-Dabi.

The preceding Hadith are translated by Sahih Bukhari and are quoted from and online source
found at http://www.usc.edu/dept/MSA/fundamentals/hadithsunnah/bukhari/ .
Interesting Quotes from the Hadith about Forgiveness

Salvation in Islam is by grace and works. I have been told by Muslims that there are
requirements for forgiveness of sins: faith in Allah and Muhammad as his prophet; submission to Allah
and his commandments; and sincere repentance from sin. Allah is not obligated to forgive anyone of
his sins. But, Allah is gracious to those who have been faithful and sincere. The Muslim can then have
a hope of surviving the Day of Judgment and going to Paradise -- if his good deeds outweigh his bad
ones.
In these Hadith quotes, notice how forgiveness is so often dependent upon the attitude and actions
of the person. This is different than in Christianity where forgiveness of sins is received by faith, not
by deed (Eph. 2:8-9). We confess our sins (1 John 1:7-9) and repent (Matt. 4:17), and of course we
are to be sincere. But forgiveness of sins has nothing to do with our deeds. We repent because we
are forgiven, not to get forgiven.

1. Punishment expiates sins


A. The Prophet added: "Whoever among you fulfills his pledge will be rewarded by Allah. And
whoever indulges in any one of them (except the ascription of partners to Allah) and gets
the punishment in this world, that punishment will be an expiation for that sin. And if one
indulges in any of them, and Allah conceals his sin, it is up to Him to forgive or punish him
(in the Hereafter)." - Volume 1, Book 2, Number 17: Narrated 'Ubada bin As-Samit:
2. Sincere prayers result in forgiveness of sins
A. Allah's Apostle said, "Whoever establishes the prayers on the night of Qadr out of sincere
faith and hoping to attain Allah's rewards (not to show off) then all his past sins will be
forgiven." , Volume 1, Book 2, Number 34, Narrated Abu Huraira
B. Allah's Apostle said: "Whoever establishes prayers during the nights of Ramadan faithfully
out of sincere faith and hoping to attain Allah's rewards (not for showing off), all his past
sins will be forgiven." - , Volume 1, Book 2, Number 36, Narrated Abu Huraira
3. Fasting and sincere prayer result in forgiveness
A. Allah's Apostle said, "Whoever observes fasts during the month of Ramadan out of sincere
faith, and hoping to attain Allah's rewards, then all his past sins will be forgiven." - Volume
1, Book 2, Number 37, Narrated Abu Huraira.
4. Ablutions and pure thoughts during prayer result in forgivness.
A. (the slave of 'Uthman) I saw 'Uthman bin 'Affan asking for a tumbler of water (and when it
was brought) he poured water over his hands and washed them thrice and then put his right
hand in the water container and rinsed his mouth, washed his nose by putting water in it and
then blowing it out. then he washed his face and forearrlns [sic] up to the elbows thrice,
passed his wet hands over his head and washed his feet up to the ankles thrice. Then he
said, "Allah's Apostle said 'If anyone Performs ablution [washing] like that of mine and offers
a two-rak'at prayer [spiritual communication with Allah] during which he does not think of
anything else (not related to the present prayer) then his past sins will be forgiven.' " After
performing the ablution 'Uthman said, "I am going to tell you a Hadith which I would not
have told you, had I not been compelled by a certain Holy Verse (the sub narrator 'Urwa
said: This verse is: "Verily, those who conceal the clear signs and the guidance which we
have sent down...)" (2:159). I heard the Prophet saying, 'If a man performs ablution
perfectly and then offers the compulsory congregational prayer, Allah will forgive his sins
committed between that (prayer) and the (next) prayer till he offers it. Volume 1, Book 4,
Number 161, Narrated Humran, See also 165
5. Prayer offered in a congregation is 25 times better than prayer offered alone and each
step towards a mosque results in forgiveness of a sin.
A. The Prophet said, "The prayer offered in congregation is twenty five times more superior (in
reward) to the prayer offered alone in one's house or in a business center, because if one
performs ablution and does it perfectly, and then proceeds to the mosque with the sole
intention of praying, then for each step which he takes towards the mosque, Allah upgrades
him a degree in reward and (forgives) crosses out one sin till he enters the mosque. When
he enters the mosque he is considered in prayer as long as he is waiting for the prayer and
the angels keep on asking for Allah's forgiveness for him and they keep on saying: 'O Allah!
Be Merciful to him, O Allah! Forgive him, as long as he keeps on sitting at his praying place
and does not pass wind. (See Hadith No. 620). Volume 1, Book 8, Number 466, Narrated
Abu Huraira,
6. Prayers coinciding with that of angels results in forgiveness of sins.
A. The Prophet said, "Say Amin" when the Imam says it and if the Amin of any one of you
coincides with that of the angels then all his past sins will be forgiven." Ibn Shihab said,
"Allah's Apostle used to Say "Amin." Volume 1, Book 12, Number 747, Narrated Abu
Huraira, See also 748
7. If you recite a certain prayer and die that day, your sins will all be forgiven.
A. The Prophet said "The most superior way of asking for forgiveness from Allah is: 'Allahumma
anta Rabbi la ilaha illa anta, Anta Khalaqtani wa ana abduka, wa ana 'ala ahdika wa wa'dika
mastata'tu, A'udhu bika min Sharri ma sana'tu, abu'u Laka bini'matika 'alaiya, wa Abu Laka
bidhanbi faghfirli innahu la yaghfiru adhdhunuba illa anta." The Prophet added. "If somebody
recites it during the day with firm faith in it, and dies on the same day before the evening,
he will be from the people of Paradise; and if somebody recites it at night with firm faith in
it, and dies before the morning, he will be from the people of Paradise." Volume 8, Book 75,
Number 318, Narrated Shaddad bin Aus, See also 335:
8. Saying a certain prayer 100 times gets the same reward as freeing 10 slaves, and
forgives sins.
A. Allah's Apostle said,"Whoever says: "La ilaha illal-lah wahdahu la sharika lahu, lahu-l-mulk
wa lahu-l-hamd wa huwa 'ala kulli shai'in qadir," one hundred times will get the same reward
as given for manumitting ten slaves; and one hundred good deeds will be written in his
accounts, and one hundred sins will be deducted from his accounts, and it (his saying) will
be a shield for him from Satan on that day till night, and nobody will be able to do a better
deed except the one who does more than he." Narrated Abu Huraira: Volume 8, Book 75,
Number 412.
B. Allah's Apostle said, "Whoever says, 'Subhan Allah wa bihamdihi,' one hundred times a day,
will be forgiven all his sins even if they were as much as the foam of the sea," Volume 8,
Book 75, Number 414, Narrated Abu Huraira,

The preceding Hadith are translated by Sahih Bukhari and are quoted from and online source
found at http://www.usc.edu/dept/MSA/fundamentals/hadithsunnah/bukhari/ .
Interesting Quotes from the Hadith on Jesus

1. Muhammad said Jesus was of red complexion.


A. The Prophet said, "I saw Moses, Jesus and Abraham (on the night of my Ascension to the
heavens). Jesus was of red complexion, curly hair and a broad chest. Moses was of brown
complexion, straight hair and tall stature as if he was from the people of Az-Zutt." Volume
4, Book 55, Number 648, Narrated Ibn Umar.
2. Muhammad said Jesus said he can't intercede for people.
A. The Prophet said, "Allah will gather the believers on the Day of Resurrection in the same way
(as they are gathered in this life), and they will say, 'Let us ask someone to intercede for us
with our Lord that He may relieve us from this place of ours.' ...They will go to Abraham who
will reply, 'I am not fit for this undertaking,' and mention to them the mistakes he made,
and add, 'But you'd better go to Moses, a slave whom Allah gave the Torah and to whom He
spoke directly' They will go to Moses who will reply, 'I am not fit for this undertaking,' and
mention to them the mistakes he made, and add, 'You'd better go to Jesus, Allah's slave and
His Apostle and His Word (Be: And it was) and a soul created by Him.' They will go to Jesus
who will say, 'I am not fit for this undertaking, but you'd better go to Muhammad whose sins
of the past and the future had been forgiven (by Allah).' So they will come to me and I will
ask the permission of my Lord, and I will be permitted (to present myself) before Him."
Volume 9, Book 93, Number 507. Narrated Anas. Also, Volume 6, Book 60, Number 236.
i. This is contrary to the Bible which states that Jesus is our mediator (1 Tim. 2:5) and
always lives to intercede for us: Heb. 7:25, says, "Hence, also, He is able to save
forever those who draw near to God through Him, since He always lives to make
intercession for them.
3. Muhammad implies that the Christian doctrine of Jesus as the son of God means God
had a wife, demonstrating his lack of understanding of the issue.
A. On the Day of Resurrection, a call-maker will announce, "Let every nation follow that which
they used to worship." Then none of those who used to worship anything other than Allah
like idols and other deities but will fall in Hell (Fire), till there will remain none but those who
used to worship Allah, both those who were obedient (i.e. good) and those who were
disobedient (i.e. bad) and the remaining party of the people of the Scripture. . . Afterwards
the Christians will be called upon and it will be said to them, 'Who do you use to worship?'
They will say, 'We used to worship Jesus, the son of Allah.' It will be said to them, 'You are
liars, for Allah has never taken anyone as a wife or a son,' . . . Allah will say, 'I am your
Lord.' They will say twice or thrice, 'We do not worship any besides Allah.' " Volume 6, Book
60, Number 105. Narrated Abu Said Al-Khudri,
i. Christian theology has never taught that Jesus was literally the offspring of God the
Father. Neither has Christianity ever taught that God has a wife. (The cult of
Mormonism has stated such erring doctrines, but it is not considered Christian.) The
term Son of God signified Jesus' special and unique relationship with the Father as the
Father's representative (Heb. 1:1-3).
4. Muhammad said that Jesus worshiped Allah.
A. Regarding the explanation of the Verse: 'Those whom they call upon (worship) (like Jesus
the Son of Mary, angels etc.) desire (for themselves) means of access to their Lord (Allah) as
to which of them should be the nearer and they hope for His Mercy and fear His torment.'
(17.57) They themselves (e.g. Angels, saints, Apostles, Jesus, etc.,) worshipped Allah, Those
Jinns who were worshipped by some Arabs became Muslims (embraced Islam), but those
human beings stuck to their (old) religion. Al-Amash said extra: 'Say, (O Muhammad): Call
unto those besides Him whom you assume (to be gods).' (17.56) Volume 6, Book 60,
Number 238. Narrated Abdullah.
i. Such a statement is only a conjecture. Jesus worshiped YHWH (Yahweh). See John 17.
5. Muhammad says to say "Jesus is Lord," is a great sin.
A. Whenever Ibn 'Umar was asked about marrying a Christian lady or a Jewess, he would say:
"Allah has made it unlawful for the believers to marry ladies who ascribe partners in worship
to Allah, and I do not know of a greater thing, as regards to ascribing partners in worship,
etc. to Allah, than that a lady should say that Jesus is her Lord although he is just one of
Allah's slaves." Volume 7, Book 63, Number 209. Narrated Nafi'.
i. It is not a great sin and it is taught in the Bible that such a confession is necessary a
sign of salvation (Rom. 10:9-10).
6. Muhammad said Jesus spoke while in the cradle
A. The Prophet said, "None spoke in cradle but three: (The first was) Jesus, (the second was),
there a man from Bani Israel called Juraij. Volume 4, Book 55, Number 645, Narrated Abu
Huraira.

The preceding Hadith are translated by Sahih Bukhari and are quoted from and online source
found at http://www.usc.edu/dept/MSA/fundamentals/hadithsunnah/bukhari/ .
Interesting Quotes from the Hadith on Jihad

Jihad means striving. It is used to describe the inner struggle against sin as well as the outer
struggle against those who would oppose Islam. Did Muhammad teach Holy War upon those who
reject Islam? Yes he did.
Following are various quotes from the Hadith dealing with killing people.

1. The second best deed is to participate in Jihad


A. Allah's Apostle was asked, "What is the best deed?" He replied, "To believe in Allah and His
Apostle (Muhammad). The questioner then asked, "What is the next (in goodness)? He
replied, "To participate in Jihad (religious fighting) in Allah's Cause." The questioner again
asked, "What is the next (in goodness)?" He replied, "To perform Hajj (Pilgrim age to Mecca)
'Mubrur, (which is accepted by Allah and is performed with the intention of seeking Allah's
pleasure only and not to show off and without committing a sin and in accordance with the
traditions of the Prophet)." Volume 1, Book 2, Number 25, Narrated Abu Huraira:
2. Muhammad said if someone leaves Islam, to kill him
A. Ali burnt some people and this news reached Ibn 'Abbas, who said, "Had I been in his place I
would not have burnt them, as the Prophet said, 'Don't punish (anybody) with Allah's
Punishment.' No doubt, I would have killed them, for the Prophet said, 'If somebody (a
Muslim) discards his religion, kill him.'" Volume 4, Book 52, Number 260, Narrated Ikrima.
Also, see Volume 9, Book 84, Number 64, Narrated 'Ali.
3. Muhammad approves of killing someone who hurt him and having the killer lie
A. "Allah's Apostle said, "Who is willing to kill Ka'b bin Al-Ashraf who has hurt Allah and His
Apostle?" Thereupon Muhammad bin Maslama got up saying, "O Allah's Apostle! Would you
like that I kill him?" The Prophet said, "Yes," Muhammad bin Maslama said, "Then allow me
to say a (false) thing (i.e. to deceive Kab). "The Prophet said, "You may say it." Then
Muhammad bin Maslama went to Kab and said, "That man (i.e. Muhammad demands Sadaqa
(i.e. Zakat) from us, and he has troubled us, and I have come to borrow something from
you." On that, Kab said, "By Allah, you will get tired of him!" Muhammad bin Maslama said,
"Now as we have followed him, we do not want to leave him unless and until we see how his
end is going to be. .." Volume 5, Book 59, Number 369, Narrated Jabir bin 'Abdullah.
4. Paradise was guaranteed to the first to take part in a naval battle
A. That 'Umair bin Al-Aswad Al-Anasi told him that he went to 'Ubada bin As-Samit while he
was staying in his house at the sea-shore of Him with (his wife) Um Haram. 'Umair said. Um
Haram informed us that she heard the Prophet saying, "Paradise is granted to the first batch
of my followers who will undertake a naval expedition." Um Haram added, I said, 'O Allah's
Apostle! Will I be amongst them?' He replied, 'You are amongst them.' The Prophet then
said, 'The first army amongst' my followers who will invade Caesar's City will be forgiven
their sins.' I asked, 'Will I be one of them, O Allah's Apostle?' He replied in the
negative." Volume 4, Book 52, Number 175 Narrated Khalid bin Madan:
5. Those who fight in Jihad have the right to the spoils of the conquered or Paradise if he
dies
A. The Prophet said, "The person who participates in (Holy battles) in Allah's cause and nothing
compels him to do so except belief in Allah and His Apostles, will be recompensed by Allah
either with a reward, or booty (if he survives) or will be admitted to Paradise (if he is killed
in the battle as a martyr). Had I not found it difficult for my followers, then I would not
remain behind any sariya going for Jihad and I would have loved to be martyred in Allah's
cause and then made alive, and then martyred and then made alive, and then again
martyred in His cause." Volume 1, Book 2, Number 35, Narrated Abu Huraira.
B. ". . . "This is the Will of Allah, "After the people returned, the Prophet sat and said, "Anyone
who has killed an enemy and has a proof of that, will posses his spoils." I got up and said,
"Who will be a witness for me?" and then sat down. The Prophet again said, "Anyone who
has killed an enemy and has proof of that, will possess his spoils." I (again) got up and said,
"Who will be a witness for me?" and sat down. Then the Prophet said the same for the third
time. I again got up, and Allah's Apostle said, "O Abu Qatada! What is your story?" Then I
narrated the whole story to him. A man (got up and) said, "O Allah's Apostle! He is speaking
the truth, and the spoils of the killed man are with me. So please compensate him on my
behalf." On that Abu Bakr As-Siddiq said, "No, by Allah, he (i.e. Allah's Apostle ) will not
agree to give you the spoils gained by one of Allah's Lions who fights on the behalf of Allah
and His Apostle." The Prophet said, "Abu Bakr has spoken the truth." So, Allah's Apostle
gave the spoils to me. I sold that armor (i.e. the spoils) and with its price I bought a garden
at Bani Salima, and this was my first property which I gained after my conversion to Islam."
Volume 4, Book 53, Number 370, Narrated Abu Qatada.
C. ". . . . While we were in this state, the Lord of the Heavens and the Earths, Elevated is His
Remembrance and Majestic is His Highness, sent to us from among ourselves a Prophet
whose father and mother are known to us. Our Prophet, the Messenger of our Lord, has
ordered us to fight you till you worship Allah Alone or give Jizya (i.e. tribute); and our
Prophet has informed us that our Lord says:-- Whoever amongst us is killed (i.e. martyred),
shall go to Paradise to lead such a luxurious life as he has never seen, and whoever amongst
us remain alive, shall become your master." (Al-Mughira, then blamed An-Numan for
delaying the attack and) An-Nu' man said to Al-Mughira, "If you had participated in a similar
battle, in the company of Allah's Apostle he would not have blamed you for waiting, nor
would he have disgraced you. But I accompanied Allah's Apostle in many battles and it was
his custom that if he did not fight early by daytime, he would wait till the wind had started
blowing and the time for the prayer was due (i.e. after midday)." Volume 4, Book 53,
Number 386, Narrated Jubair bin Haiya.
D. Allah's Apostle said, "Allah guarantees (the person who carries out Jihad in His Cause and
nothing compelled him to go out but Jihad in His Cause and the belief in His Word) that He
will either admit him into Paradise (Martyrdom) or return him with reward or booty he has
earned to his residence from where he went out." Volume 9, Book 93, Number 555,
Narrated Abu Huraira:

The preceding Hadith are translated by Sahih Bukhari and are quoted from and online source
found at http://www.usc.edu/dept/MSA/fundamentals/hadithsunnah/bukhari/ .
Interesting quotes from the Hadith about Muhammad

1. Muhammad was a white man


A. While we were sitting with the Prophet in the mosque, a man came riding on a camel. He
made his camel kneel down in the mosque, tied its foreleg and then said: "Who amongst you
is Muhammad?" At that time the Prophet was sitting amongst us (his companions) leaning
on his arm. We replied, "This white man reclining on his arm." The an then addressed him,
"O Son of 'Abdul Muttalib." Volume 1, Book 3, Number 63. Narrated Anas bin Malik:
2. Muhammad owned a black slave
A. I came and behold, Allah's Apostle was staying on a Mashroba (attic room) and a black slave
of Allah's Apostle was at the top if its stairs. I said to him, "(Tell the Prophet) that here is
'Umar bin Al-Khattab (asking for permission to enter)." Then he admitted me. Volume 9,
Book 91, Number 368. Narrated 'Umar:
3. Muhammad had people killed
A. Allah's Apostle entered Mecca in the year of its Conquest wearing an Arabian helmet on his
head and when the Prophet took it off, a person came and said, "Ibn Khatal is holding the
covering of the Ka'ba (taking refuge in the Ka'ba)." The Prophet said, "Kill him." Volume 3,
Book 29, Number 72. Narrated Anas bin Malik:
4. Muhammad said to die in battle for Allah grants you Paradise
A. The Prophet said, "The person who participates in (Holy battles) in Allah's cause and nothing
compels him to do so except belief in Allah and His Apostles, will be recompensed by Allah
either with a reward, or booty (if he survives) or will be admitted to Paradise (if he is killed
in the battle as a martyr). Volume 1, Book 2, Number 35. Narrated Abu Huraira:
5. Muhammad was a sinner
A. Allah's Apostle used to keep silent between the Takbir and the recitation of Qur'an and that
interval of silence used to be a short one. I said to the Prophet "May my parents be
sacrificed for you! What do you say in the pause between Takbir and recitation?" The
Prophet said, "I say, 'Allahumma, ba'id baini wa baina khatayaya kama ba'adta baina-l-
mashriqi wa-l-maghrib. Allahumma, naqqim min khatayaya kama yunaqqa-ththawbu-l-
abyadu mina-ddanas. Allahumma, ighsil khatayaya bil-ma'i wa-th-thalji wal-barad (O Allah!
Set me apart from my sins (faults) as the East and West are set apart from each other and
clean me from sins as a white garment is cleaned of dirt (after thorough washing). O Allah!
Wash off my sins with water, snow and hail.)" Volume 1, Book 12, Number 711. Narrated
Abu Huraira:
B. I heard Allah's Apostle saying." By Allah! I ask for forgiveness from Allah and turn to Him in
repentance more than seventy times a day." Volume 8, Book 75, Number 319, Narrated
Abu Huraira:
6. Muhammad said more women were in hell than men and that women lacked
intelligence.
A. Once Allah's Apostle went out to the Musalla (to offer the prayer) o 'Id-al-Adha or Al-Fitr
prayer. Then he passed by the women and said, "O women! Give alms, as I have seen that
the majority of the dwellers of Hell-fire were you (women)." They asked, "Why is it so, O
Allah's Apostle ?" He replied, "You curse frequently and are ungrateful to your husbands. I
have not seen anyone more deficient in intelligence and religion than you. A cautious
sensible man could be led astray by some of you." The women asked, "O Allah's Apostle!
What is deficient in our intelligence and religion?" He said, "Is not the evidence of two
women equal to the witness of one man?" They replied in the affirmative. He said, "This is
the deficiency in her intelligence. Isn't it true that a woman can neither pray nor fast during
her menses?" The women replied in the affirmative. He said, "This is the deficiency in her
religion." Volume 1, Book 6, Number 301, Narrated Abu Said Al-Khudri:
7. Muhammad had some people drink camel urine as medicine.
A. Some people from the tribe of 'Ukl came to the Prophet and embraced Islam. The climate of
Medina did not suit them, so the Prophet ordered them to go to the (herd of milch) camels of
charity and to drink, their milk and urine (as a medicine). They did so, and after they had
recovered from their ailment (became healthy) they turned renegades (reverted from Islam)
and killed the shepherd of the camels and took the camels away. The Prophet sent (some
people) in their pursuit and so they were (caught and) brought, and the Prophets ordered
that their hands and legs should be cut off and that their eyes should be branded with
heated pieces of iron, and that their cut hands and legs should not be cauterized, till they
die. Volume 8, Book 82, Number 794, Narrated Anas.
8. Muhammad has his chest opened and washed by the angel Gabriel
A. Allah's Apostle said, "While I was at Mecca the roof of my house was opened and Gabriel
descended, opened my chest, and washed it with Zam-zam water. Then he brought a golden
tray full of wisdom and faith and having poured its contents into my chest, he closed it. Then
he took my hand and ascended with me to the nearest heaven, when I reached the nearest
heaven, Gabriel said to the gatekeeper of the heaven, 'Open (the gate).' The gatekeeper
asked, 'Who is it?' Gabriel answered: 'Gabriel.' He asked, 'Is there anyone with you?' Gabriel
replied, 'Yes, Muhammad is with me.' He asked, 'Has he been called?' Gabriel said, 'Yes.' So
the gate was opened and we went over the nearest heaven and there we saw a man sitting
with some people on his right and some on his left. When he looked towards his right, he
laughed and when he looked toward his left he wept. Then he said, 'Welcome! O pious
Prophet and pious son.' I asked Gabriel, 'Who is he?' He replied, 'He is Adam and the people
on his right and left are the souls of his offspring. Those on his right are the people of
Paradise and those on his left are the people of Hell and when he looks towards his right he
laughs and when he looks towards his left he weeps.' Volume 1, Book 8, Number 345,
Narrated Abu Dhar:

The preceding Hadith are translated by Sahih Bukhari and are quoted from and online source
found at http://www.usc.edu/dept/MSA/fundamentals/hadithsunnah/bukhari/ .

_______________
Note: I am indebted to Morey, Robert, The Islamic Invasion, Harvest House Publishers, Eugene
Oregon, 1992, pages 177-208, for many references in the Hadith concerning Muhammad.
Interesting Quotes from the Hadith on Satan

1. Muhammad said that yawning is from Satan.


A. The Prophet said, "Yawning is from Satan and if anyone of you yawns, he should check his
yawning as much as possible, for if anyone of you (during the act of yawning) should say:
'Ha', Satan will laugh at him." Volume 4, Book 54, Number 509: Narrated Abu Huraira:
2. Muhammad said that yawning is from Satan and that Allah likes sneezing and dislikes
yawning.
A. The Prophet said, "Allah likes sneezing and dislikes yawning, so if someone sneezes and then
praises Allah, then it is obligatory on every Muslim who heard him, to say: May Allah be
merciful to you (Yar-hamuka-l-lah). But as regards yawning, it is from Satan, so one must
try one's best to stop it, if one says 'Ha' when yawning, Satan will laugh at him." Volume 8,
Book 73, Number 242: Narrated Abu Huraira: See also Volume 8, Book 73, Number 245:,
Narrated Abu Huraira:
3. Muhammad said a good dream is from Allah and a bad dream is from Satan.
A. The Prophet said, "A good dream is from Allah, and a bad or evil dream is from Satan; so if
anyone of you has a bad dream of which he gets afraid, he should spit on his left side and
should seek Refuge with Allah from its evil, for then it will not harm him." Volume 4, Book
54, Number 513: Narrated Abu Qatada:
B. I heard the Prophet saying, "A good dream is from Allah, and a bad dream is from Satan. So
if anyone of you sees (in a dream) something he dislikes, when he gets up he should blow
thrice (on his left side) and seek refuge with Allah from its evil for then it will not harm
him." Volume 7, Book 71, Number 643: Narrated Abu Qatada:
C. The Prophet said, "A good dream is from Allah, and a bad dream is from Satan. So whoever
has seen (in a dream) something he dislike, then he should spit without saliva, thrice on his
left and seek refuge with Allah from Satan, for it will not harm him, and Satan cannot appear
in my shape." Volume 9, Book 87, Number 124: Narrated Abu Qatada:
4. Muhammad said that Satan stayed in the upper part of a nose all night.
A. The Prophet said, "If anyone of you rouses from sleep and performs the ablution, he should
wash his nose by putting water in it and then blowing it out thrice, because Satan has stayed
in the upper part of his nose all the night." Volume 4, Book 54, Number 516: Narrated Abu
Huraira:
5. Muhammad said a cock crows because it has seen an angel and donkeys bray because
they have seen Satan.
A. The Prophet said, "When you hear the crowing of cocks, ask for Allah's Blessings for (their
crowing indicates that) they have seen an angel. And when you hear the braying of donkeys,
seek Refuge with Allah from Satan for (their braying indicates) that they have seen a
Satan." Volume 4, Book 54, Number 522: Narrated Abu Huraira:
6. Muhammad said that Satan touches all human offspring making them cry.
A. Abu Huraira said, "I heard Allah's Apostle saying, 'There is none born among the off-spring
of Adam, but Satan touches it. A child therefore, cries loudly at the time of birth because of
the touch of Satan, except Mary and her child." Then Abu Huraira recited: "And I seek refuge
with You for her and for her offspring from the outcast Satan" (3.36) Volume 4, Book 55,
Number 641:Narrated Said bin Al-Musaiyab:Also, same thing at Volume 6, Book 60, Number
71:, Narrated Said bin Al-Musaiyab:
7. Muhammad said that devils spread out at night time and that Satan does not open a
closed door.
A. Allah's Apostle said, "When night falls (or when it is evening), stop your children from going
out, for the devils spread out at that time. But when an hour of the night has passed, release
them and close the doors and mention Allah's Name, for Satan does not open a closed door.
Tie the mouth of your water-skin and mention Allah's Name; cover your containers and
utensils and mention Allah's Name. Cover them even by placing something across it, and
extinguish your lamps." Volume 7, Book 69, Number 527: Narrated Jabir bin 'Abdullah

8. Muhammad said a child born from a union where a parent said a "phrase," Satan cannot
harm the child.
A. The Prophet said, "If anyone of you, when intending to have a sexual intercourse with his
wife, says: 'Bismillah, Allahumma jannibna-sh-shaitan, wa jannibi-sh-shaitan ma razaqtana,'
and if the couple are destined to have a child (out of that very sexual relation), then Satan
will never be able to harm that child." Volume 8, Book 75, Number 397: Narrated Ibn
'Abbas:
9. Muhammad said that during sleep Satan ties knots at the back of the head.
A. Allah's Apostle said, "During your sleep, Satan knots three knots at the back of the head of
each of you, and he breathes the following words at each knot, 'The night is, long, so keep
on sleeping,' If that person wakes up and celebrates the praises of Allah, then one knot is
undone, and when he performs ablution the second knot is undone, and when he prays, all
the knots are undone, and he gets up in the morning lively and gay, otherwise he gets up
dull and gloomy." Volume 4, Book 54, Number 491: Narrated Abu Huraira:
10. Muhammad said that Satan peed in the ear of a man.
A. It was mentioned before the Prophet that there was a man who slept the night till morning
(after sunrise). The Prophet said, "He is a man in whose ears (or ear) Satan had urinated."
Volume 4, Book 54, Number 492: Narrated 'Abdullah:
11. Muhammad said Satan can not impersonate him in a dream.
A. The Prophet said, "Name yourselves with my name (use my name) but do not name
yourselves with my Kunya name (i.e. Abu-l Qasim). And whoever sees me in a dream then
surely he has seen me for Satan cannot impersonate me. And whoever tells a lie against me
(intentionally), then (surely) let him occupy his seat in Hell-fire." Volume 1, Book 3, Number
110: Narrated Abu Huraira: Also at Volume 8, Book 73, Number 217: Narrated Abu Huraira:
12. Muhammad said earthquakes and afflictions come out of the side of the head of Satan.
A. (The Prophet) said, "O Allah! Bless our Sham and our Yemen." People said, "Our Najd as
well." The Prophet again said, "O Allah! Bless our Sham and Yemen." They said again, "Our
Najd as well." On that the Prophet said, "There will appear earthquakes and afflictions, and
from there will come out the side of the head of Satan." Volume 2, Book 17, Number 147:
Narrated Ibn 'Umar:
13. Muhammad choked Satan
A. The Prophet once offered the prayer and said, "Satan came in front of me and tried to
interrupt my prayer, but Allah gave me an upper hand on him and I choked him. No doubt, I
thought of tying him to one of the pillars of the mosque till you get up in the morning and
see him. Then I remembered the statement of Prophet Solomon, 'My Lord ! Bestow on me a
kingdom such as shall not belong to any other after me.' Then Allah made him (Satan)
return with his head down (humiliated)." Volume 2, Book 22, Number 301, Narrated Abu
Huraira
14. Muhammad said Satan passes wind and runs so he may not hear the Adhan (call to
prayer).
A. Allah's Apostle said, "When the Adhan for the prayer is pronounced, then Satan takes to his
heels passing wind so that he may not hear the Adhan and when the Muadh-dhin finishes, he
comes back; Volume 2, Book 22, Number 313: Narrated Abu Huraira,
B. Allah's Apostle said, "When the call for prayer is made, Satan takes to his heels passing wind
so that he may not hear the Adhan and when the call is finished he comes back, and when
the Iqama is pronounced, Satan again takes to his heels, Volume 2, Book 22, Number 323,
Narrated Abu Huraira:
15. Muhammad said that when praying while standing, Satan puts doubts into the person.
A. Allah's Apostle said, "When anyone of you stands for the prayers, Satan comes and puts him
in doubts till he forgets how many Rakat he has prayed. So if this happens to anyone of you,
he should perform two prostrations of Sahu while sitting. Volume 2, Book 22, Number 324:
Narrated Abu Huraira:
16. Muhammad said that Satan can reach into a person's body the same as blood reaches it.
A. Safiya, the wife of the Prophet told me that she went to Allah's Apostle to visit him in the
mosque while he was in Itikaf in the last ten days of Ramadan. She had a talk with him for a
while, then she got up in order to return home. The Prophet accompanied her. When they
reached the gate of the mosque, opposite the door of Um-Salama, two Ansari men were
passing by and they greeted Allah's Apostle . He told them: Do not run away! And said, "She
is (my wife) Safiya bint Huyai." Both of them said, "Subhan Allah, (How dare we think of any
evil) O Allah's Apostle!" And they felt it. The Prophet said (to them), "Satan reaches
everywhere in the human body as blood reaches in it, (everywhere in one's body). I was
afraid lest Satan might insert an evil thought in your minds." Volume 3, Book 33, Number
251: Narrated Ali bin Al-Husain: Also, see 254
17. Muhammad said affliction comes out of the side of Satan's head.
A. The Prophet stood up and delivered a sermon, and pointing to 'Aisha's house (i.e.
eastwards), he said thrice, "Affliction (will appear from) here," and, "from where the side of
the Satan's head comes out (i.e. from the East)." Volume 4, Book 53, Number 336:
Narrated 'Abdullah:
B. I saw Allah's Apostle pointing towards the east saying, "Lo! Afflictions will verily emerge
hence; afflictions will verily emerge hence where the (side of the head of) Satan appears."
Volume 4, Book 54, Number 499: Narrated 'Abdullah bin 'Umar:

The preceding Hadith are translated by Sahih Bukhari and are quoted from and online source
found at http://www.usc.edu/dept/MSA/fundamentals/hadithsunnah/bukhari/ .
Bibliography

_____, The Holy Qur'an, Mushaf Al-Madinah An-Nabawiyah, Revised and Edited by THE PRESIDENCY
OF ISLAMIC RESEARCH, IFTA, Call and Guidance, the Custodian of the Two Holy Mosques King Fahd
Complex, for the Printing of the Holy Qur-an. (The Custodian of the Two Holy Mosques King Fahd Ibn
Abdul Aziz Al- Saud, King of the Kingdom of Saudi Arabia, Has the Honour to Order the Printing of This
Holy Qur-An And the Translation of Its Meanings and Commentary.)
_____, The Koran, translated by M.H. Shakir, Online searchable Koran found at Humanities Text
Initiative. Published by Tahrike Tarsile Qur'an, Inc., in 1983.
_____, The Quran, translated by Abdullah Yusufali. Online searchable Koran found at the Scholarly
Technology Group, The Islamic Computing Centre, London, distributed by Mohammad Jamil Sawar,
CBLU, sawar@cbl.leeds.ac.uk, Leeds University, Leeds LS2 9JT, UK, per Web page.
_____, The Qur'an, online version, an Authorized English Version, Translated from the original by Dr.
Rashad Khalifa, Ph.D
Ali, Maulana Muhammad, M.A., LL.B., A Manual of Hadith, The Ahmadiyya Anjuman Ishaat Islam,
Lahore.
Grisell, Dr. Ronald, Sufism, Ross Books, Berkely, CA., 1983.Mushaf
Miller, William M., A Christian's Response to Islam, Presbyterian and Reformed Publishing, Phillipsburg,
New Jersey, 1976.
Geisler, Norman, Baker Encyclopedia of Christian Apologetics, Grand Rapids, Michigan, Baker Books,
1999.
Glasse, Cyril, The Concise Encyclopedia of Islam, Harper & Row, Publishers, Inc. San Francisco, 1989.
Hahn, Ernest, How To Respond to Muslims, CPH Publishers, St. Louis, 1984.
Ismaeel, Saeed, Al-Harmain Islamic Foundation, Kingdom of Saudi Arabia, Riyadh, 1995.
Morey, Robert, The Islamic Invasion, Harvest House Publishers, Eugene Oregon, 1992.
Naipaul, V. S., Beyond Belief, Vintage Books, New York, 1998.
Peters, Rudolph, Jihad in Classical and Modern Islam, Markus Wiener Publishers, Princeton, NJ, 1996.
Watt, W. Montgomery, Islamic Surveys: Bell's Introduction to the Qur'an, Aldine Publishing Company,
Chicago, 1970.
Heresies

Introduction

Heresies abound throughout church history. The ancient errors are alive and well today as well? Do
you know which present day groups are represented by the following heresies?

1. What is arianism? p. 179


2. What is gnosticism? p. 181
3. What is modalism? p. 182
4. What is pelagianism? p. 184
Adoptionism

Adoptionism is an error concerning Christ that first appeared in the second century. Those who held
it denied the preexistence of Christ and, therefore, His deity. Adoptionists taught that Jesus was tested
by God and after passing this test and upon His baptism, He was granted supernatural powers by God
and adopted as the Son. As a reward for His great accomplishments and perfect character Jesus was
raised from the dead and adopted into the Godhead.
This error arose out of an attempt by people to understand the two natures of Jesus. The scriptures
tell us that Jesus is both God and man: "for Him dwells all the fullness of deity in bodily form," (Col.
2:9). This is known as the doctrine of the Hypostatic Union where in the one person of Christ, there
are two natures: God and man.
Theodotus of Byzantium was the most prominent adherent to this error.
Adoptionism was condemned as a heresy by Pope Victor (A.D. 190-198).

8th Century revision

Adoptionism was later revived in the 8th Century in Spain by Elipandus, archbishop of Toledo, and
Felix, bishop of Urgel. This was a variation of the first error but it held that Christ was the Son of God
in respect to his divine nature, but that as a man, he was only adopted as the first born of God.
In 798 Pope Leo III held a council at Rome that condemned adoptionism as a heresy.

Albigenses

A heresy during the middle ages that developed in the town Albi in Southern France. This error
taught that there were two gods: the good god of light usually referred to as Jesus in the New
Testament and the god of darkness and evil usually associated with Satan and the "God of the Old
Testament." Anything material was considered evil including the body which was created by Satan.
The soul, created by the good god, was imprisoned in the evil flesh and salvation was possible only
through holy living and doing good works. At death, if the person has been spiritual enough, salvation
comes to the believer. But, if the person has not been good enough, he is reincarnated as an animal
or another human. The Albigenses denied the resurrection of the body since it was considered evil.
The Albigenses taught that Jesus was God but that He only appeared as a man while on earth. It
also taught that the Catholic church of the time was corrupted by its power and wealth. Their
asceticism and humility compared to the great affluence of the clergy helped to bring many converts to
this evangelistic movement.
There were two types of Albigenses: believers and Perfects. Believers were Albigenses who had
not taken the initiation rite of being a Perfect. Perfects denounced all material possession. They
abstained from meat, milk, cheese, eggs, and sexual relations. To become a Perfect a believer had to
go through consolamentum, an initiation rite involving the laying on of hands that was supposed to
bring the baptism of the Holy Spirit. Infrequently, suicide was practiced as a way to rid oneself of the
evil human body.
In 1208, Peter de Castelnau, an official representative of the Pope, was murdered by an
Albigenses. Since they had been growing in number, becoming a threat, and would not convert to
Christianity, Pope Innocent III ordered them to be wiped out. The persecution was fierce and the
movement was stopped.
Apollinarianism

Apollinarianism was the heresy taught by Apollinaris the Younger, bishop of Laodicea in Syria about
361. He taught that the Logos of God, which became the divine nature of Christ, took the place of the
rational human soul of Jesus and that the body of Christ was a glorified form of human nature. In
other words, though Jesus was a man, He did not have a human mind but that the mind of Christ was
solely divine. Apollinaris taught that the two natures of Christ could not coexist within one person. His
solution was to lessen the human nature of Christ.
Apollinarianism was condemned by the Second General Council at Constantinople in 381. This
heresy denies the true and complete humanity in the person of Jesus which in turn, can jeopardize the
value of the atonement since Jesus is declared to be both God and man to atone. He needed to be
God to offer a pure and holy sacrifice of sufficient value and He needed to be a man in order to die for
men.
Jesus is completely both God and man. This is known as the Hypostatic Union.

• "In the beginning was the Word, and the Word was with God, and the Word was God...and the
Word became flesh and dwelt among us..." (John 1:1,14).
• "for Him dwells all the fullness of deity in bodily form," (Col. 2:9).

Arianism

Arianism developed around 320, in Alexandria Egypt concerning the person of Christ and is named
after Arius of Alexandar. For his doctrinal teaching he was exiled to Illyria in 325 after the first
ecumenical council at Nicaea condemned his teaching as heresy. It was the greatest of heresies within
the early church that developed a significant following. Some say, it almost took over the church.
Arius taught that only God the Father was eternal and too pure and infinite to appear on the earth.
Therefore, God produced Christ the Son out of nothing as the first and greatest creation. The Son is
then the one who created the universe. Because the Son relationship of the Son to the Father is not
one of nature, it is, therefore, adoptive. God adopted Christ as the Son. Though Christ was a
creation, because of his great position and authority, he was to be worshipped and even looked upon
as God. Some Arians even held that the Holy Spirit was the first and greatest creation of the Son.
At Jesus' incarnation, the Arians asserted that the divine quality of the Son, the Logos, took the
place of the human and spiritual aspect of Jesus, thereby denying the full and complete incarnation of
God the Son, second person of the Trinity.
In asserting that Christ the Son, as a created thing, was to be worshipped, the Arians were
advocating idolatry.
Docetism
Docetism was an error with several variations concerning the nature of Christ. Generally, it taught
that Jesus only appeared to have a body, that he was not really incarnate, (Greek, "dokeo" = "to
seem"). This error developed out of the dualistic philosophy which viewed matter as inherently evil,
that God could not be associated with matter, and that God, being perfect and infinite, could not
suffer. Therefore, God as the word, could not have become flesh per John 1:1,14, "In the beginning
was the Word, and the Word was with God, and the Word was God...And the Word became flesh, and
dwelt among us.. " This denial of a true incarnation meant that Jesus did not truly suffer on the cross
and that He did not rise from the dead.
The basic principle of Docetism was refuted by the Apostle John in 1 John 4:2-3. "By this you
know the Spirit of God: every spirit that confesses that Jesus Christ has come in the flesh is from God;
3
and every spirit that does not confess Jesus is not from God; and this is the spirit of the antichrist, of
which you have heard that it is coming, and now it is already in the world." Also, 2 John 7, "For many
deceivers have gone out into the world, those who do not acknowledge Jesus Christ as coming in the
flesh. This is the deceiver and the antichrist."
Ignatius of Antioch (died 98/117) and Irenaeus (115-190), and Hippolatus (170-235) wrote against
the error in the early part of the second century.
Docetism was condemned at the Council of Chalcedon in 451.

Donatism
Donatism was the error taught by Donatus, bishop of Casae Nigrae that the effectiveness of the
sacraments depends on the moral character of the minister. In other words, if a minister who was
involved in a serious enough sin were to baptize a person, that baptism would be considered invalid.
Donatism developed as a result of the persecution of Christians ordered by Diocletian in 303 in
which all churches and sacred scriptures of the Christians were to be destroyed. In 304 another edict
was issued ordering the burning of incense to the idol gods of the Roman Empire. Of course,
Christians refused, but it did not curtail the increased persecution. Many Christians gave up the sacred
texts to the persecutors and even betrayed other Christians to the Romans. These people became
known as "traditors," Christians who betrayed other Christians. (Note: traditor, not traitor)
At the consecration of bishop Caecilian of Carthage in 311, one of the three bishops, Felix, bishop
of Aptunga, who consecrated Caecilian, had given copies of the Bible to the Roman persecutors. A
group of about 70 bishops formed a synod and declared the consecration of the bishop to be invalid.
Great debate arose concerning the validity of the sacraments (baptism, the Lord's Supper, etc.) by one
who had sinned so greatly against other Christians.
Ater the death of Caecilian, Aelius Donatus the Great became bishop of Carthage and it is from his
name that the movement is called. The Donatists were gaining "converts" to their cause and a division
was arising in the Catholic Church. They began to practice rebaptism which was particularly
troublesome to the church at the time and was condemned at the Synod of Arles in 314 since it
basically said the authority in the Catholic Church was lost.
The Donatist issue was raised at several ecumenical councils and finally submitted to Emporer
Constantine in 316. In each case the consecration of bishop Caecilian was upheld. However,
persecution fuels emotions and by 350 the Donatists had gained many converts and outnumbered the
Orthodox in Africa. But it was the apologetic by Augustine that turned the tide against the Donatist
movement which eventually died out in the next century.

The problem with Donatism is that no person is morally pure. The effectiveness of the baptism or
administration of the Lord's Supper does not cease to be effective if the moral character of the
minister is in question or even demonstrated to be faulty. Rather, the sacraments are powerful
because of what they are, visible representations of spiritual realities. God is the one who works in
and through them and He is not restricted by the moral state of the administrant.
Gnosticism

Gnosticism traces its roots back just after the beginning of the Christian Church. Some researchers
state that evidence of its existence even predates Christianity. Whichever the case, the error of
gnosticism had affected the culture and church of the time and possibly even a earned a mention in 1
John 4.
The word "gnosticism" comes from the Greek word "gnosis" which means "knowledge." There were
many groups that were Gnostic and it isn't possible to easily describe the nuances of each variant of
Gnostic doctrines. However, generally speaking, Gnosticism taught that salvation is achieved through
special knowledge (gnosis). This knowledge usually dealt with the individual's relationship to the
transcendent Being.
A more detailed Gnostic theology is as follows. The unknowable God was far too pure and perfect
to have anything to do with the material universe which was considered evil. Therefore, God
generated lesser divinities, or emenations. One of these emanations, Wisdom desired to know the
unknowable God. Out of this erring desire the demiurge an evil god was formed and it was this evil
god that created the universe. He along with archons kept the mortals in bondage in material matter
and tried to prevent the pure spirit souls from ascending back to god after the death of the physical
bodies. Since, according to the Gnostics, matter is evil, deliverance from material form was attainable
only through special knowledge revealed by special Gnostic teachers. Christ was the divine redeemer
who descended from the spiritual realm to reveal the knowledge necessary for this redemption. In
conclusion, Gnosticism is dualistic. That is, it teaches there is a good and evil, spirit and matter, light
and dark, etc. dualism in the universe.
What we know about Gnosticism is gained from the writings of Irenaeus, Hippolytus, Tertullian,
Origen, and some later manuscripts discovered in the eighteenth century such as the "Codex Askew,
Codex Bruce, the Berlin Gnostic Codes and, most recently, the Nag Hammadi collection." 195 Nag
Hammadi is a town in Upper Egypt near ancient Chenoboskion and 13 codices discovered were
discovered about 1945.
The danger of gnosticism is easily apparent. It denies the incarnation of God as the Son. In so
doing, it denies the true efficacy of the atonement since, if Jesus is not God, He could not atone for all
of mankind and we would still be lost in our sins.
There is debate whether or not this is a Christian heresy or simply an independent development.
The evidence seems to point to the later. Nevertheless, the Gnostics laid claim to Jesus as a great
teacher of theirs and as such requires some attention. It is possible that 1 John was written against
some of the errors that Gnosticism promoted.

195
Achtemeier, Paul J., Th.D., Harper’s Bible Dictionary, (San Francisco: Harper and Row, Publishers, Inc.) 1985.
Kenosis

"Kenosis" is derived from the Greek word "kenoo" which means "to empty." It is used in Phil. 2:7.
The text of Phil. 2:5-8 is worth recording here.

"Have this attitude in yourselves which was also in Christ Jesus, 6who, although He existed in the form
of God, did not regard equality with God a thing to be grasped, 7but emptied Himself, taking the form
of a bond-servant, and being made in the likeness of men. 8And being found in appearance as a man,
He humbled Himself by becoming obedient to the point of death, even death on a cross," (Phil. 2:5-8).

The kenosis theory states that Jesus gave up some of His divine attributes while He was a man
here on earth. These attributes were omniscience, omnipresence, and omnipotence. Christ did this
voluntarily so that He could function as a man in order to fulfill the work of redemption. This view was
first introduced in the late 1800s in Germany with Gottfried Thomasius (1802-75), a Lutheran
theologian.
Phil. 2:5-8 does not teach that Jesus gave up any of His divine attributes since it says nothing of
those attributes. Instead, it is speaking of His humility that moved him, according to the will of the
Father, to leave His majestic state in heaven and enter into the humble position of human nature.
There is, however, a problem the orthodox must deal with that the Kenosis theory seems to more
adequately address. Take Mark 13:32 for example. In it, Jesus said, "But of that day or hour no one
knows, not even the angels in heaven, nor the Son, but the Father alone." If Jesus knew all things, as
is implied in His divine nature, then why did He not know the day or hour of His own return. The
answer is that Jesus cooperated with the limitations of humanity and voluntarily did not exercise His
attribute of omniscience. He still was divine but was moving and living completely as a man.
The Kenosis theory is a dangerous doctrine because if it were true then it would mean that Jesus
was not fully divine. If Jesus was not fully divine, then His atoning work would not be sufficient to
atone for the sins of the world.
The correct doctrine is the Hypostatic Union, that Jesus is both fully God and fully man (Col. 2:9)
and did not give up any divine attributes while as a man on earth.

Modalism

Modalism is probably the most common theological error concerning the nature of God. It is a
denial of the Trinity which states that God is a single person who, throughout biblical history, has
revealed Himself in three consecutive modes, or forms. Thus, God is a single person who first
manifested himself in the mode of the Father in Old Testament times. At the incarnation, the mode
was the Son. After Jesus' ascension, the mode is the Holy Spirit. These modes are consecutive and
never simultaneous. In other words, the Father, the Son, and the Holy Spirit never all exist at the
same time, only one after another. Modalism denies the distinctiveness of the three persons in the
Trinity even though it retains the divinity of Christ.
Present day groups that hold to this error are the United Pentecostal and United Apostolic
Churches. They deny the Trinity, teach that the name of God is Jesus, and require baptism for
salvation. These modalist churches often accuse Trinitarians of teaching three gods. This is not what
the Trinity is. The correct teaching of the Trinity is one God in three eternal coexistent persons: The
Father, the Son, and the Holy Spirit.
Monarchianism

Monarchianism (mono - "one"; arche - "rule") was an error concerning the nature of God that
developed in the second century A.D. It arose as an attempt to maintain Monotheism and refute
tritheism. Unfortunately, it also contradicts the orthodox doctrine of the Trinity. Monarchianism
teaches that there is one God as one person: the Father. The Trinity is that there is one God in three
persons: Father, Son, and the Holy Spirit. The Trinity is monotheistic, not polytheistic as some of its
critics like to assert. Monarchians were divided into two main groups, the dynamic monarchians and
the modal monarchians.
Dynamic Monarchianism teaches that God is the Father and that Jesus is only a man, denied the
personal subsistence of the Logos and taught that the Holy Spirit was a force or presence of God the
Father. Present day groups in this category are the Jehovah's Witnesses, Christadelphians, and
Unitarians. Additionally, some ancient dynamic monarchianists were also known as Adoptionists who
taught that Jesus was tested by God and after passing this test and upon His baptism, He was granted
supernatural powers by God and adopted as the Son. Ancient teachers of dynamic monarchianism
were Theodotians, a Tanner in Byzantium around 190 A.D., and Paul of Samosata a bishop of Antioch
in Syria around 260 AD.
Modal monarchianism teaches that the Father, the Son, and the Holy Spirit are just modes of the
single person who is God. In other words, the Father, Son, and Holy Spirit are not simultaneous and
separate persons, but consecutive modes of one person. Praxeas, a priest from Asia Minor, taught this
in Rome around 200 AD. Modern groups in this general category are the Oneness Pentecostal groups
known as the United Pentecostal and United Apostolic Churches. However, the present day modalists
maintain that God's name is Jesus. They also require baptism "in Jesus' name" not "in the name of
the Father, the Son, and the Holy Spirit" for salvation.

Monophysitism

Monophysitism is an error concerning the nature of Christ that asserts Jesus had only one nature,
not two as is taught in the correct doctrine of the hypostatic union: Jesus is both God and man in one
person. In monophysitism, the single nature was divine, not human. It is sometimes referred to as
Eutychianism, after Eutyches 378-452, but there are slight differences. Monophysitism arose out of a
reaction against Nestorianism which taught Jesus was two distinct persons instead of one. Its roots
can even be traced back to Apollinarianism which taught that the divine nature of Christ overtook and
replaced the human one.
Monophysitism was confined mainly to the Eastern Church and had little influence in the West. In
451, the Council of Chalcedon attempted to establish a common ground between the monophysitists
and the orthodox, but it did not work and divisions arose in the Eastern church which eventually
excommunicated the monophysitists in the 6th century.
The denial of the human nature of Christ is a denial of the true incarnation of the Word as a man.
Without a true incarnation there can be no atonement of sin for mankind since it was not then a true
man who died for our sins.
It was condemned as heresy at the Sixth Ecumenical Council in 680-681.
Nestorianism

Nestorianism is the error that Jesus is two distinct persons. The heresy is named after Nestorius,
who was born in Syria and died in 451 AD, who advocated this doctrine. Nestorius was a monk who
became the Patriarch of Constantinople and he repudiated the Marian title "Mother of God." He held
that Mary was the mother of Christ only in respect to His humanity. The council of Ephesus was
convened in 431 to address the issue and pronounced that Jesus was one person in two distinct and
inseparable natures: divine and human.
Nestorius was deposed as Patriarch and sent to Antioch, then Arabia, and then Egypt.
Nestorianism survived until around 1300.
The problem with Nestorianism is that it threatens the atonement. If Jesus is two persons, then
which one died on the cross? If it was the "human person" then the atonement is not of divine quality
and thereby insufficient to cleanse us of our sins.

Pelagianism

Pelagianism derives its name from Pelagius who lived in the 5th century A.D. and was a teacher in
Rome, though he was British by birth. It is a heresy dealing with the nature of man. Pelagius, whose
family name was Morgan, taught that people had the ability to fulfill the commands of God by
exercising the freedom of human will apart from the grace of God. He denied original sin, the doctrine
that we have inherited a sinful nature from Adam. He said that Adam only hurt himself when he fell
and all of his descendents were not affected by Adam's sin. Pelagius taught that a person is born with
the same purity and moral abilities as Adam was when he was first made by God. He taught that
people can choose God by the exercise of their free will and rational thought. God's grace, then, is
merely an aid to help individuals come to Him.
Pelagianism fails to understand man's nature and weakness. We are by nature sinners (Eph. 2:3;
Psalm 51:5). We all have sinned because sin entered the world through Adam: "Therefore, just as sin
entered the world through one man, and death through sin, and in this way death came to all men,
because all sinned" (Rom. 5:12, NIV). Therefore, we are unable to do God's will (Rom. 6:16; 7:14).
We were affected by the fall of Adam, contrary to what Pelagius taught.
Pelagius was condemned by the Ecumenical Council of Ephesus and excommunicated in 417 A.D.
by Pope Innocent I.
Socinianism.

This is a heresy concerning the nature of God. It is derived from two brothers of the surname
Sozinni who lived in the 1500's in Poland. Socinianism denies the doctrine of the Trinity claiming it
denies the simplicity of God's unity. Instead, God is a single person with the Holy Spirit as the power
of God. Since it emphasizes the unity of God, there could be no divine and human union in a single
person as Christ. Therefore, Socinianism denies the incarnation and deity of Christ as well as Christ's
pre-existence. It teaches that Jesus was only a man. However, as is separate from the unitarians, it
taught that Jesus was a deified man and was to be adored as such. Nevertheless, since Jesus is not
divine by nature, His sacrifice was not efficacious; that is, it did not result in the redemption of people
who would trust in it. Instead it was an example of self sacrifice. The followers of Socinianism also
rejected infant baptism, hell, and taught the annihilation of the wicked. The Bible was authoritative
but was only properly understood through rationalism.
Of course, this system of belief is wrong since it denies the doctrine of the Trinity and deity of
Christ.

Tritheism
Tritheism is the teaching that the Godhead is really three separate beings forming three separate
gods. This erring view is often misplaced by the cults for the doctrine of the Trinity which states that
there is but one God in three persons: Father, Son, and Holy Spirit. The doctrine of the trinity is, by
definition, monothestic. That is, it is a doctrine that affirms that there is only one God in all the
universe.
Tritheism has taken different forms throughout the centuries. In the early church the Christians
were accused of being tritheists by those who either refused to understand or could not understand
the doctrine of the Trinity. In the late 11th century a Catholic monk of Compiègne in France, Roscelin
considered the three Divine Persons as three independent beings and that it could be said they were
three gods. He maintained that God the Father and God the Holy Ghost would have become incarnate
with God the Son unless there were three gods.
Present day Mormonism is tritheistic -- but with a twist. Mormonism teaches that there are many
God's in the universe but they serve and worship only one of them. The godhead for earth is to them
really three separate gods: the Father, the Son, and the Holy Ghost. The Father used to be a man on
another world who brought one of his wives with him to this world - they both have bodies of flesh and
bones. The son is a second god who was literally begotten between god the father and his goddess
wife. The Holy Ghost is a third god. Therefore, in reality, Mormonism is polytheistic with a tritheistic
emphasis.
Of course, tritheism clearly contradicts the teaching of the Bible regarding monotheism.

• “You are My witnesses,” declares the Lord, “And My servant whom I have chosen, In order that
you may know and believe Me, And understand that I am He. Before Me there was no God
formed, And there will be none after Me," (Isaiah 43:10).
• “Thus says the Lord, the King of Israel And his Redeemer, the Lord of hosts: ‘I am the first and
I am the last, And there is no God besides Me," (Isaiah 44:6).

• ‘Do not tremble and do not be afraid; Have I not long since announced it to you and declared
it? And you are My witnesses. Is there any God besides Me, Or is there any other Rock? I know
of none,’” (Isaiah 44:8).
New Age Movement

Introduction

The New Age movement is a false movement that denies the essentials of Christianity. It also exalts
mankind to godhood. It is self-centered and occultic.

1. What is the New Age Movement? p. 188


2. What are the basic beliefs of the New Age movement? p. 188
3. What is the New Age view of man? p. 192
4. Generically, what is the New Age view of salvation? p. 192
5. What are some verses that demonstrate that man is not divine? p. 193
What is the New Age Movement

1. What is the New Age Movement?


A. The New Age movement has many sub-divisions, but it is generally a collection of Eastern-
influenced metaphysical thought systems, a conglomeration of theologies, hopes, and
expectations held together with an eclectic teaching of salvation, of "correct thinking," and
"correct knowledge." It is a theology of "feel-goodism," "universal tolerance," and "moral
relativism."
B. In the NAM. Man is central. He is viewed as divine, as co-creator, as the hope for future
peace and harmony. A representative quote might be: "I am affected only by my thoughts.
It needs but this to let salvation come to all the world. For in this single thought is everyone
released at last from fear." (A course in Miracles, The Foundation for Inner Peace,
Huntington Station, N.Y. Lesson 228, p. 461.)
C. Unfortunately for the NAM. the fear they want to be released from might very well be the
fear of damnation, of conviction of sin, and it is even, sometimes, fear of Christianity and
Christians. Though the NAM. is tolerant of almost any theological position, it is opposed to
the "narrow-mindedness" of Christianity that teaches Jesus is the only way and that there
are moral absolutes.
D. The NAM. is difficult to define because "there is no hierarchy, dogma, doctrine, collection
plate, or membership." It is a collection, an assortment of different theologies with the
common threads of toleration and divergence weaving through its tapestry of "universal
truth."
C. - The term "New Age" refers to the "Aquarian Age" which, according to New Age
followers, is dawning. It is supposed to bring in peace and enlightenment and reunite man
with God. Man is presently considered separated from God not because of sin (Isaiah 59:2),
but because of lack of understanding and knowledge concerning the true nature of God and
reality.
2. The New Age Movement is a religious system with two basic beliefs: Evolutionary
Godhood and Global Unity.
A. What is Evolutionary Godhood?
i. It is the next step in evolution. It will not be physical, but spiritual:
a. For the most part, the NAM espouses evolution, both of body and spirit. Man is
developing and will soon leap forward into new spiritual horizons. Many New Age
practices are designed to push one ahead into that horizon. Some of them are astral
projection which is training your soul to leave your body and travel around;
contacting spirits so they may speak through you or guide you; using crystals to
purify your body's and mind's energy systems; visualization where you use mental
imagery to imagine yourself as an animal, in the presence of a divine being, or
being healed of sickness, etc.
ii. Evolutionary Godhood also means that mankind will soon see itself as god, as the "Christ
principle."
a. The NAM teaches that Man's basic nature is good and divine. This opposes God's
Word which says...
i. that we are sinners: "Therefore, just as sin entered the world through one man,
and death through sin, and in this way death came to all men, because all
sinned" (Rom. 5:12, NIV).
ii. and that our nature is corrupt: "All of us also lived among them at one time,
gratifying the cravings of our sinful nature and following its desires and
thoughts. Like the rest, we were by nature objects of wrath" (Eph. 2:3, NIV).
b. It teaches that since man is divine by nature, he then has divine qualities.
i. This is an important part of NAM thinking. Because the average New Ager
believes himself to be divine, he can then create his own reality. If, for
example, a person believes that reincarnation is true, that's fine because that is
his reality. If someone he knows doesn't believe in it, that is alright too because
that is someone else's reality. They can each have a reality for themselves that
"follows a different path."
c. In contrast to this, the Bible says that God alone is the creator: "This is what the
LORD says -- your Redeemer, who formed you in the womb: I am the LORD, who
has made all things, who alone stretched out the heavens, who spread out the earth
by myself" (Isa. 44:24, NIV).
d. The New Ager who believes in his own divinity and ability to create usurps the
authority and position of God. He also is still listening to the lie of the devil who
spoke to Eve and said she would be like God (Gen. 3:5).
iii. Reincarnation
a. Though not all New Agers adhere to reincarnation, most believe in some form or
another. And, many believe the Bible was changed to remove any verses that might
have taught reincarnation. But this accusation only shows the limitation of their
knowledge. The Bible never had any references to reincarnation.
b. Reincarnation opposes the Word of God which says that it is appointed for man to
die once, and after this comes judgment (Heb. 9:27).
B. The second major element of the New Age Movement is Global Unity which consists of three
major divisions: Man with Man; Man with Nature; and Man with God.
i. Man with man.
a. The NAM teaches that we will all learn our proper divine relationship with one
another and achieve harmony and mutual love and acceptance through the
realization and acceptance of this divine proper knowledge.
i. Within this hoped harmony is economic unity. The average New Ager is looking
for a single world leader who, with New Age principles, will guide the world into
a single harmonious economic whole.
ii. It is also hoped that this leader will unite the world into a spiritual unity; that is,
a one world religion.
b. The New Age hope is reminiscent of the Scriptures that speak of the coming
Antichrist:
i. 2 Thess. 2:3-4, "Don't let anyone deceive you in any way, for that day will not
come until the rebellion occurs and the man of lawlessness is revealed, the man
doomed to destruction. He will oppose and will exalt himself over everything
that is called God or is worshiped, so that he sets himself up in God's temple,
proclaiming himself to be God." See also Rev. 13:17,14:9,11; 16:2; 19:20.
ii. Man with nature
a. Since the NAM. says that God is all, and all is God, then nature is also part of God.
Man must then get in tune with nature and learn to nurture it and be nurtured by it.
In this, all people can unite.
b. American Indian philosophies are popular among New Agers because they focus on
the earth, on nature, and man's relationship to them.
c. New Age philosophy generally seeks to merge with those philosophies that put man
and nature on an equal level. We are no more or less important or different than our
cousin the animal, bird, or fish. We must live in harmony with them, understand
them, and learn from them, is the general philosophy of the New Age.
i. This is opposed to the Scriptural teaching of man's superiority over animals
(Gen. 1:26-27; 2:19). This does not mean that Man must abuse what he is
over, but Man is given the responsibility of caring for and being stewards of
God's creation (Gen. 2:15). God will hold Christians responsible for the
stewardship that has been entrusted to them.
d. The New Agers have a name for the earth. It is Gaia. Gaia is to be revered and
respected. Some New Agers even worship the earth and nature.
i. This opposes the Scripture that says we are not to have any other Gods before
God (Ex. 20:3).
iii. Man with God
a. Since the NAM. teaches that man is divine by nature, all people, once they see
themselves as such, will be helped in their unity of purpose, love, and development.
The goal is to fully realize our own goodness. It is obvious that this contradicts
Scriptures, c.f., Rom. 3:10-12: "As it is written: ‘There is no one righteous, not
even one; there is no one who understands, no one who seeks God. All have turned
away, they have together become worthless; there is no one who does good, not
even one."
C. Additional beliefs of the NAM. view of God are:
i. He (it) is impersonal, omnipresent, and benevolent -- therefore he (it) won't condemn
anyone.
a. The New Age god is impersonal. An impersonal God will not reveal himself nor will
he have specific requirements as to morality, belief, and behavior. This is why
reincarnation appeals so much to them. With it, there is no judgment, there is a
second chance, a third chance, and fourth, etc.
b. It should be noted that because the New Ager seeks to elevate himself to godhood,
he must lower the majesty and personhood of the true God. In other words, the
universe isn't big enough for one true God, but it is big enough for a bunch of little
ones.
ii. There are no moral absolutes in the New Age. Therefore, they claim to have a spiritual
tolerance for all "truth systems." They call this "harmonization."
a. There is an obvious problem here. To say that there are no moral absolutes is an
absolute in itself which is self contradictory. Also, if morality is relative, then stealing
may be right sometimes, along with lying, adultery, cheating, etc. Living in a world
of moral relativism would not bring a promising future.
b. It would follow that if reality is relative and truth is too, then driving a car would be
difficult. After all, if one New Ager thinks the light is red and another thinks its
green, when they collide, their different realities will come crashing down on them.
That is something most interesting about New Agers, they don't live what they
believe. That is because in reality, New Age thinking doesn't work.
c. The New Age movement does espouse honesty, integrity, love, peace, etc. It just
wants to do it without the true God. It wants to do it not on His terms, but on its
own.
More on the New Age Movement
1. What the New Age Movement does.
A. It is a sponge that attempts to absorb all religions, cultures, and governments.
B. It seeks to unify all systems into one spiritual, socio-economic unity.
C. It uses various means to have mystical experiences with God and/or nature and/or self.
Some of the methods were described in Omni Magazine (How to Have a Mystical Experience.
Dec. 1988, p. 137-145) as imagining, where you are told to imagine your own reality;
transcendence, going beyond the limits of time; sleep deprivation, with the purpose of
inducing a mystical experience; focusing, "to experience all of reality as unified and not as
a collection of disparate objects"; avoidance, where communication with the outside world
is stopped in order to reinterpret the world without its influence on you; identification, "To
trade places mentally with a dog or a cat, canary, or animal in the zoo"; reflection, an
exercise designed to help you to view the year to come, differently; and star-gazing, "to
induce a sense of objectivity about your life and a feeling of connectedness to the rest of
cosmos."
2. What the New Age Movement does not do.
A. It does not teach that man is a sinner - Rom. 5:12; Eph. 2:3.
B. It does not teach that man is dependent upon God for all things - Isaiah 43:7; James 1:17.
C. It does not teach that punishment is eternal - Rev. 14:11.
D. It does not teach that the wages of sin is eternal separation from God - Rom. 6:23; Isaiah
59:2.
E. It does not teach that Jesus is the only way to God - Matt. 11:27; John 14:6.
F. It does not accept Christianity as the truth - 2 Tim. 3:16.
3. New Age Terminology
A. According to the Bible, Man is the image bearer of God (Gen. 1:26), and as such, is a
creature of speech. Remember, God said, "...Let there be light" (Gen. 1:3).
B. In the New Age, words are very important. In fact, the New Age has some of its own special
words. Some of them are: Holistic, holographic, synergistic, unity, oneness, harmony, at-
one-ment, transformation, personal growth, human potential, awakening, networking,
energy, and consciousness. These words are prevalent in New Age conversations and
writings.
C. In fact, if you were to go to a New Age Bookstore and read the titles or their books, you
would see a disproportionate amount of them containing the word "self."
4. The New Age Interpretation of Christianity
A. God is not a personal heavenly Father but an impersonal force.
B. God is all and all is God. God is not the "wholly other" creator of all, but part of all that
exists.
C. There is nothing that is not God. (This is pantheism.)
D. There is no sin, only incorrect understanding of truth. Knowledge is what saves, not Jesus.
E. Hell is not a place but an experience here on Earth; it is a state of mind.
F. Jesus was just one of many way showers of divine truth. He exemplified the Christ
consciousness probably better than anyone else.
G. Christ is a consciousness, a form of the higher self. It is possessed by all because everyone
is divine. "It is not Christ that can be crucified" (Miracles, Lesson 303, p. 441).
H. "A miracle is a correction...It merely looks on devastation, and reminds the mind that what it
sees is false. It undoes error" (Miracles, p. 164). A miracle to a New Ager is not God's
intervention into this world to perform His will but the realization of the true reality that God
is all and that you are God.
5. The New Age View of Man
A. Since all is God, and man is part of all, then man is God. This is pantheism.
i. This is an eastern mystical belief system that has crept into mainstream America.
ii. God is not part of creation. He is separate from it and made it (Isaiah 44:24).
iii. Man is not God, he is a creation (Gen. 1:26).
B. Therefore, man is good by nature.
i. Man is not good by nature (Eph. 2:3).
C. Man has infinite potential.
i. This arrogant conclusion based upon false concepts of grandiose self worth, is a
deceptive, self-satisfying indulgence into pride. As Satan wanted to be like God (Isaiah
14:12-17) and encouraged Adam and Eve to be like God also (Gen. 3:1-5), the New
Ager listens to the echo of that Edenic lie and yields to it willingly.
D. Man is one with the universe.
i. Again the difference between man and creation is blurred. Man is made in the image of
God (Gen. 1:26). The universe is not. Man is different than creation.
6. The New Age View of Salvation
A. Salvation in the NAM means to be in tune with the divine consciousness.
i. In tune means to be in harmony with reality and whatever is perceived to be true.
B. Since the NAM doesn't acknowledge sin or sinfulness, there is no need for a redeemer like
Jesus. Salvation, to them, is simply the realization of our divine nature. "I am not a body. I
am free. For I am still as God created me. Salvation of the world depends on me" (Miracles,
Lesson 206, p. 380). Such arrogance is mind-boggling.
C. It is a form of knowledge, of achieving correct thought. Therefore we need to be saved from
ignorance, not sin.
D. Salvation, in the New Age sense, is self achieved through understanding your natural
godlikeness and goodness, combined with proper knowledge.

As you can see, the New Age Movement is not a biblical teaching by any means. It is a false
religious system authored by the evil one. It contradicts Christianity in almost all of its main tenets. It
is to be avoided, to be guarded against, and to be destroyed. And so it will on that glorious day when
the Lord Jesus returns.
A Biblical Responses to the New Age Movement
1. God is personal. If God were impersonal, then the following qualities could not be His.
A. God speaks and has a self given name: "I AM" (Exodus 3:14).
B. God is long suffering, (Psalm 86:15; 2 Peter 3:15).
C. God is forgiving (Daniel 9:9; Ephesians 1:7; Psalm 86:5).
D. God hates sin (Psalm 5:5-6; Habakkuk 1:13).
2. Man is not divine, but a sinner (Romans 3:23).
A. He is deceitful and desperately sick (Jer. 17:9).
B. He is full of evil (Mark 7:21-23).
C. He loves darkness rather than light (John 3:19).
D. He is unrighteous, does not understand, does not seek for God (Rom. 3:10-12).
E. He is helpless and ungodly (Rom. 5:6).
F. He is dead in his trespasses and sins (Eph. 2:1).
G. He is by nature a child of wrath (Eph. 2:3).
H. He cannot understand spiritual things (1 Cor. 2:14).
3. Salvation is not correct thought, but deliverance from the consequence of our sin
(Romans 6:23; Ephesians 2:8-9).
A. Salvation is God's deliverance from damnation (Eph. 2:8-9; Rom. 1:18; 2:5; 5:9).
B. This salvation is found in no one but Jesus alone (Acts 4:12).
4. Miracles are from God not from the mind of man (Matthew 8:1-4; Mark 6:30-44; Luke
17:12-19; John 2:1-11).
A. Miracles imply an action by someone that is greater than ourselves. If God is impersonal,
miracles cannot occur. But they do occur today as well as in Bible times and are not simply
proper thoughts or understanding.
5. Christ means "anointed. Jesus was the Christ, the anointed one."
A. It does not mean a consciousness or quality of people. Jesus was the Christ, the Messiah,
the Deliverer from sin. Jesus is the Christ (Matt. 16:16,20; Luke 9:20).
B. "Was it not necessary for the Christ to suffer these things and to enter into His glory" (Luke
24:26).
C. "Thus it is written, that the Christ should suffer and rise again from the dead the third day"
(Luke 24:46 ).
D. "...we have found the Messiah (which translated means Christ)" (John 1:41).
E. "He [David] looked ahead and spoke of the resurrection of the Christ..." (Acts 2:31).
F. "...God has made Him both Lord and Christ this Jesus whom you crucified" (Acts 2:36).
G. "For while we were still helpless, at the right time Christ died for the ungodly" (Rom. 5:6).
H. "Therefore we have been buried with Him through baptism into death, in order that as Christ
was raised from the dead through the glory of the Father, so we too might walk in newness
of life" (Rom. 6:4).
I. Christ is crucified (1 Cor. 1:23). You sin against Christ (1 Cor. 8:12). The blood of Christ (1
Cor. 10:16).
6. Only the Bible has the message of Grace. Grace is the unmerited favor of God upon His
people.
A. Grace is the undeserved kindness of God. Grace is getting the blessings we do not deserve.
At the death of Christ we are blessed; we are given grace; we are given eternal life and
forgiveness of sins. Only Christianity has the message of free forgiveness given. Every other
religious system on earth has some form of salvation dependent totally or in part on what
the adherents do. Not so with Christianity.
7. Humanity is not unlimited, but just the opposite: it is under bondage (Romans 5:12).
A. Sin is its master and a deadly and deceitful one at that.
8. True morality is that which is revealed by God in the Bible (Exodus 20). Anything else is
only an imitation, a set of ideas laid down by man that originate from the mind of sinful man.
9. The Bible opposes almost all the tenets of the New Age Movement.
A. As Christians, we should be watchful to recognize what is false and teach what is true. We
should be wary because the Edenic lie still rings strong in the hearts of the deceived -- and
they want us to believe as they.

Witnessing to New Agers

1. Ask questions
A. If we are all God, then why do we act so badly?
i. They might say it is because we all have not come to a full realization of our true divine
potentials. It is ignorance that leads to bad deeds.
ii. Then ask them how, if we are divine, our mere ignorant self could so easily override our
divine goodness.
B. Why do our "realities" contradict each other?
i. They might say that they don't contradict each other. They are simply different shades
of light on the same picture (or something vague like that).
ii. Then ask if truth contradicts itself. It does not. The logic is that if we are all in different
forms of truth, then these different truths can't ultimately contradict each other--or they
wouldn't be true. However,...
a. The NAM says that Jesus is only one of many ways to God. But Jesus said He was
the only way to God (John 14:6). They can't both be right; therefore, the NAM
teaching that we can create our own truths can't be true.
2. Don't let them take Christian words and use them out of the context of biblical
meaning.
A. New Agers recognize the tremendous influence and spotless reputation of Jesus. They want
Him to be associated with their beliefs. As a result, you might find yourself facing a New
Ager who uses Christian words--but with non-Christian definitions. Listen carefully, and don't
let them steal what is Christian and transplant it into their system.
B. You must question the terms they use. You need to make sure that what they mean by
Christian terms is the same thing that you mean by them.
3. Listen for internal contradictions.
A. As mentioned above, truth does not contradict itself. You must listen to what they are saying
and ask questions. Sooner or later you catch on to inconsistencies.
B. Inconsistencies usually arise when discussing the relationship between reality and belief.
i. For example, a New Ager might say that you can create your own reality. I might reply,
"Good. Then if I believe red lights are really green, would you want to go driving with
me?"
4. Tell them that God is personal, that he loves them, and that Jesus died for sin.
A. The Word of God will not come back empty without accomplishing what God wishes it to
(Isaiah 55:11). If you focus on Jesus, tell them the truth about sin and salvation, and use
Scripture, then, at least, they will have heard the truth. Praise be to Jesus the Christ.
B. Remember, God's word is powerful. Whether or not they accept it isn't the issue. You
simply need to present the truth in an accurate and loving manner (Col. 4:5-6; 1 Tim. 1:5).
Interesting Quotes from New Age Sources
(The following quotes are from various newspapers as well as New Age Sources.)

"I am an entity much like you, Barbara. I simply don't have a body at this time."
It was Dr. Carstairs talking. Dr. Carstairs is an English physician from the 1860's. His spirit
generally spends its time floating around the astral plane, I am told, but at this particular moment it
had taken up residence in the Body of Bonney Meyer, a registered nurse from San Diego. [The San
Diego Union, Their spirits are willing, Section C. pg. 1 Nov. 12, 1987.]

There are many entities about such as myself. Man is not the only living, thinking, breathing
creature in the galaxy. Entities that come from other planets right now are about your planet waiting
for the time to introduce themselves. [The San Diego Union, Their spirits are willing, Section C. pg. 1
Nov. 12, 1987.]

Jach Pursel, a former Florida insurance agent living in Los Angeles, squints his eyes and speaks
with the voice of Lazaris, a spiritual entity of uncertain origins.
"How old are you?" he asked.
"In our reality, we have no time," says Lazaris.
"Why are you making your presence known to man?"
"Because you are ready now..."
"Is the world about to end?"
"No. In a word, no. This is not the ending. This is the beginning." [Time, New Age Harmonies,
page 66, Dec. 7, 1987.]

"Jo Ann Karl is a tall blond who...gets $15 a customer for channeling the archangel Gabriel and a
spirit named Ashtar. [Time, New Age Harmonies, page 66, Dec. 7, 1987.]

Shirley Maclaine said, "We are at any given moment living the totality of everything....The
vibrational oscillation of nature is quickening....Just remember that you are God, and act accordingly."

"In Egypt, a few bemused camel drivers and tourist guides looked on as a lone young man in white
shorts and a glittering shawl danced near the pyramids at Giza. ‘I am God, I am God,' he shouted."
[Paul Nussbaum and Rick Lyman, The San Diego Union, "5,000 greet new age at Mt. Shasta." Aug. 17,
1987, A-2.]

"War is not man's great and terrible disease; war is a symptom, a result. The real disease is the
virus of national sovereignty." [The Urantia Book, 1491.1]

"The goal of eternity is ahead! The adventure of divinity attainment lies before you! The race for
perfection is on! Whosoever will may enter, and certain victory will crown the efforts of every human
being who will run the race of faith and trust, depending every step of the way on the leading of the
indwelling Adjuster and on the guidance of the good spirit of the Universal Son, which so freely has
been poured out upon all flesh." [The Urantia Book, 365.4]

"T.I.C.'s (The Inner Christ) purpose is to assist every individual to know themselves as a Christ, to
clearly and safely channel guidance and prophecy for themselves from their own Inner Christ and to
gain dominion over their life circumstances through prayer and self-revelation. T.I.C. makes the
intellectual concept that we are God a reality." [From a tract from the Teaching of the Inner Christ,
What is T.I.C.?]

"The Pleiadians are a collective of extraterrestrials from the star system the Pleiades. The Pleiadian
culture is ancient and was ‘seeded' from another universe of love long before earth was created. They
have formed a tremendous society which operates with love, with ideas and ideals with which we are
yet unfamiliar. Although the Pleiadians exist in what we would call our future, they call themselves our
ancient family because many of us came here from the Pleiades to participate in the experiment of
earth. As they once promised us, the Pleiadians have returned to earth and are now here to help guide
us during this time of planetary awakening as earth moves through her transition from the third to the
fourth dimension and to assist each of us in our personal journeys of remembering, deepening
awareness and knowing." [From a handout at the San Diego Convention Center, The Pleiadians,
Channeled by Barbara J. Marciniak]

"Representatives of some of the nation's largest corporations, including I.B.M., A.T.&T, and
General Motors, met in New Mexico in July [1986] to discuss how metaphysics, the occult and Hindu
mysticism might help executives compete in the world marketplace." [The New York Times, Spiritual
Concepts Drawing a Different Breed of Adherent. Section Y page 8. Sept. 29, 1986.]

At Stanford University's well-regarded Graduate School of Business, the syllabus for a seminar on
"Creativity in Business" includes meditation, chanting, "dream work," the use of tarot cards and
discussion of the "New Age Capitalist." [The New York Times, Spiritual Concepts Drawing a Different
Breed of Adherent. Section Y page 8. Sept. 29, 1986.]

One concept commonly transmitted in these sessions [seminars and workshops on human
potential]...is that because man is a deity equal to God he can do no wrong; thus, there is no sin, no
reason for guilt in life. [The New York Times, Spiritual Concepts Drawing a Different Breed of
Adherent. Section Y page 8. Sept. 29, 1986.]

The following excerpt is from a handout at a New Age Convention:


"Awareness. . . Building a Better World. In Seven Days of Self-Transformation, You Will: reclaim
your infinite potential; Learn how to resolve any emotional issue which limits the quality of YOUR life,
in any way; Discover what Higher Consciousness is all about; Re-establish an intimate, knowledgeable
relationship with YOUR HIGHER SELF. Restore all the Peace, Love, & Joy YOUR LIFE DESERVES."

Learn the Ancient secrets of Creating Perfect Health within Your Being. The Essenes were the
writers of the Dead Sea Scrolls. Their ability as Natural Physicians and Healers, their Psychic expertise,
their knowledge of Nature, and their Mastery of the Order of the Universe is legendary. Among their
adepts are Pythagoras and Jesus. The Essenes knew the secrets of healing utilizing all the elements of
nature; including sun, air, water, touch, breath and centeredness. [From a handout for a seminar by
David Carmos, Lecture 5/58/95, San Diego Convention Center]

"The ancient prophecies of Mesoamerica pinpoint the return of Quetzalcoatl, Lord of the Dawn to
the time that correlates with August 16/17, 1987 in the Gregorian Calendar. Quetzalcoatl represents
the force of cosmic intelligence, the spiraling, serpentine pattern that governs the movement of all
things in this universe. Quetzalcoatl is the enlightened state, the kundalini energy soaring to the crown
chakra. Quetzalcoatl lives as potentiality, a seed within each one of us. 144,000 Human Beings will
emerge to be the sprouting of that seed on the day of Harmonic Convergence, and will grow to flower
and seed again towards the awakening of all humanity in the years that follow." [Handout from the
International Sacred Sites Festival - 3/15/87]

"The ‘Key to Visualization Course’ teaches:

• The Most Powerful and Correct use of visualization


• The strongest and most effective visualization techniques
• How to easily apply the meditative technique
• How to remove blockages successfully
• How to stay focused without using mind control
• The process of falling apart/falling together
• How to release with confidence and feel light
• Relationships - blending of male/female energy systems
• The power exercise and being a transparency.
[Handout on Inner Vision Dynamics, 3419 Via Lido, Suite 346, Newport Beach, CA 92663]

"The following is a guideline for interpreting the meaning of the colors in your energy field:
Red - Red is the color for vitality and physical health. Red also can represent anger or the use of anger
to create change.
Orange: Orange is a healing color, if you have a lot of orange in your energy field then you are a
natural healer or you are doing some healing work on yourself.
Yellow: Yellow represents personal power and/or a highly developed intellect.
Green: Green represents high affinity between the body and soul. Or green could represent growth
and/or changes ahead of in present time.
Blue: Blue in your energy field indicates high certainty or a lot of creativity. In most cases, this
represents that the individual’s creative channels are clear.
Violet: very psychic, much spiritual information and/or in the process of spiritual transmutation.
White: This individual is a highly evolved being who has the ability to focus or concentrate his/her
energy in or around the body.

The following is the description for a New Age workshop. Fee $5.
"FULL MOON MEDITATION. — This Full Moon is called the Moon of Humanity, and the Festival of
Goodwill. This time is a gateway to higher worlds, through acts of service. The Moon is in Sagittarius,
sign of higher mind, philosophy, the traveler, and teacher. The Sun is in the opposite sign of Gemini,
mental sign of communication and the exchange of ideas. This combination creates a time where the
mind is stimulated and there is an understood duality, balance, change, resolve opposition, and seek
inspiration especially outdoors in nature. The symbol is the winged Caduceus the magic wand where
positive and negative energies are beautifully balanced producing light."
INDEX

1 Jesus' Res. Body, atonement, islam........................................287


1 Cor. 15_3-4.............................................................................. 423 logical absolutes as proof for God......................................... 343
144,000....................................................................................... 778 Mormon and Salvation and works......................................... 369
Pain can make us doubt God................................................. 300
A reg. possibility of Jesus' resurrection..................................... 315
A priori....................................................................................... 159 right to say others believe falsely?.........................................307
A.D............................................................................................. 159 sickness argued against God's existence................................ 338
Aaronic Priesthood..................................................................... 870 What is salvation?................................................................. 361
Abaddon..................................................................................... 159 with a JW about relationship with Jesus................................ 326
Absolution.......................................................................... 159, 658 With a Mormon on God's nature............................................357
Abyss..........................................................................................159 with an obnoxious atheist...................................................... 341
Active Obedience....................................................................... 159 With Catholic on interpreting the Bible................................. 348
Adam................................................................................................ with gnostic on Jesus' Res..................................................... 289
was God according to Brigham Young..................................905 with Muslim on good works.................................................. 328
Adiaphora................................................................................... 159 Apostasy..................................................................................... 162
Adoptionism..................................................................... 159, 1103 and Mormonism.................................................................... 887
Adrammelech............................................................................. 475 in the Christian Church..........................................................249
Advent........................................................................................ 159 Apostle....................................................................................... 162
age.................................................................................................... Apparition.................................................................................. 161
end of this age....................................................................... 624 Arad............................................................................................394
Agnosticism................................................................................159 Aramaic...................................................................................... 524
Ahaziah...................................................................................... 489 Archaeology............................................................................... 394
aionion.............................................................................................. Archangel................................................................................... 162
In the NT............................................................................... 615 Argumentum ad hominem.......................................................... 162
Albigenses........................................................................ 159, 1103 Argumentum ad judicium........................................................... 162
Allah......................................................................................... 1045 Argumentum ad populum........................................................... 162
Amillennialism................................................................... 137, 160 Argumentum ad verecundiam..................................................... 162
Anabaptists................................................................................. 160 Arianism........................................................................... 162, 1104
Anammelech...............................................................................475 Aristophanes............................................................................... 405
Angel.......................................................................................... 160 Aristotle.............................................................................. 405, 474
Angels.............................................................................................. Ark of the Covenant............................................................162, 481
fallen angels.......................................................................... 128 Armageddon............................................................................... 163
Animism..................................................................................... 160 Arminianism............................................................................... 163
Annas......................................................................................... 533 Articles, Various...............................................................................
Annihilationism.......................................................................... 160 Angels................................................................................... 127
Anthropic Principle.................................................................... 160 God....................................................................................... 126
Anthropomorphic....................................................................... 160 Is Baptism necessary for salvation?............................... 103, 735
Antichrist.................................................................................... 160 Man....................................................................................... 123
Antinomianism........................................................................... 161 Moloch.................................................................................. 131
Antipas....................................................................................... 397 Pharisee................................................................................. 130
Apocalypse................................................................................. 161 Ascended Master........................................................................ 163
Apocrypha.................................................................................. 161 Ascension................................................................................... 163
Apollinarianism................................................................ 161, 1104 Assumption......................................................................... 163, 658
Apollonius.................................................................................. 457 Assurance................................................................................... 163
Apollyon.....................................................................................161 Astral Projection......................................................................... 163
Apologetics.................................................................................161 Astrology.................................................................................... 163
Classical................................................................................ 278 Atheism.............................................................................. 163, 929
Cosmological argument......................................................... 282 and a predictable universe..................................................... 989
Evidential.............................................................................. 281 Answers to positions held by atheists.................................... 977
introduction to....................................................................... 271 Basic Tenets of...................................................................... 930
paryer in................................................................................ 276 Concerning atheist attacks on theism.....................................981
Apologetics Dialogues...................................................................... Who created God...................................................................996
A satanist judges God............................................................ 334 Atonement.................................................................................. 163
Agnostic questions the Trinity............................................... 302 Aura............................................................................................164
and Jesus' resurrection........................................................... 298 Autograph................................................................................... 164
Atheist knows there is no God...............................................336 Autonomy................................................................................... 164
Can God be one person as flesh and spirit............................. 331
Claims to be a god................................................................. 303 B
claims to be one of two witnesses of Revelation...................310 Baal............................................................................................ 164
Condeming words................................................................. 318 Babel.......................................................................................... 164
Did Joseph Smith see God the Father?..................................364 Babel, Tower of.......................................................................... 164
evolutionist says evolution is fact.......................................... 323 Babies...............................................................................................
Feelings, gods, and Joseph Smith.......................................... 366 Massacre of........................................................................... 416
Is Baptism nec. for salvation?................................................294 Where do they go when they die?..........................................471
6.Baptism............................................................................164, 658
1 Pet. 3_21.............................................................................113 Lk 14, 1 Jn 3, Are we to hate or not?..................................... 531
Acts 2_38.............................................................................. 117 Mal 3, Gen 6, John 3, Does the Lord change or not?.............496
Gal. 3_27............................................................................... 112 Mal. 3_6, Does God change?.................................................477
in Jesus name?....................................................................... 581 Mk 1, Jn 1, What did Jesus do after meeting John the Baptist?
John 3_5................................................................................ 115 ......................................................................................... 517
Mark 16_16........................................................... 107, 567, 739 Mk 16, Is the ending of Mark really scripture?......................530
Rom. 6_3-5............................................................................109 Mt 1, Lk 3, Why different genealogies for Jesus?..................497
Baptism necessary?.................................................................... 373 Mt 11, Jn 1, Was John the Baptist Elijah?..............................509
Baptismal Regeneration..............................................................164 Mt 11, Lk 7, Jn 1, When did John find out Jesuswas the
Bartimaeus..................................................................................519 Messiah?.......................................................................... 507
Basics............................................................................................... Mt 12 & 28, Mk 16, Lk 24, Jn 20, How long was Jesus in the
of Christianity....................................................................... 262 tomb?............................................................................... 510
baths........................................................................................... 489 Mt 12, Mk 3, Act 13, Forgiven of all sins or not?..................518
Bethel......................................................................................... 394 Mt 17, Mk 9, Lk 9, After how many days did Jesus take the
Bible........................................................................................... 164 men up the moutain?........................................................ 512
as history............................................................................... 406 Mt 20, Mk 10, Who requested to sit beside Jesus?................512
Been rewritten too many times?............................................ 411 Mt 21, Mk 11, Did the tree wither immediately or not?.........520
Can it be trusted?...................................................................413 Mt 21, Mk 11, Lk 19, One or two animals brought to Jesus?.519
Day of Darkness.................................................................... 420 Mt 21, Mk 11, Lk 19, When did Jesus cleanse the Temple?. .513
evidence for Jesus' resurrection............................................. 428 Mt 26, Mk 14, Lk 22, Jn 18, Who arrested Jesus?.................522
Evidence of inspiration..........................................................402 Mt 27, Act 1, How did Judas die?..........................................515
Evidence Supporting............................................................. 387 Mt 27, Mk 14, Lk 23, Jn 19, were the women close or far from
Greek and Hebrew Alphabet....................................................66 thecross?.......................................................................... 525
Is the NT reliable?................................................................. 405 Mt 27, Mk 15, Lk 23, Jn 19, Last words of Jesus?................525
Lost books of......................................................................... 419 Mt 27, Mk 15, Lk 23, Jn 19, What written on the sign on the
New Testament books..............................................................74 cross?............................................................................... 524
Papyri, p1-p76......................................................................... 68 Mt 27, Mk 15, Lk 23, Jn 19, Who carried Jesus' cross?.........523
The Bible...................................................................................2 Mt 27, Mk 15, vinegar, gall, or myrrh given to Jesus?...........515
Was NT written long after Christ?......................................... 408 Mt 28, Mk 16, Lk 24, Jn 20, Did the women tell what happened
Bible Difficulties.............................................................................. or not?.............................................................................. 529
1 Cor 1, Pro 4, Will wisdom stand or not?............................. 537 Mt 28, Mk 16, Lk 24, Jn 20, What did the angels tell Mary?.526
1 Ki 7, 2 Chr 4, How many baths?.........................................489 Mt 4, Lk 4, Where did the Devil first take Jesus?..................499
1 Sam 17 & 2 Sam 21, Who killed Golaith?.......................... 484 Mt 5 & 23, Ps 14, Call someone a fool or not?......................504
1 Sam 31 & 2 Sam 1, Who killed Saul?.................................484 Mt 5, Lk 11, Who did Jesus tell the Lord's prayer to?............502
2 Chr 36, 2 Ki 24, Age of Jehoiachin when he became king..490 Mt 8, Lk 7, Who brougt Centurion's request to Jesus?...........504
2 Chr. 11 & 13, Who was King Abijah's mother....................490 Mt 9, Mk 2, Lk 5, Was the taxman named Matthew or Levi?506
2 Ki 24, 2 Chr 36, How long did Jehoiachin rule?................. 490 Ps 104, Ecc 1, Is 65, 2 Pt 3, Earth abide forever or not?........541
2 Sam 10, 1 Chr 19, How many charioteers killed?...............485 Ps 145, Lam 3, Is God good or bad to people?......................494
2 Sam 23, 1 Chr 21, How many fighting men?......................486 Ps 5, Prv 6, Hos 9, Does God hate or love people?................492
2 Sam 24, 1 Chr 21, Did God or Satan move David to count... Rm 15, Is 2, Ex 14, Jel, 3, Is the Lord of Peace or War?........536
......................................................................................... 486 Rm 3, Ps 14, Job 1, Have allpeople sinned or not?................535
2 Sam 24, 1 Chr. 21, How many years of famine?................487 Rm 5, Eph 2, Act 2, 1 Pt 3, Saved by faith or baptism?.........535
2 Sam 6 & 21, Did Michal have children or not?..................485 Rm 9, Did God harden Pharaoh's heart?................................ 536
Act 9 & 26, After seeing the light, did Paul fall to ground or What hour was Jesus crucified?.............................................524
not?.................................................................................. 533 Bible Difficulties1 Cor 15................................................................
Acts 5, Rm 13, 1 Pt 2, Obey God's Law or man's?................533 Baptism for the dead............................................................. 537
Acts 9 & 22, Men with Paul hear voice or not?.....................534 Bible Knowledge Commentary on 1 Cor. 15....................................
Col 1, Is. 44, Is Jesus or God the creator of all things?..........540 29. Dallas Seminary Faculty........................................................
Dt. 6, Is. 43, no. of gods........................................................ 475 Gal 6, Do we bear one another's burderns or not?............539
Eph 2, Rm 3, Jm 2, Saved by grace or works?.......................539 Gen 22, Heb 11, Gal 4, How many children did Abraham
Ex 20, 23, & 31, Keep the sabbath or not?............................481 have?...........................................................................538
Ex 20, Dt. 5 & 24, Ez 18, Do sons bear father's sins?............482 Bible Verses......................................................................................
Ex 20, Lev. 26, Dt. 5 & 27, Graven images or not?...............480 1 Chron. 29_20...................................................................... 821
Ex. 6 & 24 & 33 & John 1, Seen God or not?........................479 John 20_28............................................................................ 825
Ezra 2, Neh 7, Why are statistics different?........................... 491 John 8_58.............................................................................. 827
Gen 47, Heb 11, Did Joseph worship at bed or on a staff......540 Matt. 3_3............................................................................... 823
Gen 5, 2 Ki 2, Jn 3, Did anyone ascend into heaven?............531 Blasphemy.................................................................................. 165
Gen. 1 & 2, creation accounts................................................474 Book of Life............................................................................... 165
Gen. 3, Adam where are you?................................................476 Book of Mormon........................................................................ 869
Gen. 5, Did they live that long?............................................. 477 12 Essential LDS doctrines not found in...............................859
Gen. 6 & 7, How many kinds in ark?....................................478 errors in................................................................................. 888
Is 7, maiden or virgin?........................................................... 493 praying about it..................................................................... 858
Jer 22, Mt 1, Did Coniah have children or not?.....................495 Praying about it..................................................................... 886
Jer 32, Jud 1, Is the Lord omnipotent or not?.........................495 verses Mormon doctrine........................................................ 858
Jn 5 & 8, Was Jesus a witness of Himself or not?..................532 Born Again................................................................................. 165
Job 1, Why did God allow Satan to afflict Job?..................... 492 Brigham Young................................................................... 868, 897
Jonah 3_10, Does God change?.............................................477 damned if you deny polygamy.............................................. 904
Jonah 3, Did God destroy Nineveh or not?............................ 496 shed your own blood to atone for some sins.......................... 904
the Father and Mary \............................................................ 905 Copyist Errors..................................................................................
Bruce McConkie......................................................... 869, 897, 908 Dittography........................................................................... 473
C Fission................................................................................... 473
Caesar......................................................................................... 405 Fusion....................................................................................473
Caiaphas..................................................................................... 533 Haplography.......................................................................... 473
Caliph....................................................................................... 1046 Homophony...........................................................................473
Calvinism................................................................................... 165 Metathesis............................................................................. 473
Canon......................................................................................... 165 Cornuke...................................................................................... 418
Canopy theory............................................................................ 477 Cosmological argument.............................................................. 168
Capernaum................................................................................. 394 Cosmology................................................................................. 168
Capital sins................................................................................. 165 Covenant.............................................................................168, 578
Catholic...................................................................................... 165 Covenant..........................................................................................
Causality.....................................................................................165 Eternal..................................................................................... 26
Ceasar......................................................................................... 474 of Grace...................................................................................27
Ceasar, Tiberius.......................................................................... 398 Covenant Theology.....................................................................168
CELESTIAL...............................................................................870 Creation...................................................................................... 168
Centurion.................................................................................... 504 Cross................................................................................................
Chakra........................................................................................ 165 Did Jesus die on a cross?....................................................... 447
Channeling................................................................................. 166 Cult............................................................................................. 169
charioteers.................................................................................. 485 Cults.................................................................................................
Charismatic Gifts........................................................................ 166 and non-verifiability.............................................................. 754
Chemosh.....................................................................................475 How do you get people out of?..............................................756
Chiliasm..................................................................................... 166 How people are kept in..........................................................755
Chorazin..................................................................................... 394 Recruitment techniques......................................................... 755
Christ.......................................................................................... 166 What is a cult?....................................................................... 754
Christ................................................................................................ Who is vulnerable to?............................................................ 755
Deity of................................................................................... 12 D
Christadelphianism..................................................................... 677 Dagon......................................................................................... 475
What does it teach?............................................................... 678 Damnation.................................................................................. 169
Christian..................................................................................... 166 Dan............................................................................................. 394
Christian Church.............................................................................. Dawah...................................................................................... 1046
to the..................................................................................... 239 Death.......................................................................................... 169
What is it?............................................................................. 241 of Jesus, why necessary?....................................................... 470
Christian Science.............................................................................. Decalogue................................................................................... 169
Science and Health with Key to the Scriptures......................706 Decrees, of God.......................................................................... 169
Terms and Definitions of....................................................... 707 Deduction................................................................................... 169
Christianity....................................................................................... Deism......................................................................................... 169
Is it the only true religion?.....................................................468 Demon........................................................................................ 170
Why believe in it above all others?........................................460 Demosthenes.............................................................................. 405
Christology................................................................................. 166 Deontology................................................................................. 170
Church........................................................................................ 166 Depravity.................................................................................... 169
Church.............................................................................................. Determinism............................................................................... 170
Church, the................................................................................ 8 Deuterocanonical........................................................................ 419
Circumcision.............................................................................. 166 Devil........................................................................................... 170
Codex......................................................................................... 167 Dialectic..................................................................................... 170
Cognitive relativism................................................................. 1018 Dichotomy.......................................................................... 123, 170
colt..............................................................................................520 Didactics.....................................................................................170
Common Grace...........................................................................167 Digs..................................................................................................
Communion................................................................................ 167 and archaeology.................................................................... 394
Condemnation............................................................................ 167 Diocese....................................................................................... 170
Conditional immortality............................................................. 167 Disciple...................................................................................... 170
Confession.......................................................................... 167, 658 Dispensation, dispensationalism................................................. 170
Confessional............................................................................... 167 Dittography.................................................................................384
Confirmation.............................................................................. 167 Divination................................................................................... 170
Confusion......................................................................................... Divinity...................................................................................... 170
God the author of or not?.......................................................471 Docetism........................................................................... 171, 1105
Coniah........................................................................................ 495 Doctrine...................................................................................... 171
Consecration............................................................................... 658 3 essential doctrines................................................................ 12
Conspiracy....................................................................................... Basic Doctrine......................................................................... 11
concerning the NT writers..................................................... 432 Christian.................................................................................... 1
Consubstantiation....................................................................... 167 Plurality of Godhead............................................................. 135
Contradictions.................................................................................. Trinity....................................................................................... 4
More, of Watchtower literature..............................................816 Doctrine and Covenants..............................................................869
Contrition............................................................................168, 658 Dogma........................................................................................ 171
Conversion................................................................................. 168 Donatism...........................................................................171, 1105
Conviction.................................................................................. 168 donkey........................................................................................ 520
Coptic......................................................................................... 168 Dowsing..................................................................................... 171
Dualism...................................................................................... 171 G
E gall..............................................................................................515
Ecclesiology............................................................................... 171 Gaza........................................................................................... 395
Edify........................................................................................... 171 Gehenna......................................................................................175
Efficacy...................................................................................... 171 Genealogy.................................................................................. 497
Eisegesis..................................................................................... 171 Gentile........................................................................................ 175
Elder................................................................................................. Gifts, Spiritual Gifts................................................................... 175
office of................................................................................. 132 Gnosticism..................................................................................176
Qualifications for.................................................................. 132 God..................................................................................... 126, 176
Responsibilities of................................................................. 132 God........................................................................................... 3
Elect, Election............................................................................ 172 Holy Spirit, the.......................................................................... 7
election....................................................................................... 639 Seen in the OT.......................................................................584
Elijah.......................................................................................... 509 Gods, False................................................................................. 176
Empiricism................................................................................. 172 Goliath........................................................................................ 484
Enoch......................................................................................... 542 Gospel........................................................................................ 176
Ephesus...................................................................................... 395 What is it?............................................................................. 587
Epimenides................................................................................. 542 What is the Gospel?....................................................... 104, 736
Epistemology.............................................................................. 172 Grace.......................................................................................... 177
Eschatology................................................................................ 172 Graven Images............................................................................480
Eschatology...................................................................................... Great White Brotherhood............................................................177
Final Judgment.......................................................................... 9 Guilt........................................................................................... 177
New Heaveans & Earth............................................................. 9 H
Rapture, the............................................................................... 9 Hades..........................................................................................177
Essentials of the faith..................................................................249 Hadith................................................1046, 1082, 1083, 1090, 1092
Eternal life.................................................................................. 172 and Jihad............................................................................. 1065
Eternal Security.......................................................................... 172 Interesting quotes from........................................................ 1086
Ethics..........................................................................................172 Interesting quotes on Jihad.................................................. 1094
Eucharist............................................................................. 172, 658 Quotes about Muhammad....................................................1096
Euripedes.................................................................................... 474 Hagar.......................................................................................... 478
Euripides.................................................................................... 405 Hajj........................................................................................... 1044
Eutychianism.............................................................................. 172 Halbrook.....................................................................................418
Evangelism....................................................................................... Hamartiology.............................................................................. 177
Do's and Don'ts......................................................................216 Haplography............................................................................... 384
Isaiah 55_11.......................................................................... 208 Hazor.......................................................................................... 395
John 12_32............................................................................ 209 Heaven....................................................................................... 177
Lamp Analogy....................................................................... 211 the three heavens................................................................... 532
Law and Gospel.....................................................................217 Heber C. Kimball................................................................897, 908
Romans 1_16.........................................................................209 Hedonism................................................................................... 177
Evil............................................................................................. 173 Hegira............................................................................. 1035, 1046
Evolution.................................................................................... 173 Hell............................................................................................. 177
Evolutionary Godood................................................................ 1113 According to Mormonism..................................................... 871
Excommunication....................................................................... 173 Loving God would never send people to...............................466
Existentialism............................................................................. 173 unquenchable fire.................................................................. 629
Expiation.................................................................................... 173 Henotheism.................................................................................178
Extraordinary Claims....................................................................... Heresies............................................................................................
require extraordinary evidence.............................................. 425 Donatism............................................................................. 1105
F Heresy........................................................................................ 178
Faith........................................................................................... 173 Herod..........................................................................................397
Fall, The..................................................................................... 173 Herodotus........................................................................... 405, 474
False Prophet.............................................................................. 173 Hesbon....................................................................................... 395
Falsifiability............................................................................... 174 Heterodoxy................................................................................. 178
Family........................................................................................ 232 High Priest.................................................................................. 533
Fast, Fasting............................................................................... 174 High Priest........................................................................................
Fatalism...................................................................................... 174 Annas and Caiaphas.............................................................. 533
Federal Headship........................................................................ 482 Higher Self................................................................................. 178
Fellowship.................................................................................. 174 Historians.........................................................................................
Fideism....................................................................................... 174 Josephus, Flavius...................................................................391
fig tree........................................................................................ 520 Lucian............................................................................. 85, 393
Filioque...................................................................................... 174 Thallus.............................................................................84, 392
Firstborn..................................................................................... 174 Holy Orders........................................................................ 178, 658
Flesh and Blood.......................................................................... 449 Holy Spirit........................................................................................
Fool.................................................................................... 175, 504 Holy Spirit.................................................................................7
Foreknow, Foreknowledge..........................................................175 Holy Spirit, The.......................................................................... 178
Forgiveness.................................................................................174 Holy Water..........................................................................178, 658
Free Knowledge......................................................................... 175 Holy, Holiness............................................................................ 178
Free will..................................................................................... 175 Homer................................................................................. 405, 474
Freethinker................................................................................. 175 Homiletics.................................................................................. 178
Homophony................................................................................ 384 Jehovah.......................................................................................181
Homosexuality............................................................................231 Jehovah's Witnesses..........................................................................
Homosexuality................................................................................. 1914, 607 BC, & 586BC....................................................... 812
What does the Bible say about it?..........................................230 and 1 Cor. 1_2....................................................................... 835
Humanism.................................................................................. 178 and Col. 1_15........................................................................ 841
Humility..................................................................................... 178 and Col. 1_16........................................................................ 842
Hypostatic Union........................................................................ 179 and Heb. 1_6......................................................................... 844
I and Heb. 1_8 & Psalm 45_6.................................................. 846
Idol, Idolatry............................................................................... 179 and John 1_1......................................................................... 824
Iliad.................................................................................... 405, 474 and John 17_3....................................................................... 833
Image of God.............................................................................. 179 and John 8_58 & 10_30-33................................................... 831
Immaculate Conception.............................................................. 179 and mental health.................................................................. 802
Immortality................................................................................. 179 and NWT translation............................................................. 801
Immutability............................................................................... 179 and resurrection of Jesus....................................................... 818
Impute, Imputation..................................................................... 180 and the 144,000..................................................................... 791
In facto....................................................................................... 180 and the Trinity....................................................................... 789
In fieri.........................................................................................180 biblical response to................................................................ 803
Incarnation..................................................................................180 Controlled by the Watchtower?............................................. 785
Induction.................................................................................... 180 deny Trinity........................................................................... 789
Indulgence.......................................................................... 180, 658 doctrine not from Bible alone................................................ 814
Inerrancy.................................................................................... 180 Faithful and Discreet Slave?.................................................. 793
Infant baptism............................................................................. 180 Follow the Watchtower..........................................................781
Infidel......................................................................................... 181 in a nutshell........................................................................... 776
Infinity........................................................................................ 181 Questions for......................................................................... 790
Infralapsarianism........................................................................ 181 Russell, Charles Taze.............................................................794
Injil........................................................................................... 1046 soul, immortality................................................................... 809
Inspiration.................................................................................. 181 Jericho................................................................................ 395, 397
Intermediate state........................................................................181 Jesus................................................................................... 181, 764
International Church of Christ.......................................................... According to Mormonism..................................................... 871
Authority and Submission..................................................... 731 Deity of................................................................................. 138
Baptism................................................................................. 732 Did He inherit original sin?................................................... 450
Church Structure................................................................... 730 Did He really die on a cross?................................................. 447
Discipleship...........................................................................731 Did Jesus rise from the dead?................................................ 448
Doctrines of........................................................................... 730 die on stake or cross.............................................................. 820
History of.............................................................................. 730 Disciples faked His resurrection............................................ 438
Irenaeus...................................................................................... 400 His miracles explained naturally?..........................................451
ISAAC........................................................................................ 538 His resurrection improbable.................................................. 430
Isalm................................................................................................. Is he the everlasting Father?.................................................. 586
Salat.................................................................................... 1044 Jehovah is Jesus.......................................................................35
Ishmaelites..................................................................................478 Jesus.......................................................................................... 6
Islam................................................................................................. Jesus is God verses.................................................................. 33
Abu-Bakr.............................................................................1036 only appeared to die.............................................................. 436
Adam was 90 feet tall.......................................................... 1086 Resurrection and Ascension...................................................584
and angels............................................................................1088 The True Jesus......................................................................... 36
and Paul the Apostle............................................................ 1049 Two Natures of........................................................................ 32
and the Bible....................................................................... 1048 Was He a magician?.............................................................. 435
Angels................................................................................. 1052 Was He just a myth?.............................................................. 444
Comparison grid with Christianity......................................1052 Who did He pray to?............................................................. 583
Crucifixion.......................................................................... 1052 Jesus Only Movement................................................................ 182
fight all people until they worship Allah..............................1086 Jews............................................................................................ 182
God......................................................................................1052 Jihad......................................................................1046, 1064, 1094
Hajj......................................................................................1044 Jihad.................................................................................................
Jesus.................................................................................... 1053 and the Hadith..................................................................... 1065
Jihad.................................................................................... 1064 and the Quran...................................................................... 1064
Quraish................................................................................ 1035 Jinn................................................................................. 1046, 1088
Salvation............................................................................. 1053 Joab............................................................................................ 487
Saum................................................................................... 1044 Job.............................................................................................. 492
Shahada............................................................................... 1044 John Taylor................................................................................. 868
Sufi...................................................................................... 1042 John the Baptist...........................................................398, 507, 509
Sunnites............................................................................... 1032 Joppa.......................................................................................... 396
Women can be mortgaged................................................... 1088 Joseph Fielding Smith.........................................................869, 897
Zakat................................................................................... 1044 Joseph Smith.............................................................................. 868
Interested quotes from........................................................... 903
J need consent of to enter heaven............................................. 904
Jairus.......................................................................................... 506 1.Josephus, Flavius....................................................................... 83
James Talmage....................................................................869, 909 mentions biblical cities and people........................................ 397
Jeconiah...................................................................................... 495 Regarding quotes about Jesus................................................ 421
Jehoiachin................................................................................... 490 Josephus, FlaviusXE ................................................................397
Judas........................................................................................... 515 Molech....................................................................................... 475
Judgment.................................................................................... 182 Monarchianism................................................................. 186, 1108
Just, Justice................................................................................. 182 Monergism................................................................................. 186
9.Justification................................................................................. 8 Monism...................................................................................... 186
Council of Trent.................................................................... 668 Monolatry................................................................................... 186
Verses showing j. by faith......................................................733 Monophycitism...........................................................................186
Justify, Justification.................................................................... 182 Monophysitism......................................................................... 1108
K Monotheism................................................................................186
Ka'aba............................................................................. 1035, 1046 Moral government theology........................................................186
Karma......................................................................................... 182 Moral/ethical relativism............................................................ 1018
Kenosis............................................................................. 182, 1107 Mormonism................................................................................ 187
Khadijah................................................................................... 1038 a look at the Book of Mormon...............................................858
Kingdom of God......................................................................... 183 Aaronic Priesthood................................................................ 870
Koran.............................................................................. 1037, 1053 and Preexistence.................................................................... 872
and terminology.................................................................... 885
L and the Bible......................................................................... 870
Laity........................................................................................... 183 and the gospel........................................................................886
Lamanites................................................................................... 855 and the priesthood................................................................. 887
Latin........................................................................................... 524 Baptism................................................................................. 870
Law............................................................................................ 183 Brigham Young said...................................................... 868, 897
different from the gospel....................................................... 217 Bruce McConkie said.................................................... 869, 897
Law of non-contradiction........................................................... 183 Celestial Heaven....................................................................870
Laying on of hands..................................................................... 183 Doctrine and Covenants........................................................ 869
Lent............................................................................................ 658 Exaltation.............................................................................. 870
Liberalism.................................................................................. 183 God once a man.....................................................................903
Limited atonement...................................................................... 184 Godhead................................................................................ 871
Livy............................................................................................ 405 Gospel................................................................................... 871
Logic.......................................................................................... 184 Heaven.................................................................................. 871
Common ground with unbelievers?....................................... 275 Heber C. Kimball said........................................................... 897
Is it enough?.......................................................................... 275 Holy Ghost............................................................................ 871
Logos..........................................................................................184 Holy Spirit............................................................................. 871
Lord's Supper..............................................................................184 James Talmage said............................................................... 869
Lucretius.....................................................................................405 John Taylor said.................................................................... 868
M Joseph Fielding Smith said............................................ 869, 897
Maacah....................................................................................... 490 Joseph Smith said.................................................................. 868
Madina......................................................................................1035 many gods according to.........................................................903
Man.................................................................................... 123, 184 Mormon objections answered................................................894
Manuscript..................................................................................184 Mormon words done mean the same thing as Christain ones 870
Manuscripts...................................................................................... praying about the BOM......................................................... 895
of the Bible............................................................................ 390 S.H.I.E.L.D.S........................................................................ 873
Martyr.........................................................................................184 their testimony.......................................................................885
Mary................................................................................................. Trinity is three gods...............................................................903
Catholic titles for................................................................... 660 verses the Book of Mormon.................................................. 859
Did she have other children?................................................. 661 Mormonism Unvailed.......................................................................
Mass................................................................................... 184, 659 Mr. Howe.............................................................................. 915
Materialism.................................................................................184 Philastus Hurlbut................................................................... 911
Means of Grace.......................................................................... 185 Stafford,Barton..................................................................... 918
Mecca............................................................................. 1036, 1037 Mortal Sin........................................................................... 187, 659
Mediation, Mediator................................................................... 185 Mosque..................................................................................... 1046
Medina............................................................................ 1037, 1046 Muhammad................................................. 1034, 1046, 1053, 1096
Melchizedek Priesthood................................................................... first muslim was who?.........................................................1077
According to Mormonism..................................................... 872 said more women in hell than men...................................... 1096
Mercy......................................................................................... 185 was a sinner, Hadith............................................................ 1096
Messiah...................................................................................... 185 Muslims............................................................................................
Metaphysics................................................................................185 Shi'ite.................................................................................. 1041
Metathesis.................................................................................. 384 Sunni................................................................................... 1041
Methuselah................................................................................. 477 Myrrh......................................................................................... 515
Micaiah.......................................................................................490 N
Michal........................................................................................ 485 Natural knowledge......................................................................187
Middle Knowledge..................................................................... 185 Naturalism.................................................................................. 187
Midianites................................................................................... 478 Neo-orthodoxy............................................................................187
Millennium............................................................................. 9, 185 Nephites......................................................................................855
Minuscule................................................................................... 186 Nestorianism..................................................................... 187, 1109
Miracle....................................................................................... 186 New Age Movement.........................................................................
Miracles............................................................................................ Terminology........................................................................ 1116
Cannot happen.......................................................................443 Two basic beliefs................................................................. 1113
Mithra......................................................................................... 458 view of man......................................................................... 1117
Modalism.......................................................................... 186, 1107
New Testament........................................................................... 405 Hindrances to........................................................................ 215
i.New World Translation............................................................. 783 Pre-Adamites.............................................................................. 191
and \...................................................................................... 797 Pre-existence.............................................................................. 191
Nineveh.............................................................................. 396, 496 Predestination............................................................................. 192
Non-essentials............................................................................ 249 Premillennialism................................................................. 137, 192
Nun............................................................................................. 187 Presuppositions...........................................................................425
O Preterition................................................................................... 192
Objectivism................................................................................ 187 Priest................................................................................... 192, 659
Occam's Razor............................................................................ 187 Prophecy.....................................................................................402
Occult......................................................................................... 187 Prophet....................................................................................... 192
Omnipotence.............................................................................. 188 Propitiation................................................................................. 192
Omnipresence............................................................................. 188 Pseudepigraphal..........................................................................419
Omniscience............................................................................... 188 Purgatory.................................................................... 192, 659, 666
Oneness Pentecostal................................................................... 562 Purgatory..........................................................................................
and the word 'person'............................................................. 560 Problems with the doctrine.................................................... 663
Questions for......................................................................... 589 Q
What does it teach?............................................................... 559 Q 414
What is oneness?................................................................... 558 Quotes..............................................................................................
Witnessing to them................................................................ 588 From Mormons......................................................................903
Ontological Argument................................................................ 188 Quraish..................................................................................... 1047
Ontology.....................................................................................188 7.Quran........................................................................... 1060, 1087
Oracles....................................................................................... 188 Interesting quotes in............................................................ 1079
Ordination.................................................................................. 188 verses in cancelled?............................................................. 1087
Ordo salutis................................................................................ 188 R
Original Sin........................................................................ 188, 659 Ramadhan................................................................................. 1047
Orson Pratt................................................................................. 909 Rapture....................................................................................... 193
Orthodoxy.................................................................................. 188 Rationalism.................................................................................193
P Reconcile, Reconciliation........................................................... 193
Panentheism............................................................................... 189 Redemption................................................................................ 193
Pantheism................................................................................... 189 Regeneration...............................................................................193
Papias......................................................................................... 400 Reincarnation............................................................................. 193
Papyri............................................................................................... Relativism.................................................................................. 193
Miniscules 404 - 999............................................................... 70 An illustration......................................................................1025
Papyrus....................................................................................... 189 Cognitive............................................................................. 1018
Parable........................................................................................ 189 Ethical................................................................................. 1018
Parables...................................................................................... 140 Situational........................................................................... 1018
Great Banquet, Luke 14_15-24..............................................141 Religion...................................................................................... 193
Unjust Steward, Luke 16_1-8................................................ 153 Repentance................................................................................. 194
Paradise...................................................................................... 189 11.Resurrection...............................................................................8
Parapsychology.......................................................................... 189 Christians mistaken about Jesus'............................................431
Parousia...................................................................................... 189 No non-biblical accounts of Jesus'.........................................441
Paul.................................................................................................. Resurrection of Christ.......................................................................
Fall to ground after seeing Jesus or not?................................533 Christ, resurrection of..............................................................13
Pedobaptism............................................................................... 189 Resurrection, resurrection bodies................................................194
Pelagianism....................................................................... 189, 1109 Revelation.................................................................................. 194
Penance...............................................................................189, 659 Righteousness............................................................................. 194
Pentateuch.................................................................................. 190 Roman Catholicism.................................................................... 651
Pentecost.................................................................................... 190 2nd Vatican Council.............................................................. 654
Permissive decree....................................................................... 190 and Islam............................................................................... 675
Perseverance............................................................................... 190 Comparison Grid with cults...................................................674
Person......................................................................................... 190 Does Purgatory deny sufficiency of Christ's Sacrifice?.........664
Pharisee...................................................................................... 190 Mary...................................................................................... 660
Philosophy.................................................................................. 190 Purgatory and 1 Cor 3_15......................................................666
Plato....................................................................................405, 474 Sacred Tradition.................................................................... 654
Pliny..................................................................................... 84, 405 Terminology.......................................................................... 658
Pliny the Younger....................................................................... 392 Tradition................................................................................ 653
Pluralism.................................................................................... 190 view on justification.............................................................. 670
Pneumatology............................................................................. 190 Why necessary to write about?.............................................. 652
Polygam........................................................................................... Rosary.................................................................................194, 659
in Mormonism....................................................................... 904 S
Polytheism.................................................................................. 191 Sabbath............................................................................... 481, 510
Pontius Pilate.............................................................................. 398 Sacerdotalism............................................................................. 194
Pope.................................................................................... 191, 659 Sacrament........................................................................... 194, 659
Postmillennialism....................................................................... 191 Sadducee.................................................................................... 195
Postmodernism........................................................................... 191 Salat..........................................................................................1044
Pragmatism................................................................................. 191 Salvation............................................................................. 195, 764
Prayer................................................................................. 191, 210 according to Mormonism...................................................... 872
according to Watchtower....................................................... 795 Temple..............................................................................................
is of God................................................................................218 Destruction in 70AD............................................................. 399
Is speaking in tongues necessary for?.................................... 582 Temptation..................................................................................198
Salvation................................................................................... 8 Testament................................................................................... 198
Salvation by Grace....................................................................... 13 Tetragrammaton (YHWH).......................................................... 198
Sanctify, Sanctification............................................................... 195 The Gallic Wars.......................................................................... 474
Sanhedrin....................................................................................195 The Roman Road........................................................................ 219
Satan........................................................................................... 492 Theodicy.....................................................................................198
According to Mormonism..................................................... 872 Theology.................................................................................... 198
Saum.........................................................................................1044 Theophany.................................................................................. 199
Scholasticism.............................................................................. 195 Three heavens............................................................................. 532
Science............................................................................................. Thucydides................................................................................. 405
and the Bible......................................................................... 234 Tithe........................................................................................... 199
Scriptures....................................................................................195 Total Depravity........................................................................... 199
Second Coming, The.................................................................. 196 Tradition..................................................................................... 659
Secularism........................................................................................ Tradition...........................................................................................
in the church.......................................................................... 254 Bible verses........................................................................... 653
Septuagint, The........................................................................... 196 Transcendence............................................................................ 199
shahda.......................................................................................1032 Transfiguration........................................................................... 199
Shechem..................................................................................... 396 Transubstantiation...............................................................199, 659
Shi'ite........................................................................................1041 Tribulation, The.......................................................................... 199
Shiites....................................................................................... 1033 Trichotomy................................................................................. 200
Shirk......................................................................................... 1047 Trinity..................................................... 200, 540, 763, 1067, 1068
Simon of Cyrene......................................................................... 523 Trinity...............................................................................................
Sin.............................................................................................. 196 According to Mormonism..................................................... 872
All forgiven or not?............................................................... 518 subordination in?..................................................................... 56
Sinaiticus.................................................................................... 530 Tritheism........................................................................... 201, 1110
Situational relativism................................................................ 1018 Truth.................................................................................................
Skepticism.................................................................................. 196 known only thru the Watchtower........................................... 786
Socinianism.............................................................................. 1110 Type, Typology........................................................................... 200
Sola Fide.....................................................................................196 U
Sola Gratia.................................................................................. 196 Ummah..................................................................................... 1047
Sola Scriptura............................................................................. 196 Unbelievers......................................................................................
Son of God........................................................................196, 1070 leading worship?................................................................... 258
sons of God.................................................................................492 Uncial......................................................................................... 201
Sophocles................................................................................... 405 Unitarianism............................................................................... 201
Soteriology................................................................................. 196 Unity in the church..................................................................... 245
Soul Sleep...................................................................................196 Universalism...............................................................................201
Sovereignty.................................................................................197 Mark 3.........................................................................................
Spiritual Gifts............................................................................. 197 28_29 Translations........................................................... 603
Staff............................................................................................ 518 what does Satan want?.......................................................... 635
Subjectivism............................................................................... 197 Uz 492
Suetonius.................................................................................... 405
Suffering........................................................................................... V
Why does God allow it?........................................................ 463 Vaticanus.................................................................................... 530
Sufi................................................................................. 1042, 1047 Vellum........................................................................................ 201
Sunnah...................................................................................... 1047 Venial Sin........................................................................... 201, 659
Sunni........................................................................................ 1041 Verses...............................................................................................
Sunnis....................................................................................... 1047 Isa. 43..........................................................................................
Sunnites.................................................................................... 1032 10 475
Supralapsarianism.......................................................................197 Vicar of Christ............................................................................ 659
Surah........................................................................................ 1037 Vicarious Atonement.................................................................. 201
Susa............................................................................................ 396 vinegar........................................................................................ 516
Suzerain-Vassal.......................................................................... 482 W
Synagogue.................................................................................. 197 Watchtower......................................................................................
Synergism................................................................................... 197 contradictions in literature..................................................... 815
synoptic gospels......................................................................... 507 Controls JW's thinking?.........................................................785
Synoptic Gospels........................................................................ 197 points to itself........................................................................ 784
T What is salvation?.......................................................................373
Tabernacle.................................................................................. 197 What is sin?................................................................................ 373
Table................................................................................................. Word, The................................................................................... 202
of Cleansing the Temple........................................................ 543 Worship...................................................................................... 202
Tacitus.......................................................................... 83, 392, 405 Wrath.......................................................................................... 202
Taqwah..................................................................................... 1047 Y
Teleological argument................................................................ 198 Yin and Yang.............................................................................. 202
Teleological Argument................................................................283 Yoga........................................................................................... 202
Teleology.................................................................................... 198 Z
Zakat............................................................................... 1044, 1047 Zerubbabel..................................................................................495
Zaqqum.................................................................................... 1043 Zodiac.........................................................................................203
Zebedee...................................................................................... 512

You might also like